Você está na página 1de 3277

IBO PAPERS

INTERNATIONAL
BIOLOGY OLYMPIAD
Past Papers (1990-2013)

Science Olympiad Blog

INTERNATIONAL BIOLOGY OLYMPIAD


THEORY PROBLEMS

2013, Bern, Switzerland















All IBO examination questions are published under the following Creative Commons license:



CC BY-NC-SA (Attribution-NonCommercial-ShareAlike) https://creativecommons.org/licenses/by-nc-sa/4.0/
The exam papers can be used freely for educational purposes as long as IBO is credited and
new creations are licensed under identical terms. No commercial use is allowed.

IBO 2013 Theoretical Exam 1


Translation version: English (Oicial), downloaded by: sta1

Cell-, molecular- and microbiology

Some bacteria possess an additional mechanism to regulate the production of enzymes involved
in tryptophan (Trp) biosynthesis. The (Trp) operon possesses, prior to the actual genes (trpA-E), a
leader sequence (trpL), coding for a leader peptide. trpL contains two tryptophan codons next to
each other.
Promoter

Operator

At high tryptophan concentrations, the ribosome translates the mRNA of the leader peptide and
stalls at its stop codon, thereby masking segment 2 of the mRNA and allowing segments 3 and 4
to form a stem loop. A stem loop followed by poly-U is a termination signal for the RNA
polymerase (RNA pol), which falls o the DNA.
complete leader peptide

ribosome
RNA polymerase

mRNA

Trp codons

DNA

However, at low tryptophan concentrations, the ribosome gets stalled at the tryptophan codons
allowing 2 and 3 to form a stem loop. The whole tryptophan operon can be transcribed by the
RNA polymerase.
ribosome
incomplete leader peptide

2
1

mRNA

RNA polymerase

Trp codons
DNA

Indicate if each of the following statements is true or false.


A. The same regulation mechanism also works for nuclear genes in eukaryotes.
B. With reduced concentration of the aminoacyl-tRNA synthetase (attaches tryptophan to
tRNATrp), transcription of the trpA-E genes will be inactivated at a lower tryptophan
concentration.
C. After deletion of one of the two tryptophan codons in the gene coding for the leader peptide,
transcription of the trpA-E genes will be inactivated at a lower tryptophan concentration.
D. In case of a mutation destabilizing the stem loop 2-3, transcription of the trpA-E genes will be
inactivated at a lower tryptophan concentration.
A. False

B. False

C. True

D. True

Original commentary
Correct answers
A false
the mechanism works only if the translation begins before the transcription nishes. In eukaryotes, the transcription
happens in the nucleus, then the mRNA is exported to the cytoplasm where it is translated and this mechanism cannot
work in this form.
B false
by reduced concentrations of the tryptophan's aminoacyl-tRNA synthetase, tRNA loaded with tryptophan will be
formed slower than in the normal case, so that less tRNA-Trp will be present. To inactivate the transcription of the
trpA-E genes, a higher tryptophan concentration than in the normal case is needed.
C true
with only 1 Trp codon, a lower concentration of Trp can still allow translation of the leader peptide and therefore
inhibit the synthesis of the enzymes.
D true
a mutation destabilizing stem loop 1-2 will promote the formation of stem loop 2-3 even at low tryptophan
concentrations, inhibiting the transcription of the trpA-E genes.
References
Walsh et al, Biochemistry (1979)

Cell-, molecular- and microbiology

Bacterial count of a liquid culture can be determined by dierent methods: 1) Cells can be
counted under a microscope using a counting chamber, 2) the absorbance of the culture can be
measured in a spectrophotometer (with A600 = 1 corresponding to 8x108 bacterial cells/ml) or 3)
several dilutions of the culture can be plated on agar and the colonies can be counted (see
picture) to calculate the number of colony forming units per milliliter (cfu/ml).

too many to count

Colony count

Indicate if each of the following statements is true or false.


A. A culture with A600 = 0.1 and a doubling time of 30 minutes will reach 4x108 cells/ml in less
than two hours of growth.
B. Counting colonies on plates gives a smaller estimate of the number of bacterial cells than
counting cells under a microscope.
C. Using the plate giving the most accurate results (from the gure), the culture X is estimated
to contain 1.6x105 cfu/ml.
D. When repeating the plating of Dilution f (from the gure) many times, some plates will show
colonies.
A. True

B. True

C. True

D. True

Original commentary
Correct answers
A true
A culture with an OD600 = 0.1 and a doubling time of 30 minutes will have an OD600 = 0.2 after 30 minutes, 0.4 after 1
hour, and 0.8 after 1:30 hour, therefore it will will reach 4*108 cells/ml in less than 2 hours (4x108) cells correspond to
an OD = 0.5).
B true
Counting colonies on plates gives the number of cfu/ml, and only living cells can form colonies, whereas under the
microscope, dead cells are also counted.
C true
The plate giving the most accurate results is the one with 160 colonies that corresponds to a 1:1000 dilution of the
starting culture X. 160*1000 = 1.6x105
D true
The dilution f contains 0.16 cells/ml. Statistically, by plating it more than six times, colonies should grow.

Cell-, molecular- and microbiology

In a living organism, cells die either through apoptosis (programmed cell death) or necrosis (cells
swell and burst).
Indicate if each of the following statements is true or false.
A. Apoptosis is induced in immature T-cells that recognize self antigens.
B. Intestinal epithelial cells losing contact with the basal lamina undergo apoptosis.
C. Neural stem cells undergoing apoptosis expose on their surface a signal promoting
phagocytosis.
D. Necrosis often induces an inammatory immune response.
A. True

B. True

Original commentary

C. True

D. True

Cell-, molecular- and microbiology

The chorismate pathway leading to the synthesis of aromatic amino acids in yeast is presented
here.

Prephenate

Erythrose-4-P
Chorismate

Phosphoenolpyruvate
CM = Chorismate mutase
AS = Anthranilate synthase

Anthranilate

The velocity of the enzyme chorismate mutase (CM) was measured in the presence of either
tryptophan (+Trp) or tyrosine (+Tyr), as well as in the absence of both (--).

Trp

Tyr

4
6
[Chorismate]

10

Based on these results and the pathway scheme, indicate if each of the following
statements is true or false.
A. Tryptophan increases the activity of chorismate mutase.
B. Tryptophan, but not tyrosine inhibits the synthesis of chorismate.
C. A high concentration of tyrosine is likely to increase the synthesis of tryptophan.
D. The prephenate and the anthranilate branches compete for chorismate.
A. True

B. False

C. True

D. True

Original commentary
Correct answers
A true
Under addition of tryptophane, at the same chorismate concentrations, the speed is higher
B false
The synthesis of erythrose-4-P is neither inhibited by tryptophan, nor by tyrosine. It would not make sense that only
tryptophan inhibits E4P, since E4P is needed for the synthesis of both tryptophan and tyrosine and their synthesis is
regulated dierently.

C true
When the tyrosine concentration is increased, CM gets slower, and the chorismate will be used by AS instead to
produce tryptophane.
D true
Both pathway branches have chorismate as a starting point and the positive/negative regulation by Trp/Tyr hints that
chorismate is not present in unlimited supply.
References
Schnappauf et al, Biochemistry (1998)

Cell-, molecular- and microbiology

The following schematic shows a stage of cell division for an eukaryotic diploid cell.

Indicate if each of the following statements is true or not.


A. The schematic may represent a stage of mitosis.
B. The schematic may represent a stage of meiosis II.
C. The cell would have failed to reach this stage if microtubular motor proteins were inhibited.
D. Transcription of histone genes peaks during this stage.
A. False

B. True

C. True

D. False

Original commentary
Correct answers
A false
as the chromosomes are dierent from each other in the picture, they cannot arise from pairs of chromosomes, which
would need to be the case for mitosis of a eukaryotic diploid cell.
B true
during meiosis II, the 2 chromatids are distributed between daughter cells.
C true
to reach anaphase, molecular motors working on microtubule are needed for the expansion of microtubuli
D false
in this stage, the DNA is highly condensed and not available to transcription. Furthermore, new histone proteins are
particularly necessary during the S-phase where the DNA is duplicated and needs to be packaged, not in the
anaphase.

Cell-, molecular- and microbiology

The sensitivity of an enzyme for dierent inhibitors is assessed. The rate of product formation
was measured at dierent concentrations of substrate with 10 nM enzyme. The initial velocity v i

Initial velocity

(at t = 0 s) was calculated and plotted as a function of the substrate concentration in the absence
or presence of two dierent inhibitors.

no inhibitor
Inhibitor 1
Inhibitor 2

Substrate concentration

Indicate if each of the following statements is true or false.


A. In the absence of any inhibitor, the KM (Michaelis constant) of the enzyme is 0.15 M.
B. The eect of Inhibitor 1 can be partially compensated for by adding more substrate.
C. Inhibitor 2 reduces the vmax (maximal velocity) of the enzyme.
D. The turnover number (maximum number of molecules processed per second by one enzyme
molecule) under inhibition by Inhibitor 2 is about 10-20/s.
A. False

B. True

C. True

D. True

Original commentary
Correct answers
A false
Km is the [substrate] at which half the maximal velocity is reached, in this case v max is 300 nM/s, vmax/2 = 150 nM/s,
which corresponds to 0.5 M
B true
With inhibitor 1, only the KM is aected, not the vmax. The reaction to proceed at the same speed as without inhibitor if
more substrate is added (corresponds to a competitive inhibitor).
C true
The vmax is reduced (150 nM/s instead of 300 nM/s without inhibitor): corresponds to an uncompetitive inhibitor.
D true
kcat = vmax / [Enzyme] = (150 nM/s) / (10 nM) = 15/s

Cell-, molecular- and microbiology

Three yeast strains (Saccharomyces cerevisiae) have been engineered to each contain a gene for
a dierent enzyme from glycolysis (Tpi, Eno or Pyk) under the control of a doxycycline-repressed
promoter such that addition of doxycycline down-regulates the synthesis of the corresponding
enzyme. Concentrations of some metabolites are measured in each yeast strain grown on glucose
at dierent doxycycline concentrations, relative to the concentrations without doxycycline.
Relevant steps of the glycolysis are shown above the measurements with metabolites and
enzymes abbreviated as follows:
Metabolites

Enzymes
ATP-dependent phosphofructokinase

fructose 6-phosphate
fructose 1,6-biphosphate

Aldolase

dihydroxyacetone phosphate

Triose phosphate isomerase

2- and 3-phosphoglycerate

Enolase

phosphoenolpyruvate

Pyruvate kinase

Relative metabolite concentrations

Tpi strain

Eno strain

Pyk strain

Doxycycline concentration

Indicate if each of the following statements is true or false.


A. Down-regulating any of these enzymes disrupted metabolite concentrations in the whole
pathway.
B. Down-regulating any of these enzymes aected the concentration of its substrate more than
the concentration of its product.
C. The equilibrium of the reaction from 2+3-PG to PEP is more on the side of the product than
the equilibrium of the reaction from FBP to DHAP.
D. The concentration of F6P is expected to be unaected by a down-regulation of Al.
A. False

B. True

C. False

D. True

Original commentary
Correct answers
A false
even if the substrate of the down-regulated enzyme undergoes a big concentration change, the concentrations of other
metabolites stays similar.
B true
as seen from the graphs, the metabolite undergoing the biggest change is the substrate of the down-regulated enzyme.
C false
down-regulation of Tpi aects strongly (more than 10 fold) the levels of DHAP, its substrate, but the concentration of

FBP stays constant, thus the equilibrium of FBP DHAP is strongly on the side of DHAP. In contrary, down-regulation
of Pyk does not only aect the concentration of PEP, its direct substrate, but also of 2+3PG, the substrate of the
previous enzyme in the pathway, Eno. This means that the increased level of PEP is enough to shift the equilibrium of
2+3PG PEP back to 2+3PG, therefore the equilibrium is not as strongly on the side of the glycolytic product than
FBP DHAP.
D true
During the reaction of F6P to FBP, ATP is hydrolyzed to phosphorylate F6P. Therefore, the equilibrium is strongly on
the side of FBP. Down-regulation of Al would result in an increase of the FBP concentration, but since the equilibrium
is strongly on the side of FBP (even more strongly than for the reaction FBP DHAP), the concentration of F6P will
stay unaected.
References
Fendt et al, Molecular Systems Biology (2010)

Cell-, molecular- and microbiology

Some substances need to be transported (actively or passively) from their site of synthesis to the
location where they are active.
Indicate for each of the following substances if they are transported from the cytoplasm
to the nucleus.
A. tRNAs
B. Histone proteins
C. Nucleotides
D. ATP-synthase subunits
A. False

B. True

C. True

D. False

Original commentary
Correct answers
A false
tRNAs are synthesized in the nucleus, but need to get to the cytoplasm to be used by the ribosome.
B true
Histones are proteins synthesized in the cytoplasm, but need to get to the nucleus to bind to the DNA.
C true
Nucleotides are obtained by endo/picocytosis or synthesized in the cytoplasm, but need to get to the nucleus to be
used in DNA replication and transcription.
D false
The ATP-synthase is a membrane protein synthetised in the cytoplasm (on the ER membrane) and transported to the
plasma membrane, but not to the nucleus

Cell-, molecular- and microbiology

Progression through the cell-cycle is mediated by Cyclin-Dependent Kinases (CDKs), which


become active only when bound to their respective cyclin and phosphorylated at the ThrC (core
threonine). Phosphorylation or dephosphorylation of other amino acids further modulates their
activity. The following pathway represents the proteins involved in the entry into the M-phase of
the cell-cycle.
CyclinH
activation

inhibition

CDK7

CyclinB
CDK1

entry in M-phase

phosphorylation

dephosphorylation

Indicate for each of the following mutations if it would promote entry into the M-phase
by activating the CyclinB/CDK1 complex.
A. A mutation that reduces the dephosphorylating activity of Cdc25.
B. A mutation that reduces the phosphorylating activity of Wee1.
C. A mutation that changes the ThrC of CDK1 to a valine that cannot be phosphorylated.
D. A mutation that inhibits the binding of CyclinH to CDK7.
A. False

B. True

C. False

D. False

Original commentary
Correct answers
A false
Cdc25 by dephosphorylating CDK1 at Tyr15 and Thr14 activates CDK1 (removes the inactivation). By reducing its
activity, CyclinB/CDK1 would be less active
B true
Wee1 inactivates CDK1 by phosphorylating it at Tyr15.
C false
to be active, CDK1 needs to have Thr161 phosphorylated, with a valine at position 161, CDK1 would be always inactive
D false
CDK7 needs to be bound to CyclinH and phosphorylated at Thr170 to be active and able to phosphorylate (and
therefore activate) CDK1 in complex with CyclinB. This mutation in CyclinH would make it unable to bind CDK7 which
would stay in an inactive state.
References
Fussenegger et al, Biotechnol. Prog. (1998)

10

Cell-, molecular- and microbiology

To determine the precise transcription start site (TSS) of a newly discovered bacterial gene
promoter, a radioactively labeled primer complementary to the 3'-end of the gene is used both for
Sanger sequencing of the DNA construct and for primer extension of the mRNA. Primer extension
(similar to cDNA synthesis) is repeated on mRNA transcribed with addition of the transcription
factor .
Sanger sequencing

TSS

DNA

promoter

primer

transcription

Primer extension
TSS

primer

mRNA

The fragments obtained are separated by gel electrophoresis, a radiography is presented below.
Sanger sequencing

Primer extension

Indicate if each of the following statements is true or false.


A. Dierent polymerases are used for the Sanger sequencing and primer extension assays.
B. mRNAs of this gene with CUCAUGAC as the rst eight bases after the TSS are found in these
cells.
C. Multiple TSS exist for this gene.
D. Transcription is modulated by transcription factor .
A. True

B. True

C. True

D. False

Original commentary
Correct answers
A true
For Sanger sequencing, a DNA polymerase is needed, whereas for primer extension, reverse transcriptase is used.
B true
The PE lane on the radiography gives the length of the mRNA. The rst (most 5') base of the main mRNA corresponds
to the fragment from the sequencing with the same length (here: G). The second most 5' base of the mRNAc
corresponds from the sequencing one nucleotide shorter (=one band lower on the radiography) than PE (here: A).

C true
For this gene, a main and a secondary TSS exist.
D false
Transcription in the absence or presence of produces the same ratio of mRNA starting from the main and the
secondary TSS.

11

Animal anatomy and physiology

High levels of triglycerides in the bloodstream have been linked to higher risks of heart diseases.
An agonist (activating molecule) S of receptor Y was observed to reduce the triglycerides levels.
How could one convincingly prove that the eect of S is mediated specically via
receptor Y?
A. Generate genetically modied mice which overexpress (more than physiological) receptor Y.
B. Generate genetically modied mice in which receptor Y is knocked out (deleted).
C. Treat the mice with an antagonist (inactivating molecule) specic for receptor Y.
D. Treat the mice with an antibody that sequesters (removing from circulation) S.
A. False

B. True

C. True

D. False

Original commentary
Correct answers
A false
Even if the decrease in triglycerides levels observed would be more important than with wild-type mice, this is not
enough to prove that S-Y interaction is either suicient or necessary for the decrease.
B true
If in these mice, triglycerides levels decrease after addition of S, it is not mediated by Y. If no decrease can be
observed, the interaction of S and Y is necessary for the decrease in triglycerides levels
C true
If Y is inactivated by an antagonist, treatment with S should not decrease triglycerides levels, it shows that S-Y
interaction is necessary for the decrease in triglycerides levels. If a decrease would be observed, it would show that S
mediates triglycerides levels decrease via an other mechanism.
D false
Even if no decrease in triglycerides levels would be observed, this would only prove that S is necessary to decrease
triglycerides, but would not prove that it acts via Y.

12

Animal anatomy and physiology

Bilirubin is a breakdown product of heme catabolism which is transported to the liver where it is
conjugated to two glucuronic acid molecules by the enzyme UGT (see gure below). Conjugated
bilirubin is then secreted in the small intestine as a component of the bile.

Indicate for each of the following statements if it is true or false.


A. Conjugation to glucuronic acid increases the solubility of bilirubin in water.
B. A tumor obstructing the bile duct near the junction into the small intestine leads to a decrease
in the blood levels of conjugated bilirubin.
C. A point mutation reducing signicantly the activity of UGT leads to a decreased level of
unconjugated bilirubin in the blood.
D. An increased level of conjugated bilirubin in the blood is a symptom of a malaria infection.
A. True

B. False

C. False

D. True

Original commentary
Correct answers
A true
Glucuronic acid is a hydrophilic molecule, whereas bilirubin is hydrophobic and insoluble in water. Conjugation with
glucuronic acid increases its solubility in water.
B false
When the bile cannot enter the small intestine, the conjugated bilirubin accumulates in the bile duct and moves back
upwards in the intrahepatic bile ducts and enters the blood. As a result the conjugation is reduced or even stops
causing an accumulation of unconjugated bilirubin too.
C false
If the UGT is not working properly (like in patients suering from Morbus Meulengracht), the conjugation of bilirubin
is reduced and the level of unconjugated bilirubin is increased.
D true
At one stage, Plasmodium falciparum reproduces in erythrocytes. These erythrocytes burst when releasing ospring
parasites, leading to a liberation of hemoglobin, which in turn increases the level of bilirubin in the blood, which will
be conjugated.

13

Animal anatomy and physiology

While osteoblasts are secreting new bone material, they can trigger osteoclasts to break down
existing bone by excreting the protein RANKL, which activates its receptor RANK in osteoclasts.
This pathway is stimulated by either vitamin D (D3) or parathyroid hormone (PTH). In the
presence of oestrogen (E2), however, osteoblasts inhibit that process by secreting
osteoprotegerin (OPG), which sequesters RANKL.
Osteoblast

PTH

DNA

mRNA

DNA

mRNA

Osteoclast

cAMP

Bone formation
Bone resorption
Bone

Indicate for each of the following statements if it is true or false.


A. Oestrogen replacement therapy prevents bone resorption after menopause.
B. A symptom of hyperparathyroidism (excessive function of parathyroid gland) is loss of bone
mass.
C. D3 and E2 are hydrophilic molecules whereas PTH is lipophilic.
D. A consequence of Ca2+ loss through the urine is a decrease in PTH plasma levels.
A. True

B. True

C. False

D. False

Original commentary
Correct answers
A true
After menopause the estrogen level declines. Estrogen replacement therapy increases OPG levels and prevents
thereby RANKL to bind to RANK, which would activate osteoclasts.
B true
Loss of bones mass is a symptom of hyperparathyroidism, where increased production of PTH leads to increased levels
of RANKL and increased osteoclast activity
C false
Estrogen and Vitamin D are lipophilic hormones as shown above they have to cross the cell membrane to operate
whereas PTH needs to bind to a extracellular receptor as it is hydrophilic and cannot cross the membrane
D false
Renal loss of calcium leads to a decrease of plasma calcium level which causes a elevation of PTH. PTH indirectly
activates osteoclasts which resorb bone, process during which calcium is released into the blood. This reestablishes
the calcium plasma level.
References
Seeman et al, NEJM (2006)
Stavros et al, NEJM (1995)
Weinstein et al, NEJM (2009)

14

Animal anatomy and physiology

Wing development in chickens starts with the formation of a wing bud which will develop into a
full wing consisting of three digits.

digit 2
digit 3
digit 4

To decipher development of digits, the zone III of the left wing was grafted as an additional zone
III in the wing bud of the right wing during early development. The resulting digit morphology
relative to the somite position is presented below for dierent positions of grafting (indicated by
Z).
control

graft experiments

somites
wing buds

resulting digit pattern

position at which Z was grafted

Indicate if each of the following statements is true or false.


A. Zones I or II are necessary to produce digits.
B. Zone III seems to produce a signal whose concentration inuences the digit type.
C. Cells at the position of somite 19 are unlikely to form a digit 4 if zone III was transplanted
from somite 19/20 to 17.
D. Formation of digits occurs by sequential induction: formation of digit 2 is induced by digit 3,
whose formation is induced by digit 4.
A. True

B. True

C. True

D. False

Original commentary
Correct answers
A true
No digits could develop at position of somites 14/15-16 where zone I is absent.
B true
A high concentration of the signal (near zone III) leads to formation of digit 4, a middle concentration to digit 3, a low
one to digit 2. Grafting a second zone III close enough to the normal one increases the concentration of the signal and
promotes the formation of digits 3/4 (see e.g. when Z is grafted at somite 17, the digit pattern posterior of it is 4-3-3-4,
without digit 2 being formed.
C true
The results suggest that digit number 4 is only formed next to a zone III. In fact, such a transplantation would result in
no digit being formed by somite 19.
D false

If it was the case, a digit 2 could only be formed next to a digit 3 itself near to a digit 4. As seen when Z is grafted at
somites 15 or 15/16, this is not the case, a digit 2 can be formed even in the absence of a digit 3/4 next to it.
References
Tickle et al, Nature (1975)

15

Animal anatomy and physiology

A neuron is kept in a solution similar to the extracellular uid of brain tissue under a pure oxygen
atmosphere. After a few minutes, cyanide, a substance that blocks the electron transport chain, is
added to the solution.
Indicate if each of the following statements is true or false.
A. The concentration of K+ ions in the cell increases.
B. The probability of a spontaneous action potential increases.
C. The concentration of H+ ions in the intermembrane space of the mitochondria increases.
D. The concentration of bicarbonate in the solution decreases.
A. False

B. True

C. False

D. True

Original commentary
Correct answers
A false
Addition of cyanide leads to a rapid depletion of ATP in the neuron. As a consequence, the Na + / K+ pump will no
longer exchange Na+ against K+ inside the cell. Through diusion, the distribution of ions will be equalized across the
cell membrane, and hence the concentration of K+ will decrease in the cell.
B true
With an increase of the membrane potential due to diusion (see A), the probability of a spontaneous action potential
increases.
C false
The concentration of H+ is kept high in the intermembrane space of the mitochondria by the electron transport chain.
After blocking this chain, the concentration quickly decreases through the production of ATP at the ATP-synthetase.
D true
After adding cyanide, the cell stops emitting CO2. The CO2 dissolved as bicarbonate in the solution from before adding
cyanide is entering the atmosphere with an extremely low partial pressure of CO 2 (as it was initially pure O2).

16

Animal anatomy and physiology

Concentration in blood

The eectiveness of antibiotic treatments depends on the time and concentration at which
bacteria are exposed to a specic drug, which in turn depends on the dosage, the intake interval
and the rate of drug elimination from the body. Two antibiotics X and Y, both somewhat toxic to
humans, are excreted by the kidney, but Y is also eliminated in the liver through cytochrome
complexes. While X makes the bacterial cell wall permeable for ions, Y inhibits the synthesis of
the cell wall during cell division. The gure below shows the average concentration of drugs X
(red) and Y (blue) measured in healthy individuals after a single intake of 500 mg or 250 mg,
along with the minimal concentration at which target bacteria are inhibited to grow in vitro (MIC,
solid black line).

MIC

10

12

14

16

Time after intake (hours)

Indicate if each of the following statements is true or false.


A. To safely treat patients with signicantly reduced renal function, intake intervals of X have to
be prolonged.
B. When doubling the dosage of X to 500 mg, doubling the intake interval prevents an
accumulation of X while ensuring that the concentration stays above the MIC in the blood.
C. Patients treated with Y should increase their dosage when consuming fruits containing
inhibitors of cytochrome complexes (e.g. grapefruit).
D. Ensuring a drug concentration above the MIC in the blood is more important for X than Y.
A. True

B. False

C. False

D. False

Original commentary
Correct answers
A true
Drug X is eliminated only renally and thus in renal insuicient patients, the consideration of the risk of accumulation
of drug X is important. By increasing the intake interval, the kidney has more time to eliminate the drug and hence the
risk of accumulation is reduced.
B false
As shown in the graph a dosage of drug X of 250 mg causes blood concentration higher than MC only for 2-3 h but a
dosage of drug X of 500 mg causes blood concentration higher than MC for approx. 8 h, meaning more than double of
a dosage of 250 mg and meaning the interval has to be more than doubled.
C false
Substances inhibiting the cytochrome complexes (e.g. grapefruit) leads to slower inactivation/excretion of drug Y and
hence patients treated with this drug have to be given lower dosages or asked to increase the intake intervals to
prevent accumulation and intoxication.
D false
As bacteria cell division is a continuing process and bacteria divide not simultaneously the concentration of drug Y has

to be as a therapeutic level (higher than MC). A change in the membrane permeability of ions causes rapid bacterial
damage and death consecutively.
References
Tulane University; MIC & Time- vs Concentration-Dependent Killing

17

Animal anatomy and physiology

Dysfunctions of endocrine glands can be classied into three types, depending on the hormone
directly aected:
Primary endocrine dysfunctions alter the production of hormones with direct systemic eects
on metabolism or development.
Secondary endocrine dysfunctions alter the production of tropic hormones that act on other
glands.
Tertiary endocrine dysfunctions aect the hypothalamus.
Indicate for each of the following statements if it is true or false.
A. A patient with an elevated cortisol level, a reduced corticotropin-releasing hormone (CRH)
level and a elevated adrenocorticotropic hormone (ACTH) level is most likely aected by a
primary dysfunction.
B. An overproduction of the thyroid stimulating hormone TSH can be due to a primary
dysfunction.
C. An increased blood concentration of cortisol may be due to a tumor implying a primary or a
secondary dysfunction.
D. In the case of a tumor leading to a secondary endocrine dysfunctions, the blood concentration
of the corresponding releasing hormone is changed.
A. False

B. True

C. True

D. True

Original commentary
Correct answers
A false
The most likely explanation is a secondary dysfunction leading to an overproduction of ACTH, which in turn leads to an
elevated Cortisol level and, due to feedback, an decreased CRH level.
B true
Due to feedback interactions, a primary underproduction leads to an increase in the corresponding tropic hormone.
C true
A hormone-producing tumor of the adrenal gland as a primary dysfunction elevates the cortisol. The same is caused
secondary due a overproduction of ACTH causing a stimulation of the adrenal gland. The latter results in a
overproduction of cortisol.
D true
A secondary dysfunction aects the tropic hormone through a feedback mechanism.
References
Campbell Biology

18

Animal anatomy and physiology

Inhibiting platelet aggregation after a coronary intervention has been shown to greatly reduce
the risk of complications. The eectiveness of two competing inhibitors was assessed in a clinical
study in which 13,608 patients with symptoms of a myocardial infarction were randomly assigned
to a treatment with either inhibitor A or B after the intervention. The gure below shows the
fraction of patients that suered from a second cardiovascular incident such as an infarction or a
stroke, as well as the fraction of those that suered from major bleeding in the 400 days post
intervention.

Fraction of patients

Secondary cardiovascular incidence

Major bleeding

Days after intervention

Indicate if each of the following statements is true or false.


A. This study suggests that using inhibitor B instead of A reduces the risk of a secondary
cardiovascular event, but does not reduce the risk of major bleeding.
B. This study suggests that a switch from inhibitor B to the cheaper inhibitor A after 3 days is
increasing the risk of a secondary cardiovascular event.
C. When using inhibitor B instead of A, the total number of patients suering from a secondary
cardiovascular incidence within 3 days after a coronary intervention is reduced by more than
10%.
D. Repeating the study with a placebo control group is advised.
A. True

B. True

C. True

D. False

Original commentary
Correct answers
A true
Inhibitor B does indeed decrease the risk of a secondary cardiovascular incident, but at the same time increases the
risk of a major bleeding.
B true
The study suggests that patients treated with inhibitor B have a reduced risk of a secondary cardiovascular incidence
even after day 3. This can be read from the gure since the absolute dierence in the fraction of patients with a
secondary cardiovascular incident is increasing from day 3 to day 400. If there were no dierence in the eects after
day 3, the risk to suer a secondary cardiovascular incident in the following 397 days would be the same in both
treatments. Hence the number of patients with such an incident between days 3 and 400 can be calculated as
n*(1-f(3))*r where f(3) is the fraction at day 3, n the total number of patients and r the rate. The total fraction of
patients at day 400 is then given by f(400)=(f(3)*n + (1-f(3))*n*r)/n = f(3)+(1-f(3))*r=f(3)*(1-r)+r. The absolute
dierence at day 400 is thus (fA(3)*(1-r)+r) (fB(3)*(1-r)+r)=(1-r)*(fA(3)-fB(3)), which is always smaller than fA(3)f(B(3) unless r=0. But note that the students do not need to make these calculations but just realize that the absolute
dierence increases.
C true
To calculate the reduction in the total amount of aected patients, one need to compute the reduction from 5.8% to
4.9% of all patients, which is a reduction by 1-4.9/5.8=15.5%. Note that the exact numbers the students read o the
graph do not matter, as to get a reduction of less than 10%, the student would need to misread the percentage of
inhibitor A to be 9% or more.

D false
Given the known reduction in the risk of complications when inhibiting platelet aggregation it would not be ethical not
to give some patient any platelet inhibitor. In addition, the use of a placebo could only reairm the benecial use of
such an inhibitor but not add value to the comparison between inhibitors A and B.
References
Wiviott et al, NEJM (2007)

19

Animal anatomy and physiology

There is variability in the proteins transporting O2, and their ainities for O2, both within and
between organisms.
Indicate if each of the following statements is true or false.
A. At the same partial pressure of O2, the saturation of fetal hemoglobin is higher than the
saturation of the maternal hemoglobin.
B. Hemoglobin has a lower ainity to O2 in the vicinity of cells performing anaerobic glycolysis
heavily.
C. The hemoglobin of deep-diving mammals has a higher oxygen ainity than the hemoglobin of
mammals adapted to high altitude.
D. Hemoglobin is more eicient in transporting O2 than is hemocyanin, the equivalent protein of
many arthropods that binds O2 non-cooperatively.
A. True

B. True

C. False

D. True

Original commentary
Correct answers
A true
This is an adaptation of the fetal hemoglobin to recruit oxygen from the maternal blood.
B true
Cells relying heavily on anaerobic glycolysis recycle their NADH by fermenting lactate, which increases the acidity of
the blood in the vicinity. In a more acidic environment, hemoglobin changes its conformation, which in turn reduces its
ainity for oxygen (Bohr eect). This is an eective way to release oxygen where it is needed most.
C false
The opposite is true. Mammals that are deep divers need hemoglobin that releases most of the oxygen in the blood.
Mammals adapted to high altitude, in contrast, need to ll their hemoglobin with oxygen in the lungs even at low
partial pressure, and hence have hemoglobin with higher ainity.
D true
The cooperativity between the dierent hemoglobin subunits allow for a larger dierence in ainity between the place
of oxygen uptake and oxygen release (a sigmoid dissociation curve).

20

Animal anatomy and physiology

The Hepatitis B virus contains the antigens HBs, HBc and HBe, of which HBs is commonly used
as a vaccine. HBe is expressed in only some strains. The following table shows the presence (+)
or absence (-) of viral antigens and antibodies measured in some patients. A question mark (?)
indicates that the respective test was not performed.
Patient
+
+
?

?
?
-

?
+
?
-

+
+
?
-

?
+
?
+

+
+
-

?
+
?
+
?

Indicate for each of the following statements if it is likely to be true or false.


A. Patient P1 was vaccinated a while ago, but never suered from a Hepatitis B infection.
B. Patient P2 successfully overcame a Hepatitis B infection.
C. Patients P3 and P4 are currently suering from a Hepatitis B infection.
D. Patient P5 has been vaccinated recently.
A. True

B. True

C. True

D. True

Original commentary
Correct answers
A true
Since vaccination is done using HBs, a vaccinated person is producing anti-HBs IgG after several weeks. A Hepatitis
infection, however would also lead to anti HBc and often anti-HBe antibodies, which were not found in P1.
B true
While no antigens were found, P2 produces IgG against all three antigens, even those not used in vaccination.
C true
The antigen HBe was found in P3, which is a good indication that the virus is present. In addition, the immune system
of P3 started its rst response by producing IgM antibodies. Since P4 shows anti-HBc and anti-HBe IgG, he or she
must have been infected at one point. The presence of HBs strongly suggest that the infection is still in progress since
the only alternative would be a very recent vaccination, which is unlikely to be applied to an already immunized
person.
D true
The presence of anti-HBs IgM suggest a recent and exposure to HBs. However, since HBc and HBe antigens are not
present, the likely source of exposure was a recent vaccination.

21

Animal anatomy and physiology

Participants of the IBO 2013 are going to visit mount Niederhorn. Before and right after a rapid
ascent by cable car from 400 m to 2000 m and just before descent 3 hours later, physiological
parameters of several participants will be measured and compared to the same measurements
obtained from an alpine herder who stayed on the Niederhorn for more than two consecutive
months. The participants are expected to hyperventilate at this altitude and get dehydrated.
Indicate if each of the following statements is true or false.
A. The heart rate of an IBO participant just after arriving on top is expected to be higher than
just before the ascent.
B. The pH of the blood of the alpine herder is expected to be higher than the pH of the blood of
an IBO participant when arriving on top.
C. The pH of the urine of an IBO participant is expected to be higher just before descent than
just after arriving on top.
D. Some IBO participants are expected to show a transient increase in the hemoglobin
concentration while on top.
A. True

B. False

C. True

D. True

Original commentary
Correct answers
A true
The hypoxemia due to the lower partial concentration of oxygen will be compensated by an increased heart and
respiratory rate .
B false
The blood pH is tightly controlled and is not expected to change substantially. If any change is observed, then the pH
of the IBO participant is expected to increase due to hyperventilation, followed by a rapid reduction in the
carbondioxyde concentration in the blood.
C true
Due to hyperventilation, the carbondioxyde concentration in the blood is decreased rapidly. To prevent an increase of
the blood pH, the body excretes basic metabolites through the urine, leading to an increase of the urine pH.
D true
To avoid an increase of the blood pH (see answer to C), the kidney excretes basic metabolites, often leading to
dehydration, which is followed by an increase of the hemoglobin concentration in the blood. What is more
hyperventilation (especially on high altitudes) leads to dehydration, too.
References
Campell Biology

22

Animal anatomy and physiology

The rapid evolution of the inuenza antigen Hemagglutinin (HA) is a major challenge for the
development of eicient treatments. In order to identify antibodies eective against a wide
variety of inuenza strains, 13,000 plasma cells of a vaccinated person were individually isolated
and triggered to produce antibodies. These antibodies were tested against dierent types of HA
(H5 VN/04, H7 NE/03 and the mix present in the vaccine) by measuring their binding strength
(gure A). The binding eicacy of two particularly promising antibodies X and Y against several
HA types from groups 1 (red) and 2 (blue) was further assessed by measuring the required
concentration to achieve half the maximal binding eicacy (EC50, gure B). As a comparison, the
EC50 was also measured for an anti-HIV antibody (HIV) and an antibody X* produced by plasma
cells originally producing X but in which all mutations have been reverted to obtain the
corresponding sequence originally inherited from the parents.

group 1

H7 NE/03

>

group 2

Vaccine

Binding strength

Binding strength
H5 VN/04

HIV

Indicate if each of the following statements is true or false.


A. An immune response against the inuenza vaccine is mediated by a diverse set of antibodies.
B. While eicient against HA from group 2 strains, antibody Y does not confer immunity against
all group 1 strains tested.
C. The widespread immunity found for antibody X has in part arisen from somatic mutations.
D. An injection of antibody X confers eective immunity against a wide range of inuenza strains
for several years.
A. True

B. True

C. True

D. False

Original commentary
Correct answers
A true
As can be seen in the lower panel of gure A, there are a large number of plasma cells producing antibodies against
the HA present in the vaccine.
B true
This can be seen in gure B where antibody Y does not bind all group 1 HA better than the control antibody against
HIV.
C true
Since the germ line copies of all genes do not concur immunity, the dierence between X and X* antibodies must be
due to somatic mutations that occurred in the cell line leading to the plasma cell producing X. Note that somatic
mutations and rearrangements are common in antibody producing cells, a likely adaptation to deal with a wider range
of antigens.
D false
While an injection of antibody X concurs passive immunity against a wide range of inuenza strains, it does not
provide active immunity. Therefore, immunization will not last for more than a couple of weeks or maximally month.
References
Corti et al, Science (2011)

23

Animal anatomy and physiology

The following schematics illustrate two severe congenital heart malformations occasionally found
in newborns.

II

Indicate if each of the following statements is true or false.


A. In Malformation I, the oxygen saturation is higher in the pulmonary artery than in the carotid
artery.
B. Surgically swapping the aorta and the pulmonary artery in the case of Malformation I restores
proper blood circulation.
C. In Malformation II, the blood pressure in the carotid artery is increased compared to healthy
individuals.
D. Surgically swapping the venae cavae and the pulmonary vein in the case of Malformation II
restores proper blood circulation.
A. True

B. True

C. True

D. False

Original commentary
Note
In the heart Malformation I, the aorta comes out of the right ventricle (instead of the left one) and the pulmonary
artery out of the left ventricle (instead of the right one). In Malformation II, the aorta is narrowed.
Correct answers
A true
This is true because there is no connection between the pulmonary blood circulation and the systemic one.
B true
As mentioned in the answer note the origins of the aorta and pulmonary artery are swapped.
C true
The narrowing of the isthmus of the aorta causes an increased resistance at this location leading to a reduced ow
downwards the aorta which increases the blood ow in the arteriae of the upper extremities and the head/brain. The
latter increases the blood pressure consecutively.
As a second mechanism the decreased blood ow in the aorta descendens/aorta abdominalis and in the ow renal
arteries consecutively. As a physiological mechanism the kidney rises the circular blood pressure to try to increase the
renal blood ow.
D false
The suggested operation does not change the patients problem. What is more it would create the same separation of
the pulmonary and systemic circulation as in Malformation I.
References
Universittsklinikum Bonn; D-Transposition der groen Arterien

Universittsklinikum Bonn; Aortenisthmusstenose

24

Plant anatomy and physiology

In a community of marine brown algae (Phaeophyta), two multicellular life forms are observed
among the species:
1) A tall and fast-growing form that is strongly aected by environmental seasonality and
shows a high mortality.
2) A tiny and slow-growing form that is less sensitive to seasonality and shows a low mortality.
All species alternate between a haploid and a diploid generation. Isomorphic species show the tall
form both in the haploid and diploid generation. The life form of heteromorphic species, however,
depends on ploidy.
Indicate if each of the following statements is true or false.
A. Proportion of isomorphic species is likely to increase with more intense seasonality.
B. In heteromorphic species, the tall and fast-growing life form is observed during the more
productive season.
C. Heteromorphic species are limited to one generation each season (winter/summer).
D. In these algae, haploid full siblings from diploid parents are genetically less related than
diploid full siblings from haploid parents.
A. False

B. True

C. True

D. True

Original commentary
Correct answers
A false
The description of the small life form is giving a hint that it is adapted to endure a tougher, less productive season
(winter). A species with the tall and fast growing form alone will have an increasing handicap in regions with a more
intense winter compared to heteromorphic species more adapted to endure a tough season.
B true
As the tall life form is adapted to fast growth during summer, it will be more competitive during summer as compared
to individuals with the small life form present during summer.
C true
The small life form is adapted to outlive the unproductive winter - during summer the small life form would implicate a
loss of productivity. On the same time, the tall life form would be very vulnerable during winter and most likely not
survive. So heteromorphic species only make one new generation at the end of each season.
D true
Haploid plants derive from spores which are products of a meiosis and have only one parent. Therefore siblings share
half of their genome in average. Diploid plants derive from a zygote out of two gametes, resulting themselves from a
meiosis of their parents. All gametes from each parent are genetically identical, as are the full siblings among each
other.
References
Bessho et al, Evolutionary Ecology Research (2009)

25

Plant anatomy and physiology

A growing plant can be described by units called metamers (illustrated by boxes) that are
produced by a vegetative meristem. Each metamer consists of a stem segment and an additional
meristem that is inactive at rst, but may become active and develop into a vegetative meristem.
Vegetative mersitems can develop into a owering meristem. Vegetative and owering meristems
produce auxin, which is constantly transported downwards to lower metamers. The gure below
shows a plant at dierent ages ending up owering and illustrates the auxin concentration found
in each metamer.
metamer
inactive

owering

vegetative

root

auxin concentration

Based on the observed auxin concentrations, indicate if each of the following statements
is true or false.
A. Exceeding a minimal auxin threshold invariably activates meristems.
B. An apex turning to a owering stage is losing its apical dominance.
C. A high auxin concentration is enough to trigger the development of owers.
D. Auxin from dierent apical metamers can have cumulative eects on subsequent metamers.
A. False

B. True

C. False

D. True

Original commentary
Correct answers
A false
The opposite is true, below a certain threshold the apical dominance is lost and the uppermost inactive meristem is
activated.
B true
A metamer turning into a ower is reducing its production of auxin, so the concentration of auxin sinks in the
subsequent metamer and falls under the threshold needed to suppress meristem activation.
C false
If this was true, all meristems would turn into a owers.
D true
The residual auxin from all four owering apices accumulates along the stem and prevents the subsequent metamer
from being activated. /br>
References
Przemyslaw et al, PNAS (2009)

26

Plant anatomy and physiology

Frequency (number of species)

Two stable carbon isotopes 12C and 13C are present in the atmosphere, but 12C is approximately
100 times more frequent. Diverse metabolic processes discriminate against 13C in favor of 12C,
leading to a smaller proportion of 13C in biomass than in the atmosphere. The relative dierence
between expected and observed proportion is indicated by 13C, with a more negative value
indicating a stronger discrimination. The gure shows the distribution of 13C-values found in
plant species with C3 and C4 metabolism.

C4

C3

Indicate if each of the following statements is true or false.


A. RuBisCO is discriminating more strongly against
pressure of CO2.

13C

under higher than under lower partial

B. Fixation of CO2 into oxaloacetate is discriminating more strongly against

13C

than the

reaction of RuBisCO.
C. Meat from cattle feeding on a meadow in the Swiss mountains is likely to have a lower
content than from cattle feeding in a central African savanna.

13C

D. It is possible to distinguish between puried sugar from sugar cane (C 4) and sugar beet (C3)
based on their mass.
A. False

B. False

C. True

D. True

Original commentary
Correct answers
A false
Actually the opposite is true. The aim of the C4 metabolism is to increase the partial pressure of CO2 for RuBisCO to
increase the proportion of the carboxylase reaction compared to the oxigenase reaction. The higher partial pressure is
actually the reason of weaker discrimination of 13C in C4 plants.
B false
This reaction is the rst xation step in C4-plants which are less discriminative than C3 plants.
C true
C4 plants are much more present in tropical ecosystems than in temperate or cold ecosystem. The isotope ratio is
reported upwards in the food chain to herbivores and predators.
D true
As 13C is slightly heavier than 12C, the mean weight of a sugar molecule from cane is slightly higher.

27

Plant anatomy and physiology

Seeds of most plants are more resistant to environmental stress before germination has been
initiated. To demonstrate this, seeds of wheat (Triticum aestivum) were exposed to one of the
following four treatments.
Treatments
A
B
C
D

Soaked
5 h at room T
5 h at room T
-

Incubated
5 h; -20C
5 h; 30C
5 h; 4C
5 h; 50C

Transferred to wet paper and kept at room T


+
+
+
+

Indicate for each of the following treatments, if the seeds are expected to produce
sprouts (true), or not (false).
A. Treatment A.
B. Treatment B.
C. Treatment C.
D. Treatment D.
A. False

B. True

C. True

D. True

Original commentary
Correct answers
A false
B true
C true
D true
References
Campbell Biology, 9th ed., p. 84-85 and gure 5.22 ("What determines protein structure?") and pages 807. and gure
39.11 ("Seed Development, Form, and Function").

28

Plant anatomy and physiology

The gure below shows the leaf temperature of two groups of common bean (Phaseolus vulgaris)
plants exposed to infrared light. One group (open squares) was kept at optimal water supply and
the other group (triangles) was drought stressed for 4 month before the experiment.

Temperature in leaves

42

Time of heat exposure (min)

Based on these results, indicate if each of the following statements is likely to be true or
false.
A. After 8 minutes of heat exposure, drought stressed plants kept more stomata open than
control plants.
B. The ability to regulate the opening and closing of stomata decreases over time in plants of
both groups.
C. Following about 15 minutes of heat exposure, leaves of drought stressed plants absorbed
roughly the same amount of thermal energy as they emitted.
D. Plants experience a trade-o between preventing water loss and protection from
over-heating.
A. False

B. False

C. True

D. True

Original commentary
Note
If stomata are open, transpiration increases and therefore leaf temperature decreases (through evaporative cooling).
The opposite is true for closing of stomata.
Correct answers
A false
B false
C true
D true
References
Reynolds-Henne et al, Environmental and Experimental Botany et al (2009)
Campbell, Biology (9th ed.) page 778 ("Eects of Transpiration on Wilting and Leaf Temperature").

29

Plant anatomy and physiology

In a buered suspension of freshly isolated thylakoids incubated in light, the rate of the Hill
reaction (photolysis) can be measured using DCPIP. DCPIP is reduced at Photosystem I and
changes its colour from blue to colourless.
Indicate for each of the following modications of the experimental setting if it would
signicantly reduce the rate of this reaction.
A. Raising the temperature of the solution from 20 C to 30 C.
B. Removing soluble gases from the buer solution prior to adding thylakoids.
C. Adding DCMU, a herbicide that binds to Photosystem II.
D. Adding 2,4-D, a herbicide acting as a synthetic auxin.
A. False

B. False

C. True

D. False

Original commentary
Correct answers
A false
Temperature remains in the physiological optimum and the rate is expected to increase with temperature.
B false
No oxygen nor CO2 is needed for electron transport chain.
C true
If the electron transport chain is interrupted, DCPIP will not be reduced and the suspension will not turn colourless.
D false
Auxin has no eect on electron transport chain.
References
Campbell Biology, 9th ed., p. 193-194 (electron transport chain).

30

Plant anatomy and physiology

Infrared pictures are used to visualise the temperature of a plant surface. The gure below shows
the photograph of a plant and the corresponding infrared picture.

Based on the gure, indicate for each of the following statements if it is true or false.
A. Due to growing in the shade of older leaves, younger leaves of this plant are cooler than older
leaves.
B. Plant parts with high metabolic activity get several degrees warmer than parts with lower
metabolic activity.
C. Transpiration in leaf veins is signicantly lower than in leaf blades.
D. The high temperature of leaf A indicates that this plant begins suering drought stress.
A. False

B. False

C. True

D. False

Original commentary
Correct answers
A false
Younger leaves are actually warmer than older leaves because they are transpiring less. The really old leaves in
senescing state are warmer, but do not provide shade.
B false
While metabolism may indeed increase the temperature of plant parts, this is usually a negligible factor. In addition,
the hottest parts of the plant (shoot and veins) are actually not those parts with the highest metabolism. Those would
rather be leaves producing starch, actively growing meristems and roots (which are not visible). Finally,
non-metabolizing structures such as the pot, the substrate or the small pole with a tag get equally warm as the hottest
parts of plants.
C true
The temperature of leaf veins is higher than that of the leaf blade because their transpiration is very low.
D false
Leaf A is senescing and thus not transpiring any more. Other leaves are healthy and transpiring, hence the reason
cannot be that the plant is suering from drought stress.
References
Campbell, Biology (9th ed.)page 778 ("Eects of transpiration on wilting and leaf temperature")

31

Genetics and evolution

According to the current model of recombination, a recombination event is initiated by a double


strand break (DSB) in one of the two sister chromatids, followed by a trimming of the 5' ends.
During the repair process after recombination, the information immediately anking the DSB site
is lost and supplied by the other chromatid.

Consider the situation in which two Alleles A and B of a locus have probabilities rA and rB to
initiate a DSB and are initially present at equal frequencies in a large, isolated population.
Indicate if each of the following statements is true or false.
A. If rA is twice as large as rB, the frequency of Allele A is expected to change faster when
rB=0.05 than when rB=0.01.
B. If rB is large and rA is much smaller, the frequency of Allele A is expected to reach xation
(frequency=1) almost linearly.
C. If rA=rB, the frequency of Allele A will remain constant even if the population were small.
D. Unless there is an additional mechanism involved, recombination in the population is
expected to decrease over time.
A. True

B. False

C. False

D. True

Original commentary
Note
It should be clear from both the gure and the text that the allele initiating the DSB is not transmitted, which leads a
bias in transmission in heterozygotes if the probabilities rA and rB are dierent.
Correct answers
A true
The larger the recombination rate, the more often the described mechanism can actually play. Hence the allele
frequency changes more rapidly.
B false
While such a setting will lead to a rapid increase in allele A, the increase cannot be linear because the process
depends on the frequency of heterozygous individuals, which become rapidly rare as the frequency of A approaches 1.
Thus, the frequency of A is expected to increase asymptotically.
C false
In the case of rA=rB, this mechanisms will not necessarily lead to a change. However, due to genetic drift, an allele
frequency is never expected to remain constant, unless the population is extremely (innitely) large.
D true
This mechanism eectively leads to a reduction in the recombination rate since alleles with a lower recombination rate
are favored.

32

Genetics and evolution

In female fruit ies (Drosophila melanogaster), the oocyte is located between maternal nurse
cells and follicle cells providing nutrients, proteins and mRNA crucial for the development of the
embryo. In one of the genes whose mRNA is transported to the oocyte, a mutation X has been
found that leads to deformed, non-viable embryos.
Indicate if each of the following statements is true or false.
A. If the mutation is dominant, the female ospring of a heterozygous male and a wild type
female will be viable.
B. If the mutation is dominant, no adult individuals homozygous for X can be observed.
C. If the mutation is recessive, only the female embryos of a mother heterozygous for X will be
deformed.
D. If the mutation is recessive and two individuals heterozygous for X are crossed to produce the
F1, 1/6 of the F2 will be homozygous for X.
A. True

B. True

C. False

D. True

Original commentary
Correct answers
A true
Female with a mutation in a maternal eect gene are viable, even if they are sterile.
B true
To be homozygous for the mutation, an individual need to receive a mutant allele from each parent, however, females
with a dominant mutation are sterile and therefore cannot transmit the mutation to ospring.
C false
If the mutation is recessive, all ospring of a heterozygous mother will be viable.
D true
To be homozygous for the mutation, an individual need to receive a mutant allele from each parent. In the F1 of a cross
between heterozygotes, the genotypes are distributed 1:2:1. Males can give the mutant allele with a probability of 1/2,
however, the homozygous mutant female are sterile, therefore, only heterozygous females, representing 2/3 of the
fertile females can give the mutant allele to their ospring, this with a probability of 1/2. The nal probability of being
homozygous in the F2 is 1/2 * 2/3 * 1/2 = 1/6.

33

Genetics and evolution

Assume a one-dimensional, homogeneous habitat with carrying capacity K that is inhabited by an


annual plant species only at the left most point at generation t=0. In each generation, a fraction
m=0.01 of the seeds disperses to neighbouring locations while the remaining 1-m seeds remain at
the same location. The following gure displays the plant density over the habitat at dierent
generations.

Population density

Direction of expansion

Space

Indicate if each of the following statements is true or false.


A. A mutation doubling m would increase the speed of the expansion.
B. A benecial mutation appearing at time t=100 at Position A will almost certainly become xed
in the population.
C. A neutral mutation appearing at time t=50 in Position A has a higher probability to become
xed in the whole population than a neutral mutation appearing at time t=150 in Position B.
D. A harmful but not lethal mutation appearing at time t=150 is expected to persist in the
population longer if it appears in Position B than Position A.
A. True

B. False

C. True

D. True

Original commentary
Correct answers
A true
The speed of the expansion is primarily dependent on the growth rate and, up to a limit, on the migration rate. If the
migration rate is very small (as is the case here), it will, on average, take multiple generations until the next deme is
colonized. Hence, an increase in the migration rate does lead to an increased colonization speed.
B false
The most likely fate of every mutation appearing in a population is that it is lost by genetic drift.
C true
While both mutations have the same probability to become common at their receptive positions, a mutation that
becomes common at position B will have a very low probability to become common at position A. In contrast, a
mutation that becomes frequent at position A is likely contributing to the newly colonized demes and will thus have a
higher chance to get xed in the whole population.
D true
Since the population density will increase rapidly at Position B, selection is very weak and genetic drift is not eicient
in losing new alleles. In contrast, both drift and selection are acting more strongly on a new mutation at position A
since the population is large and stable in size. Hence, a deleterious mutation is much more likely to persist in the
population when appearing at position B than when appearing at position A.

34

Genetics and evolution

The red colour of a haploid fungus is produced by a pathway converting a precursor pigment
through several intermediates. To study this pathway, several mutant strains (I through IV) of
various colours have been obtained. The following table lists their colours along with those
observed in the haploid progeny of crosses among them.
Strain or cross

Colours observed
red

beige

yellow

pink

wild type
I

II

III

IV

I x wild type

II x III

II x IV

III x IV

X
X

Indicate if each of the following statements is true or false.


A. At least four genes are involved in this pathway.
B. Strain I has mutations in more than one gene involved in this pathway.
C. In the pathway, the enzyme turning the pigment pink is located upstream of the enzyme
turning it beige.
D. Red progeny can be observed when crossing Strains I and IV.
A. False

B. True

C. False

D. True

Original commentary
Correct answers
A false
In this pathway, 3 steps catalyzed by a total of 3 enzymes are enough to explain the results.
B true
Crossing I (beige) with the wild-type produces, besides beige and red (wild-type) ospring, also yellow ospring which
can only be explained if strain I is mutant both for the enzyme metabolizing the beige intermediate and the one
transforming the yellow one. Since strain I appears beige, it also means that the yellow pigment is situated further
downstream in the pathway as the beige one.
C false
Crossing II (beige) with IV (pink) does not produce any red (wild-type) ospring, this means that at least one of the two
strains is mutant for both alleles. Crossing III (yellow) with IV (pink) does not produce beige ospring. Since we know
from B that beige is upstream of yellow, this can only be explained if IV is only mutant for the gene metabolizing the
pink intermediate, therefore strain II is mutant both for the enzyme metabolizing beige and the one metabolizing pink.
Since II appears beige and not pink, the beige intermediate is upstream of the pink one.
D true
Ospring can only be red if between the 2 parents, at least one copy of each gene is wild-type. We know from B that I
is mutant only for the enzymes converting yellow and beige and from C that IV is only mutant for the enzyme
converting, therefore red (wild-type) ospring can be observed when crossing I and IV.

35

Genetics and evolution

Three male members (A, B and C) of a family consult a genetic counsellor. The genealogy of the
family is given below with individuals aected by two genetic diseases X (black) and Y (orange)
are indicated. While Disease X is extremely rare, the disease causing allele of Disease Y has a
frequency of 6% in the population.

A B C
Indicate if each of the following statements is true or false under the most likely modes
of inheritance.
A. Individual B must be a carrier of Disease X.
B. Individual C cannot be a carrier of Disease Y.
C. If Individual A was to have a son with an unrelated and unaected woman, the probability
that the son will be aected by Disease X is 50%.
D. If Individual B was to have a son with an unrelated and unaected woman, the probability
that the son will be aected by Disease Y is > 5.65%.
A. False

B. True

C. True

D. True

Original commentary
Note
Mode of inheritance disease X
Since two aected individuals have a healthy son, disease X must be dominant and since both males and females are
aected, it cannot be Y-linked. Finally, since we know that the disease is extremely rare and thus most likely absent in
the family of the mother of A and B, the disease must be autosomal then an X-linked disease can not be inherited from
father to son (individual A).
Mode of inheritance of disease Y
Dominant inheritance can be excluded because two healthy individual have an aected son (individual B). Autsomal
recessive is unlikely because the father of individuals A and B has healthy parents, grand parents, uncles, aunts and
siblings. Hence the most likely mode of inheritance is X-linked recessive.
Correct answers
A false
Given a dominant inheritance of disease X, individual B cannot be a carrier of this disease. Note that it is enough to
realize that the disease must be dominant but not to distinguish between autosomal or gonosomal to answer this
questions.
B true
A male cannot be carrier of a X-linked recessive disease.
C true
Since the disease is dominant, individual A will pass on the aected allele in 50% of the cases.
D true
While individual B will pass on his Y-chromosome and hence not the disease causing allele, the probability that the
mother will pass it on is non zero. Given the allele frequency of 6% in the population and given that the mother is

healthy, she is a carrier with a probability of (2*0.06*0.94)/(1-0.06*0.06) = 11.32%, in which case she has a probability
of 50% to transmit the disease causing allele. Hence, the probability to have an aected son is 5.67%. A common
mistake is to assume the probability that the mother is a carrier to be 2*0.06*0.94*0.5 = 5.64%, and hence to ignore
the fact that we already know that the mother is not aected.

36

Genetics and evolution

Several inbred lines with recessive mutations are studied in a plant species. Wild type owers are
purple due to a mix of a red and a blue pigment synthesized by two separate biochemical
pathways I and II involving enzymes encoded by genes A-D (all colorless compounds are named
"white"):

I)

white

II)

white

blue

yellow

red

white

Indicate if each of the following statements is true or false.


A. If all genes were unlinked, less than 25% of all F2 individuals of a cross between a red and a
blue inbred line are expected to be red.
B. If back-crossing the F1 of a purple and a yellow inbred line to their yellow parents results in
160 yellow, 40 red, 40 green and 160 purple individuals, genes A and D are 20 cM apart on
the same chromosome.
C. If B was more closely linked to A than to C and A was more closely linked to C than to B, gene
B must be between genes A and C.
D. If the distance between B and C was 28.5 cM and crossing purple F1 individuals from a cross
of two purple inbred lines gives rise to blue individuals in the F2, their frequency is less than
5%.
A. True

B. True

C. False

D. True

Original commentary
Correct answers
A true
A red inbred line is homozygous for a loss of function mutation in gene A (genotype aaBBCCDD). A blue inbred line is
homozygous for a loss of function mutation both in gene B and gene C (genotype AAbbccDD), as otherwise the plants
turn green. The F1 of such a cross has the genotype AaBbCcDD. F2 individuals with red owers must be homozygous
for a (which they are with probability 1/2*1/2=1/4) and may not be homozygous for both b and c at the same time,
which they are with probability 1-(1/2)^4=15/16. The total frequency of red individuals among the F2 is thus 1/4*15
/16=15/64=23.9%. However, note that the red inbred line might also be homozygous for either b or c, in which case
the F1 individuals have genotypes AabbCcDD or AaBbccDD. In this setting, the probability that an F2 individual is not
homozygous for both b and c is reduced to 1-(1/2)^2=3/4, and hence the frequency of red F2 individuals would be
1/4*3/4=3/16=18.75%. So independent of the assumptions, the frequency is always < 25%.
B true
Yellow inbred lines must be homozygous for loss of function mutations a and d. In a back-cross setting with an
individual homozygous for A and D (as the purple inbred line must be), the genetic distance can easily be computed as
the frequency of non-parental phenotypes (green and red) among the progeny (see Campbell chapter 15). Thus, the
distance is 80/400=20cM.
C false
From them stem we know that B and C are linked. We further know that B is closer to A than to C. This leaves us with
the orders A-B-C or B-A-C. Since A is closer to C than to B, only B-A-C remains (or C-A-B read from the other side).
D true
The only possible inbred purple genotypes that result in a purple F1 but some blue F2 when crossed are AAbbCCDD
and AABBccDD. Under this setting, the F1 is AABbCcDD, and hence purple. A blue F2 individual has then the genotype

AabbccDD, which requires a recombination to happen in both F1 parents (from the haplotypes AbCD and ABcD to
AbcD and ABCD). This occurs with probability 0.285 (since the two genes B and C are 28.5 cM apart). After
recombination, the needed haplotype is transmitted with a 50% chance. Hence the total probability for an F2
individual to be blue is (0.285*0.5)2=2.03%.

37

Genetics and evolution

Body height [cm]

A common strategy to detect genes underlying a particular trait is to test for statistical
associations between the phenotype and a very large number of SNVs (single nucleotide variants)
in a large sample of individuals. The charts below give the results of such an approach for body
height and ve independent SNVs typed in 20,000 random individuals from Switzerland.

Indicate if each of the following statements is true or false.


A. In contrast to SNV2, SNVs 1 and 3 are closely linked to a gene with an allele aecting body
height dominantly.
B. If the frequency of the C allele at SNV 4 decreases from 50% to 30% in the population, the
average body height increases.
C. Since the median body height in the population is 175 cm, the frequency of allele A at SNV 1
has to be below 30%.
D. These results are suicient to demonstrate that most of the variation in body height is
genetically determined.
A. True

B. True

C. True

D. False

Original commentary
Correct answers
A true
All three SNVs seem to be linked to a gene with an allele aecting body height. The dierence to SNV 2 is only that
the pattern observed at SNVs 1 and 3 is very likely due to the eect of a dominant-recessive locus in close proximity
since the heterozygous genotype results in a very similar phenotype as one of the homozygous genotypes, but the
pattern observed at SNV 2 is best explained with incomplete dominance.
B true
If the frequency of the C allele decreases from 50% to 30, the number of heterozygous individuals decreases from 50%
to 42%, and hence the average body height is expected to increase.
C true
SNV 5 has no eect on body height, and hence gives a direct estimate of the average height in the population (about
175 cm). If the A allele at SNV 1 has a frequency of 30%, the frequency of the dominant C/C genotype is only 49%,
which is not possible since the average height for this genotype had then to be > 180cm to obtain a population
average of 175cm. Higher allele frequencies of A would even make it worse. Note: the true allele frequency in the
example is 7%.
D false
All SNVs together can explain only 10% of the variation. The students are not able to calculate this value from the
data. However, they are able to judge that SNVs 1 through 4 each explain about 5cm max, which makes about 20cm
dierence between the most extreme genotypes ([A/A, T/T, G/G, A/C] vs [C/C, G/G, A/A, A/A]). However, the body height
in the populations spans a multiple of this dierence. Hence the conclusion from this data that a majority of the
variation s genetically determined is false. But note that in fact body height in humans is 60-80% heritable, yet one
needs dierent data to show this.

38

Genetics and evolution

In cats, there is a genetic locus with two alleles (A, a). In a population, 1300 cats have genotype
AA, 7400 are heterozygous and 1300 individuals carry the recessive genotype aa.
Indicate if each of the following statements is true or false.
A. The frequency of Allele A in the population is 0.5.
B. Under Hardy-Weinberg equilibrium, only 6000 cats are expected to be heterozygous for this
locus.
C. If this population was isolated and mating randomly, the next generation of cats is expected to
be in Hardy-Weinberg equilibrium.
D. Sterility of homozygous individuals can explain this pattern.
A. True

B. False

C. True

D. False

Original commentary
Correct answers
A true
The frequency of allele A is given by (2*1300+7400)/(2*(1300+7400+1300))=10000/20000=0.5.
B false
Under Hardy-Weinberg, 2*p*q=2*0.5*0.5=5000 cats are expected to be heterozygous.
C true
A population is always in the Hardy-Weinberg equilibrium after only a single generation of random mating.
D false
If only heterozygous individuals were fertile, the allele frequency would indeed be 0.5. However, the ospring would
still be expected to be heterozygous in only 50% of the cases.

39

Ecology, ethology and systematics

To elucidate the phylogenetic relationship among three y species of the Lauxaniidae, the
nucleotide sequence of the 18S RNA and the cytochrome oxidase gene was determined in all
species. Dots indicate the same nucleotide as the rst sequence (Minettia) and hyphens represent
deletions or insertions of one or more base pairs.
18S RNA
Minettia
Lauxania
Lyciella
Cytochrome Oxidase
Minettia
Lauxania
Lyciella

Based on these data, indicate if each of the following statements is true or false.
A. The gene for cytochrome oxidase accumulates mutations faster than the gene for 18S RNA.
B. Sequences of Minettia are evolutionarily more conserved than those of Lauxiana or Lyciella.
C. The fact that the cytochrome oxidase sequence of Minettia is 8 nucleotides longer than the
sequences of both Lauxiana and Lyciella suggest that the latter two taxa are more closely
related than either is with Minettia.
D. Single nucleotide substitutions of 18S RNA and cytochrome oxidase suggest dierent
relationship between Minetti, Lauxiana and Lyciella
A. True

B. False

C. False

D. False

Original commentary
Correct answers
A true
Cytochrome oxidase has13 point mutations and 4 deletions, 18S RNA has 6 point mutations and 1 deletion.
B false
Minettia was arbitrarily chosen as reference to align the other two species and do not mean that it is the most
primitive one.
C false
Between Lyciella and Minettia is only one deletion, Lauxiana has 3 independent deletions compared with Minettia and
Lyciella.
D false
False, 18S RNA and cytochrom oxidase do show the same general topology of the genetic tree.

40

Ecology, ethology and systematics

Fruit set

Flowers of cherry trees (Prunus avium) can be pollinated by both domesticated honey bees and
wild insects such as wild bees or bumblebees. To study the inuence of these pollinators on the
fruit set (percentage of owers of a tree that develop into fruits), ower visits of domestic honey
bees and wild insects were counted during a standardized observation period for cherry trees
worldwide. The gure below shows a linear model best explaining the data.

Number of visits
(honey bees)
Number of visits
(wild insects)

Indicate if each of the following statements is true or false.


A. Cherry trees produce no fruits when domesticated honey bees and wild insect are completely
absent.
B. Domestic honey bees were more eicient pollinators than wild insects, needing fewer visits to
increase fruit set.
C. To maximize fruit set, cherry farmers are advised to limit the number of visits by wild insects
when domestic honey bees are visiting.
D. An isolated cherry tree is likely to have a higher fruit set when situated in a ower-rich
backyard than in the middle of a wheat eld.
A. False

B. False

C. False

D. True

Original commentary
Correct answers
A false
The regression predicts that about 10% of cherries would still be pollinated. This might be due to wind pollination or
self-pollination.
B false
The slope of wild bees~fruit set is steeper than the one for honey bees, thus wild insects are more eicient.
C false
Both regressions (domestic bees~fruit set and wild insect~fruit set) are linear. Wild pollinators enhance fruit set
regardless of the abundance of domestic honey bees.
D true
In a ower-rich backyard the density of both wild pollinators and domestic bees is expected to be higher than in a
monoculture of wheat because the backyard is likely to oer a more breeding sites and a more diverse owers
spectrum to feed on.
References
Garibaldi et al, Science (2013)

41

Ecology, ethology and systematics

Individuals / L

In an accident in spring 2003, a large quantity of fertilizer was spilled into a small lake in
Switzerland. The gure shows the abundance of four species of zooplankton measured during
August for several years before and after the accident. The accident is indicated with an arrow

Years

Indicate if each of the following statements is true of false.


A. Species C reacts on the accident with a quick decline in population density.
B. The fertilizer is likely to be poisonous for species A.
C. Species D is more useful as a bioindicator than is species B or C.
D. The relative species densities in the community are re-established within ten years of the
accident.
A. False

B. False

C. True

D. False

Original commentary
Correct answers
A false
The decline in density of species C after the accident is in the range of its normal annual uctuation. A causal relation
with the accident is very unlikely.
B false
Zooplankton has short generation times. If the fertilizer itself were poisonous, the eect would be a drastic reduction
already within the rst two years after the accident.
C true
After the accident, species A and D show drastic and stable shifts in population densities. Those shifts are much bigger
than the stochastic uctuation before and some years after the accident and therefore seem to be reliable. The shift in
population density of species B is much smaller. Species C does not seem to react at all.
D false
After 10 years, species is likely to still have a much smaller density than before the accident. During the years 7-9 after
the accident, no signicant increase in population density happened.

42

Ecology, ethology and systematics

To study the eects of re events on forest ecosystems around a Mediterranean lake, the amount
of microscopic (smaller than 10 m) charcoal particles and pollen of three dierent tree species
were counted in several slices of sediment layers dating from approximately 6000 years before
present. Since the response of forest ecosystems may be visible only after many years, the
correlation between the abundance of charcoal and pollen of a tree species was analyzed for
dierent time lags. The hypothetical tree species in the gure below, for instance, reaches the
highest abundance many years after the actual re event. Hence, the highest correlation is
achieved when the charcoal abundance is shifted with a specic time lag.
time lag

Abundance

pollen
charcoal
charcoal
(time lag shift)
Time

Correlation

The gure below shows the results of such an analysis for three tree species, of which Abies alba
has recently become locally extinct. Correlation coeicients exceeding the threshold (dotted
lines) are statistically signicant at =0.05.

Time lag (years)

Indicate if each of the following statements is true or false.


A. An increase in the frequency of re events may have contributed to the extinction of Abies
alba
B. The pattern observed for Alnus trees can be explained by smoke from the re events
stimulating their owers to produce more pollen.
C. The dominance of Quercus ilex in recent Mediterranean forests can be explained by its
tolerance to periodic re events.
D. The abundance of Abies-pollen is aected by re events more quickly than the abundance of
Quercus-pollen.
A. True

B. False

C. False

D. True

Original commentary
Correct answers
A true
According to the data shown the amount of Abies pollen is negatively correlated with the amount of charcoal for a few
decades after a re. Therefore the species seems not to be tolerant to re. An increase of frequency of res will lead to
successive regression of this species.
B false
The positive correlation between charcoal and pollen approximately 20-40 years after re event cannot be explained
by a reaction of individual owers or even trees to re stimulus. The period of increased pollen production lasts
several decades what is much too long for being explained by a punctual stimulus.
C false
As it is for Abies, Quercus seems to be less present during the period after a re event. The recent dominance must
have other reasons than tolerance to re.

D true
The negative correlation to re events is signicant after a time lag of about 10-15 years in Abies, but only after about
40 years in Quercus.
References
Colombaroli et al, Journal of Ecology (2007)

43

Ecology, ethology and systematics

Many chironomid species (non-biting midges) are known to be abundant only within well-dened
ecological niches, as is shown below for three species commonly found in Switzerland.
Optimal trophic state

Mean T of air in July

Species 1

Oligotrophic

7.1 - 12.9 C

Most common adjacent vegetation


Alpine grassland

Species 2

Mesotrophic

9.3 - 17.6 C

Mixed forests

Species 3

Hypertrophic

10.7 - 19.2 C

Farmland

Fossil chironomids in lake sediments can be used to reconstruct the past climatic and ecological
conditions in the vicinity of the lake. In a sequence of sediment layers of a small swiss lake, head
capsules of all chironomid species have been identied and counted. The relative abundance of
the three listed species in each sediment layer is shown in the gure.
Species 2

Species 3

Age
(yrs before present)

Species 1

Relative abundance

Indicate if each of the following statements is true or false.


A. Human impact is evident since 9000 years BP.
B. An intermittent cooler period can be presumed between 2000 years BP and the present.
C. Species 3 seems to be a better indicator species for the trophic state than for average
temperature.
D. Fluctuation in abundance of Species 2 is likely to be best explained by the Lotka-Volterra
(predator-prey) model including an insectivorous predator.
A. False

B. True

C. True

D. False

Original commentary
Correct answers
A false
The apparent increase in temperature and change of vegetation type at about 9000 years BP coincides with the end of
the last ice age, when alpine vegetation was replaced by forests. However, human impact seems to be clearly indicated
by appearance of species 3, whose optimal niche is in human made farmland that appeared about 3000 years BP.
B true
Since 2000 years, species 1 reappears in the sediment after an absence of several thousand years. It is the same
species that was predominant at the end of the ice age. This indicates that the average temperature fell below a
maximal level for this species.
C true
The sediment of the past 2000 years show that Species 3 is able to live in abundance at the same place as species 1,
which has a much dierent temperature range. Therefore it is not a very good indicator species for temperature. But
species 3 seems to be very closely linked to human farming activity, and predominates since the decline of forest

indicated by species 2.
D false
Interactions between predators and prey show uctuation periods of several years at maximum (insects are short
living), but will not be visible on a time scale of several hundreds of years.
References
Heiri et al, Palaeogeography, Palaeoclimatology, Palaeoecology (2003)

44

Ecology, ethology and systematics

Cladistic analysis is an approach to infer the evolutionary relationship among organisms based on
the presence or absence of morphological or molecular characters. In order for cladistics to work
correctly, the following three assumptions have to be met:
Along a lineage, characters change over time.
Any pair of two organisms shares a common ancestor.
Evolutionary lineages split in a bifurcating manner.
Indicate if each of the following statements is in line with these assumptions (true), or
whether one or more of them are violated (false).
A. The presence or absence of nematocysts can be used in a cladistic analysis to reconstruct
phylogenetic relationship of metazoa. Nematocysts are complex cells of cnidaria (e.g. sea
anemone) that certain sea slugs incorporate into their own body for self defense by feeding on
sea anemones.
B. Cladistics can be used to reconstruct the phylogenetic relationship of ecologically distinct
plant species, of which one arose by hybridisation of two distinct parental species.
C. Cladistics can be used to reconstruct the phylogenetic relationship of two nch species that
arose from a generalist nch that colonized a remote island. The species dier in bill length
and depth, tarsus length and plumage color.
D. Cladistics can be used to reconstruct the phylogenetic relationship of lichen multicellular
eukaryotes. Lichen are a symbiotic interaction between green algae or cyanobacteria with
fungi.
A. False

B. False

C. True

D. False

Original commentary
Correct answers
A false
Although both taxa have nematocysts, this trait cannot be used to put both of them in a distinct clade within metazoa,
as sea slugs do not produce nematocyst themselves. In regard of this trait the evolutionary lineages do not split in
bifurcating manner.
B false
Here, evolutionary lineages do not split in bifurcating manner, but the origin of one lineage is the fusion of two initially
separated lineages.
C true
This example is in agreement with all assumptions.
D false
Lichen behave functionally a single organism, but consists of two independent organisms with a completely dierent
phylogenetical background. Therefore the principle of splitting lineages in bifurcating manner is violated.

45

Ecology, ethology and systematics

Fish species of the family Mormyridae are known for their ability to locate objects and
communicate by weak electric elds called electric organ discharges (EOD). They are also able to
sense EODs of other Mormyridae. The gure shows body shape, relative body size and and
EOD-waveform used for communication (white lines) for 16 Mormyridae species living in a
central African rainforest drainage system.

Indicate if each of the following statements is true or false.


A. Mormyridae show characteristics typical for sh specialized on preying on other sh of
similar size.
B. Mormyridae show characteristics typical for a group of sh warning their predators of an
electric shock via shared visual warning signs (Mllerian mimicry).
C. Mormyridae show characteristics typical for sh living in highly turbid water or are mainly
nocturnal.
D. Mormyridae show characteristics typical of sh that attract mates with non-visual cues.
A. False

B. False

C. True

D. True

Original commentary
Note
Since the students cannot be expected to know these sh, the question focuses on testing if the students can think of
typical features of sh of a specic lifestyle, and are then asked to check if the Mormyridae show such features. This
gets us around asking the students to judge the life style of Mormyridae.
Correct answers
A false
Fish specialized on preying other sh of similar size typically have a relatively large mouth with large conical teeth,
both absent from all species of Mormyridae.
B false
If Mllerian mimicry would be important to reduce pressure from predators, all shown Mormyrids should show very
similar, extremely contrasting colours most likely in combination of yellow and black or orange/red and black. In
contrast, all shown Mormyridae are of dull / camouage color. Furthermore, Mormyridae do not produce harmful
electric shocks which could be used as an anti-predation behavior (mentioned in the text).
C true
Mormyridae live in turbid water and are mostly nocturnal. They have very small eyes compared to body size what
indicates that vison is limited and seems to play a minor role for these sh. Large eyes are common in diurnal sh
species that live in clear water. The complex systems for object localization and communication via the generation and
reception of weak electric elds are very useful for nocturnal sh in turbid water.
D true
All Mormyroiadea are of dull color and have impaired vision. Hence, a system to attrackt mates using non-visual cues

is highly expected. In addition, the electric signals produced vary greatly between species and are used to attract
mates. /br>
References
Hopkins, Electroreception (1986)















All IBO examination questions are published under the following Creative Commons license:



CC BY-NC-SA (Attribution-NonCommercial-ShareAlike) https://creativecommons.org/licenses/by-nc-sa/4.0/
The exam papers can be used freely for educational purposes as long as IBO is credited and
new creations are licensed under identical terms. No commercial use is allowed.

IBO 2013 Theoretical Exam 2


Translation version: English (Oicial), downloaded by: sta1

Cell-, molecular- and microbiology

A newly discovered gene promoter is characterized by a reporter gene assay using


chloramphenicol acetyltransferase (CAT) as a reporter gene. Linear pieces of double-stranded
DNA of four potential promoter elements (white boxes, numbered 1 to 4) are placed upstream of
the CAT reporter gene. --restored!

activity of CAT

After transfection of individual constructs into cells, the [new text here] following CAT activities
were determined.

The following constructs have not tested yet.

Indicate if each of the following statements is true or false.


A. Construct a is a stronger promoter than construct I.
B. Construct a is a stronger promoter than construct b.
C. Construct c is a stronger promoter than construct b.
D. Construct c is a stronger promoter than construct d.
A. False

B. False

C. False

D. False

Original commentary
Notes
I = II -> 2 plays no role,
I > III -> deletion of 4 reduces the promoter strength -> 4 enhances promoter activity,
I < IV -> removal of 3 increases promoter strength -> 3 reduces promoter activity,
V shows almost no activity -> 1 provides basal promoter activity
Correct answers
A false
4 enhances promoter activity, removal of it in a decreases the activity of a.
B false
This case is similar to III versus IV, since 2 does not aect promoter strength.
C false
Without 1, c will show almost no activity, further decreased by the presence of 3.
D false
2 does not play a role, c and d have the same promoter strength.

Cell-, molecular- and microbiology

In genetic engineering, it is often desired to increase the yield of secreted proteins.


Indicate if each of the following strategies is expected to increase the yield in a
mammalian cell.

A. Overexpression of chaperones (proteins assisting others to fold) present in the endoplasmatic


reticulum.
B. Deletion of the genes coding for glycosylating enzymes present in the endoplasmatic
reticulum.
C. Overexpression of proteins facilitating the fusion of secretory vesicles with the cell
membrane.
D. Duplicating the gene encoding the desired protein.
A. True

B. False

C. True

D. True

Original commentary
Correct answers
A true
secretory proteins are folded in the ER, chaperones assist them in their folding and can increase the yield
B false
most secretory proteins are glycosylated, without glycosylation they are not recognised and are not traicked further,
and without glycosylating enzymes, the cell's membrane proteins cannot be glycosylated, the cells are not so healthy,
this will also reduce the yield.
C true
secretory proteins are transported from the Golgi to the plasma membrane by vesicles
D true
Since each of the gene copies can be transcribed independently, genes with multiple copies are generally expressed
more.
References
Peng et al, Biotechnology and Bioengineering (2009)
Tigges et al, Metabolic engineering (2006)

Cell-, molecular- and microbiology

Pupylation is a post-translational protein modication found in some Actinobacteria in which the


short protein Pup is ligated to a lysine side chain of a target protein by the Pup ligase.

Target protein

Pup ligase

To determine if a protein X is pupylated, puried X is incubated with Pup and the Pup ligase over
night. Trypsin, which hydrolyses (by addition of water of 0.018kDa) proteins next to lysines (K)
and arginines (R), is added and the masses of the peptides are determined by mass spectrometry
(red) . A control reaction where Pup was not added was processed similarly (black). The masses
were rounded to two decimals. Note that the Pup fragments are out of the range to be detected.

Relative intesity

Control

+ Pup

Molecular weight [kDa]


The sequences of protein X and Pup as well as the molecular weight (m a) of the peptides are
indicated below.
trypsin

Protein X

0.96

trypsin

2.11

trypsin

1.05

trypsin

1.22

1.33

trypsin

6.70

0.26

Indicate if each of the following statements is true or false.


A. Trypsin eiciency is such that several peptides are left partly uncut.

B. Hydrolysis by trypsin is inhibited next to lysines that are pupylated.


C. Under these conditions, pupylation is specic to a single lysine.
D. The target protein is pupylated at about 90%.
A. True

B. True

C. True

D. False

Original commentary
Note
The masses given in the second gure correspond to tryptic monopeptides. During trypsin hydrolysis, water (m a 18.02
Da) is added, therefore, when calculating the mass of a dipeptide, the mass of water needs to be substracted.
Correct answers
A true
Masses corresponding to dipeptides can be observed in the black sepctra, e.g. the mass 3.05 kDa corresponds to a
dipeptide formed by the two rst tryptic peptides.
B true
The peak of the pupylated peptide at 2.78 kDa corresponds to a pupylated dipeptide and no pupylated monopeptide
can be observed (the peak would be at 1.46 kDa for this lysine).
C true
Only two peaks (2.78 and 3.81 kDa) are present in the red spectra (with Pup) but not in the black one. The peak at
2.78 kDa corresponds to a pupylated dipeptide formed by the last two peptides of the protein, the peak at 3.81
corresponds to a pupylated tripeptide formed by the last three peptides of the protein.
D false
Two peaks (1.22 and 1.33 kDa) are reduced after ligation with Pup. The peak at 1.33kDa goes from 0.6 to about 0.25
relative intensity. Therefore, the protein is pupylated to about 1-(0.25/0.6) = 58%, not 90%. Furthermore, the peak
corresponding to a pupylated peptide (2.78 kDa), has a relative intensity of only 0.4, added to the peak corresponding
to the pupylated tripeptide (3.81 kDa) with a relative intensity of about 0.15, the pupylation level would be only about
55%.

Cell-, molecular- and microbiology

Upon light activation, rhodopsin proceeds to Meta-states, of which Meta-I and Meta-II are in a
dynamic equilibrium. This shifts towards Meta-II in the presence of arrestin (Arr) as some Meta-II
binds to arrestin. To study the minimal functional unit of rhodopsin, the dierence in absorption
of rhodopsin (absorption after light activation minus absorption before light activation) is
measured at dierent arrestin concentrations ([Arr]) for both monomeric and dimeric rhodopsin,
as shown below. Meta-II is known to show a stronger absorption at 380 nm than Meta-I.
Dimeric

A(after) - A(before)

Monomeric

Wavelength (nm)

Based on these results, indicate if each statement is true or false.


A. Before light activation, rhodopsin absorbs light at 500 nm.
B. The fraction of rhodopsin in the Meta-II state increases linearly with the concentration of
arrestin.
C. Dimeric rhodopsin binds tighter to arrestin than monomeric rhodopsin.
D. Monomeric rhodopsin is the minimal functional unit.
A. True

B. False

C. False

D. True

Original commentary
Correct answers
A true
the negative dierence absorption at 500 nm shows that dark-state rhodopsin absorbs light at 500 nm.
B false
as can be seen from the graph, the dierence in absorption between 2 and 4 M is smaller than the one between 1 and
2 M. If Meta-II would increase linearly with the arrestin, it should be equal to the double. Furthermore, one can
think, that as more arrestin is added, less free rhodopsin is present, therefore, less increase in absorption is observed.
C false
with monomeric rhodopsin, the increase of absorption at 380 nm is bigger than with oligomeric rhodopsin
D true
monomeric rhodopsin is enough for arrestin-binding, it is the minimal functional unit

Cell-, molecular- and microbiology

The following picture depicts part of the binding site of a protein which recognizes RNA
specically over DNA, in complex with two RNA bases (B1 and B2).

Color code for atoms

Ala

C of the protein
C of the RNA
O
N
P

Tyr

B2

A
Asn

Base

Ribose

B1
Phosphate
B
Arg

Indicate for each of the following interactions if it provides specicity for RNA over
DNA.
A. Hydrogen bond A with Tyr
B. Hydrogen bonds B with Arg
C. Hydrogen bond C with Ala
D. Hydrophobic stacking D between Tyr and B2
A. True

B. False

C. False

D. False

Original commentary
Correct answers
A true
Only RNA, not DNA, has a hydroxyl group at the 2' position in the sugar.
B false
Both RNA and DNA have phosphate groups in the backbone.
C false
This hydrogen bond is specic for B2, but B2 is a guanine present both in RNA and DNA (the only dierence in term of
bases between RNA and DNA is uracil vs. thymine, both are pyrimidines, small bases with only 1 ring).
D false
Hydrophobic stacking is a non specic interaction which is found in interactions both with DNA and RNA.
References
Handa et al, Nature (1999)

Cell-, molecular- and microbiology

The following gure depicts the shikimate pathway, which is part of the synthetic pathway
producing aromatic amino acids in bacteria.
HO

COO-

COO-

COO-

OH

OH

OH NADPH+H+

NADP+ HO

OH

OH

OH
Y
Z

COO-

c
COO-

PO4 3-

2- O3

PO
COO-

2- O3

PO

OH

COO-

2- O3

OH

PO

OH

Indicate if each statement is true or false.


A. In reaction a, X represents H2O.
B. In reaction b, the substrate is reduced.
C. In reaction c, Y represents ADP or GDP.
D. Reaction d is a phosphorylation of the substrate.
A. True

B. True

C. False

D. False

Original commentary
Correct answers
A true
A is a dehydration, the -OH group as well as a hydrogen are removed and form water.
B true
NADPH+H+ is a reducing agent. The oxidation number of the carbon of the carboxyl in the substrate changes from +2
to +1 in the product of b: this is a reduction.
C false
GDP cannot phosphorylate substrates, Y represents here ATP.
D false
In d, a phosphate group is released as part of the condensation of the 2 substrates.

Cell-, molecular- and microbiology

To separate DNA fragments on an agarose gel, one liter of 10x TAE buer (consisting of Tris
base, Acetic acid and EDTA) has to be prepared. The desired concentrations for TAE as well as
the available stocks are as follows:
Chemical

Desired concentration

Available stocks

Tris base

Powder (121 g/mol)

Acetic acid

Solution 100 %

EDTA

Solution 0.50 M

Distilled water

Indicate if each of the following statements is true or false.


A. 4.84 g of Tris base are required.
B. 11.4 ml of acetic acid are required.
C. 0.2 ml EDTA are required.
D. Tris base, EDTA and acetic acid have to be added to 1 l of distilled water.
A. False

B. True

C. False

D. False

Original commentary
Correct answers
A false
Tris base: 0.4 mol/l * 1 l * 121 g/mol =48.4 g
B true
acetic acid: dilution: V1 = c2*V2/c1 = 1.14% * 1l/100% = 11.4 ml
C false
EDTA: dilution: c1*V1 = c2*V2 <=> V1 = c2*V2/c1 = 0.01 mol/l * 1 l / 0.5 mol/l
D false
Tris, EDTA and acetic acid are rst added to a smaller amount of water. When Tris is completely dissolved, the volume
is adjusted to 1 l with water. If they were added to 1 l of water, the nal volume would be greater than 1 l and the
concentrations would be wrong.

Cell-, molecular- and microbiology

A protein complex consisting of two polypeptides A and B needs to be expressed. To ensure


proper folding and complex formation, a eukaryotic host is employed. To achieve this, eukaryotic
cells are co-transfected with two artical gene constructs. In the rst construct (above), the gene
coding for Transcription Factor X is under the control of Promoter P1. In the second construct
(below), the genes coding for A and B are separated by an internal ribosomal entry site (IRES)
and under the control of a specic Promoter P2. In the presence of tetracycline, X can bind to the
Operator O and activate P2, as shown below (pA indicates polyadenylation sites).

Indicate if each of the following statements is true or false.


A. The expression of the genes coding for A and B can only be induced by tetracycline if P1 is
active.
B. In the presence of tetracycline, the gene coding for B is expressed at higher levels than the
gene of A if the ribosome binds stronger to the 5'-cap than to the IRES.
C. If the DNA-binding domain of X is removed, the genes coding for A and B are expressed even
in the absence of tetracycline.
D. The length of the polyadenine tail of the mRNA of X inuences the amount of synthesis of
protein X in the cell.
A. True

B. False

C. False

D. False

Original commentary
Correct answers
A true
A and B can only be expressed if X is present in the cell, for this, P1 needs to be active.
B wrong
gene B would be more expressed than A if the ribosome would bind stronger to the IRES than the 5'-cap
C false
if the DNA-binding domain of X was removed, X could not bind the DNA and activate gene expression, so neither A nor
B would be produced.
D true
an mRNA with a longer or shorter poly-A tail may survive longer in the cell and can be translated during a longer time,
so more X-protein could be produced.
References
Fussenegger et al, Biotechnol. Prog. (2001)

Cell-, molecular- and microbiology

The toxicity of three dierent chemicals 1-3 was tested in vitro on neural crest cells by counting
the number of viable cells (black circles) and the number of cells migrating (red triangles). The
average and standard deviation of multiple replicates obtained at dierent concentrations are
plotted relative to the average counts from untreated cultures.
Response relative to untreated

Chemical

untreated

Chemical

untreated

Chemical

untreated

Concentration

Indicate for each of the following statements if it is true or false.


A. These results are compatible with Chemical 1 aecting receptors receiving migration signals.
B. These results are in agreement with Chemical 2 aecting cell viability only.
C. 100 to 500 nM of Chemical 3 are likely benecial for the viability of neural crest cells.
D. Measuring viability of neural crest cells is suicient to establish maximum permissible
concentrations of these chemicals.
A. True

B. True

C. False

D. False

Original commentary
Correct answers
A true
Chemical 1 aects only migration, not viability of neural crest cells. This could be explained e.g. by chemical 1
inhibiting receptors involved in migration.
B true
Even if the migration is reduced with increasing concentrations of chemical 2, it is so with the same amplitude as the
viability is reduced. The reduction in viability is enough to explain the reduction in migration (since dead cells do not
migrate).
C false
Even if the viability in the presence of 100 to 500nM of chemical 3 is higher than the viability of untreated neural crest
cells, this is due to the large standard deviation in the measurements, not to a benecial eect of chemical 3.
D false
As for chemical 1, the response in cell viability and migration is not always similar. Since during embryonic
development, neural crest cells need to migrate to create dierent tissues, it is important to take cell migration into
account when establishing maximum permissible concentrations.

10

Animal anatomy and physiology

The contraction of a muscle ber is triggered by an electric impulse that leads to the release of
Ca2+ ions from the sarcoplasmic reticulum (SR) into the cytosol. After contraction, the Ca 2+ ions
are pumped back to the SR by sarcoplasmic reticulum Ca2+ ATPases (SERCA). To better
understand the relative energy consumption of the actual contraction and the pumping of Ca 2+
ions, skeletal muscle bers of the frog species Xenopus laevis were treated with the chemicals
BTS and cyanide. BTS inhibits cross-bridging between myosin and actin and cyanide inhibits the
cytochrome c oxidase. The gure below shows the basal and peak cytosolic Ca 2+ concentration as
well as the pH of individual bers during repeated contractions under natural conditions (open
circles), in the presence of BTS (lled circles) and in the presence of both BTS and cyanide (red
triangles).
Peak

Basal

200

400

600

Time after onset of repeated contractions (s)

Indicate if each of the following statements is true or false.


A. The hydrolysis of ATP at both myosin heads and SERCA contributes to tiring of muscle bers.
B. In these bers, aerobic respiration is an almost exclusive source of ATP after about 200 sec of
repeated contractions.
C. The phosphorylation of ADP from creatine phosphate releases H+ ions.
D. In the presence of cyanide only, the acidication of the cytosol over time was expected to be
intermediate between the reduction observed under natural conditions and in the presence of
both BTS and cyanide.
A. True

B. False

C. False

D. False

Original commentary
Correct answers
A true
This can be seen easily from the right gure where the pH decreases over time (due to anaerobic glycolysis) is
substantial even when no ATP is hydrolyzed at actin heads (in the presence of BTS).
B false
The decrease in pH over time is due to anaerobic glycolysis. Since the pH decreases well beyond 200 seconds, aerobic
respiration may be contributing substantially to the production ATP, but is far from being the only important source. In
the rst 60 seconds, substantial sources of ATP are the stock of ATP and the phosphorylation of ADP from creatine
phosphate.
C false
It actually absorbs H+ ions (ADP + PCr + H+ = ATP + Cr). This can easily be seen in the right gure since the pH is
increasing at rst.
D false
In the absence of aerobic respiration, the drop in pH is expected to be even quicker than for the natural conditions.
References
Nogeira et al, AJPRICP (2013)

Walsh et al, Experimental Physiology (2008)

11

Animal anatomy and physiology

Myasthenia gravis is an autoimmune disease caused by autoantibodies that competitively bind


and block postsynaptic nicotinic acetylcholine receptors on neuromuscular junctions of skeletal
muscles.
Indicate for each of the following statements if it is true or false.
A. Reduced motility of the intestines is a likely symptom of this disease.
B. Repetitive muscle contractions without relaxation (tetanic contractions) are a likely symptom
of this disease.
C. Drugs slowing down the degradation of acetylcholine in the synaptic clefts are likely to
reduce symptoms.
D. Drugs inhibiting cytotoxic T-cell proliferation are likely to reduce symptoms.
A. False

B. False

C. True

D. False

Original commentary
Correct answers
A false
The guts contain no skeletal but smooth muscles. The latter are usually not aected by Myastenia gravis due to the
absence of neuromuscular junctions.
B false
The antibodies block the access of acetylcholine to the post-synaptic receptor (as mentioned in the stem), which cause
a paralyzation or a reduced postsynaptic neuronal stimulation.
C true
Slowing down the degradation of Acetylcholin (for instance by inhibiting the Acetylcholinesterase) allows it to act
longer on the post synaptic receptors, which leads to a stronger signal since antibodies bind reversible on the ACh
Receptors (they are in competition).
D false
B-Lymphozytes not T-Lymphozytes produce antibodies.

12

Animal anatomy and physiology

In blood capillaries, uid movement (Jv) across the capillary membrane, between the lumen of the
capillary and the interstitial space, depends on the dierence in hydrostatic and oncotic
pressures between these two compartments. (Oncotic pressure is a form of osmotic pressure
exerted by proteins.)
The following equation applies for Jv
Jv = Kf x [(Pc Pi) (c i)]
with
Pc: the capillary hydrostatic pressure
Pi: the interstitial hydrostatic pressure
c: the capillary oncotic pressure
i: the interstitial oncotic pressure
Kf: the ltration coeicient
: the reection coeicient
Indicate for each of the following situations if the risk of edema (accumulation of uid
in the interstitial space) is increased.
A. Tennis competition in the sun leading to dehydration
B. Wearing anti thrombotic stockings
C. Inammation leading to increased blood vessel permeability
D. Proteinuria (excessive protein excretion by the kidneys)
A. False

B. False

C. True

D. True

Original commentary
Correct answers
A false
Excessive sweating causes a reduction of intravasal uid which lowers the hydrostatic pressure in the capillaries.
B false
Anti thrombotic stockings increase the interstitial hydrostatic pressure and peripheral edema are decreased
consecutively.
C true
Increased blood vessel permeability/leak -> plasma proteins move to the interstitium increasing the interstitial oncotic
pressure -> more water movement to the interstitial space.
D true
Proteinuria reduces the concentration of proteins in the blood resulting in a reduced capillary oncotic pressure.

13

Animal anatomy and physiology

Leukocytes

Neutrophils (%)

Proper treatment of chronic bacterial infections of articial implants (e.g. prosthetic joints)
requires expensive and demanding surgery. Unfortunately, such infections are often diicult to
distinguish from non-infected implant failures. To propose new diagnostic tests, the absolute
leucocyte count (A) and the fraction of neutrophils among white blood cells (B) were measured in
34 patients with true prosthetic joint infections and 99 patients with known non-infected implant
failures. For each test, dotted lines indicate proposed cut o values to diagnose a true infection.

prosthetic joint infection

prosthetic joint infection


non-infected implant failure

non-infected implant failure

Indicate if each of the following statements is true or false.


A. Raising the cut o of absolute leucocyte counts to 7000/l would lower the risk of false
positive results for a prosthetic joint infection.
B. With the leucocyte cut o at 1500/l, more than 90% of patients with a prosthetic joint
infection are expected to be correctly diagnosed.
C. Lowering the cut o of the fraction of neutrophils to 50% would prevent missing prosthetic
joint infections in the future.
D. These results suggest that the fraction of neutrophils is a better diagnostic test than leucocyte
count.
A. True

B. True

C. False

D. True

Original commentary
Correct answers
A true
By rising the cut o less patients without an infection would get the incorrect diagnosis (but less real infections would
be detected).
B true
With the cut o of 1700 /l, 2/34 Pat with a real joint infections get missed, so 32/34 = 94% were correctly diagnosed.
C false
By lowering the cut o all patients with a true infection in the study would be detected. However, given that 2 out of
34 patients show low proportion of neutrophils, there is no guarantee that in a larger sample no individual with an
even lower proportion was present. Given the distribution it seems even likely.
D true
Using the neutrophil count, more patient with a prosthetic joint infection were correctly diagnosed (33/34 vs. 32/34 for
the leucocyte count) and less patients without an infection got false-positive results (2/99 vs. 12/99 for the neutrophil
count).
References
Trampuz et al, AJM (2004)

14

Animal anatomy and physiology

While the glomerular ltration rate (GFR) is determined by blood pressure in the glomerulus, it
does not directly reect the systemic blood pressure. Instead, a stable GFR is maintained by
either dilation or constriction of the aerent (ow in) and eerent (ow out) arterioles of
glomeruli by an autoregulation mechanism. Several drugs interfere with this mechanism as
side-eects. Those include non-steroidal anti-inammatory drugs (NSAIDs) that reduce the
capability of the aerent arterioles to dilate, as well as angiotensin-converting-enzyme inhibitors
(ACEIs) that inhibit the production of Angiotensin II, and therefore reduce the capability of the
eerent arterioles to constrict.
Indicate for each of the following statements if it is true or false.
A. NSAID intake reduces glomerular blood ow.
B. ACEI intake reduces glomerular blood pressure.
C. The eects of NSAID and ACEI on the GFR may compensate each other when taking both
drugs together.
D. While a chronic overproduction of aldosterone can be treated with ACEIs, using an
Aldosterone antagonist aects the autoregulation mechanism less.
A. True

B. True

C. False

D. True

Original commentary
Correct answers
A true
A side eect of NSAIDs is to inhibit dilation of the aerent arterioles. Hence the glomerulus is unable to increase blood
ow through that mechanism.
B true
If the glomerular ltration pressure is not high enough, Renin is produced, which coverts the pre-hormone
Angiotensinogen into Angiotensin I, which is then converted into Angiosin II by the Angiotensin-converting-enzyme. By
inhibiting this enzyme, the ability to constrict the eerent arteriols is reduced (as mentioned in the stem). In addition,
the production of Angiotensin II rises the blood pressure by rising the vascular pressure and rising the production of
Aldosterone, which itself rises the renal reuptake of Natrium and water. Inhibiting this further decreases blood
pressure system wide.
C false
While NSAID decrease the blood ow into the glomerulus, ACEI increase the outow of blood. Hence both lead to a
reduction on the glomerular pressure and lead to a reduction of the GFR.
D true
An Aldosteron overproduction can be treated by inhibiting the Angiotensin-converting-enzyme, as this leads to a lower
level of Angiotensin II, which is a stimulant for the production of Aldosteron. However, due to the larger role of
Angiotensin, a direct antagonist of Aldosteron implies a smaller eect, including the one described in this question.
References
Campell Biology

15

Animal anatomy and physiology

The respiratory quotient (RQ) of an adult woman was measured at 0.7, along with an oxygen
concentration of 170ml/l in her exhaled air. The RQ is the ratio between the eliminated amount of
carbon dioxide and the absorbed amount of oxygen of the body. The metabolization of glucose and
palmitic acid is as follows:
Glucose: C6 H12O6 + 6 O2 6 CO2 + 6 H2O
Palmitic acid: C16H32O2 + 23 O2 16 CO2 + 16 H2O
Indicate for each of the following statements if it is true or false.
A. The woman adds about 119ml of CO2 to every liter of exhaled air.
B. If the woman was metabolizing solely glucose, her RQ would be larger than when solely
metabolising palmitic acid.
C. The measurements are consistent with the woman solely metabolizing palmitic acid.
D. If the woman was forced to sprint for a few minutes, her RQ is expected to decrease rapidly.
A. False

B. True

C. True

D. False

Original commentary
Correct answers
A false
The adult woman is exhaling 170ml of O2 per liter of air. Since the concentration of oxygen in the atmosphere is about
210 ml/l, the woman is absorbing around 40 ml of oxygen per liter of air. Given an RQ of 0.7, 28 ml of carbon dioxide
(and not 119ml) must be eliminated per liter of exhaled air.
B true
An RQ of about 1 is expected when metabolizing glucose, but only 0.7 in the case of palimitc acid. Note that the
students do not need to make the full calculations to answer this question, but just observe that there is an imbalance
of O2 absorbed and CO2 eliminated when metabolizing palimitc acid, compared to a balance when metabolizing
glucose.
C true
Metabolizing solely palmitic acid requires an absorption of 23 O2 per 16 CO2 eliminated (see equation 2). Hence the
expected RQ is 16/23=0.69565, or 0.7 when rounded to the signicant digits provided in the question.
D false
Short but heavy exercise leads to a large usage of anaerobic metabolism by muscle cells. At rst, this is not expected
to change the RQ at all as lactate fermentation does neither eliminate CO 2 nor absorb O2. However, through time,
Lactate build up leads to an increase in RQ due to an inhibition of fatty acid metabolism. While the students are not
required to know the latter, they should know that anaerobic metabolism cannot lead to a decrease in RQ as less O 2 is
absorbed.

16

Animal anatomy and physiology

Mammalian herbivores use dierent strategies to digest cellulose. Ruminants (e.g. cattle) use
multiple stomachs, whereas monogastrics rely on an extended caecum or colon.
Indicate if each of the following statements is true or false.
A. The relative abundances of dierent amino acids in the small intestine of ruminants dier
from the relative abundances of the swallowed food.
B. Ruminants eat their faeces digested in the caecum to cover their nutritional need.
C. In monogastric herbivores, the absorption of nutrients occurs primarily in the colon.
D. The majority of the bacteria in the stomach of monogastric herbivores are able to produce
cellulase.
A. True

B. False

C. False

D. False

Original commentary
Correct answers
A true
Microorganisms metabolise anorganic nitrogen to build their own proteins which have a dierent amino-acid-pattern
than the food they swallowed. In the abomasum, microorganisms are killed by the HCL and their proteins are digested
from the ruminant.
B false
Not ruminants but monogastric herbivores like rabbits have to eat their faeces from the coecum.
C false
In most monogastric herbivores, it is still the small intestine where most nutrients are absorbed. That is why several
monogastric herbivores eat their feaces.
D false
Monogastric stomaches are not supposed to host bacterias.

17

Animal anatomy and physiology

Flow (l/s)

To assess respiratory function, the ow and the volume of the exhaled air are measured during a
forced expiration (positive ow) followed by a full inspiration (negative ow). Shown below are
the measurements from four dierent patients with an airway problem. The black dotted line
indicates normal respiratory function.

Volume (l)

Indicate if each of the following statements is true or false.


A. The results of Patient 1 are expected if he had his left lung removed.
B. The eect of the medication to reduce the symptoms of Patient 2 on the airways is similar to
that of the parasympathetic nervous system.
C. No diagnosis can be established for Patient 3 because of a cough attack.
D. Patient 4 is likely suering from an airway obstruction in the thorax.
A. True

B. False

C. True

D. False

Original commentary
Correct answers
A true
Restrictive lung diseases reduce the lung volume which is shown/documented in patients ow/volume graph #1.
B false
The patient suers from a obstructive disease. Hes volume is normal, but the ow is reduced. Sympathic eect on the
bronchial system causes an dilatation, the parasympathic eect an obstruction. An adaequate drug has either to
inactivate the parasympathetic nervous system or activate the sympathetic nervous system.
C true
The results shown are indeed typical for a cough attack in which a rhythmic pulse of fast expiration are observed.
D false
As mammals have a negative pressure breathing (rise of intrathoracal volume causes a negative intrathoracal pressure
leading to inspiration), an extrathoracal stenosis (obstruction) causes a xed air-ow reduction apparent during
inspiration.
The opposite is true for an intrathoracal obstruction, because the negative pressure during inspiration widens the
intrathoracal air ways. Vice-versa in expiration.

18

Animal anatomy and physiology

The following gure shows an experiment in which a dorsal lip from a darkly pigmented donor
embryo was transplanted to the ventral ectoderm of a lightly pigmented recipient embryo which
was allowed to develop into a tadpole. The developing second body axis consisted mostly of
non-pigmented cells.

Indicate for each of the following statements if it is true or false.


A. The second body axis came solely from the transplanted dorsal lip.
B. Transplanting the presumptive ectoderm to a host in the neurula stage would likely result in a
tadpole with two body axes.
C. Cell fate is predetermined and results from cell-intrinsic properties.
D. If the presumptive endoderm was transplanted instead of the ectoderm, the secondary body
axis would consist mostly of pigmented cells.
A. False

B. False

C. False

D. False

Original commentary
Correct answers
A false
The experiment shows, that the resulting tadpole on the ventral side had a lightly pigmented surface too, so this cells
originate from the receiving embryo.
B false
A second body axis can only be induced when the cells are still able to dierentiate in all tissues, this is not anymore
the case at the neurula stage.
C false
The grafted cells are able to induce neurulation in the receiving embryo where cell fate was not to neurulate.
D false
The endoderm would not undergo neurulation and hence would not induce the cells to develop a secondary body axis
at all.

19

Animal anatomy and physiology

Three blood groups have been characterized in cats, all of which are encoded by a single gene
with three alleles, of which allele A is dominant over allele B, and allele AB is dominant over B,
but recessive to A. Most cats with blood groups A or B have anti-B or anti-A antibodies,
respectively. Cats with blood group AB do not produce either antibodies anti-A nor anti-B.

Antigens

Produced antibodies
Anti-A
Anti-B
Anti-AB
+
+
-

A
B
AB

The gure below shows the results of blood transfusion compatibility tests performed for a
mother cat (M), her kitten (K) and two potential father cats (P1 and P2). The cards consist of
three circles that contain anti-A (A) and anti-B (B) antibodies, or no antibodies at all as a negative
control (-). When adding a drop of blood to the circles, the occurrence of an agglutination
reaction becomes visible (red dots).

P1

P2

Indicate if each of the following statements is true or false.


A. Mixing blood of kitten K with serum from P2 should lead to agglutination.
B. M could receive erythrocytes from P2.
C. A back-cross between mother M and kitten K might donate erythrocytes to P2.
D. These results suggest that P1 is the more likely father of K than P2.
A. True

B. True

C. False

D. False

Original commentary
Correct answers
A true
P2 produces anti-B, which are present in its serum and would cause an agglutination of erythrocytes of kitten K.
B true
The P2 serum has no anti-A and causes therefor no agglutination with antigens of erythrocytes in M.
C false
A cat with the blood group A has anti-B, which react both with AB and B antigens on erythrocytes. Since the mother
has genotype AB/B and the kitten B/B, any ospring of them has either blood group AB or B.
D false
The negative control indicates that the test failed. So the genotype of P1 is unknown and hence these results do not
suggest anything. Note, however, that if P1 had indeed blood group AB, he would have the same probability as P2 to

be the father of K. The corresponding probability for P2 is either 0% if he had genotype A/B or A/AB or 25% if he had
genotype A/B.

20

Animal anatomy and physiology

The urea-to-creatinine ratio is used to assess renal function. It is calculated by dividing blood
urea concentration by blood creatinine concentration. Urea and creatinine are both able to freely
pass the glomerular ltration barrier. However, while creatinine is not reabsorbed, a percentage
of urea is reabsorbed in the collecting ducts. An elevated rate of reabsorption is only observed
when the total blood volume is increased.
Indicate if each of the following statements is true or false. Compared to a healthy
individual, the urea-to-creatinine ratio is expected to be higher
A. ... in a patient suering from an acute obstruction of the urethra (urinary retention).
B. ... in a patient suering from an acute necrosis of the collecting duct epithelium.
C. ... in a patient suering from dehydration.
D. ... in a healthy individual after intensive exercise but with suicient water intake.
A. False

B. False

C. True

D. False

Original commentary
Correct answers
A false
Urinary retention aects urea and creatinine equally, and hence does not lead to a change in the ratio.
B false
Less urea reabsorption leads to a decrease in the ratio.
C true
Due to volume depletion, a higher percentage of urea is reabsorbed in the kidney, which leads to a larger ratio.
D false
In case of intensive exercise, muscles release more creatinine, and hence the ratio is decreased.

21

Animal anatomy and physiology

Figure A illustrates the results of an examination of the visual eld of a patients left and right
eyes, whereby dark areas indicate poor, and white areas good visual reception. The way sensory
information is received by the eyes and transferred to the visual cortex is presented in gure B.
Visual elds
Left

Left

Right

Right

Indicate if each of the following statements is true or false.


A. After an injury of the left visual cortex, a patient would lose vision on its left side.
B. A tumor of the hypophysis, situated below the optic chiasma, usually causes a loss of view of
the lateral visual elds of both eyes.
C. The visual eld of the patient above is most likely caused by a problem between its optic
chiasm and both retinae.
D. Total visual loss from only one eye may be caused by a trauma of the eyeball or an
inammation of the optic nerve of the blind eye.
A. False

B. True

C. False

D. True

Original commentary
Correct answers
A false
It causes a loss of function of the retinal receptors on the left side of both eyes which causes a visual loss of the right
side from patients view.
B true
The hypophysis is situated right below the optic chiasm. Tumors of the hypophysis consequently mainly aect those
nerves crossing at the optic chiasm, which are the nerves innervating the retinal receptors an the medial part of the
retina causing lateral visual eld defects.
C false
a lesion between the optic chiasm and retinae would aect both eyes but would most likely cause blindness in both
visual elds (right and left) of both eyes. A dysfunction of both visual pathways between the optic chiasm and the
retinae is much more unlikely to be the reason for the indicated visual eld than a single or multiple lesion(s) aecting
both orange part of the optic nerve (and not aecting the blue ones at the same time). The visual elds given are
typical for a lesion between the optic chiasm and the left visual cortex.
D true
A lesion aecting 100% visual eld of only one eye is usually located between the optic chiasm and the retina.
References
Duanes's Ophtalmology: E-book: Evaluation of Visual Function

22

Plant anatomy and physiology

The gure shows a schematic and representative cross section through a leaf of an angiosperm
plant. Vascular bundles are represented by circles and sclerenchyma bers by black surfaces.
Additionally, the position of trichomes and stomata is indicated. The relative position of the
vascular bundles is constant along the leaf.

Indicate if each of the following statements is true or false.


A. The leaf shown most likely represents a monocotyledon plant.
B. The plant is most likely to be found in wetlands.
C. The overall leaf shape is expected to be long and thin rather than oval or round.
D. In a freshly cut section of this leaf the sclerenchyma bers can be recognized by its intensely
green color.
A. True

B. False

C. True

D. False

Original commentary
Note
The section belongs to a fescue (Festuca) out of the family of Poaceae.
Correct answers
A true
The regular position of vascular bundles that is constant all along the leaf indicates a parallel pattern of leaf veins
without branching. This is a typical trait of monocotyledons.
B false
The stomata are situated in cavities of the leaf surface and protected by trichomes. This is a typical trait of drought
adapted plants, reducing transpiration.
C true
The amount of supporting tissue (sclerenchyma) within the thin leaf is a hint to a very long length. A relatively short
(laceolate or round) leaf would not need such a tissue.
D false
Sclerenchyma contains dead cells without cytoplasma. Therefore they do not contain chlorophyll and appear colorless
in a fresh section.

23

Plant anatomy and physiology

Plant organelles can be isolated from plant lysate by multiple rounds of centrifugation and
washing. To distinguish between dierent organelles, centrifugation fractions are subjected to
simple assays before and / or after incubation for 30 minutes under specic conditions. Possible
assays include:
1) measuring the concentration of glucose and other aldoses using a Fehling reaction,
2) detecting the presence of DNA by measuring absorption at 260 nm,
3) observing gas bubbles.
Indicate for each of the following statements if it is true or false.
A. A Fehling assay before and after incubating in light distinguishes fractions of chloroplasts
from those of amyloplasts.
B. A Fehling assay before and after incubating with glucose distinguishes fractions of Golgi from
those of mitochondria.
C. Fractions of endoplasmic reticulum and nuclei can be distinguished by incubating with lipases
and proteases, followed by centrifugation and measuring absorbance at 260 nm in both
supernatants.
D. The presence or absence of gas bubbles after incubating with H2O2 distinguishes fractions of
peroxisomes from those of endosomes.
A. True

B. False

C. True

D. True

Original commentary
Correct answers
A true
Amyloplasts convert glucose in starch that does not react with the Fehling reagent, whereas in light, chloroplasts
produce sugars that will turn the Fehling solution blue.
B false
Neither Golgi nor Mitochondria are involved in the central sugar metabolism.
C true
Nuclei treated with lipases and proteases will liberate DNA that stays in the supernatant after pelleting the nuclei
debris. Endoplasmic reticulum does not contain DNA, therefore even if lipases break it down, no change in absorption
at 260nm will be observed.
D true
Peroxysomes transform H2O2 in H2O and O2, thereby liberating O2 bubbles, whereas endosomes do not.
References
Lang, Plant Cell Rep (2011)
Peroxisome Database

24

Plant anatomy and physiology

According to the ABCE-Model of ower development, activity of genes from dierent classes A, B,
C or E determines the identity of oral parts. Expression of class A genes is needed to determine
future sepals and petals, class B genes to determine future petals and stamen and class C genes
to determine future stamen and carpels. A and C genes inhibit each other's expression.
Dierentiation of each oral part additionally requires activity of class E genes. The gure
illustrates the ABCE-model and shows ower samples of Arabidopsis (A and B), the alpine grass
Poa alpina (C) and two owers of the snapdragon Antirrhinum majus (D; the arrow indicating the
bilateral wildtype, while the radial symmetric to the right is a mutant).
Sepal

Petal

Stamen

Carpel

Indicate if each of the following statements is true or false.


A. The phenotype of Arabidopsis A is best explained by a loss of function of class B genes.
B. The phenotype of Arabidopsis B is best explained by a loss of function of class A and C genes.
C. The phenotype of Grass C is best explained by a loss of function of class E genes.
D. The symmetry of the mutant ower of Snapdragon D is best explained by a loss of function of
class C genes.
A. True

B. False

C. False

D. False

Original commentary
Correct answers
A true
B false
The best explanation is the loss of only class C genes.
C false
The leaves emerging from the owers represent grass seedlings and not altered owers with leaves at the position of
oral parts. The picture shows a viviparous plant with grains germinating before falling o the mother plant.
D false
The mutant shows a loss of bilateral symmetry and not an alteration in the dierentiation of oral parts.
References
Krizek, Nature (2005)

25

Plant anatomy and physiology

The pattern of leaf primordia (future leaves) at the apical meristem is determined by active auxin
transport. Auxin is transported towards the meristem tip. Young primordia act as auxin sinks
through the auxin eux carriers PIN1 and thus decrease the auxin level in nearby meristem cells.
A new primordium will be induced at the place with the highest remaining auxin level. The image
below shows the meristem of Arabidopsis with the primordia 1-9, with 1 being the oldest.

Indicate if each of the following statements is true or false.


A. Assuming PIN1 was only present in the two youngest primordia, its activity must be dierent
in them.
B. The next primordium will emerge at position A.
C. If PIN 1 is inhibited at the stage shown in the gure, the next primordium is most likely to
emerge at position B.
D. In a mutant where only the youngest primordium is acting as an auxin sink, leaves will grow
opposite to each other.
A. True

B. False

C. False

D. True

Original commentary
Correct answers
A true
The characteristic angle of 137 between two subsequent primordia can be only explained with the youngest
primordium being a stronger auxin sink than the second youngest.
B false
According to the order of older primordia the next primordium will emerge between primordium 2 & 5.
C false
In this mutant the primordial pattern would be aberrant, as well as the leaf shape. But it would emerge at a random
position. There is no reason to assume that it will be position B.
D true
In this case the highest auxin concentration would be present directly opposite of the youngest primordium.
References
Reinhard, Nature (2003)

26

Plant anatomy and physiology

Many plants use, among other means, the ratio of red/far-red light to detect other plants
competing for light and react with adaptive growth to avoid shade. The ratio of red to far-red
light is detected by the photoreceptor phytochrome. Phytochrome is converted between two
forms PX and PY, depending on the wavelength of the photon it has absorbed. The ratio between
both forms reects the red/far-red ratio in the environment. A high proportion of P X mediates the
expression of genes responsible for shade-avoiding growth. The gure shows the spectra of
normal daylight (solid line) and daylight ltered through a tobacco canopy (dashed line). Tobacco
plant A has been grown under normal sunlight, plant B under a canopy of older plants.
Far-Red

Spectral photon uence rate

Red

Wavelength

Indicate if each of the following statements is true of false.


A. The genes leading to the shade-avoiding phenotype are expressed when the red/far-red ratio
is between 3 : 1 and 4 : 1.
B. A high proportion of red light increases the proportion of PX to PY.
C. Upward orientation of leaves is a typical feature for shade-tolerant species.
D. The shade-avoiding phenotype includes activation of lateral meristems.
A. False

B. False

C. False

D. False

Original commentary
Correct answers
A false
The maximal ratio is observed in normal light and is slightly above 1.
B false
The opposite is true. Plant B has the genes activated with a red/far-red ration of about 0.5. As the far-red-converted
phytochrome form is dominant, this must be the active form.
C false
Shade-tolerant plants do not need to express the plant B-phenotype of growing out of the shade. In the shade they
keep the normal leaf exposition, allowing to maximize the amount of photons collected by the leaves.
D false
A plant trying to avoid shade by enhanced growth will invest in few elongated stems and not in a multitude of branches
that stay in the shade. Plant B doen't show any branching.
References
Vandenbussche, Current Opinion in Plant Biology (2005)

27

Plant anatomy and physiology

In the wood of trees of temperate climates, annual growth rings are present and reect dierent
growth conditions between years and individuals. Three coniferous trees of the same species
were cut at the same stem height in the same year. The stem cuts in the gure are drawn to the
same scale.

Based on these stem sections, indicate for each of the following statements if it is true
or false.
A. Trees I and II likely grew in the same region, whereas Tree III likely grew in a more distant
region.
B. Tree III is likely to have experienced more climate variation between years than Tree I.
C. Trees I and II may originate from the same forest.
D. The asymmetric pattern of Tree III may be due to constant exposure to strong wind beginning
roughly ten years ago.
A. True

B. False

C. True

D. True

Original commentary
Correct answers
A true
I and II show the same pattern of two periods of narrow rings (bad growth conditions) 2-6 and 9-13 years before they
have been cut. Thus they have been growing in the same climatic conditions present in one same region. Tree III
shows a dierent, much more regular pattern and did not suer those two periods of bad years.
B false
Tree III has relatively regular rings throughout the lifetime on one side of the section. The asymmetric pattern can be
explained by very local eects like a physical obstacle or shadow on one side of the tree.
C true
Local ecological factors such as available light or soil conditions can result in very dierent growth rates even for
neighboring trees.
D true
The rst rings are regular, indicating an equilibrated light supply. The last rings are more and more asymmetric. A
likely explanation is that faster growing trees are competing for sunlight on one side of the tree, whereas on the other
side the tree is still suiciently exposed to sunlight.

28

Plant anatomy and physiology

The second leaf (Leaf 2) of a young and growing plant of wheat (Triticum aestivum) was fed via a
rectangular ap cut symmetrically in the middle of the lamina and brought into a tube with a
feeding solution containing radioactive nickel (63Ni), manganese (54Mn) and zinc (65Zn). After 1,
2, 7 and 28 days, the contents of radioactive elements were measured in dierent parts of the
plant. Concentrations measured in the ap and Leaf 2 are shown in orange in the gure.
1 Day

2 Days

7 Days

28 Days

Leaf 7
Leaf 6
Leaf 5
Leaf 4
Leaf 3
Leaf 2
Flap
Leaf 1
Roots

Leaf 2

Leaf 7
Leaf 6
Leaf 5
Leaf 4
Leaf 3
Leaf 2
Flap
Leaf 1
Roots

Tube with ap inside

Leaf 7
Leaf 6
Leaf 5
Leaf 4
Leaf 3
Leaf 2
Flap
Leaf 1
Roots

Content [cpm (counts per minute) per plant part]

Indicate if each of the following statements is in agreement with the results shown
above.
A. The plants absorbed the entire feeding solution provided before the rst measurement.
B. Nickel is mostly transported to growing organs.
C. Manganese has higher phloem mobility than zinc or nickel.
D. Leaf 2 is turning into a net sugar exporter after Day 1.
A. True

B. True

C. False

D. False

Original commentary
Correct answers
A true
B true
Nickel is rst accumulated in leaf 3, then in leaf 4. After several days when these leaves are grown up nickel is
exported again towards leaves 5-7.
C false
Manganese (known to have low phloem mobility) stays in the second leaf, whereas Zn and Ni is reduced in leaf 2 and
appears in signicant amounts in other parts of the plant.
D false
Leaf two is an "adult" leaf already at the beginning of the experiment. At day one a fraction of nickel already has been
transferred from leaf 2 to 3. As the transfer is done by phloem transport, leaf 2 must already have a net sugar export.
References
Riesen, Journal of Plant Nutrition (2003)
Plant Physiology 5th ed., Lincoln Taiz and Eduardo Zeiger, section Essential nutrients, deciencies, and plant
disorders page 108 and following.
Campbell, Biology (9th ed.), table 37.1 page 791.

29

Plant anatomy and physiology

In xylem, water conduits occasionally undergo embolism, characterized by an inow of air into
the conduit lumen, followed by collapse of the water column. Each event of embolism emits a
sound called ultrasonic acoustic emission (UAE) that can be detected with adequate sensors. The
gure below shows such measurements of an oak tree (Quercus pubescens) during four
subsequent summer days. Radiation of sunlight (RN) and air temperature (T) were measured
simultaneously.

Day

Indicate if each of the following statements is true or false.


A. Embolisms occur during periods of elevated water tension in xylem conduits.
B. The sound intensity of a single UAE event is correlated with the dierence in water potential
between the air and the leaves.
C. The decrease in UAE events over time is likely due to an increase in cloud cover.
D. Embolisms reduce the conductivity of xylem and therefore the water supply of distal tissues.
A. True

B. True

C. True

D. True

Original commentary
Correct answers
A true
The UAEs are recorded during periods of high radiation and temperatures. During this time the water potential in the
surrounding air is very low, leading to increased evaporation. During increased evaporation the dierence in water
potential between leafs and roots is big and therefore the tension forces within xylem is high.
B true
The audibility is indicated by the decibel value. At mornings and evenings the few measured UAEs have lower decibel
values and are therefore less loud. Keep in mind that the decibel scale is logarithmic. The dierence between the
water potential of air and leaves depends on radiation and temperature and is maximal at noon and early afternoon
but lesser at morning and evening. Therefore a relation is given.
C true
The radiation diagram shows how the cloud covers increases (huge variation within short time), leading to a decrease
in overall radiation, and hence to a decrease in water tension within water conduits.
D true
Water transport depends on a permanent water clomn. An embolism is leading to the interruption of a xylem conduit.
Until it is relled, further water transport through this conduit will be null or very limited.
References
Zweifel et al, New Phytologist (2008)

30

Genetics and evolution

In a plant species, the level


for which only a "dark" and
gene G on chromosome 3,
through X5) are genotyped
same chromosome.

of anthocyanin pigments produced is controlled by a single gene G,


a "light" allele are present. To more accurately map the position of
two inbred lines (P1 and P2) are crossed and F2 individuals (X1
at ve single nucleotide variant loci (SNV1 through SNV5) on the
Anthocyanin

Based on these results, indicate if each of the following statements is true or false.
A. One recombination event happened in each parent of X2 between the genotyped loci.
B. F1 individuals are likely showing intermediate levels of anthocyanin.
C. Among the studied loci, SNV3 is closest to gene G.
D. The phenotype ratio in the progeny of a cross between X4 and X5 is 2:1.
A. False

B. False

C. True

D. False

Original commentary
Correct answers
A false
Most likely, a single recombination event happened in only one of the parents, as one of the haplotype is parental
(ACATC) and the other shows a recombination between SNV3 and SNV4.
B false
Gene G is linked to SNV3, and the table lists a heterozygous individual (X1) with an elevated anthocyanin level (which
is the dominant allele). But note that the students do not need to identify the most closely linked locus as for each of
them heterozygous individuals are given and the conclusion would remain unchanged.
C true
This is the only locus for which the genotypes match the phenotypes in a Mendelian fashion.
D false
A 2:1 ratio is impossible for any crossing, as we clearly have dominant-recessive inheritance. So it is possible to
answer this question even if the truly linked locus is not identied.

31

Genetics and evolution

A plant species grows in three dierent habitats A, B and C, and populations from dierent
habitats also dier genetically. To test if some of these genetic dierences are driven by local
adaptation, a so-called reciprocal transplant experiment is carried out where seeds collected from
dierent habitats are grown in all three habitats. After a year, the tness of the plants are
measured by counting the number of viable seeds produced per individual.

Number of viable seeds produced

Native habitat

Experimental habitat

Based on these results, indicate if each of the following statements is true or false.
A. Plants from Habitat A are locally adapted to their native environment.
B. Plants from Habitat B are locally adapted to their native environment.
C. Plants from Habitat C are locally adapted to their native environment.
D. Habitat C is less suitable for these plant species than habitats A or B.
A. True

B. True

C. False

D. True

Original commentary
Correct answers
A true
Plants grown from seeds collected in habitat A show, on average, a much higher tness when grown in habitat A than
seeds collected form other habitats.
B true
Plants grown from seeds collected in habitat B show, on average, a much higher tness when grown in habitat B than
seeds collected form other habitats. Note that the fact that seeds from habitat B perform better in habitat A than in
habitat B is likely due generally better conditions in habitat A.
C false
Plants grown from seeds collected in habitat C follow the habitat quality, but do not indicate that they are specically
adapted to environment C since they do not perform better there than plants adapted to dierent environments.
D true
All plants perform much worse in environment C than environments A or B, inclusing the individuals originating from
this environment.

32

Genetics and evolution

Consider a large and constant population of a diploid organism with non-overlapping generations
and sexual reproduction happening in spring. In addition, there is no dierence in allele
frequencies between sexes and there is no migration or natural selection acting. In such a
population, the frequencies of alleles uctuate at a given rate between generations due to the
random nature of reproduction.
Indicate if each of the following statements is true or false. Allele frequencies are
expected to uctuate at ...
A. ... a higher rate if the population was growing exponentially.
B. ... a lower rate if all individuals had the same number of ospring.
C. ... a similar rate even if there was strong inbreeding.
D. ... a higher rate if the population crashed every winter.
A. False

B. True

C. True

D. True

Original commentary
Correct answers
A false
If a population is growing exponentially, stochasticity in reproduction is reduced due to a larger number of ospring
sampling alleles from the parent generation, and hence allle frequencies uctuate at a lower rate.
B true
If all individuals had the same number of ospring, allele frequencies are almost constant since each individual leaves
a xed number of copies of his alleles in the population. The only stochastic variation left comes from heterozygous
individuals randomly passing one of their alleles per ospring.
C true
While inbreeding decreases the frequency of heterozygotes, it does not lead to a faster change in allele frequencies.
D true
A population crash in winter leads to a recurrent bottleneck which removes alleles randomly from the population and
leaves a smaller number of individuals reproducing in spring. This leads to an increased stochasticity.

33

Genetics and evolution

The eect of various mutations in a gene x coding for a protein X, essential for the synthesis of
leucine, is studied in a haploid yeast. The beginning and the end of the complete sequence of the
coding strand of x is given below.

Indicate if each of the following statements is true or false.


A. Cells with a C T mutation at position 13 produce shorter mRNA of x.
B. Cells with a A T mutation at position 16 are able to grow on a medium lacking leucine.
C. Cells with a T A mutation at position 31 only produce an alternative protein X* missing the
rst 10 amino acids.
D. Cells with an additional G between positions 33 and 34 produce functional X.
A. False

B. False

C. False

D. False

Original commentary
Note
Students are expected to know the start and stop codons. Nevertheless, for those who don't, they can nd out by
looking at the sequence, that ATG is a start codon and TAG a stop codon.
Correct answers
A false
Introducing the mutation C13T results in the TAG stop codon, but the transcription stops only at the transcription
termination sequence independently of the codons
B false
Introducing the mutation A16T results in the TAG stop codon. With a stop codon at the beginning of the sequence, no
functional protein is produced and no leucine can be synthesized by the yeast that would need to take it up from the
medium.
C false
Introducing the mutation T31A results in the ATG start and methionine codon. This will simply result in a methionine
at this position. The eukaryotic ribosome binds at the 5'-cap of the mRNA and starts translation at the rst AUG
encountered. Further AUG are simply translated into methionine.
D false
Introducing a G between positions 33 and 34 results in a shift of the translation frame. Instead of reading ...GAT AGC...
(positions 34-39), the ribosome will read ...GGA TAG... and reaches a stop codon. Translation stops there, the protein
will not be functional.

34

Genetics and evolution

An operon encoding enzymes 1 and 2 is regulated by metabolite X and consists of four sequences
A, B, C and D of unknown function. To elucidate their function, the eect of mutations in the
sequences A-D on the synthesis of the enzymes is assessed in the presence and absence of X.
X present
Mutation in sequence

Enzyme 1

Enzyme 2

X absent
Enzyme 1

Enzyme 2

no mutation

Indicate if each of the following statements is true or false.


A. Enzymes 1 and 2 are likely part of the synthetic pathway for X.
B. The sequence A codes for enzyme 2.
C. The sequence B is the promoter region.
D. The sequence D is the regulatory gene.
A. True

B. False

C. False

D. False

Original commentary
Correct answers
A true
This operon is repressed by X. This kind of negative transcriptional regulation is mostly found in biosynthetic
pathways.
B false
In the absence of X and mutation in A, enzyme 2 is still produced, enzyme 1 isn't, therefore A codes for enzyme 1.
C false
In the case of a mutation in B, enzymes 1 and 2 are produced even in the absence of X, therefore B corresponds to the
regulatory gene.
D false
In the case of a mutation in D, neither enzyme is produced, even in the absence of X, therefore D corresponds to the
promoter region.

35

Genetics and evolution

In cells, most plasmids are supercoiled (a in gure below). While such plasmids can be uncoiled
to relaxed circles (b) using Topoisomerase IA (TopoIA), using restriction enzymes (R) linearizes
plasmids through cutting (c). A linearized plasmid may spontaneously self-anneal and
subsequently be ligated to form a relaxed circle. The ligation reaction can be inhibited by the
addition of phosphatase (AP).

b
AP
Topo IA
a
R

c
In an experiment, a plasmid was treated with dierent restriction enzymes (R1, R2, R3) under
similar conditions and separated on an agarose gel together with an untreated sample (P0) and a
marker consisting of linear DNA pieces. Topoisomerase IA and AP treatments in combination with
R3 were also analyzed, but the tubes were mixed up (R3+E1 and R3+E2 on gel).
(bp)

(bp)

Indicate if each of the following statements is true or false.


A. On this plasmid, the restriction site for R2 is closer to the one for R3 than to the one for R1.
B. The plasmid is about 5000 bp long.
C. R1 cuts more eiciently than R3.
D. E1 represents AP, E2 represents Topo IA.
A. False

B. True

Original commentary
Correct answers

C. True

D. True

A false
R1 and R2 are distant from 1000 bp, R2 and R3 from 2000 bp, even if 2 vector maps are possible:
R1 1000 bp R2 2000 bp R3 2000 bp
R1 1000 bp R2 3000 bp R3 1000 bp
B true
Since the shape of DNA plays a role in the migration, the length of DNA can only be determined by comparing
fragments with the same shape. The plasmid is linearized when using only 1 restriction enzyme. The supercoiled
plasmid from P0 (thick band) is converted to linearized DNA which migrates around 5000 bp (see R1 or R2).
C true
In R3, some uncut supercoiled is still present, only about half was cut, whereas in the same time, R1 could cut
everything
D true
Treatment with AP removes prevents re-formation of relaxed circles, the slowest migrating band disappeared in
R3+E1. Treatment with E2 converted the uncut supercoiled plasmid into relaxed circles, this corresponds to Topo IA.

36

Genetics and evolution

A child aected by a rare genetic disease is born to two healthy parents. The child has a healthy
sister.
Indicate if each of the following statements is true or false.
A. If the disease was known to be segregating in the family of the father, the disease is more
likely autosomal than sex-linked recessive.
B. If the disease was sex-linked recessive, the probability of the sister being a carrier is 50%.
C. If the disease was autosomal recessive, the probability of the sister being a carrier is 50%.
D. If the disease was not known to be segregating in either family, the causal mutation is either
autosomal recessive or, within the family, unique to the child.
A. True

B. True

C. False

D. True

Original commentary
Correct answers
A true
X-linked recessive would imply that the father was aected.
B true
In this case, the mother is carrier and the father healthy. Hence the sister inherits an unaected X chromosome from
the father and has a 50% chance to inherit the aected chromosome from the mother.
C false
In this case, both parents are carriers. But since we know that the sister is healthy, the probability is 2/3 (and not 1/2).
D true
A rare autosomal recessive disease is likely not to show up in an outbred family for many generations. An alternative
explanation for the disease, however, might indeed be a spontaneous mutation being unique to the child.

37

Genetics and evolution

In a small pasture, 500 individuals of two closely related snail species were sampled. Genetic
analyses detected a locus at which none of the individuals were found to be heterozygous, despite
the presence of two alleles in each species. The two species and all genotypes were randomly
distributed in the homogenous habitat.
Snail species

Genotype

Number of snails

Indicate if each of the following statements is a likely explanations of the observed


pattern.
A. These snails generally self-fertilize.
B. Both species experience strong genetic drift due to low population sizes.
C. These snails reproduce hermaphroditically.
D. These snails mate preferably with individuals of the same genotype.
A. True

B. False

C. False

D. True

Original commentary
Correct answers
A true
Self-fertilization would indeed lead to all snails being homozygous.
B false
If the populations were experiencing strong drift through low population sizes, this would lead to a reduction in
diversity. However, there is no reason why this should aect heterozygous individuals preferentially.
C false
Hermaphroditic reproduction does not have any impact on allele frequencies as long as mating is random.
D true
If snails mate only with snails of the same genotype, heterozygous individuals would be very rare in the population.

38

Genetics and evolution

A yeast-two-hybrid assay (Y2H) allows to test if a protein X interacts with another protein Y. In
this assay, the gene coding for X is fused to the gene of a DNA-binding domain (BD) of a
transcription factor T. The gene coding for Y is fused to the gene of the activating domain (AD) of
T. The resulting plasmids are transformed into a yeast strain containing the lacZ gene under the
control of a promoter P, which is specically recognized by BD. Plated on agar with X-gal, the
colonies turn blue if they are expressing LacZ, which is only the case if BD and AD are in close
proximity and hence if X and Y interact with each other.
interacting

not interacting

lacZ

lacZ

blue

white

Indicate if each of the following statements is true of false.


A. Y2H also works if BD alone is suicient to activate transcription.
B. Y2H allows studying interactions between integral plasma membrane proteins.
C. Y2H can give false-positive results if X and Y interact indirectly via a third protein.
D. Y2H can give false-negative results if the binding site for Y on X is situated close to the
terminus at which BD is attached.
A. False

B. False

C. True

D. True

Original commentary
Correct answers
A false
The assay works correctly only if BD and AD by themselves are not enough to activate transcription, but are both
needed.
B false
This in vivo assay works only for soluble proteins. To study interaction of membrane proteins, the membrane proteins
need to be properly folded, which is only possible if they are in lipidic environment.
C true
If I binds to a third protein which binds to II, the assay can give positive results, even if I and II do not interact directly.
D true
The binding site can be distorted if it is too close to the junction to the transcription factor domain.

39

Ecology, ethology and systematics

Consider a currently stable system in which three predatory insect species P1, P2 and P3, feed
exclusively on three herbivore insect species H1, H2 or H3, respectively, all of which feed on the
same limited plant resource R.

Indicate if each of the following statements is true or false.


A. If the plant resource was doubled, the abundance of P2 will increase.
B. If H1 is a stronger competitor than H2, removing P1 leads to a increase of P2.
C. If H3 was severely limited by its predator, the removal of P3 would lead a decrease in the
abundance of P2.
D. The introduction of a top-predator feeding on P1 and P2 is likely to increase the abundance of
P3.
A. True

B. False

C. True

D. False

Original commentary
Correct answers
A true
Doubling the resource should lead to a doubling of all abundances in the system in the long term since the interactions
remain unchanged.
B false
Population size of predators depend on populations size of herbivores. H1 now lacking a predator will increases on
cost of H2. As a consequence the abundance of P2 decreases as well. This is expected unless H2 is limited almost
completely by its predator and not by competition with H1, but even in that case the abundance is expected to remain
unchanged, but not increase.
C true
Population size of predators depend on populations size of herbivores. H3 when having its predator removed, is able to
reduce H2, P3 must be a limiting factor for H3. H2 when having its predator removed does not seem to be able to
aect H3. Therefore the eect of the predator is not limiting enough to lead to signicant loss of competition against
H3.
D false
The top-predator will decrease the abundance of P1 and P2 and therefore lead to an increase of H1 and H2. As both
those herbivores are in competition with H3, the abundance of H3 is likely to decrease, leading to a dcrease in the
abundance of P3.

40

Ecology, ethology and systematics

The following gure illustrates the result of an experiment during which a person was alone in a
room and was allowed to freely choose the awake and sleep periods by turning a bright light on
and o. The consecutive time of light for each day is shown as a rectangle with times at which the
person chose to eat a meal indicated by black bars. While the person had no time cues from the
outside world during the days shown in orange, the room was exposed to natural light during the
days shown in white.

Days

Time of the day

Indicate if each of the following statements is true or false.


A. Without external cues, the person chose increasingly longer periods of light.
B. The endogenous clock of this person cycles on a 28.5 h rhythm.
C. These results are in agreement with bright light being a cue to delay the sleeping phase.
D. These results suggest that the endogenous clock of this person can readjust completely within
two days.
A. False

B. False

C. True

D. False

Original commentary
Correct answers
A false
While the periods of light were extended in the absence of external cues, there us no indication the those periods got
longer during the experiment.
B false
Firstly, this experiment does not measure the endogenous clock since the bright light is an eective, external cue to
delay the endogenous clock. Secondly, the the observed rhythm is on the order of about 26 hours (average over the 17
days).
C true
The person chose extended periods of light, without extending the periods of sleep much. This is an indication that the
bright light resets the clock and delays the desire to go to sleep. A good example can seen at the beginning of the
experiment where the person stay up for more than 20 hours on the rst two days and apparently got very tired on the
third day.
D false
The transition back to normal conditions took the person at least four days. This can be seen best with the irregular
eating times.
References
Charles et al, Science (1999)

41

Ecology, ethology and systematics

A herbivorous insect H is known to exclusively feed on the seedlings of two tree species X and Y.
In an experiment, patches of a forest were subjected to a treatment, or not. In the treatment,
seedlings of X and Y were protected from being fed on by H (open circles). Patches in which X
and Y were unprotected served as a control (lled circles). Panel A shows the average number of
all species for which seedlings were found in the patches. Panel B shows the average relative
abundance of seedlings of X and Y observed in the patches.

Number of species

Relative abundance

Indicate if each of the following statements is true or false.

Months since onset of the experiment

A. Seedlings of species Y are weak competitors.


B. The regulation of this tree community involves a top-down process.
C. Seedlings of species X are strongly regulated by an additional herbivore.
D. The herbivore insect functions as a key stone species.
A. False

B. True

C. False

D. True

Original commentary
Note
The two prey species must be excellent competitors, not inferior ones; otherwise they would not be able to dominate
the experimental plot communities in the absence of the predator (78% vs. 27%). When community organization is
regulated in this way, as opposed to the availability of nutrients recall seedling growth was not limited by light
availability it follows a top-down model and not a bottom-up model (p. 1206). A top-down regulatory role for the
predator is predicated on there being interspecic competition between prey and non-prey species for space when the
prey are largely sessile, which plants are (p.1205 top of Fig. 54.17). In this way, the predator pre-empts interspecic
competition among seedlings to promote local alpha diversity in the forest.
Correct answers
A false
Competition of species Y is only controlled by herbivore H. Without this control it dominates the tree seedlings in the
forest within a few years. Moreover it is clearly able to outcompete species X when both species are lacking herbivore
H.
B true
The diversity of tree species is regulated by the presence of herbivore H, a species at a higher position in the food
chain. This feature qualies a top-down process.
C false
After removing herbivore H, species X increases rapidly over two years, before diminishing again when being
exceeded by species Y. This feature is best explained by competition between species X and Y. A potential regulation by
another herbivore would be likely to prevent the initial strong increase of species Y.
D true
The predator not very abundant in the community, as both its host species only have a cumulative abundance of about
10%, but it clearly plays a pivotal ecological role in structuring the community by preventing potential dominance by
prey, so it therefore qualies as a "keystone species" (p. 1204; Campbell Biology 9th ed., 2010)

42

Ecology, ethology and systematics

The meta-population concept describes the population size in a habitat patch as a function of
birth rate, mortality and migration. Consider the hypothetical meta-population given below,
consisting of two big source patches X and Y where the birth rate exceeds mortality and three
sink patches (A, B and C) where reproduction does not occur. Yearly net migration (individuals)
between dierent sub-populations is constant and indicated with arrows in the gure. In each
sink patch, 6 individuals die at the end of each migration season. Individuals cannot migrate
further than to the next patch within a year. The initial population sizes in the sink patches are
A=22, B=9 and C=12.

Indicate if each of the following statements is true or false.


A. The subpopulation in Patch A dies out for the rst time after 8 years.
B. Subpopulations X and Y will be genetically isolated within few years.
C. If in Patch C, 50% of the individuals (instead of 6 individuals) die each year, the Subpopulation
C will not decrease below 7 individuals.
D. A conservation measure to reduce mortality in A by 50% (3 individuals a year) is suicient to
preserve all subpopulations.
A. True

B. True

C. False

D. True

Original commentary
Correct answers
A true
Net migration for patch A is 3. With a mortality of 6 individuals a year the sub-population declines 3 individuals each
year. After 8 years, the population will reach 0.
B true
With the given migration and mortality, all sink population have a negative long-term trend and will die out. First,
population A will die out so migration between A and B will be interrupted. Without this migration, population B and
subsequently population C will die out as well. So no individual will be able any more to migrate between X and Y.
C false
With a constant net migration of 6 individuals into patch C, the population will fall below 7 individuals within three
years, but will not fall below 6 individuals.
D true
This reduction in mortality prevents A from dying out. Therefore all other subpopulations will not die out either.

43

Ecology, ethology and systematics

You are given four drawings referring to typical representatives of four major groups of metazoa.

Indicate if each of the following statements is true or false.


A. Organism I belongs to a taxon characterized by a digestive system with a single opening and
the lack of a specialized respiratory system.
B. Organism II belongs to a taxon characterized by a hydrostatic skeleton, a thick cuticle, a
tubular digestive system with openings at both ends and often a genetically xed number of
somatic cells.
C. Organism III belongs to a taxon characterized by a larval stage with nerves forming a neural
tube and a sessile adult stage with a body surrounded by a tunic.
D. Organism IV belongs to a taxon characterized by an endoskeleton with calcareous pads and a
vascular system of uid-lled canals used for various functions, including locomotion by
tubular feet.
A. True

B. True

C. True

D. False

Original commentary
Correct answers
A true
Both the drawing and the description are referring to the group of Plathelminthes (Tubularia).
B true
Both the drawing and the description are referring to the group of Nematoda.
C true
Both the drawing and the description are referring to the group of Tunicata (Chordata)
D false
The drawing shows a representative of the group of Cnidaria (Hydra) with typical polyps, whereas the description
refers to a typical representative of the group of Echinodermata.

44

Ecology, ethology and systematics

The following gure shows the phylogenetic relationship among several sympatric sh species of
the family Mormyridae, which are known to use weak electric communication signals no predator
is capable of sensing and the transmission of which does not depend on environmental factors.

For each species, measurements informative about the trophic level (A), body shape (B) and the
shape and frequency of their electric signals (C) were determined for several individuals. The
gure below shows the position of each measured sh individual in a principal components space
for each of the three groups of measurements where all individuals of a species are enclosed in a
polygon. The colors refer to the phylogenetic positions shown above. Principal components
analysis is a statistical procedure that maximizes the variance on the rst few axes.

Indicate if each of the following statements is true or false.


A. Among these species, the phylogenetic distance is highly informative about the divergence in
communication systems between two taxa.
B. Speciation of recently diverged sister species was likely driven by sexual selection on the
communication system rather than by natural selection by ecological dierences.
C. The morphological variation among taxa is in agreement with an increase in habitat types at
the onset of the radiation of the red and blue clade.
D. Shape or frequency of communication signals in these species is strongly constrained by
morphological trade-os.
A. False

B. True

Original commentary

C. False

D. False

Correct answers
A false
Recently evolved sister species are as dierent or even more dierent in their communication signals than
phylogenetically distinct pairs.
B true
Currently diverged sister species are very distinct in their communication signals but show only limited dierences in
trophic ecology and morphology. Information (no predator with receptors able to track EODs, no eect of typic
ecological parameters like water turbidity and pH on communication system) given in the text make ecological driven
eects responsible for the pronounced dierences in signals very unlikely.
C false
Morphospace is mainly determined by phylogenetically very distinct taxa and therefore not created by recent changes
of habitats. Recently evolved taxa do dier only marginally in body shape.
D false
There is no obvious correlation between body shape and electric signals of Mormyridae. In addition, morphologically
very similar species dier heavily in their communication signals.

45

Ecology, ethology and systematics

Trichoplax adhaerens is the only known animal (metazoa) of the phylum Placozoa. It appears as a
at disc with a very simple structure made of only very few distinct cell types. Nerves, sensory
cells and muscle cells are absent. Shown below is an electron microscope image of T. adhaerens
and a dendrogram based on molecular data indicating the phylogenetic position of T. adhaerens
in relation to other taxa.

200 m

Indicate if each of the following statements is true or false.


A. Drosophila is more closely related to Trichoplax than humans are.
B. Taxa A listed in the dendrogram is more likely to be a sponge (Porifera) than a snail
(Mollusca).
C. Trichoplax has no coelom and no gastric tube system.
D. Trichoplax is likely to be a representative of bilateria (animals with distinct dorsal and ventral
sides as well as front and back side).
A. False

B. True

C. True

D. False

Original commentary
Correct answers
A false
As Placozoa are a sister group of all Eumetazoa, they are equally related to both humans and Drosophila.
B true
Taxa A has a more basal position than Cnidaria. Porifera are known to be the most basal group of metazoa, whereas
Mollusca are representatives of bilateria and therefore in the same group as humans or Drosophila are.
C true
Coelom and gastric tube are traits of higher Metazoa. Cnidaria do not yet have those organs, and Placozoa have even a
more basal position than Cnidaria. Moreover, these organs would demand for more distinct cell types than Placozoa
has.
D false
Placozoa have a more basal position than Cnidaria, which is a group of organisms that do not yet belong to Bilateria.
References
Srivastava, Nature (2008)

46

Ecology, ethology and systematics

While some mammals are known for their large dierence in body size between males and
females, there is no apparent dierence in size between sexes in other mammal species. This
sexual dimorphism can often be well explained by the ecology and mating system of a species.
Indicate if each of the following statements is true of false. A signicant sexual size
dimorphism with larger males is expected in ...
A. ... a very small antelope (< 5 kg), in which a male and a female together defend their small
area (< 10 ha) with rich food resources, water and shelter.
B. ... a seal species where males are known to travel large distances oshore to feed on
dispersed food and to copulate with any oestrous female they encounter.
C. ... an antelope species, in which males gather on a sandy lake shore during the dry season,
where each of them defends a 20 m 2 area vigorously.
D. ... a small carnivore (about 20 cm in length) living in a predator-rich habitat in mixed sex
groups. The ospring rely heavily on parental care from both sexes and males are known for
their remarkably large testicles.
A. False

B. False

C. True

D. False

Original commentary
Correct answers
A false
Both sexes defend the small territory as a full time job year-round against conspecics, therefore selection pressures
are the same for both sexes.
B false
This particular case of ssion-fusion mating selects more for agile, fast travelling males and resource-rich females than
for heavily built competitive males.
C true
This territory is evidently too small and of bad food quality to serve as home range. Male-male interaction is
permanent and highly competitive, females choose under many males - selection to show good genes and heavy
bodies.
D false
This species is social and group members have similar functions. Male-male competition is limited since cooperation is
very important to limit own mortality and to increase survival rate of ospring. In this system it is likely that females
mate with several partners that cause sperm competition.
References
Floyd, Journal of Mammology(1998)

47

Ecology, ethology and systematics

In ecology, two dierent measures of biodiversity are commonly used: alpha diversity
characterizes the biodiversity at a given location and beta diversity characterizes the diversity
found between habitats. A good estimate of alpha diversity is the Shannon index, which is
computed as

where the sum runs over all species 1, , S present in a habitat and p i is the relative abundance
of Species i.
The table shows the abundance of adult trees of eight tree species (A through H) in four plots of a
temperate ecosystem in both pristine and disturbed states.

Plot

State
pristine
pristine
disturbed
disturbed

Indicate if each of the following statements is true of false.


A. Alpha diversity is higher in Plot 1 than in Plot 2.
B. Disturbance seems to increase beta diversity in this ecosystem.
C. Species B is likely to be a pioneer species.
D. Seedlings of species D perform best in the presence of adult D trees.
A. True

B. True

C. True

D. True

Original commentary
Correct answers
A true
While the number of species present is the same in both plots, their relative abundance is rather dierent in that
species D dominates the system much more in plot 2 than plot 1, leading to a lower alpha diversity in plot 2.
B true
Beta diversity is much higher among plots 2 and 3 than plots 1 and 2. The students should be able to see this without
calculating anything since exactly the same species are present in plots 1 and 2, but several species are restricted to
either plot 3 or 4.
C true
Pioneer species are the rst species to colonize a damaged / disturbed habitat. While other species are also present
uniquely in either plots 3 and 4, none of them is present in both nor in that number, suggesting that species B.
D true
This is a typical characteristics of a species capable of dominating tree ecosystems, as species D is capable of doing in
the pristine state. Seedling of other trees only get a chance after disturbance through external factors.

INTERNATIONAL BIOLOGY OLYMPIAD


PRACTICAL PROBLEMS

2013, Bern, Switzerland















All IBO examination questions are published under the following Creative Commons license:



CC BY-NC-SA (Attribution-NonCommercial-ShareAlike) https://creativecommons.org/licenses/by-nc-sa/4.0/
The exam papers can be used freely for educational purposes as long as IBO is credited and
new creations are licensed under identical terms. No commercial use is allowed.

IBO 2013, SWITZERLAND

Practical Exam Molecular Cell Biology


Student Code:

th

24 International Biology Olympiad


14th-21st July, 2013
Bern, Switzerland

Practical Exam 1
Molecular Cell Biology
Total points: 100
Duration: 90 minutes

1 / 10

IBO 2013, SWITZERLAND

Practical Exam Molecular Cell Biology

Dear participants,
This test consists of two tasks:
Introduction [2 points]
Task 1: Presence of the -glucuronidase [12 points]
Part 1.1: Determine the presence of the -glucuronidase [12 points]
Task 2: Presence of the procyclin protein [86 points]
Part 2.1: How to use the counting chamber [1 point]
Part 2.2: Washing the magnetic beads
Part 2.3: Density of trypanosomes not binding to magnetic beads [37.5 points]
Part 2.4: Total trypanosome density [25.5 points]
Part 2.5: Success of the deletion of the procyclin gene [9 points]
Part 2.6: Verification of binding to beads [9 points]
Part 2.7: Interpretation of your results [4 points]

4
5
5
6
6
7
7
7
8
10
10

Please write your student code into the box on the title page.
There is no separate answer sheet. Please fill in your answers into the specific answers boxes indicated
with a gray background. Only answers given inside these boxes will be evaluated.
The answers have to be given either with a tick () or with Arabic numbers. The numbers "1" and "7" can
look very similar in handwriting. To make sure that those two numbers can be well distinguished by the
IBO staff, please write them as you normally would into the following box.
1=

7=

Stop answering and put down your pen IMMEDIATELY when the bell rings at the end of the exam. Put
the entire protocol with all the answers back into the exam envelope.

2 / 10

IBO 2013, SWITZERLAND

Practical Exam Molecular Cell Biology

Material and equipment


Make sure that you have received all the materials and equipment listed for each task. If any of these
items are missing, please raise your hand.

Equipment
Water bath at 37C (used in common)
1 Micropipette P1000
1 Micropipette P200
1 Micropipette P20
1 Box of pipette tips for P1000
1 Box of pipette tips for P200 and P20
1 Eppendorf holder
1 Tube holder
1 Solid waste container
1 Liquid waste tube [LW]
1 Polystyrene (styrofoam) box filled with ice
1 Timer
1 Marker
1 Microscope
3 Cell counting chambers
3 Microscope slides
3 Cover slips
21 Eppendorf tubes
1 Magnetic Eppendorf holder
Blank paper
1 Flag to call an assistant
1 Yellow sheet labeled with your student code

Chemicals
1 Eppendorf tube with magnetic beads [MB]
1 Tube with phosphate buffer [PBSB]
1 Eppendorf tube with Fixation Buffer [FB]
1 Eppendorf tube with Substrate Buffer [SB]
1 Eppendorf tube with Substrate (X-gluc) [S]

Trypanosome suspensions
1 Eppendorf tube with suspension of Strain 1 [T1]
1 Eppendorf tube with suspension of Strain 2 [T2]
1 Eppendorf tube with suspension of Strain 3 [T3]

3 / 10

IBO 2013, SWITZERLAND

Practical Exam Molecular Cell Biology

Introduction [2 points]
Trypanosoma brucei is a parasite causing sleeping sickness in humans and nagana in animals. It is
transmitted between individuals via the tsetse-fly and is almost exclusively found in Africa south of the
Sahara. To better understand the function of different proteins implicated in the life cycle and infection
pathway of T. brucei, it is the goal to create mutant strains that lack procyclin, but express another
protein of interest instead. Procyclin is a surface protein found in T. brucei but not in other trypanosome
species and is hypothesized to have an effect on the infection pathway. Different trypanosome species
rely on different surface proteins for their infectivity. For example, T. congolense relies on a surface
protein called GARP.
In this practical you will work with strains of the subspecies T. brucei brucei, which can infect domestic
and wild animals, but is not dangerous for humans. In a first step, cells were transfected or not with a
single construct coding for both GARP and -glucuronidase and grown as pure strains. -glucuronidase,
which is absent in wild-type T. brucei, is a protein that can cleave X-gluc, an artificial substrate, into a
blue product that can easily be observed by eye. In this setting, -glucuronidase serves as a convenient
reporter gene that will allow you to recognize the strains carrying a successfully introduced construct by
simply incubating the strains with X-gluc (Task 1).
The strains were then subjected to a protocol to delete the gene coding for the protein procyclin. This
would allow verifying whether procyclin is indeed important for the infectivity cycle and whether GARP
can compensate for the procyclin function. Since the deletion efficiency is not 100%, you will separate
the cells where the procyclin gene was successfully deleted from the ones in which the deletion did not
work. To achieve this, trypanosomes pre-incubated with antibodies specific against procyclin will be
separated with magnetic beads coated with protein A that specifically binds to the Fc part of antibodies,

as is illustrated below.
Before starting the practical work, indicate for each of the following statements if it is true or false with
a tick (). [2 points]
true
false
If the suspension of a strain incubated with X-gluc turns blue, the
-glucuronidase gene was successfully introduced.
Q1
Inferring the successful introduction of the GARP gene based on
the presence of the reporter gene (-glucuronidase) may result in
false positives or negatives if only one gene of the construct was
successfully inserted.
The location in the genome where the construct is introduced
affects the level of gene expression of the introduced genes.
A similar approach with magnetic beads and specific antibodies
can be used to separate cells, with successful deletion of a gene
coding for an intracellular protein, from cells where the deletion
did not work.

4 / 10

IBO 2013, SWITZERLAND

Practical Exam Molecular Cell Biology

Task 1: Presence of -glucuronidase [12 points]


Part 1.1: Determine the presence of -glucuronidase [12 points]
Prepare the following reaction mix for each of the three trypanosome strains T1, T2 and T3 in a separate
Eppendorf tube and mix by pipetting up and down:
1. 20 l of the trypanosome suspension. Since the trypanosomes sink to the bottom of the tube,
make sure to mix the tubes by inverting prior to pipetting.
2. 100 l substrate buffer (SB)
3. 10 l substrate (S)
Label each tube with the strain you used as well as with your three letter country code (as indicated on
your badge).
The reaction mixes must be incubated for at least 1 hour at 37C. Place your flag into the tube on your
partition wall to call an assistant who will put your tubes in a water bath. Also, use your flag to indicate
to the assistant that you want to get your tubes back from the water bath. Consider working on the
other task during the incubation.
Put your tubes on the yellow sheet with your student code in the corresponding box, they will be
photographed and evaluated.
Indicate with a tick () for each of the three strains if the sample turned blue or remained colourless
after incubation. [12 points]
Strain T1

Strain T2

Strain T3

Blue
Q2

Colourless

5 / 10

IBO 2013, SWITZERLAND

Practical Exam Molecular Cell Biology

Task 2: Presence of the procyclin protein [86 points]


Part 2.1: How to use the counting chamber [1 point]
You will use counting chambers to determine the density of trypanosomes in parts 2.3 and 2.4. Two
counting chambers that can be individually filled are organized on a single slide. Please be aware that
these chambers cannot be cleaned and that no extra counting chambers will be provided. Also, these
counting chambers do not need a cover slip.

The following steps should be performed when determining the density of trypanosomes in a
suspension:
1. Pipette 10 l of the suspension into a counting chamber.
2. Wait at least 2 minutes for the cells to sink to the bottom.
3. Put the slide under the microscope and count the number of trypanosomes individually in three
of the four larger squares highlighted in gray.

You may use either the 10x or the 40x objective, whichever you prefer. It is advised to follow a
serpentine (snake-like) route to go through each of the smaller squares to avoid losing orientation. In
order to prevent a potential bias, count trypanosomes within the square and those crossing the left or
bottom limit (filled circles), but not those outside the square or crossing the right or top limit (open
circles).
To obtain the trypanosome density from the number of counted trypanosomes, first determine the
volume in which the cells are counted and indicate it in the table below. Note that the height of the
counting chamber is exactly 0.1 mm and each of counting cell is exactly 1 mm wide (see figure above) [1
point].
Volume of 1 counting square (mm)
Q3

Volume of 1 counting square (ml)

6 / 10

IBO 2013, SWITZERLAND

Practical Exam Molecular Cell Biology

Part 2.2: Washing the magnetic beads


Wash the magnetic beads (MB) twice as follows:
1. Add 1 ml cold phosphate buffer (PBSB) to the tube and mix by pipetting up and down.
2. Place the Eppendorf tube in the magnetic holder. Wait at least 1 minute for the magnetic beads
to get pulled down.
3. Pipette the supernatant into the liquid waste tube (LW).
Finally, resuspend the magnetic beads in 35 l PBSB buffer and put them on ice.

Part 2.3: Density of trypanosomes not binding to magnetic beads [37.5 points]
Pull down trypanosomes expressing procyclin in a sample of each of the three trypanosome strains T1,
T2 and T3 as follows:
1. Pipette 190 l of the trypanosome suspension in a fresh Eppendorf tube. Mix the trypanosome
suspension by inverting prior to pipetting.
2. Add 10 l of washed magnetic beads. Make sure the beads are resuspended prior to pipetting.
3. Incubate 30 minutes on ice. Resuspend the magnetic beads very gently every 3-5 minutes by
inverting and finger-flicking the tube. Consider working on other parts of Task 2 during the
incubation.
4. Pull down the magnetic beads using the magnetic holder.
5. Transfer the entire supernatant, while the tube is still in the magnetic holder, into a fresh
Eppendorf tube and put on ice.
6. Immediately resuspend the magnetic beads in 50 l phosphate buffer (PBSB) very gently and
put on ice.
7. Prepare 100 l of a 1:10 dilution of the supernatant in phosphate buffer (PBSB) in a fresh
Eppendorf tube.
8. Pipette 36 l of this dilution into a separate Eppendorf tube and add 4 l of Fixation Buffer (FB).
Mix well by pipetting up and down.
9. Count the number of trypanosomes according to the protocol in Part 2.1 and enter the values in
the table below.
10. Calculate the mean of the number of trypanosomes per square (precision: 3 positions after the
decimal point). You will use these numbers in Part 2.5. [37.5 points]
Strain T1

Strain T2

Strain T3

Square 1
Q4
Square 2
Square 3
Mean counts per square

Part 2.4: Total trypanosome density [25.5 points]


To determine the total trypanosome density in the original suspension, perform the following steps for
each of the three trypanosome strains T1, T2 and T3:
1. Prepare 100 l of a 1:10 dilution of the original suspension in phosphate buffer (PBSB) in a fresh
Eppendorf tube. Mix the trypanosome suspension by inverting prior to pipetting.

7 / 10

IBO 2013, SWITZERLAND

Practical Exam Molecular Cell Biology

2. Pipette 36 l of this dilution into a fresh Eppendorf tube and add 4 l of Fixation Buffer (FB). Mix
well by pipetting up and down.
3. Count the number of trypanosomes according to the protocol in Part 2.1 and report the values
in the table below.
4. Calculate the mean and the standard deviation of the number of trypanosomes per square
(precision: 3 positions after the decimal point). You will use these numbers in Part 2.5. [25.5
points]
The formula for the standard deviation (SD) is given below with
value of the replicate and the mean.

Strain T1

being the number of replicates,

Strain T2

the

Strain T3

Square 1
Q5
Square 2
Square 3
Mean counts per square
Standard deviation of counts
per square

Part 2.5: Success of the deletion of the procyclin gene [9 points]


The ultimate goal is to calculate and compare the density of trypanosomes not binding to magnetic
beads to the total trypanosome density in the starting suspension from the average counts observed.
However, you will first have to determine the standard error of the mean (SEmean) to decide on the
correct number of significant digits. Under the assumption that the counts are normally distributed, the
SEmean is given by
Calculate SEmean for the strain for which you observed the largest standard deviation among the counts
of trypanosomes per cell from Part 2.4 and enter your result in the table below (precision: 3 positions
after the decimal point). [0.6 points]

SEmean
Q6

8 / 10

IBO 2013, SWITZERLAND

Practical Exam Molecular Cell Biology

The SEmean tells you the accuracy with which you estimated the mean number of trypanosomes per
square. Use this estimate to decide the correct number of significant digits (all digits including the first
for which you are uncertain) by comparing the estimated mean plus the SEmean with the estimated mean
minus SEmean. For instance, if the mean is 1234.567 and the SEmean 98.765, you will have to compare
1234.567 + 98.765 = 1333.332 with 1234.567 - 98.765 = 1135.802. In this case, there are two significant
digits and the mean should be reported as 1.2x103. Indicate the number of significant digits you should
use with your data. [1.5 points]

Number of significant digits


Q7
Report in the table below the mean counts per square with and without pull-down for all three strains
using the number of significant digits you indicated above. [0.6 points]
Strain T1

Strain T2

Strain T3

Mean counts per square of


Q8 trypanosomes not binding to magnetic
beads (from Part 2.3)
Mean counts per square of total
trypanosomes (from Part 2.4)
Now use these values to estimate the density of trypanosomes in the dilutions used for counting, and
report your values in the table below with the same number of significant digits. [3.7 points]
Strain T1

Strain T2

Strain T3

Trypanosomes not binding to


Q9 magnetic beads /ml in dilution used
for counting (from Part 2.3)
Total trypanosomes / ml in dilution
used for counting (from Part 2.4)
Finally, calculate both the density of trypanosomes not binding to magnetic beads as well as the total
trypanosome density in the original suspension, and report your values in the table below with the same
number of significant digits. [1.1 points]
Strain T1

Strain T2

Strain T3

Trypanosomes not binding to


magnetic beads /ml in original
Q10
suspension (from Part 2.3)
Total Trypanosomes / ml in original
suspension (from Part 2.4)

In order to assess the success rate of the gene deletion experiment, calculate the percentage of
trypanosomes that did not bind to magnetic beads for each of the three strains. Use the estimates for
the densities in the original suspension for your calculations and indicate your results in the table below
(precision: only full percentages). [1.5 points]

9 / 10

IBO 2013, SWITZERLAND

Practical Exam Molecular Cell Biology


Strain T1

Strain T2

Strain T3

Percentage of trypanosomes not


Q11 binding to beads

Part 2.6: Verification of binding to beads [9 points]


You will next verify under the microscope if a reduction in trypanosomes observed after pull down is
indeed due to binding of trypanosomes to the magnetic beads. To do so, perform the following steps for
each of the three strains:
1. Pipette 10 l of the beads, you resuspended in Part 2.3, on a microscope slide.
2. Cover the drop with a cover slip.
Make a rough assessment of the fraction of trypanosomes that are attached to a magnetic bead. A good
indication that a trypanosome is bound to a bead is when the bead wiggles as the trypanosome moves.
In the table below, indicate with a tick () for each of the three strains which description best fits your
observation. [9 points]
Strain T1

Strain T2

Strain T3

Practically no bound trypanosomes in the sample


Q12

<50% of the trypanosomes present in the sample are


bound
>50% of the trypanosomes present in the sample are
bound

Part 2.7: Interpretation of your results [4 points]


Indicate with a tick () the statement best describing the reduction in trypanosomes in the supernatant
after pull down for each strain. [3 points]
Strain T1
Q13

Strain T2

Strain T3

A reduction, at least in part due to binding to


beads
No or only a purely stochastic change

Indicate with a tick () the strain where the deletion was most efficient. [1 point]
Strain T1
Q14

Strain T2

Strain T3

Highest deletion efficiency

End of the Practical Exam

10 / 10

Student Code:

24th International Biology Olympiad


14th-21st July, 2013
Bern, Switzerland

Practical Exam 2
Plant Physiology, Morphology and Ecology
Total points: 88
Duration: 90 minutes

IBO 2013, SWITZERLAND

Practical Exam Plant Physiology, Morphology and Ecology

Dear participants,
This test consists of three tasks:
Task 1: Determination of glucose content in plant extracts [44 points]
Part 1.1: Calibration curve [13.5 points]
Part 1.2: Glucose content [15 points for measured raw values]
Part 1.3: Data analysis [15.5 points]
1.3.1 NADH calibration curve
1.3.2 Glucose concentration in plant extracts
1.3.3 Interpretation of your results
Task 2: Staining of transitory starch [9 points]
Task 3: Floral morphology and pollination ecology [35.5 points]
Part 3.1: Floral morphology
3.1.1 Type of inflorescence
3.1.2 Number of floral parts
3.1.3 Fusion of floral parts
3.1.4 Ovary position
3.1.5 Floral symmetry
Part 3.2: Pollination ecology
3.2.1 Floral shape
3.2.2 Pollinators

Please write your student code into the box on the title page.
You are strongly advised to start working with Task 1. During this task, you will have to incubate your
probes for 20 minutes and then allow for some time for the lab assistants to perform the necessary
measurements. During this waiting time you may work on Tasks 2 and 3.
There is no separate answer sheet. Please fill in your answers into the specific answers boxes indicated
with a gray background. Only answers given inside these boxes will be evaluated.
The answers have to be given either with a tick () or with Arabic numbers. The numbers "1" and "7" can
look very similar in handwriting. To make sure that those two numbers can be well distinguished by the
IBO staff, please write them as you normally would into the following box.

1=

7=

Stop answering and put down your pen IMMEDIATELY when the bell rings at the end of the exam. Put
the entire protocol with all the answers back into the exam envelope.

2/16

IBO 2013, SWITZERLAND

Practical Exam Plant Physiology, Morphology and Ecology

Material and equipment


Make sure that you have received all the materials and equipment listed for each task. If any of these
items are missing, please raise your hand.

Task 1
Plant material:
6 x 250 l plant extracts [WT light, WT dark, sex1 light, sex1 dark, pgm1 light, pgm1 dark]
Solutions and reagents:
1.5 ml Master mix [MM]
1 ml NADH solution 500 M [NADH]
200 l G6PDH [G6PDH]
10 ml H2O [H2O]
Technical material:
1 micropipette 50 l-200 l (can be used down to 20 l)
1 96-well Microplate (dont touch the bottom of the plate!)
1 Timer
Container for used materials
2 blank paper sheets for notes
Flag to call the lab assistant

Task 2
Plant material:
8 tubes with de-pigmented plants [A, B, C, D, E, F, G, H]
Solutions and reagents:
10 ml Lugols solution [Lugol]
Technical material:
8 plastic dishes
Plastic Pasteur pipette
Waterproof pen for writing on plastic dishes

Task 3
Plant material:
5 tubes with floral specimen in 70% ethanol [V, W, X, Y, Z]
Color print with photos of the flowers V-Z
Technical material:
1 dissecting microscope
1 razor blade
1 forceps
2 toothpick
1 plastic dish filled with water

3/16

IBO 2013, SWITZERLAND

Practical Exam Plant Physiology, Morphology and Ecology

Task 1: Determination of glucose content in plant extracts [44 points]


Under sufficient light, plants synthesize carbohydrates from atmospheric CO2 by photosynthesis. While a
fraction of these photoassimilates is exported to the cytosol and then transformed to soluble glucose,
another fraction is retained within the chloroplasts and stored transiently as starch. This starch is
subsequently degraded and mobilized to provide a supply of carbon and energy during dark periods.
Several mutants of Arabidopsis have been identified that are unable to either synthesize or degrade
starch. In this practical, you will work with plant extracts from two of them, along with extract from wild
type [WT] plants:
1. WT : unmutated wildtype plants
2. sex1 : starch degradation mutants
3. pgm1 : starch synthesis mutants

All plants were grown for 4 weeks with 8 hours light and 16 hours of darkness per day and then either
exposed to complete darkness for 48 hours (dark-incubated or dark) or to 10 hours of light (lightincubated or light) immediately prior to harvesting.
You are going to quantify the glucose content in those plant extracts by measuring the absorbance of
NADH. As shown below, one molecule of NAD+ is converted into one molecule of NADH per molecule of
glucose in a two-step reaction that is catalyzed by the enzymes hexokinase (HXK) and glucose-6phosphate dehydrogenase [G6PDH]. The master mix [MM] contains NAD+, HXK and ATP.

You will work with a 96-well microplate (see figure below). Only touch its edges! Positions on this plate
are indicated by a number (1-12) and a letter (A-H) specifying columns and rows, respectively. Use only
the wells which are listed in the protocol (contained within the area indicated by the dotted line).

4/16

IBO 2013, SWITZERLAND

Practical Exam Plant Physiology, Morphology and Ecology

Part 1.1: Calibration curve [13.5 points]


In order to quantify glucose by measuring the absorbance of NADH, you will first have to make a
calibration curve with different concentrations of NADH. In the table below, indicate the required
volumes of the 500 M NADH stock solution and H2O that are needed to achieve the desired
concentrations of NADH in a total volume of 200 l. Then pipette the required volumes into the
corresponding well on the microplate (A1-A7) and mix by pipetting up and down three times. [3 points
for calculations + 10.5 points for measured raw values]
Well
[NADH] (M)

A1
0

A2
50

A3
100

A4
150

A5
200

A6
250

A7
300

500 M
Q 1 NADH solution (l)
H2O (l)

Part 1.2: Glucose content [15 points for measured raw values]
To determine the concentration of glucose in the different plant extracts, you will now prepare two
solutions for each plant extract. The Wells B1-B6 will serve as a blank to quantify the base absorption
and differ from the Wells C1-C6 which will contain a solution of G6PDH. Begin by pipetting the following
into Wells B1-B6 and C1-C6:
1. 100 l Master mix into the Wells B1-B6 and C1-C6
2. 20 l H2O into the Wells B1-B6
20 l G6PDH solution into the Wells C1-C6
3. 80 l of the corresponding plant extract as indicated in the table below and mix by pipetting
three times up and down
Wells
Plant extract

B1 and C1
WT light

B2 and C2
WT dark

B3 and C3
sex1 light

B4 and C4
sex1 dark

B5 and C5
pgm1 light

B6 and C6
pgm1 dark

Incubate for at least 20 and up to 60 minutes at room temperature. After incubation, call an assistant by
placing your flag into the tube on your left partition wall. The assistant will measure the absorbance at
340 nm and bring you a printout of the absorbance values measured for each well. You will need these
values for the analysis in Part 1.3.
NOTE: Due to the limited number of microplate readers, you might have to wait up to 15 minutes to
have your microplate measured after putting up your flag. During incubation time and waiting time you
may work on Tasks 2 and 3.

5/16

IBO 2013, SWITZERLAND

Practical Exam Plant Physiology, Morphology and Ecology

Part 1.3: Data analysis [15 points]


Important: Label the printout with your name and your student code. At the end of the exam, put it into
your exam envelope.

1.3.1 NADH calibration curve


Calculate the scaled extinction coefficient (s) for all measured NADH concentrations according to

where A0 is the background absorbance value at 0 M NADH (Well A1) and Ac the absorbance value at
concentration cNADH. s is equivalent to the extinction coefficient multiplied with l, the path length of
light through the solution. Write the calculated values in the answer fields (precision: five positions after
the decimal point). [3 points]
Well
[NADH] (M)

A2
50

A3
100

A4
150

A5
200

A6
250

A7
300

-1
Q 2 s ( M )

Calculate the mean value of all calculated scaled extinction coefficients

and write your result in the

corresponding field below (precision: five positions after the decimal point). [1 point]
Q3

( M-1)

1.3.2 Glucose concentration in plant extracts


Calculate the glucose concentration cglucose in each well (diluted plant extract) measured as

where AG6PDH is the absorbance measured for the sample incubated with G6PDH (Wells C1-C6), AH2O is
the absorbance measured for the sample incubated with H2O only (Wells B1-B6), and s is the mean
scaled extinction coefficient you have calculated above. Then, use these values to calculate the initial
glucose content that was present in the fresh leaves, indicated as mmol/g. Each plant extract was made
of 25 g leaf material per liter. Report your values in the table below (precision: one position after the
decimal point). [7 points]

6/16

IBO 2013, SWITZERLAND


Sample

Practical Exam Plant Physiology, Morphology and Ecology


Concentration in well ( M)

Content in leaves (mmol/g)

WT light
Q4

WT dark
sex1 light
sex1 dark
pgm1 light
pgm1 dark

1.3.3 Interpretation of your results


Indicate with a tick () for each of the following statements if it is true or false based on your
measurements. [4 points]
true

Q5

false

Plants light-incubated prior to extraction contain more glucose than those


incubated in the dark.
Dark-incubated plants have used up all their carbohydrate energy sources in the
dark period.
Glucose levels in light-incubated wild-type plants are lower than in darkincubated wild-type plants.
Both pgm1 light and sex1 light samples contain more glucose than WT light
samples.
The sex1 light sample contains more glucose than the pgm1 light sample.
The difference in glucose concentrations between dark- and light-incubated
plants is bigger in wild type plants than in mutant plants.
sex1 plants are likely to grow faster than pgm1 plants.
Measuring samples without addition of G6PDH is needed to correct the effect of
the background concentration of 6-P-Gluconolactone.

7/16

IBO 2013, SWITZERLAND

Practical Exam Plant Physiology, Morphology and Ecology

Task 2: Staining of transitory starch [9 points]


Transitory starch can be easily visualized in leaves by staining it with Lugols solution containing iodine
in ethanol-destained leaves. On your lab bench you find two groups of plant samples.
GROUP I

GROUP II

While all plants of one group have been harvested after incubating for 12 hours in light, all plants of the
other group have been harvested after incubating for 12 h in complete darkness. Each group contains at
least one wild type plant (WT), one starch degradation mutant (sex1) and one starch synthesis mutant
(pgm1).
Repeat the following steps for each of the plants A-H to stain them:
1. Transfer the plant specimen into a fresh plastic dish.
2. Remove residual liquid from the dish, with the Pasteur pipette.
3. Stain the plants by covering them with some drops of Lugols solution using the Pasteur
pipette.
4. Incubate at room temperature for 1 minute before doing your observations.
Indicate the observed staining pattern of leaves of each plant specimen in Group I (Plants A-D) and
Group II (Plants E-H) in the table below with a tick (). [4 points]
Group I
Plant

Q6

Light
orange

Dark
brown

Intense
black

Plant

Group II
Light
orange

Dark
brown

Intense
black

Based on the observed plants and staining pattern, indicate the corresponding growth condition for
Group I and Group II in the table below with a tick (). [1 points]
Growth condition
12 h in light

12 h in darkness

Q 7 Group I
Group II

8/16

IBO 2013, SWITZERLAND

Practical Exam Plant Physiology, Morphology and Ecology

Based on the observed plants and staining pattern, indicate the corresponding strain for each of the
plant specimen (A-H) in the table below with a tick (). [4 points]

Group
I
Q8

Plant

WT

Strain
sex1

pgm1

A
B
C
D

II

E
F
G
H

9/16

IBO 2013, SWITZERLAND

Practical Exam Plant Physiology, Morphology and Ecology

Task 3: Floral morphology and pollination ecology [35.5 points]


Angiosperm flowers show an enormous variety in structure, shape and ecological function. While the
number, shape and disposition of different floral parts is often phylogenetically determined, functional
characteristics such as the general shape of the flower depend more on ecological factors such as the
mode of pollination. In this task, you will study the morphologic traits of five different floral samples (VZ) and attribute them ecological traits linked with pollination.
To help understand the terminology used below, the following figure gives you an overview of the
different parts in a schematic flower.
A: carpel
B: stigma
C: style
D: ovary
E: pedicel
F: stamen
G: petal
H: sepal
I: tepal (when sepals and petals are undifferentiated)
J: floral axis
K: bract

Part 3.1: Floral morphology


Look at the five specimens V-Z preserved in ethanol, containing a flower or an entire inflorescence of
plant species commonly found in Switzerland. The flowers are all hermaphrodite, with male and female
parts present in the same flowers. The habit, shape and color of the corresponding plants are shown on
the color printout.
For each specimen V-Z, you will be asked to make a series of morphological observations. It is
recommended to do first all observations for one specimen before going over to the next one.
Take the specimen out of the tube and place it in a Petri dish filled with water for easier observation.
Please close the tube again to avoid excessive ethanol vapor in the room.
Handle the flower specimens carefully, as you have to get along with the given plant material to do all
your observations. In the case a specimen consists of more than one flower, isolate individual flowers to
observe specific parts. For such plants you may want to observe a specific part in several flowers as
some floral parts (notably stamen and carpels) are easiest observed in flowers in a specific state of
maturation.
Observe small details with the dissecting microscope. For a part of the observations you might have to
dissect the flower, for which you can use the given material (razor blade, forceps and toothpicks). Make
all dissections in the Petri dish.

3.1.1 Type of inflorescence


The figure shows schematic illustrations of different types of inflorescences. Each small circle represents
a single flower and is often accompanied by a bract.
10/16

IBO 2013, SWITZERLAND

Practical Exam Plant Physiology, Morphology and Ecology

Indicate the most appropriate type of inflorescence for each Specimen V-Z with a tick ().[2.5 points]
V

Inflorescence type 1
Q9

Inflorescence type 2
Inflorescence type 3
Inflorescence type 4
Inflorescence type 5
Inflorescence type 6
Inflorescence type 7

3.1.2 Number of floral parts


Determine the number of sepals, petals (or tepals), stamen and carpels per flower for each of the
Specimens V-Z and write your results in the table below. When more than 10 parts of a specific floral
part are present within a single flower, indicate ">10". Omit counting floral parts indicated by a black
cell in the table. [11.5 points]
V

number of sepals
Q 10
number of petals or tepals
number of stamens
number of styles (count branched
styles only once)

number of distinct ovaries

3.1.3 Fusion of floral parts


As illustrated in the figure with schematic petals and stamen, floral parts can either be free (A), partially
fused (B) or completely fused (C). A: free (fused part <10%), B: partially fused (fused part 10-90%), C:
completely fused (fused part >90%).
11/16

IBO 2013, SWITZERLAND

Practical Exam Plant Physiology, Morphology and Ecology

Indicate the correct state of fusion of sepals, petals and stamen for flowers of plant Specimens V-Z with
a tick () in the table below. [6.5 points]
V
Sepals
Q 11

Free (fused part <10%),


Partially fused (fused part 10-90%)
Completely fused (fused part >90%)

Petals /
Free (fused part <10%)
tepals
Partially fused (fused part 10-90%)
Completely fused (fused part >90%)
Stamens

Free
Partially fused to other stamens
at the base or the top (fused part 10-90%)
Partially fused with petals / tepals (fused part
10-90%),
Partially fused to other stamens AND with
petals / tepals (fused part 10-90%)

3.1.4 Ovary position


The position of the ovary of a flower can be classified as superior (A) or inferior (B), as is shown in the
figure below.

Indicate the ovary position of the flowers for each specimen V-Z with a tick () in the table below. [2.5
points]

12/16

IBO 2013, SWITZERLAND

Practical Exam Plant Physiology, Morphology and Ecology


V

A (superior)
Q 12
B (inferior)

3.1.5 Floral symmetry


The symmetry of a flower can be classified into radial (A), bilateral (B) or asymmetrical (C). The following
figure illustrates these symmetries both with example flowers as well as floral diagrams (second row).
Floral diagrams show from outside to inside sepals (filled and curved), petals or tepals (open and
curved), stamen (open and round) and carpels (grey and round).

Indicate the floral symmetry for flowers of each of the specimens V-Z with a tick () in the table below.
[2.5 points]
V

A (radial)
Q 13

B (bilateral)
C (asymmetrical)

Part 3.2: Pollination ecology


3.2.1 Floral shape
Floral shapes evolved as adaptations to the morphology and behavior of different pollinators. Despite
the enormous diversity of flowers and inflorescences, most of them can be classified into one of several
broad classes shown in the table below. Note that the relevant unit for attracting pollinators can be a
single flower or an entire inflorescence.

13/16

IBO 2013, SWITZERLAND


n Floral shape

Open
flower

Practical Exam Plant Physiology, Morphology and Ecology

Illustration of example flowers or General characteristics


inflorescences

disk

Pollen and/or nectar freely presented


Flat landing place.

Tubular flower
with
radial
symmetry

Radial symmetry
Pollen and/or nectar partially or
completely hidden in a narrow tube
Tube may be built from free or fused
floral parts

Tubular flower
with bilateral
symmetry

Bilateral symmetry
Pollen partially or completely hidden in
a narrow tube
Tube may be built from free or fused
floral parts

Brush flower

Absent or inconspicuous perianth /


petals
Numerous and predominant stamens

Based on the morphological characteristics you determined above, indicate the most appropriate floral
shape class for each specimen V-Z with a tick () in the table below. [5 points]
V

Floral shape 1
Q 14

Floral shape 2
Floral shape 3
Floral shape 4

14/16

IBO 2013, SWITZERLAND

Practical Exam Plant Physiology, Morphology and Ecology

3.2.2 Pollinators
As is shown for a subset of common pollinators in the following table, the floral shape and other
characteristics of the flowers or inflorescences are often strongly associated with their pollinators.
n

Pollinator

Wind

Characteristics of pollinator

Common shapes
inflorescences

of

flowers

Unspecific

Inconspicuous

Non-directional

Lack of optically attractive floral


parts
Nectar and scent absent
Huge amount of pollen
Anthers and stigma well exposed to
the wind

Bees/ bumblebees

Day-active

Either

Biting mouthparts

Bilateral

Long tongue
Attracted by pollen and nectar

Brightly colored (often yellow,


violet or blue)

Pollen gathering

Nectar hidden in a tube

Able to hang upside down

Offering a landing place


Or
Flat
Brightly colored (often yellow,
violet or blue)
Offering a lot of pollen

Moths

Night-active

White or nearly white

Long proboscis

Fragrant scent

Do not need a landing platform

Open during the night

Seeking for nectar


4

Butterfly

Day-active

Upright position

Long proboscis

Tight tube

Do not need a landing platform

Deeply hidden nectar

Seeking for nectar


5

Flies

Licking mouthparts

Either
Small and flat

Like bright flat surfaces to sit in


the sun

Without scent
Nectar reward freely presented
Or
Brown/purple advertisement
Carrion- or excrement-scented

15/16

IBO 2013, SWITZERLAND

Practical Exam Plant Physiology, Morphology and Ecology

The following table lists additional floral traits of the plant specimen that cannot be observed from the
prepared samples (+: present, ++ : strongly present, -: (nearly) absent).
V

presence of nectar

++

++

scent

++

Based on the characteristics of the flowers or inflorescences you observed and given in the table above,
indicate with a tick () for each of the different plant specimens V-Z its most likely group of pollinators in
the table below. Use the numbers indicated for each group in the table above. If several groups are
equally likely, pick any one of them. [5 points]
V

Pollinator 1
Q 15

Pollinator 2
Pollinator 3
Pollinator 4
Pollinator 5

End of the Practical Exam

16/16

IBO 2013, SWITZERLAND

Practical Exam Evolutionary ethology


Student Code:

24th International Biology Olympiad


14th-21st July, 2013
Bern, Switzerland

Practical Exam 3
Evolutionary Ethology
Total points: 94
Duration: 90 minutes

1 / 13

IBO 2013, SWITZERLAND

Practical Exam Evolutionary ethology

Dear participants,
This test consists of three tasks:
Task 1: Quantifying aggressiveness
Part 1.1: Experimental setup
Part 1.2: Aggressive behavior of Neolamprologus pulcher
Part 1.3: Examples of ramming or biting attacks
Part 1.4: Quantify ramming or biting attacks of N. pulcher [30 points]
Part 1.5: Additional replicates
Part 1.6: Statistical analysis of bite or ram attacks [20 points]
Task 2: Puffed throat behavior

4
4
4
5
5
6
6
10

Part 2.1: Puffed throat behavior of N. pulcher

10

Part 2.2: Examples of puffed throat behavior


Part 2.3: Quantify puffed throat behavior of N. pulcher [21 points]

10
10

Part 2.4: Interpret your results [4 points]

11

Task 3: Social groups

12

Part 3.1: Quantifying task sharing in social groups of N. pulcher [15 points]

12

Part 3.2: Interpret your observations [4 points]

13

Please write your student code into the box on the title page.
There is no separate answer sheet. Please fill in your answers into the specific answers boxes indicated
with a gray background. Only answers given inside these boxes will be evaluated.
The answers have to be given either with a tick () or with Arabic numbers. The numbers "1" and "7" can
look very similar in handwriting. To make sure that those two numbers can be well distinguished by the
IBO staff, please write them as you normally would into the following box.

1=

7=

Stop answering and put down your pen IMMEDIATELY when the bell rings at the end of the exam. Put
the entire protocol with all the answers back into the exam envelope. Make sure you wrote your
student code into the box on the title page.

Material and equipment


Make sure that you have received all the materials and equipment listed for each task. If any of these
items are missing, please raise your hand.

Equipment
1 Tablet with preloaded movies
scratch paper
2 / 13

IBO 2013, SWITZERLAND

Practical Exam Evolutionary ethology

Introduction
Most organisms are limited in resources such as food, mating partners or shelters. As a result, available
resources are often defended by aggressive interactions with conspecific (same species) and
heterospecific (different species) competitors. A very common phenomenon to monopolize resources is
the establishment of a well-defended territory. But territorial defense is costly as it may expose the
territorial resident to an increased risk of injury or predation or reduce the time available for other
activities such as foraging, mating or brood care. Given this, a territorial resident that is able to adjust its
aggressive response to match the degree of threat posed by an intruder is expected to have a selective
advantage.
In this practical you will test if individuals of the territorial African cichlid Neolamprologus pulcher are
using visual and / or olfactory cues of a competing conspecific to adjust their aggressive response. N.
pulcher is native to Lake Tanganyika where it forms breeding pairs, which defend a small contiguous
territory (mean area = 0.3 m2) used for feeding, breeding, and shelter from predators.
All experiments were conducted under laboratory conditions in aquaria and recorded on video. Your
task is now to analyze these recordings by quantifying the behavior of different individuals. All videos
are preloaded on a tablet computer and can be played at your convenience.

3 / 13

IBO 2013, SWITZERLAND

Practical Exam Evolutionary ethology

Task 1: Quantifying aggressiveness


In this task, you will quantify the aggressive behavior of N. pulcher males by statistically testing if males
use visual and / or olfactory cues to adjust their level of aggressiveness to the threat posed by a
competing male.

Part 1.1: Experimental setup


In each experiment, two males are freshly exposed to each other in a previously uninhabited test
aquarium that is divided by a glass wall that either seals off the two compartments completely
(Situations A and B) or is porous and allows for an exchange of water between the two compartments
(Situations C and D). The focal male is always in the right compartment and is exposed to either a
considerably smaller male (Situations A and C) or a male of equal size (Situations B and D).

Throughout this task, you will only have to consider the behavioral response of the focal male in the
right compartment (indicated in gray).

Part 1.2: Aggressive behavior of Neolamprologus pulcher


In this task, you will count the frequency of a particularly conspicuous behavioral response of N. pulcher
males to conspecifics: ramming or biting attacks. Aggressive behavior is counted as ram or bite
(ramming with its mouth open) when the fish is obviously attacking its competitor by swimming towards
it and hitting the glass wall separating them. Each hit of the snout against the glass wall is counted as a
single event, even if males ram or bite repeatedly within a few seconds. The following figure illustrates
this behavior.

4 / 13

IBO 2013, SWITZERLAND

Practical Exam Evolutionary ethology

Part 1.3: Examples of ramming or biting attacks


Activate your tablet by tapping and sliding with your fingers and open the Folder E_1. In that folder you
will find three movies. Movie I contains a series of obvious biting or ramming attacks. Watch it to get a
feeling of what to look for. Next, watch Movies II and III which indicate how these attacks are to be
counted. Watch them carefully. Proceed with the protocol as soon as you feel confident with how to
count this behavior in N. pulcher.

Part 1.4: Quantify ramming or biting attacks of N. pulcher [30 points]


Each of the Folders A, B, C and D contain three movies, showing sequences from experiments conducted
under the corresponding Situations A, B, C and D indicated in the figure in Part 1.1 and the table below.
Count the number of ram or bite attacks of the male in the right compartment for each of the twelve
movies and report your results in the table below. It is recommended to count by using tally marks on
the provided scratch paper.

Q1

Situation
Separation
Size

A
sealed
different

B
sealed
matched

C
porous
different

D
porous
matched

Replicate 1
Replicate 2
Replicate 3

5 / 13

IBO 2013, SWITZERLAND

Practical Exam Evolutionary ethology

Part 1.5: Additional replicates


The table below provides the results of three additional replicates for each experimental setup. These
results are to be combined with your results for the statistical analysis below.
Situation
Separation
Size
Replicate 4
Replicate 5
Replicate 6

A
sealed
different
24
34
27

B
sealed
matched
45
41
38

C
porous
different
10
17
12

D
porous
matched
15
8
16

Part 1.6: Statistical analysis of bite or ram attacks [20 points]


You will conduct an analysis of variance (or ANOVA) to analyze the obtained results. The goal of this
analysis is to infer what part of the variance in your measures is due to stochastic differences between
the individuals studied, and what part can be explained by the two factors manipulated in this
experiment: 1) the type of separation (sealed vs. porous) and 2) the difference in the size of males
(matched vs. different). This is done by partitioning the total variance into different components and
testing the relative importance of these partitions
Variance within groups
Compute the mean and variance among the replicates 1 through 6 of each of the four experimental
setups A through D. Report your results in the tables below with a precision of one digit after the
decimal point. The variance is calculated according to

where i runs over all replicates 1 through n and Ri are the observed counts in replicate i and M is the
mean across all replicates 1 through n.
Situation
Separation
Size

A
sealed
different

B
sealed
matched

C
porous
different

D
porous
matched

Mean (M)
Q2
sample variance ( 2)

Now compute the average sample variance within groups (Vg) as the average of the four variances.
Report your results in the box below with a precision of one digit after the decimal point.

6 / 13

IBO 2013, SWITZERLAND

Practical Exam Evolutionary ethology

average sample variance within groups (Vg)


Q3

Variance explained by the type of separation and difference in the size of males
Next compute the variance explained by the type of separation (Vseparation) and the variance explained by
the difference in size of males (Vsize). To do so, you will first have to compute the overall mean of bite or
ram attacks across all 24 replicates. Report your result in the box below with a precision of one digit
after the decimal point.

overall mean across all replicates (MABCD)


Q4
Next, compute the mean number of bite or ram attacks among all replicates for each of the type of
separation and the difference in the size of males, independent of the other category. The mean
number of bite or ram attacks among all replicates with sealed separations, for instance, is simply given
by the average of MA and MB, where MA and MB refer to the mean number of bite or ram attacks
observed in Situations A and B, respectively, which you calculated above. Compute all these means and
report your results in the table below with a precision of one digit after the decimal point.
Type of separation
B
sealed
porous
Q5

Difference in the size of males


D
different
matched

mean within category

The variance explained by factor x (either separation or size) is now computed as

where i runs over both categories of the factor considered, n is the number of replicates within each
category (12 in your case), Mi is the mean within category i and MABCD the overall mean computed in Q4.
Compute the variance explained by each factor and report your results in the table below with a
precision of one digit after the decimal point.
Type of separation

Difference in the size of males

variance explained by factor (VX)


Q6

If a factor does not explain any of the observed variance, we expect Vx to be zero. However, due to the
stochastic nature of this experiment, deviations from zero are expected. You will now test if the

7 / 13

IBO 2013, SWITZERLAND

Practical Exam Evolutionary ethology

deviations from zero you observed for both Vseparation and Vsize are statistically significant, which would
indicate that these factors are significantly explaining part of your observations. This is done by
computing the probability of observing deviations as large as or even larger than what you observed. To
do so, you will need a F-test, for which you now need to compute the test statistic Fx, which is given by
the variance explained by factor x (either separation or size) divided by the average sample variance
within groups (Vg) you calculated in Q3.

Calculate Fx for both factors and report your results in the table below with a precision of one digit after
the decimal point.

Type of separation
B

Difference in the size of males


D

test statistic Fx
Q7

You can now use the properties of the F-distribution to translate the test statistic into the probability of
observing the Vx values you calculated or even more extreme Vx values if factor x does not explain any
of the variance observed. This probability is commonly referred to as the p-value. The table below
shows the corresponding F-values for a wide range of p-values.
Fx
0.47
2.96
4.32
8.02
14.59
22.89
33.28
46.27
62.46
82.65

p-value
0.5
0.1
0.05
0.01
10-3
10-4
10-5
10-6
10-7
10-8

Use this table to translate your test statistic values into p-values. For each factor, indicate in the table
below, the smallest p-value for which the p-value corresponding to the calculated F-value is smaller.
Type of separation

Q8

Difference in the size of


males

p-value <

8 / 13

IBO 2013, SWITZERLAND

Practical Exam Evolutionary ethology

Indicate with a tick () for each factor if it explains a fraction of the total variance significantly (p-value <
0.05) or not in the table below.
Type of separation

Q9

Difference in the size of


males

Explains part of the total variance


Does not explain part of the total
variance

Indicate with a tick () if each of the following statements is a valid conclusion from your results or not.
valid

Q10

not valid

Males of N. pulcher use visual cues of a competing conspecific to


adjust their aggressive response.
Males of N. pulcher use olfactory cues of a competing conspecific to
adjust their aggressive response.

9 / 13

IBO 2013, SWITZERLAND

Practical Exam Evolutionary ethology

Task 2: Puffed throat behavior


In this task, you will check if a second aggressive behavior indicates the same pattern you found for the
number of ramming or biting attacks in the previous task by concentrating on the puffed throat
behavior.

Part 2.1: Puffed throat behavior of N. pulcher


N. pulcher males use puffed throats to impress conspecifics, possibly because it makes them appear
larger. A behavior is counted as puffed throat whenever the focal fish ares out its operculum and lower
jaw and swims in the direction of his competitor on the other side of the glass wall. Note that males may
flare out their operculum and lower jaw cavity for a very short time or an extended period of several
seconds. Each flaring out of the operculum and lower jaw is counted as a single event. Be aware that
some males may flare out their operculum and lower jaw multiple times within a few seconds, each of
which is counted as a single event.

Part 2.2: Examples of puffed throat behavior


Activate your tablet by tapping and sliding with your fingers and open the Folder E_2. In that folder you
will find three movies. Movie I contains a series of obvious puffed throat behaviors. Watch it to get a
feeling what to look for. Next, watch Movies II and III which indicate how this behavior is to be counted.
Watch them carefully. Proceed with the protocol as soon as you feel confident with how to count this
behavior in N. pulcher.

Part 2.3: Quantify puffed throat behavior of N. pulcher [21 points]


Watch the three movies found in each of the Folders A, B, C and D you already analyzed in the first task.
Count the number of puffed throat behaviors of the male in the right compartment for each of the
twelve movies and report your results in the table below. It is recommended to count by using tally
marks on the provided scratch paper.

Situation
Separation
Q11 Size

A
sealed
different

B
sealed
matched

C
porous
different

D
porous
matched

Replicate 1
Replicate 2
Replicate 3

10 / 13

IBO 2013, SWITZERLAND

Practical Exam Evolutionary ethology

Part 2.4: Interpret your results [4 points]


Based on your observations, indicate with a tick () if each of the following statements is true or false.
true

Q12

false

These results are in line with N. pulcher males using visual cues of a competing
conspecific to adjust their aggressive response.
The results are in line with N. pulcher males using puffed throat behavior to bluff
about their size, which is most effective if males are of equal size.

11 / 13

IBO 2013, SWITZERLAND

Practical Exam Evolutionary ethology

Task 3: Social groups


In nature, N. pulcher lives almost exclusively in social groups made up of a dominant breeding pair and 1
to 20 smaller subordinates called helpers (average group size = 7 to 9). In each social group, the
breeding male is always the largest individual (5.67.0 cm in length), the breeding female is usually the
next largest fish in the social group (4.86.0 cm) and subordinate helpers are generally smaller (1.56.4
cm). Both breeders and helpers show three main behaviors:
1. territory defense (against predators and conspecic or heterospecic territory competitors)
2. territory maintenance (by digging and removing debris)
3. brood care (by cleaning and fanning eggs and defending the young)
The goal is to understand task sharing in such groups by observing such a group for several minutes.

Part 3.1: Quantifying task sharing in social groups of N. pulcher [15 points]
Activate your tablet by tapping the screen with your fingers and open the Folder X. This folder contains
the Movie x with two sequences showing typical territory maintenance and brood care behavior of the
simplest social group of N. pulcher consisting of a large breeding male, a slightly smaller breeding female
and a small helper. You can recognize each individual easily in the very first scene of the sequence (see
figure below): the breeding male (A) is standing in the ceramic nesting cave in the background on the
right and the breeding female (B) is swimming in open water on the left. The only eggs in the aquarium
are attached to the top of the inner wall of the ceramic cave (C) within which the helper (D) hides.

While watching this movie, you will focus on two typical behaviors:
Digging behavior is a typical territory maintenance behavior in which the fish takes up sand,
carries it away in its mouth and spits it out at another location. Count the number of digging
sequences performed for each of the three individuals.
N. pulcher individuals perform egg care by cleaning the eggs from small particles such as fungi or
biofilms. This behavior is visible by back-and-forth movements in front of the eggs and
simultaneous mouth movements. Estimate the total time spent on this behavior by each
individual.

Report the total number of digging behavior for each individual in the table below.

12 / 13

IBO 2013, SWITZERLAND

Practical Exam Evolutionary ethology

Breeding male
Q13

Breeding female

Total number of digging


behavior events displayed

Helper individual
D

Indicate with a tick () the time spent on egg care for each of the three individuals in the table below.
Breeding male

Breeding female

no egg caring or caring for < 5s

Helper individual
D

egg caring for > 5s and < 30s


Q14

egg caring for > 30s

Part 3.2: Interpret your observations [4 points]


Indicate with a tick () if each of the following statements is true or false.
Assuming that the observed social group is a good representation of the majority
of social groups in nature, we would conclude that

true

false

helpers are likely to have a larger effect on the survival rates of clutches of
breeding pairs after an oligotrophic (nutrient-poor) environment was suddenly
converted into a highly eutrophic (hypertrophic) environment.
large-bodied breeding males are crucial in maintaining breeding caves by
digging large quantities of sand.
... the presence of helpers allows the breeding female to spend most of her time
patrolling the territory rather than on territory maintenance.
Q15

... helper individuals accommodate the large breeding male by following it and
maintaining the cave currently occupied by it.

End of practical exam.

13 / 13

IBO 2013, SWITZERLAND

Practical Exam Comparative and Functional Biosystematics


Student Code:

24th International Biology Olympiad


14th-21st July, 2013
Bern, Switzerland

Practical Exam 4
Comparative and Functional Biosystematics
Total points: 100
Duration: 90 minutes

1 / 17

IBO 2013, SWITZERLAND

Practical Exam Comparative and Functional Biosystematics

Dear participants,
This test consists of three tasks:
Task 1: Determine presence or absence of characters [42 points]
Part 1.1: Description of morphological characters
Part 1.2: Determine all character states [42 points]
Task 2: Resolve phylogenetic relationships [24 points]
Part 2.1: How maximum parsimony works
Part 2.2: Resolve the phylogenetic relationship using parsimony [10 points]
Part 2.3: Identify characters reflecting diet [4 points]
Part 2.4: How UPGMA works
Part 2.5: Resolve the phylogenetic relationship using UPGMA [10 points]
Task 3: Rate of morphological evolution [34 points]
Part 3.1: Obtain necessary skull measurements [15 points]
Part 3.2: Calculate the morphological distance between species pairs [15 points]
Part 3.3: Plot the phylogenetic versus the morphological distance [2 points]
Part 3.4: Interpret the results [2 point]
Appendix: Character states (presence or absence) of all specimens

5
5
5
6
6
7
8
9
11
13
13
13
15
16
17

Please write your student code into the box on the title page.

There is no separate answer sheet. Please fill in your answers into the specific answers boxes indicated
with a gray background. Only answers given inside these boxes will be evaluated.
The answers have to be given either with a tick () or with Arabic numbers. The numbers "1" and "7" can
look very similar in handwriting. To make sure that those two numbers can be well distinguished by the
IBO staff, please write them as you normally would into the following box.

1=

7=

Stop answering and put down your pen IMMEDIATELY when the bell rings at the end of the exam. Put
the entire protocol with all the answers back into the exam envelope. Make sure you wrote your
student code into the box on the title page.

2 / 17

IBO 2013, SWITZERLAND

Practical Exam Comparative and Functional Biosystematics

Material and equipment


Make sure that you have received all the materials and equipment listed for each task. If any of these
items are missing, please raise your hand.

Equipment
1 sliding caliper (150 mm)
1 ruler (300 mm)
1 calculator
scratch paper

Skull specimen
Arvicola terrestris
Capreolous capreoulus
Felis catus
Lepus europaeus
Meles meles
Sciurus vulgaris
Vulpes vulpes

arte
caca
feca
leeu
meme
scvu
vuvu

European water vole


European roe deer
domestic cat
European/brown hare
European badger
Eurasian red squirrel
red fox

Additional taxa for which data is provided


Ceratotherium simum
Didelphis marsupialis
Equus ferus
Galago senegalensis
Genetta genetta
Hippopotamus amphibius
Inia geoffrensis
Manis pentadactyla
Macropus rufus
Mustela nivalis
Pipistrellus pipistrellus
Procavia capensis
Pteropus vampyrus
Sorex minutus
Tachyglossus aculeatus
Tolypeutes matacus

cesi
dima
eqfe
gase
gege
hiam
inge
mape
maru
muni
pipi
prca
ptva
somi
taac
toma

white or square-lipped rhinoceros


common opossum
horse
Senegal bushbaby
common genet
hippopotamus
Amazon river dolphin
Chinese pangolin
red kangaroo
least weasel
common pipistrelle
rock or cape hyrax
Malaysian large flying fox
Eurasian pygmy shrew
short-beaked echidna
southern three-banded armadillo

Print outs
skull diagrams
color print outs illustrating the presence (1) and absence (-) of the morphological characters.
Note legend on top.

3 / 17

IBO 2013, SWITZERLAND

Practical Exam Comparative and Functional Biosystematics

Introduction
Mammals, which consist of more than 5700 extant species, are omnipresent on earth and often play key
roles in shaping terrestrial and aquatic ecosystems, both as predators and herbivores. Mammals are
morphologically very diverse. The smallest mammals, for instance, weigh only about 2 g, whereas the
blue whale, the largest animal that ever existed, is more than 100 million times larger.
In this practical exam, you will attempt to resolve the phylogeny of some representatives of major
mammal groups based on morphological characters of the skull (teeth and jaw), as well as molecular
data. You will use cladistics, an approach to infer evolutionary relationships, which assumes that
members of a taxonomic group have a common evolutionary history. They have therefore inherited a
set of derived traits (synapomorphies) from their last common ancestor, which distinguishes members
of this group from other groups.
Cladistic analysis consists of identifying specific traits and determining their state in all organisms
included in the analysis. Traits can be both molecular (e.g. the specific amino acid at a specific position in
a protein) or morphological (e.g. the presence or absence of a particular bone feature). The outcome of
a cladistic analysis is a cladogram - a tree-shaped diagram (dendrogram) that represents the inferred
phylogenetic relationships between organisms.

4 / 17

IBO 2013, SWITZERLAND

Practical Exam Comparative and Functional Biosystematics

Task 1: Determine presence or absence of characters [42 points]


Part 1.1: Description of morphological characters
The following table lists all morphological characters for which the presence (1) or absence (-) will be
considered in this practical. Examples for each character are found on a color print out (note the legend
on top of each page). For Characters 7 through 10, only examples of presence (1) are shown. All
abbreviations given in brackets (e.g. [P]) refer to positions indicated in the skull diagrams. Note that the
positions of incisors [I], canines [C], premolars [P] and molars [M] are enumerated from the snout to the
back of the skull using superscript (upper jaw) or subscript (lower jaw). For example, [P3] refers to the
third premolar on the upper jaw.
Number

Description

Incisors [I] are present on the upper jaw.

Elongated incisors [I] with abrasive (rubbed) edges indicating continuously growth through
life on upper and/or lower jaw.

Incisors [I] on the front side coated with dental enamel (orange color).

Third upper incisors [I3] reduced and positioned behind the second incisors.

Diastema on upper jaw. A diastema is defined as a large gap (more than two times the
width of the largest tooth adjacent to the gap) between the position of incisors [I] or
canines [C] and the premolars [P] or molars [M]. Some of these teeth may be absent.

Canines [C] are present on the upper jaw.

Carnassial dentition, characterized by [P4] in the upper jaw and [M1] in the lower jaw
forming a scissor-like structure which allows cutting and breaking, but not chewing.

Selenodont teeth, characterized by low crowns and longitudinal crescent-shaped cusps,


when viewed from above. They form a series of triangular cusps when seen from the side.

Lophodont teeth, characterized by elongated ridges that run between cusps. Ridges are
perpendicular to the jaw (like an old-fashioned washboard).

10

Teeth characterized by a well-developed V- or W-shaped ectoloph (crest).

11

Pointed teeth forming conical tips.

12

The angular extension [Ang. Pr.] (Processus Angularis) of the lower jaw is bent medial
(towards the centre) and therefore more medial than [Cond. Pr.] (Processus condularis).

Part 1.2: Determine all character states [42 points]


For the later analysis, it is convenient to first determine the presence or absence of each of the
morphological characters considered here in each of the species included in the analysis, and to report
them in the table provided in Appendix (last page). Indicate presence with a 1 and absence with a -.
Please be extra careful when handling the skulls. All specimens are originals and part of the collection
of the Natural History Museum of Bern.

5 / 17

IBO 2013, SWITZERLAND

Practical Exam Comparative and Functional Biosystematics

Task 2: Resolve phylogenetic relationships [24 points]


The goal of this task is to resolve the phylogenetic relationship among major mammalian groups. You
will use two methods that will be explained in detail. In Part 2.1-2.3, you will apply the concept of
maximum parsimony to the presences and absences determined in Task 1 to estimate the phylogenetic
relationship between several taxa representing different major groups of mammals. In Part 2.4 2.5,
you will use the UPGMA (Unweighted Pair Group Method with Arithmetic Mean) approach to resolve
the phylogeny among Carnivora to complete a more detailed phylogeny obtained from molecular data.

Part 2.1: How maximum parsimony works


A common approach to resolve phylogenetic relationships between species is to identify the most
parsimonious (the simplest) among competing phylogenies. A phylogenetic tree is called the most
parsimonious if the fewest number of character state changes are invoked. To illustrate that concept,
consider the following two competing phylogenies I and II for three taxa A, B and C.

To choose among those phylogenies, the presence or absence of morphological characters 1 through 4
was assessed for each taxa and reported in the table below. All of these characters are known to be
absent in closely related taxa, indicating that each character was absent at the root of the phylogenies
(indicated by a black dot in the diagrams above).

A
B
C

1
1
1

2
1
1

3
1
-

4
1
1
-

First, the minimal number of state changes has to be determined for each character and phylogeny
individually. Trait 4, for instance, is present in Taxa A and B, but not in Taxon C, indicating that the state
of this character changed from absent to present along the lineages leading to A and B. While this
change could have happened independently on Branches A and B, the explanation leading to the
minimal number of state changes (and hence the most parsimonious solution) is a single change on the
branch leading to the most recent common ancestor of A and B.
As shown in the following figure, two equally parsimonious Solutions Ia and Ib can be found for
Phylogeny I when considering all characters. In Solution Ia, Characters 1 and 2 are assumed to have
arisen independently on Branches B and C. Alternatively (Solution Ib), these characters arose on the
branch coming from the root of the phylogeny, and were lost on the Branch A. Note that throughout
this practical, open circles indicate gains (changes from absent present) and black bars indicate
losses (state changes from present absent).

6 / 17

IBO 2013, SWITZERLAND

Practical Exam Comparative and Functional Biosystematics

A key observation is that any character for which the state in a single taxon differs from the state at the
root (such as Trait 3 in this example) invokes a single character change, regardless of the phylogeny
considered. Such characters and those for which the state is identical in all taxa should be regarded as
uninformative and discarded prior to the analysis.
The following figure finally shows one of the most parsimonious solutions for each of the competing
phylogenies, along with the number of character state changes invoked by these solutions given in the
box. Thus, under the concept of maximum parsimony, Phylogeny II is chosen as the best estimate of the
true phylogenetic relationship between Taxa A, B and C.

Part 2.2: Resolve the phylogenetic relationship using parsimony [10 points]
You will now use maximum parsimony to resolve the phylogenetic relationship among the following
taxa, representing a major mammal clade:
Arvicola terrestris (arte)
Ceratotherium simum (cesi)
Equus ferus (eqfe)
Genetta genetta (gege)
Lepus europaeus (leeu)
Capreolus capreolus (caca)
Sorex minutus (somi)
First, indicate with a tick () if Characters 1 through 12 (page 5) are informative to resolve the phylogeny
of these taxa, or not. Note that all characters are known to be absent at the root of these taxa, with the
exception of 1, 6 and 11, which are present at the root.
Character

10

11

12

informative
Q2

not informative

7 / 17

IBO 2013, SWITZERLAND

Practical Exam Comparative and Functional Biosystematics

Next, indicate any of the most parsimonious solutions regarding character state changes for each of the
two competing phylogenies I and II directly into the figure. Indicate gains with empty circles and losses
with bars and write the number of the corresponding characters above it. Finally, indicate the total
number of state changes necessary in the box provided.

Q3

Indicate the phylogeny that represents the best estimate of the true phylogenetic relationship among
those 7 taxa according to the concept of maximum parsimony with a tick () in the table below
I

Q4

II

Preferred phylogeny according to the concept of maximum parsimony

Part 2.3: Identify characters reflecting diet [4 points]


Unless a large number of characters are included, maximum parsimony is often too simplistic an
approach to correctly disentangle the phylogeny of distantly related taxa. This is particularly true when
some character state differences between taxa reflect opposing adaptations to diet rather than a
phylogenetic signal. To evaluate this hypothesis, identify the characters that are typical for either an
carnivorous or a herbivorous lifestyle.
Among the species included in this practical, the following are very distantly related but are highly
carnivorous and prey on living Arthropoda, Mollusca, Annelida and/or Vertebrata:
Didelphis marsupialis (dima)
Galago senegalensis (gase)
Inia geoffrensis (inge)
Mustela nivalis (muni)
Pipistrellus pipistrellus (pipi)
Sorex minutus (somi)
Tolypeutes matacus (toma)

8 / 17

IBO 2013, SWITZERLAND

Practical Exam Comparative and Functional Biosystematics

The following are very distantly related species but are known to be highly herbivorous and feed on
grass, leaves, fruits and roots:
Arvicola terrestris (arte)
Capreolous capreolus (caca)
Equus ferus (eqfe)
Lepus europaeus (leeu)
Macropus rufus (maru)
Procavia capensis (prca)
Pteropus vampyrus (ptva)
Indicate with a tick () in the table below for each Character 1 through 12 whether its presence or
absence is typical for either an carnivorous or herbivorous lifestyle. Here, a typical character state is
defined as any character state shared by at least three of the taxa from one of the two groups but
absent from all members of the other group.
Character

10

11

12

typical for carnivores


Q5

typical for herbivores


not typical for either

Part 2.4: How UPGMA works


An approach that is purely based on the presence or absence of morphological characters does not lend
itself readily for estimating evolutionary distance between taxa quantitatively. This is where molecular
data, which provides a large set of characters with similar state change probabilities, becomes very
handy. A simple, iterative approach to estimate a phylogeny along with the relative length of each
branch from molecular data is UPGMA (Unweighted Pair Group Method with Arithmetic Mean), in
which the pair of clusters with the shortest distance is combined into a higher level cluster at each
iteration.
To illustrate that concept, consider the molecular distances (e.g. number of base pair differences)
between the Taxa A, B, C and D.

A
B
C
D

A
0
6
7
8

B
0
8
9

C
0
4

D
0

Iteration 1: First, the pair of clusters with the smallest distance is the pair C and D, which is combined
into a higher-order cluster (C,D). The relative age of the newly formed cluster is computed as half of the
distance between the two clusters combined. In this case, the relative age of the cluster is thus 2.

9 / 17

IBO 2013, SWITZERLAND

Practical Exam Comparative and Functional Biosystematics

Next, a new matrix of all distances is generated by computing the distance between clusters as the
average distance between all taxa from one cluster to all taxa of the other cluster. The distance between
cluster A and cluster (C,D), for instance, is computed as the average between d(A,C) and d(A,D), where
d(x,y) is a notation to indicate the distance between clusters x and y.

A
B
(C,D)

A
0
6
7.5

B
0
8.5

(C,D)
0

Iteration 2: The pair of clusters with the smallest distance is now the pair of A and B, which is thus
combined into a higher level cluster (A,B) with a relative age of 3. Again, distances are recomputed as
indicated above. The distance d((A,B),(C,D)) is thus the average between d(A,C), d(A,D), d(B,C) and
d(B,D), which is the same as the average between d(A,(C,D)) and d(B,(C,D)).

(A,B)
(C,D)

(A,B)
0
8

(C,D)
0

Iteration 3: In the last iteration, the two remaining taxa are combined into the new cluster ((A,B),(C,D))
and the relative age of this cluster is 4.
Computing branch length: After all taxa have been combined into a single cluster, the resulting tree is
plotted with the relative age of each cluster. From these relative times, it is then straight forward to
compute individual branch length.

10 / 17

IBO 2013, SWITZERLAND

Practical Exam Comparative and Functional Biosystematics

Part 2.5: Resolve the phylogenetic relationship using UPGMA [10 points]
Resolve the phylogenetic relationship of all five Carnivora species included in this practical iteratively
using the UPGMA method. Standardized molecular distances between all pairs of these taxa are given in
the table below.

vuvu
muni
gege
meme
feca

Vulpes vulpes
Mustela nivalis
Genetta genetta
Meles meles
Felis catus

A
B
C
D
E

A
0.00
4.18
4.96
4.18
4.96

B
0.00
4.96
0.48
4.96

C
0.00
4.96
2.82

D
0.00
4.96

E
0.00

Proceed iteratively and always fill in the table with the molecular distances considered. Make sure to
always report the code of the cluster in the top and left most row. Use the notation introduced above to
denote higher level clusters. The term ((x,y),z), for instance, should refer to a cluster made of the
clusters (x,y) and z. Also, report the relative age of the newly formed cluster in each iteration (precision:
two digits after the decimal point).
Iteration 1
Relative age of newly
formed cluster

0.00
Q6

0.00
0.00
0.00
Iteration 2
Relative age of newly
formed cluster

0.00
Q7

0.00
0.00
Iteration 3
Relative age of newly
formed cluster

0.00
Q8

0.00

Iteration 4
Relative age of newly
formed cluster
Q9

11 / 17

IBO 2013, SWITZERLAND

Practical Exam Comparative and Functional Biosystematics

Now draw the resulting phylogeny of the five species to scale and indicate the length of each branch on
the top and the species code on the right of it. Note that 5 mm indicates 0.1 unit of relative age of your
calculated tree.

Q10

12 / 17

IBO 2013, SWITZERLAND

Practical Exam Comparative and Functional Biosystematics

Task 3: Rate of morphological evolution [34 points]


If morphological changes happen at a constant rate, the morphological difference between pairs species
measured at multiple characters should be highly correlated with the phylogenetic distance between
the species as estimated from molecular data. In this task, you will test this hypothesis by using a series
of skull measurements. While the results are provided for many additional species pairs, you will
perform the necessary measurements and calculations for the species pair Arvicola terrestris (arte) and
Lepus europaeus (leeu).

Part 3.1: Obtain necessary skull measurements [15 points]


The table below indicates a series of skull features which you will have to measure for the two species
mentioned above. All measurements are also indicated in the skull diagrams.

M1
M2
M3
M4
M5
M6
M7

Description
distance in mm between [I1] and [NF1].
distance in mm between [Ang. Pr.] and end of masseter depression (masseter
muscle attachment point).
distance in mm between [Ang. Pr.] and [Cond. Pr.].
distance in mm between [I] and [Cond. Pr.].
distance in mm between end of eyes and end of [PA]
distance in mm between eyes and [PMX].
distance in mm between [PMX] or [NA] or [I] and [Cond. Pr.].

While results for features M5 and M6 will be provided below, measure the five features (M1 through M4
and M7) for both specimens (arte and leeu). Focus on the right side of the skull (the left side when facing
the skull). Report your results in mm in the table to a precision of 0.1 mm
M1
Q11

M2

M3

M4

M5

M6

M7

arte
Arvicola terrestris
leeu
Lepus europaeus

Part 3.2: Calculate the morphological distance between species pairs [15 points]
A major hurdle when studying different skull features is that they are primarily determined by the
overall size of the specimen. To study skull shape, begin by correcting for the overall size of the skull by
dividing each measurement by the length of the skull (M7) and taking the logarithm to base 10 from this
ratio. For each measurement Mi = {M1, ..., M4}, calculate:

Report your results in the table below with a precision of three digits after the decimal point.

13 / 17

IBO 2013, SWITZERLAND

Practical Exam Comparative and Functional Biosystematics

arte
Q12 Arvicola terrestris
leeu
Lepus europaeus

Next, compute the absolute morphological distance


measurement i as

-0.367

-0.478

-0.547

-0.382

between the two species x and y for each

where
is the standardized measure i of species x. The absolute difference is the difference
regardless of whether it is negative or positive (indicated by | |). Report your results with a precision of
three digits after the decimal point in the table below.

arte and leeu


Q13

When combining the different measures into a single distance, it is important to first standardize the
distances in order to give them equal weight. A simple procedure to standardize is to divide the absolute
differences
by the median difference across all species pairs considered:

The appropriate medians are indicated in the table below.

0.084

0.139

0.067

0.026

0.110

0.082

Standardize all distances and report your results with a precision of three digits after the decimal point
in the table below.

arte and leeu


Q14

The combined morphological distance


standardized distances of that pair:

of a pair (x,y) is defined as the sum across all

14 / 17

IBO 2013, SWITZERLAND

Practical Exam Comparative and Functional Biosystematics

Calculate
for arte and leeu and report your results with a precision of three digits after the
decimal point in the table below.

arte and leeu


Q15

Part 3.3: Plot the phylogenetic versus the morphological distance [2 points]
The graph below shows the phylogenetic distance
against the morphological distance
for pairs of some of the species listed in the Appendix, along with a linear regression best
explaining the observed values. Add your calculated distances to this plot by using a dot with a circle
around, similar to the other points. The phylogenetic distance between arte and leeu is 6.80.

Q16

15 / 17

IBO 2013, SWITZERLAND

Practical Exam Comparative and Functional Biosystematics

Part 3.4: Interpret the results [2 point]


The plot of the phylogenetic versus the morphological distance shows two obvious outliers, points P and
T. Indicate with a tick () for each of the following hypotheses whether it could explain these outliers or
not.
yes

Q17

no

Both species of pair P are closely related but were geographically separated
and evolved convergent adaptations to very similar habitats.
The ancestor of one species of pair P colonized a totally different
environment with very different selection pressures, changing its skull
morphology dramatically.
Both species of pair T are part of a fast, ecologically driven adaptive
radiation.
While only distantly related, both species of pair T feed on very similar
resources, for which their skulls evolved convergent adaptations.

End of practical exam

16 / 17

IBO 2013, SWITZERLAND

Practical Exam Comparative and Functional Biosystematics

Appendix: Character states (presence or absence) of all specimens


Presences are indicated with a 1 and absences with a -.
Character
arte
Q1

caca
feca
leeu
meme
scvu
vuvu
cesi
dima
eqfe
gase
gege
hiam
inge
mape
maru
muni
pipi
prca
ptva
somi
taac
toma

Arvicola
terrestris
Capreolus
capreolus
Felis
catus
Lepus
europaeus
Meles
meles
Sciurus
vulgaris
Vulpes
vulpes
Ceratotherium
simum
Didelphis
marsupialis
Equus
ferus
Galago
senegalensis
Genetta
genetta
Hippopotamus
amphibius
Inia
geoffrensis
Manis
pentadactyla
Macropus
rufus
Mustela
nivalis
Pipistrellus
pipistrellus
Procavia
capensis
Pteropus
vampyrus
Sorex
minutus
Tachyglossus
aculeatus
Tolypeutes
matacus

10

11

12

17 / 17

INTERNATIONAL BIOLOGY OLYMPIAD


THEORY PROBLEMS

2012, Singapore















All IBO examination questions are published under the following Creative Commons license:



CC BY-NC-SA (Attribution-NonCommercial-ShareAlike) https://creativecommons.org/licenses/by-nc-sa/4.0/
The exam papers can be used freely for educational purposes as long as IBO is credited and
new creations are licensed under identical terms. No commercial use is allowed.

IBO2012 Singapore

Theoretical Test Paper 1

Country: _____________________

Student Code: ________________

23rd INTERNATIONAL BIOLOGY OLYMPIAD


8th 15th July, 2012
SINGAPORE

THEORETICAL TEST PAPER 1


Write all answers in the ANSWER SHEET

Page 1 of 49

IBO2012 Singapore

Theoretical Test Paper 1

Dear Participants

You have a total of 3 hours (180 minutes) for answering this theory paper.

Use the Answer Sheet, which is provided separately, to answer all the questions.

The answers written in the Question Paper will NOT be evaluated.

Write your answers legibly. Note that there may be more than one correct/incorrect
answer and every cell should be filled.
For example:

NOTE: Some of the questions may be marked Skipped / Deleted. DO NOT attempt
these questions. Also, read the question completely before attempting it as some
questions may continue from one page to the next.

The maximum number of points for this paper is 89.3.

Stop answering and put down your pen IMMEDIATELY when the bell rings.

Your Answer Sheets as well as the Theoretical Test question paper will be collected at
the end of the test period.

Good Luck!

Page 2 of 49

IBO2012 Singapore

Theoretical Test Paper 1

CELL BIOLOGY
1.

The Table below shows the genetic codes of amino acids.

Some viruses (e.g. tobacco mosaic virus (TMV)) have RNA sequences that contain a "leaky"
stop codon. In TMV 95% of the time the host ribosome will terminate the synthesis of the
polypeptide at this codon but the rest of the time it continues past it.
The following sequences show part of a mRNA from TMV. Indicate the sequence(s) that may
result in two polypeptides in the indicated frame with a tick () and those that will not with a
cross (). (1.8 points)
a.

5-AUG-UCU-UGU-CUU-UUC-ACC-CGG-GGG-UAG-UAU-UAC-CAU-GAU-GGU-UAA-3

b.

5-AUG-ACC-CGG-GGG-UUU-CUU-UUC-UAG-UAU-GAU-CAU-GAA-GGU-UGU-UAA-3

c.

5-AUG-CUU-UUC-UCU-UAU-UAG-CAU-GAU-GGU-UGU-ACC-CGG-GGG-CCC-UAA-3

d.

5-AUG-CAU-GUU-CUU-UUC-UCU-UAU-UGU-GGU-UGU-ACC-CGG-GGG-UUC-UAA-3

e.

5-AUG-CAU-GAU-GGU-UGU-ACC-CGG-GGG-UAG-CUU-UUC-UCU-UAU-UGC-UAA-3

f.

5-AUG-UCU-UAU-UGG-CAU-GAU-GGU-UGU-CUU-UUC-ACC-CGG-GGG-AAA-UAA-3

Page 3 of 49

IBO2012 Singapore

2.

Theoretical Test Paper 1

Mitochondria are mainly concerned with the following functions:


a.

thermogenesis

b.

apoptosis

c.

production of ATP

d.

fatty acid metabolism

Indicate the extensive presence of mitochondria with a tick (), intermediate presence (-) and
absence of mitochondria with a cross (). Match the key function(s) of mitochondria (a to d)
suited to the respective cells. (1.8 points)

3.

Arrange the order of the DNA molecules from lowest to highest in terms of their melting
temperature (Tm). (0.9 points)
a.

5-AAGTTCTCTGAA-3
3-TTCAAGAGACTT-5

b.

5-AGTCGTCAATGCGG-3
3-TCAGCAGTTACGCC-5

c.

5-GGACCTCTCAGG-3
3-CCTGGAGAGTCC-5

Page 4 of 49

IBO2012 Singapore

4.

Theoretical Test Paper 1

There are various mechanisms by which a cell can commit suicide a phenomenon known as
apoptosis. One of the mechanisms is triggered by reactive oxygen species. The outer
membrane of mitochondria normally expresses a protein Bcl-2 on its surface. Another protein
Apaf-1 binds Bcl-2. Reactive oxygen species cause Bcl-2 to release Apaf-1 and a third protein
Bax to penetrate the mitochondrial membrane, releasing cytochrome c. The released
cytochrome c forms a complex with Apaf-1 and caspase 9. This complex sequentially activates
many proteases that digest cellular proteins.

What will be the fate of a cell exposed to reactive oxygen species in the following conditions?
I.

The cell has expressed a mutant form of Apaf-1 that constitutively (always) bind Bcl-2.

II.

The cell does not express Bcl-2 at all.

III.

The cell overexpresses a form of Bcl-2 that is targeted to cell membrane only.

IV.

A chemical which extends the half life of Bcl-2 is added to the cell.

Match the following fates of the cell with the conditions (I to IV). (2 points)
a. The cell resists apoptosis.
b. The cell is forced towards apoptosis.
c. The fate of the cell cannot be predicted.

Page 5 of 49

IBO2012 Singapore

5.

Theoretical Test Paper 1

The Table below shows the chemical structure, pK1, pK2 and pKR of some amino acids.
pK1
-COOH

pK2
-NH2

pKR
side chain

Glycine (Gly)

2.35

9.78

---

Alanine (Ala)

2.35

9.87

---

Serine (Ser)

2.19

9.21

---

Aspartic acid (Asp)

1.99

9.9

3.9

Glutamic acid (Glu)

2.1

9.47

4.07

Lysine (Lys)

2.16

9.06

10.54

Amino acid

Structural formula

5.1. Determine the predominant form (ionic or neutral) for heptapeptides, A to C, at pH 1, pH 7


and pH 12. Calculate their corresponding net charges (with an integer approximation).
(3.6 points)

5.2. What is the best pH for the electrophoretic separation of these three peptides from each
other?
Indicate the best pH with a tick () and the other pH values with a cross (). (0.6 points)

Page 6 of 49

IBO2012 Singapore

6.

Theoretical Test Paper 1

Which of the following sequence(s) of cell-cycle phases is/are characteristic of eukaryotes [G:
gap; S: synthesis; M: mitosis]? Indicate correct sequence(s) with a tick () and incorrect ones
with a cross (). (0.5 points)

7.

a.

G1 - S - G2 - G0 - M

b.

G0 - G1 - S - G2 - M

c.

G1 - G0 - G2 - S - M

d.

G1 - G0 - G1 - G2 - S - M

e.

G1 - G0 - G1 - S - G2 - M

About the G2 phase


7.1. Which of the statement(s) describe(s) a cell in the G2 phase? Indicate correct statement(s)
with a tick () and incorrect ones with a cross (). (0.4 points)
a.

The homologous chromosomes are lined up on the equator.

b.

The homologous chromosomes have been pulled to their respective poles by the
spindle apparatus.

c.

The homologous chromosomes have not been replicated yet.

d.

The homologous chromosomes are now in the haploid or n condition.

7.2. How many chromatin threads are there in a human somatic cell in the G2 phase? (0.5
points)

Page 7 of 49

IBO2012 Singapore

8.

Theoretical Test Paper 1

The morphology of three species of bacteria (A to C) are shown below:

8.1. Bacteria in nature prefer to attach to surfaces and exist in a form known as biofilms.
During the attachment stage, before reaching the surface for attachment, bacteria will
encounter a zone of repulsive force as they come very close to the surface. Which bacteria
are likely to have an advantage to overcome this repulsive zone? Indicate the correct
answer(s) with a tick () and incorrect answer(s) with a cross (). (0.6 points)
a.

Bacterium A

b.

Bacterium B

c.

Bacterium C

8.2. After overcoming the repulsive zone and reaching the surface, the strength of attachment
on the surface of the three bacteria is likely to be different. Arrange the correct order of
attachment strength of the three bacteria in the Answer Sheet. (0.6 points)

Page 8 of 49

IBO2012 Singapore

Theoretical Test Paper 1

8.3. A stagnant pool of water was originally rich in organic content, but the nutrient
concentration soon became diluted with rainwater. All three bacteria were affected and
would attempt to survive the best way they could under this condition. Indicate true
statement(s) with a tick () and incorrect statement(s) with a cross (). (0.6 points)

9.

a.

Bacterium A allows the fastest relative diffusion of nutrients into the interior of its cell.

b.

Bacterium B can extend its flagellum to reach nutrients above the water level.

c.

Bacterium C has a capsule which can actively absorb more nutrients.

A laboratory technician stained unknown bacterial cells with different dyes before observing
them. The dyes (stains) used are known to target (i) lipopolysaccharide, (ii) nuclear envelope,
(iii) DNA, (iv) cytoplasm and (v) ribosomes. Which dyes are likely to stain positive no matter
what type of bacteria there may be in the sample? Indicate with a tick () if they will be stained
and with a cross () if they will not be stained. (1 point)

Page 9 of 49

IBO2012 Singapore

Theoretical Test Paper 1

PLANT ANATOMY AND PHYSIOLOGY


10. Study the transverse section of a root in the figure below.

10.1. Match the codes (1 18) given in the table below with the labeled parts (A to H) in the
above figure. (1.6 points)
No.

Part

No.

Part

Hypodermis

10

Sclerenchyma cell

Epithelial cell

11

Casparian strip

Xylem parenchyma

12

Central vacuole

Epidermal cell

13

Phloem parenchyma

Xylem fiber

14

Pericycle

Root hair

15

Companion cell

Exodermal cells

16

Phloem fiber

Xylem vessel

17

Endodermal cell

Cortical parenchyma cell

18

Collenchyma cell

Page 10 of 49

IBO2012 Singapore

Theoretical Test Paper 1

10.2. The following are three pathways of ion and water absorption:
I.

a symplastic pathway

II.

an apoplastic pathway

III. a transmembrane pathway

Draw continuous lines and label (with I, II and III) the three different pathways from the
outside to H in the figure provided in the Answer Sheet. (3 points)

11. Match plant structures (1 10) with the corresponding function (A J). (3 points)
Plant cell / Tissue structure

Function(s) / Feature(s)

Thylakoid membranes

Vascular cambium

Central vacuole

Production of new plant tissues/organs

Plasmodesmata

Modified parenchyma cell without nucleus

Apical meristem

Periderm

Mechanical support

Sieve tube

Presence of electron transport proteins

Trichome

Production of secondary vascular tissues

Secondary cell wall

Secondary protective tissue

Micropyle

Protection and absorption

10

An intercellular communication network


Storage of water, digestive enzymes and other
inorganic and organic substances

Small opening in the surface of an ovule,


through which the pollen tube penetrates.

Page 11 of 49

IBO2012 Singapore

Theoretical Test Paper 1

12. Study the graph below and determine which of the statements (a to h) are correct.
Indicate correct answer(s) with a tick () and incorrect ones with a cross (). (1.4 points)

a.

It is a photosynthetic O2 response curve.

b.

Point A is light saturation point.

c.

Point B is light compensation point.

d.

C is the maximal photosynthetic rate.

e.

Plants stop growth when they grow under the irradiance greater than the value shown at
point B.

f.

Respiration rate is greater than photosynthetic rate when plants are grown under the light
below the value shown at point A.

g.

Plants grow (accumulate biomass) when their growth light environments are higher than
the photon flux shown at point A.

Page 12 of 49

IBO2012 Singapore

Theoretical Test Paper 1

13. Study the light response curves for leaf photosynthesis of C4 and C3 plants shown below.
Indicate correct statement(s) with a tick () and incorrect statement(s) with a cross (). (1.2
points)

a.

Figure A demonstrates the characteristics of C4 plants.

b.

C3 plants have a competitive advantage over C4 plants at high temperature and under full
sunlight because of a reduction in photorespiration.

c.

C3 plants have a competitive advantage over C4 plants at low temperature and under low
light because of the higher quantum yield.

Page 13 of 49

IBO2012 Singapore

Theoretical Test Paper 1

14. Some statements about photosynthesis are given below. Indicate true statement(s) with a tick
() and false statement(s) with a cross (). (1.0 points)
a.

Photophosphorylation involves ATP formation during the light reaction of photosynthesis.

b.

The essential initial role of light in initiating the light reaction of photosynthesis is to produce
free oxygen.

c.

In a plant cell, the ATP synthase complexes are only located in the thylakoid membrane.

d.

Photosystem II is required for cyclic photophosphorylation.

e.

It is currently believed that the specific enzymes necessary for the fixation of CO 2 into
sugar are located in the chloroplast stroma.

15. Arrange the following plants A to C in evolutionary order starting with the most primitive
specimen to the most modern. (1.5 points)

Vascular Tissues

Page 14 of 49

IBO2012 Singapore

Theoretical Test Paper 1

16. Match the description or effect (A J) with their corresponding terms (1 10). (1.8 points)

Term

Description / effect
Physiological reaction of organisms to the length of day or

Ethylene

A
night

Photoperiodism

Inhibition of the growth of lateral buds

Apical dominance

Stem elongation in intact plants


Prolonged exposure to cold temperatures promotes

Thigmotropism

D
flowering

Phyllotaxy

Leaf and fruit abscission

Cytokinin

Bending of growing stems toward light sources

Gibberellin

The arrangement of leaves on a stem

Statolith

The response of plants to touch

Vernalization

Delayed senescence

Gravitropism

Page 15 of 49

IBO2012 Singapore

Theoretical Test Paper 1

ANIMAL ANATOMY AND PHYSIOLOGY


17. Referring to the events (I to VIII) below, fill in the correct sequence of events during cardiac
excitation-contraction coupling. (1.5 points)

[DELETED]

I.

action potential triggers opening of Ca2+ channels in the endoplasmic reticulum

II.

release of intracellular Ca2+ store

III.

action potential triggers opening of L-type Ca2+ channels

IV.

Ca2+ influx from the extracellular space

V.

cytosolic Ca2+ binds to troponin

VI.

cytosolic Ca2+ binds to tropomyosin

VII.

crossbridge forms as myosin heads bind to actin

VIII.

crossbridge forms as actin heads bind to tropomyosin

18. Given below are data on the breathing rate, heart rate and body temperature of four different
mammals, A to D.

Animals

Breathing rate
(inhalations/min)

Heart rate
(beats/min)

Body temperature
(C)

160

500

36.5

15

40

37.2

28

190

38.2

28

35.9

18.1. Rank Animals A to D in descending order for surface area per unit volume of the body.
(0.8 points)

18.2. Rank Animals A to D in descending order for total volume of blood in the body. (0.8
points)

Page 16 of 49

IBO2012 Singapore

Theoretical Test Paper 1

19. Evaluate the following two statements regarding the respiratory processes of amphibians,
reptiles, birds and mammals. Indicate true statement(s) with a tick () and false statement(s)
with a cross (). (1.6 point)
I. Negative pressure used to force air into lungs
II. Lungs are completely ventilated during each breathing cycle

20. Gas exchange in animal taxa involves various respiratory organs (a d), as well as, the
circulatory system (open and closed). For each animal, indicate open circulatory system(s) with
a tick () and closed circulatory system(s) with a cross (). Match the appropriate organs (a
d) with the animals (adult). (2.6 points)

a.

lungs

b.

gills

c.

skin

d.

trachea

Page 17 of 49

IBO2012 Singapore

Theoretical Test Paper 1

21. Urine production is the result of continuous filtration of plasma through the kidneys. Indicate
true statement(s) about the mammalian kidney with a tick () and false statement(s) with a
cross (). (2 points)

a.

The kidneys have a direct effect on blood pressure.

b.

The kidneys help regulate total blood volume in circulation.

c.

The loops of Henle remove water, ions and nutrients from the blood.

d.

Those able to excrete the most hyperosmotic urine, such as the kangaroo rats living in the
desert, have relatively short loops of Henle.

e.

The kidneys partner the lungs in controlling the pH in plasma.

f.

The kidneys help maintain blood pH by excreting hydrogen ions and reabsorbing
bicarbonate ions as needed.

g.

The kidneys dispose of volatile acids produced in metabolism.

h.

Ammonia (NH3) is produced in proximal tubule cells during acidosis.

i.

The glomerular filtration rate is affected by blood pressure.

j.

The kidneys produce ADH (antidiuretic hormone).

22. The amount of saliva secreted by a mammal is related to how much chewing is required on
feeding. Match the following animals (a e) to the quantity of saliva secreted as given in the
table in the Answer Sheet. (0.8 points)
a.

wolf

b.

horse

c.

cattle

d.

human

Page 18 of 49

IBO2012 Singapore

Theoretical Test Paper 1

23. Allergy is a hypersensitive human immune system reaction which is a result of repeated antigen
exposure. In comparison, although pseudoallergy is identical to allergy in clinical terms, there is
no immunological stage in its development.

The underlying pathological processes are listed below as observations:


a.

General level of IgE class antibodies in the serum is raised.

b.

Specific IgE class antibodies in the serum is detected.

c.

Histamine the main mediator of inflammation is released.

d.

A minimal amount of the antigen is needed to demonstrate the reaction.

Indicate for allergy, as well as pseudoallergy, the observation(s) that apply with a tick () and the
observation(s) that do not with a cross () in the Answer Sheet.(0.8 points)

24. The age of animal fossils can be determined by measuring the content of carbon isotope 14C in
the bones. How is 14C accumulated in the bones? Indicate correct statement(s) with a tick ()
and incorrect statement(s) with a cross (). (0.6 points)

a.

through consumption and assimilation of organic compounds in the bones

b.

through converting CO2 into organic compounds in the bones

c.

through accumulation of residual CO2 during respiration deposited in the bones

Page 19 of 49

IBO2012 Singapore

Theoretical Test Paper 1

25. European (freshwater) eel usually obtains oxygen by gills but can spend long periods of time
out of water using dermal respiration. The graph below shows the level of blood saturation by
oxygen and oxygen supply through different organs when the eel was removed from the water
(in arbitrary units):

A, B, C, D

Match the following statements (I to IV) to the corresponding lines (A D) shown above. (1.2
points)

I.

Total blood saturation by oxygen

II.

Oxygen supply through gills

III. Oxygen supply through skin


IV. Oxygen supply from air bladder

Page 20 of 49

IBO2012 Singapore

Theoretical Test Paper 1

26. Anatomical characteristics of animals are adapted for their different modes of feeding (a d).
a.

carnivores

b.

omnivores

c.

non-ruminant herbivores

d.

ruminant herbivores

26.1. Match the different modes of feeding (a d) with the corresponding dental features (I
IV). (1.2 points)
I.

no upper incisors, have dental pad, molars allow only lateral movements

II.

canine teeth highly developed and used for tearing

III. grinding teeth patterns on posterior teeth (molars)


IV. incisors for nipping, molars slightly angled, jaws move circularly (vertical and lateral)

26.2. The gastrointestinal (GI) tract surface area to the body surface area ratio differs between
herbivores, omnivores and carnivores. Match the different modes of feeding (a d) with
the corresponding GI tract surface/body surface area ratio as listed in the table in the
Answer Sheet. (1.2 points)

Page 21 of 49

IBO2012 Singapore

Theoretical Test Paper 1

27. Match the adaptations of the digestive systems (a c) with the corresponding anatomical
descriptions. (0.9 points).
Anatomical descriptions:
I.

simple stomach, limited utilization of foliage-based diets

II.

simple stomach incapable of utilization of foliage-based diets

III.

highly developed sacculated stomach capable of extensive and effective utilization of


foliage-based diets

Digestive adaptations:
a.

extensive fermentation after primary sites of digestion and absorption

b.

extensive fermentation before primary sites of digestion and absorption

c.

unable to digest some of the substances in grains, fruits and vegetables

28. Fishes are specially adapted for aquatic life in different parts (e.g., surface, middle, bottom) of
the water column and various special habitats (e.g., sea grass beds, rock crevices). Their
swimming speeds are also partly dependent on their body morphology. Match the fishes (A H,
not drawn to scale) with their respective habitats and indicate the two fastest swimmers and the
two slowest swimmers. (2.4 points)

Page 22 of 49

IBO2012 Singapore

Theoretical Test Paper 1

29. In all classes of vertebrates, there are at least a few species that may occasionally take to the
air (e.g., flying fish, frog, lizard, and squirrel). They are not capable of true flight but make use
of non-flapping locomotion such as gliding and parachuting to slow their descent.

29.1. Animals that glide minimize drag (D) and use lift (L) to produce a more favourable lift-todrag ratio (L/D ratio). In contrast, animals that parachute maximize D as they often have
no significant surface area to produce enough L. When an animal has a steady glide,
several forces act upon it (see figure below). Resistance (R) of the outstretched body
against the airstream produces L. Drag (D) in the direction opposite to travel is also
present, and weight (mg) acts as well. The descending path makes an angle () with the
ground.

Match the correct animal, (a or b), with the expected values of L/D and in the table in
the Answer Sheet. (1 point)

29.2. In flying lizards, the patagium is a fold of skin connecting the forelimbs and hind limbs.
Russell and Dijkstra (2001) compared the patagia and accessory aerodynamic surfaces
between two species of lizards, Draco volans (flying dragon) and Ptychozoon kuhli
(flying gecko).

Page 23 of 49

IBO2012 Singapore

Theoretical Test Paper 1

------ Draco volans;

Ptychozoon kuhli

Indicate correct conclusion(s) that can be drawn from the study about the morphological
adaptations of the lizards for aerial locomotion with a tick (), and incorrect conclusion(s)
with a cross (). (2 points)
a. Although the mean weight of D. volans is smaller than that of P. kuhli, their body area
per unit mass is very similar.
b. Comparison of the patagial area to mass indicates that the patagia of P. kuhli are
larger than those of D. volans of equivalent mass.
c. The accessory structures contribute more significantly to total available aerodynamic
area in D. volans than they do in P. kuhli.
d. The total body area of the two taxa is very similar.
e. The proportional area that is contributed by the patagium is much larger in D. volans
than in P. kuhli, which compensates by the addition of extensive accessory flaps and
folds.

Page 24 of 49

IBO2012 Singapore

Theoretical Test Paper 1

30. In the hypophysis, several regulatory peptides are built from the propeptide, Proopiomelanocortin (POMC). POMC is cleaved proteolytically (A E) into various products.
Every polypeptide below is represented with N- terminal on the left and C- terminal on the right.
Note that each enzyme digests only matured precursor peptide.

30.1. Write down the minimum number of enzymes needed by a cell to produce -MSH from
POMC. (1 point)

30.2. Write down the minimum number of enzymes needed by a cell to produce -MSH from
POMC. (1 point)

Page 25 of 49

IBO2012 Singapore

Theoretical Test Paper 1

ETHOLOGY
31. Male fiddler crabs use their enlarged claws chelipeds (major chelipeds) for signalling (e.g.,
fighting for burrows, waving at females, etc.). A student studied male-female interactions by
using mirrors to reflect two different-sized images of the same waving male crab to females.
Mirror combinations used in the experiment were: 10x : 3x (Treatment I), 3x : 1x (Treatment II)
and 10x : 1x (Treatment III). Ten waving males were presented to 20 females in three trials for
each treatment. She recorded the percentage of females (Graph A) and time taken by each
female to approach each reflection (Graph B) for each treatment as well as whether the male
was right or left-handed (Graph C).

Larger image;

Smaller image;

Right-handed male;

Left-handed male

Page 26 of 49

IBO2012 Singapore

Theoretical Test Paper 1

Indicate correct conclusion(s) that can be drawn about the interactions between male and
female crabs with a tick (), incorrect conclusion(s) with a cross () and the statement(s) that
cannot be concluded with a dash (). (1.5 points)

a.

Female fiddler crabs generally prefer larger males.

b.

In mate-choice selection, male handedness is an important criterion.

c.

Males that wave faster generally attracted more females.

d.

The mean time taken for females to make a choice differed between Treatments II and III.

e.

An obvious difference in cheliped size of males may be necessary before females become
more decisive.

Page 27 of 49

IBO2012 Singapore

Theoretical Test Paper 1

32. Cormorants (Phalacrocorax carbo) feed on fish. They dive in the water and chase fish by sight,
so water clarity is important. Normally cormorants fish individually, but if the water is murky
they may develop a cooperative hunting method in a group. (1.2 points).

Indicate the process(es) that play(s) a role in developing the collaborative hunting strategy of
the cormorants with a tick () and use a cross () for inappropriate process(es).

a. competition
b. conditioning
c. habituation
d. social learning
e. imprinting
f.

trial and error

Page 28 of 49

IBO2012 Singapore

Theoretical Test Paper 1

33. The figure below from Dittman et al. (1999) shows the behavioural response of mature
hatchery-reared Coho salmon, Oncorhynchus kisutch to an artificial odorant, -phenylethyl
alcohol (PEA) placed in one arm of a two-arm arena. These salmon were exposed to PEA at
different specific developmental stages: alevin (Stage I), parr (Stage II) and smolt (Stage III) in
the hatchery before maturity and experimentation. Control fish had never been exposed to PEA.

PEA absent;

PEA present

Indicate correct conclusion(s) with a tick (), incorrect conclusion(s) with a cross () and the
statement(s) that cannot be concluded with a dash (). (1.2 points)

a.

Salmon that had previous exposure to PEA had equal preference for both arms of the
arena.

b.

Age of salmon is the most important criterion for navigation to natal stream.

c.

Chemical cues play a secondary role in salmon homing behaviour.

d.

There is a critical period for olfactory imprinting in the Coho salmon.

Page 29 of 49

IBO2012 Singapore

Theoretical Test Paper 1

GENETICS AND EVOLUTION


34. Bateson and Punnett (1908) studied the flower colour and pollen grain shape in the sweet pea
(Lathyrus odoratus, which is related to the garden pea, Pisum sativum, which Mendel studied).
They crossed a true-breeding purple-flowered plant that had long pollen grains with a truebreeding red-flowered plant that had round pollen grains, and tabulated the following results for
the F2 progeny:
Phenotype

Observed

Purple flowers, long pollen grains

296

Purple flowers, round pollen grains

19

Red flowers, long pollen grains

27

Red flowers, round pollen grains

85

Total number of progenies

427

34.1. If the genetic traits are assorted independently, what phenotype ratio would you expect
to see? Fill in the expected values for the respective phenotype and test for independent
assortment by calculating the 2 value. (4 points)

df

3.841

5.991

7.815

9.488

11.070
Table:

2 values for (p value) = 0.05

34.2. Indicate the likely explanation with a tick () and inappropriate explanations with a cross
() for the above observation. (0.8 points)
Page 30 of 49

IBO2012 Singapore

Theoretical Test Paper 1

35. The DNA sequence of the Atlantic salmon (Salmo salar) genome, which contains 28 pairs of
autosomal chromosomes and a pair of sex chromosomes (XY), has been recently completed.
DNA microinjection technique was used to successfully transfer a growth hormone transgene
construct into the zygotic stage of salmon fish embryos. Subsequently, 4 transgenic individuals
(F0 founders), 2 males and 2 females, were obtained. The growth hormone transgene is under
the regulation of a liver-specific enhancer and all 4 transgenic founders have high plasma
growth hormone levels leading to accelerated growth. It was confirmed that the transgene is
inserted as a single copy within their genomes. Stable lines of transgenic salmon with
accelerated growth will be established through crossing. For the establishment of the F1
generation, both the male and female transgenic founders (F0) are outcrossed to the respective
gender of wild-type (non-transgenic) salmon

Page 31 of 49

IBO2012 Singapore

Theoretical Test Paper 1

35.1. For the establishment of the F2 generation you have been asked to carry out a sibling-pair
cross in order to recover homozygous transgenic individuals carrying the growth hormone
transgene. What would be the expected genotype ratio expressed as a %? (1.5 points)

35.2. When you check the ratio of males versus females of the F2 generation you found that
there are always more females (70%) than males (30%) regardless of whether the growth
hormone transgene is present as null, heterozygous or homozygous within the individuals
of the F2 generation. Indicate the appropriate reason(s) with a tick () and the
inappropriate one(s) with a cross () from the list below. (0.8 points)

a.

Epigenetic silencing in some male individuals has shut down the growth hormone
transgene.

b.

Integration of the growth hormone transgene is no longer stable.

c.

Besides the XY sex chromosomes, environmental factors might have a secondary


role in sex determination.

d.

The growth hormone transgene has translocated onto the sex chromosomes leading
to sex reversal of some males.

Page 32 of 49

IBO2012 Singapore

Theoretical Test Paper 1

36. Chicken with short wings and legs are called creepers. When creepers are mated with normal
birds they produce creepers and normal chickens with equal frequency. When creepers are
mated with creepers they produce two creepers to one normal. Crossing between normal birds
produce only normal progeny.

36.1. What is the simplest genetic basis for creepers and normal chicken? Indicate the correct
answer(s) with a tick () and incorrect answer(s) with a cross (). (0.6 points)

36.2. Indicate the correct phenotype of chickens carrying two creeper alleles with a tick () and
incorrect phenotypes with a cross (). (0.5 points)

37. The black hair of guinea pigs is produced by a dominant gene B and white by its recessive
allele b. Assume that II1 and II4 do not carry the recessive allele.

37.1. What is the probability of II3 being heterozygous? (1 point)

37.2. What is the probability that one particular offspring of III1 x III2 will have white hair? (1
point)

Page 33 of 49

IBO2012 Singapore

Theoretical Test Paper 1

38. Some allele combinations can result in a particular mental disorder in humans. The Table
shows the enzyme activities of different genotypes (reported as percentage of the normal
activity).

Allele 1
Allele 2
R231X

P292L

R407W

R231X

<1

P292L

<1

<1

R407W

<1

<1

<1

IVS-12

<1

<1

<1

E290K

IVS-12

E290K

R158Q

R271Q

<1
~2

<3

R158Q

~6.5

10

R271Q

~20

30

40

Y424C

Y424C

50

All individuals homozygous or heterozygous for any combination of the first 5 alleles listed
above exhibit the classical symptoms of the disease. Individuals heterozygous between Y424C
and any of the first four alleles however have mild symptoms. R158Q/R158Q homozygous
show classical symptoms of the disease, while R271Q/R271Q homozygous and R271Q/Y424C
heterozygous have mild symptoms.

Page 34 of 49

IBO2012 Singapore

Theoretical Test Paper 1

38.1. What is the enzyme activity in individuals for genotype combinations marked by X
(R271Q/E290K) and Y (Y424C/ R158Q)? (2 points)

38.2. What is the critical range defining those with classical symptoms from those with mild
symptoms? (1 point)

39.

In a particular crop plant, several genes govern the production of anthocyanin. In the
absence of anthocyanin, the seedling leaves show only green chlorophyll pigmentation.
In the presence of anthocyanin, the seedlings have a purple cast on top of the green
coloration. A gene locus called Colourless 1 (C1) appears to function as a transacting
inducer locus for at least two other loci (ChsA and ChsJ) that encode two enzymes in the
pathway for anthocyanin synthesis (see figure below). The pathway for anthocyanin is
blocked in homozygous recessive genotypes as shown for chsA in the figure. Assume
the following conditions exist: (i) independent assortment applies to all three loci; and (ii)
homozygosity for the recessive alleles at any of the three loci causes green seedlings.

Indicate the expected phenotypic progeny ratios for the two crosses listed in the table in
the Answer Sheet with a tick () and inappropriate ones with a cross (). (2.0 points)

Page 35 of 49

IBO2012 Singapore

Theoretical Test Paper 1

40. The marsupial moles (order Notoryctemorphia) inhabit the sandy desert regions of South
Australia, Western Australia and the Northern territory. They tunnel through the sand, filling in
the tunnel behind them and giving the appearance of swimming through the sand. The eyes
of the marsupial mole are completely covered by skin, measure about 1 mm in diameter and
lack a lens or pupil. The optic nerve is greatly reduced. Indicate the correct statement(s) (a c)
with a tick () and incorrect statement(s) with a cross (). (0.6 points)

a.

The lack of a lens is homologous to the lack of ommatidia in cave flies.

b.

The greatly reduced optic nerve is a vestigial (rudimentary) structure.

c.

The eyes of the marsupial mole are analogous to the eyes of kangaroos.

Page 36 of 49

IBO2012 Singapore

Theoretical Test Paper 1

41. Interphotoreceptor retinoid binding protein (IRBP) is a single-copy gene, the product of which
plays a role in the regeneration of rhodopsin in the visual cycle in mammals. This gene was
sequenced in several marsupials and the resulting sequences were aligned for comparison. A
portion of the sequence of the coding strand of IRBP is shown below. Note that this is not the
beginning of the gene and that the correct reading frame has been indicated.

41.1. Starting with the codon involving the frameshift mutation, write down three consecutive
amino acids coded for by this gene for Vombatus and Notoryctes. Use the genetic code
table provided in Question 1. (1.8 points)

41.2. Indicate true statement(s) with a tick (), false statement(s) with a cross () and
inconclusive statement(s) that cannot be concluded with a dash (-). (0.9 point)

Compared with that of Vombatus, the protein product of the IRBP gene in Notoryctes will:
a.

contain multiple amino acid substitutions.

b.

not begin to be translated as it lacks a START codon.

c.

be truncated as it contains a STOP codon at an earlier point.

Page 37 of 49

IBO2012 Singapore

Theoretical Test Paper 1

ECOLOGY
42. The zonation patterns of littorinid snails on the rocky shores in Singapore were studied at a
vertical cliff and a sloping rock. Two taxa of littorinids were found on the vertical cliff (see A)
and an additional third species was found on the sloping rock (see B). The snail distribution
was recorded in July, September and December 2002 when the mean temperatures of the rock
surfaces were 42 C, 34 C and 27 C respectively.

Kite diagram showing the distributions of littorinid species on (A) a vertical cliff: (a) July, (b)
September, (c) December 2002; (B) a sloping rock.
EM: Echinolittorina malaccana; EV: E. vidua; LL: Littoraria sp.;

Mean High Water Spring

(MHWS) tide level.

Page 38 of 49

IBO2012 Singapore

Theoretical Test Paper 1

42.1. Indicate correct conclusion(s) about the distribution patterns of the littorinids with a tick
(), incorrect conclusion(s) with a cross () and inconclusive statement(s) that cannot be
concluded with a dash (). (2 points)
a.

Sampling period has no influence on distribution pattern of the two littorinid taxa at
the vertical cliff.

b.

Echinolittorina vidua and Littoraria sp. have similar zones of distribution.

c.

The upper limits of the Echinolittorina malaccana zone at the vertical cliff were
constant regardless of sampling period.

d.

The preferred zone of occupation of Echinolittorina malaccana is smaller than that of


Littoraria sp. at the sloping rock.

e.

Echinolittorina vidua is less heat-tolerant than its sympatric species, E. malaccana.

42.2. Snails were collected from the field and extracts of six tissue samples of each of the three
littorinid taxa were incubated separately at different temperatures. Glutamate
oxaloacetate transaminase (GOT) activity was determined and the results of the enzyme
assays are presented in the graph below. Indicate the correct taxon (EM, EV or LL) that
corresponds to the graph shown in the table in the Answer Sheet. (1 point)

Page 39 of 49

IBO2012 Singapore

Theoretical Test Paper 1

43. The ecology of a group of dung beetles was studied in West Africa by Krell-Westerwalbesloh et
al. (2004). The beetle community was divided into four categories: (i) rollers, (ii) tunnellers, (iii)
dwellers, and (iv) obligatory kleptoparasites. The rollers rapidly form balls from the faeces (in <
1 hour), roll them away from the food source, and deposit them in or on the soil to ensure
exclusive use of the dung. The tunnellers make nests directly under the food source and
transport dung into the nest where they form dung balls ( a few hours). Dwellers feed and
reproduce directly in the dung pat. Kleptoparasites use faeces portions monopolised by other
groups, e.g., by penetrating dung balls made by the rollers or the dung mass in the
subterranean nests of the tunnellers. The abundance (see Table) and flight activity (see Figure)
of these beetles (pooled data of 15 samples) at six different periods of the day were recorded.

0200-0600 h

0600-1000 h

1000-1400 h

1400-1800 h

1800-2200 h

2200-0200 h

Period
N

Dwellers

51

6.46

31

1.45

0.03

Obligatory
kleptoparasites

51

6.46

536

25.01

1351

34

4.30

997

46.52

654

82.78

579

27.02

Rollers
Tunnellers

Rollers;

78

2.09

1795

27.91

172

48.45

10.87

1230

33.00

253

3.93

24

6.76

8559

68.87

1243

33.35

22

0.34

45

12.68

2514

20.23

1176

31.55

4362

67.82

114

32.11

Tunnellers;

Dwellers;

Kleptoparasites

Page 40 of 49

IBO2012 Singapore

Theoretical Test Paper 1

Indicate correct conclusion(s) that can be drawn from the study with a tick (), incorrect
conclusion(s) with a cross () and inconclusive statement(s) that cannot concluded with a dash
(). (1.8 points)
a.

There is intense competition in the dung beetle community.

b.

Rollers dominate the community of dung beetles.

c.

The four groups of dung beetles cannot co-exist as they all exploit the same resource in
similar ways.

d.

One or more of the groups will eventually be out-competed and eliminated in the
community.

e.

The results of this study support the principle of competitive exclusion and provide
evidence for resource partitioning.

f.

The realized niche of each group is similar to their respective fundamental niche.

Page 41 of 49

IBO2012 Singapore

Theoretical Test Paper 1

44. Mount St Helens in southwest Washington state (USA) erupted catastrophically on May 18,
1980. The eruption produced a landscape with low nutrient availability, intense drought and
frequent surface movements. Permanent plots were established at several sites above the
treeline around the crater to monitor recovery after the eruption. The figure below shows the
number of species and percentage cover at one of the sites from 1981 to 1998.

Indicate the correct conclusion(s) that can be drawn from the figure above with a tick () and
incorrect conclusion(s) with a cross (). (1.2 points)

a.

The eruption killed all of the vegetation above the treeline.

b.

Secondary succession occurred rapidly after the eruption.

c.

Neither space nor light are limiting resources for plants in this environment.

d.

Only a few additional species invaded after 1982.

e.

Total plant cover in this area has increased relatively slowly due to harsh conditions on the
volcanic deposits.

f.

A stable plant community of 20 species has been reached at the study site indicating the
climax stage in the succession process.

Page 42 of 49

IBO2012 Singapore

Theoretical Test Paper 1

45. The schematic figure below shows a simulation of a marine community done with the objective
to study the relationships between its populations.

Starfish Asteroidea; Mussels Lamellibranchia; Barnacles Cirripedia;


Gooseneck barnacles Cirripedia; Rockweed Phaeophyta.

Based on the above figure, indicate correct statement(s) with a tick () and incorrect
statement(s) with a cross (). (1.8 points)

a.

The community, in its natural state, includes four species of the Kingdom Animalia.

b.

All the animals of this community have three germ layers and are deuterostomates.

c.

Phyla of animals represented here are Echinodermata, Mollusca and Arthropoda.

d.

In their natural environment, starfish is a keystone species

e.

In their natural environment, mussel density is larger in the middle intertidal zone than in
the lower intertidal zone because starfish live in the lower intertidal zone.

f.

At the end of the study, the community collapses and only one population increases its
ecological niche.

g.

The competitive exclusion of the other populations by the mussels was demonstrated.

h.

Mussels occupy the fundamental niche including both the middle intertidal zone and lower
intertidal zone.

i.

Natural conditions include biotic interactions like inter-specific competition and predation.

Page 43 of 49

IBO2012 Singapore

Theoretical Test Paper 1

46. Growth rate of most intertidal organisms generally declines in an upshore direction. Lim and
Green (1991) studied a population of the Baltic clam, Macoma balthica (a common bivalve),
from two levels of the shore at Hudson Bay, Canada. Annual shell growth rings are distinct in
the two sub-populations (see figure below) and the clams from the two zones do not differ
genetically. The Baltic clam normally buries itself and is generally hidden from predators. It is
the intermediate host to trematodes; the daughter sporocysts of the parasites are found mainly
in the gonads of the clam, causing partial or total disappearance of the gonads. Shorebirds, the
definitive host of these trematodes, are present in large numbers at the upper shore during ebb
tide. The clams in Hudson Bay have been observed to make conspicuous tracks on the sand
flats at ebb tide. The number of metacercariae (the next stage in the parasites life cycle)
encysted on the inner shell surface of crawling and burrowing clams from the two shore regions
were counted.

Symbols represent number of clams: , 1; , 2; x, 3. The vertical line divides clams that were
smaller and larger than their median length at their respective tidal level.

Page 44 of 49

IBO2012 Singapore

Theoretical Test Paper 1

Indicate valid conclusion(s) about the behaviour and biology of the Baltic clam with a tick ()
and invalid conclusion(s) with a cross (). (2.8 points)

a.

The growth rate of Macoma balthica in Hudson Bay conforms to the general rule observed
for most intertidal organisms that lower intertidal organisms grow faster than those at the
upper shore.

b.

A relatively higher proportion of clams was parasitized higher up the shore.

c.

Crawling behaviour of the clams could enhance the completion of the parasites life cycle.

d.

Clams that are buried in the sand generally have more metacercaria cysts regardless of
shore level.

e.

Increased exposure of the clams at the upper shore to shorebirds, the final host of the
trematodes, could probably account for the difference in parasite load between the two
sub-populations.

f.

High parasite load promotes increased somatic growth as reproductive output is reduced
due to host castration by the trematodes.

g.

Environmental factors probably played a greater role in determining clam growth rate of the
sub-populations than heredity.

Page 45 of 49

IBO2012 Singapore

Theoretical Test Paper 1

BIOSYSTEMATICS
47. Match the following characteristic features with the correct organisms. (1.2 points)
Features:
I. book lung, claws formerly made of three parts, but now reduced to only two, gizzard
II. reduced ribs, undergoes a metamorphosis during ontogenesis
III. hind extremities covered with scales, respiration organ using ventillating air bags, movable
upper jaw (maxilla) and lower jaw (mandibula)
IV. thin, tube-like excretion organs ending between mid- and hind-gut, body made of three
parts (tagmata), a pair of antennae
V. specialized epithelic muscle cells, nettle cells, radial symmetric body
VI. uses ampullae of Lorenzini to sense electric fields and temperature differences, cartilage
skeleton, spiraculum

Organisms:
a.

white shark (Carcharodon carcharias)

b.

house fly (Musca domestica)

c.

bird, common redstart (Phoenicurus phoenicurus)

d.

brain corals (Faviidae)

e.

european garden spider (Araneus diadematus)

f.

common frog (Rana temporaria)

Page 46 of 49

IBO2012 Singapore

Theoretical Test Paper 1

48. One of the known hypothesis of the origin and evolution of plastids is shown in the Figure below.

The processes that facilitated evolution are represented by the numbers (1 to 4) in the diagram
above: (1) for primary endosymbiosis, (2) for loss of primary plastids, (3) and (4) for secondary
endosymbiosis. These processes resulted in the presence or absence of certain plastids in
various taxa.

Match the taxa (a d) with the corresponding type of plastids in the Answer Sheet. (1.2 points)

Page 47 of 49

IBO2012 Singapore

Theoretical Test Paper 1

49. Cladistic systematic researchers apply comparisons among groups in order to differentiate
derivative characters and shared primitive characters. In doing this, they use an external group,
closely related with the one they are studying.
Condition: The external group is less related to any member of the studied group than the
members of the last one are between themselves.
Supposition: The primitive characters that precede the divergence of both groups are
homologies.
Taking into account these theoretical concepts, some researchers studied a group and the
information they obtained is presented in the following table.
Animals represented by double digit codes
Characters

A1

A2

A3

A4

A5

A6

a. Hair

b. Amniotic egg with extra


embryonic membranes

c. Four legs for locomotion

d. Jointed jaws

e. Vertebral spine

f.

Notocord

Note: 0: absence of character; 1: presence of character

49.1. After analyzing the information above, identify the external group. (0.2 points)

49.2. Identify the characters (a f) that are shared between the external and internal groups.
(0.2 points)

Page 48 of 49

IBO2012 Singapore

Theoretical Test Paper 1

49.3. Identify the primitive character present in all members of the internal group alone.
(0.2 points)

49.4. Identify the last point of divergence of the cladogram given in the Answer Sheet
using the most appropriate character. (0.2 points)

49.5. Fill in the cladogram which best represents the relationships between A1 to A6
using the information analyzed in the Answer Sheet. (1.8 points)

END OF PAPER

Page 49 of 49















All IBO examination questions are published under the following Creative Commons license:



CC BY-NC-SA (Attribution-NonCommercial-ShareAlike) https://creativecommons.org/licenses/by-nc-sa/4.0/
The exam papers can be used freely for educational purposes as long as IBO is credited and
new creations are licensed under identical terms. No commercial use is allowed.

IBO2012 Singapore

Theoretical Test Paper 2

Country: _____________________

Student Code: ________________

23rd INTERNATIONAL BIOLOGY OLYMPIAD


8th 15th July, 2012
SINGAPORE

THEORETICAL TEST PAPER 2


Write all answers in the ANSWER SHEET

Page 1 of 46

IBO2012 Singapore

Theoretical Test Paper 2

Dear Participants

You have a total of 3 hours (180 minutes) for answering this theory paper.

Use the Answer Sheet, which is provided separately, to answer all the questions.

The answers written in the Question Paper will NOT be evaluated.

Write your answers legibly. Note that there may be more than one correct/incorrect
answer and every cell should be filled.
For example:

NOTE: Some of the questions may be marked Skipped / Deleted. DO NOT attempt
these questions. Also, read the question completely before attempting it as some
questions may continue from one page to the next.

The maximum number of points for this paper is 91.8.

Stop answering and put down your pen IMMEDIATELY when the bell rings.

Your Answer Sheets as well as the Theoretical Test question paper will be collected at
the end of the test period.

Good Luck!

Page 2 of 46

IBO2012 Singapore

Theoretical Test Paper 2

CELL BIOLOGY
1.

Four mixtures of microorganisms were collected from different sites around a school and each
microbial mixture was inoculated into a medium that contained all essential elements (in the
form of ionic compounds) except carbon. The medium was at first clear (i.e., not turbid), and
this was left to be cultured with agitation in the dark for 24 h (Stage I). The culture was
subsequently continued in bright light for 24 h (Stage II) and then a further 24 h in the dark
(Stage III). The turbidity of the four samples was monitored at the end of each stage and the
following results were obtained.
End of Stage
Sample
I

II

III

Clear

Clear

Clear

Clear

Slightly turbid

Slightly turbid

Slightly turbid

More turbid

Very turbid

Slightly turbid

Slightly turbid

Slightly turbid

Which of the following organisms (a-d) are likely to be present in samples 1 to 4? Use a tick ()
to indicate presence and a cross () to indicate absence in the Answer sheet. (3.2 points)
a.

photoautotrophic microorganisms

b.

chemo-organotrophic microorganisms

c.

microorganisms that carry cellular storage granules such as inclusion bodies

d.

microorganisms that carry thylakoid membranes in their cells

Page 3 of 46

IBO2012 Singapore

2.

Theoretical Test Paper 2

A rod-shaped bacterial cell is observed to have numerous pili and fimbriae growing all over its
surface. These structures appear to be able to lengthen and shorten. Indicate the correct
function(s) (a d) of these structures with a tick () and incorrect answer(s) with a cross (). (1
point)

3.

a.

recombination

b.

attachment to surfaces

c.

active motility in solution

d.

for defence

Two bacterial cells were cross sectioned, and Bacterium A showed a single membrane
covering its cell, while Bacterium B is covered by two membranes which are separated by a
narrow space containing peptidoglycan material. Identify which bacterium is Gram positive and
Gram negative respectively in the Answer sheet. (0.4 points)

Page 4 of 46

IBO2012 Singapore

4.

Theoretical Test Paper 2

Research was conducted to examine the presence of regulator element in the upstream of
transcription start site from eukaryotic gene. As a preliminary study, a researcher performed in
silico analysis by multiple alignment of nucleotide -37 to -26 from 900 different genes. The
resulting homology percentage data are shown in the table below.
-26

-37
(5)

(3)

21

16

91

95

67

97

52

41

16

24

Base
frequency

23

39

10

35

37

(%)

28

35

12

40

38

30

28

10

83

100

33

36

10

11

4.1. Based on the given data, predict the most likely nucleotide sequence -35 to -29 within the
conserved area which is essential for its regulator function. In the Answer Sheet, fill the
boxes with A, C, T, and G, at the appropriate positions. (1.4 points)

4.2. Deletion of nucleotides -50 to -26 of several genes resulted in dramatically decreased RNA
polymerase binding within the gene. Which type(s) of sequence element may be
represented by nucleotides -50 to -26? Indicate appropriate answer(s) with a tick () and
inappropriate answer(s) with a cross () in the Answer Sheet. (1.0 point)

Page 5 of 46

IBO2012 Singapore

5.

Theoretical Test Paper 2

Hormones regulate physiological processes in various specialised cells. Match the hormones
listed below (a e) with the physiological processes (I VII) that they regulate. Note: some
processes can be regulated by more than one corresponding hormone. (2.8 points)
Hormone

6.

Physiological process

a.

insulin

I.

gluconeogenesis

b.

cortisol

II.

glycogenesis

c.

glucagon

III.

glycogenolysis

d.

thyroid hormone

IV.

lipolysis

e.

epinephrine

V.

lipogenesis

VI.

protein catabolism

VII.

protein anabolism

Which of the following allow(s) cell membranes to remain fluid under cold temperatures?
Indicate correct answer(s) with a tick () and incorrect answer(s) with a cross (). (0.8 point)
a.

by using active transport

b.

by co-transport of glucose and proton

c.

by increasing the percentage of unsaturated phospholipids in the membrane

d.

by decreasing the number of hydrophobic proteins in the membrane

Page 6 of 46

IBO2012 Singapore

7.

Theoretical Test Paper 2

Cellular abnormality can often lead to manifestation of disease or disorders in our body. Match
the following cellular abnormalities (I V) each with their most likely disorder (a e). (1.5
points)

Cellular abnormalities:
I.

altered cellular receptor

II.

uncontrollable cell division

III.

abnormal membrane transport protein

IV.

enzyme absence

V.

absence of structural protein

Disorders:
a.

A child has chronic respiratory infections; secretions of the gut and lung are thick in this
child, and his sweat is altered, with high Na+ and Cl- levels.

b.

A young man has pain in his chest and biopsy test demonstrates presence of abnormal,
unspecialized, metastatic cells.

c.

A phenotypically normal boy is diagnosed as having androgen insensitivity; his


chromosomes are 46, XY.

d.

A young boy has progressive weakness and muscle wasting atrophy of calf muscles.

e.

Affected children gradually lose skills and sight; have massive accumulation of lipid in brain
cells. Death occurs at young age. There is no treatment.

Page 7 of 46

IBO2012 Singapore

8.

Theoretical Test Paper 2

A replicating cell population was stained with a dye that became fluorescent when bound to
DNA. The DNA content of its individual cells was then determined by fluorescence-activated
cell sorting (FACS) which is shown in the graph below.

a. From the graph, which group of cells (A C) are in the S phase of the cell cycle? (0.9 point)
b. Which group of these cells (A C) are in the LONGEST phase of the cell cycle? (0.9 point)
Indicate appropriate answer(s) with a tick () and inappropriate answer(s) with a cross ().

Page 8 of 46

IBO2012 Singapore

9.

Theoretical Test Paper 2

Phospholipids are a class of lipids that are a major component of all cell membranes as they
can form lipid bilayers.
9.1. Indicate true statement(s) with a tick () and false statement(s) with a cross (). (0.8 point)
a.

The hydrophobic tails are oriented towards the interior of the cell membrane.

b.

The fatty acids present in the membrane do not have double bonds.

c.

Once phospholipids are incorporated they remain in the cell membrane permanently.

d.

The bilayers are randomly interspersed with proteins.

9.2. Mammalian plasma membranes are characterized by the presence of different types of
phospholipids (SM, PC, PE, PS and PI). The graph below shows the percentage
distribution of each phospholipid across the plasma membrane of human erythrocytes.

Page 9 of 46

IBO2012 Singapore

Theoretical Test Paper 2

Indicate the correct statement(s) with a tick () and incorrect statement(s) with a cross ().
The numbers indicated are approximate figures. (0.8 point)

a.

Membranes, in general, can be concluded to be asymmetric.

b.

24% of the total membrane phospholipids contain SM and 4% contain PI.

c.

80% of the inner total membrane phospholipids contain PE and 16% contain PC.

d.

Most PC is confined to the outer surface of the erythrocytes while most of the PE and
PS are confined to the inner surface of the erythrocytes.

10. In a study of a rice plant, it is found that gibberellins (GA) play an important role in the growth of
seedlings. The GA repressor protein (GARP) controls the expression of GA induced genes.
This protein contains two domains: (i) regulatory domain (DELLA) and (ii) the repressor domain
(GRAS). GA bound to its receptor, attaches to DELLA domain and facilitates association of
repressor domain with SCF ubiquitin ligase complex. It results in GARP getting targeted and
degraded by proteasome. This leads to GA induced gene expression and seedling growth.
Based on this information, indicate growth of the seedlings with a loss of function mutation in the
DELLA domain or the GRAS domain with a tick () under the presence or absence of
gibberellins, and use a cross () if no growth is predicted in the Answer Sheet. (1.2 points)

Page 10 of 46

IBO2012 Singapore

Theoretical Test Paper 2

11. The following enzymes(1 6) catalyse the formation of the chemical bonds, I to VI.
Enzyme

Chemical bond

1. DNA ligase

I.

Carbon-oxygen bond

2. magnesium chelatase

II.

Carbon-sulfur bond

3. acetate-CoA synthase

III.

Carbon-nitrogen bond

4. amino acid-tRNA
synthase

IV.

Carbon-carbon bond

5. pyruvate carboxylase

V.

Phosphoric ester bond

6. glutathione synthase

VI.

Nitrogen-metal bond

Reactions that the enzymes catalyse are listed below:

Match the enzyme and corresponding enzymatic reactions with the respective bond types. (2.4
points)

Page 11 of 46

IBO2012 Singapore

Theoretical Test Paper 2

PLANT ANATOMY AND PHYSIOLOGY


12. The cell wall in plants limits cell expansion. Growing cell walls extend faster in acidic conditions
and a group of proteins called expansins are key regulators of wall extension during growth. In
an experiment, excised cucumber hypocotyls of the same length were subjected to the
following treatments (1 4) before being attached to an extensometer and the extension of the
hypocotyl measured.

Treatment protocol
1

Treated with fusicoccin (a drug which activates H+-ATPase in the plasma


membrane) and placed into a buffer at pH 7
Treated with fusicoccin (a drug which activates H+-ATPase in the plasma
membrane), heated, and placed into a buffer at pH 4.5
Heated, then placed in a buffer at pH 4.5
Heated, then placed in a buffer pH at 4.5 with the addition of an homogenate
extracted from a region just behind the growing tip of another cucumber hypocotyl

The length of the hypocotyl can either increase or remain the same. Indicate the outcomes with
an arrow () for an increase in length and an equal sign () for the same length for the different
treatments. (1.2 points)

Page 12 of 46

IBO2012 Singapore

Theoretical Test Paper 2

13. The ABC model explains how three homeotic genes control floral organ identity:

activity of gene A alone specifies sepals

activity of both gene A and B is required for the formation of petals

activity of genes B and C results in the formation of stamens

activity of gene C alone specifies carpels

gene A and gene C mutually repress each other

Indicate what the floral parts (I IV) develop into in a mutant with a loss of activity of its B gene?
Write the resultant outcome in the Answer Sheet. (1.2 points)
a.

sepal

b.

petal

c.

stamen

d.

carpel

14. Indicate the type of cell division involved in the organ/cell formation and the ploidy of the cells.
Use I for mitosis and II for meiosis. (2 points)

Page 13 of 46

IBO2012 Singapore

Theoretical Test Paper 2

15. Delves et al. (1986) studied the influence of different organs on nodulation phenotype. To
analyze whether shoot or root factors have a regulatory role in the nodule formation, they
grafted wildtype plant (Bragg cultivar) with two plant mutants (nts382 and nts1116). The grafts
were inoculated with Bradyrhizobium japonicum strain USDA110 and harvested 9 weeks later.
Nodules were picked and counted from each plant and dry weights obtained. The results are
summarized in Table 1.

Table 1. Supernodulation control by Bragg cultivar

Indicate correct deduction(s) with a tick () and incorrect deduction(s) with a cross (). (1.8
points)
a. Genetic factors expressed in the shoot are affecting the number of nodules present in
the root.
b. Genetic factors expressed in the root are affecting total nodule mass.
c. Grafts are compensating an increased number of nodules with a lower mass per nodule.

Page 14 of 46

IBO2012 Singapore

Theoretical Test Paper 2

16. The bar chart shows the concentrations of various minerals in the nutrient solution () and in
the root cells () after 2 weeks of plant growth. Based on the graph given below, indicate
appropriate answer(s) with a tick () and inappropriate ones with a cross () in the Answer

Log [concentration]

Sheet. (2.4 points)

NO3-

K+

Mg2+

Fe3+

17. Chemical Z, produced in germinating barley (Hordeum) seeds plays a role in the -amylase
synthesis by the aleurone layer cells. To investigate the role of Z more closely, the promoter of
the -amylase gene was fused with the gene for -glucuronidase (enzyme producing blue
product with certain substrate). The transgenic plants were then tested under various
conditions for the presence of blue color in the aleurone layer cells, upon addition of the glucuronidase substrate. The results are shown in the table below:

Experimental condition

Aleurone layer cells


with blue colour

Normal seed

Present

Seed with embryo removed

Absent

Seed without embryo + Z

Present

Isolated protoplasts of aleurone layer cells

Absent

Isolated protoplasts of aleurone layer cells + Z

Present

Page 15 of 46

IBO2012 Singapore

Theoretical Test Paper 2

Indicate correct deduction(s) about Z with a tick () and incorrect deduction(s) with a cross ().
(1.0 point)
a.

It is likely to be a transcription factor for the -amylase gene in barley.

b.

It is produced in the aleurone layer.

c.

It is produced in the embryo.

d.

It is produced in the pericarp.

e.

It is likely to be ethylene.

Page 16 of 46

IBO2012 Singapore

Theoretical Test Paper 2

ANIMAL ANATOMY AND PHYSIOLOGY


18. The O2-affinity curve for human haemoglobin at the physiological blood-pH of 7.4 is represented
by (2). Under various conditions, the curve would shift towards (1) or (3). Indicate the
appropriate curve (1 or 3) under the conditions listed in the table in the Answer Sheet.
(2 points)

19. Tom ran after a snatch thief and caught him after a 80m chase. Which of the following
biochemical pathways was important in his muscles during the chase? Indicate the correct
answer with a tick () and incorrect answers with a cross (). (1 point)

a.

fatty acid oxidation

b.

glycolysis

c.

gluconeogenesis

d.

glycogenolysis

e.

proteolysis

Page 17 of 46

IBO2012 Singapore

Theoretical Test Paper 2

20. The schematic drawing below was traced from the horizontal section of a chick embryo showing
the axon outgrowth pattern of the motorneurones after an experimental manipulation. N is the
neural tube which will normally develop into the spinal cord. The segmented structures flanking
the neural tube are the somites which will contribute to the muscles and vertebrae development.
Somites are subdivided into anterior (a) and posterior (p) segments.
The control side (C) has somites in the original orientation whereas the experimental side (E)
has some somites surgically rotated. The objective of the experiment was to determine if the
outgrowth pattern of the motor axons is dependent on the orientation of the somites.

Based on the above figure, indicate correct deduction(s) with a tick () and incorrect
deduction(s) with a cross (). (2 points)

a.

The axons grow out of the neural tube regardless of the orientation of the somites.

b.

The axons preferentially grow through the anterior segment of the somite.

c.

The axons preferentially grow through the posterior segment of the somite.

d.

The segmented axon outgrowth pattern is an intrinsic property of the motor neurons.

e.

The somite segmentation pattern determines the motor axon segmentation pattern.

Page 18 of 46

IBO2012 Singapore

Theoretical Test Paper 2

21. Dorsal root ganglia are formed by neural crest cells migrating away from the neural tube during
embryo development. The crest cells forming the ganglia differentiate into sensory neurones.
In an experiment, a two-day old chick embryo had the anterior part of two somites surgically
removed (arrowed) and allowed to develop until the dorsal root ganglia (d) had formed in day 5.
In the image below, the right side of the embryo was the experimental side (E) and the control
side (C) is to the left. Note the ganglia scattered on the E side of the embryo (next to the
arrows).

Based on the image above, indicate correct deduction(s) with a tick () and incorrect
deduction(s) with a cross (). (1.6 points)

a.

The ganglia segmentation pattern is secondary to the somites segmentation.

b.

The ganglia segmentation is dependent on the presence of the anterior part of the somite.

c.

Ectopic ganglia formed possibly because of somites lacking the anterior segments
disrupted the normal crest cell migration pathway.

d.

The somites can regenerate after surgical interference.

Page 19 of 46

IBO2012 Singapore

Theoretical Test Paper 2

22. Combining the observations and deductions derived from Questions 20 and 21, it is quite
obvious that the anterior segment of the somites are conducive for the outgrowth of
motorneurones and the migration of neural crest cells. Indicate valid inference(s) with a tick ()
and invalid inference(s) with a cross (). (1.5 points)

a. The anterior segments of the somites are likely to express extracellular matrix molecules
that guide the axons and the crest cells.
b. The molecules expressed in the anterior segments of the somites are likely to be
adhesive proteins for contact inhibition.
c. The posterior segments of the somites may produce repulsive molecules that axons and
crest cells avoid.

23. A transverse section through the spinal cord is examined under high magnification of the
microscope. Indicate which figure (A or B: not drawn to scale) corresponds to grey and white
matter respectively. (0.6 points)

Page 20 of 46

IBO2012 Singapore

Theoretical Test Paper 2

24. Blood glucose concentration is regulated by homeostasis. Indicate the concentrations at which
the following responses are elicited. Use tick () and a cross () for high and low blood
glucose concentrations respectively. (1.2 points)

a.

detected by -cells in islets of Langerhans

b.

increase in insulin secretion

c.

convert glycogen to glucose

d.

speeds up rate of glucose uptake by cells from blood

e.

promotes fat synthesis

f.

stimulates formation of glucose from amino acids

25. Match the digestive systems (I III) with the corresponding animal feeding adaptation (a c).
(1.5 points)

a.

carnivore with limited post-gastric fermentation

b.

herbivore with extensive post-gastric fermentation

c.

herbivore with extensive pre-gastric fermentation

Page 21 of 46

IBO2012 Singapore

Theoretical Test Paper 2

26. The graph below shows the different responses of marine animals to salinity changes.

An extremely heavy and sustained week-long rainfall resulted in the salinity at a river mouth to
fall from 28 ppt to 8 ppt. This caused many soft-bodied intertidal organisms to die. Which group
of organism survived best? Write your answer in the Answer Sheet. (1 point)

Page 22 of 46

IBO2012 Singapore

Theoretical Test Paper 2

27. Q10 values are often used to describe the effects of temperature on the rate of many reactions
involved in biological processes. The Q10 value is the ratio of the velocity constants k1 (at t +
10 C) and k2 (at t C), [k1/k2].
27.1. Using the data on the oxygen consumption of the three organisms below, calculate the
Q10 values for temperature intervals of (i) 10 20 C, (ii) 15 25 C, and (iii) 20 30 C.
(3.6 points)

Note: A: --- X---

B: O

C:

27.2. Classify the organisms (A C) as ectotherm(s) or endotherm(s). (0.9 point)

Page 23 of 46

IBO2012 Singapore

Theoretical Test Paper 2

28. Ghost crabs (Ocypode ceratophthalmus) are common on tropical shores, with the adults being
nocturnal (staying in their burrows in the day) and the juveniles, diurnal in activity behaviour.
Adult crabs generally excavate burrows higher up the shore than juveniles. Heart beat rates of
juvenile and adult ghost crabs were measured at various temperatures.

juvenile crabs; adult crabs

28.1. Graphs (A D) below represent the trend of typical physiological response of organisms
to temperature change as measured by Q10 values. Identify the correct graph for the two
stages of crab development. (1 point)

Page 24 of 46

IBO2012 Singapore

Theoretical Test Paper 2

28.2. The heart beat patterns of the crabs at 30 C over a period of 5 s are shown in the figure
below. Match the patterns with the correct stage of crab development. (1 point)

28.3. Indicate correct statement(s) with a tick () and incorrect statement(s) with a cross ().
(1.6 points)

a.

As heart rate and metabolic rate are strongly correlated, this study provides evidence
that smaller organisms have higher basal metabolisms regardless of temperature.

b.

The higher heart rates of juvenile crabs are compensated by more heat lost via the
proportionately larger surface area to volume ratio.

c.

Adult crabs show endogenous nocturnal activity as they are not so heat tolerant,
preferring to stay in their burrows during the day.

d.

Physiological responses to thermal stress remain constant as ghost crabs mature.

Page 25 of 46

IBO2012 Singapore

Theoretical Test Paper 2

29. The types of cleavage pattern and blastulas (1 6) for some animal embryos are shown below:

29.1. Match the cleavage patterns and blastulas (1 6) with the corresponding animals.
Indicate P for protostome and D for deuterostome. (1.2 points)

29.2. Identify the main factor that influences the cleavage. (1 point)
a.

the ratio of the egg cytoplasm to nucleus

b.

the thickness of the egg membrane

c.

the amount of the yolk content

d.

the overall volume of the zygote

Page 26 of 46

IBO2012 Singapore

Theoretical Test Paper 2

ETHOLOGY
30. Vervet monkeys (Cercopithecus aethiops) warn fellow monkeys by producing unique warning
signals according to the type of predators such as eagles, leopards and snakes. Depending on
the type of signals, monkeys in the group choose the appropriate method to escape. A
newborn Vervet monkey is capable of producing all of these signals but it does not know which
signal should be used in each case. If a baby monkey produces the signal for eagles when a
sparrow is flying over, adult monkeys look up at the sky and then ignore the signal. However, if
an eagle is indeed hovering, the entire group joins in the warning. Sometimes, baby monkeys
are punished by their mothers for producing wrong signals.

Which of the following learning types are associated with the warning signal development in
baby monkeys? Indicate the correct answer(s) with a tick () and incorrect answer(s) with a
cross () in the table provided in the Answer

Sheet. (0.8 point)

a.

imprinting

b.

associative learning

c.

problem solving

d.

social learning

Page 27 of 46

IBO2012 Singapore

Theoretical Test Paper 2

31. The Asian honey-bee, Apis cerana is the primary pollinator of Portulaca grandiflora. Bees
pollination of four flower varieties of P. grandiflora: white flowers (single and double-petalled)
and pink flowers (single and double-petalled), was studied. Bees visitations to the four flower
varieties at six observation periods of half hour duration from 0930 h to 1230 h were recorded.
Pollen production per flower in the four flower varieties was determined. Three different floral
patch sizes (small, medium, and large: 20, 40, and 80 flowers respectively) placed at a distance
of 1.5 m apart from each other, were presented to the bees.

Single white,

Single pink,

Double white,

Double pink

Page 28 of 46

IBO2012 Singapore

Theoretical Test Paper 2

Indicate correct conclusion(s) about the honey bees behaviour with a tick () and incorrect
conclusion(s) with a cross (). (2 points)

a.

Apis cerana preferred the single-petalled Portulaca grandiflora flowers regardless of


observation period.

b.

Pink flowers were always preferred over white flowers as they had more pollen grains for
the bees to harvest.

c.

Flower colour provide an indirect cue to reward size.

d.

Pollinators did not respond to diminishing rewards but floral patch size influenced their
foraging patterns significantly.

e.

Larger floral patch size provided stronger advertising signals and the promise of larger
rewards.

Page 29 of 46

IBO2012 Singapore

Theoretical Test Paper 2

32. Fruit flies usually find food by following the odour of ethanol produced from fruits. The fruit also
serves as the place for male and female flies to mate and reproduce. The graph below shows
the relationship between the number of allured flies and the concentration of ethanol.

Based on the graph, indicate the correct statement(s) with a tick () and incorrect statement(s)
with a cross (). (1 point)

a.

The male/female ratio in the number of flies occupying the food source varies depending
on the ethanol concentration.

b.

The number of mating animals would be the lowest when ethanol concentration is 9.

c.

The competition between males would be most severe when ethanol concentration is 7.

d.

The number of laid eggs would be the highest when ethanol concentration is around 6
and 7.

e.

The number of attracted flies would be the highest when ethanol concentration is 8.

Page 30 of 46

IBO2012 Singapore

Theoretical Test Paper 2

GENETICS AND EVOLUTION


33. A two-step metabolic pathway in a diploid organism has the following components:
Gene 1 (G1) encodes enzyme E1 that converts substrate A to product B. Its mutant allele g1
produces a defective e1 that has 46% activity of normal E1. Similarly, gene 2 (G2) encodes the
enzyme E2 that converts the metabolic intermediate B to the product C, while its mutant allele
g2 produces a defective enzyme e2 with 36% activity of normal E2. For both enzymes, each
allele contributes 50% towards the protein pool in the cell and both reactions have the same
rate in a wild-type cell.

In the F2 progeny of a cross between G1G1g2g2 and g1g1G2G2 individuals, what fraction is
expected to show an elevated level of the metabolic intermediate B? (2 points)

34. In a particular breed of dogs, the hairless condition is produced by the heterozygous genotype.
Normal dogs are homozygous recessive. Puppies homozygous for the H allele are usually born
dead with abnormalities of the mouth and absence of external ears. If the average litter size at
weaning is 6 in matings between hairless dogs, what would be the average expected number of
hairless and normal offspring at weaning for matings between hairless dogs and between
hairless and normal dogs? (1.8 points)

Page 31 of 46

IBO2012 Singapore

Theoretical Test Paper 2

35. There are two types of red-green colour blindness deuteranopia and protanopia, governed by
two linked loci on the X chromosome. Among 18,121 Norwegian children examined in a study,
9049 were males, of which, 725 were males with colour blindness, 551 had deuteranopia and
174 had protanopia. Of the 40 females with colour blindness, 37 had deuteranopia and 3 had
protanopia.
df

3.841

5.991

7.815

9.488

11.070

Table:

2 values for = 0.05

Estimate the allele frequencies for deuteranopia (independent of protanopia) from the data on
males. Use these values to test the phenotypic distribution in females for compatibility with the
Hardy Weinberg equilibrium (HWE) using the

test.

Indicate compatibility with HWE with a tick () and incompatibility with a cross (). (4.0 points)

Page 32 of 46

IBO2012 Singapore

Theoretical Test Paper 2

36. The diagram below presents the patterns of clan membership for a kinship group of Native
Americans. The diagram shows the conditions of marriage and the way the clan of the father
determines the clan of the children. The broken lines point from the fathers clan to the clan of
his children and the solid lines point from a mans clan to the clan of a potential wife.

The pattern of clan membership between the four clans A, B, C and D.

A disease X, that is prevalent in this clan community, is a dominant sex linked trait and is
carried on the Y chromosome. Answer the following questions relevant to disease transmission
between the four clans.

36.1. Maska of clan D has disease X. After two generations both including sons, indicate the
clans that will be affected if only Maska carried the disease, with a tick () and nonaffected clans with a cross (). (2 points)

36.2. What is the probability of Tala (clan A) and Yonato (clan B) having a son with disease X in
clan D if Yonatos father had disease X? (2 points)

Page 33 of 46

IBO2012 Singapore

Theoretical Test Paper 2

37. A number of nutritional mutant strains were isolated from wild-type red bread mold Neurospora
crassa that responded to the addition of certain supplements in the culture medium by growth
(+) or no growth (0). Given in the Table below are the responses for single-gene mutants.

Supplements added to minimal culture medium


Strain
Citrulline

Glutamic
semialdehyde

Arginine

Ornithine

Glutamic acid

37.1. Indicate the sequence (1 5) of the five metabolites within the metabolic pathway in the
Answer Sheet. (1.5 points)

37.2. Indicate the strain that is blocked at each of the four steps in the metabolic pathway in
the Answer Sheet. (1.2 points)

38. Given that A1A1 = lethal, A1A2 = gray, A2A2 = black, B1B1 = long hair, B1B2 = short hair, B2B2 =
very short hair (fuzzy), and parents that are A1A2B1B2.

38.1. What is the fraction of adult offspring that is expected to be gray and fuzzy? (1 point)

38.2. In the case when fuzzy is also a lethal trait, what is the fraction of adult progeny expected
to be black and short? (1 point)

Page 34 of 46

IBO2012 Singapore

Theoretical Test Paper 2

39. You are given the following number of F1 flies.


Male

Female

Total

wild type

80

60

140

mutant

30

30

60

If the mutation was inherited via a simple autosomal recessive mode, what is the most likely
parental genotype? Indicate the correct answer(s) with a tick () and incorrect answer(s) with a
cross (). (1 point)

a.

w w X w+w+

b.

w+w X w+w

c.

w+w X w w

d.

w+w X w+w+

e.

w+ w+ X w+ w+

Page 35 of 46

IBO2012 Singapore

Theoretical Test Paper 2

40. If a paternal chromosome has alleles L, M, and n and the maternal chromosome has l, m, and
N. Which of the following chromosomes could possibly be produced as a result of a single
crossing over? Indicate the correct answer(s) with a tick () and incorrect answer(s) with a
cross (). (1 point)

I.

LMN

II.

LMn

III. LmN
IV. Lmn
V. lmn

41. The closest living relative of human (Homo sapiens) is widely considered to be the chimpanzee
(Pan troglodytes) and bonobo (Pan paniscus). Together with the orangutans (Pongo pygmaeus
and P. abelii) and gorilla (Gorilla gorilla and G. beringei) they form the subfamily Homininae.
This evolutionary relationship can be presented as shown below.

Page 36 of 46

IBO2012 Singapore

Theoretical Test Paper 2

41.1. The phylogenetic tree demonstrating the evolutionary relationship of the higher primates
is constructed using molecular data and is considered to be reliable because:

a.

DNA mutations and polymorphisms can be readily identified and analyzed.

b.

DNA sequences can be converted into protein sequences for comparative analysis.

c.

the availability of large amount of molecular data permits rigorous computational


analyses to be conducted.

d.

physically intact and unfragmented genomic DNA can be recovered after millions of
years.

Indicate true statement(s) with a tick () and false statement(s) with a cross (). (0.8 point)

41.2. Most of the oldest fossils belonging to the subfamily Homininae are found in the African
continent. This is similarly reflected by the living species, with orang utans being the only
species not found in Africa. This observation of fossil distribution supports the:

a.

"Out of Africa" hypothesis whereby the first human beings evolved in Africa and
subsequently migrated to other continents.

b.

"Out of Asia" hypothesis as the oldest living species, the orang utans, are found only
in South-East Asia.

c.

"multi-centric origin" hypothesis.

Indicate true statement(s) with a tick () and false statement(s) with a cross (). (0.6
point)

Page 37 of 46

IBO2012 Singapore

Theoretical Test Paper 2

ECOLOGY
42. Borges and Brown (1999) studied the arthropod species richness in three islands (Pico, Santa
Maria and Terceira), in the Azorean archipelago. The figure below shows the location of the
islands with respect to major land masses, Africa and Europe (in the east) and North America
(in the west).

Some other characteristics of the islands are given in the table below.
Island
Pico
Santa Maria
Terceira

Altitude (m)

Geological age (Myr)

Distance from mainland (km)

2351

0.037 0.300

1866

587

8.12

1585

1023

0.300 2

1770

Page 38 of 46

IBO2012 Singapore

Theoretical Test Paper 2

42.1. Match the following x axes with the correct graphs (I to IV). (2.4 points)
a.

Log10 (area in km2)

b.

Log10 (altitude in m)

c.

Log10 (distance from mainland in km)

d.

Log10 (geological age in years)

42.2. Estimate from the graphs, the number of endemic species (to the nearest whole number)
in the three islands. (1.2 points)

Page 39 of 46

IBO2012 Singapore

Theoretical Test Paper 2

42.3. From the data indicate correct conclusion(s) that can be drawn from the study with a tick
() and incorrect conclusion(s) with a cross (). (1.5 points)
a.

Diversity was greater on the island with the lowest maximum altitude and decreased
with increasing altitude.

b.

Results of this study support the species-area hypothesis.

c.

Isolation alone cannot be used to explain the species richness patterns found in the
arthropod assemblage in the three islands.

d.

Species richness patterns in these islands are influenced by both ecological and
evolutionary factors.

e.

MacArthur and Wilsons theory of island biogeography is fully supported by the


results of this study.

Page 40 of 46

IBO2012 Singapore

Theoretical Test Paper 2

43. Daniel boiled water and hay in a beaker for some time, and left it uncovered for some days.
During that period only heterotrophic bacteria were found in the beaker. He then added a few
drops of ditch water and covered it loosely. The water from the ditch only contained
heterotrophic unicellular organisms (no bacteria or fungi). Daniel regularly determined the size
of the populations of the different species (p u) present in the beaker over a period of time.

X = Time (days); Y = number of individuals per ml at water surface


Looking at the results, some of Daniels students claimed that in the beaker:
I.

competition is likely to occur

II.

succession is taking place

III.

total biomass is increasing in the period between Day 40 to Day 50

Some other students even went on to predict that:


IV.

the number of dividing bacteria will decrease to zero

V.

the number of the other dividing heterotrophic unicellular organism will decrease to zero

VI.

a climax stage will develop, comprising bacteria and other heterotrophic unicellular
organisms, in a stable natural equilibrium

Indicate correct statement(s) with a tick () and incorrect statement(s) with a cross (). (1.2
points)

Page 41 of 46

IBO2012 Singapore

Theoretical Test Paper 2

44. Increasing concentration of carbon dioxide (a greenhouse gas) has been linked to global
climate change. Carbon dioxide can be removed from the atmosphere and deposited in a
reservoir in order to either mitigate or defer global warming and avoid dangerous climate
change. The removal process includes carbon dioxide uptake from the atmosphere by all
chlorophyllous plants, through photosynthesis. Indicate correct statement(s) concerning the
amount of carbon stored per unit area in different ecosystems with a tick () and incorrect
statement(s) with a cross (). (1 point)

a.

net primary productivity (NPP) (or net carbon absorption rate) of coniferous forests
temperate forests tropical forests

b.

carbon stocks (the amount of carbon stored) of coniferous forests temperate forests
tropical forests

c.

net primary productivity (NPP) of temperate grasslands savannas (grasslands with


scattered trees) tundra

d.

net primary productivity (NPP) of secondary forests climax forests or old growth forests

e.

as compared to other ecosystems, coral reefs have very high net primary productivity, and
their contribution to global production of biomass is large

Page 42 of 46

IBO2012 Singapore

Theoretical Test Paper 2

45. Male fiddler crabs have an enlarged cheliped (the major cheliped) to attract mates and defend
territories. However, the major cheliped is useless for foraging; hence males are left with only
one feeding appendage the minor cheliped. Many strategies have been proposed by
researchers to explain how male fiddler crabs compensate for this apparent disadvantage. A
student conducted a study to investigate some of these strategies. She videotaped the foraging
bouts of males and females on the first five patches of sediment upon first emergence at low
tide. Sediment samples from these five patches were collected after the crabs have vacated
the patch and chlorophyll a content in these samples was determined. Various dimensions of
the feeding cheliped in males and females were also compared. The mean number of scoops
per second for the two sexes was determined from the videotapes: females, 2.39 0.08 scoops;
males, 1.60 0.06 scoops.

Females;

Males

Page 43 of 46

IBO2012 Singapore

Theoretical Test Paper 2

Indicate correct conclusion(s) that can be drawn from the students study with a tick () and
incorrect conclusion(s) with a cross (). (2.4 points)

a.

Male fiddler crabs do not compensate for the one-feeding-cheliped-handicap by feeding at


a faster rate than females.

b.

There is sexual dimorphism in the minor cheliped.

c.

In general, male fiddler crabs compensate for the one-feeding-cheliped-handicap by


staying for a longer period of time in a foraging patch.

d.

Male fiddler crabs generally leave a foraging patch at a higher threshold of chlorophyll a
content than females.

e.

Males compensate for the one-feeding-cheliped-handicap by having larger scoops of


sediment per lift of the cheliped.

f.

Results of this study do not support the principles of the optimal foraging theory.

Page 44 of 46

IBO2012 Singapore

Theoretical Test Paper 2

BIOSYSTEMATICS
46. Morphological characters (1 6) shared by two or more organisms and their recent common
ancestor are shown in the cladogram below.

46.1. Which of the morphological characters (1-6) are the synapomorphies for the
corresponding taxonomic groups indicated in the Answer Sheet. (1.2 points)

46.2. Identify the taxonomic groups (W Z) in the table provided in the Answer Sheet. (1.2
points)

Page 45 of 46

IBO2012 Singapore

Theoretical Test Paper 2

46.3. Based on your knowledge of these organisms and groups, match the morphological
characters listed below (I VI) among the labelled morphological characters from 1 to 6.
(1.2 points)
I.

body organised into head, and elongated (and segmented) trunk; numerous pairs (>
12 pairs) of legs

II. body organised into head, thorax, and abdomen; three pairs of legs (from thorax)
III. jointed/segmented appendages
IV. antennae (one or more pairs); mandibles (chewing mouthparts)
V. two pairs of antennae; biramous (two branches) appendages
VI. no antennae; no mandibles

46.4. Classify the groups listed below (I IV) according to their respective phylogenetic origins
in the Answer Sheet. (1.2 points)
I.

W and X

II.

X and Y

III. Y and Z
IV. W, X, Y, and Z

END OF PAPER

Page 46 of 46















All IBO examination questions are published under the following Creative Commons license:



CC BY-NC-SA (Attribution-NonCommercial-ShareAlike) https://creativecommons.org/licenses/by-nc-sa/4.0/
The exam papers can be used freely for educational purposes as long as IBO is credited and
new creations are licensed under identical terms. No commercial use is allowed.

IBO2012 Singapore

Paper 1
Answer Key

Theoretical Test

Paper 1

Answer Key
1.

2.

(1.8 points)
a

(1.8 points)

Cell

Functions (a d) if present

Mitochondria present

Sperm cell
Brown fat cell
Red muscle fibers
Intestine epithelia

3.

4.

(0.9 points)
Lowest Tm

Medium Tm

Highest Tm

(2 points)
Condition

II

III

IV

Cell fate

Page 1 of 15

IBO2012 Singapore

5.

Paper 1
Answer Key

(4.2 points)
5.1. (3.6 points)

Heptapeptide
Peptide A

Asp-Ala-Glu-Asp-Gly-Ser-Ser
Peptide B

Gly-Lys-Asp-Ala-Ala-Ser-Gly
Peptide C

Ser-Lys-Ser-Lys-Gly-Asp-Ala

pH 1 net
charge

pH 7 net
charge

pH 12 net
charge

+1

-3

-4

+2

-2

+3

+1

-2

5.2. (0.6 points)

6.

7.

pH 1

pH 7

pH 12

(0.5 points)
a

(0.9 points)
7.1. (0.4 points)
a

7.2. (0.5 points)


The number is ____92________.

Page 2 of 15

IBO2012 Singapore

8.

Paper 1
Answer Key

(1.8 points)
8.1. (0.6 points)
Bacterium A

Bacterium B

Bacterium C

8.2. (0.6 points)


____C____

>

____B____

>

____A____

8.3. (0.6 points)

9.

ii

iii

iv

(1 point)

Page 3 of 15

IBO2012 Singapore

Paper 1
Answer Key

10. (4.6 points)


10.1. (1.6 points)
A

12

11

17

14

10.2. (3 points)

11. (3 points)
1

10

12. (1.4 points)


a

Page 4 of 15

IBO2012 Singapore

Paper 1
Answer Key

13. (1.2 points)


a

14. (1.0 points)

15. (1.5 points)


Most primitive

Intermediate

Most modern

16. (1.8 points)


1

17. (1.5 points)

___III__

___IV___

___II___

___V____

___VII__

Page 5 of 15

IBO2012 Singapore

Paper 1
Answer Key

18. (1.6 points)


18.1. (0.8 points)

____A____

>

____C____

>

____B____

>

____D____

>

____B____

>

____C____

>

____A____

18.2. (0.8 points)

____D____

19. (1.6 point)


Animal

Amphibians

Reptiles

Birds

Mammals

II

20. (2.6 points)


Animal

Frog

Salmon

Crayfish

Lizard

Earthworm

Dragonfly

Circulatory
system

Respiratory
organ

a, c

21. (2 points)
a

Page 6 of 15

IBO2012 Singapore

Paper 1
Answer Key

22. (0.8 point)


Saliva secreted/day
(litres)
Animal

< 0.75

0.75 1.5

10 12

130 180

23. (0.8 point)


Allergy

Pseudoallergy

24. (0.6 points)


a

II

III

IV

25. (1.2 points)

Page 7 of 15

IBO2012 Singapore

Paper 1
Answer Key

26. (2.4 points)


26.1. (1.2 points)
I

II

III

IV

26.2. (1.2 points)


GI tract surface area/ body surface area ratio
0.6:1

1.2:1

2:1

3:1

27. (0.9 points)


I

II

III

28. (2.4 points)


Part of water column / Habitats

Swimming speed

Surface

Middle

Bottom

Sea grass
beds

Rock
crevices

Fast

Slow

D, H

A, C, E

D, H

A, G

Page 8 of 15

IBO2012 Singapore

Paper 1
Answer Key

29. (3 points)
29.1. (1 point)
L/D < 1

> 45

29.2. (2 points)
a

30. (2 points)
30.1. (1 point)
The minimum number of enzymes needed to produce -MSH = ______3_______.

30.2. (1 point)
The minimum number of enzymes needed to produce -MSH = ______3_______.
31. (1.5 points)
a

32. (1.2 points)

Page 9 of 15

IBO2012 Singapore

Paper 1
Answer Key

33. (1.2 points)


a

34. (4.8 points)


34.1. The expected ratio = ______ 9:3:3:1____________ (1 point)
Phenotype

Observed

Purple flowers, long pollen grains

Expected

296

Purple flowers, round pollen grains

19

Red flowers, long pollen grains

27

Red flowers, round pollen grains

85

Total number of progenies

427

(1 points)

2 value = ____________ (2 points)


34.2. (0.8 points)
Complimentary
epistasis

Dominant epitasis

Linkage

Maternal
inheritance

35. (2.3 points)


35.1. (1.5 points)
homozygous

heterozygous

wild type

25

50

25

35.2. (0.8 points)


a

Page 10 of 15

IBO2012 Singapore

Paper 1
Answer Key

36. (1.1 point)


36.1. (0.6 points)
Homozygous
dominant

Heterozygous

Homozygous
recessive

Normal

Creepers

36.2. (0.5 points)

Normal

Short wings

Short legs

Short wings
and legs

Lethal

37. (2 points)
37.1. (1 point)
The fraction expected is = _____2/3_______.

37.2. (1 point)
The fraction expected is = _____1/12______.

38. (3 points)
38.1. (2 points)
The estimated enzyme activity of X (R271Q/E290K) is _ 16.5 (any value between 15 to
17)____________.
The estimated enzyme activity of Y (Y424C/ R158Q) is __ 30 (any value between 28 to
32)_________________.
38.2. (1 point)
The critical range is somewhere between __10___ % to __25___ % of normal activity.

Page 11 of 15

IBO2012 Singapore

Paper 1
Answer Key

39. (2 points)
Progeny ratio (purple to green)
Cross
3:1

9:7

15:1

1:7

1:1

ChsA chsA ChsJ chsJ C1C1 X

i.

ChsA chsA ChsJ chsJ C1C1


ChsA chsA ChsJ chsJ C1c1 X

ii.

chsA chsA chsJ chsJ c1c1

40. (0.6 point)


a

41. (2.7 points)


41.1. (1.8 points)
Vombatus

Tyr

Asp

Arg

Notoryctes

Leu

STOP

Pro

41.2. (0.9 points)


a

42. (3 points)
42.1. (2 points)

Page 12 of 15

IBO2012 Singapore

Paper 1
Answer Key

42.2. (1 point)
Line

Taxon

EM

43. (1.8 points)


a

44. (1.2 points)

45. (1.8 points)

46. (2.8 points)

47. (1.2 points)


I

II

III

IV

VI

Page 13 of 15

IBO2012 Singapore

Paper 1
Answer Key

48. (1.2 points)


Type of plastids

Taxa

Two-membrane rhodoplast

Two-membrane chloroplast

Four-membrane rhodoplast

Three-membrane chloroplast

49. (2.6 points)


49.1. (0.2 points)
Answer: ___A1____.

49.2. (0.2 points)


Answer: ____f_____.

49.3. (0.2 points)


Answer: ____e____.

49.4. (0.2 points)


Answer: ____b_____.

Page 14 of 15

IBO2012 Singapore

Paper 1
Answer Key

49.5. (1.8 points)

A6

A2
/A
5

A2
/A
5

A3
A4
A1

END OF PAPER

Page 15 of 15

IBO2012 Singapore

Paper 2
Answer Key

Theoretical Test

Paper 2

Answer Key
1.

(3.2 points)
Types of organism
Sample

2.

3.

(1 point)
a

(0.4 points)

Bacterium __________ is likely to be Gram negative.


Bacterium __________ is likely to be Gram positive.

Page 1 of 13

IBO2012 Singapore

4.

Paper 2
Answer Key

(2.4 points)
4.1. (1.45 points)

-35

-34

-33

-32

-31

-30

-29

A/T

A/T

4.2. (1 point)
Sequence element

5.

6.

7.

Operator

Promoter

Origin of
replication
(ORI)

Telomere

Enhancer

(2.8 points, 0.4 point per cell)


I

II

III

IV

VI

VII

b, c

c, e

b, d, e, f

a, d, f

(0.8 point)
a

II

III

IV

(1.5 points)

Page 2 of 13

IBO2012 Singapore

8.

Paper 2
Answer Key

(1.8 points)
Group
Cell cycle

9.

a. S phase

b. Longest phase

(1.6 points)
9.1. (0.8 points)

9.2.

(0.8 points)

10. (1.2 points)


Gibberellin

DELLA mutant

GRAS mutant

Present

Absent

11. (6 x 0.4= 2.4 points) (0.4pts per column)


I

II

III

IV

VI

Ligase

Reaction

Page 3 of 13

IBO2012 Singapore

Paper 2
Answer Key

12. (1.2 points)


1

13. (1.2 points)

II

III

IV

14. (2 points) (10 x 0.2, mark by row)


Organ / Cell

Type of cell division

Ploidy of cells

Endosperm of
angiosperm

3n

Pollen grain

II

Central cells

n+n

Egg of angiosperm

II

Spore of moss

II

Protonema

Sperm of moss

Fern gametophyte

Spore of fern

II

Egg of fern

15. (1.8 points)


Bragg

nts382

nts1116

Page 4 of 13

IBO2012 Singapore

Paper 2
Answer Key

16. (8 x 0.3 = 2.4 points)

Required as trace
element

Absorbed by passive
transport

2+

3+

Nutrient
-

NO3
K

Mg
Fe

17. (1.0 point)


a

18. (2 points)
Condition

Curve

In actively working muscles

In the lung

In human fetus

With increased temperature

With increased CO2 content

19. (1 point- all or none)


a

Page 5 of 13

IBO2012 Singapore

Paper 2
Answer Key

20. (2 points)
a

21. (1.6 points)

22. (1.5 points)

23. (0.6 point)


Grey matter

White matter

24. (1.2 points)


a

25. (1.5 points)


I

II

III

26. (1 point)
Answer: _____C_____

Page 6 of 13

IBO2012 Singapore

Paper 2
Answer Key

27. (4.5 points)


27.1. (3.6 points)
Q10 value

(i)

(ii)

(iii)

2.3

2.4

2.5

3.0

2.9

3.3

5.0

3.3

6.0

27.2. (0.9 points)


Ectotherm

Endotherm

A, B

Juvenile

Adult

28. (3.6 points)


28.1. (1 point)

28.2. (1 point all or none)


Juvenile

Adult

28.3. (1.6 points)


a

Page 7 of 13

IBO2012 Singapore

Paper 2
Answer Key

29. (2.2 points)


29.1. (1.2 points)
Animal

Cleavage pattern and blastulas

Type of coelom formation

Mouse

Snail

Toad

Chicken

Seastar

Fruit fly

29.2. (1.0 point)


The main factor is ______C_______.

30. (0.8 points)


a

31. (2 points)
a

Page 8 of 13

IBO2012 Singapore

Paper 2
Answer Key

32. (1 point)
a

33. (2 points)( allow if given in decimal points or in lowest form or in percentage)


33.1. (1 point)
Answer: _____3/16_______.

33.2. (1 point)
Answer: _____4/16_______.

34. (1.8 points)


Lethal

Hairless

Normal

Hairless (Hh) X Hairless (Hh)

Hairless (Hh) X normal (hh)

35. (1 + 2 + 1 points = 4 points)


Frequency of disease causing allele (%)

6.09

0.338

(3 decimal places)

Hardy-Weinberg Equilibrium

36. (4 points)
36.1. (2 points)
A

Page 9 of 13

IBO2012 Singapore

Paper 2
Answer Key

36.2. (2 points)
The probability is _____0_____.

37. (2.7 points)


37.1. (1.5 points)
Metabolites

Order in the pathway

Citrulline

Glutamic semialdehyde

Arginine

Ornithine

Glutamic acid

37.2. (1.2 points)


Step in metabolic pathway
Strain (carrying defect in this step)

12

2 3

34

45

38. (2 points) ( allow if given in decimal points or in lowest form or in percentage)


38.1. (1 point)
Answer: _____1/6______.

38.2. (1 point)
Answer: _____2/9______.

39. (1 point) (all or none)


a

Page 10 of 13

IBO2012 Singapore

Paper 2
Answer Key

40. (1 point)
I

II

III

IV

41. (1.4 points)


41.1. (0.8 point)
a

II

III

IV

41.2. (0.6 point)

41.3. (1 point)

42. (5.1 points)


42.1. (2.4 points)

42.2. (1.2 points)


Island
Number of endemic species

Pico

Santa Maria

Terceira

48 - 52

100 - 104

63 - 68

Page 11 of 13

IBO2012 Singapore

Paper 2
Answer Key

42.3. (1.5 points)


a

43. (1.2 points)


I

II

III

IV

VI

44. (1 point)

45. (2.4 points)

46. (4.8 points)


46.1. (0.4 x 3 =1.2 points)
Taxonomic
group

X+Y

W+Z

Morphological
character(s)

46.2. (1.2 points)


Chelicerata

Crustacea

Hexapoda

Myriapoda

Page 12 of 13

IBO2012 Singapore

Paper 2
Answer Key

46.3. (1.2 points)


1

III

IV

VII

VI

II

46.4. (1.2 points)


Monophyletic

Paraphyletic

I, IV

II, III

Polyphyletic

END OF PAPER

Page 13 of 13

INTERNATIONAL BIOLOGY OLYMPIAD


PRACTICAL PROBLEMS

2012, Singapore















All IBO examination questions are published under the following Creative Commons license:



CC BY-NC-SA (Attribution-NonCommercial-ShareAlike) https://creativecommons.org/licenses/by-nc-sa/4.0/
The exam papers can be used freely for educational purposes as long as IBO is credited and
new creations are licensed under identical terms. No commercial use is allowed.

IBO2012
SINGAPORE
CELL & MOLECULAR BIOLOGY

PRACTICAL TEST 1

Country: _____________________

Student Code: ________________

23rd INTERNATIONAL BIOLOGY OLYMPIAD


8th 15th July, 2012
SINGAPORE

PRACTICAL TEST 1
CELL & MOLECULAR BIOLOGY
Total points: 100

Duration: 90 minutes

Page 1 of 9

IBO2012
SINGAPORE
CELL & MOLECULAR BIOLOGY

PRACTICAL TEST 1

Dear Participants

In this test, you have been given the following task:


Task: Gene mapping by restriction endonuclease digestion of DNA fragments
Part A. Confirmation of insertion of human DNA in a cloning plasmid. (80 points)
Part B. Determination of orientation by which the fragment is inserted. (20 points)

Use the Answer Sheet, which is provided separately, to answer all the questions.

The answers written in the Question Paper will NOT be evaluated.

Write your answers legibly in ink.

Please make sure that you have received all the materials and equipment listed for each task.
If any of these items are missing, please raise your hand immediately.

Stop answering and put down your pen IMMEDIATELY when the bell rings.

At the end of the test, place the Answer Sheet and Question paper in the envelope provided.
Our Assistants will collect the envelope from you.

Have fun and Good Luck!

Page 2 of 9

IBO2012
SINGAPORE
CELL & MOLECULAR BIOLOGY

PRACTICAL TEST 1

Materials and equipment:

Materials and equipment

Quantity

Unit

restriction endonuclease RE1 (Ndel) (kept on ice)

4 l

tube

restriction endonuclease RE2 (EcoRl) (kept on ice)

4 l

tube

10 l x 4

tube

miliQ water (labelled W)

tube

DNA electrophoresis gel tank and power supply

set

micropipettes and tips in boxes (p10, p100)

piece

stopwatch

piece

DNA ladder (as internal size markers, L1 for 100 bp range and
L2 for 1 kbp range) (on ice)

tube

DNA loading dye (blue in colour)

tube

pre-cast gel in holder (already placed in running buffer)

piece

large petri dish (for placing the gel for imaging purposes)

piece

card with your country code (in a clip holder): for signalling for
assistance

piece

floating rack (labelled with your country code)

piece

micro-centrifuge

set

water-bath 37 C (there is one assigned for your usage)

set

gel doc (there is one assigned for your usage)

set

DNA test samples in enzyme buffer (labelled T) (on ice)

Page 3 of 9

IBO2012
SINGAPORE
CELL & MOLECULAR BIOLOGY

PRACTICAL TEST 1

Task (100 points)


Gene mapping by restriction endonuclease digestion of DNA fragments
Introduction
Genetic mapping is routinely used in analysing the order and the identities of DNA fragments. This
technique is based on the unique profiles of DNA fragments generated after DNA digests with
specific combination of restriction endonucleases (RE) and revealed by DNA gel electrophoresis. It
is extremely powerful for gene cloning, studying gene function and regulation, for finding candidate
genes for diseases and their diagnosis and also as a forensic tool.

Part A. Confirmation of insertion of human DNA in a cloning plasmid. (80 points)


Using this technique, you are now tasked to confirm that a fragment of human DNA X
(approximate size: 760 base pairs) has been inserted into a cloning plasmid or vector V (circular
and approximate size: 2570 base pairs). You are required to design and carry out DNA digests by
incubating DNA T with the restriction endonucleases by following the general protocol of
incubation and electrophoresis given (details described below). After the gel electrophoresis, your
results will be revealed by DNA staining (this will be performed by lab technicians), analysed and
data interpreted.

Protocol and Procedures


1.

Design your DNA digests (you may do a maximum of 4 tubes) in a total volume of 20 l by
using the Table in the Answer Sheet.
Q1.1

(20 points) Record the desired amounts of reagents in your plan. One example is
already given for Tube 2 in the table provided. All units are in l.

Page 4 of 9

IBO2012
SINGAPORE
CELL & MOLECULAR BIOLOGY

2.

PRACTICAL TEST 1

Prepare the mixtures by carefully pipetting the correct amount of the reagents and gently mix
them by pipetting them up and down in each tube. Label the tubes. Do not contaminate one
sample with another when preparing the mixture. Use a clean pipette tip for each operation.
Note: use p10 micropipette (white-coded) for pipetting reagents of less than 10 l. [NOTE:
there will be a penalty of 20 points if additional samples are requested. Please prepare the
samples carefully.]

3.

Spin down the mixture by placing all four tubes in the micro-centrifuge (please balance the spin
by placing tubes opposite to each other). During preparation and after spinning, always keep
the tubes on ice.

4.

After all the tubes have been prepared, remove them from the ice and place them into the
labelled floatation rack and incubate them for 20 minutes (stopwatch is provided) at 37 C in the
water bath assigned to you. Make sure that you retrieve your own samples after the 20 minutes
incubation time.

5.

During this 20 minute incubation duration, answer the following questions in the Answer Sheet:
Q1.2

(2 points 5 = 10 points) Indicate true statement(s) with a tick () and false


statement(s) with a cross ().
a.

Each RE cuts DNA at a specific sequence.

b.

Each RE cuts DNA only at the 3 and 5ends.

c.

RE are most effective in digesting DNA at 4 C.

d.

RE can be kept at room temperature for months.

e.

Unlike exonucleases, RE only cuts DNA internally.

Page 5 of 9

IBO2012
SINGAPORE
CELL & MOLECULAR BIOLOGY

Q1.3

PRACTICAL TEST 1

(2 points 5 = 10 points) Which of the following principles is true of separating DNA


by gel electrophoresis? Indicate true statement(s) with a tick () and false statement(s)
with a cross ().
a.

DNA fragments are overall positively charged.

b.

The smaller DNA fragments move faster across the gel under the electric current.

c.

The smaller DNA fragments are lesser charged than the larger fragments hence
they move faster across the gel.

d.

The relative density of the gel matrix affects how long the separation takes.

e.

The voltage applied to the electrophoresis is determined by how much DNA is


loaded in the gel.

6.

When the 20 minutes of DNA digests duration is up, retrieve your own tubes from the water
bath.

7.

Add 4 l of DNA loading-dye (blue colour). Mix them well by pipetting the mixture up and down
and spin down any residual liquid using the micro-centrifuge.

8.

Using the p100 micropipette (yellow-coded), load 15 l of the sample mixture of DNA digests
with the loading dye into the wells of the agarose gel provided. Make sure that you position
the pipette tips carefully on top of the wells and gently deliver the mixture to the wells without
spilling them. Load 15 l of each of the Markers, L1 and L2. Add your samples according to
the following scheme of lanes, starting from the left end of the gel.
Marker L1

9.

Tube 1

Tube 2

Tube 3

Tube 4

Marker L2

Cover the gel with a lid and connect the power supply to run at 100 volts for 20 min. Please be
careful and do not touch any part of the electrodes and power supply.

Page 6 of 9

IBO2012
SINGAPORE
CELL & MOLECULAR BIOLOGY

PRACTICAL TEST 1

10. Check regularly that the samples have entered the wells and are indeed running towards the
positive electrode. If you need help from the technicians to ensure proper runs for the samples,
please signal for assistance by clipping your signal card at the edge of right wall of your cubicle.

11. While waiting for the gel run, answer the following questions in the Answer Sheet:

Q1.4

(20 points) Consider the following scenario: A piece of linear human DNA (1 kbp) was
digested by a particular enzyme RE3, resulting in 2 fragments of 650 bp and 350 bp.
The same piece of 1 kbp DNA was digested with another enzyme RE4, releasing 2
fragments of 800 bp and 200 bp. And when this 1 kbp DNA was digested with RE3 and
RE4 together, 3 fragments of DNA were generated, 650 bp, 200 bp and 150 bp.

Sketch a linear map of this piece of DNA by indicating the position of RE3 and RE4
digests in the space provided. An example of such a sketch is provided below as a
guide.

12. When the 20 minutes of gel running time is up, turn off the power supply and remove the lid of
the gel tank. Carefully remove the gel (still on the gel tray) and place it on the petri dish
provided. Bring your gel to the gel doc that has been assigned for your usage and the
technician will photograph it for you.

13. Bring your gel and the photograph back to your cubicle and use your signal card to get
assistance for an invigilator to staple it in the space provided on the Answer Sheet.
Q1.5

(10 points) Your skills in running a gel will be assessed by the quality of the gel
produced.
Page 7 of 9

IBO2012
SINGAPORE
CELL & MOLECULAR BIOLOGY

PRACTICAL TEST 1

14. Based on the gel results, answer the following questions in the Answer Sheet:
Q1.6

(1 point 5 = 5 points) Using the DNA ladder markers (in basepairs) provided below
as the reference, estimate the sizes of the fragments/bands. You may draw a line
across the band of your query and the size marker to do the estimation. How many
fragment(s) of DNA were generated by RE1 and RE2? And what is/are the estimated
size(s)? Answer using numerals.

L1: 100 bp DNA Ladder

Q1.7

L2: 1 kb DNA Ladder

(1 point) What is the estimated size of the test DNA sample (T)? Answer using
numerals.

Page 8 of 9

IBO2012
SINGAPORE
CELL & MOLECULAR BIOLOGY

Q1.8

PRACTICAL TEST 1

(1 point) Based on your results, is the test DNA sample (T) larger, smaller or the same
size as the empty vector? Indicate your answer with a tick () in the correct box.

Q1.9

(1 point) Does the test DNA sample (T) contain any insert? Indicate yes with a tick ()
and no with a cross ().

Q1.10 (2 points) Uncut DNA appears to move faster than any of the samples digested with
RE2. Why? Indicate your answer with a tick () in the correct box.
a.

The smaller fragment size of uncut DNA is due to DNA degradation.

b.

The uncut DNA is more compact and therefore moves faster through the gel.

c.

RE2 still binds to the DNA and therefore slows down their movement through gel.

Part B. Determination of orientation by which fragment was inserted. (20 points)


Q1.11 (20 points) Construct possible restriction map(s) for the DNA T by indicating the relative
position of RE1 and RE2 and the distance between them in the Answer Sheet. An example
of such a map is provided below as a guide.

END OF PAPER

Page 9 of 9

IBO2012
SINGAPORE
MICROBIOLOGY & BIOCHEMISTRY

PRACTICAL TEST 2

Country: _____________________

Student Code: ________________

23rd INTERNATIONAL BIOLOGY OLYMPIAD


8th 15th July, 2012
SINGAPORE

PRACTICAL TEST 2
MICROBIOLOGY & BIOCHEMISTRY
Total points: 100

Duration: 90 minutes

Page 1 of 13

IBO2012
SINGAPORE
MICROBIOLOGY & BIOCHEMISTRY

PRACTICAL TEST 2

Dear Participants

In this test, you have been given the following two tasks:
Task 1:

Bacteriophage: an effective agent in the killing of pathogenic bacteria. (50 points)


Part A: Effects of Phage and antibiotics on the killing of antibiotic-resistant E. coli (31
points)
Part B: Phage titre and multiplicity of infection (19 points)

Task 2:

Titration of an amino acid. (50 points)

Use the Answer Sheet, which is provided separately, to answer all the questions.

The answers written in the Question Paper will NOT be evaluated.

Write your answers legibly in ink.

Please make sure that you have received all the materials and equipment listed for each task.
If any of these items are missing, please raise your hand immediately.

Stop answering and put down your pen IMMEDIATELY when the bell rings.

At the end of the test, place the Answer Sheets and Question paper in the envelope provided.
Our Assistants will collect the envelope from you.

Have fun and Good Luck!

Page 2 of 13

IBO2012
SINGAPORE
MICROBIOLOGY & BIOCHEMISTRY

PRACTICAL TEST 2

Materials and equipment:


For Task I: Bacteriophage: an effective agent in the killing of bacteria
Materials and equipment

Quantity

Unit

micropipette tips 10l

box

micropipette tips 200l

box

micropipette tips 1000l

box

micropipette 1 - 10l

piece

micropipette 2 - 20l

piece

micropipette 20 - 200l

piece

micropipette 100 - 1000l

piece

microfuge tube rack

piece

cuvette rack

piece

many

tube

stock E. coli culture (1 107 cells/ml) in LB broth

tube

LB broth (in a 50 ml Falcon tube)

tube

sterile deionized water in microfuge tube

tube

ampicillin stock (1 mg/ml) dissolved in deionized water

tube

bacteriophage stock (108 pfu/ml) in deionized water

tube

cuvettes (in a beaker)

piece

stopwatch

piece

floating rack (labelled with your country code)

piece

water-bath 37 C (there is one assigned for your usage)

set

UV-VIS Spectrophotometer (there is one assigned for your usage)

set

photographs of E. coli plates (A to H)

set

Quantity

Unit

25 ml burette

piece

25 ml pipette

piece

100 ml beakers

piece

magnetic stirring bar

piece

magnetic stirrer

set

microfuge tubes (in a beaker)

For Task II: Titration of an amino acid


Materials and equipment

Page 3 of 13

IBO2012
SINGAPORE
MICROBIOLOGY & BIOCHEMISTRY

PRACTICAL TEST 2

pH meter with electrode

set

pipette bulb

piece

Kimwipe papers

box

retort stand with clamps

set

0.3024 M standardized NaOH solution

100

ml

Amino acid Z solution of unknown concentration

80

ml

Page 4 of 13

IBO2012
SINGAPORE
MICROBIOLOGY & BIOCHEMISTRY

PRACTICAL TEST 2

Task I (50 points)


Bacteriophage: an effective agent in the killing of bacteria
Part A. Effects of Phage and antibiotics on the killing of antibiotic-resistant E. coli (31 points)
Introduction
A bacteriophage is a virus that infects bacterial cells. Certain bacteriophages can kill bacteria cells
by lysis. The bacteriophage is now recognized as an effective agent in the killing of pathogenic
bacteria. This provides a good alternative to antibiotics in our combat against disease-causing
bacteria that might be resistant to traditional antibiotics. You are required to design a simple
experiment, with proper controls, to examine the killing efficiency of phage of an ampicillin-resistant
E. coli. Answer the following questions in the Answer Sheet and follow the instructions given
below.

Q1.1

(1 point) To dilute the E. coli culture from 1 107 cells/ml to 2 105 cells/ml, what would be
the dilution factor needed?

Q1.2

(1 point) For 1 ml of E coli culture at a cell density of 2 105 cells/ml, the final concentration
of ampicillin used should be 10 g/ml. What would be the volume of ampicillin stock (1mg/ml)
used?

Q1.3

(1 point) For 1 ml of E coli culture at a cell density of 2 105 cells/ml, final titre of phage
used should be 106 pfu/ml. What would be the volume of phage stock (108pfu/ml) used?

Q1.4

(1 point 15 = 15 points) With the above calculated dilution factors, fill in the table in the
Answer Sheet with your experimental plan. One example is already given for Tube 1 in the
table provided. All units are in l. Carry out your proposed experiment by incubating the
four tubes (placed in the labelled floating rack) for 40 minutes (stop watch provided) in the 37

Page 5 of 13

IBO2012
SINGAPORE
MICROBIOLOGY & BIOCHEMISTRY

PRACTICAL TEST 2

C water bath assigned to you. Hand over the floating rack to the technician at the water
bath.

After incubation, transfer your samples to the cuvettes labelled 1 to 4. In order to observe
the killing of bacteria cells, measure the absorbance at 595 nm wavelength. Bring your
samples to the spectrophotometer that is allocated for your use and hand over your samples
to the technician. You are to record your own readings as the samples are measured.

Q1.5

(0.75 2 + 1.5 points 6 = 10.5 points) Fill in the absorbance reading at 595 nm of the
different tubes of reactions in the table provided in the Answer Sheet. Taking 1 absorbance
unit of the E. coli cells at 595 nm to be equivalent to 1 107 cells/ml, what are the cell
densities of the E. coli in the respective reaction tubes?

Q1.6

(0.5 points 5 = 2.5 points) Which of the following are correct? Indicate correct answer(s)
with a tick () and incorrect answer(s) with a cross ().
a.

Due to the ampicillin resistance, the bacteria cell wall prevented easy penetration of the
antibiotics, but allowed the phages to enter the E. coli cells to cause lysis.

b.

The ampicillin resistance in the E. coli did not prevent the ability of the phage to adsorp
onto the bacteria cells.

c.

The bacteriophage likely has a lytic life cycle of around 20 to 30 mins and hence lysis of
the E. coli was observable during the short experiment.

d.

The temperature of 37 C was not the correct temperature for ampicillin to kill the E. coli
cells.

e.

The phages competed with the E. coli for the nutrients in the LB broth and the bacteria
cells lysed due to insufficient nutrients.

Page 6 of 13

IBO2012
SINGAPORE
MICROBIOLOGY & BIOCHEMISTRY

PRACTICAL TEST 2

Part B. Phage titre and multiplicity of infection (19 points)


The Table below shows the legends for photographs of E. coli lawns that are untreated and infected
with bacteriophages. The E. coli culture used had a starting cell density of 0.5 104 cells/ml. 0.5 ml
of phage was used to infect the E. coli cells. Serial dilutions of the phage culture were made as
indicated and used for infection. (The photographs labelled A to H will be provided as part of
materials for the lab task).

-6

B = 10 dilution

-4

D = 10 dilution

E = 10 dilution

-2

F = 10 dilution

G = neat phage

H = E. coli lawn
uninfected by phage

A = 10 dilution

C = 10 dilution

-5

-3

-1

Page 7 of 13

IBO2012
SINGAPORE
MICROBIOLOGY & BIOCHEMISTRY

Q1.7

PRACTICAL TEST 2

(2 points 4 = 8 points) Based on the number of plaques observed in the photos, calculate
the number of plaques that would be observed if the original undiluted phage culture were
used.

Q1.8

(3 points) To estimate the titre of a phage culture, serial dilutions as shown in the photos (A
to H) are normally performed. Based on the number of plaques shown, indicate with a tick ()
which is the best dilution to confirm the phage titre.

Q1.9

(4 points 2 = 8 points) Using the information given and your answers above, determine:
a.

the plaque forming units per milliliter (pfu/ml) of the phage culture used and

b.

the multiplicity of infection(defined as the ratio of phages to E. coli),

at the best dilution determined in Q1.8.

Page 8 of 13

IBO2012
SINGAPORE
MICROBIOLOGY & BIOCHEMISTRY

PRACTICAL TEST 2

Task II (50 points)


Titration of an Amino Acid
Introduction
Amino acids are organic molecules possessing both carboxyl and amino groups. Table 1 shows the
20 amino acids that cells use to build their thousands of proteins. The majority of the standard
amino acids are diprotic molecules since they have two dissociable protons: one on the amino
group and the other on the carboxyl group; there is no dissociable proton in the R group.
Recall: For an acid HA, the acid dissociation constant for the equilibrium of HA

H+ + A is Ka.

Ka = [H+][A] / [HA]

More often, the strength of acids is expressed in terms of the pKa of the acid:
pKa = log Ka

In the titration of such a diprotic amino acid, the titration will thus occur in two steps as the more
acidic carboxyl group (lower pKa1) and the less acidic amino group (pKa2) successively lose their
protons.

In addition, the pH at which the net charge on the molecule is zero is called the isoelectric point (pI)
of the molecule, a useful constant in characterizing and purifying molecules. Using a titration curve,
the pI can be empirically determined as the inflection point between the pKas of the anionic and
cationic forms.

Page 9 of 13

IBO2012
SINGAPORE
MICROBIOLOGY & BIOCHEMISTRY

PRACTICAL TEST 2

The apparent pKa values for the two dissociation steps may be extrapolated from the midpoints of
each step. This can be shown by the Henderson-Hasselbalch equation:
pH = pKa + log { [A] / [HA] }
The pKa1 (pKa for the carboxyl acid group) is where half the acid group has been titrated. Therefore
the equation becomes:
pH = pKa
Similarly, the pKa2 (pKa for the amino group) can be determined.
In this experiment, you will titrate an unknown amino acid Z and determine its pI, pKa1 and pKa2.

Procedure
1.

Fill the burette with the standardized NaOH solution. Record the exact concentration of this
standardized NaOH solution in the Answer Sheet.

2.

Pipette 25 ml of the unknown amino acid solution Z into a clean 100 ml beaker.

3.

Carefully place the pH probe and a magnetic stirring bar into the amino acid solution, so that
the probe is far enough into the solution, but not touching the stirring bar or beaker. Clamp and
adjust the pH probe such that the stirring bar will not hit the probe while stirring. DO NOT
TOUCH THE CALIBRATION.

4.

Titration 1
Rinse the pH probe with deionized water. Dry the probe gently with a piece of Kimwipe paper.
Determine the pH of the amino acid solution Z before the addition of NaOH. Next, titrate the
amino acid solution with the NaOH from the burette. Add approximately 1.00 ml of the NaOH to
the amino acid at a time. Record the exact volume dispensed and the pH of the solution after
every 1.00 ml interval in the Answer Sheet. Continue until approximately 25 ml of NaOH has
been added.

5.

Repeat the titration (Titration 2)


Rinse the pH probe with deionized water. Dry the probe gently with a piece of Kimwipe paper.
Refill the burette with the standardized NaOH solution and repeat steps 2 4.
Page 10 of 13

IBO2012
SINGAPORE
MICROBIOLOGY & BIOCHEMISTRY

Q2.1

PRACTICAL TEST 2

(3 points 3 = 9 points) Table 1 shows the chemical structures of the twenty standard
amino acids. With reference to these structures, draw structures to show the complete
dissociation of glycine, proline and asparagine.

Q2.2

(3 points 2 = 6 points) For both titrations, record the volume of NaOH (ml) added during
the titration and the observed pH value for the unknown amino acid.

Q2.3

(5 points 2 = 10 points) Using your data, plot the graphs of each titration run (pH versus
Vol. of NaOH (ml)) in Graphs 1 and 2 provided in the Answer Sheet.

Q2.4

(2 points 2 = 4 points) From your titration curves, find the pI and label it on each graph.
Q2.4.1 (2 points) What is the mean pI?

Q2.5

(4 points 2 = 8 points) Find and label the pKa1 and pKa2 on each graph.
Q2.5.1 (2 points 2 = 4 points) What is the mean pKa1 and pKa2?

Q2.6

(5 points) 0.9210 g of the unknown amino acid Z was dissolved in 80 ml of deionized water.
Determine the molecular weight of the unknown amino acid Z. Note: In order to start with a
fully protonated amino acid, HCl solution has been added. This is equivalent to 3.2 ml of the
NaOH solution. To determine the actual number of moles of NaOH needed to reach the pl,
subtract 3.2 ml from the volume of NaOH used to reach the first end point.

Page 11 of 13

IBO2012
SINGAPORE
MICROBIOLOGY & BIOCHEMISTRY

Q2.7

PRACTICAL TEST 2

(2 points) Based on Table 2, identify amino acid Z.


a.

glycine

b.

proline

c.

asparagine

d.

tyrosine

e.

tryptophan

Page 12 of 13

IBO2012
SINGAPORE
MICROBIOLOGY & BIOCHEMISTRY

PRACTICAL TEST 2

Table 1. Structures of the 20 standard amino acids.

Table 2. Molecular weights of amino acids


Amino acid

MW (g/mole)

Glycine

75

Proline

115

Asparagine

132

Tyrosine

181

Tryptophan

204

END OF PAPER
Page 13 of 13

IBO2012
SINGAPORE
PLANT DIVERSITY, ANATOMY & PHYSIOLOGY

Country: _____________________

PRACTICAL TEST 3

Student Code: ________________

23rd INTERNATIONAL BIOLOGY OLYMPIAD


8th 15th July, 2012
SINGAPORE

PRACTICAL TEST 3
PLANT DIVERSITY, ANATOMY & PHYSIOLOGY
Total points: 100

Duration: 90 minutes

Page 1 of 14

IBO2012
SINGAPORE
PLANT DIVERSITY, ANATOMY & PHYSIOLOGY

PRACTICAL TEST 3

Dear Participants

In this test, you have been given the following two tasks:
Task I: Plant diversity and anatomy. (60 points)
Part A: Morphology of seedlings (14.25 points)
Part B: Seed morphology and anatomy (27.25 points)
Part C: Ficus propagule dissection (5 points)
Part D: Functional, ecological and phylogenetic aspects of seeds and seedlings (13.5
points)
Task II: Plant anatomy and physiology. (40 points)
Part A: Anatomy of a plant stem (13 points)
Part B: Study of leaf epidermis and physiology (15 points)
Part C: Interpretation of photosynthetic data (12 points)

Use the Answer Sheet, which is provided separately, to answer all the questions.

The answers written in the Question Paper will NOT be evaluated.

Write your answers legibly in ink (you may use a pencil for diagrams).

Please make sure that you have received all the materials and equipment listed for each task.
If any of these items are missing, please raise your hand immediately.

Stop answering and put down your pen IMMEDIATELY when the bell rings.

At the end of the test, place the Answer Sheets and Question paper in the envelope provided.
Our Assistants will collect the envelope from you.

Have fun and Good Luck!

Page 2 of 14

IBO2012
SINGAPORE
PLANT DIVERSITY, ANATOMY & PHYSIOLOGY

PRACTICAL TEST 3

Materials and equipment:


For Task I: Plant diversity and anatomy
Materials and equipment

Quantity

Unit

Seedlings: A, B, C and D (in plastic cups)

specimen

Seeds/propagules: 1 to 7 (in labelled plastic bags)

specimen

Specimen E (in labelled plastic bag)

specimen

razor blade (use ONLY for Seed 3 and Seed 5)

piece

Scissors (use for Seed 4 and Specimen E)

pair

hand lens

piece

Materials and equipment

Quantity

Unit

Leaves, L (in petri dish L)

piece

Stems, S (in petri dish S)

piece

concentrated HCl (in bottle labelled H, placed within a beaker)

bottle

Phloroglucin stain (in bottle labelled P)

bottle

water (in wash bottle)

bottle

filter paper

sheet

forceps

pair

razor blade

piece

plastic dropper

piece

petri dish (with water, labelled W)

piece

petri dish (labelled LL, LU and SS with the correct student code)

piece

beaker (small)

piece

glass slides

piece

cover slips

piece

compound microscope

set

For Task II: Plant anatomy and physiology

Page 3 of 14

IBO2012
SINGAPORE
PLANT DIVERSITY, ANATOMY & PHYSIOLOGY

PRACTICAL TEST 3

Task I (60 points)


Plant diversity and anatomy
Part A. Morphology of seedlings (14.25 points)
Q1.1

(0.5 points 20 = 10 points; 2 points for quality of drawing; 2.25 points for not
damaging specimens) Make a simple schematic diagram of each seedling (A D) in the
corresponding space provided in the Answer Sheet and label the following (if present) with
a to e and indicate if any are absent:
a.

cotyledons

b.

epicotyl

c.

hypocotyl

d.

leaves

e.

seed coat

Part B. Seed morphology and anatomy (27.25 points)


Each seedling (A to D) from Part A comes from seeds 1-4 respectively.
Q1.2

(0.25 points 11 = 2.75 points) Draw each whole seed (1 6) in the corresponding space
provided in the Answer Sheet (Note: (1) you do not need to draw Seed 7; (2) seed coat of
Seed 5 has been removed). Label the following (if present) with a and b and indicate if any
part is absent:
a.

seed coat

b.

site of attachment of funiculus

Page 4 of 14

IBO2012
SINGAPORE
PLANT DIVERSITY, ANATOMY & PHYSIOLOGY

Q1.3

PRACTICAL TEST 3

(0.5 points 24 = 12 points; 1 point for quality of drawing) After drawing the exterior of
each seed, dissect in longitudinal section and draw the sectioned seed in the corresponding
space provided in the Answer Sheet. (Note: Some seeds have been pre-cut for your
convenience. The seed coat of Seed 5 has been removed.). Label/indicate the following
items on your diagram with a to d:

Q1.4

a.

cotyledons

b.

food storage

c.

hypocotyl

d.

seed coat

(0.5 points 23 = 11.5 points) Indicate the likely ploidy (1N, 2N or 3N) of each of the items
labelled a to d in Q1.3 in the Answer Sheet.

Part C. Ficus propagule (dispersal unit) (5 points)


Q1.5

(1 + 1 + 3 points) Dissect Specimen E longitudinally and draw the longitudinal section of the
propagule (dispersal unit). Draw and label an enlarged section to show in detail the features
of a to c.
a.

fruit

b.

seed

c.

stigma

Part D. Functional, ecological and phylogenetic aspects of seeds and seedlings (13.5 points)
Q1.6

(0.5 points 9 = 4.5 points) Based on the observations in Parts A, B and C, and the
information given in the Table provided in the Answer Sheet, indicate the primary function
of the cotyledon with P (for photosynthesis) or S (for storage of nutrients) and the probable

Page 5 of 14

IBO2012
SINGAPORE
PLANT DIVERSITY, ANATOMY & PHYSIOLOGY

PRACTICAL TEST 3

germination pattern with O (orthodox: seed can undergo dormancy) or R (recalcitrant: seed
does not undergo dormancy).

Q1.7

(1 point 5 = 5 points) Based on the information in the Table and the diagrams that you
have drawn, determine if each of the following statements is true () or false ().
a.

All tropical plants have recalcitrant seeds.

b.

Gymnosperms have at most two cotyledons.

c.

Cotyledon function is phylogenetically constrained within plant families.

d.

Seed size varies considerably in plant families and is probably not phylogenetically
constrained.

e.

Q1.8

Large seed size may be advantageous for some tropical rainforest plants.

(1 point 4 = 4 points) The phylogenies of Malvaceae and Moraceae, and a simplified


phylogeny of seed plants are provided on the next page. Using this information, as well as
the specimens that you have observed today, determine if each of the following statements
is true () or false (), or if there is insufficient evidence to conclude () in the Answer
Sheet.

a.

Large seeds have evolved independently in multiple lineages.

b.

Large seeds have evolved on more than one occasion in some lineages.

c.

Recalcitrant seeds are more associated with tropical plants than with temperate plants.

d.

Gymnosperms are unable to produce fleshy structures associated with animal dispersal
because they lack ovaries.

Page 6 of 14

IBO2012
SINGAPORE
PLANT DIVERSITY, ANATOMY & PHYSIOLOGY

PRACTICAL TEST 3

Phylogeny of Malvaceae

Phylogeny of Moraceae

Simplified phylogeny of seed plants

Gymnosperms
(Seeds 5, 6, 7)

Monocots

Moraceae
+ related families
(Seed 2, Specimen E)

Malvaceae, Sapindaceae
+ related families
(Seeds 1, 3, 4)

Asteraceae + related families

Page 7 of 14

IBO2012
SINGAPORE
PLANT DIVERSITY, ANATOMY & PHYSIOLOGY

PRACTICAL TEST 3

Task II (40 points)


Plant anatomy and physiology
Part A. Anatomy of a plant stem (13 points)
Procedure:
1.

Using the razor blade, cut as thin as possible transverse sections of the stem, S.

2.

Float the cross-sections in water in petri dish, W.

3.

Place a drop of water onto a glass slide and transfer the best cross-section to the centre of the
slide.

4.

Place a small drop of phloroglucin stain (P) onto the cross-section, followed by a small drop of
concentrated hydrochloric acid, HCl (H). Be very careful when handling H as it is corrosive.

5.

Leave for 1 minute and then remove excess stain with the filter paper.

6.

Cover the cross-section with a coverslip and examine it using the compound microscope under
low magnification (4 objective lens).

Answer the following questions in the Answer Sheet:


Q2.1

(1 point) Is S a monocot stem or dicot stem? Indicate M for monocot and D for dicot.

Q2.2

(1 point) Is there any pith? Indicate presence of pith with a tick () and absence with a
cross ().

Q2.3

(1 point) Where is the vascular bundle in the stem located? Indicate centre with C and
periphery with P.

Page 8 of 14

IBO2012
SINGAPORE
PLANT DIVERSITY, ANATOMY & PHYSIOLOGY

PRACTICAL TEST 3

Q2.4 (0.5 points 3 = 1.5 points) Based on your observations, is S the stem of a shrub, tree,
or herb? Indicate correct answer(s) with a tick () and incorrect answer(s) with a cross
() in the Answer Sheet.

Q2.5

(0.5 points) Identify the tissue (a e) that is stained red. Indicate the correct answer
with a tick () in the Answer Sheet.

7.

a.

cortex

b.

endodermis

c.

epidermis

d.

phloem

e.

xylem

When you have completed Part A, place your slide with the stem section into the petri dish SS
for grading purposes.
Q2.6

(8 points)

Part B. Study of leaf epidermis and physiology (15 points)


(i)

Lower epidermis

Procedure:
1.

Peel off the lower epidermis of the leaf, L, with a pair of forceps.

2.

Place it in a drop of water on a glass slide and cover the peeled layer with a cover slip.

3.

Examine it using the compound microscope under10 objective lens.

Page 9 of 14

IBO2012
SINGAPORE
PLANT DIVERSITY, ANATOMY & PHYSIOLOGY

PRACTICAL TEST 3

Answer the following questions in the Answer Sheet:


Q2.7

(2 points) Do you see any stomata? Indicate presence of stomata with a tick ()
and absence with a cross ().

Q2.8

(3 points) Measure the lengths and widths of FIVE (5) epidermal cells that are
representative of the majority of the cells. Smallest unit in the eyepiece micrometer
as seen under 10 objective lens is 10 m. Calculate the mean values and fill in your
answer in the table provided.

4.

When you have completed Part B (i), place your slide with the epidermal peel into the
petri dish LL for checking purposes. The slide will also be used to check for accuracy of
your measurement of epidermal cell dimensions.

(ii) Upper epidermis


Procedure:
1.

Peel off the upper epidermis of the leaf, L, with a pair of forceps. You may either use
the same leaf as before or a fresh leaf from petri dish L.

2.

Place it in a drop of water on a glass slide and cover the peeled layer with a cover slip.

3.

Examine it using the compound microscope under 10 objective lens.

Page 10 of 14

IBO2012
SINGAPORE
PLANT DIVERSITY, ANATOMY & PHYSIOLOGY

PRACTICAL TEST 3

Answer the following questions in the Answer Sheet:


Q2.9

(2 points) Do you see any stomata? Indicate presence of stomata with a tick ()
and absence with a cross ().

Q2.10 (3 points) Measure the lengths and widths of FIVE (5) epidermal cells that are
representative of the majority of the cells. Smallest unit in the eyepiece micrometer
as seen under 10 objective lens is 10 m. Calculate the mean values and fill in your
answers in the table provided.

4.

When you have completed Part B (ii), place your slide with the epidermal peel into
the petri dish LU for checking purposes. The slide will also be used to check for
accuracy of your measurement of epidermal cell dimensions.

Q2.11 (0.5 point 3 = 1.5 points) Based on your observations in Part B (i) and (ii), indicate
the correct answer(s) with a tick () and incorrect answer(s) with a cross ().
a.

There are more stomata in the lower epidermis than in the upper epidermis.

b.

Epidermal cells of the upper epidermis are smaller than those of the lower
epidermis.

c.

Stomata are separated from each other by at least one cell.

Q2.12 (1 point) Based on your observations, determine what type of plant this is. Indicate
the correct answer with a tick () in the Answer Sheet.
a. hydrophyte
b. mesophyte
c. xerophyte

Page 11 of 14

IBO2012
SINGAPORE
PLANT DIVERSITY, ANATOMY & PHYSIOLOGY

PRACTICAL TEST 3

Q2.13 (0.5 point 5 = 2.5 points) Some statements about stomatal structure, function and
development are given below. Indicate true statement(s) with a tick () and false
statement(s) with a cross ().
a. Stomata consist of a pair of highly specialized guard cells that are usually
surrounded by a pair of larger subsidiary cells.
b. Guard cells differ significantly from other epidermal cells in that they have
chloroplasts.
c. Chloroplasts of guard cells differ from mesophyll chloroplasts in that they lack
grana.
d. The number of stomata on any leaf surface is under genetic control and is not
modified by any environmental factors.
e. Stomatal development involves asymmetric cell divisions.

Part C. Interpretation of photosynthetic data from plants measured at different CO2 concentrations
(12 points)

Introduction
Single leaves from plants A and B that had been grown under full sunlight in the same greenhouse
were studied. The responses of their net photosynthetic CO2 assimilation rates to varying levels of
ambient CO2 under saturating light intensity of 1,200 mol quanta m2 s1 at 25 C and 21% O2
measured on leaves in the laboratory are given on the next page:

Page 12 of 14

IBO2012
SINGAPORE
PLANT DIVERSITY, ANATOMY & PHYSIOLOGY

PRACTICAL TEST 3

Net photosynthetic CO2 assimilation rate


-1

Ambient CO2 concentration (l l )


(mol CO2 m2 s1)
Plant A

Plant B

20

0.5

-4

40

11

-1

60

19

2.5

80

28

5.5

100

33

180

41

18

300

44

27

400

44

32

600

44

40

800

44

44

1000

44

45.5

Q2.14 (4 points). Plot a graph by using the data above for Plant A and B in Graph 1 provided in
the Answer Sheet. Use an X-axis scale from 0 to 1000 l l-1.

Based on Graph 1, answer the following questions in the Answer Sheet:


Q2.15 (1 point) Indicate whether plants A and B are C3 or C4 plants. Indicate the correct answers
with a tick () in the Answer Sheet.

Q2.16 (2 points) What is the net photosynthetic CO2 assimilation rate for Plant A and Plant B
measured at 200 l l-1 of CO2 concentration?

Page 13 of 14

IBO2012
SINGAPORE
PLANT DIVERSITY, ANATOMY & PHYSIOLOGY

PRACTICAL TEST 3

Q2.17 (2 points) Plot another graph by using CO2 assimilation rate from 20 to 100 l l-1 of CO2
concentration only (i.e., at low CO2 concentrations) for Plant B in Graph 2 provided in the
Answer Sheet. Use an X-axis scale from 0 to 100 l l-1.

Q2.18 (1 point) Based on Graph 2, what is the CO2 compensation point for plant B? Write the
value in the Answer Sheet.

Q2.19 (1 point) Compared to the data in Graph 2, would the CO2 compensation point increase,
decrease or remain unchanged if the measurements were carried out at 35 C and 21% O2?
Indicate the correct answer(s) with a tick () in the Answer Sheet.

Q2.20 (1 point). Compared to the data in Graph 2, would the CO2 compensation point increase,
decrease or remain unchanged if the measurements were carried out at 25 C and 2% O2?
Indicate the correct answer(s) with a tick () in the Answer Sheet.

END OF PAPER

Page 14 of 14

IBO2012
SINGAPORE
ANIMAL ANATOMY & ECOLOGY

PRACTICAL TEST 4

Country: _____________________

Student Code: ________________

23rd INTERNATIONAL BIOLOGY OLYMPIAD


8th 15th July, 2012
SINGAPORE

PRACTICAL TEST 4
ANIMAL ANATOMY & ECOLOGY
Total points: 100

Duration: 90 minutes

Page 1 of 12

IBO2012
SINGAPORE
ANIMAL ANATOMY & ECOLOGY

PRACTICAL TEST 4

Dear Participants

In this test, you have been given the following two tasks:
Task I: Anatomy of molluscs. (20 points)
Task II: Rank-abundance plots, ABC curves and community structure. (80 points)

Use the Answer Sheet, which is provided separately, to answer all the questions.

The answers written in the Question Paper will NOT be evaluated.

Write your answers legibly in ink.

Please make sure that you have received all the materials and equipment listed for each task.
If any of these items are missing, please raise your hand immediately.

Stop answering and put down your pen IMMEDIATELY when the bell rings.

At the end of the test, place the Answer Sheet and Question paper in the envelope provided.
Our Assistants will collect the envelope from you.

Have fun and Good Luck!

Page 2 of 12

IBO2012
SINGAPORE
ANIMAL ANATOMY & ECOLOGY

PRACTICAL TEST 4

Materials and equipment:


For Task I: Anatomy of molluscs
Materials and equipment

Quantity

Unit

Mollusc 1 (in vial)

specimen

Mollusc 2 (in vial)

specimen

stereomicroscope

set

scissors

pair

forceps

pairs

plastic tray

piece

1000

mL

sheet

water (in beaker)


paper towels

For Task II: Rank-abundance plots, ABC curves and community structure
Materials

Quantity

Unit

Community 1

bag

Community 2

bag

Table A

sheet

Table B

sheet

Page 3 of 12

IBO2012
SINGAPORE
ANIMAL ANATOMY & ECOLOGY

PRACTICAL TEST 4

Task I (20 points)


Anatomy of molluscs

Introduction
Members of the class Bivalvia are successful molluscs with a long evolutionary history. They
possess hinged left and right shell valves that enclose a headless animal within.
Vials labelled 1 and 2 contain two species of marine bivalves that are common in tropical Asia but
live in different habitats. The specimens were partly boiled and preserved in 70% ethanol.
Follow the instructions below to open the animals up for detailed examination under the
stereomicroscope.

Locate the anterior (if present) and posterior adductor muscles that join the left and right
valves of the animal.

Use the pair of scissors to cut the adductor muscles so that the valves can be separated to
expose the internal parts of the animal.

Observe the specimens under water in the trays provided.

Answer the following questions in the Answer Sheet:


Q1.1

(3 points 2 = 6 points) In which habitat (a d) would you expect to find species 1 and 2
respectively?
a.

attached to rocks or other hard surfaces

b.

boring into coral

c.

buried in sand or mud

d.

lying unattached on a sandy substratum

Page 4 of 12

IBO2012
SINGAPORE
ANIMAL ANATOMY & ECOLOGY

Q1.2

PRACTICAL TEST 4

(2 points 2 = 4 points) How many pairs of ctenidia (gills) are there in species 1 and 2
respectively? Answer using numerals.

Q1.3

(2 points 2 = 4 points) How many pairs of labial palps are there in species 1 and 2
respectively? Answer using numerals.

Q1.4

(2 points 2 = 4 points) Locate the anus near the posterior end of the animal in each
species. The anus empties its contents into the path of the exhalant water flow. Starting
with the anus, trace the path of the intestine forwards towards the stomach. Indicate the
position of the intestine in relation to the heart (a e) in the two species respectively.
a.

intestine passes dorsally over the heart

b.

intestine passes under (ventral to) the heart

c.

intestine passes through the heart

d.

intestine passes to the right of the heart

e.

intestine passes to the left of the heart

Q1.5

(0.4 points 5 = 2 points) The following is a list (a-e) of anatomical features in

molluscs. Indicate with a tick () if the feature may be present in bivalves and with a cross
() if it is always absent.
a.

crystalline style

b.

eye

c.

foot

d.

penis

e.

radula

Page 5 of 12

IBO2012
SINGAPORE
ANIMAL ANATOMY & ECOLOGY

PRACTICAL TEST 4

Task II (80 points)


Rank-abundance plots, ABC curves and community structure

Introduction
Changes in community structure may be visualized using a variety of graphs. A rank-abundance
plot or Whittaker plot is used by ecologists to display relative species abundance, a component of
biodiversity. In this type of graph, the rank of each species is plotted along the X axis. The most
abundant species is ranked 1, the second most abundant species is ranked 2, and so forth. The
abundance of each species is plotted on the log scale of the Y axis. The shape of the curve can
provide an indication of dominance or evenness.

The Abundance-Biomass Comparison (ABC) method was proposed by Warwick (1986) as a


technique for monitoring disturbance on benthic invertebrate communities. ABC curves have a
theoretical background in classical theory of r- and K-selection. The relative positions of the
abundance curve and biomass curve serves to indicate the level of disturbance in the community
(see graph below).

Page 6 of 12

IBO2012
SINGAPORE
ANIMAL ANATOMY & ECOLOGY

Q2.1

PRACTICAL TEST 4

(16 points 2 = 32 points) In an environmental impact assessment (EIA) study on the


impact of salmon cage farming on benthic communities, samples were collected from the
various stations along two transects, A1 to A3, and B1 to B7 (see figure below).

, location of floating salmon cages;

, indicates direction of current flow along the coast.

You are a summer intern at the marine laboratory in which this EIA study is conducted. Your
responsibilities include the processing of benthic samples. You are given two bags
containing Community 1 and 2 and your job is to process the samples and collate
information similar to that carried out by a senior research assistant for Community 3 and 4
(see Table A, page 11 for Summary of the information). Each community may contain any of
the 17 species (A Q) listed in Table B (page 12); the respective mean fresh biomass per
individual of each species is also provided in Table B.

Page 7 of 12

IBO2012
SINGAPORE
ANIMAL ANATOMY & ECOLOGY

PRACTICAL TEST 4

The abundance of each species in Community 1 and 2 is indicated with different-sized chips
(see photograph below); e.g., there are 61 individuals of species A shown here:

1 individual

10 individuals

50 individuals

Determine the abundance of each species in Community 1 and record your data in
Table 1 in the Answer Sheet.

Fill in the rest of the required information (to 2 decimal places) in the table. Please note
that lg in the table and figure represents log10 and in the calculator, this is represented
by the log button.

Q2.2

Repeat the entire procedure for Community 2.

(3 points 4 = 12 points) Using your data in Tables 1 and 2, as well as the data provided
in Tables 3 and 4, plot the rank-abundance curves for Community 1 to 4 on Graphs 1 to 4
provided in the Answer Sheet.
Answer the following questions in the Answer Sheet. Indicate correct answer(s) with a tick
() and incorrect answer(s) with a cross ().
Q2.2.1 (1 point 5 = 5 points) Low evenness is:
a.

indicated by a steep slope in the rank-abundance curve.

b.

shown in Community 1.

c.

shown in Community 2.

d.

shown in Community 3.

e.

shown in Community 4.

Page 8 of 12

IBO2012
SINGAPORE
ANIMAL ANATOMY & ECOLOGY

PRACTICAL TEST 4

Q2.2.2 (2.5 points 4 = 10 points) Indicate the correct ABC curves (A H) that
correspond to Community 1 to 4.

abundance; biomass

Page 9 of 12

IBO2012
SINGAPORE
ANIMAL ANATOMY & ECOLOGY

PRACTICAL TEST 4

Q2.2.3 (1.5 points 4 = 6 points) Rank Community 1 to 4 in decreasing levels of


disturbance.

Q2.2.4 (1 point 10 = 10 points) A careless summer intern, mixed up the labels for the
sampling stations (A1 A3; B1 B7) when the samples were transferred from
leaking containers to new bottles. From which stations could the benthic samples
containing Community 1 to 4 likely to be collected?

Q2.2.5 (2.5 points) Which of the 17 species (A Q) is likely to be a bivalve?

Q2.2.6 (2.5 points) Which of the17 species (A Q) has the potential to be a bioindicator of
organic enrichment?

Page 10 of 12

IBO2012
SINGAPORE
ANIMAL ANATOMY & ECOLOGY

PRACTICAL TEST 4
Table A

Community 3

Community 4

Species

Abundance

Rank

Log10 (lg)
Abundance

Cumulative
%
Abundance

Cumulative
%
Biomass

Species

Abundance

Rank

Log10 (lg)
Abundance

Cumulative
%
Abundance

Cumulative
%
Biomass

200

2.30

14.31

28.30

320

2.51

60.49

7.83

180

2.26

27.18

41.03

78

1.89

75.24

38.37

175

2.24

39.70

50.76

50

1.70

84.69

60.39

150

2.18

50.43

58.34

32

1.51

90.74

71.35

120

2.08

59.01

68.65

25

1.40

95.46

85.42

112

2.05

67.02

73.74

10

1.00

97.35

91.53

98

1.99

74.03

81.17

0.70

98.30

93.98

80

1.90

79.76

86.02

0.60

99.05

96.62

75

1.88

85.12

88.29

0.48

99.62

98.53

62

10

1.79

89.56

92.36

10

0.30

100.00

100.00

35

11

1.54

92.06

94.40

30

12

1.48

94.21

96.45

28

13

1.45

96.21

97.44

25

14

1.40

98.00

98.45

15

15

1.18

99.07

98.48

16

0.90

99.64

98.99

17

0.70

100.00

100.00

Total

1398

Total

529

Page 11 of 12

IBO2012
SINGAPORE
ANIMAL ANATOMY & ECOLOGY

PRACTICAL TEST 4

Table B. Fresh biomass per individual for species A to Q.

Species

Mean fresh biomass (g)

0.70

0.90

1.40

2.80

1.30

1.15

0.80

1.35

1.25

0.05

4.00

1.10

1.50

1.00

0.60

1.70

1.20

END OF PAPER

Page 12 of 12















All IBO examination questions are published under the following Creative Commons license:



CC BY-NC-SA (Attribution-NonCommercial-ShareAlike) https://creativecommons.org/licenses/by-nc-sa/4.0/
The exam papers can be used freely for educational purposes as long as IBO is credited and
new creations are licensed under identical terms. No commercial use is allowed.

IBO2012
SINGAPORE
PRACTICAL TEST 1
ANSWER KEY

CELL & MOLECULAR BIOLOGY

Country: _____________________

Student Code: ________________

23rd INTERNATIONAL BIOLOGY OLYMPIAD


8th 15th July, 2012
SINGAPORE

PRACTICAL TEST 1
CELL & MOLECULAR BIOLOGY
ANSWER KEY
Total points: 100

Duration: 90 minutes

Page 1 of 5

IBO2012
SINGAPORE
PRACTICAL TEST 1
ANSWER KEY

CELL & MOLECULAR BIOLOGY

Task (100 points)


Gene mapping by restriction endonuclease digestion of DNA fragments
Part A. Confirmation of insertion of human DNA in a cloning plasmid. (80 points)
Q1.1

Q1.2

Q1.3

Q1.4

(2 points 8 + 1 point 4 = 20 points)

Tube 1

Tube 2

Tube 3

Tube 4

DNA T (with buffer)

10

10

10

10

RE1 (Ndel)

RE2 (EcoRI)

water

10

Total

20

20

20

20

(2 points 5 = 10 points)
a

(2 points 5 = 10 points)
a

(20 points)

Either orientation
Page 2 of 5

IBO2012
SINGAPORE
PRACTICAL TEST 1
ANSWER KEY

CELL & MOLECULAR BIOLOGY

Q1.5

(10 points)

3kbp
0.75kbp
500bp
300bp

Lane 1: 1 kbp ladder (1 point)


Lane 2: uncut (supercoiled and relaxed form) (2 points)
Lane 3: Nde I actual size: 2.57 kbp, 0.76 kbp (2 points for 2 bands).
Lane 4: EcoR I 3.33 kbp (1 point for 1 band)
Lane 5: Nde I/EcoR I actual sizes: 2.57 kbp, 0.49 kbp, 0.27 kbp (3 points for 3
bands)
Lane 6: 100-bp ladder (1 point)

Q1.6

(1 point 5 = 5 points)

Number of fragments
Estimated size

RE1

RE2

2 (1 point)

1 (1 point)

Any value between 2.5 to 3.0 kbp for the


larger fragment, and 0.75 kbp for the
smaller fragment but their total must
be close to the total given in the

3.33 kbp
(any value between
3.0 to 3.5 kbp)
(1 point)

question, i.e., 3.33 kbp) (2 points)

Page 3 of 5

IBO2012
SINGAPORE
PRACTICAL TEST 1
ANSWER KEY

CELL & MOLECULAR BIOLOGY

Q1.7

(1 point)
Answer: ___3.33 kbp (between 3 and 3.5 kbp)_______ .

Q1.8

(1 point) [for having one correct tick; no points if there are more than 1 ticks]

Larger

Smaller

Same
size

Q1.9

(1 point)
Answer: _____ .

Q1.10 (2 points) [for having one correct tick; no points if there are more than 1 tick]
a

Page 4 of 5

IBO2012
SINGAPORE
CELL & MOLECULAR BIOLOGY

PRACTICAL TEST 1
ANSWER KEY

Part B. Determination of orientation by which fragment was inserted. (20 points)


Q1.11 (20 points)
1. 15 points. 9 points for translating the observation in gels to writing down the estimate
sizes of the 3 fragments generated by RE1 and RE2, i.e 2.5 to 3.0 kbp for the largest (3
points), 0.5 to 0.6 for the medium size (3 points) and 0.3 to 0.4 for the smallest fragment
(3 points). However, their total size must be close to the total given in the question, i.e.,
3.0 to 3.5 kbp (3 points) and the values must be consistent with the ones quoted in their
answers to Q6 (3 points). Use enzyme names or RE1 and RE2).
2. 5 points for proposing an alternative map (i.e., the relative position of EcoRI site to the
flanking NdeI sites can be in two arrangements, see answers below).

With alternative model;

Page 5 of 5

IBO2012
SINGAPORE
PRACTICAL TEST 2
ANSWER KEY

MICROBIOLOGY & BIOCHEMISTRY

Country: _____________________

Student Code: ________________

23rd INTERNATIONAL BIOLOGY OLYMPIAD


8th 15th July, 2012
SINGAPORE

PRACTICAL TEST 2
MICROBIOLOGY & BIOCHEMISTRY
ANSWER KEY
Total points: 100

Duration: 90 minutes

Page 1 of 9

IBO2012
SINGAPORE
PRACTICAL TEST 2
ANSWER KEY

MICROBIOLOGY & BIOCHEMISTRY

Task I (50 points)


Bacteriophage: an effective agent in the killing of pathogenic bacteria
Part A. Effects of Phage and antibiotics on the killing of antibiotic-resistant E. coli (31 points)
Q1.1

(1 point)
Answer: ____50-fold or 1/50_

Q1.2

(1 point)
Answer: _____10______l

Q1.3

(1 point)
Answer: _____10______l

Q1.4

(5 points 3 = 15 points)
Tube 1

Tube 2

Tube 3

Tube 4

Diluted E. coli (2 105 cell/ml) in


LB broth

20

20

20

bacteriophage stock (108 pfu/ml) in


deionized water

10

ampicillin stock (1 mg/ml) in


deionized water

10

deionised water

10

10

LB broth

1000

970

970

970

Total (l)

1000

1000

1000

1000

(5 points)

(5 points)

(5 points)

Page 2 of 9

IBO2012
SINGAPORE
PRACTICAL TEST 2
ANSWER KEY

MICROBIOLOGY & BIOCHEMISTRY

Q1.5

Q1.6

(0.75 points 2 + 1.5 points 6 = 10.5 points)

Tube

Absorbance reading at
595 nm

Blanked absorbance at
595 nm

Cell density (cells/ml)

0.143

0.312

0.166

1.69 x 106

0.313

0.165

1.71 x 106

0.157

0.015

1.42 x 104

(0.5 points 5 = 2.5 points)


a

Part B. Phage titre and multiplicity of infection (19 points)


Q1.7

Q1.8

(2 points 4 = 8 points)
Plate

Dilution factor

Number of plaques
observed

Calculated number of plaques


in the original phage culture

10-6

10 -5

200,000

10 -4

15

150,000

10 -3

153

153,000

10 -2

1560

156,000

(3 points)
10-6

10 -5

10 -4

10 -3

10 -2

Q1.9

(4 points 2 = 8 points)
a

3.06 x 105 pfu/ml

61
Page 3 of 9

IBO2012
SINGAPORE
PRACTICAL TEST 2
ANSWER KEY

MICROBIOLOGY & BIOCHEMISTRY

Task II (50 points)


Titration of an Amino Acid
Q2.1

(3 points 3 = 9 points)
Glycine dissociation:

Proline dissociation:

Asparagine dissociation:

Page 4 of 9

IBO2012
SINGAPORE
PRACTICAL TEST 2
ANSWER KEY

MICROBIOLOGY & BIOCHEMISTRY

Q2.2

(3 points 2 = 6 points)

Titration 1
Concentration of standardized NaOH: _____0.3024 M_______________
Starting volume of NaOH: ____0.00 ml________________

Vol. NaOH added (ml)

pH

0.00

1.3

1.00

1.4

2.00

1.4

3.00

1.5

4.00

1.6

5.00

1.7

6.00

1.8

7.00

2.0

8.00

2.1

9.00

2.3

10.00

2.6

11.00

3.0

12.00

9.1

13.00

9.8

14.00

10.1

15.00

10.3

16.00

10.5

17.00

10.8

18.00

11.0

19.00

11.2

20.00

11.5

21.00

11.7

22.00

11.9

23.00

12.0

24.00

12.1

25.00

12.2

Page 5 of 9

IBO2012
SINGAPORE
PRACTICAL TEST 2
ANSWER KEY

MICROBIOLOGY & BIOCHEMISTRY

Titration 2
Concentration of standardized NaOH: _____0.3024 M______________
Starting volume of NaOH: _____0.00 ml_______________

Vol. NaOH added (ml)

pH

0.00

1.4

1.00

1.4

2.00

1.5

3.00

1.6

4.00

1.7

5.00

1.8

6.00

1.9

7.00

2.1

8.00

2.2

9.00

2.4

10.00

2.7

11.00

3.2

12.00

9.4

13.00

9.9

14.00

10.2

15.00

10.4

16.00

10.6

17.00

10.8

18.00

11.0

19.00

11.3

20.00

11.6

21.00

11.8

22.00

12.0

23.00

12.1

24.00

12.2

25.00

12.3

Page 6 of 9

IBO2012
SINGAPORE
MICROBIOLOGY & BIOCHEMISTRY

Q2.3

PRACTICAL TEST 2
ANSWER KEY

(6 points 2 = 12 points) Graphs 1 and 2.


Shape of graph: 2 points (buffering region containing pKa1); 2 points (inflexion point region
containing pI); 2 points (buffering region containing pKa2)

Q2.4

(2 points 2 = 4 points) Graphs 1 and 2.


Arrows to indicate: 2 points (finding and labeling pI in middle)
Q2.4.1 (2 points)
Mean pI: __6.0____ (2 pts: 5.8 6.2; 1 pt: 5.6 5.7 or 6.3 6.4)

Q2.5

(4 points 2 = 8 points) Graphs 1 and 2.


Arrows to indicate: 2 points (finding and labeling pKa1 in middle); 2 points (finding and
labeling pKa2 in middle).
Q2.5.1 (2 points 2 = 4 points)
Mean pKa1: ____1.9___ (2 pts: 1.5 2.3; 1 pt: 1.1 1.4 or 2.4 2.7)
Mean pKa2: ___10.9____ (2 pts: 10.5 11.3; 1 pt: 10.1 10.4 or 11.4 11.7)

Q2.6

(5 points)
Answer: ___115_____.(5 pts: based on volume of NaOH used to obtain value of pI in Q2.4,
create formula in Excel to check if MW has been calculated correctly)

Q2.7

(2 points)
Answer: ___b_______.

Page 7 of 9

IBO2012
SINGAPORE

Student code: ___________________

MICROBIOLOGY & BIOCHEMISTRY

PRACTICAL TEST 2
ANSWER SHEET

Graph 1

Page 8 of 9

IBO2012
SINGAPORE

Student code: ___________________

MICROBIOLOGY & BIOCHEMISTRY

PRACTICAL TEST 2
ANSWER SHEET

Graph 2

Page 9 of 9

IBO2012
SINGAPORE
PRACTICAL TEST 3
ANSWER KEY

PLANT DIVERSITY, ANATOMY & PHYSIOLOGY

Country: _____________________

Student Code: ________________

23rd INTERNATIONAL BIOLOGY OLYMPIAD


8th 15th July, 2012
SINGAPORE

PRACTICAL TEST 3
PLANT DIVERSITY, ANATOMY & PHYSIOLOGY
ANSWER KEY
Total points: 100

Duration: 90 minutes

Page 1 of 13

IBO2012
SINGAPORE
PRACTICAL TEST 3
ANSWER KEY

PLANT DIVERSITY, ANATOMY & PHYSIOLOGY

Task I (60 points)


Plant diversity and anatomy
Part A. Morphology of seedlings (14.25 points)
Q1.1

(0.5 points 20 = 10 points; 2 points for quality of drawings; 2.25 points for not
damaging specimens)
A

Absent: _b, d for some samples_________


C

Absent: _d, e______________

Absent: _____c________
D

Absent: _____c________

Page 2 of 13

IBO2012
SINGAPORE
PRACTICAL TEST 3
ANSWER KEY

PLANT DIVERSITY, ANATOMY & PHYSIOLOGY

Part B. Seed morphology and anatomy (27.25 points)


Q1.2

(0.25 points 11 = 2.75 points)


1

Absent: _____________
3

Absent: _____________
5

Absent: ____a_________

Absent: _____________
4

Absent: _____________
6

Absent: ____b_________

Page 3 of 13

IBO2012
SINGAPORE
PRACTICAL TEST 3
ANSWER KEY

PLANT DIVERSITY, ANATOMY & PHYSIOLOGY

Q1.3

(0.5 points 24 = 12 points; 1 point for quality of drawings)


1

Page 4 of 13

IBO2012
SINGAPORE
PRACTICAL TEST 3
ANSWER KEY

PLANT DIVERSITY, ANATOMY & PHYSIOLOGY

Q1.4

(0.5 points 23 = 11.5 points)


Seed

2N

2N

2N

2N

2N

2N

2N

2N

2N

3N

2N

2N

2N

2N

2N

2N

2N

1N

2N

---

2N

1N

2N

2N

Page 5 of 13

IBO2012
SINGAPORE
PLANT DIVERSITY, ANATOMY & PHYSIOLOGY

PRACTICAL TEST 3
ANSWER KEY

Part C. Ficus propagule (5 points)


Q1.5

(1+1+3 points)
E

Enlarged section

Page 6 of 13

IBO2012
SINGAPORE
PRACTICAL TEST 3
ANSWER KEY

PLANT DIVERSITY, ANATOMY & PHYSIOLOGY

Part D. Functional, ecological and phylogenetic aspects of seeds and seedlings (13.5 points)
Q1.6

(0.5 points 9 = 4.5 points)

Table 1
Seeds

Family

Primary

Seed dry

Probable

function of

weight as %

germination

cotyledon*

fresh weight

pattern+

Climate of original habitat

Malvaceae

60%

Tropical / wet

Moraceae

45%

Tropical / wet

Malvaceae

80%

Tropical-Subtropical / dry

Sapindaceae

65%

Tropical / wet

Pinaceae

80%

Temperate / subtropical

Ginkgoaceae

***

55%

Tropical / wet

Moraceae

85%

Tropical / wet

***Ginkgo cotyledons remain embedded in the seed during germination

Q1.7

Q1.8

(1 point 5 = 5 points)
a

(1 point 4 = 4 points)
a

--

--

Page 7 of 13

IBO2012
SINGAPORE
PRACTICAL TEST 3
ANSWER KEY

PLANT DIVERSITY, ANATOMY & PHYSIOLOGY

Task II (40 points)


Plant anatomy and physiology
Part A. Anatomy of a plant stem (13 points)

Q2.1 Q2.3

Q2.4

Q2.5

(1 point 3 = 3 points)

Q2.1 (M or D)

Q2.2 ( or )

Q2.3 (C or P)

(0.5 points 3 = 1.5 points)

Shrub

Tree

Herb

(0.5 points)

Page 8 of 13

IBO2012
SINGAPORE
PRACTICAL TEST 3
ANSWER KEY

PLANT DIVERSITY, ANATOMY & PHYSIOLOGY

Q2.6

(8 points)

Quality of stem section


(for examiners use only)

CRITERIA
Completeness of stem section

SCORE
/10

Complete: 10 points; Incomplete: 5 points


Staining of stem section

/10

Yes: 10 points
No: O point
Thickness of stem section

/40

Single layer (throughout): 40 points


Single layer in (some areas): 30 points
2-3 layers of cells: 20 points
>3 layers of cells: 10 points
5 bonus points if more than half the section meets criteria
% intact cells in stem section

/20

100% of cells: 20 points


80% of cells: 15 points
50% of cells: 10 points
<5% of cells: 5 points
% air bubbles in stem section

/20

0 bubbles: 20 points
<10 small bubbles: 15 points
>10 small bubbles and some large bubbles: 10 points
Numerous large bubbles, section obscured: 0 point
TOTAL

100

Page 9 of 13

IBO2012
SINGAPORE
PRACTICAL TEST 3
ANSWER KEY

PLANT DIVERSITY, ANATOMY & PHYSIOLOGY

Part B. Study of leaf epidermis and physiology (15 points)


(i)

Lower epidermis

Q2.7

(2 points)
Answer: ___________ .

Q2.8

(1.5 points 2 = 3 points)


1

Mean

Length (m)

120

100

100

150

150

124

Width (m)

100

80

110

70

80

88

Acceptable Answers:
Length: 120225 m; Width: 80130 m
(Length: 125225 m; Width: 80125 m, Chimpan & Sipos; 2009)

(ii) Upper epidermis


Q2.9

(2 points)
Answer: ___________ .

Q2.10 (1.5 points 2 = 3 points)


1

Mean

Length (m)

180

220

240

210

200

210

Width (m)

180

240

210

210

160

200

Acceptable Answers:
Length: 220 270 m; Width: 160 230 m
(Length: 225 250 m; Width: 175 225 m, Chimpan & Sipos; 2009)

Page 10 of 13

IBO2012
SINGAPORE
PRACTICAL TEST 3
ANSWER KEY

PLANT DIVERSITY, ANATOMY & PHYSIOLOGY

Q2.11 (0.5 point 3 = 1.5 points)


a

Q2.12 (1 point)

Q2.13 (0.5 points 5 = 2.5 points)


a

Part C. Interpretation of photosynthetic data from plants measured at different CO2 concentrations (12
points)
Q2.14 (4 points) Graph 1

CRITERIA:
1. Plot
a. Accuracy (1 mark) one point off, - 0.5
mark , two points off, -1 point
b. Differentiation of curves by different symbols
or labels ( 1 point)
2. Smoothness of curves: 2 points, 1 point for
each curve

Page 11 of 13

IBO2012
SINGAPORE
PRACTICAL TEST 3
ANSWER KEY

PLANT DIVERSITY, ANATOMY & PHYSIOLOGY

Q2.15 (0.5 points 2 = 1 point)


C3

C4

Q2.16 (2 points)

Net photosynthetic CO2


assimilation rate

42 0.5

20.5 0.5

mol CO2 m2 s1

mol CO2 m2 s1

(Note: without a unit, 0.5 points will be deducted)

Q2.17 (2 points) Graph 2

CRITERIA:
1. Plot
Accuracy (1 point)
one point off, 0.5 point,
two points off, 1 point
2. Straight line (1 point)

Q2.18 (1 point)
Answer: 46 1 mol CO2 m2 s1
(Note: without a unit, 0.5 points will be deducted.)

Page 12 of 13

IBO2012
SINGAPORE
PLANT DIVERSITY, ANATOMY & PHYSIOLOGY

PRACTICAL TEST 3
ANSWER KEY

Q2.19 (1 point)
increase

decrease

remain unchanged

decrease

remain unchanged

Q2.20 (1 point)
increase

Page 13 of 13

IBO2012
SINGAPORE
PRACTICAL TEST 4
ANSWER KEY

ANIMAL ANATOMY & ECOLOGY

Country: _____________________

Student Code: ________________

23rd INTERNATIONAL BIOLOGY OLYMPIAD


8th 15th July, 2012
SINGAPORE

PRACTICAL TEST 4
ANIMAL ANATOMY & ECOLOGY
ANSWER KEY
Total points: 100

Duration: 90 minutes

Page 1 of 9

IBO2012
SINGAPORE
PRACTICAL TEST 4
ANSWER KEY

ANIMAL ANATOMY & ECOLOGY

Task I (20 points)


Anatomy of molluscs
Answer the following questions:
Q1.1

Q1.2

Q1.3

Q1.4

Q1.5

(3 points 2 = 6 points)
1

(2 points 2 = 4 points)
1

(2 points 2 = 4 points)
1

(2 points 2 = 4 points)
1

(0.4 points 5 = 2 points)


a

Page 2 of 9

IBO2012
SINGAPORE
PRACTICAL TEST 4
ANSWER KEY

ANIMAL ANATOMY & ECOLOGY

Task II (80 points)


Rank-abundance plots, ABC curves and community structure
Q2.1

(8 points 4 = 32 points) See Tables 1 2.


For each table:
4 pts for all correct answers in each column (Species and Abundance); minus pt for
every wrong answer until 0 point is reached.
2 pts for all correct answers in each column (Log10 Abundance, Cumulative %
Abundance and Cumulative % Biomass); minus pt for every wrong answer until 0
point is reached.
Pivot points: Total Abundance and Total biomass of species 1 pt for correct answer; 0
pt for wrong answer.
Note: Error Carried Forward (ECF) will be taken into consideration (correlated answers
generated by Excel).

Q2.2

(3 points 4 = 12 points) See Graphs 1 4.


Q2.2.1 (1 point 5 = 5 points)
a

Q2.2.2 (2.5 points 4 = 10 points)


1

Q2.2.3 (1.5 points 4 = 6 points)


Highly disturbed undisturbed
2

Q2.2.4 (1 point 10 = 10 points)


1

B1, B6

A1, A2, B3, B4

A3, B7

B2, B5

B7

B2, B5

or
A3, B1, B6

A1, A2, B3, B4

Q2.2.5 and Q2.2.6 (2.5 points 2 = 5 points)


Q2.2.5

Q2.2.6

Page 3 of 9

IBO2012
SINGAPORE
PRACTICAL TEST 4
ANIMAL ANATOMY & ECOLOGY
ANSWER KEY

Table 1. Community 1

Species

Abundance

Rank

Log10 (lg)
Abundance

%
Abundance

Cumulative %
Abundance
(+/- 0.10)

Biomass of
individual

Total
biomass of
species

% Biomass

Cumulative %
Biomass
(+/-0.10)

120

2.08

17.39

17.39

1.00

120.00

10.05

10.05

115

2.06

16.67

34.06

2.80

322.00

26.98

37.03

100

2.00

14.49

48.55

1.25

125.00

10.47

47.50

85

1.93

12.32

60.87

1.70

144.50

12.11

59.61

78

1.89

11.30

72.17

1.15

89.70

7.51

67.12

62

1.79

8.99

81.16

4.00

248.00

20.78

87.90

50

1.70

7.25

88.41

1.30

65.00

5.45

93.34

25

1.40

3.62

92.03

0.90

22.50

1.88

95.23

20

1.30

2.90

94.93

1.50

30.00

2.51

97.74

15

10

1.18

2.17

97.10

1.35

20.25

1.70

99.44

12

11

1.08

1.74

98.84

0.05

0.60

0.05

99.49

12

0.70

0.72

99.57

0.80

4.00

0.34

99.82

13

0.48

0.43

100.00

0.70

2.10

0.18

100.00

Total

690

1193.65

Page 4 of 9

IBO2012
SINGAPORE
PRACTICAL TEST 4
ANIMAL ANATOMY & ECOLOGY
ANSWER KEY

Table 2. Community 2

Species

Abundance

Rank

Log10 (lg)
Abundance

%
Abundance

Cumulative %
Abundance
(+/- 0.10)

Biomass of
individual

Total
biomass of
species

% Biomass

Cumulative %
Biomass
(+/-0.10)

1200

3.08

96.93

96.93

0.05

60.00

64.38

64.38

15

1.18

1.21

98.14

0.70

10.50

11.27

75.65

0.90

0.65

98.79

0.90

7.20

7.73

83.38

0.70

0.40

99.19

0.80

4.00

4.29

87.67

0.60

0.32

99.52

1.00

4.00

4.29

91.96

0.48

0.24

99.76

1.25

3.75

4.02

95.98

0.30

0.16

99.92

1.30

2.60

2.79

98.77

0.00

0.08

100.00

1.15

1.15

1.23

100.00

Total

1238

93.20

Page 5 of 9

IBO2012
SINGAPORE
PRACTICAL TEST 4
ANIMAL ANATOMY & ECOLOGY
ANSWER KEY

Graph 1

Provision made for Error Carried Forward (ECF).


Minus point for every incorrect plotted point (within ONE square proximity) until 0 point is reached.

Page 6 of 9

IBO2012
SINGAPORE
PRACTICAL TEST 4
ANIMAL ANATOMY & ECOLOGY
ANSWER KEY

Graph 2

Provision made for Error Carried Forward (ECF).


Minus point for every incorrect plotted point (within ONE square proximity) until 0 point is reached.

Page 7 of 9

IBO2012
SINGAPORE
PRACTICAL TEST 4
ANIMAL ANATOMY & ECOLOGY
ANSWER KEY

Graph 3

Provision made for Error Carried Forward (ECF).


Minus point for every incorrect plotted point (within ONE square proximity) until 0 point is reached.

Page 8 of 9

IBO2012
SINGAPORE
PRACTICAL TEST 4
ANIMAL ANATOMY & ECOLOGY
ANSWER KEY

Graph 4

Provision made for Error Carried Forward (ECF).


Minus point for every incorrect plotted point (within ONE square proximity) until 0 point is reached.

Page 9 of 9

INTERNATIONAL BIOLOGY OLYMPIAD


THEORY PROBLEMS

2011, Taipei, Chinese Taipei















All IBO examination questions are published under the following Creative Commons license:



CC BY-NC-SA (Attribution-NonCommercial-ShareAlike) https://creativecommons.org/licenses/by-nc-sa/4.0/
The exam papers can be used freely for educational purposes as long as IBO is credited and
new creations are licensed under identical terms. No commercial use is allowed.

IBO 2011
TAIWAN
THEORETICAL TEST
PART A

Student Code:

22nd INTERNATIONAL BIOLOGY OLYMPIAD


July 10-17, 2011
Taipei, Taiwan

THEORETICAL TEST: PART A


Duration: 120 minutes

IBO 2011
TAIWAN
THEORETICAL TEST
PART A

Dear participants,

Check your Student Code on the Answer Sheet before starting the test.
The questions in Part A have only one correct answer. Fill your answer in the Answer Sheet.
Mark the correct answer with X on the Answer Sheet clearly, as shown below.

No.

A0

You can use a ruler and a calculator provided.


Write down your results and answers in the Answer Sheet. Answers written in the Question
Paper will not be evaluated.
Some of the questions may be marked DELETED. DO NOT answer these questions.
The maximal point of Part A is 116 (2 points each for each question).
Stop answering and put down your pen IMMEDIATELY after the end bell rings.
:

Good Luck!!

IBO 2011
TAIWAN
THEORETICAL TEST
PART A

I. Cell Biology
A1. Endorphin is a natural analgesic secreted by the pituitary gland. Upon binding to its receptor
in brain cells, endorphin can relieve pain and create a sense of euphoria. Morphine can
achieve similar pain relief effects by binding to the endorphin receptor. Why do both
endorphin and morphine bind to the endorphin receptors in brain cells?
(A) Sizes of both molecules are similar.
(B) Molecular weights of both molecules are similar.
(C) Both are isomers.
(D) Shapes of both molecules are similar.
(E) Net charges of both molecules are identical.
A2. Most biological macromolecules are made by the polymerization of small principal
components. The major structural polysaccharide of the insect exoskeleton is a polymer.
Which of the following statements regarding this kind of polysaccharide is NOT correct?
(A) It is made by polymerization of glucose.
(B) It contains C, H, O and N atoms.
(C) Its structure is similar to that of cellulose.
(D) It can be used to produce chitosan and glucosamine in industry.
(E) This polymer can also been found in the cell wall of fungi.

IBO 2011
TAIWAN
THEORETICAL TEST
PART A

A3. In some cells, synthesis of isoleucine from threonine is catalyzed by the sequential action of
five enzymes a, b, c, d and e which produce 4 intermediates A, B, C, D, and the end product
isoleucine, respectively. What is most likely to happen when isoleucine is overproduced and
there is an ample supply of threonine in cells?
(A) Isoleucine associates with threonine to inhibit the activity of enzyme a.
(B) Isoleucine associates with intermediate D to inhibit the activity of enzyme e.
(C) Isoleucine binds to enzyme a and inhibits its activity.
(D) Isoleucine binds to enzyme e and inhibits its activity.
(E) Threonine is converted into isoleucine continuously through the 5 enzymes.
A4. In some prokaryotic organisms, SO42- is used as the final electron receptor at the end of
electron transport chain during cellular respiration. Which of the following statements
regarding cellular respiration in these prokaryotic organisms is NOT correct?
(A) It is anaerobic respiration.
(B) The reception of electron by SO42- is accompanied by the production of H2O.
(C) Operation of the electron transport chain builds up a proton motive force.
(D) ATP can be produced.
(E) Production of ATP is correlated with the mobility of H+.

IBO 2011
TAIWAN
THEORETICAL TEST
PART A

A5. Three stages in bacteria growth are:


I.

Lag phase

II. Log phase


III. Stationary phase
In which phase or phases can penicillin inhibit the synthesis of the bacterial cell wall?
(A) Only I
(B) Only II
(C) Only III
(D) Only I and II
(E) Only I and III
(F) I, II and III
A6. Which structural or physiological feature of bacteria can be used as a target for developing
drugs to kill bacteria effectively but with no harm to human cells?
(A) Glycolysis
(B) Components of plasma membrane
(C) Components of ribosome
(D) Components of the electron transport chain in aerobic respiration
(E) Requirement of oxygen

IBO 2011
TAIWAN
THEORETICAL TEST
PART A

A7. Histones are small basic proteins that assemble with DNA molecules to form chromosomes.
There are five histones, including H1, H2A, H2B, H3 and H4, in eukaryotic cells. Which of
the following structural features of chromosomes is associated with Histone H1?
(A) Telomere
(B) Nucleosome fiber (10-nm fiber)
(C) 30-nm fiber
(D) Looped domains
(E) Centromere
A8. DNA is a double helix molecule containing four different types of nitrogen bases. Which of
the following statements regarding both the replication and chemical composition of DNA
is correct?
(A) Base sequences of both strands are the same.
(B) The amount of purine is equal to that of pyrimidine in a double-stranded DNA.
(C) Both strands are synthesized continuously in 53 direction.
(D) The first base of the newly synthesized DNA is catalyzed by DNA polymerase.
(E) The proof-reading activity of DNA polymerase proceeds in the 53 direction.
A9. Mister Spiderman has compared the DNA, the corresponding RNA and protein sequences of
many human genes. What conclusion can be drawn from the sequence comparison?
(A) The number of exons is always more than that of introns.
6

IBO 2011
TAIWAN
THEORETICAL TEST
PART A

(B) The translation start codon is located within the first exon.
(C) The translation stop codon is located within the last exon.
(D) The G nucleotide of RNA capping is the first nucleotide transcribed from DNA.
(E) The polyA tail is transcribed from the polydT of DNA.
A10. Miss Ling-Ling conducts DNA synthesis and transcription reactions in two separate test
tubes. Which of the following substances needs to be added to both reactions?
(A) ATP
(B) DNA template
(C) RNA primer
(D) DNA polymerase
(E) DNA ligase
A11. The Nobel Prize in Physiology or Medicine 2009 was awarded jointly to Blackburn, Greider
and Szostak for the discovery that chromosomes are protected by telomeres and the enzyme
telomerase is highly correlated with aging and cancer in animals. Which of the following
statements regarding telomere and telomerase is correct?
(A) Telomerase is a DNA exonuclease.
(B) Telomerase is an RNA polymerase.
(C) Embryonic cells possess long telomeres and high telomerase activity.
(D) Telomeres are longer and telomerase is inactive in cancer cells.
7

IBO 2011
TAIWAN
THEORETICAL TEST
PART A

(E) Telomeres are longer and telomerase is highly active in somatic cells.
A12. EcoRI restriction enzyme is a DNA endonuclease that can recognize the sequence GAATTC.
It was first discovered in E. coli, therefore it was named EcoRI. To produce a large quantity
of the endonuclease, the DNA fragment encoding the gene was subcloned into an
expression plasmid and the resultant recombinant plasmid was transformed into E. coli cells
to produce recombinant enzyme for a study. Why is the host DNA not cleaved by the
recombinant EcoRI?
(A) The host DNA does not contain EcoRI cleavage sites.
(B) EcoRI is secreted out of the host cells.
(C) Environmental factors such as temperature and pH value inhibit EcoRI activity.
(D) The E. coli host produces inhibitors to block EcoRI activity.
(E) The EcoRI cleavage sites within the host DNA are modified.

IBO 2011
TAIWAN
THEORETICAL TEST
PART A

II. Plant anatomy and physiology


A13. Hypersensitive response is one of the plant defense responses to pathogens. Each of four
pathogen strains, a to d, produce a distinct range of effectors. One of the effectors, Avr,
recognized by a specific receptor protein encoded by the resistance (R) gene in the host
plant is present in strains b and c. Host plants B and D produce the R protein. Which plant(s)
are likely to develop a hypersensitive response after the host plants A to D are infected by
pathogens a to d (a A, b B, c C, d D), respectively?
(A) A only
(B) B only
(C) C only
(D) D only
(E) B and C
(F) B and D
A14. Plant movement occurs when plant organs change their spatial distribution after being exposed to
stimuli, and may be caused either by differential growth or by differential turgor change among
cells within the organs. Which of the following plant movements uses a mechanism distinct from
the others?
(A) The gravitropic movement of corn roots
(B) The closure of soybean leaflets during night time
9

IBO 2011
TAIWAN
THEORETICAL TEST
PART A

(C) The tentrils of cucumber moving along the trellis


(D) The phototropic movement of mungbean seedlings
(E) The downward bending of the tomato leaves after flooding treatment
Questions 15 and 16 are a problem set
A15. In the model plant Arabidopsis, the DXS,
DXR, CMS, CMK, MCS, HDS and HDR
enzymes are involved in the methyl
erythritol phosphate (MEP) pathway of
isopentenyl diphosphate (IPP) and
dimethylallyl diphosphate (DMAPP)
biosynthesis. The Arabidopsis white devil
albino mutant is impaired in the enzyme
HDS. Assuming Arabidopsis can
efficiently take up the intermediate
metabolites of the MEP pathway, the
white devil albino mutant will grow and
turn green if given which of the following compound?
(A) MEP
(B) CDP-ME
10

IBO 2011
TAIWAN
THEORETICAL TEST
PART A

(C) CDP-MEP
(D) ME-cPP
(E) HMBPP
A16. The plant MEP pathway is located in which of the following organelle?
(A) nucleus
(B) vacuole
(C) chloroplast
(D) mitochondrion
(E) endoplasmic reticulum
A17. Dennis dissected a plant leaf and found bundle sheath cells full of starch granules. Which of the
following characteristics can be observed in this plant?
I. Stomata open at night
II. Presence of PEP carboxylase in mesophylls
III. Presence of Rubisco in bundle sheath cells
IV. High photorespiration rate on hot summer days
V. Light reaction and carbon fixation occur in different cell types
VI. Carbon assimilation rate is saturated in the early morning on summer days
(A) Only I, III
(B) Only II, IV
(C) Only II, IV, V
11

IBO 2011
TAIWAN
THEORETICAL TEST
PART A

(D) Only II, III, V


(E) Only II, III, V, VI
(F) Only II, IV, V, VI
A18. It has been estimated that around 124 million children are vitamin A deficient, causing
about 500,000 children to go blind each year. To help children who suffer from vitamin A
deficiency, scientists have developed a variety of rice (Oryza sativa L.), Golden Rice,
through genetic engineering. The original Golden Rice was produced using the japonica
variety Taipei 309, which is genetically enriched in
(A) auxin
(B) starch
(C) -carotene
(D) iron
(E) anthocyanins
A19. Abscisic acid (ABA) is one of the important growth regulators of plants.
antagonizes the functions of hormones that promote growth.

It often

Plant biologists have been

interested in elucidating the signaling pathway of ABA by genetic approaches.

They

screened mutants of the model plant Arabidopsis thaliana that respond abnormally to ABA
treatment to identify the involving components of the pathway.
phenotype is ABA-insensitive (abi).

One type of mutant

Which of the following phenotypes are likely to be

12

IBO 2011
TAIWAN
THEORETICAL TEST
PART A

observed in the abi mutants?


(1) Seeds germinate at the presence of exogenous ABA.
(2) Seeds become dormant at the presence of exogenous ABA.
(3) Stomata do not close in response to drought.
(4) More tolerant to drought than the wild-type plants.
(5) Leaf does not abscise when it becomes senescent.
(6) Leaf is prematurely abscised even when greenish.
(A) Only (1), (3)
(B) Only (2), (3)
(C) Only (2), (5)
(D) Only (2), (4), (5)
(E) Only (1), (3), (6)
(F) Only (2), (4), (6)
A20. Ethylene is a hormone that influences plants growth and development. It is known that
treatment with 10 ppm of 1-methylcyclopropene (MCP) can block the signal transduction of
ethylene. If certain plant tissues were treated with 10 ppm MCP, which of the following
phenotypes could be observed in MCP-treated tissues?
(A) Shorter hypocotyl in etiolated mung bean seedling
(B) Increased degradation of chlorophyll in detached leaves
13

IBO 2011
TAIWAN
THEORETICAL TEST
PART A

(C) Increased synthesis of ethylene in banana fruits


(D) Inhibition of the ripening of tomato fruits
(E) Induction of the senescence of carnation cut flowers
A21. During leaf development in water lily, the sclereid-initials grow and elongate along the
palisade mesophyll cells or the intercellular space between them. After elongation they
gradually form calcium oxalate crystals in the cell wall along the cell membrane. Thereafter,
they form the secondary cell wall. Four cell wall structures are: (I) primary cell wall; (II)
secondary cell wall; (III) middle lamella; (IV) calcium oxalate crystals. What is the final
sequence of structures in the mature sclereids of water lily, starting from the plasma
membrane as the innermost layer to the outermost layer?
(A) I IV II III
(B) III IV I II
(C) I IV II III
(D) III I IV II
(E) II IV I III
A22. Agrobacterium tumefaciens-mediated transformation, a widely used method to transfer
foreign genes into the plant genome, has contributed to the considerable successes that
plant biotechnology has already achieved. For instance, a gene encoding the coat protein
(CP) of papaya ringspot virus (PRSV) was used to generate the virus-resistant transgenic
14

IBO 2011
TAIWAN
THEORETICAL TEST
PART A

SunUp papaya in Hawaii. The construct used for transformation includes the CP gene and a
selectable marker gene (nptII) conferring kanamycin resistance. Both CP and nptII genes
are driven by a constitutive cauliflower mosaic virus (CaMV) 35S promoter. According to
the above information, which of the following statements is NOT correct?
(A) The SunUp papaya is resistant to kanamycin.
(B) The SunUp papaya contains some DNA sequences from CaMV.
(C) The SunUp papaya contains some genomic DNA of Agrobacterium tumefaciens.
(D) The SunUp papaya contains a portion of the Ti plasmid termed T-DNA.
(E) The SunUp papaya contains the nptII gene.

15

IBO 2011
TAIWAN
THEORETICAL TEST
PART A

III. Animal anatomy and physiology


A23. Which of the following is the only vertebrate in which blood flows directly from respiratory
organs to body tissues without returning to the heart first?
(A) Fish
(B) Amphibians
(C) Mammals
(D) Reptiles
(E) Birds
A24. How does the hemocyanin of arthropods differ from the hemoglobin of mammals?
(A) The oxygen dissociation curve of hemocyanin is not a S-shape
(B) Hemocyanin carries considerably more carbon dioxide
(C) Hemocyanin is a single-chain respiratory pigment
(D) Hemocyanin is a protein coupled with magnesium
(E) Hemocyanin is a protein coupled with copper
A25. A shark is more likely to survive for an extended period of food deprivation than is a
dolphin with equivalent size because
(A) The shark maintains a higher basal metabolic rate
(B) The shark expends more energy/kg body weight than the dolphin
(C) The shark invests much less energy in temperature regulation
16

IBO 2011
TAIWAN
THEORETICAL TEST
PART A

(D) The shark metabolizes its stored energy more readily than the dolphin does
(E) The shark has a better insulation on its body surface
A26. Increased arteriolar resistance contributes to hypertension. Which one of the following
factors contribute to the increased vascular resistance most significantly?
(A) Vessel length
(B) Blood viscosity
(C) Vascular diameter
(D) Total leukocyte counts
(E) Heart rate
A27. A method to estimate an mammals blood volume uses a specific radioactive isotope of
iodine(123I). This isotope, usually produced synthetically, has a half-life time of 13 hours. It
decays to 123Te, which is almost perfectly stable. To estimate the blood volume, 10 mL of
iodine solution are injected into the animals vein. The activity of the solution at the
injection is 2mSv. A sample of 10 mL of the animals blood, taken 13 hours after the
injection, is 0.0025mSv. The estimate volume of the animals blood volume is?
(A) 10.0 L
(B) 8.0 L
(C) 4.0 L
(D) 2.5 L
17

IBO 2011
TAIWAN
THEORETICAL TEST
PART A

(E) 1.25 L
A28. Which of the following events will result in an excitatory postsynaptic potential?
a.

Increasing sodium influx.

b. Blocking potassium out-flux.


c.

Increasing calcium influx.

d. Closing a chloride channel.


(A) Only a & b
(B) Only b & c
(C) Only a, c & d
(D) Only b, c & d
(E) a, b, c & d.

18

IBO 2011
TAIWAN
THEORETICAL TEST
PART A

A29. Which of the following is the correct effect of hyperthyroidism (hypersecretion of thyroid
hormone) on Thyrotropin-releasing hormone (TRH), thyroid-stimulating hormone (TSH), and
Thyroid hormones T3 and T4?
: increase : decrease

: remains unchanged

TRH

TSH

T3

T4

A30. Which of following receptors/molecules are required for the activation of Helper T cells
trigged by antigen-presenting cells.
1. CD8
2. CD4
3. Class I MHC molecule
4. Class II MHC molecule
5. T cell receptor
(A) Only 1, 3 & 5
19

IBO 2011
TAIWAN
THEORETICAL TEST
PART A

(B) Only 2, 4 & 5


(C) Only 3, 4 & 5
(D) Only 2 & 4
(E) Only 1 & 3
A31. Inspect the following table which is revealed to the function of kidneys in vertebrate.

What

1. Urine concentration by NaCl reabsorption


2. Urine concentration by urea reabsorption
3. Aquaporin mediated water reabsorption

How

4. Countercurrent multiplier system


5. Countercurrent exchange

Where

6. Loop of Henle
7. Collecting duct
8. Proximal tubule

Figure out which of the following alternatives show a correct combination.


(A) 1-4-6
(B) 1-4-8
(C) 2-4-6
(D) 2-5-8
20

IBO 2011
TAIWAN
THEORETICAL TEST
PART A

(E) 3-5-7
A32. When people lose blood quickly as happens in a car accident, which of the following
situations will NOT occur
(A) Stroke volume increases; cardiac output increases
(B) Blood volume decreases but interstitial fluid increases
(C) Increase in the resistance of blood vessel
(D) Decrease of sodium concentration in urine
(E) Decrease in the proportion of red blood cells in blood.
A33. The following figure illustrates the membrane potential changes measured at three different
sites (A, B, C) along a sensory neuron and the release of neurotransmitters from the axon
termini when depolarizing electrical stimuli with varied intensities were applied to the
dendrite. Based on the information provided in the figure below, choose the correct
statements in the following box.

21

IBO 2011
TAIWAN
THEORETICAL TEST
PART A

1. The membrane potential changes evoked at A site would be proportional to the


intensity of the electrical stimuli applied to the dendrite.
2. An action potential would be recorded at B site only when the intensity of the applied
current stimulus causes the membrane potential to be higher than the threshold
potential in the axon hillock.
3. The frequency of the action potentials at B site is independent of the intensity of the
applied current stimulus at A.
4. The quantity of the neurotransmitters released from the axon termini is unlikely to
depend on the frequency of the action potential at C site.
(A) Only 1 and 2
(B) Only 1 and 3
(C) Only 2 and 3
(D) Only 3 and 4
(E) Only 1, 2, and 3
A34. The perception of the messages from the environmental changes is carried out by
specialized sensory cells. Their structural composition is in a strict accordance to their
functions. Please analyze the following pictures and answer the question below:

22

IBO 2011
TAIWAN
THEORETICAL TEST
PART A

Which one of the above receptors in human receptors will be activated by a stimulation and
trigger the opening of a potassium channel?
(A) 1
(B) 2
(C) 3
(D) 4
(E) 5

23

IBO 2011
TAIWAN
THEORETICAL TEST
PART A

IV. Ethology
A35. There are three types of chemical substances that organisms emit to mediate interspecific
interactions: kairomone, allomone, and synomone. Kairomone benefits individuals of
another species which receives it but is disadvantageous to the emitter. Allomone benefits
the emitter, and does not benefit or harm the receiver. Synomone benefits both the emitter
and receiver. A plant species emits a volatile essential oil that attracts a phytophagous beetle
to feed and lay eggs on its leaves. At the same time, it also attracts a parasitoid wasp, and
helps this parasitic natural enemy of the beetles to locate the beetle larvae within which they
can lay their own eggs. Which of the following descriptions of the role that this essential oil
plays is correct?
(A) It acts as a synomone between the plant and the beetle, and an allomone between the
plant and the parasitoid wasp.
(B) It acts as a kairomone between the plant and the parasitoid wasp, and a synomone
between the beetle and the parasitoid wasp.
(C) It acts as a kairomone between the plant and the beetle, and a synomone between the
plant and the parasitoid wasp.
(D) It acts as a kairomone between the plant and the beetle, and an allomone between the
beetle and the parasitoid wasp.
(E) It acts as a kairomone between the plant and the parasitoid wasp, as well as between the
24

IBO 2011
TAIWAN
THEORETICAL TEST
PART A

beetle and the parasitoid wasp.


A36. In terms of the benefits and harms received by each of the two parties interacting, which of
the following pairs of biological interactions are most similar to each other?
(A) Clownfish and sea anemones; mistletoes and apple trees.
(B) Sea stars and bivalves; locusts and grasshoppers.
(C) Lichens and maples; mistletoes and oaks.
(D) Caterpillars and parasitic wasps; food plants and caterpillars.
(E) HIV virus and human; mushrooms and rotten woods.
A37. Great tits (Parus major) inhabiting forests and woodlands (patchy forest) have different
song patterns.

It is documented that high frequency sounds become less degraded in open

habitat than in places with dense vegetation. Consider the following graphs showing song
characteristics of great tits from 6 locations. Which of the following statements is correct?

25

IBO 2011
TAIWAN
THEORETICAL TEST
PART A

(A) There is less variability in song frequency in low-latitude regions.


(B) Forest inhabitants are more varied in song frequency than woodland inhabitants.
(C) Songs of forest inhabitants have more notes per phrase than those of woodland
inhabitants.
(D) The variation of song type has nothing to do with habitat type
(E) If an individual moves from forests to open grassland, the mean frequency of the song is
likely to increase.
A38. A hypothetical insect species employs the polygynous mating system, in which the males
are capable of multiple mating, but the females mate only once. The adults of this species
occur in the fall when matings take place. All adults of this species die off soon after the
mating season. The sex ratio of this species is 1:1. The below shows the comparison of the
mating success, defined as the number of mating in a given season, for both male and
female in this species. Which one of the graphs below (A to E) best describes the mean and
variance of both male and female in this species? Solid dots represent means, and lines
denote the ranges of variance.

26

IBO 2011
TAIWAN
THEORETICAL TEST
PART A

27

IBO 2011
TAIWAN
THEORETICAL TEST
PART A

V. Genetics and evolution


A39. Frank has subcloned a cDNA fragment from an animal into an expression plasmid. The
recombinant plasmid was transformed into bacteria to produce recombinant protein., What
is the major reason that the expressed protein is non-functional ?
(A) Differences in codon usage between animals and bacteria
(B) Differences in protein modification between animals and bacteria
(C) Components of bacterial culture media
(D) Modulators of gene transcription
(E) Secretion signal of proteins
A40. Gregor Mendel discovered that segregation of genes on non-homologous chromosomes is
independent of each other in his garden pea hybridization experiments. Four alleles A, B, C
and D are located on four non-homologous chromosomes. Which of the following
genotypes will have the highest chance to produce the dominant trait in all four loci when it
mates with an organism with the genotype AaBbCcDd?
(A) aabbccdd
(B) AaBbCcDd
(C) AaBBccDd
(D) AaBBCCdd
(E) aaBBCCdd
28

IBO 2011
TAIWAN
THEORETICAL TEST
PART A

A41. An X-linked allele determines the coat color of cats with orange being dominant and black
being recessive. Which of the following statements regarding the inheritance pattern of
orange/black mosaic cats is correct?
(A) Half of all male cats are mosaic.
(B) The mosaic phenotype is a consequence of gene interaction.
(C) The mosaic phenotype is correlated with genomic imprinting.
(D) The mosaic phenotype results from random X-chromosomal inactivation.
(E) The offspring from matings of orange males and black females are mosaic.
Questions 42 and 43 are a problem set
A42. On a remote island, Dr. Yeh discovered a new plant species, which can produce either white
or blue flowers. This species is self fertilized or cross pollinated by insects. Genetic
experiments showed that the white-flower phenotype is recessive to the blue-flower
phenotype. Statistical analysis revealed that 91% of these plants on the island produce blue
flowers. If one is to randomly select two blue-flower plants and cross them, then what is the
approximate probability that their F1 offspring will produce white flowers?
(A) 0.09

(B) 0.21

(C) 0.42

(D) 0.49

(E) 0.91

A43. Dr. Yeh treated the seeds of the above-mentioned homozygous blue-flower plants with
chemical mutagen to produce a mutant population. Three recessive mutants, wf1, wf2, and
wf3, produced white flowers were selected. He crossed the mutants and obtained the
29

IBO 2011
TAIWAN
THEORETICAL TEST
PART A

following results: wf1 x wf3 produced F2 offspring with only white flowers, and wf2 x wf3
produced F2 offspring with blue and white flowers in a ratio of 9:7. According to these data,
which of the statements below is NOT correct?
(A) wf1 and wf3 are unable to complement each other.
(B) wf2 and wf3 are able to complement each other.
(C) wf1 and wf3 are in the same locus.
(D) wf2 and wf3 are not in the same locus
(E) The F1 offspring from crossing wf1 and wf2 will all produce white flowers

30

IBO 2011
TAIWAN
THEORETICAL TEST
PART A

VI. Ecology
A44. Biogeography researchers found that continental islands tend to have a species composition
similar to the mainland, but a lower degree of species differentiation comparing to oceanic
islands. If one compares the biome of an oceanic island (X) to a continental island (Y),
assuming the two have approximately the same area, and are located in the same latitudinal
range, which of the following descriptions is correct?
Proportion of endemic species

Total number of species

X<Y

X>Y

X>Y

X>Y

X>Y

X<Y

X<Y

X<Y

X=Y

X<Y

A45. In the figure below, A to E denote five different species in an ecosystem. Which of the
species is most likely to be a keystone species?

31

IBO 2011
TAIWAN
THEORETICAL TEST
PART A

A46. A group of students would like to know how the discharge of waste water from a factory
might influence water quality of a river. The picture shows 7 potential sampling locations
( to ) in relation to the locations of the factory and the river. Which locations are
essential to be included in the sampling in order to draw valid conclusions about the
pollution of the river by the factory?
(A) Locations 1, 2, 4, 7
(B) Locations 1, 3, 4, 7
(C) Locations 1, 2, 5, 7
(D) Locations 2, 3, 4, 6
(E) Locations 2, 5, 6, 7

A47. Biogeography researchers have long recognized that terrestrial biomes on islands are often
associated with dispersal and colonizing ability of different organisms. Based on dispersal
and colonizing ability of the following groups of organisms, which one is least likely to
occur on an oceanic, tropical island with a large area, numerous mountains, a dense
vegetation cover, and a high level of biodiversity?
(A)

Insects

(B)

Birds
32

IBO 2011
TAIWAN
THEORETICAL TEST
PART A

(C)

Ferns

(D)

Amphibians

(E)

Reptiles

A48. A male guppy (Poecilia reticulata) with large, bright spots on the body is more likely to
attract females, which increases his opportunity to reproduce. In the meantime, he is also
more easily detected by the natural enemy, which increases his predation risk. Consider
male guppies from three different rivers: X, Y and Z, males from X have the largest spots,
males from Y have the intermediate-sized spots, and males from Z have the smallest spots.
Which of the following descriptions about the guppies in the three rivers is correct?
The density of
(A) male guppies in X is higher than in the other rivers.
(B) male guppies in Z is higher than in the other rivers.
(C) natural enemy of guppies in X is higher than in the other rivers.
(D) natural enemy of guppies in Z is higher than the other rivers.
(E) female guppies in X is higher than the other rivers.
A49. Species M had been introduced multiple times to an ecosystem outside its native
distribution, but could not establish itself. Although no parameter in ecosystem changed
between the different attempts, the final one introduction was eventually successful, and led
to a rapid and wide-spread expansion of species M in the ecosystem. Which of the
33

IBO 2011
TAIWAN
THEORETICAL TEST
PART A

followings is the most plausible explanation for why species M was not natively distributed
in this ecosystem?
(A) There are too many competitors of species M in the ecosystem.
(B) There are too many predators of species M in the ecosystem.
(C) Species M is not able to disperse to the ecosystem on it own.
(D) The abiotic environment in the ecosystem is not suitable for the growth of species M.
(E) The ecosystem is frequently under disturbance, which creates an unfavorable condition
for species M to sustain.
A50. A large forest is cleared. The land is rapidly colonized by species with which of the
following characteristics?
(1) long lifespan, (2) rapid reproduction, (3) fast growth, (4) strong dispersal ability, (5)
strong defense against natural enemies or predators.
(A) Only 1, 2, 3
(B) Only 1, 2, 5
(C) Only 1, 4, 5
(D) Only 2, 3, 4
(E) Only 3, 4, 5
A51. A large proportion of angiosperms are pollinated by animals. Assign the following flower
descriptions (I to V) to the most likely pollinator (a to e).
I. Flower white, open during night, intensive fragrant, nectar hidden in long, tight tubes.
II. Flower often with ultraviolet coloring pattern, open during daytime, pleasant fragrant.
III. Flower large and coarse, bright red, open during daytime, no fragrance but large
34

IBO 2011
TAIWAN
THEORETICAL TEST
PART A

amounts of nectar
IV. Flower large and coarse, far opened, open during night, intensive fragrant, large
amounts of nectar
V. Flower reddish brown, no nectar, smell of rotten flesh
a. bats
b. birds
c. bees
d. flies
e. moths
Which of the following statement is correct?
(A) Ia, IIb, IIIc, IVe, Vd
(B) Ib, IIc, IIId, IVa, Ve
(C) Id, IIe, IIIa, IVb, Vc
(D) Ie, IIc, IIIb, IVa, Vd
(E) Ie, IId, IIIc, IVb, Va
A52. It has been demonstrated that house roaches show less threat to human health than
mosquitos in terms of serving as disease vectors. Which feature possessed by roaches given
below may explain this observation?
(A) piercing mouthpart, injecting saliva into the tissue which it feeds upon
35

IBO 2011
TAIWAN
THEORETICAL TEST
PART A

(B) chewing mouthpart, swallowing food without saliva


(C) mouthpart sponge-like, secreting saliva upon the food they feed on
(D) microhabitats they prefer much more cleaner than those by mosquitos
(E) by natural they are anthropophobia

36

IBO 2011
TAIWAN
THEORETICAL TEST
PART A

VII. Biosystematics
[Questions 53-55] The following table shows the main characteristics of 8 different animals (taxa
1 to 8).

A + sign indicates that the animal possesses such characteristic, and a blank

indicates that the animal does not possess such characteristic:


Taxon

Characteristic
1

Amnion
Limbs with fingers

Mammary glands
Lateral line system

+
+

Cycloid scales

Sternum

Semicircle canals

+
+

+
+

Ventral nerve cords

+
+

Please answer questions A53 to 55using the information in the table above.
A53. Which of the following taxa most likely belongs to the same Class as Taxon 4?
(A) Taxon 1
(B) Taxon 2
(C) Taxon 3
(D) Taxon 5
(E) Taxon 6

37

IBO 2011
TAIWAN
THEORETICAL TEST
PART A

A54. Taxon 8 is least likely to be which of the following organisms?


(A) Earthworm
(B) Grasshopper
(C) Lobster
(D) Sea star
(E) Spider
A55. Taxon 1 is most likely to be which of the following organisms?
(A) Shark
(B) Eel
(C) Sea lion
(D) Turtle
(E) Frog
A56. Table A is a data matrix for characters of four kinds of animals. The number entries denote
shared characters if the same values (0 or 1) are given, not shared if different values (0 and 1)
are given.
Character

1 2 3 4 5 6 7 8 9 10 11 12 13 14 15 16 17 18 19

Animal A

0 0 0 0 0 0 0 0 0 0

Animal B

0 0 0 0 0 0 0 0 0 0

Animal C

0 0 0 0 0 0 0 0 1 1

Animal D

1 1 1 1 1 1 1 1 0 0

38

IBO 2011
TAIWAN
THEORETICAL TEST
PART A

If relationship among organisms can be inferred from the degree of similarity, and the degree
of similarity is defined as a coefficient S:

S = quantity of shared characters/(quantity of shared characters + different charcters)


According to the data matrix given by Table A, which animal is the most closely related to
animal A, and which one is to C? Please give your animal in the format of (the animal most
closely related to A, the animal most closely related to C).
(A) (BA)
(B) (BB)
(C) (CB)
(D) (CA)
(E) (DA)
A57. Mary bought rice, potatoes, tomatoes, kelp, pine nuts, mushrooms, dates, bird nest fern, bananas,
and corn cobs from the supermarket. Based on hierarchical classification, how many different
phyla do these items belong to?
(A) 4
(B) 5
(C) 6
(D) 7
(E) 8

39

IBO 2011
TAIWAN
THEORETICAL TEST
PART A

A58. A scientist unearthed four plant fossils (I to IV) with some prominent structures intact.
These are listed in the following table:
Structure
Spore

Ovary

Embryo

Pollen

Xylem

Ovule

Fossil #
I
II
III
IV
According to this table, which sequence below correctly represents the order of evolution of
these plants?
(A) IIIIIIIV
(B) IIIIIIVI
(C) IIIIVIII
(D) IVIIIIII
(E) IIIIVIII
(F) IIIIIVII

40















All IBO examination questions are published under the following Creative Commons license:



CC BY-NC-SA (Attribution-NonCommercial-ShareAlike) https://creativecommons.org/licenses/by-nc-sa/4.0/
The exam papers can be used freely for educational purposes as long as IBO is credited and
new creations are licensed under identical terms. No commercial use is allowed.

IBO 2011
TAIWAN
THEORETICAL TEST
PART B

Student Code:

22nd INTERNATIONAL BIOLOGY OLYMPIAD


July 10-17, 2011
Taipei, Taiwan

THEORETICAL TEST: PART B


Duration: 150 minutes

IBO 2011
TAIWAN
THEORETICAL TEST
PART B

Dear participants,

Check your Student Code on the Answer Sheet before starting the test.

The questions in Part B may have more than one correct answer. Fill your answers in the
Answer Sheet. The marks, numbers, or characters to answer questions in Part B vary
depending on questions. Mark the correct answers with and incorrect answers with
on the Answer Sheet clearly, as shown below.

No.

B0.

Write down your results and answers in the Answer Sheet. Answers written in the Question
Paper will not be evaluated.

Some of the questions may be marked DELETED. DO NOT answer these questions.

The maximal points of Part B is 120 (3 points for each question)

All answers must be correct in each question. Then you will get the points.

Stop answering and put down your pencil IMMEDIATELY after the end bell rings.

:
Good Luck!!

IBO 2011
TAIWAN
THEORETICAL TEST
PART B

I. Cell biology
Problem set: Figure 1 depicts the cross-section of a certain cell surface structure observed by
electron microscope. Answer questions 1 and 2.

B1. Which of the following possess the above structure?


(A) Paramecium
(B) Escherichia coli
(C) Tracheid of gymnosperm
(D) Sieve tube element of angiosperm
(E) Human tracheal cell
(F) Human intestinal epithelial cell
B2. What is/are the functions and what is the major chemical composition of the structure?
Function options:
(A) Attachment

IBO 2011
TAIWAN
THEORETICAL TEST
PART B

(B) Locomotion
(C) Transportation
(D) Secretion
(E) Absorption
Composition options:
(P) Cellulose
(Q) Protein
(R) Mucin
(S) Lipid
(T) Nucleic acid
B3. Some pathogens produce exotoxins that can cause human diseases. One type of exotoxins
consists of two polypeptides, subunits A and B. Subunit B can bind to surface receptors on
the target cells and cause the transport of the subunit A or associated molecules across the
plasma membrane into the cell. Once the subunit A enters the cell, it inhibits protein
synthesis and destroys the cells. Which of the following statements regarding exotoxins is/are
correct?
(A) Subunit A alone can cause disease.
(B) Subunit B alone can bind to target cells.
(C) Subunit A may carry other molecules to kill target cells.
4

IBO 2011
TAIWAN
THEORETICAL TEST
PART B

(D) Subunit B may carry other molecules and assist these molecules to enter target cells.
(E) When conjugated with an antibody against breast cancer cells, subunit A can kill breast
cancer cells.
Problem set: Some leucocytes can ingest invaded pathogens by phagocytosis. Digestive enzymes
that kill pathogens only function in acidic conditions. Please answer questions 4 and 5.
B4. Based on the information provided in the following table, complete the synthesis process of
digestive enzymes during phagocytosis:
Replication

Endoplasmic reticulum

Translation

Vesicle

Transcription

Lysosome

Mitochondrion

Golgi apparatus

(1) mRNA-ribosome complex is transferred to


(2) Synthesized enzymes enter

and

(3) The modified enzymes are stored in

to continue

for modification.

(A) a:

b:

c:

d:

e:

(B)

a:

b:

c:

d:

e:

(C) a:

b:

c:

d:

e:

(D) a:

b:

c:

d:

e:

(E) a:

b:

c:

d:

e:

IBO 2011
TAIWAN
THEORETICAL TEST
PART B

B5. Tom isolated phagocytes from a blood sample. He cultured these phagocytes in a test tube for
a period of time. To observe phagocytosis, E. coli was co-cultured with phagocytes. What
will be the consequence if an antacid is added to the culture?
(A) Phagocytes can ingest and kill E. coli.
(B) Ingestion of E. coli by phagocytes is inhibited.
(C) E. coli is viable in phagolysosome.
(D) If phagolysosomes are formed, the digestive enzymes in them are inactive.
(E) Phagocytes can secrete ingested debris out of the cells.
Problem set: Jessica is dissecting a signal transduction pathway (depicted in the following figure)
that leads to oncogenesis in cancer cells, in the hope that she can find inhibitors to block the
signaling pathway and use them as chemotherapy drugs for cancer treatment. Please answer
questions B6-B8.

IBO 2011
TAIWAN
THEORETICAL TEST
PART B

B6. Components of signal transduction, including A, B and C, usually are activated through
phosphorylation or dephosphorylation reactions. What are the mechanisms by which
proteins A , B and C are phosphorylated or dephosphorylated?
(A) Receptors may contain enzyme domains which can catalyse
phosphorylation/dephosphorylation reactions.
(B) Enzymes that participate in phosphorylation/dephosphorylation reactions may exist in
the cytoplasm.
(C) Proteins A, B and C may contain enzyme domains which can catalyse
phosphorylation/dephosphorylation reactions.
(D) Phosphorylation or dephosphorylation may not be mediated through enzymatic
reactions.
(E) A phosphate group is transferred from the receptor to protein A.
(F) The phosphate group can only be provided by H3PO4.
B7. Which of the following experiment can prove that the signal transduction pathway is BC,
but not CB?
(A) Adding an A antagonist will activate B.
(B) Adding an A agonist will activate C.
(C) Adding a B agonist will activate C.
(D) Adding a B antagonist will activate C.
7

IBO 2011
TAIWAN
THEORETICAL TEST
PART B

(E) Increasing the expression level of B will generate more active C molecules.
(F) Cell response can be observed when B antagonist and active C molecules are added to
the cell.
B8. If this is a highly activated signal transduction pathway in cancer cells, which of the
following processes does the signaling pathway involve?
(A) Inhibiting cell division
(B) Inhibiting cell differentiation
(C) Hypomethylation of some tumor suppressor genes
(D) Activating the transcription of an oncogene
(E) Arresting the cell cycle at S phase
(F) Inhibiting the expression of some DNA repair genes
B9. In protein synthesis, there are 64 codons, 61 codons specify the 20 amino acids and the other
3 for termination STOP (Table). The following sequence of amino acids occurred in the
structure of a polypeptide found in a wild-type organism:
Ser-Arg-Ile-Leu-Ala-Ala-Lys-Tyr. Which of the following may generate the mutant amino
acid sequence Ser-Arg-Ile-Trp-Arg-Gln-Lys-Tyr?

IBO 2011
TAIWAN
THEORETICAL TEST
PART B

(A) 1 nucleotide mutation


(B) 1 nucleotide insertion
(C) 1 nucleotide deletion
(D) 2 nucleotide mutation
(E) 2 nucleotide insertion
(F) 3 nucleotide mutation

IBO 2011
TAIWAN
THEORETICAL TEST
PART B

II. Plant anatomy and physiology


B10. At the time of pollination, the living pollen grain typically consists of only the tube cell and
the generative cell. During the germination of pollen grain, a pollen tube is produced and
the nucleus of generative cell divides and forms two sperms. Directed by a chemical
attractant (such as GABA) produced by the synergids, the tip of pollen tube enter the ovule
through the micropyle. Then in the embryo sac, double fertilization occurs by the two
sperms. Which of the followings are correct as concerning the pollination and double
fertilization?
(A) Tube cell, sperm, and synergid are haploid, while generative cell and zygote are
diploid.
(B) During the pollination, a gradient in GABA content is formed from the stigma (low) to
the ovary (high).
(C) The two sperms fertilize two eggs, but only one forming zygote.
(D) After fertilization, one zygote and one endosperm initial are formed.
(E) Germinated pollen grain is male gametophyte, while embryo sac is female
gametophyte.

10

IBO 2011
TAIWAN
THEORETICAL TEST
PART B

B11. Mary divided 30 pots of plant X of similar condition into 10 plants per group, with each
group being treated with different types of light regime. After a month, the flowering
phenotypes of each group are shown in the table below:
Treatment

Light regime

Flowering result

(I)

12 hr

12 hr

All 10 pots flowered

(II)

14 hr

10 hr

9 pots flowered, and 1 pot failed to flower

(III)

16 hr

8 hr

All 10 pots fail to flower

Light

Darkness

According to the information above, which of the following descriptions of plant X are
correct?
(A) Plant X is a short day plant
(B) The critical dark-length required by plant X for flowering is less than 10 hours
(C) If group III is given an one-minute dark treatment in the middle of the light period,
after one month, most plants in this group will flower
(D) If group II is given an one-minute red light treatment in the middle of the dark period,
most plants in this group will not flower right after one month
(E) If the apical buds of group I plants are removed before giving the light regime treatment,
then most plants will not produce florigen required for flowering after giving light
regime treatment.

11

IBO 2011
TAIWAN
THEORETICAL TEST
PART B

B12 and B13 are a problem set


Dr. Wang carried out experiments with the model organism Arabidopsis thatliana, and identified
the two proteins Phototropin 1 and Phototropin 2 as regulators of stomata opening. His
experimental results are depicted in the following figure, illustrating the stomata of plants during
the day.

B12. Which of the following pathways potentially depicts the relationship of Phototropin 1 and
Phototropin 2 on a molecular level?

12

IBO 2011
TAIWAN
THEORETICAL TEST
PART B

B13. Which of the following processes could be regulated and/or mediated by Phototropin 1 and
2?
(A). K+ ion efflux

(B). K+ ion influx

(C). Na+ ion influx

(D). H2O efflux

(E). H+-ATPase activity

(F). Blue light sensing

B14. Phytochromes exist in two isoforms, Pr and Pfr. In darkness, they are synthesized as Pr form,
then turned into Pfr form after absorbing red light (most effective at 666 nm). When
irradiated with far red light, Pfr transforms back to Pr. According to the description above,
which of the following are likely to be the absorption spectra of phytochrome?

(A)

(B)

(C)

(D)

(E)

13

IBO 2011
TAIWAN
THEORETICAL TEST
PART B

B15. The AGAMOUS (AG) gene is involved in flower development.

Plant mutants without a

functional AG would produce flowers with only sepals and petals. A scientist generated a
transgenic plant harboring a green fluorescence protein (GFP) gene driven by the AG
promoter in a wild type background that produces normal flowers. In which of the
following flower parts, you are likely to observe strong GFP fluorescent signals?
(A) Receptacle
(B) Sepal
(C) Petal
(D) Stamen
(E) Carpel

14

IBO 2011
TAIWAN
THEORETICAL TEST
PART B

III. Animal anatomy and physiology


B16 to B18 are a problem set
B16. In the following figure, the structure of fish gills and the direction of water flow in the
ventilation are illustrated. Answer the questions.

Which of the following statements are correct?


(A) Vessel A carries oxygenated blood
(B) Vessel B carries deoxygenated blood
(C) Vessel A is an arteriole
(D) Vessel B is a venule
(E) Vessel A and B are portal vessels

15

IBO 2011
TAIWAN
THEORETICAL TEST
PART B

B17. During evolution, the gas exchange in gills has become more effective by
(A) A decrease in the thickness of the structure C
(B) A decrease in the number of cell layers in structure C
(C) An increase in the metabolic rate of the structure C
(D) An increase in the cell volume of the structure C
(E) An increase in the surface area of the structure C

B18. Scientists found a kind of epithelial cell (X cell) in the structure of D with which fish can
maintain body fluid osmolarity. Consequently, X cells are supposed to
(A) Absorb salt actively in freshwater fish
(B) Excrete salt actively in seawater fish
(C) Excrete water actively in freshwater fish
(D) Absorb water actively in seawater fish
(E) be rich in mitochondria

16

IBO 2011
TAIWAN
THEORETICAL TEST
PART B

B19. The following image represents a gastric fold from the interior surface of the stomach. The different
structures are indicated by roman numerals:

The list below describes the function for each structure.


a. Secretes hydrochloric acid
b. Secretes mucus which lubricates and protects the cells that cover the stomach.
c. Contains a series of ridges or deep pits which lead to the glands
d. Secretes pepsinogen
e. Contains three different types of cells that secrete the components of gastric acid.
Please indicate the correct set of answers relating structure with corresponding function.
Function code

Structure code (I ~ V)

a.
b.
c.
d.
e.

17

IBO 2011
TAIWAN
THEORETICAL TEST
PART B

B20. The graph below depicts the different pulmonary volumes and capacities:

Below, you will find two charts, please correlate with each other and with the graph above:

1. Tidal volume (TV)


2. Residual volume (RV)
3. Vital capacity (VC)
4. Inspiratory capacity (IC)
5. Expiratory Reserve volume (ERV)
6. Total lung capacity (TLC)
7. Inspiratory reserve volume (IRV)
8. Functional residual capacity (FRC)

a. The maximum volume of air inhaled in a


normal inspiration. It comprises tidal volume
and inspiratory reserve.
b. The maximum amount of air inhaled over
de resting level of spontaneous inspiration.
c. The volume of air remaining in lung after a
strong forced expiration.
d. The volume of air present in lung alter a
maximum inspiration.
e. Total amount of air flowing between
inspiration and expiration at maximal rate. It
includes tidal volume, inspiratory reserve
volume and spontaneous expiration rate.
f. Amount of air in excess of tidal expiration
that can be exhaled with maximum effort.
g. The volume of air present in the lungs, at
the end of passive expiration. It is the sum of
residual volume and expiratory reserve
volume
h. The lung volume representing the normal
volume of air displaced between normal
inspiration and expiration with or without
extra effort applied.
18

IBO 2011
TAIWAN
THEORETICAL TEST
PART B

II

III

IV

VI

VII

VIII

Letter
Number

B21. Which of the following statements about thermo-adaption in animals are correct?
(A) Blue-fin tuna is able to raise their core temperature. Therefore, it is an endothermic
animal.
(B) Some icefish are able to survive in the freezing ice-laden water and maintain a very
stable body temperature. Therefore, icefish are homeothermic animals.
(C) Shivering can help mammals to generate heat, and it is regulated by hypothalamus in
mammals.
(D) Brown adipose tissues help mammals to generate heat by supplying energy to skeletal
muscles.
(E) Brown adipose cells are rich in mitochondria for heat generation.

19

IBO 2011
TAIWAN
THEORETICAL TEST
PART B

B22. Maintenance of the blood glucose level is important for normal physiological function. It is
modulated by both neural and endocrine systems. The diagram below shows two different
situations resulting from physiological stress or low blood glucose level. Complete the table
in the answer sheet by using appropriate letters shown below.
Structure/hormone

Answer

cells of the pancreatic islets


Insulin
Liver
Adrenal medulla
Cortisol

20

IBO 2011
TAIWAN
THEORETICAL TEST
PART B

B23 and B24 are a problem set


B23. Normal spermatognesis and androgen secretion are delicately regulated by hormones in
males. The occurrence of infertility in the male could be resulted from the disturbance of
hormonal regulation. The following figure partly illustrates the cross interactions among
hypothalamus, pituitary gland, and male gonads. The symbols (-) indicate negative
feedback inhibitions. As shown in the table below, some hormones, cells, or tissues are tried
to match to the terms in this figure (a to h).

21

IBO 2011
TAIWAN
THEORETICAL TEST
PART B

Structure/hormone

Answer

Sertoli cells
Anterior pituitary
Gonadotropin
releasing hormone
FSH
Inhibin
B24. The application of available and suitable hormonal therapy to the male patients with gonad
failure is very important. Consider how the following case may be improved by a hormonal
treatment.
Patient A suffered from testicular cancer and had both testis removed.
According to the figure of Q22, select the most appropriate letter to patient A.
Patient

Answer

22

IBO 2011
TAIWAN
THEORETICAL TEST
PART B

B25 and B26 are a problem set


B25. A to E in the diagram below represents the five major steps of synaptic transmission.

A.

Release of neurotransmitter.

B.

Activation of presynaptic calcium channel.

C.

Activation of postsynaptic sodium channel.

D.

Re-uptake of neurotransmitter.

E.

Degradation of neurotransmitter.

Scientists study drug effects on synaptic transmission by using electrophysiological recording.


Briefly the postsynaptic current will be recorded and used to determine the possible
mechanism which may account for the drug effects. Figure 1 is the typical tract of postsynaptic
current before drug administration. Match the figure number with the correct drug effects
below

Mechanism

Figure number

Blocking of step A
Facilitation of step B
Blocking of step C
Facilitation of step D
Blocking of step E
(I)

(IV)
23

IBO 2011
TAIWAN
THEORETICAL TEST
PART B

(II)

(V)

(III)

B26. Epilepsy is a common neurological condition. Patients suffer convulsions which result from
hyperactivity of certain cerebral areas. Symptoms can be reduced by using antiepileptic drugs. If
the receptor activated in the above figure was a chloride channel instead of a sodium channel,
which of the following mechanism(s) may form the basis for an antiepileptic drug?
24

IBO 2011
TAIWAN
THEORETICAL TEST
PART B

Mechanism

Answer

Blocking of step A
Facilitation of step B
Blocking of step C
Facilitation of step D
Blocking of step E

B27. The following diagram indicates the basic structure of a sarcomere.

Choose and fill in the appropriate answering code shown below to each statement in the
following table. The statements are about a muscle fiber undergoing an isotonic contraction.
Statement

Answer

a. D remain the same distance apart


b. A move closer to the ends of the B
c. C become shorter
d. B become wider
e. D lines move closer to the end of the B

25

IBO 2011
TAIWAN
THEORETICAL TEST
PART B

IV. Ethology
B28. There are two types of bird hatchlings: precocial and altricial. In general, precocial birds are
covered with feathers when they hatch, and can find their own food with help and
instructions from their mothers. In contrast, altricial hatchlings require feeding and caring
for from the parents. Based on this, which of the following statements are correct?
(A) Precocial hatchlings usually take longer to hatch than altricial hatchlings.
(B) Altricial hatchlings usually develop imprinting earlier than precocial hatchlings.
(C) Parents invest more in precocial hatchlings than in altricial hatchlings during nestling
period.
(D) For a group of young birds that hatch at the same time, altricial hatchlings tend to
develop the ability to fly earlier than precocial hatchlings.
(E) Parents of precocial and altricial hatchlings have the same level of investment in
reproduction during the breeding season.
B29. An entomologist found that a species of cockroach use the dramatic change of light intensity
at dusk (given as 6:00 pm) to reset its biological clock. He also found out that the circadian
(daily) rhythm is 25 hrs. If a student cages a cockroach of this species into a dark box at
6:00 pm, at what time the cockroach will be active after 12 days? Fill in the time in the
given space, then circle am or pm in the Answer Sheet.
B30. A biologist discovered that a species of moth is capable of flying in a straight line at night
because it keeps a constant angle (80) between its body axis and the direction of moon light
26

IBO 2011
TAIWAN
THEORETICAL TEST
PART B

using photoreceptors as a tool. If the moth encounters a bright light in a dark night, what
kind of flight path of the moth one will expect to see in relation with the light source?
(A)

(D)

(B)

(E)

(C)

B31. A researcher monitored 10 pairs of adult birds during the breeding season. For each of the
pairs, he recorded body lengths of the male and female, and their nesting date (Table 1).
Table 1. The body lengths of the males and females, and the nesting dates for the 10 pairs of
birds (A to J)

Female
body length
(cm)
Male body
length (cm)
Nesting
date

A
26.4

B
27.8

C
25.1

D
25.0

E
27.0

F
28.1

G
25.5

H
25.9

I
28.3

J
27.4

28.3

28.4

28.9

29.0

27.9

30.2

29.6

27.4

29.7

30.5

5/6

5/3

5/4

4/28

5/1

4/27

4/29

5/2

5/1

4/26

27

IBO 2011
TAIWAN
THEORETICAL TEST
PART B

Based on the data in Table 1, the mean body length is 26.65 cm for the females, and 28.99 cm
for the males. In comparison with the mean body length of 26.10 cm in females and 27.60 cm
in males in the total adult ( breeders + non-breeders) population (N=30) of the study area,
which of the following statements are correct?
(A) All males in this species are larger than females.
(B) Females tend to pair with males that are larger than themselves.
(C) Male body lengths do not affect female mate choices.
(D) Nesting dates are linked to male body lengths.
(E) The chance of breeding of this species is most likely affected by the body size.

28

IBO 2011
TAIWAN
THEORETICAL TEST
PART B

V. Genetics and Evolution


B32. The black, brown and white coat colors of mice are determined by the interaction of B/b and
C alleles. B and b alleles control the synthesis of black and brown pigments, respectively. In
the presence of the C allele, black and brown pigments are deposited in the fur. In a crossing
between BbCc and bbCc, which of the following statements are correct?
(A) The coat colors of parental mice are black and brown respectively.
(B) The ratio of black and brown offspring is 1:1.
(C) 3/4 of the offspring are black.
(D)1/4 of the offspring are brown.
(E) 1/4 of the offspring are white.
(F) Alleles C and B/b are co-dominant.

29

IBO 2011
TAIWAN
THEORETICAL TEST
PART B

B33. Fur of Guinea pigs can have different colors (black and white).
Hairs can be rough and smooth. Alleles Q and q are coding for
color, alleles R and r for type of hair. A number of cavias with
exactly the same genotype (parents group) are allowed to mate and the result is a big F1
offspring. Most of these have a black rough fur. A small number has white smooth fur.
Besides about the same number of offspring is white and rough, or black and smooth.
B33.1. Using the given letters, indicate the genotype of the guinea pigs in the parents
group:
B33.2.

If

1024

smooth?

.
F1

offspring

were

born,

how

many

of

them

are

black

and

B33.3. Guinea pigs often have a fur spot pattern. According to a simplified model spot pattern is
determined by one gene with two alleles: G and g. If G is present the guinea pig is spotted.
Students investigate the population Guinea pigs in a territory and find out that 84 % was
spotted. Presuming this population is in (Hardy Weinberg) equilibrium. Calculate the
frequency of G. Give your answer in one decimal.

B33.4. At one day all unspotted guinea pigs are caught and transported to another territory. What
will be the number of unspotted guinea pigs in the next generation? Give your answer as a
percentage without decimals.

30

IBO 2011
TAIWAN
THEORETICAL TEST
PART B

B34. It is believed that land plants are evolved from charophytes. Which of the following
statements support this hypothesis?
(A) Both have alternation of generation in life cycles
(B) Both contain chlorophyll a and chlorophyll b
(C) Both have peroxisomes that contain photorespiration related enzymes
(D) Both can form phragmoplasts during cytokinesis
(E) Both contain cellulosic cell walls
(F) Both have the cellulose synthase arranged on the plasma membrane in a rosette pattern

Questions B35a-B35c are a problem set


Dr. Chen was investigating the function of gene X in rice by using a mutant with a T-DNA
inserted in the exon 2 as diagrammed below. The size of the T-DNA is approximately 5 kilo base
pairs (Kbp). She used PCR analysis for genotyping of five individual plants (A, B, C, D, E ) with
primers I, II, and III as indicated in the diagram. The gel on the right shows the PCR result. The
DNA molecular size markers are shown in lane M. Lane A-E are the PCR products from leaf
samples of the plant A-E, respectively. It is known that the polymerase being used is unable to
effectively amplify DNA fragments above 5 Kbp.

31

IBO 2011
TAIWAN
THEORETICAL TEST
PART B

primer
primer

primer

Based on the above information, answer questions B34a-B34b :


B35a. Which pair of primers (I+II, I+III, or II+III) amplified the DNA band in lane B?
B35b. Which plant(s) (A, B, C, D, or E) is/are homozygous mutant(s)?
B35c. Which plant(s) (A, B, C, D, or E)

is/are the F1 offspring of homozygous mutant crossed

to the wild type?


B36. Doctor Lin isolated a rice mutant with a late flowering time phenotype. The mutant allele
responsible for this phenotype was identified by map-based cloning technique and named
LFTm. When Doctor Lin sequenced the whole LFTm gene, including the promoter region,
she could not find any difference in the nucleotide sequence as compared to the wild-type
allele. Which of the following phenomena are likely to be responsible for this observation?

(A) The mRNA level of LFT in the mutant is the same as that in the wild-type at the same
developmental stage
(B) The LFT protein found in the wild type plants cannot be detected or is lower in the mutant
32

IBO 2011
TAIWAN
THEORETICAL TEST
PART B

(C) The DNA methylation patterns on LFT are altered in the mutant
(D) The levels of histone proteins are dramatically altered in the mutant
(E) Introducing the cloned LFTm into the wild-type plant generates a transgenic plant with
late flowering phenotype

33

IBO 2011
TAIWAN
THEORETICAL TEST
PART B

VI. Ecology
B37. The population size of a dragonfly in a pond was estimated to be 50,000 during a survey.
Their sex ratio is 1:1. Each female lays approximately 400 eggs. A second survey of the
next generation revealed that the population size is still 50,000 and the sex ratio is still 1:1.
What is the average survival rate (surviving to adult stage) of the eggs?
(A) 0.2%
(B) 0.25%
(C) 0.5%
(D) 1%
(E) 5%
B38. Biologists found that the threshold temperature for development of a mosquito species is
15. They also found that the product of (1) the number of days it takes to complete
development, and (2) the difference between the temperature during development and
threshold temperature, is a constant. That is, the result of multiplying (1) and (2) is a
constant. It is known that this mosquito requires 15 days to complete development at 30,
and given that there was an unusually warm May in India this year with an average
temperature of 40, how many days it should take this mosquito to complete development
this May in India?
B39. Assuming human, birds, and fish respectively have Type I, II, and III survival curves (the
34

IBO 2011
TAIWAN
THEORETICAL TEST
PART B

vertical axis is survival rate, and horizontal axis is time), which of the following figures
most accurately describes mortality curves (obtained by replacing survival rate with
mortality rate) for these three groups of organisms (Type I, II and III)?

35

IBO 2011
TAIWAN
THEORETICAL TEST
PART B

B40. The figure below shows a nutrient cycle, and the six groups of organisms (A to F) involved
in this cycle.

Which of the following descriptions are correct?


(A) C and F are carnivores.
(B) C and E are carnivores.
(C) A and B are herbivores.
(D) D includes bacteria and fungi.
(E) F includes bacterivores and fungivores.
36

IBO 2011
TAIWAN
THEORETICAL TEST
PART B

B41. When comparing closely-related bird species, mortality of breeding individuals appears to
be higher for species in temperate regions than in tropical regions. Therefore, predation
risks to parents themselves, their young and eggs are given different priorities for species in
different regions. In an experiment where specimen of three different predators (crows,
owls, and hawks) were placed at close distances to the nests during the day in the breeding
period, which of the following responses from the parents can be expected? Note that crows
are predators of the young and eggs, owls are nocturnal predators, and hawks are diurnal
predators of adult birds.
(A) The owl specimen is more strongly avoided by the parents than the hawk specimen.
(B) With the crow specimen, the parents of tropical species reduce the frequency of
returning to the nests and feeding the young to a lesser degree than parents of temperate
species.
(C) With the hawk specimen, the parents of tropical species reduce the frequency of
returning to the nests and feeding the young to a lesser degree than parents of
temperature species.
(D) With the hawk specimen, the parents of tropical species reduce the frequency of
returning to the nests and feeding the young to a greater degree than parents of
temperature species.
(E) The degree to which the parents reduce their frequency of returning to the nests and
feeding the young when predator specimen is present is not affected by the type of
predators or the latitudes in which the species is distributed.
37

IBO 2011
TAIWAN
THEORETICAL TEST
PART B

B42. The figure below shows a scheme of the assimilation efficiency (A/I) and production
efficiency (P/A) of two groups of mammals in a meadow ecosystem.

A = energy intake in blood after digestion


F = loss of energy through excretion
I = take in of energy present in eaten organic compounds
P = production
R = loss of energy by dissimilation

Suppose I = 100 J
Which of the following descriptions are correct?

(A) Digestion of plant materials costs relatively less energy than digestion of animal
materials.
(B) Plant materials that are reabsorbed have less organic contents than animal materials that
are reabsorbed.
(C) R ranges between 40 and 60 J in carnivores.
38

IBO 2011
TAIWAN
THEORETICAL TEST
PART B

(D) R ranges between 40 and 60 J in herbivores.


(E) R is lower in Bull frogs than in mammals.
B43. Some fungi form symbiotic associations with the roots of vascular plants, which are called
mycorrhizae. In such associations, mycorrhizae help plants to absorb water, phosphate salt
and other mineral nutrients. Depending on whether the fungus colonizes the roots
extracellularly or intracellularly, mycorrhizae can be grouped into ectomycorrhizae or
arbuscular mycorrhizae. Which of the following descriptions of mycorrhizae are correct?

(A) Seedlings with mycorrhizae grow more rapidly than seedlings without mycorrhizae in
low-phosphorus soils.
(B) Hyphae of arbuscular mycorrhizae can penetrate the cortical cells of the roots and cell
membranes to form symbiotic associations intracellularly.
(C) Hyphae of ectomycorrhizae can penetrate the cortical cells of the roots, and form hyphal
sheaths around the roots.
(D) Mycorrhizae have similar functions as root hairs of plants, and therefore, plants with
mycorrhizae have less-developed root hairs.
(E) Each mycorrhiza forms symbiotic associations with the roots of specific plants.

39

IBO 2011
TAIWAN
THEORETICAL TEST
PART B

VII. Biosystematics
B44. Eggleton et al (2007) studied the phylogeny of Dictyoptera, as shown below. According to
the proposed phylogeny, determine whether the following statements are true (T) or false (F).

(A) Mantids sister group is cockroaches.


(B) Cockroaches form a paraphyletic group.
(C) Termites should be viewed as highly modified cockroaches.
(D) Mantids should be viewed as highly modified cockroaches.
(E) Termites evolved from mantids.

40

IBO 2011
TAIWAN
THEORETICAL TEST
PART B

B45a. Systematic positions of some butterflies such as satyrids (ringlets, etc.), nymphalids (frush
foots, etc), danaids (milkweed butterflies, etc.) were controversial. Some researchers
regarded them as distinct families, the others disagreed. Recent studies supported the view
to pool them into a single family. Below is a phylogeny of these butterflies reconstructed
by Freitas & Brown (2004). Answer the following questions based on this phylogeny.

Determine whether the following statements are true (T) or false (F).
(A) Danaid butterflies may still be a distinct family according to Freitas & Browns
phylogeny.
(B) Although Calinaginae butterflies resemble danaid butterflies in appearance, they should
be classified as Nymphaloid butterflies.
41

IBO 2011
TAIWAN
THEORETICAL TEST
PART B

(C) Satyrinae and Brassolinae were considered distinct families, but Apaturinae was always
placed in Nymphalidae. If we want to keep Apaturinae in Nymphalidae, both Satyrinae
and Brassolinae should be lumped into Nymphalidae.
(D) Danaid butterflies may be considered as the ancestors of Nymphaloid + Satyroid.
B45b. According to the phylogeny diagram above, fill in the blanks.
If Libytheinae is treated as the outgroup on the phylogeny, and 5 major groups (clades) are
recognized for the ingroup. The maximum number of lineages of such a major group shown
on THE PHYLOGENY is

(A) , minimum number is

42

(B)___.















All IBO examination questions are published under the following Creative Commons license:



CC BY-NC-SA (Attribution-NonCommercial-ShareAlike) https://creativecommons.org/licenses/by-nc-sa/4.0/
The exam papers can be used freely for educational purposes as long as IBO is credited and
new creations are licensed under identical terms. No commercial use is allowed.

INTERNATIONAL BIOLOGY OLYMPIAD


PRACTICAL PROBLEMS

2011, Taipei, Chinese Taipei















All IBO examination questions are published under the following Creative Commons license:



CC BY-NC-SA (Attribution-NonCommercial-ShareAlike) https://creativecommons.org/licenses/by-nc-sa/4.0/
The exam papers can be used freely for educational purposes as long as IBO is credited and
new creations are licensed under identical terms. No commercial use is allowed.

IBO 2011
TAIWAN
PRACTICAL TEST 1
BIOCHEMISTRY AND CELL BIOLOGY

Student Code:

22nd INTERNATIONAL BIOLOGY OLYMPIAD


July 10-17, 2011
Taipei, Taiwan

PRACTICAL TEST 1
BIOCHEMISTRY AND CELL BIOLOGY
Total Points: 100
Duration: 90 minutes

IBO 2011
TAIWAN
PRACTICAL TEST 1
BIOCHEMISTRY AND CELL BIOLOGY

Dear Participants,

In this test, you have been given the following 3 tasks:


Task I: Protein electrophoresis (35 points)
Task II: Protein quantification (35 points)
Task III: Protein purification (30 points)

Check your Student Code on the Answer Sheet before starting the test.

Write down your results and answers in the Answer Sheet. Answers written in the Question
Paper will not be evaluated.

Make sure that you have received all the materials listed for each task. If any of the listed items is
missing, raise your sign.

Use pen only.

You should organize your work efficiently but ensure that you complete Task II early
enough to obtain the spectrophotometer readings to answer the questions that follow.

Stop answering immediately after the end bell rings.

After test, enclose the Answer sheets, Question paper, and Data printout in the provided
envelope. Our lab assistants will collect it promptly.

NO paper or materials should be taken out of the laboratory.

Good Luck!!

IBO 2011
TAIWAN
PRACTICAL TEST 1
BIOCHEMISTRY AND CELL BIOLOGY

Shared instruments:
Camera, spectrophotometer, printer

Equipment and Materials:


Equipment:

Quantity

1 Power supply

2 Electrophoresis tank (with gel and buffer)

3 Micropipettes P20 and P200

1 each

4 80-well microcentrifuge tube rack

5 Wire test tube rack with 15-mL centrifuge tubes (6) (yellow cap)

6 4-way test tube rack

7 Plastic droppers in 15-mL centrifuge tubes

8 Micropipette tips (for P20 and P200)

1 each

9 Timer

10 96-well microplate

11 Marker pen & paper label

1 each

12 600-mL beaker for waste disposal

13 Scissors and a ruler

1 each

14 Double-sticker to attach the results

15 Student Code sticker

16 Tissue paper

17

Mini centrifuge (if you need to spin down the samples in the
microcentrifuge tubes)

IBO 2011
TAIWAN
PRACTICAL TEST 1
BIOCHEMISTRY AND CELL BIOLOGY

Materials:

Quantity

1 Loading dye (microcentrifuge tube-L) (pink tube with orange label)

2 Pre-stained protein molecular weight marker (microcentrifuge tube-M) (pink 1


tube with orange label)
3 Unknown pre-stained protein samples (microcentrifuge tubes-U1 and U2)

(pink tube with orange label)


4 CBG reagent in 50-mL centrifuge tube

5 Bovine serum albumin (BSA) concentration standard (0.5 mg/mL) in

microcentrifuge tube (green tube with yellow label)


6 Enzyme E in two microcentrifuge tubes: concentrations X and Y (green tube

with yellow label)


7 Distilled water (microcentrifuge tube-ddH2O) (green tube with yellow label)

8 Protein sample (microcentrifuge tube-C) (blue tube with blue label)

9 Anion exchange chromatography column on 15-mL centrifuge tube

10 Anionic buffers A and B (5 mL each in two separated 15-mL centrifuge

tubes) (green cap)


11 Coomassie brilliant blue G-250 (CBG) reagent 1 mL in each of six 15-mL
centrifuge tubes (A1 to A3 & B1 to B3, red cap)

IBO 2011
TAIWAN
PRACTICAL TEST 1
BIOCHEMISTRY AND CELL BIOLOGY

Task I (35 points)


Protein electrophoresis
Introduction:
Polyacrylamide gel electrophoresis (PAGE) is a common technique for protein study. It can be
used to separate different proteins based on their charges or sizes. A type of PAGE is termed
SDS-PAGE, in which the negatively charged chemical, SDS, is added before protein
electrophoresis. The amount of SDS that binds to proteins is proportional to the size of the protein
which gives each protein a similar charge-to-mass ratio and renders the intrinsic charge of the
protein insignificant, at least for this experiment. Thus, the major factor that affects the migration
of protein is the molecular weight (MW) of the protein during SDS-PAGE. The relative mobility
(Rf) of the protein can be calculated as the ratio of the distance migrated by the protein to that
migrated by the dye-front. The value of Rf is negatively proportional to the log of its molecular
weight.
In the problem set, you will perform the following experiment:
1. An electrophoresis tank has been set up for SDS-PAGE, in which a polyacrylamide gel has been
secured on an electrode assembly and electrophoresis buffer bath has been filled. There are 10
wells for sample loading on the top of the gel. To load the sample, use the P20 micropipette
with tip to withdraw a protein sample, and carefully place the tip on the top of the well. By

IBO 2011
TAIWAN
PRACTICAL TEST 1
BIOCHEMISTRY AND CELL BIOLOGY

injecting slowly the sample will sink to the bottom of the well by gravity (Figure 1).
2. If you need to practice, use the P20 micropipette with tip to withdraw 10 L of loading dye
from microcentrifuge tube L (pink tube with orange label) on rack. Load the dye into wells 1 to
3 or 7 to 10.
3. Each of the microcentrifuge tubes M, U1 and U2 (pink tube with orange label) contains 15 L
of protein molecular weight marker, unknown protein U1 and unknown protein U2, respectively.
Use micropipette P20 to withdraw 10 L solution from each tube and load the samples into
wells 4 to 6 as shown in Figure 1.
4. As soon as you finish sample loading, Lift the sign, lab assistants will connect the power cord
to power supply and set the voltage to 200 V for you. The gel will run for 25 minutes. The
timer will be set up by an assistant to countdown.
5. After finishing electrophoresis, Lift the sign, lab assistants will disassemble the electrophoresis
set-up and give back your gel. Wipe clean the surface of the gel with tissue papers and label the
gel with your Student Code sticker. Lab assistants will take the photo of your gel. Put the
photo on the answer sheet using double-sticker (5 points).

IBO 2011
TAIWAN
PRACTICAL TEST 1
BIOCHEMISTRY AND CELL BIOLOGY

Figure 1
Answer the following questions:
Q.1.1. (2 points) Figure 2 shows a photograph of a SDS-PAGE gel. The electrophoresis start point
and dye-front are indicated. Which side of the gel should be connected to the anode (+ charge) of
the power supply? Mark your answer (X) on the answer sheet.

IBO 2011
TAIWAN
PRACTICAL TEST 1
BIOCHEMISTRY AND CELL BIOLOGY

Figure 2

Q.1.2. (8 points) Based on the information provided in Figure 2, plot the molecular weight of the

five marker proteins versus their relative migration-Rf values on the graph paper provided (4

points). Use the graph to estimate the molecular weights of unknown proteins on lanes A and B (4

points). Write down your answers on the answer sheet.

Q.1.3. (5 points) A protein complex of molecular weight 246 kDa is composed of multiple

subunits bound by non-covalent interaction. Two protein bands of 57 and 33 kDa were identified

after SDS-PAGE. How many 57-kDa and 33-kDa subunits, respectively, are included in the
8

IBO 2011
TAIWAN
PRACTICAL TEST 1
BIOCHEMISTRY AND CELL BIOLOGY

protein complex? Write down your answers on the answer sheet.

Q.1.4. (5 points) The average molecular weight of amino acid residues is about 110 daltons. How

many amino acids are there in the 33-kDa protein subunit? How many nucleotides of RNA are

translated into the protein? Write down your answers on the answer sheet.

Q.1.5. (5 points) Suppose the average molecular weight of nucleotides is 330 daltons. Excluding
introns and the stop codon, what is the mass ratio of dsDNA that encodes the 33-kDa protein, to
the 33-kDa protein? Write down your answer on the answer sheet.
Q.1.6. (5 points) Suppose a protein P can bind to a protein Q (MW = 1000 daltons). The binding
can be revealed by gel-mobility shift assay. 200 pmol of protein P were mixed with various
amounts (0 to 500 ng) of protein Q. These mixtures were resolved by 10% (w/v) polyacrylamide
gel. The gel was stained by Coomassie blue and is shown in Figure 3. Calculate the binding
molar ratio of proteins P and Q. Write down your answer on the answer sheet.

IBO 2011
TAIWAN
PRACTICAL TEST 1
BIOCHEMISTRY AND CELL BIOLOGY

Figure 3

10

IBO 2011
TAIWAN
PRACTICAL TEST 1
BIOCHEMISTRY AND CELL BIOLOGY

Task II (30 Points)


Protein quantification
Introduction:
Coomassie Brilliant Blue G-250 (CBG) is a protein staining reagent. It appears a different
color under different pH conditions. It looks reddish brown in acidic solution, whereas it turns blue
under neutral or alkaline conditions. Since proteins can provide a relatively neutral environment,
CBG will turn blue with the maximum absorbance at a wavelength of 595 nm when binding to
protein. The more protein there is in a sample, the more CBG will bind to it, and thus, the higher
intensity the blue color will be. In other words, the absorbance at 595 nm is proportional to the
amount of protein in a sample. Based on this, one can determine the concentration of a protein by
measuring the blue intensity of a sample.
In the problem set, you will perform the following experiment:
1. To make BSA concentration standards (Table 1), add 0, 2, 4, 6, 8 and 10 L of 0.5 mg/mL BSA
(green color) in A1 to A6 wells of a microplate (Figure 4). Make duplicated BSA concentration
standards in B1 to B6 wells. If you make a mistake, you can repeat the procedure in wells A7 to
A12 and/or B7 to B12. Adjust the total volume of each BSA solution to 10 L by adding an
appropriate volume of H2O (Table 1).

2. Add 200 L of CBG reagent per well in A1 to A6 and B1 to B6. Mix and observe the color

11

IBO 2011
TAIWAN
PRACTICAL TEST 1
BIOCHEMISTRY AND CELL BIOLOGY

change.

3. To determine the two concentrations X and Y of enzyme E, add various amounts (2, 4, 6, 8 and
10 L) of enzyme E (green color) in duplicate to empty wells and bring up the volume to 10
L with H2O.

4. Add 200 L of CBG reagent per well to the diluted enzyme E. Mix and observe the color
change.

5. Lift the sign, lab assistants will accompany you to measure the absorbance values of your
samples at 595 nm using spectrophotometer. Put your Student Code on the print-out data
with marker pen.

6. Return to your work bench, and put the result on the answer sheet using double-sticker.
Table 1
Well of a microplate
Materials

A1 & B1

A2 & B2

A3 & B3

A4 & B4

A5 & B5

A6 & B6

0.5 mg/mL BSA (L)

10

H2O (L)

10

Diluted BSA

concentration (mg/mL)

12

IBO 2011
TAIWAN
PRACTICAL TEST 1
BIOCHEMISTRY AND CELL BIOLOGY

A1

A2

A3

A4

A5

A6

B1

B2

B3

B4

B5

B6

Figure 4

13

IBO 2011
TAIWAN
PRACTICAL TEST 1
BIOCHEMISTRY AND CELL BIOLOGY

Answer the following questions:


Q.2.1. (10 points) Calculate the concentrations of BSA in each sample (10 L) and fill in the
blanks in the table on the answer sheet (Q.2.1.1. 5 points). Use these values to plot a standard
curve of BSA concentrations (X-axis) versus mean absorbance values of duplicated standards
(Y-axis) on the answer sheet (Q.2.1.2. 5points).
Q.2.2. (12 points) For both X and Y choose the best absorbance value within the range of the
BSA standard curve to determine concentration and fill in the table on the answer sheet.
Q.2.3. (8 points) Based on the values you chose, calculate the original concentrations (X and Y)
of enzyme E from the standard curve of BSA concentration. The concentrations should be
expressed in units of mg/mL. Write down your answers on the answer sheet.

14

IBO 2011
TAIWAN
PRACTICAL TEST 1
BIOCHEMISTRY AND CELL BIOLOGY

Task III (35 points)


Protein purification
Introduction:
Column chromatography is commonly used for purification of proteins. The column is made
by packing solid porous material (stationary phase) in a column filled with buffer solution
(mobile phase). The protein solution to be separated is loaded on top of the column and allowed
to percolate into the solid matrix (stationary phase). A reservoir at the top supplies elution buffer
constantly which flows through the matrix and passes out of the column at the bottom (the
eluent). Since proteins interact with the solid matrix to different degrees, individual proteins
migrate faster or more slowly through the column depending on their properties. Therefore, one
can obtain purified proteins by collecting eluent at different times (Figure 5).
Ion-exchange chromatography can be used to separate proteins with different electric charge
at a given pH. In anion exchange chromatography, negatively charged proteins bind to the
positively charged stationary phase. The bound proteins will be eluted using a solution
containing anions to compete with proteins for the adsorption of solid matrix. In the practical,
proteins are eluted first with buffer containing a lower concentration of anion, than with buffer
containing a higher concentration of anions. Since differently-charged proteins interact with the
stationary phase with different strengths, they can be separately eluted by different

15

IBO 2011
TAIWAN
PRACTICAL TEST 1
BIOCHEMISTRY AND CELL BIOLOGY

concentrations of anionic buffers.

Figure 5

16

IBO 2011
TAIWAN
PRACTICAL TEST 1
BIOCHEMISTRY AND CELL BIOLOGY

In the problem set, you will perform the following experiment (5 points):
1. Label six 15-mL centrifuge tubes (yellow cap) a1 to a3 and b1 to b3 accordingly, with a
marker pen.
2. Take the anion chromatography column (Figure 6A), un-plug the tube and allow the solution
to be drained by gravity. Plug the tube intermediately when the liquid surface reaches the top
of the disc (Figure 6A, white arrow). Do not over-dry the gel as it may affect protein
purification.
3. Withdraw 200 L of protein solution from microcentrifuge tube C (blue tube with blue label)
using a P200 micropipette, and apply the sample to the chromatography column slowly by
touching the filled pipette tip lightly against the inside wall of the tube (Figure 6B).
4. Un-plug the column and allow the protein sample buffer to drain out, then transfer the column
to centrifuge tube a1 (yellow cap). Withdraw 3 mL of anion buffer A (blue cap) with a plastic
dropper and apply the solution to the gel by touching the pipette tip against the wall of the
tube (Figure 6C).
5. Collect ~1 mL eluent in centrifuge tubes a1 to a3 (yellow cap) sequentially. It takes about 2
to 3 minutes for each tube.
6. Allow the contents of the column to drain entirely out then transfer the column to centrifuge
tube b1 (yellow cap). Withdraw 3 mL of anion buffer B (blue cap) with a plastic dropper and

17

IBO 2011
TAIWAN
PRACTICAL TEST 1
BIOCHEMISTRY AND CELL BIOLOGY

apply the solution to gel by touching pipette tip against the wall of the tube (Figure 6C).
7. Collect ~1 mL eluent in centrifuge tubes b1 to b3 (yellow cap) sequentially. It takes about 2 to
3 minutes for each tube.
8. Withdraw 50 L of eluent from tubes a1 to a3 & b1 to b3 (yellow cap) and transfer to
centrifuge tubes A1 to A3 & B1 to B3 (red cap), respectively. Mix and observe color change.
The CBG reagent (see introduction in Task II) in tubes A1 to A3 & B1 to B3 will turn blue
when it reacts with the eluted protein.
9. After finishing all the experiments, Lift the sign, lab assistants will take a photo of your
experiment results and put a stamp mark on your answer sheet. Without a stamp mark you
will not be evaluated for Q3.1.1 AND Q3.1.2

Figure 6

18

IBO 2011
TAIWAN
PRACTICAL TEST 1
BIOCHEMISTRY AND CELL BIOLOGY

Q.3.1. (7 points) Mark the deepest color change (X) on the answer sheet (Q.3.1.1. 5 points).
Which of the following buffers (buffer A or buffer B) can be used to elute the protein? Mark your
answer (X) on the answer sheet (Q.3.1.2. 2 points).
Q.3.2. (5 points) Enzyme A is a protein whose surface is evenly distributed with electric charges.
If enzyme A can be eluted from anionic exchange chromatography by a high concentration of
anionic buffer, what is the property of enzyme A with respect to electric charge? Mark (X) the
answer on the answer sheet.
(A) High negative net charges
(B) Low negative net charges
(C) Zero net charge
(D) Low positive net charges
(E) High positive net charges

19

IBO 2011
TAIWAN
PRACTICAL TEST 1
BIOCHEMISTRY AND CELL BIOLOGY

Q.3.3. (4 points) Amino acids differ in the chemical nature of the R group (side chain). Figure 7
shows four amino acids A, B, C, and D in their prevailing ionic forms at pH 7.2, with the side
chain marked in a white box. Which of the following amino acids in Figure 7 would be present
more frequently on enzyme A? Write down your answer (X) on the answer sheet.

Figure 7
Q.3.4. (5 points) Hydrophobic interaction chromatography can be used to separate proteins
based on their hydrophobicity. To perform the chromatography, protein samples were first treated
with buffer containing a high concentration of salts such as ammonium sulfate (NH4)2SO4, which
will remove water molecules from the protein surface. This causes the hydrophobic area on the
surface of the protein to be exposed. When the salt-treated proteins are subjected to
chromatography, they will be adsorbed on the stationary phase through hydrophobic interactions.
The higher the hydrophobicity of the protein, the stronger the adsorption. As salt concentration
can affect the hydrophobic interaction between the protein and the stationary phase, different
proteins can be separately eluted by using different concentrations of salt-containing buffers. If
enzyme A is highly hydrophobic, which of the following buffers should be used to separate

20

IBO 2011
TAIWAN
PRACTICAL TEST 1
BIOCHEMISTRY AND CELL BIOLOGY

enzyme A from other proteins by chromatography? Mark (X) the answer on the answer sheet.
(A) Low-salt buffer
(B) High-salt buffer
(C) Buffer without salt
(D) Low-salt buffer first then high-salt buffer
(E) High-salt buffer first then low-salt buffer
Q.3.5. (4 points) If enzyme A is highly hydrophobic, which of the amino acids in Figure 7
would be present more frequently on enzyme A? Mark (X) the answer on the answer sheet.
Q.3.6. (5 points) Gel filtration chromatography separates proteins based on their sizes. The gel,
or stationary phase, consists of cross-linked polymer beads with engineered pores of a particular
size. Small proteins enter the pores and move slowly through a complex path. Large proteins
cannot enter the pores and so take a short path through the column, around the beads. Table 2 is
a list of gels and their fractionation ranges. Suppose both enzyme A (22 kDa) and protein B (44
kDa) are single-subunit proteins. Which gel is best suited for the task of purifying enzyme A
from a mixture containing enzyme A and protein B, using gel filtration chromatography. Mark
your answer (X) on the answer sheet.

21

IBO 2011
TAIWAN
PRACTICAL TEST 1
BIOCHEMISTRY AND CELL BIOLOGY

Table 2
Types of stationary phase

Fractionation range (MW, Da)

G-10

<700

G-15

<1500

G-25

1,000-6,000

G-50

1,500-30,000

G-75

3,000-70,000

G-100

4,000-150,000

G-150

5,000-400,000

G-200

5,000-800,000

Q.3.7. (5 points) Assume that the concentration of total proteins in the original solution is 1
mg/mL and the activity of enzyme A is 0.5 units in 1 mL protein sample. The concentration of
total proteins after purification is 0.1 mg/mL and the activity of enzyme A is 1 unit in 1 mL
protein sample. Calculate the purification factor (number of times purified) of enzyme A. Write
down your answer on the answer sheet.

22

IBO 2011
TAIWAN
PRACTICAL TEST 1 ANSWER KEY
BIOCHEMISTRY AND CELL BIOLOGY

STUDENT CODE:

22nd INTERNATIONAL BIOLOGY OLYMPIAD


10th 17th, 2011
Taipei, Taiwan

PRACTICAL TEST 1
BIOCHEMISTRY AND CELL BIOLOGY
Total Points: 100
Duration: 90 minutes

ANSWER KEY

IBO 2011
TAIWAN
PRACTICAL TEST 1 ANSWER KEY
BIOCHEMISTRY AND CELL BIOLOGY

STUDENT CODE:

Q.1.1. (2 points)

Start point

Dye-front

Anode (+ charge)

X
Q.1.2. (4+4 points)

IBO 2011
TAIWAN
PRACTICAL TEST 1 ANSWER KEY
BIOCHEMISTRY AND CELL BIOLOGY

Protein

STUDENT CODE:

Lane A

Lane B

56 (53-59)

72 (68-76)

Molecular weight
(kDa)
Q.1.3. (5 points)

Number

57 kDa

33 kDa

Q.1.4. (5 points)

Number

Amino acid

Nucleotide

300

900

Q.1.5. (5 points)

DNA

18 :

Protein

Q.1.6. (5 points)
P

IBO 2011
TAIWAN
PRACTICAL TEST 1 ANSWER KEY
BIOCHEMISTRY AND CELL BIOLOGY

STUDENT CODE:

Task I photo - protein electrophoresis (5 points)

IBO 2011
TAIWAN
PRACTICAL TEST 1 ANSWER KEY
BIOCHEMISTRY AND CELL BIOLOGY

STUDENT CODE:

Task II result sheet - Protein quantification

BSA
X
Y

Q.2.1.1. (5 points)

Well of a microplate
Materials

A1 & B1

A2 & B2

A3 & B3

A4 & B4

A5 & B5

A6 & B6

0.5 mg/mL BSA (L)

10

H2O (L)

10

0.1

0.2

0.3

0.4

0.5

Diluted BSA
concentration (mg/mL)

IBO 2011
TAIWAN
PRACTICAL TEST 1 ANSWER KEY
BIOCHEMISTRY AND CELL BIOLOGY

STUDENT CODE:

Q.2.1.2. (5 points) Standard curve for BSA

Q.2.2. (12 points)


Solution X

Solution Y

Sample volume (L)

10

10

H2O (L)

Optical density - OD595


nm

0.859

0.452

out of range

0.521 0.586 0.631 0.691

IBO 2011
TAIWAN
PRACTICAL TEST 1 ANSWER KEY
BIOCHEMISTRY AND CELL BIOLOGY

STUDENT CODE:

Q.2.3. (8 points)

Concentration of X

Concentration of Y

(mg/mL)

(mg/mL)

2.0

0.26

(1.6-2.4)

(0.21-0.31)

IBO 2011
TAIWAN
PRACTICAL TEST 1 ANSWER KEY
BIOCHEMISTRY AND CELL BIOLOGY

STUDENT CODE:

Q.3.1.1. (5 points)

Tube

A1

A2

A3

B1

Color
change

Q.3.1.2. (2 points)

Buffer

Q.3.2. (5 points)

Q.3.3. (4 points)

B2

B3

IBO 2011
TAIWAN
PRACTICAL TEST 1 ANSWER KEY
BIOCHEMISTRY AND CELL BIOLOGY

STUDENT CODE:

Q.3.4. (5 points)

Q.3.5. (4 points)

Q.3.6. (5 points)

G-10

G-15

G-25

G-50

G-75

Q.3.7. (5 points)

20

G-100

G-150

G-200

IBO 2011
TAIWAN
PRACTICAL TEST 2
ANIMAL PHYSIOLOGY AND ANATOMY

Student Code:

22nd INTERNATIONAL BIOLOGY OLYMPIAD


July 10-17, 2011
Taipei, Taiwan

PRACTICAL TEST 2
ANIMAL PHYSIOLOGY AND ANATOMY
Total Points: 100
Duration: 90 minutes

IBO 2011
TAIWAN
PRACTICAL TEST 2
ANIMAL PHYSIOLOGY AND ANATOMY

Dear Participants,

In this test, you have been given the following 2 tasks:


Task I: The observation of the sciatic nerve of American bullfrog (58 points)
Task II: The observation of tissue morphology and the match of their functionality (42 points)

Check your Student Code on the Answer Sheet before starting the test.

Write down your results and answers in the Answer Sheet. Answers written in the Question
Paper will not be evaluated.
Make sure that you have received all the materials listed for each task. If any of the listed items is

missing, raise your sign.


Use pen ONLY.
You must complete Task I first.
Stop answering and put down your pen immediately after the end bell rings.
After test, enclose both the Answer sheets and Question paper test sheets in the provided envelope.
Our Lab assistants will collect it promptly.
No paper or materials should be taken out from the laboratory.

Good Luck!!

IBO 2011
TAIWAN
PRACTICAL TEST 2
ANIMAL PHYSIOLOGY AND ANATOMY

Equipment and Materials:


For task I: The observation of the sciatic nerve of American bullfrog.
Instruments/materials

Quantity

unit

Bullfrog specimen

piece

Dissecting tray

piece

Round plastic petri dish

piece

Ringers solution (in wash bottle)

500

mL

Pin (in a glass bottle)

10

pieces

Cotton thread (in a glass bottle)

pieces

Electric stimulating device

set

Wet paper

set

Gloves

pair

Dissecting equipment : scissors (large)


scissors (small)

1
1

pair
pair

pairs

fine forceps

For task II: The observation of tissue morphology and the match of their functionality
Instruments/materials

Quantity

unit

Microscope

set

Prepared tissue sections (marked A to J)

10

slides

Coloured pictures of tissue sections (numbered 1 to 9) on 3


sheets of A4 paper

set

IBO 2011
TAIWAN
PRACTICAL TEST 2
ANIMAL PHYSIOLOGY AND ANATOMY

Task I (58 points)


The observation of the sciatic nerve of the American bullfrog.
Introduction:
The sciatic nerve is a branch of the sacral plexus. It is the thickest and longest nerve tract in the
body, extending from the vertebral column to the foot. The sciatic nerve includes the distributed
sensory and motor nerves that control most sensory and motor activities of the lower extremities.
Mediated by the sciatic nerve, sensory signals from the lower limbs are transmitted to the brain.
Similarly, muscle contraction of the lower extremities can be stimulated by nerve impulses from
the brain. The aim of this experiment is to observe and isolate the sciatic nerve from the bullfrog.
Experiment Procedure:
Step 1 to 5: (Do not allow the tissues to dry out. To keep the tissues wet, a small amount of
Ringers solution may be added onto the tissue at anytime.)
1. Carefully check if all the experiment instruments/materials are fully provided.
Raise your sign if you have any problem.
2. Put the bullfrog specimen on the dissecting tray.
3. First, carefully observe the 10 pairs of spinal nerves extending from the vertebrae of the
bullfrog. Next, locate the sciatic nerve that is formed by pairs of spinal nerves VII, VIII and
IX (as shown in Fig. 1).
4. Turn on the switch (on/off) on the electric stimulating device. The red light will be lit up
immediately, indicating that the device is functioning.
5. Simultaneously stimulate the sciatic nerve with the two electric wires that are separately
connected to the (+) and (-) electrodes of the electric stimulating device. Fig. 3 indicates the
position of the sciatic nerve emerging from the spinal cord. Observe the contracting response
of the hind limb.
Electrodes (+) (-)
Spinal Nerve I
Spinal Nerve II
Spinal Nerve III
Spinal Nerve IV
Spinal Nerve V
Spinal Nerve VI
Spinal Nerve VII

Region

Switch

Spinal Nerve VIII

OFF ON

Spinal Nerve IX

of

Spinal Nerve X

stimulation

Sciatic Nerve

Fig. 1. Spinal nerve

Red LED

Fig. 2. Electric stimulating Fig. 3. Sciatic nerve


device

IBO 2011
TAIWAN
PRACTICAL TEST 2
ANIMAL PHYSIOLOGY AND ANATOMY

Q.1.1. (9 points) When you have finished the above five steps, raise the sign to notify the Lab
assistant to videotape the contraction.
Step 6 to 10: (To keep the tissues wet, a small amount of Ringers solution may be added
onto the tissues at any time)
6. Use a pair of scissors to make a circular cut through the skin spanning the circumference of
the upper part of one thigh of the bullfrog. Starting from the cutting point, completely peel off
the skin by hand to remove it from the hind limb (Fig. 4) It may be necessary to cut some
connections between the skin and underlying tissue.
7. Lay the bullfrog on the dissecting tray with its back facing up.
8. Push two pins separately into both ends of the gastrocnemius and separate it from the
tibiofibular (shinbone) (Fig. 5).
9. The sciatic nerve is located in a trough surrounded by thigh muscles. Carefully separate the
muscles on both sides of the trough and to expose the pale yellow colored sciatic nerve.
Pass a cotton thread underneath the sciatic nerve to label it.
10. Stimulate the cotton thread-labeled sciatic nerve with the provided electric stimulating device
and observe the contracting response of the gastrocnemius.
Cotton
thread

Skin

Pin
Sciatic nerve

Gastrocnemius
Fig. 4. Demonstration of skin peeling

Fig. 5 Sciatic nerve and gastrocnemius

Q.1.2. (8 points) When you have finished steps 6 to 10, raise the sign to notify the Lab assistant
to videotape the contraction.

IBO 2011
TAIWAN
PRACTICAL TEST 2
ANIMAL PHYSIOLOGY AND ANATOMY

Step 11 to 12: (To keep the tissues wet, a small amount of Ringers solution may be added
onto the tissues at anytime)
11. Completely separate and isolate the INTACT sciatic nerve gastrocnemius muscle
preparation from the bullfrog specimen and place it in a petri dish, as shown in Fig. 6. (The
sciatic nerve must be at least 2cm long).
12. Stimulate the sciatic nerve with the electric stimulating device and observe the contraction
response of the gastrocnemius.

Tendon (A)
Gastrocnemius
Sciatic nerve-gastrocnemius connection (B)
Sciatic nerve

Student code and scale bar


Fig. 6 Isolation of sciatic nerve-gastrocnemiustissue
Q.1.3. (40 points) When you have finished steps 11-12, raise the sign to notify the lab assistant
to check your specimen integrity and to videotape the contraction.

IBO 2011
TAIWAN
PRACTICAL TEST 2
ANIMAL PHYSIOLOGY AND ANATOMY

Task II (42 points)


Identify tissues based on their morphology and match their functionality
Introduction:
The vertebrate physiological system is established by the functional coordination of 11 organ
systems, which include the skin, skeletal, muscle, nervous, endocrine, cardiovascular, lymphatic,
respiratory, digestive, urinary, and reproductive systems.
Identify the specimens on the slides (30 points)
Slides A to J are tissue sections from vertebrates. Identify the tissues or cell types, based on their
characteristic features, using the microscope.
1. Vein
6. Blood
(human)
11. Smooth
muscle
16. Pancreas

2. Artery

3. Ganglion

4. Neuron

5. Blood ( frog )
10. Skeletal
muscle

7. Ovary

8. Testis

9. Lung

12. Cardiac
muscle

13. Kidney

14. Cartilage

15. Bone

17. Intestine

18. Gastric
tissue

19. Skin

20. Rectum

Q.2.1. (30 points) Match each slide specimen (A to J) with its correct name from the 20 different
tissue/organ names listed in table above. (Note: there is ONLY one correct answer for each
specimen). Fill in the correct number in the answer sheets.
Identify the photographed tissue sections and match their correct functions (12 points)
Fig. 1-9 are enlarged pictures of tissue sections of different mammalian tissues. Based on their
structural features, identify the tissue and answer the questions below.
The functions of 11 organs are listed in the following table. Each specific function is assigned an
alphabetic letter (A to K).
Symbol Functional description
A

Producing vitamin D3

Producing erythropoietin

Producing urea

Producing surfactant to reduce surface tension

Regulating the homeostasis of the pH of body fluid

Helping vein compression and promoting blood flow back to the heart

Digesting proteins

Secreting secretin

Producing inhibin

Major organ for the storage of calcium and phosphate

Producing progesterone
7

IBO 2011
TAIWAN
PRACTICAL TEST 2
ANIMAL PHYSIOLOGY AND ANATOMY

Q.2.2. (13 points) Correctly write down the functional symbols, i.e. the alphabetic letters (A
to K), on the answer sheets. Match the organ with their functionalities. Note: some organs may
have more than one function (1 point will be deducted for each incorrect answer and the
minimum score will not be less than zero).

Fig 1.

H&E stain (400 X)

Fig 2.

H&E stain (100 X)

Fig 3.

H&E stain (200 X)

Fig 4.

H&E stain (200 X)

Fig 5.

H&E stain (200 X)

Fig 6.

H&E stain (40 X)

Fig 7.

(200 x)

Fig 8.

H&E stain (100 X)

Fig 9.

H&E stain (200 X)

IBO 2011
TAIWAN
PRACTICAL TEST 2 ANSWER KEY
ANIMAL PHYSIOLOGY AND ANATOMY

STUDENT CODE:

22nd INTERNATIONAL BIOLOGY OLYMPIAD


July 10-17, 2011
Taipei, Taiwan

PRACTICAL TEST 2
ANIMAL PHYSIOLOGY AND ANATOMY
Total Points: 100
Duration: 90 minutes

ANSWER KEY

IBO 2011
TAIWAN
PRACTICAL TEST 2 ANSWER KEY
ANIMAL PHYSIOLOGY AND ANATOMY

STUDENT CODE:

Q.1.1. (9 points) When you have finished the above five steps, raise the sign to notify
the Lab assistant to videotape the contraction.

The 1st checkpoint

Points

Leg limb muscle contraction

0 or 9

Lab assistant
signature

Q.1.2. (8 points) When you have finished the steps 6 to 10, raise the sign to notify the
Lab assistant to videotape the contraction.

The 2nd checkpoint

Points

Indicate the scietic nerve correctly


0 or 4
(nerve must be intact)

Gastrocnemius contraction

0 or 4

Lab assistant
signature

IBO 2011
TAIWAN
PRACTICAL TEST 2 ANSWER KEY
ANIMAL PHYSIOLOGY AND ANATOMY

STUDENT CODE:

Q.1.3. (40 points) When you have finished the steps 11-12, write down the results of
your observation on the answer sheets. And, raise the sign to notify the Lab assistant
for checking the results and videotape the contraction.

The 3rd checkpoint

Lab assistant
Points
signature

Isolation of sciatic nerve


0 or 15
(must be more than 2 cm)
Isolation of gastrocnemius
(must be intact; including A

0 or 15

and B parts)
Contraction of the isolated
sciatic nerve-gastrocnemius

0 or 10

tissue.

IBO 2011
TAIWAN
PRACTICAL TEST 2 ANSWER KEY
ANIMAL PHYSIOLOGY AND ANATOMY

STUDENT CODE:

Q.2.1. (30 points) Match each slide specimen (A to J) with its correct name from 20
different tissue/organ names listed in above table. (Note: only one correct answer for
each specimen). Fill in the correct number in the answer sheets.
Answers3 points each

Slide specimen
A.

B.

C.

15

D.

10

E.

14

F.

11

G.

18

H.

I.

12

J.

17

IBO 2011
TAIWAN
PRACTICAL TEST 2 ANSWER KEY
ANIMAL PHYSIOLOGY AND ANATOMY

STUDENT CODE:

Q.2.2. (13 points) Correctly write down the functional symbols, i.e. the alphabetic
letters (A to K), on the answer sheets.

Match the organ with their functionalities.

Note: some organs may have more than one function (1 point will be deducted for
each incorrect answer, minimum score will not be less than zero).

Figure

Functional symbol

1.

B,E

2.

3.

4.

D,E

5.

6.

I, K

7.

8.

G,H

9.

IBO 2011
TAIWAN
PRACTICAL TEST 3
ECOLOGY AND SYSTEMATICS

Student Code:

22nd INTERNATIONAL BIOLOGY OLYMPIAD


July 10-17, 2011
Taipei, Taiwan

PRACTICAL TEST 3
ECOLOGY AND SYSTEMATICS
Total Points: 100
Duration: 90 minutes

Dear Participants,

In this test, you have been given the following 2 tasks:


1

IBO 2011
TAIWAN
PRACTICAL TEST 3
ECOLOGY AND SYSTEMATICS

Task I: Reconstruct the phylogenetic tree for the given spiders (60 points)
Task II: Test of species association in a community (40 points)

Check your Student Code on the Answer Sheet before starting the test.
Write down your results and answers in the Answer Sheet. Answers written in the Question
Paper will not be evaluated.
Please make sure that you have received all the materials listed for each task. If any of the listed

items is missing, please raise your sign.


Use pen only. You can use a ruler and a calculator provided.
Check the condition of the spiders in the first 5 minutes. If any of the legs is missing, please
raise your sign. No replacement of the spiders is possible after 5 minutes.

Stop answering and put down your pen immediately after the end bell rings.
After test, our lab assistants will check the condition of the spiders and fill out the spider checklist at

the end of your answer sheet. Each undamaged spider in the original vial will get you one bonus
point. Please put down student code and sign after the check is done.
Enclose both the Answer Sheets and Question Paper in the provided envelope after the spider
check is finished. Our Lab Assistant will collect it promptly.

Good Luck!!

IBO 2011
TAIWAN
PRACTICAL TEST 3
ECOLOGY AND SYSTEMATICS

Equipments and Materials:


Equipment:
1 Dissecting microscope
2 Four sheets of colored pictures and one sheet of black and white

pictures:
4
1

Figures (figure 1-3 to 1-12)


Figure (figure 2-1)
3 Forceps
4
5
6
7

Petri dish
70% ethanol
Plastic dropper
1-m quadrat cardboard (represented by a small cardboard in a zip
lock bag)

Materials:
1 Four spider samples in glass vials (W, X, Y, Z)

2
1
1
1

IBO 2011
TAIWAN
PRACTICAL TEST 3
ECOLOGY AND SYSTEMATICS

TASK I: (60 points)


Reconstruct the phylogenetic tree for the given spiders

cervical

B
Figure 1-1 External morphology of spider. A. Dorsal view. B. Ventral view.
4

IBO 2011
TAIWAN
PRACTICAL TEST 3
ECOLOGY AND SYSTEMATICS

A: two transverse eye rows,


with 4 eyes on each eye row

B: two rows, with 6 eyes on


the first eye row

C: four rows, with 2 first


row eyes smallest and 6
posterior eyes arranged in
a hexagonal configuration

D: three rows, with the 4 first E: three rows, with the 2


row eyes much smaller than
anterior median eyes much
the others
larger than the others

F: three rows, with 4 eyes


on the second eye row

G: eight eyes with one diad


(a cluster of 2 eyes) and two

I: six eyes in three diads

H: six eyes in two triads

triads (a cluster of 3 eyes)

Figure 1-2 Eye arrangements (A key is provided on pages 7-9)


Legends and Abbreviations of figures 1-3 to 1-12
Figure 1-3 Book lungs. A. Two pairs. B. One pair.
Figure 1-4 Spinnerets. A. Three pairs. B. Two pairs.
Figure 1-5 Cribellum. A. Absent. B. Present.
Figure 1-6 Calamistrum on metatarsus IV. A. Absent. B. Present.
Figure 1-7
Figure 1-8
Figure 1-9
Figure 1-10

Tarsi claw. A. Three claws. B. Two claws.


Claw tufts. A. Absent. B. Present.
Base of anterior spinnerets (AS). A. Widely separated.
Grades of legs. A. Prograde. B. Laterigrade.
5

B. Close or in contact.

IBO 2011
TAIWAN
PRACTICAL TEST 3
ECOLOGY AND SYSTEMATICS

Figure 1-11 Tibia and metatarsus of legs I and II have a series of long spines interspersed with
much shorter setae. A. Absent. B. Present.
Figure 1-12 Double-rowed trichobothria on femora IV. A. Absent. B. Present.

1-1 Key to species of some common spiders


There are many living creatures in the world. For unfamiliar creatures, scientists usually
choose a suitable key, the most commonly used tool, to find out its name. A key uses
dichotomous statements (a or b) of diagnosed characters to divide a larger group of taxa into two
smaller subgroups (indicated by numbers or taxon names). Beginning with the number 1,
choose a more likely statement (a or b) for the specimen and then go to the number shown at the
end of the statement, and so on. Go through the key, until a taxon name is shown. A key for
some common spiders of the world is given below.

IBO 2011
TAIWAN
PRACTICAL TEST 3
ECOLOGY AND SYSTEMATICS

Key to species of some common spiders


1a Two pairs of book lungs (Fig. 1-3A) 2
1b One pair of book lungs (Fig. 1-3B) 3
2a Three pairs of spinnerets (Fig. 1-4A) A. aus
2b Two pairs of spinnerets (Fig. 1-4B) M. bus
3a With a cribellum in front of the spinnerets (Fig. 1-5B), and a calamistrum on
metatarsus IV (Fig. 1-6B) Z. cus
3b Without the cribellum and calamistrum (Figs. 1-5A, 1-6A) 4
4a With six eyes 5
4b With eight eyes 6
5a six eyes in three diads (Fig. 1-2I) S. dus
5b six eyes in two triads (Figs. 1-2H) P. eus
6a Tarsi with two claws (Fig. 1-7B), with or without claw tufts 7
6b Tarsi with three claws (Fig. 1-7A), never with claw tufts (Fig. 1-8A) 10
7a Eyes in three or four rows (Figs. 1-2C, D, E, F) 8
7b Eyes in two rows (Figs. 1-2A, B) 9
8a Eyes arranged in three rows in 2-4-2 conformation; with a pair of remarkably large anterior
median eyes (AMEs) (Fig. 1-2E)T. fus
8b Eyes arranged in 2-4-2 three rows (Figs. 1-2F); AMEs not as above C. gus
9a Base of both anterior spinnerets separated from each other or wide apart (Fig.
1-9A); Legs prograde (Fig. 1-10A) Z. hus
9b Bases of both anterior spinnerets in contact (Fig. 1-9B); Legs laterigrade (Fig.
1-10B) T. kus
10a Eye group hexagonal, eyes arranged in 4 rows, in a 2-2-2-2 pattern (Fig.
1-2C) O. lus
10b Eye group not hexagonal 11
11a Eyes in two rows (Figs. 1-2A, B) 12
11b Eyes in three rows (Figs. 1-2D, E, F)P. mus
12a Tibia and metatarsus of legs I and II armed with series of long spines interspersed
with much shorter setae (Fig. 1-11B) M. nus
12b Legs I and II without such spine arrangement 13
13a Femora IV with a proximal cluster of double-rowed trichobothria (Fig.
1-12B) L. ous
13b Femora IV without such trichobothria (Fig. 1-12A) N. pus
You have four spider specimens coded W to Z, respectively.

Identify some of their

characters with the aid of figures 1-1 to 1-12 and identify all spiders using the key.
7

(Caution!

IBO 2011
TAIWAN
PRACTICAL TEST 3
ECOLOGY AND SYSTEMATICS

You may take out the specimen from the vials for identification. When you do so, you should
place a spider in the petri dish with some 70% alcohol to examine its characters under the
stereomicroscope. Because the spiders body is very fragile, the best way to handle the
specimen is to gently grasp its legs with a pair of forceps to move it in or out from the vial.
Dont break the spiders body or its legs. Undamaged spiders in their original vials will get
extra bonus points. Please handle everything with care!
alcohol at all times to prevent desiccation).

Spiders should be kept in 70%

Q1.1.1 (4 points for each correct spider; 16 points total) Match each spider code with the
correct taxon name respectively in your Answer Sheet Note: each spider code can only be used
once; repeated taxa cells will not be given any points.
Q1.1.2 (0.65 points for each cell; 13 points total) If a spider has the characters listed in the
left column of the table in your Answer Sheet, indicate with a and if the character is absent,
indicate with a . (Penalty of 0.2 points for each wrong answer, minimum 0 point)
1-2 Reconstruct a phylogenetic tree for eight spiders
Data matrix 1-1 represents character entries (a to t) for a group of hypothetical organisms A
to H. Based on Data Matrix 1-1, Taxon A serves as the outgroup and the other 7 organisms (Taxa
B to H) are ingroups. Character state 0 represents the pleisiomorphy (ancestral character) and
states 1-6 are apomorphies (derived characters). - represents missing character. We may
reconstruct a cladogram (cladistic tree) by using synapomorphies (shared derived characters).
Each change represents one step of the evolutionary events (indicated by the character and its
state, e.g., e-5, t-4). The following tree (Figure 1-13) is the only resulting most parsimonious
cladogram that shows all the character changes on the tree. Numbers 1 to 15 represent 15 steps
of the tree.
Data Matrix 1-1
Taxa

Character
a

IBO 2011
TAIWAN
PRACTICAL TEST 3
ECOLOGY AND SYSTEMATICS

Figure 1-13

The most parsimonious cladogram reconstructed from data matrix 1-1.

Q1.2. (1.2 points for each cell; 18 points total) Fill in the character code and state (e.g., e-1) in
the Answer Sheet for each of the 15 steps.
1-3 Based on the cladogram (figure 1-13), answer the following questions:
Q1.3.1. (2 points) How many steps of the cladogram are there in total?
Q1.3.2. (2 points) Besides character e-1, which character is homoplasious (i.e., not an
homologous character)?
Q1.3.3. (2 points) Which of the following taxon is the sister group of taxon {C, D}?
(A) {E, F} (B) {H, B, G} (C) {F} (D) {H} (E) {B, G}
Q1.3.4. (Each correct answer will get 0.4 points, 2 points total) Mark with an X in the
True cell in the Answer Sheet if the characters given below appeared prior to the evolution of
character m-1 in the cladogram; otherwise, mark the False cell.
Character
s-1
s-2
a-1
g-1
d-1

IBO 2011
TAIWAN
PRACTICAL TEST 3
ECOLOGY AND SYSTEMATICS

Q1.3.5. (1 point for each cell; 5 points total) To what kind of grouping do the following taxa
belong? Use code I for polyphyletic, II for paraphyletic, or III for monophyletic grouping.
Taxon
{H}
{B, C, G, H}
{C, D, E, F}
{B, G, H}
{B, E, G}

10

IBO 2011
TAIWAN
PRACTICAL TEST 3
ECOLOGY AND SYSTEMATICS

TASK II: (40 points)


Test of species association in a community
The basic idea of community organization is that species tend to be associated in a
non-random manner. One way to understand their association conditions is to use a 2 2
contingency table (Table 2-1-0). If a sample contains both species x and y, it is defined as type
a. If a sample contains only species y, species x, or no species, then it is defined as type b,
c, or d respectively.
Table 2-1-0
Species x
Species y

Present

Absent

Total

Present

a+b

Absent

c+d

a+c

b+d

Total

n=a+b+c+d
Probability of obtaining species x, P(x) = (a+c)/n
Probability of obtaining species y, P(y) = (a+b)/n
Joint probability (JP): the probability of both species x and y being present,
JP = P(x) P(y)
Expected joint occurrences = n x JP
Significance level for Chi-squared statistical test (2)
Significance level () 0.05

0.01

df
1

3.841

6.635

5.991

9.210

7.815

11.345

Figure 2-1 (the figure on a separate paper) is a distribution map of two plant species, Plant-A ()
and Plant-B (), and a sympatric spider species, Spider (), in a hypothetical community. Each
square is 0.5 0.5 m2.

11

IBO 2011
TAIWAN
PRACTICAL TEST 3
ECOLOGY AND SYSTEMATICS

2-1 Association between Plant-A () and Spider (): analyzed by quadrat method.
Put a 1-m square quadrat on Figure 2-1 using the following 40 randomly assigned coordinates as
the center (i.e., 2 2 complete squares) and determine the type of each quadrat.
N-11, S-8, F-10, Q-18, O-16, K-2, L-4, M-17, M-4, H-17
X-2, K-11, T-19, M-8, P-10, G-8, B-19, M-19, S-10, O-12
J-18, D-7, B-17, I-11, B-10, G-13, V-16, C-3, F-5, R-15
L-2, Q-11, R-5, G-11, K-10, T-10, X-9, R-3, O-3, F-16
For example, a coordinate of C-3 would look like this:

Q2.1.1. (1 point each; 9 points total) Write down your results in Table 2-1-1 and complete all
the blank cells.
Answer the following in your Answer Sheet:
Q2.1.2a. (0.6 points) Calculate P (Plant-A).
Q2.1.2b. (0.6 points) Calculate P (Spider).
Q2.1.2c. (0.6 points) Calculate JP (Plant-A and Spider)
Q2.1.2d. (0.6 points) Calculate the expected joint occurrences.
Q2.1.2e. (0.6 points) Two species are more likely to be positively associated if the actual
observation of the joint occurrence is greater than the expected one, and negatively associated if
the actual observation is smaller than the expected one. What kind of association exists between
Plant-A and Spider? [Answer Code: P for positive association, N for negative association.]
2-1-3 A simple Chi-squared statistical test (2) with one degree of freedom (df = 1) is calculated
as follows:
n=a+b+c+d
n (ad-bc)2
2=

(a+b)(c+d)(a+c)(b+d)
Q2.1.3. (2 points) Based on Table 2-1-1, calculate 2 (to the fourth decimal place).
12

IBO 2011
TAIWAN
PRACTICAL TEST 3
ECOLOGY AND SYSTEMATICS

2-1-4 The strength of the association between the two species can be estimated from a coefficient
(V), defined as follows:

ad bc
(a b)(c d )(a c)(b d )

The V value varies from -1 (strongly negative association) to +1 (strongly positive


association) and it is zero when there is no association.
Q2.1.4a. (2 points) Calculate the V value from Table 2-1-1 (to the fourth decimal place).
Q2.1.4b. (2 points) According to the V value, what can be hypothesized about the strength of
the association exists between the two species? (Mark your answer with an X in the cell)
2-2 The following table shows data derived from 40 randomly placed 2-m square quadrats.
Table 2-2-1
Spider ()
Plant-A()

Present

Absent

Total

Present

14

16

30

Absent

10

Total

22

18

40

The expected joint occurrence is 16.5.


The Chi-squared statistical test (2) with one degree of freedom (df = 1) is
calculated as 2 = 3.3670.
V = -0.2901
Based on Table 2-2-1 answer the following questions:
Q2.2.1a. (2 points) According to the expected joint occurrence, what can be hypothesized about
the kind of association between Plant-A and Spider? [Answer Code: P for positive association,
N for negative association]
Q2.2.1b. (2 points) According to the V value, what can be hypothesized about the strength of the
association between the two species? (Mark your answer with X in the cell on the Answer
Sheet)
2.2.2. (6 points total) Answer the questions: Mark with an X on the Answer Sheet whether
each of the following statements are true or false.
Q2.2.2a. (2 points) Both tests of association using 1-m and 2-m square quadrats (sections 2-1
and 2-2), allowed us to reject the null hypothesis of random distribution.
13

IBO 2011
TAIWAN
PRACTICAL TEST 3
ECOLOGY AND SYSTEMATICS

Q2.2.2b. (2 points) The larger the quadrat size used, the more accurate the results.
Q2.2.2c. (2 points) Increasing the sampling efforts in the quadrat method should improve the
accuracy for the results for species association.
2-3 Association between Plant-A () and Plant-B (): analyzed by the nearest neighbor method.
Tally up the frequencies of the nearest neighbor of each plant systematically for all individuals.
Fill in the totals in the table printed in the Answer Sheet.
Q2.3.1. (0.5 points for each cell; 3 points total) Write down your results in Table 2-3-1 and
complete all the blank cells.
Q2.3.2a. (2 points) Based on Table 2-3-1, with one degree of freedom (df = 1), calculate 2
(to the fourth decimal place).
Q2.3.2b. (3 points) Are these two plant species randomly distributed, associated or segregated?
(Mark your answer with an X in the cell)
2-4 Mark with an X on the Answer Sheet whether each of the following are true or false.
(4 points total)
Q2.4.1. (2 points) The null hypothesis of the 2 test for the nearest neighbor method is that both
Plant-A and Plant-B are randomly distributed.
Q2.4.2. (2 points) Using the nearest neighbor method to test species association can avoid the
quadrat-size effect.

14

IBO 2011
TAIWAN
PRACTICAL TEST 3 FIGURE
SYSTEMATICS AND ECOLOGY

A: two pairs

B: one pair

Figure 1-3 Book lungs


A: three pairs

B: two pairs

Figure 1-4 Spinnerets


A: absent

Figure 1-5

B: present

Cribellum

IBO 2011
TAIWAN
PRACTICAL TEST 3 FIGURE
SYSTEMATICS AND ECOLOGY

A: absent

Figure 1-6

B: present

Calamistrum on metatarsus IV

A: three claws

B: two claws

Figure 1-7 Tarsi claws


A: absent

B: present

Figure 1-8 Claw tufts

IBO 2011
TAIWAN
PRACTICAL TEST 3 FIGURE
SYSTEMATICS AND ECOLOGY

A: widely separated

B: close or in contact

Figure 1-9 Base of anterior spinnerets (AS)


A : prograde

B: laterigrade

Figure 1-10 Grades of Legs


A: absent

B: present

Figure 1-11 Tibia and metatarsus of legs I and II with series of long spines interspersed
with much shorter setae.

IBO 2011
TAIWAN
PRACTICAL TEST 3 FIGURE
SYSTEMATICS AND ECOLOGY

A: absent

B: present

Figure 1-12 Double-rowed trichobothria on femora IV

IBO 2011
TAIWAN
PRACTICAL TEST 3 ANSWER KEY
ECOLOGY AND SYSTEMATICS

STUDENT CODE:

22nd INTERNATIONAL BIOLOGY OLYMPIAD


July 10-17, 2011
Taipei, Taiwan

PRACTICAL TEST 3
ECOLOGY AND SYSTEMATICS
Total Points: 100
Duration: 90 minutes

ANSWER KEY

IBO 2011
TAIWAN
PRACTICAL TEST 3 ANSWER KEY
ECOLOGY AND SYSTEMATICS

STUDENT CODE:

Q.1.1.1. (4 points for each correct spider; 16 points total)


Note: each spider code can only be used once, or the grades of these cells will not be
counted.
Taxon name

Spider code

Taxon name

A. aus

P. mus

C. gus

P. nus

L. ous

O. lus

S. dus

M. bus
N. pus

Spider code

T. fus

Y
W

T. kus
Z. cus

P. eus

Z. hus

Q.1.1.2. (0.65 points for each right answer cell; 13 points total)
(Penalty of 0.2 point for each wrong answer, minimum 0 point)

Taxon

Eyes in two rows

Tarsi with three claws

Bases of both anterior


spinnerets in contact

Calamistrum present on

metatarsus IV
Present a cluster of
double-rowed trichobothria
on femora IV

Please put all spider specimens back to the original vials.

IBO 2011
TAIWAN
PRACTICAL TEST 3 ANSWER KEY
ECOLOGY AND SYSTEMATICS

STUDENT CODE:

Up to 4 points bonus will be granted to students who keep the intact specimens.

Q1.2. (1.2 points for each cell; 18 points total)


1

a-1 or b-1

b-1 or a-1

s-2

h-1 or m-1 or
n-1

h-1 or m-1 or
n-1

10

h-1 or m-1 or
n-1

d-1

e-4

s-1

t-3

11

12

13

14

15

d-1

e-6

e-3

o-1

g-1

Q1.3.1.

Q1.3.2.

Q1.3.3.

19

d-1

(2 points for each cell)

Q1.3.4. (Each correct answer will get 0.4 points, 2 points total)

Character

True

s-1

s-2

a-1

g-1

False

IBO 2011
TAIWAN
PRACTICAL TEST 3 ANSWER KEY
ECOLOGY AND SYSTEMATICS

d-1

STUDENT CODE:

Q1.3.5. (1 point for each cell; 5 points total)


Taxon

Kind of grouping

{H}

III
II
II
III
I

{B, C, G, H}
{C, D, E, F}
{B, G, H}
{B, E, G}

Q2.1.1. (1 point each; 9 points total)


Table 2-1-1
Spider()
Plant-A()
Present
absent
Total

Present

Absent

Total

2
4
6

10
24
34

12
28
40

Q2.1.2a.

Q2.1.2b.

Q2.1.2c.

Q2.1.2d.

0.3

0.15

0.045

1.8

(0.6 points for each cell)

Q2.1.3.(2 points)

0.0373

Q2.1.2e.

IBO 2011
TAIWAN
PRACTICAL TEST 3 ANSWER KEY
ECOLOGY AND SYSTEMATICS

STUDENT CODE:

Q2.1.4a. (2 points)

0.0306

Q2.1.4b. (2 points)
Association Strong
V value

-1=V-0.6

Moderate

None

Moderate +

Strong +

-0.6V-0.2

-0.2V0.2

0.2V0.6

0.6V=1

Q2.2.1a. (2 points)

Q2.2.1b. (2 points)
Association

Strong

Moderate

None

Moderate +

Strong +

V value

-1=V-0.6

-0.6V-0.2

-0.2V0.2

0.2V0.6

0.6V=1

IBO 2011
TAIWAN
PRACTICAL TEST 3 ANSWER KEY
ECOLOGY AND SYSTEMATICS

STUDENT CODE:

Q2.2.2a. (2 points)

Q2.2.2b. (2 points)

True

Q2.2.2c. (2 points)
X

False

Q2.3.1. (0.5 points for each cell; 3 points total)


Table 2-3-1
Species

Species of nearest neighbor


Plant-A ()

Plant-B ()

Total

24
21
45

16
19
35

40
40
80

Plant-A ()
Plant-B ()
Total

Q2.3.2a. (2 points)

0.4571

Q2.3.2b. (3 points)
randomly distributed

associated
segregated

True

Q2.4.1 (2 points)

Q2.4.2 (2 points)

False

IBO 2011
TAIWAN
PRACTICAL TEST 3 ANSWER KEY
ECOLOGY AND SYSTEMATICS

STUDENT CODE:

Check list of the spider condition in their original vials


(Filled out by the LAB ASSISTANTS after test)
Taxon

Damaged
Undamaged

Signed by Inspector:

Student Code:

(Without Student Code written here, the 4 bonus points will not be awarded)

IBO 2011
TAIWAN
PRACTICAL TEST 4
PLANT ANATOMY, PHYSIOLOGY, AND GENETICS

Student Code:

22nd INTERNATIONAL BIOLOGY OLYMPIAD


July 10-17, 2011
Taipei, Taiwan

PRACTICAL TEST 4
PLANT ANATOMY, PHYSIOLOGY, AND GENETICS
Total Points: 100
Duration: 90 minutes

IBO 2011
TAIWAN
PRACTICAL TEST 4
PLANT ANATOMY, PHYSIOLOGY, AND GENETICS

Dear Participants,

In this test, you have been given the following 2 tasks:


Task I: Plant anatomy (60 points)
Task II: Plant physiology and genetics (40 points)
Check your Student Code on the Answer Sheet and Template paper before starting the
test.

Write down your results and answers in the Answer Sheet. Answers written in the Question
Paper will not be evaluated.
Make sure that you have received all the materials listed for each task. If any of the listed items is

missing, lift the sign.


Ensure that you organize the sequence of your tasks efficiently.

Stop answering immediately when the end bell rings.


After test, enclose the Answer sheets, Question paper, Data printout, and the stamped Template
paper (without slides) in the provided envelop. Our lab assistant will collect it promptly.
No paper or materials should be taken out of the laboratory.

Good Luck!!

IBO 2011
TAIWAN
PRACTICAL TEST 4
PLANT ANATOMY, PHYSIOLOGY, AND GENETICS

Task I (60 points)


Plant Anatomy
Equipment:

Quantity
20

1. Slides (in carrier box)


2.
3.
4.
5.
6.
7.
8.
9.
10.

Cover slips (in carrier box)


Compound microscope (with 4X, 10X, and 40X objective lenses)
Ocular micrometer (installed within the lens)
Single sided razor blade (in carrier box)

30
1
1
5

Petri dish (in carrier box)


Forceps (in carrier box)
Marker pen
Kimwipes
Paper towel

1
1
1
1
1

11. Waste basket

Materials:
1. Double distilled water (labeled as ddH2O in carrier box)
2. 1 M hydrochloric acid (HCl) (in carrier box)
3. Transparent nail polish (in carrier box)
4. Section slides X, Y, and Z of the root of plant K in slide box K
5. Four-compartment plastic petri dish
(Containing tissue samples from plants V, W, M, N, P, Q, R, S
in each compartment)
6. Template paper (with student code) for placing the slides
with sections you made and for documenting

Quantity
20 mL/vial
5-10 mL/vial
1 vial
1 slide each
2 petri dishes

1 sheet

IBO 2011
TAIWAN
PRACTICAL TEST 4
PLANT ANATOMY, PHYSIOLOGY, AND GENETICS

Part A: Structure of Plant Root (5 points total)


Introduction :

Figure 1

The structure of a typical plant root tip

There are three slides (X, Y, and Z), which are transverse sections (located within the circular
label on the slides) from different regions of the root of plant K. You need to determine whether
plant K is monocot or dicot to answer some questions later. Observe these sections under a
compound microscope and answer the following questions.
Q1.A.1.2 points each, 6 points total
Sections X, Y, and Z each correspond to a part of the root depicted in Figure 1. Check [X] the
correct answers on the answer sheet.
Q.1.A.2. single answer, 4 points
What is the direction of maturation of primary xylem in the root (tissues mature earlier tissues
mature later)? Check [X] the correct answers on the answer sheet.

Part B: Structure of Plant Stem6 points total


Introduction :
Carefully prepare transverse sections of proper thickness from the stem segments of plants V and
W and place them on separate slides. Add a drop of water to the sections and cover with cover
slips. Observe under the microscope, and answer the following questions. You need to determine
whether plants V and W are monocot or dicot to answer some questions later. When you finish
this part, place your slides with sections on the template paper, lift the sign and the lab
assistant will stamp in the boxes with the slides on the Template Sheet.
.
Q.1.B.3 points for each plant, points are given when all correct answers are selected; 6
points total
What are the distribution patterns of vascular bundles in the stems of plant V and W? Check [X]
the correct answers on the answer sheet.
4

IBO 2011
TAIWAN
PRACTICAL TEST 4
PLANT ANATOMY, PHYSIOLOGY, AND GENETICS

Part C: Structure of Plant Leaf14 points total


Introduction :
First identify the upper and lower epidermis of the leaves of plants M and N. Answer the
following questions. You need to determine whether plants M and N are monocot or dicot to
answer some questions later. When you finish this part, place your slides with sections on the
template paper, lift the sign and the lab assistant will stamp in the boxes with the slides on
the template sheet.
.
Q.1.C.18 points
Observation of stomata of plant M:
Prepare upper and lower epidermis strips of the leaves, either by peeling them off with bare hands
or by scraping off the unwanted tissues and leaving only the epidermis. Place these epidermal
tissues on separate slides, with the epidermal side facing upward. Add a drop of water to each
tissue sample and cover with a cover slip. Observe under the microscope and use the ocular
micrometer for measurements. The smallest unit of scale length of the micrometer is
approximately 30 m when observing under the 4X objective lens. Answer the following
questions in the answer sheet.
(a) Measurement of stomatal size on upper epidermis:
i) Under the 40X objective lens, what is the length of each smallest scale unit of the
ocular micrometer? (1 point)
ii) Measure the lengths of 3 guard cells, then average their lengths. (3 points)
(b) Measurement of stomatal density on lower epidermis:
i) Under the 40X objective lens, what is the approximate area of the field of view? (1
point)
ii) Observe 3 fields of view, and calculate the number of stomata in each. Then work
out the average stomatal density. (3 points)
Q.1.C.2 6 points, points are given when all correct answers are selected
Observation of leaf tissue of plant N:
Use the provided transparent nail polish to paint the upper and lower epidermis of leaves. When
the nail polish has dried, carefully peel off the layers and place on separate slides, with the
epidermal side facing upward. Add a drop of water on each sample, cover with cover slips, and
examine under the appropriate objective lens. Determine the stomatal distribution of the upper
and lower epidermis, and deduce the habitat of plant N. Check [X] the correct answers on the
answer sheet.
5

IBO 2011
TAIWAN
PRACTICAL TEST 4
PLANT ANATOMY, PHYSIOLOGY, AND GENETICS

Part D: Monocot or Dicot Plants (5 points)


Q.1.D ( 1 point each, 5 points total)
Determine whether plants K, V, W, M, and N are monocot or dicot. Check [X] the correct
answers on the answer sheet.

Part E: Calcium Crystals in Plant Cells20 points


Introduction :
Some plants have idioblasts that can form polygonal calcium oxalate crystals or calcium
carbonate crystals. Use the four plant materials (P, Q, R, S) to carefully prepare transverse
sections of appropriate thickness with a clean razor blade and place the sections on separate slides.
Add a drop of water to each section, and cover with cover slips. Observe under the microscope
and check for the presence of crystals. If crystals are present, locate the region of crystal
distribution in the tissue (most crystals present in or absent from the cells of vascular bundles),
carefully open the cover slip, remove the excess water around the sections, and add a few drops
of HCl. Add cover slips again and observe the samples under the microscope and deduce the
types of crystals that are present. When you finish this part, place your slides with sections on
the template paper, lift the sign and the lab assistant will stamp in the boxes with the slides
on the template sheet.
Q.1.E 6 points for each plant with the presence of crystals, points are given when all
correct answers are selected; 2 points for the plant with crystals absent; 20 points total
Using your observations, fill in the corresponding letters in the table in the answer sheet.
Plants: P, Q, R, S
Location of crystals: A (most crystals present in cells of vascular bundles)
B (crystals absent from the cells of vascular bundles)
Crystal type: C (polygonal calcium oxalate crystal); D (calcium carbonate crystal)

Documentation (5 points total)


When you finish all the parts (A to E) of Task I, double check whether all your slides have been stamped
for this task. If not, lift the sign and the lab assistant will check your Template Sheet (0.5 point for
each slide present, 5 points total).

IBO 2011
TAIWAN
PRACTICAL TEST 4
PLANT ANATOMY, PHYSIOLOGY, AND GENETICS

Task II (40 points)


Plant Physiology and Genetics
Shared Equipment
ELISA reader

Equipment:
1.
2.
3.

Quantity
1 each
1 box each

Micropipettes P200 and P1000


Micropipette tips for P200 and P1000
96-well microplate

4. 1.5 mL microcentrifuge tubes


(for preparation of standard solutions, use those labeled 0 M, 25 M
50 M, 100 M, 200 M, 400 M )
5. 80-well microcentrifuge tube rack / 4-way test tube rack
6. Vortex mixer

7. Marker pen
Materials:
1.
2.

Phosphate detection solution (labeled as Solution A)


400 M KH2PO4 solution (labeled as Solution B)

3.
4.

Double distilled water (labeled as ddH2O)


6 samples to be tested
(allotted in microcentrifuge tubes, labeled as sample
#1, #2, #3, #4, #5, & #6)

12 (6 extra
unlabeled)
1 each

1
1
Quantity
10 mL/tube
10 mL/tube
50 mL/vial

Introduction:
Phosphate is an important plant nutrient found in cell membranes, nucleic acids, and
energy compounds like ATP. When lacking phosphates, plant growth and development can be
dramatically affected. Plants can sense changes in phosphate concentration in the environment
and accordingly regulate their gene expression, changing the activity of phosphate transport
proteins on the cell membranes to maintain homeostasis of phosphate concentration within the
cells. Using the model plant Arabidopsis, scientists discovered that root cells respond to
phosphate-sufficient (Pi-sufficient; e.g. 1mM) or phosphate-deficient (Pi-deficient; e.g. 10M)
conditions as depicted below in Figures 2 and 3, respectively:

IBO 2011
TAIWAN
PRACTICAL TEST 4
PLANT ANATOMY, PHYSIOLOGY, AND GENETICS

Figure 2

Figure 3

When Arabidopsis is in Pi-sufficient conditions (Figure 2), the protein Z negatively


regulates the level of the protein T on the plasma membrane, responsible for the transport of
phosphate into the cell. This regulation avoids excessive phosphate absorption that leads to
toxicity. On the other hand, when the plant is in Pi-deficient condition (Figure 3), the
transcription factor X will enhance the expression of gene Y and thus increase the level of protein
Y. Protein Y can promote the degradation of protein Z, leading to an increase of protein T level,
and consequently a higher absorption of phosphate. In general, the phosphate level in the shoot of
a plant is proportional to the uptake efficiency of phosphate in the root.
Phosphate level in plants will be considerably affected when the expression of genes
encoding T, X, Y, and Z is perturbed by a mutation or insertion of a transgene. Therefore, plant
biologists can utilize such mutant or transgenic plants to determine the role and relationship of
these genes in the regulatory mechanism of phosphate homeostasis.
There are 6 samples in microcentrifuge tubes, which are extracts from the shoots of five
Arabidopsis lines (A to E) grown under either Pi-sufficient (1 mM) or Pi-deficient (10 M)
conditions (listed in Table 1). Arabidopsis A is wild type and B to E are from either a knockout
(KO; null mutant with complete loss of function of the gene) mutant line or an over-expression
lines of gene T, X, Y, or Z. You will be measuring the phosphate level in each sample and
determine the identities of the samples based on the principles shown in Fig. 2 and 3. Each
sample is derived from 20 seedlings of fresh weight shown in Table 1 and brought to final volume
of 10 mL with ddH2O.
8

IBO 2011
TAIWAN
PRACTICAL TEST 4
PLANT ANATOMY, PHYSIOLOGY, AND GENETICS

Table 1
Sample
No.

Plant

[Pi] in
medium

Fresh weight of
seedlings (mg)

1 mM

40.4

10 M

17.3

1 mM

28.0

1 mM

39.2

1 mM

30.6

1 mM

33.8

Use the provided equipment and solutions to measure the phosphate concentration in each sample
in accordance with the experimental procedures.
Experimental Procedures:
1. Use the 400-M KH2PO4 solution (Solution B), ddH2O, and pre-labeled 1.5-mL
microcentrifuge tubes to prepare the following concentrations of phosphate solutions for a
standard curve: 0, 25, 50, 100, 200, 400 M. Vortex the samples to mix solutions thoroughly.
For each concentration, there should be at least 0.5 mL of solution. Use the P200 micropipette
with fresh tips to transfer 0.1 mL of each standard solution into the 96-well microplate at
specified positions (as in Figure 4; make 2 replicates for each standard).
Figure 4 Positions of standards and samples in 96-well microplate
Standards
Samples
Standards
A

#1

#1

25

#2

#2

25

50

#3

#3

50

100

#4

#4

100

200

#5

#5

200

400

#6

#6

400

G
H
1

10

11

12

2. Transfer 0.1 mL of each sample into the 96-well microplate at specified positions (as in
Figure 4, make 2 replicates for each sample).
3. Add 0.1 mL of the phosphate detection solution (Solution A) into the wells that contain the
standards and the samples. Mix by gently tapping the side of the plate.

IBO 2011
TAIWAN
PRACTICAL TEST 4
PLANT ANATOMY, PHYSIOLOGY, AND GENETICS

4.

Lift the sign after you finish Step 3, and wait for lab assistants to guide and help you with
measuring the absorbance of the reaction mixtures with the ELISA reader at 820 nm.

5. The lab assistant will print out the data for you. Put your student code on the print-out.
6. Answer the following questions:
Q.2.1. (18 points total)
Calculate the mean values of the absorbance for each sample and standard. Use the graph paper
on the answer sheet to plot a standard curve. (0.5 point for each standard point correctly
plotted)
Determine the phosphate concentrations of the samples in M and the nmol phosphate per mg of
seedling fresh weight for sample # 1 to # 6. Fill in your results in the table of answer sheet. (2
points for each phosphate concentration measured, 0.5 point for each nmol/mg of phosphate
calculated)
Q.2.2. (Multiple answers, 4 points. Points given when all correct answers are selected)
For each of the following statements, determine whether they are true or false explanations for
those plants having higher phosphate content (nmol/mg) than the wild type. Check [X] the correct
answers on the answer sheet.
(A) X cannot be activated in the plant, thus leading to an increase in phosphate uptake.
(B) Loss of function of Gene Y in the plant causing an increase in phosphate uptake.
(C) Loss of function of Gene Z in the plant causing an increase in phosphate uptake.
(D) The plant harbors a transgene that over-expressed gene Y, causing the loss of inhibition
of protein T, leading to higher activity in phosphate uptake.
(E) Protein T of the plant has a defect and is unable to transport phosphate efficiently.
(F) The transcription factor X of the plant has a mutation, and is unable to bind the
promoter of gene Y.
Q.2.3. (2.5 points each, 10 points total)
Using the results from the experiment, assign the corresponding plant (B, C, D, or E) to the
correct description on the answer sheet.
Q.2.4. (Single answer, 4 points)
If a wild type plant (W), a gene X knockout mutant (X), and a gene Y knockout mutant (Y) were
all grown in the same Pi-deficient condition, what would be the phosphate level in their shoots
(rank from the lowest to highest)? Check [X] the correct answer on the answer sheet.
(A) X < W < Y
(B) Y < W < X
(C) W < X < Y
(D) W < Y < X
(E) X < Y < W
(F) Y < X < W
10

IBO 2011
TAIWAN
PRACTICAL TEST 4
PLANT ANATOMY, PHYSIOLOGY, AND GENETICS

Q.2.5. (Single answer, 4 points)


Western blot is a technique to detect protein levels with the use of specific antibodies. Which of
the following would be the most likely result of the Western blot analysis of protein Y and Z from
the total protein extracts of samples # 1 and # 2? Check [X] the correct answer on the answer
sheet.

11

BO 2011
TAIWAN
PRACTICAL TEST 4 ANSWER KEY
PLANT ANATOMY, PHYSIOLOGY, AND GENETICS

STUDENT CODE:

22nd INTERNATIONAL BIOLOGY OLYMPIAD


July 10-17, 2011
Taipei, Taiwan

PRACTICAL TEST 4
PLANT ANATOMY, PHYSIOLOGY, AND GENETICS
Total Points: 100
Duration: 90 minutes

ANSWER KEY

BO 2011
TAIWAN
PRACTICAL TEST 4 ANSWER KEY
PLANT ANATOMY, PHYSIOLOGY, AND GENETICS

STUDENT CODE:

Q1.A.1.2 points each, 6 points totalCheck [X] the correct answers


Part A

Part B

Part C

Part D

Section X

Section Y

Section Z

Q.1.A.2. Single answer, 4 pointsCheck [X] the correct answers

X
Q.1.B.3 points for each plant, points are given when all correct answers are
selected; 6 points total
Check [X] the correct answers
Distribution pattern of vascular bundles in the stem
arranged in a ring

Plant V

Plant W

scattered in ground tissue

solid vascular cylinder with star-like xylem

central core of parenchyma cells surrounded by


rings of xylem and phloem
2

BO 2011
TAIWAN
PRACTICAL TEST 4 ANSWER KEY
PLANT ANATOMY, PHYSIOLOGY, AND GENETICS

STUDENT CODE:

Q.1.C.18 points
Fill in the correct answers
(a)
i)

_m. (1 point)
m. (3 points)

ii) ___40 _m ( 10)


(b)
i)

___ 0.2

ii) ___ 150

( 0.05)
( 50)

___mm2. (1 point)
_ __(stomatal number /mm2). (3 points)

Q.1.C.2 6 points, points are given when all correct answers are selected
Check [X] the correct answers
True

Few or no stomata on the upper epidermis

Many stomata on the lower epidermis

An aquatic plant

False

BO 2011
TAIWAN
PRACTICAL TEST 4 ANSWER KEY
PLANT ANATOMY, PHYSIOLOGY, AND GENETICS

STUDENT CODE:

Q.1.D ( 1 point each, 5 points total)


Check [X] the correct answers
Plant

Monocot

Dicot

BO 2011
TAIWAN
PRACTICAL TEST 4 ANSWER KEY
PLANT ANATOMY, PHYSIOLOGY, AND GENETICS

STUDENT CODE:

Q.1.E 6 points for each plant with the presence of crystals, points are given
when all correct answers are selected; 2 points for the plant with the absence of
crystals; 20 points total
Fill in the corresponding letters

Presence of
crystals

Absence of

Plant

Location of crystals

Crystal type

(P, Q, R, or S)

(A or B)

(C or D)

crystals

Documentation of the template paper


(0.5 point for each slide present, 5 points total)

BO 2011
TAIWAN
PRACTICAL TEST 4 ANSWER KEY
PLANT ANATOMY, PHYSIOLOGY, AND GENETICS

Q.2.1. (18 points total)


(0.5 point for each standard point correctly plotted)

STUDENT CODE:

BO 2011
TAIWAN
PRACTICAL TEST 4 ANSWER KEY
PLANT ANATOMY, PHYSIOLOGY, AND GENETICS

STUDENT CODE:

(2 points for each phosphate concentration measured, 0.5 point for each nmol/mg
of phosphate calculated)
Sample Plant

Fresh weight

Average phosphate

nmol of phosphate per mg of

of seedlings

concentration of

seedling fresh weight

(mg)

extract (M)

(nmol/mg)

40.4

160 10%

39.6 10%

17.3

33 10%

19.0 10%

28.0

75 10%

26.8 10%

39.2

150 10%

38.3 10%

30.6

380 10%

124.2 10%

33.8

300 10%

88.8 10%

Q.2.2. (Multiple answers, 4 points. Points given when all correct answers are
selected)

Check [X] the correct answers


(A)

(B)

True
False

(C)

(D)

(E)

(F)

BO 2011
TAIWAN
PRACTICAL TEST 4 ANSWER KEY
PLANT ANATOMY, PHYSIOLOGY, AND GENETICS

Q.2.3. (2.5 points each, 10 points total)

STUDENT CODE:

Check [X] the correct answers


B

Knockout mutant plant of gene X

Plant with defective protein T

Transgenic plant with gene Y over-expression

(A)

(B)

(C)

(D)

Check [X] the correct answers


(E)

(F)
X

Q.2.5. (Single answer, 4 points)


(A)

(B)

(C)

(D)

Knock out mutant plant of gene Z

Q.2.4. (Single answer, 4 points)

Check [X] the correct answers


(E)

INTERNATIONAL BIOLOGY OLYMPIAD


THEORY PROBLEMS

2010, Changwon, Korea















All IBO examination questions are published under the following Creative Commons license:



CC BY-NC-SA (Attribution-NonCommercial-ShareAlike) https://creativecommons.org/licenses/by-nc-sa/4.0/
The exam papers can be used freely for educational purposes as long as IBO is credited and
new creations are licensed under identical terms. No commercial use is allowed.

ENVELOPE COVER SHEET

Student Code: __________________

The 21st INTERNATIONAL BIOLOGY OLYMPIAD


Changwon, KOREA

11th 18th July, 2010

THEORETICAL TEST: PART A


Time available: 120 minutes
GENERAL INSTRUCTIONS

1. Open the envelope after the start bell rings.


2. A set of questions and an answer sheet are in the envelope.
3. Write your 4-digit student code in every student code box.
4. Mark only one correct answer with on the Answer Sheet clearly, as shown below.
A

5. Use pencils and erasers. You may use a scale and a calculator provided.
6. Some of the questions may be crossed-out. DO NOT answer these questions.
7. Stop answering and put down your pencil IMMEDIATELY after the end bell rings.
8. At the end of the test session you should leave all papers at your table. It is not allowed to take
anything out.

IBO2010 KOREA
THEORETICAL TEST Part A

Country: _______________

Student Code:

___________

IBO2010 KOREA
THEORETICAL TEST Part A

The 21st INTERNATIONAL BIOLOGY OLYMPIAD


11th 18th July, 2010

Changwon, KOREA

THEORETICAL TEST: PART A


Time available: 120 minutes
GENERAL INSTRUCTIONS
1. Write your 4-digit student code in every student code box.
2. Mark the correct answer with in the Answer Sheet clearly, as shown below.
A

3. Use pencils and erasers. You can use a ruler and a calculator provided.
4. Some of the questions may be crossed-out. Do not answer these questions.
5. The maximal point of Part A is 51 (1 point for each question).
6. Stop answering and put down your pencil immediately after the end bell rings.
7. At the end of the test session you should leave all papers at your table. It is not allowed to take
anything out.

CELL BIOLOGY

IBO2010 KOREA
THEORETICAL TEST Part A

A1.

Select the chemical property that is shared by all types of lipids forming the plasma membrane.
A. Polar head
B. Sugar component
C. Glycerol backbone
D. Phosphate group
E.

Hydrophobic region

IBO2010 KOREA
THEORETICAL TEST Part A

A2.

The following photograph shows filamentous growth of a kind of cyanobacteria, Nostoc sp. The
bacteria form heterocysts (thick-walled cells), when nitrogen sources such as ammonia or nitrates are
deficient in the environment.

Which of the following statements describing these heterocysts is/are true?

I.

Nitrogen is fixed in the heterocyst.

II.

Photosystem I does not function in the heterocyst.

III.

Photosystem II does not function in the heterocyst.

A. Only I
B.

Only II

C.

Only I and II

D. Only I and III


E. Only II and III

IBO2010 KOREA
THEORETICAL TEST Part A

A3.

A generegulatory protein X controls cell proliferation. Protein X is found in the cytosol and has
no typical nuclear localization signal (NLS). When cells are treated with a specific growth hormone,
protein X re-localizes from the cytoplasm into the nucleus where it activates the transcription factors
involved in cell proliferation.
Recently, a protein (Y) that interacts with protein X has been identified in unstimulated cells. To
investigate the function of protein Y, a mutant lacking the gene encoding protein Y was generated.
Fractionation of cells from the wild type and mutant produced membrane (M), cytoplasmic (C), and
nuclear (N) fractions for each cell type. Proteins extracted from each fraction were separated by
SDS-PAGE and analyzed by Western blotting for the presence of proteins X and Y.

On the basis of the results shown above, which of the following statements is the most plausible
characterization of protein Y?
A. In the absence of growth hormone, protein Y associates with protein X, and the X/Y
complex is subjected to a degradation pathway.
B.

In the presence of growth hormone, protein Y interacts with protein X, and the complex
remains in the cytoplasm.

C.

Protein X interacts with protein Y in the absence of growth hormone. Upon growth
hormone treatment, protein X is released from protein Y and re-localizes to the nucleus.

D. Protein Y is a membrane-associated protein and re-localizes with protein X to the


nucleus upon the growth hormone treatment.
E. Protein Y is one of the nuclear import proteins and the growth hormone does not
induceprotein Y to translocate protein X to the nucleus.

IBO2010 KOREA
THEORETICAL TEST Part A

A4.

A GFP (green fluorescent protein) tagged form of protein P was expressed in fibroblast cells. The
subcellular distribution of protein P can be observed using fluorescence microscopy. To determine
the precise movement mechanism of protein P in the cells, fluorescence recovery after
photobleaching (FRAP) was performed. As shown below, protein P is expressed in the nucleus (ROI
1) and in the cytoplasm (ROI 2). Protein P in the ROI 1 area was photobleached using a laser beam.
Photobleaching causes an irreversible loss of flouorescence. Changes in the fluorescence intensity of
protein P in ROI 1 and ROI 2 following photobleaching are shown in the graph and figures below.

Which of the following is the best explanation for the distribution and movement of protein P?
A. P is a nuclear membrane protein.
B.

P is imported to the nucleus through a nuclear pore.

C.

P binds to the nuclear pore complex.

D. P is imported to the nucleus via vesicular trafficking through Golgi and ER.
E.

P is capable of moving from the nucleus to the cytoplasm.

IBO2010 KOREA
THEORETICAL TEST Part A

A5.

The domain structure of protein Z, which is composed of 180 amino acids, is shown in the upper
part of the figure below. Protein Z is palmitoylated at a cysteine residue (the third amino acid)
through the mechanism shown in the box.

Which of the following diagrams shows the correct topology of protein Z in the plasma membrane?

IBO2010 KOREA
THEORETICAL TEST Part A

A6.

The figure below shows the nucleotide sequence of the mouse -globin gene. The DNA nucleotide
sequence represents the coding strand, and the 3-letter abbreviations below represent the amino acid
sequence. The 79th cAp marked with the black arrow is the 5 capping site, and the 1467th pA is the
site where the poly-A tail is attached.

Which of the following statements about this gene structure is correct?


A. This gene has 3 introns and 4 exons.
B. The size of the mature mRNA, not including the poly-A tail, is about 1389 nt.
C. Transcription starts at nucleotide 132.
D. The region between the nucleotide sequence 1336 and 1467 is the 3 untranslated
region of the mRNA.
E. The promoter of this gene resides in the region up to nucleotide 131.

IBO2010 KOREA
THEORETICAL TEST Part A

A7.

Which one of the following graphs shows the relative change in the amount of mitochondrial DNA
of a cell undergoing mitosis?

IBO2010 KOREA
THEORETICAL TEST Part A

A8. DNA helicase, a key enzyme for DNA replication, separates double-stranded DNA into
single-stranded DNA. The following describes an experiment to find out the characteristics of this
enzyme.

A linear 6 kb ssDNA was annealed with a short (300 bp) complementary ssDNA that is
labeled with radioactive nucleotides (a). The annealed DNA was then treated in one of three
ways: with DNA helicase, boiling without helicase, or boiled helicase. Treated DNA
samples were electrophoresed on an agarose gel. The gel in b shows the DNA bands that
could be detected in the gel by autoradiography. (It is assumed that the ATP energy needed
for this enzyme reaction was provided during the treatment of DNA helicase).

Which of the following explanation about this experiment is correct?


A.

The band appearing in the top part of the gel is the 6.3 kb ssDNA only.

B.

The band appearing in the lower part of the gel is the labelled 300 bp DNA.

C.

If the annealed DNA is treated only with DNA helicase and the reaction is complete, the
band pattern looks like the lane 3 in b.

D.

If the annealed DNA is treated only with the boiling without helicase treatment, the band
pattern will look like lane 2 in b.

E.

If the annealed DNA is treated only with boiled helicase, the band pattern will
look like lane 1 in b.

10

IBO2010 KOREA
THEORETICAL TEST Part A

A9.

As shown in the picture below, microarray was used to find genes whose expression is regulated
when a plant is treated with the ABA hormone.

Which of the following explanations is not correct about the microarray experiment?
A. All cDNAs of the expressed mRNA from both the experimental group and the control group
hybridizes competitively with the corresponding genes on the DNA chip.
B. Genes whose expressions are induced by ABA appear red after hybridization.
C. Because we used different colored probes with each sample, we can measure the relative
amount of genes which are expressed differentially.
D. We can only know the expression profile of genes which are included on the microarray.
E. This process includes reverse transcription and hybridization.

11

IBO2010 KOREA
THEORETICAL TEST Part A

PLANT ANATOMY AND PHYSIOLOGY


A10.

Self-incompatibility (SI) in flowering plants is the most common mechanism preventing

self-pollination, which is mediated by a single S locus with multiple alleles. In gametophytic


self-incompatibility (GSI), the incompatibility of pollen is determined by the haploid pollen
genotype at the S locus. In sporophytic self-incompatibility (SSI), the incompatibility is determined
by the diploid S genotype of the parent pollen wall. The table below shows the SI type and
pollen/style S-gene genotypes of two plants crossed for fertilization. S1 and S2 alleles are codominant
in pollen wall.

Expressed genotype
SI type

Pollen of plant 1

Style of plant 2

GSI

S1 or S2

S2S3

II

GSI

S2 or S3

S2S3

III

SSI

S1 or S2

S1S3

IV

SSI

S1 or S2

S3S4

Which of these crosses (I, II, III, and IV) result in successful fertilizations?
A. I and II
B. I and III
C. I and IV
D. II and III

E. II and IV

12

IBO2010 KOREA
THEORETICAL TEST Part A

A11. Phytochrome is one of the plant photoreceptors involved in photoperiodism. It exists in


two spectophotometrically different forms: red-light absorbing Pr and far-red light
absorbing Pfr. An investigation explored how plant flowering was affected by different
light flashes [white (W), red (R), or far-red (FR) light] applied during the dark period or
darkness in the light-period of plant growth. The figure below shows the experimental
results.

Based on this experiment, find the most accurate explanation or expectation for the light control of
flowering in this plant,
A. This plant flowers whenever the total night length exceeds a 12 hr threshold (out of the 24 hr
night/light period) with or without light interruption.
B. This plant is likely to be a short-day plant that requires a certain length of uninterrupted light
period for flowering.
C. The plant in experiment 3 will flower if it is irradiated with a flash of far-red light, instead of
white..
D. The plant in experiment 4 will flower.
E. The plant in experiment 5 will not flower.

13

IBO2010 KOREA
THEORETICAL TEST Part A

A12. Which statement correctly describes the differentiation and development of cells and organs in
flowering plants?
A. Organomorphogenesis involves cell movement as one of the important mechanisms.
B. Post-embryogenesis is a growth process, as all of the plant organs are pre-formed during
embryogenesis.
C. Totipotency of plant tissues provides the original source of power to develop a complete plant
by re-differentiation, without going through the de-differentiation process.
D. The direction of cell division determines cell type and function.
E. Lineage information obtained by genetic inheritance overides environmental factors in
determining the time for organ development.

14

IBO2010 KOREA
THEORETICAL TEST Part A

A13. The graphs below show sucrose (Suc)- and/or indole 3-acetic acid (IAA, an auxin)-induced cell
growth (Figure a) and the kinetics of IAA-induced cell elongation and cell wall acidification in
coleoptiles (Figure b). Based on these results, together with the fact that these processes are
delayed by cold treatments or inhibitors of protein synthesis, the "acid-growth hypothesis" was
proposed as the best model to explain auxin-induced cell growth.

Which of the following statements is most accurate?


A. IAA-driven protons, pumped into the cell wall, are utilized to synthesize the ATP required
for cell elongation.
B. IAA-induced acidification of the cell wall is an ATP-dependent process, and can be delayed
by a treatment of a metabolic inhibitor.
C. IAA-induced loosening of the cell wall is mainly caused by an acidification-induced
weakening of the covalent bonds in cell wall proteins.
D. IAA- or sucrose-induced cell elongation shares a common action mechanism, such as an
increase in the cell wall acidity and the following change in turgor pressure.
E. Cell wall acidification and stimulation in the elongation is an IAA-specific process, thus it is
not induced by treatment with Fusicoccoin, an activator of the proton pump, in the absence
of the IAA.

15

IBO2010 KOREA
THEORETICAL TEST Part A

A14.

Rubisco is an enzyme crucial for carbon fixation in plants. In addition to the predominant

carboxylation reaction, this enzyme catalyzes an oxidation reaction as well. For an aquatic plant, the
frequency of the oxidation reaction depends on the relative concentrations of the reagents CO 2 and
O2 in the aquatic solution, which in turn are coupled to temperature. The figures show the absolute
(a) and relative (b) concentrations of CO2 and O2 dissolved in water that is at equilibrium with the
atmosphere.

Choose the following statement that is correct.


A. The frequency of the oxidation reaction decreases with increasing temperature.
B. In water at equilibrium with the atmosphere, the relative concentration change with
temperature of CO2 is larger than of O2.
C. Rubisco has a higher affinity for O2 than for CO2.
D. At a temperature of 90C, Rubisco catalyzes only one of the above two reactions in vascular
plants.
E. This sensitivity to temperature matters for submerged aquatic plants only.

16

IBO2010 KOREA
THEORETICAL TEST Part A

A15. As depicted in the following figure, an oat seedling was germinated in the dark. A blue light was
given unilaterallyto the right side of the coleoptile, and an agar block containing Ca 2+ was attached to
the right side of root tip below the elongation zone.

What do you expect the bending responses of the oat seedling will be in a few days?
Coleoptile

Root

Bending towards the light.

Bending towards the Ca2+ block.

Growing upright.

Bending towards the Ca2+ block.

Bending away from the light.

Bending towards the Ca2+ block.

Bending towards the light.

Growing downwards.

Growing upright.

Bending away from the Ca2+ block.

17

IBO2010 KOREA
THEORETICAL TEST Part A

ANIMAL ANATOMY AND PHYSIOLOGY


A16. As shown in the left-hand picture below, neuron (N) receives signals directly from two separate
nerve terminals (a and c). Nerve terminal (b) is synaptically connected to nerve terminal (a). The
right-hand graph shows various postsynaptic potentials recorded in neuron (N) caused by input
signals from the three presynaptic terminals.

Which of the following statements about the signal transmissions of these synapses are correct?

I.

Action potentials would be generated in neuron (N) if nerve terminals (a) and (c)
were stimulated simultaneously.

II. The neurotransmitter released from nerve terminal (b) is inhibitory.


III. When nerve terminal (b) is stimulated alone, an inhibitory postsynaptic potential
(IPSP) would be recorded in neuron (N).
IV. When nerve terminals (b) and (c) are stimulated simultaneously, the excitatory
postsynaptic potential (EPSP) recorded in neuron (N) is smaller compared to when
only nerve terminal (c) is stimulated.

A. Only I and II

B.

Only I and IV

D. Only III and IV

E.

I, II and III

18

C. Only II and III

IBO2010 KOREA
THEORETICAL TEST Part A

19

IBO2010 KOREA
THEORETICAL TEST Part A

A17. Animal cap cells from the animal pole were removed from a Xenopus blastula embryo. These
cells were then incubated in culture media containing different concentrations of activin. As seen in
the table below, the cells differentiated into various tissues or cells depending on the concentration of
activin.
Tissues
Concentration of
or cells
activin in medium
differentiated
0 (control)

epithelial cells

~ 0.1 ng/mL

blood cells

~ 1 ng/mL

muscles

~ 10 ng/mL

notochord

~ 100 ng/mL

heart

Which of the following statement(s) regarding this experiment is/are correct?

I. Ectodermal tissues are induced to differentiate into endodermal tissues


according to the level of activin concentration.
II. The fate of the animal cap cells was determined prior to the blastula stage.
III. Initially animal cap cells differentiate into epithelial tissue.
IV. Cells from the vegetal pole are also able to differentiate into muscle or heart
tissues if exposed to high activin concentrations.

A. Only I

B. Only III

C. Only I and III

D. Only II and IV

E. II, III, and IV

20

IBO2010 KOREA
THEORETICAL TEST Part A

A18. This illustration shows the molecular mechanism of the signal transduction pathway that occurs
in rod cell membranes when rod cells receive light.

Which of the following statements are correct?


When rod cells receive light, retinal molecules are converted to their active form, and protein
(a) is activated.
Component (b) is a G-protein which activates enzyme (c).
Component (c) is an adenylyl cyclase which increases the intracellular concentration of
cAMP when activated.
Component (d) is a Na+ channel that causes the membrane to depolarize when the rod cell
receives light.

Only I and II
Only I and III
Only III and IV
I, II, and IV
II, III, and IV

21

IBO2010 KOREA
THEORETICAL TEST Part A

A19. The figure shows muscle fibers, muscle spindle, and their nerve innervations of biceps of human
arm.

a: afferent nerves innervating muscle fibers of the spindle


b: efferent nerve innervating muscle fibers outside of spindle
c: efferent nerve innervating muscle fibers of the spindle
d: muscle spindle
e: nerve endings of (a)
f: muscle fibers outside of spindle

Nerve (a) is sensitive to the stretch of muscle fibers outside of the spindle when muscle fibers within
the spindle are relaxed. Choose a case when the afferent signals in nerve (a) increase?

A. Signals in (b) are increased.


B. Signals in (c) are decreased.
C. Triceps are contracted.
D. (f) are contracted.
E. The lenght of (d) remained constant.

22

IBO2010 KOREA
THEORETICAL TEST Part A

A20. The following experiments are designed to investigate the differentiation mechanism of skeletal
muscle.
<Experiment 1> Cultured mouse muscle cells were chemically induced to fuse with undifferentiated
human cells.
Result 1:

Many of the fused cells had human muscle-specific proteins.

Result 2:

Unfused cells had no human muscle-specific proteins.

<Experiment 2> Cytoplasmic portions of human muscle cells were injected into undifferentiated
mouse stem cell.
Result: The cells injected with the cytoplasm of human muscle cells transiently
expressed mouse muscle-specific genes. However, the expression of muscle
specific gene was disappeared after 24 hours.

What do these experiments suggest?


A. The nucleus of muscle cell should be fused with human cell nucleus to induce human
muscle-specific proteins.
B. The expression of muscle specific gene in human undifferentiated cell is suppressed by
cytoplasmic factor.
C. The continuous production of cytoplasmic factor(s) is indispensible for maintaining the
differentiation state of the muscle cell.
D. The cytoplasm of muscle cell induced a mutation of DNA to differentiate into muscle cell.
E. The induction of muscle differentiation is a species-specific phenomenon.

23

IBO2010 KOREA
THEORETICAL TEST Part A

A21. The following figures indicate changes in phosphate concentration as the filtrate passes through
regions a and b, according to the increase in plasma phosphate concentration.

Using this information, choose the most appropriate graph that depicts the changes in the renal
reabsorption rate of phosphate ions according to the increase of its concentration in the plasma.

24

IBO2010 KOREA
THEORETICAL TEST Part A

A22. The table below shows the results of experimental tests on skin graft rejection between two
different mouse strains. (Strains [A] and [B] are genetically identical except for the MHC loci.)

Skin donor
mouse

Exp.

Skin recipient
mouse

Skin rejection
6~8 d

10~13 d

[A]

[A]

did not occur

did not occur

II

[A]

[B]

did not occur

occurred weakly

[B] mouse which had


previously received
strain [A] skin

occurred strongly

[B] mouse which has


received lymphocytes from
a strain-[A]-skin-grafted
[B] mouse

occurred strongly

III

[A]

IV

[A]

Which of the following explanations for the results is not correct?


A. Graft rejection is considered to be the result of immune responses.
B. MHC genes are mainly responsible for the graft rejection.
C. If strain [B] skin is grafted onto mouse [A], the result would be the same as the result of Exp.
II.
D. If strain [A] skin is grafted onto an offspring from a mating between [A] and [B] mice (e.g.
F1, [A] x [B], the result would be the same as that of Exp. III.
E. The result observed in Exp. III is due to the formation of memory cells in the
previously-exposed [B] mouse against strain [A] MHC antigens.

25

IBO2010 KOREA
THEORETICAL TEST Part A

A23. The phagocytic activity of macrophages against a certain pathogenic bacteria is described below.

Which of the following conclusions best explains the results above?


A. Non-specific immunity enhances acquired immunity.
B. Humoral immunity enhances acquired immunity.
C. Humoral immunity enhances non-specific immunity.
D. Cell-mediated immunity enhances humoral immunity.
E. Cell-mediated immunity enhances non-specific immunity.

26

IBO2010 KOREA
THEORETICAL TEST Part A

A24. Oskar, nanos and hunchback are three major genes that establish the anterior-posterior (A-P) axis
during Drosophila embryogenesis. The diagram below shows how the mRNA and protein products
of these three genes are distributed in Drosophila eggs (darker shades represent higher
concentrations). The table below lists the outcome of disrupting one of these genes, as reflected in
protein distribution and anterior-posterior development.

Mutation in
nanos

Mutation in
hunchback

Mutation in oskar

Distribution of
the Nanos protein

normal

abnormal

Distribution of
the Hunchback protein

abnormal

abnormal

Distribution of
the Oskar protein

normal

normal

Establishment of normal A-P


polarity

abnormal

abnormal

abnormal

27

IBO2010 KOREA
THEORETICAL TEST Part A

Based on these data, which of the given statements correctly describes the interaction among the
three genes?

A. The transcription of hunchback gene is suppressed by the Nanos protein.


B. The Oskar protein activates the translation of nanos mRNA.
C. The Hunchback protein suppresses the translation of oskar mRNA in the anterior.
D. The Oskar protein suppresses the transcription of hunchback gene in the posterior.
E. The Hunchback protein suppresses the transcription of nanos gene in the anterior.

28

IBO2010 KOREA
THEORETICAL TEST Part A

A25. The diagram on the left depicts the different areas of the spinal cord from the cervical region to
the coccyx. The statements on the right provide descriptions about the spinal cord.

1. Cutaneous sensory information


from the skin ascends through the
corresponding side of the spinal
cord.
2. Pain information from the skin only
ascends through the contralateral
side of the spinal cord.
3. Motor neurons of the spinal cord
cause muscle contraction on the
corresponding side of the body.
4. Cervical nerves innervate the upper
limb.

Suppose that an athlete injures the left half of the spinal cord T4 during a football game.
Choose the correct statement concerning this patient's sensory or motor function.
A. Abnormal touch sensation in the right foot.
B. Disability in the movement of the right leg.
C. Normal pain perception in the left leg.
D. No cutaneous sensation in the left hand.
E. Normal pain perception in the right leg.

29

IBO2010 KOREA
THEORETICAL TEST Part A

A26. This picture illustrates monthly changes in the human ovary during the reproductive cycle.

Which of the following statements most accurately describes each structure?


A. Before puberty, the oocyte (a) has not started the process of meiosis.
B. The hormone produced by structure (b) causes thinning of the uterine cervical mucus to allow
passage of sperm.
C. During ovulation, structure (c) stays at the interphase between meiosis I and meiosis II.
D. The hormone produced by structure (d) stimulates the pituitary gland to secrete luteinizing
hormone.
E. The hormone produced by structure (e) causes the proliferation of the uterine endometrium.

30

IBO2010 KOREA
THEORETICAL TEST Part A

A27. The diagram below represents the development of a human zygote from fertilization to the late
blastocyst stage.

Choose a correct statement from the following choices.


A. If two sperm penetrate the oocyte membrane at the time of fertilization, conjoined twins with
shared body parts will be born.
B. During the process of in vitro fertilization with embryo transfer (IVF-ET), the embryo is
transferred at the 2-cell stage to the mother's uterus.
C. The

most

appropriate

stage

for

the

collection

of

Embryonic

stem

cells

regenerative-therapeutic purposes is the 8-cell stage.


D. The outer cells (structure a) of the early-blastocyst embryo will eventually form the fetus.
E. During the late blastocyst stage, the embryo is implanted in the uterine endometrium.

31

for

IBO2010 KOREA
THEORETICAL TEST Part A

A28. Figure I illustrates a skeletal muscle and its innervation. Figures II and III depict cross and
longitudinal sections of the muscle, respectively. Figure IV shows an electron micrograph of
neuromuscular junction.

Which of the following statements gives the most accurate description of each structure?
A. The number of muscle cells innervated by a single motor neuron is larger in a muscle that
controls fine movement than in one that controls unskilled movement.
B. During embryonic development, structure (a) is derived from a single cell.
C. Within the same muscle, the population of small-diameter cells (b) is increased after several
weeks of intense exercise.
D. Structure (c) is called a myofibril, which is the structural unit of the skeletal muscle.
E. The main mechanism to terminate the action of secreted acetylcholine at the neuromuscular
junction is neurotransmitter reuptake into the nerve terminal (d).

32

IBO2010 KOREA
THEORETICAL TEST Part A

A29. Figures I~III depict the excretory systems of planaria, earthworms, and grasshoppers. Figure IV
illustrates the habitat of the salmon life cycle.

Which of the following statements concerning excretory structures is correct?


A. In a planarian, the beating of cilia (a) within each flame bulb releases filtrate in the direction of
the arrows.
B. A pair of mesonephridia within each segment of the earthworm collects coelomic fluid from the
adjacent anterior segment and excretes that collected fluid.
C. Structure (b) is called a collecting duct, which collects and excretes concentrated urine that is
hyper-osmotic to body fluids.
D. In a grasshopper, reabsorption of the filtrate occurs mainly in the Malpighian tubules (c), where
most solutes are pumped back into the hemolymph, with water following by osmosis.
E. In freshwater, salmon take up salt from the gills and produce dilute urine; in the ocean, they
excrete excess salt through their gills.

33

IBO2010 KOREA
THEORETICAL TEST Part A

A30. Which of the following statements is correct concerning gas exchange organs in animals?
A. In starfish, the gill plays a role in gas exchange, but the tube feet do not play a role in that
process.
B. In grasshoppers, well-developed muscles surrounding the tracheae control movement of air
inward and outward through an external opening.
C. In fish, blood flows through the gill-filament capillaries in the same direction as that of water
exiting from the mouth and pharynx to the outside.
D. In birds, during exhalation both air sacs deflate, forcing air to the outside, whereas the lung is
filled with air.
E. In humans, surfactants are required to increase the surface tension in the trace amount of fluid
coating the inner alveolar surface; in the absence of surfactants, the alveoli collapse during
exhalation, blocking the entry of air during inhalation.

34

IBO2010 KOREA
THEORETICAL TEST Part A

A31.

The following picture of the midsagittal section of the human brain illustrates diencephalic

structures.

Choose a correct statement.


A. Structure (a) plays a role in the maintenance of temperature, hunger, and thirst.
B. Structure (b) produces melanin, a hormone that is involved in photoperiodic function.
C. Almost all of the incoming, somatosensory information is sorted in structure (c) and is sent to
the appropriate cerebral centers for further processing.
D. Structure (d) is under the direct control of the suprachiasmatic nucleus.
E. Structure (e) is derived from epithelial cells; thus, its embryological origin is different from that
of structure (d).

35

IBO2010 KOREA
THEORETICAL TEST Part A

A32. Which of the following statements about the regulation of male reproduction is not correct?
A. Follicle stimulating hormone (FSH) promotes the activity of Sertoli cells located within the
seminiferous tubules.
B. Luteinizing hormone (LH) regulates Leydig cells located in the interstitial space between the
seminiferous tubules.
C. In response to LH, Leydig cells secrete testosterone and other androgens, which promote
spermatogenesis in the tubules.
D. Testosterone regulates blood levels of gonadotropin-releasing hormone (GnRH), FSH, and LH
through inhibitory effects on the hypothalamus and anterior pituitary.
E. Inhibin, a hormone produced by Leydig cells, acts on the anterior pituitary gland to reduce LH
secretion.

36

IBO2010 KOREA
THEORETICAL TEST Part A

ETHOLOGY
A33. An experiment is designed to test this hypothesis: the number of yellowjacket wasps at a feeding
site visually affects the feeding-site choices of workers collecting nectar.. Four feeders with zero, one,
two or eight individual decoys are prepared, as shown in the figure below. One nectar dish is placed
in the middle of each feeder. You then observe the feeding-site choice made by each worker.

Which of the followings should not be included in this experimental design?


A. Using live individuals as decoys.
B. Placing the four feeders with nectar dishes randomly and alternatively.
C. Using the nectar solutions of equal concentration among the feeders.
D. Preventing other species from visiting the feeders.
E. Preventing successive visits by the same worker.

37

IBO2010 KOREA
THEORETICAL TEST Part A

A34. As seen in the left-hand graph, a population of moth species A exhibits individual variation in
body color. The environment in which this population lives includes predators, such as birds, which
find species A palatable. The environment also includes other moth species unpalatable to birds: one
individual from each of three species (1~3) is shown in the right-hand illustration. Species 1, 2, and 3
are similar to different phenotypes found within species A: species 1 to lighter individuals, species 2
to individuals with intermediate phenotype, and species 3 to darker individuals. After capturing and
tasting species 1, 2 and 3, birds learn to avoid eating them. Species A is considered a Batesian mimic
of the other species. If species 3 becomes most abundant in this habitat, which graph accurately
predicts what you would observe in species A? (The dotted line represents the mean value of the
original population of species A.)

38

IBO2010 KOREA
THEORETICAL TEST Part A

A
.

B
.

39

IBO2010 KOREA
THEORETICAL TEST Part A

A35. Eusocial honeybees have a specific system of sex determination. Females are diploid (2n) and
develop from fertilized eggs; males are haploid (n) and develop from unfertilized eggs. Assuming
that the queen copulated with a single male, which of the following is/are most likely true for this
social group?

I. The males have mothers but not fathers.


II. A female should foster her brothers to increase her inclusive fitness rather than trying to
increase her direct reproduction.
III. It is advantageous to females' (workers) fitness if the queen produces sons and
daughters in equal proportions.
IV. A female should remove the eggs of other females (workers) from the nest to increase
her fitness.

A. Only I and II
B.

Only I and III

C.

Only I and IV

D. Only II and III


E.

Only III and IV

40

IBO2010 KOREA
THEORETICAL TEST Part A

GENETICS AND EVOLUTION


A36. Which of the following can children only inherit from their mother?
A mutation:
A. on the X chromosome.
B. on the Y chromosome.
C. in the mitochondrial genome.
D. in a maternally imprinted gene.
E. in the hypervariable region of an antibody gene.

41

IBO2010 KOREA
THEORETICAL TEST Part A

A37. In Drosophila melanogaster, yellow body and white eye are both X-linked recessive genes.
Wild-type males were crossed with yellow females with white eyes, and F1 progenies were produced
in the numbers and phenotypes shown in the table below.

Progeny group

Progeny phenotype and sex

Progeny number

(a)

wild-type female

3,996

(b)

yellow males with white eyes

3,997

(c)

yellow females with white eyes

(d)

wild-type male

Which of the following is the best explanation for how progeny groups (c) and (d) were produced?
A. Genetic recombination during meiosis I.
B. Genetic recombination during meiosis II.
C. Somatic mutations in the eye and body of wild-type flies.
D. Nondisjunction of sex chromosome.
E. Dosage compensation for X-linked genes.

42

IBO2010 KOREA
THEORETICAL TEST Part A

A38. Four mutant strains of bacteria (1~4) all require substance S to grow (each strain is blocked at
one step in the S-biosynthesis pathway). Four plates were prepared with minimal medium and a trace
of substance S, to allow a small amount of growth of mutant cells. On plate a, mutant cells of strain 1
were spread over entire surface of the agar to form a thin lawn of bacteria. On plate b, the lawn was
composed of mutant cells of strain 2, and so on. On each plate, cells of each of the four mutant types
were inoculated over the lawn, as indicated in the figure by the circles. Dark circles indicate excellent
growth. A strain blocked at a later step in the S substance metabolic pathway accumulates
intermediates that can feed a strain blocked at an earlier step.

What is the order of genes (1~4) in the metabolic pathway for synthesis of substance S?
A. 2 4 3 1
B. 2 1 3 4
C. 1 3 4 2
D. 1 2 4 3
E. 3 4 2 1

43

IBO2010 KOREA
THEORETICAL TEST Part A

A39.

By using modern technology, the gene that determined height in Mendels pea plants was

discovered to be the gene Le that codes for the enzyme involved in biosynthesis of the gibberellin
hormone GA1. The two alleles for this gene, T and t, differ in only one nucleotide. The enzyme
produced by the recessive t allele has efficiency as low as 1/20 of the normal enzyme. Which of the
following statements is correct?

A. GA1 is directly involved in auxin biosynthesis in the pea plant.


B. The product of the T allele is responsible for the normal gibberellin hormone.
C. A F1 plant from a cross between TT and tt plants will have 1/20 enzyme activity of that of
normal plant.
D. Treatment of a tt plant with gibberellin hormone fails to make it grow to become a tall plant.
E. The mutation is due to a deletion of the gene Le.

44

IBO2010 KOREA
THEORETICAL TEST Part A

A40. Glucose-6-phosphate dehydrogenase (G6PDH) is encoded by a single X-linked gene in humans.


There are multiple functional alleles for this gene, such as A1, A2, etc.

G6PDH dimers are made in

cells and secreted into blood.


If a woman has both A1 and A2 allele for G6PDH, what type(s) of dimers is/are found in her blood?
A. Only A1A1
B. Only A2A2
C. Only A1A2
D. Only A1A1 and A2A2
E. A1A1, A2A2 and A1A2

45

IBO2010 KOREA
THEORETICAL TEST Part A

A41. The direction of shell coiling in the snail Limnaea peregra is either dextral or sinistral. Coiling
direction is determined by a pair of autosomal alleles. The allele for dextral (S+) is dominant over the
allele for sinistral (s). Experimental results of two reciprocal monohybrid crosses are shown below.

What is the genetic phenomenon that explains the inheritance pattern for coiling direction?
A. Cytoplasmic inheritance.
B. Epistasis.
C. Genetic imprinting.
D. Maternal effect.
E. Sex-limited inheritance.

46

IBO2010 KOREA
THEORETICAL TEST Part A

A42. Some fruit flies (Drosophila melanogaster) have a mutation that makes them shake. These fruit
flies are called shakers.

An experimental cross is shown below:

What kind of inheritance best explains the inheritance pattern for the shaker gene?
A. Somatic dominant.
B. Somatic recessive.
C. X-linked dominant.
D. X-linked recessive.
E. Y-linked dominant.

47

IBO2010 KOREA
THEORETICAL TEST Part A

A43.

'Coefficient of relatedness' (or 'genetic relatedness') refers to the probability of two related

individuals inheriting a particular allele of a single gene from their common ancestor.

In this family tree of diploid individuals, which of the following 'coefficient of relatedness' is not
true?
Coefficient of relatedness of
A.

A being 1/2

B.

B being 1/2

C.

C being 1/4

D.

D being 1

E. E being 1/4

48

IBO2010 KOREA
THEORETICAL TEST Part A

ECOLOGY

A44. The figure below shows the change in the abundance pattern of three trophic levels in a lake when
it was polluted by city sewage. Ground-feeding carps increase in frequency because they benefit
directly from additional mineral nutrients.

Which ecological control methods could improve the water quality of the lake?

<Ecological control methods>


Mechanism
I.

Control methods

Top-down control: Attempt to introduce predatory fish on carp.

II.

Top-down control: Attempt to reduce the nutrients in the river entering the lake.

III.

Bottom-up control: Attempt to inhibit recycling of nutrients


accumulated in the substrate of the lake.

IV.

Bottom-up control: Attempt to reduce primary producers as


well as consumers by introducing more carp.

A. Only I and II

B. Only I and III

D. Only II and III

E. Only II and IV

49

C. Only I and IV

IBO2010 KOREA
THEORETICAL TEST Part A

A45. The figure below depicts life-history strategies for three plant species (a~c) along 3
axes: strength of competition with other organisms, level of disturbance in the habitat, and
level of environmental stress in the habitat. Species a grows in habitats where competition
among species is high but disturbance and stress are low. Species b grows in habitats with
high environmental stress but with low interspecies competition. Species c grows in
highly disturbed habitats with low environmental stress.

Which of the statements below is/are correct?


I
II

Characteristics of a-type plants are slow growth rate and short-lived leaves.
Desert annual plants are b-type species. They have rapid growth and produce large
amount of seeds in a short time after rains.

III

Most plants belonging to c-type species would be herbaceous while a-type and
b-types species are likely to be trees or shrubs.

A. Only II
B. Only I and II
C. Only I and III
D. Only II and III
E. I, II, and III

50

IBO2010 KOREA
THEORETICAL TEST Part A

A46. In the photic zone of freshwater and marine environments, where light penetrates, cyanobacteria
are found in the upper part of the zone, and purple and green bacteria are in the lower part of the
zone. Which of the following statements best explains the vertical distribution of the photosynthetic
bacteria?
A. Green and purple bacteria are anaerobic, while cyanobacteria are aerobic.
B. Green and purple bacteria are better able to use light wavelengths that cyanobacteria do not use
as efficiently.
C. Habitat isolation develops due to competition for nutrient and oxygen.
D. Cyanobacteria are better able to use oxygen as an electron donor for photosynthesis.
E. It is the result of adaptation to lower temperatures in purple and green bacteria.

51

IBO2010 KOREA
THEORETICAL TEST Part A

A47. The following figure shows nitrogen cycling in an ecosystem. Numbers I~V represent different
chemical conversion steps in the cycle.

Which process (I ~ V) is correctly paired with the organismal group performing that step?
A. I- photoautotrophs.
B. II- bacteria symbiotic with plants.
C. III- anaerobic bacteria living in conditions such as wetland ecosystem.
D. IV- eukaryotic organisms.
E. V - nitrogen fixing bacteria such as Rhizobium or Cyanobacteria.

52

IBO2010 KOREA
THEORETICAL TEST Part A

A48. The following graph shows the relationship between the frequency or strength of disturbance and
species diversity.

Which of the following statements is not correct?


A. The low species diversity of community (a) is due to the presence of dominant species in the
community.
B. In community (c), species diversity is low because there is not enough time between
disturbances for a wide variety of species to colonize.
C. The degree of competitive exclusion among species is highest in community (b).
D. In community (c), late successional species will be replaced rapidly by early successional
species.
E. Community (c) consists of environmental stress-tolerant species.

53

IBO2010 KOREA
THEORETICAL TEST Part A

A49. Which of following statements is/are correct?


With increasing atmospheric CO2,
I.

inorganic nutrients in soil will be increasingly limiting for plant growth.

II. C4 plants will grow increasingly better than C3 plants in environments where water is
limiting.
III. the increased C:N ratio in litter will cause an increase in decomposition rate by soil
microorganisms.

A. Only I
B. Only II
C. Only III
D. Only I and II
E. Only II and III

54

IBO2010 KOREA
THEORETICAL TEST Part A

BIOSYSTEMATICS

A50. The following summaries describe recently published research results.

Research 1. Wu and Li (1985): The comparative analysis of homologous genes between human and
mouse genomes suggests that the evolutionary rate of homologous genes was higher in the
mouse lineage than in the human lineage.
Research 2. Smith and Donohue (2008): The plant families Caprifoliaceae, Asclepiadaceae, and
Lamiaceae are composed of both herbaceous and arborescent species. The comparative analysis
of homologous genes between the herbaceous and the arborescent species within a single plant
family suggest that the evolutionary rates of homologous genes in herbaceous lineages were
faster than that of arborescent lineages in all three plant families.
Research 3. Gilman et al. (2009): The comparative analysis of 130 homologous mitochondrial genes
between a sister species pair of vertebrates from the temperate region and from the tropical
region indicate that the base substitution rates of homologous genes from the tropical region are
1.7 times faster than that of the temperate region.

55

IBO2010 KOREA
THEORETICAL TEST Part A

Based on these studies, which of the following statements best describes the common evolutionary
processes in plant and animal genes?
A. The evolutionary rates of genes are accelerated in short-lived animals and plants.
B. The evolutionary rates of genes are accelerated in higher animals and plants.
C. The evolutionary rates of genes are accelerated in animals and plants which lived in higher
temperature regions.
D. Direct comparisons of homologous genes between animals and plants show that plants evolve
faster than animals.
E. The fast evolutionary rates of mitochondria genes make them ideal for phylogenetic
comparison between distant lineages.

56

IBO2010 KOREA
THEORETICAL TEST Part A

A51. Which of the following pairs does not show a monophyletic group - paraphyletic group
relationship?
A. Monocots - Dicots
B. Tetrapods - Bony fishes
C. Echinoderms - Chordata
D. Birds - Reptiles
E. Vascular plants - Nonvascular plants

57

IBO2010 KOREA
THEORETICAL TEST Part A

A52. The following figure shows a hypothetical evolutionary tree of species a~ e along with the
variability between pairs of these species.

Choose a statement that is correct.


A. The speciation rate shows a linear relationship to evolutionary time.
B. Species variation shows a linear relationship to evolutionary time.
C. The species pair a - b and the pair c- d shows sister group relationship.
D. The tree contains three monophyletic groups.
E. Species a can be used as an outgroup for the other four species.

58

IBO2010 KOREA
THEORETICAL TEST Part A

A53. Which of the following statements about speciation is correct?


A. Sympatric speciation occurs more gradually and more slowly than allopatric speciation.
B. The divergence of two maggot fly races is an example of allopatric speciation due to mating
time differences.
C. The evolution of cultivated wheat is associated with polyploidization. This is an example of
sympatric speciation.
D. Allopatric speciation is usually associated with stronger secondary reproductive barriers than
sympatric speciation.
E. Different species of Drosophila inhabit the different islands of Hawaii. This is an example of
sympatric speciation.

59

IBO2010 KOREA
THEORETICAL TEST Part A

A54. The following figure shows a mushroom belonging to the Basidiomycetes.

Which of the following combination is correct for the nuclear ploidy states of structures a~ c?

Ploidy state of structure


a

2n

n+n

2n

n+n

n+n

2n

n+n

n+n

60















All IBO examination questions are published under the following Creative Commons license:



CC BY-NC-SA (Attribution-NonCommercial-ShareAlike) https://creativecommons.org/licenses/by-nc-sa/4.0/
The exam papers can be used freely for educational purposes as long as IBO is credited and
new creations are licensed under identical terms. No commercial use is allowed.

Country Code: ___________

Student Code: ___________

Theoretical Test Part A


Answer Key
No.

A1

No.

No.

A19

A37

A38

A2

A20

A3

A21

A39

A22

A40

A41

A4

A5

A6
A7

A23

A24

A25

A8

A26

A9

A27

A10

A28

A11

A29

A12

A42

A43

A44

A30

A45

A46

A47

A48

A13

A31

A49

A14

A32

A50

A15

A33

A16

A34

A17
A18

A51

A52

A35

A53

A36

A54















All IBO examination questions are published under the following Creative Commons license:



CC BY-NC-SA (Attribution-NonCommercial-ShareAlike) https://creativecommons.org/licenses/by-nc-sa/4.0/
The exam papers can be used freely for educational purposes as long as IBO is credited and
new creations are licensed under identical terms. No commercial use is allowed.

ENVELOPE COVER SHEET

Student Code: __________________

The 21st INTERNATIONAL BIOLOGY OLYMPIAD


Changwon, KOREA

11th 18th July, 2010

THEORETICAL TEST: PART B


Time available: 150 minutes
GENERAL INSTRUCTIONS
1. Open the envelope after the start bell rings.
2. A set of questions and an answer sheet are in the envelope.
3. Write your 4-digit student code in every student code box.
4. The questions in Part B may have more than one correct answer. Fill the Answer Sheet with
checkmarks (), numbers, or characters to answer each question.
5. Use pencils and erasers. You can use a ruler and a calculator provided.
6. Some of the questions may be crossed-out. DO NOT answer these questions.
7. Stop answering and put down your pencil IMMEDIATELY after the end bell rings.
8. At the end of the test session you should leave all papers at your table. It is not allowed to take
anything out.

IBO2010 KOREA
THEORETICAL TEST Part B

Country: ________________

Student Code:

___________

The 21st INTERNATIONAL BIOLOGY OLYMPIAD


11th 18th July, 2010

Changwon, KOREA

THEORETICAL TEST: PART B


Time available: 150 minutes
GENERAL INSTRUCTIONS
1. Write your 4-digit student code in every student code box.
2. The questions in Part B may have more than one correct answer. Fill the Answer Sheet with
checkmarks (), numbers, or characters to answer each question.

3. Use pencils and erasers. You can use a scale and a calculator provided.
4. Some of the questions may be crossed-out. Do not answer these questions.
5. The maximal point of Part B is 107.1.
6. Stop answering and put down your pencil immediately after the end bell rings.
7. At the end of the test session you should leave all papers at your table. It is not allowed to take
anything out.

IBO2010 KOREA
THEORETICAL TEST Part B

CELL BIOLOGY

B1. (2.7 points) The Western blot below shows migration distances of five signal molecules (a~e)
involved in a growth hormone-regulated cell-signaling pathway.

To determine the order of molecules (a~e) in the signal cascade that occurs upon the growth hormone
treatment, cells were treated with different inhibitors (I~IV) of cell signaling. The following blots
show the changes in signal molecule expression patterns resulting from inhibitor treatment.

B1.1. (1.5 points) Fill in the boxes in the answer sheet to show the order of proteins (a~e) in the
signaling cascade.
B1.2. (1.2 points) Fill in the circles in the answer sheet to show the site where each inhibitor (I~IV)
exerts its action.

IBO2010 KOREA
THEORETICAL TEST Part B

B2. (2.7 points) Match the molecular constituents (a~f) on the right with the cellular structures
(A~D) that maintain cell morphology on the left. Each cellular structure can have more than one
molecular constituent.

a. Cadherin
A. Cytoskeleton

b. Cellulose

B. Cell wall

c. Collagen

C. Desmosome junction

d. Actin

D. Extracellular matrix

e. Keratin
f. Lignin

IBO2010 KOREA
THEORETICAL TEST Part B

B3. (1.5 points)

In the figure, the letter in each box represents an organ or tissue.

Match each listed organ or tissue in the answer sheet to the correct box in the figure.

IBO2010 KOREA
THEORETICAL TEST Part B

B4. (2.2 points)

When E. coli is grown on a medium containing a mixture of glucose and lactose, it

shows complex growth kinetics, as shown in the graph below.

IBO2010 KOREA
THEORETICAL TEST Part B

B4.1. (1 point) Which pair of graphs correctly shows the changes in glucose concentrations in the
medium and -galactosidase activity within the cells?

IBO2010 KOREA
THEORETICAL TEST Part B

B4.2. (1.2 points) The graph below shows the expression pattern of lac mRNA in wild-type and mutant
E. coli cells after lactose is added to a glucose-depleted medium.

Indicate with a checkmark () in the answer sheet whether each mutant is able or unable to show the
mutant expression pattern.

Mutant
I. An E. coli mutant in which the repressor is not expressed.
II. An E. coli mutant in which the repressor can bind to the operator, but not to lactose.
III. An E. coli mutant in which the operator is mutated so that the repressor cannot bind
to the operator.
IV. An E. coli mutant in which RNA polymerase cannot bind to the promoter of the lac
operon.

IBO2010 KOREA
THEORETICAL TEST Part B

B5. (1.5 points)

Transcription and translation of a gene in a prokaryote cell are depicted in the picture

below.

Indicate with a checkmark () in the answer sheet whether each description is true or false.

Description
I. The direction of transcription is from (B) to (A).
II. Location (C) of the mRNA is the 5' - end.
III. The polypeptide on ribosome (D) is longer than the polypeptide on ribosome (E).

IBO2010 KOREA
THEORETICAL TEST Part B

B6. (2 points)

A part of the nucleotide sequence of one strand of a double-stranded DNA molecule and

the corresponding amino acid sequence are shown. The table shows a portion of the genetic code.

Codon position
DNA strand
Polypeptide

5'........ TTT AAG TTA AGC .......3'


........

Phe

Lys

Codon

Amino acid

UUU

Phe

UUA

Leu

AAG

Lys

AGC

Ser

Leu

Ser .......

Indicate with a checkmark () in the answer sheet whether each description in true or false.
(Assume that the length of the DNA is the same as that of its primary transcript.)
Description
I. The DNA strand shown is a template strand.
II. If the G+C content of the DNA strand shown is 40%, then the A+T content of its
complementary DNA strand is 60%.
III. If the G+C content of the DNA strand shown is 40%, then the A+U content of the
primary transcript is 60%.
IV. The nucleotide sequence of mRNA is 5' ....... UUU AAG UUA AGC ....... 3'.

IBO2010 KOREA
THEORETICAL TEST Part B

B7. (2 points)

The picture below shows the process of generating a transgenic plant harboring gene X

using the Agrobacteria Ti-plasmid.

B7.1. (1 point) Which explanation about this process is true or false?


Explanation
I.
II.
III.

Restriction enzymes and ligase are used to make the recombinant DNA.
Plant tissue culture techniques are used to differentiate the leaf discs into a plant.
The whole recombinant Ti-plasmid harboring gene X gets integrated into the plant
genome.

IV.

The introduction of gene X into the transgenic plant genome can be confirmed by
using genomic PCR or genomic Southern blot analysis.

V.

The expression of the introduced gene X in the plant cell can be checked by using
RT(reverse transcriptase)-PCR, Northern blot analysis, or Western blot analysis.

10

IBO2010 KOREA
THEORETICAL TEST Part B

B7.2. (1 point) Evaluate whether the following description is true or false for a plant expression vector
in general?
Description
I.

It should include the selection marker gene that is needed for selecting the
transformed cell.

II.

It should include a promoter that can express the introduced gene within the plant cell.

III.

It usually contains a multiple cloning site used for insertion of the foreign gene.

IV.

It should have the same nucleotide sequence with the specific part of the plant genome
because the foreign gene is inserted by homologous recombination.

V.

It should have the replication origin needed for cloning during the process of making
the recombinant vector.

11

IBO2010 KOREA
THEORETICAL TEST Part B

B8. (1.5 points) Caulobacter bacteria undergo a special cell division. Division of the mother cell results
in two different daughter cells: a roaming (r) cell and a pedicle (p) cell. Roaming cells permit
Caulobacter to spread out. Pedicle cells stay and use the pedicle to stick at that place. The picture
below shows how roaming and pedicle cells divide.

The division cycle period when starting with a roaming cell (r = 90 min) is longer than when starting
with a pedicle cell (p = 60 min). The extended length of period (r) is because the roaming cell
A. produces more DNA than the pedicle cell.
B. produces a pedicle before division.
C. produces a flagellum during division.
For each of the above explanations, indicate with a checkmark () on the answer sheet whether it is
true or false.

12

IBO2010 KOREA
THEORETICAL TEST Part B

B9. (2 points) In the experiment described below, cells (1) were put in a medium with a salt concentration
lower than the cytoplasm, causing them to swell and rupture at one location (2). Ruptured cells were
then washed out and resealed to form ghosts (3). This process also produced smaller vesicles whose
membrane was either right-side-out (4) or inside-out (5), depending on the ionic conditions of the
solution used for the disruption procedure.

Prepared ghosts/vesicles were then mixed with a radioactive labeling reagent that is water-soluble and
could covalently attach to proteins (3~5). The proteins embedded in the membrane were then
solubilized with detergent and analyzed by SDS polyacrylamide-gel electrophoresis. Segregated
proteins were visualized by Coomassie Blue staining (I) and autoradiography (II).

13

IBO2010 KOREA
THEORETICAL TEST Part B

Which of the proteins (a~e) is/are transmembrane protein(s)?


A. Protein b
B. Protein c
C. Protein d
D. Proteins a~e
E. Protein a and protein e

14

IBO2010 KOREA
THEORETICAL TEST Part B

B10. (1.5 points)

Subcellular organelles and their cellular components can be easily separated by the

size-fractionating differential centrifugation method, as depicted below. During the process, four
pellets (nucleus and 1~3) are formed.

The table below shows descriptions about subcellular organelles collected in different centrifugation
pellets.
Pallets

Description

Nucleus

An organelle containing a linear DNA harboring telomeric sequences.

Pellet 1

An organelle inheriting its genetic information by maternal inheritance.

Pellet 2

An organelle performing glycosylation of most proteins.

Pellet 3

An organelle composed of two subunits and synthesizing proteins.

15

IBO2010 KOREA
THEORETICAL TEST Part B

Provided that the subcellular structures are not disrupted during the centrifugation process, determine
whether descriptions A, B and C of different subcellular structures in the same pellets are true or false
taking above information as a reference. Mark the appropriate box with a checkmark () in the answer
sheet.
Pellet

Description

Pellet 1

An organelle containing a bunch of proteases, lipases, and nucleases.

Pellet 2

An organelle carrying an enzyme catalyzing the conversion of hydrogen


peroxide (H2O2) to water and oxygen.

Pellet 3

The infected intracellular virus covered with viral coat.

16

IBO2010 KOREA
THEORETICAL TEST Part B

B11. (2 points)

The SalI and XhoI restriction map of a 5 kb linear DNA molecule is shown below.

The 3.5 kb DNA fragments obtained from a XhoI digestion were ligated with the 1.0 kb DNA
fragments obtained from a SalI digestion. The resulting 4.5 kb DNA molecules were digested with
SalI. Write down all the different lengths of DNA fragments you can get from this digestion. (Assume
that restriction enzymes completely cut all the DNA molecules, and ignore blunt-end ligation.)

17

IBO2010 KOREA
THEORETICAL TEST Part B

B12. (1.5 points)

The following graphs show the quantitative change in DNA content at each of four

stages in the cell cycle (G1, S, G2, M).

Select the graph (A~D) representing the stages described in I~III.


Cellular activity and response
I. Taxol treatment, which prevents microtubule deploymerization, arrests the cell at this
stage.
II. With a mitogen treatment, such as an epidermal growth factor, an arrested cell at this
stage proceeds to the next stage of the cell cycle.
III. The cell cycle check point at this stage confirms that DNA duplication is complete
before the cell proceeds to the next stage.

18

IBO2010 KOREA
THEORETICAL TEST Part B

PLANT ANATOMY AND PHYSIOLOGY

B13. (2 points) A transgenic Arabidopsis plant (2n) has a total of two copies of a kanamycin-resistant
gene in its nuclear genome, one on chromosome 1 and the other on chromosome 3. For each
description of this plant, indicate with a checkmark () in the answer sheet whether the description is
true or false.

Description
I. All pollen grains of this plant have kanamycin-resistant genes.
II. Endosperms formed by self-fertilization of this plant have 0~6 copies of the
kanamycin-resistant gene.
III. If seeds from self-fertilization of this plant are germinated, the ratio of kanamycin-resistant
to kanamycin-sensitive seedlings is 3 to 1.
IV. A cell containing 4 copies of the kanamycin-resistant gene exists among root cells at
prophase of mitosis in this plant.

19

IBO2010 KOREA
THEORETICAL TEST Part B

B14. (1.5 points) Figure a shows an ABA signal transduction pathway in a guard cell. Figure b shows
changes occurring after ABA treatment in (1) the cytoplasmic Ca2+ concentrations of guard cell and (2)
stomata aperture size.

For each description about ABA action, indicate with a checkmark () in the answer sheet whether the
description is true or false.
Description
I.

With ABA treatment, Ca2+ is moved from outside of the guard cell into the cell interior.

II.

With ABA treatment, the concentration of K+ is increased in the cytoplasm of guard cells.

III.

The K+ channel (I) is outward, and the K+ channel (II) is inward.

20

IBO2010 KOREA
THEORETICAL TEST Part B

B15. (3 points)

The chloroplast, a plant organelle, originated from ancestors of the cyanobacteria;

however, many proteins in the chloroplast are encoded from genes in the nuclear genome.

B15.1. (1.2 points) For each property of chloroplast DNA, indicate a checkmark () in the answer sheet
whether the property is similar to that of prokaryote or eukaryote genomic DNA.
Property
I. The DNA is a circular double strand.
II. Introns are found.
III.

Component of 70S ribosome is encoded.

IV. Usually, polycistronic mRNA is transcribed.

21

IBO2010 KOREA
THEORETICAL TEST Part B

B15.2. (1.8 points) Protein X, a thylakoid lumen protein, is transcribed in the nucleus and translated in
the cytoplasm. Next, the protein is translocated into the stroma of the chloroplast by signal peptide
I. In the stroma, signal peptide I is cleaved, and the remaining protein is targeted to the thylakoid
lumen by signal peptide II. In the thylakoid lumen, signal peptide II is cleaved, and the remaining
polypeptide III is usually observed.

Several recombinant vectors of protein X are transformed into the nuclear genome and expressed.
For each recombinant vector, fill the blanks in the 2nd column with the cellular location (A~D)
where the expressed proteins are mainly observed. Fill the blanks in the 3rd column with the
polypeptides (E~H) observed in that location.

< Cellular location of expressed proteins >


A. Cytoplasm

B. Stroma

C. Thylakoid membrane

D. Thylakoid lumen

< Observed polypeptides >


E.

I-II-III

F.

I-III

G.

22

II-III

H.

III

IBO2010 KOREA
THEORETICAL TEST Part B

B16. (1.5 points)

Figure a shows organogenesis of plant calluses incubated on media containing

different concentrations of IAA (an auxin) and kinetin (a cytokinin). In nature, Agrobacterium, a soil
bacterium, induces crown gall tumors on the roots of legume plants. The bacterium induces these
tumors by integrating its T-DNA into the plant genome and by expressing a group of genes necessary
for gall formation (Figure b).

If an infecting Agrobacterium lacks or over-expresses the auxin-biosynthetic genes or


cytokinin-biosynthetic genes, determine the most expected callus phenotype (A~D) for mutations (I,
II, and III) described in the table below. Indicate with a checkmark () in the appropriate box in the
answer sheet.
< Expected callus phenotypes >
A. Shooty callus

B. Rooty callus

C. Undifferentiated callus

D. Propagation-deficient callus

Gene mutation
I. Deletion of iaaH, overexpression of ipt.
II. Overexpression of iaaH, deletion of ipt.
III. Deletion of iaaH and ipt.

23

IBO2010 KOREA
THEORETICAL TEST Part B

B17. (2.4 points)

Plant root cell type is determined by the division and differentiation of a particular

stem cell (meristematic cell). Figure a shows the whole microscopic structure of a longitudinallysectioned Arabidopsis primary root. Figure b is an enlarged diagram corresponding to a region of the
inset in Figure a, showing the arrangement of root primordia (stem cells).

Fill in the table to best match the listed function with the correct root cell type (1~6 in Figure a) and
with the corresponding initial cell (7~11) in Figure b.

24

IBO2010 KOREA
THEORETICAL TEST Part B

B18. (1.5 points) The figures below show the inner structures of pine and persimmon seeds.

Indicate with a checkmark () whether the following statements are true or false.
I.
II.

Structures a and b are the same in ploidy, but they differ in genetic composition.
Structures a, b, and c consist of two different sporophytic structures and one
gametophytic structure.

III.

Structures x and y are the same in both ploidy and genetic composition.

IV.

Structure z is three-times higher in ploidy than structure c.

V.

Structures a and x are both surrounded by the ovary.

25

IBO2010 KOREA
THEORETICAL TEST Part B

ANIMAL ANATOMY AND PHYSIOLOGY

B19. (1.8 points) Human blood can be separated into three parts using a table top centrifuge, as shown
in the following figure.

Of these blood parts (a~c), select the part that contributes most to the listed functions of blood.
Answer by placing a checkmark () in the appropriate box in the answer sheet.
Function
I. Antibody production.
II. Transport of carbon dioxide.
III. Transport of iron.
IV. Transport of oxygen.
V. Formation of blood clot.
VI. Neutralizing snake venom.

26

IBO2010 KOREA
THEORETICAL TEST Part B

B20. (2.2 points) The picture depicts the adult human skeleton and the table lists different types of
joints.

B20.1. (1.2 points) Choose the type of each joint by placing a checkmark () in the appropriate box in
the answer sheet.

B20.2. (1 point) For each statement concerning the function of joints and bones, indicate with a
checkmark () whether the statement is true or false.

Function
I. The joint between the skull and the first cervical vertebra enables the rotation of the
head.
II. The fibula, as well as the tibia, plays an important role in supporting the body weight.

27

B21. (2.4 points)

Chordates are distinguished from other animals by 4 distinctive key

morphological characters.
B21.1. (1.2 points) Choose the 4 key morphological characters from the following list and write their
numbers in the left-hand column of the table in the answer sheet.
Morphological character
1. Cirri,

2. Brain,

3. Pharyngeal slits,

4. Gills,

5. Notochord,

6. Intestine,

7. Tubular dorsal nerve cord,

8. Anus,

9. Tail.

B21.2. (1.2 points) The morphological characters of a lancelet (Branchiostoma) are shown in the
illustration below. Find each of the morphological characters that you listed in the table (from
B21.1) - write the corresponding letter code in the right-hand column of the table in the answer
sheet.

28

IBO2010 KOREA
THEORETICAL TEST Part B

B22. (2 points)

The graph below depicts the pressure changes in an aorta, left ventricle, and left atrium

that occur concurrently during the mammalian cardiac cycle. Below the graph are sketches of the
heart illustrating blood flow and valve state (opened/closed).

Match each numbered event in the cardiac cycle graph with the letter of its corresponding heart sketch.
Write the corresponding letter code in the right-hand column of the table in the answer sheet.

29

IBO2010 KOREA
THEORETICAL TEST Part B

B23. (1.5 points)

Fig. I shows the relationship between weight and the specific metabolic rate of the

indicated animal species, and Fig. II shows the O2 consumption rate of the indicated species as a
function of running speed (on a treadmill machine).

30

IBO2010 KOREA
THEORETICAL TEST Part B

Read each of the following explanations, and indicate with a checkmark () in the answer sheet
whether the explanation is true of false.
Explanation
A. At rest, smaller animals consume more energy per weight than the bigger animals
consume.
B. Using the same amount of food per body weight, a smaller animal can travel a longer
distance than a bigger animal can travel.
C. Using the same amount of food, bigger animals generate more ATP than the smaller ones
generate.

31

IBO2010 KOREA
THEORETICAL TEST Part B

B24. (1.8 points)

If an astronaut lived on a heavier and larger planet than Earth, he would experience

stronger gravitational forces. In that case, what would you expect to happen in this astronauts body?
For each symptom listed below, indicate with a checkmark () whether the symptom is expected or
unexpected.(Assume that the composition of the atmosphere of the planet is the same as that of Earth.)

Symptom

A. Increase in blood pressure.

B. Decrease in the respiration rate.

C. Increase in muscle mass.

D. Increase in bone density.

E. Decrease in the number of red blood cell.

F. Increase in oxygen content in the blood.

32

IBO2010 KOREA
THEORETICAL TEST Part B

B25. (1.5 points)

The following dissection figure shows the blood vessels in liver tissue. The three main

blood vessels are indicated by capital letters (A~C).

Following statements describes properties of blood that flows through particular blood vessels. For
each description, indicate with a checkmark () in the appropriate box with matching vessel where
that blood would be found.
Description
I. Blood with the highest oxygen content.
II. The blood shows the first increase in lipid content after the meal.
III. The blood shows the first increase in glucose content after the meal.

33

IBO2010 KOREA
THEORETICAL TEST Part B

B26. (3 points) A Korean professor, Charlie Shin, was bilingual, such that he is fluent in Korean and
English. He was also good at communicating using sign language. Unfortunately, he had a stroke
while taking part in discussions at the 2010 IBO International Jury meeting. Dr. Oliver diagnosed that
Charlie had damage in his left cerebral cortex which controls some part of his language output area
and whole arm areas.

B26.1. (1 point) A novice nurse examined Charlies language ability. Select a correct diagnosis among
below.
A. Charlie had difficulty in understanding Dr. Olivers talk.
B. Charlie had difficulty in understandings of the 2010 IBO theoretical questions written on a
paper.
C. Charlie had a hard time to understand a word LOVE written on his back by Dr. YT Kim.
D. Charlies ability to speak Korean fluently had disappeared.
E. Charlies ability to write Korean poems with his right hand remained intact.

B26.2. (1 point) The ability of Charlies sign language and the movement of upper extremity were also
carefully examined by Dr. Oliver. The results showed that he was also incapable of proper
execution of sign language expression in either arm and of moving his right arm. What can we
conclude from this?
A. The damaged language area is responsible for both sign as well as spoken language.
B. Motor neurons in the right cerebral cortex govern the muscles of the right side.
C. The language comprehension region is located in the right hemisphere.
D. His visual system is also damaged.
E. His sign language expression with left arm is normal.
34

IBO2010 KOREA
THEORETICAL TEST Part B

B26.3. (1 point) A brain-machine interface (BMI) study using a monkey was reported in the Science
journal. An array of micro-wire recording electrodes was implanted in associative, arm movement
planning area in the frontal cortex of a normal monkey. During upper arm movements
electromyogram (EMG), recordings were taken from upper extremities, and at the same time
neural recordings were made from implanted recording electrodes in the frontal cortex.
Correlations between EMG and neural signals were obtained every 200 msec and used as
commands for robot arm movement. The monkey intentionally controlled the robot arm with
almost 100% success rate, without using arm muscles. Evaluate whether the following would be
true or false if this BMI technology is used for human.

Description
I

Immunological reaction is one of obstacles to overcome for future development of a


prosthetic device for patients such as Charlie.

II

For accurate decoding of motor planning information, the number of simultaneously


recorded neurons should be increased.

III

It is more difficult to design prosthetic robot fingers than a robot arm using this kind
of BMI technology.

IV

This BMI technology is applicable to overcome Charlies language disability by


decoding motor production information.

The described BMI technology can be classified as a motor (output) BMI, while
artificial cochlea can be classified as an sensory (input) BMI.

35

IBO2010 KOREA
THEORETICAL TEST Part B

B27. (3 points) A spinal nerve has four different kinds of axons carrying out physiological functions like
muscle contractions and cutaneous sensory, thermal and pain sensations. Myelinated, large-diameter
axons carry motor information, while unmyelinated, small-diameter axons carry pain information. An
electrophysiological experiment was carried out using an isolated rat spinal nerve. Four different
intensities of electrical stimulation were delivered to the nerve. Since the stimulation caused
simultaneous activation of all axons in the nerve, including both small and large diameter axons, we
observed different peaks (a to d) in the compound action potential (CAP) traces on an oscilloscope.
The averaged post-stimulus time delays of these CAP peaks were: a, 2 ms; b, 2.5 ms; c, 12 ms; and d,
55 ms. The length of the spinal nerve was 10 cm.

B27.1. (1 point) Calculate the conduction velocity (m/sec) of the CAP peak a.

36

IBO2010 KOREA
THEORETICAL TEST Part B

B27.2. (1 point) After the middle part of the nerve is exposed to a local anesthetic that blocks Na+
channels, which of the following is expected to occur?
A. The height of all CAP peaks is reduced.
B. The post-stimulus time delays of all CAP peaks are shortened.
C. Peaks are reduced and delays are shortened selectively in CAP peaks c and d.
D. Peaks are reduced and delays are shortened selectively in CAP peaks a and c.
E. Peaks are reduced and delays are shortened selectively in CAP peaks b and c.

B27.3. (0.5 point) Which CAP peak is response to painful stimulation?

B27.4. (0.5 point) Which CAP peak is responsible for muscle contractions?

37

B28. (2.7 points)

The figures below present the skeletal structures of Tetrapod anterior limbs. In the

figure, (a) corresponds to an early amphibian limb. The numbers and letter codes with each limb
represent different bones as indicated in the legend under the figure.

H: Humerus,
M: Metacarpals,

U: Ulna,

R: Radius,

S: Sesamoid bone,

38

C: Carpals,
1~5: Phalanges.

IBO2010 KOREA
THEORETICAL TEST Part B

B28.1. (1.8 points) Among the following statements, decide which statements are true or false?
I.

(c) and (e) show loss or fusion in the skeletons as compared to the ancestral
condition.

II.

(b) and (g) show adaptation for life in the ocean.

III.

(b) and (d) show convergent evolution of the skeleton.

IV.

(i) shows adaptation for grasping.

V.
VI.

The sesamoid bones in (f) and (g) are evolutionary reversals.


The figures show homologous characteristics of Tetrapod anterior limbs.

B28.2. (0.9 point) Which of the anterior limbs in the figure above show adaptation for flight or no
adaptation for flight? Indicate with checkmarks () in the appropriate box the answer sheet.

39

IBO2010 KOREA
THEORETICAL TEST Part B

ETHOLOGY

B29. (3 points)

In matriphagy, a spider female is cannibalized by her offspring who attack and eat her

body, when they reach a specific age. The young then live in a group for a short time period and
disperse from the nest individually after the third molt. However, some mothers avoid matriphagy. If a
mother is not eaten by the first clutch, there is a 30% probability that she will be able to produce a
second clutch. The table presents demographic data for this species.
Clutch size
at emergence

Survival rate at
rd

the 3 molting

Body mass

Survival rate

at dispersal

from emergence
until
reproductive age

1st clutch with


matriphagy

100

95%

3.5 mg

20%

100

70%

2.0 mg

10%

40

95%

3.5 mg

20%

1st clutch without


matriphagy
2nd clutch with
matriphagy

B29.1. (1 point) If spiders avoid matriphagy and attempt to produce a second clutch, what is the total
clutch size, on average, that these spiders would produce?

40

IBO2010 KOREA
THEORETICAL TEST Part B

B29.2. (1 point) Calculate and write down the reproductive success of the two strategies in which a
female spider
(i)

produces only a single clutch and is cannibalized, or

(ii) avoids being eaten and attempts to produce a second clutch?


(Reproductive success refers to the mean number of reproductively viable offspring one individual
produces.)

B29.3. (1 point) From an evolutionary perspective and given the constraints above, which behavior
would be selected for?
A. The female does not allow matriphagy because the behavior decreases her survivorship.
B. The female leaves the nest before emergence of the young from the egg sac.
C. The female is eaten by its second clutch after leaving the first clutch just before
matriphagy.
D. The female is eaten by its first clutch.
E. The female does not produce the offspring which cannibalize the mother.

41

IBO2010 KOREA
THEORETICAL TEST Part B

B30. (2.6 points) Honeybee workers (Apis species) perform dances to transmit information about the
distance and direction of the food source.

B30.1. (1 point) What is the primary sensory mechanism involved in this communication between
colony members in the nest?
A. Acoustic
B. Gustatory
C. Olfactory
D. Tactile
E. Visual

42

IBO2010 KOREA
THEORETICAL TEST Part B

B30.2. (1.6 points) The figure below shows the location of 8 food sources (1~8) relative to the hive.

The next figure shows a waggle dance pattern for food source 1. The dotted line indicates the
direction of gravity.

Match each food source direction with its corresponding waggle dance pattern in the following
figures.

43

IBO2010 KOREA
THEORETICAL TEST Part B

B31. (1.5 points) Over a number of generations, two strains of rats were selected in a normal environment
for their increased or decreased maze-learning ability: Maze-bright rats vs. 'Maze-dull rats. For the
experimental test, rats from each strain were reared in three environments that differed in the amount
of visual stimuli present: restricted, normal, and enriched. The graph below shows the behavioral
performance of adults in terms of the number of errors committed in running a maze for the
maze-bright and maze-dull rats.

Mark whether each of the conclusions below is true or false by putting a checkmark () in the
appropriate box in the answer sheet.
Conclusion
I.

This experiment proves that selection for a behavioral trait leads to genetic differences
between strains.

II.

If the two strains of rats are raised in a normal environment, the two strains make a
similar number of errors.

III.

This experiment shows that exposure to visual cues during early development
influences behavioral performance in adult rats.

IV.

The threshold amount of visual stimuli that markedly improves adult behavioral
performance is different for maze-dull and maze-bright rats.

44

IBO2010 KOREA
THEORETICAL TEST Part B

GENETICS AND EVOLUTION

B32. (2 points)

The fruit fly Drosophila melanogaster has a XX(female)-XY(male) system of sex

determination. The Y chromosome determines maleness in humans, but not in Drosophila. Instead,
sex determination in Drosophila depends on the ratio of the number of X chromosomes to the number
of autosomal haploid sets in an individual fly.

The table below describes five mutants whose sex-chromosome complements and haploid sets of
autosomes differ from the normal condition.

Indicate with a checkmark () the sex phenotype of all the mutant flies.

Sex-chromosome complement

Haploid sets of autosomes

XXY

XXX

XXXY

XX

45

IBO2010 KOREA
THEORETICAL TEST Part B

B33. (2.4 points) The following statements concern evolutionary patterns of animal morphological traits.
Mark whether each statement is true or false by putting a checkmark () in the appropriate box on the
answer sheet.

Statements
I. Evolution is invariably a phenomenon with direction; therefore, morphological
complexities evolved from simplicities.
II. Genetic mutations always lead to morphological changes.
III. Increases in animal body size are not universal within evolutionary lineages.
IV. Morphological changes of individuals do not result from allometric growth, the
differential growth of body parts.
V. Chordate species are more similar in the embryonic stages rather than in the adult stages.
VI. Phylogenetic analyses have revealed trends of morphological evolution in some lineages.

46

IBO2010 KOREA
THEORETICAL TEST Part B

B34. (3 points)

The following tables present results of plant crosses involving three linked genes: F is a

flower-color gene, S is a seed-color gene, and L is a plant-height gene. Each gene has two alleles with
one allele exhibiting complete dominance over the other allele. Dominant phenotypes are red flowers,
yellow seeds, and tall plants; recessive phenotypes are white, green, and short, respectively. Assume
that crossing-over between two genes occurs once.

Parents

Red flower / Yellow seed (FfSs)

X White flower / Green seed (ffss)

Red flower /

White flower /

Red flower /

White flower /

Yellow seed

Green seed

Green seed

Yellow seed

F1 phenotypes

Frequency of F1

0.49

Parents

0.49

0.01

0.01

Tall height / Yellow seed (LlSs) : self fertilization


Tall height /

Tall height /

Short height /

Short height /

Yellow seed

Green seed

Yellow seed

Green seed

0.51

0.24

0.24

0.01

F1 phenotypes

Frequency of F1

B34.1. (0.9 point) Indicate with a checkmark () in the answer sheet whether each description is true
of false.
Description
I.
II.
III.

S is closer to L than to F.
Some of F1 plants with tall height / green seed are due to crossing-over.
Crossing-over occurs at prophase of meiosis I.

47

IBO2010 KOREA
THEORETICAL TEST Part B

B34.2. (0.8 point) How many genotypes can be observed in F1 plants having tall height/yellow seed?

B34.3. (1.3 points) Calculate the map unit between gene L and gene S. (One map unit = distance of 1%
recombination)

48

IBO2010 KOREA
THEORETICAL TEST Part B

B35. (2 points) Shown below is a pedigree for the genetic trait PKU (phenylketonuria) that is caused by
a recessive mutation of the PAH gene (that encodes phenylalanine hydroxylase). Under the pedigree
is the RFLP (Restriction fragment length polymorphism) pattern of each individual for the PAH gene.
II-2 individual has the PKU.

B35.1. (1 point) The RFLP phenotype for individual II-2 is not given. From the gel shown below
(A~D), choose all the patterns that would be a correct match for II-2.

B35.2. (1 point) The RFLP phenotype for individual II-4 is not given. From the gel shown below,
determine whether each molecular phenotype (A~D) could be a possible match for II-4.
<Example>

49

IBO2010 KOREA
THEORETICAL TEST Part B

B36. (2 points) 105 cells of a triple-mutant yeast strain (leu his trp ) were spread either on minimal
medium or on minimal medium supplemented with various combinations of histidine, leucine, or
tryptophan. The cultures were grown at either 25 or 37 for 3 days. Colony numbers in each plate
were counted, and the data are listed in the following table.

Number of colonies

Supplements
added on minimal medium

25

37

None

None

None

His, Trp

None

None

Leu, His

Leu, Trp

Confluent

11

Leu, His, Trp

Confluent

Confluent

B36.1. (1 point) What kind of mutation most probably causes the his phenotype?
A. Conditional mutation
B. Deletion mutation
C. Point mutation
D. Missense mutation
E. Nonsense mutation

50

IBO2010 KOREA
THEORETICAL TEST Part B

B36.2. (1 point) What type of mutation most probably causes the leu phenotype?
A. Conditional mutation
B. Deletion mutation
C. Point mutation
D. Missense mutation
E. Nonsense mutation

51

IBO2010 KOREA
THEORETICAL TEST Part B

B37. (2 points)

Human ABO blood type is determined by two genes (H and I). First, the H gene codes

for the antigen precursor. The dominant allele (H) leads to expression of the precursor; the recessive
allele (h) does not. Second, the I gene has three allele forms, IA, IB and i, and determines blood type
(A, B, O or AB).

A male with blood type A and a female with blood type B marry. Each of them is heterozygous for
both the H gene and the I gene. What is the probability of having a son with blood type O? Give your
answer as a percentage (%) rounded to an integer (without any decimals).

52

IBO2010 KOREA
THEORETICAL TEST Part B

B38. (2 points)

The presence of a beard on some goats is determined by the B (beard) gene, which has

two alleles: beardless (B+) and bearded (Bb). The Bb allele is dominant in males but recessive in
females. F1 progeny were born from a cross between a beardless male and a bearded female; F 2
progeny were produced by crossing two F1 individuals.

A beardless male () A beard female ()

F1 F1

F2

Mark whether each statement is true or false by putting a checkmark () in the appropriate box on the
answer sheet.

Description
A. F1 females have beards.
B. One half of F2 progeny have beards.
C. One fourth of F2 females have beards.
D. The beard gene is sex-linked.
E. The beard gene is inherited according to Mendel's principles.

53

IBO2010 KOREA
THEORETICAL TEST Part B

B39. (3 points)

You sequence a 16 bp DNA molecule with the Sanger DNA sequencing procedure.

Shown below is the high resolution electrophoretic pattern of the fragments. As you can see, the
ddCTP lane was damaged.

B39 .1. (1 point) Indicate with a checkmark(() which of the following components are required in the
reaction mixture containing ddGTP?
Component
A. DNA polymerase
B. Primer
C. dATP
D. dGTP
E. Template DNA to be sequenced

54

IBO2010 KOREA
THEORETICAL TEST Part B

B39.2. (1 point) How does the absence of a 3-OH group in ddNTPs affect DNA synthesis?

A. It promotes DNA breakage.


B. It prevents the proper base pairing.
C. It destabilizes the phosphodiester bond.
D. It activates nucleases.
E. It prevents phosphodiester bond formation.

B39.3. (1 point) What would be the correct DNA sequence?


A. 5'-AGGCTACCAGAAATCC-3'
B. 5'-CCTAAAGACCATCGGA-3'
C. 5'-GGATTTCTGGTAGCCT-3'
D. 5'-TCCGATGGTCTTTAGG-3'
E. 5'-TGATGGTTTTAGG-3'

55

IBO2010 KOREA
THEORETICAL TEST Part B

B40. (2 points)

Answer the next two questions using the genetic code table provided below.

B40.1. (1 point) Which of the following mutations would create new template DNA from which the
shortest peptide would be translated?

Template strand DNA

5-

ATG

GCT

GGC

AAT

CAA

CTA

TAT

TAG

-3

3-

TAC

CGA

CCG

TTA

GTT

GAT

ATA

ATC

-5

13

16

sequence
Nucleotide number

10

A. a deletion of nucleotide number 7.


B. a GC transversion of nucleotide number 9.
C. a GA transition of nucleotide number 13.
D. insertion of -GGT- after nucleotide number 5.
E. a TA transversion at nucleotide number 18.

56

19

22

IBO2010 KOREA
THEORETICAL TEST Part B

B40.2. (1 point) A series of point mutations occurred in a bacterial gene, resulting in the substitution of
amino acid residues in the order shown in the diagram below.

Which amino acid in the diagram can have more than one option for its codon given this particular
process of point mutation?

A. Gly
B. Arg
C. Ile
D. Leu
E. Lys

57

IBO2010 KOREA
THEORETICAL TEST Part B

B41. (2 points)

Suppose you have a population of flour beetles with 1,000 individuals. Normally the

beetles are a red color; however, this population is polymorphic for a mutant autosomal body color,
black, designated by b/b. Red is dominant to black, so B/B and B/b genotypes are red. Assume the
population is in HardyWeinberg equilibrium, with f(B) = p = 0.5 and f(b) = q = 0.5.

B41.1. (1 point) What would be the expected B and b allele frequencies, respectively, if 1,000 black
individuals migrated into the population? (Assume that all other HardyWeinberg conditions were
met.)

B41.2. (1 point) What would be the frequencies of B and b alleles respectively, if a population
bottleneck occurred and only four individuals survived: one female red heterozygote and three
black males?

58

IBO2010 KOREA
THEORETICAL TEST Part B

ECOLOGY

B42. (2 points)

Island biogeography theory states that the number of species on an island is determined

by immigration rates of new species to the island and extinction rates of species on the island.
Immigration rates to an island decline as its distance from the mainland increases, and extinction rates
decrease with increasing island size. When the immigration and extinction rates on an island are equal,
the number of species on the island reaches equilibrium.

Give the correct equilibrium number of species (S1~S4), in the answer sheet, for each of four islands
with different combinations of distance (near and far) and area (small and large) as shown in the
figure above.

59

IBO2010 KOREA
THEORETICAL TEST Part B

B43. (2 points)

The contents of 3 soils (a, b, and c) were examined for soil pH and amounts of acidic

cations (H+, Al3+) and other cations (Ca2+, Mg2+, K+, Na+). The figure below shows the results of that
examination: the white and shaded portions of each column represent the amount of acidic and other
cations, respectively. (Values given are in units of centimoles/kg.)

For each description below, indicate with a checkmark () whether it is true or false.
Description
I. Aluminum toxicity tends to be most severe in soil a.
II. Soil b contains the most nutrient minerals plants can use.
III. Anions such as NO3- and PO4- tend to be retained in soil more than cations are retained.
IV. As more H+ displaces other cations, the soil becomes more acidic.

60

IBO2010 KOREA
THEORETICAL TEST Part B

B44. (2.2 points)

The figure below shows standing biomass pyramids of two ecosystems, each with

four trophic levels.

B44.1. (1.2 point) Which of the following explanations are true or false? Indicate with a checkmark ()
in the answer sheet.
Explanation
I.

Pyramid a reflects energy losses due to respiration within trophic levels and energy
losses during energy transfer between trophic levels.

II.

Pyramid b represents an ecosystem with fast turnover in the primary producer level.

III.

For each ecosystem, its energy pyramid is opposite to its biomass pyramid.

IV.

For both ecosystems, production efficiency becomes higher as the trophic level
increases.

B44.2. (1 point)

Assuming an ecological efficiency of 10% between trophic levels, how much net

primary productivity is required to harvest 2 g C/m2 annually from the tertiary consumer level?

61

IBO2010 KOREA
THEORETICAL TEST Part B

B45. (2.8 points)

Recent changes in the mean global temperature are largely attributed to increases in

levels of some atmospheric gases and aerosols (small particles suspended in air), many of which have
been generated by human activities.

B45.1. (0.8 point) Evaluate whether the following statements are true or false in relation to the role of
these gases and aerosols changing global temperature.
I.
II.

These gases scatters short-wave radiation emitted from the sun.


These gases absorb and re-radiate infrared radiation emitted from the earths
surface.

III.

Aerosols prevent heat convection into space.

IV.

Regardless of the presence of gases or aerosols, solar radiation itself has increased
recently.

62

IBO2010 KOREA
THEORETICAL TEST Part B

B45.2. (2 points) For each statement below, choose from the following list of gases the one that is most
likely to be related to that description.

<List of gases>
a. Hydrofluorocarbons (HFCs)
d. N2

b. CH4

c. CO2

e. O3

f. N2O

Description
I. The gas largely derived from fossil fuels and clearing of forests that contributes the
most to global warming.
II. The gas with the highest global warming potential (compared to CO2).
III. A gas that in the stratosphere is essential to support human life on earth, while in the
troposphere it exerts harmful effects on humans.
IV. A gas that is not thought to contribute to global warming.
V. A gas derived from landfills and the livestock sector that has increased most rapidly in
the past 200 years.

63

IBO2010 KOREA
THEORETICAL TEST Part B

B46. (2 points) The picture shows a schematic representation of the production of three well-known trees
of a deciduous forest. The production is indicated in kg dry mass per hectare per year.

Calculate how much of the total production comes from above ground woody parts.
Give your answer as a percentage (%) rounded to an integer (without any decimals).

64

IBO2010 KOREA
THEORETICAL TEST Part B

BIOSYSTEMATICS
B47. (2 points)

Figures a and b show the characteristics of a cactus from the American desert and a

spurge from the African desert, respectively. An evolutionary mechanism has been proposed to
explain the morphological similarities between these nonrelated species. That same evolutionary
mechanism has been reported to operate at the DNA sequence level.

a. Cactus

b. Spurge

c. Molecular evolution model

Which of the molecular evolutionary trees shown in Figure c is the best molecular model of the
morphological evolutionary mechanism that we observe in the cactus and spurge? The symbols A, C,
G, and T on the molecular evolutionary trees represent DNA bases.
A. Tree (a), 1 with 4.
B. Tree (b), 1 with 2.
C. Tree (c), 1 with 5.
D. Tree (c), 2 with 3.
65

B48. (2 points)

The following figure represents a recent phylogenetic tree for the animal kingdom.

Carefully observe the tree topology, and answer the following questions.

B48.1. (1 point) What are the most appropriate synapomorphic characters for the numbers (1) and (2),
respectively? Mark appropriate boxes with a checkmark ().
A. Segmented body
B. True tissue differentiation
C. Embryogenesis
D. Bilateral symmetry
E. Exoskeleton development

66

IBO2010 KOREA
THEORETICAL TEST Part B

B48.2. (1 point) Which of the following groups are members of Deuterostomia (taxon number (3))?
A. Echinodermata, Arthropoda.
B. Echinodermata, Chordata.
C. Mollusca, Arthropoda.
D. Annelida, Mollusca.
E. Chordata, Mollusca.

67

IBO2010 KOREA
THEORETICAL TEST Part B

B49. (2 points) The Influenza A virus is responsible for annual flu epidemics and for occasional flu
pandemics. Influenza A has a genome composed of eight RNA strands that encode a total of 11
proteins. Influenza A strains can be classified based on the combination of two coat proteins,
Hemagglutinin (H1~H13) and Neuraminidase (N1~N9). In this way, various flu types such as H1N1,
H3N1, H7N2, etc. can be recognized. The virus strains also can be classified by the host animal. The
following figure represents the phylogeny of flu viruses based on the nucleoprotein gene of the flu
virus genome. Indicated for each viral strain is the host species from which it was isolated, the year,
and the type of Hemagglutinin and Neuraminidase it carries. Indicate with checkmarks ().

whether

following statements are true or false.

I.

The avian flu virus consists of the most diverse types, and some avian flu types also are
found in some mammalian species such as whales and dolphins. Therefore, the avian flu
virus represents the most archaic type of flu virus.

II.

The phylogenetic tree suggests that the host shift and genetic recombination of flu virus
have occurred between birds and pigs.

III.

The virulence of virus can be changed rapidly by host shifts and mutations. Therefore,
vaccine developments are relatively difficult compared to other common diseases.

IV.

Swine flu strains are phylogenetically more closely related to the human flu strains than to
other strains.

68

IBO2010 KOREA
THEORETICAL TEST Part B

69

IBO2010 KOREA
THEORETICAL TEST Part B

B50. (1.5 points)


plants.

Following table summarizes the main characteristics of the four major phyla of seed

Check () in the answer sheet whether each characteristics is absent () or present (+) for

A~E.
Development of
Character

Flagellated

Double

Vessel in

Flowers

sperm

fertilization

xylem

and fruits

the secondary
Phylum

xylem
Cycadophyta

Ginkgophyta

Pinophyta

Magnoliophyta

70

IBO2010 KOREA
THEORETICAL TEST Part B

B51. (2.4 points)

All organisms use carbon as well as energy in order to live and function. Organisms

can be divided into four nutrition modes based upon the species main sources of energy and of
carbon.

B51.1. (1.2 points) From the following list of nutrition modes, fill in the answer sheet with the correct
term corresponding to each combination of carbon and energy source.

<Nutrition mode>
I. Photoautotroph,

II. Chemoautotroph,

III. Photoheterotroph,

IV. Chemoheterotroph.

B51.2. (1.2 points) From the list of organisms provided, choose two organisms belonging to each
nutrition mode.
<Organisms>
a. Cyanobacteria,

b. Green nonsulfur bacteria,

c. Purple nonsulfur bacteria,

d. Fungi,

e. Most archaebacteria,

f. Most plants,

g. Animals,

h. Nitrifying bacteria.

71

IBO2010 KOREA
THEORETICAL TEST Part B

B52. (2 points)

The following figure represents a recent phylogeny for the plant kingdom.

For each number (1)~(4), select the appropriate apomorphic trait from the list provided.

< Apomorphic traits >


A. Leaves with well-developed vascular bundles,
B. Embryos,
C. Seeds,
D. Vascular tissues,
E. Phragmoplast.

72















All IBO examination questions are published under the following Creative Commons license:



CC BY-NC-SA (Attribution-NonCommercial-ShareAlike) https://creativecommons.org/licenses/by-nc-sa/4.0/
The exam papers can be used freely for educational purposes as long as IBO is credited and
new creations are licensed under identical terms. No commercial use is allowed.

Country Code: ___________

Student Code: ___________

Theoretical Test Part B


Answer Key
B1 (2.7 points)

B1.1 (1.5 points = 0.3 5)

B1.2 (1.2 points = 0.3 4)

B2. (2.7 points = 0.3 9)

B
C

B3. (1.5 points = 0.3 5)

Organ and tissue

Choose from a~e

Brain

Liver

Heart muscle

Skeletal muscle

Adipose tissue

B4. (2.2 points)


B4.1 (1 point)

B4.2 (1.2 points = 0.3 4)

Mutant

Able to show pattern

Unable to show pattern

II

III

IV

B5. (1.5 points = 0.5 3)

Description

True

II

III

false

B6. (2 points = 0.5 4)

Description

True

False

I
II

III

IV

B7. (2 points)
B7.1 (1 points = 0.2 5)

Explanation

True

II

false

III
IV

B7.2 (1 point = 0.2 5)

Description

True

II

III

False

IV

B8. (1.5 points = 0.5 3)

Explanation

True

False

B9. (2 points)

B10. (1.5 points = 0.5 3)

True

False

B11. (2 points)

1.0, 3.5 kb

B12. (1.5 points = 0.5 3)

Graphs

Cellular activity
and response

I
II
III

B13. (2 points = 0.5 4)

Description

True

I
II

III
IV

False

B14. (1.5 points = 0.5 3)

Description

True

False

II

III

B15. (3 points)
B15.1 (1.2 points = 0.3 4)

Property

Prokaryote

Eukaryote

II
III

IV

B15.2 (1.8 points = 0.3 6)

Recombinant gene

Cellular location of

Observed polypeptides

expressed proteins
I-II-III

I-III

II-III

III

B16. (1.5 points = 0.5 3)

Gene mutation

Callus phenotype
A

II

III

B17. (2.4 points = 0.4 6)

Characterization

Cell type (1-6)

Initials for this cell type (7-11)

A. Origin of root hairs

11

B. Storage parenchyma

C. Perception of gravity

10

D. Origin of lateral roots

B18. (1.5 points = 0.5 3)

True
I

II

False

III

IV

B19. (1.8 points = 0.3 6)

Function

I
II

III

IV
V

VI

10

B20. (2.2 points)


B20.1 (1.2 points = 0.3 4)

Labels in

Types of joint

the Figure

B20.2 (1 point = 0.5 2)

Function

true

False

II

11

B21. (2.4 points = 0.3 8)

B21.1 (1.2 points = 0.3 4)

B21.2 (1.2 points = 0.3 4)

Morphological character (1~9)

Character in figure (a~g)

B22. (2 points = 0.5 4)

Number in the Graph

Heart
E
C
A
D

12

B23. (1.5 points = 0.5 3)

Explanation

True

False

B24. (1.8 points = 0.3 6)

Symptom

Expected

B
C

E
F

Unexpected

13

B25. (1.5 points = 0.5 3)

Blood vessel
Description
A
I

II

III

B26. (3 points)
B26.1 (1 point)

B26.2 (1 point)

14

B26.3 (1 point = 0.2 5)

Description

True

II

III

False

IV

B27. (3 points)
B27.1 (1 point)

50

m/sec

B27.2 (1 point)

B27.3 (0.5 point)

d
15

B27.4 (0.5 point)

B28. (2.7 points)


B28.1 (1.8 points = 0.3 6)

True
I

II
III

IV

V
VI

False

16

B28.2 (0.9 point = 0.1 9)


Adaptation for flight

No adaptation for flight

(a)

(b)

(c)

(d)

(e)

(f)

(g)

(h)

(i)

B29. (3 points)
B29.1 (1 point)

112

17

B29.2 (1 point = 0.5 2)

(i)

20

(ii)

12.4

B29.3 (1 point)

B30. (2.6 points)


B30.1 (1 point)

B30.2 (1.6 points = 0.4 4)

18

B31. (1.5 points = 0.3 5)

True

False

II

III

IV

B32. (2 points = 0.4 5)

Mutant

Male

Female

19

B33. (2.4 points = 0.4 6)

True

False

II

III

IV
V

VI

B34. (3 points)
B34.1 (0.9 point = 0.3 3)

True

False

I
II

III

B34.2 (0.8 point)

4
20

B34.3 (1.3 points)

20

B35. (2 points)
B35.1 (1 point)

B35.2 (1 point)

Possible

Impossible

B36. (2 points)
B36.1 (1 point)

21

B36.2 (1 point)

B37. (2 points)

22

B.38 (2 points = 0.4 5)

Description

True

False

A
B

22

B39. (3 points)
B39.1 (1 point = 0.2 5)

Required
A

Not required

B39.2 (1 point)

B39.3 (1 point)

23

B40. (2 points)
B40.1 (1 point)

B40.2 (1 point)

B41. (2 points)
B41.1 (1 point)

0.25

0.75

B41.2 (1 point)

0.125

0.875

24

B42. (2 points = 0.5 4)

Small island

Large island

Island near mainland

S3

S4

Island far from mainland

S1

S2

B43. (2 points = 0.5 4)

Description

True

II

III
IV

False

25

B44. (2.2 points)


B44.1 (1.2 points = 0.3 4)

True
I

II

False

III

IV

B44.2 (1 point)

2000

g C/m2

B45. (2.8 points)


B45.1 (0.8 point = 0.2 4)

True

I
II

False

III

IV

26

B45.2 (2 points = 0.4 5)

Description

Gas (a~f)

II

III

IV

B46. (2 points)

49

B47. (2 points)

27

B48. (2 points)
B48.1 (1 point)

(1)

(2)

B48.2 (1 point)

B49. (2 points = 0.4 5)

True

False

I
II

III

IV

28

B50. (1.5 points = 0.3 5)

B51. (2.4 points)


B51.1 (1.2 points = 0.3 4)

En

Energy source
Oxidation of inorganic molecules

Light

CO2

II

Organic molecules

IV

III

Carbon source

B51.2 (1.2 points = 0.3 4)

Organisms

Nutrition mode
nutrition
I

II

III

IV

29

B52. (2 points = 0.5 4)

(1)

(2)

(3)

(4)

30

INTERNATIONAL BIOLOGY OLYMPIAD


PRACTICAL PROBLEMS

2010, Changwon, Korea















All IBO examination questions are published under the following Creative Commons license:



CC BY-NC-SA (Attribution-NonCommercial-ShareAlike) https://creativecommons.org/licenses/by-nc-sa/4.0/
The exam papers can be used freely for educational purposes as long as IBO is credited and
new creations are licensed under identical terms. No commercial use is allowed.

IBO 2010 KOREA


PRACTICAL TEST 1
PLANT AND ANIMAL SYSTEMATICS
_________________________________________________________________________________

Country Code: ___________

Student Code: ___________

The 21st INTERNATIONAL BIOLOGY OLYMPIAD


11th 18th July, 2010
Changwon, KOREA

PRACTICAL TEST 1
PLANT AND ANIMAL SYSTEMATICS

Total Points: 50
Duration: 90 minutes

IBO 2010 KOREA


PRACTICAL TEST 1
PLANT AND ANIMAL SYSTEMATICS
_________________________________________________________________________________

Dear Participants,

In this test, you have been given the following 3 tasks:


Task I: Reconstruct the phylogenetic tree of six plant species using parsimony method
(25 points)
Task II: Reconstruct the phylogenetic tree of six insect species using the UPGMA method
(18 points)
Task III: Co-evolution between plants and insects (7 points)

Write down your results and answers in the Answer Sheet. Answers written in the Question
Paper will not be evaluated.

Please make sure that you have received all the materials listed for each task. If any of the listed
items is missing, please raise your hand.

Stop answering and put down your pencil immediately after the end bell rings. The supervisor will
collect the Question Paper and the Answer Sheet.

Good Luck!!

IBO 2010 KOREA


PRACTICAL TEST 1
PLANT AND ANIMAL SYSTEMATICS
_________________________________________________________________________________

PLANT AND ANIMAL SYSTEMATICS

This practical test is composed of 3 tasks.

TASK I. (25 points) Reconstruct the phylogenetic tree of six plant species
using parsimony method
This task is composed of 4 parts.
Plant taxa (species)
A, B, C, D, E, and F.

Plant materials
Each set consists of the following materials:
1) The flowers of six species preserved in 70% ethanol (Flowers A-F).
2) The fruits of six species preserved in 70% ethanol (Fruits A-F).
3) Six dried flowering specimens (Flowering specimens A-F).
4) Six dried fruiting specimens (Fruiting specimens A-F).
5) Three prepared pollen slides (Each slide contains the pollen grains of two species,
labeled A-B, C-D and E-F.).

Instruments
Stereomicroscope (20X), microscope (400X), razor blade, dissecting forceps, dissecting
needles (2), petri-dishes (2), 20-cm ruler.

IBO 2010 KOREA


PRACTICAL TEST 1
PLANT AND ANIMAL SYSTEMATICS
_________________________________________________________________________________

Part I-1. (9 points) Using the suggested materials and methods in Table 1, observe the
following 10 characters. Enter each character state in the Data Matrix 1. Each
character state should be recorded as the appropriate number (0, 1, or 2) based
on the following descriptions. Figure 1 is provided as a reference of the
descriptive terminology.

IBO 2010 KOREA


PRACTICAL TEST 1
PLANT AND ANIMAL SYSTEMATICS
_________________________________________________________________________________

Table 1. Character descriptions for plants (See Figure 1 for illustration of the character states for
the character 1, 2, 4, 7, 8, 9 and 10.)
No
1.

Character
Flower petal

2.

1:

Polypetalous

0:

Corymb or umbel

1:

Raceme

2:

Axillary or terminal

0:

Longer than 1 cm

1:

Shorter than 1 cm

0:

Superior

1:

Half-inferior

2:

Inferior

0:

Shrub

1:

Tree

Trichomes on
the fruit surface

0:

Hairless

1:

Densely haired

Fruit shape and


size at maturity

0:

0:

Cylindrical or circular
with emarginated tip
(Dia.<1 cm)
Circular with pointed tip
(Dia.< 1 cm)
Circular-elliptic with
pointed tip (Dia. 1 cm)
Pome or capsule

1:

Drupe

0:

Tetrad

1:

Monad

0:

Entire or undulate

1:

Serrate or dentate

Inflorescence

3.

Fruit stalk

4.

Ovary position

5.

Habit

6.

7.

Character state
0: Sympetalous

1:
2:
8.

Fruit types

**9

Pollen grains

.
10.

*If
**

Leaf margin at
maturity

Materials
Flowers

Methods
Naked eye

Flowering specimens

Naked eye

Fruiting specimens

Naked eye

Flowers

Dissect with
razor blade,
Stereomicroscope (20X)

Given answer

Given answer

Fruiting specimens

Naked eye

Fruits and Fruiting


specimens

Naked eye

Fruits

*Dissect

Pollen slides

Fruiting specimens

with
razor blade,
Naked eye
Microscope
(400X)
Naked eye

the endocarp is hard, carefully remove exocarp and mesocarp to identify the fruit type.

Search pollen grains within a red circle of each specimen.

IBO 2010 KOREA


PRACTICAL TEST 1
PLANT AND ANIMAL SYSTEMATICS
_________________________________________________________________________________

Figure 1. Illustration of character states for the character 1, 2, 4, 7, 8, 9 and 10.

IBO 2010 KOREA


PRACTICAL TEST 1
PLANT AND ANIMAL SYSTEMATICS
_________________________________________________________________________________

Figure 1. continued

IBO 2010 KOREA


PRACTICAL TEST 1
PLANT AND ANIMAL SYSTEMATICS
_________________________________________________________________________________

Q1. (9 points) Fill the empty cells of Data Matrix 1 on your Answer Sheet. The sheet consists of
6 x 10 cells. Taxon F and character 5 are already filled for your reference.
Character

10

Taxa

Part I-2. (4 points) Refer to your completed Data Matrix 1 to answer the following questions
Q2.1. (1 point) Indicate with checkmarks () which of the characters are phylogenetically
informative.

Q2.2. (1 point) Indicate with checkmarks () which of the characters are polymorphic (more
than two states).

Q2.3. (2 points = 1 2) Use the following numerical formulae to define the number of
possible trees for a given number of taxa (n).
The number of possible unrooted trees = (2n-5)!/2n-3(n-3)!
The number of possible rooted trees = (2n-3)!/2n-2(n-2)!
The symbol ! in the formulae indicate the factorial.
What is the numbers of rooted and unrooted trees in this case?

IBO 2010 KOREA


PRACTICAL TEST 1
PLANT AND ANIMAL SYSTEMATICS
_________________________________________________________________________________

Part I-3. (8 points) Cladistic analysis can be used to construct a phylogenetic tree of this
species group. The primitive character state (plesiomorphy) is hypothesized to be
the same as the state found in the outgroup F. Therefore, the character state 0
represents the primitive state for all given characters. Any change in state from
that primitive trait is considered to be a derived character state, representing an
evolutionary event (apomorphy). The character states of 1 and 2 represent
derived condition. In this analysis, all characters are given equal weight. Tree
construction is done in a step-by-step process. Place the appropriate character
numbers and group members on the tree at each step.

Q3.1. (1 point) The initial Tree 1 can be created if we separate the ingroup (A, B, C, D, E)
from the outgroup (F) using the two characters that distinguish all members of the
ingroup (shared derived traits) from the outgroup F.

Identify these two characters (a and b) shown in Tree 1 and write them in the Answer
Sheet. Character state should be given in parenthesis if the character is a polymorphic
one.

IBO 2010 KOREA


PRACTICAL TEST 1
PLANT AND ANIMAL SYSTEMATICS
_________________________________________________________________________________

Q3.2. (2 points) The step-by-step method to create the final tree from this initial tree can be
illustrated by the concept of membership. Analysis proceeds to progressively less
inclusive groupings supported by other derived characters. A less inclusive group(s) can
be separated from the more inclusive group by the supported character change(s) on the
tree. In the second step, the five ingroup taxa can be further divided into two subgroups
(GI and GII) based on three synapomorphic characters.

Identify the subdivided species group members of GI and GII and the three character
numbers (c, d and e) shown on Tree 2 and write them in the Answer Sheet. Character
state should be given in parenthesis if the character is a polymorphic one.

Q3.3. (3 points) The group II (GII) can be further divided into two less inclusive subgroups
(GII1 and GII2) by four and one synapomorphic character(s), respectively.

Identify the members of GII1 and GII2 and write the corresponding character numbers
(shown in locations f-j in Tree 3) in the Answer Sheet. Character state should be given in
parenthesis if the character is a polymorphic one.

10

IBO 2010 KOREA


PRACTICAL TEST 1
PLANT AND ANIMAL SYSTEMATICS
_________________________________________________________________________________

Q3.4. (2 points) At the final stage of tree construction, all autapomorphic (singly derived)
characters should be located on the tree, and any conflicting characters should be
adjusted using the parsimony principle. There are two autapomorphic characters (l and
m) and only a single conflicting character (k) in this case.

List the taxon name for each of the five ingroup species on the fully resolved Tree 4, and
give the character numbers that correspond to k, l and m, respectively, in the Answer
Sheet. Character state should be given in parenthesis if the character is a polymorphic
one.

11

IBO 2010 KOREA


PRACTICAL TEST 1
PLANT AND ANIMAL SYSTEMATICS
_________________________________________________________________________________

Part I-4. (4 points) Use the complete phylogenetic tree to answer the following questions.

Q4.1. (1 point) What is the number of character changes (steps) on the maximum parsimonious
tree?

Q4.2. (1 point) The consistency index (CI) is defined as the minimum number of character
state changes required in an absolutely consistent tree (all character states changed only
once) divided by the observed number of character state changes in the final tree.
What is the CI of the final Tree 4?

Q4.3. (1 point) What is the maximum number of genera that can be recognized from the final
tree if taxa C and D are congeneric species?

Q4.4. (1 point) How many monophyletic groups can be recognized from the final tree?

12

IBO 2010 KOREA


PRACTICAL TEST 1
PLANT AND ANIMAL SYSTEMATICS
_________________________________________________________________________________

TASK II. (18 points) Reconstruct the phylogenetic tree of six insect species
using the Unweighted Pair Group Method with Arithmetic mean
(UPGMA) method

This task is composed of 3 parts.

Insect taxa: Six beetles (Coleoptera)


T1, T2, T3, T4, T5 and T6

Insect materials
Pinned and dried specimens of six beetles, labeled T1~T6.

Experiment tools
Insect stage, ruler, stereomicroscope (20X)
Note: Please be careful. Do not break the legs or antennae of the beetle specimens. There will
be three point deduction penalty if you break the parts of any specimen.
Most insect specimens and their parts can be observed directly from the plastic case
after removing the cover.

13

IBO 2010 KOREA


PRACTICAL TEST 1
PLANT AND ANIMAL SYSTEMATICS
_________________________________________________________________________________

Part II-1. (8 points) The character states are defined in Table 2. Carefully observe the
morphological characters of the beetle specimens using the naked eye and the
stereomicroscope. Then, complete the Data Matrix 2. Figure 2 is provided as a
reference for the Coleoptera body parts.
No.

Table 2. Character descriptions for Coleoptera


Character
Character state
1.

Longitudinal
discontinuous
ridges on elytra

0: Present

Methods
Stereomicroscope

1: Absent
2.

Horns on head
and pronotum

0: Absent

Naked eye

1: Present
3.

Compound eye

4.

Mandible length

5.

Antennae length

0: Does not surround


antennal socket
1: Surrounds about half of
antennal socket
0: Shorter than prothorax
length
1: Longer than prothorax
length
0: Shorter than body length

Stereomicroscope

Naked eye, stereomicroscope if the


part is small

Naked eye

1: Longer than body length


6.

7.

Antennae shape

0: Filiform or serrate

Antennae

1: Distal segments clubbed


or lamellated
0: Not elbowed

Naked eye, stereomicroscope if the


part is small

Naked eye

1: Elbowed
8.

Hind tarsi

0: 5 segments

Stereomicroscope

1: 4 segments or less
*9.

Notopleural
sutures of the
prothorax

0: Fused

Given answer

1: Not fused
*10.

1st sternum and


hind coxa

0: Separated

Given answer

1: Not separated
*11.

Food preference

0: Zoophagy or
saprophagy
1: Phytophagy

Given answer

* Character states are provided in the Answer Sheet.

14

IBO 2010 KOREA


PRACTICAL TEST 1
PLANT AND ANIMAL SYSTEMATICS
_________________________________________________________________________________

Figure 2. The body parts of Coleoptera to be observed.

Q5. (8 points) Complete the Data Matrix 2 in your answer sheet. The sheet consists of 6 x 11
cells. Characters 9, 10, and 11 are already filled in.
Data matrix 2.
Character
C1

C2

C3

C4

C5

C6

C7

C8

C9

C10

C11

T1

T2

T3

T4

T5

T6

Taxa

15

IBO 2010 KOREA


PRACTICAL TEST 1
PLANT AND ANIMAL SYSTEMATICS
_________________________________________________________________________________

Part II-2. (3 points) Create a character difference matrix between all possible pair-wise
taxa from the completed Data Matrix 2. The difference value (Dij) between
taxon i and j is the sum of the character (C) numbers for which Ci Cj.
Calculate the difference values, and fill the table below (Difference Matrix 1).
The values of three pair-wise comparisons (T1/T2, T3/T4, and T5/T6) are
already provided as examples.

Q6. (3 points) Complete the following Difference Matrix 1 on your answer sheet.

Difference Matrix 1. The pair-wise difference matrix calculation.


Dij

T1

T2

T3

T4

T5

T6

T1

T2

T3

T4

T5

T6

16

IBO 2010 KOREA


PRACTICAL TEST 1
PLANT AND ANIMAL SYSTEMATICS
_________________________________________________________________________________

Part II-3. (7 points) Construct a phylogenetic tree based on the UPGMA algorithm using
the pair-wise difference matrix that you created in Part 2. During the procedure,
you will create nested clusters of taxa (smaller clusters into larger clusters) using
successive difference matrices and phenograms until you construct a single
cluster. The order for clustering is: 1) Pick the smallest entry Dij. 2) Join those
two species into a cluster, 3) Compute new distances from that cluster to the
other taxon k, using the Unweighted Pair Group Method with Arithmetic mean
(UPGMA). A new distance between a new species k and a cluster (i and j) is
defined as D(k(ij)) = (1/2)((D(ki)+D(kj)). Repeat the clustering process 1) ~ 3) to
create the next cluster. This process should be continued to construct a single
cluster for the entire group.

Q7.1. (1 point) Two alternate initial trees (a and b) can be constructed from the Difference
Matrix 1 as shown below. Combine the two alternative trees and draw as a single tree
(Tree 1). Draw Tree 1 on your Answer Sheet.

Initial Trees a and b: (T1, T2, T3, T4, (T5, T6)) or (T1, T2, (T3, T4), T5, T6)
a

17

IBO 2010 KOREA


PRACTICAL TEST 1
PLANT AND ANIMAL SYSTEMATICS
_________________________________________________________________________________

Q7.2. (2 points) Complete the Difference Matrix 2. Calculate new difference values
between cluster and taxon (or between cluster and cluster) using UPGMA algorithm and
fill the cells in the answer sheet. Find the taxon pair(s) that shows the lowest difference
values and make a newly clustered tree (Tree 2). Draw the tree in the answer sheet.
Difference Matrix 2:
Dij or Dk(ij)

T1

T2

T(3,4)

T(5,6)

T1

T2

T(3,4)
T(5,6)

Q7.3. (2 points) Complete the Difference Matrix 3. Again calculate the new difference
values between cluster and taxon (or between cluster and cluster) using UPGMA
algorithm and fill the cells in the answer sheet. Find the taxon pair that shows the lowest
difference values and make a newly clustered tree (Tree 3). Draw the tree in the answer
sheet.

Difference Matrix 3:
Dij or Dk(ij)
-

18

IBO 2010 KOREA


PRACTICAL TEST 1
PLANT AND ANIMAL SYSTEMATICS
_________________________________________________________________________________

Q7.4. (2 points) Complete the Difference Matrix 4. Calculate the new difference values
between cluster and cluster using UPGMA algorithm and fill the cells in the answer
sheet. Make a complete clustered tree (Tree 4) and draw it in the answer sheet.

Difference Matrix 4:
Dij or Dk(ij)
-

19

IBO 2010 KOREA


PRACTICAL TEST 1
PLANT AND ANIMAL SYSTEMATICS
_________________________________________________________________________________

TASK III. (7 points) Co-evolution between plants and insects


Plant-herbivore relationships are one of the core explanations for the rapid diversification of
insects and flowering plants. For this task, use the final plant phylogeny (Task 1) and beetle
phylogeny (Task 2). Under the assumption that the larvae of each insect species feed on a single
plant species, compare the insect and plant phylogenies and answer the following questions.

Q8. (3 points) If the insect species T2, T3, and T5 feed on the plant species E, D, and A,
respectively, what kinds of plant species are fed upon by insect species T1, T4, and T6,
respectively?

Q9. (2 points) Which plant and insect species pair shows different phylogenetic positions in the
trees?

Q10. (2 points) Which are the two best possible reasons to explain the differences between the
insect and plant phylogenies? (Select the two best answers).
A. Host shift of insect species
B. Adaptive radiation of plant species
C. Genetic bottleneck during the insect species evolution
D. Different tree reconstruction methods
E. Genetic drift of plant species

20

Country Code: ___________

Student Code: ___________

PRACTICAL TEST 1
Answer Sheet
PLANT AND ANIMAL SYSTEMATICS
Total Points: 50
Duration: 90 minutes

TASK I. (25 points)


Part I-1. (9 points)
Q1. (9 points)
Data Matrix 1. (One point per character)
Character
Taxa

10

10

Part I-2. (4 points) Place "" symbol on the right characters.

Q2.1. (1 point)
Character
Number

Phylogenetically
informative

Q2.2. (1 point)
Character
Number

10

Polymorphic
Q2.3. (2 points = 1 x 2)
The number of possible unrooted trees: ___________
The number of possible rooted trees: ____________

Part I-3. (8 points) Character state should be given in the parenthesis if the character
is polymorphic one.
Q3.1. (1 point)
Character numbers
Character a and b

Q3.2. (2 points). Character number and taxon name will be assign one point, respectively.
Character numbers
Character c, d
and e

Taxon name(s)
GI
GII

Q3.3. (3 points)
Character number(s)

Taxon name(s)

Character f

GII1

Character g, h, i
and j

GII2

Q3.4. (2 points)
Character number

Taxon name

Character k

GI

Character l

GII1a

Character m

GII1b
GII2a
GII2b

Part I-4. (4 points)


Q4.1. (1 point)
steps

Q4.2. (1 point)
CI =

Q4.3 .(1 point)

Q4.4. (1 point)

Task II. (18 points)


Part II-1. (8 points)
Q5. (8 points)
Data Matrix 2
Character
C1
Taxa

C2

C3

C4

C5

C6

C7

C8

C9

C10

C11

T1

T2

T3

T4

T5

T6

Part II-2. (3 points)


Q6. (3 points)
Difference Matrix 1
Dij

T1

T2

T3

T4

T5

T6

T1

T2

T3

T4

T5

T6

Part II-3 (7 points)


Q7.1. (1 point)
Tree 1: Combine and draw the two alternative trees as a single tree
T1

T2

T3

T4

T5

T6

Q7.2. (2 points)
Difference Matrix 2
Dij or Dk(ij)

T1

T2

T(3, 4)

T(5, 6)

T1

T2

T(3, 4)
T(5, 6)

Tree 2
T1

T2

T3

T4

T5

T6

Q7.3. (2 points)
Difference Matrix 3:
Dij or Dk(ij)
-

Tree 3
T1

T2

T3

T4

T5

T6

Q7.4. (2 points)
Difference Matrix 4
Dij or Dk(ij)
-

Tree 4
T1

T2

T3

T4

T5

T6

Task III. (7 points)


Q8. (3 points)
Insect Species

Plant Species

T1
T2

T3

T4
T5

T6

Q9. (2 points)
Insect species

Plant species

Q10. (2 points)
A

PRACTICAL TEST 1
Answer Key
PLANT AND ANIMAL SYSTEMATICS
Total Points: 50
Duration: 90 minutes

TASK I. (25 points)


Part I-1. (9 points)
Q1. (9 points)
Data Matrix 1. (1 point per character) / (0.2 point/box)
Character

10

10

Taxa

Part I-2. (4 points) Place "" symbol on the right characters.

Q2.1. (1 point) (No partial score per character)


Character
Number
Phylogenetically
informative

Q2.2. (1 point) (No partial score)


Character
Number

Polymorphic

10

Q2.3. (2 points = 1 x 2) (1 point per box)


The number of possible unrooted trees:
The number of possible rooted trees:

105
945

Part I-3. (8 points) Character state should be given in the parenthesis if the character
is polymorphic one.
Q3.1. (1 point)

(0.5 point per character number)


Character numbers

Character a and b

1, 9

Q3.2. (2 points)
Character numbers
Character c, d
and e

4(1), 8, 10

Taxon name(s)
GI

GII

A, B, C, D

1. 1 point for Character numbers. (Deduct 0.3 point per wrong answer. Both character
number and state should be correct for characters c,d, and e.)
2. 1 point for Taxon name(s) (0.5 point for GI, 0.5 point for GII)

Q3.3. (3 points)
Character number(s)

Taxon name(s)

Character f

7(1)

GII1

A, B

Character g, h,
i and j

2(2), 3, 6, 7(2)

GII2

C, D

1. 1 point for Character number(s) of character f. (no partial score.)


2. 1 point for Character number(s) of character g~j. (0.25 point per answer. Both
character number and state should be correct.)
3. 1 point for Taxon name(s). (0.5 point for GII1, 0.5 point for GII2)
Q3.4. (2 points)
Character number

Taxon name

Character k

GI

Character l

4(2)

GII1a

Character m

2(1)

GII1b

GII2a

GII2b

1. 1 point for Character number(s) of characters k~m. (Deduct 0.3 point per wrong
answer. Both character number and state should be correct.)
2. 1 point for Taxon name. (0.2 point per box.)
Part I-4. (4 points)
Q4.1. (1 point)
14

steps

Q4.2. (1 point)
CI = 13/14
1. Any decimals possible. (0.92857..)
Q4.3. (1 point)
4

Q4.4. (1 point)
5

Task II. (18 points)


Part II-1. (8 points)
Q5. (8 points)
Data Matrix 1
Character
C1
Taxa

C2

C3

C4

C5

C6

C7

C8

C9

C10

C11

T1

T2

T3

T4

T5

T6

1. 1 point per each character. (Deduct 0.15 point per box with wrong number.)

Part II-2. (3 points)


Q6. (3 points = 0.25 12)
Difference Matrix 1
Dij

T1

T2

T3

T4

T5

T6

T1

T2

T3

T4

T5

T6

Part II.3 (7 points)


Q7.1. (1 point) (No partial score)
Tree 1: Combine and draw the two alternative trees as a single tree.

Q7.2. (2 points )
Difference Matrix 2

Tree 2

(1 point = 0.2 5)

Dij or Dk(ij)

T1

T2

T(3,4)

T(5,6)

T1

T2

T(3,4)

T(5,6)

(1 point) (No partial score)

Q7.3. (2 points)
Difference Matrix 3

(1 point) (Deduct 0.1 point per box with wrong answer.)

Dij or Dk(ij)

T1

T(2(3,4))

T(5,6)

T1

T(2(3,4))

5.5

T(5,6)

4.5

Tree 3

(1 point) (No partial score)

Q7.4. (2 points)
Difference Matrix 4

(1 point = 0.2 5)

Dij or Dk(ij)

T1

T(2(3,4),(5,6))

T1

T(2(3,4),(5,6))

5.75

Tree 4 (1 point) (No partial score)

Task III. (7 points)


Q8. (3 points = 1 3)
Insect Species

Plant Species

T1

T2

T3

T4

T5

T6

Q9. (2 points = 1 2)
Insect species

Plant species

T2

Q10. (2 points)
A

(No partial score)


C

IBO 2010 KOREA


PRACTICAL TEST 2
PHYSIOLOGY AND ANATOMY
_________________________________________________________________________________

Country Code: ___________

Student Code: ___________

The 21st INTERNATIONAL BIOLOGY OLYMPIAD


11th 18th July, 2010
Changwon, KOREA

PRACTICAL TEST 2
PHYSIOLOGY AND ANATOMY
Total Points: 49
Duration: 90 minutes

IBO 2010 KOREA


PRACTICAL TEST 2
PHYSIOLOGY AND ANATOMY
_________________________________________________________________________________

Dear Participants,

In this test, you have been given the following 2 tasks:


Task I: The response of the rat cerebral cortex to skin stimulation (25 points)
Task II: Anatomy of spider (24 points)

Write down your results and answers in the Answer Sheet. Answers written in the Question
Paper will not be evaluated.

Please make sure that you have received all the materials listed for each task. If any of the listed
items is missing, please raise your hand.

If you have any problem with your computer, raise your hand.

Stop answering and put down your pencil immediately after the end bell rings. The supervisor will
collect the Question Paper and the Answer Sheet.

Note: All animals used in the pictures and the described experiments were treated according
to guidelines approved by the institutional animal care and use committee and
conformed to the NIH guidelines on care and use of animals in research.

IBO 2010 KOREA


PRACTICAL TEST 2
PHYSIOLOGY AND ANATOMY
_________________________________________________________________________________

PHYSIOLOGY AND ANATOMY

This practical test is composed of 2 tasks.

TASK I. (25 points) The response of the rat cerebral cortex to skin stimulation

Welcome to the Electro-Physiology Laboratory!


Today you are going to examine one of the principles of how the brain works. This test is
composed of 4 parts: one background section on how electrophysiological experiments are
conducted and three experimental sections. You are required to answer a total of 15 questions by
analyzing data presented on screen.
The home-page photo of the notebook computer shows the tools and equipment used in an
electrophysiology laboratory.

Press

above the photo

The primary somatosensory (S1) cortex receives tactile information from a specific body
surface region. These specialized receptive areas in the human brain is shown in Figure 1. A
similar body representation within the rat S1 (Fig. 2) will be created from these experiments.

Press

or

IBO 2010 KOREA


PRACTICAL TEST 2
PHYSIOLOGY AND ANATOMY
_________________________________________________________________________________

1.

Background information
1.1 Skull immobilization with brain exposure
The stereotaxic device is used to immobilize the skull (Fig. 3). The incisor bar is adjusted to

make the skull surface horizontal (Fig. 4). Following a scalp incision, a hole is drilled in the skull
over the location of S1, and a recording electrode (a red, moving needle) is inserted into the brain
(Fig. 5). A micro-driver is used to move the electrode downward (25 m/step) from the surface
into the brain (Fig. 6).

Press

or

1.2 S1 neuronal response following skin stimulation


The rat skin can be stimulated either mechanically with a cotton probe or electrically with an
electrode. Following the electrical stimulation of forepaw digit (Fig. 7; a white, moving arrow),
S1 neuronal activity is recorded using an electrode (Fig. 7; a red, moving needle). Using an
oscilloscope (Fig. 8), S1 neuronal activity can be visualized (Fig. 9).

Press

or

1.3 Response histogram


When an S1 neuron is responsive to the stimulation of a body part, the body part is within the
receptive field (RF) of the neuron; a neuron does not show any response to the stimulation of
body parts outside of its specified RF.
Using the amplifier (Fig. 10) and the analyzer (Fig. 11), activities of many S1 neurons
surrounding the electrode can be recorded (Fig. 12; left panel). Subsequently, single neuronal
activities can be isolated (Fig. 12; spikes on the right panel). To quantify the S1 neuronal
responses, the stimulation of the body part is repeated within a certain period of time, and the

IBO 2010 KOREA


PRACTICAL TEST 2
PHYSIOLOGY AND ANATOMY
_________________________________________________________________________________

action potentials are accumulated to produce a histogram (Fig. 13). In the histogram, the X-axis
stands for time (ms) before (-), the exact moment of (0), and after (+) stimulation. The Y-axis
represents the mean firing rate (Hz) within the recorded neuron.

Press

or

2. S1 neuronal response to forepaw digit stimulation


2.1. Forepaw digit region in S1
For the location of recording electrode, x-y coordinates are drawn over the skull (Fig. 14). The
point where three bones meet (the bregma) is the origin (0, 0) of the coordinate system. Previous
investigations reported that the point (0.3, 4.3) (Fig. 15) is one of the responding spots for
stimulation of the 2nd forepaw digit (Fig. 16).

Press

or

2.2. Mechanical stimulation


To find the general boundaries of the S1 region responding to stimulation of a specific skin
area, it is better to perform mechanical stimulation prior to electrical stimulation. A recording
electrode is positioned above the coordinate (0.3, 4.3) and is lowered stepwise at 25 m/step (Fig.
17; a red, moving needle). The responses to mechanical stimulation of the 2nd forepaw digit are
given in Table 2.2

IBO 2010 KOREA


PRACTICAL TEST 2
PHYSIOLOGY AND ANATOMY
_________________________________________________________________________________

Table 2.2
Fig.

Depth

Area of

S1 response to

S1 Response to

(number of steps)

skin stimulation

skin stimulation

joint movement

18

0-30

Broad

Weak

no

19

31-48

Tip

Strong

no

20

49-60

Broad

Weak

strong

Press

or

2.3 Electrical stimulation


A stimulating electrode is inserted into the 2nd forepaw digit, whereas the recording electrode is
inserted into the S1. The measured distance from the stimulating electrode to the recording one is
12 cm. The response of the S1 neuron to weak and strong stimuli is shown in Table 2.3. and
Figures 21 and 22. (Note the pop-up histogram at the bottom in both actions.).
Table 2.3
Action

Stimulus to 2nd forepaw digit

Response of S1 neuron

Cursor on 21

Weak (0.1 mA)

No conspicuous spike

Cursor on 22

Strong (2 mA)

One conspicuous spike

IBO 2010 KOREA


PRACTICAL TEST 2
PHYSIOLOGY AND ANATOMY
_________________________________________________________________________________

Q1. (1 point) Based on the results of mechanical and electrical stimulation, which of the
following statements is correct?
A. The strongest response to mechanical stimulation is observed in neurons at 0.50.75 mm deep from the surface.
B. Neurons at a depth of 0.775-1.2 mm respond to the smallest skin area.
C. Neurons at a depth of 0.775-1.5 mm respond only to skin touch.
D. The thickness of the S1 cortex is less than 1mm.
E. The firing rate (Hz) of S1 neurons has no correlation with stimulus intensity.

Q2. (1 point) Calculate the minimum (p) and maximum (q) velocities (unit: m/sec) of
information transmission from the digit to the S1.
Q3. (1 point) During the period of 6-15 ms after stimulation, what is the net increase in the mean
value (

) of firing rate (Hz) evoked by strong (2 mA) stimulation?

2.4. Response to a gamma-aminobutyric acid (GABA) antagonist


GABA is a neurotransmitter in the brain. The response of the S1 neuron to weak and strong
stimuli following the topical application of a GABA antagonist (i.e., inhibitor of GABA action)
to the S1 cortex is shown in Table 2.4 and Figures 23 and 24. (Note the pop-up histogram at the
bottom in both cases.).
Table 2.4
Action

Stimulus to 2nd forepaw

Cursor on 23

Weak (0.1 mA)

No conspicuous spike

Cursor on 24

Strong (2 mA)

Two conspicuous spikes

Response of S1 neuron

IBO 2010 KOREA


PRACTICAL TEST 2
PHYSIOLOGY AND ANATOMY
_________________________________________________________________________________

Q4. (2 points) Based on the results of before and after the antagonist application, which of the
following statements is correct?
A. The net increase in the mean firing rate (Hz) of the first peak in histogram 24 is about
2.14 times of that of the peak in histogram 22.
B. After the antagonist application, the mean firing rates (Hz) always increase regardless of
stimulation intensity.
C. The GABA antagonist inhibits excitatory synaptic activity in the S1.
D. Based on histogram 24, a net increase in the mean firing rate (Hz) for the first peak is 4.5
times of the one for the second peak.
E. The second peak in histogram 24 is not associated with S1 processing of the cutaneous
input from the digit.

Press

or

3. S1 neuronal response to hindpaw digit stimulation


3.1 Electrical stimulation
Previous investigations reported that the point (-1.0, 2.5) is one of the responding spots for
hindpaw digit stimulation (Fig. 25).
A recording electrode is lowered stepwise (25 m/step) downward from the brain surface.
Responses of neurons at three locations (a=25 steps, b=41 steps, c=52 steps) along the vertical
track are recorded (Fig. 26).
Following strong (2mA) electrical stimulation of the 2nd, 3rd, and 4th hindpaw digits (Fig. 27),
responses of the three neurons at a, b, and c are recorded (Fig. 29).

IBO 2010 KOREA


PRACTICAL TEST 2
PHYSIOLOGY AND ANATOMY
_________________________________________________________________________________

3.2 Response to local anesthesia


A local anesthetic drug applied to the 3rd hindpaw digit (Fig. 28, grey color) causes a sensory
loss within 2 minutes, and the effect lasts for 30 minutes. Afterward, recovery of sensation
gradually occurs. The drug effect completely disappears by 60 minutes post-application. When
strong (2 mA) electrical stimulation is applied to the digit 40 minutes after drug application, the
response of the three neurons is changed (Fig. 30).

Q5. (1 point) Based on neural response before anesthesia (Fig. 29), choose the correct statement.
Case

Neurons

Stronger or longer response

Weaker or shorter response

Locations a, b and c

2nd digit

3rd digit

Locations a, b and c

4th digit

3rd digit

Location b

4th digit

2nd digit

During 3rd digit stimulation, neurons at locations a and c have longer response
durations than neuron at location b.

Location a

4th digit

Other digits

Q6. (1 point) Based on neural response before anesthesia (Fig. 29), choose the correct statement.
A. All three neurons respond to 4th digit stimulation.
B. A single S1 neuron responds to the stimulation of only one digit.
C. Neurons at location a respond to the stimulation of more of the hindpaw digits than
neurons at location b.
D. Neurons at location c respond to the stimulation of more of the hindpaw digits than
neurons at location b.
E. All three neurons receive convergent sensory information from two or more digits.

IBO 2010 KOREA


PRACTICAL TEST 2
PHYSIOLOGY AND ANATOMY
_________________________________________________________________________________

Q7. (1 point) Based on the responses shown by the neurons in all three locations in Figs. 29 and
30, choose the incorrect statement.
Case

Location of stimulation

Timing of response

Magnitude of response

2nd digit

40 min after drug application

Increased

3rd digit

40 min after drug application

Decreased

4th digit

40 min after drug application

Increased

2nd and 4th digits

Before and after drug

Greater in 4th than in 2nd

application

digit

A neuron not responding to a certain stimulus may respond to it under certain


conditions

Q8. (2 points) Based on the response after anesthesia (Fig. 30), select an appropriate inference.
A. The drug is absorbed into the blood and is transferred to the S1.
B. The drug has changed the structure of peripheral nerve branches.
C. Neuronal response is not altered after local anesthesia.
D. The drug causes reversible, temporary changes in S1 neuronal synapses.
E. The change in response after anesthesia is due to newly-synthesized proteins within the
S1.

Press

or

10

IBO 2010 KOREA


PRACTICAL TEST 2
PHYSIOLOGY AND ANATOMY
_________________________________________________________________________________

4. S1 body map
4.1 Normal S1 map
Following repeated stimulation/recording procedures, the normal S1 body map (Fig.
31) is obtained (Note: the electrode is moved along the x or y axis by the distance of
0.5 mm). If the computer cursor is laid on each symbol (, , , ), the

abbreviation for appropriate body surface is shown as a note and, at the same time,
the

equivalent

body

Abbreviations
fl

forelimb

position

fp

forepaw

depicted at the bottom.

fpd 1-5

forepaw digits 1-5

fm

forelimb muscle

hl

hindlimb

hp

hindpaw

hpd 1-5

hindpaw digits 1-5

hm

hindlimb muscle

trunk

mv

mystacial vibrissa

rv

rostral vibrissa

forelimb

The

hindlimb

will

following

provides
anatomical

be

table
the

term

for

each abbreviation used

trunk

vibrissa

11

in the figure.

IBO 2010 KOREA


PRACTICAL TEST 2
PHYSIOLOGY AND ANATOMY
_________________________________________________________________________________

Q9. (5 points = 0.5 10) Find the following points (n=10) from Fig. 31 and fill in the
blanks with

abbreviations (i.e., notes within the boxes of the screen) for body surfaces.

Q10. (1point) Based on the answers to Q9, which of the following statements is correct?
A. The fpd4 region is medial to the fpd2 region.
B. The hpd2 region is medial to the hpd4 region.
C. The fl region is rostral to the hp region.
D. The fl region is caudal to the t region.
E. The mvB2 region is lateral to the mvA3 region.
Q11. (1 point) Based on the normal S1 map, what can you conclude about the following areas?
Case

Smaller area

Larger area

Forelimb (fl + fp + fpd + fm)

Hindlimb (hl + hp + hpd + hm)

Forelimb (fl + fp + fpd + fm)

Trunk (t)

Hindlimb (hl + hp + hpd + hm)

Trunk (t)

12

IBO 2010 KOREA


PRACTICAL TEST 2
PHYSIOLOGY AND ANATOMY
_________________________________________________________________________________

Mystacial vibrissa (mv)

Rostral vibrissa (rv)

Forelimb (fl + fp + fpd + fm)

Vibrissa (mv + rv)

Q12. (1 point = 0.5 2) In the hindlimb region, S1 neurons receiving sensory information overlap
with the motor neurons that cause muscle contraction. Find a coordinate (unit: mm) which
supports this observation.

Press

or

4.2 Change in S1 body map after digit amputation


By reducing the distance between checkpoints (Note: the electrode is moved along the x or y
axis by the distance of 0.2 mm), a more precise map for the hindpaw region is obtained (Fig. 32).
Surgery is performed to remove the 4th hindpaw digit. At 4 weeks after digit amputation, a new
body map is obtained (Fig. 33).

Q13. (4 points = 0.5 8) Put the cursor on the corresponding spots within Figs. 32 and 33, and
notice where the post-amputation response is different from the normal response. For the
locations where alterations occurred, fill in the appropriate table boxes with the
abbreviations (i.e., notes within the boxes of the screen) for the digit numbers (you will fill
in 4 boxes on each table, for a total of 8 boxes).

Normal

hpd4 amputated

lateral

lateral

2.8

2.8

2.6

2.6

2.4

2.4

13

IBO 2010 KOREA


PRACTICAL TEST 2
PHYSIOLOGY AND ANATOMY
_________________________________________________________________________________

2.2
(mm)

2.2
0.6

0.8

1.0

(mm)

1.2

0.6

0.8

caudal

1.0

1.2

caudal

Q14. (1 point) What changes occurred in the S1 body map after digit amputation?
Case

Activation of neurons by stimulation of

Became responsive to

hpd3

hpd2

hpd3

hpd2 or hpd5

hpd4

hpd2

hpd4

hpd3 or hpd5

hpd5

hpd2 or hpd3

Press

or

4.3 Biochemical and histological changes after digit amputation


(1) Biochemical changes (Fig. 34)
Glutamate is a neurotransmitter. To explore the molecular basis of S1-body-map
reorganization following amputation, changes in the amount of glutamate- and GABA- receptors
in S1 tissue were tracked over an extended period of time. The amount of glutamate-receptors
(green curve) increased by 250% of control (dotted line) at 1 week after the 4th hindpaw digit
amputation; whereas, the amount of GABA-receptors (blue curve) rose to 180% of control at 4
weeks post-amputation.
(2) Histological changes (Fig. 35)

14

IBO 2010 KOREA


PRACTICAL TEST 2
PHYSIOLOGY AND ANATOMY
_________________________________________________________________________________

Using transverse sections of S1 tissue, the location of glutamate- or GABA-receptors on the


neuronal surface can be visualized using antibodies against those receptors. Immunostaining of
the S1 cell surface (asterisks) shows that glutamate-receptors (a and c, arrows) increase at 1
week post-amputation, whereas GABA-receptors (b and d, arrows) rise at 4 weeks postamputation.

Q15. (2 points) Based on Figs. 33, 34, and 35, choose the incorrect statement.
A. An increase in neuronal excitability is observed at 1 week after amputation.
B. An increase in neuronal inhibition is observed 4 weeks after amputation.
C. In the normal state, the S1 body map is maintained by a balance between excitatory
sensory input and local inhibition within the cortex.
D. During 1-4 weeks after amputation, the balance between excitatory input and local
inhibition is always maintained.
E. Electrophysiological changes at 4 week after amputation are accompanied by biochemical
and histological changes in S1 tissue.

Hope youve got interested in Neuroscience.

Press
Lets dissect a spider and be a Spiderman!

15

IBO 2010 KOREA


PRACTICAL TEST 2
PHYSIOLOGY AND ANATOMY
_________________________________________________________________________________

TASK II. (24 points) Anatomy of spider

Caution:

Handle carefully, because only one spider will be provided for each student.

Please note that the vials are labeled Venom gland, Silk gland, Heart, and Book lung in
English.

This task is composed of 2 parts.

Part I. (14 points) Exploration of the spider cephalothorax.

Q16. Both spiders and insects are members of phylum Arthropoda. In general, insects have two
kinds of eyes; compound eye and single eyes (ocelli). Examine the spider specimen
carefully under the microscope and answer the following questions.

Q16.1. (2 points) Record the types and total number of the spider's eyes.

Q16.2. (2 points) Generally, spiders eyes are arranged around its head in two distinct rows; i.e.
the anterior and posterior rows. Within each row, the inner pair of eyes are designated
as medial, while the outer pair is described as lateral (Table 1). Each eye is defined
using two anatomical terms: anterior vs. posterior and medial vs. lateral. Examine the
specimen and draw the relative position of eyes in the figure on the Answer Sheet.
Label the drawn eyes with specific codes given in Table 2.

16

IBO 2010 KOREA


PRACTICAL TEST 2
PHYSIOLOGY AND ANATOMY
_________________________________________________________________________________

Table 1. Terms of anatomical position


Anterior

situated near or toward the head

Posterior

opposite of anterior

Medial

toward the midline of the body

Lateral

away from the midline

Table 2. Codes for spider eyes


Code

Terminology of spider eyes

AME

Anterior Medial Eye

ALE

Anterior Lateral Eye

PME

Posterior Medial Eye

PLE

Posterior Lateral Eye

Q17. Spiders can be divided into two suborders based on the positions of the cheliceral fangs.
Using the forceps, examine the movement of the spider fangs under the dissecting
microscope. Then, answer the following questions.

Q17.1. (1 point) What is the striking direction of the fangs?


A

from forwards to downwards

from downwards to forwards

from inside to outside

from side to center

from center to side

17

IBO 2010 KOREA


PRACTICAL TEST 2
PHYSIOLOGY AND ANATOMY
_________________________________________________________________________________

Q17.2. (1 point) The fang forms an articulation (or joint) with the chelicerae. What type of
joint is the articulation?
A

Plane joint

Pivot joint

Hinge joint

Saddle joint

Ball-and-socket joint

Q18. (1 point) As arthropods, spiders have segmented bodies with jointed limbs. The head is
composed of several segments that fuse during development. Being chelicerates, their
bodies consist of two segments the cephalothorax and the abdomen (Figure 1).

Figure 1. Diagram of spider

Which of the following (1~4) correctly represents the segmental differentiation of the
cephalothorax in spiders compared to Trilobite, an ancient chelicerate?

18

IBO 2010 KOREA


PRACTICAL TEST 2
PHYSIOLOGY AND ANATOMY
_________________________________________________________________________________

Eye
Eye

Eye

Eye

Eye

A
L

Body

Body

Body

Body

Body

Trilobite

<Abbreviations>
A: Antenna,

C: Chelicera,

L: Leg,

P: Pedipalp

Caution: From now on, you will dissect the internal organs (venom glands, silk gland, heart, and
book lung) of the spider. Using the Ringers solution provided, you need to keep the
dissected organs from drying. You will be scored based on the correctness and the
intactness of the preparation. Points will be deducted when there is a failure to remove
the correct organ.

Q19. Most spiders possess venom that is injected into prey through the fangs of the chelicerae.
Spiders have a pair of venom glands that lie either in the chelicerae or in front of the
cephalothorax (see the diagram of spider in Figure 1). The venom gland consists of an
outermost muscle layer, an underlying secretion layer and a duct. Locate the venom glands
of the spider provided.

19

IBO 2010 KOREA


PRACTICAL TEST 2
PHYSIOLOGY AND ANATOMY
_________________________________________________________________________________

Q19.1. (2 points) Dissect out the pair of venom glands from the spider and put it in the vial
labeled Venom gland after the following examination. It is not required to separate the
venom glands from the chelicerae.

Q19.2 (1 point) Examine the outermost muscle layer of the venom gland under the microscope.
What is the direction of the muscular orientation?
A

Longitudinal direction

Circular direction

Spiral direction

Bilateral direction

Irregular direction

Q20. (2 points) In most spiders, each leg has several segments and the tip of the last segment has
claws. Remove the 1st and 2nd legs from the spider body. Using the microscope, count the
number of segments and claws on each leg.

Q21. Many spider species exhibit sexual dimorphism. In sexually mature male spiders, the final
segment of the pedipalp develops into a complicated structure that is used to transfer sperm
to the female during mating. This apparatus makes the male palp so enlarged that it is often
described as resembling a boxing glove.

Q21.1. (1 point) Examine the external morphology of the spider specimen provided and
identify the sex of the spider.

20

IBO 2010 KOREA


PRACTICAL TEST 2
PHYSIOLOGY AND ANATOMY
_________________________________________________________________________________

Q21.2. (1 point) Pedipalps of spiders also have segmentation like the legs. Using the
microscope, count the number of the segments and claws in each pedipalp.

Part II. (10 points) Exploration of the spider abdomen.

Q22. (1 point) The abdomen and cephalothorax of a spider are connected by a thin waist called
the pedicel, which allows the abdomen to move in all directions (see the diagram of spider
in Figure 1).
Which of the following organ systems does not pass through the pedicel?
A

Nervous system

Respiratory system

Circulatory system

Digestive system

Integumentary system

Q23. The silk-spinning apparatus of the spider is located at the posterior end of the ventral
abdomen. This apparatus is composed of three pairs of spinnerets. Generally, the spinnerets
are arranged in two distinct rows; anterior and posterior. Anatomically, the inner pair of
spinnerets is defined as medial, and the outer pair is lateral (Table 3). Accordingly, the
position of a spinneret is defined using these two positional terms.

21

IBO 2010 KOREA


PRACTICAL TEST 2
PHYSIOLOGY AND ANATOMY
_________________________________________________________________________________

Q23.1. (1 point) Compare the external morphology of the spinnerets with the following
diagram. Label each spinneret in the answer sheet using the codes given in Table 3.

Table 3. Spider spinnerets


Code

Position of spinneret

Anterior

AM

Anterior medial

AL

Anterior lateral

Posterior

PM

Posterior medial

PL

Posterior lateral

Q23.2. (1 point) Identify the structure posterior to the spinneret under the microscope .

Anus

Spermatheca

Spiracle

Copulatory organ

Spinneret

22

IBO 2010 KOREA


PRACTICAL TEST 2
PHYSIOLOGY AND ANATOMY
_________________________________________________________________________________

Q24. Spiders produce various kinds of silk fibers from the silk glands. There are seven gland
types in the specimen provided, each producing a different type of silk (Table 4).

Table 4. Silk glands of the spider


Code of silk gland

Number of pairs

Connection to spinneret

Numerous

Middle & posterior

Posterior

Posterior

Anterior

Middle

Numerous

Anterior

Middle (1) & posterior (2)

Q24.1. (1 point) Dragline silk is produced by the largest silk glands of this spider (Figure 2).
Use the codes in Table 4 to locate the largest silk gland.

Figure 2. The silk gland which produces dragline silk

23

IBO 2010 KOREA


PRACTICAL TEST 2
PHYSIOLOGY AND ANATOMY
_________________________________________________________________________________

Q24.2. (2 points) Dissect one complete silk gland which produces dragline silk from this spider.
After dissecting the silk gland, place the organ in the vial labeled Silk gland.

Q25. (2 points) With reference to Figure 1, dissect the heart tube from the abdomen and place it
in the vial labeled Heart.

Q26. (2 points) With reference to Figure 1, locate and dissect one complete book lung. Place the
organ in the vial labeled Book lung.

24

Country Code: ___________

Student Code: ___________

PRACTICAL TEST 2
Answer Sheet
PHYSIOLOGY AND ANATOMY
Total Points: 49
Duration: 90 minutes

TASK I. (25 points)


Q1. (1 point)
A

Q2. (1 point = 0.5 x 2)


p
q
Q3. (1 point)
Hz

Q4. (2 points)
A

Q5. (1 point)
A

Q6. (1 point)
A

Q7. (1 point)
A

Q8. (2 points)
A

Q9. (5 points = 0.5 x 10)

Q10. (1 point)
A

Q11. (1 point)
A

Q12. (1 point = 0.5 x 2)


x
y

Q13. (4 points = 0.5 x 8)


Normal

hpd4 amputated

Q14. (1 point)
A

Q15. (2 points)
A

TASK II. (24 points)


Part I. (14 points)
Q16.1. (2 points = 1 2)
Type of eye

Total number of eye

Compound eye
Ocellus

Q16.2. (2 points = 0.5 4)

.
Frontal view of the head

Q17.1. (1 point)
A

Q17.2. (1 point)
A

Q18. (1 point)
1

Q19.1. (2 points)

(Place the organ in the provided vial, labeled Venom gland in English)

Q19.2. (1 point)
A

Q20. (2 points = 0.5 4)


1st leg

2nd

Number of segments
Number of claws

Q21.1. (1 point)
Sex of the spider provided
Male

Female

Q21.2. (1 point = 0.5 2)


Number of the segments
Number of claws

leg

Part II. (10 points)


Q22. (1 point)
A

Q23.1. (1 point = 0.5 2)

Q23.2. (1 point)
A

Q24.1. (1 point)
Code of silk
gland
Q24.2. (2 points)

(Place the organ in the provided vial, labeled Silk gland in English.)

Q25. (2 points)

(Place the organ in the provided vial, labeled Heart in English.)


Q26. (2 points)

(Place the organ in the provided vial, labeled Book lung in English.)
7

PRACTICAL TEST 2
Answer Key
PHYSIOLOGY AND ANATOMY
Total Points: 49
Duration: 90 minutes

TASK I. (25 points)


Q1. (1 point)
A

Q2. (1 point = 0.5 x 2)


p

20

Q3. (1 point)
Hz

40

Q4. (2 points)
A

Q5. (1 point)
A

Q6. (1 point)
A

Q7. (1 point)
A

Q8. (2 points)
A

Q9. (5 points = 0.5 x 10)

1. For answers mvA3 or mvB2, A3 or B2 is evaluated as a correct answer, because the


picture of the rat shows A3 or B2 only.
Q10. (1 point)
A

Q11. (1 point)
A

Q12. (1 point = 0.5 x 2)


X

2.5
2

Q13. (4 points = 0.5 x 8)

1. Numbers without hpd are also evaluated as correct answers, because the picture of the rat
contains digit numbers only.
Q14. (1 point)
A

Q15. (2 points)
A

TASK II. (24 points)


Part I. (14 points)
Q16.1. (2 points = 1 2)
Type of eye

Total number of eye

Compound eye

Ocellus

Q16.2. (2 points)

Frontal view of the head.


1. 1 point will be given if you draw 4 pairs of eyes at proper position.
1 point to the 4 correct codes (1 point = 0.25 4).
2. 1 point will be given if you draw 4 pairs of eyes at proper position without correct codes or
with incorrect codes.
3. 0 point will be given if you draw incorrect number of eyes.
Q17.1. (1 point)
A

Q17.2. (1 point)
A

Q18. (1 point)

Q19.1. (2 points)

(Place the organ in the provided vial, labeled Venom gland in English)
1. 2 points will be given if you dissect out a pair of venom glands with or without chelicerae.
2. 1 point will be given if you dissect out the chelicerae with one venom gland.
3. 0 point will be given if you dissect incorrect organ.
Q19.2. (1 point)
A

Q20. (2 points = 0.5 4)


1st leg

2nd leg

Number of segments

Number of claws

Q21.1. (1 point)
Sex of the spider provided
Male

Female

Q21.2. (1 point = 0.5 2)


Number of the segments

Number of claws

Part II. (10 points)


Q22. (1 point)
A

Q23.1. (1 point = 0.5 2)

PL

Q23.2. (1 point)
A

Q24.1. (1 point)
Code of silk
gland

Q24.2. (2 points)

(Place the organ in the provided vial, labeled Silk gland in English.)
1. 2 points will be given if you dissect proper silk gland with both regions of ampulla and tail.
2. 1 point will be given if you dissect proper silk gland with ampulla region only.
3. 0 point will be given if you dissect incorrect silk gland.
Q25. (2 points)

(Place the organ in the provided vial, labeled Heart in English.)


Q26. (2 points)

(Place the organ in the provided vial, labeled Book lung in English.)

IBO 2010 KOREA


PRACTICAL TEST 3
GENETICS AND CELL BIOLOGY
_________________________________________________________________________________

Country Code: ___________

Student Code: ___________

The 21st INTERNATIONAL BIOLOGY OLYMPIAD


11th 18th July, 2010
Changwon, KOREA

PRACTICAL TEST 3
GENETICS AND CELL BIOLOGY

Total Points: 50
Duration: 90 minutes

IBO 2010 KOREA


PRACTICAL TEST 3
GENETICS AND CELL BIOLOGY
_________________________________________________________________________________

Dear Participants,

In this test, you have been given the following 2 tasks:


Task I (35 points)
(1) Study of promoter-driven regulation of gene expression. (20 points)
(2) Characterization of the relationship between genotypes and phenotypes
(15 points).
Task II: Observation of meiotic cells in preserved rye anthers (15 points)

Write down your results and answers in the Answer Sheet. Answers written in the Question
Paper will not be evaluated.

Please make sure that you have received all the materials listed for each task. If any of the listed
items is missing, please raise your hand.

Stop answering and put down your pencil immediately after the end bell rings. The supervisor will
collect the Question Paper and the Answer Sheet.

Good Luck!!

IBO 2010 KOREA


PRACTICAL TEST 3
GENETICS AND CELL BIOLOGY
_________________________________________________________________________________

GENETICS AND CELL BIOLOGY


This practical test is composed of 2 tasks.
TASK I. (35 points)
(1) Study of the promoter-driven regulation of gene expression
(2) Characterization of the relationship between genotypes and
phenotypes

This task is composed of 2 parts.


Materials and Equipments
On individual Table
1. Fluoro-spectrophotometer
2. Microfuge tubes containing 50 L each of nine differently-labeled plant extracts; two
identically labeled tubes are provided for each type of extract (2 9 = 18 tubes). The
transparent tubes are for the protein assay, and the black tubes are for fluorescence
measurements.
Label

Treatment

Label

Treatment

WT-0

Plant WT + distilled water

WT-1

Plant WT + 1 M hormone H

WT-100

Plant WT + 100 M hormone H

dA-1

Plant dA + 1 M hormone H

dA-100

Plant dA + 100 M hormone H

dAB-1

Plant dAB + 1 M hormone H

dAB-100

Plant dAB + 100 M hormone H

dABC-1

Plant dABC + 1 M hormone H

dABC-100

Plant dABC + 100 M hormone H

IBO 2010 KOREA


PRACTICAL TEST 3
GENETICS AND CELL BIOLOGY
_________________________________________________________________________________

3. 12 mL Bradford reagent in a 15 mL plastic tube (Bradford reagent is used to determine


concentration of protein)
4. 1 mL of 1 mM MUG (fluorescence substrate to measure GUS activity) in a microfuge
tube
5. 12 mL of stop reagent for the GUS (enzyme -glucuronidase which converts MUG into
MU) reaction in a 15 mL plastic tube
6. Two DNA size-marker tubes (labeled M, 50 L each) and eight tubes containing EcoRIdigested DNA (labeled P1~P8, 50 L each)
7. Two microfuge tubes labeled as GUS BL and Pro BL, respectively.
8. Three micropipettes (one each for 10-100 L and 100-1000 L, and a fixed volume
pipette for 20 L)
9. A box of yellow tips for the 20 L and the 10-100 L micropipettes
10. A box of blue tips for the 100-1000 L micropipette
11. A DNA electrophoresis apparatus, equipped with a 1% agarose gel in 1X TAE gel
running buffer. If your gel is broken, raise your hand for assistance.
12. A tip disposal container
13. Polygloves
14. 25 cuvettes for the Fluoro-spectrophotometer
15. A calculator
16. A timer
17. A Scotch tape
18. An ice bucket filled with ice
19. Microfuge tube racks
20. Green card
On the common equipment table
1. Gel documentation system equipped with a UV source

IBO 2010 KOREA


PRACTICAL TEST 3
GENETICS AND CELL BIOLOGY
_________________________________________________________________________________

Handling of Micropipettes

Adjustment method
Turn the plunger to set the volume to the desired value, which can be seen in the display
window. Remember that each micropipette has designated range of volumes as indicated on
the pipette. Do not exceed the limits of this range.

Usage method
1) Secure the pipette tip to the tip holder. Gently push down the plunger to the first stop.
2) Hold and lower the tip down into the solution to a depth of 2~4 mm. Release the plunger
slowly to allow it to return to its original position.
3) Remove the pipette from the liquid, and transfer the contents to the desired tube. Push the
plunger to the first stop and then push further to the second stop to discharge the solution
completely from the tip.
4) Remove the pipette from the tube and release the plunger. Eject the used tip into the tip
disposal container by pressing the tip-ejector.

IBO 2010 KOREA


PRACTICAL TEST 3
GENETICS AND CELL BIOLOGY
_________________________________________________________________________________

Operating Instruction for the Fluoro-Spectrophotometer (measures fluorescence of MU and


absorbance of proteins at 595 nm)

Usage method
Important: Please be sure not to touch the light path of cuvettes.
1) Press the PWR (

) button to turn on the machine. The display window will be turned on

after a beep.
2) To set the blank sample to zero, insert the blank cuvette in an appropriate holder (use cuvette
holder A to measure protein concentration, and cuvette holder B to measure GUS activity).
The cuvette indicator will be turned on (

for the holder A and

for the holder B).

Note: Two blank samples for measurement of GUS activity and amounts of proteins are
provided in the microfuge tubes labeled as GUS BL and Pro BL, respectively.
3) Press the BL button, and the blank indicator ( ) will appear when the blank is set at 0.0.
4) To measure a sample, remove the blank cuvette and insert the test cuvette in the same
cuvette holder, and press the TS button. The result will be displayed after 5-10 seconds, and
the indicator will appear in the display window ( )
5) To end the machine, keep the PWR button pressed till beep is heard.

IBO 2010 KOREA


PRACTICAL TEST 3
GENETICS AND CELL BIOLOGY
_________________________________________________________________________________

Operating Instruction for the DNA Gel Electrophoretic Apparatus


1) Load the samples to the wells using the 20 L micropipette.

2) After verifying that the operation switch of the power supply is OFF, close the migration
tank lid.

Do this as follows:
(1) First, insert the 2 tabs on the cover into the holes in the migration tank.
(2) Then, rotate the cover forward to close it.

3) Set the voltage to Half using the output selection switch.

4) Push the operation switch to start the migration.

5) In this experiment, the gel running time should be 30 min. Make sure to turn the operation
switch OFF when the running is finished.

IBO 2010 KOREA


PRACTICAL TEST 3
GENETICS AND CELL BIOLOGY
_________________________________________________________________________________

Part I. (20 points) Using the gene X-fused GUS reporter gene to analyze hormonal effects on
gene expression and to characterize the hormone-responsive elements in the
promoter.

Plants respond to their hormones by regulating hormone-responsive genes. Within a gene


promoter, a specific DNA sequence(s), the cis-element, dictates the proper time and amount of
gene expression. Regulation is primarily controlled by an hormone-responsive transcription
factor(s) that binds specifically to this region, resulting either in gene activation or suppression.
In this task, you will examine the mode of hormonal regulation in the hormone-responsive
gene X of Arabidopsis. To find the hormone-responsive regions, in the promoter and to
understand the mode of hormonal regulation of gene X expression, the promoter of gene X is
divided into A~C (each of these domain may function as enhancer, silencer or minimal
promoter). Then, a variety of Arabidopsis transgenic plants expressing the GUS (glucuronidase) reporter gene under the control of the different regions of the promoter, as
diagramed below, was generated. The GUS will be produced when the promoter of gene X is
activated. The GUS enzyme converts MUG into MU, and its activity can be measured by
quantifying MU fluorescence using a fluoro-spectrophotometer.

< Four Arabidopsis transgenic plants carrying different reporter constructs>

IBO 2010 KOREA


PRACTICAL TEST 3
GENETICS AND CELL BIOLOGY
_________________________________________________________________________________

Q1. The purpose of the first experiment is two-fold: (1) to find the promoter region containing a
hormone-responsive cis-element and (2) to investigate the effects of different hormone H
concentrations on gene X expression. All transgenic plants (WT, dA, dAB, and dABC) were
treated with either 1 M or 100 M of hormone H. To assess the level of GUS expression,
plant extracts were prepared from these treated plants. (See the table in the materials and
method section.)
Using the methods described in the next section, measure the fluorescence value and
absorbance at 595 nm of each 50 L plant extracts. Based on these measurements,
calculate the amount of MU (nmole MU/50 L plant extracts), the amount of proteins
(g/50 L plant extracts), and the resulting GUS activity (nmole MU/g protein/min) for
each extract. Record your results in Table 1 in the answer sheet to find answers for Q1.1,
Q1.2, and Q1.3.

Measurement of fluorescence and determination of MU amount


1)-1. Turn on and set the fluoro-spectrophotometer to zero with 500 L of the blank sample
labeled GUS BL.
1)-2. Take a microfuge tube of plant extracts (each tube contains 50 L extracts) prepared from
each WT-O or hormone-treated transgenic plant, and mix well (by gentle tapping) with
50 L of 1 mM MUG solution. Start with tube labeled WT-O and proceed in an order
shown in the table in Materials and Equipments.
1)-3. Incubate the reaction mixtures at room temperature for 10 min.
1)-4. Stop the reaction by adding 900 L of stop reagent (1M sodium carbonate in GUS
extraction buffer) into each 100 L reaction solution in the same order you added MUG.
Mix well by tapping.
1)-5. Take 500 L of the finished mixture from each tube, and measure the fluorescence using
the fluoro-spectrophotometer.

IBO 2010 KOREA


PRACTICAL TEST 3
GENETICS AND CELL BIOLOGY
_________________________________________________________________________________

1)-6. Calculate the amount of MU in the sample using the formula provided below. Record the
fluorescence value and the calculated amount of MU in Table 1 in the answer sheet. This
is the amount of MU produced from each of 50 L plant extracts.

Y = 0.04 X + 2.5
Y: the amount of MU (nmoles/ 50 L plant extracts)
X: the measured fluorescence value [from step 1)-5]

Measurement of absorbance at 595 nm and determination of protein amount


2)-1. Turn on and set the fluoro-spectrophotometer to zero with 500 L of the blank sample
labeled Pro BL.
2)-2. Take a microfuge tube with extracts (each tube contains 50 l extracts) prepared from
each WT-O or hormone-treated transgenic plant, and mix well with 950 L of Bradford
reagent. Incubate at room temperature for 5 min.
2)-3. Take 500 L of the reaction mixture from each tube, and measure the absorbance at 595
nm using the fluoro-spectrophotometer.
2)-4. Calculate the amount of proteins using the formula provided below. Record the
absorbance at 595 nm and the calculated amount of proteins in Table 1 in the answer
sheet. This is the amount of proteins contained in each of 50 L plant extracts.
Y = 98X + 2.8
Y: the amount of protein (g/50 L plant extracts) X: the measured absorbance at
595 nm of the solution [from step 2)-3]

10

IBO 2010 KOREA


PRACTICAL TEST 3
GENETICS AND CELL BIOLOGY
_________________________________________________________________________________

Calculation of GUS activity


3)-1. Considering that this GUS enzyme reaction was performed for 10 min [refer to 1)-3],
calculate GUS activity in nmole MU/g protein/min and record the value in Table 1 in the
answer sheet.

Table 1 is worth of 9 points.

Q1.1. (4 points) Based on your results in <Table 1>, put a checkmark () in the appropriate
box of each plant in Table Q1.1 in the answer sheet.
Note: - stimulation: more than 3-fold increase in gene X expression
- no effect: less than 3-fold increase in gene X expression

Q1.2. (6 points = 2 3) Based on your previous conclusions in Q1.1, determine the regulatory
function (enhancer, silencer, or minimal promoter) of each cis-element (A~C). Put a
checkmark () in the appropriate box in Table Q1.2 in the answer sheet.

Q1.3. (1 point) How does 100 M of hormone H regulate the expression of gene X? Based on
your finding from <Table 1>, determine the mode of action of hormone H. Put a
checkmark () in the appropriate box in Table Q1.3 in the answer sheet.

11

IBO 2010 KOREA


PRACTICAL TEST 3
GENETICS AND CELL BIOLOGY
_________________________________________________________________________________

Part II. (15 points) A co-relationship analysis between genotype and phenotype, and the
prediction of gene pool frequencies using Hardy-Weinberg mathematics.

Q2. Gene Y encodes a protein that regulates plant growth. The schematic figure below depicts
the region of gene Y in genomic DNA and a point mutation within.

There are eight plants with homozygous (YY or yy) or heterozygous (Yy) genotype, showing
either wild type or dwarf phenotypes (Y: wild type allele, y: mutant allele. The alleles Y and
y do not specify whether they are dominant or reccessive). To analyze the genotype of these
plants, the
1 kb region of gene Y was amplified by PCR. This fragment was then digested with EcoRI
restriction enzyme, which cuts GAATTC sequence. Other than the EcoRI site created by the
point mutation, there is no other EcoRI recognition sequence in gene Y. Using the protocol
described below, perform a gel electrophoresis of the EcoRI-digested PCR products.

Genotyping of gene Y by gel electrophoresis


Note: Always wear polygloves during the experiment !!!

12

IBO 2010 KOREA


PRACTICAL TEST 3
GENETICS AND CELL BIOLOGY
_________________________________________________________________________________

(1) A total of ten microfuge tubes are provided: two DNA size marker tubes (M) and eight
tubes containing EcoRI-treated PCR product from Plants 1~8 (P1~P8, respectively).
Starting from left, in the order of M, P1~P8, M, load 20 L out of 50 L DNA solution
into each well of a prepared agarose gel in the electrophoresis apparatus. Use the 20 L
micropipette to load samples. Change pipette tip for each sample.

Note: The DNA size marker solution contains 0.4, 0.6, and 1.0 kb DNA fragments.
DNA loading buffer and DNA-staining dye are already included in each tube.
(2) Refer to <Operating instructions for DNA gel electrophoretic apparatus> to put the cover
on the electrophoresis apparatus, to turn on the apparatus, and to run the electrophoresis.
Note: Upon starting the electrophoresis, make sure that the output indicator LED is lit and
that bubbles are forming on the platinum electrodes.
(3) Run the gel for 30 min at Half voltage.
* IMPORTANT: While the gel is running, proceed to TASK II !!!

(4) Turn off the apparatus. Then, raise the green card to request help for photography of the
agarose gel.
Note: The assistant will bring a gel transfer box to you. Make sure that your student code is
on the box.
(5) When you receive the agarose gel picture, attach it to Q2.1 of the answer sheet using
Scotch tape. Label the number of each plant (P1~P8) on each lane of the gel picture.
(6) In Table Q2.2 in the answer sheet, put checkmarks () to designate the size of DNA
fragments and the genotype of each plant.

13

IBO 2010 KOREA


PRACTICAL TEST 3
GENETICS AND CELL BIOLOGY
_________________________________________________________________________________

Q2.1. (3 points) Attach the agarose gel picture to a space given on the answer sheet. And
label the number of each plant (P1~P8) on each lane of the gel picture.

Q2.2. (4 points) Determine the size of DNA fragment(s) and the genotype (YY, Yy or yy)
of each plant. Put a checkmark () in the appropriate box in Table Q2.2 in the
answer sheet.

Q2.3. (2 points) Based on the genotype and phenotype of each plant given in Q2.2, deduce
the characteristic of the mutation. Put a checkmark () in the appropriate box in the
Table Q2.3 in the answer sheet.

Q2.4. (2 points) If you cross Plant 1 with Plant 3 (from Q2.2), what is the probability (%)
that an offspring will be a dwarf plant? Write your answer in the answer sheet.

Q2.5. (4 points) The eight plants in Q2.2 represent a population. If this population produces
10,000 plants in the next generation, what would be the expected number of
heterozygous and dwarf offspring, respectively? (Assume that this population is in
Hardy-Weinberg equilibrium.)

14

IBO 2010 KOREA


PRACTICAL TEST 3
GENETICS AND CELL BIOLOGY
_________________________________________________________________________________

TASK II. (15 points) Observation of meiotic cells in preserved rye anthers

Materials, instruments and tools

Numbers

1. Light microscope with objective lenses of


4X, 10X, 40X, and 100X

2. Preserved rye anthers in a vial

3. Dissecting needle set

4. Slides and cover slips

5 each

5. Filter paper (7 cm diameter)

6. Forceps

7. Ceramic tile

8. Petri-dish (6 cm diameter)

9. Acetocarmine solution with a dropper

10. Pencil

11. Eraser

12. Disposable plastic pipet

13. Red card

Background

Using a light microscope, you will observe meiotic cells in preserved rye anthers. Anthers
at a specific stage of meiosis were selected and were preserved in 70% ethanol.

15

IBO 2010 KOREA


PRACTICAL TEST 3
GENETICS AND CELL BIOLOGY
_________________________________________________________________________________

Requirements Overview

Using the microscope, identify anther cells undergoing meiosis. In the space given in the
answer sheet, sketch an image of meiotic cell you observe at 400X magnification (Q3.2)

Procedure
1) Before you start observation, check for the presence of two small preserved anthers in the
vial.
2) Take out the ceramic tile out of the tray, and put one glass slide on it.
3) Observe your specimen under the microscope at 100X magnification, and find at least one
cell undergoing meiosis. Then, observe one cell at 400X magnification and draw this
image in the given area of the answer sheet (Q3.2). Make sure that this cell is at the center
of your field of view. After you finish drawing, raise the red card. The lab assistant will
come to you and will take a photograph of the slide.

< Procedure for observation of meiotic cells in preserved rye anthers>

16

IBO 2010 KOREA


PRACTICAL TEST 3
GENETICS AND CELL BIOLOGY
_________________________________________________________________________________

Notes :
1. In step (1), if the anthers won't come out, put the solution back into the vial using the
disposable plastic pipet and repeat step (1).
2. Be careful not to break the anther in step (2).
3. You may use a filter paper to remove excess 70% ethanol in step (3).
4. Do not press too hard, or you may break the cells and/or the cover slip in step (7).
5. You are provided with two anthers to prepare your specimen. If you fail to make good
specimen with the first anther, please repeat the procedure and make another
preparation using the other. However keep in mind that the time for your experiment is
limited.

17

IBO 2010 KOREA


PRACTICAL TEST 3
GENETICS AND CELL BIOLOGY
_________________________________________________________________________________

Q3. Answer the following questions.


Important: You will see two types of cells under the microscope as shown in Figure Q3. The
circled ones are examples of cells undergoing meiosis, and the rest are cells of the
anther wall.

Figure Q3. Examples of cells undergoing meiotic cell division


observed under a microscope.

Q3.1. (1 point) What kind of cells in the anther undergoes meiosis? Put a checkmark () in the
appropriate box in the answer sheet.

Q3.2. (8 points) Draw one cell undergoing meiosis at 400X magnification in the answer sheet.
Do not label the drawing.

Important : This cell must be at the center of your field of view when the picture is taken.

18

IBO 2010 KOREA


PRACTICAL TEST 3
GENETICS AND CELL BIOLOGY
_________________________________________________________________________________

Q3.3. (4 points) At what meiotic stage are the cells? Put a checkmark () in the appropriate
box in the answer sheet.

Q3.4. (2 points) What is the amount of DNA in the cell undergoing meiosis that you observed
and a cell of the anther wall, respectively? Put checkmarks () in the appropriate boxes
in the answer sheet.

19

Country Code: ___________

Student Code: ___________

PRACTICAL TEST 3
Answer Sheet
GENETICS AND CELL BIOLOGY
Total Points: 50
Duration: 90 minutes

TASK I. (35 points)

Part I. (20 points)


Table 1. (9 points=1X9)

<Table 1> GUS expression, after treatment with different concentrations of hormone H, in
transgenic plants containing various deletions within the gene X promoter

Plants

Measured
fluorescence

Amount of MU*
produced by 50 L

Measured
absorobance

Amount of
proteins* in 50 L

GUS activity*
[nmole MU/g

[value from
1)-5]

plant extracts
[nmole MU, value

at 595 nm
[value from

plant extracts
[g, value from

protein/min,
value from 3)-1]

from 1)-6]

2)-3]

2)-4]

WT-0
(control)
WT-1
dA-1
dAB-1
dABC-1
WT-100
dA-100
dAB-100
dABC100
* The calculated values should be rounded to the nearest hundredth.

Q1.1. (4 points = 0.5 x 8)


Plant treated with
hormone H

Effect of hormone treatment in plants


Stimulation

WT-0

No effect
Control

WT-1
dA-1
dAB-1
dABC-1
WT-100
dA-100
dAB-100
dABC-100

Q1.2. (6 points = 2 x 3)
Function (enhancer, silencer, or minimal promoter)
Region in gene X promoter
enhancer
A
B
C

Q1.3. (1 point)
Action mode
Transcriptional positive feedback regulation
Transcriptional negative feedback regulation

silencer

minimal promoter

Part II. (15 points)


Q2.1. (3 points)
<Attach the agarose gel picture here>

Q2.2. (4 points = 0.5 x 8)

Plant

Size of the DNA fragment(s)


(kb)
0.4

0.6

1.0

Genotype
Phenotype
YY

Yy

yy

Plant 1

Wild type

Plant 2

Wild type

Plant 3

dwarf

Plant 4

dwarf

Plant 5

Wild type

Plant 6

Wild type

Plant 7

Wild type

Plant 8

dwarf

Q2.3. (2 points)
Dominant
Characteristic of the mutation
Recessive
Q2.4. (2 points)
Probability of dwarf offspring

(%)

Q2.5. (4 points = 2 x 2)
Number of heterozygous (Yy) offspring
Number of dwarf offspring

TASK II.

(15 points)

Q3.1. (1 point)
synergid cells
egg cells
megaspore mother cells
Pollen (microspore)mother cells
pollen
antipodal cells

Q3.2. (8 points)

400X

Q3.3. (4 points)
Meiosis I

Meiosis II

Prophase Metaphase Anaphase Telophase Prophase Metaphase Anaphase Telophase

Q3.4. (2 points = 1 2)
The amount of DNA
The cell undergoing meiosis
1C
2C
3C
4C

C: the amount of DNA in a haploid complement

Cells constituting anther wall

PRACTICAL TEST 3
Answer Key
GENETICS AND CELL BIOLOGY
Total Points: 50
Duration: 90 minutes

]TASK I. (35 points)

Part I. (20 points)


Table 1. (9 points = 0.2 45)
<Table 1> GUS expression, after treatment with different concentrations of hormone H, in
transgenic plants containing various deletions within the gene X promoter

Plants

WT-0

Measured
fluorescence

Amount of MU*
produced by 50 L

Measured
Amount of
absorbance at proteins* in 50 L

GUS activity*
[nmole MU/g

[value from
1)-5]

plant extracts
[nmole MU, value

595 nm
[value from

protein/min,
value from 3)-1]

from 1)-6]

2)-3]

plant extracts
[g, value from
2)-4]

600-900

26.5-38.5

0.5-0.7

51.8-71.4

0.04-0.07

WT-1

6,000-9,000

242.5-362.5

0.5-0.7

51.8-71.4

0.34-0.70

dA-1

6,000-9,000

242.5-362.5

0.5-0.7

51.8-71.4

0.34-0.70

dAB-1

600-900

26.5-38.5

0.5-0.7

51.8-71.4

0.04-0.07

dABC-1

600-900

26.5-38.5

0.5-0.7

51.8-71.4

0.04-0.07

WT-100

600-900

26.5-38.5

0.5-0.7

51.8-71.4

0.04-0.07

dA-100

6,000-9,000

242.5-362.5

0.5-0.7

51.8-71.4

0.34-0.70

dAB-100

600-900

26.5-38.5

0.5-0.7

51.8-71.4

0.04-0.07

dABC100

600-900

26.5-38.5

0.5-0.7

51.8-71.4

0.04-0.07

(control)

1. A correct answer for each measurement or calculation within ranges indicated in the
Table 1 is worth of 0.2 point.

Q1.1. (4 points = 0.5 x 8)


Plant treated with
hormone H

Effect of hormone treatment in plants


Stimulation

WT-0

No effect
Control

WT-1

dA-1

dAB-1

dABC-1

WT-100

dA-100

dAB-100

dABC-100

1. Answers that are not supported by the GUS activity data in Table 1 will be
considered as wrong answers.
2. Plural choices for each hormone-treated plant are null.

Q1.2. (6 points = 2 x 3)
Function (enhancer, silencer, or minimal promoter)
Region in gene X promoter
enhancer

silencer

minimal promoter

1. An answer that is not supported by the data in Table 1 and Q1.1 will be considered
as a wrong answer.
2. Plural choices for each promoter region are null
Q1.3. (1 point)
Action mode
Transcriptional positive feedback regulation
Transcriptional negative feedback regulation

1. The answer that is not supported by the answers for WT-0, WT-1 and WT-100 in
Table 1 and Q1.1 will be considered as a wrong answer.

Part II. (15 points)


Q2.1. (3 points)
<Attach the agarose gel picture here>

1. 3 points :
At the least one of the two DNA marker lanes was loaded, together with ALL of
the plant samples.
2. 2 points :
1) Both marker lanes were loaded but one of the plant sample is missing.
Or
2) All plant samples were loaded but both of the marker lanes are missing.
In all cases, electrophoresis should be performed long enough to allow the
genotyping. Otherwise no point will be given.

Q2.2. (4 points = 0.5 x 8)

Plant

Size of the DNA fragment(s)


(kb)
0.4

0.6

Genotype
Phenotype

1.0

YY

Yy

yy

Plant 1

Wild type

Plant 2

Wild type

Plant 3

dwarf

Plant 4

dwarf

Plant 5

Wild type

Plant 6

Wild type

Plant 7

Wild type

Plant 8

dwarf

Q2.3. (2 points)
Dominant
Characteristic of the mutation
Recessive
Q2.4. (2 points)
Probability of dwarf offspring

50 (%)

Q2.5. (4 points = 2 x 2)
Number of heterozygous (Yy) offspring

3750

Number of dwarf offspring

4375

TASK II.

(15 points)

Q3.1. (1 point)
synergid cells
egg cells
megaspore mother cells

pollen (microspore) mother cells


pollen
antipodal cells

Q3.2. (8 points)

400X
**
1. 8 points will be given if a cell undergoing meiosis with proper chromosome is drawn
and photo evidence is available.
2. 4 points will be given if drawing is good but photo evidence is not available.
3. 2 points will be given if only anther wall cell(s) were drawn.
4. 0 point will be given if i) no cell is drawn, ii) no chromosomes is discernable, iii)
only cell debris are drawn.
3

Q3.3. (4 points)
Meiosis I

Meiosis II

Prophase Metaphase Anaphase Telophase Prophase Metaphase Anaphase Telophase

1. 4 point will be given if the meiotic stage drawn in Q3.2 is correctly checked.
Q3.4. (2 points = 1 2)
The amount of DNA
The cell undergoing meiosis

Cells constituting anther wall

1C

2C
3C
4C

C: the amount of DNA in a haploid complement

IBO 2010 KOREA


PRACTICAL TEST 4
ECOLOGY
________________________________________________________________________________

Country Code: ___________

Student Code: ___________

The 21st INTERNATIONAL BIOLOGY OLYMPIAD


11th 18th July, 2010
Changwon, KOREA

PRACTICAL TEST 4
ECOLOGY

Total Points: 51
Duration: 90 minutes

IBO 2010 KOREA


PRACTICAL TEST 4
ECOLOGY
________________________________________________________________________________

Dear Participants,

In this test, you have been given the following 4 tasks:


Task I: Characteristics of Coastal Animal Communities (16 points)
Task II: Mark and Recapture Method (8 points)
Task III: Interspecific Interaction (14 points)
Task IV: Prey Choice Model (13 points)

Write down your results and answers in the Answer Sheet. Answers written in the Question
Paper will not be evaluated.

Please make sure that you have received all the materials listed for each task. If any of the listed
items is missing, please raise your hand.

Stop answering and put down your pencil immediately after the end bell rings. The supervisor will
collect the Question Paper and the Answer Sheet.

Good Luck!!

IBO 2010 KOREA


PRACTICAL TEST 4
ECOLOGY
________________________________________________________________________________

TASK I. (16 points) Characteristics of coastal animal communities

Quantity

Materials
1. Community model board (40 x 37 cm)

2. Transparent quadrat board (37 x 37 cm)

3. Electronic calculator

Introduction
A population is defined as a group of individuals of a single species inhabiting a specific
area, and a community is a group of populations of different species inhabiting a specific area.
Identification of the characteristics of populations and communities is a basic element in
predicting ecological change due to environmental factors.

Using Calculator

IBO 2010 KOREA


PRACTICAL TEST 4
ECOLOGY
________________________________________________________________________________

1. Press ON to turn on the calculator


2. Calculation Examples
To calculate 1 + 1 , press 1 + 1 =
To calculate ln 90 (= loge 90), press ln 9 0 =
To calculate

2 2 , press
52

( 2 x 2 ) ab/c ( 5 x 2 ) =

3. To correct characters, move the cursor by pressing or , and press DEL to delete the
character or SHIFT DEL to insert character
4. To clear all of the calculation you have entered, press AC.
5. Press Shift AC to turn off the calculator. Calculator will automatically turn off if you do
not perform any operation for about 10 minutes.

Q1. (4 points) The model provided on the board is a coastal community consisting of nine animal
species. Determine the population size (abundance, N) of each species in the community
using a complete enumeration survey and the population density (per unit area, 1 m2) of each
species. The size of each quadrat is 1 m x 1 m. Round values to the nearest hundredth (two
decimal places) during your calculations, and record the values in the answer sheet.

IBO 2010 KOREA


PRACTICAL TEST 4
ECOLOGY
________________________________________________________________________________

Species

Population size

Starfish

Razor clam

Sea slater

15

Sea urchin

Fiddler crab

13

Octopus

Oyster

Mudskipper

Sea anemone

13

IBO 2010 KOREA


PRACTICAL TEST 4
ECOLOGY
________________________________________________________________________________

Q2. (2 points) The table below records species population sizes in two different coastal
communities. Calculate the proportion of relative abundance of each species. Round
values to the nearest hundredth (two decimal places) during your calculations, and record
the values in the answer sheet.
Community A

Community B

Species

Population size

Species

Population size

Starfish

13

Fiddler crab

Razor clam

18

Barnacle

18

Sea slater

13

Sea anemone

15

Sea urchin

12

Sea cucumber

Fiddler crab

11

Hermit crab

Gastropod

Gastropod

Oyster

12

Mudskipper

Sea anemone

10

Total

106

Total

50

Q3. (4 points) A rank-abundance curve is a chart that displays the species in a community
ordered from most abundant to rare based on relative abundance. Using the relative
abundances you previously calculated (in Q2), make a rank-abundance curve for each
community on the grid-line in the answer sheet. Indicate community A as A and
community B as B on the curve, and write appropriate titles and scales for the X-axis and
the Y-axis.

IBO 2010 KOREA


PRACTICAL TEST 4
ECOLOGY
________________________________________________________________________________

Q4. (4 points) Calculate the Shannon-Wiener species diversity index (H) for each of the two
coastal communities using the following equation. Round values to the nearest hundredth
(two decimal places) during your calculations. Put the values in the box in the answer sheet.

where,
pi = the proportion of the ith species
ln pi = the natural logarithm of pi
n = the number of species in the community

Q5. (1 point) Which statement is/are correct for your rank-abundance curves? Put checkmark(s)
() in all appropriate boxes in the answer sheet.
A. Species evenness is higher in community A than in community B.
B. Species evenness is lower in community A than in community B.
C. Species richness is higher in community A than in community B.
D. Species richness is lower in community A than in community B.

Q6. (1 point) Which statement is correct for the species diversity index of the two communities?
Put a checkmark () in the appropriate box in the answer sheet.
A. The area with the higher diversity index (H') should be conserved.
B. The species diversity index (H') indicates the species number inhabiting the coastal area.
C. The species diversity index (H') is inversely proportional to species evenness in an area.
D. The species diversity index (H') depends on both species richness and species evenness.

IBO 2010 KOREA


PRACTICAL TEST 4
ECOLOGY
________________________________________________________________________________

TASK II. (8 points) Mark and recapture method

Quantity

Materials
1. Pottery with beads

2. Sampling net (100 ml)

3. Electronic calculator

Introduction
A few individuals are captured, marked and released back into the population. The population is
sampled again and the numbers of marked individuals in this sample counted. Assuming an equal
recapture rate for all individuals and without repetitive counting of the same individual, the
population size can be simply estimated by using a modified Lincoln Index as follows:

N: Estimation of population size


M: Number of individuals marked
S: Number of individuals captured in the second sample
R: Number of marked individuals recaptured

In this task, the pottery represents a pond with a diving beetle population (the beads). One bead
represents one diving beetle. This population contains 40 individuals marked with a red sticker that
had been captured during the first sampling. You will be performing the second sampling of this
population.

IBO 2010 KOREA


PRACTICAL TEST 4
ECOLOGY
________________________________________________________________________________

Q7. (4 points) Using the sampling net, capture a sample of diving beetles from the pond (the second
sampling). Take two full scoops and combine them. (Assume this population does not have birth,
death, emigration, or immigration of individuals between the first and the second sampling events).
Estimate population size to the nearest tenth (one decimal place) and record your result in the
answer sheet.

Q8. (4 points) The mark and recapture method has a degree of uncertainty because it is an estimation
by sampling, not by a total population count. We can measure uncertainty through the calculation
of standard error (SE). Standard error (SE) can be obtained by the function given below.

The 95% confidence interval can be obtained by this calculation: N t SE. The 95% confidence
interval means that the size of original population is within the range of the confidence interval with
95% certainty. The t-value is the Student's t- value when the degree of freedom is infinity. (At
infinity, the Students t-value is also refer to as Z-value). The critical values of the Student's t
distribution are provided.
Find the appropriate t in the table and calculate SE and the 95 % confidence interval for your
estimate of population size. Enter the numbers you obtain in the table in the answer sheet. Round
your value to the nearest hundredth (two decimal places) during your calculations and record
your values in the answer sheet.

IBO 2010 KOREA


PRACTICAL TEST 4
ECOLOGY
________________________________________________________________________________

Critical Values of the Students t Distribution


= p = P ( t > tcritical)

Degree of freedom
0.1

0.05

0.01

0.001

1
2
3

6.31
2.92
2.35

12.71
4.30
3.18

63.66
9.93
5.84

636.62
31.60
12.92

4
5

2.13
2.02

2.78
2.57

4.60
4.03

8.61
6.87

1.94

2.45

3.71

5.96

7
8
9
10

1.89
1.86
1.83
1.81

2.37
2.31
2.26
2.23

3.50
3.36
3.25
3.17

5.41
5.04
4.78
4.59

11
12

1.80
1.78

2.20
2.18

3.11
3.06

4.44
4.32

13

1.77

2.16

3.01

4.22

14

1.76

2.14

2.98

4.14

15
16
17

1.75
1.75
1.74

2.13
2.12
2.11

2.95
2.92
2.90

4.07
4.02
3.97

18

1.73

2.10

2.88

3.92

19
20
21
22

1.73
1.72
1.72
1.72

2.09
2.09
2.08
2.07

2.86
2.85
2.83
2.82

3.88
3.85
3.82
3.79

23
24

1.71
1.71

2.07
2.06

2.82
2.80

3.77
3.75

25
26

1.71
1.71

2.06
2.06

2.79
2.78

3.73
3.71

27
28
29
30

1.70
1.70
1.70
1.70

2.05
2.05
2.05
2.04

2.77
2.76
2.76
2.75

3.69
3.67
3.66
3.65

40
60
120

1.68
1.67
1.66

2.02
2.00
1.98

2.70
2.66
2.62

3.55
3.46
3.37

1.65

1.96

2.58

3.29

10

IBO 2010 KOREA


PRACTICAL TEST 4
ECOLOGY
________________________________________________________________________________

TASK III. (14 points) Interspecific interaction

Quantity

Materials
1. Two species model board (30 32 cm)

2. Transparent quadrat board (30 30 cm)

3. Electronic calculator

Introduction
Spiral shellfishes and clams live in the same habitat. In order to know whether there is an
interaction between these two species, we examine the distribution of each species in that habitat.

Q9. (2 points) Using the given quadrat board, observe whether the spiral shellfish and the clam are
absent and/or present in each quadrat. Write the number of quadrats you have observed in the box
in the answer sheet.

Q10. (2 points) The significance of the species distributions measured in this habitat can be examined
by using the Chi-square (2) test. The null hypothesis for the 2 test in this situation is that the
distribution of each species:
A. is nonrandom.
B. is independent of each other.
C. shows a mutually negative influence.
D. shows a mutually positive influence.
E. is influenced by a third species.
Put a checkmark () in the appropriate box in the answer sheet.

11

IBO 2010 KOREA


PRACTICAL TEST 4
ECOLOGY
________________________________________________________________________________

Q11. (4 points) To perform the 2 test, first determine the expected counts for each observational class.
For example, the expected counts of quadrats where both species are present is calculated by
multiplying the number of quadrats where one species is present with the number of quadrats
where the other species is present divided by the total number of quadrats. Compute the other
expected counts similarly to the nearest tenths (one decimal place) and fill the table in the answer
sheet.

Q12. (2 points) Using the function below, calculate the 2 value for this data set. Record your value to
the nearest hundredth (two decimal places) in the answer sheet.

Q13. (1 point) In order to evaluate the Chi-square value (2), the degree of freedom for the data set must
be determined (df). What is the degree of freedom for this data set? Record the value in the answer
sheet.

Q14. (2 points) Decide whether to reject or not reject the null hypothesis using the significance level
(probability, p) of 0.05. In the given 2 table, locate the degree of freedom in the appropriate
column. Compare your calculated 2 test statistic to the tabular 2 value to make your decision. Put
a checkmark () in the appropriate box in the answer sheet.

12

IBO 2010 KOREA


PRACTICAL TEST 4
ECOLOGY
________________________________________________________________________________

Q15. (1 point) Considering the spatial pattern of the distribution, what kind of interactionis likely to be
taking place between the two species? Choose all possible options and put a checkmark () in the
appropriate box in the answer sheet.
A. No interaction
B. Commensalism
C. Competition
D. Parasitism
E. Exclusion

13

IBO 2010 KOREA


PRACTICAL TEST 4
ECOLOGY
________________________________________________________________________________

Chi-square Table
Probability, p
Degree of freedom
1
2
3
4
5
6
7
8
9
10
11
12
13
14
15
16
17
18
19
20
21
22
23
24
25
26
27
28
29
30

0.99

0.95

0.05

0.01

0.001

0.000
0.020
0.115
0.297
0.554
0.872
1.239
1.646
2.088
2.558
3.05
3.57
4.11
4.66
5.23
5.81
6.41
7.02
7.63
8.26
8.90
9.54
10.20
10.86
11.52
12.20
12.88
13.57
14.26
14.95

0.004
0.103
0.352
0.711
1.145
1.635
2.167
2.733
3.325
3.940
4.58
5.23
5.89
6.57
7.26
7.96
8.67
9.39
10.12
10.85
11.59
12.34
13.09
13.85
14.61
15.38
16.15
16.93
17.71
18.49

3.84
5.99
7.82
9.49
11.07
12.59
14.07
15.51
16.92
18.31
19.68
21.03
22.36
23.69
25.00
26.30
27.59
28.87
30.14
31.41
32.67
33.92
35.17
36.42
37.65
38.89
40.11
41.34
42.56
43.77

6.64
9.21
11.35
13.28
15.09
16.81
18.48
20.09
21.67
23.21
24.73
26.22
27.69
29.14
30.58
32.00
33.41
34.81
36.19
37.57
38.93
40.29
41.64
42.98
44.31
45.64
46.96
48.28
49.59
50.89

10.83
13.82
16.27
18.47
20.52
22.46
24.32
26.13
27.88
29.59
31.26
32.91
34.53
36.12
37.70
39.25
40.79
42.31
43.82
45.32
46.80
48.27
49.73
51.18
52.62
54.05
55.48
56.89
58.30
59.70

14

IBO 2010 KOREA


PRACTICAL TEST 4
ECOLOGY
________________________________________________________________________________

TASK IV. (13 points) Prey choice model


Quantity
Materials
1. Prey model board (22 24 cm)

2. Electronic calculator

Introduction
A foraging animal encounters various types of prey items. Each type of prey can be characterized
by its energy content (E), the time required to search for that prey (searching time, Ts), and the time
required to capture and consume it (handling time, Th). Therefore, we can measure prey profitability
by the function E/(Ts+Th). In this situation, according to optimality theory, natural selection would
favor behaviors that maximize an animals net energy intake per amount of foraging time.
The behavioral options for a forager are whether to accept or to reject an item of a given prey type
when it is encountered. Assume that there are two kinds of prey item, Type 1 and Type 2. Let the
profitability be higher for Type 1 that is, E1/(Ts1+Th1) > E2/(Ts2+Th2). Thus, Type 1 items should
always be accepted. Prey profitability is density-dependant. That is, profitability of a prey species
changes if the prey species becomes less abundant.
On the boards for Site I and Site II, there are three prey items for gulls:
Prey A: Spiral shellfish

Prey B: Clam

Prey C: Razor clam

15

IBO 2010 KOREA


PRACTICAL TEST 4
ECOLOGY
________________________________________________________________________________

Q16. (2 points) For Site I, record the density of each of the prey species A, B, C (number of individuals
per m2, assuming that each quadrat is 1m x 1m). Calculate searching time (Ts) for each of the prey
species, where the species-specific searching time at density = 1 has been provided. Ts=
(1/density)a (sec). The value a is a species-specific constant. Calculate the values to nearest
hundredth (two decimal places).
Ts (sec)
Prey species
when the prey density is 1
Prey A

10

Prey B

15

Prey C

Q17. (2 points) After capturing a prey item, gulls fly high and drop the item to break its shell. The
forager repeats the behavior if the shell does not break. The table below indicates the drop height
and the average number of drops required at that height to break the preys shell. For each prey
type, indicate with a checkmark (), in the answer sheet, the optimal drop height that gulls should
choose, if they are optimal foragers.
Height of drop (m)

Average number of drops required to break shell

60

40

20

10

15

Prey A

16

IBO 2010 KOREA


PRACTICAL TEST 4
ECOLOGY
________________________________________________________________________________

Height of drop (m)

Average number of drops required to break shell

60

20

10

15

Height of drop (m)

Average number of drops required to break shell

30

10

10

15

Prey B

Prey C

Q18. (2 points) Gulls fly one meter up or down in 0.5 seconds. Given the optimal drop height for each
prey species, calculate the handling time (Th) for each prey item. Record the number in the box in
the answer sheet.

17

IBO 2010 KOREA


PRACTICAL TEST 4
ECOLOGY
________________________________________________________________________________

Q19. (3 points) The table below lists the average energy gain from eating an individual of each prey
species (kilojoules (KJ) per prey). Calculate the profitability of each prey species at Site I to the
nearest hundredth (two decimal places), and record the number in the box in the answer sheet.

Prey species

Energy (KJ per prey)

Prey A

Prey B

25

Prey C

Q20. (2 points) Of the following choices, what would be the optimal decision for the gulls at site I? Put
a checkmark () in the most appropriate box in the answer sheet.
A. Eat all of prey A.
B. Eat all of prey B.
C. Eat all of prey C.
D. Eat prey A at first and then switch to prey B.
E. Eat prey B at first and then switch to prey C.

18

IBO 2010 KOREA


PRACTICAL TEST 4
ECOLOGY
________________________________________________________________________________

Q21. (2 points) A gull finds an item of prey C in Site II. The gull can, however, decide not to take
this item and fly to Site I where it can search for prey B. Given that Site I requires 50
seconds of flying time from Site II, what should the gull do in order to maximize the
profitability of the next prey item, if it is an optimal forager? Distribution of the prey items in
Site II has been provided to you. Put a checkmark () in the most appropriate box in the answer
sheet.
A. The gull will eat the prey C in Site II.
B. The gull will move to Site I to search for prey B.
C. The gull will search for prey B in Site II.
D. The gull will move to site I to search for prey C.
E. The gull will search for prey A in Site II.

19

Country Code: ___________

Student Code: ___________

PRACTICAL TEST 4
Answer Sheet
ECOLOGY
Total Points: 51
Duration: 90 minutes

TASK I. (16 points)

Q1. (4 points)
Species

Population size

Starfish

Razor clam

Sea slater

15

Sea urchin

Fiddler crab

13

Octopus

Oyster

Mudskipper

Sea anemone

13

Density
(number of individuals/m2)

Q2. (2 points)
Community A

Species

Population
size

Starfish

Community B
Proportion of
relative
abundance

Species

Population
size

13

Fiddler crab

Razor clam

18

Barnacle

18

Sea slater

13

Sea
anemone

15

Sea urchin

12

Sea
cucumber

Fiddler
crab

11

Hermit crab

Gastropod

Gastropod

Oyster

12

Mudskipper

Sea
anemone

10

Total

106

Total

50

Proportion of
relative
abundance

Q3. (4 points)

Q4. (4 points)
Species diversity index
of community A (H'A)

Species diversity index


of community B (H' B)

Q5. (1 point)
A

Q6. (1 point)
A

TASK II. (8 points)


Q7. (4 points)
Number of individuals captured during the second sampling
Number of marked individuals recaptured
Estimate of the population size

Q8. (4 points)
t-value
SE
Confidence interval of the estimated population size

TASK III. (14 points)

Q9. (2 points)
Spiral shellfish
Observed count
Present
Clam

Present
Absent

Q10. (2 points)
A

Absent

Q11. (4 points)
Spiral shellfish

Expected count

Present

Absent

Present
Clam
Absent

Q12. (2 points)
2

Q13. (1 point)
Degree of freedom
(df)
Q14. (2 points)
Fail to reject
Null hypothesis

Reject

Q15. (1 point)
A

TASK IV. (13 points)

Q16. (2 points)
Prey species

Density
2

(number of individuals/m )

Ts (sec)

Ts (sec)

when the prey density is 1

at the Site I

Prey A

10

Prey B

15

Prey C

Q17. (2 points)

Prey A

Height of drop

Average number of drops

(m)

required to break shell

60

40

20

10

15

7
7

Optimal height for handling

Height of drop

Average number of drops

(m)

required to break shell

60

20

10

15

Height of drop

Average number of drops

(m)

required to break shell

30

10

10

15

Prey B

Prey C

Q18. (2 points)
Prey species

Handling time per prey (sec)

Prey A
Prey B
Prey C

Optimal height for handling

Optimal height for handling

Q19. (3 points)
Energy

Prey species

Prey profitability

(KJ per prey)


Prey A

Prey B

25

Prey C

Q20. (2 points)
A

Q21. (2 points)
A

PRACTICAL TEST 4
Answer Key
ECOLOGY
Total Points: 51
Duration: 90 minutes

TASK I. (16 points)

Q1. (4 points)
Population size

Density
(number of individuals/m2)

Starfish

15

0.31

Razor clam

20

0.41

Sea slater

15

0.31

Sea urchin

13

0.27

Fiddler crab

13

0.27

Octopus

10

0.20

Oyster

14

0.29

Mudskipper

11

0.22

Sea anemone

13

0.27

Species

1. One point will be subtracted for any error in rounding value and error in decimal
place.
2. In case of calculation error for any value, one point is subtracted for each error.
3. Only one point is subtracted for incorrect answers within each row of the table.

Q2. (2 points)
Community A

Community B

Species

Population
size

Proportion of
relative
abundance

Species

Population
size

Proportion of
relative
abundance

Starfish

13

0.12

Fiddler crab

0.04

Razor clam

18

0.17

Barnacle

18

0.36

Sea slater

13

0.12

Sea
anemone

15

0.30

Sea urchin

12

0.11

Sea
cucumber

0.04

Fiddler crab

11

0.10

Hermit crab

0.10

Gastropod

0.08

Gastropod

0.16

Oyster

12

0.11

Mudskipper

0.09

Sea
anemone

10

0.09

Total

106

Total

50

1. 0.5 point is subtracted for any error in rounding value and error in decimal place.
2. In case of calculation error for any value, 0.5 point is subtracted for each error.

Q3. (4 points)

1. Full points will be given for marks on the appropriate curve, and appropriate titles
and scales for the Y-axis.
2. Full points are given if participant make appropriate graph using data in Q2.
3. For incorrect marks or no marks, 2 points are subtracted.
4. For no title or scales on Y-axis, 2 points are subtracted.

Q4. (4 points)
Species diversity index
of community A (H' A)

Species diversity index


of community B (H' B)

2.15

1.51

1. Full points will be given for values between 2.10 2.19 for community A and 1.501.59 for community B.
2. 2 points for each value.
3. 1 point are substracted for each error in rounding value and decimal place.
4. Full Points are given if participant make appropriate calculation using data in Q2.
Q5. (1 point)
A

1. 1 point is given if participant make appropriate answer using the graph of Q3.
2. No point is given if participant mark only one out of both answers.
Q6. (1 point)
A

TASK II. (8 points)


Q7. (4 points)
Number of individuals captured during the second sampling

Participants Value

Number of marked individuals recaptured

Participants Value

Estimate of the population size

Use of the Excel Table

1. 1 point will be given if the participant wrote the first and second answers.
2. 0.5 point is subtracted if the participant did not round off the numbers.
3. 0.5 point is subtracted if the participant did not record one decimal place or recorded
more decimal places.
Q8. (4 points)
t-value

1.96

SE

Use of the Excel Table

Confidence interval of the estimated population size

Use of the Excel Table

1. Values found in Q.1 must be applied.


2. 0.5 point is subtracted if the participant did not round off the numbers.
3. 0.5 point is subtracted if the participant did not record one decimal place or recorded
more decimal places.
4. It is OK if the confidence interval is written in the range (X~X) or in the form of YY or
Y.
5. 1 point is subtracted when the participant wrote one wrong answer.
6. Confidence interval is accepted for the point within the range of the excel calculation
0.05

TASK III. (14 points)

Q9. (2 points)
Spiral shellfish
Observed count

Clam

Present

Absent

Present

15

12

Absent

16

1. 1 point is subtracted if the participant wrote one wrong answer.

Q10. (2 points)
A

1.

Plural choice is null.

Q11. (4 points)
Expected count

Spiral shellfish
Present

Absent

Present

11.6

15.4

Absent

9.4

12.6

Clam

1. 1 point is subtracted for each wrong answer.


2. 0.5 point is subtracted if the participant did not round off the numbers.
3. 0.5 point is subtracted if the participant did not record one decimal place or recorded
more decimal places.
4. Use the excel table for the evaluation when the observation counts are wrong (full point,
in the case of exact calculation).
Q12. (2 points)
2

3.96 or 3.89

1. Use the excel table for the evaluation when the answers of Q9 and Q11 are wrong (full
point in the case of exact calculation).
2. Full point is given if the value is within the range of 0.05
Q13. (1 point)
Degree of freedom
(df)

Q14. (2 points)
Fail to reject

Reject

Null hypothesis

If Q12 < 3.84, fail to reject is a correct answer.

Q15. (1 point)
A

1. 1 point is obtained if the participant chose only B or D.


2. If answer of Q14 is fail to reject, correct answer is A.

TASK IV. (13 points)

Q16. (2 points)
Density

Ts (sec)

Ts (sec)

(number of individuals/m2)

when the prey density is 1

at the Site I

Prey A

1.00

10

10

Prey B

0.75

15

20

Prey C

0.50

10

Prey species

1. 0.5 point is subtracted if the participant did not round off the numbers.
2. 0.5 point is subtracted if the participant did not record one decimal place or recorded
more decimal places.
3. 1 point is subtracted if the participant wrote one or two wrong answer within each row in
the table.

Q17. (2 points)

Prey A

Prey B

Prey C

Height of drop

Average number of drops

(m)

required to break shell

60

40

20

10

15

Height of drop

Average number of drops

(m)

required to break shell

60

20

10

15

Height of drop

Average number of drops

(m)

required to break shell

30

10

10

15

Optimal height for handling

Optimal height for handling

Optimal height for handling

Q18. (2 points)
Prey species

Handling time per prey (sec)

Prey A

80

Prey B

35

Prey C

30

1. 1 point is subtracted if the participant wrote one wrong answer.


Q19. (3 points)
Energy

Prey species

(KJ per prey)

Prey profitability

Prey A

0.08

Prey B

25

0.45

Prey C

0.13

1. 1 point is subtracted if the participant wrote one wrong answer.


2. 0.5 point is subtracted if the participant did not round off the numbers.
3. 0.5 point is subtracted if the participant did not record one decimal place or recorded
more decimal places.
4. Use the excel table for the evaluation using answers of Q16 and Q18 (full point, in the
case of exact calculation).
Q20. (2 points)
A

Q21. (2 points)
A

Plural choice is null.


10

INTERNATIONAL BIOLOGY OLYMPIAD


THEORY PROBLEMS

2009, Tsukuba, Japan















All IBO examination questions are published under the following Creative Commons license:



CC BY-NC-SA (Attribution-NonCommercial-ShareAlike) https://creativecommons.org/licenses/by-nc-sa/4.0/
The exam papers can be used freely for educational purposes as long as IBO is credited and
new creations are licensed under identical terms. No commercial use is allowed.

ENVELOPE COVER SHEET

Student Code: __________________

20th INTERNATIONAL BIOLOGY OLYMPIAD


Tsukuba, JAPAN

12th 19th July, 2009

THEORETICAL TEST: PART A


Time available: 120 minutes
GENERAL INSTRUCTIONS
1. Open the envelope after the start bell rings.
2. A set of questions and an answer sheet are in the envelope.
3. Write your 4-digit student code in every student code box.
4. The questions in Part A have only one correct answer. Mark the correct answer with X
on the Answer Sheet clearly, as shown below.
No. A
A0

5. Use pencils and erasers. You can use a scale and a calculator provided.
6. Some of the questions may be marked DELETED. DO NOT answer these questions.
7. Stop answering and put down your pencil IMMEDIATELY after the end bell rings.

IBO-2009 JAPAN
THEORETICAL TEST Part A

Student Code:

___________

20th INTERNATIONAL BIOLOGY OLYMPIAD


Tsukuba, JAPAN

12th 19th July, 2009

THEORETICAL TEST: PART A


Time available: 120 minutes
GENERAL INSTRUCTIONS
1. Write your 4-digit student code in every student code box.
2. The questions in Part A have only one correct answer. Mark the correct answer with X
on the Answer Sheet clearly, as shown below.
No. A
A0

3. Use pencils and erasers. You can use a ruler and a calculator provided.
4. Some of the questions may be marked DELETED. DO NOT answer these questions.
5. The maximal points of Part A is 81 (1.5 point each question).
6. Stop answering and put down your pencil IMMEDIATELY after the end bell rings.
GOOD LUCK!!
1

IBO-2009 JAPAN
THEORETICAL TEST Part A

Cell Biology

A1. Which treatment is most effective in breaking as many hydrogen bonds as possible in an
aqueous solution (pH 7.0) of 1 mg/mL DNA and 10 mg/mL protein?

A. Addition of hydrochloric acid to make the pH 1.0.


B. Addition of sodium hydroxide solution to make the pH 13.0.
C. Addition of urea to a concentration of 6 mol/L.
D. Addition of sodium dodecyl sulfate (a detergent) to a concentration of 10 mg/mL.
E. Heating the solution to 121C.
F. Freezing the solution to -80C.

IBO-2009 JAPAN
THEORETICAL TEST Part A

A2. For the elongation of biopolymer molecules, there are two basic mechanisms, as shown
below. In Type I elongation, the activation group (marked with an X) is released from the
chain of growth. In Type II elongation, the activation group is released from the unit
which is coming into the chain of growth. By which of these mechanisms are DNA (D),
RNA (R),and protein (P) biosynthesized?

Type I

Type II

(D)

(R), (P)

(P)

(D), (R)

none

(D), (R), (P)

(R), (P)

(D)

(D), (R)

(P)

(D), (R), (P)

none

IBO-2009 JAPAN
THEORETICAL TEST Part A

A3. The movement of a ciliated protozoan is controlled by a protein called RacerX. When
this protein binds to another protein, Speed, found at the base of the cilia, it stimulates
the cilia to beat faster and the protozoan to swim faster. Speed can only bind to RacerX
after phosphorylation of a specific threonine residue. How would you expect the mutant
protozoan to behave if this threonine residue in Speed is replaced by an alanine
residue?

A. Swims fast occasionally.


B. Always swims fast.
C. Never swims fast.
D. Switches rapidly back and forth between fast and slow swimming.
E. Cannot move at all.

IBO-2009 JAPAN
THEORETICAL TEST Part A

A4. It is suggested that Alzheimer's disease is manifested by increased accumulation of a


small peptide known as -amyloid (A-, 40-42 residues). Production of A- occurs by
proteolytic cleavage from a much longer protein APP, a membrane-inserted protein, by
two proteases. The figure below shows the hypothesis for the production of the A-
molecule (the gray shaded box), displaying the sequential action of -secretase to form
the N-terminus of A- and -secretase to cleave its substrate within a phospholipid
membrane to produce the C-terminus of A-. The produced A- monomers then
associate to form insoluble oligomers and toxic fibrils.

Sites of association of APP with the membrane

APP
-secretase -secretase

A- (-amyloid)

Fibrils

Low oligomers

Which of the following is effective as an anti-Alzheimer therapy based on the above


mechanisms?

I. Inhibiting the activity of -secretase


II. Inhibiting the membrane targeting of -secretase
III. Inhibiting the oligomerization of A-
5

IBO-2009 JAPAN
THEORETICAL TEST Part A

IV. Enhancing the cellular mechanism of removal and degradation of A- oligomers

A. Only I, II, IV
B. Only I, II, III
C. Only I, III, IV
D. Only II, III, IV
E. I, II, III, IV

IBO-2009 JAPAN
THEORETICAL TEST Part A

A5. Human acetaldehyde dehydrogenase acts as a tetramer. Two alleles, N encoding a


normal polypeptide and M encoding a mutant polypeptide, are known for the gene of this
enzyme. Tetramers containing one or more mutant polypeptides have effectively no
enzymatic activity. If the acetaldehyde dehydrogenase activity of the NN homozygote
cells is 1, what is the activity of the NM heterozygote cells, assuming that both alleles are
expressed at equal rates?

A. 1/2
B. 1/4
C. 1/8
D. 1/16
E. 1/32

IBO-2009 JAPAN
THEORETICAL TEST Part A

A6. In 1961 Mitchell proposed a highly original explanation for ATP synthesis, which he
called the chemiosmotic coupling model. Which of the following is correct?

A. ATP synthesis in mitochondria can be explained by the chemiosomotic model, but


in chloroplasts it cannot.
B. ATP synthesis in mitochondria and chloroplasts can be explained by the
chemiosomotic model only when the concentration of H+ ions in the cell is higher
than 0.1 mmol/L.
C. The energy source for mitochondria is electrons from nutrients, but for chloroplasts
the energy source is electrons from water.
D. In mitochondria H+ ions are pumped into the matrix, but in chloroplasts they are
pumped into the thylakoid lumen.
E. H+ ions are transferred through ATP synthase both in mitochondria and
chloroplasts.

IBO-2009 JAPAN
THEORETICAL TEST Part A

A7. A scientist, studying the process of photosynthesis, illuminates a culture of unicellular


green algae for a certain period of time. Then she turns off the light and adds radioactive
CO2 by bubbling it in the culture for 30 minutes. Immediately she measures radioactivity
in the cells. What is she likely to observe?

A. No radioactivity in the cells, because light is necessary to produce sugars starting


from CO2 and water.
B. No radioactivity in the cells, because CO2 is used to produce O2 during the
light-dependent reactions.
C. No radioactivity in the cells, because CO2 is taken by the plant cells only during
illumination.
D. Radioactivity in the cells, because CO2 is used to produce sugars even in the dark.
E. Radioactivity in the cells, because CO2 is incorporated into NADPH in the dark.

IBO-2009 JAPAN
THEORETICAL TEST Part A

A8. Which of the following are true for the relative permeabilities of human red blood cells
and artificial phospholipid bilayer vesicles (called artificial vesicles hereafter) to glucose
and ethanol?

I. Both red blood cells and artificial vesicles are more permeable to glucose than to
ethanol.
II. Both red blood cells and artificial vesicles are more permeable to ethanol than to
glucose.
III. In both red blood cells and artificial vesicles, the permeability to ethanol is almost
the same as that to glucose.
IV. While red blood cells and artificial vesicles show almost the same permeability to
glucose, red blood cells have a higher permeability to ethanol than artificial
vesicles.
V. While red blood cells and artificial vesicles show almost the same permeability to
ethanol, red blood cells have a higher permeability to glucose than artificial
vesicles.

A. I, IV
B. I, V
C. II, IV
D. II, V
E. III, IV
F. III, V

10

IBO-2009 JAPAN
THEORETICAL TEST Part A

A9. A previously unknown organism that lacks nuclear membrane and mitochondria has just
been discovered. Which of the following would this organism most likely possess?

A. Lysosome
B. Cilium
C. Endoplasmic reticulum
D. Chloroplast
E. Ribosome

11

IBO-2009 JAPAN
THEORETICAL TEST Part A

A10. In eukaryotic cells, the oxidative phosphorylation reactions are catalyzed by various
enzymes. Which of the following is correct?

A. All of these enzymes are coded in nuclear DNA, synthesized in ribosomes and
imported into mitochondria.
B. Some of these enzymes are coded in mitochondrial DNA. Their messenger RNA is
exported outside mitochondria and the enzymes are synthesized in ribosomes.
The enzymes are then imported back into mitochondria.
C. Some of them are coded in mitochondrial DNA and synthesized in mitochondrial
ribosomes.
D. All of them are coded in mitochondrial DNA and synthesized in mitochondrial
ribosomes.
E. A copy of mitochondrial DNA is exported outside mitochondria. The synthesized
enzymes are imported into mitochondria.

12

IBO-2009 JAPAN
THEORETICAL TEST Part A

A11. Jellyfish-derived genes encoding fluorescent proteins, such as green fluorescent


protein (GFP), are widely used in molecular biological studies particularly for the
purpose of tagging and visualizing proteins of interest. PLX is a plant gene encoding an
unknown protein. A chimeric gene consisting of the PLX gene and the GFP gene was
constructed to produce a PLX-GFP fusion protein under an inducible promoter, and
introduced into mesophyll protoplasts by electroporation. The following figures show
schematic images of fluorescence micrographs of the same protoplast at various times
after the induction of PLX-GFP expression.

Before the induction


(dotted line indicates the
protoplast outline)

Shortly after the


induction

Long after the


induction

In consideration of the change in the spatial pattern of the fluorescent signals,


speculate which of the following cell structures most likely corresponds to the
fluorescent signals in the middle picture.

A. Nucleoli
B. Mitochondria
C. Golgi apparatuses
D. Nuclear pores
E. Chloroplasts
F. Peroxisomes

13

IBO-2009 JAPAN
THEORETICAL TEST Part A

A12. The recognition sequence for the restriction endonuclease AvaI is CYCGRG, where Y
is any pyrimidine and R is any purine. What is the expected distance (in bp = base pairs)
between the restriction sites of AvaI in a long, random DNA sequence?

A. 4096 bp
B. 2048 bp
C. 1024 bp
D. 512 bp
E. 256 bp
F. 64 bp

14

IBO-2009 JAPAN
THEORETICAL TEST Part A

A13. The arabinose operon of Escherichia coli is not expressed in the absence of arabinose.
This is attributable to the AraC protein, which binds to the promoter of the arabinose
operon and acts as a suppressor to prevent its transcription. Normally the arabinose
operon is expressed in the presence of arabinose. In mutants that lack the AraC gene,
however, the arabinose operon is not expressed even in the presence of arabinose.
Based on this information, which of the following can be reasonably inferred with respect
to AraC?

A. The transcription of the AraC gene is induced by arabinose.


B. The transcription of the AraC gene is blocked by arabinose.
C. The AraC protein is converted into an activator in the presence of arabinose.
D. The AraC protein is degraded in the presence of arabinose.

15

IBO-2009 JAPAN
THEORETICAL TEST Part A

A14. Nucleotide sequence duplications in a gene cause severe effects on its function in
some cases while they do not in other cases. Which of the following duplication events
would most likely result in the synthesis of a non-functional protein?

A. A base pair is duplicated just before the translation initiation site.


B. Three base pairs are duplicated just before the translation initiation site.
C. A base pair is duplicated in the coding region near the translation initiation site.
D. Three base pairs are duplicated in the coding region near the translation initiation
site.
E. A base pair is duplicated in the coding region near the stop codon.
F. Three base pairs are duplicated in the coding region near the stop codon.

16

IBO-2009 JAPAN
THEORETICAL TEST Part A

Plant Anatomy and Physiology

A15. Cell walls of vessels and tracheids of vascular plants contain a phenolic polymer called
"lignin", which together with cellulose confers mechanical strength to these
water-conducting tissues. If vessels/tracheids are deficient in lignin, they:

A. burst outward when transpiration is very active.


B. burst outward when transpiration is very inactive.
C. collapse inward when transpiration is very active.
D. collapse inward when transpiration is very inactive.

17

IBO-2009 JAPAN
THEORETICAL TEST Part A

A16. The following micrograph shows a part of the transverse section of the stem of a dicot
plant. Which arrow indicates the direction towards the center of the stem?

18

IBO-2009 JAPAN
THEORETICAL TEST Part A

A17. The plant tissue shown below is likely to be from a:

A. xerophyte
B. mesophyte
C. halophyte
D. hydrophyte
E. epiphyte

19

IBO-2009 JAPAN
THEORETICAL TEST Part A

A18. To examine the effect of phytohormones P1 and P2 in plant tissue culture, leaf
segments were excised from plants grown under the light, placed on medium that
contained P1 and/or P2, and cultured in the dark. As a control experiment, leaf segments
were cultured without P1 or P2 in the dark.
(a) When only P1 was added to the medium, adventitious roots formed on the explants.
(b) When only P2 was added to the medium, neither organogenesis nor callus formation
occurred. The explants retained green color for a longer period than the explants of
the control experiment.
(c) When both P1 and P2 were added to the medium, callus formed on the explants.
Based on this information, P1 and P2 were:
P1

P2

Auxin

Gibberellin

Auxin

Cytokinin

Gibberellin

Auxin

Gibberellin

Cytokinin

Cytokinin

Gibberellin

Cytokinin

Auxin

20

IBO-2009 JAPAN
THEORETICAL TEST Part A

A19. Exalbuminous (endospermless) seeds of a certain plant species were immersed in


pure water, germinated, and grown in the dark. Total nitrogen content and soluble
nitrogen content (nitrogen in low-molecular-weight compounds such as amino acids)
were measured for cotyledons and the other parts of the seedlings. The results are
shown in the following figures. With respect to the nitrogen metabolism in seedlings of

Soluble nitrogen content (mg)

Total nitrogen content (mg)

this plant, which of the following statements is the most appropriate explanation?

Rest of
seedlings

Cotyledons

Days after germination

Rest of
seedlings

Cotyledons

Days after germination

Proteins in cotyledons were degraded to produce amino acids,


A. which were eventually consumed as nitrogen sources for the growth of cotyledons.
B. which were eventually excreted from seedlings as wastes.
C. which were translocated and provided almost all of the nitrogen sources required
for the initial growth of seedlings.
D. which were translocated and provided about half of the nitrogen sources required
for the initial growth of seedlings.

21

IBO-2009 JAPAN
THEORETICAL TEST Part A

A20. Two alleles G and g are present at a particular locus of a fern species. Spores were
collected from a heterozygous sporophyte with Gg genotype of the fern species.
Gametophytes were grown from the spores and self-fertilized by isolating each sexually
matured gametophyte. What is the expected ratio of the GG : Gg : gg genotypes of the
sporophytes?

A.

1:2:1

B.

2:1:1

C.

3:0:1

D.

0:3:1

E.

1:0:1

F.

0:1:1

22

IBO-2009 JAPAN
THEORETICAL TEST Part A

A21. Totally submerged aquatic plants can cause a pH change in the surrounding water
when they carry out photosynthesis. What pH change happens and what causes it?

A. The pH falls because carbon dioxide is absorbed.


B. The pH rises because carbon dioxide is absorbed.
C. The pH falls because oxygen is released.
D. The pH rises because oxygen is released.

23

IBO-2009 JAPAN
THEORETICAL TEST Part A

A22. If the ambient temperature rises by 5C, photorespiration would:

A. Increase in rice, decrease in maize


B. Increase in maize, decrease in rice
C. Increase in rice, little effects on maize
D. Increase in maize, little effects on rice
E. Increase in both species
F. Decrease in both species

24

IBO-2009 JAPAN
THEORETICAL TEST Part A

Animal Anatomy and Physiology

A23. When fertilized sea urchin eggs were reared in sea water containing actinomycin D, an
inhibitor of transcription, eggs developed normally until the blastula stage, but stopped
development after that. This is due to the fact that in embryos the process of
transcription does not take place during the cleavage period, and the proteins necessary
for the development are translated from mRNA stored in the eggs.

If protein synthesis is measured during this experiment, which of the following graphs
would be obtained?

Protein synthesis

Onset of gastrulation

Time after fertilization


Normal sea water
Sea water containing actinomycin D

25

IBO-2009 JAPAN
THEORETICAL TEST Part A

A24. At the 16-cell stage, the sea urchin embryo consists of three types of cells: eight
mesomeres, four macromeres and four micromeres, from animal pole to vegetal pole.
When four micromeres were labeled by fluorescent dye, all the spicule forming cells in
the resulting 2-day-old larva were fluorescent (see figure).

Thus, in normal larvae, spicule forming cells are derived solely from micromeres.
However, even if all the micromeres are removed from a 16-cell embryo,
spiculogenesis still occurs in 2-day-old larva. From this we can conclude that:

A. all the cells in a 16-cell-stage embryo can form spicules when receiving an
appropriate signal from micromeres.
B. all the cells in a 16-cell-stage embryo can form spicules when the micromeres are
removed.
C. micromeres or their descendent cells send a spiculogenesis-inhibiting signal to
other cells.
D. micromeres or their descendent cells send a spiculogenesis-inducing signal to
other cells.

26

IBO-2009 JAPAN
THEORETICAL TEST Part A

A25. The crab-eating frog is a unique amphibian which has adapted to the marine habitat
and lives in mangroves. Different from marine bony fish, these frogs deal with the
osmotic problem by:

A. drinking sea water and excreting excess salt.


B. excreting a large amount of excess water as urine.
C. excreting nitrogen waste as ammonia.
D. storing urea in their body fluid.

27

IBO-2009 JAPAN
THEORETICAL TEST Part A

A26. Which of the following states occurs if the lung alveoli lose their elasticity?

I. Residual volume decreases.


II. pO2 in the air inhaled has to increase in order to keep the saturation of hemoglobin
at the same level.
III. Blood pH increases.

A. Only I
B. Only II
C. Only III
D. I and II
E. I and III
F. II and III

28

IBO-2009 JAPAN
THEORETICAL TEST Part A

A27. Which of the following statements about skeletal muscle is NOT correct?

A. The length (distance) of a single muscle contraction depends on the concentration


of Ca2+ ions in the sarcoplasmic reticulum.
B. Muscles with short sarcomeres contract faster than muscles with long sarcomeres.
C. The velocity of muscle contractions is determined by myosin-ATPase activity.
D. Tetanus is the effect of repeated stimulations within a very short interval.
E. Rigor mortis (death rigidity) appears when the concentration of Ca2+ in cytoplasm is
high but ATP is lacking.

29

IBO-2009 JAPAN
THEORETICAL TEST Part A

A28. Which of the following would occur if a neuron was experimentally stimulated
simultaneously at both ends?

A. The action potentials would pass in the middle and travel to the opposite ends.
B. The action potentials would meet in the middle and then be propagated back to
their starting positions.
C. The action potentials would stop as they meet in the middle.
D. The stronger action potential would override the weaker action potential.
E. Summation would occur when the action potentials meet in the middle, resulting in
a larger action potential.

30

IBO-2009 JAPAN
THEORETICAL TEST Part A

A29 What happens when the pancreatic duct of a certain mammal is temporarily ligated for
an experiment? Note that carbohydrate and other nutrients in the diet are in proper
amounts and ligation of the pancreatic duct is not critical for survival of the animal.

The amount of carbohydrate:


A. increases in feces, decreases in urine.
B. increases in feces, does not change in urine.
C. decreases in feces, increases in urine.
D. decreases in feces, does not change in urine.
E. increases both in feces and urine.
F. decreases both in feces and urine.

31

IBO-2009 JAPAN
THEORETICAL TEST Part A

A30. Shown is the change of glucose concentration in the blood, measured by taking small
blood samples from the fingertip of a person who drank a solution containing 50 g of
glucose.

Time after drinking the solution

Glucose conc. in the blood

(min)

(mmol/L)

4.9

15

6.1

30

7.7

45

6.4

60

4.2

90

4.2

120

4.0

150

4.8

Has the glucose concentration at any time during the experiment been equal to or
higher than 7.7 mmol/L in the hepatic portal vein and the hepatic vein?

hepatic portal vein

hepatic vein

A.

no

no

B.

no

yes

C.

yes

no

D.

yes

yes

32

IBO-2009 JAPAN
THEORETICAL TEST Part A

A31. A substance from the plant Gymnema sylvestre blocks the sweet taste of sugar and
also blocks absorption of sugar by the small intestine. What can be assumed from these
two phenomena?

A. It metabolizes sucrose to glucose and fructose.


B. It polymerizes sugar into oligosaccharides.
C. It binds with sugar receptors and transporters.
D. It binds with certain neurotransmitter receptors and transporters.
E. It binds with insulin receptors.

33

IBO-2009 JAPAN
THEORETICAL TEST Part A

A32. When a species of poisonous fish was fertilized in vitro and cultured in an indoor plastic
tank filled with artificial seawater, they were never poisonous. Young fish grown in this
tank were next divided into two groups and placed in separate pens in a bay where
they were exposed to real seawater. One pen had a horizontal net that prevented the
fish from reaching the sea bottom, while the other pen had no horizontal net.
Subsequently, no poison was detected from the fish cultured in the pen with the net, but
poison was found in fish from the other pen.

What do you conclude from this experiment? To be toxic:

I. some component not in artificial seawater but in natural seawater is necessary.


II. it is necessary that they grow up to adults.
III. it is necessary that they can reach the sea bottom.

A. Only I
B. Only II
C. Only III
D. Both I and II
E. Both I and III
F. Both II and III

34

IBO-2009 JAPAN
THEORETICAL TEST Part A

A33. What can be most likely inferred from the following statements (1 to 4) about a disease
of patient X?

1. Patient X has a disease that makes her very sensitive to infection by bacteria and
viruses.
2. The IgG gene of this patient is normal.
3. This disease is caused by the abnormality of gene x which does not work at all.
4. When T cells of a normal person and B cells of patient X are mixed and cultured in
the presence of reagents that activate these cells, IgG is secreted into the culture
medium. However, when B cells of a normal person and T cells of patient X are
combined, IgG is not secreted.

A. Gene x needs to be expressed in B cells for the production of IgG.


B. T cells of patient X are normal.
C. IgG is produced by T cells.
D. Gene products of gene x are necessary for T cells to induce B cells to produce
IgG.
E. The genome of B cells does not contain gene x, while that of T cells does.

35

IBO-2009 JAPAN
THEORETICAL TEST Part A

A34. The diagram shows a simplified kidney tubule and associated blood vessels, and the
table shows the presence or absence of substances (X, Y, Z) in each part (16) of the
diagram.
X

Present

Present

Present

Present

Present

Present

Absent

Present

Absent

Absent

Present

Absent

Present

Present

Absent

Present

Present

Absent

Identify the substances X to Z.


X

Urea

Glucose

Proteins

Urea

Proteins

Glucose

Glucose

Urea

Proteins

Glucose

Proteins

Urea

Proteins

Glucose

Urea

Proteins

Urea

Glucose

36

IBO-2009 JAPAN
THEORETICAL TEST Part A

Ethology

A35. The vampire bat of Costa Rica is often not able to acquire blood from a mammal on a
given night. Wilkinson (1984) trapped bats which were not allowed to feed for a night
and found that they were given regurgitated blood by certain cave-mates. Based on this
knowledge, which of the following observations are indispensable to confirm the
occurrence of reciprocal altruism in this species?

Data showing that:


I. blood is exchanged only between kin.
II. blood is exchanged between non-kin.
III. weak bats are frequently given blood even if they cannot give it to others.
IV. bats who are given blood donate it to those who have given it to them before.

Combinations:
A. Only I
B. Only IV
C. I, III
D. I, IV
E. II, III
F. II, IV

37

IBO-2009 JAPAN
THEORETICAL TEST Part A

A36. In a certain bird species, territory-holding males are sexually mature, have red chest
feathers and aggressively drive out intruders. Several models, shown below, were built
to test territory defense in this species. What is the most likely sequence of attack on
these models in decreasing order of aggression?

I. A model of a normal juvenile bird with brown chest feathers


II. A model of a normal adult bird with red chest feathers
III. A model of an adult bird with brown chest feathers
IV. A model of a juvenile bird with red chest feathers

Sequences
A. I III IV II
B. I IV III II
C. II III IV I
D. II IV III I

38

IBO-2009 JAPAN
THEORETICAL TEST Part A

Genetics and Evolution

A37. A man with a genetic disease marries a phenotypically normal woman. They have four
girls and four boys; all of the girls have the same disease as their father, but none of the
boys does. What is the most likely explanation?

The disease is caused by:


A. an autosomal dominant allele.
B. an autosomal recessive allele.
C. an X-linked dominant allele.
D. an X-linked recessive allele.
E. a Y-linked allele.

39

IBO-2009 JAPAN
THEORETICAL TEST Part A

A38. There is a degenerative disease which develops in people between 35 and 45 years
old. It is caused by a dominant allele. A couple has two children, who are both younger
than 20 years old. One parent has the disease (heterozygote), but the other parent, who
is 50 years old, does not. What is the probability that the both children will develop this
disease when they become older?

A. 1/16
B. 3/16
C. 1/4
D. 9/16
E. 3/4

40

IBO-2009 JAPAN
THEORETICAL TEST Part A

A39. There are n+1 alleles at a particular locus on an autosome. The frequency of one allele
is 1/2 and the frequencies of the other alleles are all 1/(2n). Under the assumption of
Hardy-Weinberg equilibrium, what is the total frequency of heterozygotes?

A. (n 1)/(2n)
B. (2n 1)/(3n)
C. (3n 1)/(4n)
D. (4n 1)/(5n)
E. (5n 1)/(6n)

41

IBO-2009 JAPAN
THEORETICAL TEST Part A

A40. At a locus for an enzyme which is inherited independently of sex, the frequencies of
genotypes in a population were as follows.

FF

FS

SS

Female

30

60

10

Male

20

40

40

Predict the frequency of the FS genotype in the next generation, assuming that they will
mate randomly.

A. 0.46
B. 0.48
C. 0.50
D. 0.52
E. 0.54

42

IBO-2009 JAPAN
THEORETICAL TEST Part A

A41. How does the occurrence of self-fertilization relative to cross-fertilization affect the
fixation of an advantageous and recessive allele that newly appeared in a population by
mutation?

A. The allele will be fixed most quickly when the relative occurrence of self-fertilization
is highest.
B. The allele will be fixed most quickly when the relative occurrence of self-fertilization
is lowest.
C. The allele will be fixed most quickly when the relative occurrence of self-fertilization
is moderate.
D. The relative occurrence of self-fertilization does not affect the fixation of the allele.
E. The relative occurrence of self-fertilization affects the fixation of the allele only
when the population is very small.

43

IBO-2009 JAPAN
THEORETICAL TEST Part A

A42. The following table shows the number of estimated nucleotide substitutions that have
occurred in a gene among seven species.

The number of estimated nucleotide substitutions between each pair of species

a
b
c

39

72

128

126

159

269

81

130

128

158

268

129

127

157

267

56

154

271

151

268

d
e
f

273

Which is the most appropriate tree that shows the phylogenetic relationship among
these seven species?

44

IBO-2009 JAPAN
THEORETICAL TEST Part A

A.

B.

C.

D.

E.

F.

45

IBO-2009 JAPAN
THEORETICAL TEST Part A

A43. Suppose that at a neutrally evolving genomic region of a species the mutation rate from
the base pair GC to AT is three-times the mutation rate from AT to GC. What is the
expected GC content at equilibrium?

A. 1/2
B. 1/3
C. 1/4
D. 1/5
E. 1/6

46

IBO-2009 JAPAN
THEORETICAL TEST Part A

A44. A species of insect was found to have developed resistance to a commonly used
insecticide. Which of the following is the most likely explanation?

A. Stabilizing selection caused development of resistance in the insect population.


B. The original gene pool included genes that conferred resistance to the insecticide.
C. The insecticide stimulated development of resistance in certain individuals and this
was inherited.
D. The insecticide caused a mutation that was favorable to resistance and this was
inherited.

47

IBO-2009 JAPAN
THEORETICAL TEST Part A

A45. Darwin's finches are a prime example of adaptive radiation. Which of the following best
describes this adaptive radiation correctly?

A. The genetic variability that can be found among individuals of the same species.
B. The evolutionary process by which different forms, adapted to different niches,
arose from a common ancestor.
C. A sudden diversification of a group of organisms from closely related species.
D. The evolutionary process that allows for the changes that occur within the same
lineage.
E. The evolutionary process of adaptation of species through a kind of
polymorphism.

48

IBO-2009 JAPAN
THEORETICAL TEST Part A

A46. Multigene families are groups of two or more identical or very similar genes. Which of
the following statements about multigene families is correct?

A. Globin gene families do not have pseudogenes, because globins are essential for
oxygen transport.
B. Ribosomal RNA gene families in multicellular eukaryotes have many identical
genes, because many ribosomes are required for active protein synthesis.
C. Compared with multicellular eukaryotes, prokaryotes have many multigene
families, because prokaryotes have to reproduce very quickly.
D. The number of genes in a multigene family always increases by unequal crossing
over.

49

IBO-2009 JAPAN
THEORETICAL TEST Part A

Ecology

A47. The following table shows the net primary productivity and biomass without soil organic
matter in five ecosystems.
Ecosystem

Net primary productivity (g/m2/year)

Biomass (kg/m2)

Tropical rainforest

2200

45

2000

15

II

1200

30

III

900

Boreal forest

800

20

Choose from A to F in the table below the most appropriate combination of ecosystems for I,
II and III above.
I

II

III

African dry savanna

Tropical swamp & marsh

Temperate deciduous forest

African dry savanna

Temperate deciduous forest

Tropical swamp & marsh

Temperate deciduous forest

African dry savanna

Tropical swamp & marsh

Temperate deciduous forest

Tropical swamp & marsh

African dry savanna

Tropical swamp & marsh

African dry savanna

Temperate deciduous forest

Tropical swamp & marsh

Temperate deciduous forest

African dry savanna

50

IBO-2009 JAPAN
THEORETICAL TEST Part A

A48. The diagram below represents the relationships between organisms in a remote pond
ecosystem.

Consumer 4

Consumer 3

Consumer 1

Producer 1

Detritivore 1

Consumer 2

Producer 2

Producer 3

1
From this information, which of the following is the most likely to be correct?

A. DDT present in the ecosystem would accumulate to the highest concentrations in


the tissues of Detritivore 1.
B. The introduction of Consumer 4 individuals from an external population would lead
to a temporary increase in numbers of Producer 2.
C. Disease in the Producer 1 population would lead to an increase in the Producer 3
population.
D. Extermination of Consumer 3 would cause a sustained increase in the population
of Consumer 2.
E. Consumer 1 is more adaptable with regard to its food source than Consumer 3.

51

IBO-2009 JAPAN
THEORETICAL TEST Part A

A49. The table below shows the results of measurements of production in two ecosystems in
the temperate zone: a rainforest and a field with an annual crop. All results are stated in
MJ/m2 / year (1 MJ = 106J).

[I] Rainforest

[II] Field with an


annual crop

Gross Primary Production (GPP)

188

102

Respiration (autotrophs)

134

38

Respiration (heterotrophs)

54

Of these two ecosystems, which has a higher ratio of respiration by heterotrophic


organisms to net primary production (NPP)? What is the reason? Choose the correct
option from A to F.

A. [I] < [II]The rainforest has larger GPP and more consumers than the crop field.
B. [I] < [II]The rainforest has larger NPP and more consumers than the crop field.
C. [I] < [II]The rainforest has larger NPP and less consumers than the crop field.
D. [II] < [I]The rainforest has smaller GPP and more consumers than the crop field.
E. [II] < [I]The rainforest has smaller NPP and more consumers than the crop field.
F. [II] < [I]The rainforest has smaller NPP and less consumers than the crop field.

52

IBO-2009 JAPAN
THEORETICAL TEST Part A

A50. What does the energy input into most food webs typically depend on? Choose the most
likely factor from the following.

A. Grazing rate of the primary consumers


B. Material cycling efficiency rate of the whole ecosystem
C. Efficiency rate of producers converting solar radiation energy into chemical energy
D. Action of nitrogen-fixing bacteria
E. Heat-energy costs due to respiration within each trophic level

53

IBO-2009 JAPAN
THEORETICAL TEST Part A

A51. Which factor most promotes the stability of population dynamics in a developed
terrestrial ecosystem?

A. Food webs that have many trophic levels each of which consists of few species
only
B. A few species of producers with very high production rates
C. Rapid nutrient recycling by active decomposers
D. Food webs that have very few trophic levels and limited niche overlaps
E. A few eminent and competitively-dominant species

54

IBO-2009 JAPAN
THEORETICAL TEST Part A

A52. Animal species X and Y have a temporal negative correlation of population


abundances, in which arrows indicate the anti-clock-wise (counter-clock-wise) orbit of
population dynamics. Choose the most likely combination of explanation and its
reasoning.

No. of individuals
in species Y

No. of individuals in species X

Relationship between
Reasoning
species X and Y
interspecific competition Y decreases at high density of X
A.
between X and Y

Y increases at low density of X

interspecific competition Y increases at intermediate density of X


B.
between X and Y

C.

X decreases at intermediate density of Y

predator (X) and prey (Y)


Y decreases when X increases from low density
Y increases when X decreases from high density

D.

prey (X) and predator (Y)

55

IBO-2009 JAPAN
THEORETICAL TEST Part A

Biosystematics

A53. The following phylogenetic tree shows the relationships among Antarctic icefish and
their relatives. Icefish refer to all the species in the tree that have lost hemoglobin and
thus possess clear blood. Some icefish species also lost myoglobin which is usually
found in muscle cells. In these species, myoglobin lost its function due to distinct
mutations. In addition, icefish and relatives possess an anti-freezing glycoprotein to
arrest the growth of ice crystals in their tissues. To the right of the tree, whether or not
each species possess hemoglobin, myoglobin and the anti-freezing glycoprotein is
shown. What conclusion can you draw from the tree?

56

IBO-2009 JAPAN
THEORETICAL TEST Part A

A. Anti-freezing glycoprotein has originated in the icefish clade relatively recently.


B. Myoglobin was lost multiple times in the icefish clade.
C. The anti-freezing glycoprotein was necessary before the icefish could lose
hemoglobin.
D. The loss of hemoglobin appears to be a more recent trait than the loss of
myoglobin.
E. Because myoglobin can substitute for the functions of hemoglobin, icefish could
lose hemoglobin.

57

IBO-2009 JAPAN
THEORETICAL TEST Part A

A54. A list of the shared derived characters for some metazoan phyla is shown below.
Identify all the phylogenetic tree(s) which are consistent with the statements below.

I. Presence of trochophore larva is a shared derived character of the Mollusca and


the Annelida.
II. Molting is a shared derived character of the Arthropoda and the Nematoda.
III.Presence of a notochord is a shared derived character of the Urochordata and
Cephalochordata.
IV. Developmental fate of blastopore to form the anus is a shared derived character of
the Urochordata, Cephalochordata and Echinodermata.

58

IBO-2009 JAPAN
THEORETICAL TEST Part A

A. a
B. a, c
C. a, d
D. b, c
E. a, b, d
F. a, c, d.

*****

END OF PART A

59

*****















All IBO examination questions are published under the following Creative Commons license:



CC BY-NC-SA (Attribution-NonCommercial-ShareAlike) https://creativecommons.org/licenses/by-nc-sa/4.0/
The exam papers can be used freely for educational purposes as long as IBO is credited and
new creations are licensed under identical terms. No commercial use is allowed.

IBO-2009 JAPAN
THEORETICAL TEST Part B

Country Code___________

20th INTERNATIONAL BIOLOGY OLYMPIAD


Tsukuba, JAPAN 12th 19th July, 2009

THEORETICAL TEST: PART B


Time available: 150 minutes
GENERAL INSTRUCTIONS
1. Write your 4-digit student code in every student code box.
2. The questions in Part B may have more than one correct answer. Fill in all your all
answers in the Answer Sheet for Part B. The marks, numbers, or characters to answer
questions in Part B vary depending on questions. The ways to answer are indicated
along with the questions.
3. Use pencils and erasers. You can use a scale ruler and a calculator provided.
4. Some of the questions may be marked DELETED. DO NOT answer these questions.
5. The maximal points forof Part B is are 108 (points are indicated in each question).
6. Stop answering and put down your pencil IMMEDIATELY after the end bell rings.

IBO-2009 JAPAN
THEORETICAL TEST Part B

Cell Biology

B1. (3 point) On a dry matter basis, is the average proportion of the following elements
significantly higher in herbaceous vascular plants or in mammals? For each element
mark X in the appropriate box.

A.

Nitrogen

B.

Oxygen

C.

Calcium

D.

Potassium

E.

Sodium

F.

Phosphorus

IBO-2009 JAPAN
THEORETICAL TEST Part B

B2. (2.5 points) Match each of the following properties of water with a benefit to organisms
by putting a letter (A to E) in the appropriate box.

Property
I.

Low light absorption in the visible region

II.

High heat capacity

III.

High heat of released during fusion

IV.

High heat of vaporization

V.

Polarity of molecules

Benefit to organisms
A. Biological membranes composed of lipid molecules are thermodynamically stable.
B. Terrestrial plants and animals can cool themselves with minimum loss of water
content.
C. Temperature changes in plants and animals are minimized under fluctuating
environmental conditions.
D. Plants can efficiently utilize solar radiation for photosynthesis.
E. Plants and animals are protected against freezing at low temperatures.

IBO-2009 JAPAN
THEORETICAL TEST Part B

B3. (3 points) A coding region of a gene consists of 735 base pairs without a stop codon.
Calculate the molecular mass of the protein from this gene. Assume the average
molecular mass of the free amino acid in this protein is 122. Five disulfide bonds are
present in the protein. Calculate the molecular mass of the protein from this gene. Show
your calculations.

IBO-2009 JAPAN
THEORETICAL TEST Part B

B4. (3.5 points) Glycolysis is essential for all organisms.

(1) The figure below shows the reactions of glycolysis. The numbers in the figure indicate
enzymes which catalyze the reactions. Categorize each enzyme into the enzyme
type listed below and put each reaction number in an appropriate box. Note that
some enzyme types may be missing.

Enzyme type:
A.

Oxidoreductase

B.

Transferase

C.

Hydrolase

D.

Lyase

E.

Isomerase

F.

Ligase

IBO-2009 JAPAN
THEORETICAL TEST Part B

(2) A cell culture of muscle cells was incubated in oxygenated medium that was then
quickly made anoxic. The concentrations of three compounds which are important in
glucose metabolism were measured immediately after oxygen removal (marked as
time 0) and shown in the graph below:
Match each curve of the graph (1, 2, and 3) with the metabolite whose concentration
change it depicts:

Metabolites:
A. Glucose-6-phosphate
B. Lactate
C. Fructose-1,6-bisphosphate

IBO-2009 JAPAN
THEORETICAL TEST Part B

B5. (2 points) Different patterns of cell cycling (A to D) are shown below. Correctly match
them with the given cell types they represent.

Cell types
I.

Human epithelial cell

II.

Sea urchin embryonic cells up to the 128-cell stage

III.

Drosophila salivary gland cell

IV.

Plasmodium of slime mold

IBO-2009 JAPAN
THEORETICAL TEST Part B

B6. (3 points) A cell suspension of a microorganism was fed with [3H]-labeled uridine and
incubated. Cell components were fractionated from these cells and radioactivity in the
mRNA fraction was measured, which revealed that 2.5 picomoles of uridine were
incorporated into mRNA in 1 x 106 cells. Assuming that the base composition of mRNA
is random and that the average length of mRNA is 3,000 bases, calculate how many
molecules of mRNA were synthesized in each individual cell during incubation.
(Avogadros number: 6 x 1023)

IBO-2009 JAPAN
THEORETICAL TEST Part B

B7. (4 points) From the model plant Arabidopsis, 0.3, 0.6, 0.9, 1.2, and 1.5-kbp genomic
fragments upstream of the translation start site of gene Z were isolated and designated
Za, Zb, Zc, Zd, and Ze, respectively. These fragments were fused to the structural gene
of -glucuronidase (GUS) of Escherichia coli. Arabidopsis was transformed with the
resultant chimeric genes Za-GUS, Zb-GUS, Zc-GUS, Zd-GUS, and Ze-GUS, and
examined for GUS activity by in-situ chromogenic reaction. The following figure
schematically shows construction of the chimeric genes and the GUS activity patterns
in heart-shaped embryos of the transgenic Arabidopsis carrying these chimeric genes.

10

IBO-2009 JAPAN
THEORETICAL TEST Part B

Based on this result, identify the function of each upstream region of Z.

Upstream region
I.

1,500 to 1,201

II.

1,200 to 901

III.

900 to 601

IV.

600 to 301

Functions
A.

essential for gene expression

AB.

makes promotes gene expression stronger in a tissue-non-specific manner

BC.

promotes gene expression in cotyledons only

CD.

promotes gene expression in tissues other than cotyledons only

DE.

suppresses gene expression in cotyledons

EF.

suppresses gene expression in tissues other than cotyledons

GF.

little influence on gene expression

11

IBO-2009 JAPAN
THEORETICAL TEST Part B

Plant Anatomy and Physiology

B8. (3 points) Deficiency of a particular mineral element in the soil elicits a specific pattern
of leaf discoloration in plants (chlorosis), which is related to metabolic roles and mobility
(translocation) of the mineral element in the plant. The following describes the
deficiency symptoms (leaf discoloration), metabolic roles, and mobility of magnesium
(Mg), iron (Fe), and nitrogen (N).

Deficiency symptoms
A.

Deficiency of this mineral causes chlorosis initially in young leaves

B.

Deficiency of this mineral causes chlorosis initially in old leaves

Mineral mobility
C.

This mineral is highly mobile in plants.

D.

This mineral is largely immobile in plants.

Metabolic roles
E.

This mineral is involved as a component in the electron transfer system and is


also required for the synthesis of some of chlorophyll-protein complexes.

F.

This mineral serves as a constituent of many plant cell components including


amino acids, nucleic acids, and chlorophyll.

G.

This mineral is involved in the activation of various enzymes and serves as a


part of the ring structure of chlorophyll.

.
12

IBO-2009 JAPAN
THEORETICAL TEST Part B

Connect each mineral element to the appropriate descriptions of the above three
categories (A or B for Deficiency symptoms; C or D for Mineral mobility; E, F, or G for
Metabolic roles).

13

IBO-2009 JAPAN
THEORETICAL TEST Part B

B9. (3 points) Growing plant roots were analyzed with respect to spatial patterns of cell
division and elongation growth. The roots were marked with graphite particles (P) at various
positions along the root axis, where x was the distance from the root apex just behind the
root cap to Px.

For each Px, the following data were collected.


I.

The cumulative number of totalcumulative number of epidermal cells present

between P0 and Px
II.

The cumulative cumulative number of total mitotic epidermal cells present

between P0 and Px
III.

Velocity of displacement (movement away) of Px from P0

14

IBO-2009 JAPAN
THEORETICAL TEST Part B

When the data are plotted against x, what types of profiles do these data sets show?
For each data set, choose the most appropriate profile from the followings.

15

IBO-2009 JAPAN
THEORETICAL TEST Part B

16

IBO-2009 JAPAN
THEORETICAL TEST Part B

17

IBO-2009 JAPAN
THEORETICAL TEST Part B

B10. (4 points) Henbane (Hyoscyamus niger) is a medicinal plant. Two varieties of this
plant, one of which is annual and the other biennial, were characterized for flowering.
In the first experiment, effects of cold treatment and day- length on flowering were
examined in the annual and biennial varieties. For this purpose, cold-treated and
untreated plants were grown under the short-day condition or the long-day condition.
The following table indicates whether the plants flowered or not.

18

IBO-2009 JAPAN
THEORETICAL TEST Part B

In the second experiment, cold-treated and untreated plants of the annual and
biennial strains were grafted as shown in the following figure, and then grown under
the long-day condition. Whether or not the stock and scion flowered or not was
recorded. The results of the two types of grafts (#1 and #2) are summarized in the table
on the next page.

Assuming the involvement of florigen in flowering of this species, identify the


properties of the shoot apical meristems and leaves of the annual and biennial plants,
19

IBO-2009 JAPAN
THEORETICAL TEST Part B

based on the above results. Mark the appropriate boxes with "X" about in relation to
florigen response (1) and florigen productivity (2).

20

IBO-2009 JAPAN
THEORETICAL TEST Part B

B11. (3 points) Plants and animals accumulate starch and glycogen as a storage
polysaccharide, respectively. Starch consists of two sorts of large, water-insoluble
polymers of glucose, amylose and amylopectin. Amylose is essentially unbranched and
linear while amylopectin is highly and regularly branched, which forms branch clusters.
Glycogen is also a branched glucose polymer, but unlike amylopectin, it is relatively
small and water-soluble. In the glycogen molecule, branches are shorter, irregular, and
not clustered.

21

IBO-2009 JAPAN
THEORETICAL TEST Part B

(1) Biosynthesis of starch involves three classes of enzymes: chain elongation enzymes,
branching enzymes, and debranching enzymes. Sugary, a rice mutant, is deficient in a
particular debranching enzyme. The endosperm of this mutant is characterized by the
accumulation of glycogen-like polysaccharide instead of amylopectin. In consideration
of this information, the role of the wild type debranching enzyme in starch biosynthesis
is:

A.

to remove all branches from amylopectin to form amylose.

B.

to shorten every branch of amylopectin.

C.

to regulate the branching pattern of amylopectin.

D.

to cut 1 4 glycosidic bonds of amylopectin.

22

IBO-2009 JAPAN
THEORETICAL TEST Part B

(2) The seeds of the Sugary mutant of rice are not different from the wild-type seeds in
the size and appearance before desiccation which is associated with seed maturation.
During desiccation, however, the Sugary seeds become shrunk and wrinkled. This
phenomenon suggests that before desiccation, as compared with the wild-type seeds,
the Sugary seeds contain:

(3) Bacteria, including cyanobacteria, accumulate a glycogen-like polysaccharide for


storing glucose. Which of the following can reasonably explain the evolution of storage
polysaccharides?

The common ancestor of plants and animals could synthesize:


A.

both amylopectin and glycogen, but plants have lost the ability of glycogen
synthesis during evolution.

B.

both amylopectin and glycogen, but animals have lost the ability of
amylopectin synthesis.

C.

amylopectin but not glycogen, and animals have acquired the ability of
glycogen synthesis.

D.

glycogen but not amylopectin, and plants have acquired the ability of
amylopectin synthesis.

23

IBO-2009 JAPAN
THEORETICAL TEST Part B

B12. (3 points) Soybean roots form nodules when infected by Rhizobium. HN is a


recessive mutant of soybean that exhibits a hyper-rnodulating phenotype. As
shown in Figure 1, the roots of the HN mutant form more nodules than the wildtype (WT) roots, and the shoot growth of the HN mutant is retarded compared to
WT. Figure 2 schematically shows the nodulation phenotypes observed in grafting
experiments with WT and the HN mutant. In the absence of Rhizobium, the HN
mutant is not phenotypically different from WT in any aspects.

24

IBO-2009 JAPAN
THEORETICAL TEST Part B

From the above informationprovided, what can be reasonably inferred? For each of the
following statements, mark X in the appropriate box choosing the option in the
bracket.

I.

In the HN mutant, the determines the hyper-nodulation phenotype.

II.

The shoot of WT the number of nodules.

III.

In the HN mutant, hypernodulation is of retarded growth of the shoot.

25

IBO-2009 JAPAN
THEORETICAL TEST Part B

Animal Anatomy and Physiologyy

B13. (3 points) Three patients I, II and III show symptoms of low thyroxine levels. Defects
are found in the hypothalamus for patient I, in the anterior pituitary for patient II, and in
the thyroid for patient III. After thyroid-stimulating-hormone-releasing hormone (TRH) is
given to these patients, the concentration of thyroid-stimulating hormone (TSH) before
and after (30 min) TRH administration is measured in each patient.

26

IBO-2009 JAPAN
THEORETICAL TEST Part B

Fill Choose the letter of the appropriate data (AC) for each patient (IIII).

27

IBO-2009 JAPAN
THEORETICAL TEST Part B

B14. (2.5 points) The graph below shows the blood glucose level after three hormones I, II
and III are administered separately or together.

(1) How do you classify these hormones?

A.

Hypoglycemic

B.

Hyperglycemic

(2) Choose the type of interaction between these hormones.

A.

Additive

B.

Antagonistic

C.

Synergistic

D.

None

(3) Pick the three possible hormones that are consistent with the results shown in the
graph.

A.

Insulin

B.

ADH (Vasopressin)

C.

Adrenalin (Epinephrine)

D.

Renin

E.

Glucagon

F.

Angiotensinogen

G.

Cortisol
28

IBO-2009 JAPAN
THEORETICAL TEST Part B

H.

Calcitonin

I.

Atrial natriuretic peptide

29

IBO-2009 JAPAN
THEORETICAL TEST Part B

B15. (4 points) The oocytes of a starfish grow within the provided follicle cells in the gonad.
Eventually they cease meiosis at prophase I, and wait as a state of immature eggs. The
immature eggs resume meiosis when stimulated and lose their nuclear envelop as
shown below.
In order to understand the mechanism of this resumption, the following experiments
were conducted.
Experiment 1: When extract from the nerve tissue of adult starfish was added to
immature eggs surrounded by follicles, meiosis resumed.

Experiment 2: When extract from the nerve tissue of adult starfish was added to
immature eggs from which follicles were removed, meiosis did NOT resume.

Experiment 3: When extract from the nerve tissue of adult starfish was added to follicles
after they had been separated from immature eggs, and subsequently the medium was
added to immature eggs without follicles, meiosis resumed.

Experiment 4: When extract from the nerve tissue of an adult starfish was added to
follicles after separationed from immature eggs, and the medium was injected into
immature eggs without follicles, meiosis did NOT resume.

Based on these results, four hypotheses were developed.

30

IBO-2009 JAPAN
THEORETICAL TEST Part B

Hypothesis 1: The extract from the nerve tissue contains a substance which directly
acts on immature eggs causing them to resume meiosis.

Hypothesis 2: The extract from the nerve tissue contains a substance which acts on
immature eggs to resume meiosis, but the follicle blocks the substance from reaching
the immature eggs.

Hypothesis 3: The extract from the nerve tissue contains a precursor of a substance
that causes meiosis to resume, which is processed by the follicle into an active
compound that causes immature eggs to resume meiosis.

Hypothesis 4: The extract from the nerve tissue induces follicles to secrete a substance
which then acts on the cell surface of an immature egg to cause a resumption of
meiosis.

Indicate whether each hypothesis is rejected or not.

31

IBO-2009 JAPAN
THEORETICAL TEST Part B

B16. (2 points) After the nucleus is removed from a fertilized frog egg, it is re-transferred
back into the enucleated egg. In another experiment, the nucleus from a gut epithelial
cell is transferred into an enucleated egg. In both cases, the eggs grow well and
develop normally into tadpoles.

(1) Choose the correct statement from A to E below.

During differentiation from fertilized eggs to tadpole gut epithelial cells:


A.

gene expression patterns do not change.

B.

some genes are not expressed, but the genes themselves are not lost during
development.

C.

all the genes are expressed.

D.

the amount of proteins does not change.

E.

the amount of RNAs does not change.

(2) In the experiment above, frog gut epithelial cells were used. If this experiment were
performed in mammals, theoretically almost all cell types can be used as a nucleus
donor, but a few cell types cannot. Which of the following cell types is NOT
appropriate as a donor cell?

A.

B lymphocyte

B.

Liver cell

C.

Mammary gland cell

D.

ES (embryonic stem) cell

E.

Cone cell
32

IBO-2009 JAPAN
THEORETICAL TEST Part B

B17. (2 points) The figure below represents a cross section of a vertebrate neurula stage
embryo.

(1) The following are statements about the tissues and organs derived from (a), (b), (c)
and (d) of the figure. Identify whether each statement is True or False and mark X in
the appropriate box.

A.

Tissues derived from (a) are always associated with those from (b).

B.

The developmental fate of (c) sometimes changes.

C.

(d) differentiates into the backbone (vertebra).

D.

Most of the circulatory system arises from (b).

33

IBO-2009 JAPAN
THEORETICAL TEST Part B

(2) Neural tube arises from (e). The following are statements about the formation and
later development of the neural tube. Identify whether each statement is True or
False and mark X in the appropriate box.

A.

Cells in the wall of neural tube later differentiate into glial cells as well as
nerve cells (neurons).

B.

Lumen in the neural tube is later completely occluded.

C.

Almost all nervous tissue derived from neural tube is central nervous system.

D.

The retinal pigment epithelium in the eye derives from optic vesicle formed
from the neural tube.

34

IBO-2009 JAPAN
THEORETICAL TEST Part B

B18. (3 points) For intracellular infectious bacteria and viruses to successfully invade a cell,
they must bind to receptors on the cell surface. HIV, specifically infects helper T cells,
which express the CD4 molecule, but not CD8 on their cell surface, making it possible
to distinguish helper T cells from other lymphocytes. Thus, CD4 is hypothesized to be a
receptor for HIV.

(1) Which two of the following experiments would confirm this hypothesis?

Experiments that examine whether:


A.

an antibody against CD4 added to a co-culture system of CD4-positive T cells


and HIV can inhibit HIV infection of T cells

B.

an antibody against CD8 added to a co-culture system of CD8-positive T cells


and HIV can inhibit HIV infection of T cells

C.

an antibody against HIV added to a co-culture system of CD4-positive T cells


and HIV can inhibit HIV infection of T cells

D.

forced expression of the CD4 gene in HIV-resistant CD4-negative T cells


causes a recovery of susceptibility to HIV infection

E.

forced expression of the CD8 gene in HIV-resistant CD8-negative T cells


causes a recovery of susceptibility to HIV infection

35

IBO-2009 JAPAN
THEORETICAL TEST Part B

(2) It is known that HIV cannot infect mice, although the mouse has CD4-positive helper T
cells, because mouse CD4 cannot bind to HIV. To study further the mechanism of HIV
infection in human cells, the following experiments were carried out, and the results
are as follows:

1. When the human CD4 gene is expressed in mouse T cells, HIV can bind to the
cells but cannot infect them.
2. When human chemokine receptor (CXCR4) is expressed in addition to human
CD4 in mouse cells, HIV is able to infect the cells.
3. When human CD4 and CXCR4 genes are expressed in mouse cells and the cells
are cultivated in the presence of SDF-1a, a ligand of CXCR4, infection by HIV is
perturbed (altered).

Which of the following sentences states the correct conclusion based on the above
experiments?

A.

If CXCR4 is expressed in mouse cells, CD4 is not required for the infection of
HIV.

B.

Human CD4 is required for the binding with HIV, and the binding is enhanced
by the SDF-1a ligand.

C.

Even if human CD4 is expressed in mouse T cells, CXCR4 is required for


binding of HIV to the T cells.

D.

Human CD4 is required for the binding with HIV, but infection of HIV into cells
requires help of CXCR4.

36

IBO-2009 JAPAN
THEORETICAL TEST Part B

B19. (3 points) The majority of humans have erythrocytes that express the Rh (Rhesus)
antigen on their cell surface, but some are negative for the Rh antigen.
An Rh-negative woman marries to a heterozygous Rh-positive man and has three
children.

(1) What is the probability that all three of their children become Rh-positive?

A.

B.

1/2

C.

1/4

D.

1/8

E.

(2) In which combination below could the second child suffer from hemolytic disease?

First child

Second child

A.

Rh-positive

Rh-negative

B.

Rh-negative

Rh-positive

C.

Rh-negative

Rh-negative

D.

Rh-positive

Rh-positive

37

IBO-2009 JAPAN
THEORETICAL TEST Part B

(3) Which molecules or cells are mainly involved in causing hemolytic disease in the fetus
and newborn infant in case of Rh blood group antigen-incompatibility? Choose TWO
correct options from A to F.

A.

T cells

B.

IgM antibody

C.

Complement

D.

Interferon gamma

E.

IgG antibody

F.

Perforin

38

IBO-2009 JAPAN
THEORETICAL TEST Part B

Ethology

B20. (3 points)
(1) Foraging honeybees usually perform a waggle dance (Figure 1) when they find an
attractive food source 100 m or more away from their hive. The duration of the waggle
dance indicates the distance to the food source.
The duration of the waggle dance was studied in two honeybee species, Apis cerana
cerana (Acc) and Apis mellifera ligustica (Aml), when food was placed at varying
distances from the hives and the data shown in the graph below.

What were the distances (m) indicated when the average duration of the waggle
dances of Acc and Aml both lasted 800 msec? Answer the distance for each species
from the following numbers.

130

160

39

190

220

250

280

310

340

370

400

IBO-2009 JAPAN
THEORETICAL TEST Part B

(2) Mixed colonies of Acc and Aml were successfully established by introducing Aml
pupae into Acc colony and vice versa. The young individuals of both species were
accepted by the colony members of the other species. When the same experiment
(Figure 2) was performed on the mixed colonies, the introduced Acc and Aml workers
each showed exactly the same patterns that these species had shown earlier.
In the final experiment, food was placed at 400, 500 and 600 m, all in the same
direction, and the introduced Aml bees trained to forage at the food source 500 m
away. When these bees recruited Acc bees from the hive, the latter were found to
forage at the food site exactly 500 m away. This was also seen when the reverse
experiment was done with Acc bees recruiting Aml bees.
From these experiments, what can we conclude about the transfer of the encoded
and decoded information between the actor and receiver bees, respectively?

40

IBO-2009 JAPAN
THEORETICAL TEST Part B

41

IBO-2009 JAPAN
THEORETICAL TEST Part B

B21. (2 points) Red harvester ants (Pogonomyrmex barbatus) are social insects and live in
underground colonies, in which various functions are carried out by different groups of
ants. Below is a picture of one such ant colony. The open circle in the center is the nest
entrance. The four types of lines (i to iv) indicate paths followed by different groups of
these ants. Match the appropriate groups (A to D) with these lines:

Groups:
A.

Foragers

B.

Patrollers

C.

Nest maintenance ants

D.

Midden workers or refuse pile sorters (those who remove (pile) fecal matter
from outside the nest)

42

IBO-2009 JAPAN
THEORETICAL TEST Part B

B22. (2 points) In birds, there are many ways of singing. This is caused by the fact that
brain regulates the action of the syrinx (vocal organ of birds). In a certain species of
birds, two kinds of vocalization can be recognized: longer songs produced by males in
the breeding season, and other simpler calls heard outside the breeding season.

(1) If the young chicks of such birds are reared in an environment without sound, adult
birds cannot produce the exact longer songs. Which of the following is the most
appropriate as explanation of the above statement?

A.

In an environment without sound, differentiation between males and females


cannot be attained.

B.

The song is a mode of behavior which is determined by learning after


hatching.

C.

In an environment without sound, imprinting of the gene responsible for the


song cannot occur.

D.

In an environment without sound, the auditory sense cannot develop.

43

IBO-2009 JAPAN
THEORETICAL TEST Part B

(2) Although chicken and quail are closely related, their calls are different. An experiment
was carried out in which the presumptive brain region of 5-day-old white chicken
embryo was substituted by that of a brown quail embryo of the same age. Then the
host chicken embryo was incubated. The hatched chicken had some brown parts in its
brain, which indicates that these parts were derived from a quail. The calls of this
chicken were more similar to that of a quail rather than that of a chicken. Which of the
following is the most appropriate conclusion deduced from the experiment?

I.

Calls are species-specific and are determined genetically.

II.

Calls are determined after hatching.

III.

Calls are determined by the structure of the syrinx.

A.

Only I

B.

Only II

C.

Only III

D.

I and II

E.

I and III

F.

II and III

44

IBO-2009 JAPAN
THEORETICAL TEST Part B

Genetics and Evolution

B23. (4 points) In an experiment on the members of a family with the pedigree shown
below, blood plasma and blood cells from different individuals were mixed in pairs to
test the presence (p) or absence (a) of coagulation. In this pedigree AB- means that the
phenotypes of individual 1 (mother) are AB type and Rh negative (Rh-), and B+ means
that the phenotypes of individual 2 (father) are B type and Rh positive (Rh+).

The results of this experiment are shown below. A blank box in this table indicates a
combination that was not tested in this experiment.

(1) What are the phenotypes of individual 6?

A.

A type and Rh+

B.

A type and Rh-

C.

B type and Rh+

D.

B type and Rh-

E.

AB type and Rh+

F.

AB type and Rh-

(2) Which member of this family is probably homozygous with respect to both the ABO
blood group and the Rh loci?

45

IBO-2009 JAPAN
THEORETICAL TEST Part B

A.

Individual 2

B.

Individual 3

C.

Individual 4

D.

Individual 5

E.

Individual 6

46

IBO-2009 JAPAN
THEORETICAL TEST Part B

B24. (4 points) In maize a single locus determines the color of the seed; allele A results in
colored seeds, and allele a in colorless seeds. Another locus determines the shape of
the seeds; allele B results in a smooth shape of the seeds, and b in wrinkled seeds.

In a crossbreeding between the plant that grew from a colored and smooth seed and
the plant that grew from a colorless and wrinkled seed, the offspring were documented
as:
376

had colored and smooth seeds

13

had colored and wrinkled seeds

13

had colorless and smooth seeds

373

had colorless and wrinkled seeds

(1) What are the genotypes of the parents?

A.

AABb x aaBb

B.

AaBb x aabb

C.

AAbb x aaBB

D.

AaBb x AaBb

E.

aabb x AABB

47

IBO-2009 JAPAN
THEORETICAL TEST Part B

(2) What is the frequency of recombinants?

A.

0.335%

B.

1.68%

C.

3.35%

D.

6.91%

E.

48.52%

(3) Three loci C, D and E are located on the same chromosome in this order. Using
similar experiments to the above, we found that the frequency of recombinants
between C and D is 10% and that between D and E it is 20%. Assuming that crossing
over occurs randomly on the chromosome, what is the expected frequency of
recombinants between C and E?

48

IBO-2009 JAPAN
THEORETICAL TEST Part B

B25. (3 points) The evolutionary distance is defined as the number of nucleotide


substitutions per nucleotide site between two DNA sequences, and the evolutionary
rate is defined as the number of nucleotide substitutions per nucleotide site per year.
We sampled two DNA sequences from two species (one sequence from each species),
and found that the evolutionary distance between the two sequences is 0.05. We
assume that the evolutionary rate is 10-8.

(1) How many years ago did the two sequences diverge?

(2) What is the relationship between the divergence time between the two sequences
(T1) and the divergence time between the two species (T2) in general?

A.

T1 < T2

B.

T1 = T2

C.

T1 > T2

49

IBO-2009 JAPAN
THEORETICAL TEST Part B

B26. (3 points) Preproinsulin is the primary product of the insulin gene, and consists of 4
major parts: signal, B-chain, C, and A-chain peptides. After several modifications
including removal of the signal and C peptides, insulin is obtained.

(1) Which of the following peptides is responsible for the transport of polypeptide into the
endoplasmic reticulum?

A.

A-chain peptide

B.

B-chain peptide

C.

C peptide

D.

signal peptide

(2) Comparisons of amino acid sequences among mammals show that the sequence
similarity between species varies substantially among the peptides. Which of the
following is the most likely explanation?

A.

directional selection

B.

frequency-dependent selection

C.

overdominant selection (heterozygote advantage)

D.

purifying selection (selection against deleterious mutations)

50

IBO-2009 JAPAN
THEORETICAL TEST Part B

(3) Which peptide is likely to differ the most among mammals?

A.

A-chain peptide

B.

B-chain peptide

C.

C peptide

D.

signal peptide

51

IBO-2009 JAPAN
THEORETICAL TEST Part B

B27. (4 points) In order to quantify genetic diversity of an endangered plant species, genetic
variation in subpopulations (IIV) was examined at the protein level. Subpopulation I is
the largest in this species, and the number of individuals in all other subpopulations II,
III and IV are each 1/7 of that in subpopulation I. In each subpopulation 5 individuals
were sampled. The diagram below shows the results of proteins separated by gel
electrophoresis. The band pattern in each lane, which consists of alleles F and/or S,
represents the genotype of each individual at a certain locus.

Subpopulation I

Subpopulation II

Subpopulation III

Subpopulation IV

(1) Estimate the frequency of F in this species.

(2) Which subpopulation is thought to be the most isolated group?

52

IBO-2009 JAPAN
THEORETICAL TEST Part B

(3) After several generations, we found that the frequency of F changed substantially in
subpopulations II, III and IV, compared with that of subpopulation I. What is the most
likely explanation?

A.

Genetic drift

B.

Migration

C.

Mutation

D.

Natural selection

53

IBO-2009 JAPAN
THEORETICAL TEST Part B

B28. (3 points) Islands are considered as experimental sites for biological evolution and
community assembly. The diagram below shows two phylogenetic trees, each
consisting of 9 species (aAi and jJr) and community assemblies on 6 islands.
Phenotypic traits of the species are represented by size and color.

Which of the following explanations are responsible for the mechanisms of community
assembly on these islands? Choose THREE correct options from A to H.

54

IBO-2009 JAPAN
THEORETICAL TEST Part B

55

IBO-2009 JAPAN
THEORETICAL TEST Part B

56

IBO-2009 JAPAN
THEORETICAL TEST Part B

Ecology

B29. (3 points) The following diagram shows the cycle of nitrogen compounds in an
ecosystem.

(1) BacteriaIn which of the processes do NOT participate in which of the above two
processesbacteria participate? Choose TWO from A to G.

(2) Which of the processes may include a symbiotic relationship between a species of
plant and a species of bacterium?

(3) Which of the processes do farmers want to inhibit in agricultural land?

57

IBO-2009 JAPAN
THEORETICAL TEST Part B

B30. (3 points) The relationship between population density (Nt) and population growth rate
(R = Nt+1 / Nt ) in a certain animal species is shown below.

Choose from the following graphs the appropriate population growth patterns that
would be obtained if the population is at the densities (I, II, III) shown in the graph
above. Note that the y-axis in A to D is relative density that cannot be compared to the
absolute density in the figure.

58

IBO-2009 JAPAN
THEORETICAL TEST Part B

B31. (2.5 points) Competitive exclusion among species is regulated by various ecological
factors. Identify whether the following statements are True or False about this process,
and mark X in the appropriate boxes.

Competitive exclusion:
A.

is intense among species with similar ecological niches.

B.

is occasionally interrupted by environmental disturbances.

C.

is promoted by species succession.

D.

is alleviated reduced by habitat segregation among species.

E.

occurs because of keystone species.

59

IBO-2009 JAPAN
THEORETICAL TEST Part B

B32. (3 points) The diagram below shows the results of an experiment on the vine Ipomoea
tricolor, in which root competition and shoot competition were separated. The average
dry mass is indicated by open bars, and the coefficient of variation (ratio of standard
deviation / mean) of mass among plants is indicated by hatched bars. Based on the
data presented, iIdentify whether the following statements are True or False about the
competition mode of this plant species, and mark X in the appropriate boxes

60

IBO-2009 JAPAN
THEORETICAL TEST Part B

A. Competition for light has more influence on the average mass than competition for
soil nutrients.
B. Under conditions of competition, individual plants have stronger potential to
monopolize soil nutrients over others than they can monopolize light.The
differences in competitive strength among these plants are larger when competing
for soil nutrients than for light.
C. When grown individually, soil nutrients constitute a limiting factor for growth, but
light does not.

61

IBO-2009 JAPAN
THEORETICAL TEST Part B

Biosystematics

B33. (3 points) At which branches A to E in this phylogenetic tree of green plants were the
traits I to VI listed below acquired?

I.

Pollen

II.

Tracheid

III.

Cuticle

IV.

Seed

V.

Carpel

VI.

Multicellular embryo

62

IBO-2009 JAPAN
THEORETICAL TEST Part B

B34. (5 points) The universal phylogenetic tree based on molecular sequencing analysis
shows three major groups of living organisms as shown below. Woese proposed the
concept of three domains in living organisms in the 1990s based on such a tree.

(1) What was the molecule used for the construction of the universal phylogenetic tree?
What was the benefit of this molecule for the universal tree? Choose the combination
of the molecule and benefit.
(2) The two broken arrows indicate hypothesized endosymbiotic events whereby members
of Domain I became endosymbionts of Domain II. What are the two organisms that were
involved in these events, what did they become in the cells of Domain II and what is their
current biological function in the Domain II organisms?

(3) Which of the following corresponds to domains I, II or III?

A. Archaea
B. Bacteria
C. Eukarya

63

IBO-2009 JAPAN
THEORETICAL TEST Part B

B35. (4 points) Joseph Camin, a taxonomist, invented artificial non-existing creatures, the
Caminalcules, for his students. Below are depicted four different Caminalcules.

Take a close look at the following four Caminalcules:

(1) For these four Caminalcules, choose an appropriate cladogram by focusing upon the
following characteristics. The most likely tree should be the one where the largest
number of characters can be mapped in the internal branch.

1.

Antenna

2.

Belly spots

3.

Elbow

4.

Fingers

5.

Neck

6.

Spots Lines at the side

7.

Posterior legs

64

IBO-2009 JAPAN
THEORETICAL TEST Part B

(2) Choose characteristics from the list in question (1) which presumably evolved
convergently (independently lost or acquired) in two species of the four.

(3) Assuming that Caminalcule a is a sister taxon of the other species, choose an
appropriate rooted tree from the following.

*****

65

END OF PART B

*****















All IBO examination questions are published under the following Creative Commons license:



CC BY-NC-SA (Attribution-NonCommercial-ShareAlike) https://creativecommons.org/licenses/by-nc-sa/4.0/
The exam papers can be used freely for educational purposes as long as IBO is credited and
new creations are licensed under identical terms. No commercial use is allowed.

Report of the 20th International Biology Olympiad, Tsukuba, Japan

Answer Key to the Theoretical Tests


1. Part A
No.

A1
A2

E
X

A3

No.

A21

A23

A4

A5

A6

A27

A8

A28

A9

A29

No.

A41

A43

X
X
X

A44

A45

A46

A47
X

A48

X
X

A49

A30

A11

A31

A51

A12

A32

A52

A13

A33

A14

A34

A15

A35

A16

A36

A18
A19
A20

X
X
X
X

A50

A53

A54
X
X

A37

A38

A39

A40

- 114 -

A10

A17

A42

A26

A24

A7

A25
X

A22
X

X
X

Report of the 20th International Biology Olympiad, Tsukuba, Japan

2. Part B
B1. (0.5 points x 6 = 3 points)
A
B
C
Plants
X
Mammals
X
X

D
X

B2. (0.5 points x 5 = 2.5 points)


I
II
III
IV
V
D
C
E
B
A
B3. (0.5 points x 6 = 3 points)
735

735
122
1 18 {5 (2 1)}

3
3

= 245 122 244 18 5 2


= 29890 4392 10
= 25488
When the N-terminal Met residue is assumed to have been removed,
735

735

1 122
11 18 {5 (2 1)}

= 244 122 243 18 5 2


= 29768 4374 10
= 25384
Partial point:
(0.5) Division by 3.
(0.5) Subtraction of water mass of peptide bonds.
(0.5) Number of peptide bonds: number of amino acids minus 1.
(0.5) Correct molecular mass of one water molecule.
(0.5) Subtraction of ten hydrogen molecules of five disulfide bonds.
(0.5) Correct calculations.
B4. (3.5 points)
(1) (0.3 points x 10 = 3 points)
A
6
B
1, 3, 7, 10
C
D
4, 9
E
2, 5, 8
F
(2) (0.5 point : if all are correct)

- 115 -

Report of the 20th International Biology Olympiad, Tsukuba, Japan

1
B

2
C

3
A

B5. (0.5 points x 4 = 2 points)


I
II
III
D
A
C

IV
B

B6. (3 points)
2000
(or 2007 by using more precise Avogadros number.)
B7. (1 points x 4 = 4 points)
I
II
III
A
F
E

IV
A

B8. (3 points)
Deficiency symptoms
(0.3x3)
Mg
B
Fe
A
N
B

Mineral mobility
(0.3x3)
C
D
C

Metabolic roles
(0.4x3)
G
E
F

B9. (1 points x 3 = 3 points)


I
II
III
C
D
F
B10. (4 points)
(1) (0.5 points x 4 = 2 points)
can
respond
Florigen.
cold-treated
annual plants
untreated
annual plants
The
shoot
cold-treated
apical
biennial
meristems of
plants
untreated
biennial
plants

X
X
X
X

- 116 -

to cannot respond
Florigen.

to

Report of the 20th International Biology Olympiad, Tsukuba, Japan

(2) (0.5 points x 4 = 2 points)


produce
Florigen do not produce Florigen
under the long-day in either photoperiodic
condition.
condition.

The leaves of

B11. (3 points)
(1) (1 point)
A
B
C
X
(2) (1 point)
A
B

(3) (1 point)
A
B

cold-treated
annual plants
untreated
annual plants
cold-treated
biennial
plants
untreated
biennial
plants

X
X
X
X

C
X

D
X

B12. (1 points x 3 = 3 points)


A
B
C
I
X
II
X
III
X
B13. (1 points x 3 = 3 points)
I
II
III
A
C
B
B14. (2.5 points)
(1) (0.5 point)
A
B
X
(2) (0.5 point)
A
B

D
- 117 -

Report of the 20th International Biology Olympiad, Tsukuba, Japan

X
(3) (0.5 points x 3 = 1.5 points)
C
E
G
B15. (1 points x 4 = 4 points)
Hypothesis
1
Rejected
X
Not rejected
B16. (2 points)
(1) (1 point)
A
B
X
(2) (1 point)
A
B
X

Hypothesis
2
X

B17. (2 points)
(1) (0.2 point x 4 = 0.8 points)
A
B
C
TRUE
X
X
FALSE
X

D
X

(2) (0.3 points x 4 = 1.2 points)


A
B
C
TRUE
X
X
FALSE
X

D
X

B18. (3 points)
(1) (1 points x 2 = 2 points)
A
B
C
D
E
X
X
(Choosing more than 2 gives no point.)
(2) (1 point)
A
B

D
X

B19. (3 points)
(1) (0.5 point)
A
B

E
- 118 -

Hypothesis
3

Hypothesis
4

Report of the 20th International Biology Olympiad, Tsukuba, Japan

X
(2) (0.5 point)
A
B

D
X

(2) (1 points x 2 = 2 points)


A
B
C
D
E
F
X
X
(Choosing more than 2 gives no point.)
B20. (3 points)
(1) (1 points x 2 = 2 points)
Acc
m
190
Aml
m
340
(2) (1 point)
A
B
X

B21. (0.5 points x 4 = 2 points)


i
ii
iii
Iv
C
D
B
A
B22. (2 points)
(1) (1 point)
A
B
C
X
(2) (1 point)
A
B
X

B23. (4 points)
(1) (2 points)
A
B
C
X
(2) (2 points)
A
B

C
X

B24. (4 points)
(1) (1 point)
- 119 -

Report of the 20th International Biology Olympiad, Tsukuba, Japan

B
X

(2) (1 point)
A
B

(3) (2 points)
26

C
X

B25. (3 points)
(1) (2 points)
2.5
X
106
(2,500,000)
years
6
5 X 10 (5,000,000) years: 1 point
(2) (1 point)
A
B

C
X

B26. (3 points)
(1) (1 point)
A
B
C

(2) (1 point)
A
B

(3) (1 point)
A
B

B27. (4 points)
(1) (2 points)
34
or 0.34

D
X

D
X

C
X

(2) (1 point)
I
II

III

IV
X

(3) (1 point)
A
B

D
- 120 -

Report of the 20th International Biology Olympiad, Tsukuba, Japan

X
B28. (1 points x 3 = 3 points)
A
B
C
D
E
X
X

F
X

B29. (3 points)
(1) (0.5 point x 2 = 1 points)
A
B
C
D
E
F
G
X
X
(Choosing more than 2 gives no point.)
(2) (1 point)
A
B
X

(3) (1 point)
A
B
X

B30. (1 points x 3 = 3 points)


I
II
III
D
C
B
B31. (0.5 points x 5 = 2.5 points)
A
B
C
D
TRUE
X
X
X
FALSE
X

E
X

B32. (1 points x 3 = 3 points)


A
B
C
TRUE
X
FALSE
X
X
B33. (0.5 points x 6 = 3 points)
I
II
III
IV
V
D
C
B
D
E
B34. (5 points)
(1) (1 point)
A
B
C
X

VI
B

- 121 -

Report of the 20th International Biology Olympiad, Tsukuba, Japan

(2) (0.5 points x 6 = 3 points)


Domain I Domain II
Older
3
1
Newer
1
4

Function
4
1

(3) (1 point: if all are correct)


I
II
III
B
C
A
B35. (4 points)
(1) (2 points)
A
B
C
X
(2) (1 point)
2, 6
Since the number of correct choice is not stated in the question, the points are
calculated by (number of the correct answer) x 0.5 (number of the wrong answer) x
0.3.
(3) (1 point)
A
B

E
F
G
H
I
X
Since the answer of (3) is consequence of the question (1), the combination of the
wrong answer (1) A and the answer of (3) B gives 0.5 point for (3), and that of (1) C and
(3) H, gives 0.5 point for (3).

- 122 -

INTERNATIONAL BIOLOGY OLYMPIAD


PRACTICAL PROBLEMS

2009, Tsukuba, Japan















All IBO examination questions are published under the following Creative Commons license:



CC BY-NC-SA (Attribution-NonCommercial-ShareAlike) https://creativecommons.org/licenses/by-nc-sa/4.0/
The exam papers can be used freely for educational purposes as long as IBO is credited and
new creations are licensed under identical terms. No commercial use is allowed.

IBO 2009
JAPAN
PRACTICAL TEST 1
ANIMAL AND PLANT ANATOMY
_______________________________________________________________________

Student Code: ___________

20th INTERNATIONAL BIOLOGY OLYMPIAD


12th 19th July, 2009
Tsukuba, JAPAN

PRACTICAL TEST 1
ANIMAL AND PLANT ANATOMY
Total Points: 100
Duration: 90 minutes

IBO 2009
JAPAN
PRACTICAL TEST 1
ANIMAL AND PLANT ANATOMY
_______________________________________________________________________

Dear Participants,

In this test, you have been given the following 2 tasks:


Task 1: Animal anatomy ( 50 points)
Task 2: Plant anatomy (50 points)

You must write down your results and answers in the ANSWER SHEET. Answers
written in the Question Paper will not be evaluated.

Please make sure that you have received all the materials and equipment listed for each
task. If any of these items are missing, please raise your hand.

At the end of the test, put the Answer Sheet and Question Paper in the envelope. The
supervisor will collect this envelope.

Good Luck!!

IBO 2009
JAPAN
PRACTICAL TEST 1
ANIMAL AND PLANT ANATOMY
_______________________________________________________________________

Task 1 (50 points)


Animal Anatomy

Materials and Equipment

Quantity

1. Vessel containing two caterpillars anesthetized

2. Vessel containing one caterpillar non-anesthetized

3. Dissecting plate

4. Forceps

5. Scissors

6. Disposable pipette

7. Dissecting needle equipped with holder

8. Dissecting pins

20

9. Compound binocular microscope (equipped with illuminator)

10. Set of color pencils: one O (orange), one B (blue), and one G (green)

11. Photo of a dissected caterpillar (included in your envelope)

12. A Petri dish for discarding dissected larva

Introduction
Even in insects which undergo complete metamorphosis, the body structure of the adult and
larva are basically common. After closely observing a non-anesthetized caterpillar and
dissecting and closely observing anesthetized caterpillars or moth (Bombyx mori Linn)
larvae (silk worm), answer the following questions. When you dissect the caterpillars, do it in

IBO 2009
JAPAN
PRACTICAL TEST 1
ANIMAL AND PLANT ANATOMY
_______________________________________________________________________

the dissecting plate filled with water, using suitable equipments such as forceps, scissors,
dissecting needle with holder, dissecting pins.

Q.1.1. (1 point2 = 2 points) The insect body is composed of three regions, the head,
thorax and abdomen. Show the boundary between the head and thorax by drawing an orange
line with orange color pencil O and the boundary between the thorax and abdomen by
drawing a blue line with blue color pencil B on the photo of the caterpillar in the Answer
Sheet.

Q.1.2. (3 points) On each side of the caterpillars head, you will find an eye patch. How
many small eyes are in the eye patch of one side of the caterpillar head in front of you?
Answer using numerals.

Q.1.3. (3 points) Insects breathe by means of a tracheal system, with external openings called
spiracles. How many pairs of spiracles do the caterpillars in front of you have? Answer using
numerals.

Q.1.4. (6 points +[2+2 ]3 points = 18 points) The photo in your envelope shows a dorsal
view of a dissected caterpillar. Dissect an anesthetized caterpillar by yourself exactly as
shown in photo. (You may use the second caterpillar if required) When you have finished
the dissection, call your assistant by raising your hand. Your assistant will take a photograph
of your specimen for evaluation (6 points). You should check the photograph of your
dissected specimen after it has been taken.

IBO 2009
JAPAN
PRACTICAL TEST 1
ANIMAL AND PLANT ANATOMY
_______________________________________________________________________

Closely observe the internal structures of the caterpillar, focusing on where the
tubular structures A, B and C arise. Answer the name and function of each of the tubular
structures A, B and C by choosing the appropriate answer for the name from numerals 1-10
and function from the alphabet a-j.

Names

1: salivary gland; 2: oviduct; 3: malpighian tubule; 4: appendix;


5: trachea; 6: prothoracic gland; 7: silk gland; 8: corpora allata;
9: fat body; 10: seminal duct

Functions

a: secretion of juvenile hormone; b: support of digestion;


c: respiration; d: secretion of silk; e: secretion of prothoracic hormone;
f: restoration of fat; g: excretion; h: transport of egg;
i: transport of sperm; j: secretion of saliva

Q.1.5. (2 points3 = 6 points) The insect body contains different kinds of internal organ
systems. Closely observing non-anesthetized and dissected caterpillars, show the positions of
the central nervous system, digestive system (gut) and circulatory system (heart), by drawing
them into the image of the caterpillar prepared in the Answer Sheet using the colors as
indicated below.
Central nervous system - orange color pencil O
Digestive system - blue one B
Circulatory system - green one G.
Notice: If you can show the positions of the systems in the image of the caterpillar, there is
no need to copy their exact shapes: however, in drawing the digestive systems, you should
clearly show both ends.

IBO 2009
JAPAN
PRACTICAL TEST 1
ANIMAL AND PLANT ANATOMY
_______________________________________________________________________

Q.1.6. (4 points) The central nervous system of insects is composed of the aggregations of
cell bodies or the ganglia and the bundles of nerve fibers or the nerve cords connecting
ganglia. How many ganglia does the dissected caterpillar have? Answer using numerals.

Q.1.7. (4 points3 = 12 points) Show the positions of the anteriormost, anterior-second and
posteriormost ganglia by drawing arrows and labeling with A for anteriormost, 2 for
anterior second and P for posteriormost with black pencil in the image of the caterpillar
used in Q.1.5.

Q.1.8. (2 points) How many nerve cords are there between each pair of ganglia? Answer
using numerals, choosing the correct number from 1 to 4.

IBO 2009
JAPAN
PRACTICAL TEST 1
ANIMAL AND PLANT ANATOMY
_______________________________________________________________________

Task 2 (50 points)


Plant Anatomy

In this task, fruit and flower morphology are examined and the developmental
process is studied.

Part A Seed morphology and reserve substances

Materials and equipment

Quantity

1.Petri dishes containing seeds labeled I to IV


2. Compound binocular microscope (used in Task 1)

4
1

3. Forceps (used in Task 1)

4. Knife

5. Scalpel

6. Bottles of staining and rinsing solutions (IKI, IKI-R, CBB, CBB-R, OR, OR-R) 6
7.Small Petri dishes for staining

12

Introduction
Morphology and reserve substances vary across plant species. Reserve substances can be
distinguished by staining.

IBO 2009
JAPAN
PRACTICAL TEST 1
ANIMAL AND PLANT ANATOMY
_______________________________________________________________________

Q.2.A.1. (27 points)


There are 4 kinds of seed (I to IV) in Petri dishes. The seeds labeled IV are Vigna angularis,
a kind of legume which are given as an example. The seeds have been soaked for 24 hours.
From some seeds, the seed coat was removed. Dissect the seeds using scalpel or knife, and
stain each of them and their sections separately using all three staining solutions. Then,
observe the stained seed samples including the sections of tissues under the stereomicroscope,
and examine the degree of staining. Look at the samples carefully and fill the degree of
staining in the Box of Q.2.A.1. in the answer sheet using the following symbols: + for
weak staining, + for medium staining, ++ for strong staining. Use - for samples not
stained, and N for seeds which do not have the indicated tissue..

Caution
-Some seeds are potential allergens. Wear gloves and do not touch them with your bare hands.
-Do not allow the staining solutions to contact your skin. If they touch your skin, rinse the
area thoroughly with distilled water.

Staining and rinsing solutions:


Staining solution Rinsing solution Stain for Color

Property

IKI

IKI-R

Starch

Purple

Aqueous solution

CBB

CBB-R

Protein

Blue

Contain ethanol and acetic acid

OR

OR-R

Lipid

Red

Contain ethanol

IBO 2009
JAPAN
PRACTICAL TEST 1
ANIMAL AND PLANT ANATOMY
_______________________________________________________________________

Staining method:
- Use small Petri dishes for staining and rinsing.
- Stain for 5 to 10 minutes in staining solution.
- Then, rinse the specimens well with rinsing solution.

IBO 2009
JAPAN
PRACTICAL TEST 1
ANIMAL AND PLANT ANATOMY
_______________________________________________________________________

Part B Development of fruits


Materials and equipment
1. Tomato fruits labeled (A)

2. An apple fruit labeled (B)

3. Drawings of flowers labeled (I and II) and strawberry fruits (included


in your envelope)

4. Forceps (used in Task 1)

5. Knife

6. Colored pencils (orange (O), blue (B), green (G)) (used in Task 1)

7. White tray

Introduction
A fruit may develop from some part of a single flower. Therefore, the morphological
features of a fruit are closely related to those of its flower.

Q.2.B.1. (4 points)
There are fruits of tomato (A) and apple (B). Cut the fruits transversely and vertically on a
paper towel in the white tray. Compare the fruits and flower drawings (I and II).
Enter the number of the flower (I or II) that corresponds to each fruits (A, B) in the Box of
Q.2.B.1. in the Answer Sheet.

Q.2.B.2. (11 points)


Using a black pencil, draw and indicate ovules (or seeds), carpels (and/or tissue derived from
carpel), and sepals on the vertical illustrations of the fruits (A1 and B1) of Q.2.B.2. in the

10

IBO 2009
JAPAN
PRACTICAL TEST 1
ANIMAL AND PLANT ANATOMY
_______________________________________________________________________

Answer Sheet. Then, color the following tissues on the same fruit drawings (A1 and B1) in
the colors designated. Refer to the strawberry drawings.
Ovule (or seeds): color pencil O (orange)
Carpels (and/or tissue derived from carpel): color pencil G (green)
Sepals: color pencil B (blue)

Q.2.B.3. (8 points)
Complete the drawings of the transverse illustrations of the fruits (A2 and B2) of Q.2.B.3. in
the Answer Sheet. Draw additional lines and color the ovules (or seeds) and carpels (and/or
tissue derived from carpel) in the colors designated.
Ovule (or seeds): color pencil O (orange)
Carpels (and/or tissue derived from carpel): color pencil G (green)

11

Task 1
Photo of dissected caterpillar, dorsal view

B
C

Task 2
Flowers I and II

Sample drawings of a strawberry

IBO 2009
JAPAN
PRACTICAL TEST 1 ANSWER KEY
ANIMAL AND PLANT ANATOMY

STUDENT CODE:

___________________________________________________________________

Student Code: ___________

20th INTERNATIONAL BIOLOGY OLYMPIAD


12th 19th July, 2009
Tsukuba, JAPAN

PRACTICAL TEST 1
ANIMAL AND PLANT ANATOMY
Total Points: 100
Duration: 90 minutes
ANSWER KEY

IBO 2009
JAPAN
PRACTICAL TEST 1 ANSWER KEY
ANIMAL AND PLANT ANATOMY

STUDENT CODE:

___________________________________________________________________
Q.1.1. (1 point 2 = 2 points

Q.1.2. (3 points)
6

Q.1.3. (3 points)
9

* The answer "8" is scored as one point: the spiracles in the first thoracic segment is
apt not to be noticed, because the first thoracic spiracles are unique in the insects.
Q.1.4. (6 points +[name: 2 points + function: 2 point] 3 points= 18 points)
Photograph of specimen (6 points)
* The students skillfully made a dissection exactly as shown in the photo
prepared are given full marks. Even if the dissection is not so skillful, the dissection,
which may be good enough for the students themselves to distinguish and identify the
tubular structures concerned, is given 4 points. The dissection without sufficient
quality is only given 3 points.
name

Function

IBO 2009
JAPAN
PRACTICAL TEST 1 ANSWER KEY
ANIMAL AND PLANT ANATOMY

STUDENT CODE:

___________________________________________________________________
Q.1.5. (2 points3 = 6 points) , Q.1.7. (4 points3 = 12 points)

A
P

* Q.1.5.: The students have to answer the questions, based on their own direct
observations on the dissected and non-anesthetized caterpillars. As for the circulatory
system, the students could observe only the heart beating in the dorsum of the
abdomen, and this has to be clearly shown in the drawings: the other information is
not evaluated. As for the central nervous system, the following points are important:
1) the crossing with the gut around the oesophagus, 2) the brain in the head, and 3)
ventral localization . As for the digestive system, the anterior and posterior ends have
to be clearly shown. For each structure, the answers missing only one of key points
mentioned-above are given 1 point.
* Q.1.7. The anteriomost and the anterior-second gangia are clearly shown in the head
and just posterior to the point crossing with the gut, respectively. The posteriormost
ganglion shoud be shown in the seventh abdominal segment; the other answer is given
2 points, so far as it is shown in the range of the post abdomen.

Q.1.6. (4 points)
13

* The correct answer is "13", but the "12", "11" and "10 are respectively given 3, 2
and 1 points.

Q.1.8. (2 points)
2

IBO 2009
JAPAN
PRACTICAL TEST 1 ANSWER KEY
ANIMAL AND PLANT ANATOMY

STUDENT CODE:

___________________________________________________________________
Q.2.A.1. (27 points)
Starch
seeds

Embryo

endosperm

II

III

++

IV

seeds

Embryo

endosperm

II

III

IV

seeds

Embryo

endosperm

++

II

++

III

IV

Protein

Lipid

3 point x 9 = 27 points
barley
sunflower
buckwheat
azuki bean

Hordeum vulgare
Helianthus annuus
Fagopyrum esculentum
Vigna angularis

IBO 2009
JAPAN
PRACTICAL TEST 1 ANSWER KEY
ANIMAL AND PLANT ANATOMY

STUDENT CODE:

___________________________________________________________________
Starch
correct combination of symbols
pattern of the
seeds
correct answers
embryo
endosperm
I
II
III

-,
+
or +
-,
-,,+

+
++
N
+
++

or +, N

1. - andare added.
2. Partially point (1 point) are given to - , N, and ++,N pairs.
Protein
seeds
I
II
III

correct combination of symbols


embryo
endosperm

-,
+
+, ++, (+)

+
++

+
++
N

+
++

pattern of the
correct answers

+, N

1. and++are added.
2. Two points are given to +, N pair.
3. One point is given to + in II-embryo box.
4. One point is given to N in II-endosperm box.
5. No points are given when the both box of a seed are filled with N.
Lipid
seeds

II
III

correct combination of symbols


embryo
endosperm

+
++
+
++
+
++

+
N
N
- , (+)
-, (+)

pattern of the
correct answers

>
+, N or ++,N
+,- or ++,-

1. Technical errors are corrected and a new pair is adopted.


2. One point is given to N in II-endosperm box.
3. No points are given when the both box of a seed are filled with N.
4. Partially points (2 points) are given to +,+ or ++, + pairs.

IBO 2009
JAPAN
PRACTICAL TEST 1 ANSWER KEY
ANIMAL AND PLANT ANATOMY

STUDENT CODE:

___________________________________________________________________
Q.2.B.1. ( 4 points)
Fruit

Flower

II

1. The base of the points are changed from 4 points x 1 = 4 points to


2 points x 2 = 4 points.

IBO 2009
JAPAN
PRACTICAL TEST 1 ANSWER KEY
ANIMAL AND PLANT ANATOMY

STUDENT CODE:

___________________________________________________________________
Q.2.B.2. ( 11 points)

or

1 point x 11 = 11 points

IBO 2009
JAPAN
PRACTICAL TEST 1 ANSWER KEY
ANIMAL AND PLANT ANATOMY

STUDENT CODE:

___________________________________________________________________
A1:
1 Sepals are drawn.
2 Sepals are painted in blue
The point is given when a student color the edible part of an apple in blue.
3 Ovules (or seeds) are drawn
4 Ovules (or seeds) are painted in orange
5 Carpels (and/or tissue derived from carpel) are painted in green
B1:

6 Sepals are drawn.


7 Sepals are painted in blue
8 Ovules (or seeds) are drawn
9 Ovules (or seeds) are painted in orange
10 A line indicating the border of carpels are drawn
11 Carpels (and/or tissue derived from carpel) are painted in green
(The answer shown below is also acceptable)

IBO 2009
JAPAN
PRACTICAL TEST 1 ANSWER KEY
ANIMAL AND PLANT ANATOMY

STUDENT CODE:

___________________________________________________________________
Q.2.B.3. ( 8 points)

or

2 points x 4 = 8 points

A2:

B2:

1 Two carpels are shown (there are three carpels in some fruits)
Carpels are shown (1 point), and painted in green (1 point).
2 Ovules (or seeds) are drawn
Ovules are shown (1 point), and painted in orange (1 point).
3 Five carpels are shown (there are more or less carpels depends on fruits)
(The answer shown below is also acceptable)
Carpels are shown (1 point), and painted in green (1 point).
4 Ovules (or seeds) are drawn
Ovules are shown (1 point), and painted in orange (1 point).

IBO 2009
JAPAN
PRACTICAL TEST 1 ANSWER KEY
ANIMAL AND PLANT ANATOMY

STUDENT CODE:

___________________________________________________________________
************ END OF PRACTICAL TEST 1 *****

10

IBO 2009
JAPAN
PRACTICAL TEST 2
BIOCHEMISTRY
_______________________________________________________________________

Student Code: ___________

20th INTERNATIONAL BIOLOGY OLYMPIAD


12th 19th July, 2009
Tsukuba, JAPAN

PRACTICAL TEST 2
BIOCHEMISTRY
Total Points: 100
Duration: 90 minutes

IBO 2009
JAPAN
PRACTICAL TEST 2
BIOCHEMISTRY
_______________________________________________________________________

Dear Participants,

In this test, you have been given the following 2 tasks:


Task 1: Measurement of acid phosphatase activity
Task 2: Protein determination

(70 points)

(30 points)

You must write down your results and answers in the ANSWER SHEET. Answers
written in the Question Paper will not be evaluated.

Please make sure that you have received all the materials and equipment listed for each
task. If any of these items are missing, please raise your hand.

At the end of the test, put the Answer Sheet and Question Paper in the envelope. The
supervisor will collect this envelope.

Good Luck!!

IBO 2009
JAPAN
PRACTICAL TEST 2
BIOCHEMISTRY
_______________________________________________________________________

How to use the spectrophotometer


1. The screen of spectrophotometer (Shimadzu UVmini-1240) must show 400 nm (Fig. 1).
If not, raise your hand. ABS value shown may not be 0.000.
2. Fill a plastic semi-micro cuvette with distilled water (DW) at least up to the shoulders
inside (Fig. 2)
3. Insert the cuvette into the cuvette holder of the instrument, with the transparent surfaces
facing to the left and right (Fig. 3).
4. Shut the lid (Fig. 4).
5. Press AUTO ZERO button (Fig. 5). By this manipulation, the instrument regards the
level of absorbance by the cuvette plus water as zero. This will be used as the blank
control for the rest of this experiment.
6. Now, you are ready to measure absorbance of samples.
7. Replace the water with a sample solution and read an ABS value after the lid is shut. The
absorbance is caused by solutes in the sample solution.
8. You do not have to wash the cuvette after every measurement, if you measure a series of
samples from the dilute to the concentrated.

IBO 2009
JAPAN
PRACTICAL TEST 2
BIOCHEMISTRY
_______________________________________________________________________

shoulders

Fig. 1

Fig. 2

Lid
Cuvette
holder

Light

Fig. 3

Fig. 4

Fig. 5

IBO 2009
JAPAN
PRACTICAL TEST 2
BIOCHEMISTRY
_______________________________________________________________________

Introduction
Acid phosphatase liberates phosphate ions from phosphorylated molecules under acidic
conditions. The purpose of this experiment is to determine the specific activity of the acid
phosphatase. You will measure activities of the acid phosphatase using a crude extract from
potato in Task 1, and determine a protein concentration of the crude extract in Task 2.
Specific activity, which is the activity per unit time per unit weight of protein, is obtained
from Tasks 1 and 2. Specific activity is an index of purity; it increases as the enzyme is
purified.

Caution
1.

You will be handling small amounts of toxic substances (p-nitrophenol and


NaOH).You can choose to wear disposable gloves and safety goggles in the
experiments if you like.

2.

In calculations where answers to previous questions are needed, partials marks will be
given if calculated formulas are correct, even if answers are incorrect.

Materials and Equipments

Quantity

1. Spectrophotometer

2. Micropipettes (P1000)

3. Micropipettes (P200)

4. Tips (one box each for P1000 and P200)

5. Plastic cuvette

6. Test tube holder that accommodates 6-1 to 6-6

IBO 2009
JAPAN
PRACTICAL TEST 2
BIOCHEMISTRY
_______________________________________________________________________

6-1. Crude extract of acid phosphatase (4 ml in a 15-ml plastic tube, labeled 1x


enzyme)
1
6-2. 0.5 M Na acetate buffer (pH 5.6) (2 ml in a 15-ml plastic tube)

6-3. 5 mM pNPP (4 ml in a 15-ml plastic tube)

6-4. 0.5 M NaOH (8 ml in a 15-ml plastic tube)

6-5. 3% NaCl (10 ml in a 15-ml plastic tube)

6-6. Test tubes (Glass)

Task 1 (70 points)


Measurement of acid phosphatase activity
The activity of acid phosphatase is measured by an enzymatic reaction that converts pnitrophenyl phosphate (pNPP) to p-nitrophenol (pNP), liberating phosphate. The product,
pNP, absorbs light whose wavelength is 400 nm with an absorption coefficient (400 nm) of
19000 M-1 cm-1 at extremely alkaline pH. Reaction mixture for an acid phosphatase is
slightly acidic. Thus, it must be alkalinized for quantification of pNP. In Task 1, you will
measure a time course of the reaction and obtain absorbance change per minute that is caused
by 1 ml of crude extract. The absorbance change is converted to concentration change by
using 400 nm. Then, you will calculate a mol number of pNP molecules produced during the
reaction by multiplying the concentration change by a volume of sample that is subjected to
the measurement of absorbance.

IBO 2009
JAPAN
PRACTICAL TEST 2
BIOCHEMISTRY
_______________________________________________________________________

IBO 2009
JAPAN
PRACTICAL TEST 2
BIOCHEMISTRY
_______________________________________________________________________

What is an absorption coefficient

A, absorbance
, absorption coefficient (M-1 cm-1)
C, concentration (M=mol litre-1 )
L, light path length (cm)
I0, intensity of incident light
I, intensity of transmission light

Absorbance (A) is a physico-chemical property of solution that expresses to what extent


a solute absorbs light at a specific wavelength. Absorbance is in proportion to
concentration (C) and light path length (L). The constant in the equation is a value
characteristic to the solute, and is termed the absorption coefficient (). Thus, the
relationship is formulated as A= C (M=mol litre-1) L (cm). Absorbance can be
converted to concentration, since is given and L is 1 cm in this experiment. The
dimension of is M-1 cm-1, because absorbance is an absolute number without units.

IBO 2009
JAPAN
PRACTICAL TEST 2
BIOCHEMISTRY
_______________________________________________________________________

Two enzyme concentrations are to be examined in Task 1. Find the test tube on which 1x
enzyme is labeled, which contains a crude extract of acid phosphatase. Next, find the 15-ml
tube that contains 3% NaCl and remove 1ml of the solution so that the tube now contains 9
ml of 3% NaCl. Add 1 ml of the 1x enzyme solution to it by using a micropipette, which
makes 0.1x enzyme solution. Relabel the tube as 0.1x. Next, find 6 empty test tubes.
Label each tube with an enzyme concentration and a reaction time as follows.

0.1x, 20 min
1x, 20 min
0.1x, 10 min
1x, 10 min
0.1x, 1 min
1x, 1 min

IBO 2009
JAPAN
PRACTICAL TEST 2
BIOCHEMISTRY
_______________________________________________________________________

Q.1.1. (10 points) First, make an experimental schedule in order to perform all reactions, by
describing start () and stop () signs for each reaction in the table in the Answer Sheet,
allowing at least 1 min between the beginning of each reaction. An example for the reaction
of 0.1x, 20 min has been described in the table in the Answer Sheet.

Q.1.2. (15+10 points) Perform the enzymatic reactions according to the protocol described
below and the schedule you made in Q.1.1. Use a new pipette tip in every manipulation.
Agitate a mixture by tapping a test tube immediately after an addition. After you perform all
the reactions, measure A400 of the samples. Write the obtained values in the table in the
Answer Sheet, and plot them in the graph. Please note that since water has been used as
blank, the line will not pass through 0 (zero) on Y-axis (origin).

Protocol for measurement of acid phosphatase activity


Mix 0.12 ml of 0.5 M Na acetate buffer (pH 5.6) and 0.24 ml of 5 mM pNPP in a test
tube. Start the reaction by adding 0.24 ml of an enzyme solution.
After the reaction times of 1, 10, and 20 min, respectively, stop the reaction by adding
0.6 ml of 0.5 M NaOH. NaOH stops the reaction and converts the pNP produced into a
yellow-colored (A400-absorbing) form.
After all reactions are stopped, measure A400 of the samples.

Assay of potato acid phosphatase

0.5 M Na acetate buffer(pH 5.6)0.12


5 mM pNPP
0.24
Enzyme
0.24
0.5 M NaOH
0.6
Sum
1.2
10

ml
ml
ml
ml
ml

IBO 2009
JAPAN
PRACTICAL TEST 2
BIOCHEMISTRY
_______________________________________________________________________

Q.1.3. (15 points) Which enzyme concentration gave better linearity in the relationship
between time and A400? Circle the correct one on the Answer Sheet. Read the slope of this
straight line from the graph.

Q.1.4. (5 points) Using the slope obtained in Q. 1.3, calculate the activity in the form of
A400 change per min per 1 ml of an enzyme solution of concentration 1x. The length of the
light path (L) is 1cm. Your answer should be written with your calculations and the
appropriate unit in the Answer Sheet.

Q.1.5. (5 points) Convert the absorbance change obtained in Q.1.4 to a concentration change
by assuming the 400 of pNP to be 19000 M-1 cm-1. Your answer should be written with your
calculations and the unit per min per 1 ml of 1x enzyme solution in the Answer Sheet.

Q.1.6. (5 points) Convert the concentration change obtained in Q.1.5. to a change in number
of moles of pNP. Your answer should be written with your calculations in moles per min per
ml of 1x enzyme solution in the Answer Sheet.

Q.1.7. (5 points) Calculate the total activity (in moles per min) in 4 ml of 1x enzyme solution that
was initially given .

11

IBO 2009
JAPAN
PRACTICAL TEST 2
BIOCHEMISTRY
_______________________________________________________________________

Task 2 (30 points)


Protein determination

Protein concentration is determined by using a standard protein such as bovine serum


albumin (BSA). In Task 2, you will determine a BSA-equivalent concentration of the 1x
enzyme solution by the Bradford method. The Bradford method takes advantage of an
increase in absorption of Coommassie Brilliant Blue at 595 nm when it is bound to protein.

By diluting a concentrated BSA solution (0.4 mg protein ml-1) with 3% NaCl, a 1/2dilution series was made (0.4, 0.2, 0.1, and 0.05 mg protein ml-1). The dilution series of BSA
and the 0.1x enzyme solution, which was made in Task 1, were all similarly treated with dye.
Optical density at 595 nm (OD595) of these samples was measured and recorded in the table
below.
Table

Sample

[BSA]
(mg ml -1 )
0.00
0.05
0.1
0.2
0.4

0.1x enzyme solution

OD595
0.000
0.070
0.143
0.261
0.521
0.180

Optical density (OD), a measure of the extent to which a substance transmits light or the
absorbance of suspension of particles.

Q.2.1.(10 points) Plot OD595 against BSA concentration in the graph in the Answer Sheet
and depict an approximate straight line.

12

IBO 2009
JAPAN
PRACTICAL TEST 2
BIOCHEMISTRY
_______________________________________________________________________

Q.2.2.(10 points) Estimate a protein concentration of the 0.1x enzyme solution from the
graph, and obtain the protein concentration of the 1x enzyme solution.

Q.2.3.(10 points) Calculate the specific activity (activity per min per mg protein) of the 1x
enzyme solution. Your answer should be written with your calculations and the unit per min
per mg protein in the Answer Sheet.

13

IBO 2009
JAPAN
PRACTICAL TEST 2 ANSWER KEY
BIOCHEMISTRY

STUDENT CODE:

___________________________________________________________________

Country Code: ___________


Country: ________________

Student Code: ___________


Name: ________________

20th INTERNATIONAL BIOLOGY OLYMPIAD


12th 19th July 2009
Tsukuba, JAPAN

PRACTICAL TEST 2
BIOCHEMISTRY
Total Points: 100
Duration: 90 minutes
ANSWER KEY

IBO 2009
JAPAN
PRACTICAL TEST 2 ANSWER KEY
BIOCHEMISTRY

STUDENT CODE:

___________________________________________________________________
Q.1.1. (10 points)
Enzyme conc.
Time (min)
0
1
2
3
4
5
6
7
8
9
10
11
12
13
14
15
16
17
18
19
20
21
22
23

0.1x
20

1x
20

0.1x
10

1x
10

0.1x
1

1x
1

Taking a time course of enzymatic reactions is time-consuming if individual reactions are


performed in series. Thus, this kind of time schedule is necessary to save time. The key
points are that the 1-min reactions are done within the time of a 10-min reaction and the 10min reactions are done within the time of a 20-min reaction.
Q.1.2. (15+10 points)

Time Enzyme concentration


(m in )
1x
0.1x
1
0.766
0.338
10
3.491
0.825
20
3.578
1.342
When new samples are tested for an enzymatic reaction, it is necessary to perform the
reaction at different enzyme concentrations. In the case of Q.1.2., one cannot estimate the
initial rate of the reaction from the data of 1x enzyme; one cannot linearly link the 1-min
point and the 10-min point, because the 20-min point is not on the line projected from the 1min and 10-min points. The saturation of absorbance observed with 1x enzyme is due to
inability of spectrophotometers towards too much concentrated samples. The reaction with
0.1x enzyme proceeded linearly within the time range, and the initial rate of the reaction is
calculated from these data.

IBO 2009
JAPAN
PRACTICAL TEST 2 ANSWER KEY
BIOCHEMISTRY

STUDENT CODE:

A400

___________________________________________________________________
4.0
3.5
3.0
2.5
2.0
1.5
1.0
0.5
0.0
0

10
20
Tim e (m in)

30

In this case, time is the independent variable, which must be plotted on the X-axis, while
absorbance is dependent variable, which must be plotted on the Y-axis. Principally, both axes
should be labeled with unit, but absorbance is an absolute number having no units.

Q.1.3. (15 points)


Linearity

1x

0.1x

Slope= (1.342-0.338)/(20-1)
=
0.053 min-1

The slope is calculated from the data point as described in the above box or directly read from
the graph.

Q.1.4.(5 points)
A= Ans(Q..3.) *(1/0.24)*10
=
0.053*
4.17
*10
=
2.2 ml -1 min-1

The slope obtained with 0.24 ml of 0.1x enzyme is proportionally converted to what would be
obtained with 1 ml of 1x enzyme solution.

IBO 2009
JAPAN
PRACTICAL TEST 2 ANSWER KEY
BIOCHEMISTRY

STUDENT CODE:

___________________________________________________________________
Q.1.5. (5 points)

CA/L=Ans(Q.1.4.)/=2.2/19000=1.2 x 104 M min1 ml1

The rate obtained as the absorbance change per min per ml of 1x enzyme is converted to a
concentration change by using the absorption coefficient of pNP, 19000 M -1 cm-1. Note that L
is 1 cm, as described in the box in the Question paper.

Q.1.6. (5 points)

N=Cx0.0012 (L)=Ans(Q.1.5.)x0.0012=1.4 x107 (mol min1 ml1)

Mol number is calculated by multiplying the concentration change by the volume of mixture
that was subjected to the measurement of absorbance, that is 1.2 ml. Remember that M
(molar) is mol/liter, and the unit of the volume must be liter, not milliliter.

Q.1.7. (5 points)

Total activity=Ans(Q.1.6.)x4=5.6 x107 (mol min1)

Total activity is calculated by multiplying the activity per ml of 1x enzyme solution by the
volume of 1x enzyme solution.

IBO 2009
JAPAN
PRACTICAL TEST 2 ANSWER KEY
BIOCHEMISTRY

STUDENT CODE:

___________________________________________________________________
Q.2.1. (10 points)
0.6
0.5

OD595

0.4
0.3
0.2
0.1
0
0

0.1

0.2
0.3
protein (m g/m l)

0.4

0.5

Optical density is the absorbance of suspension, and can be treated like absorbance in Task
2*. Protein concentration is the independent variable, which must be plotted on the X-axis,
while optical density is the dependent variable, which must be plotted on the Y-axis. Both
axes should be labeled with unit, but optical density is an absolute number having no units
like absorbance.

*Absorbance is a term for solutions, but cannot be used for suspensions. In the Bradford
method, mixing of soluble proteins with the Bradford dye yields insoluble materials that
absorb 595-nm light and are precipitated by low speed centrifugation.

Q.2.2. (10 points)


Concentration of 0.1x enzyme solution= 0.135 mg ml1
Concentration of 1x enzyme solution =-1.35 mg ml1

The plot of 5 point gives a straight line. The intersection of the straight line and an OD=0.18
line shows that the concentration of 0.1x enzyme is 0.135 mg ml1. The concentration of 1x
enzyme solution can be obtained by multiplying the concentration of 0.1x enzyme by 10.

Q.2.3. (10 points)


Specific activity= Ans(Q.1.6.)/Ans(Q.2.2.)
=1.4 x 107 (mol min1) (1.35 mg protein)1
1.0 x107 (mol min1 mg1protein)

Specific activity is the activity per unit weight (mg) of protein. The 1x enzyme
solution has the activity of 1.4 x 107 (mol min1 ml 1) and the protein concentration of
1.35 mg ml1. Thus, the specific activity is calculated by dividing the former by the latter.

IBO 2009
JAPAN
PRACTICAL TEST 3
GENETICS
_______________________________________________________________________

Student Code: ___________

20th INTERNATIONAL BIOLOGY OLYMPIAD


12th 19th July, 2009
Tsukuba, JAPAN

PRACTICAL TEST 3
GENETICS
Total Points: 98
Duration: 90 minutes

IBO 2009
JAPAN
PRACTICAL TEST 3
GENETICS
_______________________________________________________________________

Dear Participants,

This test includes the following 5 tasks:


Task 1: Phenotypic observation of mutant flies

(9 points)

Task 2: Inheritance of white eye mutation

(33 points)

Task 3: Separation of eye pigments

(18 points)

Task 4: Reading chromatography

(14 points)

Task 5: Analysis of White Protein

(24 points)

You must write down your results and answers in the ANSWER SHEET. Answers
written in the Question Paper will not be evaluated.

Please make sure that you have received all the materials and equipment listed for each
task. If any of these items are missing, please raise your hand.

At the end of the test, put the Answer Sheet and Question Paper in the envelope. The
supervisor will collect this envelope.

This series of practicals are time consuming. You will need to be well organized and work
quickly to complete the five tasks.

Good Luck!!

IBO 2009
JAPAN
PRACTICAL TEST 3
GENETICS
_______________________________________________________________________

Task 1 (9 points)
Phenotypic observation of mutant flies
Materials and Equipment

Quantity

1.

Petri dishes numbered (1)-(4) containing live fruit flies

1 set

2.

Stand loupe (magnifying glass)

Introduction
Fruit flies are commonly used materials in genetics studies. Petri dish (1) contains the wild
type, and each of the Petri dishes (2)-(4) contains different mutant flies. Observe the flies
carefully by using the loupe (magnifying glass), but do not open the lid of the dishes. You may
adjust the height and angle of the loupe for your observations.

Q.1.1. (9 points) In the case of each mutant, what kind of trait differs from the wild type?
Choose the characteristic phenotype of the mutant trait from the following list.
A. eye color

B. eye shape

C. wing shape

D. bristle length

E. antenna shape

F. bristle shape

G. leg shape

H. proboscis shape

I. body color

J. abdomen length

IBO 2009
JAPAN
PRACTICAL TEST 3
GENETICS
_______________________________________________________________________

Task 2 (33 points)


Inheritance of white eye mutation

Materials and Equipment

Quantity

1. 1.5 ml tubes containing anesthetized fruit flies labeled


(5a) and (5b), (6a) and (6b), and (7)

1 set

2. Empty Petri dishes

3. White cardboard (place under the Petri dishes for easy observation)

4. Forceps

5. Stand loupe (magnifying glass) (used in Task 1)

6. 1.5 ml tube rack

Introduction
Wild type fruit flies (WT) have red eyes, while the mutant flies (w) have white eyes. w is a
recessive mutation and located on the X chromosome. Each of tubes (5a) and (5b) or (6a) and
(6b) separately contains male or female flies obtained from two different crossings. Tube (7)
contains flies of both sexes from another crossing. Note that flies can be sexed by their patterns
of the posterior dorsal abdomen, which is uniformly black in males.

Female

Male

IBO 2009
JAPAN
PRACTICAL TEST 3
GENETICS
_______________________________________________________________________

Q.2.1. (8 points) Remove the flies from tubes (5a) and (5b) into different Petri dishes, and
observe them by using the loupe (magnifying glass). Examine sex and eye color, and complete
the table with the numbers of flies, including zero.

Q.2.2. (8 points) Remove the flies from tubes (6a) and (6b) into different Petri dishes, and
observe them by using the loupe (magnifying glass). Examine sex and eye color, and complete
the table with the numbers of flies, including zero.

Q.2.3. (8 points) Remove the flies from tube (7) into a Petri dish, and observe them by using
the loupe (magnifying glass). Examine sex and eye color, and complete the table with the
numbers of flies, including zero.

IBO 2009
JAPAN
PRACTICAL TEST 3
GENETICS
_______________________________________________________________________

Q.2.6. (9 points) Which of the following crossings produce the flies of tubes (5a) and (5b),
(6a) and (6b), and (7)? Choose all possible cases and answer with symbols.
A. Homozygous red-eyed females and hemizygous red-eyed males
B. Homozygous white-eyed females and hemizygous white-eyed males
C. Homozygous red-eyed females and hemizygous white-eyed males
D. Homozygous white-eyed females and hemizygous red-eyed males
E. Heterozygous females and hemizygous red-eyed males
F. Heterozygous females and hemizygous white-eyed males

IBO 2009
JAPAN
PRACTICAL TEST 3
GENETICS
_______________________________________________________________________

Task 3 (18 points)


Separation of eye pigments

Materials and Equipment

Quantity

In addition to the materials and equipment used in Task 2, you will use the following set of
equipment in this task.
1. 1.5 ml tubes (8) and (9) containing eye-pigments extraction solution

1 set (1 spare set)

2. Empty 1.5 ml tubes (10) and (11)

1 set (1 spare set)

3. Micropestles (in 15 ml tube)

2 (1 spare)

4. Centrifuge

5. Micropipette (P20)

6. Pipette tips (for P200 and P20)

1 pack

7. Empty 1.5 ml tubes (no numbers written on the lid)

2 (2 spares)

8. Cellulose/plastic sheet

1 (1 spare)

9. Micropipette (P2)

10. Pipette tips (P2)

1 pack

11. 50 ml tube containing solvents

12. Tube rack for the 50 ml tube

IBO 2009
JAPAN
PRACTICAL TEST 3
GENETICS
_______________________________________________________________________

Procedure
1. Select five red-eyed and five white-eyed flies classified in Task 2 (either females or males),
and remove their heads from the bodies using two pairs of forceps.
*Be sure not to crush eyes and abdomen of the flies.
2. By using forceps transfer the heads of red-eyed flies into tube (8), the heads of white-eyed
flies into tube (9), the bodies of red-eyed flies into tube (10), and the bodies of white-eyed
flies into tube (11). Tubes (10) and (11) will be used in Task 5.
3. Insert a micropestle in each of tubes (8) and (9) and grind fly heads by revolving and
pressing the pestle against the bottom of the tube with your hand. Use different pestles for
different samples.

4. Centrifuge tubes (8) and (9) at 14,000 rpm for 3 min (see the Instruction for the
centrifuge at the end of this test, pages 18-19, and ask the supervisor for assistance if
required).
5. Transfer 5 l of supernatant from tubes (8) and (9) into new tubes.
6. Look at the cellulose/plastic sheet. The shorter sides of the cellulose/plastic sheet are the
top and the bottom, and the non-glazy surface is the cellulose surface, which is used in this
8

IBO 2009
JAPAN
PRACTICAL TEST 3
GENETICS
_______________________________________________________________________

experiment. Write your student code with pencil at the top of the cellulose surface.
7. First, spot 1l of the red-eyed heads extract at 1/3 from the left side and about 2 cm from
the bottom of the sheet. Do not draw a line using a pencil or a marker pen, which may
scratch the cellulose coating.
8. Then, spot 1l of the white-eyed heads extract at 1/3 from the right side and about 2 cm
from the bottom of the sheet.
9. When the spots dry, set the sheet into the 50 ml tube so that the bottom of the sheet touches
the solvent, and close the cap tightly. Make sure the spots are not touched by the solvent.
Open and close the cap of the tube quickly to minimize the leak of vapor.
10. Keep the tube straight on the tube rack to start solvent development. You can continue with
task 4 and 5 in the test and come back to this section. Please read part 11 below before
you continue.
11. When the solvent front on the sheet reaches the 30 ml graduation mark of the tube, take the
sheet out from the tube, let it dry on a piece of paper towel and close the cap of the tube.
Raise your hand once the cellulose sheet is dry. (Your assistant will collect your sheet to
evaluate the result.) (18 points)

IBO 2009
JAPAN
PRACTICAL TEST 3
GENETICS
_______________________________________________________________________

Task 4 (14 points)


Reading chromatography
Introduction
Although some of the eye pigments involved in the compound eyes of fruit flies are invisible to
our eyes, they can be visualized under UV lamp. Figure 1 shows an example of eye pigment
spots resolved by chromatography and recorded under UV light. Note that the samples include
not only WT (wild type) and w (white eyes) but also se (sepia eyes), bw (brown eyes), and cn
(cinnabar eyes).
There are two pathways of eye pigment production in fruit flies, ommochrome pathway
and pteridin pathway. The wild type eye color is formed if all pigments produced in both of
the pathways are normally transferred to the compound eyes. Eyes are white if both the
ommochrome and the pteridin pigments are absent. Of the pigments and their intermediate
compounds involved in the two pathways, only those of the pteridin pathway can be separated
by chromatography of this experiment.
The migration of each pigment during chromatography is determined by the chemical
nature of the compound, the solubility of the compound to the solvent, and the migration
distance of the solvent. The migration distance of a given pigment depends on the developing
time of chromatography, but the Rf value is constant for each pigment, which is calculated by
the following equation.

Distance from the base line to the center of the spot


Rf = --------------------------------------------------------------------Distance from the baseline to the solvent front

10

IBO 2009
JAPAN
PRACTICAL TEST 3
GENETICS
_______________________________________________________________________

Table 1 summarizes color under UV lamp and Rf value of each pigment separated from
the compound eyes of fruit flies.

Table 1

Characters of pteridin pigments in compound eyes of fruit flies

Code

Name

Color under UV lamp

Rf value

2-amino-4-hydroxypteridin

blue

0.57

biopterin

blue

0.61

drosopterin

orange

0.21

sepiapterin

yellow

0.52

isoxanthopterin

yellow

0.69

xanthopterin

green-blue

0.38

isosepiapterin

violet-blue

0.25

Q.4.1. (5 points) Choose the pigment from Table 1 that corresponds to each of the spots
separated in the Figure 1 chromatography. Answer with the code in the table. How are the
compositions of the pteridin eye pigments of the mutants different from that of the wild type?
Estimate the approximate amount of each pigment deduced from the Figure 1 chromatography.
Write ++ if there is a lot more of the pigment as compared with the wild type, + if the
pigment is present in similar amounts as in wild type, and - if the pigment is not present.

Q.4.2. (9 points) Given the eye color and the results of chromatography shown in Figure 1,
which of the following abnormalities do se (sepia eyes), bw (brown eyes), and cn (cinnabar
eyes) have? Write the corresponding alphabet.
A.

Ommochrome pigments must be absent.

B.

All pteridin pigments are absent but ommochrome pigments must be present.

C.

Both ommochrome and pteridin pigments are absent.

D.

Constituent of pteridin pigments differs from the wild type.

11

IBO 2009
JAPAN
PRACTICAL TEST 3
GENETICS
_______________________________________________________________________

Solvent front

Spot 1 (yellow)
Spot 2 (blue)
Spot 3 (blue)
Spot 4 (yellow)

Spot 5 (green blue)


Spot 6 (violet blue)
Spot 7 (orange)
Base line

WT

se

bw

cn

Figure 1. Chromatography of eye pigments from wild type and mutant flies

12

IBO 2009
JAPAN
PRACTICAL TEST 3
GENETICS
_______________________________________________________________________

Task 5 (24 points)


Analysis of White Protein

Materials and Equipment

Quantity

1.

1.5 ml tube A: Protein extraction buffer

2.

1.5 ml tubes (two are (10) and (11) of Task 3)

3.

Micropestles (in 15 ml tube)

2 (1 spare)

4.

Electrophoresis apparatus with precast gel

5.

Micropipetter (P200)

6.

Micropipetter (P20)

7.

Pipette tips (for both P200 and P20)

1 pack

8.

1.5 ml tube rack

9.

1.5 ml tube C: Protein electrophoresis marker

13

IBO 2009
JAPAN
PRACTICAL TEST 3
GENETICS
_______________________________________________________________________

Protein extraction and electrophoresis


1. Add 50 l protein extraction buffer (tube A) in the tube (10) (bodies of red-eyed flies) and
(11) (bodies of white-eyed flies) prepared in Task 3. Crush the flies with the micropestle.
Use different micropestles for wild type and mutant samples.
2. Centrifuge tubes (10) and (11) at 14,000 rpm for 3 min, and then transfer supernatant into
fresh 1.5 ml tube (see the Instruction for the centrifuge at the end of this test, pages
18-19, and ask the supervisor for assistance if required).
3. The assistant has prepared a gel for you and it is ready for use.
Load 5 l of each sample on the slots in the middle of the gel plate in the order of molecular
weight marker, red eye and white eye (from left to right). When you have finished sample
loading, raise your hand for the supervisor. Your assistant will take care of the apparatus and
start electrophoresis.
4. After 5 min, call your assistant by raising your hand. Your assistant will collect the lower
part of the apparatus and take a photograph of the gel for evaluation (18 points). Please
check the image on the camera with your assistant.

Analysis of protein electrophoresis data


M1, M2 and M3 flies are different mutant lines for the eye pigment genes. After separating
proteins of these mutant flies through SDS polyacrylamide gel, proteins were transferred onto a
nylon filter to be probed with antibody that specifically recognizes the protein encoded by the
white gene. The following result was obtained.

14

IBO 2009
JAPAN
PRACTICAL TEST 3
GENETICS
_______________________________________________________________________

WT

M1

M2

M3

Q.5.1. (3 points) Which of the following defects of eye pigment genes causes the
electrophoresis results of M1, M2 and M3? Choose the corresponding symbols from A, B and
C.

A. The mRNA initiation site of the white gene is deleted, and the gene is not
expressed.
B. A stop codon mutation has occurred in the coding region of the White protein,
resulting in failure of translation of carboxyl terminal peptide sequence
corresponding to molecular weight 20 kDa.
C. Although a normal White protein is synthesized, genes involved in the synthesis of
ommochrome pigments are defective.

Q.5.2. (3 points) Choose another defect of eye pigment gene from A, B and C that would cause
the same phenotypes as M1, M2 and M3.
15

IBO 2009
JAPAN
PRACTICAL TEST 3
GENETICS
_______________________________________________________________________

A. The coding sequence of the white gene is fused with the coding sequence of
another gene by chromosomal translocation, resulting in a novel sequence encoding
a fusion protein that retains antibody reacting sites but exhibits about 30 % lower
molecular weight.
B. A single base substitution has occurred in the protein-coding region of the white
gene changing an amino acid coding sequence into another amino acid coding
sequence.

However, immunological reactivity of the altered protein for the

antibody is not lost.


C. A large deletion exists in the chromosomal region that involves the entire white
gene.

16

IBO 2009
JAPAN
PRACTICAL TEST 3
GENETICS
_______________________________________________________________________

Instructions for the centrifuge


Ask the supervisor for assistance if required
1. Press the OPEN button at the upper-right of the operation panel (Fig. 1 - 1) to open the
centrifuge lid (2).
2. The rotor is covered by a plastic cap (Fig. 2 - 3). To remove the cap, hold the cap with one
hand, and unfasten the central black screw (4) anti-clockwise with the other hand.
3. There are 24 holes inside the rotor (Fig. 3). Set the sample tubes in a symmetric position,
considering their balance.
4. Turn the rotor cap screw (4) clockwise to fasten the cap on the rotor.
5. Close the centrifuge lid firmly. You should hear a beep that tells complete closure.
6. The centrifuge speed (140 x 100 rotation per minute) and time (3 min) is preset. Confirm
the set parameters in the windows (5) and (7) by pressing the DISP/CE button (6), and
press the START button (8) to start centrifugation.
7. When centrifugation is finished, the lid (2) is automatically unlocked. Then, open the lid (2)
fully and remove the rotor cap by unfastening the screw (4) anti-clockwise while holding
the rotor cap with the other hand.
8. In order to not disturb the precipitates, take out the sample tubes carefully from the rotor.
Leave them on the tube stand.
9. Replace the rotor cap (3) and fasten the screw (4) clockwise, and close the centrifuge lid
(2).

17

IBO 2009
JAPAN
PRACTICAL TEST 3
GENETICS
_______________________________________________________________________

18

IBO 2009
JAPAN
PRACTICAL TEST 3 ANSWER SHEET
GENETICS

STUDENT CODE:

___________________________________________________________________

Student Code: ___________

20th INTERNATIONAL BIOLOGY OLYMPIAD


12th 19th July, 2009
Tsukuba, JAPAN

PRACTICAL TEST 3
GENETICS
Total Points: 100
Duration: 90 minutes
ANSWER KEYS AND CRITERIA

IBO 2009
JAPAN
PRACTICAL TEST 3 ANSWER SHEET
GENETICS

STUDENT CODE:

___________________________________________________________________
Q.1.1. (9 points)

3
(2)

(3)

(4)

Q.2.1.(8 points)
Four points for complete row
Four points for complete row
Four points for each complete row.
Two points if other numbers for 10 is written.
No points if other numbers for 0 is written.
red females

white females

red males

white males

(5a)

10

(5b)

10

Q.2.2.(8 points)
Four points for each complete row.
Two points if other numbers for 10 is written.
No points if other numbers for 0 is written.
red females

white females

red males

white males

(6a)

10

(6b)

10

Q.2.3.(8 points)
Two points for each box.
For red females, one point for the numbers 8, 9, 11 or 12.
For white females, only zero is acceptable and other numbers
receive no point.
For red males and white males, one point for 4 or 6.

(7)

red females

white females

red males

white males

10

IBO 2009
JAPAN
PRACTICAL TEST 3 ANSWER SHEET
GENETICS

STUDENT CODE:

___________________________________________________________________
Q.2.4.(1 point)
D
Q.2.5.(1 point)
E

Q.2.6. (9 points)
Three points for each.
For (5a) and (5b), one point if only one answer is chosen.

(5a) and (5b)

A, C

(6a) and (6b)

(7)

Task 3 (18 points)


Criteria
At lest one sample spot is confirmed on the sheet.

4 points

2.

Solvent front is moved as specified (~3 cm)

4 points

3.

Pigments are migrated in near straight line.

4 points

4.

At least two pigment spots are separated.

6 points

1.

IBO 2009
JAPAN
PRACTICAL TEST 3 ANSWER SHEET
GENETICS

STUDENT CODE:

___________________________________________________________________
Q.4.1. (5 points)
One point for each column with correct answers for all
Spot No.

WT

se

bw

cn

Pigment
(A-G)
E

++

Q.4.2. (9 points)
3 points for each
se

bw

cn

Photograph of the gel (18 points)


Criteria
Loading of the molecular weight marker and two samples

1.
2.

3 x 3 points

Separation of the molecular weight marker with several bands visible.


3 points

3.

BPB dye bands are well migrated for the two samples

2 x 3 points

IBO 2009
JAPAN
PRACTICAL TEST 3 ANSWER SHEET
GENETICS

STUDENT CODE:

___________________________________________________________________
Q.5.1. (3 points)
3 points only when all answers are correct. No partial points.

M1

M2

M3

Q.5.2. (3 points)
3 points only when all answers are correct. No partial points.

M1

M2

M3

************ END OF PRACTICAL TEST 3 ************

IBO 2009
JAPAN
PRACTICAL TEST 4
CELL PHYSIOLOGY
_______________________________________________________________________

Student Code: ___________

20th INTERNATIONAL BIOLOGY OLYMPIAD


12th 19th July, 2009
Tsukuba, JAPAN

PRACTICAL TEST 4
CELL PHYSIOLOGY
Total Points: 10191
Duration: 90 minutes

IBO 2009
JAPAN
PRACTICAL TEST 4
CELL PHYSIOLOGY
_______________________________________________________________________

Dear Participants,

In this test, you have been given the following 2 tasks:


Task 1: Study on the cell cycle (62 61 points)
Task 2: Study on the motile mechanism of unicellular algae (39 30 points)

You must write down your results and answers in the ANSWER SHEET. Answers
written in the Question Paper will not be evaluated.

Please make sure that you have received all the materials and equipment listed for each
task. If any of these items are missing, please raise your hand.

At the end of the test, put the Answer Sheet and Question Paper in the envelope. The
supervisor will collect this envelope.

Good Luck!!

IBO 2009
JAPAN
PRACTICAL TEST 4
CELL PHYSIOLOGY
_______________________________________________________________________

Task 1 (62 61 points)


Study on the cell cycle

Introduction
In many unicellular organisms, gene duplication and segregation occur in a controlled
manner as the cell body grows. When the environmental conditions in which cells are
growing become less favorable or stressful, genetic exchange is often seen via cell
conjugation (mating) between cells of different mating the same cell types. That
phenomenon is essential for life and is controlled by both internal and external condition of
the cells. To date, we have tried to reveal these mechanisms by studying mutants in several
model organisms. For example, the investigation of mutants in the fission yeast,
Schizosaccharomyces pombe has provided us with invaluable information. Wild-type S.
pombe cells proliferate by repeated cell elongation followed by symmetric cell division. On
the other hand, under stressful conditions such as starvation, cells undergo arrested growth at
an appropriate stage of the cycle, and spore formation is induced via cell conjugation to
overcome the stressful conditions.

The following task involves examining cell proliferation using S. pombe.

IBO 2009
JAPAN
PRACTICAL TEST 4
CELL PHYSIOLOGY
_______________________________________________________________________

Materials and equipment

Quantity

1. Fixed culture of wild-type strain; a

2. Fixed culture of wild-type strain; b

3. Fixed culture of wild-type strain; c

4. Fixed culture of wild-type strain; d

5. Micro tube stand

6. Microscope

7. Disposable cell counter

8. Counter

9. 1.5ml microtube

10. Box of glass slides

11. Box of coverslips

12. Micropipette P-20 (capacity 2-20L)

13. Box containing micropipette tips

14. Fixed culture of wild-type strain incubated at 25C; W25

15. Fixed culture of wild-type strain incubated at 36C; W36

16. Fixed culture of cdc25 mutant strain incubated at 25C; M25

17. Fixed culture of cdc25 mutant strain incubated at 36C; M36

18. Photograph of cells stained with Calcofluor and DAPI

IBO 2009
JAPAN
PRACTICAL TEST 4
CELL PHYSIOLOGY
_______________________________________________________________________

Part A
The growth curve of S. pombe wild-type haploid (n=1) incubated at 25C is shown below.
Sampling of culture medium has been carried out at time points indicated by an arrow.
Culture media a, b, c and d on the bench correspond to a sample of the culture taken at a
certain time of cultivation I, II, III or IV. Observe each of the media with a microscope, and
answer the following questions. Please stir the microtube just before observation.

Growth curve of S. pombe

Density of culture log.

II

III

IV

Time after cultivation

Q.1.A.1. (5 2x2points) Compare the cells in sample a with those in sample b. Determine
whether the following statements are true or false. Put a cross mark (x) in the appropriate
boxes in the answer sheet. , and answer the following questions.
1

There is no difference between samples a and bIn which sample are the cells

rounder?

IBO 2009
JAPAN
PRACTICAL TEST 4
CELL PHYSIOLOGY
_______________________________________________________________________

Cells in sample a are rounder than those in sample bIn which sample is there a

higher population of cells undergoing cytokinesis?


Cytokinesis is defined as the part of the cell cycle from initiation of septum formation to the
separation of daughter cells.
3

Cells in sample b are rounder than those in sample a

Population of cells containing septa in sample b are higher than those in sample a

Population of cells containing septa in sample a are higher than those in sample b

Q.1.A.2. (6 points) Measure the number of cells per 1 ml culture medium in sample a by
using the cell counter as indicated below. Daughter cells that have not separated should be
counted as a single cell. Write your Answer on the Answer Sheet. Notice that each student
has received one cell counter but each counter has two counting chambers. You can make
two measurements with this counter.

IBO 2009
JAPAN
PRACTICAL TEST 4
CELL PHYSIOLOGY
_______________________________________________________________________

Q.1.A.3. (5 points) Cytokinesis is defined as the part of the cell cycle from initiation of
septum formation to the separation of daughter cells. Measure the percentage of cells
undergoing cytokinesis in the culture medium in sample a. You should count more than 100
cells in total by choosing several optical fields at random. You must write the percentage of
cells undergoing cytokinesis AND the total number of cells you counted on the Answer
Sheet.

Q.1.A.4. (4 points) In the growth curve shown above, cells divided asynchronously during
the logarithmic phase. Estimate the time period required for one round of the cell cycle of
cells in logarithmic phase, provided that it takes 25 min from the beginning of septum
formationcytokinesis to the separation of the daughter cells. Enter both the formula and your
answer in the Answer Sheet.

Q.1.A.5. (3 points) What applies to the cells in culture medium c?


A

vigorously growing

forming spores

conjugating

most of cells are dead

undergoing meiosis

Q.1.A.6. (8 points) Which culture medium (I, II, III, or IV ) corresponds to a, b, c and d,
respectively? Fill in the Answer Sheet by putting I, II, III, or IV.
7

IBO 2009
JAPAN
PRACTICAL TEST 4
CELL PHYSIOLOGY
_______________________________________________________________________

IBO 2009
JAPAN
PRACTICAL TEST 4
CELL PHYSIOLOGY
_______________________________________________________________________

Part B
Both wild-type and cdc25-mutant strains were incubated at 36C for 4 hrs after logarithmic
growth at 25C. cdc25-mutant strain is not able to grow at 36C while it normally grows as
well as wild-type strain at 25C. The number of nuclei per cell is shown below. Daughter
cells that have not separated were counted as a single cell.

IBO 2009
JAPAN
PRACTICAL TEST 4
CELL PHYSIOLOGY
_______________________________________________________________________

Q.1.B.1.(3 points) By observing the phenotypes of the cultures W25, W36, M25 and M36,
what can we conclude?

Condition

Most of cdc25 mutant cells

Wild type cells

A 25C

Do not undergo cytokinesis

Undergo cytokinesis

B 25C

Undergo cytokinesis

Do not undergo cytokinesis

C 36C

Do not undergo cytokinesis

Undergo cytokinesis

D 36C

Undergo cytokinesis

Do not undergo cytokinesis

25C and

No significant difference in cytokinesis between cdc25 mutant and wild

36C

type cells

10

IBO 2009
JAPAN
PRACTICAL TEST 4
CELL PHYSIOLOGY
_______________________________________________________________________

Q.1.B.2.(4 points) To measure cell length, your microscope is equipped with a micrometer
in the eyepiece lens. In order to calibrate the eyepiece micrometer, a second micrometer,
called the stage micrometer, is place on the stage of the microscope. The distance between
any two adjacent lines on the stage micrometer is known to be 10.0m. By matching the
lines on both micrometers, we can determine the distance between two adjacent lines of the
eyepiece micrometer. Determine this distance inm to two decimal places using the figure
shown below.

11

IBO 2009
JAPAN
PRACTICAL TEST 4
CELL PHYSIOLOGY
_______________________________________________________________________

Q.1.B.3.(12 points) Measure the longitudinal length of more than 10 cells selected at
random in culture media of M36. Graph your results in the Answer Sheet according to the
example indicated below. The scale of your eyepiece micrometer is 4m. Do not forget to
indicate the unit of length.

Q.1.B.4.(2 points) What can you conclude from your observations of each culture?
cdc25 cells are longer than wild-type cells at:

A both 25C and 36C.


B 36C but not 25C.
C 25C but not 36C.
D There is no significant difference in cell length between wild-type and cdc25 cells at
both 25C and 36C.

12

IBO 2009
JAPAN
PRACTICAL TEST 4
CELL PHYSIOLOGY
_______________________________________________________________________

Part C
The following experiment was done using wild-type cells and 5 mutant strains (A-E). These
mutant strains grow at 25C as well as wild-type cells but are not able to grow at 36C.
All cells undergoing logarithmic growth at 25C were then incubated at 36C for an
additional 4 hrs before chemical fixation. Fixed cells were stained with both Calcofluor
(stains septa) and DAPI (stains DNA) for observation using fluorescence microscopy (as seen
in the photograph provided on the bench).

Q.1.C.1.(10 points) The following statements describe the phenotype of the mutants
incubated at 36C. Identify the descriptions that correspond with each of the mutant strains
(A-E), respectively.
1.

Cytokinesis is repeated independently of progression of the cell cycle.

2.

Cell cycle progresses but cytokinesis has not begun.

3.

Cell cycle is arrested at interphase.

4.

Karyokinesis is severely defective.

5.

Completion of cytokinesis is suppressed.

13

IBO 2009
JAPAN
PRACTICAL TEST 4
CELL PHYSIOLOGY
_______________________________________________________________________

Task 2 (39 30 points)


Study on the motile mechanism of unicellular algae

Introduction
Some unicellular algae and zygotes of multicellular algae swim actively. This behavior is
important for migration to appropriate conditions for growth and sexual reproduction.
Chlamydomonas reinhardtii, an unicellular green alga, swims using flagella movement.
Flagella often fall out when in contact with some stimuli, and some are absorbed into the cell
body at a specific stage of the cell cycle.

This task concerns the machinery of flagella movement and flagella regeneration in C.
reinhardtii.

Materials and equipment

Quantity

1. C. reinhardtii wild-type cells (wt)

2. C. reinhardtii oda1mutant (oda)

3. C. reinhardtii pf17 mutant (pf)

4. Microscope

5. Box of glass slides

6. Box of glass coverslips

7. Acetic acid solution (A)

8. Neutralizing solution (N)

9. Disposable pipette1 ml

10

14

IBO 2009
JAPAN
PRACTICAL TEST 4
CELL PHYSIOLOGY
_______________________________________________________________________

10. 1.5 ml microtube

11. Vinyl tape

12. Scissors

Caution
C. reinhardtii flagella frequently stick to glass slides. As a result, the swimming ability of
the cell is hindered. Therefore, cells immobilized on a glass slide should be excluded from
observations for cell movement. It is recommended to make a chamber as indicated below
for the observation. Slips of vinyl tape are stuck on a glass slide in parallel, and a coverslip is
mounted on the slips after the samples are loaded by pipette. This chamber will provide a
space for the cells to swim.

15

IBO 2009
JAPAN
PRACTICAL TEST 4
CELL PHYSIOLOGY
_______________________________________________________________________

Part A
Microscopically compare the wild-type (wt) and pf17 mutant (pf) cells. This mutant has a
normal shape and cellular structure but lacks a component of the radial spoke head in its
flagella.

Q.2.A.1. (6 points) In comparison to wild-type cells, pf17 mutant cells:


A

swim in the same manner

swim but more slowly

swim but more rapidly

do not swim at all

Q.2.A.2. (4 2 points) What can you conclude about the function of the radial spoke head?
A

essential for flagella movement

no effect on flagella movement

suppresses flagella movement

coordinates flagella movement

16

IBO 2009
JAPAN
PRACTICAL TEST 4
CELL PHYSIOLOGY
_______________________________________________________________________

Part B
Microscopically compare the wild-type (wt) and oda1 mutant (od). This mutant lacks a kind
of dynein in flagella whereas the shape and other cellular structures are normal.

Q.2.B.1. (6 points)
In comparison to wild-type cells, oda1 mutant cells swim:
A

in the same manner

more slowly and smoothly

more slowly and jerkily

more rapidly and smoothly

rapidly and jerkily

Q.2.B.2. (4 2 points)
What can you conclude about the function of the dynein lost in the oda1 mutant?
A

essential for flagella movement

no effect on flagella movement

suppresses increases flagella movement

coordinates flagella movement

17

IBO 2009
JAPAN
PRACTICAL TEST 4
CELL PHYSIOLOGY
_______________________________________________________________________

Part C
Study the effect of acetic acid on flagella as follows:
(i) Measure the percentage (A) of wild-type cells containingwith flagella in 20 cells by
choosing the most appropriate strain for this study.
(ii) Transfer about 1 ml of the culture selected in (i) into a 1.5 ml microtube by disposable
pipette, and add one drop of acetic acid solution
(iii) Add one drop of neutralizing solution after 30 seconds
(iv) Measure the percentage (B) of cells containing flagella in 20 cells after the treatment

Q.2.C.1.( (6 4points x 2)=12 8 points) Calculate the percentage of cells containing flagella
in the pretreatment (A) and posttreatment (B) samples.

18

IBO 2009
JAPAN
PRACTICAL TEST 4
CELL PHYSIOLOGY
_______________________________________________________________________

Part D
Wild-type cells with their flagella removed were incubated under different conditions (i, ii or
iii). The following graph indicates the flagella length relative to its original length at
different time points.

(i) control (incubated without inhibitors)


(ii) incubated with cycloheximide, an inhibitor of protein synthesis
(iii) incubated with colchicine, an inhibitor of microtubule formation

19

IBO 2009
JAPAN
PRACTICAL TEST 4
CELL PHYSIOLOGY
_______________________________________________________________________

In addition, photographs of cells after incubation for 120 min are shown.

20

IBO 2009
JAPAN
PRACTICAL TEST 4
CELL PHYSIOLOGY
_______________________________________________________________________

Q.2.D.1.(5 4 points) Are the following statements supported by observing the results of cells
incubated with cycloheximide? Put a cross mark (x) in the appropriate boxes in the answer
sheet.
1

All proteins incorporated in regenerated flagella are synthesized de novo

Regenerated flagella show no motility because of a lack of dynein

The protein component of flagella is stored before flagella removal

43

De novo synthesis of protein is essential for the complete regeneration of flagella.

54

De novo synthesis of protein is essential for the formation of the basal body of
flagella

Q.2.D.2.(2 points)

Based on your observations of cells incubated with colchicines, what is

required for the regeneration of flagella?


A

Polymerization of tubulin

Polymerization of actin

Polymerization of keratin

Depolymerization of tubulin

Depolymerization of actin

Depolymerization of keratin

21

IBO 2009
JAPAN
PRACTICAL TEST 4 ANSWER SHEET
CELL PHYSIOLOGY

STUDENT CODE:

___________________________________________________________________

Country Code: ___________


Country: ________________

Student Code: ___________


Name: ________________

20th INTERNATIONAL BIOLOGY OLYMPIAD


12th 19th July, 2009
Tsukuba, JAPAN

PRACTICAL TEST 4
CELL PHYSIOLOGY
Total Points: 91
Duration: 90 minutes
ANSWER Key

IBO 2009
JAPAN
PRACTICAL TEST 4 ANSWER SHEET
CELL PHYSIOLOGY

STUDENT CODE:

___________________________________________________________________
Q.1.A.1. (4 (22) points)
1

b
a

Q.1.A.2. (6 points)

4.8

e.g.

cells/ml
1. Actual length of cell is 4.790.60 106 cells /ml. This value is determined in two
independent experiments by 10 graduate students.
2. Full point is given for 2.4106 9.6106 cells /ml.
3. 3 point is given for answer between 1106 9.9106 cells /ml except range
prescribed in 2.
4. 1 point is given for answer between 1105 9.9107 cells /ml except range
prescribed in 2 and 3.
Q.1.A.3. (5 points)

Total cells counted


e.g.

19

120

1. Actual percentage of cells undergoing cytokinesis is 19.45.1%. This value is


determined in two independent experiments by 10 graduate students.
2. To fill in a number more than 100 is required for the mark. In the case of under
100 or no entry, 2 points are subtracted.
3. Points are given as follows,
For 5 points, 0.1-38%*
3 points, 39-50%
Minimum value 0.1% is provided. Because, students who are not
familiar with adjustment of iris in microscope may underestimate a
percentage of cells undergoing cytokinesis because they tend to miss a
cell containing thin septum. Under 0.1% may not be acceptable because
students are unable to practically count more than 1,000 cells.
Maximal value for full marks is twice 19%.
50% is a limit for marks because it has never occurred more than half of
cells are undergoing cytokinesis in an asynchronous culture in wild-type
of S. pombe.
2

IBO 2009
JAPAN
PRACTICAL TEST 4 ANSWER SHEET
CELL PHYSIOLOGY

STUDENT CODE:

___________________________________________________________________
Q.1.A.4. (4 points)
formula

solution

e.g. 100% 19% 25 min

132 min

1. Formula or calculation process must be shown for fill marks.


2. Value found in Q.1.A.3 must be applied.
3. Miscalculation is subtracted 2 points.

Q.1.A.5. (3 points)

B (C, D, E are also possible*)


*More than half of cells in culture medium c are forming spores. But a few
percentages of cells are conjugating or undergoing meiosis. Moreover, some dead
cells are included there. Answer of B, C, D, or E with A is given 1 point because
there is no cell vigorously growing.

Q.1.A.6. (8 (24) points)


a

II

IV

III

Q.1.B.1.(3 points)

C
Plural choice is null.
Q.1.B.2.(4 points)

3.85
m
Two points are given for 3.7-4.0.

IBO 2009
JAPAN
PRACTICAL TEST 4 ANSWER SHEET
CELL PHYSIOLOGY

STUDENT CODE:

___________________________________________________________________
Q.1.B.3.(12 points)

1. It is essential for full mark that students make a graph similar to one of above four.
Top is a summarized result by 10 graduate students when they used objective lens

IBO 2009
JAPAN
PRACTICAL TEST 4 ANSWER SHEET
CELL PHYSIOLOGY

STUDENT CODE:

___________________________________________________________________
(40). Three lower graphs may be possible if 4, 10, or 20, are applied.
Because, there is no description which objective lens should be used for experiment
in the problem.
2. 3 points are subtracted if student does not fill the appropriate unit in the bottom
bracket.

Q.1.B.4.(2 points)

A
Plural choice is null.

Q.1.C.1. (10 (25) points)


1

IBO 2009
JAPAN
PRACTICAL TEST 4 ANSWER SHEET
CELL PHYSIOLOGY

STUDENT CODE:

___________________________________________________________________
Q.2.A.1. (6 points)

D
Plural choice is null.

Q.2.A.2. (2 points)

A
Plural choice is null.

Q.2.B.1. (6 points)

C
Plural choice is null.

Q.2.B.2. (2 points)

C and/or D
Q.2.C.1. (8 (4x2) points)
A

e.g. 95

0
%

1. In A 4 points are given for value more than 70% since almost all cells contain
flagella. Value under 70% is subtracted 2 points because of miss probably caused
by inadequate technique for observation.
2. In B 4 points are given for 0%. Because effect of acetic acid on removal of flagella
is absolute in this experimental condition. Other value smaller than A is subtracted
2 points.

IBO 2009
JAPAN
PRACTICAL TEST 4 ANSWER SHEET
CELL PHYSIOLOGY

STUDENT CODE:

___________________________________________________________________

Q.2.D.1. (4 (1x4) points)


Supported

Not-supported

1
2
3

Q.2.D.2. (2 points)

A
Plural choice is null.

************ END OF PRACTICAL TEST 4 ************

INTERNATIONAL BIOLOGY OLYMPIAD


THEORY PROBLEMS

2008, India, Mumbai















All IBO examination questions are published under the following Creative Commons license:



CC BY-NC-SA (Attribution-NonCommercial-ShareAlike) https://creativecommons.org/licenses/by-nc-sa/4.0/
The exam papers can be used freely for educational purposes as long as IBO is credited and
new creations are licensed under identical terms. No commercial use is allowed.

IBO 2008
INDIA
THEORETICAL TEST PART A
_______________________________________________________________________

Student Code: ____________

Country:
___________________

19th INTERNATIONAL BIOLOGY OLYMPIAD


13th 20th July, 2008
Mumbai, INDIA

THEORETICAL TEST PART A

Write all answers in the ANSWER SHEET.

IBO 2008
INDIA
THEORETICAL TEST PART A
_______________________________________________________________________

Dear Participants

You have a total of 2 hours for answering Part A.

The questions in Part A have only one correct answer. Mark the correct
answer with X on the Answer Sheet, which is provided separately. The
correct way of marking the cross is shown below. Use a pen or a dark pencil
to mark your answers.
Q.
NO.

20

The answers written in the Question Paper will not be evaluated.

Mark your answers clearly. Avoid any corrections in the Answer Sheet.

NOTE: Some of the questions may be marked Skipped / Deleted. DO NOT


attempt these questions. Also, read the question completely before
attempting it as some questions may continue from one page to the next.

The maximum number of points is 63.

Your Answer Sheets will be collected at the end of the examination.


Good Luck!!

Country: ___________________________________
First name: _________________________________
Middle name: _______________________________
Family name: ________________________________
Student Code: _______________________________

IBO 2008
INDIA
THEORETICAL TEST PART A
_______________________________________________________________________

PART A
CELL BIOLOGY (13 points)
1. (1 point) The central dogma originally proposed by Francis Crick has seen
changes reflecting new insights obtained from time to time. Which one of the
following schematics correctly depicts our current understanding of the
replication of genetic material in various organisms and the flow of
information in biological systems?

IBO 2008
INDIA
THEORETICAL TEST PART A
_______________________________________________________________________

2. (1 point) In an experiment, mice were injected intravenously with uniformly


labeled [14C] glucose. The molecules in the body where the
found are:
a. essential amino acids and proteins.
b. lipids and all vitamins.
c. proteins and lipids.
d. proteins and all vitamins.

14

C would be

IBO 2008
INDIA
THEORETICAL TEST PART A
_______________________________________________________________________

3. (1 point) The following schematics depict the orientation of F1FO-ATPase


along with the direction of H+-transport and ATP synthesis/hydrolysis.

Of the above schematics,


a. Only I is correct.
b. Only II is correct.
c. Only III is correct.
d. Both I and III are correct.

IBO 2008
INDIA
THEORETICAL TEST PART A
_______________________________________________________________________

4. (1 point) A given DNA sample has 60% purines. The source of this DNA is
most likely to be:
a. a eukaryotic cell.
b. a bacterial cell.
c. a bacteriophage with double-stranded DNA.
d. a bacteriophage with single-stranded DNA.

IBO 2008
INDIA
THEORETICAL TEST PART A
_______________________________________________________________________

5. (1 point) The stage of cell division shown in the figure below represents:

a. Meiotic metaphase I with n = 4


b. Meiotic metaphase II with n = 4
c. Meiotic metaphase II with n = 8
d. Meiotic metaphase I with n = 2

IBO 2008
INDIA
THEORETICAL TEST PART A
_______________________________________________________________________

6. (1 point) Polymerase Chain Reaction (PCR) is a technique for rapid


amplification of DNA segments. If you are given double-stranded DNA with
appropriate forward and reverse primers as shown in the figure below, the
minimum number of cycles you will require to obtain at least one copy of the
desired fragment PQ will be:

a. 1
b. 3
c. 4
d. 40

IBO 2008
INDIA
THEORETICAL TEST PART A
_______________________________________________________________________

7. (1 point) Which of the primer pairs is the correct one to amplify the gene
sequence below with PCR?
5-GCGTTGACGGTATCAAAACGTTAT TTTACCTGGTGGGCTGTTCTAATC-3

a. 5-GCGTTGACGGTATCA-3 and 5-TGGGCTGTTCTAATC-3


b. 5-CGCAACTGCCATAGT-3 and 5-TGGGCTGTTCTAATC-3
c. 5-GCGTTGACGGTATCA-3 and 5-GATTAGAACAGCCCA-3
d. 5-TGATACCGTCAACGC-3 and 5-GATTAGAACAGCCCA-3

IBO 2008
INDIA
THEORETICAL TEST PART A
_______________________________________________________________________

8. (1 point) Equal concentrations of urea, ethyl urea, and dimethyl urea were
separately added to a suspension of red blood cells (RBC). The relative rates
of diffusion of these molecules into RBCs will be:

a. 1 > 2 > 3
b. 1 > 2 = 3
c. 3 > 2 > 1
d. 3 = 2 > 1

10

IBO 2008
INDIA
THEORETICAL TEST PART A
_______________________________________________________________________

9. (1 point) A region of a double-stranded DNA is represented in the following


schematic and the hyphens denote sequences of unspecified lengths:

The region of DNA enclosed within the box undergoes inversion. Which
one of the following correctly depicts the above DNA after inversion?

11

IBO 2008
INDIA
THEORETICAL TEST PART A
_______________________________________________________________________

10. (1 point) A rare genetic disease is characterized by immuno-deficiency,


developmental and growth delay, and microcephaly. Suppose you extract
DNA from a patient with this syndrome and find almost equal quantities of
long and very short DNA strands, which enzyme is likely to be defective in
this patient?

a. DNA ligase
b. Topoisomerase
c. DNA polymerase
d. Helicase

12

IBO 2008
INDIA
THEORETICAL TEST PART A
_______________________________________________________________________

11. (1 point) A scientist has suggested that a homolactic fermenting organism


grows anaerobically on glycerol 3-phosphate as the sole source of carbon,
exclusively using the following pathway:

However, the scientific community rejected this suggestion because:


a. the number of ATP molecules produced is insufficient to support growth.
b. the number of NAD+ that are reduced is not same as the number of NADH
that are oxidized in the pathway.
c. the carbon source is not as reduced as glucose and hence, cannot support
growth.
d. the number of negative charges on lactic acid (which is being produced) is not
the same as that on glycerol 3-phosphate (which is being consumed).

13

IBO 2008
INDIA
THEORETICAL TEST PART A
_______________________________________________________________________

12. (1 point) The growth curve of a bacterial culture grown in a rich medium at
37C is shown in Figure A. The same organism when exposed to 45C for 30
min and then inoculated into a rich medium at 37C, exhibited a growth curve
shown in Figure B.

Time (hours)

Time (hours)

Turbidity
OD600

Which of the following statements is most likely to explain the growth


pattern in Figure B?
a. Heat kills the original bacterial population and the growth pattern observed is
due to a contaminating bacterial strain.
b. Heat causes growth arrest at a particular stage, thereby synchronizing cells
and resulting in all cells dividing at the same time.
c. Heat exposure alters surface properties of cells causing errors in turbidity
measurements.
d. The increase in turbidity is not due to growth but caused by increasing lysis of
heat-treated cells with time.

14

IBO 2008
INDIA
THEORETICAL TEST PART A
_______________________________________________________________________

13. (1 point) Absorption of a drug in the gastro-intestinal tract depends on a


number of factors. Penicillin V, whose structure is shown below, is a weak
acid (pKa = 2.7). The pH in stomach is about 2.0 and that in the intestine is
7.5. Most of the drug is absorbed in the intestine.

Choose the most likely reason for this from the following statements:
a. The molecule of drug being hydrophobic in nature passes through gastric and
intestinal membranes to a very small extent. However, because of the much
greater surface area in the intestine, the major quantity of the drug is
absorbed here.
b. The un-ionized form of the drug prevails in stomach, which slows down its
absorption. Hence, the drug gets preferentially absorbed in the intestine.
c. The ionized form of the drug prevails in the intestine which hinders/slows
down its absorption. However, owing to the large surface area available in the
intestine, the drug is absorbed mainly here.
d. Due to rapid churning movement and the low pH in the stomach, the drug is
completely broken down into smaller fragments, which are subsequently
absorbed in the intestine.

15

IBO 2008
INDIA
THEORETICAL TEST PART A
_______________________________________________________________________

PLANT SCIENCES (9 points)


14. (1 point) Which of the following will harm a dicotyledonous plant the most?
a. Removal of the central pith
b. Removal of the cork
c. Removal of the bark
d. Removal of the cork cambium

16

IBO 2008
INDIA
THEORETICAL TEST PART A
_______________________________________________________________________

15. (1 point) The transverse sections of the leaves A and B given below
represent, respectively:

a. a xerophyte and a mesophyte.


b. a xerophyte and a floating hydrophyte.
c. a floating hydrophyte and a submerged hydrophyte.
d. a submerged hydrophyte and a xerophyte.

17

IBO 2008
INDIA
THEORETICAL TEST PART A
_______________________________________________________________________

16. (1 point) Certain plant species such as Red Oak (Quercus rubra) can tolerate
severe drought over a long period of time without affecting its photosynthesis.
Which of the following adaptations is likely to contribute to this ability?

a. Stomatal closure
b. Large negative leaf water potential
c. Kranz leaf anatomy
d. Fibrous root system that increases root surface area

18

IBO 2008
INDIA
THEORETICAL TEST PART A
_______________________________________________________________________

17. (1 point) The net assimilation of CO2 of a plant is 0.5 moles when illuminated
during the day. The net consumption of O2 is 0.12 moles during the night.
Assuming that all the gas exchange is due to photosynthesis and respiration
of the biomass (equivalent molecular mass of 30), what is the net production
or consumption of biomass in grams during a complete 12 h day:12 h night
diurnal cycle?

a. 3.6 g/day
b. 7.8 g/day
c. 11.4 g/day
d. 15.0 g/day

19

IBO 2008
INDIA
THEORETICAL TEST PART A
_______________________________________________________________________

18. (1 point) Choose the figure that correctly represents the photosynthetic
efficiencies of C3 and C4 plants.
b.

a.

C4
Rate of photosynthesis
per unit leaf area

Rate of photosynthesis
per unit leaf area

C4

C3

C3

1%

50%
Sunlight

100%

1%

50%
Sunlight

100%

0C

25C

40C

0C

25C

40C

c.

d.
C3
Rate of photosynthesis
per unit leaf area

Rate of photosynthesis
per unit leaf area

C4

C3

1%

0C

50%
Sunlight
25C

100%

40C

20

C4

1%

50%
Sunlight

100%

0C

25C

40C

IBO 2008
INDIA
THEORETICAL TEST PART A
_______________________________________________________________________

19. (1 point) Plant chloroplasts are believed to have evolved from cyanobacterialike progenitors through endosymbiosis. Which of the following statements
support this hypothesis?

I. Chloroplasts and cyanobacteria share similar photosynthetic pigments and


thylakoid membranes.
II. Cyanobacteria exhibit an oxygenic photosynthesis.
III. Chloroplasts are maternally inherited.
IV. Chloroplasts have their own DNA and ribosomes.
V. Viable chloroplasts can be isolated from cells but cannot be cultured in
vitro.
VI. Prokaryotic genes express well in chloroplasts.

a. I, III, IV and V
b. I, II, IV and VI
c. I, II, III and V
d. II, IV, V and VI

21

IBO 2008
INDIA
THEORETICAL TEST PART A
_______________________________________________________________________

20. (1 point) Powdery mildew is a plant disease caused by an ectoparasitic


fungus. The fungal infection can spread to neighboring host cells in the
following ways:

R
Conidia

Conidiophore

Germination

Hypha S

Germ tube

Ascospore

Antheridia

Ascogonia

Ascus
Plasmogamy
Karyogamy

The ploidy levels of the structures Q, R and S are, respectively:


a. 2n, n, n
b. n, n, n
c. 2n, n, 2n
d. n, n, 2n

22

IBO 2008
INDIA
THEORETICAL TEST PART A
_______________________________________________________________________

21. (1 point) A few characteristics of photoautotrophs are tabulated below.


Group Light compensation point

Light saturation point

CO2 compensation

(klux units)

(klux units)

point (ppm)

13

> 80

II

12

50 80

> 40

III

0.2 0.5

5 10

> 40

IV

Data not available

12

Data not available

The four groups (I IV) represent, respectively:

a. I: C4 plants
III: Shade-loving C3 plants

b. I: Sun-loving C3 plants
III: C4 plants

c. I: C4 plants
III: Sun-loving C3 plants

d. I: C4 plants
III: Deep-sea algae

II: Sun-loving C3 plants


IV: Deep-sea algae

II: Shade-loving C3 plants


IV: Bryophytes

II: Bryophytes
IV: Shade-loving C3 plants

II: Sun-loving C3 plants


IV: Bryophytes

23

IBO 2008
INDIA
THEORETICAL TEST PART A
_______________________________________________________________________

22. (1 point) The stem of a lily plant was placed in water tinted with red ink to
monitor the movement of water through it. Two transverse sections of stems
are given below. In which of the labeled structures would you expect the red
color?
A

a. A
b. B
c. C
d. D
e. E

24

IBO 2008
INDIA
THEORETICAL TEST PART A
_______________________________________________________________________

ANIMAL SCIENCES (11 points)


23. (1 point) A few intact skeletons of birds were found during a field trip to the
Pampas in Argentina. In all the skeletons examined, the sternum lacked a
keel bone. These skeletons most likely belonged to:
a. terrestrial birds capable of short and powerful flight.
b. flightless aquatic birds.
c. insectivorous flying birds.
d. flightless terrestrial birds.

25

IBO 2008
INDIA
THEORETICAL TEST PART A
_______________________________________________________________________

24. (1 point) Which one of the following is a feature of a heterothermic


endotherm?
a. Its body temperature can vary, but it produces heat from its own tissues.
b. Its body temperature varies because it gains most of the heat from sources
outside its body.
c. Its body temperature does not vary because it produces heat from its own
tissues.
d. Its body temperature does not vary even though it gains heat from sources
outside its body.

26

IBO 2008
INDIA
THEORETICAL TEST PART A
_______________________________________________________________________

25. (1 point) Which of the following will be an advantage of breathing in air over
breathing in water?
I. As air is less dense than water, less energy is required to move air over
respiratory surfaces.
II. Oxygen diffuses faster through air than it does through water.
III. The oxygen content of air is greater than that of an equal volume of water.

a. Only I and II
b. Only I and III
c. Only II and III
d. I, II and III

27

IBO 2008
INDIA
THEORETICAL TEST PART A
_______________________________________________________________________

26. (1 point) Which characteristics would allow you to declare an organism found
on a beach as an echinoderm?
a. Radially symmetric adults with presence of spines and tube feet.
b. Radially symmetric adults with dorsal hollow notochord.
c. Exoskeleton with pharyngeal gill-slits and tube feet.
d. Radially symmetric adults with mantle cavity.

28

IBO 2008
INDIA
THEORETICAL TEST PART A
_______________________________________________________________________

27. (1 point) In an individual X, the pituitary gland was found to function normally
while the adrenal glands were atrophied. In another individual Y, both the
pituitary and adrenal glands were found to be underdeveloped. If
adrenocorticotropic hormone (ACTH) is administered to these individuals as a
remedial measure, it will be effective in:
a. individual X alone.
b. individual Y alone.
c. both X and Y.
d. neither X nor Y.

29

IBO 2008
INDIA
THEORETICAL TEST PART A
_______________________________________________________________________

28. (1 point) Which of the following are associated with stereoscopic vision?
I. Effect of the blind spot of one eye is cancelled by the other eye.
II. Total visual field of 360 and frontal visual field of 30.
III. More likely to be observed in predatory birds.
IV. Centrally situated fovea that gives good visual acuity.

a. I, II and IV
b. I, II and III
c.

II, III and IV

d. I, III and IV

30

IBO 2008
INDIA
THEORETICAL TEST PART A
_______________________________________________________________________

29. (1 point) The glycoside Phloridzin present in apple peel can block the
normal reabsorption of glucose from kidney tubules. As a result, sugar is
almost completely excreted through the urine. A mouse fed with Phloridzin
along with sodium succinate will develop:
a. hypoglycemia and no sugar will be detected in the urine sample.
b. hyperglycemia and urine test for sugar will be positive.
c. hyperglycemia and no sugar will be detected in the urine sample.
d. hypoglycemia and urine test for sugar will be positive.

31

IBO 2008
INDIA
THEORETICAL TEST PART A
_______________________________________________________________________

30. (1 point) Cardiac output is defined as the amount of blood pumped by each
ventricle. It is determined by multiplying the heart rate and the stroke volume.
The stroke volume is the amount of blood ejected by each ventricle with each
beat. If the heart of a woman beats 56 times in a minute, the volume of blood
in her heart is 120 ml at the end of diastole and 76 ml at the end of systole,
what would be her cardiac output?
a. 10.976 L/min
b. 2.464 L/min
c. 6.720 L/min
d. 4.256 L/min

32

IBO 2008
INDIA
THEORETICAL TEST PART A
_______________________________________________________________________

31. (1 point) A population is drinking water contaminated with a modified


bisphenol-A, which is not degraded in the body. As a result, there are
measurable levels of this compound in the blood. Which of the following
would result if the modified bisphenol-A were an oestrogen-mimicking
compound?
a. Males would have decreased sperm production.
b. Males would have elevated levels of follicle-stimulating hormone.
c. Females would have elevated levels of gonadotropin-releasing hormone.
d. Males would have elevated levels of blood testosterone.
e. Follicle stimulation would increase in females.

33

IBO 2008
INDIA
THEORETICAL TEST PART A
_______________________________________________________________________

32. (1 point) If a molecule of carbon dioxide released into the blood in your left
foot travels out of your nose, it must pass through all of the following
structures except the:
a. right atrium
b. pulmonary vein
c. alveolus
d. bronchus
e. pulmonary artery

34

IBO 2008
INDIA
THEORETICAL TEST PART A
_______________________________________________________________________

33. (1 point) The process of artificial kidney dialysis is shown schematically using
the following symbols:
: erythrocyte
: urea

: salts
: semi-permeable membrane
: proteins

Which of the following correctly depicts the process?

a.

b.

c.

d.

35

IBO 2008
INDIA
THEORETICAL TEST PART A
_______________________________________________________________________

GENETICS AND EVOLUTION (17 points)


34. (1 point) A mutation results in the absence of sweat glands, a disease called
anhidrotic ectodermal dysplasia. A woman suffering from this disease has a
mosaic of skin patches lacking sweat glands. The woman is likely to be:
a. homozygous for an autosomal recessive mutation.
b. heterozygous for an autosomal dominant mutation.
c. homozygous for a X-linked recessive mutation.
d. heterozygous for a X-linked recessive mutation.

36

IBO 2008
INDIA
THEORETICAL TEST PART A
_______________________________________________________________________

35. (1 point) A mink breeder allows random mating among his minks. He
discovers that, on an average, 9% of his minks have rough fur that fetches
less money when sold. So he decides to focus upon smooth fur and does not
allow minks with rough fur to mate. Rough fur is linked to an autosomal
recessive allele. What is the percentage of minks with rough fur that he will
obtain in the next generation?

a. 7.3
b. 5.3
c. 25
d. 23

37

IBO 2008
INDIA
THEORETICAL TEST PART A
_______________________________________________________________________

36. (1 point) In a breed of rabbits, multiple alleles with the following dominance
relationships control coat coloration:
C (agouti) > cch (chinchilla) > ch (Himalayan) > c (albino).
An experimental cross between agouti and Himalayan produced 50% agouti
and 50% Himalayan progeny. Which of the following crosses could produce
this result?
I. Cch X chch
II. Cc X chc
III. Cch X chc
IV. Cc X chch

a. I, II and III
b. II, III and IV
c. I, III and IV
d. I, II and IV

38

IBO 2008
INDIA
THEORETICAL TEST PART A
_______________________________________________________________________

37. (1 point) Alleles IA and IB present on chromosome 9 are responsible for blood
groups A and B, respectively. Blood group O results when these alleles are
either absent or not expressed. The alleles IA and IB are expressed only if the
H allele is present on chromosome 19, either in the homozygous or
heterozygous condition, where h stands for the recessive allele.
Gilbert belongs to the AB blood group. His sister Helen belongs to the A
group while their father belongs to the O group. Identify the maternal and
paternal genotypes.

Mother

Father

a. H/H, IA/IB

H/h, IO/IO

b. H/h, IB/IO

h/h, IA/IO

h/h, IO/IO

h/h, IA/IO

d. H/H, IA/IO

H/h, IB/IO

e. h/h, IB/IO

H/h, IO/IO

c.

39

IBO 2008
INDIA
THEORETICAL TEST PART A
_______________________________________________________________________

38. (1 point) The phenotypes of three experimental populations of plants are

Relative frequency

Relative frequency

Relative frequency

shown in the following graphs.

Height
X

Height

Height

The three populations X, Y and Z represent, respectively:


a. F1, F2 and F3 generations
b. P, F1 and F2 generations
c. F2, P and F1 generations
d. F3, F1 and F2 generations

40

IBO 2008
INDIA
THEORETICAL TEST PART A
_______________________________________________________________________

39. (1 point) In a population of mice, 40% of males showed a dominant X-linked


trait. Assuming random mating, the most frequent mating is expected
between the genotypes:
a. XBXb and XbY
b. XBXB and XbY
c. XBXb and XBY
d. XbXb and XbY

41

IBO 2008
INDIA
THEORETICAL TEST PART A
_______________________________________________________________________

40. (1 point) Hunting of Northern elephant seals reduced their population size to
as few as 20 individuals at the end of the 19th century. Their population has
since rebounded to over 30,000. But their genomes still carry the marks of
this bottleneck when compared to the population of Southern elephant seals
that was not so intensely hunted. Such bottlenecks are manifested in the form
of:
I. abundance of unique mutations.
II. increased frequency of deleterious recessive alleles.
III. reduced genetic variation.
IV. increased population size.

a. Only I and II
b. Only III
c. I, II and IV
d. II and III

42

IBO 2008
INDIA
THEORETICAL TEST PART A
_______________________________________________________________________

41. (1 point) What is true for both genetic drift and natural selection?
I. They are mechanisms of evolution.
II. They are entirely random processes.
III. They result in adaptations.
IV. They affect the genetic make-up of the population.

a. I and II
b. I and III
c. II and III
d. I and IV

43

IBO 2008
INDIA
THEORETICAL TEST PART A
_______________________________________________________________________

42. (1 point) The frequencies of two codominant alleles with similar fitness values
in a laboratory population of mice were 0.55 and 0.45. After 5 generations,
the values changed to 0.35 and 0.65, respectively. Which of the following
phenomena may be responsible for this observation?
I. Point mutation
II. Nonrandom mating
III. Genetic drift
IV. Selection pressure

a. I and IV
b. II and IV
c. I and III
d. II and III

44

IBO 2008
INDIA
THEORETICAL TEST PART A
_______________________________________________________________________

43. (1 point) In pea plants, the allele for green color of seeds (G) is dominant
over that for yellow color (g) while the allele for round seeds (R) is dominant
over that for wrinkled seed (r). The results of an experimental cross with such
garden pea plants are tabulated below:
Seed phenotype

Number

Green and round

32

Green and wrinkled

28

Yellow and round

12

Yellow and wrinkled 9


The parental genotypes are likely to be:
a. GgRr and Ggrr
b. Ggrr and GgRR
c. GgRr and GgRr
d. GgRR and ggRr

45

IBO 2008
INDIA
THEORETICAL TEST PART A
_______________________________________________________________________

44. (1 point) A population has 6 times as many heterozygous as homozygous


recessive individuals. The frequency of the recessive allele will be:
a. 1/3
b. 1/4
c. 1/2
d. 1/6

46

IBO 2008
INDIA
THEORETICAL TEST PART A
_______________________________________________________________________

45. (1 point) If you have data on genotypic frequencies for several generations of
a population and if you apply the Hardy-Weinberg equation to it, which of the
following can be deduced?
I. Whether evolution has occurred in the population.
II. The direction of evolution, if it has occurred.
III. The cause of evolution, if it has occurred.

a. Only I and II
b. Only I and III
c. Only II and III
d. I, II and III

47

IBO 2008
INDIA
THEORETICAL TEST PART A
_______________________________________________________________________

46. (1 point) The residues of mines often contain such high concentrations of
toxic metals (e.g., copper, lead) that most plants are unable to grow on them.
However, in a particular study, certain grasses were found to spread from the
surrounding uncontaminated soil onto such waste heaps. These plants
developed resistance to the toxic metals while their ability to grow on
uncontaminated soil decreased. As grasses are wind-pollinated, breeding
between the resistant and non-resistant populations went on. But eventually,
the less resistant plants growing on contaminated soil and the more resistant
plants growing on uncontaminated soil died out. This process is indicative of:

a. mass extinction of species.


b. bottleneck effect.
c. divergent evolution.
d. disruptive selection.

48

IBO 2008
INDIA
THEORETICAL TEST PART A
_______________________________________________________________________

47. (1 point) Sickle cell anemia is an autosomal recessive trait. Individual 2 in the
following pedigree is a carrier for this trait. Assuming that individuals 3 and 4
are normal homozygous, what is the probability that individual 6 will have the
disease?

2
: Male

AA

Aa

a. 1/16
b. 1/32
c. 1/64
d. 1/128

49

: Female

IBO 2008
INDIA
THEORETICAL TEST PART A
_______________________________________________________________________

48. (1 point) Note the following genotypes and corresponding phenotypes:


AB

Agouti

Abb

Albino

aaB

Black

aabb

Albino

The biochemical process that can explain the above pattern is:

a.

product of B gene

Colorless precursor

b.

product of A gene

agouti pigment

product of B gene

Colorless precursor

black pigment

product of A gene

black pigment

agouti pigment

c.
product of A gene

Colorless precursor

d.

product of B gene

agouti pigment

product of B gene

Colorless precursor

black pigment

product of A gene
agouti pigment

50

black pigment

IBO 2008
INDIA
THEORETICAL TEST PART A
_______________________________________________________________________

49. (1 point) In a population, 90% of the alleles at the Rh locus are R. Another
alternative form of this allele is r. Forty children from this population go to a
particular play school. The probability that all are Rh positive is:
a. 400.81
b. 0.9940
c. 400.75
d. 1-0.8140

51

IBO 2008
INDIA
THEORETICAL TEST PART A
_______________________________________________________________________

50. (1 point) Study the pedigree and answer the following question.

3
1

: Male

: Female

: Dizygotic twins

The genetic relatedness between individuals 1 and 2 and between individuals


5 and 6, respectively, is:

a. 0.5 and 0.25


b. 0.25 and 0.5
c. 1.0 and 0.5
d. 1.0 and 0.25

52

IBO 2008
INDIA
THEORETICAL TEST PART A
_______________________________________________________________________

ECOLOGY (7 points)
51. (1 point) A typical biomass pyramid is represented in the figure below.

D
E

C
B
A

If A represents a primary producer, then E is likely to be a:


a. photo-litho-heterotroph.
b. chemo-organo-heterotroph.
c. chemo-litho-autotroph.
d. photo-organo-heterotroph.

53

IBO 2008
INDIA
THEORETICAL TEST PART A
_______________________________________________________________________

52. (1 point) Comparative sensitivity of three groups of organisms to single large


doses of x-or -rays delivered at short intervals is shown in the figure below.

P
Q
R

102

103

104
105
Dose in rads

The three groups P, Q, R respectively are:


a. insects, mammals and bacteria
b. mammals, bacteria and insects
c.

bacteria, mammals and insects

d. mammals, insects and bacteria

54

106

IBO 2008
INDIA
THEORETICAL TEST PART A
_______________________________________________________________________

53. (1 point) Hay is boiled in water and cooled. Some pond water, containing only
heterotrophic protozoa, is added to it and kept in the dark for a long time.
Which of the following are true?
I. Heterotrophic succession of protozoa will occur with increase in total
biomass.
II. The energy of the system is maximum at the beginning.
III. Succession will occur, eventually reaching a steady state in which energy
flow is maintained.
IV. The ecosystem may undergo succession but finally all organisms will die
or go into resting stages.

a. I and III
b. II and IV
c. II and III
d. I and IV

55

IBO 2008
INDIA
THEORETICAL TEST PART A
_______________________________________________________________________

54. (1 point) An ecologist is comparing the growth of a herbaceous plant species


growing in two different sites A and B. To compare the populations from the
two sites, she has harvested 30 individuals from each site, then measured the
root length, root biomass, and shoot biomass of each individual. A summary
of those measurements are as follows:

Location

Mean root length

Mean root

Mean shoot

(cm)

biomass (g)

biomass (g)

Site A

27.2 + 0.2

348.7 + 0.5

680.7 + 0.1

Site B

13.4 + 0.3

322.4 + 0.6

708.9 + 0.2

Based on the data presented, which of the following statements is likely to be


true?
a. Soil water availability is lower in Site B than in Site A.
b. Plant productivity is higher in Site A than in Site B.
c. Soil water availability is lower in Site A than in Site B.
d. Soil nutrient availability is lower in Site B than in Site A.

56

IBO 2008
INDIA
THEORETICAL TEST PART A
_______________________________________________________________________

55. (1 point) In an aquatic ecosystem, the total dry biomass of each of three
groups of organisms is as follows:
I. Ciliates: 1.1041 g
II. Midge larvae: 0.9845 g
III. Oligochaetes: 1.005 g
The correct food chain that they represent is:

a. I II III
b. II I III
c. I III II
d. III II I
e. II III I

57

IBO 2008
INDIA
THEORETICAL TEST PART A
_______________________________________________________________________

56. (1 point) The reproductive effort of a plant is defined as the ratio of the dry
weight of its reproductive organs to that of its above-ground tissues. The
reproductive effort of two plant species M and N, as compared to their leaf

Reproductive effort

biomass is plotted in the graph below.

0.5
M

0.1
0

0.1
0.5
Leaf biomass / total biomass

Choose the correct interpretation.


a. M is a r-strategist adapted to a highly disturbed habitat.
b. N is a k-strategist adapted to a highly disturbed habitat.
c. N is a r-strategist growing under favorable environmental conditions.
d. M is a k-strategist growing under favorable environmental conditions.

58

IBO 2008
INDIA
THEORETICAL TEST PART A
_______________________________________________________________________

57. (1 point) Prey-predator relationships are often considered analogous to a lifedinner relationship in behavioral ecology. Which of the following statements
best describe this analogy and the relative evolutionary rates of the prey and
predator species in a population?
I. This analogy indicates the fact that the prey species serves as the dinner
for the predator species, the life of which depends on the former.
II. This analogy indicates that a prey species caught by a predator loses its
life while a predator that fails to catch a prey only loses a dinner.
III. The prey species is usually under greater selection pressure from its
predators and tends to evolve faster than does a predator species.
IV. The predator species is usually under greater selection pressure because
of its dependence on a prey species for food and tends to evolve faster
than does a prey species.

a. I and III
b. I and IV
c. II and III
d. II and IV

59

IBO 2008
INDIA
THEORETICAL TEST PART A
_______________________________________________________________________

ETHOLOGY (4 points)
58. (1 point) Animals can use their circadian clocks to determine direction from
the position of the sun. In a particular experiment conducted in Iceland, a bird,
kept in a cage open to the sky, was trained to seek food on the western side.
Its circadian rhythm was then phase-delayed by 6 hours and after phase
shifting, the bird was returned to its open cage at 12.00 noon real time. It was
observed to seek food in the:
a. north.
b. south.
c. east.
d. west.

60

IBO 2008
INDIA
THEORETICAL TEST PART A
_______________________________________________________________________

59. (1 point) Coho Salmon is a fish found in the freshwater streams of North
America. The males of this species have two reproductive strategies to
fertilize the eggs laid by females. Larger males are able to fight with each
other successfully but smaller males are unable to do so. The latter adopt
another strategy, that of sneaking, in which they hide behind rocks and
quickly approach females to fertilize the eggs before the larger males are able
to do so. Which of the following graphs depicts the correct strategies?

a.

Fighting

Sneaking

Proximity to female

Male body size

b.

Proximity to female

c.

Sneaking

Fighting

Male body size

Sneaking

Fighting

Proximity to female

Male body size

d.

Proximity to female

Fighting

Sneaking

Male body size

61

IBO 2008
INDIA
THEORETICAL TEST PART A
_______________________________________________________________________

60. (1 point) Young laughing gull chicks peck at the tip of the parents beak which,
in turn, induces the adult gull to regurgitate food. Experiments were
conducted with one-day old and three-day old chicks, the latter being reared
with their parents. These chicks were presented with the following models of
the parent head and the following responses were obtained:

Model

Mean number of pecks / 30 sec


0
5
10
15
One-day old

Parental
Experienced

II

III

IV

62

IBO 2008
INDIA
THEORETICAL TEST PART A
_______________________________________________________________________

Choose the correct interpretation of the experiment.

a. Pecking behavior is a fixed action pattern where any long pointed object acts
as an equally effective stimulus.
b. The pecking rate of laughing gull chicks increases with age.
c. The response of one-day old chicks is more pronounced when the model is
closer to that of the parent.
d. Act of pecking is an innate behavior while the discriminatory capacity of the
chicks is a result of learning.

63

IBO 2008
INDIA
THEORETICAL TEST PART A
_______________________________________________________________________

61. (1 point) While studying the frogs of a certain species in their natural habitat
during the mating season, you observe a chorus of male frogs in which some
individuals are calling while others remain silent. On further observation, you
see the silent frogs are sitting closer to those that are calling.
Which of the following is most likely to explain the behavior of this chorus of
frogs?
a. The individuals who are not calling are staggering their calls with those of the
others and are likely to call later in the season after the latter have finished
mating.
b. The silent frogs are close genetic relatives of the calling individuals and do
not expend valuable energy in calling as the matings that the latter will
receive would provide adequate indirect fitness to them.
c. The silent frogs have evaluated that their calls are inadequate in attracting
females, as compared to those of the calling individuals, and lie in wait to
sneak matings with the females that approach the calling males.
d. The silent frogs do not expend energy in themselves calling as the female
frogs that are attracted to the calls of the others are anyway likely to visually
inspect the closely-spaced males and then choose their mating partners.

64

IBO 2008
INDIA
THEORETICAL TEST PART A
_______________________________________________________________________

BIOSYSTEMATICS (2 points)
62. (1 point) Although Echidna lays eggs, it has been classified as a mammal
due to the presence of mammary glands. Which of the following additional
features support its inclusion in the class Mammalia?

Echidna

I. Hair over parts of the body.


II. Presence of pituitary and thyroid gland.
III. Complete separation of pulmonary and systemic circulation in a 4 chambered heart.
IV. A diaphragm separating thoracic and abdominal cavities.
V. Regulation of body temperature irrespective of ambient temperature.
VI. Enucleated red blood cells.

a. III and VI
b. I, IV and V
c. Only I and IV
d. I and II
e. I, IV and VI

65

IBO 2008
INDIA
THEORETICAL TEST PART A
_______________________________________________________________________

63. (1 point) Study the adjoining schematically drawn evolutionary lineage. The

Mammalia

Aves

Reptilia

Amphibia

Pisces

Agnatha

Protochordata

Invertebrates

derived characters A, B and C represent, respectively:

TIME

a. vertebral column and cranium, jaw, lung.


b. tail, heart, teeth.
c. heart, gill, cranium.
d. cranium, cloaca, hepatic portal system.

*************** END OF PART A ***************

66















All IBO examination questions are published under the following Creative Commons license:



CC BY-NC-SA (Attribution-NonCommercial-ShareAlike) https://creativecommons.org/licenses/by-nc-sa/4.0/
The exam papers can be used freely for educational purposes as long as IBO is credited and
new creations are licensed under identical terms. No commercial use is allowed.

IBO Answers India Paper 1 2008


Part A
Question
1
2
3
4
5
6
7
8
9
10
11
12
13
14
15
16
17
18
19
20
21

Answer
B
C
B
D
D
B
C
C
C
A
B
B
C
C
A
B
C
A
B
B
A

Question
22
23
24
25
26
27
28
29
30
31
32
33
34
35
36
37
38
39
40
41
42
= OMITTED

Answer
D
D
A
D
A
D
D
D
B
A
B
C
D
B
C
B
C
A
B
D
D

Question
43
44
45
46
47
48
49
50
51
52
53
54
55
56
57
58
59
60
61
62
63

Answer
A
B
A
D
C
B
B
A
B
- *
B
C
C
A
-*
A
D
D
C
E
A















All IBO examination questions are published under the following Creative Commons license:



CC BY-NC-SA (Attribution-NonCommercial-ShareAlike) https://creativecommons.org/licenses/by-nc-sa/4.0/
The exam papers can be used freely for educational purposes as long as IBO is credited and
new creations are licensed under identical terms. No commercial use is allowed.

IBO 2008
INDIA
THEORETICAL TEST PART B
_______________________________________________________________________

Student Code: ____________

Country:
___________________

19th INTERNATIONAL BIOLOGY OLYMPIAD


13th 20th July, 2008
Mumbai, INDIA

THEORETICAL TEST PART B

Write all answers in the ANSWER SHEET.

IBO 2008
INDIA
THEORETICAL TEST PART B
_______________________________________________________________________

Dear Participants

You have a total of 2 hours 30 minutes for answering Part B.

The questions in Part B may have more than one correct answer. Fill your
answers in the Answer Sheet for Part B. The marks for the questions in Part
B vary depending on the number of answers and the complexity of the
question. These marks have been indicated along with the question.

Mark your answers clearly. Avoid any corrections in the Answer Sheet.

NOTE: Some of the questions may be marked Skipped / Deleted. DO NOT


attempt these questions. Also, read the question completely before
attempting it as some questions may continue from one page to the next.

The maximum number of points is 120.5

Your Answer Sheets will be collected at the end of the examination.


Good Luck!!

Country: ___________________________________
First name: _________________________________
Middle name: _______________________________
Family name: ________________________________
Student Code: _______________________________

IBO 2008
INDIA
THEORETICAL TEST PART B
_______________________________________________________________________

CELL BIOLOGY (26 points)


In the following questions use a value of 3 for , 6 x 1023 for Avogadros
number and 660 for the molecular weight of 1 bp of DNA. The volume of a
sphere of radius r is 4/3r3. 10.5bp of linear DNA has a length of 3.4nm.
All calculations should be done on the question booklet.
1. (2+1+1=5 points) A bacterium has a single copy of a 4 x106 bp circular
genomic DNA.

a. If the diameter of this spherical cell is 1 m, what would be the molar


concentration of DNA in this cell?

Answer: _________________Molar

b. If the DNA assumed a conformation as proposed by Watson and Crick, what


would be the linear length of the bacterial DNA?

Answer: _________________metre

c. How many bacterial cells one should take to get 1 mg of DNA?

Answer: ____________________

IBO 2008
INDIA
THEORETICAL TEST PART B
_______________________________________________________________________

2. (3 points) Smooth endoplasmic reticulum (SER) is mainly concerned with the


following functions:
I. Lipid synthesis
II. Drug detoxification
III. Ca++ storage
IV. Gluconeogenesis
Fill in the following table with a tick mark () wherever appropriate and
indicate the function/s of SER wherever it is extensively present, by choosing
from options I IV above.

Organ/Cell

SER

SER not

Function/s (if

extensively

extensively

extensively

present

present

present)

a. Adrenal gland

b. Sebaceous glands

c. Intestinal villi

d. Muscles

e. Liver

f.

Pancreas

IBO 2008
INDIA
THEORETICAL TEST PART B
_______________________________________________________________________

3. (2 points) There are various mechanisms by which a cell can commit suicide
a phenomenon known as apoptosis. One of the mechanisms is triggered
by reactive oxygen species. The outer membrane of mitochondria normally
expresses a protein Bcl-2 on its surface. Another protein Apaf-1 binds Bcl-2.
Reactive oxygen species cause Bcl-2 to release Apaf-1 and a third protein
Bax to penetrate the mitochondrial membrane, releasing cytochrome c. The
released cytochrome c forms a complex with Apaf-1 and caspase 9. This
complex sequentially activates many proteases that digest cellular proteins.
Finally, the cell is phagocytosed.

What will be the fate of a cell exposed to reactive oxygen species in the
following situations? (Choose from options on the next page)

Situation I: The cell receives a signal for inhibition of expression of Apaf-1


protein. ______

Situation II: The cell expresses low-affinity Bcl-2 proteins. _______

Situation III: A competitive inhibitor of Apaf-1 for Bcl-2 binding is added to the
cell in excess quantity. _______

Situation IV: A chemical which significantly lowers the ratio of Bax to Bcl-2 is
added to the cell. ________

IBO 2008
INDIA
THEORETICAL TEST PART B
_______________________________________________________________________

Choose from the following options:


A. The cell resists apoptosis.
B. The cell is forced towards apoptosis.
C. The fate of the cell cannot be predicted.

IBO 2008
INDIA
THEORETICAL TEST PART B
_______________________________________________________________________

4. (3 points) The stoichiometry of aerobic and anaerobic degradation


of glucose by yeast are as follows:

C6H12O6 + 6O2 6CO2 + 6H2O


C6H12O6 2C2H5OH + 2CO2

In an experiment, the complete utilization of 0.5 mol of glucose, partly under


aerobic and partly under anaerobic conditions, yielded 1.8 mol of CO 2.

a. Calculate the fraction of glucose that is utilized aerobically.

Answer: _________________%

b. Calculate the Respiratory Quotient, which is defined as the molar ratio of the
CO2 produced to the O2 utilized.

Answer: __________________

IBO 2008
INDIA
THEORETICAL TEST PART B
_______________________________________________________________________

5. (2.5 points) In order to study the effect of a hormone on the breakdown of a


polysaccharide in liver tissue, fresh liver was homogenized in an isotonic
buffer system. Part of this homogenate was centrifuged to obtain a clear
supernatant and a pellet.

The following experiments were then conducted.

Experiment

Reaction mixture

Result
Quantity of Activity of
enzyme

enzyme

Liver homogenate

++++

II

Liver homogenate + hormone

++++

++++

III

Supernatant + hormone

++++

IV

Pellet + hormone

Supernatant + small quantity

++++

++++

++++

++++

++++

of reaction mixture from


Experiment IV

VI

Supernatant + small quantity


of heated reaction mixture from
Experiment IV

VII

Supernatant + small quantity


of heated pellet + hormone

IBO 2008
INDIA
THEORETICAL TEST PART B
_______________________________________________________________________

Complete the signal transduction pathway for the breakdown of the


polysaccharide in the following schematic.

Precursor

Polysaccharide

Monosaccharide

Options:
A. Membrane-bound protein
B. Heat-stable molecule
C. Inactive enzyme
D. Active cytosolic enzyme
E. Hormone
F. Organic inhibitor
G. Heat shock protein
Fill in the appropriate letters in the table from the options given.
1

IBO 2008
INDIA
THEORETICAL TEST PART B
_______________________________________________________________________

6. (4 points) Sugars and fatty acids are important biomolecules that provide
energy to the majority of living systems. The estimated utilization of palmitic
acid and glucose in the human body is shown in the equation below:

129 Pi + 129 ADP 129 ATP


(A) C15H31COOH + 23 O2

16 CO2 + 16 H2O

38 Pi + 38 ADP

38 ATP

(B) C H O + 6 O
6 12 6
2

6 CO2 + 6 H2O

Answer the following questions:


(Atomic weights of H: 1, C: 12 and O: 16)

I. ATP yield (in moles) per mole of oxygen in Reaction A :__________

II. ATP yield (in moles) per mole of oxygen in Reaction B:__________

III. ATP yield (in moles) per gram of fuel in Reaction A: ____________

IV. ATP yield (in moles) per gram of fuel in Reaction B:____________

V. Based on the above reactions, state whether the following statements are
true or false by putting tick marks () in the appropriate boxes.

10

IBO 2008
INDIA
THEORETICAL TEST PART B
_______________________________________________________________________

Statements:
a. Under conditions of mild-intensity exercise and abundance of oxygen, the
Respiratory Quotient tends to be < 1.
b. High-intensity exercise is primarily fuelled by fat when oxygen
concentration is limiting.
c. Reaction A represents the energy-acquiring process of nervous tissue
while Reaction B is more common in skeletal muscles involved in rapid
movement.
d. Under conditions of hypoxia, the shift of tissue metabolism from fatty acid
oxidation to glucose oxidation will yield more ATP.

True False
a.
b.
c.
d.

11

IBO 2008
INDIA
THEORETICAL TEST PART B
_______________________________________________________________________

7. (1+1+2 = 4 points) Leena is a molecular biology student. She purifies two


fragments of DNA, 800 and 300 base pairs long. These were obtained from a
plasmid after digesting it with HindIII. Each of these fragments has a single
EcoRI recognition site.
Leena wants to join these two fragments to get a 1.1kb gene as shown in
Figure 7.1. She suspects that this gene has a unique protein-coding
sequence.

HindIII

HindIII

HindIII
300bp
200bp

EcoRI

800bp
200bp

700bp

EcoRI

Figure 7.1

She, therefore, mixes the two fragments in the presence of excess DNA
ligase in an appropriate buffer and incubates the mixture. She removes an
aliquot (a small part of the reaction mixture) after 30 minutes and loads it on
an agarose gel to check the results. She is surprised to find many bands
along with the expected 1.1kb band (as shown in the figure 7.2, next page) in
the gel.

12

IBO 2008
INDIA
THEORETICAL TEST PART B
_______________________________________________________________________

1.9kb

1.1kb
0.8kb
0.6kb
0.3kb

Figure 7.2

I. Which one of the following statements can explain this result?


a. The two fragments used for ligation were not sufficiently purified.
b. The multiple bands on the gel are due to the degradation of DNA in the
reaction mixture.
c. The observed band pattern is a result of ligation of randomly-selected
fragments.
d. DNA ligase did not function, and hence, it led to the random catenation of the
DNA molecules.
Put a tick mark () in the appropriate box.
a.

b.

13

c.

d.

IBO 2008
INDIA
THEORETICAL TEST PART B
_______________________________________________________________________

II. If another aliquot of the reaction mixture is removed after 8 hours, which one
of the following would be expected?
a. Prominent bands of high molecular weight.
b. Prominent bands of low molecular weight.
c. Large number of molecules of varying lengths leading to a smearing on the
gel.
d. The gel pattern would remain the same. Only the intensity of bands would
increase.

Put a tick mark () in the appropriate box.

a.

b.

14

c.

d.

IBO 2008
INDIA
THEORETICAL TEST PART B
_______________________________________________________________________

III. Leena is interested in the 1.1kb fragment shown in Figure 7.1. Hence, she
elutes the 1.1kb fragment from the gel shown in Figure 7.2 and subjects part
of this sample to HindIII digestion. She obtains the expected pattern with two
bands, 800 and 300 base pairs long. To confirm its restriction map, she
subjects the remaining sample to complete EcoRI digestion. Which pattern of
bands would she obtain?
2 kb

2 kb

2 kb

2 kb

1 kb

1 kb

1 kb

1 kb

0.9

0.9

0.9

0.9

0.7

0.7

0.7

0.7

0.5

0.5

0.5

0.5

0.3

0.3

0.3

0.3

0.1

0.1

0.1

0.1

Put a tick mark () in the appropriate box.


a.

b.

15

c.

d.

IBO 2008
INDIA
THEORETICAL TEST PART B
_______________________________________________________________________

8. (2.5 points) Protein function can be regulated at many levels. By interpreting


the graphs below, find out how each of these proteins (A to D) is regulated.
They are all enzymes involved in the same physiological process, their
activity is induced by the same treatment and their respective activities in a
sample can be measured with specific assays. The arrows indicate the
beginning of the activating treatment. See options on next page.

16

IBO 2008
INDIA
THEORETICAL TEST PART B
_______________________________________________________________________

Match the proteins A to D with their mode(s) of regulation (I to IV) by putting


tick marks () in the appropriate boxes.
I. Post-translational modification, but not phosphorylation
II. Transcriptional regulation
III. Proteasomal degradation and rapid turnover
IV. Phosphorylation

Protein
I

Mode of regulation
II
III
IV

A
B
C
D

17

IBO 2008
INDIA
THEORETICAL TEST PART B
_______________________________________________________________________

PLANT SCIENCES (15 points)


9. (4 points) Study the schematics of the plant tissues/cells shown below and fill
in the blank column with appropriate letter/s.

No.
I

Answer
Cell/s that is/are not alive when
functional.

II

Plasmodesmata can be found


associated with this/these cell/s.

III

When you eat potato, you eat


the tissue formed of this/these
cell/s.

IV

Cell/s that harden/s the nut skin.

18

IBO 2008
INDIA
THEORETICAL TEST PART B
_______________________________________________________________________

10. (1.5 points) On the basis of the photoperiod required for flowering, plants can
be described as:
I. Short-day plants
II. Long-day plants
III. Day-length indifferent plants
The effect of varying light periods on flowering in these three types of plants is
depicted in the graphs below, where TM, trophic minimum, is the minimum
light that is required to produce the organic matter indispensable to its

Number of days before flowering

TM CP
24
Hours of light/day
Number of days before flowering

Number of days before flowering

metabolism and CP is the critical period for flowering.

TM CP
24
Hours of light/day

19

24
(TM = CP)
Hours of light/day

IBO 2008
INDIA
THEORETICAL TEST PART B
_______________________________________________________________________

Choose the plant type (I, II or III) for each of the three graphs and fill in the
table.
Graph Plant type
A
B
C

20

IBO 2008
INDIA
THEORETICAL TEST PART B
_______________________________________________________________________

11. (2 points) (A) A mesophyte was planted in soil with high salt concentration
and watered. It showed wilting. Assign the appropriate values of water
potentials to the regions marked P, Q, and R in the schematic representation
of this plant.

Choose from the options given below and fill in the table:

-1 atm

-5 atm

-8 atm
Region Water potential

______ atm

______ atm

______ atm

21

IBO 2008
INDIA
THEORETICAL TEST PART B
_______________________________________________________________________

(B) Which of the following remedial measures will completely reverse wilting in
this plant? Put a tick mark () in the appropriate box.

a. Increasing environmental humidity.


b. Irrigation to flush out the excess salts.
c. Applying wax on the surface of leaves.
d. Placing the plant in shade.

a.

b.

22

c.

d.

IBO 2008
INDIA
THEORETICAL TEST PART B
_______________________________________________________________________

12. (4 points) A few characteristics of some organisms are listed in the table. Put
a tick mark () against the appropriate organisms.

Chlamydomonas Cyanobacteria

Phototrophic autotrophs

Photosystem II absent

Respiratory enzymes located on


plasma membrane

Chlorophyll a as the major


photosynthetic pigment

23

Green-

Purple-

sulphur

sulphur

bacteria

bacteria

IBO 2008
INDIA
THEORETICAL TEST PART B
_______________________________________________________________________

13. (3.5 points) The total respiration (R) of a young growing plant is described by
the following expression:
R = 0.27 P + 0.015 W,
where P is the total amount of glucose produced per day and W is the
average mass of the plant.

Of the processes listed below, some influence the factor 0.27 of the above
equation whereas the others do not.

1. Movement of water within the cells


2. Reduction of nitrate (NO3-) ions to ammonium (NH4+) ions
3. Uptake of K+ ions through the plasma membrane of endodermal cells
4. Uptake of CO2 in cells of palisade parenchyma
5. Opening and closing of stomata
6. Lengthening of a polypeptide chain
7. Absorption of light by chlorophyll a

24

IBO 2008
INDIA
THEORETICAL TEST PART B
_______________________________________________________________________

Indicate with a tick mark ( ) in the appropriate column in the table below,
which of these processes do or do not affect the factor 0.27.

Process

Does
affect

1
2
3
4
5
6
7

25

Does not
affect

IBO 2008
INDIA
THEORETICAL TEST PART B
_______________________________________________________________________

ANIMAL SCIENCES (18 points)


14. (2 points) The tidal volume is defined as the volume of air entering the lungs
in a single inspiration (inhalation), which is approximately equal to the volume
exhaled during subsequent expiration (exhalation) on normal quiet breathing.
Exchange of gases with the blood occurs in the alveoli of lungs. In the
conducting airways (e.g. trachea), which also contain a volume of air, no
exchange takes place. The space within these airways is called the anatomic
dead space. Thus the volume of fresh air entering the alveoli during each
inspiration equals the tidal volume minus the volume of air in the anatomic
dead space. The total volume of fresh air entering the alveoli per minute is
called the alveolar ventilation and is expressed in ml/min; it varies directly with
the respiration rate.

Consider the hypothetical breathing patterns of three individuals A, B and C:


Individual

Tidal volume

Frequency

Anatomic dead space

(ml/breath)

(breaths/min)

(ml/breath)

800

12

600

500

16

350

600

12

200

Which of the following holds true about the alveolar ventilation of these three
individuals? See options on next page.

26

IBO 2008
INDIA
THEORETICAL TEST PART B
_______________________________________________________________________

a. B has considerably greater alveolar ventilation than C.


b. A has considerably greater alveolar ventilation than C.
c. C has considerably greater alveolar ventilation than B.
d. A has considerably greater alveolar ventilation than B.

Put a tick mark () for the correct statement(s) in the appropriate box of the
table.
a.

b.

27

c.

d.

IBO 2008
INDIA
THEORETICAL TEST PART B
_______________________________________________________________________

15. (2 points) The relative growth rates of four organs of the human body are
shown in the following graphs.
Q
% of mass in
adults

% of mass in
adults

P
100

100

10
20
Age (years)

10
20
Age (years)
S

% of mass in
adults

% of mass in
adults

R
100

100

10
20
Age (years)

10
20
Age (years)

Match the graphs with the organs by putting a tick mark () in the appropriate
box of the table.

P
Liver
Brain
Thymus
Gonads

28

IBO 2008
INDIA
THEORETICAL TEST PART B
_______________________________________________________________________

16. (2 points) A few statements regarding the respiratory processes in vertebrates


are given below:
a. Amphibians use negative pressure to force air into the lungs.
b. Reptiles, birds, and mammals use positive pressure to force air into the
lungs.
c. Lungs of amphibians and mammals are incompletely ventilated during
each breathing cycle.
d. Lungs of birds are completely ventilated during each breathing cycle.
Mark whether each statement is true or false by putting a tick mark () in the
appropriate box of the table.

True False
a.
b.
c.
d.

29

IBO 2008
INDIA
THEORETICAL TEST PART B
_______________________________________________________________________

17. (2 points) The oxygen saturation of hemoglobin when plotted versus pO 2 is


sigmoid and this is conventionally referred to as the oxygen dissociation
curve. Many parameters such as pH, pCO2, temperature, and metabolic
activity of the cell affect the oxygen dissociation curve.

% saturation of
hemoglobin

Two such curves, I and II, are depicted in the following graph

I
II

pO2 (mm of Hg)


Determine whether the curves could represent the sets of conditions given
below. Put a tick mark () in the appropriate boxes in the table.
Set

Condition

Curve I. Normal blood pH and Curve II. Acidosis

Curve I. 40C and Curve II. 30C

Curve I. Elephant hemoglobin and Curve II. Cat hemoglobin

Curve I. Fetal hemoglobin and Curve II. Maternal hemoglobin

30

True False

IBO 2008
INDIA
THEORETICAL TEST PART B
_______________________________________________________________________

18. (2 points) Given below are the data on breathing rate, heart rate and body
temperature of four different mammals A, B, C, and D.

Animals

Breathing rate
(inhalations/min)

Heart rate
(beats/min)

Body temperature
(C)

160

500

36.5

15

40

37.2

28

190

38.2

28

35.9

Study the data and rank these animals in descending order of surface area per
unit volume as well as the total volume of blood by filling in the boxes with
appropriate letters (A to D).

Surface area per unit volume of the body

Total volume of blood in the body

31

IBO 2008
INDIA
THEORETICAL TEST PART B
_______________________________________________________________________

19. (5 points) In order to determine the nature of factors involved in humoral


immunity, three groups of mice were immunized according to the scheme
below:
Immunization scheme
1. Mice

Isolate serum (S1) after 2 weeks

2. Mice

Immunized with pathogen P

Isolate serum (S2) after 2 weeks

3. Mice

Immunized with pathogen Q

Isolate serum (S3) after 2 weeks

Using sera from the above immunization schemes, the following experiments
were conducted to test the response of these sera towards pathogens P or Q:
Number

Experiment

Serum S1 Add pathogen P or Q No lysis of pathogen P or Q

II

Serum S2 Add pathogen P Lysis of pathogen P

III

Serum S3 Add pathogen Q Lysis of pathogen Q

IV

Serum S2 Add pathogen Q No lysis of pathogen Q

Serum S3 Add pathogen P No lysis of pathogen P

VI

Serum S2 Heat at 55C for 30 min Add pathogen P No lysis of


pathogen P

VII

Serum S3 Heat at 55C for 30 min Add pathogen Q No lysis of


pathogen Q

VIII

Serum S2 Heat at 55C for 30 min Add serum S1 Add pathogen


P Lysis of pathogen P

IX

Serum S2 Heat at 55C for 30 min Add serum S1 heated at 55C


for 30 min Add pathogen P No lysis of pathogen P

Serum S2 Heat at 55C for 30 min Add serum S3 Add pathogen


P Lysis of pathogen P

32

IBO 2008
INDIA
THEORETICAL TEST PART B
_______________________________________________________________________

Answer the following questions:


(A) If serum S3 is heated at 55C for 30 min, and mixed with serum S1, which of
the following pathogen would it lyse?
a. Only P
b. Only Q
c. P and Q both
d. Neither P nor Q
Put a tick mark () in the appropriate box.

a.

b.

c.

d.

(B) If serum S2 is heated at 55C for 30 min, and mixed with serum S3, which of
the following pathogen would it lyse?
a. Only P
b. Only Q
c. P and Q both
d. Neither P nor Q
Put a tick mark () in the appropriate box.
a.

b.

33

c.

d.

IBO 2008
INDIA
THEORETICAL TEST PART B
_______________________________________________________________________

(C) Which of the following statements are TRUE or FALSE for the above
experiment?
a. The lysis of pathogen requires only one component, which is heat-labile.
b. The lysis of pathogens requires at least two components. One component is
induced by the pathogen, while the other is non-inducible and is pathogen
non-specific.
c. The pathogen-induced component is heat-labile whereas the non-specific
component is heat-stable.
d. The pathogen-induced component is heat-stable whereas the non-specific
component is heat-labile.
e. The pathogen-specific components cannot function if present together.
f. The non-specific component has to be derived from the same mice in which
the pathogen-specific component would be induced.

Put a tick mark () in the appropriate boxes.


Options

True

a.
b.
c.
d.
e.
f.

34

False

IBO 2008
INDIA
THEORETICAL TEST PART B
_______________________________________________________________________

20. (3.5 points) In air-breathing animals, bicarbonate ions present in the blood
play an important role of buffering. Various equilibria that occur in lungs and
plasma are shown below.

Plasma
H+ + HCO3

Lungs

H2CO3

CO2
(dissolved)

CO2
(gaseous)

Indicate the events that will occur in sequence as a result of following


activities by filling in the boxes with the appropriate numbers I to VI of the
options given on the next page:

A. A person is hyperventilating as a result of rapid breathing.

B. A person continues vigorous exercise:

35

IBO 2008
INDIA
THEORETICAL TEST PART B
_______________________________________________________________________

Options:
I. Decrease in plasma carbon dioxide concentration
II. Decrease in blood bicarbonates
III. Acidosis
IV. Increase in blood bicarbonates
V. Increase in exhalation of carbon dioxide
VI. Alkalosis

36

IBO 2008
INDIA
THEORETICAL TEST PART B
_______________________________________________________________________

GENETICS AND EVOLUTION (20.5 points)


21. (2 points) Cystic fibrosis is an autosomal recessive trait. If parents who are
both carriers for this gene have 3 children, what is the probability that exactly
two will be phenotypically normal?

Answer: _________

37

IBO 2008
INDIA
THEORETICAL TEST PART B
_______________________________________________________________________

22. (2 points) E. coli cells were grown in a medium containing glucose and
lactose, and a growth curve was obtained which is shown below.

Fill in the table using tick marks () to indicate which of the listed events
would predominate during the three phases of growth (I to III).

I
Lactose hydrolysis by -galactosidase
Reduction of lac repressors affinity for the lac operator

Binding of the CAP-cAMP complex to the lac promoter

Utilization of glucose

38

II

III

IBO 2008
INDIA
THEORETICAL TEST PART B
_______________________________________________________________________

23. (2 points) In a small tribal population, the frequencies of two alleles A and a
at a particular locus were 0.3 and 0.7, respectively. However, not all the
individuals with genotype aa could live up to the reproductive age and the
relative fitness of this genotype was found to be 0.5. The remaining
genotypes had a relative fitness of 1.

What is the expected percentage of heterozygotes among newborns in the


next generation?

Answer: ______________%

39

IBO 2008
INDIA
THEORETICAL TEST PART B
_______________________________________________________________________

24. (2 points) In the following pedigree, the probability that the individual marked
as

will be affected is:

Normal female

Normal male

Affected female

Affected male

Answer: _______________

40

IBO 2008
INDIA
THEORETICAL TEST PART B
_______________________________________________________________________

25. (2 points) If two heterozygotes (Aa) mate, what is the minimum number of
offspring they should have such that the probability of at least one offspring
having genotype aa is greater than 90%?

Answer: _______________

41

IBO 2008
INDIA
THEORETICAL TEST PART B
_______________________________________________________________________

26. (2 points) A celebrity has been named in a paternity suit. The defendant
(labeled D in the autoradiogram), the mother (labeled M), and the baby
(labeled B) have each been typed for two loci VNTR1 and VNTR2, as shown
in the autoradiograms below. Each of these VNTR loci has four alleles. For
VNTR1, the frequencies of the alleles 1, 2, 3, and 4 in the general population
are 0.2, 0.4, 0.3, and 0.1, respectively. For VNTR2, the frequencies of alleles
1, 2, 3, and 4 are 0.1, 0.1, 0.2, and 0.6, respectively.

VNTR 1
M

VNTR 2
M

a. Do the autoradiograms indicate that D could be the father of the baby B?


Put a tick mark () in the appropriate box.

Yes No

b. What is the probability that another male in the general population could be
the father of the baby B?

Answer: _________________

42

IBO 2008
INDIA
THEORETICAL TEST PART B
_______________________________________________________________________

27. (2 points) In some populations, inbreeding takes place amongst first cousins.
Inbreeding leads to a reduction in the frequency of heterozygotes and is
measured as the inbreeding co-efficient, F, where

heterozygo tes
heterozygo tes
f observed
f expected
1 F
The symbol f denotes frequency.
If F = 1 (complete inbreeding), the population consists entirely of
homozygotes.

In a population of 150 individuals, the observed numbers of MN blood group


genotypes are: 60 MM, 36 MN, 54 NN.

a. Calculate F.
Answer: __________

b. If, for another population of the same species, the allelic frequencies remain
the same but the value of F is half of that calculated in a, what will be the
frequency of the heterozygotes (MN) observed in this group?
Answer: ____________

43

IBO 2008
INDIA
THEORETICAL TEST PART B
_______________________________________________________________________

28. (2 points) The transmission pattern of a disease caused by an autosomal


dominant gene is shown in the following pedigree:

Generation
I

II

III

Each family member has been typed for a seven-allele microsatellite


polymorphism. Based on the genotypes in Generation III, calculate the
recombination frequency between the disease and microsatellite loci.

Answer: _________________

44

IBO 2008
INDIA
THEORETICAL TEST PART B
_______________________________________________________________________

29. (2 points) The figure given below depicts a region of double stranded DNA, in
a bacterium, containing a polycistronic operon with three of your favorite
genes yfgA, yfgB and yfgC, as shown. The positions of certain bases in the
nucleotide sequence around yfg operon, with respect to position A are
marked in the figure.
The yfg operon

TTGACA
-35 region

TATAAT
-10 region
Transcription termination site
B

C
yfgA

200

300

810

yfgB

yfgC

1230

1560

1750

Length of DNA in base pairs

Answer the following questions:


I. What is the expected minimum number and length of the transcript(s) from
this operon?
a. A single transcript of 1260b
b. A single transcript of 1450b
c. A single transcript greater than 1451b but less than 1550b
d. Three transcripts of 330b, 420b and 510b

45

IBO 2008
INDIA
THEORETICAL TEST PART B
_______________________________________________________________________

Put a tick mark () in the appropriate box.

a.

b.

c.

d.

II. From the above figure, calculate the maximal expected mass of the YfgA
protein .
________________kDa
(assume the mass of an amino acid to be 110 Da)

46

IBO 2008
INDIA
THEORETICAL TEST PART B
_______________________________________________________________________

30. (2.5 points) The map distance between two genes on a chromosome can be
calculated using the frequency of crossing over between them. In case of a
genetic cross involving three genes, the crossover (CO) classes of progeny
can be categorized as
(i) Single crossover I (SCO I),
(ii) Single crossover II (SCO II), and
(iii) Double crossover (DCO).
DCO requires the simultaneous occurrence of the two SCOs.

Among the progeny of a test cross, the number of non-crossovers (NCO) is


the highest followed by SCO I and II. The DCO is the least frequent.

A Drosophila fly, heterozygous for alleles p, q and r, when crossed with a


homozygous recessive fly, had the following progeny:
(p+, q+, and r+ indicate wild-type alleles whereas p, q, and r indicate the
mutant alleles.)
Genotype

Number of progeny

p q+ r
p+q r+
pqr
p+ q+ r+
p+ q+ r
p q r+
p q+ r+
p+ q r

375
355
50
45
75
85
8
7
Total = 1000

47

IBO 2008
INDIA
THEORETICAL TEST PART B
_______________________________________________________________________

The middle gene is the one that has altered position in the DCO classes
compared to that in the NCO classes.

(A) Which is the middle gene in the given cross? Put a tick mark () in the
appropriate box.
p
q
r

(B) Assuming 1% crossover as one map unit (mu), calculate the distance
between p, q, and r.
Distance between p and q ________ mu

Distance between p and r

_________ mu

Distance between q and r

_________ mu

48

IBO 2008
INDIA
THEORETICAL TEST PART B
_______________________________________________________________________

ECOLOGY (16.5 points)


31. (1.5 points) Consider a population of r-selected insects in the early part of its
growth season. Choose the appropriate growth curve at this stage,
survivorship curve and current age structure (from each pair of graphs) that

Total life span

Growth Curve

Total life span

Number of organisms

Log (number surviving)

Number of organisms

Log (number surviving)

represent this population and put the corresponding letter in the boxes below.

Time

Time

Survivorship Curve

49

Age structure

IBO 2008
INDIA
THEORETICAL TEST PART B
_______________________________________________________________________

32. (3 points) Nitrogen, as a mineral nutrient, has the greatest effect on plant
growth. The atmosphere contains nearly 80% nitrogen gas (N2), yet plants
have to be provided with ammonium salts or nitrates as fertilizers for optimum
growth and yield. Certain nitrogen-fixing bacteria (rhizobia, cyanobacteria,
etc.) can convert atmospheric N2 into ammonia using nitrogenase by the
following reaction:
N2 + 8 e- + 8 H+ + 16 ATP

2 NH3 + H2 + 16 ADP + 16 Pi

Such bacteria can be used as biofertilizers in agriculture. In soil, ammonia is


protonated to ammonium (NH4+). This, in turn, is converted to nitrate (NO3)
and then to N2 gas by the action of nitrifying and denitrifying bacteria,
respectively. Plants require nitrogen mainly in the form of nitrate, which is
exported from roots to shoots, reconverted to ammonium and assimilated as
amino acids.

(A) Plants do not themselves fix N2, because:


a. it is easily available from the soil.
b. they lack the nitrogenase enzyme complex.
c. the process has a very high requirement of ATP per mole of N2 fixed.
d. hydrogen evolved in the process is deleterious to plants.

50

IBO 2008
INDIA
THEORETICAL TEST PART B
_______________________________________________________________________

Put a tick mark () in the appropriate box.


a.

b.

c.

d.

(B) Processes related to nitrogen conversion to different chemical forms in the


soil, carried out by the nitrogen-fixing bacteria, nitrifying bacteria and
denitrifying bacteria can be, respectively, described as:

a. reduction, oxidation and oxidation.


b. reduction, oxidation and reduction.
c. reduction, reduction and oxidation.
d. oxidation, oxidation and reduction.
Put a tick mark () in the appropriate box.
a.

b.

c.

d.

(C) Based on the given information, which type of soil bacteria will NOT be
beneficial for plants?

a. Nitrogen-fixing bacteria
b. Nitrifying bacteria
c. Denitrifying bacteria

51

IBO 2008
INDIA
THEORETICAL TEST PART B
_______________________________________________________________________

Put a tick mark () in the appropriate box.


a.

b.

52

c.

IBO 2008
INDIA
THEORETICAL TEST PART B
_______________________________________________________________________

33. (2 points) The relationship between members of different species is termed as


interspecific relationships. Some such relationships are listed below. -

1. Mosses (A) grow on the trunks and branches of trees (B).

2. A female moth (A), the only pollinator, arrives at a Yucca flower (B) with a
ball of Yucca pollen. She places her pollen ball on the stigma and then
lays eggs in some, but not all, of the ovules. Offspring development kills
the seeds on which they feed. If too many seeds are killed, the fruit is
aborted by the plant, killing the developing moth larva.

3. Wolbachia, a rickettsia-like bacterium (A) infects some insects (B). The


infected males are either killed or develop as females, leading to distorted
sex ratios (female bias) in the population.

4. Certain plants (A) attract ants (B) through extra-floral nectaries to deter
herbivores.

Indicate in the table given below whether the species (A and B) involved in
each of these are benefited (indicate by +), harmed (indicate by ) or not
affected (indicate by 0). Identify also the type of interaction by choosing from
the options I to VI given below.

53

IBO 2008
INDIA
THEORETICAL TEST PART B
_______________________________________________________________________

Options:
I.

Amensalism

II.

Commensalism

III.

Competition

IV.

Mutualism

V.

Parasitism

VI.

Predation

Number A

Type of
interaction

1.
2.
3.
4.

54

IBO 2008
INDIA
THEORETICAL TEST PART B
_______________________________________________________________________

34. (4 points) Mathematical models can be applied to many aspects of predator


behavior.
In a simple mathematical model, it is assumed that a predator can feed on
two prey species, Prey1 and Prey2 and that it captures and consumes every
prey it encounters. For this predator, the variables Ts, N1, N2, E1, E2, TH1, and
TH2 are defined as follows:
Ts: Total time spent searching for the prey species
N1: Number of Prey1 encountered per unit time
N2: Number of Prey2 encountered per unit time
E1: Energy gained from a single Prey1
E2: Energy gained from a single Prey2
TH1: Handling time needed for each Prey1. This includes time required for
capturing and consuming the prey.
TH2: Handling time needed for each Prey2

(A) Once a prey has been captured, the profitability (calories gained per unit time)
of each prey species for the predator is, respectively:

a.

E1
TH1

and

E2
TH2

b.

E1
E2
and
TH1 TH2
TH1 TH2

c.

E1
E2
and
N1TH1
N 2 TH2

55

IBO 2008
INDIA
THEORETICAL TEST PART B
_______________________________________________________________________

d.

E1
E2
and
TH1 TH2 TS
TH1 TH2 TS
Put a tick mark () in the appropriate box
a.

b.

c.

d.

(B) The total energy gain E for the predator will be:
a. E E1 E 2 TS
b. E E1N1 E2 N2
c. E E1N1 E2 N2 TS
d. E

E1N1 E 2 N 2
TS

Put a tick mark () in the appropriate box.


a.

b.

c.

d.

(C) The total time (T) spent to gain the total energy E will be:
a. T TS TS N1TH1 N 2 TH2
b.

T TS TH1 TH2

c.

T 1 N1TH1 N 2 TH2

d.

T TS N1TH1 N 2 TH2

56

IBO 2008
INDIA
THEORETICAL TEST PART B
_______________________________________________________________________

Put a tick mark () in the appropriate box.


a.

b.

c.

d.

.
(D) In one situation, the following data were obtained:
Ts = 60 minutes
Prey1

Prey2

N1 = 2/min

N2 = 5/min

TH1 = 10 min

TH2 = 20 min

E1 = 1000 cal

E2 = 700 cal

Which of the following hypotheses does the above mathematical model


support?
a. The predator should specialize on Prey1 as it leads to a better rate of energy
gain.
b. The predator should specialize on Prey2 as it leads to a better rate of energy
gain.
c. The predator should not specialize on one particular prey as a combination of
both prey species is more beneficial
d. The predator should specialize on both prey species as any one of them may
be likely to be unavailable in future.

57

IBO 2008
INDIA
THEORETICAL TEST PART B
_______________________________________________________________________

Put a tick mark () in the appropriate box.

a.

b.

58

c.

d.

IBO 2008
INDIA
THEORETICAL TEST PART B
_______________________________________________________________________

35. (6 points) A female gall fly (Eurosta solidaginis) typically lays a single egg in
the bud of some plants. After the egg hatches, the larva burrows its way
through the bud and produces a tumor-like structure called a gall. Larvae
inside these galls present a very nutritious food source for many birds.

(A) After observing some galls, a student proposed a hypothesis that birds
choose larger galls in preference to smaller ones. In order to gather the data
to test this hypothesis, she conducted a survey of one such site and
measured the widths of disturbed (fed on by the birds) as well as undisturbed
galls. The results are as follows:

Disturbed galls

Undisturbed galls

Gall number Width (mm) Gall number Width (mm)


1.

12

1.

18

2.

15

2.

15

3.

30

3.

22

4.

20

4.

12

5.

23

5.

20

You need to put this hypothesis to test. (Some of the required statistical
formulae as well as the Student-t and Chi-square probabilities are provided in
the Appendix at the end of Part B-Question Paper.)

59

IBO 2008
INDIA
THEORETICAL TEST PART B
_______________________________________________________________________

I. Which of the following is the correct null hypothesis?


a. The birds do not choose galls of smaller size.
b. The birds do not choose galls of larger size.
c. The birds do not choose galls based on size.
d. The birds do not choose galls of smaller size in preference to larger size.
Put a tick mark () in the appropriate box.
a.

b.

c.

d.

II. The test that you will require to do is:


a. Students t test
b. Chi-square test
c. Both Students t test and chi-square test
d. Either Students t test or chi-square test
Put a tick mark () in the appropriate box
a.

b.

c.

d.

III. The degree/s of freedom is/are: Answer:___________

IV. The value/s of the statistic/s (up to two decimal points):


Answer: _______________________

60

IBO 2008
INDIA
THEORETICAL TEST PART B
_______________________________________________________________________

V. Mark the correct interpretation:


a. At p < 0.05, the null hypothesis cannot be rejected.
b. At p < 0.05, the null hypothesis is rejected.
Put a tick mark () in the appropriate box
a.
b.

(B) After observing more sites, another student came up with a hypothesis that
patches with high density of galls are foraged more than those with low
density. To test this hypothesis, he surveyed six patches. The results are as
follows:
Gall

Site

Site

Site

Site

Site

Site

description

II

III

IV

VI

Density

High

Low

High

High

Low

Low

Foraged

15

10

14

60

Undisturbed

39

Total

20

17

22

14

17

99

I. The null hypothesis will be:


a. The birds do not choose galls in less dense areas.
b. Density of galls is not more important than the size of the gall.

61

Total

IBO 2008
INDIA
THEORETICAL TEST PART B
_______________________________________________________________________

c. Choosing of galls by birds is independent of the gall density in the patch.


d. Choosing of galls by birds is not dependent on the size of galls but on the
density of the patch.
Put a tick mark () in the appropriate box.
a.

b.

c.

d.

II. To test the hypothesis, the test that will be required is:
a. Students t test
b. Chi-square test
c. Both Students t test and chi-square test
d. Either Students t test or chi-square test
Put a tick mark () in the appropriate box.

a.

b.

c.

d.

III. The degree/s of freedom is/are: ____________

IV. The value/s of the statistic/s (up to two decimal points):


___________________

62

IBO 2008
INDIA
THEORETICAL TEST PART B
_______________________________________________________________________

V. Based on the value you obtain, the correct interpretation is:


a. At p < 0.05, the null hypothesis cannot be rejected.
b. At p < 0.05, the null hypothesis can be rejected.

Put a tick mark () in the appropriate box.

a.
b.

63

IBO 2008
INDIA
THEORETICAL TEST PART B
_______________________________________________________________________

ETHOLOGY (11 points)


36. (2 points) In a population of prey animals, most individuals are solitary and
stay well apart from others. But some mutant types arise that search out
others, use them as shields against predators and take away fitness from the
solitary types by making them more conspicuous to their predators. Let the
fitness payoff for a solitary individual living in a population consisting of only
solitary types be P. But when a solitary individual is found and used by a
social type, the solitary animal loses some fitness (B) to the social type. There
is also an additional cost C to being social in terms of the time required to find
a solitary individual to hide behind and that arising from the resulting
increased conspicuousness to predators. When two social types interact,
assume that they each have an equal chance of hiding behind the other when
the predator attacks. A game theory diagram summarizes these interactions
as follows:

Payoff for

In presence of
Solitary

Social
PB

Solitary

Social

P + B C P + B/2 B/2 C = P C

64

IBO 2008
INDIA
THEORETICAL TEST PART B
_______________________________________________________________________

(A) If B is greater than C, what behavioral type will predominate in the


population over time?
a. Solitary
b. Social
Put a tick mark () in the appropriate box.
a.
b.

(B) The average fitness payoff of a prey


(i) when it enters a population composed entirely of solitary types and
(ii) when it enters a population composed entirely of social types
would, respectively, be:

a. P B/2 C/2, P + B/2 C/2


b. P B/2, P + B/2 C
c. P + B/2 C/2, P B/2 C/2
d. P + B/2, P B/2 C

Put a tick mark () in the appropriate box.


a.

b.

65

c.

d.

IBO 2008
INDIA
THEORETICAL TEST PART B
_______________________________________________________________________

37. (3 points) Game theory models have been borrowed from economics and
often applied to behavioral ecology in order to understand the strategies that
animals use against each other while competing for resources. In a HawkDove game, for example, in which there were two kinds of competing
individuals, Hawks and Doves, with different behavioral strategies, John
Maynard Smith suggested the following pay-offs:

Winner

+50

Injury

-100

Loser

Display

-10

(A) Assuming that (a) Hawks always win against Doves, (b) Hawks win on half
the occasions when they meet other Hawks but suffer injury during the other
half, (c) Doves always display when they meet other Doves, but win on only
half of these occasions, and (d) Doves never display to hawks. What would
be the average pay-off to the attacker in different fights as listed in the
following matrix?

66

IBO 2008
INDIA
THEORETICAL TEST PART B
_______________________________________________________________________

Opponent
Hawk Dove
Hawk
Attacker
Dove

(B) An Evolutionary Stable Strategy (ESS) is one that will always win against any
other strategy and no other strategy can be successful within the population.
Evaluate whether the following statements are true or false given the pay-offs
for the Hawk and Dove strategies listed above.
a. Hawk is an ESS and when all individuals in a population play this strategy, a
mutation to Dove can never be successful.
b. Dove is an ESS and when all individuals in a population play this strategy, A
mutation to Hawk can never be successful.
Put a tick mark () in the appropriate box.
Statement True
a.
b.

67

False

IBO 2008
INDIA
THEORETICAL TEST PART B
_______________________________________________________________________

38. (2 points) White crown sparrows that live in temperate regions show a
complex annual cycle of behavior.

Winter

Autumn

D
B

Spring
Migration to north

Increased rate of feeding


Territory establishment
Summer
Care of young

Match the physiological changes (I to V) in these birds with the appropriate


points in the behavioral cycle (A to D). Choose from the following options and
fill in the table given below with the appropriate numbers:

Options:
I. Moulting (molting)
II. Gonadal regression (Shrinking of reproductive tissue)
III. Gonadal development (Development of the reproductive tissue)
IV. Fat deposition
V. Hibernation

68

IBO 2008
INDIA
THEORETICAL TEST PART B
_______________________________________________________________________

Points in the

Physiological

behavioral cycle

changes

A
B
C
D

69

IBO 2008
INDIA
THEORETICAL TEST PART B
_______________________________________________________________________

39. (4 points) The dunnock is a common bird of the British Islands. The females
of this species establish territories represented by solid lines in the figure
below, which may be defended by one or two ( and ) unrelated males
(dashed lines). The numbers in the figure refer to the average number of
young raised per season by males and females in the different mating
combinations. The arrows indicate the directions in which the behaviour of the
males and females encourage changes in the mating system.
Male: 5.0

Monogamy

Female: 5.0

II

III

IV

-Male: 3.7, -Male: 3.0


Male: 7.6

Female: 3.8

Female: 6.7

Female: 3.8

Polygyny

Polyandry

70

IBO 2008
INDIA
THEORETICAL TEST PART B
_______________________________________________________________________

(A) Identify the specific individuals, which would attempt to change the mating
system in the directions shown by the arrows.
a. I: Male, II: Female, III: Female, IV: -Male
b. I: Female, II: Male, III: -Male, IV: -Male
c. I: Female, II: Male, III: Female, IV: -Male
d. I: Male, II: Female, III: -Male, IV: -Male
Put a tick mark () in the appropriate box.

a.

b.

c.

d.

(B) Which of the following statements are true?


I. The benefit of polygyny to males is the increased amount of food brought
for the chicks by two females instead of one.
II. The cost of polygyny to females is shared male care because the
contribution of the males feeding efforts is essential for the survival of the
chicks.
III. The cost of polyandry to females is the aggression that often results
between the two males who have mated with her.
IV. The cost of polyandry to males is shared paternity.

71

IBO 2008
INDIA
THEORETICAL TEST PART B
_______________________________________________________________________

a. I and II
b. II and III
c. I and IV
d. II and IV

Put a tick mark () in the appropriate box.


a.

b.

72

c.

d.

IBO 2008
INDIA
THEORETICAL TEST PART B
_______________________________________________________________________

BIOSYSTEMATICS (13)
40. (2 points) The schematic diagram below represents group-in-group
relationships. The T3 taxon, represented by the largest circle, includes three
T2 taxa. Each of these three T2 taxa has one T1 taxon, represented by
circles filled with dots; the dots represent individuals.

T2a

T2c

T1a

T1c

T3
T2b
T1b

According to the above scheme, assign the correct taxa from the options
given below to each of the circles. Fill in your answers by writing the
appropriate number in the table. Points will only be awarded if the entire
table is correctly filled.

73

IBO 2008
INDIA
THEORETICAL TEST PART B
_______________________________________________________________________

Options:
I.

Annelida

II.

Lepidoptera

III.

Polychaeta

IV.

Mollusca

V.

Orthoptera

VI.

Insecta

VII.

Arthropoda

VIII.

Crustacea

IX.

Gastropoda

X.

Arachnida

XI.

Lumbricus (earthworm)

XII.

Hirudo (leech)

XIII.

Gryllus (cricket)

XIV.

Unio (freshwater mussel)

XV.

Euscorpias (scorpion)

XVI.

Daphnia (water flea)

Taxon
T3
T2a
T1a
T2b
T1b
T2c
T1c

74

Option

IBO 2008
INDIA
THEORETICAL TEST PART B
_______________________________________________________________________

41. (2 points) The sequence of a pentanucleotide DNA segment of four species


P, Q, R and S are given.

Species

Sequence site
1

The most parsimonious phylogenetic classification of these species would be:

a.

P
Q

4 5

4 5
3

P
R

3 1

d.

1 4 5
Q

1 3
R

1 3

3
P

4 5
1

75

145

1 3

c.

b.

IBO 2008
INDIA
THEORETICAL TEST PART B
_______________________________________________________________________

Put a tick mark () in the appropriate box.


a.

b.

76

c.

d.

IBO 2008
INDIA
THEORETICAL TEST PART B
_______________________________________________________________________

42. (5 points) A classification chart based on certain characteristics of


invertebrates is shown below:

no bilateral

invertebrates

aquatic

Fill in the appropriate numbers from the classification chart against the
respective groups in the table below:

77

IBO 2008
INDIA
THEORETICAL TEST PART B
_______________________________________________________________________

Group

Number

Group

Annelida (Earthworms)

Mollusca (Land snails)

Arthropoda (Crayfishes)

Mollusca (Squids)

Cnidaria (Jellyfishes)

Nematoda (Roundworms)

Echinodermata (Starfishes)

Platyhelminthes (Tapeworms)

Mollusca (Bivalvia)

Porifera (Sponges)

78

Number

IBO 2008
INDIA
THEORETICAL TEST PART B
_______________________________________________________________________

43. (4 points) The genetic distances between four species are provided in a
matrix below. The numbers represent the percentage differences between
each pair of species.

C D

A -

B 5

C 13 14 -

D 15 16 6

(A) Which of the following tree structures represent the matrix data most
appropriately? (Squares in the generalized figure represent species, and the
lines represent the relationship between them).

a.

c.

b.

d.

Put a tick mark () in the appropriate box.

79

IBO 2008
INDIA
THEORETICAL TEST PART B
_______________________________________________________________________

a.

b.

c.

d.

(B) Based on the answer selected in the previous question and using the data
given in the matrix, construct the tree that correctly shows the genetic
relatedness of the four species where the numbers on the lines should
approximate the genetic distance between them.

****************** END OF PART B ******************

80















All IBO examination questions are published under the following Creative Commons license:



CC BY-NC-SA (Attribution-NonCommercial-ShareAlike) https://creativecommons.org/licenses/by-nc-sa/4.0/
The exam papers can be used freely for educational purposes as long as IBO is credited and
new creations are licensed under identical terms. No commercial use is allowed.

INTERNATIONAL BIOLOGY OLYMPIAD


PRACTICAL PROBLEMS

2008, India, Mumbai















All IBO examination questions are published under the following Creative Commons license:



CC BY-NC-SA (Attribution-NonCommercial-ShareAlike) https://creativecommons.org/licenses/by-nc-sa/4.0/
The exam papers can be used freely for educational purposes as long as IBO is credited and
new creations are licensed under identical terms. No commercial use is allowed.

IBO 2008
INDIA
PRACTICAL TEST 1
PLANT ANATOMY AND PHYSIOLOGY
_______________________________________________________________________

Country: __________________

Student Code: ____________

19th INTERNATIONAL BIOLOGY OLYMPIAD


13th 20th July, 2008
Mumbai, INDIA

PRACTICAL TEST 1
PLANT ANATOMY AND PHYSIOLOGY
Total Points: 47
Duration: 60 minutes

IBO 2008
INDIA
PRACTICAL TEST 1
PLANT ANATOMY AND PHYSIOLOGY
_______________________________________________________________________

Dear Participants,

In this test, you have been given the following two tasks:
Task 1: Study of factors affecting the activity of stomata (33 points)
Task 2: Study of plant anatomy and its correlation with the habitat
(14 points)

You have to write down your results and answers in the ANSWER SHEET.
Answers written in the Question Paper will not be evaluated.

Please make sure that you have received all the materials and equipment listed
for each task. In case any of these items is missing, please raise the yellow card.

At the end of the test, put the Answer Sheet and Question Paper in the envelope.
The supervisor will collect this envelope.

Good Luck!!

Country: ________________________
Country Code: ____________________
First Name: _______________________
Middle Name: _____________________
Family Name: _____________________
Student Code: ____________________

IBO 2008
INDIA
PRACTICAL TEST 1
PLANT ANATOMY AND PHYSIOLOGY
_______________________________________________________________________

Task 1 (33 points)


Study of factors affecting the activity of stomata
You should try and complete this task in 30 minutes.

Materials and equipment

Quantity

1. Specimens labeled 1 to 8 (in red capped vials)

2. Compound binocular microscope

3. Glass microslides

4. Box of coverslips

5. Watchglass

6. Forceps

7. Brush

8. Wash bottle containing distilled water

9. Permanent marker pen

10. Tissue paper roll

11. Container for washing and discard

Introduction
Stomata are specialized microscopic structures found in all vascular plants.
These microscopic pores allow exchange of gases between the environment and the
plant cells. Stomata are also the sites from where water evaporates from the plant.
Various environmental factors such as temperature, humidity and light intensity can
affect the opening or closing of the stomata.

IBO 2008
INDIA
PRACTICAL TEST 1
PLANT ANATOMY AND PHYSIOLOGY
_______________________________________________________________________

Q. 1.1. (3 points) Some statements about stomata are given below. Indicate
whether the statements are true or false by putting a tick mark ( ) in the
appropriate boxes in Q. 1.1. in the Answer Sheet.

Statement

True False

a. Guard cells covering stomata are the only epidermal cells that
contain chloroplasts.
b. The Stomatal Index of any plant species is the ratio of number of
stomata in a given area of the leaf to the total number of stomata
and other epidermal cells in that same area.
c. Stomata are characteristic of angiosperms alone.
d. Larger the stomatal pore, greater is the rate of transpiration per
unit area of the pore.
e. The Stomatal Index is always constant for a given species.
f.

A plant with stomata only on the upper surface of its leaves is most
likely to be a submerged hydrophyte.

Stomata can be observed by taking an epidermal peel of a leaf. Alternatively, An


imprint of the stomata can be obtained, without damaging the leaf tissue, as follows:

A thin coat of transparent colourless nail polish is applied on the leaf surface. The
coat is allowed to dry and peeled off using a pair of forceps. This imprint is placed on

IBO 2008
INDIA
PRACTICAL TEST 1
PLANT ANATOMY AND PHYSIOLOGY
_______________________________________________________________________

a microslide with a drop of water, a coverslip is placed on it and it is observed under


the microscope.

Open and closed stomata can be easily distinguished from these imprints.
Representative microscopic images of the imprints are given below.

OPEN stomata: (10X)

CLOSED stomata: (10X)

IBO 2008
INDIA
PRACTICAL TEST 1
PLANT ANATOMY AND PHYSIOLOGY
_______________________________________________________________________

In this task, you will study the effect of various factors on the opening and closing of
the stomata. These experiments are conducted by immersing leaves in solutions of
different chemicals under specified conditions.

Q. 1.2. (0.5 point) The best choice for such an experiment would be:
a. a mesophyte
b. a hydrophyte
c. a xerophyte
d. a halophyte

Put a tick mark () in the appropriate box in Q. 1.2. in the Answer Sheet.

a.

b.

c.

d.

Q. 1.3. (2 points) In one such experiment, a solution of pH 4.0 containing 100 mM


+

K and 0.1 mM Ca

++

is required. Choose the correct way to prepare this solution

from the following options.


Atomic weight of K is 39.1 and of Cl is 35.5.
a. Dissolve 3.73 g of KCl in 90 ml citrate buffer of pH 4.0, add 1 ml of 10 mM CaCl 2,
and make up the volume to 500 ml.

IBO 2008
INDIA
PRACTICAL TEST 1
PLANT ANATOMY AND PHYSIOLOGY
_______________________________________________________________________

b. Dissolve 0.373 g of KCl in 40 ml distilled water. Add 0.5 ml of 10 mM CaCl 2 to 5


ml distilled water. Mix both the solutions and adjust the pH to 4.0 with HCl. Make
up the volume to 50 ml.

c. Dissolve 7.46 g of KCl in 50 ml distilled water. To this, add 10 ml of 10 mM CaCl 2.


Adjust the pH to 4.0 with HCl and make up the volume to 100 ml.

d. Dissolve 0.746 g of KCl in 100 ml of distilled water, add 1 ml of 10 mM CaCl 2, and


adjust the pH to 4.0 with HCl.

Put a tick mark () in the appropriate box in Q. 1.3. in the Answer Sheet.

a.

b.

c.

d.

Experiment

In order to study the effect of various factors on stomata, leaves of a plant were
treated in eight different ways. The imprints obtained after each of these treatments
(1 to 8 given below) are provided in red-capped vials labeled 1 to 8, respectively.

IBO 2008
INDIA
PRACTICAL TEST 1
PLANT ANATOMY AND PHYSIOLOGY
_______________________________________________________________________

Treatment 1: Leaves were immersed in a solution containing 100 mM KCl, 0.1 mM


CaCl2, pH 7.0 and kept in light for 2 h.

Treatment 2: Leaves were immersed in a solution containing 10 mM KCl, 0.1 mM


CaCl2, pH 7.0 and kept in dark for 2 h.

Treatment 3: Leaves were immersed in a solution containing 0.5 M mannitol, 100


mM KCl, 0.1 mM CaCl2, pH 7.0 and kept in light for 2 h.

Treatment 4: Leaves were immersed in a solution containing 10 mM KCl, 0.1 mM


CaCl2, pH 4.0 and kept in dark for 2 h.

Treatment 5: Leaves were immersed in solution containing 10 mM KCl, 0.1 mM


CaCl2, pH 7.0 containing an Unknown Chemical and kept in dark for 2 h.

Treatment 6: Leaves were immersed in a solution containing 100 mM KCl, 0.1 mM


CaCl2, pH 4.0 and kept in light for 2 h.

Treatment 7: Leaves were immersed in a solution containing 100 mM KCl, 0.1 mM


CaCl2, 10 M abscisic acid, pH 7.0 and kept in light for 2 h.

Treatment 8: Leaves were immersed in a solution containing 100 mM KCl, 0.1 mM


CaCl2, 10 M abscisic acid, pH 4.0 and kept in light for 2 h.

IBO 2008
INDIA
PRACTICAL TEST 1
PLANT ANATOMY AND PHYSIOLOGY
_______________________________________________________________________

Q. 1.4. (8 points): Observation of the imprints


(i) Pick up the imprint gently from the container using a brush. You may need to
gently swirl the vial in order to locate the imprint.
(ii) Place the imprint in a drop of water on a glass microslide.
(iii) Place a coverslip and observe under the 10X objective of the microscope. Note
that you will be observing unstained specimens. Hence, make appropriate
adjustments in the microscope.
(iv) Note down the observations in Table 1.4. in the Answer Sheet. You need to
count at least 20 stomata per imprint.

Table 1.4.
Treatment

Total

Number of

Number of

number of

open stomata

closed

stomata

stomata

counted
1
2
3
4
5
6
7
8

IBO 2008
INDIA
PRACTICAL TEST 1
PLANT ANATOMY AND PHYSIOLOGY
_______________________________________________________________________

Interpretations
Q. 1.5. (10 points)
Based on the results obtained from the given set of experiments, answer the
questions Q. 1.5.A to Q. 1.5.C in the Answer Sheet. Fill in the blanks with the
appropriate serial numbers from the options given below.
A. The factor/s that clearly lead to stomatal closure is/are: ______________
B. The factor/s that clearly lead to stomatal opening is/are: ______________
C. The factor/s that clearly have no effect on stomatal opening/closing is/are:
______________
Options:
1. Light alone
2. Darkness alone
3. Acidic pH
4. Mannitol
5. Unknown Chemical
6. 10mM KCl alone
7. 100mM KCl alone
8. CaCl2
9. Abscisic acid alone
10. Abscisic acid and acidic pH

10

IBO 2008
INDIA
PRACTICAL TEST 1
PLANT ANATOMY AND PHYSIOLOGY
_______________________________________________________________________

Q. 1.6. (2.5 points) The correct explanation for the observations in Treatments 7 and
8 is:
+

a. Acidification of guard cells leads to opening of K channels of the plasma


+

membrane. This results in entry of K and water molecules to the guard cells.
b. As the pKa of abscisic acid is close to 5.0, most of the molecules remain
undissociated at pH 4.0. This hastens their entry into the guard cells.
c. No effect was observed in either treatment because there was no water stress.
d. Abscisic acid is a strong acid and works best at highly acidic pH.

Put a tick mark () in the appropriate box in Q. 1.6. in the Answer Sheet

a.

b.

c.

d.

Q. 1.7. (2.5 points) Which of the following correctly explains the effect of mannitol on
the stomatal aperture?
a. Mannitol is a highly hydrophilic substance and restricts the entry of water
molecules into the guard cells.
b. High concentration of mannitol in the extracellular fluid forces K +, Cl- and Ca++ to
enter the guard cells. This leads to entry of water molecules into the cells as well.

11

IBO 2008
INDIA
PRACTICAL TEST 1
PLANT ANATOMY AND PHYSIOLOGY
_______________________________________________________________________

c. Entry of mannitol into guard cells increases their solute potential leading to
uptake of water.
d. High solute concentration of mannitol results in withdrawal of water from guard
cells.
e. Entry of mannitol in the guard cells is counter-balanced by the efflux of K+ and
Ca++ leading to the withdrawal of water from the guard cells.

Put a tick mark () in the appropriate box in Q. 1.7. in the Answer Sheet

a.

b.

c.

d.

e.

Q. 1.8. (2.5 points) You have already observed the effect of the Unknown Chemical
on stomata (Treatment 5). These results suggest that the chemical could be useful
for:
a. weed control by increasing the rate of respiration.
b. keeping plant cuttings fresh over long periods by preventing water loss.
c. weed control by acting as a wilting toxin.
d. increasing crop yield in arid lands by increasing rate of photosynthesis.
e. increasing plant growth by reducing photorespiration.

12

IBO 2008
INDIA
PRACTICAL TEST 1
PLANT ANATOMY AND PHYSIOLOGY
_______________________________________________________________________

Put a tick mark () in the appropriate box in Q. 1.8 in the Answer Sheet.

a.

b.

c.

d.

e.

Q. 1.9. (2 points) In this task, you studied the effect of various factors on the
opening and closing of the stomatal aperture. Similar experiments were performed
by scientists and they discovered that light activates zeaxanthin molecules, present
in the guard cells, which in turn, activate an ATP-powered proton pump of the guard
cell membrane. With this background information and the observations made by you
in this task, you have to arrange the sequence of events involved in the response of
stomata to light. Fill in the correct options against each step in Q. 1.9. in the Answer
Sheet.
Mechanism:
Step I: _____ 1_______
Step II: _____2_______
Step III: _____________
Step IV: ____6________
Step V: _____________
Step VI: _____________
Step VII: _____________

13

IBO 2008
INDIA
PRACTICAL TEST 1
PLANT ANATOMY AND PHYSIOLOGY
_______________________________________________________________________

Options:
1) Activation of zeaxanthin by light
2) Activation of ATP-powered proton pump
3) Closing of the stomata
4) Influx of K+
5) Efflux of K+
6) Change in membrane potential
7) Efflux of Ca++
8) Efflux of protons
9) Influx of water molecules
10) Efflux of water molecules
11) Opening of the stomata

14

IBO 2008
INDIA
PRACTICAL TEST 1
PLANT ANATOMY AND PHYSIOLOGY
_______________________________________________________________________

Task 2 (14 points)


Study of plant anatomy and its correlation with the habitat
You should try and complete this task in 30 minutes.

Materials and equipment

Quantity

1. Fresh plant specimens


(i) Leaf in a Petri dish (labeled X)

(ii) Stem in a Petri dish (labeled Y)

2. Compound binocular microscope

3. Razor blades

4. Glass microslides

5. Box of coverslips

6. Watchglasses

7. Safranin staining solution (labeled S)

8. Brush

Introduction
Plants growing in different habitats exhibit various adaptations. These adaptations
can be studied macroscopically as well as microscopically and correlated to their
habitats.

In this task, you will study the anatomy of the given specimens using the following
method.

15

IBO 2008
INDIA
PRACTICAL TEST 1
PLANT ANATOMY AND PHYSIOLOGY
_______________________________________________________________________

Method
1. Take thin transverse sections of the leaf specimen X.
2. Stain with Safranin staining solution for 45 seconds.
3. Wash the section with distilled water and mount on a clean glass microslide in a
drop of water.
4. Place a coverslip and observe under 10X objective of the microscope.
5. Repeat Steps 1- 4 for the stem specimen Y.

Observations on the leaf specimen X:


Observe the leaf section and answer Questions Q. 2.1. and Q. 2.2.

Q. 2.1. (4 points) Choose the appropriate letters from the Dichotomous Keys 1 and
2 given in Annexure 2.1. and fill in Q. 2.1.I. and Q. 2.1.II. in the Answer Sheet.

Note: Schematic representations of some of the plant structures are given in


Annexure 2.2. for your reference.

I. Trichomes

II. Stomata

16

IBO 2008
INDIA
PRACTICAL TEST 1
PLANT ANATOMY AND PHYSIOLOGY
_______________________________________________________________________

Q. 2.2. (4 points) Based on your observations on the leaf section, put a tick mark ( )
in the appropriate boxes in Q. 2.2. in the Answer Sheet.

Present

Absent

1. Cuticle
2. Sclerenchyma
3. Collenchyma
4. Air spaces
5. Water storage tissue
6. Glands:
a. Oil gland
b. Salt gland
c. Digestive gland

Observations on the stem specimen Y:


Observe the stem section and put a tick mark () in the appropriate boxes in Q. 2.3.
in the Answer Sheet.

17

IBO 2008
INDIA
PRACTICAL TEST 1
PLANT ANATOMY AND PHYSIOLOGY
_______________________________________________________________________

Q. 2.3. (3.5 points)


Present

Absent

1. Cuticle
2. Sclerenchyma
3. Collenchyma
4. Air spaces
5. Water storage tissue
6. Vascular bundle:

Open

Collateral

Closed

Bicollateral

Q. 2.4. (2.5 points) Based on your observations, identify the type of plant to which
the specimens belong.

a. Mesophyte
b. Succulent xerophyte
c. Submerged hydrophyte
d. Floating hydrophyte
e. Insectivorous mesophyte
f. Parasitic mesophyte

18

IBO 2008
INDIA
PRACTICAL TEST 1
PLANT ANATOMY AND PHYSIOLOGY
_______________________________________________________________________

g. Halophyte
h. Freshwater hygrophyte
Put a tick mark () in the appropriate box in Q. 2.4. in the Answer Sheet.
(In this question, the correct interpretation will be given points if it is
consistent with your observations.)

a.

b.

c.

d.

e.

**************

19

f.

g.

h.

IBO 2008
INDIA
PRACTICAL TEST 1
PLANT ANATOMY AND PHYSIOLOGY
_______________________________________________________________________

ANNEXURE 2.1
Dichotomous Key 1

Trichomes (Epidermal appendages)

Present

Absent
K

Hair

On adaxial
surface

Papillae

On abaxial
surface

On adaxial
surface
I

Peltate A

Peltate E

Capitate B

Capitate

Branched C

Branched G

Simple and
uniserrate D

Simple and
uniserrate H

On abaxial
surface
J

Dichotomous Key 2
Stomata

Present

On adaxial
surface

Stomatal
crypt present
M

Stomatal
crypt absent

Absent
Q
On abaxial surface

Stomatal
crypt present
O

N
20

Stomatal
crypt absent
P

IBO 2008
INDIA
PRACTICAL TEST 1
PLANT ANATOMY AND PHYSIOLOGY
_______________________________________________________________________

ANNEXURE 2.2.

Epidermal cells

Figure 2: Digestive Gland

Figure 1: Salt Gland

Figure 3: Oil Gland

Papilla

Figure 4: Papillose Epidermis

Figure 5: Peltate Hair

Sub-stomatal
Chamber

Figure 6: Sunken Stoma


Figure 7: Stomatal Crypt

Figure 8: Capitate Hair

Figure 9: Uniserrate Trichome Figure 10: Branched Trichome

************END OF PRACTICAL TEST 1************


21

IBO 2008
INDIA
PRACTICAL TEST 2
ANIMAL ANATOMY AND PHYSIOLOGY
__________________________________________________________________________

Country: ________________

Student Code: ____________

19th INTERNATIONAL BIOLOGY OLYMPIAD


13th 20th July, 2008
Mumbai, INDIA

PRACTICAL TEST 2
ANIMAL ANATOMY AND PHYSIOLOGY
Total Points: 66
Duration: 60 minutes

IBO 2008
INDIA
PRACTICAL TEST 2
ANIMAL ANATOMY AND PHYSIOLOGY
__________________________________________________________________________

Dear Participants,

In this test, you have been given the following two tasks:
Task 1: Study of animal skeletal systems (54 points)
Task 2: Semi-quantitative estimation of nitrogenous waste products
(12 points)

You have to write down your results and answers in the ANSWER
SHEET. Answers written in the Question Paper will not be
evaluated.

Please make sure that you have received all the materials and
equipment listed for each task. In case any of these items is missing,
please raise the yellow card.

At the end of the test, put the Answer Sheet and Question Paper in the
envelope. The supervisor will collect this envelope.
Good Luck!!

Country: ________________________
Country Code: ____________________
First Name: _______________________
Middle Name: _____________________
Family Name: _____________________
Student Code: ____________________

IBO 2008
INDIA
PRACTICAL TEST 2
ANIMAL ANATOMY AND PHYSIOLOGY
__________________________________________________________________________

Task 1 (54 points)


Study of animal skeletal systems
You should try and complete this task in 45 minutes.

Materials and equipment

Quantity

1. Set of skeletal specimens labeled 1 to 9 in sealed boxes

(Please do not open the boxes!)


2. A set of photographs of three skulls labelled 1A, 2A and 3A

3. Magnifying hand-lens

Introduction
The skeletal system provides physical support and a scaffold for the body and
defines its architecture in animals. The three types of skeletal systems include
an external (exoskeleton), internal (endoskeleton) and a fluid-based
(hydrostatic skeleton) system.

The internal skeleton in vertebrates determines its body shape, provides


support for its weight and offers sites for muscle attachment. Although
structural modifications in the skeleton may occur in different groups of
animals, the basic plan by and large remains the same.

In this task, you will observe and compare the internal skeletal systems of
three present-day vertebrates. The models of the skeletal parts provided to

IBO 2008
INDIA
PRACTICAL TEST 2
ANIMAL ANATOMY AND PHYSIOLOGY
__________________________________________________________________________

you include the skull, the vertebral column and the limb bones. At the end of
the task, you will match these parts to form the complete skeletal system of
each of the three vertebrates.
Part A: Comparative study of skulls
(i) Types of skull:
The skull of vertebrates is a bony structure that serves as the general
framework for the head. Structurally, the skull comprises four regions frontal,
parietal, occipital and temporal (Figure 1). There are various openings in
different regions of the skull, including the nostrils, eye sockets and the
temporal openings. The placement of the eyes with respect to each other
determines the field of vision of the animal.

Figure 1

IBO 2008
INDIA
PRACTICAL TEST 2
ANIMAL ANATOMY AND PHYSIOLOGY
__________________________________________________________________________

The number of temporal fenestrae (openings) and the position of these


openings are used to broadly classify vertebrate skulls into the following four
major categories:

(A) Anapsid skull: Anapsids get their name from the fact that they have no
additional openings in their skulls apart from their eye sockets and nostrils.
The temporal region is covered completely by bone. This type of skull is
characteristic of fishes, amphibians, and early reptiles (Figure 2).

Eye Socket

Nostril

Figure 2
(B) Synapsid skull: It has a single pair of temporal openings. It was found in
mammalian ancestors and represented an early divergence from the
anapsids. The skull of present day mammals represents a modified synapsid
pattern (Figure 3).

Eye Socket

Nostril
Temporal
Opening

Figure 3

IBO 2008
INDIA
PRACTICAL TEST 2
ANIMAL ANATOMY AND PHYSIOLOGY
__________________________________________________________________________

(C) Diapsid skull: It is characterized by two pairs of temporal openings. This


type diverged from the anapsids and has undergone extensive modification. It
is found in pterosaur and other dinosaur fossils, as well as in birds and all
living reptiles. One of the highly modified forms of the diapsid skull is found in
lizards, where the lower temporal opening is not as distinct as the upper one
(Figure 4).
Eye Socket

Nostril

Temporal
openings

Figure 4

(D) Euryapsid skull: It has a single pair of temporal openings. The euryapsid
skull seems to be derived from diapsid ancestors by loss of the lower temporal
openings. Two groups of Mesozoic marine reptiles (plesiosaurs and
ichthyosaurs) possessed this type of skull (Figure 5).
Eye Socket
Temporal
opening

Nostril

Figure 5

IBO 2008
INDIA
PRACTICAL TEST 2
ANIMAL ANATOMY AND PHYSIOLOGY
__________________________________________________________________________

Q. 1.A.1. (2 points): Based on the information provided earlier, choose the


cladogram that most likely depicts the evolution of skulls by putting a tick mark
() in Q. 1.A.1. in the Answer Sheet.

Modern mammalian
skull

Synapsid
a.

Anapsid

Modern reptilian skull

Diapsid

Euryapsid

Synapsid

b.
Anapsid

Modern mammalian skull


Euryapsid

Diapsid

Modern terrestrial reptilian


skull

c.
Synapsid
Anapsid

Diapsid
Euryapsid
Early reptilian and
modern avian skull

Modern mammalian skull

IBO 2008
INDIA
PRACTICAL TEST 2
ANIMAL ANATOMY AND PHYSIOLOGY
__________________________________________________________________________

d.
Synapsid

Modern mammalian skull

Anapsid
Early reptilian and
modern avian skull

Diapsid

Euryapsid

a.
b.
c.
d.

(ii) Dentition: Dentition refers to the type and arrangement of teeth in an


animal and is an adaptation to its feeding habit. Based on the dentition,
vertebrates can be broadly classified as homodont or heterodont. Depending
on the number of times the teeth are replaced during the life span of an
animal, they can be further classified as diphyodont or polyphyodont.

Q. 1.A.2. (6 points) Observe the Specimens 1, 2 and 3 for the type of skull
and the respective photographs 1A, 2A, and 3A for their dentition. Put tick
marks () in the appropriate boxes in Table 1.A.2. in the Answer Sheet.

IBO 2008
INDIA
PRACTICAL TEST 2
ANIMAL ANATOMY AND PHYSIOLOGY
__________________________________________________________________________

Table 1.A.2.

Character
Type of skull

Anapsid
Diapsid
Synapsid
Euryapsid

Type of dentition

Homodont
Heterodont

Q. 1.A.3. (6 points) Observe the specimens for position of orbit (the eye
sockets), and for types of teeth. Fill in the Table 1.A.3. in the Answer Sheet
by putting tick marks () in the appropriate boxes.
Table 1.A.3.

Features
Vision

Predominantly stereoscopic vision


Predominantly non-stereoscopic vision

Feeding habit Predominantly carnivorous


Predominantly herbivorous

IBO 2008
INDIA
PRACTICAL TEST 2
ANIMAL ANATOMY AND PHYSIOLOGY
__________________________________________________________________________

Part B: Comparative study of vertebral columns and ribs


The vertebral column and ribs are components of the axial skeletal system.
The vertebral column defines the major body axis and comprises a series of
separate bones (vertebrae) joined to form a backbone (Figure 6).

Sacral vertebrae
Caudal vertebrae

Figure 6

Cervical vertebrae, the first set of vertebrae, are characterized by highly


reduced transverse processes in contrast to the following set of vertebrae, the

10

IBO 2008
INDIA
PRACTICAL TEST 2
ANIMAL ANATOMY AND PHYSIOLOGY
__________________________________________________________________________

thoracic vertebrae. The number of cervical vertebrae is usually correlated with


the degree of neck movement.

In higher animals, the thoracic vertebrae are important because they articulate
with the ventral sternum and ribs to form a rib cage.

Ribs also provide sites for secure muscle attachment, help suspend the body,
form a protective case around the viscera and sometimes serve as accessory
breathing devices (Figure 7).

True rib

False rib
Floating rib
Figure 7

The sternum is a mid-ventral skeletal structure that offers a site of origin for
chest muscles and secures the ventral tips of true ribs to complete the
protective rib cage. A well-developed rib cage is characteristic of mammals.
11

IBO 2008
INDIA
PRACTICAL TEST 2
ANIMAL ANATOMY AND PHYSIOLOGY
__________________________________________________________________________

The classification of ribs in tetrapods is based on the type of association they


establish with the sternum. The three kinds of ribs are:

True ribs these ribs meet ventrally with the sternum.


False ribs these ribs articulate with each other but not with the sternum.
Floating ribs these ribs do not articulate with the sternum or any other
structure. These ribs, when present in large numbers offer flexibility to the
body during locomotion.

Q. 1.B.1. and Q.1.B.2. (8 + 3 = 11 points) Observe the Specimens 4, 5 and 6


and put tick marks () in the appropriate boxes in Tables 1.B.1. and 1.B.2. in
the Answer Sheet.

Table 1.B.1.
Characters
Ribs

Present
Absent

Major type of ribs

True
False
Floating

Tail

Present
Reduced /Absent

12

IBO 2008
INDIA
PRACTICAL TEST 2
ANIMAL ANATOMY AND PHYSIOLOGY
__________________________________________________________________________

Table 1.B.2.
Feature
Neck movement

Restricted
Free

Part C: Comparative study of limb bones


The transition of vertebrates from aquatic to terrestrial and from terrestrial to
aerial has had an impact upon the design and redesign of the appendicular
system. The appendicular skeleton includes the paired fins or limbs and the
girdles. Schematic figures of representative limb arrangements are given
below (Figures 8 and 9).
Limb bone

girdle

Body

Figure 8: Schematic representation of the articulation of a sprawled limb


girdle

Body

Limb bone

Figure 9: Schematic representation of the articulation of an underneath limb

13

IBO 2008
INDIA
PRACTICAL TEST 2
ANIMAL ANATOMY AND PHYSIOLOGY
__________________________________________________________________________

Q. 1.C.1. (12 points) Carefully study the Specimens 7, 8 and 9, and fill in
Table 1.C.1. in the Answer Sheet by putting tick marks () in the appropriate
boxes.

Characters
Position of

Sprawled

limb with
respect to

Underneath

body
Length of

Similar

fore- and

Fore limbs longer

hindlimbs
Claws

Hind limbs longer


Present
Absent

Modifications Tibia and fibula


completely fused
Tibia and fibula
partially separate

Q. 1.C.2. (8 points) Based on your observations, fill in Table 1.C.2. in the


Answer Sheet by putting tick marks () in the appropriate boxes.

14

IBO 2008
INDIA
PRACTICAL TEST 2
ANIMAL ANATOMY AND PHYSIOLOGY
__________________________________________________________________________

Table 1.C.2.
Features

Limb movement during

Swinging (rotational

locomotion

movement)

Pendulum-like
Habit of the animal

Saltatorial (jumping)
Cursorial (walking)
Fossorial (digging)

Part D: Assembly of the skeletal systems

Q. 1.D.1 (6 points) The nine specimens (three skulls, three vertebral columns
and three sets of limb bones) belong to three different animals (I, II, and III). In
the schemes given below, write the respective specimen numbers (4 to 9) in
the appropriate boxes to construct the three animals in Q. 1.D.1. in the
Answer Sheet.

Skull

Vertebral
column

Pair of limbs

15

IBO 2008
INDIA
PRACTICAL TEST 2
ANIMAL ANATOMY AND PHYSIOLOGY
__________________________________________________________________________

Animal I

Animal II

Animal III

Q. 1.D.2 (3 points): Assign each of the three animals to the most probable
class. Choose from the options given below and write appropriate letter in Q.
1.D.2. in the Answer Sheet.

Animal I:

Class: ____________

Animal II:

Class: ____________

Animal III:

Class: ____________

Options:
A. Mammalia
B. Reptilia
C. Aves
D. Amphibia
E. Pisces
16

IBO 2008
INDIA
PRACTICAL TEST 2
ANIMAL ANATOMY AND PHYSIOLOGY
__________________________________________________________________________

Task 2 (12 points)


Semi-quantitative estimation of nitrogenous waste products
You should try and complete this task in 15 minutes.
Materials

Quantity

1.

Porcelain spot plates, each with 6 cavities

2.

Toothpicks

20

3.

Permanent marker pen

4.

Tissue paper roll

5.

Container for wash and discard

6.

Reagents (given in a plastic box)


Label

Reagent

Phosphotungstic acid

Sodium carbonate (20% w/v)

Uric acid (standard solution)

Ehrlichs reagent

Urea (standard solution)

Sodium nitroprusside

Oxidizing solution

Phenol solution

Ammonia (standard solution)

S1

Simulated Sample 1

S2

Simulated Sample 2

S3

Simulated Sample 3

H2O

Distilled water

17

1 bottle each

IBO 2008
INDIA
PRACTICAL TEST 2
ANIMAL ANATOMY AND PHYSIOLOGY
__________________________________________________________________________

Introduction
Vertebrates have acquired different modes of excretion of nitrogenous wastes,
which are mostly derived from degradation of proteins and nucleic acids. They
have adopted different strategies of excretion of these wastes during their
transition from an aquatic to terrestrial mode of life. The three major forms of
these wastes are ammonia, urea and uric acid. While ammonia is highly
soluble in water, uric acid is the least soluble. Ammonia, being most toxic,
needs to be excreted in a highly diluted form. Uric acid is mostly excreted as
semisolid crystals.

Three simulated samples (S1, S2 and S3), representing nitrogenous wastes


from three groups of animals, are provided. Follow the protocols given below
to find out the relative levels of uric acid, urea and ammonia in these samples.

General Instructions

1. For each test, run a positive control and a negative control using
the standard solutions and distilled water, respectively.

2. Grade the colour of positive control as +++ and that of negative


control as .

3. Please note that the recording of the results for the positive and
negative controls carries NO marks.

18

IBO 2008
INDIA
PRACTICAL TEST 2
ANIMAL ANATOMY AND PHYSIOLOGY
__________________________________________________________________________

Protocols for estimation


1. Estimation of uric acid by phosphotungstic acid reduction method
Principle
Under alkaline conditions, uric acid reduces phosphotungstic acid to give a
blue- coloured product.
Method
(i)

Take three drops each of Samples S1, S2 and S3 in separate


cavities of a given spot plate.

(ii)

Add one drop each of solutions A followed by B to each cavity. Mix


with separate toothpicks and observe the developed colour.

(iii)

Grade the colour of the positive control as +++ and that of the
negative control as .

Q. 2.1.1. (3 points) Record the results in Table 2.1. in the Answer Sheet by
putting +++, ++ or + for positive results depending on the intensity of the
colour developed and for negative results.

2. Estimation of urea using Ehrlichs reagent


Principle
Under strong acidic conditions, urea reacts with Ehrlichs reagent (pdimethylaminobenzaldehyde) to form a yellow-coloured dye (protonated
Schiffs base).
Method
(i)

Take three drops each of Samples S1, S2 and S3 in separate


cavities of a given spot plate.
19

IBO 2008
INDIA
PRACTICAL TEST 2
ANIMAL ANATOMY AND PHYSIOLOGY
__________________________________________________________________________

(ii)

Add one drop of solution D to each cavity. Mix with separate


toothpicks.

Q. 2.1.2. (3 points) Record your results immediately in Table 2.1. in the


Answer Sheet by putting +++, ++ or + for positive results depending on
the intensity of the colour developed and - for negative results. For
comparison, grade the colour of the positive control as +++ and that of the
negative control as .

3. Estimation of ammonia by indophenol blue method


Principle
In an alkaline solution, ammonium ions react with hypochlorite to form
monochloramine. In the presence of phenol and an excess of hypochlorite,
the monochloramine forms a blue-coloured product, indophenol, when
nitroprusside is used as a catalyst.

Method
(i)

Take three drops each of Samples S1, S2 and S3 in separate


cavities of a given spot plate.

(ii)

Add one drop each of solutions F, followed by G and finally H to


each cavity. Mix with separate toothpicks.

20

IBO 2008
INDIA
PRACTICAL TEST 2
ANIMAL ANATOMY AND PHYSIOLOGY
__________________________________________________________________________

Q. 2.1.3. (3 points) Record your results after two minutes in Table 2.1. in
the Answer Sheet by putting +++, ++ or + for positive results depending
on the intensity of colour developed and - for negative results. For
comparison, grade the colour of positive control as +++ and that of negative
control as .
Table 2.1.

Samples

Uric acid test Urea test Ammonia test

S1
S2
S3
Positive control
Negative control

Q. 2. 2. (3 points): Based on the results obtained, match each of the samples


with the appropriate class of vertebrates listed below. Fill in your answer by
putting the appropriate alphabet in Q. 2.2. in the Answer Sheet.

Answer: ________

21

IBO 2008
INDIA
PRACTICAL TEST 2
ANIMAL ANATOMY AND PHYSIOLOGY
__________________________________________________________________________

a. S1: Pisces

S2: Mammalia

S3: Aves

b. S1: Mammalia

S2: Aves

S3: Pisces

c. S1: Mammalia

S2: Reptilia

S3: Aves

d. S1: Mammalia

S2: Pisces

S3: Aves

e. S1: Aves

S2: Pisces

S3: Mammalia

f. S1:Reptilia

S2: Amphibia

S3: Mammalia

g. S1: Mammalia

S2: Reptilia

S3: Amphibia

************ END OF PRACTICAL TEST 2 ************

22

IBO 2008
INDIA
PRACTICAL TEST 3
BIOCHEMISTRY AND CELL BIOLOGY
__________________________________________________________________________

Country: ________________

Student Code: ____________

19th INTERNATIONAL BIOLOGY OLYMPIAD


13th 20th July, 2008
Mumbai, INDIA

PRACTICAL TEST 3
BIOCHEMISTRY AND CELL BIOLOGY
Total Points: 43
Duration: 60 minutes

IBO 2008
INDIA
PRACTICAL TEST 3
BIOCHEMISTRY AND CELL BIOLOGY
__________________________________________________________________________

Dear Participants,

In this test, you have been given the following task:


Task 1: A: Study of -lactamase activity and its inhibition (35 points)
B: Correlating -lactamase expression to resistance (4 points)
C: Correlating Ki values of pesticides to bacterial growth (4 points)

You have to write down your results and answers in the ANSWER SHEET.
Answers written in the Question Paper will not be evaluated.

Please make sure that you have received all the materials and equipment listed
for the task. In case any of these items is missing, please raise the yellow card.

At the end of the test, put the Answer Sheet and Question Paper in the envelope.
The supervisor will collect this envelope.
Good Luck!!

Country: _________________________
Country Code: ____________________
First Name: _______________________
Middle Name: _____________________
Family Name: _____________________
Student Code: ____________________

IBO 2008
INDIA
PRACTICAL TEST 3
BIOCHEMISTRY AND CELL BIOLOGY
__________________________________________________________________________

Task 1
PART A (35 points)
Study of -lactamase activity and its inhibition

Materials and equipment

Quantity

1. Colorimeter, with a set of seven cuvettes

2. Test tubes

3. Test tube stand

4. Micropipette (10 100 l capacity)

5. Micropipette (100 1000 l capacity)

6. Micropipette tips (10 100 l capacity)

20

7. Micropipette tips (100 1000 l capacity)

20

8. Photographs of Petri plates

9. Permanent marker

10. Tissue paper roll

11. Wash bottle containing distilled water

12. Container for wash and discard

Reagents (please see the next page)

IBO 2008
INDIA
PRACTICAL TEST 3
BIOCHEMISTRY AND CELL BIOLOGY
__________________________________________________________________________

Label

Reagent

Container

Lactamase enzyme (1.85 mg/ml)

Vial

Inhibitor (100 mM)

Vial

Penicillin G (0.54 mM)

Blue-stoppered tube

Sodium phosphate buffer, pH 7.0 (10 mM)

Blue-stoppered tube

CuSO4-Neocuproine reagent

Blue-stoppered tube

HCl (2 M)

White-stoppered tube

Handling of micropipette:

Plunger
Tip-ejector

Display

Tip-holder

Figure 1

IBO 2008
INDIA
PRACTICAL TEST 3
BIOCHEMISTRY AND CELL BIOLOGY
__________________________________________________________________________

Adjustment method
Turn the plunger (Figure 1) to set the value to the desired volume, which can be
seen in the display.

Remember that each micropipette has a fixed range of volumes as indicated


on the pipette. DO NOT CROSS THE LIMITS OF THIS RANGE.

Usage method
Secure the pipette tip to the tip holder (Figure 1). Gently push down the plunger to
the first stop, hold, and dip the tip into the solution vertically to a depth of 2 - 4 mm.
Release the plunger slowly and make it return to the original position. Remove the
pipette from the liquid and transfer the contents to the desired tube. Make sure that
the tip is close to the inner wall of the tube. Push the plunger to the first stop and
then push further to discharge the solution completely from the tip. Remove the
pipette from the tube. Eject the used tip into the discard container by pressing the tipejector.

IBO 2008
INDIA
PRACTICAL TEST 3
BIOCHEMISTRY AND CELL BIOLOGY
__________________________________________________________________________

Operating Instructions for the colorimeter:


Figure 2

Wavelength selector

Cuvette holder

Mode selector

Set zero knob


Top view of colorimeter

Power Switch

Rear view of colorimeter

IBO 2008
INDIA
PRACTICAL TEST 3
BIOCHEMISTRY AND CELL BIOLOGY
__________________________________________________________________________

1) Turn the power switch (Figure 2) of the colorimeter ON.


2) Set the instrument to Absorbance mode (ABS) using the mode selector.
3) Set the wavelength to 465 nm using the wavelength selector.
4) Put the blank solution in a cuvette. Clean the outside surface of the cuvette with
tissue paper and insert it into the cuvette holder. Gently push the cuvette all the way
down.
5) Rotate the set zero knob to set the reading to zero. The instrument is now ready
for measuring the absorbance of the test solutions.

Introduction
Penicillins are antibiotics with a characteristic -lactam ring in their structure. This
antibiotic kills bacteria by inhibiting the cell wall synthesis. However, these molecules
are rendered inactive by some bacteria, which synthesize an enzyme called lactamase. These bacteria, which produce -lactamases, are resistant to penicillins.
Due to this, penicillin treatment is ineffective in patients infected with such resistant
bacteria. One approach to overcome this problem is to develop effective -lactamase
inhibitors.

The effectiveness of a -lactamase inhibitor can be evaluated by determining its IC50


and Ki values. The IC50 of an inhibitor is defined as the concentration of the inhibitor
required to inhibit the enzyme activity by 50 percent. The K i of an inhibitor is a
measure of its binding affinity for the enzyme.

IBO 2008
INDIA
PRACTICAL TEST 3
BIOCHEMISTRY AND CELL BIOLOGY
__________________________________________________________________________

Principle of -lactamase assay


-Lactamase inactivates penicillin by catalyzing the following reaction:

H
N

Lactamase

+ H2O

Penicillin G

H
N
O
O

S
N
H

Penicilloic acid

O
O

The penicilloic acid generated is complexed with CuSO 4 in the presence of


neocuproine. The yellow-colored product formed can be monitored by measuring its
absorbance at 465 nm using a colorimeter.

In this task, you will:

determine the IC50 value of a given inhibitor by generating a dose-response


curve, and

calculate the Ki value for the inhibitor.

A dose-response curve for the inhibitor is generated by measuring the activity of lactamase in the presence of varying concentrations of the inhibitor at a fixed
concentration of the substrate.

IBO 2008
INDIA
PRACTICAL TEST 3
BIOCHEMISTRY AND CELL BIOLOGY
__________________________________________________________________________

Q. 1.A.1. (18 points) Follow the protocol given below and enter the absorbance
values in Table 1.A.1. in the Answer Sheet.

I. Prepare the following reaction mixtures:

Test tube

Sodium

Inhibitor

-lactamase

Distilled

phosphate

(100 mM)

enzyme

water

buffer, pH 7.0
1

1.48 ml

20 l

1.46 ml

20 l

20 l

1.44 ml

40 l

20 l

1.42 ml

60 l

20 l

1.40 ml

80 l

20 l

1.38 ml

100 l

20 l

Blank

1.43 ml

50 l

20 l

II. Mix gently and incubate at room temperature for 5 minutes.


III. Add 1 ml of penicillin G (0.54 mM) to each tube and mix gently. Incubate at room
temperature for 10 minutes.
IV. Add 1.5 ml of the CuSO4-neocuproine reagent to each tube and mix gently.
Incubate at room temperature for 5 minutes.
V. Stop the color development by adding 100 l of HCl to each tube and mix gently.
VI. Set the colorimeter to 465 nm.

IBO 2008
INDIA
PRACTICAL TEST 3
BIOCHEMISTRY AND CELL BIOLOGY
__________________________________________________________________________

VII. Use the Blank to set the absorbance to zero.


VIII. Measure the absorbance values of the solutions in Test tubes 1 to 6, and enter
these values in the table. You should get any one absorbance reading
countersigned by the supervisor. To call the supervisor, raise the yellow
card.

Table 1.A.1.
Test tube

Absorbance

1
2
3
4
5
6

Data analysis and interpretation


Q. 1.A.2. (6 points)
I. Calculate the concentrations (in mM) of the inhibitor [I] in 2.5 ml of the enzyme
reaction in Test tubes 1 to 6 and enter these values in Table 1.A.2. in the Answer
Sheet.

10

IBO 2008
INDIA
PRACTICAL TEST 3
BIOCHEMISTRY AND CELL BIOLOGY
__________________________________________________________________________

II. Consider the absorbance values to be the rates of hydrolysis of penicillin G. Now
calculate vi/v0, where:
v0 is the rate of hydrolysis of penicillin G by -lactamase in the absence of the
inhibitor, and vi is the rate of penicillin G hydrolysis in the presence of the inhibitor.
Note that for Test tube 1, vi = v0.

Enter these values (up to two decimals) in Table 1.A.2. in the Answer Sheet.
Table 1.A.2.

Test tube

[I] (mM)

vi/v0

1
2
3
4
5
6

Q. 1.A.3. (5 points) Plot a graph of vi/v0 versus [I] in the Graph Paper attached to
the Answer Sheet.

11

IBO 2008
INDIA
PRACTICAL TEST 3
BIOCHEMISTRY AND CELL BIOLOGY
__________________________________________________________________________

Determination of the IC50 and Ki value of the inhibitor


Q. 1.A.4. (3 points) Determine the IC50 value by interpolation of the data points in
the graph. Write the value (up to two decimals) in the box in the Answer Sheet.

IC50 = _________mM

Q. 1.A.5. (3 points) Calculate the dissociation constant Ki of the inhibitor using the
equation:

S
IC50 K i 1

Km
where Km is the Michaelis-Menten constant of -lactamase for penicillin G and [S] is
the initial concentration of substrate (penicillin G) present in the enzyme reaction
mixture.
Assume the Km of -lactamase for penicillin G to be 0.05 mM. Write down your
answer (up to two decimals) in the box in the Answer Sheet.

Ki = _________mM

12

IBO 2008
INDIA
PRACTICAL TEST 3
BIOCHEMISTRY AND CELL BIOLOGY
__________________________________________________________________________

PART B (4 points)
Correlating -lactamase expression to resistance
When penicillin-resistant bacteria are grown in liquid culture media, -lactamase is
secreted into the medium. The supernatant of such a medium can be assayed for lactamase activity. Culture supernatants from four different organisms (P, Q, R and
S), which are suspected to be penicillin-resistant, were obtained and 20 l of each
was assayed for -lactamase activity. The corresponding absorbance values were
measured at 465 nm and are given in the table below.

Organism

Absorbance

0.090

0.450

0.075

0.220

These four organisms were tested for their resistance to penicillin G by the disc
diffusion plate assay as follows:
1. Each organism was separately inoculated into warm growth medium and
poured into a sterile Petri plate. On cooling, the medium solidified.
2. Filter paper discs impregnated with varying concentrations of penicillin G were
then placed on the surface of the medium.
3. The plates were incubated allowing penicillin to diffuse and organisms to
grow.

13

IBO 2008
INDIA
PRACTICAL TEST 3
BIOCHEMISTRY AND CELL BIOLOGY
__________________________________________________________________________

4. Organisms sensitive to penicillin will not be able to grow in the vicinity of the
antibiotic disc and hence, a clear zone will be obtained around the disc.

You have been given labeled photographs of six plates I - VI.


Plate I is a control plate showing uniform mat growth of organisms in the absence of
penicillin G.
Plate II is also a control plate that contains media without the growth of any
organism.
Plates III to VI show the growth of the four organisms in the presence of penicillin G.
2.5, 5, 7.5, 10 and 12.5 are the micrograms of penicillin G present in the respective
discs.

Q. 1.B.1. (4 points) Observe these plates and infer which organism is growing in
each plate. Write your answers in Table 1.B.1. in the Answer Sheet.

Table 1.B.1.
Plate

Organism

III
IV
V
VI

14

IBO 2008
INDIA
PRACTICAL TEST 3
BIOCHEMISTRY AND CELL BIOLOGY
__________________________________________________________________________

PART C (4 points)
Correlating Ki values of pesticides to bacterial growth
Four pesticides P1 to P4 are reversible inhibitors of an enzyme E that is essential for
the growth of a bacterium B. Their Ki values are given in the table below. Each of
these four pesticides is used in four geographically different
regions R1 to R4. The residual concentrations of these four pesticides in the
respective regions are also shown in the table below:

Region

R1

R2

R3

R4

Pesticide

P1

P2

P3

P4

Ki for the enzyme E

1 nM

5 nM

0.45 M 0.55 M

Residual concentration 60 nM 100 pM 30 nM

5.5 M

Q. 1.C.1. ( 4 points) Indicate whether the bacterium B would grow or not in each of
the four regions by putting tick marks ( ) in the appropriate boxes in the Table
1.C.1. in the Answer Sheet.

Table 1.C.1.
Region

R1 R2 R3 R4

Bacterium B grows
Bacterium B does not grow

************ END OF PRACTICAL TEST 3 ************

15

IBO 2008
INDIA
PRACTICAL TEST 4
ANIMAL BEHAVIOR
__________________________________________________________________________

Country: ________________

Student Code: ____________

19th INTERNATIONAL BIOLOGY OLYMPIAD


13th 20th July, 2008
Mumbai, INDIA

PRACTICAL TEST 4
ANIMAL BEHAVIOR
Total Points: 48
Duration: 60 minutes

IBO 2008
INDIA
PRACTICAL TEST 4
ANIMAL BEHAVIOR
__________________________________________________________________________

Dear Participants,

In this test, you have been given the following two tasks:
Task 1: Part A: Study of the olfactory response of Drosophila melanogaster
larvae: Experimental design (6 points)
Part B: Study of the olfactory response of Drosophila melanogaster
larvae: Larval plate test (18 points)
Part C: Study of olfactory adaptation in Drosophila larvae (11 points)
Task 2: Study of fish behavior (13 points)

The duration of the test is 10 minutes for Part A of Task 1 and 50 minutes for
the remaining paper.

You have to answer Part A in 10 minutes after which the buzzer will ring
and the Answer Sheet for Part A will be collected from you. Only then will
the Question Paper and Answer Sheet for Task I Parts B and C, and Task
2 be given to you.

Please do not switch on your computer before your Answer Sheet for Part
A is collected.

You have to write down your results and answers in the ANSWER SHEET.
Answers written in the Question Paper will not be evaluated.

At the end of the test, put both the Question Papers as well as the Answer Sheet
for Task 1 Parts B and C, and Task 2 in the envelope. The supervisor will
collect this envelope.
Good Luck!!

IBO 2008
INDIA
PRACTICAL TEST 4
ANIMAL BEHAVIOR
__________________________________________________________________________

Country: ________________________
Country Code: ____________________
First Name: _______________________
Middle Name: _____________________
Family Name: _____________________
Student Code: ____________________

IBO 2008
INDIA
PRACTICAL TEST 4
ANIMAL BEHAVIOR
__________________________________________________________________________

Practical Test 4
Animal Behavior
Task 1 Part A (6 points)
Study of the olfactory response of Drosophila melanogaster larvae
Experimental design
You have been given Part A of this task. You have to answer this part in
10 minutes after which the buzzer will ring and the Answer Sheet for this
part will be collected from you. Only then will the rest of the paper be
given to you.

Introduction
Insects have a strong sense of smell. Adult moths, for example, can find their mates
by smelling pheromone molecules at very low concentrations. The sense of smell is
associated with a discriminatory behavior as well. This is evident from the fact that
insects are able to choose their food by odor. The nature of an odor stimulus can be
categorized into three types: (1) attractive, (2) repulsive, and (3) neutral.

The odor discriminatory behavior of insects such as Drosophila melanogaster, the


common fruit fly, can be assessed using either the adults or the larvae. Drosophila
larvae respond to odor stimuli by crawling either towards or away from them. It is,
therefore, possible to design an experiment to test the larval response towards
different chemicals on a Petri plate.

IBO 2008
INDIA
PRACTICAL TEST 4
ANIMAL BEHAVIOR
__________________________________________________________________________

Q. 1.A.1. (3 points) Suppose you want to determine the response of Drosophila


melanogaster larvae to three chemical odorants T1, T2, and T3. Five possible
experimental designs for this purpose are given below:
Design I: All the three chemicals are placed at equidistant positions on the periphery
of a Petri plate and the larvae are introduced at the centre.
T2

T1
Larvae

T3

Design II: One of the chemicals and the larvae are placed together at the centre of a
plate. Three such plates are set up for the three chemicals.

T1 and Larvae

T2 and Larvae

T3 and Larvae

Design III: One of the chemicals and an odor-free (neutral) chemical are placed at
two ends of a plate. Larvae are introduced in the centre. The test is repeated for the
remaining chemicals.

IBO 2008
INDIA
PRACTICAL TEST 4
ANIMAL BEHAVIOR
__________________________________________________________________________

T1

Neutral T2

Neutral T3
Larvae

Larvae

Neutral
Larvae

Design IV: The three test chemicals and a neutral chemical are placed at equidistant
positions on the periphery of a plate and the larvae are introduced at the centre.
T2

Neutral

T1
Larvae

T3

Design V: The three test chemicals and the larvae are placed at equidistant
positions on the periphery of a plate.
T2

Larvae

T1

T3

Choose the most appropriate experimental design and put a tick mark () against it
in Q. 1.A.1. in the Answer Sheet.

Design I
6

IBO 2008
INDIA
PRACTICAL TEST 4
ANIMAL BEHAVIOR
__________________________________________________________________________

Design II
Design III
Design IV
Design V
Please note that the next question (Q. 1.A.2.) will be evaluated only if your
answer to this question (Q. 1.A.1.) is correct.

Q. 1.A.2. (3 points) Choose the most appropriate statement/s from the following
options to support your choice of design.
I.

It allows the larvae to choose between two or more different chemicals


presented simultaneously and thus acts as a discriminatory test.

II.

It can clearly distinguish between attractants and repellants by testing


them one at a time against the neutral chemical.

III.

The entire experiment (i.e., testing all the chemicals) can be completed
using a single test and thus inter-experimental variation can be avoided.

IV.

It can clearly distinguish the repulsive and attractive nature of the stimuli
as each can enhance the response of the larvae to the other(s).

V.

There will not be any mixing of the odors and hence more reliable results
will be obtained.

VI.

All the chemicals can be tested against the same control in a single plate.

VII.

It can effectively test the odorant even if it is weak in nature.

VIII.

Larvae can disperse in any direction without any hindrance.

IBO 2008
INDIA
PRACTICAL TEST 4
ANIMAL BEHAVIOR
__________________________________________________________________________

a.

I, III, IV

b.

V, VII, VIII

c.

I, III, IV, VI, VIII

d.

II, V, VII, VIII

e.

I, III, IV, VIII

a.

b.

c.

d.

e.

Put a tick mark () in the appropriate box in Q. 1.A.2. in the Answer Sheet.

**********END OF PART A**********

IBO 2008
INDIA
PRACTICAL TEST 4
ANIMAL BEHAVIOR
__________________________________________________________________________

Practical Test 4
Animal Behavior
Task 1 Parts B and C
Study of the olfactory response of Drosophila melanogaster larvae
You should try and complete Parts B and C of this task in 35 minutes.

Part B (18 points)


Larval plate test

Design
Five experiments were conducted to test the response of Drosophila melanogaster
larvae to chemical and light stimuli. Four chemical odorants, A, B, C, and D were
used in the tests. Of these, D was known to be a neutral chemical while A, B, and C
could be an attractant, repellent or a neutral chemical. The design for the experiment
is as shown in the figure:

Filter paper
disc with
chemical

Filter paper
disc with
chemical

Larvae

Zone II

Zone I

Neutral Zone (N)

IBO 2008
INDIA
PRACTICAL TEST 4
ANIMAL BEHAVIOR
__________________________________________________________________________

Method
Third instar larvae were used in these experiments. These larvae were obtained by
washing the 6-day-old Drosophila culture with 15% sucrose solution. The larvae that
float in this solution were washed free of sucrose and immediately used in the tests.
The tests were performed in Petri plates containing a layer of 1% agarose.

In each experiment, two chemicals were spotted on separate filter paper discs that
were placed in two Zones, Z I and Z II (marked as semicircular areas) at two
diametrically opposite ends of a Petri plate. Approximately 30-40 larvae were placed
at the centre of each plate and their movement over the next five minutes was
recorded. Five such experiments were conducted. The recordings of these
experiments have been provided to you as video films. Experiments No. 1, 2, and 4
were conducted under uniform light conditions. In Experiments No. 3 and 5, half the
plate was covered with black paper and the remaining half was left exposed to light.

Q. 1.B.1. (10 points): Observation of video films


1. Double click on the video file labelled 1 on the computer monitor to observe
the movement of larvae.
2. The duration of the video film is 5 minutes, compressed to 2.5 minutes. You
may forward or rewind the video film, if required.
3. At the end of the experiment, count the number of larvae in Zone I (N ZI) and
Zone II (NZII).
4. Record your readings in Table 1.B.1. in the Answer Sheet.
5. Repeat steps 1 to 4 for the video files labelled 2 to 5.

10

IBO 2008
INDIA
PRACTICAL TEST 4
ANIMAL BEHAVIOR
__________________________________________________________________________

Table 1.B.1.
Experiment

Chemical

Number of

Chemical Number of

in ZI

larvae in ZI

in ZII

(NZI)

larvae in ZII

N ZI
N ZI N ZII

N ZII
N ZI N ZII

(NZII)

A (in dark)

B (in dark)

Q. 1.B.2. (3 points) What is the likely nature of the three chemicals A, B and C? Put
a tick mark () in the appropriate box in the Answer Sheet.
Chemical Attractant Repellant Neutral
chemical

Nature cannot be
determined

A
B
C

Q. 1.B.3. (5 points) Based on your observations, mark whether the following


statements are true or false by putting a tick mark () in the appropriate box in the
Answer Sheet.

11

IBO 2008
INDIA
PRACTICAL TEST 4
ANIMAL BEHAVIOR
__________________________________________________________________________

a. Larvae exhibit a stronger positive movement towards the attractant odorant


tested than towards darkness.
b. Light is a stronger repelling stimulus for the larvae than the repellant odorant
tested.
c. The positive phototaxis shown by the larvae is stronger than the movement
towards the attractant odorant.
d. In the presence of light, the larvae do not exhibit chemotaxis.
e. The repellant odorant has a stronger influence on the larvae than does
darkness.
True
a.
b.
c.
d.
e.

12

False

IBO 2008
INDIA
PRACTICAL TEST 4
ANIMAL BEHAVIOR
__________________________________________________________________________

Part C (11 points)


Study of olfactory adaptation in Drosophila melanogaster larvae

Continuous stimulation of the olfactory system with a given odor tends to result in
adaptation, also known as desensitization. As a result, the larvae fail to respond to
the odor to which they have been adapted. A researcher working on olfaction in
Drosophila melanogaster larvae wanted to study adaptation in these larvae. She
selected the following odorants for her study:
1.

Ethyl acetate

2.

Pentyl acetate

3.

Hexyl acetate

4.

Heptyl acetate

Pre-stimulation experiment: In each experiment (except in Experiment 1), the


larvae were pre-stimulated by placing them in a Petri plate containing 40 microlitres
of one of the above odorants for 25 min. The same larvae were then picked up from
this plate and tested for their response to the same or different odorants using the
protocol described earlier in Part B of Task 1.
The data obtained from these tests are tabulated below.

13

IBO 2008
INDIA
PRACTICAL TEST 4
ANIMAL BEHAVIOR
__________________________________________________________________________

Data from the pre-stimulation experiment


Experiment

Prestimulation

Test odorant
Experiment

Experiment

Experiment

Ethyl
acetate

Pentyl
acetate

Hexyl
acetate

Experiment
D
Heptyl
acetate

NZ I

NZ II NZ I

NZ II

NZ I

NZ II

NZ I

NZ II

None

21

18

14

12

13

Ethyl

14

11

15

11

13

10

15

16

15

12

11

19

14

17

17

14

16

13

13

15

10

13

13

10

13

acetate
3

Pentyl
acetate

Hexyl
acetate

Heptyl
acetate

NZI and NZII are the number of larvae in Zone I and Zone II, respectively.

* Zone II in all the experiments contained a neutral chemical.


The data given in the table are a set of average responses. The actual numbers
varied up to 10% on either side of the average.

14

IBO 2008
INDIA
PRACTICAL TEST 4
ANIMAL BEHAVIOR
__________________________________________________________________________

Q. 1.C.1. ( 5 points) Calculate the Response Index (RI) for each experiment
according to the formula:

RI

N ZI N ZII
100
N ZI N ZII

Fill in the RI values in Table 1.C.1. in the Answer Sheet.

Table 1.C.1.

Experiment Prestimulation

None

Ethyl acetate

Pentyl acetate

Hexyl acetate

Heptyl acetate

Test odorant
Experiment

Experiment

Experiment

Experiment

Ethyl

Pentyl

Hexyl

Heptyl

acetate

acetate

acetate

acetate

RI

RI

RI

RI

15

IBO 2008
INDIA
PRACTICAL TEST 4
ANIMAL BEHAVIOR
__________________________________________________________________________

Q. 1.C.2. (2 points) To which odorant have the larvae adapted the most?
Put a tick mark () in the appropriate box in the Answer Sheet.

Ethyl acetate
Pentyl acetate
Hexyl acetate
Heptyl acetate

Q. 1.C.3. ( 2 points) To which odorant have the larvae adapted the least?
Put a tick mark () in the appropriate box in the Answer Sheet.

Ethyl acetate
Pentyl acetate
Hexyl acetate
Heptyl acetate

16

IBO 2008
INDIA
PRACTICAL TEST 4
ANIMAL BEHAVIOR
__________________________________________________________________________

Q.1.C.4. (2 points) In which one of the experiments do you find that larval sensitivity
to the odorant has been reversed?
Put a tick mark () the appropriate box in the Answer Sheet.

Experiment

Experiment
A

1
2
3
4
5

17

IBO 2008
INDIA
PRACTICAL TEST 4
ANIMAL BEHAVIOR
__________________________________________________________________________

Task 2 (13 points)


Study of fish behavior
You should try and complete this task in 15 minutes.

Introduction
The Siamese fighting fish, Betta splendens, is one of the most popular species of
freshwater aquarium fish. This fish shows varied responses when exposed to
different stimuli.

Operculum

You have been provided with a video recording of an experiment using a male
Siamese fighting fish.

Q. 2.1. (11 points) Double click on the video file 6 on the computer monitor and
observe the behavior of the fish before and after introduction of a mirror.

18

IBO 2008
INDIA
PRACTICAL TEST 4
ANIMAL BEHAVIOR
__________________________________________________________________________

After observing the film, write a + for the particular behaviors that were displayed by

the fish and for those that were not displayed. Record your observations in Table
2.1. in the Answer Sheet.

Table 2.1.
Number Behavior

1.

Pectoral fin beating

2.

Rapid zigzag movement of the body and the

Before the

After the

introduction of

introduction of

the mirror

the mirror

tail fin
3.

Appearance of horizontal stripes on body

4.

Pecking at the base of the aquarium

5.

Brightening of body coloration

6.

Erection of dorsal, anal and caudal fin

7.

Appearance of vertical stripes on body

8.

Operculum display (opening of the operculum)

9.

Bleaching of body color

10.

11.

Lateral display

Gasping for air

*The lateral display is a behavior where the fish exhibits the lateral surface of its
body, expands its dorsal and caudal fins and vibrates/quivers its body.
19

IBO 2008
INDIA
PRACTICAL TEST 4
ANIMAL BEHAVIOR
__________________________________________________________________________

Q. 2.2. (1 point) The differences in behavior of the fish that you observed before and
after the mirror was introduced into the aquarium could be due to:

a. an apparent increase in the size of the territory that the fish now has to defend.
b. an urge to display courtship behavior towards a conspecific individual that the fish
now perceives in its territory.
c. an urge to establish dominance over a conspecific individual that the fish now
perceives in its territory.
d. a startle response displayed by the fish when confronted with a mirror.

Put a tick mark () in the appropriate box in Q. 2.2. in the Answer Sheet.

a.

b.

c.

d.

Q. 2.3. (1 point) Different behaviors in animals have certain benefits and costs
associated with them. For example, prolonged extension of the gill cover or
operculum display may indicate its physical strength but may also severely limit the
ability of the fish to ventilate its gills. In the light of your observation, what could be
the rationale for the experimental fish displaying or not displaying this particular
behavior?

20

IBO 2008
INDIA
PRACTICAL TEST 4
ANIMAL BEHAVIOR
__________________________________________________________________________

a. Fish always prefer to maintain regular opercular movement without any display,
independent of the presence or absence of another conspecific individual, to
maintain the oxygen supply for the body at its optimum.
b. Fish will exhibit the operculum display advertising its ability to tolerate oxygen
stress in presence of another conspecific individual to establish its dominance.
c. Operculum display, being an energetically costly behavior, is usually not
exhibited by a fish under most circumstances. Males of this species, however,
may display this behavior in the presence of a conspecific female because the
potential reproductive success that it will acquire will more than compensate for
the energetic cost of the display.
d. Operculum display is likely to be determined only by abiotic environmental factors
such as level of dissolved oxygen in the water. Thus, fish in sufficiently aerated
water will always show this response so as to declare its territory and maintain its
dominance.

Put a tick mark () in the appropriate box in Q. 2.3. in the Answer Sheet.

a.

b.

c.

d.

************ END OF PRACTICAL TEST 4 ************

21

INTERNATIONAL BIOLOGY OLYMPIAD


THEORY PROBLEMS

2007, Saskatoon, Canada















All IBO examination questions are published under the following Creative Commons license:



CC BY-NC-SA (Attribution-NonCommercial-ShareAlike) https://creativecommons.org/licenses/by-nc-sa/4.0/
The exam papers can be used freely for educational purposes as long as IBO is credited and
new creations are licensed under identical terms. No commercial use is allowed.

Final Report &


Planning Suggestions

Part Two: Exams, Answers & Stats


(Report Not Included)

Nov. 2007

18th INTERNATIONAL BIOLOGY OLYMPIAD


JULY 15 - 22, 2007

PRACTICAL EXAMINATION 1
ANIMAL ANATOMY, SYSTEMATICS AND ECOLOGY

This examination is composed of 3 tasks.


TASK A: Dissection of two annelids

26 marks

TASK B: Identification of annelids using a dichotomous key

10 marks

TASK C: Defining the structures, body plan, life style and


classification of 10 worm-like animals.

27 marks

TOTAL MARKS = 63

TOTAL TIME AVAILABLE = 90 minutes

IBO 2007. Practical Exam 1 - Animal Biology

GENERAL INSTRUCTIONS

Before starting the exam, the invigilator will show you a red card and a green
card to test for red-green color blindness. If you are unable to see the difference
between the two cards, raise your hand, and you will be provided with
assistance immediately.

Read the exam paper carefully before commencing the exam.

It is recommended that you allocate your time according to the mark value of each
task and question.

IMPORTANT INFORMATION FOR TASK A


You must commence with Task A. When Task A is completed, raise your hand
and a lab assistant will take a photo of your dissections, record the time, sign the
pan labels and remove the dissections for marking.

IMPORTANT INFORMATION FOR TASKS B AND C


!

All answers for Tasks B and C must be recorded in the answer booklet
provided.

Ensure that your 4-digit student code number is written on ALL pages of
your answer booklet.

Use the pencil provided to fill in the appropriate circle for each question in
the answer booklet.

IBO 2007. Practical Exam 1 - Animal Biology

Task A. Annelid Dissection (26 marks)


Objective: To locate key features in a marine and a terrestrial annelid.
Materials:
! dissecting tray containing annelid 1 (tray labeled with blue sticker)
! dissecting tray containing annelid 2 (tray labeled with yellow sticker)
! 1 pair of dissecting scissors
! 1 pair of forceps
! 1 scalpel
! 20 steel pins on foam board
! 14 colored pins on foam board (2 red-orange, 2 blue, 2 yellow, 2 black, 2 white, 2 pink,
2 green)
! 1 pair disposable gloves
! 1 dissecting microscope and external lamp
! 2 specimen cards (1 labeled with blue sticker, the other labeled with yellow sticker)
! water bottle for keeping specimens wet
! 15 cm ruler from student pencil case
NOTE: Before beginning your dissection, ensure that you have all of the materials listed above. If
you do not, immediately notify a lab assistant by raising your hand. After all materials lists are
confirmed, timing will begin.
Procedure:
1.

Fill out each of the two specimen cards with your student number and name and set aside. You
will sign these cards upon completion of your dissections.

2.

Put on your gloves and remove the wet paper towel that is covering the specimen.
Throughout the dissection, use the water bottle to regularly wet your specimen and any
parts removed. This will ensure that the parts do not dry out.

3.

Note the differences in the external features of each worm, namely the increased number of
sensory structures and the presence of multifunctional appendages on annelid 1.

4.

From the mid portion of the body of annelid 1, detach an entire parapodium. Parapodia
function as limbs and gills for the worm. Details of the parapodia allow zoologists to
distinguish between different species of this annelid. Each parapodium consists of a ventral
division called the neuropodium and a bilobed dorsal division called the notopodium. Each
notopodium is supported by a chitinous and stiff rod called an aciculum. A dorsal and a
ventral cirrus project

IBO 2007. Practical Exam 1 - Animal Biology

from the notopodium and the neuropodium, respectively. Setae extend beyond the parapodia.
5.
Use the pins provided to pin the detached parapodium in one corner of the annelid 1
dissecting
pan. Ensure that it is pinned on wet paper towel. Pin as follows:
! red-orange pin for the neuropodium

(2 marks)

! blue pin for the notopodium (2 marks)


* Before continuing, use the water bottle to moisten the parapodium & cover it with a wet
piece of paper towel *
st

6.

Stretch out each worm in its dissecting pan, dorsal side up. Place one steel pin through the 1
segment of the body and one pin through the last segment of the body to secure it in place.

7.

Cut open the body wall of annelid 1 from the anterior tip down the body 3-5 cm. Separate
the body wall from the internal structures and pin the body wall to the dissecting tray by using
the steel pins.

8.

Cut open the body wall of annelid 2 from the anterior tip, and continue the cut posteriorly
approximately 5 cm. Separate the body wall from the internal structures. To open up the
worm, pin the body wall to the dissecting tray by using the steel pins.

9.

Starting at the anterior end of each worm, locate the muscular pharynx. In annelid 1 the
pharynx also contains jaws that are useful in its predatory lifestyle. In both specimens, pin
the following structure:

10.

11.

! yellow pin for the pharynx on annelid 1

(2 marks)

! yellow pin for the pharynx on annelid 2

(2 marks)

Moving posteriorly in both specimens, locate the long and tubular intestine used in
digestion. In both specimens, pin the following:
! black pin for the intestine on annelid 1

(2 marks)

! black pin for the intestine on annelid 2

(2 marks)

Other major features of the annelid digestive system can be seen in annelid 2. Immediately
posterior to the reproductive organs in annelid 2 lie the soft crop and the tougher-walled
gizzard. In annelid 2, pin the following:
! pink pin for the crop on annelid 2

(2 marks)

! green pin for the gizzard on annelid 2

(2 marks)

IBO 2007. Practical Exam 1 - Animal Biology

12.

13.

Both annelids possess a closed circulatory system with tubular hearts and a dorsal and ventral
blood vessel. In both specimens, pin the following:
! white pin for the dorsal blood vessel on annelid 1

(2 marks)

! white pin for the dorsal blood vessel on annelid 2

(2 marks)

Although both specimens are annelids, annelid 1 is sexually dioecious, whereas annelid 2 is
hermaphroditic. Hermaphroditism is an advantage for this slow-moving organism. Examine
the anterior internal structures in annelid 2, and any external features found on the body wall.
In annelid 2 only, pin the following:
! plain steel pin for clitellum

(2 marks)

! red- orange pin for seminal vesicle

(2 marks)

! blue pin for seminal receptacle

(2 marks)

14. After finishing the task, place a wet paper towel over the dissected specimens. Raise your
hand. A lab assistant will take a photo of your dissection. Both the lab assistant and
yourself will sign your dissection pan labels and record the time. Your dissection will then
be taken in and graded as you move onto the next section of the practicum.

IBO 2007. Practical Exam 1 - Animal Biology

Task B. Identification of annelids using a dichotomous key (10 marks)


Objective: To use a dichotomous key to identify ten annelids to the genus-level.
Materials:
! line drawings of 10 annelids (labeled as 1 to 10). ALL of the organisms are drawn in the
SAME orientation
Procedure:
Use the dichotomous key below to identify the genus to which each annelid belongs. Indicate your
selections in the answer booklet by filling in the most appropriate circle for each annelid.

Dichotomous Key
1a. Has a prominent posterior sucker ......
1b. Lacks a posterior sucker ........

go to 2
go to 3

2a. Posterior half of body much wider than the anterior end ..
2b. Body more ribbon like, anterior part tapered ....

Glossiphonia
Eropobdella

3a. Has a prominent clitellum ......


3b. Clitellum absent .....

Lumbricus
go to 4

4a. Each segment has a pair of lateral appendages (parapodia) ......


4b. Parapodia are reduced, modified and/or not present on each segment ..

go to 5
go to 8

5a. Worm bears dorsal scales (elytra) ........


5b. Worm lacks dorsal scales .......

Lepidontus
to 6

6a. More than 15 body segments.........


6b. Less than 15 body segments; prostomium with a pair of club-shaped palps.

go to 7
Nerillidopsis

7a. Segment 2 bears a pair of long parapodial cirri .


7b. Lacks long parapodial cirri on segment 2 .....

Tomopteris
Nereis

8a. Possesses numerous tentacles ....


8b. Lacks tentacles ......

Neoamphitrite
go to 9

9a. Parapodia of the mid-body region modified as tufted branchia (gills) .


9b. Body divided into distinct regions; anterior end modified for filter-feeding....

Arenicola
Chaetopterus

IBO 2007. Practical Exam 1 - Animal Biology

Task C. Form and function of worm-like animals (27 marks)


Introduction
The following 10 animals all resemble worms in habit or appearance based on their general
tubular or worm-like body plans. Most people without scientific training would initially use the
term worms to describe these ten animals but with our zoological knowledge we know that these
animals actually belong to several very different phyla and are only related superficially by their
worm-like body plan. These 10 animals have structural characteristics that are adapted to their
particular environments and life styles.

Objective: Using the pictures provided, determine which adaptations (form) these animals have
that helps them in their environment and life styles (function)

Materials:
! laminated, colour photographs of 10 animals (labeled A to J). Note: there are two
photographs of each animal.

Procedure:
There are two parts to this task. Fill in the tables in your answer booklet.
1.

In Part I, select the best response for each of 6 characteristics (body shape; structures used in
locomotion or for attachment to a host; structures used in feeding; type of digestive tract; body
segmentation; type of sensory structures) from the choices provided.

2.

In Part 2, use your observations from Part 1 to select the best response from the choices
provided for the life style of each animal, the phylum to which it belongs and its body plan.
For each part, indicate your choices by filling in the circles in the appropriate section of the
answer booklet.

- THE END

1
2

3
4

Images for Task B

10

ANSWER BOOKLET FOR PART 1: TASKS B & C

Do not write in the box below. For examiners use only.


Task/Part
B
C-I
C-2
Total

Marks

STUDENT NUMBER: ________________

Task B (Dichotomous key to annelid worms)


Indicate the genus for each of the ten specimens (labeled 1-10) by filing in the most appropriate circle

Specimen number:
1

10

Glossiphonia

Eropobdella

Lumbricus

Lepidontus

Nerillidopsis

Tomopteris

Nereis

Neoamphitrite

Arenicola

Chaetopterus

STUDENT NUMBER: ________________

Task C PART 1 (Characteristics of worm-like organisms; A to J)


Select the best response for each characteristic. Total marks = 12 (0.2 marks/specimen/characteristic)

characteristic 1
Body Shape:
Specimen
flattened
not flattened
A
B
C
D
E
F
G
H
I
J

Specimen
A
B
C
D
E
F
G
H
I
J

O
O
O
O
O
O
O
O
O
O

characteristic 2
Structures used in locomotion or for attachment to a host:
muscular
jointed
sucker, scolex
none of the
foot
appendages
and/or teeth other choices

O
O
O
O
O
O
O
O
O
O

O
O
O
O
O
O
O
O
O
O

O
O
O
O
O
O
O
O
O
O

characteristic 3
Structures used in feeding:
mandibles
sucker
radula none of the
or teeth
other choices
O
O
O
O
O
O
O
O
O
O

O
O
O
O
O
O
O
O
O
O

O
O
O
O
O
O
O
O
O
O

O
O
O
O
O
O
O
O
O
O

none

O
O
O
O
O
O
O
O
O
O

O
O
O
O
O
O
O
O
O
O

characteristic 4
Digestive tract:
incomplete
complete
(single opening for (separate openings
mouth and anus)
for mouth and anus)

O
O
O
O
O
O
O
O
O
O

O
O
O
O
O
O
O
O
O
O

O
O
O
O
O
O
O
O
O
O

STUDENT NUMBER: ________________

Task C PART 1 continued

Do not write in the boxes below. For examiners use only.


Question No. correct
1
2
3
4
5
6
Total
X 0.2 =

/12

STUDENT NUMBER ______________

Task C PART 2 (Characteristics of worm-like organisms; A to J)


Select the best response for each characteristic. Total marks = 15 (0.5 marks/specimen/characteristic)
characteristic 7
Specimen
A
B
C
D
E
F
G
H
I
J

Specimen
A
B
C
D
E
F
G
H
I
J

characteristic 8

Lifestyle:
Specimen belongs to the phylum
parasitic non-parasitic
free-living Arthropoda Nematoda Mollusca Annelida Platyhelminthes Other
O
O
O
O
O
O
O
O
O
O

O
O
O
O
O
O
O
O
O
O

O
O
O
O
O
O
O
O
O
O

O
O
O
O
O
O
O
O
O
O

O
O
O
O
O
O
O
O
O
O

O
O
O
O
O
O
O
O
O
O

O
O
O
O
O
O
O
O
O
O

O
O
O
O
O
O
O
O
O
O

characteristic 9
Body plan:
acoelomate pseudocoelomate coelomate
O
O
O
O
O
O
O
O
O
O

O
O
O
O
O
O
O
O
O
O

O
O
O
O
O
O
O
O
O
O

Do not write in the


boxes below. For
examiners use only.
Question

No. correct

7
8
9
Total
X 0.5 =
/15
5

18th INTERNATIONAL BIOLOGY OLYMPIAD


JULY 15 - 22, 2007

PRACTICAL EXAMINATION 2
PLANT ANATOMY, MORPHOLOGY AND PHYSIOLOGY
EXAM BOOKLET 1

TASK A.

Identification of plant structures and organs

16 marks

Time allowed: 20 minutes

WRITE YOUR 4-DIGIT STUDENT NUMBER IN THE BOX


BELOW

STUDENT CODE

IBO 2007. Practicum 2 - Plant Biology, Booklet 1

STUDENT CODE _____________________

PLANT MORPHOLOGY
TASK A.

Identification of plant structures and organs from images shown in a PowerPoint


presentation (16 marks)

In this task, you are required to answer the following questions, each of which relates to a slide that
you will be shown. Each slide will be shown twice.
In the first showing, each slide will be displayed for 45 seconds, then the second slide will be
shown for 45 seconds and so on until all 16 slides have been once. The second showing is to give
you the opportunity to review your answers. In this showing, each slide will be displayed for 15
seconds.

FOR EACH QUESTION,


WRITE THE LETTER OF YOUR ANSWER IN THE SPACE PROVIDED

1. What mutualistic relationship between roots of land plants and specific soil fungi is displayed in
this slide?
a. mycorrhizae
b. mycelium
c. lichens
ANSWER: ___a____
d. root hairs
2. This leafs venation is commonly found in which group of plants?
a. hornworts
b. dicotyledons
c. ferns
d. monocotyledons
ANSWER: ____b___
e. gymnosperms
3. The aerenchyma stem tissue shown here is characteristic of its adaptation as a:
a) mesophyte
b) xerophyte
c) halophyte
ANSWER: ____d___
d) hydrophyte

Plant Biology Lab Exam, Booklet 1

STUDENT CODE ___________________

4. What type of plant do these leaf cross-sections represent?


a) a monocot
b) a lycopod
c) a eudicot
d) a tree
ANSWER: ____a____
e) a fern

5.

In this picture of a fern sorus, what is the ploidy level of the structure indicated by the arrow?
a) triploid
b) diploid
ANSWER: ____c____
c) haploid

6. In this longitudinal section of a dicot angiosperm stem, name the structure indicated by X.
a) shoot apical meristem
b) axillary bud
c) lateral inflorescence
d) lateral root
ANSWER: _____b____
e) leaf primordium

7. The arrow in this slide is indicating:


a) sclerenchyma fibre
b) sieve tube element
c) vessel element
d) chlorenchyma
e) sclereid

ANSWER: _____c_____

8. What is the function(s) of the structure indicated by the arrow?


a) to prevent an insect proboscis reaching phloem sap
b) to provide mechanical support to the xylem tissue
c) to initiate the formation of interfascicular vascular cambium
d) all of the above
ANSWER: _____a______
e) none of the above

IBO 2007. Practicum 2 - Plant Biology, Booklet 1

STUDENT CODE _____________________

9. The name of the meristem responsible for generating the tissues labelled X is:
a) vascular cambium
b) shoot apical meristem
c) root apical meristem
d) cork cambium
ANSWER: ____d_____
e) lenticel

10. The presence of the following illustrated cells gives Pyrus communis L. (pears) their gritty
texture:
a) chlorenchyma
b) guard cell
c) tracheary element
d) collenchyma
ANSWER: ____e_____
e) sclereid

11. This type of phyllotaxy is best described as:


a) whorled
b) distichous
c) opposite
d) alternate

ANSWER: ____a_____

12. The hormone auxin, produced at the terminal meristem, inhibits lateral shoot growth which
results in the illustrated phenomenon, known as:
a) conical shape
b) deciduousness
c) apical dominance
ANSWER: _____c____
d) axillary dominance
13. These tendrils and spines represent evolutionary adaptations of what structures?
a) leaves
b) branches
c) axillary shoots
d) adventitious roots
ANSWER: _____a_____
e) trichomes

Plant Biology Lab Exam, Booklet 1

STUDENT CODE ___________________

14. Which plant cell type shown has the potential to give rise to all of the other cells in the
section?
a) parenchyma
b) companion
c) collenchyma
d) sclereid
ANSWER: _____a_____
e) tracheary element

15. In embryo development, the structure indicated by the arrow is known as the:
a) filament
b) endosperm
c) heart-shaped embryo
d) suspensor
ANSWER: _____d____
e) basal cell

16. What plant group does this slide represent?


a) angiosperms
b) bryophytes
c) tracheophytes
d) pteridophytes

ANSWER: _____b_____

- THE END

HAVE YOU WRITTEN YOUR STUDENT CODE ON THE FIRST PAGE OF


THIS EXAM BOOKLET AND ON THE TOP OF EVERY OTHER PAGE?

INTERNATIONAL BIOLOGY OLYMPIAD 2007

#1

#2
#2

#3

#4

#5
#5

#6
#6

#7
#7

#8
#8

#9
#9

#10
# 10

#11
# 11

#12
# 12

##13
13

# 14
#14

#15
# 15

#17
# 16

18th INTERNATIONAL BIOLOGY OLYMPIAD


JULY 15 - 22, 2007

PRACTICAL EXAMINATION 2
PLANT ANATOMY, MORPHOLOLOGY AND PHYSIOLOGY
EXAM BOOKLET 2
Task B. Identification of flowering plants

23 marks

Task C. Dissection of a seed and a flower

25 marks

Task D. Plant evolution

5 points

Task E. Graphing and interpretation of data

8 marks

Time allowed:

70 minutes

(Total time allowed for Practical Examination 2 = 90 minutes)

WRITE YOUR 4-DIGIT STUDENT CODE IN THE BOX BELOW


AND ON THE TOP OF EACH PAGE OF THIS EXAM BOOKLET

STUDENT CODE

IBO 2007. Practical Exam 2 - Plant Biology, Booklet 2

STUDENT CODE __________________

GENERAL INSTRUCTIONS
IMPORTANT
! Before starting the exam, the invigilator will show you a red card and a green card
to test for red-green color blindness. If you are unable to see the difference
between the two cards, raise your hand, and you will be provided with assistance
immediately.
! Read the exam paper carefully before commencing the exam.
! It is recommended that you allocate your time according to the mark value of the Task.
! Write your answers in the exam booklet.
! Do not forget to hand in your graph prepared in Task E with your exam booklet.

BE SURE THAT YOU HAVE WRITTEN YOUR 4-DIGIT STUDENT


CODE ON THE FIRST PAGE OF EACH EXAM BOOKLET
and
ON THE TOP OF EACH PAGE OF THIS BOOKLET
IMPORTANT INFORMATION FOR TASKS B AND C
! Handle the plant samples with care. Some samples will be used in more than one
TASK.
! When you have completed Part 7 of Task B, please indicate so by placing your
plastic bag cover back on top of the microscope and a lab assistant will grade the
quality of your section.
! Make sure that you have completed Parts 5, 6 and 7 of TASK B before
commencing TASK C.
! It is important that you cover your dissection board with paper towel to indicate you
have completed Task C1 (Seed Dissection) and Task C3 (Flower Dissection). In
each case, a lab assistant will ask you to sign your specimen board, photograph your
dissection and then remove the dissection for marking.

IBO 2007. Practical Exam 2 - Plant Biology, Booklet 2

STUDENT CODE __________________

PLANT ANATOMY and MORPHOLOGY


Materials
! 10 petri dishes containing plant samples 1 to 10
! 1 foam core specimen board labeled SEED DISSECTION with four coloured pins
(1 black, 1white, 1 yellow, 1 blue)
! 1 foam core specimen board labeled FLOWER DISSECTION with seven coloured pins
(1 orange, 1 white, 1 yellow, 1 blue, 1 pink, 1 green, 1 black)
! 1 single-edge razor blade
! 1 dissecting kit
! 6 glass microscope slides
! 1 box of cover slips
! 1 drop bottle containing toluedine blue staining solution
! 1 drop bottle containing distilled water
! 3 tissues
! 1 light microscope
! 1 pair disposable gloves
! Paper towel
NOTE: Before beginning this task, be sure that you have all the materials listed above. If
you do not, notify a lab assistant immediately by raising your hand.

TASK B..

Identification and classification of flowering plant samples based on


their anatomy and morphology. (23 marks)

Procedure:
1. Using the razor blade, cut a thin cross section of each of the samples 1 to 4.
2. Transfer each section to a microscope slide and place 1 drop of toluedine blue
staining solution and 1 drop of water on the section.
3. Put a cover slip on the section (try to avoid air bubbles when placing the cover slip) and
remove excess stain by placing the corner of a piece of tissue paper against one edge of the
cover slip.
4. Starting first with the 4X objective and then using either the 10X or the 40X objective,
examine each slide under the microscope and observe the structure of the tissue.

IBO 2007. Practical Exam 2 - Plant Biology, Booklet 2

STUDENT CODE __________________

5. Based on your observation of each slide prepared for samples 1 to 4, identify the
tissue/organ sectioned. For each sample, enter the appropriate letter from the
column labeled KEY in the table below. (8 marks)

6.

Sample
1

Tissue/Organ
B

B = stem

C = root

D = petiole

A = leaf

Examine the sections you prepared for Samples 1 to 4, and the plant parts of
Samples 5 to 10. Identify whether each sample is from a monocot plant or from a dicot
plant and indicate your answer by writing an X in the appropriate column of the
following table. (10 marks)

Plant Sample

Monocot

Dicot

10

7.

KEY

Once you have completed Part 6, place the slide with your best section on the
microscope, focus the microscope and place your plastic bag cover back on the
microscope indicating to the lab assistant that you are finished. A lab assistant will
examine the slide and grade your sectioning technique (5 marks).

IBO 2007. Practical Exam 2 - Plant Biology, Booklet 2

STUDENT CODE __________________

TASK C. SEED AND FLOWER ANATOMY AND MORPHOLOGY (25 marks)


IMPORTANT.
TASK C1.

Make sure that you have completed TASK B before starting TASK C.

SEED ANATOMY (8 marks)

Procedure
1. Write your student number on the specimen board labeled SEED DISSECTION.
2. Using Sample 5 cut the seed longitudinally with the razor blade and dissect the seed into its
component parts.
3. Use the pins provided to pin the correct seed part on the specimen board
!

black pin for the testa (seed coat)

white pin for the cotyledon

yellow pin for the plumule (foliage leaves)

blue pin for the radicle.

4. After finishing this task, cover the board with a paper towel indicating to the lab
assistant that you are finished. The lab assistant and yourself will sign the label on the
specimen board, and the lab assistant will photograph your dissection. Your dissection will
then be removed for grading.
TASK C2.

FLOWER MORPHOLOGY (2 marks)

Procedure
1. Examine the flower in Sample 6. Write an X against the correct answer for the
following questions:
(a)

The sepals alone make up the


i) corolla

_________

ii) calyx

____X_____

iii) perianth

_______

iv) hypanthium

_______

IBO 2007. Practical Exam 2 - Plant Biology, Booklet 2

(b)

TASK C3.

STUDENT CODE __________________

The petals alone make up the whorl known as


i) corolla

____X___

ii) calyx

_______

iii) perianth

_______

iv) hypanthium

_______

FLOWER ANATOMY (15 marks)

IMPORTANT.

You must have completed TASK B before commencing this dissection.

Procedure
1. Write your student number on the specimen board labeled FLOWER DISSECTION.
2. Dissect the flower (Sample 6) into its component parts.
3. Use the pins provided to pin the correct flower part on the specimen board

4.

orange pin for a sepal (2 mark)

white pin for a petal (2 marks)

yellow pin for an anther (2 marks)

pink pin for the filament (2 marks)

green pin for the style (2 marks)

blue pin the stigma (2 marks)

black pin for the ovary (2 marks)

Use an X to indicate the correct classification of the placentation within the ovule
of this flower (1 mark).
i) marginal

_______

ii) axile

___X____

iii) parietal

_______

iv) free-central

_______

IBO 2007. Practical Exam 2 - Plant Biology, Booklet 2

STUDENT CODE __________________

5. After finishing this task, cover the board with a paper towel to indicate to the lab
assistant that you are finished. A lab assistant will photograph your dissection. Both the
lab assistant and yourself will sign the label on the board. Your dissection will then be taken
by the lab assistant for grading.

IBO 2007. Practical Exam 2 - Plant Biology, Booklet 2

STUDENT CODE __________________

PLANT EVOLUTION
TASK D. Identification of the Time of Evolution of Higher Plants (5 marks)
Materials
! Plant samples in dishes labeled H to M.

DO NOT OPEN THE PETRI DISHES.

! Photograph of the evolutionary time scale (Figure 1)


NOTE: Before beginning this task, be sure that you have all the materials listed above. If
you do not, notify a lab assistant immediately by raising your hand.
Procedure
These plant samples possess characteristics representative of their ancestral lineages. Read the
descriptions in Box A and identify the description that is most correct for each plant sample.
1. Using the codes (1 to 6) representing the different time periods in the evolutionary time scale
shown in Figure 1), indicate the geologic time period that best corresponds to each
description.
2. Enter the two codes (one related to the description and one related to the evolutionary time
period) for each sample in Box B.
NOTE:

Not all descriptions in Box A will be used and no letter should be used more than
once. The answer for Sample M is provided.

IBO 2007. Practical Exam 2 - Plant Biology, Booklet 2

STUDENT CODE __________________

BOX A
Plant Sample Lineage Characteristics
a. This spore-bearing plant group has persisted relatively unchanged for hundreds of

millions of years. In this time period, it was likely an important dietary element of
herbivore dinosaurs.
b. The first macrofossil evidence of the evolution of grasses appears in the fossil record at
the time of the diversification of mammals.
c. In this time period, the indehiscent integumented megasporangium (ovules/seeds)
originated. It is represented in the samples by modern plants producing naked
seeds on a scale.
d. This group of spore-bearing plants included members with tree-like stature (Sample M)
and were common in coal-producing swamp floras (answers provided).
e. Coniferous seed plants, as represented by the sample, were driven to extinction by the
diversification of the superior characteristics of the angiosperms, in this time period.
f. The dichotomous branching and sporangia of this plant were characteristics of the first
terrestrial tracheophytes, which left some of the earliest land plant macrofossils at this
time in history.
g. The evolution of flowering plants, as represented by this angiosperm, first appears in the
fossil record in this time period.

STUDENT CODE __________________

IBO 2007. Practical Exam 2 - Plant Biology, Booklet 2

BOX B
Enter the correct codes for each sample
Sample

Description

Time Period

H __

M __

10

STUDENT CODE __________________

IBO 2007. Practical Exam 2 - Plant Biology, Booklet 2

11

PLANT PHYSIOLOGY
TASK E. Interpretation of photosynthetic data from plants grown at different light
levels (8 marks)

Materials
! 2 sheets of graph paper, each with the axes labeled differently
NOTE: Before beginning this task, be sure that you have all the materials listed above. If
you do not, notify a lab assistant immediately by raising your hand.
Introduction
Single leaves from two different plants, one grown in full sun, the other in shade, were removed
and placed in separate, clear boxes. The leaves were exposed to increasing light levels and the
rate of O2 release was measured.
The data obtained from this experiment are presented in the following table:
Light Level
(!mol photons m-2 s-1)
0
10
25
50
100
250
500
600

Rate of O2 production
(!mol O2 m-2 s-1)
Leaf A
Leaf B
-20
-2
-10
-0.5
-5
1.5
-1
3
5
6
15
10
28
12
30
11

Procedure
1. Select the sheet of graph paper that has the X-axis and the Y-axis labeled correctly for the set
of data above. (1 mark)
2. Write your name and your student number on the label on the sheet of graph paper you have
chosen.
3. Mark the scale of the units on each axis.
4. Plot the data presented in the table for each leaf to compare the photosynthetic rates (O2
production) of the leaves with respect to light. Clearly identify which line represents Leaf A
and which line represents Leaf B. (2 marks)

STUDENT CODE __________________

IBO 2007. Practical Exam 2 - Plant Biology, Booklet 2

12

5. Examine the graphs you have drawn and determine which leaf (Leaf A or Leaf B)
demonstrates the characteristics of a shade-adapted leaf and which demonstrates the
characteristics of a sun-adapted leaf. Indicate your answer in the table below by writing an
X in the correct cell. (1 mark)
Leaf A

Leaf B

Shade-adapted

Sun-adapted

6. Use the data plots on your graph to answer the following questions:
(a)

Is the light compensation point of Leaf A higher than the light compensation
point of Leaf B? Circle the correct answer. (0.5 mark)
YES

(b)

Can the light compensation point be defined as the light level at which the
photosynthetic response reaches saturation? (0.5 mark)
YES

(c)

NO

NO

Which of the answers below most correctly identifies the light compensation
point of Leaf A? Circle the letter of that answer. (1 mark)
i)

between -10 and -5 !mol O2 m-2 s-1

ii)

between 10 and 20 !mol O2 m-2 s-1

iii)

between 25 and 50 !mol photons m-2 s-1

iv)

between 50 and 75 !mol photons m-2 s-1

v)

between 500 and 600 !mol photons m-2 s-1

IBO 2007. Practical Exam 2 - Plant Biology, Booklet 2

(d)

STUDENT CODE __________________

13

Which of the answers below best describes the maximum rate of


photosynthesis of the sun leaf? Circle the letter of that answer. (1 mark)

(e)

i)

12 !mol O2 m-2 s-1

ii)

15 !mol O2 m-2 s-1

iii)

30 !mol O2 m-2 s-1

iv)

between 250 and 600 !mol photons m-2 s-1

v)

greater than 600 !mol photons m-2 s-1

This graph gives information about the photosynthetic response to light. Can it also be
used to estimate the response of respiration rate with regards to light? Circle the
correct answer. (1 mark)
YES

NO

- THE END

HAVE YOU WRITTEN YOUR STUDENT CODE ON THE FIRST PAGE OF


THIS EXAM BOOKLET AND ON THE TOP OF EACH PAGE?

REMEMBER TO HAND IN YOUR GRAPH PAPER WITH THIS EXAM


BOOKLET.

FIGURE 1.

DIAGRAM FOR TASK D.

Geologic Time Scale


A. MacRae 1998

PERIOD
CODE

66
55
44

33
22
11

18th INTERNATIONAL BIOLOGY OLYMPIAD


JULY 15 - 22, 2007

PRACTICAL EXAMINATION 3
Cell Biology/Biochemistry
TASK A.

Thiocyanate analysis in cauliflower

27 marks

TASK B.

Determination of the amount of cauliflower needed


to be consumed to cause toxicity

5 marks

Regulation of gene expression

18 marks

TASK C.

Time allowed: 90 minutes

WRITE ALL ANSWERS IN THIS EXAM BOOKLET

WRITE YOUR 4-DIGIT STUDENT CODE IN THE BOX BELOW AND


ON THE TOP OF EACH PAGE OF THIS BOOKLET

STUDENT CODE

IBO 2007. Practical Exam 3 - Cell Biology/Biotechnology.

STUDENT CODE: _______________

Introduction
The cabbage family contains a class of compounds known as glucosinolates. Some glucosinolates such
as glucoraphanin have desired medicinal properties helping to prevent cancers while others such as
glucosinalbin have toxic metabolites.
One of the products of the toxic glucosinolates is the thiocyanate ion (SCN-). SCN- interferes with
iodine metabolism resulting in thyroid hormone deficiency. Eating plants of the crucifer family such as
cauliflower will result in the production of a limited amount of thiocyanate ion from glucosinolates such
as glucosinalbin.
The glucosinolate glucoraphanin is metabolized to sulforaphane. Sulforaphane is an inducer of phase 2
proteins. One consequence of phase 2 protein induction is an increased ability of cells to scavenge free
radicals and other oxidants. A consequence of decreased oxidant levels is a lower probability of
activation of pathways that lead to inflammation. One such pathway is through activation of a protein
complex such as NFkappaB.

TASK A. To determine the amount of thiocyanate ion released from cauliflower using a
spectrophotometric assay. (27 marks)

OBJECTIVE:

To use a spectrophotometer to determine how much thiocyanate ion is released


from cauliflower. This assay is based upon the principle that in an acid
environment thiocyanate reacts with Fe3+ to form a stable Fe2+-SCN red-coloured
complex with a maximum absorption at 447 nm.

Materials
! Eppendorf pipettor: one 20-200 microlitre capacity set to 100 microlitres.
! Eppendorf pipette tips.
! Spectrophotometer cuvettes containing 900 microlitres of ferric nitrate reagent as noted above,
this reagent is in a strong acid.
CAUTION: The ferric nitrate reagent solution you will be using is dissolved in 1.0 M nitric
acid. Wear gloves and use goggles to protect your eyes before starting the
experiment.
! Thiocyanate standards in tubes at the following concentrations: 0 micromoles/mL (this is your
blank), 0.5 micromoles/mL, 1.0 micromoles/mL, 2.0 micromoles/mL and 4.0 micromoles/mL.
! One tube of filtered cauliflower homogenate. 1.0 g of cauliflower was homogenized and the
homogenate was diluted to a total volume of 4.0 mL water. This is your unknown and you will be
required to determine how many micromoles of thiocyanate are present in one millilitre of this
homogenate.

IBO 2007. Practical Exam 3 - Cell Biology/Biotechnology.

STUDENT CODE: _______________

! Marker pen to label the frosted side of each cuvette.


! Gloves and protective glasses
! On your bench is a spectrophotometer set to an absorbance of 447 nm.
NOTE: Before beginning this task, be sure that you have all the materials listed above. If you do not,
notify a lab assistant by raising your hand.

Procedure
1. Put on the gloves and the protective glasses.
2. To each of the cuvettes containing the ferric nitrate reagent add 100 microliters of each of the
thiocyanate standards. The standards are: 0, 0.5, 1.0, 2.0 and 4.0 micromoles thiocyanate/mL. A
coloured reaction should become visible except for the 0 micromole thiocyanate standard which
serves as your blank. Be sure to label the cuvettes on the frosted surface.
3. To each of the remaining 3 cuvettes add 100 microlitres of the cauliflower homogenate.
4. Carefully carry the cuvettes to the spectrophotometer which has been set to absorb at 447 nm. Open
the lid to the light path in the spectrophotometer and insert the 0 micromole thiocyanate/mL standard
(i.e., blank) cuvette. The arrow indicates the light path. Ensure that the walls of the cuvettes
through which the light passes is transparent.
Close the lid and push the set reference button on the top right hand of the panel on the
spectrophotometer see the diagram below. Do not touch any of the other buttons!

5. Insert each of the standards and record the reading. Then insert each of the cuvettes containing the
unknown and record the spectrophotometer reading. Leave the cuvettes at the spectrophotometer
and the laboratory assistants will take care of them.

IBO 2007. Practical Exam 3 - Cell Biology/Biotechnology.

STUDENT CODE: _______________

Spectrophotometer reading (absorbance) for each standard:


0.5 micromole/mL thiocyanate:

___________

1.0 micromole/mL thiocyanate:

___________

2.0 micromole/mL thiocyanate:

___________

4.0 micromole/mL thiocyanate:

___________

Spectrophotometer reading (absorbance) for the unknown:


1._____

2._____

(10 marks)

(4 marks)

3._____

6. Plot, on the graph paper (page 5), the absorbance measurements for your thiocyanate
standards against the concentration (micromoles/mL) of the standards.
(6 marks)
7. Calculate the average absorbance of your cauliflower homogenate.
ANSWER:

____________________

8. What is the concentration of thiocyanate present in the cauliflower homogenate


solution?
ANSWER:

(2 marks)

(5 marks)

_____________________

9. What is the standard deviation of the absorbance of the unknown?


ANSWER: ________________________

(2 marks)

IBO 2007. Practical Exam 3 - Cell Biology/Biotechnology.

STUDENT CODE: _______________

IBO 2007. Practical Exam 3 - Cell Biology/Biotechnology.

STUDENT CODE: _______________

TASK B. To determine the amount of cauliflower needed to be consumed for it to cause toxic
effects because of the presence of thiocyanate (5 marks)

Introduction
The LD50 is a toxicology term that describes the dose (i.e., moles of toxin/kg animal) of a compound that
will kill 50% of the animals tested. In the rat, the LD50 of sodium thiocyanate consumed is reported to
be 9 millimoles/kg. Using the data of the experiment you have just performed, calculate how much
cauliflower a rat that weighs 500 g would have to eat in a short time to reach the LD50 of thiocyanate.

Procedure
Circle the letter of the range that best fits your calculated value. Show your calculations on this page.
Continue on the back of this page if necessary.
(a)

1 g to 5 g

(b)

50 g to 250 g

(c)

500 g to 1 kg

(d)

1.5 kg to 14 kg

(e)

15 kg to 25 kg

TASK C. To interpret the regulation of gene expression. (18 marks)


Introduction
The glucosinolate glucoraphanin is metabolized to sulforaphane. Sulforaphane is an inducer of phase 2
proteins. One consequence of phase 2 protein induction is an increased ability of cells to scavenge free
radicals and other oxidants. A consequence of decreased oxidant levels is a lower probability of
activation of pathways that lead to inflammation. One such pathway is through activation of a protein
complex such as NFkappaB.
NFkappaB is a transcription factor complex comprised of two proteins (p50 and p65) bound to a third
protein known as IkappaB that is normally present in the cytoplasm. Activation of NFkappaB involves
the degradation of IkappaB resulting in the NFkappaB p50/p65 heterodimer translocating to the nucleus
where it binds to specific promoter elements increasing the transcription of pro-inflammatory genes such
as inducible nitric oxide synthase (iNOS). One indicator of activation of NFkappaB is that the ratio of
the p65 to IkappaB protein increases.
One of the consequences of increased iNOS activity is excessive production of the nitric oxide free
radical (NO.). Nitric oxide reacts with the superoxide anion (O2.-) to form peroxynitrous acid.
Peroxynitrous acid is a very strong oxidant.
Increased oxidant levels often results in activation of NFkappaB while lowering oxidant levels often
results in decreased activation of NFkappaB and, hence, lowered levels of expression of proinflammatory genes.

IBO 2007. Practical Exam 3 - Cell Biology/Biotechnology.

STUDENT CODE: _______________

Procedure
1. Examine the figures provided in each of the following sections.
2. Using the data presented, identify which data set is derived from animals fed a diet high in
glucoraphanin and provide the basis for your answer.
SECTION A. (5 marks)
Below is a figure that gives data on NFkappaB activation in spontaneously hypertensive stroke-prone
(SHRsp) male rats that were fed one of two diets: a control diet or an experimental diet containing
glucoraphanin. In the experimental diet, the animals consumed 10 micromoles glucoraphanin/kg body
weight.
After several months on these diets, the animals were euthanized, nuclei from the kidney cells were
isolated and prepared for SDS polyacrylamide electrophoresis. Following separation of the proteins on
the gel, the proteins were transferred to nitrocellulose membrane and probed with an antibody that
recognized the NFkappaB p65 protein.

Relative p65 Protein Level

A representative Western blot is shown below (on the left) and next to it is a graph that depicts the
quantification of blots from 5 different animals per diet group.

Answer the following questions:


1. Which group of animals (A or B) were on the glucoraphanin-containing diet?
ANSWER: _______________________

(1 mark)

IBO 2007. Practical Exam 3 - Cell Biology/Biotechnology.

STUDENT CODE: _______________

2. Which of the following statements gives the best explanation for your answer?
Circle the letter of that statement.

(4 marks)

(a)

Less oxidative stress results in less NFkappaB activation and hence less p65 in the nuclei.

(b)

Less oxidative stress results in less NFkappaB activation and hence more p65 in the nuclei.

(c)

More oxidative stress results in less NFkappaB activation and hence less p65 in the nuclei.

(d)

More oxidative stress results in less NFkappaB activation and hence more p65 in the nuclei.

(e)

More oxidative stress results in more NFkappaB activation and hence less p65 in the nuclei.

SECTION B.

(8 marks)

Relative N-Protein Levels

Below is a figure that gives Western blot data on a 45 kD nitrosylated protein (N-protein) in the kidneys
of two male SHRsp rats that were put on one of two different diets: a diet containing glucoraphanin and
control diets.

The top part of the figure is a representative Western blot while the bottom part of the figure is the
quantification of Western blots from 5 different animals per diet group.
Answer the following questions:
1. Which group (A or B) represents the animals fed a diet containing glucoraphanin?
ANSWER: __________________________

(1 mark)

IBO 2007. Practical Exam 3 - Cell Biology/Biotechnology.

STUDENT CODE: _______________

2. Circle the letter of the statement below that best explains your answer.

(4 marks)

(a) More oxidative stress results in more NFkappaB activation that results in more iNOS expression
and more peroxynitrous acid formation and thus more nitrosylation of proteins.
(b) More oxidative stress results in more NFkappaB activation that results in more iNOS expression
and more peroxynitrous acid formation but less nitrosylation of proteins.
(c) More oxidative stress results in more NFkappaB activation that results in more iNOS expression
but less peroxynitrous acid formation and thus less nitrosylation of proteins.
(d) More oxidative stress results in less NFkappaB activation but results in less iNOS expression and
less peroxynitrous acid formation and thus less nitrosylation of proteins.
(e) Less oxidative stress results in less NFkappaB activation that results in less iNOS expression and
less peroxynitrous acid formation and thus less nitrosylation of proteins.
3)

The figure above shows the amount of an additional protein, actin. Why is the level of this
protein measured?
(3 marks)
a) To quantify the expression of N-protein relative to a protein that is assumed to be equally
expressed in kidney cells independent of the treatment.
b) To show that the expression level of N-protein is independent of the effect of the treatment on
the filtration rate of the kidney.
c) To ensure that N-protein and actin are not bound to each other in the cells of rats that received
the treatment.
d) The level of a protein whose expression is assumed to be independent of the treatment is
measured to ensure that the binding of the antibody used to detect N-protein is unaffected by the
treatment.

IBO 2007. Practical Exam 3 - Cell Biology/Biotechnology.

STUDENT CODE: _______________

10

SECTION C. (5 marks)
The nitric oxide radical (NO.), that is released by endothelial cells of blood vessels, diffuses to smooth
muscle cells where it causes smooth muscle cells to relax and thus decreases blood pressure. The
amount of NO. that can diffuse to the smooth muscle depends upon the overall level of superoxide
radicals. If there are excessive superoxide radicals present, then NO. reacts with superoxide forming
peroxynitrous acid.
Below is a graph depicting the systolic blood pressures of female SHRsp rats placed on one of two diets,
a control diet and a diet containing glucoraphanin. Animals were placed on these diets at the age of 5
weeks post-natal and blood pressures were monitored from 6 to 19 weeks post-natal.

Answer the following questions:


1. Which group of animals, A or B, were fed the glucoraphanin-containing diet?
ANSWER: _______________________________

(1mark)

IBO 2007. Practical Exam 3 - Cell Biology/Biotechnology.

STUDENT CODE: _______________

2. Circle the letter of the statement below that best explains your answer.

(4 marks)

(a) Less oxidative stress results in lower scavenging of nitric oxide. Decreased amounts of nitric
oxide available to vascular smooth muscle results in more vasorelaxation and thus lower blood
pressure.
(b) Less oxidative stress results in higher scavenging of nitric oxide. Decreased amounts of nitric
oxide available to vascular smooth muscle results in less vasorelaxation and thus higher blood
pressure.
(c) Less oxidative stress results in lower scavenging of nitric oxide. Increased amounts of nitric
oxide available to vascular smooth muscle results in more vasorelaxation and thus lower blood
pressure.
(d) Less oxidative stress results in higher scavenging of nitric oxide. Increased amounts of nitric
oxide available to vascular smooth muscle results in less vasorelaxation and thus higher blood
pressure.
(e) Less oxidative stress results in lower scavenging of nitric oxide. Increased amount of nitric
oxide available to vascular smooth muscle results in more vasorelaxation and thus lower blood
pressure.

- THE END

HAVE YOU WRITTEN YOUR STUDENT CODE ON THE FIRST PAGE OF


THIS EXAM BOOKLET AND ON THE TOP OF THE OTHER PAGES?

11

18th INTERNATIONAL BIOLOGY OLYMPIAD


JULY 15 - 22, 2007

PRACTICAL EXAMINATION 3
Cell Biology/Biochemistry
TASK A.

Thiocyanate analysis in cauliflower

29 marks

TASK B.

Determination of the amount of cauliflower needed


to be consumed to cause toxicity

5 marks

Regulation of gene expression

10 marks

TASK C.

Time allowed: 90 minutes

WRITE ALL ANSWERS IN THIS EXAM BOOKLET

WRITE YOUR 4-DIGIT STUDENT CODE IN THE BOX BELOW AND


ON THE TOP OF EACH PAGE OF THIS BOOKLET

STUDENT CODE

IBO 2007. Practical Exam 3 - Cell Biology/Biotechnology.

STUDENT CODE: _______________

Introduction
The cabbage family contains a class of compounds known as glucosinolates. Some glucosinolates such
as glucoraphanin have desired medicinal properties helping to prevent cancers while others such as
glucosinalbin have toxic metabolites.
One of the products of the toxic glucosinolates is the thiocyanate ion (SCN-). SCN- interferes with
iodine metabolism resulting in thyroid hormone deficiency. Eating plants of the crucifer family such as
cauliflower will result in the production of a limited amount of thiocyanate ion from glucosinolates such
as glucosinalbin.

TASK A. To determine the amount of thiocyanate ion released from cauliflower using a
spectrophotometric assay. (29 marks)

OBJECTIVE:

To use a spectrophotometer to determine how much thiocyanate ion is released


from cauliflower. This assay is based upon the principle that in an acid
environment thiocyanate reacts with Fe3+ to form a stable Fe2+-SCN red-coloured
complex with a maximum absorption at 447 nm.

Materials
! Eppendorf pipettor: one 20-200 microlitre capacity set to 100 microlitres.
! Eppendorf pipette tips.
! Spectrophotometer cuvettes containing 900 microlitres of ferric nitrate reagent as noted above,
this reagent is in a strong acid.
CAUTION: The ferric nitrate reagent solution you will be using is dissolved in 1.0 M nitric
acid. Wear gloves and use goggles to protect your eyes before starting the
experiment.
! Thiocyanate standards in tubes at the following concentrations: 0 micromoles/mL (this is your
blank), 0.5 micromoles/mL, 1.0 micromoles/mL, 2.0 micromoles/mL and 4.0 micromoles/mL.
! One tube of filtered cauliflower homogenate. 1.0 g of cauliflower was homogenized and the
homogenate was diluted to a total volume of 4.0 mL water. This is your unknown and you will be
required to determine how many micromoles of thiocyanate are present in one millilitre of this
homogenate.
! Marker pen to label the frosted side of each cuvette.
! Gloves and protective glasses
! On your bench is a spectrophotometer set to an absorbance of 447 nm.
NOTE: Before beginning this task, be sure that you have all the materials listed above. If you do not,
notify a lab assistant by raising your hand.

IBO 2007. Practical Exam 3 - Cell Biology/Biotechnology.

STUDENT CODE: _______________

Procedure
1. Put on the gloves and the protective glasses.
2. To each of the cuvettes containing the ferric nitrate reagent add 100 microliters of each of the
thiocyanate standards. The standards are: 0, 0.5, 1.0, 2.0 and 4.0 micromoles thiocyanate/mL. A
coloured reaction should become visible except for the 0 micromole thiocyanate standard which
serves as your blank. Be sure to label the cuvettes on the frosted surface.
3. To each of the remaining 3 cuvettes add 100 microlitres of the cauliflower homogenate.
4. Carefully carry the cuvettes to the spectrophotometer which has been set to absorb at 447 nm. Open
the lid to the light path in the spectrophotometer and insert the 0 micromole thiocyanate/mL standard
(i.e., blank) cuvette. Note that the clear walls of the cuvette should be in line with the arrow
indicators in the spectrophotometer chanber.
Close the lid and push the set reference button on the top right hand of the panel on the
spectrophotometer see the diagram below.
Do not touch any of the other buttons!

5. Insert each of the standards and record the reading. Then insert each of the cuvettes containing the
unknown and record the spectrophotometer reading. Leave the cuvettes at the spectrophotometer
and the laboratory assistants will take care of them.

Spectrophotometer reading (absorbance) for each standard:


0.5 micromole/mL thiocyanate:

___________

1.0 micromole/mL thiocyanate:

___________

2.0 micromole/mL thiocyanate:

___________

4.0 micromole/mL thiocyanate:

___________

(10 marks)

IBO 2007. Practical Exam 3 - Cell Biology/Biotechnology.

STUDENT CODE: _______________

Spectrophotometer reading (absorbance) for the unknown: (3 marks)


1._____

2._____

3._____

6. Plot, on the graph paper (page 5), the absorbance measurements for your thiocyanate
standards against the concentration (micromoles/mL) of the standards. (6 marks)
7. Take the average absorbance of your cauliflower homogenate and determine the thiocyanate ion
concentration using the previously plotted graph. (5 marks)
ANSWER:

____________________

8. What is the concentration of the thiocyanate ion present in your cauliflower homogenate? Be sure to
state the units. (3 marks)
ANSWER:

_____________________

9. Calculate the standard deviation of the spectrophotometer reading of the unknown? (2 marks)
ANSWER: ________________________

IBO 2007. Practical Exam 3 - Cell Biology/Biotechnology.

STUDENT CODE: _______________

IBO 2007. Practical Exam 3 - Cell Biology/Biotechnology.

STUDENT CODE: _______________

TASK B. To determine the amount of cauliflower needed to be consumed for it to cause toxic
effects because of the presence of thiocyanate (5 marks)

Introduction
The LD50 is a toxicology term that describes the dose (i.e., moles of toxin/kg animal) of a compound that
will kill 50% of the animals tested. In the rat, the LD50 of sodium thiocyanate consumed is reported to
be 9 millimoles/kg. Using the data of the experiment you have just performed, calculate how much
cauliflower a rat that weighs 500 g would have to eat in a short time to reach the LD50 of thiocyanate.

Procedure
Circle the letter of the range that best fits your calculated value. Show your calculations on this page.
Continue on the back of this page if necessary.
(a)

1 g to 5 g

(b)

50 g to 250 g

(c)

500 g to 1 kg

(d)

1.5 kg to 14 kg

(e)

15 kg to 25 kg

TASK C. To interpret the regulation of gene expression. (18 marks)


Introduction
The glucosinolate glucoraphanin is metabolized to sulforaphane. Sulforaphane is an inducer of phase 2
proteins. One consequence of phase 2 protein induction is an increased ability of cells to scavenge free
radicals and other oxidants. A consequence of decreased oxidant levels is a lower probability of
activation of pathways that lead to inflammation. One such pathway is through activation of a protein
complex such as NFkappaB.
NFkappaB is a transcription factor complex comprised of two proteins (p50 and p65) bound to a third
protein known as IkappaB that is normally present in the cytoplasm. Activation of NFkappaB involves
the degradation of IkappaB resulting in the NFkappaB p50/p65 heterodimer translocating to the nucleus
where it binds to specific promoter elements increasing the transcription of pro-inflammatory genes such
as inducible nitric oxide synthase (iNOS). One indicator of activation of NFkappaB is that the ratio of
the p65 to IkappaB protein increases.
One of the consequences of increased iNOS activity is excessive production of the nitric oxide free
radical (NO.). Nitric oxide reacts with the superoxide anion (O2.-) to form peroxynitrous acid.
Peroxynitrous acid is a very strong oxidant.

IBO 2007. Practical Exam 3 - Cell Biology/Biotechnology.

STUDENT CODE: _______________

Increased oxidant levels often results in activation of NFkappaB while lowering oxidant levels often
results in decreased activation of NFkappaB and, hence, lowered levels of expression of proinflammatory genes. The nitric oxide that is released by endothelial cells diffuses to smooth muscle
cells where it causes smooth muscle cells to relax. Hence, nitric oxide is a major regulator of blood
pressure.

Procedure
1. Examine the figures provided in each of the following sections.
2. Using the data presented, identify which data set is derived from animals fed a diet high in
glucoraphanin and provide the basis for your answer.
SECTION A. (5 marks)
Below is a figure that gives data on NFkappaB activation in spontaneously hypertensive stroke-prone
(SHRsp) male rats that were fed one of two diets: a control diet or an experimental diet containing
glucoraphanin. In the experimental diet, the animals consumed 10 micromoles glucoraphanin/kg body
weight.
After several months on these diets, the animals were euthanized, nuclei from the kidney cells were
isolated and prepared for SDS polyacrylamide electrophoresis. Following separation of the proteins on
the gel, the proteins were transferred to nitrocellulose membrane and probed with an antibody that
recognized the NFkappaB p65 protein.

Relative p65 Protein Level

A representative Western blot is shown below (on the left) and next to it is a graph that depicts the
quantification of blots from 5 different animals per diet group.

Answer the following questions:


1. Which group of animals were on the glucoraphanin-containing diet? The group represented by A or
B? (1 mark)
ANSWER: _________B______________

IBO 2007. Practical Exam 3 - Cell Biology/Biotechnology.

STUDENT CODE: _______________

2. Which of the following statements gives the best explanation for your answer?
Circle the letter of that statement. (4 marks)
(a)

Less oxidative stress results in less NFkappaB activation and hence less p65 in the nuclei.

(b)

Less oxidative stress results in less NFkappaB activation and hence more p65 in the nuclei.

(c)

More oxidative stress results in less NFkappaB activation and hence less p65 in the nuclei.

(d)

More oxidative stress results in less NFkappaB activation and hence more p65 in the nuclei.

(e)

More oxidative stress results in more NFkappaB activation and hence less p65 in the nuclei.

SECTION B. (5 marks)
Below is a figure that gives Western blot data on a 45 kD nitrosylated protein (N-protein) in the kidneys
of two male SHRsp rats that were put on one of two different diets: a diet containing glucoraphanin and
control diets.

The top part of the figure is a representative Western blot while the bottom part of the figure is the
quantification of Western blots from 5 different animals per diet group.
Answer the following questions:
1. Which group represents the animals fed a diet containing glucoraphanin?
ANSWER: ___________C_______________

(1 mark)

IBO 2007. Practical Exam 3 - Cell Biology/Biotechnology.

STUDENT CODE: _______________

2. Circle the letter of the statement below that best explains your answer.

(4 marks)

(a) More oxidative stress results in more NFkappaB activation that results in more iNOS expression
and more peroxynitrous acid formation and thus more nitrosylation of proteins.
(b) More oxidative stress results in more NFkappaB activation that results in more iNOS expression
and more peroxynitrous acid formation but less nitrosylation of proteins.
(c) More oxidative stress results in more NFkappaB activation that results in more iNOS expression
but less peroxynitrous acid formation and thus less nitrosylation of proteins.
(d) More oxidative stress results in less NFkappaB activation but results in less iNOS expression and
less peroxynitrous acid formation and thus less nitrosylation of proteins.
(e) Less oxidative stress results in less NFkappaB activation that results in less iNOS
expression and less peroxynitrous acid formation and thus less nitrosylation of proteins.
SECTION C. (5 marks)
The nitric oxide radical (NO.), that is released by endothelial cells of blood vessels, diffuses to smooth
muscle cells where it causes smooth muscle cells to relax and thus decreases blood pressure. The
amount of NO. that can diffuse to the smooth muscle depends upon the overall level of superoxide
radicals. If there are excessive superoxide radicals present, then NO. reacts with superoxide forming
peroxynitrous acid.
Below is a graph depicting the systolic blood pressures of female SHRsp rats placed on one of two diets,
a control diet and a diet containing glucoraphanin. Animals were placed on these diets at the age of 5
weeks post-natal and blood pressures were monitored from 6 to 19 weeks post-natal.

IBO 2007. Practical Exam 3 - Cell Biology/Biotechnology.

STUDENT CODE: _______________

Answer the following questions:


1. Which group of animals, A or B, were fed the glucoraphanin-containing diet?

(1mark)

ANSWER: _______________________________
2. Circle the letter of the statement below that best explains your answer.

(4 marks)

(a) Less oxidative stress results in lower scavenging of nitric oxide. Decreased amounts of nitric
oxide available to vascular smooth muscle results in more vasorelaxation and thus lower blood
pressure.
(b) Less oxidative stress results in higher scavenging of nitric oxide. Decreased amounts of nitric
oxide available to vascular smooth muscle results in less vasorelaxation and thus higher blood
pressure.
(c) Less oxidative stress results in lower scavenging of nitric oxide. Increased amounts of nitric
oxide available to vascular smooth muscle results in more vasorelaxation and thus lower blood
pressure.
(d) Less oxidative stress results in higher scavenging of nitric oxide. Increased amounts of nitric
oxide available to vascular smooth muscle results in less vasorelaxation and thus higher blood
pressure.
(e) Less oxidative stress results in lower scavenging of nitric oxide. Increased amount of nitric
oxide available to vascular smooth muscle results in more vasorelaxation and thus lower blood
pressure.

- THE END

HAVE YOU WRITTEN YOUR STUDENT CODE ON THE FIRST PAGE OF


THIS EXAM BOOKLET AND ON THE TOP OF THE OTHER PAGES?

10

18th INTERNATIONAL BIOLOGY OLYMPIAD

July 15 22, 2007

PRACTICAL EXAM 4
GENETICS
Before starting the exam, the invigilator will show you a red card and a
green card to test for red-green color blindness. If you are unable to
see the difference between the two cards, raise your hand, and you will
be provided with assistance immediately.
TASK A.

Sequence confirmation of a cDNA

23 marks

TASK B.

Genetics of coat colour in dogs

16 marks

TASK C.

Genetic control of seed coat colour and seed shape in beans


20 points

Time allowed: 90 minutes

WRITE ALL ANSWERS IN THE QUESTION BOOK.


Write your 4-digit Student Code in the Box below

Student code:

IBO 2007. Practical Exam 4 - Genetics

TASK A.

Sequence Confirmation of a cDNA (23 marks)

Objective: To isolate plasmid DNA containing a cDNA of interest and to


determine the sequence of the cDNA.
Introduction:
To over-express a gene of interest in a plant or animal you must first isolate the gene of
interest in the form of a cDNA. You have done this and in order to amplify this DNA, you have
cloned it into the pBluescript SK plasmid vector which you have subsequently used to
transform bacteria cells. You must now carry out a quick plasmid preparation to isolate the
plasmid and confirm the sequence of your cDNA insert.
Materials

Quantity

! Bacterial cell culture

4 mL

! 1.5 mL microcentrifuge tubes

! Microcentrifuge rack

! P1000 micropipettor

! Box of 200-1000 uL pipette tips

! GET buffer (1.5 mL tube)

0.5 mL

! 10% Sodium Dodecyl Sulphate (1.5 mL tube)

0.5 mL

! 2 N NaOH (1.5 mL tube)

0.5 mL

! 3 M Potassium 5 M Acetate (1.5 mL tube)

0.5 mL

! 95% ethanol (Falcon tube)

3 mL

! Distilled water (Falcon tube)

3 mL

! Timer

! Tube labels

! Marker pen

! Red card

! Garbage (tips & tubes) bag

! Access to a microcentrifuge
! Access to vortex

IBO 2007. Practical Exam 4 - Genetics

NOTE:

Before beginning this task, be sure that you have all the materials listed above.
If you do not, raise your RED card to call a lab assistant.

Procedure
1.

Pipette 1.5 mL of bacterial culture into each of two 1.5 mL microcentrifuge tubes.

2.

Centrifuge the tubes in a benchtop microcentrifuge for 1 minute - make sure that the
centrifuge rotor is BALANCED.

3.

Completely remove and discard the growth medium from each tube.

4.

Add 100 uL of GET (Glucose-EDTA-Tris) buffer pH 7.9 to the cell pellet (no need to
cap the tubes) - vortex vigorously to resuspend the pellet and leave at room
temperature for 5 minutes.

5.

In a separate 1.5 mL microcentrifuge tube, make a combined mixture of 1% SDS and


0.2 N NaOH in water to a final volume of 1 mL.

6.

To each tube from 4. above add 200 uL of this freshly prepared mixture of 1% SDS
and 0.2 N NaOH - cap the tubes and invert 4-5 times.

7.

Incubate at room temperature for 3 minutes.

8.

To each tube add 150 uL 5M KOAc (3 M potassium and 5 M acetate), cap the tubes
and shake briefly by hand to mix.

9.

Incubate at room temperature for 3 minutes.

10. Centrifuge the tubes for 3 minutes - full speed in microcentrifuge - remember to
balance the rotor.
11. Label 2 clean microcentrifuge tubes with your 4-digit student code number.
12. Pipette the supernatant from each of the centrifuged tubes into each of the clean
tubes. Discard the original tube which now contains a white pellet - this is bacterial
chromosomal DNA.
13. Add 800 uL of 95% ethanol to each tube. Cap the tubes, shake vigorously for 10 sec
and leave on bench 10 minutes.
14. Centrifuge the tubes for 5 minutes - full speed in microcentrifuge.
15. Pour off the supernatant from each tube, cap the tube and raise your RED card.
16. The lab assistant will check your pellet (10 marks for a white pellet).
17. The lab assistant will then give you the sequence trace for your plasmid and cDNA.
The cDNA was sequenced from the T3 promoter.

IBO 2007. Practical Exam 4 - Genetics

18. Check your sequence (starting at nucleotide 21) against that for the pBluescript
vector and answer the questions on page 5.

PLASMID MAP AND MULTIPLE CLONING SITE SEQUENCE FOR pBLUESCRIPT

IBO 2007. Practical Exam 4 - Genetics

Questions (13 marks)


1. The enzyme site into which you cloned your fragment of DNA is

EcoR1.

NOTE: The first letter of the enzymes name is located above the first nucleotide of its
recognition sequence. (5 marks)
2. List the first 20 nucleotides of your fragment of DNA (not including the restriction site
sequence). (2 marks)

nucleotide

10

11

12

13

14

15

16

17

18

19

20

3. Find the first start codon. Using the genetic code table provided, and starting with the
start codon, translate the first 21 nucleotides into their appropriate amino acids. (4 marks)
Start codon

Amino

MET

acid
1

VAL
3

TYR
6

LEU
9

10

11

LEU
12

13

14

LEU
15

16

17

nucleotide

4. (a) If the nucleotide at position 13 was mutated to an A, what would be the


corresponding amino acid? (1 mark)

MET

(b). If the nucleotide at position 14 was mutated to an A, what would be the


corresponding amino acid? (1 mark)

STOP

LEU
18

19

20

21

IBO 2007. Practical Exam 4 - Genetics

Task C.
Genetic Control of Seed Coat Colour and Seed Shape in
Beans (20 marks)
Material
!

1 plastic bag containing flat red parent beans DO NOT OPEN

1 plastic bag containing round red parent beans DO NOT OPEN

1 plastic bag containing F1 seeds (flat yellow) from the cross between the parent
beans DO NOT OPEN

1 plastic bag of F2 bean seeds (representing 250 F2 plants) THIS BAG MAY BE

OPENED
To help you answer the questions below, fill in the following table:
Seed shape

Seed coat colour

(round or flat)

(yellow or red)

Parent 1

FLAT

RED

Parent 2

ROUND

RED

FLAT

YELLOW

Generation

F1 from a cross between these two


parents
Answer the following questions.

1. Is the seed coat colour controlled by (circle one)


(i) one gene
(ii) more than one gene?

(1 mark)

2. a) Red seed coat colour is (circle one)


(i) dominant
(ii) partially dominant
(iii) recessive

(1 mark)

IBO 2007. Practical Exam 4 - Genetics

b) Round seed shape is (circle one)


(i) dominant
(ii) partially dominant
(iii) recessive

(1 mark)

3. (a) There are four phenotypes in your sample of F2 seeds. Classify the seeds into
these phenotypic classes and write the number of each phenotype in the table
below. (2 points)
Phenotype

Number of seeds

(seed colour/seed shape)

(= number of F2 plants)

round, red

13

flat, red

38

round, yellow

18

flat, yellow

55
Total

124

Use these F2 segregation data to answer the following questions:


4. (a)
(b)

From your data how many genes could be controlling seed shape?
(1 mark)

How many round beans and how many flat ones would you expect in a
population this size?
ROUND ___31___

FLAT ____93______

(2 marks)

(c) Is this segregation ratio significantly different from the observed ratio (circle one)?
YES
And what is the probability?

NO
0.95<p<1.0

(1 mark)
(3 marks)

IBO 2007. Practical Exam 4 - Genetics

5. (a)

From your data how many genes could be controlling seed coat colour?

_2_

(1 mark)
(b)

How many red beans and how many yellow beans would you expect in a
population this size?
RED

54.25

YELLOW

69.75

(3 marks)

(c) Is this segregation ratio significantly different from the observed ratio? (circle one)
YES
And what is the probability?

NO

(1 mark)

0.56 or 0.5<p< 0.7

(3 marks)

Chi-square Distribution
Probability
Df

0.95

0.90

0.80

0.70

0.50

0.30

0.20

0.10

0.05

0.01

0.001

0.004

0.02

0.06

0.15

0.46

1.07

1.64

2.71

3.84

6.64

10.83

0.10

0.21

0.45

0.71

1.39

2.41

3.22

4.60

5.99

9.21

13.82

0.35

0.58

1.01

1.42

2.37

3.66

4.64

6.25

7.82

11.34

16.27

0.71

1.06

1.65

2.20

3.36

4.88

5.99

7.78

9.49

13.28

18.47

- THE END

IBO 2007. Practical Exam 4 - Genetics

GENETIC CODE TABLE

This table shows the 64 codons and the amino acid each codon codes for. The direction is 5' to 3'.
2nd base
U

1st
base
A

UUU (Phe/F)Phenylalanine UCU (Ser/S)Serine

UAU (Tyr/Y)Tyrosine

UGU (Cys/C)Cysteine

UUC (Phe/F)Phenylalanine UCC (Ser/S)Serine

UAC (Tyr/Y)Tyrosine

UGC (Cys/C)Cysteine

UUA (Leu/L)Leucine

UCA (Ser/S)Serine

UAA Ochre (Stop)

UGA Opal (Stop)

UUG (Leu/L)Leucine

UCG (Ser/S)Serine

UAG Amber (Stop)

UGG (Trp/W)Tryptophan

CUU (Leu/L)Leucine

CCU (Pro/P)Proline

CAU (His/H)Histidine

CGU (Arg/R)Arginine

CUC (Leu/L)Leucine

CCC (Pro/P)Proline

CAC (His/H)Histidine

CGC (Arg/R)Arginine

CUA (Leu/L)Leucine

CCA (Pro/P)Proline

CAA (Gln/Q)Glutamine

CGA (Arg/R)Arginine

CUG (Leu/L)Leucine

CCG (Pro/P)Proline

CAG (Gln/Q)Glutamine

CGG (Arg/R)Arginine

AUU (Ile/I)Isoleucine

ACU (Thr/T)Threonine AAU (Asn/N)Asparagine

AGU (Ser/S)Serine

AUC (Ile/I)Isoleucine

ACC (Thr/T)Threonine AAC (Asn/N)Asparagine

AGC (Ser/S)Serine

AUA (Ile/I)Isoleucine

ACA (Thr/T)Threonine AAA (Lys/K)Lysine

AGA (Arg/R)Arginine

AUG (Met/M)Methionine

ACG (Thr/T)Threonine AAG (Lys/K)Lysine

AGG (Arg/R)Arginine

GUU (Val/V)Valine

GCU (Ala/A)Alanine

GAU (Asp/D)Aspartic acid GGU (Gly/G)Glycine

GUC (Val/V)Valine

GCC (Ala/A)Alanine

GAC (Asp/D)Aspartic acid GGC (Gly/G)Glycine

GUA (Val/V)Valine

GCA (Ala/A)Alanine

GAA (Glu/E)Glutamic acid GGA (Gly/G)Glycine

GUG (Val/V)Valine

GCG (Ala/A)Alanine

GAG (Glu/E)Glutamic acid GGG (Gly/G)Glycine

MARKING NOTES for TASK C of the GENETICS PRACTICAL EXAM


Q. 3.

Deviations of 3 were accepted in each category.


If these numbers were different, the remaining calculations were done with the
students numbers and marked accordingly.

Q. 4. (b)
(c)

63 :187 was also accepted if the students used 250 as the population size
P = 0.95 was not accepted unless it was the actual p-value.
This answer is based on the expected values in 4b, if the student had different
expected values their numbers were used in a chi-square calculation. This was
necessary as the students did not always get the right # of seeds/phenotype.

Q. 5. (b)

(c)

109:140 was also accepted if the students used 250 as the population size.
Full marks were given if the ratio was reversed (not convinced the translations were
always correct)
This answer is based on the expected values in 5 (b). If the student had different
expected values, those numbers were used in the chi-square calculation. This was
necessary as the students did not always get the right # of seeds/phenotype.

18th INTERNATIONAL BIOLOGY OLYMPIAD


JULY 15 - 22, 2007

THEORY EXAMINATION # 1
Total marks possible: 99.0

Time allowed: 2.5 hours (150 minutes)

WRITE YOUR 4-DIGIT STUDENT NUMBER IN THE BOX


BELOW

STUDENT CODE

IBO 2007. Theoretical Exam 1

GENERAL INSTRUCTIONS
Check that you have the correct examination paper and an answer sheet.
WHEN YOU HAVE FINISHED THE EXAM, PLACE YOUR ANSWER SHEET INSIDE
YOUR QUESTION PAPER AND HAND BOTH TO THE INVIGILATOR BEFORE
LEAVING THE EXAM ROOM.
REMEMBER TO WRITE YOUR 4-DIGIT STUDENT CODE ON THE FRONT PAGE OF
THE QUESTION PAPER.
Read each question carefully before attempting it.

INSTRUCTIONS REGARDING RECORDING YOUR ANSWERS


QUESTIONS 1 - 31.
SHEET.

RECORD YOUR ANSWERS ON THE ANSWER

QUESTIONS 32 59. RECORD YOUR ANSWERS IN THE EXAM


QUESTION BOOKLET.
IMPORTANT
! Use the answer sheet provided to record your answers.
! Ensure that your name and student code is PRINTED in the top margin of the front
page of the answer sheet. The invigilators will enter this information in the correct
places on the reverse side of the answer sheet.
! Use only the HB pencil provided to mark the answer sheet. Completely fill in the circle.

This is the correct way:

! DO NOT USE AN X OR ANY OTHER SYMBOL TO MARK YOUR ANSWER.

! If you want to change your answer, use the eraser to remove your incorrect response and
fill in the new circle you require.
! There is only one correct answer to each question.
! Questions 1 - 30 are worth one mark each The mark value for questions 31 60 varies
according to the length and difficulty of the question.
! Marks will not be deducted for incorrect answers.

IBO 2007. Theoretical Exam 1

ANSWERS TO QUESTIONS 1 TO 31 ARE TO BE RECORDED ON THE


ANSWER SHEET.
Question 1. Which of the following statements is FALSE?
For almost every antigen you may encounter.
A.
B.
C.
D.
E.

a subset of B-cells already exists in your body specific to it.


a subset of T-helper cells already exists in your body that expresses a T-cell receptor
specific to it
a subset of phagocytes already exists in your body that attacks only that antigen.
a subset of antigen-specific antibodies already exists, but are not yet produced in large
numbers.
a subset of antigen-specific memory cells can be produced upon exposure to that
antigen.

Question 2. A blood smear of a human shows higher than normal numbers of eosinophils. Which
of the following may be occurring in his body?
ibo! 7/18/07 8:20 PM
Deleted:

A.
B.
C.
D.
E.

chronic nematode infection


anaphylactic shock
reduced white blood cells (leucopenia)
initial response to invading bacteria
hemostasis

IBO 2007. Theoretical Exam 1

Question 3. The ABO blood type of humans can be determined by a coagulation reaction with
anti-A and anti-B antibodies.

Positive coagulation

Negative Coagulation

Coagulation tests of persons blood produced the results shown below:


With anti-A and
anti-B antibodies

With anti-A
antibodies

With anti-B
antibodies

None

Which of the following statements can be deduced from the above?


A.
B.
C.
D.
E.

This persons blood contains anti-B antibodies.


This persons parents had to be type-A and type-O.
This person can receive neither type-A nor type-B blood.
Type-B antigens are present on the surface of this persons red blood cells.
This persons blood can be donated to both type-B and type-O individuals.

IBO 2007. Theoretical Exam 1

Question 4. The graph below shows the result of blood glucose test from a diabetes patient.

When tested 3 hours after having a carbohydrate rich meal, the blood glucose level of this patient
was 3 times higher than that of a normal individual. However, there was no difference in the level
of insulin in the blood between the two individuals.
Which of the following could be the reason for diabetic symptoms in this patient?
A.
B.
C.
D.
E.

Degradation of pancreatic beta-islets cells.


Degradation of pancreatic alpha-islets cells.
Abnormal proliferation of pancreatic beta-islet cells.
Reduced sensitivity of insulin-receptor mediated signal transduction.
Increased sensitivity of insulin-receptor mediated signal transduction.

IBO 2007. Theoretical Exam 1

Question 5. The following graph shows the dissociation curves for hemoglobin and myoglobin.

Based on the data presented in the graph, which of the following statements is true?
A.
B.
C.
D.
E.

The high affinity of myoglobin for O2 at low partial pressures of O2 prevents


hemoglobin from unloading O2 to muscle.
Myoglobin binds to oxygen with greater affinity than hemoglobin and unloads
oxygen after hemoglobin unloading.
Myoglobin helps hemoglobin bind as much O2 as possible from lungs.
Hemoglobin binds to O2 tightly thus preventing O2 from being made available to
skeletal muscle.
The high affinity of hemoglobin for O2 at low partial pressures of O2 prevents
myoglobin from unloading O2 to muscle.

IBO 2007. Theoretical Exam 1

Questions 6 - 8. This figure indicates some of the pathways involved in the metabolism of
food.

Question 6. When a person consumes a diet rich in carbohydrate, the reactions up-regulated will be:
A.
B.
C.
D.
E.
Question 7.
be:
A.
B.
C.
D.
E.
Question 8.
are:
A.
B.
C.
D.
E.

5, 6, 7
2, 8
5, 8
1, 3, 4
2, 5. 6
When a person performs heavy exercise, the reactions that will be down-regulated will
5, 6, 7, 8
1, 3, 4
4, 5, 6, 7
1, 2, 4
7, 3, 4
If a person suffers from carnitine deficiency, the reactions that will be down-regulated
6, 8 only
1, 3, 4
4, 5, 6, 7
2, 5, 6
5, 6, 7, 8

IBO 2007. Theoretical Exam 1

Question 9. A yeast extract contains all the enzymes required for alcohol production. The extract
is incubated under anaerobic conditions in 1 liter of medium containing: 200 mM glucose, 20 mM
ADP, 40 mM ATP, 2 mM NADH, 2 mM NAD+ and 20 mM Pi (inorganic phosphates). Ethanol

production can be summarized by the following equation:


C6H12O6

2 C2H5OH + 2 CO2 + 2ATP

What is the maximum amount of ethanol that can be produced under these conditions?
A.
B.
C.
D.
E.

2 mM
20 mM
40 mM
200 mM
400 mM

Question 10. Thyroid hormone release is due to the action of thyroid-stimulating hormone (TSH)
released by the pituitary gland. Release of TSH is governed by the TSH-releasing hormone (TRH)
which is synthesized in the hypothalamus and released into the pituitary gland.
The graph below shows the concentration of TSH in human blood during the three hours following
an injection of TRH in two groups of people, A and B. One group was treated with thyroxine in the
days prior to the experiment. Consider endocrine feed-back regulation when answering the
following question.

Which of the statement(s) below are true?


I.
II.
III.
IV.

Thyroxine inhibits the release of TSH


Group B has been treated with thyroxin daily before treatment with TRH
TRH is needed for stimulating excretion of TSH
TSH inhibits the release of thyroxin

IBO 2007. Theoretical Exam 1

A.
B.
C.
D.
E.

I and II
II and III
III and IV
I and IV
I and III

Question 11. Typical intracellular concentrations of the Na+, K+ and Ca2+ ions are 15 mM, 120
mM and 100 nM, respectively. In the cell we are interested in the usual Na+, K+ leak channels that
are present as well as the Na+/K+ pump (ATPase). Extracellularly, the concentrations of Na+, K+
and Ca2+ are 140 mM, 4 mM and 2 mM, respectively. Which of the following would happen if the
extracellular K+ concentration were to be increased to 10 mM?
I.
II.
III.
IV.
V.

Intracellular Na+ would increase


There would be increased ATP utilization
There is an large increase in intracellular Ca2+
Intracellular Na+ would decrease
Intracellular K+ would increase
A.
B.
C.
D.
E.

I
I, II, III
III
III, IV, V
IV, V

Question 12. Thermogenesis is a process where heat is generated. The energy present in the reducing

equivalents such as NADH + H+ or FADH2 in mitochondria is normally used to pump protons across the
inner mitochondrial membrane to the intermembranous space. This proton gradient is the motive force for
ATP production. Examine the figures below and consider whether ATP synthesis or thermogenesis
predominates when answering the following question.
The molecules represented by I, II, III and IV represent mitochondrial electron carriers.

IBO 2007. Theoretical Exam 1

10

In which of the three situations shown in the figure above does thermogenesis predominate over
ATP synthesis?
A.
B.
C.
D.
E.

only I
only II
only III
I and II
I and III

Question 13. The figure below outlines the glycolytic pathway. There are several regulatory steps
in glycolysis. A major regulatory step in glycolysis is the conversion of fructose 6-phosphate to
fructose 1,6-biphosphate by phosphofructose kinase. This enzyme is allosterically inhibited by
ATP and allosterically activated by AMP. Thus, cellular ATP:AMP ratios are important in the
regulation of phosphofructose kinase. In addition, low pH inhibits phosphofructose kinase activity.

11

IBO 2007. Theoretical Exam 1

What effect will poisoning of mitochondrial function by the mitochondrial uncoupler dinitrophenol
(DNP) have on glycolysis?
A. It will increase the rate of glycolysis if there is a means of oxidizing NADH.
B. It will result in the immediate death of the cell.
C. It will increase the rate of glycolysis if there is a means of further increasing the
reduction of NAD+.
D. It will inhibit the conversion of phosphoenol pyruvate to pyruvic acid.
E. It will promote the formation of 1,3 biphosphoglycerate from 3-phosphoglycerate.
Question 14. Lions (Panthera leo) live in stable social groups called prides which usually have
three or more adult females, their dependent offspring and one or two dominant adult males. The
old and weak male(s) in a pride may be driven away by other strong males or by a new coalition of
males.
Which combination of the following statements is correct.?
I. Females born into a pride leave before they reproductive maturity.
II. Males born into a pride remain there for life.
III. Females born into a pride remain there for life.
IV. New dominant male try to kill only newly born females.
V. Males born into a pride leave before they reach reproductive activity.
VI. New dominant male try to kill only newly born males.
VII. Adult females in a lion pride are never related to each other.
VIII. New dominant male try to kill as many young cubs as possible.
IX. Adult females in a lion pride are often related each other.
A.
B.
C.
D.
E.

I, IV, VI, VII


III, V, VIII, IX
III, IV, V, IX
II, V, VI, VIII
I, II, VII, VIII

Proportional Abundance "

Question 15. The correct statement pertaining to the following Rank-Abundance Curve is:
Community A

Community B

Abundance Rank "

12

IBO 2007. Theoretical Exam 1

A.
B.
C.
D.
E.

Species richness in Community A is lower than in Community B


Species richness in Community A is higher than in Community B
Species diversity in Community A is lower than in Community B
Species diversity in Community A is higher than in Community B
Species evenness in Community A is higher than in Community B

Question 16. Stromatolites, layered mounds created by cyanobacteria, have been found in shallow
waters. They resemble small rocks but are organic in origin. Fossilised stromatolites are thought to
be important because they are suggestive of:
A.
B.
C.
D.
E.

the origin of earth.


the origin of photo-autotrophy.
oxidation of iron in oceans.
the appearance of the ozone layer in the atmosphere.
the origin of life.

Questions 17 18. A student studied the influence of temperature and light intensity upon CO2
flux of plants in a greenhouse. During the experiment cellular respiration is not influenced by light
intensity and cellular respiration of glucose is completely aerobic. At each temperature CO2 uptake
was measured during light exposure and loss of CO2 was measured during the dark period. The
light intensity was constant during the light period and was not a limiting factor for photosynthesis.
The data collected are presented in the following table.
Temp (C)

CO2 uptake in light*

Loss of CO2 in
dark*

5
0.5
10
0.7
15
1.2
20
1.9
25
2.3
30
2.0
35
1.5
* units: mg per gram dried weight per hour

0.2
0.5
0.9
1.5
2.6
3.9
3.3

Question 17. At which temperatures does the plant release O2 when exposed to light?
A.
B.
C.
D.
E.

only in the range 5 20 C


only in the range 20 25 C
only at temperatures over 20 C
only at temperatures over 25 C
at all temperatures

IBO 2007. Theoretical Exam 1

13

Question 18. The optimum temperature for photosynthesis and the optimum temperature of respiration
is somewhere in the range of 5 - 35 C. Which of the following statements is correct?
A. optimum temp for photosynthesis < optimum temp for dissimilation
B. optimum temp for photosynthesis = optimum temp for dissimilation
C. optimum temp for photosynthesis > optimum temp for dissimilation
Question 19. A woman with Turner syndrome is found to be haemophilic (X linked recessive
phenotype). Both her mother and her father have normal blood coagulation.
Which of the statements below gives what you consider to be the best answer to each of following
questions?
I.

How can the simultaneous origin of Turner syndrome and haemophilia by abnormal
chromosome behavior during meiosis be explained?
II. Did the abnormal chromosome behavior occur in the mother or the father?
III. During which division of meiosis did the abnormal chromosome behavior occur?
A.

The father of the woman with Turner syndrome (XXX) must have been a
carrier for haemophilia, an X-linked recessive disorder. Nondisjunction occurred in her
mother. An egg lacking a sex chromosome was fertilized with a sperm with X
chromosome carrying the haemophilic allele. The nondisjunctive event could have
occurred only during first meiotic division.

B.

One of the parents of the woman with Turner syndrome (X0) must have been a
carrier for haemophilia, an X-linked recessive disorder. Because her father has
normal blood coagulation, she could not have obtained her only X chromosome
from him. Therefore nondisjunction occurred in her father. A sperm lacking a
sex chromosome fertilized an egg with X chromosome carrying the haemophilic
allele. The nondisjunctive event could have occurred during either meiotic
division.

C.

One of the parents of the woman with Turner syndrome (XXY) is a carrier for
haemophilia, an X-linked recessive disorder. Because her mother has normal blood
coagulation, she could not have obtained her X choromosome from her mother.
Therefore, nondisjunction occurred in her mother. A sperm with a sex chromosome
carrying the haemophilic allele fertilized an egg with XX chromosome. The
nondisjunctive event could have occurred during second meiotic division.

D.

One of the parents of the woman with Turner syndrome is a carrier for haemophilia, an
X-linked recessive disorder. Because her father has normal blood coagulation, she has
obtained her only X choromosome from her mother. A nondisjunction occurred in her
father during either meiotic division. A sperm lacking a sex chromosome fertilized an
egg with X chromosome carrying the haemophilic allele.

IBO 2007. Theoretical Exam 1

14

Questions 20 21. A rare human disease afflicted a family as shown in the following pedigree.

Question 20. What is the most likely mode of inheritance of this disease?
A.
B.
C.
D.
E.

Mode of inheritance is autosomal recessive.


Mode of inheritance is autosomal dominant.
Mode of inheritance is X-linked recessive.
Mode of inheritance is X-linked dominant
Mode of inheritance could not be deduced.

Question 21. What is the probability that the first child of the marriage between cousins, 1 x 4, is
a boy with the disease?
A.
B.
C.
D.
E.

1/2
1/4
1/8
1/16
0

Questions 22 - 23. The wild-type flower color of harebell plants (genus Campanula) is blue.
Using radiation, three mutants with white petals were produced, white 1, white 2 and white 3. They
all look the same, so it was not known whether they were genetically identical. The mutant strains
are available as homozygous pure-breeding lines.

15

IBO 2007. Theoretical Exam 1

The mutant strains were crossed with the wild-type blue genotype and with each other to produce
the following results:
Parental cross

F1 phenotype

F2 segregation ratio

White 1 x blue

all blue

3/4 blue : 1/4 white

White 2 x blue

all blue

3/4 blue : 1/4 white

White 3 x blue

all blue

3/4 blue : 1/4 white

White 1 x white 2

all white

no data available

White 1 x white 3

all blue

no data available

White 2 x white 3

all blue

no data available

Question 22. Using these results, determine which statement is the correct conclusion for this
study.
A.
B.
C.
D.

The mutant genes in white 1 and 3 are allelic and are different to the mutant gene in
white 2.
The mutant genes in white 2 and 3 are allelic and are different to the mutant gene in
white 1.
The mutant genes in white 1 and 2 are allelic and are different to the mutant gene
in white 3.
The mutant genes in white 1, 2 and 3 are all allelic.

Question 23. The type of gene action operating among the crosses between the mutants in this
study is
A.
B.
C.
D.

complete dominance.
dominant epistasis.
recessive (complementary) epistasis
duplicate gene interaction.

Question 24 - 25. Hemoglobin in the erythrocytes of adults is composed of a combination of


two !-globin molecules and two "-globin molecules. Sickle-cell anemia is caused by the
substitution of a single amino acid in the "-globin subunit.
In 1957, Vernon M. Ingram and his colleagues investigated the amino acid sequences of normal and
sickle-cell anemia hemoglobins in several short peptide chains obtained by trypsin digestion. A
difference in the fourth peptide between both types of "-globin was found and further hydrolytic
digestion of the fourth peptides revealed six hydrolyzed products.

IBO 2007. Theoretical Exam 1

16

# the fourth peptide products of normal "-globin were (amino acid residues are abbreviated by
the following letters: V=valine, H= histidine, L= leucine, T= threonine, P= proline, E= glutamic
acid and K= lysine):
VH
VHL
VHLT
TPE
TPEEK
EK
# the fourth peptide products of "-globin of sickle cell anemia were
VH
VHL
VHLT
TPV
TPVEK
EK
Question 24. From these results, how many amino acids is the fourth peptide composed of and
what was the substituted position of amino acid residue counting from the N-terminus?
From the following, choose the one statement which is most appropriate. Assume that this fourth
peptide contains only one molecule of T (threonine).
A.
B.
C.
D.
E.

It was composed of 8 amino acids and the 6th amino acid was substituted.
It was composed of 8 amino acids and the 3rd amino acid was substituted.
It was composed of 7 amino acids and the 6th amino acid was substituted.
It was composed of 7 amino acids and the 3rd amino acid was substituted.
It was composed of 9 amino acids and the 6th amino acid was substituted.

Question 25. Below is a DNA sequence coding a part of the amino acid sequence in the fourth
peptide of normal "-globin. In sickle cell anemia, it is known that a mutation occurs in the region
enclosed by
.
From the following, choose one that is an appropriate DNA sequence of the mutation.
Normal! TGAGGACTCCTCTTCAGA
A.
B.
C.
D.
E.

TGAGGACCCTCTTCAGA
TGAGGACTACCTCTTCAGA
TGAGGACACCTCTTCAGA
TGAGGACCTCTTCAGA
TGAGGAACTCCTCTTCAGA

17

IBO 2007. Theoretical Exam 1

Question 26 - 28. The diagram below represents a nephron from an adult human.

3
4

Question 26. At which of the numbered points would the filtrate be hypertonic to the blood?
A.
B.
C.
D.
E.

1 and 3 only
1, 2 and 3
2 and 3 only
4 only
3 and 4

Question 27. At which of the numbered points is sodium reabsorbed from the filtrate?
A.

1 only

B.

1 and 2 only

C.

1, 2 and 3

D.

1, 2 and 4

E.

4 only

IBO 2007. Theoretical Exam 1

18

Question 28. The open arrow shows the direction of blood flow into the Glomerulus. What
happens if the diameter of the blood vessel is constricted at point X?
A.
B.
C.
D.
E.

More sodium will appear in the urine


Water reabsorption will be decreased
The rate of ultrafiltration will be increased
The rate of urine production will be reduced
Glucose will be appear in the urine

Question 29. A and B are two 70 Kg individuals with same body water volume. Both of them had
a snack that had a high salt content, and B also drank a glass of an alcoholic drink . Based on this
information, which one of following statements is true?
A.
B.
C.
D
E.

A will have a lower circulating level of antiduretic hormone (ADH) than B


B will have a lower circulating level of antiduretic hormone (ADH) than A
Both of them will have the same level of circulating ADH
A will have less body water than B
B will produce less urine than A

Question 30. Which of the following RNA sequences would hybridize most effectively with the
DNA sequence 5 - ATA CTT ACT CAT TTT 3?
A.
B.
C.
D.
E.

5 AAA AAC GUC CCC UAA 3


5 ATA CTT ACT CAT TTT 3
5 UAU GAA UGA GUA AAA 3
5 AAA AUG AGU AAG UAU 3
5 AAA ATG AGT AAG TAT 3

Question 31. What does a small standard deviation indicate about data obtained from an
experiment?
A.
B.
C.
D.
E

The data are not reliable.


More data needs to be collected.
More of the values are above the mean than below the mean
The data are grouped closely around the mean.
More of the group values are below the mean than above the mean.

19

IBO 2007. Theoretical Exam 1

IMPORTANT

ANSWERS TO QUESTIONS 32 TO 59 ARE


TO BE WRITTEN IN THIS EXAM
BOOKLET.

STARTING AT THE NEXT PAGE, WRITE


YOUR STUDENT CODE AT THE TOP OF
EVERY PAGE IN THIS EXAM BOOKLET

20

IBO 2007. Theoretical Exam 1

Question 32. For blood under each of the conditions described below, select the letter of the oxyhemoglobin dissociation curve with which it is most likely to be associated. (3 marks)

Oxygen
saturation
(%)

1.

Normal adult arterial blood

2.

Blood stored for 2 weeks

3.

Anaemic blood

Foetal blood

5.

Blood exposed to CO

6.

Blood from a person with hypothermia

7.

Blood with PaCO2 above normal

8.

Blood with an increased pH

E
A

A
C
A

21

IBO 2007. Theoretical Exam 1

Question 33. The following graph shows the concentration of thyroid-stimulating hormone (TSH)
in human blood during the 3 hours following an injection of TSH-releasing hormone (TRH) in two
groups of people (A and B). One group was treated with thyroxine daily for a week prior to the
experiment. (2 marks)

B
A

STATEMENT
a. Thyroxine treatment stimulated TSH release in Group A
b. Group A has been treated with thyroxine daily before treatment
with TRH
c. Group B has been treated with thyroxine daily before treatment
with TRH
d. Thyroxine treatment inhibited TSH release in Group B

True (1) or False (2)


2
2
1
1

22

IBO 2007. Theoretical Exam 1

Question 34. Digestion of food is facilitated by enzymes and hormones secreted at various regions
of the gastro-intestinal tract. Select the organs (identified by different letters) from the diagram
below that secretes the following enzymes and hormones: (4 marks)

B
C
D

I.

Amylase

_____A, C____

II.

Lipase

_____C, D____

III. Chymotrypsin

_____C_______

IV. Insulin

_____C_______

V.

_____D_______

Cholecystokinin (CCK)

VI. Aminopeptidase

_____D_______

VII. Gastrin

_____B_______

VIII.Carboxypeptidase

______C______

23

IBO 2007. Theoretical Exam 1

Questions 35 37. A 21 year-old student gets into a car accident and experiences brain trauma.
Use the figure below to answer the following questions.. Use the appropriate number to refer to the
region of the brain affected.

Question 35. The patient experiences lack of co-ordination and problems in balance. What
part of the brain is most likely damaged? (0.5 mark)
ANSWER:

Question 36. The patient slurs her speech and is unable to clearly read even simple passages
from a book. What part of the brain is most likely damaged? (0.5 mark)
ANSWER:
Question 37. The patient experiences double vision and images are blurry. What part of the
brain is most likely damaged? (0.5 mark)
ANSWER:

24

IBO 2007. Theoretical Exam 1

Question 38. To study hierarchial reaction in crickets (Gryllus campestris), five crickets, A, B, C,
D and E, were marked with colours and placed two by two in an experimental field. Observations
were made on their aggressive behaviour and the results are shown below:
Partner
B

Won
fights
6

Lost
fights
0

Won
fights
0

Lost
fights
6

Table 1 : Fight results for cricket A

Won
fights
9

Lost
fights
2

Partner

Table 2 : Fight results for cricket B

Won
fights
0

Lost
fights
7

Table 3 : Fight results for cricket C

Won
fights
6

Lost
fights
2

Partner

Partner

Partner

Table 4 : Fight results for cricket D

Table 5 : Fight results for cricket E

Indicate if the following statements are correct by writing the appropriate answer code in the
answer column of the following table. (3 marks)
Answer code :

1 = CORRECT

2 = INCORRECT

25

IBO 2007. Theoretical Exam 1

Statement

Answer

a. Cricket D is last in the hierarchical order.

b. Cricket E is first in the hierarchical order.

c. The hierarchy is linear: with the following order: C " E" A " B " D
d. Some crickets won fights against crickets that were higher in the
hierarchical order.

1
1

Question 39. According to the usual classification, birds are classified as vertebrates with feathers
and reptiles as epidermal scale vertebrates. A different phylogenetic classification has been
proposed and includes birds and crocodiles in the Archosaurian group.
Below are the two types of classification:
Usual classification

Phylogenetic classification

26

IBO 2007. Theoretical Exam 1

Comparison of selected anatomical characteristics of these vertebrates


Epidermal scales

Preorbital fenestra Gizzard Feathers

Eagle

covering feet

present

present

present

Ostrich

covering feet

present

present

present

present

present

none

Crocodile covering all the body


Boa

covering all the body

none

none

none

Lizard

covering all the body

none

none

none

Frog

none

none

none

none

Indicate if the following statements are correct by writing the appropriate answer code in the
answer column of the table. (2 marks)
Answer code :

No
h.
a.
b.
c.
d.
e.
f.
g.

1 = CORRECT

2 = INCORRECT

Statement

Answer

a. Birds and reptiles both have scales. Therefore, we can assume that
they share a common ancestor which is not a common ancestor of the
frog.
b. The eagle, ostrich and crocodile are homologous for the preorbital
fenestra feature
c. Possession of feathers is an ancestral characteristic, whereas the
possession of scales is a more recent modification.
d. Since crocodiles are more closely related to birds than to lizards, scales
are not a relevant characteristic to be used in this type of classification.

1
1
2

27

IBO 2007. Theoretical Exam 1

Question 40. Four tree communities were identified at four different locations to the north, south,
west and east of Ottawa, Canada. The communities are represented below, with each different tree
figure symbolizing a different species. (6 marks)
NORTH Community

SOUTH Community

WEST Community

No
a.

Community attribute
Highest species richness

b.

Lowest species richness

c.

Highest species evenness (balance)

d.

Lowest species evenness (balance)

e.

Highest species diversity

f.

Lowest species diversity

g.

Highest total abundance

h.

Lowest total abundance

EAST Community

A. North

ANSWER
B. South
C. West

D. East

!
!

!
!
!
!

28

IBO 2007. Theoretical Exam 1

Question 41. A survivorship curve depicts the age-specific mortality through survivorship.
Indicate whether the following statements about the survivorship are true. The graphs shown below
indicate different types of survivorship curves.

Circle whether each statement is TRUE or FALSE. (2 marks)


A.

Graph I represents organisms that provide good care of their offspring, such as humans
and many other large mammals.
TRUE

FALSE

B. Graph II is typical of survivorship curves for organisms such as many fishes and marine
invertebrates
TRUE

FALSE

C. Graph II is characteristics of the adult stages of birds after a period of high juvenile
mortality.
TRUE

FALSE

D. Birds may have a Graph III-type survivorship curve with a brief period of high mortality
among the youngest individuals, followed by increasing periods of lower mortality.
TRUE

FALSE

E. In populations where migration is common, survivorship is important factor in


determining changes in population size
TRUE

FALSE

29

IBO 2007. Theoretical Exam 1

Question 42. The following diagram represents the gymnosperm lifecycle.

To match the structures and processes involved in this lifecycle, match each number from the
diagram of the lifecycle with the letter of the proper term in the list below and write that letter in the
appropriate box. (3.5 marks)
Diagram
number

Term
from list

Diagram
number

Term
from list

Diagram
number

13

14

3
4

9
10

5
6

11
12

List of Terms
a. megasporangium produces eggs by meiosis
b. fertilization
c. zygote
d. embryo
e. diploid portion of life cycle
f. seedling
g. pollen cone, producing pollen

Term
from list

h. seed coat
i. mature sporophyte
j. integument
k. haploid portion of life cycle
l. megaspore mother cell
m. ovulate cone, bearing ovules
n. microsporangium produces pollen by
meiosis

30

IBO 2007. Theoretical Exam 1

Queston 43. The diagram below represents the stages in the mobilization of starch reserves in a
barley grain.
Water
I

III

IV

II

Starch
hhh

Water

Match the appropriate term with the correct Roman numeral from the diagram above. (Note: not all
terms have answers.) (3 marks)
TERM FROM
DIAGRAM

ANSWER

Alpha-amylase

III

Aleurone layer

Auxn
Gibberellic acid

II

Sugar

IV

Proten

Question 44. The structures in List B develop from the structures shown in List A. Match each
structure in List A with the appropriate structures in List B. Enter your answers in the table below.
(2.5 marks)
a.
b.
c.
d.
e.

List A
Microspore
Microsporophyll
Megaspore
Megasporangium
Megasporophyll

List B
1. Pollen sac
2. Primary cell of Embryo sac
3. Carpel
4. Nucellus
5. Pollen grain

31

IBO 2007. Theoretical Exam 1

LIST A

LIST B

a.

b.

c.

d.

e.

Question 45. Plants obtain various mineral nutrients from the soil. These nutrients have different
physiological roles in plants.
Match the elements/compounds from the left column with their functions in plants in the right
column. Write your answers in the answer table below. (5 marks)
1. Calcium

A. A cation that is important in the formation of turgor in stomata

2. Nitrogen
3. Nitrate

B. A nitrogen anionic compound that is accessible to plants in natural


ecosystems
C. Necessary for the synthesis of the side chains of cysteine and
methionine

4. Iodine

D. An element present in all amino acids, nucleotides and chlorophyll

5. Phosphate

E. A metal present in the chlorophyll molecule

6. Magnesium

F. Enables the crosslinking of pectates in the cell wall

7. Potassium

G. A component of DNA and RNA that is not a part of purine or


pyrimidine bases.

8. Sulfate

H. Is the most abundant metal in the electron transport chain proteins

9. Manganese

I. Participates in the photo-oxidization of water during photosynthesis

10. Iron

J. Is not essential for the growth of plants

Answer Table
1.
2.
F

3.

4.

5.

6.

7.

8.

9.

10.

32

IBO 2007. Theoretical Exam 1

Questions 46 - 48. The ways different substances can be transported through the biological
membrane is shown in Figure 1.

Exterior
Y

out.

in.

Dout.
out.

in.

out.

in.

Eout.

Fout.

Oout.
Mout.

Din.
P
Ein.

Fin.

P
Oin.
Min.

Interior

Figure 1. Transport of different substances through the biological membrane.

Question 46. Match the name of the transport systems to the letter(s) in Figure 1 (4 marks)
Transport mechanism
1. Conjugated active transport

Answer
M, O

2. Active transport (non-conjugated)

3. Exocytosis

4. Transport through membrane pores

5. Phagocytosis/pinocytosis;

6. Facilitated (mediated) diffusion;

7. Simple diffusion through membrane phospholipid bilayer

8. Co-transport

D, E

33

IBO 2007. Theoretical Exam 1

Question 47. Indicate which letters in Figure 1 correspond to each transport system. (4 marks)
Answer
9.

Direct membrane transport (without carrier)

10.

Mediated membrane transport (using specific systems of


carriers)

11.

Passive transport

12.

Active transport

Question 48. Identify from Figure 1 the correct example for each transport type. (4 marks)

Membrane transport type

Answer

13.

Na+, K+-$%Pase

14.

Low-density lipoproteins

15.

water, urea

16.

Inner mitochondrial membrane &+-$%Pase

17.

glucose, aminoacids

18.

Exchange of $DP for $%P across inner mitochondrial


membrane

19.

Long chain fatty acids and alcohols

20.

Hormonal secretion

M, O

D, E

34

IBO 2007. Theoretical Exam 1

Question 49. The total respiration (R) of a young growing plant can be described by the following
expression:
Total R = Maintenance R + Growth R
Some of the processes that occur during growth of this plant are:
1.
2.
3.
4.
5.
6.
7.

Movement of water within a cell


Reduction of nitrate (NO3-)-ions to ammonium (NH4 +)-ions
Uptake of K+ -ions through the plasma membrane of a endodermis cell
Uptake of CO2 in cells of palisade parenchyma
Opening and closing of stomata
Lengthening of a polypeptide chain
Absorption of light by chlorophyll A

Certain of these processes require energy, some supply energy to the plant and others are not
involved in energy use or supply. Indicate which processes require or supply energy by writing a +
(plus sign) and which processes have no energy involvement by writing a ' (minus sign) in the
correct places in the following table. (3.5 marks)
Process
number

Energy required/supplied (+)


or no energy involvement (-)

35

IBO 2007. Theoretical Exam 1

Question 50. The following diagram shows an ovule just prior to double fertilization.
Identify each of the structures indicated by an arrow and label it on the diagram with the
appropriate letter code from the table below. (4 marks)

A
F

G
D

STRUCTURE

LETTER CODE

Antipodal cell

Egg cell

Vegetative cell nucleus (Pollen tube nucleus)

Integument

Polar nuclei

Pollen tube

Sperm cell (male gamete)

Synergid cell

36

IBO 2007. Theoretical Exam 1

Question 51. A family consists of three children, David, Edna and Sophie and their parents
Alison and Alfred. One of the children is blood group A and is also red green colour blind . Edna is
blood group B and Sophie is blood group O. Of the children only David has blue eyes. Neither
parent is colour blind, but only Alfred has blue eyes and is blood group B.
Choose a possible genotype for each family member and write the appropriate capital letter for that
genotype against the name. (2.5 marks)
A = XC Xc AO Bb

B = XC Y AO bb

C = XC Xc BO Bb

D = XC Xc AO Bb

E = XC Y AO Bb

F = XC Xc OO Bb

G = XC Y BO bb

H = Xc Y AO bb

Family member

Genotype

David

Edna

Sophie

Alison

Alfred

Question 52. In Canada, 7,0 % of the male population is colorblind. This is a sex linked recessive
feature located on the X-chromosome. (1 mark)
What percentage of the female population, not being colorblind, is a carrier of alleles responsible
for colorblindness?
Answer: 13 %

37

IBO 2007. Theoretical Exam 1

Questions 53 - 54. The fox operon, which has sequences A, B, C, and D, encodes enzymes 1
and 2. Mutations in sequences A, B, C, and D have the following effects, where a plus sign (+) =
enzyme synthesized and a minus sign (-) = enzyme not synthesized. Fox is the regulator of fox
operon.
Mutation
sequence
No mutation
A
B
C
D

in

Fox absent
Enzyme 1
Enzyme 2

Fox present
Enzyme 1
Enzyme 2

+
+
+

+
+
+

Question 53. Is the fox operon inducible or repressible? Indicate your answer by writing X in the
appropriate place in the table below. (1 mark)
Inducible

Repressible

Question 54. Which sequence (A, B, C, or D) is part of the following components of the operon?
Match the correct letter against the component in the table below. (2 marks)
Component of operon

Answer

Regulator gene

Promoter

Structural gene for enzyme 1

Structural gene for enzyme 2

38

IBO 2007. Theoretical Exam 1

Question 55. The following is a list of mutational changes. For each of the specific mutations
described, indicate which of the following terms could apply, either as a description of the mutation
or as a possible cause. More than one term from the right column can apply to each statement in the
left column. (6 marks)
Write your answers in the answer table below.

Code

Description of mutation

Code

1.

An $-% base pair in the wild-type gene is


changed to a G-C pair

2.

An $-% base pair is changed to a %-$ pair

3.

Term

a.

transition

b.

base
substitution

The sequence $$GC%%$%CG is changed


to a $$GC%$%CG

c.

transversion

4.

The sequence $$GC%%$%CG is changed


to a $$GC%%%$%CG

d.

inversion

5.

The sequence $$CG%%$%CG is changed


to a $$%G%$%CG

e.

translocation

6.

The sequence $$CG%C$C$$C$C$%CG


changed to a $$CG%C$C$%CG

f.

7.

The gene map in a given chromosome arm


is changed from bog-rad-fox1-fox2-try-duf
(where fox1 and fox2 are highly
homologous, recently diverged genes) to
bog-rad-fox1-fox3-fox2-try-duf (where fox3
is a new gene with one end similar to fox1
and the other similar to fox2).

8.

The gene map in a chromosome is changed


from bog-rad-fox1-fox2-try-duf
to bog-rad-fox2-fox1-try-duf.

9.

deletion

g.

insertion

h.

deamination

i.

X-ray
irradiation

j.

intercalator

k.

unequal
crossingover

The gene map in a given chromosome is


changed from bog-rad-fox1-met-qui-txusqm to bog-txu-qui-met-fox1-rad- sqm

Answer table
1

a. b

b, c

39

IBO 2007. Theoretical Exam 1

Question 56. The wild type tryptophan synthetase enzyme of E. coli contains a glycine (Gly) at
position 38. Two trp mutants A23 and A46 have been isolated which have arginine (Arg) instead of
glycine at position 38 (A23) and glutamate (Glu) at position 38 (A46). Both mutants were plated on
minimal medium and from A23 four spontaneous revertants to prototrophy (i.e. are able to grow
without supplements) were obtained and from A46 three spontaneous revertants to prototrophy were
obtained. The tryptophan synthetase from each of the seven revertants were isolated and the amino
acids at position 38 were identified.
mutant

revertant

amino acid at position 38

A23

1
2
3
4

isoleucine (Ile)
threonine (Thr)
serine (Ser)
glycine (Gly)

A46

1
2
3

glycine (Gly)
alanine (Ala)
valine (Val)

A summary of these data is given below. Using the genetic code table provided on the next page,
deduce the codons for the wild type, mutants A23 and A46 and for the revertants and place each
designation in the box provided. (5 marks)
wild type
Gly

GGA
A23 mutant

A46 mutant
Arg

Glu

AGA

GAA

revertants
Gly
Ile
AUA

Thr
ACA

Ser
AGC/AGU

Gly
GGA

GGA

Ala

Val

GCA

GUA

40

IBO 2007. Theoretical Exam 1

GENETIC CODE TABLE

This table shows the 64 codons and the amino acid each codon codes for. The direction is 5' to 3'.
2nd base
U

UUU (Phe/F)Phenylalanine UCU (Ser/S)Serine

UAU (Tyr/Y)Tyrosine

UGU (Cys/C)Cysteine

UUC (Phe/F)Phenylalanine UCC (Ser/S)Serine

U UUA (Leu/L)Leucine

UAC (Tyr/Y)Tyrosine

UGC (Cys/C)Cysteine

UCA (Ser/S)Serine

UAA Ochre (Stop)

UGA Opal (Stop)

UUG (Leu/L)Leucine

UCG (Ser/S)Serine

UAG Amber (Stop)

UGG (Trp/W)Tryptophan

CUU (Leu/L)Leucine

CCU (Pro/P)Proline

CAU (His/H)Histidine

CGU (Arg/R)Arginine

CUC (Leu/L)Leucine

CCC (Pro/P)Proline

CAC (His/H)Histidine

CGC (Arg/R)Arginine

CUA (Leu/L)Leucine

CCA (Pro/P)Proline

CAA (Gln/Q)Glutamine

CGA (Arg/R)Arginine

CUG (Leu/L)Leucine

CCG (Pro/P)Proline

CAG (Gln/Q)Glutamine

CGG (Arg/R)Arginine

AUU (Ile/I)Isoleucine

ACU (Thr/T)Threonine AAU (Asn/N)Asparagine

AGU (Ser/S)Serine

AUC (Ile/I)Isoleucine

ACC (Thr/T)Threonine AAC (Asn/N)Asparagine

AGC (Ser/S)Serine

AUA (Ile/I)Isoleucine

ACA (Thr/T)Threonine AAA (Lys/K)Lysine

AGA (Arg/R)Arginine

AUG (Met/M)Methionine

ACG (Thr/T)Threonine AAG (Lys/K)Lysine

AGG (Arg/R)Arginine

GUU (Val/V)Valine

GCU (Ala/A)Alanine

GAU (Asp/D)Aspartic acid GGU (Gly/G)Glycine

GUC (Val/V)Valine

GCC (Ala/A)Alanine

GAC (Asp/D)Aspartic acid GGC (Gly/G)Glycine

GCA (Ala/A)Alanine

GAA (Glu/E)Glutamic acid GGA (Gly/G)Glycine

GCG (Ala/A)Alanine

GAG (Glu/E)Glutamic acid GGG (Gly/G)Glycine

C
1st
base

G GUA (Val/V)Valine

GUG (Val/V)Valine

41

IBO 2007. Theoretical Exam 1

Question 57. In a paternity suit the ABO phenotypes of the mother, the child and the two possible
fathers (F1 and F2) were determined, and a DNA profile was made for each person.
Both the mother (M) and the child (C) are type A, Rh-negative. Father F1 is type B, Rh-negative
and Father F2 is type O, Rh-negative. The DNA profiles are shown below.

Answer the following questions. (3 marks)


Question
a. The mother has the genotype Rr for the Rh
factor
b. The child has the genotype IAIo
c.

F1 cannot be the father

Answer: True (1) or False (2)


2

42

IBO 2007. Theoretical Exam 1

Question 58. DNA repair" mechanisms can be divided into 3 categories (listed below). A list of
repairing processes is also given.
Match each DNA repair mechanisms with the names of the repairing processes. (4.5 marks)

Name of repairing process


A. Mismatch repair
B. Recombinational (daughter-strand gap) repair
C. Nucleotide excision repair
D. Photoreactivation
E. Mutagenic repair (trans-lesion synthesis)

DNA repair mechanisms


1. Damage reversal
2. Damage removal
3. Damage tolerance

F. Ligation of single strand breaks


G. SOS repair
H. Base excition repair
I. Postreplicative Translesion Bypass Repair

Answer table
DNA repair mechanism
1. Damage reversal
2. Damage removal
3. Damage tolerance

Repairing process (list letter of all that apply)

43

IBO 2007. Theoretical Exam 1

Question 59. The pBR322 plasmid was cut with two different restriction enzymes. The patterns of
ethidium bromide staining of plasmid DNA after electrophoresis on agarose gels are shown.
P

+ HindIII

+BsuI

P
+BsuI + HindIII

a
b

c
d

f
g
h

Reference:
P: plasmid

Answer true or false: (2.5 marks)


1

( ..T ) The pBR322 has only one restriction site for HindIII.

(F) The restriction enzyme HindIII induces plasmid supercoiling.

(....T....) The pBR322 has two restriction sites for BsuI.

(....T....) The migration rate of a DNA molecule in an agarose gel is inversely proportional to
its size.
(....F.....) The bands in lane 4 indicate that both enzymes have the same restriction site.

44

IBO 2007. Theoretical Exam 1

Question 60. Information on the description and appearance of various chromosomal structural
arrangements is given below.

Type of chromosomal change

Definition of chromosomal change

1. Tandom duplication

A. Internal fragment of chromosome is missed.

2. Reciprocal translocation

B. Chromosomal segment is doubled in the opposite order.

3. Interstitial deletion

C. Two-way exchange of a segment of chromosome.

4. Pericentric inversion

D. Centromere containing part of chromosome is inverted.

5. Displaced duplication

E. The tip of chromosome is lost because of single break.

6. Interstitial translocation

F. Acentromeric part of chromosome is inverted.

7. Terminal deletion

G. Chromosomal segment is represented twice same as the

8. Reverse duplication
9. Paracentric inversion

original order.
H. Movement of a segment of chromosome from one to another
in one way.
K. Chromosomal segment is represented twice but it is not
adjacent to original segment

45

IBO 2007. Theoretical Exam 1

In the following table and using the appropriate letters and Roman numerals, match the correct
definition and appearance to the type of chromosomal change listed. (4.5 marks)

Type of chromosomal change

Definition

1. Tandem duplication
2. Reciprocal translocation
3. Interstitial deletion
4. Pericentric inversion
5. Displaced duplication
6. Interstitial translocation
7. Terminal deletion
8. Reverse duplication
9. Paracentric inversion

- THE END -

Appearance

18th INTERNATIONAL BIOLOGY OLYMPIAD


JULY 15 - 22, 2007

THEORY EXAMINATION # 2
Total marks possible: 53.5

Time allowed: 120 minutes

WRITE YOUR 4-DIGIT STUDENT NUMBER IN THE BOX


BELOW

STUDENT CODE

IBO 2007. Theoretical Exam 2.

GENERAL INSTRUCTIONS
Check that you have the correct examination paper and an answer sheet.

BE SURE TO RECORD ALL YOUR ANSWERS ON THE ANSWER SHEET


WHEN YOU HAVE FINISHED THE EXAM, PLACE YOUR ANSWER
SHEET INSIDE YOUR QUESTION PAPER AND HAND BOTH TO THE
INVIGILATOR BEFORE LEAVING THE EXAM ROOM.
REMEMBER TO WRITE YOUR 4-DIGIT STUDENT CODE ON THE
FRONT PAGE OF THE QUESTION PAPER.
Read each question carefully before attempting it.

IMPORTANT
! Use the answer sheet provided to record your answers.

! Ensure that your name and student code is PRINTED in the top margin of the front
page of the answer sheet. The markers will enter this information in the correct places
on the reverse side of the answer sheet.
! Use only the HB pencil provided to mark the answer sheet. Completely fill in the circle.

This is the correct way:

A
o

C
o

D
o

E
o

! DO NOT USE AN X OR ANY OTHER SYMBOL TO MARK YOUR


ANSWER.
! If you want to change your answer, use the eraser to completely erase your incorrect
response and fill in the new circle you require.
Question 1. The diagram below shows a section through a mammalian ovary.
! There is ONLY ONE CORRECT ANSWER to each question.
! Each question (except Question 31) is worth one mark.
! Marks will NOT be deducted for incorrect answers.

IBO 2007. Theoretical Exam 2.

Question 1. The diagram below shows a section through a mammalian ovary.

Which one of the following is the correct sequence of the development of the structures indicated
by the letters A to E?
A.
B.
C.
D.
E.

A, C, D, B, E
A, B, D, C, E
C, B, D, A, E
D, B, C, A, E
E, B, D, C, A

Question 2. Oogenesis differs substantially from spermatogenesis. Which of the following


statements concerning oogenesis is INCORRECT?
A.
B.
C.
D.
E.

Cytokinesis is unequal during the meiotic divisions


The sequence from secondary oocyte to ovum is interrupted by a relatively long
rest period
The first meiotic division is not completed unless the egg is reactivated by a hormone
A mature ovum has not completed its second meiotic division
The number of potential gametes is, by and large, established at birth

IBO 2007. Theoretical Exam 2.

Questions 3 7. Examine the figure below:


Hormone Levels during the Human Female Reproductive Cycle

A
B

C
D

Question 3. Which of the following correctly lists the hormones in order from A to D?
A.
B.
C.
D.
E.

estrogen, progesterone, LH, FSH


estrogen, FSH, progesterone, LH
LH, FSH, progesterone, estrogen
LH, estrogen, FSH, progesterone
LH, FSH, estrogen, progesterone

IBO 2007. Theoretical Exam 2.

Question 4. Which of the following statements is INCORRECT?


A.
B.
C.
D.

An increase in hormone B causes a decrease in hormones C and D


A steep rise in hormone C stimulates the production of hormones A and B.
A low level of hormone C inhibits the production of hormones A and B.
A high level of hormones C and D inhibits the secretion of hormones A and B

Question 5. Ovulation is triggered by a peak in the hormone whose level is shown by the
A.
B.
C.
D.

Line A
Line B
Line C
Line D

Question 6. Hormones A and B are secreted by the

A.
B.
C.
D.

uterine wall
ovary
hypothalamus
anterior pituitary

Question 7. Hormones C and D are secreted by the

A.
B.
C.
D.

uterine wall
ovary
hypothalamus
anterior pituitary

Question 8. One hypothesis predicts that most of the CO2 produced in the soil originates from
microorganisms feeding on dead plant material. To which trophic level do these microorganisms
belong?
A.
B.
C.
D.
E.

Primary producers
Secondary producers
Decomposers
First order consumers
Second order consumers

IBO 2007. Theoretical Exam 2.

Question 9. Joan and Claude (neither have cystic fibrosis) come to you seeking genetic counseling.
Claude was married before, and he and his first wife had a child with cystic fibrosis, an autosomal
recessive condition. A brother of Joans died of cystic fibrosis and Joan has never been tested for
the gene. If they marry, what is the probability that Joan and Claude will have a son that WILL
NOT be a carrier for, nor have cystic fibrosis?
A.
B.
C.
D.
E.

1/12
1/8
1/6
1/4
1/2

Question 10.
A.
B.
C.
D.
E.

Chromosomal crossing over occurs in which of the following stages of cell division?

Prophase of mitosis.
Metaphase of mitosis.
Prophase I of meiosis.
Metaphase II of meiosis.
Telophase I of meiosis.

Question 11. A man whose blood group is Type A has two boys. The plasma of one of the boys
agglutinates the red cells of his father, but the plasma from the other son does not. Which statement
is incorrect?
A.
B.
C.
D.
E.

The father must be heterozygous for the A blood type allele.


The mother of the son that agglutinates his fathers blood can be type AB.
The boy that agglutinates could have type O blood.
The mother of the son that agglutinates must possess a type O allele.
The boy that doesnt cause agglutination can be type AB.

Question 12. In peas, the allele for smooth seed coat (S) is dominant to wrinkled (s), Tall plant (T)
is dominant to short (t) and yellow coloured seed (Y) is dominant to green (y).
A plant with the genotype SsTtyy was test crossed and 145 progeny survived to maturity.
Approximately how many of these progeny are expected to be tall plants with green wrinkled seeds?
A.
B.
C.
D.

9
18
36
72

IBO 2007. Theoretical Exam 2.

Question 13. B chromosomes are additional chromosomes possessed by some, but not all,
individuals in a population. Which combination of statements is correct?
I.
II.
III.
IV.
V.

They occur only in plants.


While they are common in plants, they occur also in fungi, insects and animals.
They arise from normal chromosomes by fragmentation.
They are normal, but short, chromosomes..
In plants they are associated with reduced viability.
A.
B.
C.
D.

I, III and V
I, IV and V
II, III and V
II, IV and V

Question 14. Often the frequency of a particular deleterious allele is very different in
neighbouring populations. For example, the frequency of the allele causing cystic fibrosis is 0.02 in
Population A and 0.006 in Population B. Such a difference in allele frequencies between two close
populations is probably the result of
A.
B.
C.
D.
E.

The occurrence of the founder effect in an earlier generation


More effective repair of DNA damage caused by mutation
Selective advantage of the allele in one population but not the other
Recurring migration between the populations
Non-random mating.

IBO 2007. Theoretical Exam 2.

Question 15. The coefficient of relatedness is a theoretical value determined by the number of
alleles that would be the same between two organisms.
The diagram below shows the relationships between some honeybees. What is the coefficient of
relatedness between II-2 and II-3; and between II-2 and III-1?

II

III

ANSWER

Between II-2 and


II-3

Between II-2 and


III-1

A.

0.50

0.50

B.

0.75

0.50

C.

0.75

1.00

D.

0.25

1.00

E.

0.50

0.75

IBO 2007. Theoretical Exam 2.

Question 16. Collenchyma and sclerenchyma are plant support tissues. Which combination of the
following statements correctly differentiates these two types of plant tissue?
I.

Collenchyma occurs only in Dicotyledons; sclerenchyma is an elastic tissue that is found in


both Monocotyledons and Dicotyledons.
II. Collenchyma cells are developed during growth; sclerenchyma cells generally occur in organs
that have concluded their longitudinal growth.
III. Collenchyma and sclerenchyma may arise from the same cell type.
IV. Collenchyma cells have primary walls only while sclerenchyma cells have secondary walls.
V. Collenchyma originates from the protoderm; sclerenchyma is formed by the procambium.
A.
B.
C.
D.

I, II, III.
II, III, IV
II, IV, V
IV, V

Question 17. The following figure shows the carbon fixation reactions during
photosynthesis in a typical C4 plant :
Vascular bundle

CELL 1

CELL 2
epidermis

Which of the following answers indicate the dominant carbon fixation enzyme in each of the
two cells?
Answer
Cell 1
Cell 2
A.

Malate dehydrogenase

Sucrose synthase

B.

PEP Carboxylase

Rubisco

C.

Rubisco

PEP Carboxylase

D.

Aspartate aminotransferase

Malate dehydrogenase

E.

Malic enzyme

Pyruvate
dehydrogenase

10

IBO 2007. Theoretical Exam 2.

Question 18. Suppose that an illuminated suspension of Chlorella (a photosynthetic alga) was
actively carrying out photosynthesis when the light was suddenly switched off. How would the
levels of 3-phosphoglycerate and ribulose 1,5-bisphosphate change during the next minute?
A.
B.
C.
D.
E.

The concentration of 3-phosphoglycerate would increase and that of ribulose 1,5bisphosphate would increase.
The concentration of 3-phosphoglycerate would increase; the concentration of
ribulose1,5-bisphosphate would decrease.
The concentration of 3-phosphoglycerate would decrease; the concentration of ribulose
1,5-bisphosphate would increase.
The concentration of 3-phosphoglycerate would decrease; the concentration of ribulose
1,5-bisphosphate would decrease.
The concentration of 3-phosphoglycerate would remain the same; the concentration of
ribulose 1,5-bisphosphate would decrease.

Question 19. Which of the following statements shows the difference between the reaction sites of
photosystem I and II?
A.
B.
C
D.
E.

Chlorophyll a is only found in photosystem I; chlorophyll b is found in photosystem II.


Each preferentially absorbs slightly different wavelengths of light.
One is located in the thylakoid membrane; the other occurs in the stroma.
Only photosystem I is found in the thylakoid membranes.
None of these statements are correct.

Question 20. You are a biotechnologist designing novel eukaryotic enzymes that are regulated by
phosphorylation. Which amino acid residues shown below would you most likely use at the
regulatory site?

I.

IV.

A.
B.
C.
D.
E.

III.

II.

I, III
I, IV
I, II, III
II, IV, V
III, IV, V

V.

11

IBO 2007. Theoretical Exam 2.

Question 21. A biologist has discovered two new species of micro-organisms. Micro-organism A
was isolated from a hot spring whereas micro-organism B was obtained from a tropical forest. DNA
was isolated from both organisms and an analysis of the melting profile of each DNA sample was
carried out. The melting temperature (Tm) was 80 C for DNA from micro-organism A, and 70 C
for DNA from micro-organism B.
Which statement best describes the reason for this difference in values?
A.
B.
C.
D.
E.

DNA of micro-organism A has higher A+T content


DNA of micro-organism A has higher G+A content
DNA of micro-organism A has higher G+C content
DNA of micro-organism A has higher T+G content
DNA of micro-organism A has a higher proportion of TGA codons

Question 22. Which of the following cofactors is not redox active?


A.
B.
C.
D.
E.

Coenzyme A
Flavin coenzyme
NADH
Vitamin D
Coenzyme A and Vitamin D
A.
B.
C.
D.
E.

I
II
III
IV
I, III

Questions 23-24. Two cells have the following characteristics:


Characteristic

Cell I

Cell II

Cell wall

Present

Present

Ribosomes

Present

Present

Nucleus

Absent

Present

Ability to photosynthesize

Present

Absent

Cell respiration

Present

Present

IBO 2007. Theoretical Exam 2.

Question 23. From the characteristics presented in the table, which statement is correct?
A.
B.
C.
D.
E.

Cell I is more complex in its organization than cell II


Cell I is a prokaryote
Cells with all characteristics of cell II appeared earlier in the fossil record than cells
with all characteristics of cell I.
Cell II does not have a cell membrane
Both groups of cells are from fungi

Question 24. Cell II is a


A.
B.
C.
D.
E.

plant cell
eubacterium
archaebacterium
animal cell
cyanobacterium

Question 25. Suppose Species B disappears from an ecosystem in which the interrelationship
among the component species can be described by the food web below.

Which of the following will be a consequence of its elimination?


A.
B.
C.
D.
E.

Species X loses its only prey.


Species A loses its only prey.
Species D benefits because it is most distant from Species B.
Species C benefits because the competition between species B and species C is
reduced.
The disappearance of species B has no effect on species C or species D.

12

13

IBO 2007. Theoretical Exam 2.

Question 26. You have been asked by an international organization to initiate a biodiversity
conservation project on a tropical island off the south coast of Java, and far from any large
continental land. For this purpose, you must identify from three islands the island that has the
highest number of species.
The following information is provided:
Island Name
Boa
Ibo
Bio

Size/Area
418 km2
500 km2
420 km2

Distance from Java


220 km
800 km
450 km

Which of the following statements describes your decision?


A. Bio Island
B. Boa Island
C. Ibo Island
D. Either Boa Island or Bio Island
E. Insufficient information is provided to allow you to make a decision.
Question 27. Marine bony fishes have much lower internal osmotic concentration than the
seawater around them. Which of the following statements DOES NOT EXPLAIN the osmotic
regulation of marine bony fishes:
A.
B.
C.
D.
E.

They lose water by osmosis and gain salt by diffusion


They drink seawater
They actively absorb sodium chloride across gills
They absorb sodium chloride from stomach
They absorb water from stomach

Question 28. The best description of the relationships between fundamental niches (FN) and
realized niches (RN) of two competing species A and B that coexist is:
A.
B.
C.
D.
E.

FNA = RNA; FNB = RNB


FNA > RNA; FNB = RNB
FNA < RNA; FNB < RNB
FNA > RNA; FNB > RNB
FNA = RNA; FNB > RNB

IBO 2007. Theoretical Exam 2.

14

Question 29. Use the information given in Figures 1 and 2 to answer this question.

Figure 1. An example of a phylogeny showing characters by which taxa are recognised. Characters
1 4 are synapomorphies, 5 12 are autapomorphies and 13 is an attribute seen in the salmon and
the shark.

Figure 2. Two possible ways to organize the data from Figure 1.


Which of the following statements best describes the information presented in Figures 1 and 2?
A.
B.
C.
D.
E.

The tree shown in Figure 1 is the most parsimonious tree possible using these
characters.
The Lamprey and the Lizard are the oldest because they have the longest line.
The four groups shown in Figure 1 are equally related because they are all at the same
horizontal level.
X, Y and Z are characteristics common to all groups.
The Lamprey is more closely related to the Shark than to the Salmon or the Lizard.

IBO 2007. Theoretical Exam 2.

15

Question 30. A woman visits her doctor after noticing several changes in her body over a period
of 6 (six) months. She has noticed weight loss, intolerance to temperature variations, irregular
menstrual cycles, insomnia, and general weakness. Based on these symptoms, you would expect
the doctor to test her for:
A.
B.
C.
D.

Diabetes mellitus
Hyperthyroidism
Hypothyroidism
Hypoglycemia

Question 31. Endocrine glands


A.
B.
C.
D.

Produce hormones that are only secreted into the digestive tract
Release most hormones into the bloodstream
Release hormones that generally act as rapidly as nerve impulses
Are present only in vertebrates

Question 32. Long corolla length in tobacco is inherited as a recessive monogenic characteristic.
If in a natural population 49% of the plants have a long corolla, what is the probability that the
result of test crossing a randomly selected plant with a short corolla from this population in F1 will
have uniform progeny?
!.
B.
C.
D.
E.

100%
50%
30%
18%
0%

Question 33. From an evolutionary viewpoint, which of the five following individuals is the most fit?
!.
B.
C.
D.
E.

A child who does not become infected with any of the usual childhood diseases, such as
measles or chicken pox.
A woman of 40 with seven adult offspring
A woman of 80 who has one adult offspring
A 100-year old man with no offspring
A childless man who can run a mile in less than five minutes

IBO 2007. Theoretical Exam 2.

16

Question 34. A study of a grass population growing in an area of irregular rainfall found that
plants with alleles for curled leaves reproduced better in dry years, whereas plants with alleles for
flat leaves reproduced better in wet years. Curled and flat leaves are controlled by different alleles
at the same gene locus.
This situation tends to
A.
B.
C.
D.
E.

cause genetic drift in the grass population


cause gene flow in the grass population
lead to directional selection in the grass population
preserve variability in the grass population
lead to uniformity in the grass population

Question 35. The cohesion-tension (C-T) theory of sap ascent states that in plants sap is
transported against gravity by bulk flow through the xylem vessels or chains of tracheids. Which of
the following statements correctly describes the main factors affecting this bulk flow?
A.
B.
C.
D.

Hydrogen bonds within the water, hydrogen bonding to the hydrophilic walls of the
xylem cells, and the gradient of solute potential (! s).
The gradient of the pressure potential (! p), and solute concentration
The gradient of water potential (!), hydrogen bonds within the water and solute
concentration.
Hydrogen bonds within the water, hydrogen bonding to the hydrophilic walls of
the xylem cells, and the gradient of pressure potential (! p).

Questions 36-38. Plants maintain most Indole Acetic Acid (IAA) (an auxin) in conjugated
forms, which complicates IAA quantification.
Question 36. These forms can exist in forms such as IAA-amino acid conjugates
A.
B.
C.
D.

AA-amino acid conjugates


IAA-hydrolase conjugates
IAA-glycerol conjugates
IAA-peroxide conjugates

Question 37. These forms can exist in forms such as IAA-amino acid conjugates. Before analysis
in order to measure total IAA in a particular tissue, these conjugates must be
A.
B.
C.
D.

dehydrated
dehydrogenized
hydrolyzed
synthesized

IBO 2007. Theoretical Exam 2.

17

Question 38. Therefore free IAA, thought to be the active form of the hormone, is measured
A.
B.
C.
D.

in the same sample without enzymes


in the same sample with enzymes
in a parallel sample without enzymes
in a parallel sample with enzymes

Question 39. You find a mutant bacterium that synthesizes lactose-digesting enzymes whether or
not lactose is present. Which of the following statements or combination of statements might
explain this?
I. The operator has mutated such that it is no longer recognised by the repressor.
II. The gene that codes for the repressor has mutated and the repressor is no longer effective.
III. The gene or genes that code for the lactose-digesting enzymes have mutated.
A.
B.
C.
D.
E.

Only I
Only II
Only I, II
Only I, III
I, II, III

Question 40.
A.
B
C.
D.
E.

What mechanism is responsible for the acidification of the lysosome?

A lysosome fuses with acidic vesicle derived from Golgi aparatus


A pump transports protons from the cytosol into the lysosome
A pump transports protons from the lysosomal lumen to the cytosol
A lysosome fuses with acidic endocytosed material
A pump transports OH- ions from the cytosol to the lysosomal lumen

Question 41. Which of the following is an example of active transport?


A.
B.
C.
D.
E.

K+ through a voltage-gated K+ channel


Ca2+ through a voltage-gated ion channel
Na+ through ligand-gated ion channel
3 Na+ in exchange for 2 K+ across the plasmalemma
All of the above

IBO 2007. Theoretical Exam 2.

Question 42. The transport of glucose into the mammalian red blood cell is accomplished by
A.
B.
C.
D.
E.

simple diffusion through the phospholipid bilayer


a Na+ - K+ ATPase
esterifying the glucose to phosphatidate
first converting glucose into lactose
facilitated diffusion through a glucose transporter

Question 43. Isopods are one of the few crustacean groups that have successfully invaded
terrestrial habitats. Which of these statements is INCORRECT?
A.
B.
C.
D.

They live in dry conditions.


They must live in moist conditions.
Their abdominal appendages bear gills.
They do not have an efficient cuticular covering to conserve water.

Question 44. Hormones are essential to maintaining homeostasis mainly because


A.
B
C.
D

they catalyze specific chemical reactions in brain cells.


the body requires them for digesting food.
they cause specific responses in specific targets.
they act faster than nerve impulses.

Question 45.
A.
B.
C.
D.

Compared to nerve impulses, hormones are generally

released more slowly and have longer lasting effects.


released faster and have longer lasting effects.
release more slowly and have effects of a shorter duration.
released faster and have effects of shorter duration.

Question 46. Someone who has suffered damage to the pancreas might
A.
B.
C.
D.

have difficulty maintaining normal cortisol levels


have abnormal blood calcium levels.
have periods of very low energy.
experience fluctuating blood pressure

Question 47.
A.
B.
C.
D.

The hypothalamus

sends nerve impulses and also makes hormones.


directly stimulates the adrenal gland to produce glucocorticoids
belongs to both the nervous and circulatory systems.
regulates circadian rhythms in vertebrates.

18

IBO 2007. Theoretical Exam 2.

19

Questions 48 - 49. Ten grams of plant material were homogenized in 50 ml buffer and the
homogenate was centrifuged. Protein from 10 ml of the supernatant was precipitated by
addition of ammonium sulphate and the protein precipitated was collected by centrifugation
and re-suspended in 1 ml of buffer. The re-suspended protein was diluted 10 times for protein
determination.
Question 48. The amount of protein in 1 ml of the diluted sample was 0.4 mg. What is the total
amount of protein recovered from 10 ml of the supernatant?
A.
B.
C.
D.

2 mg
4 mg
6 mg
8 mg

Question 49. The amount of protein in 1 ml of the diluted sample was 0.4 mg. What is the amount
of protein extracted from 100 g tissue?
A.
B.
C.
D.

0.2 g
0.4 g
0.6 g
0.8 g

Question 50. Prion diseases are characterized by:


A.
B.
C.
D.
E.

cellular DNA damage.


misfolded proteins that are much more soluble than the regular form of the protein.
misfolded proteins that are prone to aggregation and are very stable.
abnormal enzyme activity.
protein chaperones in cells.

Question 51. Why are some proteinases synthesized as inactive precursors known as zymogens
(proenzymes)?
A.
B.
C.
D.
E.

Because they dont degrade a cells starch supply.


Zymogens have a higher degree of substrate specificity than most enzymes.
Zymogen synthesis ensures that proteinase activity is kept to a minimum inside
the cell where they are synthesised.
Zymogens are better at interconverting energy than regular enzymes.
Zymogens are more resistant to protein denaturation than the regular proteinase.

IBO 2007. Theoretical Exam 2.

20

Question 52. Negative feedback is a process that


Which of the following statements is INCORRECT? Negative feedback:
A. always reduces the amount of a hormone present in the blood.
A. is the main mechanism maintaining endometrial blood supply during pregnancy
B. keeps conditions near their normal state.
B. is responsible for varying urine osmolarities over the period of a day
C. lowers the body temperature below normal.
C. results in small fluctuations in physiological parameters
D. none of the above are correct.
D. is used in regulation of sympathetic and parasympathetic functions
Question 53. What is the role of the second messenger in hormone action?
A.
B.
C.
D.

it signals a cell to secrete a hormone.


it informs a gland as to whether its hormones are having an effect.
it relays a hormones message inside a target cell.
it carries a hormone while it is in the blood.

Question 54. It takes much longer for sex hormones and other steroids to produce their effect that
it takes nonsteroid hormones. Why?
A.
B.
C.
D.

Steroids are bigger, slower moving molecules.


Steroids usually must be carried longer distances by the blood.
Steroids generally cause target cells to make new proteins, which take time.
Steroids relay their message via a second messenger.

Questions 55 - 57. To test the origin of CO2 available in the soil, two experiments were
conducted on trees in a Pinus forest.
Question 55. In the first experiment, a 20 cm-wide strip of bark around the stem was removed
from trees mid-way between the ground and the lowest branch.
Which of the following statements correctly describes the effect of this treatment on the trees?
A.
B.
C.
D.
E.

Transpiration will cease. As a result, the tree will lose all its needles.
Transport of auxin in the xylem is prevented. This will cause increased auxin
concentration in the roots and root growth will be stimulated.
Phloem transport is prevented causing the roots to become deficient in nitrogen.
Transport of sugars to the roots ceases and the roots will die.
Transport of potassium and calcium from the roots to the needles will cease.

IBO 2007. Theoretical Exam 2.

21

Question 56. In the second experiment, the amount of CO2 released from the soil at the base of
trees was measured on several days during the growing period. The bark-removal experiment was
repeated on a total of 9 trees, three trees per teament. In the first treatment, the bark was removed
in early June (white triangles); in the second treatment, the bark was removed in late August (white
squares); the third treament was the control treatment where the bark was not removed (black
circles).
The data from this experiment are shown in the following graph. The black arrows indicate the
time of bark removal.

Which combination of the following statements best describe the results of this experiment?
I.

The variability of the different treatments overlap and any effect of bark removal is due to
chance.
II. The production of CO2 in the soil shows seasonal variation.
III. Bark removal in June had a much smaller effect on the total CO2 production in the soil during
the whole season than bark removal in August.
IV. The decrease in the CO2 production in the soil in the treatments where bark was removed
cannot be explained by seasonal variations alone.
V. The production of CO2 in the soil is always smaller for trees with bark removed than for
undamaged trees.
A.
B.
C.
D.
E.

Only I, II and V
Only I , II and IV
Only II, IV and V
Only II, III and IV
Only I, III and V

IBO 2007. Theoretical Exam 2.

22

Question 57. Which of the following statements is a valid conclusion for the results of the second
experiment?
A.
B.
C.
D.
E.

Most CO2 produced in the soil is due to the decomposition of dead roots.
Most CO2 produced in the soil is due to cellular respiration of root cells.
The amount of CO2 produced in the soil is not influenced by photosynthesis.
When most of the roots die, the production of CO2 in the soil is greatest.
The amount of CO2 produced in the soil depends on the soil temperature.

Questions 58 - 59. Below are six floral diagrams, labelled A to F.

Question 58. From the following, choose the combination in which the floral diagram is
associated with the correct family.
A.
B.
C.
D.
E.

A = Brassicaceae
B = Fabaceae
C = Liliaceae
D = Malvaceae
E = Rosaceae

IBO 2007. Theoretical Exam 2.

Question 59.

23

In which floral diagram do the all the floral characteristics listed below occur?

" Zygomorphic flower, fused sepals


" Axile placentation.
" Epipetalous stamen
A.
B.
C.
D.
E.

B
C
D
E
F

Question 60. Students made cross-sections of leaves collected from two different oak trees.
When they examined the sections under a microscope, they were surprised to see that the leaves
were different.
The following diagrams show cross-sections of leaves from Oak tree 1 and Oak tree 2.

Which of the following statements best explains the difference in leaf structure that the students
observed?
A.
B.
C.
D.
E.

Oak tree 1 grows in a swampy area; Oak tree 2 grows in sandy soil.
Oak tree 1 is a young tree; Oak tree 2 is a mature tree.
Oak tree 1 grows in fertile soil; Oak tree 2 grows in poor soil.
Oak tree 1 is exposed to sun for most of the day; Oak tree 2 grows in a shaded
area.
Oak tree 1 is infected by fungi, which induced the cell proliferation; Oak tree 2 was not
infected.

IBO 2007. Theoretical Exam 2.

24

Question 61. Cloning of a new DNA fragment into a circular plasmid/vector always requires:
A.
B.
C.
D.
E.

Complementary base pairing


DNA ligase activity
The presence of the same restriction site in the insert and the vector
Selectable markers and autonomous replicating sequences
All of the above

Question 62. Below is a diagram representing cholesterol in the phospholipid bilayer.

Cholesterol mixes with phospholipids in a cell membrane because cholesterol molecules are:
A.
B.
C.
D.
E.

amphipathic
steroid derivatives
entirely hydrophobic
phospholipids derivatives
bound with glycoproteins

Question 63. Which of the following molecules can diffuse through the mammalian phospholipid
bilayer without using a channel/transporter?
I.
II.
III.
IV.
V.

O2
glucose
steroid hormones
K+
amino acids
A.
B.
C.
D.
E.

I, III
I, IV
II, III, V
II, III, IV, V
All of the above.

IBO 2007. Theoretical Exam 2.

25

Question 64. What is the net charge of aspartic acid when the pH of the solution in which it is
prepared is the same as its pI value? Note the three pKa values of aspartic acid are as follows:
-COOH pKa = 2.1; -NH3+ pKa = 9.8; R group pKa = 3.9
A.
B.
C.
D.
E.

one net positive charge


two net positive charges
one net negative charge
two net negative charges
no net charge

Question 65. A quantitative amino acid analysis reveals that bovine serum albumin (BSA)
contains 0.58% tryptophan residues by weight. The molecular mass of the tryptophan molecules is
204 daltons. The molecular mass of bovine serum albumin is known to be approximately 66000
daltons. What number of tryptophan residues is present in each BSA molecule?
A.
B.
C.
D.
E.

2
3
4
5
6

Question 66. What essential function does gastrulation perform for the developing embryo?
A.
B.
C.
D.
E.

It results in the dorsal-ventral axis formation.


It gives rise directly to the germ layers of the embryo.
It gives rise to neural crest cells.
It gives rise directly to endocrine cells.
It gives rise directly to the trophectoderm.

Question 67. Which of the following is not true about the lymphatic system?
A.
B.
C.
D.
E.

It helps maintain the volume and protein concentration of the blood.


It helps defend the body against infection.
It transports fats from digestive tract to circulatory system.
Lymph composition is similar to that of interstitial fluid.
Lymph drains directly into the excretory system.

26

IBO 2007. Theoretical Exam 2.

Question 68. The most direct consequence on amphibian development upon removal of the grey
crescent would be:
A.
B.
C.
D.
E.

Inability to develop from the 2-cell stage to the 4-cell stage.


Inability to develop form the 4-cell stage to the 8-cell stage.
Inability to form a blastocoel.
Inability to form dorsal structures.
Inability to form ventral structures.

- THE END -

EXAM SUMMARY STATISTICS IBO 2007


Practical Exams
Marks

Animal
Biology
Raw
%
scores

Plant Biology

Theoretical Exams

Biochemistry

Genetics

Exam 1

Exam 2

Raw
scores

Raw
scores

Raw
scores

Raw
scores

Raw
scores

Maximum
possible

63

100

77

100

44

100

43

100

99

100

53.5

100

Maximum

62.6

99.4

74.0

96.1

44.0

100.0

39.0

90.8

92.5

93.4

50.5

94.4

Minimum

18.3

29.0

17.5

22.7

12.0

27.3

7.0

16.3

34.6

35.0

14.0

26.2

Mean

49.2

78.2

59.3

77.1

36.7

84.1

22.8

53.2

69.9

70.6

38.2

71.3

8.7

13.8

9.9

12.8

6.7

15.1

5.7

12.9

11.2

11.3

7.2

13.5

Standard
deviation
CV (%)

17.6

MEAN SCORE
DISCRIMINATING INDEX

16.6

18.0

73.1%
19.1

24.2

15.9

18.9

71.0%
17.4

DISTRIBUTION OF THEORETICAL EXAM QUESTIONS BY SUBJECT AREA


IBO 2007
(before Jury revision)
Subject Area

Recommended
distribution (%)

No. of
questions

%
distribution

Over/
under

Mark
value of
questions

%
distribution

Over/
under

Cell Biology

20

30

24

+4%

42.5

23

+3%

15

19

15

37.5

20

+5%

25

34

27

+2%

39.5

21

-4%

Ethology

-3%

-2%

Genetics and
Evolution

20

24

20

42.5

23

+3%

Ecology

10

-4%

11.5

-4%

Biosystematics

+2%

Plant Anatomy
and
Physiology
Animal
Anatomy and
Physiology

DISTRIBUTION OF THEORETICAL EXAM QUESTIONS BY SUBJECT AREA


IBO 2007
(after Jury revision)
Subject Area

Recommended
distribution (%)

No. of
questions

%
distribution

Over/
under

Mark
value of
questions

%
distribution

Over/
under

Cell Biology

20

26

25

+5%

35.0

23

+3%

15

18

18

+3%

31.5

21

+6%

25

25

25

31.5

21

-4%

Ethology

-4%

4.0

-2%

Genetics and
Evolution

20

21

21

+1%

36.0

24

+4%

Ecology

10

-6%

7.0

-5%

Biosystematics

+2%

7.5

Plant Anatomy
and
Physiology
Animal
Anatomy and
Physiology

IBO 2007 Theoretical Exam 1 Item Analysis

Question
1
2
3
4
5
6
7
8
9
10
11
12
13
14
15
16
17
18
19
20
21
22
23
24
25
26
27
28
29
30
31
Mean
SD

Point
Difficulty
Serial
Discrimination
index
correlation
Index*
(p-score)
(Rit)

% Correct response
Upper 27%
Lower 27%

0.59
0.65
0.66
0.87
0.70
0.92
Skipped
0.47
0.39
Skipped
Skipped
0.40
0.53
0.87
Skipped
0.63
0.59
0.20
Skipped
0.64
0.74
0.74
0.50
0.75
0.89
0.48
0.47
0.63
0.68
0.72
0.81

0.38
0.48
0.41
0.25
0.43
0.28

0.481
0.615
0.442
0.192
0.481
0.154

44
48
46
51
47
51

19
16
23
41
22
43

0.15
0.49

0.212
0.558

28
36

17
7

0.28
0.41
0.31

0.327
0.481
0.212

28
42
51

11
17
40

0.18
0.24
0.23

0.231
0.212
0.231

36
38
20

24
27
8

0.25
0.39
0.34
0.32
0.44
0.40
0.49
0.55
0.34
0.39
0.38
0.33

0.288
0.365
0.442
0.327
0.423
0.269
0.615
0.808
0.385
0.404
0.365
0.288

41
48
51
34
45
52
44
47
42
46
48
52

26
29
28
17
23
38
12
5
22
25
29
37

0.64
0.17

0.35
0.10

0.38
0.16

* A measure of the effectiveness of the question in discriminating between high and low scorers.

IBO 2007 Theoretical Exam 2 Item Analysis

Question
1
2
3
4
5
6
7
8
9
10
11
12
13
14
15
16
17
18
19
20
21
22
23
24
25
26
27
28
29
30
31
32
33
34
35
36
37
38
39
40
41
42
43
44

Point
Difficulty
Serial
Discrimination
index
correlation
Index*
(p-score)
(Rit)
Skipped
0.37
0.79
0.67
0.71
0.80
Skipped
0.98
0.30
0.95
0.65
0.86
Skipped
0.55
0.37
0.63
0.71
0.39
0.84
0.76
0.89
Skipped
0.96
0.46
0.78
Skipped
0.55
Skipped
Skipped
0.56
0.96
0.64
0.87
0.65
0.60
Skipped
0.88
0.32
0.86
0.84
0.81
0.80
0.73
Skipped

% Correct response
Upper 27% Lower 27%

0.43
0.40
0.25
0.25
0.47

0.46
0.37
0.23
0.29
0.44

32
50
38
46
52

8
31
26
31
29

0.24
0.40
0.23
0.39
0.29

0.06
0.44
0.12
0.48
0.21

52
30
52
48
49

49
7
46
23
38

0.20
0.19
0.20
0.33
0.33
0.31
0.44
0.18

0.29
0.29
0.23
0.37
0.46
0.31
0.46
0.15

35
28
41
46
35
50
51
50

20
13
29
27
11
34
27
42

0.30
0.34
0.21

0.12
0.44
0.21

52
36
50

46
13
39

0.45

0.56

41

12

0.18
0.28
0.42
0.31
0.28
0.24

0.23
0.08
0.50
0.29
0.29
0.33

33
52
45
52
42
40

21
48
19
37
27
23

0.33
0.27
0.52
0.36
0.37
0.33
0.36

0.23
0.27
0.44
0.23
0.35
0.33
0.42

49
23
52
47
50
51
47

37
9
29
35
32
34
25

45
46
47
48
49
50
51
52
53
54
55
56
57
58
59
60
61
62
63
64
65
66
67
68
Mean
SD

Skipped
0.75
0.82
Skipped
0.68
0.76
0.91
Skipped
0.91
Skipped
0.87
0.79
0.82
Skipped
0.68
0.72
0.06
0.49
0.68
0.68
0.87
0.82
0.88
0.63
0.72
0.19

0.34
0.29

43
48

27
30

0.51
0.38
0.56

0.31
0.35
0.00
0.62
0.40
0.31

48
50
52

16
29
36

0.46

0.27

52

38

0.44
0.44
0.31

0.29
0.35
0.25

51
47
47

36
29
34

0.34
0.37
0.23
0.35
0.36
0.36
0.30
0.20
0.39
0.25

0.42
0.44
0.13
0.46
0.42
0.42
0.19
0.25
0.27
0.29

46
48
7
37
46
46
52
49
51
41

24
25
0
13
24
24
42
36
37
26

0.33
0.10

0.31
0.14

* A measure of the effectiveness of the question in discriminating between high and low scorers.















All IBO examination questions are published under the following Creative Commons license:



CC BY-NC-SA (Attribution-NonCommercial-ShareAlike) https://creativecommons.org/licenses/by-nc-sa/4.0/
The exam papers can be used freely for educational purposes as long as IBO is credited and
new creations are licensed under identical terms. No commercial use is allowed.

18th INTERNATIONAL BIOLOGY OLYMPIAD


JULY 15 - 22, 2007

THEORY EXAMINATION # 1
Total marks possible: 99.0

Time allowed: 2.5 hours (150 minutes)

WRITE YOUR 4-DIGIT STUDENT NUMBER IN THE BOX


BELOW

STUDENT CODE

IBO 2007. Theoretical Exam 1

GENERAL INSTRUCTIONS
Check that you have the correct examination paper and an answer sheet.
WHEN YOU HAVE FINISHED THE EXAM, PLACE YOUR ANSWER SHEET INSIDE
YOUR QUESTION PAPER AND HAND BOTH TO THE INVIGILATOR BEFORE
LEAVING THE EXAM ROOM.
REMEMBER TO WRITE YOUR 4-DIGIT STUDENT CODE ON THE FRONT PAGE OF
THE QUESTION PAPER.
Read each question carefully before attempting it.

INSTRUCTIONS REGARDING RECORDING YOUR ANSWERS


QUESTIONS 1 - 31.
SHEET.

RECORD YOUR ANSWERS ON THE ANSWER

QUESTIONS 32 59. RECORD YOUR ANSWERS IN THE EXAM


QUESTION BOOKLET.
IMPORTANT
! Use the answer sheet provided to record your answers.
! Ensure that your name and student code is PRINTED in the top margin of the front
page of the answer sheet. The invigilators will enter this information in the correct
places on the reverse side of the answer sheet.
! Use only the HB pencil provided to mark the answer sheet. Completely fill in the circle.

This is the correct way:

! DO NOT USE AN X OR ANY OTHER SYMBOL TO MARK YOUR ANSWER.

! If you want to change your answer, use the eraser to remove your incorrect response and
fill in the new circle you require.
! There is only one correct answer to each question.
! Questions 1 - 30 are worth one mark each The mark value for questions 31 60 varies
according to the length and difficulty of the question.
! Marks will not be deducted for incorrect answers.

IBO 2007. Theoretical Exam 1

ANSWERS TO QUESTIONS 1 TO 31 ARE TO BE RECORDED ON THE


ANSWER SHEET.
Question 1. Which of the following statements is FALSE?
For almost every antigen you may encounter.
A.
B.
C.
D.
E.

a subset of B-cells already exists in your body specific to it.


a subset of T-helper cells already exists in your body that expresses a T-cell receptor
specific to it
a subset of phagocytes already exists in your body that attacks only that antigen.
a subset of antigen-specific antibodies already exists, but are not yet produced in large
numbers.
a subset of antigen-specific memory cells can be produced upon exposure to that
antigen.

Question 2. A blood smear of a human shows higher than normal numbers of eosinophils. Which
of the following may be occurring in his body?
ibo! 7/18/07 8:20 PM
Deleted:

A.
B.
C.
D.
E.

chronic nematode infection


anaphylactic shock
reduced white blood cells (leucopenia)
initial response to invading bacteria
hemostasis

IBO 2007. Theoretical Exam 1

Question 3. The ABO blood type of humans can be determined by a coagulation reaction with
anti-A and anti-B antibodies.

Positive coagulation

Negative Coagulation

Coagulation tests of persons blood produced the results shown below:


With anti-A and
anti-B antibodies

With anti-A
antibodies

With anti-B
antibodies

None

Which of the following statements can be deduced from the above?


A.
B.
C.
D.
E.

This persons blood contains anti-B antibodies.


This persons parents had to be type-A and type-O.
This person can receive neither type-A nor type-B blood.
Type-B antigens are present on the surface of this persons red blood cells.
This persons blood can be donated to both type-B and type-O individuals.

IBO 2007. Theoretical Exam 1

Question 4. The graph below shows the result of blood glucose test from a diabetes patient.

When tested 3 hours after having a carbohydrate rich meal, the blood glucose level of this patient
was 3 times higher than that of a normal individual. However, there was no difference in the level
of insulin in the blood between the two individuals.
Which of the following could be the reason for diabetic symptoms in this patient?
A.
B.
C.
D.
E.

Degradation of pancreatic beta-islets cells.


Degradation of pancreatic alpha-islets cells.
Abnormal proliferation of pancreatic beta-islet cells.
Reduced sensitivity of insulin-receptor mediated signal transduction.
Increased sensitivity of insulin-receptor mediated signal transduction.

IBO 2007. Theoretical Exam 1

Question 5. The following graph shows the dissociation curves for hemoglobin and myoglobin.

Based on the data presented in the graph, which of the following statements is true?
A.
B.
C.
D.
E.

The high affinity of myoglobin for O2 at low partial pressures of O2 prevents


hemoglobin from unloading O2 to muscle.
Myoglobin binds to oxygen with greater affinity than hemoglobin and unloads
oxygen after hemoglobin unloading.
Myoglobin helps hemoglobin bind as much O2 as possible from lungs.
Hemoglobin binds to O2 tightly thus preventing O2 from being made available to
skeletal muscle.
The high affinity of hemoglobin for O2 at low partial pressures of O2 prevents
myoglobin from unloading O2 to muscle.

IBO 2007. Theoretical Exam 1

Questions 6 - 8. This figure indicates some of the pathways involved in the metabolism of
food.

Question 6. When a person consumes a diet rich in carbohydrate, the reactions up-regulated will be:
A.
B.
C.
D.
E.
Question 7.
be:
A.
B.
C.
D.
E.
Question 8.
are:
A.
B.
C.
D.
E.

5, 6, 7
2, 8
5, 8
1, 3, 4
2, 5. 6
When a person performs heavy exercise, the reactions that will be down-regulated will
5, 6, 7, 8
1, 3, 4
4, 5, 6, 7
1, 2, 4
7, 3, 4
If a person suffers from carnitine deficiency, the reactions that will be down-regulated
6, 8 only
1, 3, 4
4, 5, 6, 7
2, 5, 6
5, 6, 7, 8

IBO 2007. Theoretical Exam 1

Question 9. A yeast extract contains all the enzymes required for alcohol production. The extract
is incubated under anaerobic conditions in 1 liter of medium containing: 200 mM glucose, 20 mM
ADP, 40 mM ATP, 2 mM NADH, 2 mM NAD+ and 20 mM Pi (inorganic phosphates). Ethanol

production can be summarized by the following equation:


C6H12O6

2 C2H5OH + 2 CO2 + 2ATP

What is the maximum amount of ethanol that can be produced under these conditions?
A.
B.
C.
D.
E.

2 mM
20 mM
40 mM
200 mM
400 mM

Question 10. Thyroid hormone release is due to the action of thyroid-stimulating hormone (TSH)
released by the pituitary gland. Release of TSH is governed by the TSH-releasing hormone (TRH)
which is synthesized in the hypothalamus and released into the pituitary gland.
The graph below shows the concentration of TSH in human blood during the three hours following
an injection of TRH in two groups of people, A and B. One group was treated with thyroxine in the
days prior to the experiment. Consider endocrine feed-back regulation when answering the
following question.

Which of the statement(s) below are true?


I.
II.
III.
IV.

Thyroxine inhibits the release of TSH


Group B has been treated with thyroxin daily before treatment with TRH
TRH is needed for stimulating excretion of TSH
TSH inhibits the release of thyroxin

IBO 2007. Theoretical Exam 1

A.
B.
C.
D.
E.

I and II
II and III
III and IV
I and IV
I and III

Question 11. Typical intracellular concentrations of the Na+, K+ and Ca2+ ions are 15 mM, 120
mM and 100 nM, respectively. In the cell we are interested in the usual Na+, K+ leak channels that
are present as well as the Na+/K+ pump (ATPase). Extracellularly, the concentrations of Na+, K+
and Ca2+ are 140 mM, 4 mM and 2 mM, respectively. Which of the following would happen if the
extracellular K+ concentration were to be increased to 10 mM?
I.
II.
III.
IV.
V.

Intracellular Na+ would increase


There would be increased ATP utilization
There is an large increase in intracellular Ca2+
Intracellular Na+ would decrease
Intracellular K+ would increase
A.
B.
C.
D.
E.

I
I, II, III
III
III, IV, V
IV, V

Question 12. Thermogenesis is a process where heat is generated. The energy present in the reducing

equivalents such as NADH + H+ or FADH2 in mitochondria is normally used to pump protons across the
inner mitochondrial membrane to the intermembranous space. This proton gradient is the motive force for
ATP production. Examine the figures below and consider whether ATP synthesis or thermogenesis
predominates when answering the following question.
The molecules represented by I, II, III and IV represent mitochondrial electron carriers.

IBO 2007. Theoretical Exam 1

10

In which of the three situations shown in the figure above does thermogenesis predominate over
ATP synthesis?
A.
B.
C.
D.
E.

only I
only II
only III
I and II
I and III

Question 13. The figure below outlines the glycolytic pathway. There are several regulatory steps
in glycolysis. A major regulatory step in glycolysis is the conversion of fructose 6-phosphate to
fructose 1,6-biphosphate by phosphofructose kinase. This enzyme is allosterically inhibited by
ATP and allosterically activated by AMP. Thus, cellular ATP:AMP ratios are important in the
regulation of phosphofructose kinase. In addition, low pH inhibits phosphofructose kinase activity.

11

IBO 2007. Theoretical Exam 1

What effect will poisoning of mitochondrial function by the mitochondrial uncoupler dinitrophenol
(DNP) have on glycolysis?
A. It will increase the rate of glycolysis if there is a means of oxidizing NADH.
B. It will result in the immediate death of the cell.
C. It will increase the rate of glycolysis if there is a means of further increasing the
reduction of NAD+.
D. It will inhibit the conversion of phosphoenol pyruvate to pyruvic acid.
E. It will promote the formation of 1,3 biphosphoglycerate from 3-phosphoglycerate.
Question 14. Lions (Panthera leo) live in stable social groups called prides which usually have
three or more adult females, their dependent offspring and one or two dominant adult males. The
old and weak male(s) in a pride may be driven away by other strong males or by a new coalition of
males.
Which combination of the following statements is correct.?
I. Females born into a pride leave before they reproductive maturity.
II. Males born into a pride remain there for life.
III. Females born into a pride remain there for life.
IV. New dominant male try to kill only newly born females.
V. Males born into a pride leave before they reach reproductive activity.
VI. New dominant male try to kill only newly born males.
VII. Adult females in a lion pride are never related to each other.
VIII. New dominant male try to kill as many young cubs as possible.
IX. Adult females in a lion pride are often related each other.
A.
B.
C.
D.
E.

I, IV, VI, VII


III, V, VIII, IX
III, IV, V, IX
II, V, VI, VIII
I, II, VII, VIII

Proportional Abundance "

Question 15. The correct statement pertaining to the following Rank-Abundance Curve is:
Community A

Community B

Abundance Rank "

12

IBO 2007. Theoretical Exam 1

A.
B.
C.
D.
E.

Species richness in Community A is lower than in Community B


Species richness in Community A is higher than in Community B
Species diversity in Community A is lower than in Community B
Species diversity in Community A is higher than in Community B
Species evenness in Community A is higher than in Community B

Question 16. Stromatolites, layered mounds created by cyanobacteria, have been found in shallow
waters. They resemble small rocks but are organic in origin. Fossilised stromatolites are thought to
be important because they are suggestive of:
A.
B.
C.
D.
E.

the origin of earth.


the origin of photo-autotrophy.
oxidation of iron in oceans.
the appearance of the ozone layer in the atmosphere.
the origin of life.

Questions 17 18. A student studied the influence of temperature and light intensity upon CO2
flux of plants in a greenhouse. During the experiment cellular respiration is not influenced by light
intensity and cellular respiration of glucose is completely aerobic. At each temperature CO2 uptake
was measured during light exposure and loss of CO2 was measured during the dark period. The
light intensity was constant during the light period and was not a limiting factor for photosynthesis.
The data collected are presented in the following table.
Temp (C)

CO2 uptake in light*

Loss of CO2 in
dark*

5
0.5
10
0.7
15
1.2
20
1.9
25
2.3
30
2.0
35
1.5
* units: mg per gram dried weight per hour

0.2
0.5
0.9
1.5
2.6
3.9
3.3

Question 17. At which temperatures does the plant release O2 when exposed to light?
A.
B.
C.
D.
E.

only in the range 5 20 C


only in the range 20 25 C
only at temperatures over 20 C
only at temperatures over 25 C
at all temperatures

IBO 2007. Theoretical Exam 1

13

Question 18. The optimum temperature for photosynthesis and the optimum temperature of respiration
is somewhere in the range of 5 - 35 C. Which of the following statements is correct?
A. optimum temp for photosynthesis < optimum temp for dissimilation
B. optimum temp for photosynthesis = optimum temp for dissimilation
C. optimum temp for photosynthesis > optimum temp for dissimilation
Question 19. A woman with Turner syndrome is found to be haemophilic (X linked recessive
phenotype). Both her mother and her father have normal blood coagulation.
Which of the statements below gives what you consider to be the best answer to each of following
questions?
I.

How can the simultaneous origin of Turner syndrome and haemophilia by abnormal
chromosome behavior during meiosis be explained?
II. Did the abnormal chromosome behavior occur in the mother or the father?
III. During which division of meiosis did the abnormal chromosome behavior occur?
A.

The father of the woman with Turner syndrome (XXX) must have been a
carrier for haemophilia, an X-linked recessive disorder. Nondisjunction occurred in her
mother. An egg lacking a sex chromosome was fertilized with a sperm with X
chromosome carrying the haemophilic allele. The nondisjunctive event could have
occurred only during first meiotic division.

B.

One of the parents of the woman with Turner syndrome (X0) must have been a
carrier for haemophilia, an X-linked recessive disorder. Because her father has
normal blood coagulation, she could not have obtained her only X chromosome
from him. Therefore nondisjunction occurred in her father. A sperm lacking a
sex chromosome fertilized an egg with X chromosome carrying the haemophilic
allele. The nondisjunctive event could have occurred during either meiotic
division.

C.

One of the parents of the woman with Turner syndrome (XXY) is a carrier for
haemophilia, an X-linked recessive disorder. Because her mother has normal blood
coagulation, she could not have obtained her X choromosome from her mother.
Therefore, nondisjunction occurred in her mother. A sperm with a sex chromosome
carrying the haemophilic allele fertilized an egg with XX chromosome. The
nondisjunctive event could have occurred during second meiotic division.

D.

One of the parents of the woman with Turner syndrome is a carrier for haemophilia, an
X-linked recessive disorder. Because her father has normal blood coagulation, she has
obtained her only X choromosome from her mother. A nondisjunction occurred in her
father during either meiotic division. A sperm lacking a sex chromosome fertilized an
egg with X chromosome carrying the haemophilic allele.

IBO 2007. Theoretical Exam 1

14

Questions 20 21. A rare human disease afflicted a family as shown in the following pedigree.

Question 20. What is the most likely mode of inheritance of this disease?
A.
B.
C.
D.
E.

Mode of inheritance is autosomal recessive.


Mode of inheritance is autosomal dominant.
Mode of inheritance is X-linked recessive.
Mode of inheritance is X-linked dominant
Mode of inheritance could not be deduced.

Question 21. What is the probability that the first child of the marriage between cousins, 1 x 4, is
a boy with the disease?
A.
B.
C.
D.
E.

1/2
1/4
1/8
1/16
0

Questions 22 - 23. The wild-type flower color of harebell plants (genus Campanula) is blue.
Using radiation, three mutants with white petals were produced, white 1, white 2 and white 3. They
all look the same, so it was not known whether they were genetically identical. The mutant strains
are available as homozygous pure-breeding lines.

15

IBO 2007. Theoretical Exam 1

The mutant strains were crossed with the wild-type blue genotype and with each other to produce
the following results:
Parental cross

F1 phenotype

F2 segregation ratio

White 1 x blue

all blue

3/4 blue : 1/4 white

White 2 x blue

all blue

3/4 blue : 1/4 white

White 3 x blue

all blue

3/4 blue : 1/4 white

White 1 x white 2

all white

no data available

White 1 x white 3

all blue

no data available

White 2 x white 3

all blue

no data available

Question 22. Using these results, determine which statement is the correct conclusion for this
study.
A.
B.
C.
D.

The mutant genes in white 1 and 3 are allelic and are different to the mutant gene in
white 2.
The mutant genes in white 2 and 3 are allelic and are different to the mutant gene in
white 1.
The mutant genes in white 1 and 2 are allelic and are different to the mutant gene
in white 3.
The mutant genes in white 1, 2 and 3 are all allelic.

Question 23. The type of gene action operating among the crosses between the mutants in this
study is
A.
B.
C.
D.

complete dominance.
dominant epistasis.
recessive (complementary) epistasis
duplicate gene interaction.

Question 24 - 25. Hemoglobin in the erythrocytes of adults is composed of a combination of


two !-globin molecules and two "-globin molecules. Sickle-cell anemia is caused by the
substitution of a single amino acid in the "-globin subunit.
In 1957, Vernon M. Ingram and his colleagues investigated the amino acid sequences of normal and
sickle-cell anemia hemoglobins in several short peptide chains obtained by trypsin digestion. A
difference in the fourth peptide between both types of "-globin was found and further hydrolytic
digestion of the fourth peptides revealed six hydrolyzed products.

IBO 2007. Theoretical Exam 1

16

# the fourth peptide products of normal "-globin were (amino acid residues are abbreviated by
the following letters: V=valine, H= histidine, L= leucine, T= threonine, P= proline, E= glutamic
acid and K= lysine):
VH
VHL
VHLT
TPE
TPEEK
EK
# the fourth peptide products of "-globin of sickle cell anemia were
VH
VHL
VHLT
TPV
TPVEK
EK
Question 24. From these results, how many amino acids is the fourth peptide composed of and
what was the substituted position of amino acid residue counting from the N-terminus?
From the following, choose the one statement which is most appropriate. Assume that this fourth
peptide contains only one molecule of T (threonine).
A.
B.
C.
D.
E.

It was composed of 8 amino acids and the 6th amino acid was substituted.
It was composed of 8 amino acids and the 3rd amino acid was substituted.
It was composed of 7 amino acids and the 6th amino acid was substituted.
It was composed of 7 amino acids and the 3rd amino acid was substituted.
It was composed of 9 amino acids and the 6th amino acid was substituted.

Question 25. Below is a DNA sequence coding a part of the amino acid sequence in the fourth
peptide of normal "-globin. In sickle cell anemia, it is known that a mutation occurs in the region
enclosed by
.
From the following, choose one that is an appropriate DNA sequence of the mutation.
Normal! TGAGGACTCCTCTTCAGA
A.
B.
C.
D.
E.

TGAGGACCCTCTTCAGA
TGAGGACTACCTCTTCAGA
TGAGGACACCTCTTCAGA
TGAGGACCTCTTCAGA
TGAGGAACTCCTCTTCAGA

17

IBO 2007. Theoretical Exam 1

Question 26 - 28. The diagram below represents a nephron from an adult human.

3
4

Question 26. At which of the numbered points would the filtrate be hypertonic to the blood?
A.
B.
C.
D.
E.

1 and 3 only
1, 2 and 3
2 and 3 only
4 only
3 and 4

Question 27. At which of the numbered points is sodium reabsorbed from the filtrate?
A.

1 only

B.

1 and 2 only

C.

1, 2 and 3

D.

1, 2 and 4

E.

4 only

IBO 2007. Theoretical Exam 1

18

Question 28. The open arrow shows the direction of blood flow into the Glomerulus. What
happens if the diameter of the blood vessel is constricted at point X?
A.
B.
C.
D.
E.

More sodium will appear in the urine


Water reabsorption will be decreased
The rate of ultrafiltration will be increased
The rate of urine production will be reduced
Glucose will be appear in the urine

Question 29. A and B are two 70 Kg individuals with same body water volume. Both of them had
a snack that had a high salt content, and B also drank a glass of an alcoholic drink . Based on this
information, which one of following statements is true?
A.
B.
C.
D
E.

A will have a lower circulating level of antiduretic hormone (ADH) than B


B will have a lower circulating level of antiduretic hormone (ADH) than A
Both of them will have the same level of circulating ADH
A will have less body water than B
B will produce less urine than A

Question 30. Which of the following RNA sequences would hybridize most effectively with the
DNA sequence 5 - ATA CTT ACT CAT TTT 3?
A.
B.
C.
D.
E.

5 AAA AAC GUC CCC UAA 3


5 ATA CTT ACT CAT TTT 3
5 UAU GAA UGA GUA AAA 3
5 AAA AUG AGU AAG UAU 3
5 AAA ATG AGT AAG TAT 3

Question 31. What does a small standard deviation indicate about data obtained from an
experiment?
A.
B.
C.
D.
E

The data are not reliable.


More data needs to be collected.
More of the values are above the mean than below the mean
The data are grouped closely around the mean.
More of the group values are below the mean than above the mean.

19

IBO 2007. Theoretical Exam 1

IMPORTANT

ANSWERS TO QUESTIONS 32 TO 59 ARE


TO BE WRITTEN IN THIS EXAM
BOOKLET.

STARTING AT THE NEXT PAGE, WRITE


YOUR STUDENT CODE AT THE TOP OF
EVERY PAGE IN THIS EXAM BOOKLET

20

IBO 2007. Theoretical Exam 1

Question 32. For blood under each of the conditions described below, select the letter of the oxyhemoglobin dissociation curve with which it is most likely to be associated. (3 marks)

Oxygen
saturation
(%)

1.

Normal adult arterial blood

2.

Blood stored for 2 weeks

3.

Anaemic blood

Foetal blood

5.

Blood exposed to CO

6.

Blood from a person with hypothermia

7.

Blood with PaCO2 above normal

8.

Blood with an increased pH

E
A

A
C
A

21

IBO 2007. Theoretical Exam 1

Question 33. The following graph shows the concentration of thyroid-stimulating hormone (TSH)
in human blood during the 3 hours following an injection of TSH-releasing hormone (TRH) in two
groups of people (A and B). One group was treated with thyroxine daily for a week prior to the
experiment. (2 marks)

B
A

STATEMENT
a. Thyroxine treatment stimulated TSH release in Group A
b. Group A has been treated with thyroxine daily before treatment
with TRH
c. Group B has been treated with thyroxine daily before treatment
with TRH
d. Thyroxine treatment inhibited TSH release in Group B

True (1) or False (2)


2
2
1
1

22

IBO 2007. Theoretical Exam 1

Question 34. Digestion of food is facilitated by enzymes and hormones secreted at various regions
of the gastro-intestinal tract. Select the organs (identified by different letters) from the diagram
below that secretes the following enzymes and hormones: (4 marks)

B
C
D

I.

Amylase

_____A, C____

II.

Lipase

_____C, D____

III. Chymotrypsin

_____C_______

IV. Insulin

_____C_______

V.

_____D_______

Cholecystokinin (CCK)

VI. Aminopeptidase

_____D_______

VII. Gastrin

_____B_______

VIII.Carboxypeptidase

______C______

23

IBO 2007. Theoretical Exam 1

Questions 35 37. A 21 year-old student gets into a car accident and experiences brain trauma.
Use the figure below to answer the following questions.. Use the appropriate number to refer to the
region of the brain affected.

Question 35. The patient experiences lack of co-ordination and problems in balance. What
part of the brain is most likely damaged? (0.5 mark)
ANSWER:

Question 36. The patient slurs her speech and is unable to clearly read even simple passages
from a book. What part of the brain is most likely damaged? (0.5 mark)
ANSWER:
Question 37. The patient experiences double vision and images are blurry. What part of the
brain is most likely damaged? (0.5 mark)
ANSWER:

24

IBO 2007. Theoretical Exam 1

Question 38. To study hierarchial reaction in crickets (Gryllus campestris), five crickets, A, B, C,
D and E, were marked with colours and placed two by two in an experimental field. Observations
were made on their aggressive behaviour and the results are shown below:
Partner
B

Won
fights
6

Lost
fights
0

Won
fights
0

Lost
fights
6

Table 1 : Fight results for cricket A

Won
fights
9

Lost
fights
2

Partner

Table 2 : Fight results for cricket B

Won
fights
0

Lost
fights
7

Table 3 : Fight results for cricket C

Won
fights
6

Lost
fights
2

Partner

Partner

Partner

Table 4 : Fight results for cricket D

Table 5 : Fight results for cricket E

Indicate if the following statements are correct by writing the appropriate answer code in the
answer column of the following table. (3 marks)
Answer code :

1 = CORRECT

2 = INCORRECT

25

IBO 2007. Theoretical Exam 1

Statement

Answer

a. Cricket D is last in the hierarchical order.

b. Cricket E is first in the hierarchical order.

c. The hierarchy is linear: with the following order: C " E" A " B " D
d. Some crickets won fights against crickets that were higher in the
hierarchical order.

1
1

Question 39. According to the usual classification, birds are classified as vertebrates with feathers
and reptiles as epidermal scale vertebrates. A different phylogenetic classification has been
proposed and includes birds and crocodiles in the Archosaurian group.
Below are the two types of classification:
Usual classification

Phylogenetic classification

26

IBO 2007. Theoretical Exam 1

Comparison of selected anatomical characteristics of these vertebrates


Epidermal scales

Preorbital fenestra Gizzard Feathers

Eagle

covering feet

present

present

present

Ostrich

covering feet

present

present

present

present

present

none

Crocodile covering all the body


Boa

covering all the body

none

none

none

Lizard

covering all the body

none

none

none

Frog

none

none

none

none

Indicate if the following statements are correct by writing the appropriate answer code in the
answer column of the table. (2 marks)
Answer code :

No
h.
a.
b.
c.
d.
e.
f.
g.

1 = CORRECT

2 = INCORRECT

Statement

Answer

a. Birds and reptiles both have scales. Therefore, we can assume that
they share a common ancestor which is not a common ancestor of the
frog.
b. The eagle, ostrich and crocodile are homologous for the preorbital
fenestra feature
c. Possession of feathers is an ancestral characteristic, whereas the
possession of scales is a more recent modification.
d. Since crocodiles are more closely related to birds than to lizards, scales
are not a relevant characteristic to be used in this type of classification.

1
1
2

27

IBO 2007. Theoretical Exam 1

Question 40. Four tree communities were identified at four different locations to the north, south,
west and east of Ottawa, Canada. The communities are represented below, with each different tree
figure symbolizing a different species. (6 marks)
NORTH Community

SOUTH Community

WEST Community

No
a.

Community attribute
Highest species richness

b.

Lowest species richness

c.

Highest species evenness (balance)

d.

Lowest species evenness (balance)

e.

Highest species diversity

f.

Lowest species diversity

g.

Highest total abundance

h.

Lowest total abundance

EAST Community

A. North

ANSWER
B. South
C. West

D. East

!
!

!
!
!
!

28

IBO 2007. Theoretical Exam 1

Question 41. A survivorship curve depicts the age-specific mortality through survivorship.
Indicate whether the following statements about the survivorship are true. The graphs shown below
indicate different types of survivorship curves.

Circle whether each statement is TRUE or FALSE. (2 marks)


A.

Graph I represents organisms that provide good care of their offspring, such as humans
and many other large mammals.
TRUE

FALSE

B. Graph II is typical of survivorship curves for organisms such as many fishes and marine
invertebrates
TRUE

FALSE

C. Graph II is characteristics of the adult stages of birds after a period of high juvenile
mortality.
TRUE

FALSE

D. Birds may have a Graph III-type survivorship curve with a brief period of high mortality
among the youngest individuals, followed by increasing periods of lower mortality.
TRUE

FALSE

E. In populations where migration is common, survivorship is important factor in


determining changes in population size
TRUE

FALSE

29

IBO 2007. Theoretical Exam 1

Question 42. The following diagram represents the gymnosperm lifecycle.

To match the structures and processes involved in this lifecycle, match each number from the
diagram of the lifecycle with the letter of the proper term in the list below and write that letter in the
appropriate box. (3.5 marks)
Diagram
number

Term
from list

Diagram
number

Term
from list

Diagram
number

13

14

3
4

9
10

5
6

11
12

List of Terms
a. megasporangium produces eggs by meiosis
b. fertilization
c. zygote
d. embryo
e. diploid portion of life cycle
f. seedling
g. pollen cone, producing pollen

Term
from list

h. seed coat
i. mature sporophyte
j. integument
k. haploid portion of life cycle
l. megaspore mother cell
m. ovulate cone, bearing ovules
n. microsporangium produces pollen by
meiosis

30

IBO 2007. Theoretical Exam 1

Queston 43. The diagram below represents the stages in the mobilization of starch reserves in a
barley grain.
Water
I

III

IV

II

Starch
hhh

Water

Match the appropriate term with the correct Roman numeral from the diagram above. (Note: not all
terms have answers.) (3 marks)
TERM FROM
DIAGRAM

ANSWER

Alpha-amylase

III

Aleurone layer

Auxn
Gibberellic acid

II

Sugar

IV

Proten

Question 44. The structures in List B develop from the structures shown in List A. Match each
structure in List A with the appropriate structures in List B. Enter your answers in the table below.
(2.5 marks)
a.
b.
c.
d.
e.

List A
Microspore
Microsporophyll
Megaspore
Megasporangium
Megasporophyll

List B
1. Pollen sac
2. Primary cell of Embryo sac
3. Carpel
4. Nucellus
5. Pollen grain

31

IBO 2007. Theoretical Exam 1

LIST A

LIST B

a.

b.

c.

d.

e.

Question 45. Plants obtain various mineral nutrients from the soil. These nutrients have different
physiological roles in plants.
Match the elements/compounds from the left column with their functions in plants in the right
column. Write your answers in the answer table below. (5 marks)
1. Calcium

A. A cation that is important in the formation of turgor in stomata

2. Nitrogen
3. Nitrate

B. A nitrogen anionic compound that is accessible to plants in natural


ecosystems
C. Necessary for the synthesis of the side chains of cysteine and
methionine

4. Iodine

D. An element present in all amino acids, nucleotides and chlorophyll

5. Phosphate

E. A metal present in the chlorophyll molecule

6. Magnesium

F. Enables the crosslinking of pectates in the cell wall

7. Potassium

G. A component of DNA and RNA that is not a part of purine or


pyrimidine bases.

8. Sulfate

H. Is the most abundant metal in the electron transport chain proteins

9. Manganese

I. Participates in the photo-oxidization of water during photosynthesis

10. Iron

J. Is not essential for the growth of plants

Answer Table
1.
2.
F

3.

4.

5.

6.

7.

8.

9.

10.

32

IBO 2007. Theoretical Exam 1

Questions 46 - 48. The ways different substances can be transported through the biological
membrane is shown in Figure 1.

Exterior
Y

out.

in.

Dout.
out.

in.

out.

in.

Eout.

Fout.

Oout.
Mout.

Din.
P
Ein.

Fin.

P
Oin.
Min.

Interior

Figure 1. Transport of different substances through the biological membrane.

Question 46. Match the name of the transport systems to the letter(s) in Figure 1 (4 marks)
Transport mechanism
1. Conjugated active transport

Answer
M, O

2. Active transport (non-conjugated)

3. Exocytosis

4. Transport through membrane pores

5. Phagocytosis/pinocytosis;

6. Facilitated (mediated) diffusion;

7. Simple diffusion through membrane phospholipid bilayer

8. Co-transport

D, E

33

IBO 2007. Theoretical Exam 1

Question 47. Indicate which letters in Figure 1 correspond to each transport system. (4 marks)
Answer
9.

Direct membrane transport (without carrier)

10.

Mediated membrane transport (using specific systems of


carriers)

11.

Passive transport

12.

Active transport

Question 48. Identify from Figure 1 the correct example for each transport type. (4 marks)

Membrane transport type

Answer

13.

Na+, K+-$%Pase

14.

Low-density lipoproteins

15.

water, urea

16.

Inner mitochondrial membrane &+-$%Pase

17.

glucose, aminoacids

18.

Exchange of $DP for $%P across inner mitochondrial


membrane

19.

Long chain fatty acids and alcohols

20.

Hormonal secretion

M, O

D, E

34

IBO 2007. Theoretical Exam 1

Question 49. The total respiration (R) of a young growing plant can be described by the following
expression:
Total R = Maintenance R + Growth R
Some of the processes that occur during growth of this plant are:
1.
2.
3.
4.
5.
6.
7.

Movement of water within a cell


Reduction of nitrate (NO3-)-ions to ammonium (NH4 +)-ions
Uptake of K+ -ions through the plasma membrane of a endodermis cell
Uptake of CO2 in cells of palisade parenchyma
Opening and closing of stomata
Lengthening of a polypeptide chain
Absorption of light by chlorophyll A

Certain of these processes require energy, some supply energy to the plant and others are not
involved in energy use or supply. Indicate which processes require or supply energy by writing a +
(plus sign) and which processes have no energy involvement by writing a ' (minus sign) in the
correct places in the following table. (3.5 marks)
Process
number

Energy required/supplied (+)


or no energy involvement (-)

35

IBO 2007. Theoretical Exam 1

Question 50. The following diagram shows an ovule just prior to double fertilization.
Identify each of the structures indicated by an arrow and label it on the diagram with the
appropriate letter code from the table below. (4 marks)

A
F

G
D

STRUCTURE

LETTER CODE

Antipodal cell

Egg cell

Vegetative cell nucleus (Pollen tube nucleus)

Integument

Polar nuclei

Pollen tube

Sperm cell (male gamete)

Synergid cell

36

IBO 2007. Theoretical Exam 1

Question 51. A family consists of three children, David, Edna and Sophie and their parents
Alison and Alfred. One of the children is blood group A and is also red green colour blind . Edna is
blood group B and Sophie is blood group O. Of the children only David has blue eyes. Neither
parent is colour blind, but only Alfred has blue eyes and is blood group B.
Choose a possible genotype for each family member and write the appropriate capital letter for that
genotype against the name. (2.5 marks)
A = XC Xc AO Bb

B = XC Y AO bb

C = XC Xc BO Bb

D = XC Xc AO Bb

E = XC Y AO Bb

F = XC Xc OO Bb

G = XC Y BO bb

H = Xc Y AO bb

Family member

Genotype

David

Edna

Sophie

Alison

Alfred

Question 52. In Canada, 7,0 % of the male population is colorblind. This is a sex linked recessive
feature located on the X-chromosome. (1 mark)
What percentage of the female population, not being colorblind, is a carrier of alleles responsible
for colorblindness?
Answer: 13 %

37

IBO 2007. Theoretical Exam 1

Questions 53 - 54. The fox operon, which has sequences A, B, C, and D, encodes enzymes 1
and 2. Mutations in sequences A, B, C, and D have the following effects, where a plus sign (+) =
enzyme synthesized and a minus sign (-) = enzyme not synthesized. Fox is the regulator of fox
operon.
Mutation
sequence
No mutation
A
B
C
D

in

Fox absent
Enzyme 1
Enzyme 2

Fox present
Enzyme 1
Enzyme 2

+
+
+

+
+
+

Question 53. Is the fox operon inducible or repressible? Indicate your answer by writing X in the
appropriate place in the table below. (1 mark)
Inducible

Repressible

Question 54. Which sequence (A, B, C, or D) is part of the following components of the operon?
Match the correct letter against the component in the table below. (2 marks)
Component of operon

Answer

Regulator gene

Promoter

Structural gene for enzyme 1

Structural gene for enzyme 2

38

IBO 2007. Theoretical Exam 1

Question 55. The following is a list of mutational changes. For each of the specific mutations
described, indicate which of the following terms could apply, either as a description of the mutation
or as a possible cause. More than one term from the right column can apply to each statement in the
left column. (6 marks)
Write your answers in the answer table below.

Code

Description of mutation

Code

1.

An $-% base pair in the wild-type gene is


changed to a G-C pair

2.

An $-% base pair is changed to a %-$ pair

3.

Term

a.

transition

b.

base
substitution

The sequence $$GC%%$%CG is changed


to a $$GC%$%CG

c.

transversion

4.

The sequence $$GC%%$%CG is changed


to a $$GC%%%$%CG

d.

inversion

5.

The sequence $$CG%%$%CG is changed


to a $$%G%$%CG

e.

translocation

6.

The sequence $$CG%C$C$$C$C$%CG


changed to a $$CG%C$C$%CG

f.

7.

The gene map in a given chromosome arm


is changed from bog-rad-fox1-fox2-try-duf
(where fox1 and fox2 are highly
homologous, recently diverged genes) to
bog-rad-fox1-fox3-fox2-try-duf (where fox3
is a new gene with one end similar to fox1
and the other similar to fox2).

8.

The gene map in a chromosome is changed


from bog-rad-fox1-fox2-try-duf
to bog-rad-fox2-fox1-try-duf.

9.

deletion

g.

insertion

h.

deamination

i.

X-ray
irradiation

j.

intercalator

k.

unequal
crossingover

The gene map in a given chromosome is


changed from bog-rad-fox1-met-qui-txusqm to bog-txu-qui-met-fox1-rad- sqm

Answer table
1

a. b

b, c

39

IBO 2007. Theoretical Exam 1

Question 56. The wild type tryptophan synthetase enzyme of E. coli contains a glycine (Gly) at
position 38. Two trp mutants A23 and A46 have been isolated which have arginine (Arg) instead of
glycine at position 38 (A23) and glutamate (Glu) at position 38 (A46). Both mutants were plated on
minimal medium and from A23 four spontaneous revertants to prototrophy (i.e. are able to grow
without supplements) were obtained and from A46 three spontaneous revertants to prototrophy were
obtained. The tryptophan synthetase from each of the seven revertants were isolated and the amino
acids at position 38 were identified.
mutant

revertant

amino acid at position 38

A23

1
2
3
4

isoleucine (Ile)
threonine (Thr)
serine (Ser)
glycine (Gly)

A46

1
2
3

glycine (Gly)
alanine (Ala)
valine (Val)

A summary of these data is given below. Using the genetic code table provided on the next page,
deduce the codons for the wild type, mutants A23 and A46 and for the revertants and place each
designation in the box provided. (5 marks)
wild type
Gly

GGA
A23 mutant

A46 mutant
Arg

Glu

AGA

GAA

revertants
Gly
Ile
AUA

Thr
ACA

Ser
AGC/AGU

Gly
GGA

GGA

Ala

Val

GCA

GUA

40

IBO 2007. Theoretical Exam 1

GENETIC CODE TABLE

This table shows the 64 codons and the amino acid each codon codes for. The direction is 5' to 3'.
2nd base
U

UUU (Phe/F)Phenylalanine UCU (Ser/S)Serine

UAU (Tyr/Y)Tyrosine

UGU (Cys/C)Cysteine

UUC (Phe/F)Phenylalanine UCC (Ser/S)Serine

U UUA (Leu/L)Leucine

UAC (Tyr/Y)Tyrosine

UGC (Cys/C)Cysteine

UCA (Ser/S)Serine

UAA Ochre (Stop)

UGA Opal (Stop)

UUG (Leu/L)Leucine

UCG (Ser/S)Serine

UAG Amber (Stop)

UGG (Trp/W)Tryptophan

CUU (Leu/L)Leucine

CCU (Pro/P)Proline

CAU (His/H)Histidine

CGU (Arg/R)Arginine

CUC (Leu/L)Leucine

CCC (Pro/P)Proline

CAC (His/H)Histidine

CGC (Arg/R)Arginine

CUA (Leu/L)Leucine

CCA (Pro/P)Proline

CAA (Gln/Q)Glutamine

CGA (Arg/R)Arginine

CUG (Leu/L)Leucine

CCG (Pro/P)Proline

CAG (Gln/Q)Glutamine

CGG (Arg/R)Arginine

AUU (Ile/I)Isoleucine

ACU (Thr/T)Threonine AAU (Asn/N)Asparagine

AGU (Ser/S)Serine

AUC (Ile/I)Isoleucine

ACC (Thr/T)Threonine AAC (Asn/N)Asparagine

AGC (Ser/S)Serine

AUA (Ile/I)Isoleucine

ACA (Thr/T)Threonine AAA (Lys/K)Lysine

AGA (Arg/R)Arginine

AUG (Met/M)Methionine

ACG (Thr/T)Threonine AAG (Lys/K)Lysine

AGG (Arg/R)Arginine

GUU (Val/V)Valine

GCU (Ala/A)Alanine

GAU (Asp/D)Aspartic acid GGU (Gly/G)Glycine

GUC (Val/V)Valine

GCC (Ala/A)Alanine

GAC (Asp/D)Aspartic acid GGC (Gly/G)Glycine

GCA (Ala/A)Alanine

GAA (Glu/E)Glutamic acid GGA (Gly/G)Glycine

GCG (Ala/A)Alanine

GAG (Glu/E)Glutamic acid GGG (Gly/G)Glycine

C
1st
base

G GUA (Val/V)Valine

GUG (Val/V)Valine

41

IBO 2007. Theoretical Exam 1

Question 57. In a paternity suit the ABO phenotypes of the mother, the child and the two possible
fathers (F1 and F2) were determined, and a DNA profile was made for each person.
Both the mother (M) and the child (C) are type A, Rh-negative. Father F1 is type B, Rh-negative
and Father F2 is type O, Rh-negative. The DNA profiles are shown below.

Answer the following questions. (3 marks)


Question
a. The mother has the genotype Rr for the Rh
factor
b. The child has the genotype IAIo
c.

F1 cannot be the father

Answer: True (1) or False (2)


2

42

IBO 2007. Theoretical Exam 1

Question 58. DNA repair" mechanisms can be divided into 3 categories (listed below). A list of
repairing processes is also given.
Match each DNA repair mechanisms with the names of the repairing processes. (4.5 marks)

Name of repairing process


A. Mismatch repair
B. Recombinational (daughter-strand gap) repair
C. Nucleotide excision repair
D. Photoreactivation
E. Mutagenic repair (trans-lesion synthesis)

DNA repair mechanisms


1. Damage reversal
2. Damage removal
3. Damage tolerance

F. Ligation of single strand breaks


G. SOS repair
H. Base excition repair
I. Postreplicative Translesion Bypass Repair

Answer table
DNA repair mechanism
1. Damage reversal
2. Damage removal
3. Damage tolerance

Repairing process (list letter of all that apply)

43

IBO 2007. Theoretical Exam 1

Question 59. The pBR322 plasmid was cut with two different restriction enzymes. The patterns of
ethidium bromide staining of plasmid DNA after electrophoresis on agarose gels are shown.
P

+ HindIII

+BsuI

P
+BsuI + HindIII

a
b

c
d

f
g
h

Reference:
P: plasmid

Answer true or false: (2.5 marks)


1

( ..T ) The pBR322 has only one restriction site for HindIII.

(F) The restriction enzyme HindIII induces plasmid supercoiling.

(....T....) The pBR322 has two restriction sites for BsuI.

(....T....) The migration rate of a DNA molecule in an agarose gel is inversely proportional to
its size.
(....F.....) The bands in lane 4 indicate that both enzymes have the same restriction site.

44

IBO 2007. Theoretical Exam 1

Question 60. Information on the description and appearance of various chromosomal structural
arrangements is given below.

Type of chromosomal change

Definition of chromosomal change

1. Tandom duplication

A. Internal fragment of chromosome is missed.

2. Reciprocal translocation

B. Chromosomal segment is doubled in the opposite order.

3. Interstitial deletion

C. Two-way exchange of a segment of chromosome.

4. Pericentric inversion

D. Centromere containing part of chromosome is inverted.

5. Displaced duplication

E. The tip of chromosome is lost because of single break.

6. Interstitial translocation

F. Acentromeric part of chromosome is inverted.

7. Terminal deletion

G. Chromosomal segment is represented twice same as the

8. Reverse duplication
9. Paracentric inversion

original order.
H. Movement of a segment of chromosome from one to another
in one way.
K. Chromosomal segment is represented twice but it is not
adjacent to original segment

45

IBO 2007. Theoretical Exam 1

In the following table and using the appropriate letters and Roman numerals, match the correct
definition and appearance to the type of chromosomal change listed. (4.5 marks)

Type of chromosomal change

Definition

1. Tandem duplication
2. Reciprocal translocation
3. Interstitial deletion
4. Pericentric inversion
5. Displaced duplication
6. Interstitial translocation
7. Terminal deletion
8. Reverse duplication
9. Paracentric inversion

- THE END -

Appearance

18th INTERNATIONAL BIOLOGY OLYMPIAD


JULY 15 - 22, 2007

THEORY EXAMINATION # 2
Total marks possible: 53.5

Time allowed: 120 minutes

WRITE YOUR 4-DIGIT STUDENT NUMBER IN THE BOX


BELOW

STUDENT CODE

IBO 2007. Theoretical Exam 2.

GENERAL INSTRUCTIONS
Check that you have the correct examination paper and an answer sheet.

BE SURE TO RECORD ALL YOUR ANSWERS ON THE ANSWER SHEET


WHEN YOU HAVE FINISHED THE EXAM, PLACE YOUR ANSWER
SHEET INSIDE YOUR QUESTION PAPER AND HAND BOTH TO THE
INVIGILATOR BEFORE LEAVING THE EXAM ROOM.
REMEMBER TO WRITE YOUR 4-DIGIT STUDENT CODE ON THE
FRONT PAGE OF THE QUESTION PAPER.
Read each question carefully before attempting it.

IMPORTANT
! Use the answer sheet provided to record your answers.

! Ensure that your name and student code is PRINTED in the top margin of the front
page of the answer sheet. The markers will enter this information in the correct places
on the reverse side of the answer sheet.
! Use only the HB pencil provided to mark the answer sheet. Completely fill in the circle.

This is the correct way:

A
o

C
o

D
o

E
o

! DO NOT USE AN X OR ANY OTHER SYMBOL TO MARK YOUR


ANSWER.
! If you want to change your answer, use the eraser to completely erase your incorrect
response and fill in the new circle you require.
Question 1. The diagram below shows a section through a mammalian ovary.
! There is ONLY ONE CORRECT ANSWER to each question.
! Each question (except Question 31) is worth one mark.
! Marks will NOT be deducted for incorrect answers.

IBO 2007. Theoretical Exam 2.

Question 1. The diagram below shows a section through a mammalian ovary.

Which one of the following is the correct sequence of the development of the structures indicated
by the letters A to E?
A.
B.
C.
D.
E.

A, C, D, B, E
A, B, D, C, E
C, B, D, A, E
D, B, C, A, E
E, B, D, C, A

Question 2. Oogenesis differs substantially from spermatogenesis. Which of the following


statements concerning oogenesis is INCORRECT?
A.
B.
C.
D.
E.

Cytokinesis is unequal during the meiotic divisions


The sequence from secondary oocyte to ovum is interrupted by a relatively long
rest period
The first meiotic division is not completed unless the egg is reactivated by a hormone
A mature ovum has not completed its second meiotic division
The number of potential gametes is, by and large, established at birth

IBO 2007. Theoretical Exam 2.

Questions 3 7. Examine the figure below:


Hormone Levels during the Human Female Reproductive Cycle

A
B

C
D

Question 3. Which of the following correctly lists the hormones in order from A to D?
A.
B.
C.
D.
E.

estrogen, progesterone, LH, FSH


estrogen, FSH, progesterone, LH
LH, FSH, progesterone, estrogen
LH, estrogen, FSH, progesterone
LH, FSH, estrogen, progesterone

IBO 2007. Theoretical Exam 2.

Question 4. Which of the following statements is INCORRECT?


A.
B.
C.
D.

An increase in hormone B causes a decrease in hormones C and D


A steep rise in hormone C stimulates the production of hormones A and B.
A low level of hormone C inhibits the production of hormones A and B.
A high level of hormones C and D inhibits the secretion of hormones A and B

Question 5. Ovulation is triggered by a peak in the hormone whose level is shown by the
A.
B.
C.
D.

Line A
Line B
Line C
Line D

Question 6. Hormones A and B are secreted by the

A.
B.
C.
D.

uterine wall
ovary
hypothalamus
anterior pituitary

Question 7. Hormones C and D are secreted by the

A.
B.
C.
D.

uterine wall
ovary
hypothalamus
anterior pituitary

Question 8. One hypothesis predicts that most of the CO2 produced in the soil originates from
microorganisms feeding on dead plant material. To which trophic level do these microorganisms
belong?
A.
B.
C.
D.
E.

Primary producers
Secondary producers
Decomposers
First order consumers
Second order consumers

IBO 2007. Theoretical Exam 2.

Question 9. Joan and Claude (neither have cystic fibrosis) come to you seeking genetic counseling.
Claude was married before, and he and his first wife had a child with cystic fibrosis, an autosomal
recessive condition. A brother of Joans died of cystic fibrosis and Joan has never been tested for
the gene. If they marry, what is the probability that Joan and Claude will have a son that WILL
NOT be a carrier for, nor have cystic fibrosis?
A.
B.
C.
D.
E.

1/12
1/8
1/6
1/4
1/2

Question 10.
A.
B.
C.
D.
E.

Chromosomal crossing over occurs in which of the following stages of cell division?

Prophase of mitosis.
Metaphase of mitosis.
Prophase I of meiosis.
Metaphase II of meiosis.
Telophase I of meiosis.

Question 11. A man whose blood group is Type A has two boys. The plasma of one of the boys
agglutinates the red cells of his father, but the plasma from the other son does not. Which statement
is incorrect?
A.
B.
C.
D.
E.

The father must be heterozygous for the A blood type allele.


The mother of the son that agglutinates his fathers blood can be type AB.
The boy that agglutinates could have type O blood.
The mother of the son that agglutinates must possess a type O allele.
The boy that doesnt cause agglutination can be type AB.

Question 12. In peas, the allele for smooth seed coat (S) is dominant to wrinkled (s), Tall plant (T)
is dominant to short (t) and yellow coloured seed (Y) is dominant to green (y).
A plant with the genotype SsTtyy was test crossed and 145 progeny survived to maturity.
Approximately how many of these progeny are expected to be tall plants with green wrinkled seeds?
A.
B.
C.
D.

9
18
36
72

IBO 2007. Theoretical Exam 2.

Question 13. B chromosomes are additional chromosomes possessed by some, but not all,
individuals in a population. Which combination of statements is correct?
I.
II.
III.
IV.
V.

They occur only in plants.


While they are common in plants, they occur also in fungi, insects and animals.
They arise from normal chromosomes by fragmentation.
They are normal, but short, chromosomes..
In plants they are associated with reduced viability.
A.
B.
C.
D.

I, III and V
I, IV and V
II, III and V
II, IV and V

Question 14. Often the frequency of a particular deleterious allele is very different in
neighbouring populations. For example, the frequency of the allele causing cystic fibrosis is 0.02 in
Population A and 0.006 in Population B. Such a difference in allele frequencies between two close
populations is probably the result of
A.
B.
C.
D.
E.

The occurrence of the founder effect in an earlier generation


More effective repair of DNA damage caused by mutation
Selective advantage of the allele in one population but not the other
Recurring migration between the populations
Non-random mating.

IBO 2007. Theoretical Exam 2.

Question 15. The coefficient of relatedness is a theoretical value determined by the number of
alleles that would be the same between two organisms.
The diagram below shows the relationships between some honeybees. What is the coefficient of
relatedness between II-2 and II-3; and between II-2 and III-1?

II

III

ANSWER

Between II-2 and


II-3

Between II-2 and


III-1

A.

0.50

0.50

B.

0.75

0.50

C.

0.75

1.00

D.

0.25

1.00

E.

0.50

0.75

IBO 2007. Theoretical Exam 2.

Question 16. Collenchyma and sclerenchyma are plant support tissues. Which combination of the
following statements correctly differentiates these two types of plant tissue?
I.

Collenchyma occurs only in Dicotyledons; sclerenchyma is an elastic tissue that is found in


both Monocotyledons and Dicotyledons.
II. Collenchyma cells are developed during growth; sclerenchyma cells generally occur in organs
that have concluded their longitudinal growth.
III. Collenchyma and sclerenchyma may arise from the same cell type.
IV. Collenchyma cells have primary walls only while sclerenchyma cells have secondary walls.
V. Collenchyma originates from the protoderm; sclerenchyma is formed by the procambium.
A.
B.
C.
D.

I, II, III.
II, III, IV
II, IV, V
IV, V

Question 17. The following figure shows the carbon fixation reactions during
photosynthesis in a typical C4 plant :
Vascular bundle

CELL 1

CELL 2
epidermis

Which of the following answers indicate the dominant carbon fixation enzyme in each of the
two cells?
Answer
Cell 1
Cell 2
A.

Malate dehydrogenase

Sucrose synthase

B.

PEP Carboxylase

Rubisco

C.

Rubisco

PEP Carboxylase

D.

Aspartate aminotransferase

Malate dehydrogenase

E.

Malic enzyme

Pyruvate
dehydrogenase

10

IBO 2007. Theoretical Exam 2.

Question 18. Suppose that an illuminated suspension of Chlorella (a photosynthetic alga) was
actively carrying out photosynthesis when the light was suddenly switched off. How would the
levels of 3-phosphoglycerate and ribulose 1,5-bisphosphate change during the next minute?
A.
B.
C.
D.
E.

The concentration of 3-phosphoglycerate would increase and that of ribulose 1,5bisphosphate would increase.
The concentration of 3-phosphoglycerate would increase; the concentration of
ribulose1,5-bisphosphate would decrease.
The concentration of 3-phosphoglycerate would decrease; the concentration of ribulose
1,5-bisphosphate would increase.
The concentration of 3-phosphoglycerate would decrease; the concentration of ribulose
1,5-bisphosphate would decrease.
The concentration of 3-phosphoglycerate would remain the same; the concentration of
ribulose 1,5-bisphosphate would decrease.

Question 19. Which of the following statements shows the difference between the reaction sites of
photosystem I and II?
A.
B.
C
D.
E.

Chlorophyll a is only found in photosystem I; chlorophyll b is found in photosystem II.


Each preferentially absorbs slightly different wavelengths of light.
One is located in the thylakoid membrane; the other occurs in the stroma.
Only photosystem I is found in the thylakoid membranes.
None of these statements are correct.

Question 20. You are a biotechnologist designing novel eukaryotic enzymes that are regulated by
phosphorylation. Which amino acid residues shown below would you most likely use at the
regulatory site?

I.

IV.

A.
B.
C.
D.
E.

III.

II.

I, III
I, IV
I, II, III
II, IV, V
III, IV, V

V.

11

IBO 2007. Theoretical Exam 2.

Question 21. A biologist has discovered two new species of micro-organisms. Micro-organism A
was isolated from a hot spring whereas micro-organism B was obtained from a tropical forest. DNA
was isolated from both organisms and an analysis of the melting profile of each DNA sample was
carried out. The melting temperature (Tm) was 80 C for DNA from micro-organism A, and 70 C
for DNA from micro-organism B.
Which statement best describes the reason for this difference in values?
A.
B.
C.
D.
E.

DNA of micro-organism A has higher A+T content


DNA of micro-organism A has higher G+A content
DNA of micro-organism A has higher G+C content
DNA of micro-organism A has higher T+G content
DNA of micro-organism A has a higher proportion of TGA codons

Question 22. Which of the following cofactors is not redox active?


A.
B.
C.
D.
E.

Coenzyme A
Flavin coenzyme
NADH
Vitamin D
Coenzyme A and Vitamin D
A.
B.
C.
D.
E.

I
II
III
IV
I, III

Questions 23-24. Two cells have the following characteristics:


Characteristic

Cell I

Cell II

Cell wall

Present

Present

Ribosomes

Present

Present

Nucleus

Absent

Present

Ability to photosynthesize

Present

Absent

Cell respiration

Present

Present

IBO 2007. Theoretical Exam 2.

Question 23. From the characteristics presented in the table, which statement is correct?
A.
B.
C.
D.
E.

Cell I is more complex in its organization than cell II


Cell I is a prokaryote
Cells with all characteristics of cell II appeared earlier in the fossil record than cells
with all characteristics of cell I.
Cell II does not have a cell membrane
Both groups of cells are from fungi

Question 24. Cell II is a


A.
B.
C.
D.
E.

plant cell
eubacterium
archaebacterium
animal cell
cyanobacterium

Question 25. Suppose Species B disappears from an ecosystem in which the interrelationship
among the component species can be described by the food web below.

Which of the following will be a consequence of its elimination?


A.
B.
C.
D.
E.

Species X loses its only prey.


Species A loses its only prey.
Species D benefits because it is most distant from Species B.
Species C benefits because the competition between species B and species C is
reduced.
The disappearance of species B has no effect on species C or species D.

12

13

IBO 2007. Theoretical Exam 2.

Question 26. You have been asked by an international organization to initiate a biodiversity
conservation project on a tropical island off the south coast of Java, and far from any large
continental land. For this purpose, you must identify from three islands the island that has the
highest number of species.
The following information is provided:
Island Name
Boa
Ibo
Bio

Size/Area
418 km2
500 km2
420 km2

Distance from Java


220 km
800 km
450 km

Which of the following statements describes your decision?


A. Bio Island
B. Boa Island
C. Ibo Island
D. Either Boa Island or Bio Island
E. Insufficient information is provided to allow you to make a decision.
Question 27. Marine bony fishes have much lower internal osmotic concentration than the
seawater around them. Which of the following statements DOES NOT EXPLAIN the osmotic
regulation of marine bony fishes:
A.
B.
C.
D.
E.

They lose water by osmosis and gain salt by diffusion


They drink seawater
They actively absorb sodium chloride across gills
They absorb sodium chloride from stomach
They absorb water from stomach

Question 28. The best description of the relationships between fundamental niches (FN) and
realized niches (RN) of two competing species A and B that coexist is:
A.
B.
C.
D.
E.

FNA = RNA; FNB = RNB


FNA > RNA; FNB = RNB
FNA < RNA; FNB < RNB
FNA > RNA; FNB > RNB
FNA = RNA; FNB > RNB

IBO 2007. Theoretical Exam 2.

14

Question 29. Use the information given in Figures 1 and 2 to answer this question.

Figure 1. An example of a phylogeny showing characters by which taxa are recognised. Characters
1 4 are synapomorphies, 5 12 are autapomorphies and 13 is an attribute seen in the salmon and
the shark.

Figure 2. Two possible ways to organize the data from Figure 1.


Which of the following statements best describes the information presented in Figures 1 and 2?
A.
B.
C.
D.
E.

The tree shown in Figure 1 is the most parsimonious tree possible using these
characters.
The Lamprey and the Lizard are the oldest because they have the longest line.
The four groups shown in Figure 1 are equally related because they are all at the same
horizontal level.
X, Y and Z are characteristics common to all groups.
The Lamprey is more closely related to the Shark than to the Salmon or the Lizard.

IBO 2007. Theoretical Exam 2.

15

Question 30. A woman visits her doctor after noticing several changes in her body over a period
of 6 (six) months. She has noticed weight loss, intolerance to temperature variations, irregular
menstrual cycles, insomnia, and general weakness. Based on these symptoms, you would expect
the doctor to test her for:
A.
B.
C.
D.

Diabetes mellitus
Hyperthyroidism
Hypothyroidism
Hypoglycemia

Question 31. Endocrine glands


A.
B.
C.
D.

Produce hormones that are only secreted into the digestive tract
Release most hormones into the bloodstream
Release hormones that generally act as rapidly as nerve impulses
Are present only in vertebrates

Question 32. Long corolla length in tobacco is inherited as a recessive monogenic characteristic.
If in a natural population 49% of the plants have a long corolla, what is the probability that the
result of test crossing a randomly selected plant with a short corolla from this population in F1 will
have uniform progeny?
!.
B.
C.
D.
E.

100%
50%
30%
18%
0%

Question 33. From an evolutionary viewpoint, which of the five following individuals is the most fit?
!.
B.
C.
D.
E.

A child who does not become infected with any of the usual childhood diseases, such as
measles or chicken pox.
A woman of 40 with seven adult offspring
A woman of 80 who has one adult offspring
A 100-year old man with no offspring
A childless man who can run a mile in less than five minutes

IBO 2007. Theoretical Exam 2.

16

Question 34. A study of a grass population growing in an area of irregular rainfall found that
plants with alleles for curled leaves reproduced better in dry years, whereas plants with alleles for
flat leaves reproduced better in wet years. Curled and flat leaves are controlled by different alleles
at the same gene locus.
This situation tends to
A.
B.
C.
D.
E.

cause genetic drift in the grass population


cause gene flow in the grass population
lead to directional selection in the grass population
preserve variability in the grass population
lead to uniformity in the grass population

Question 35. The cohesion-tension (C-T) theory of sap ascent states that in plants sap is
transported against gravity by bulk flow through the xylem vessels or chains of tracheids. Which of
the following statements correctly describes the main factors affecting this bulk flow?
A.
B.
C.
D.

Hydrogen bonds within the water, hydrogen bonding to the hydrophilic walls of the
xylem cells, and the gradient of solute potential (! s).
The gradient of the pressure potential (! p), and solute concentration
The gradient of water potential (!), hydrogen bonds within the water and solute
concentration.
Hydrogen bonds within the water, hydrogen bonding to the hydrophilic walls of
the xylem cells, and the gradient of pressure potential (! p).

Questions 36-38. Plants maintain most Indole Acetic Acid (IAA) (an auxin) in conjugated
forms, which complicates IAA quantification.
Question 36. These forms can exist in forms such as IAA-amino acid conjugates
A.
B.
C.
D.

AA-amino acid conjugates


IAA-hydrolase conjugates
IAA-glycerol conjugates
IAA-peroxide conjugates

Question 37. These forms can exist in forms such as IAA-amino acid conjugates. Before analysis
in order to measure total IAA in a particular tissue, these conjugates must be
A.
B.
C.
D.

dehydrated
dehydrogenized
hydrolyzed
synthesized

IBO 2007. Theoretical Exam 2.

17

Question 38. Therefore free IAA, thought to be the active form of the hormone, is measured
A.
B.
C.
D.

in the same sample without enzymes


in the same sample with enzymes
in a parallel sample without enzymes
in a parallel sample with enzymes

Question 39. You find a mutant bacterium that synthesizes lactose-digesting enzymes whether or
not lactose is present. Which of the following statements or combination of statements might
explain this?
I. The operator has mutated such that it is no longer recognised by the repressor.
II. The gene that codes for the repressor has mutated and the repressor is no longer effective.
III. The gene or genes that code for the lactose-digesting enzymes have mutated.
A.
B.
C.
D.
E.

Only I
Only II
Only I, II
Only I, III
I, II, III

Question 40.
A.
B
C.
D.
E.

What mechanism is responsible for the acidification of the lysosome?

A lysosome fuses with acidic vesicle derived from Golgi aparatus


A pump transports protons from the cytosol into the lysosome
A pump transports protons from the lysosomal lumen to the cytosol
A lysosome fuses with acidic endocytosed material
A pump transports OH- ions from the cytosol to the lysosomal lumen

Question 41. Which of the following is an example of active transport?


A.
B.
C.
D.
E.

K+ through a voltage-gated K+ channel


Ca2+ through a voltage-gated ion channel
Na+ through ligand-gated ion channel
3 Na+ in exchange for 2 K+ across the plasmalemma
All of the above

IBO 2007. Theoretical Exam 2.

Question 42. The transport of glucose into the mammalian red blood cell is accomplished by
A.
B.
C.
D.
E.

simple diffusion through the phospholipid bilayer


a Na+ - K+ ATPase
esterifying the glucose to phosphatidate
first converting glucose into lactose
facilitated diffusion through a glucose transporter

Question 43. Isopods are one of the few crustacean groups that have successfully invaded
terrestrial habitats. Which of these statements is INCORRECT?
A.
B.
C.
D.

They live in dry conditions.


They must live in moist conditions.
Their abdominal appendages bear gills.
They do not have an efficient cuticular covering to conserve water.

Question 44. Hormones are essential to maintaining homeostasis mainly because


A.
B
C.
D

they catalyze specific chemical reactions in brain cells.


the body requires them for digesting food.
they cause specific responses in specific targets.
they act faster than nerve impulses.

Question 45.
A.
B.
C.
D.

Compared to nerve impulses, hormones are generally

released more slowly and have longer lasting effects.


released faster and have longer lasting effects.
release more slowly and have effects of a shorter duration.
released faster and have effects of shorter duration.

Question 46. Someone who has suffered damage to the pancreas might
A.
B.
C.
D.

have difficulty maintaining normal cortisol levels


have abnormal blood calcium levels.
have periods of very low energy.
experience fluctuating blood pressure

Question 47.
A.
B.
C.
D.

The hypothalamus

sends nerve impulses and also makes hormones.


directly stimulates the adrenal gland to produce glucocorticoids
belongs to both the nervous and circulatory systems.
regulates circadian rhythms in vertebrates.

18

IBO 2007. Theoretical Exam 2.

19

Questions 48 - 49. Ten grams of plant material were homogenized in 50 ml buffer and the
homogenate was centrifuged. Protein from 10 ml of the supernatant was precipitated by
addition of ammonium sulphate and the protein precipitated was collected by centrifugation
and re-suspended in 1 ml of buffer. The re-suspended protein was diluted 10 times for protein
determination.
Question 48. The amount of protein in 1 ml of the diluted sample was 0.4 mg. What is the total
amount of protein recovered from 10 ml of the supernatant?
A.
B.
C.
D.

2 mg
4 mg
6 mg
8 mg

Question 49. The amount of protein in 1 ml of the diluted sample was 0.4 mg. What is the amount
of protein extracted from 100 g tissue?
A.
B.
C.
D.

0.2 g
0.4 g
0.6 g
0.8 g

Question 50. Prion diseases are characterized by:


A.
B.
C.
D.
E.

cellular DNA damage.


misfolded proteins that are much more soluble than the regular form of the protein.
misfolded proteins that are prone to aggregation and are very stable.
abnormal enzyme activity.
protein chaperones in cells.

Question 51. Why are some proteinases synthesized as inactive precursors known as zymogens
(proenzymes)?
A.
B.
C.
D.
E.

Because they dont degrade a cells starch supply.


Zymogens have a higher degree of substrate specificity than most enzymes.
Zymogen synthesis ensures that proteinase activity is kept to a minimum inside
the cell where they are synthesised.
Zymogens are better at interconverting energy than regular enzymes.
Zymogens are more resistant to protein denaturation than the regular proteinase.

IBO 2007. Theoretical Exam 2.

20

Question 52. Negative feedback is a process that


Which of the following statements is INCORRECT? Negative feedback:
A. always reduces the amount of a hormone present in the blood.
A. is the main mechanism maintaining endometrial blood supply during pregnancy
B. keeps conditions near their normal state.
B. is responsible for varying urine osmolarities over the period of a day
C. lowers the body temperature below normal.
C. results in small fluctuations in physiological parameters
D. none of the above are correct.
D. is used in regulation of sympathetic and parasympathetic functions
Question 53. What is the role of the second messenger in hormone action?
A.
B.
C.
D.

it signals a cell to secrete a hormone.


it informs a gland as to whether its hormones are having an effect.
it relays a hormones message inside a target cell.
it carries a hormone while it is in the blood.

Question 54. It takes much longer for sex hormones and other steroids to produce their effect that
it takes nonsteroid hormones. Why?
A.
B.
C.
D.

Steroids are bigger, slower moving molecules.


Steroids usually must be carried longer distances by the blood.
Steroids generally cause target cells to make new proteins, which take time.
Steroids relay their message via a second messenger.

Questions 55 - 57. To test the origin of CO2 available in the soil, two experiments were
conducted on trees in a Pinus forest.
Question 55. In the first experiment, a 20 cm-wide strip of bark around the stem was removed
from trees mid-way between the ground and the lowest branch.
Which of the following statements correctly describes the effect of this treatment on the trees?
A.
B.
C.
D.
E.

Transpiration will cease. As a result, the tree will lose all its needles.
Transport of auxin in the xylem is prevented. This will cause increased auxin
concentration in the roots and root growth will be stimulated.
Phloem transport is prevented causing the roots to become deficient in nitrogen.
Transport of sugars to the roots ceases and the roots will die.
Transport of potassium and calcium from the roots to the needles will cease.

IBO 2007. Theoretical Exam 2.

21

Question 56. In the second experiment, the amount of CO2 released from the soil at the base of
trees was measured on several days during the growing period. The bark-removal experiment was
repeated on a total of 9 trees, three trees per teament. In the first treatment, the bark was removed
in early June (white triangles); in the second treatment, the bark was removed in late August (white
squares); the third treament was the control treatment where the bark was not removed (black
circles).
The data from this experiment are shown in the following graph. The black arrows indicate the
time of bark removal.

Which combination of the following statements best describe the results of this experiment?
I.

The variability of the different treatments overlap and any effect of bark removal is due to
chance.
II. The production of CO2 in the soil shows seasonal variation.
III. Bark removal in June had a much smaller effect on the total CO2 production in the soil during
the whole season than bark removal in August.
IV. The decrease in the CO2 production in the soil in the treatments where bark was removed
cannot be explained by seasonal variations alone.
V. The production of CO2 in the soil is always smaller for trees with bark removed than for
undamaged trees.
A.
B.
C.
D.
E.

Only I, II and V
Only I , II and IV
Only II, IV and V
Only II, III and IV
Only I, III and V

IBO 2007. Theoretical Exam 2.

22

Question 57. Which of the following statements is a valid conclusion for the results of the second
experiment?
A.
B.
C.
D.
E.

Most CO2 produced in the soil is due to the decomposition of dead roots.
Most CO2 produced in the soil is due to cellular respiration of root cells.
The amount of CO2 produced in the soil is not influenced by photosynthesis.
When most of the roots die, the production of CO2 in the soil is greatest.
The amount of CO2 produced in the soil depends on the soil temperature.

Questions 58 - 59. Below are six floral diagrams, labelled A to F.

Question 58. From the following, choose the combination in which the floral diagram is
associated with the correct family.
A.
B.
C.
D.
E.

A = Brassicaceae
B = Fabaceae
C = Liliaceae
D = Malvaceae
E = Rosaceae

IBO 2007. Theoretical Exam 2.

Question 59.

23

In which floral diagram do the all the floral characteristics listed below occur?

" Zygomorphic flower, fused sepals


" Axile placentation.
" Epipetalous stamen
A.
B.
C.
D.
E.

B
C
D
E
F

Question 60. Students made cross-sections of leaves collected from two different oak trees.
When they examined the sections under a microscope, they were surprised to see that the leaves
were different.
The following diagrams show cross-sections of leaves from Oak tree 1 and Oak tree 2.

Which of the following statements best explains the difference in leaf structure that the students
observed?
A.
B.
C.
D.
E.

Oak tree 1 grows in a swampy area; Oak tree 2 grows in sandy soil.
Oak tree 1 is a young tree; Oak tree 2 is a mature tree.
Oak tree 1 grows in fertile soil; Oak tree 2 grows in poor soil.
Oak tree 1 is exposed to sun for most of the day; Oak tree 2 grows in a shaded
area.
Oak tree 1 is infected by fungi, which induced the cell proliferation; Oak tree 2 was not
infected.

IBO 2007. Theoretical Exam 2.

24

Question 61. Cloning of a new DNA fragment into a circular plasmid/vector always requires:
A.
B.
C.
D.
E.

Complementary base pairing


DNA ligase activity
The presence of the same restriction site in the insert and the vector
Selectable markers and autonomous replicating sequences
All of the above

Question 62. Below is a diagram representing cholesterol in the phospholipid bilayer.

Cholesterol mixes with phospholipids in a cell membrane because cholesterol molecules are:
A.
B.
C.
D.
E.

amphipathic
steroid derivatives
entirely hydrophobic
phospholipids derivatives
bound with glycoproteins

Question 63. Which of the following molecules can diffuse through the mammalian phospholipid
bilayer without using a channel/transporter?
I.
II.
III.
IV.
V.

O2
glucose
steroid hormones
K+
amino acids
A.
B.
C.
D.
E.

I, III
I, IV
II, III, V
II, III, IV, V
All of the above.

IBO 2007. Theoretical Exam 2.

25

Question 64. What is the net charge of aspartic acid when the pH of the solution in which it is
prepared is the same as its pI value? Note the three pKa values of aspartic acid are as follows:
-COOH pKa = 2.1; -NH3+ pKa = 9.8; R group pKa = 3.9
A.
B.
C.
D.
E.

one net positive charge


two net positive charges
one net negative charge
two net negative charges
no net charge

Question 65. A quantitative amino acid analysis reveals that bovine serum albumin (BSA)
contains 0.58% tryptophan residues by weight. The molecular mass of the tryptophan molecules is
204 daltons. The molecular mass of bovine serum albumin is known to be approximately 66000
daltons. What number of tryptophan residues is present in each BSA molecule?
A.
B.
C.
D.
E.

2
3
4
5
6

Question 66. What essential function does gastrulation perform for the developing embryo?
A.
B.
C.
D.
E.

It results in the dorsal-ventral axis formation.


It gives rise directly to the germ layers of the embryo.
It gives rise to neural crest cells.
It gives rise directly to endocrine cells.
It gives rise directly to the trophectoderm.

Question 67. Which of the following is not true about the lymphatic system?
A.
B.
C.
D.
E.

It helps maintain the volume and protein concentration of the blood.


It helps defend the body against infection.
It transports fats from digestive tract to circulatory system.
Lymph composition is similar to that of interstitial fluid.
Lymph drains directly into the excretory system.

26

IBO 2007. Theoretical Exam 2.

Question 68. The most direct consequence on amphibian development upon removal of the grey
crescent would be:
A.
B.
C.
D.
E.

Inability to develop from the 2-cell stage to the 4-cell stage.


Inability to develop form the 4-cell stage to the 8-cell stage.
Inability to form a blastocoel.
Inability to form dorsal structures.
Inability to form ventral structures.

- THE END -

INTERNATIONAL BIOLOGY OLYMPIAD


THEORY PROBLEMS

2006, Rio Cuarto, Argentina















All IBO examination questions are published under the following Creative Commons license:



CC BY-NC-SA (Attribution-NonCommercial-ShareAlike) https://creativecommons.org/licenses/by-nc-sa/4.0/
The exam papers can be used freely for educational purposes as long as IBO is credited and
new creations are licensed under identical terms. No commercial use is allowed.

17th INTERNATIONAL BIOLOGY OLYMPIAD


JULY 9-16, 2006
Ro Cuarto Repblica Argentina

THEORETICAL TEST
PART A

Student code:

GENERAL INSTRUCTIONS
Please check that you have the appropriate examination papers and answer sheet,
as well as a calculator and a black pencil.
Use the answer sheet provided to record your answers.
Remember to write down your personal code number on top of the answer sheet.
All the questions are multiple choice and there is ONLY ONE correct answer.
You must mark your answers in the answer sheet by filling in the corresponding
box.
Use the pencil provided to complete the answer sheet.
You have 2 hrs 30 min (150 minutes) to answer the questions.
The maximum score is approximately 82 points (1 point each question).

PLEASE, REMEMBER: THE ACADEMIC COMMITTEE WILL CONTROL ONLY THE


ANSWER SHEET!

GOOD LUCK IBO COMPETITORS!

17 INTERNATIONAL BIOLOGY OLYMPIAD


THEORETICAL TEST
PART A

CELULAR BIOLOGY
1- Which of the following structures and processes can exist in both prokaryotic and
eukaryotic cells?

I. Nuclear envelope.
II. Ribosomes.
III. Introns.
IV. ATP synthesis.
V. Cell membrane.
VI. DNA polymerase.
VII. Cytoskeletal elements.
VIII. rRNA 18S.

A) I, II, III and VIII.


B) II, IV, VI and VIII.
C) I, III, V and VII.
D) II, IV, V and VI.
E) II, III, VI and VIII.

2- What structure in the bacterial cell has the most enzymatic activity?

A) Cell membrane.
B) Cell wall.
C) Mesosome.
D) Capsule.
E) Flagellum.

3- Which of the following functions are carried out in the rough endoplasmic reticulum?

I. Addition of carbohydrates to protein.


II. Synthesis of lisosomal protein.
III. Addition of carbohydrates to lipids.
IV. Synthesis of membrane proteins.
V. Formation of glycolipids.

3 Answers

A) I, II and V.
B) II, III and IV.
C) II, IV and V.
D) I, II and IV.
E) III, IV and

4- Which of the following fibers bind to the cytoplasmic site of cell-matrix adherens
junctions (focal adhesion)?

A) Tubulin protein fibers.


B) Collagen fibers.
C) Actin protein fibers.
D) Elastic fibers.
E) Reticular fibers.

5- Nuclear pores permit the passage of:

A) nucleotides inward and glucose outward.


B) RNA inward and chromosome outward.
5

C) proteins inward and RNA outward.


D) potassium ions inward and protein outward.
E) nucleotides inward and RNA outward.

6- Which is the function of the nucleolus?

A) Import of cytoplasmic proteins.


B) Regulation of nuclear pores.
C) Site of ribosome assembly.
D) Storage of inactive DNA.
E) Synthesis of nuclear protein.

7- The activity of lysosomes is regulated by:

A) cytoplasmic ions.
B) lysosomal pH.
C) temperature.
D) calcium.
E) nuclear DNA.

8- In which of the following processes the membrane integral proteins are NOT involved?

A) Glucose carrier.
B) Sodium ion channel.
C) Surface receptor.
6

D) Urea carrier.
E) Sodium-potassium pump.

9- How do polypeptides find their way from the site of synthesis on the cytoplasmic
ribosome to the place of their destination in the peroxisome?

A) By specific amino-terminal targeting signals.


B) By specific transport along the cytoskeleton.
C) By specific carboxy-terminal targeting signals.
D) By specific vesicular transport.

10- What is embedded in the thylakoid membrane of the chloroplast and protruding as
knobs from the outer surface?

A) Enzymes that fix carbon dioxide.


B) Molecules of chlorophyll a and chlorophyll b.
C) P700 or P680 molecules.
D) Proton channels that synthesize ATP.
E) Water splitting complex.

11- Where are most proteins of respiratory chain in that carry out the oxidative
metabolism in the mitochondrial structure located?

A) Dissolved within the fluid of the matrix.


7

B) In the cytoplasm on the outer surface of the mitochondria.


C) In the space between the two membranes.
D) On the surface and embedded within the outer membrane.
E) On the surface and embedded within the inner membrane.

12- Two sister chromatids fail to separate because microtubules became improperly
attached to the kinetochores. What is the most likely result?

A) Both chromatids will remain at the equator of the spindle.


B) The mitosis process will stop immediately.
C) One daughter cell will lose all its chromosomes.
D) One daughter cell will lose one chromosome.

13- Which of the following lipids contain glycerol in their structure?

A) Sphingolipids, Triacylglycerides, Cholestrol


B) Phosphatidylcholine, Wax, -Carotene
C) Triacylglycerides, Phosphatidylcholine,Phosphatidylethanolamine
D) Cholestrol, Phosphatidylcholine,Phosphatidylethanolamine
E) Carotenoides, Sphingolipids, Phosphatidylcholine.

14- Which of the following reactions are likely to occur in the cytoplasm of an eukariotic
cell?

I. Krebs cycle.
II. Oxidative catabolism of fatty acids
8

III. Glycolisis
IV. Lactic fermentation
V. Etanol fermentation
VI. Glyoxylate cycle

Answers

A) I, IV, and VI.


B) III, IV and V.
C) II, I and III.
D) II, IV and V.
E) IV, V and I.

15- Which of the following statements corresponds to apoptosis (Programmed Cellular


Death)?

I. Enzymes known as caspases are involved


II. Cellular swelling and plasmatic membrane lysis are observed.
III. The nucleus degrades randomly into fragments.
9

IV. The product of the tumor-suppressing gene (p53 protein) activates in response to DNA
damage.
V. It is a process that involves loss of energy.

Answers

A) I, IV and V.
B) II, III and IV.
C) I, II and V.
D) II, IV and V.
E) I, III and V.

16- Which of the following enzymes DOES NOT correspond to the Krebs cycle?

A) Isocitrate dehydrogenase
B) Succinate dehydrogenase
C) Citrate sintetase
D) Pyruvate carboilase
E) -cetoglutarate deshidrogenase

PLANT ANATOMY AND PHYSIOLOGY

17- Which are the key terms to explain water transport in the xylem?

A) Root hairs, cations concentration, transpiration.

10

B) Transpiration, tension, guttation.


D) Tension, cavitation, guttation.
E) Transpiration, cuticle, water potential.

18- Which of the following statements about the stomatal opening is true?

A) The concentration of abscisic acid in the guard cells increases.


B) Higher K+ concentrations give guard cells a negative water potential.
C) The Level of carbon dioxide in the spaces inside the leaf increases.
D) Lower K+ concentrations give guard cells a negative water potential.
E) Potassium ions diffuse passively out of the guard cells.

19- An active sieve tube member is characterized by having:

A) secondary wall, nuclear desintegration, sieve plates.


B) primary wall, central vacuole, a nucleus.
C) secondary wall, large amounts of callosa, sieve plates.
D) primary wall, bordered pits, a nucleus.
E) primary wall, sieve plates, desintegrated nucleous and tonoplast.

20- Which of the following statements is not related to plants pollinated by wind
(anemophily)?

A) They have large, divided or plumose stygma.


B) They produce a large quantity of pollen.
C) They have not showy flowers.
11

D) They have smooth and dry pollen.


E) They present flowers of diverse colors and agglutinated pollen.

21- The function of the aleurone layer in the carypsis is:

A) to protect the embryo.


B) to produce and release enzymes that degrade the starch and proteins of the
endosperm.
C) to produce gibberellins.
D) to synthesize carbohydrate.
E) to accumulate water.

22- The potato (Solanum tuberosum) tuber is characterized by:

A) being a modified underground stem.


B) accumulating large quantity of starch.
C) being a kind of asexual reproduction.
D) containing several buds.
E) all of them are true.

23- An aggregate fruit is originated from:

A) a set of flowers clustered in a receptacle.


B) a flower with several carpels and a syncarpous gynoecium.
C) a flower with syncarpous gynoecium and axile placentation.
D) a flower with several separated carpels.
12

E) several flowers around an axis.

24- In a laboratory, stems and roots of different plants were cut. When putting them into a
box the slides mixed. Which of the following cross sections corresponds to a primary root
of Magnoliopsida?

A)

Epidermis

Cortex

Bicollateral bundles Pith

B)

Epidermis

Cortex

Pericycle

4 xylem strands alternate


with 4 phloem strands

C)

Periderm

Secondary Phloem

Cambium

Secondary Xylem

D)

Epidermis

Cortex

Pericycle

20 xylem strands
alternate with phloem

E)

Epidermis

Sclerenchyma

Scattered vascular
bundles

13

Hollow pith

25- Which of the following combinations present primary walls in an adult plant?

A)

Vessels members

Meristematic cells

Parenchyma cells

B)

Colenchyma cells

Fibres

Sieve tube member

C)

Sclereids

Colenchyma cells

Sieve cells

D)

Meristematic cells

Tracheary elements

Colenchyima cells

E)

Sieve elements

Meristematic cells

Colenchyma cells

14

26- Identify the following characteristics in the floral diagrams below: Calix and corolla with
the same number of parts in each whorl, the number of stamens are twice the number of
sepals and petals, gynoecium with 5 carpels.

II
I

III

IV

A) I, II and IV.
B) II and III.
C) III and IV.
D) I, II and III.
E) I and IV.

15

27- The following figures correspond to cross sections of leaves. Which one/s
correspond/s to a hydrophytic habitat?

I
III
II

IV

A) I, II and III.
B) II.
C) I, III, IV and V.
D) I, II and V.
E) I, III and IV.

16

28- In a 5-m-tall young Eucalyptus, a person stuck 2 long nails horizontally and opposite to
each other in its trunk, at a height of 1.0 m. Today the tree is 10 m tall. Are there changes
in the height above the ground and the distance between the two nails?

A) The height of the two nails above the ground increases due to the activity of vascular
cambium.
B) The height of the two nails above the ground remains unchanged because primary
growth occurs in the stem tip.
C)The distance between the two nails will increase due to activity of vascular cambium.
D) The height and distance between the two nails increase due to activity of the intercalary
meristem.
E) Both B and C are true.

1m

17

ANIMAL ANATOMY AND PHYSIOLOGY

** Regulation of body temperature is fundamental in the organisms. The balance


between production and loss of heat determines the body temperature. In
vertebrates like reptiles, amphibians and fish the body temperature fluctuates within
a considerable rank. In birds and mammals there exists a group of reflex responses
that integrate to keep/maintain the body temperature within a narrow rank in spite of
the fluctuations of the atmosphere.
The following 4 questions are related to this introduction.

29- Body temperature in mammals is regulated by:

A) spinal cord.
B) medulla oblongata.
C) hypothalamus.
D) cerebellum.

30- A naked person inside a room at 21 C of temperature and 80% humidity will lose heat
mainly because of:

A) an increment of the metabolism.


B) miction.
C) breathing.
D) radiation and conduction.
E) sweat evaporation.

18

31- In a dehydrated person, corporal water must be replaced by means of intravenous


infusion of:

A) distilled water.
B) a 2 % sodium chloride solution.
C) a 5% glucose solution.
D) a mixture of 1% glucose and sodium chloride solutions.

32- Select the correct combination of the temperature regulating mechanisms activated by
cold:

A) perspiration - cutaneous vessel constriction - increased breathing.


B) cutaneous vessel constriction - piloerection - increased adrenalin secretion.
C) cutaneous vessel expansion - increased breathing shivering.
D) increased adrenalin secretion perspiration - piloerection.

19

In the 18th century the Italian clergyman Lazaro


Spalanzani

(1729-1799),

developed

very

ingenious experience with the frogs in his


swimming pool. He dressed some males with tight
trousers to prevent the spreading of the semen
into the water. That summer, Spallanzani did not
have any tadpoles in his swimming pool. He
collected the semen drops from the shorts and
noticed that when he added them to the ova
deposited by the females during amplexus, the
ova developed into tadpoles.
Spallanzanis frogs

The clergyman concluded that the ovum requires


contact with semen to be fertilized. Without
knowing,

he

was

the

precursor

of

artificial

fertilization. Spallanzani also observed that in the


fertilized eggs there appeared a furrow that had also
been observed by other researchers before. That
furrow is the beginning of the frog developmental
stages.

Eggs observed by Lzaro Spallanzani

20

Since then, developmental Biology has acquired significant advances, and at present the
embryonic development of the amphibia is well known.

33- Which of the following statements about amphibia development are correct?

I. Amphibia have mesolecithal eggs.


II. The blastomeres in the animal pole are smaller than in the vegetative / vegetal pole.
III. Yolk is concentrated in the vegetative / vegetal pole.
IV. Amphibians display holoblastic cleavage.

Answers

A) I, II, and IV.


B) I and III.
C) II and IV
D) I, II, III and IV.

21

The amniotic egg is one of the major adaptations in the evolution of the vertebrates. The
following scheme corresponds to an amniotic egg.

22

34 - The embryonic membrane marked with an X corresponds to:

A) chorion.
B) allantoids.
C) amnion.
D) yolk sac.

35- One of the extra embryonic membranes produces proteins and lymphocytes which
suppress the immune response the mothers body would present against the fetus. This
membrane is:

A) the allantois.
B) the chorion.
C) the yolk sac.
D) the amnion.

36- If the corpus luteum of a pregnant woman is removed before the eleventh week of
pregnancy:

A) the pregnancy proceeds because there is no connection between the corpus luteum
and pregnancy during this period of gestation.
B) the placenta has already secreted enough progesterone and estrogen to sustain the
pregnancy.
C) the embryo is spontaneously aborted.
D) none of the given answers is correct.

23

37- The three embryonic layers established during gastrulation in mammals subsequently
differentiate into specific tissues and organs.
Select the correct pair:

A) Mesoderm liver.
B) Endoderm - dental enamel.
C) Ectoderm crystalline lens.
D) Mesoderm - thyroid.

38- From which part of the paraxial mesoderm do the vertebrae develop?

A) Sclerotome.
B) Dermatome.
C) Myotome
D) Hypomere.

39- Select the correct sequence with relation to the embryonic origin of the nervous
system.

A) Neural tube neural plate - neural fold - neural crest.


B) Neural plate - neural crest - neural tube - neural fold.
C) Neural crest - neural plate - neural fold neural tube.
D) Neural plate neural fold - neural crest - neural tube.

24

The following 5 questions are related to the scheme:

40- Upon which organs in the diagram the cholecystokinin (CCK) acts?

A) 1; 2; 3 and 4.
B) 2 and 3.
C) 2 and 4.
D) 3 and 4.
E) 1; 3 and 4.

41- Which of following proteins are synthesized by the organ marked with 1 in the
scheme?

I. Albumin.
II. Fibrinogenous.
III. Transferrin.
IV. Angiotensyn.

25

Answers

A) I, II and IV.
B) II, III and IV.
C) I and III.
D) II and IV.
E) All of them are correct.

42- The structure marked with letter c corresponds to the:

A) Cystic duct.
B) Common hepatic duct.
C) Pancreatic duct.
D) Choledoco.

43- Which of following enzymes are secreted by organ 4 in the scheme?

I. Nucleases.
II. Lypase.
III. Pepsin.
IV. Trypsin.
V. Lactase.
VI. Chymotrypsin.

26

Answers

A) I, III and VI.


B) I, II, IV and VI.
C) II, IV and VI.
D) I, III, IV and VI.

44- Organ 4 of the scheme also has an endocrinous function. One of the main hormones it
releases is insulin. Which one of the following triplets is true regarding the effects of Insulin
on fat tissue (adipose tissue), muscle and liver?

Fat tissue
A)

Muscle

Increase in glucose entry. Decrease in glucose entry

Liver
Decrease in lipid
synthesis

B)

Increase in glucose entry

Increase in glycogen

Decrease in ketogenesis

synthesis
C)

D)

Increase in fat acids

Decrease in glycogen

synthesis

synthesis

Decrease in fat acids

Decrease in aminoacid

Decrease in protein

synthesis

acquisition

synthesis

27

Increase in ketogenesis

GENETICS

** The armadillo is a placental mammal of the order Xenarthra, family Dasypodidae,


mostly known for having a bony armour shell made up of small, overlapping scales.
All species are native to the American continent, where they inhabit a variety of
environments.

45- This remarkable animal is the only known species that always produces monozygotic
multiple offspring. After 140 days of gestation a female armadillo gives birth to 4 naked
babies with soft armour. We expect that:

A) the genotype of the four babies is the same as their mothers.


B) all of these armadillo babies have the same genotype.
C) the babies are of different sexes.
D) the four babies are haploid.
E) the four babies have different phenotypes.

46- In an electrophoretic study of enzyme variation in one species of Dasypus you find 31
individuals A1A1, 24 A1A2 and 5 A2A2, in a sample of 60. Which are the frequencies of
alleles A1 and A2?

A) p(A1)= 0.72; q (A2) = 0.28.


B) p(A1)= 0.52; q (A2) = 0.48.
C) p(A1)= 0.92; q (A2) = 0.028.
D) p(A1)= 0.28; q (A2) = 0.72.
E) p(A1)= 0.48; q (A2) = 0.52.
28

47- Is the above population in Hardy-Weinberg equilibrium?

Table: 2-Distribution

df 0.995

0.99

0.975

0.95

0.90

0.10

0.05

0.025

0.01

0.005

---

---

0.001

0.004

0.016

2.706

3.841

5.024

6.635

7.879

0.010

0.020

0.051

0.103

0.211

4.605

5.91

7.378

9.210

10.597

0.072

0.115

0.216

0.352

0.584

6.251

7.15

9.348

11.345

12.838

0.207

0.297

0.484

0.711

1.064

7.779

9.488

11.143

13.277

14.860

0.412

0.554

0.831

1.145

1.610

9.236

11.070

12.833

15.086

16.750

0.676

0.872

1.237

1.635

2.204

10.645

12.592

14.449

16.812

18.548

0.989

1.239

1.690

2.167

2.833

12.017

14.067

16.013

18.475

20.278

A) Yes.
B) No.
C) It is not possible to determine.

48- If in another population the frequency of the A 1A1 genotype is 0.25 and the frequency
of the A1A2 genotype is 0.45, in Hardy-Weinberg equilibrium, the frequency of matings
between the A2A2 and A2A2 genotypes would be:

A) 0.063.
B) 0.300.
C) 0.090.
D) 0.112.
E) 0.075.

29

49- Among the offspring of a given mating of armadillos, a phenotypic ratio of 9:3:3:1 is
discovered. This is a result of:

A) epistasis.
B) linkage.
C) independent assortment.
D) pleiotropy.
E) polyploidy.

50- Two male specimens of the rodent Akodon molinae belonging to the same population
were analyzed cytogenetically: One of them had 43 and the other 42 chromosomes. The
Fundamental Number (number of chromosome arms in a somatic cell) was 44 in both of
them. This may be due to:

A) chromosome loss.
B) an inversion.
C) Robertsonian translocation.
D) presence of B chromosomes.
E) polyploidy.

30

51- Which of the following statements is NOT true with respect to an X-linked recessive
inheritance?

A) It affects mainly males.


B) There is no male-to-male transmission in the pedigree.
C) Females may be affected if the father is affected and the mother is a carrier.
D) It affects either sex, but females are more affected than males.
E) Affected males are usually born to unaffected parents.

52- In chickens there exists a genetic condition called creeper (very short crooked legs).
A cross of affected chickens, produced a progeny of 775 creepers and 388 normal chicks.
The closest phenotypic ratio is:

A) 3 : 1.
B) 2 : 1.
C) 1 : 1.
D) 3 : 2.
E) 4 : 1.

53- What is the mode of inheritance of the creeper trait in the above question?

A) Sex-limited.
B) Autosomal recessive-lethal.
C) Epistatic to the normal allele.
D) Both a and b.
E) X-linked recessive.
31

54- You carry out a cross between homozygous purple-eyed flies with vestigial wings, and
wild-type flies. The resulting F1 flies are all phenotypically wild-type. In the progeny of the
testcross of F1 females you observe the following phenotypes:
Phenotype

Progeny

Purple-eyed, vestigial wings

1193

Purple-eyed, normal wings

159

Red-eyed, vestigial wings

161

Red-eyed, normal wings

1129

What is the map distance between these two loci?

A) 12.2 map units.


B) 48.2 map units.
C) 6.2 map units.
D) 24.4 map units.
E) none of the above.

55- In order to study a viral polymerase, a scientist decided to express the encoding gene
in Escherichia coli using the recombinant DNA technology. Choose the correct option
about steps mentioned below, in the right order.

a. cloning into an expression vector.


b. disruption of cells and isolation of cytoplasmic fraction.
c. induction of protein expression.
d. isolation of viral genomic RNA from purified virions.
32

e. PCR (polymerase chain reaction).


f. reverse transcription.
g. selection of the desired clone.
h. transformation into Escherichia coli cells.

Answer

A) d, f, e, a, h, g, c, b.
B) d, b, c, a, h, e, f, g.
C) h, g, a, b, d, f, c, e.
D) d, f, e, h, g, b, c, a.

56- Which of the three options (A, B, C or D) shows the characteristics of the DNA
molecule under physiological conditions?

Base pairs (bp) per

Diameter

turn

Distance between two

Form

successive base pairs

A)

12 bp

2 nm

34 nm

B)

10 bp

1 nm

3.4 nm

C)

10 bp

2 nm

3.4 nm

D)

11 bp

1 nm

3.4 nm

33

57- A DNA molecule has 160 base pairs and 20 percent of adenine nucleotides. How
many cytosine nucleotides are present in this molecule?

A) 96 cytosine nucleotides.
B) 60 cytosine nucleotides.
C) 160 cytosine nucleotides.
D) 40 cytosine nucleotides.
E) 48 cytosine nucleotides.

58- In vivo cloning of genomic DNA requires some of the following:

I. DNA polymerase.
II. restriction endonucleases.
III. a probe.
IV. DNA ligase.
V. a host.
VI. donor DNA.
VII. methylases.
VIII. proteases.
IX. a vector.
X. Taq DNA polymerase.

34

Answer

A) I, III, IV, V and VI.


B) II, IV, V, VI and IX.
C) II, V, VI, VII and IX.
D) IV, V, VI, IX and X.

35

ECOLOGY

59- The following graph represents the hypothetical results of an experiment designed to
recognize which nutrients can act as limitants of productivity in a salinized water lake.
According to the areas of limitation, determine the lines in the graphic which correspond to
each of these nutrients: phosphorus, nitrogen, iron and copper.

References
Solid black line: primary production.
a) Iron limitation area.
b) Nitrogen limitation area.
c) Phosphorus limitation area.

A) I = iron, II = nitrogen, III = phosphorus, and IV = copper.


B) I = copper, II = phosphorus, III = nitrogen, and IV = iron.
C) I = nitrogen, II = phosphorus, III = iron, and IV = copper.
D) I = copper, II = nitrogen, III = iron, and IV = phosphorus.
E) I = iron, II = copper, III = phosphorus, and IV = nitrogen.

36

** During a period of six years, the variation in abundance for five animal species
was registered in the month of October in order to verify the recovery of the fauna
of a contaminated lake after a decontamination process initiated in 1999. The
abundance values are shown in the following figure.

The following 4 questions are related to the figure.

60- Abundance of species 3 tends to:

A) diminish with time.


B) rise with time.
C) remain constant with time.
D) fluctuate at random with time.
E) fluctuate systematically.
37

61- The highest density of species 2 was registered in the years:

A) 2000 and 2001.


B) 2002 and 2003.
C) 2003 and 2004.
D) 2004 and 2005.
E) None of the above answers is correct because there is not complete information.

62- The species that allowed evaluating the reduction of contamination were:

A) 1; 2; and 3.
B) 2; 3; and 5.
C) 3; 4; and 5.
D) 1; 4; and 5.
E) 2; 4; and 5.

63- The reduction of contamination allowed the species richness:

A) to rise with time.


B) to diminish with time.
C) not to change with time.
D) to fluctuate at random with time.
E) to fluctuate systematically with time.

38

64- In the same way as a population, a community shows several properties. Which of the
following characteristics correspond to the community level?

A) Species diversity, stratification, relative abundance of females, and trophic webs.


B) Species diversity, age distribution, deaths of individuals, and trophic webs.
C) Cohorts diversity, dominance, age distribution, and trophic webs.
D) Species diversity, dominance, relative abundance, and trophic webs.
E) Species diversity, density, deaths of individuals, and age structure.

39

** In the following figure five examples of interactions are shown.

References

R: limiting resources.
C: consumers.
E: natural enemies.

: direct interaction and negative influence.


: indirect interaction and negative influence.
: direct interaction and positive influence.

40

The following two questions are related to the above figure.

65- Identify two situations of competition, and two of apparent competition:

A) I and II; III; and IV.


B) I and V; II; and IV.
C) III and II; IV; and I.
D) I and IV; III; and II.
E) I and III; II; and V.

66- Identify the organisms with mutualistic interaction between them, and the example
representing this situation:

Mutualistic interaction

Example

between
A)

C1 C2

B)

C1 C2

II

C)

C2 C3

II

D)

III

E)

C1 C2

41

67- Organic matter decomposition depends in part on climatic factors such as temperature
and precipitation. In the following schemes, the distribution of nitrogen in mulch, root, and
soil for six different biomes is shown. Which biome is represented in each of the following
figures: I, II, and III?

References
mulch
root
soil

A) I= Tundra, II= Temperate deciduous forest, III= Tropical deciduous forest.


B) I= Tropical deciduous forest, II= Tundra, III= Temperate deciduous forest.
C) I= Temperate deciduous forest, II= Tropical deciduous forest, III= Tundra.
D) I= Tundra, II= Temperate deciduous forest, III= Tropical evergreen forest.

42

68- Which of the following statements are correct?

Statements

I. The amount of nitrogen in living organisms is very small compared to the total quantity in
the atmosphere.
II. Less than 30% of the nitrogen available for plants comes from nitrogen-fixing bacteria or
algae.
III. The gaseous nitrogen cycle is global because it implies an exchange between the
ecosystem and the atmosphere.
IV. The input mechanisms of nutrients to an ecosystem are different from the output ones.
V. The nutrients cycles can be studied introducing radioactive markers in natural or
artificial ecosystems.

Answer

A) I; II; and IV.


B) II; III; and V.
C) I; III; and V.
D) III; IV; and V.
E) II; IV; and V.

43

69- In Biology, dispersion refers to:

A) the movements of organisms between populations.


B) the movements of organisms within a population.
C) the relative positions of the organisms to one another.
D) A and B are true.
E) B and C are true.

70 - Which of the following statements are true?

Statements

I. The life table of a population does not change with the environmental conditions.
II. To project the population growth when birth and mortality rates vary according to
individual age, we must know the proportion of individuals in each age-class.
III. The life table of a population varies according to the density of the population.
IV. The age-class structure of a population is always an indicative of the temporal
heterogeneity in individuals recruitment.

Answer

A) I, and III.
B) I, and IV.
C) II, and III.
D) II, and IV.
E) III, and IV.
44

BIOSYSTEMATICS

71- Development in marine molluss may be characterized by:

A) trochophore larva only.


B) trochophore and veliger larva.
C) direct development without larval state.
D) all of them are correct.

45

72- Observe the following diagrams of invertebrates embryos illustrating the characteristics
of the body plan (bauplan).

References

Endoderm
Cross

Mesoderm
I

II

Ectoderm
III

IV

Transversal

Longitudinal

Diploblastic

Triploblastic

No coeloms

pseudocoeloms

Incomplete or blind gut

Complete gut (Tube-within-a-tube)

Without segmentation

true coeloms

With segmentation (metameric)

46

Select the correct sequence which corresponds to the Phyla represented with I, II, III, IV
and V.

A)

Cnidaria

Plathyhelminta

Annelida

Nematoda

Arthropoda

B)

Cnidaria

Plathyhelminta

Nematoda

Arthropoda

Annelida

C) Nematoda Arthropoda

Plathyhelminta

Cnidaria

Annelida

D) Annelida

Arthropoda

Plathyhelminta

Nematoda

Cnidaria

73- The following characteristics correspond to: Unicellular organisms, eukaryotic cell with
micronuclei and macronuclei, asexual reproduction by transverse binary fision and sexual
reproduction by conjugation. Most are free-living.

A) Rhizopoda
B) Apicomplexa
C) Zoomastigophora
D) Ciliophora

74- The main characteristics of the Asteraceae family are:

A) calyx in the form of a pappus, sympetalous corolla.


B) inferior ovary.
C) seed separated from the pericarp.
D) free filaments, fused anthers.
E) all of them are true.

47

75- The Orchidaceae family present:

A) superior ovary, three-carpellate, one loculed ovary.


B) showy labellum. Pollen grains united in a mass.
C) plants with terrestrial habitat only.
D) androecium with two whorls of stamens of six stamens each.
E) fruit siliqua, The embryo is surrounded by storage tissue.

76 - The flower parts in the Solanaceae are:

A) calyx of 4 sepals, corolla of 4 petals, androecium of 6 stamens usually, parietal


placentation.
B) calyx of 5 sepals, corolla of 5 petals, androecium of 5 stamens usually, axile
placentation.
C) calyx of 5 sepals, corolla of 5 petals, androecium of 10 stamens usually, marginal
placentation.
D) calyx of 5 sepals, corolla of 5 petals, androecium of 4 stamens usually, axile
placentation.
E) perigon of 6 tepals, androecium of 6 stamens, axile placentation.

48

77- The Pinophyta are characterized by:

A) seedlings with two cotyledons.


B) anatropous ovule.
C) double fertilisation.
D) prothallus (primary endosperm) as storage tissue of the seed.
E) high predominance of herbaceous species.

78- The Liliopsida are characterized by:

A) a tap root system.


B) typical netlike veins
C) flower parts are usually in multiples of three.
D) ring-like arrangement of vascular bundles.
E) secondary growth.

49

ETHOLOGY

** The following 2 questions are related to the mulita (Dasypus hybridus) behavior.

To shelter and to look after its babies, the mulita (Dasypus hybridus) excavates
cylindrical burrows where it builds its nest with vegetal matter, specially dry
grasses (herbs).

79 - The mulitas avoid the overlapping of refuge and nestling areas by means of a fragrant
sign: on moving into the burrow, they impregnate its roof with an oily and stinking liquid
that is secreted by a gland placed in the back of the caparison at the pelvis level.
Therefore every adult mulita usually dwells into its own burrow -though sometimes the
burrows may be occupied by several individuals. This behavior corresponds to:

A) an altruistic behavior.
B) a selfish or malicious behavior.
C) a territorial behavior.
D) an agonistic behavior.
E) none of the previous ones.

50

80- The above mentioned behavior is adaptative and tends:

A) to reduce the intraspecific competition.


B) to ensure the most efficient use of the resources in a habitat.
C) to control the population growth.
D) to stimulate the dispersion of the individuals.
E) all the previous ones are correct.

** The armadillo commonly known as mataco or ball quirquincho (Tolypeutes


mataco) adopts, sometimes, a typical position similar to a "ball" as you can see in
the following figure; hence its name. In these cases, they draw in their feet and nose
causing the exoskeleton plates of the body and head to fit tightly to each other.

81- The armadillo exhibits this behavior when it faces a:

A) sure hiding place.


B) food source.
C) possible predator.
D) companion or a baby.
E) fragrant mark of a companion.

51

82- Mating of Recurvirostra avosetta, a wading bird, is preceded by some peculiar


movements. Both male and female clean their feathers nervously. After some time, the
female takes a horizontal position (see picture) and this triggers the male to copulate.
The horizontal position of the female corresponds to:

A) a conditioned reflex.
B) a displacement activity.
C) an innate response.
D) a sign stimulus.
E) the super normal releaser.

52















All IBO examination questions are published under the following Creative Commons license:



CC BY-NC-SA (Attribution-NonCommercial-ShareAlike) https://creativecommons.org/licenses/by-nc-sa/4.0/
The exam papers can be used freely for educational purposes as long as IBO is credited and
new creations are licensed under identical terms. No commercial use is allowed.

ANSWER SHEET
THEORETICAL TEST PART A
STUDENT NUMBER
CORRECT:
INCORRECT: X, O;
Cellular Biology
1
A
B
C
D
E
1
2
A
B
C
D
E
1
3
A
B
C
D
E
1
4
A
B
C
D
E
1
5
A
B
C
D
E
1
6
A
B
C
D
E
1
7
A
B
C
D
E
1
8
A
B
C
D
E
1
9
A
B
C
D
E
1
10
A
B
C
D
E
1
11
A
B
C
D
E
1
12
A
B
C
D
E
1
13
A
B
C
D
E
1
14
A
B
C
D
E
1
15
A
B
C
D
E
1
16
A
B
C
D
E
1
Plant Anatomy and Physiology
17
A
B
C
D
E
1
18
A
B
C
D
E
1
19
A
B
C
D
E
1
20
A
B
C
D
E
1
21
A
B
C
D
E
1
22
A
B
C
D
E
1
23
A
B
C
D
E
1
24
A
B
C
D
E
1
25
A
B
C
D
E
1
26
A
B
C
D
E
1
27
A
B
C
D
E
1
28
A
B
C
D
E
1
Animal Anatomy and Physiology
29
A
B
C
D
E
1
30
A
B
C
D
E
1
31
A
N
U
L
A
D
32
A
B
C
D
E
1
33
A
B
C
D
E
1
34
A
B
C
D
E
1
35
A
B
C
D
E
1
36
A
B
C
D
E
1
37
A
B
C
D
E
1
38
A
N
U
L
A
D
39
A
B
C
D
E
1
40
A
B
C
D
E
1
41
A
B
C
D
E
1
42
A
B
C
D
E
1
43
A
B
C
D
E
1
44
A
B
C
D
E
1

45
46
47
48
49
50
51
52
53
54
55
56
57
58

A
A
A
A
A
A
A
A
A
A
A
A
A
A

59
60
61
62
63
64
65
66
67
68
69
70

A
A
A
A
A
A
A
A
A
A
A
A

71
72
73
74
75
76
77
78

A
A
A
A
A
A
A
A

79
80
81
82

A
A
A
A

Genetics
C
D
C
D
C
D
C
D
C
D
C
D
C
D
C
D
C
D
C
D
C
D
C
D
C
D
C
D
Ecology
B
C
D
B
C
D
B
C
D
B
C
D
B
C
D
B
C
D
B
C
D
B
C
D
B
C
D
B
C
D
B
C
D
B
C
D
Biosystematics
B
C
D
B
C
D
B
C
D
B
C
D
B
C
D
B
C
D
B
C
D
B
C
D
Ethology
B
C
D
B
C
D
B
C
D
B
C
D
B
B
B
B
B
B
B
B
B
B
B
B
B
B

E
E
E
E
E
E
E
E
E
E
E
E
E
E

1
1
1
1
1
1
1
1
1
1
1
1
1
1

E
E
E
E
E
E
E
E
E
E
E
E

1
1
1
1
1
1
1
1
1
1
1
1

E
E
E
E
E
E
E
E

1
1
1
1
1
1
1
1

E
E
E
E

1
1
1
1

TOTAL:















All IBO examination questions are published under the following Creative Commons license:



CC BY-NC-SA (Attribution-NonCommercial-ShareAlike) https://creativecommons.org/licenses/by-nc-sa/4.0/
The exam papers can be used freely for educational purposes as long as IBO is credited and
new creations are licensed under identical terms. No commercial use is allowed.

17th INTERNATIONAL BIOLOGY OLYMPIAD


9 -16 JULY 2006
Ro Cuarto Repblica Argentina

THEORETICAL TEST
PART B

Student Code:

GENERAL INSTRUCTIONS

Please check that you have the appropriate examination paper and answer sheets,
a calculator, a black pencil, and two pens: a green one and a red one.
Use the answer sheet provided to record your answers.
Remember to write down your personal code number on top of the answer sheet.
There are different kinds of questions: in some of them you will have to choose
one answer, in this case, you must fill in the corresponding box; in other questions
you will have to complete diagrams or in blank spaces.
In the examination sheets you will find the instructions to complete the answer
sheet according to each question.
Use the pencils provided to complete the answer sheet.
You have 2 hrs 30 min (150 minutes) to answer the questions.
The maximum score is approximately 79 points. The points assigned to each
individual question depends on its complexity.

PLEASE, REMEMBER: DO NOT WRITE YOUR ANSWERS ON EXAMINATION SHEET


THE ACADEMIC COMMITTEE WILL ONLY COLLECT THE ANSWER SHEET!

GOOD LUCK IBO COMPETITORS!

17 INTERNATIONAL BIOLOGY OLYMPIAD


THEORETICAL TEST
PART B

CELLULAR BIOLOGY (13 questions, 15 points)

1- In a laboratory of Molecular Biology, the amino acid sequence of an armadillo intestinal


protein has been partially determined. The tRNA molecules used in the synthesis have the
following anticodons:
3 UAC 5

3 CGA 5

3 GGA 5

3 GCU 5

3 UUU 5

3 GGA 5

Mark the DNA nucleotide sequence of the complementary chain to the DNA chain that
encodes for the armadillo intestinal protein:

A) 5-ATG-GCT-GGT-CGA - AAA-CCT-3.
B) 5-ATG-GCT-CCT-CGA - AAA-CCT-3.
C) 5-ATG-GCT-GCT-CGA - AAA-GCT-3.
D) 5-ATG-GGT-CCT-CGA - AAA-CGT-3.

2- In the eukaryotic cell, the ribosomes that are associated with the cytosol, endoplasmic
reticulum, mitochondria and chloroplast carry out the synthesis of specific proteins.
Using the answer code, mark the location of the ribosomes that carry out the synthesis of
the detailed proteins:
Answer code:
01. Cytosol.
02. Rough Endoplasmic reticulum.
03. Mitochondria.
04. Chloroplast.

PROTEINS

CODE

A) Fibronectin.
B) Lactate dehydrogenase.
C) Complex of the cytochrome b6-f.
D) Amylase.
E) Ribulose biphosphate carboxylase.
F) Cytochrome C oxidase.
G) Keratin.
H) NADH deshidrogenase.

3- The nuclear protein DNA polymerase (synthesized by cytoplasmic ribosomes of the cell)
enters into the nucleus through the nuclear pores by:

A) passive diffusion through hydrophilic channels.


B) specific signal sequence of the protein using energy.
C) receptor -mediated endocytosis.
D) specific signal sequence of the protein without energy.

4- Using the answer code, specify the characteristics of RNA synthesis, mRNA processing
and protein synthesis corresponding.

Answer code:
01. prokaryote.
02. eukaryote.
03. both.

CHARACTERISTIC
A) A single RNA polymerase catalyzes the synthesis of the three types of
RNA.
B) The assembly of RNA polymerase at the promoter requires a set of proteins
called general transcription factors which must be assembled at the promoter
before transcription can begin.
C) The structural genes are not contained in operons.
D) In the mRNA processing, a methylguanine cap is added to the 5-end and a
poly-A tail to the 3-end.
E) Most of the structural genes contain introns that are spliced before the
translation.
F) The protein synthesis starts even before the transcription has been
completed.
G) The protein synthesis always starts on free ribosomes in the cytoplasm.
H) The degradation rate of the mRNA is regulated by extracellular signs.
I) The Shine-Dalgarno sequence in the 5-end of mRNA recognizes the
ribosome and the translation starts.

CODE

5- In order to investigate protein location either in the cytoplasm or in organelles such as


endoplasmic reticulum and Golgi bodies, analyses that combine proteinases and
surfactants are frequently carried out. As a result of the target protein contacting the
proteinase, the former is decomposed and can no longer be detected.
The proteinase does not act across the biomembrane. The biomembrane is destroyed by
the detergent.
Liver cells were homogenated to produce a small vesicles and cytoplasm components. In
order to investigate the respective locations of protein A (40 kDa), protein B (50 kDa) and
protein C (80 kDa), the following procedures were performed individually, followed by
Western analysis to detect and investigate the molecular weights of proteins A, B and C.
(Note: kDa indicates a unit to express the molecular weight of proteins).

Procedure 1: Proteinase K was added followed by warming.


Procedure 2: Detergent Triton X-100 was added followed by warming.
Procedure 3: Both proteinase K and detergent Triton X-100 were added followed
by warming.
Procedure 4: Only the vesicles were separated by precipitating with ultra-highspeed centrifugation followed by removal of cytoplasm components.

The experiment results are shown below.


Procedure 1

Procedure 2

Procedure 3

Procedure 4

Protein A

40 kDa

40 kDa

Not detected

40 kDa

Protein B

Not detected

50 kDa

Not detected

Not detected

Protein C

40 kDa

80 kDa

Not detected

80 kDa

Fill in the blanks using the answer code corresponding to the location of each protein:

Answer code:
01. Suspended in the cytoplasm.
02. Attached to the outside of the vesicle.
03. Entrapped inside the vesicle.
04. Spanning the vesicle membrane with one half being exposed on the outside and the
other half entrapped inside.
05. Cannot be determined from this experiment only.

The protein is present:

CODE
Protein A
Protein B
Protein C

6- Which of the following combinations between various components of the cytoskeleton


and their characteristics is correct:

COMPONENTS OF THE CYTOSKELETON


I. Microfilaments.
II. Microtubules.
III. Intermediate Filaments.

STRUCTURAL AND FUNCTIONAL CHARACTERISTICS


01. Polymer of the protein actin.
02. Participate in animal cytokinesis.
03. Participate in the formation of cilia and flagella.
04. Polymer of the protein tubulin.
05. Provide mechanical stability to the cell.
06. Participate in cellular locomotion.
07. Non-polar polymer.
08. Form the nuclear lamina.
09. Form the mitotic spindle.

II

III

A)

01, 03, 05

02, 04, 08

02, 03, 07

B)

01, 02, 06

03, 04, 09

05, 07, 08

C)

01, 03, 08

03, 04, 05

02, 06, 09

D)

01, 06, 09

02, 04, 07

03, 05, 07

7- Using the answer code, which of the following statements about the transport across the
plasma membrane of the animal cells is correct or incorrect.

Answer code:
01. Correct.
02. Incorrect.

STATEMENT

CODE

A) Steroid hormones are taken up by the cell by endocytosis.


B) Amino acids are taken up by the cell by simple diffusion.
C) Bacteria are taken up by the cell by phagocytosis.
D) Metabolic wastes are taken up by the cell by endocytosis.
E) Ions can pass through channel proteins by passive transport.
F) Cholesterol is taken up by the cell as a low-density lipoprotein
(LDL) by receptor-mediated endocytosis.
G) In the epithelial cells of the intestine, macromolecules
transport from the lumen to the interstitial fluid is by transcytosis.
H) The Na+ / K+ pump transports 3 Na+ into the cell and 2 K+ out
of the cell.

10

8- In the following table, some components, processes and structures of mitochondria are
shown. Match both columns and identify the correct combination.

01. Porin.
02. Enzymes of mitochondrial RNA synthesis.
03. ATP synthase.

I. Outer mitochondrial membrane.

04. Monoamine oxidase.

II. Inner mitochondrial membrane.

05. Enzymes of fatty acid oxidation.

III. Mitochondrial matrix.

06. Coenzyme Q.
07. Enzymes of citric acid (Krebs) cycle.

II

III

A)

02, 06, 07

01, 04, 07

01, 05

B)

01, 05, 06

02, 03

02, 04, 07

C)

01, 04

03, 06

02, 05, 07

D)

02, 05

01, 03, 07

06, 07

11

9- Below are 9 Statements about prokaryotic and eukaryotic flagellae are:

01. prokaryotic flagellae are covered with membrane.


02. eukaryotic flagellae rotate.
03. both prokaryotic and eukaryotic flagellae use proton gradients as a direct source of the
energy for the movement.
04. prokaryotic flagellae are formed from actin, eukaryotic ones from the tubulin.
05. prokaryotic flagellae consist of three parts: the basal apparatus, the hook and the
filament.
06. all prokaryotic cells have at least one flagellum.
07. all eukaryotic flagellae are covered with a plasma membrane.
08. all functional eukaryotic flagellae contain motor-proteins (dyneins).
09. prokaryotic flagellae can rotate only in one direction.

The correct statements are:

A) 01, 04, 07.


B) 03, 07, 08.
C) 02, 05, 09.
D) 05, 07, 08.

12

10- Look at the following picture and write down the correct answer code.

Which of the structures:

CODE

A) is the place where the ribosomal subunits are formed?


B) is the place where the glycosylation of proteins and lipids are carried out?
C) can form protein not encoded by the nuclear DNA?
D) maintains the structural integrity of an axon?
E) is the most abundant structure in the cytoplasm of the pancreatic
(secretory) cells?
F) is the most abundant structure in an insect flight muscle?
G) is the place of lipid synthesis?

13

11- The following figure corresponds to part of the nitrogen cycle. Match the processes
numbered 1 to 4 in the figure with the following answer code.

Answer code:
CODE

01. Ammonifying (putrifying) bacteria.


Process 1

02. Denitrifying bacteria.

Process 2

03. Nitrate reduction.

Process 3

04. Nitrifying bacteria.

Process 4

05. Protein synthesis.

14

12- Using the answer code, select which of the following statements about carbohydrates
and the human body are correct or incorrect.

Answer code:
01. Correct.
02. Incorrect.

15

STATEMENT

CODE

A) D-Galactose and D-glucose are pentose sugars.


B) D-Galactose and D-glucose are stereo isomers.
C) D-Galactose and D-glucose are aldoses.
D) In the human body the reaction from left to right takes place in the
duodenum.
E) In the human body the reaction from left to right takes place in the
mammary glands.

13- Complete the following diagram by using the answer code. Fill in the blank boxes with
the number corresponding to the regulation of the cycle phases.

Answer code:
I. Cyclin B-CdK1.
II. Cyclin A-Cdk2.
III. Cyclin E-CdK2.
IV. Cyclin D- CdK4.

16

PLANT ANATOMY AND PHYSIOLOGY (8 questions, 12 points)

The diagram on the next page shows the life cycle of a plant. Examine this figure and its
captions carefully. Answer questions 14 to 21 using the diagram on the next page.

14- This life cycle corresponds to a plant from which of the following taxon:

A) Bryophyta.
B) Pinophyta.
C) Magnoliopsida. (dicots)
D) Liliopsida. (monocots)

15- Choose and write down the Arabic number of the figure that represents fertilization
structure.

Answer: ________________................

17

18

16- The following table shows some of possible components in diagram 14 (page 18) of
the plant life cycle. Choose the correct option.

A)

Stigma

Outer

Pollen tube

Zygote

Synergid

Funiculus

Ovary

Archegonium

Ovum

Nucellus

Pollen tube

Male

Synergid

Funiculus

Archegonium

Outer

integument
B)

C)

Pollen

Pollen

tube

chamber

Stigma

Outer
integument

D)

Pollen

Synergid

gametes
Stigma

Funiculus

chamber
E)

Style

integument
Synergid

Stigma

Zygote

Outer

Antipodal

integument

cells

17- Which of the following statements associated with this life cycle is INCORRECT?

A) The anther endothecium (inner lining of an anther) develops into the fibrous layer.
B) The megaspores are arranged in one row and, generally, three of them degenerate.
C) The mature male gametophyte consists of three cells resulting from two meiotic
divisions.
D) The seed develops from the ovule.
E) The embryo constitutes a partially developed young sporophyte.

19

18- It may be reasonable to conclude that the species that shows the life cycle
represented in this figure (page 18) have the following pool of characteristics:

Answer code:
01. Naked seed.
02. Protected seed.
03. Winged seed.
04. Seed without albumin.
05. Perfect flower.
06. Imperfect flower.
07. Free pollen grains.
08. Agglutinated pollen (pollinium).
09. Anatropous ovule.
10. Orthotropous ovule.
11. Gametophyte generation only.
12. Sporophyte generation only.
13. Two alternating generations.
14. Hypogeal germination (cotyledons remain below ground).
15. Epigeal germination (cotyledons emerge above ground).

Write down the correct combination of characteristics.

Answer: ______________________

20

19- Observe the parts of the life cycle that correspond to processes occurring in the ovule
before fertilization and mark with an X the corresponding stages at which mitotic divisions
occur.

II

III

IV

VI

VII

VIII

IX

XI

XII

20- Suppose that the plant corresponding to this cycle, besides producing seeds, can
reproduce asexually through stem pieces or cuttings. Which of the following plant
hormones would you choose to improve rooting?

A) Gibberellins.
B) Cytokinins.
C) Ethylene.
D) Auxins.
E) Abscisic acid.

21

21- In the following table, nine mineral elements essential to the plant whose life cycle is
represented in the figure are listed. Match both columns and identify the correct
combination.

01. Calcium.
02. Potassium.
03. Iron.
04. Nitrogen.
I. Macronutrients
05. Magnesium.
II. Micronutrients
06. Molybdenum.
07. Phosphorus.
08. Zinc.
09. Sulfur.

II

A)

02, 04, 05, 07, 09

01, 03, 06, 08

B)

02, 04, 07, 09

01, 03, 05, 06, 08

C)

01, 02, 04, 05, 09

03, 06, 07, 08

D)

01, 02, 04, 05, 07, 09

03, 06, 08

E)

01, 02, 03, 04, 07

05, 06, 08, 09

22

ANIMAL ANATOMY AND PHYSIOLOGY (9 questions, 16 points)

22- Diagrams A and B correspond to the mechanisms of hormonal action. Complete the
diagram by filling in the boxes with the corresponding answer code (on the answer sheet),
using each answer code once .

Answer code:
01. chemical reaction.

05. receptor

02. steroid hormone.

06. peptide hormone

03. inactive enzyme.

0.7 cyclic AMP (cAMP)

04. protein.
23

23- Complete the following table on the answer sheet about the chemical nature of the
hormones by using the codes below:
Answer code:
01. Peptides or proteins.
02. Amino acid derivatives.
03. Fatty acid derivatives.
04. Steroid.
05. Glycoprotein
HORMONE

CODE

A) Progesterone
B) Insulin
C) FSH (Follicle-stimulating hormone)
D) LH (Luteinizing hormone)
E) Prolactin
F) Oxytocin
G) Estrogen
H) Testosterone
I) ACTH (Adrenocorticotropic hormone)

J) ADH (Antidiuretic hormone or vasopressin)

24

24- The following figure shows the interactions between the hypothalamus, the anterior
hypophysis (pituitary) and the male gonads. The full arrows ( ) indicate excitatory
effects and the dotted arrows ( ) indicate inhibiting effects.

Complete the boxes on the answer sheet by using the corresponding code. Use each code
once.

Answer code:
01. Sertoli cell.
02. Testosterone.
03. FSH - Follicle-stimulating hormone.
04. Leydig cells or interstitial cells.
05. Inhibin.

25

GnRH

Hypothalamus

(Gonadotropinreleasing hormone)

Anterior hypophysis

LH

(anterior pituitary)

Testis

26

25- 48 hours after beginning a hyposodic (low sodium) diet, the renal and hormonal
conditions of a person are controlled. Which combination of the following conditions does
the patient display? Select the correct answer.

Reference signs:
+: increase.
-: decrease.
=: no changes.

Aldosterone in ADH in plasma


plasma

Na+

Water

reabsorption

reabsorption

A)

B)

C)

D)

E)

27

26- Next to each type of receptor, write down the answer code of the corresponding
structures. All possible answer codes must be used once only.

CODE
Mechanoreceptor
Chemoreceptor
Photoreceptor

Answer code:
01. Gustatory papillae.
02. Crustacean statocysts.
03. Gustatory hairs in flies.
04. Vertebrate retina.
05. Muscle spindles.
06. Arhtropod ommatia.
07. Diptera halteres.
08. Labyrinth (cochlea and vestibule) vertebrate ear.

28

27- Identify what kind of muscular tissue which corresponds to each of the following sets of
characteristics. Write the correct answer codes in the corresponding box of the answer
sheet.

Muscular kind

Characteristics

Fibre shape

Elongated;
cylindrical;
blunt ends

Elongated;
spindle-like;
pointed ends

Elongated;
cylindrical;
branched;
fibres
interconnected

Number of nuclei
per fibre

Many

One

One or two

Nucleus location

Peripheral

Central

Central

Contraction
speed

Very fast

Very slow

Intermediate

Answer code:
01. cardiac muscle.
02. skeletal muscle.
03. smooth muscle.

29

28- The volume of blood pumped by each ventricle during a beat is known as systolic
volume. If it is multiplied by the number of beats per minute, the product is the cardiac
output.

Cardiac output = systolic volume X cardiac frequency

What is the cardiac output (volume of blood pumped by each ventricle in a minute) of an
adult person at rest whose heart beats 72 times per minute and pumps 70 millilitres of
blood in each ventricular contraction?

Answer code:
A) 3 L/min.
B) 5 L/min.
C) 10 L/min.
D) 7 L/min.

30

29- The cardiac frequency can increase or diminish under the influence of several factors.
Complete the column on the right by writing a (+) sign if the factor increases the cardiac
frequency and a (-) sign if the factor diminishes it.

FACTOR

SIGN

A) Hypoxia
B) Exhalation
C) Fever
D) Excitation
E) Inspiration
F) Exercise

30- The following diagram shows some of the factors that influence the cardiac output.
Complete the blank boxes in the answer sheet by using the answer code below. Use each
code once.
Answer code:
01. Cardiac frequency.
02. Adrenal glands.
03. Sympathetic nerves.
04. Systolic volume.
05. Parasympathetic nerves.
06. Cardiac centre in the medula oblongata.

31

Emotions

Other stimuli

Hypothalamus

----------------------------

----------------------

------------------

-------------------

Adrenalin

increases

Cardiac output =

increases

diminishes

------------------------

32

----------------------

GENETICS (12 questions, 14 points)

** In a butterfly species, the wing colour is determined by a locus with three alleles:
C (black wings) > cg (grey wings) > c (white wings). In a survey of a large population
living in Ro Cuarto, the following frequencies were obtained: C= 0.5; cg = 0.4, and c
= 0.1.

31- If the butterflies continue to mate randomly, the frequencies of black-winged, greywinged, and white-winged butterflies in the next generation will be:

Black wings

Grey wings

White wings

A)

0.75

0.24

0.01

B)

0.75

0.15

0.1

C)

0.24

0.75

0.01

D)

0.83

0.16

0.01

32- If the population consists of 6,500 butterflies, how many butterflies of each phenotype
will there be?

Black wings

Grey wings

White wings

A)

3656

374

2470

B)

4875

1560

65

C)

3595

1040

65

D)

4875

156

1469

33

A small group of butterflies of the Ro Cuarto population flies to an isolated


neighboring site in Las Higueras, and starts a new population. After several
generations, there is a large randomly mating population of butterflies in which the
following phenotypic frequencies are observed:

PHENOTYPE

FREQUENCY

black wings

0.00

grey wings

0.75

white wings

0.25

33.a- The allelic frequencies for C (black wings), c

(grey wings), and c (white wings)

under Hardy-Weinberg equilibrium are:

cg

A)

0.25

0.50

0.25

B)

0.00

0.75

0.25

C)

0.00

0.50

0.50

D)

0.25

0.25

0.50

33.b- The change in the allelic frequencies in this population as compared to the original
one is an example of:

A) migration.
B) selection.
C) bottleneck effect.
D) founder effect.
34

34- A group of birds migrates into this isolated site in Las Higueras. As the birds find it
easier to spot and catch the white-winged butterflies, the relative fitness of the whitewinged is reduced to 0.2.

Phenotype

Relative fitness

grey wings

white wings

0.2

What will the genotypic frequencies be after one generation of selection?

cgcg

cgc

cc

A)

0.3125

0.625

0.0625

B)

0.25

0.5

0.05

C)

0.25

0.5

0.2

D)

0.263

0.526

0.211

35

Human lactoferrin (hLf) is an 80kD monomeric glycoprotein originally found in milk


that confers antibacterial and iron transport properties to humans. A group of
scientists decided to generate transgenic tobacco plants expressing hLf cDNA
(complementary DNA).
35- In order to prepare a cDNA library in Escherichia coli, total cellular RNA was extracted
from human mammary gland and the mRNA was isolated. Then, they converted the mRNA
into cDNA. Look at the following figure, and employing the answer code, determine the
correct order of steps (1-7) to obtain cDNA. Write your answer on the answer sheet.

Answer code:

01. C bases added to 3end.


02. Add terminal transferase + dCTP.
03. Add reverse transcriptase + 4 dNTPs + oligo dT primer (TTTT).
04. Second DNA strand synthesized from GGGG primer to 3 end.
05. ssDNA strand synthesized from TTTT primer to 3 end.
06. hydrolyzed RNA leaving DNA.
07. Add DNA polymerase + 4 dNTPs + oligo-dG primer (GGGG).

36

37

36- The scientists constructed a cDNA library in a lambda vector. They chose a lambda
vector instead of a plasmid vector because lambda vectors:

I. can be packaged in vitro.


II. can accommodate larger DNA foreign fragments than plasmid vectors.
III. do not have to be cut with restriction enzymes.
IV. are introduced in to Escherichia coli cells by transformation.
V. are introduced in Escherichia coli cells by infection.
VI. carry antibiotic-resistance genes.
VII. lambda libraries are easier to screen.
VIII. lambda vectors form larger colonies.

Select the combination of the correct options:

A) I, II, VII.
B) III, V, VI.
C) II, IV, VIII.
D) I, II, V.

37- They identified in the library the cDNA clone encoding hLf using the sequence of the
hLf gene (with a chemical label) as a probe. The screening procedure employed was:

A) Southern hybridization.
B) colony hybridization.
C) Northern hybridization.
D) plaque hybridization.
E) immuno blotting

38

38- Once the desired cDNA clone was identified, the scientists isolated and sequenced the
insert in order to be sure it was the hLf cDNA. Deduce the nucleotide sequence of the
original strand of this small fragment sequenced with the dideoxy method.

A) 5 TGGCTACC 3.
B) 3 TGGCTACC 5.
C) 5 ACCGATGG 3.
D) 3 ACCGATGG 5.

39- As the sequencing confirmed it was the hLf cDNA, they inserted it into the EcoRI site
of a plasmid. The plasmid contained: a) the wild type of hLf coding sequence including its
signal peptide (PSLf) under the control of the 35S strong promoter and the 35S terminator
of the cauliflower mosaic virus, and b) the left (LB) and right (RB) borders of T-DNA. It was
used to transform E. coli and then transferred to Agrobacterium tumefaciens by
conjugation.

39

The promoter sequence of the cauliflower mosaic virus was necessary because:

A) they wanted to sequence the hLf cDNA again.


B) they wanted to express the hLf gene throughout the transgenic tobacco plant.
C) they wanted to express the hLf gene in the cauliflower as well.
D) without a promoter the gene does not replicate.

40- Leaf discs of tobacco (Nicotiana tabacum) were infected with recombinant
Agrobacterium tumefaciens. To determine which tobacco plants were transgenic, the
scientists performed:

A) a Western blot analysis.


B) a Northern blot analysis.
C) a Southern blot analysis.
D) microscopic observation.

40

41- Twenty tobacco transgenic plants were screened for the expression of the hLf gene:
1g of fresh weight of transgenic leaf tissue was ground in liquid
nitrogen and total soluble proteins were extracted. The following figure
shows the Western analysis of the total soluble protein extract from
transgenic leaves (1. concentrated protein extract; 2. milk hLf. The
position of protein standards is shown on the left). The blot was:

A) immunostained with anti-milk hLf antibodies.


B) hybridized with hLf mRNA.
C) hybridized with the cDNA encoding human lactoferrin.
D) hybridized with the EcoRI fragment containing the hLf cDNA.

41

42- After the transformation of tobacco (N. tabacum) with a human lactoferrin cDNA under
the control of the 35S promoter from cauliflower mosaic virus, using A. tumefaciens-based
gene transfer, the authors concluded that:

I. it leads to the production of full-length 80-kDa hLf in transgenic plants.


II. the human lactoferrin obtained is different from the hLf produced by human mammary
gland.
III. the human lactoferrin obtained is not a monomeric glycoprotein.
IV. transgenic plants are able to produce human lactoferrin.
V. the transgenic tobacco plants produce milk with human lactoferrin.
VI. the tobacco hLf protein produced, presents a molecular mass closely identical to the
native protein.
VII. carbohydrate compositions of tobacco hLf and milk hLf are the same.
VIII. the human lactoferrin obtained confers antibacterial and iron transport properties to
humans.

Select the correct combination of options:

A) I, IV, VI.
B) I, V, VII.
C) III, IV, VIII.
D) II, V, VIII.

42

ECOLOGY (7 questions, 12 points)

43- In order to determine the armadillos' (Dasypus novemcinctus) eating habit, a group of
scientists conducted a vegetation survey and compared it with the food remains found in
the excrements. The scientists walked in a straight line through a sunflower and a corn
field, and a natural pasture. They conducted a vegetation census in a 1m 2 square every 50
metres recording the species diversity, the coverage and the phenological stage (stage of
the flowering plant). The following experimental techniques were available:

I. soil sampling.
II. systematic sampling.
III. a graphical presentation of life forms by means of a bar graph.
IV. coverage estimation (% of the ground covered by the species).
V. collection of vegetation biomass.
VI. use of a transect.
VII. data analysis (eg. classifying species and census).
VIII. species listing.

43

Which of the above techniques were employed?

A) I, III, V, VII.
B) II, IV, VI, VIII.
C) II, V, VI, VIII.
D) I, II, III, IV.
E) V, VI, VII, VIII.

** The results of the samples of the mulitas excrement (I), and the composition of
the vegetation in the sunflower field (II), in the corn field (III), and in the natural
pasture (IV), are shown below. Column 1 shows the percentage of faeces which
contain remains of a particular plant species and identifies the form of those
remains. In columns II, III and IV, the phenological stage and the percentage
coverage of each species of plant are presented. The excrement and vegetation
sampling was performed at the same time.

44

Plant Species

II

Species 1
Fruit, 90%
Epidermis, 100% Flower, 5%

Species 4
Species 5

IV

Fruit, 90%

Species 2
Species 3

III

Epidermis, 10%

Species 6

Flower, 10%

Flower, 2%

Flower, 10%

Flower, 6%

Flower, 3%

Vegetative, 6%

Vegetative, 2%

Vegetative, 7%

Vegetative, 5%

Vegetative, 20%

Species 7

Vegetative, 8%

Vegetative, 40%

Species 8

Epidermis, 40%

Vegetative, 5%

Vegetative, 90% Vegetative, 15%

Species 9

Seeds, 20%

Fruit, 30%

Fruit, 40%

Species 10

Epidermis, 10%

Fruit, 30%

Vegetative, 15% Fruit, 28%

Species 11

Epidermis, 60%

Flower, 10%

Flower, 6%

Species 12

Seeds, 80%

Vegetative, 90% Fruit, 90%

Fruit, 40%

Species 13

Seeds, 100%

Flower, 10%

Flower, 3%

44- In which environment have the mulitas fed?

A) Only in II.
B) Only in III.
C) Only in IV.
D) Both A and B are correct.
E) Both A and C are correct.

45

Fruit, 6%

Fruit, 5%

Flower, 30%

45- To compare the population size of mulitas between a corn field and a natural pasture,
another group of scientists carried out two samplings. In the first one, they caught 130
specimens in each environment, marked them without interfering with their survival, and
then released them. Three days later, a second random sample was taken. Of the 125
animals captured in the corn field, 72% were marked. In the natural pasture 45% of the
144 specimens caught were marked. Assuming no changes in the population size within
the three days, which environment had the largest population and what was its size?

Answer:

A) corn field; approximately 288 individuals.


B) natural pasture; approximately 180 individuals.
C) corn field; approximately 180 individuals.
D) natural pasture; approximately 288 individuals.
E) corn field; approximately 280 individuals.

46

** The following diagram corresponds to a trophic/food web in which the mulita is


present. The numbers represent other species, and the arrows, the energy transfer
pathway in an ecosystem.

10

6
4

11
MULITA

12

3
2

5
13

14

46- This mulita species is:


A) Herbivorous.
B) Carnivorous.
C) Omnivorous.
D) Saprophagous.

47- An example of a long trophic/food chain is:


A) 5, 8, 4, 11, 1.
B) 9, 3, mulita, 10, 4.
C) 13, 2, mulita, 11, 12.
D) 13, 2, mulita, 4, 1.

47

** When a plant community develops by the process of ecological succession,


population age structure also varies in the participating species causing changes in
the net productivity and in the community biomass. The following figure shows an
example of the outcome of a study of reforestation of an abandoned land.

48- On the ANSWER SHEET, complete the figure by adding the appropriate curves to
indicate:
I.

net primary production evolution (in red pencil); and

II.

biomass (in green pencil)

III

II biomass (kg/m2)

I net production (g/m2/a)

Use the graph axes help.

IV

Grassland

Shrubland

48

Forest

49- Using the answer code, say whether the following statements are correct or incorrect
according to results shown in the above figure.

Answer code:
01. Correct
02. Incorrect.

STATEMENTS
A) In advanced stages of succession, species richness increases.
B) During the first stage of succession, the number of vascular species (III)
increases.
C) Pioneer and opportunistic species (IV) would be eliminated in the
shrub/scrub phase by competition.
D) Between the 5th and 14th years there is a shift in the trajectory of the
four characteristics (I IV) analysed in this study.
E) Richness of vascular and pioneer and opportunistic species (IV) are
inversely proportional.
F) The number of vascular species fluctuates around an equilibrium.

49

CODE

BIOSYSTEMATICS (3 questions, 6 points)

50- The characteristics of eight taxonomic groups (indicated with letters A to H) are shown
in the table below:

Group Amniotic Notochord Hair

Legs

egg

Bony

Teeth/

endoskeleton

Jaws

Key:

+ feature present
- feature absent

50

Based upon these features complete the following evolutionary tree by writing the correct
taxon group letters in the corresponding branches.

51- In the table over the page you will find several statements about three families of
Liliopsida (Monocot) class. Match both columns and identify the correct answer.

51

01. The inflorescence of almost all species of this family is


an unbranched fleshy spike composed of numerous very
small flowers.

02. Most species of this family have bulbs. The bulbs are
tunicated or, sometimes, have numerous overlapping
fleshy scales.

03. The vegetative body is usually a trunk that terminates in


a crown of leaves. Histologically, the woody stem
consists of primary tissues, which were originated from
the growing tip.

04. Stems of most species of this family are short and each
one usually ends in an umbel-like inflorescence.

I. Araceae

05. Leaves are usually pinnate or palmate, and differ


enormously in size at maturity, sometimes reaching a

II. Arecaceae

length of about 25 m. They can bear a crest at the


junction of the petiole and leaf blade.

III. Liliaceae

06. Most species of this family are adapted to disperse their


fruits by animals, but some have their fruits dispersed by
water due to a very thick fibrous mesocarp.

07. The species of this family include a wide range of plant


forms, many of them are epiphytes, hemiepiphytes, or
root climbers.

08. Fenestrated or Perforated leaves are a special


characteristic of some genera of this family.

09. The fruit is usually a capsule with several seeds.

52

II

III

A)

01, 05, 07

04, 08, 09

02, 03, 06

B)

04, 07, 08

01, 05, 06

02, 03, 09

C)

01, 07, 08

02, 05, 06

03, 04, 09

D)

01, 07, 08

03, 05, 06

02, 04, 09

E)

03, 06, 08

02, 05, 07

01, 04, 09

52- In the table below you will find several statements. Using the answer code, indicate on
your answer sheet whether you consider the statement correct or incorrect.

Answer code:
01. Correct.
02. Incorrect.

53

STATEMENTS

CODE

A) Careful examination, using cleared and stained leaves of


Ginkgo biloba, reveals that the venation is net-like.
B) Lycopodiaceae family is terrestrial or epiphytic. The outer
walls

of

megaspores

and

microspores

have

taxonomic

significance.
C) The leaves of Equisetum are small and whorled. Antherozoids
are multiflagellate.
D) Pinus species have female cones with woody scales at
maturity.
E) The indusium is a structure that protects sporangia in true
ferns.

54

ETHOLOGY (2 questions, 4 points)

** Guppys are among the first fish that beginners in acquaculture get. They are
commonly called "millionaire fish" because of their abundant progeny.
In 1966, professor C. M. Breder, then director of the New York aquarium, decided to
conduct an experiment to investigate the causes of their proliferation.
In a small aquarium - with a capacity of 27.5 liters of water- sufficiently supplied with
food and oxygen to maintain up to 500 fish meticulously taken care of, he
introduced a couple of guppys (1 adult male and 1 adult female). In the course of the
following 6 months and at 4-week-intervals between each breeding (these fish are
ovoviviparous), the female produced 102, 87, 94, 71 and 89 offspring, that is a sum
total of 443 guppys. A later recount showed that only 9 out of all the newborns
remained alive: 6 females and 3 males. All the rest had been devoured by the
mother.
At the same time in another aquarium of equal size and identical conditions, the
researcher put 8 adult males, 8 adult females and 8 young fish, that is to say, a total
of 24 guppys. The females had also abundant offspring here. The data of the
proliferation survey along the 6 months following the introduction of the original
group of 24 guppys in the aquarium, are shown in the following tables.

55

FEMALE 1
4 week

8 week

12 week

16 week 20 week

Males

29

24

31

30

33

Females

58

48

64

58

68

Total

87

72

95

88

101

N of offspring

Males

recounted hours

Females

after breeding

Total

N of offspring in
each breeding

Observations: The newborns were devoured by their own mother

FEMALE 2
4 week 8 week 12 week 16 week 20 week
Males
32
26
33
28
29
N of offspring in
Females
65
50
66
56
58
each breeding
Total
97
76
99
84
87
Males
0
0
0
0
0
N of offspring
Females
0
0
0
0
0
recounted hours
after breeding
Total
0
0
0
0
0
Observations: The newborns were devoured by the own mother
FEMALE 3
4 week 8 week 12 week 16 week 20 week
Males
32
29
25
34
28
N of offspring in
Females
64
56
51
69
55
each breeding
Total
96
85
76
103
83
Males
0
0
0
0
0
N of offspring
recounted hours
Females
0
0
0
0
0
after breeding
Total
0
0
0
0
0
Observations: The newborns were devoured by the own mother

56

FEMALE 4
4 week 8 week 12 week 16 week 20 week
Males
28
25
35
30
29
N of offspring in
Females
57
49
69
61
60
each breeding
Total
85
74
104
91
89
Males
0
0
0
0
0
N of offspring
Females
0
0
0
0
0
recounted hours
after breeding
Total
0
0
0
0
0
Observations: The newborns were devoured by the own mother
FEMALE 5
4 week 8 week 12 week 16 week 20 week
Males
33
30
30
23
30
N of offspring in
Females
67
59
64
47
60
each breeding
Total
100
89
94
70
90
Males
0
0
0
0
0
N of offspring
recounted hours
Females
0
0
0
0
0
after the breeding
Total
0
0
0
0
0
Observations: The newborns were devoured by their own mother
FEMALE 6
4 week 8 week 12 week 16 week 20 week
Males
30
29
26
35
25
N of offspring in
Females
62
57
53
70
52
each breeding
Total
92
86
79
105
77
Males
0
0
0
0
0
N of offspring
recounted hours
Females
0
0
0
0
0
after breeding
Total
0
0
0
0
0
Observations: The newborns were devoured by their own mother
FEMALE 7
4 week 8 week 12 week 16 week 20 week
Males
29
24
33
28
29
N of offspring in
Females
60
50
71
57
62
each breeding
Total
89
74
104
85
91
Males
0
0
0
0
0
N of offspring
Females
0
0
0
0
0
recounted hours
after breeding
Total
0
0
0
0
0
Observations: The newborns were devoured by their own mother

57

FEMALE 8
4 week 8 week 12 week 16 week 20 week
Males
26
32
33
28
28
N of offspring in
Females
52
65
64
58
57
each breeding
Total
78
97
97
86
85
Males
0
0
0
0
0
N of offspring
Females
0
0
0
0
0
recounted hours
after breeding
Total
0
0
0
0
0
Observations: The newborns were devoured by their own mother

Original number of individuals


N recounted after a year

ORIGINAL GROUP
ADULTS
Males
Females
8
8
3

YOUNGSTERS
8
0

Observations:
- The youngsters of the original group were devoured by the adults of the original group
- Some adults of the original group died by unknown causes
53- Which of the following statements can be made from the analysis of the previous data?

I. Guppys eat their own offspring (infanticide behavior).


II. Guppys show indiscriminate cannibalism devouring their own offspring until
exterminating them.
III. Guppys show selective cannibalism, that is to say, they preserve the life of their
offspring as long as a certain population density is maintained.
IV. Guppys show indiscriminate cannibalism by devouring their own offspring, although
they always allow the survival of a random number of them.

58

Choose the corresponding combination of answers:

A) I, II.
B) I, III.
C) I, IV.
D) IV.
E) I.

54- Which of the following statements is more likely to explain the above mentioned
behavior in guppys?
I. Lack of maturity of the instincts in the young mothers (preadult).
II. Instinctive conflicts caused by a premature reawakening of female heat that, in
reaction to male heat, causes the break of mother-offspring bonds.
III. Instinctive conflicts with addictive impulses such as gluttony.
IV. The aggressiveness caused by excessive hunger.
V. An increase of the stress levels, and the consequent increase of aggressiveness due to
the overpopulation.
VI. An increase of the stress levels, and the consequent increase of aggressiveness
caused by lack of vital space.

Choose the corresponding combination of answers:

A) I, II.
B) III, IV.
C) V, VI.
D) V.
E) III.

THE END!!!
59















All IBO examination questions are published under the following Creative Commons license:



CC BY-NC-SA (Attribution-NonCommercial-ShareAlike) https://creativecommons.org/licenses/by-nc-sa/4.0/
The exam papers can be used freely for educational purposes as long as IBO is credited and
new creations are licensed under identical terms. No commercial use is allowed.

TOTAL:73,5

ANSWER SHEET
THEORETICAL TEST PART B

STUDENT NUMBER

CELLULAR BIOLOGY
1-

1 point
A

2-

CODE
02

0.2

01

0.2

04

0.2

02

0.2
0.2
0.2

01

0.2
0.2

0.4 point
A

4-

1 point

A
B
C
D
E
F
G
H
3-

13,4

1.8 points

A
B
C
D
E
F
G
H
I

CODE
01

0.2

02

0.2

02

0.2

02

0.2

02

0.2

01

0.2

03

0.2

02

0.2

01

0.2

5-

1.2 points

Protein A
Protein B
Protein C
6-

04

0.4

CODE
02

0.2

02

0.2

01

0.2

02

0.2

01

0.2

01

0.2

01

0.2

02

0.2

1 point
A

10-

0.4

1 point
A

9-

01

1.6 points

A
B
C
D
E
F
G
H
8-

0.4

1 point
A

7-

CODE
03

1.4 points

A
B
C
D
E
F
G

CODE
07

0.2

01

0.2

05

0.2

03

0.2

06

0.2

05

0.2

02

0.2

11-

1 point

Process 1.
Process 2.
Process 3.
Process 4.
12-

0.25

04

0.25

03

0.25

05

0.25

1 point

A
B
C
D
E

13-

CODE
01

CODE
02

0.20

01

0.20

01

0.20

02

0.20

01

0.20

1 point

PLANT ANATOMY AND PHYSIOLOGY

14-

12

1 point
A

15-

1 point
Answer: ___________________14_____________________________________

16-

2.5 points
A

17-

2 points
A

18-

2 points, 0.4 per code


Answer: __02, 05, 07, 13, 15

19-

1 point
I

20-

II

IV

V
x

VI
x

VII
x

VIII

1 point
A

21-

III

1.5 points
A

IX

XI

XII

ANIMAL ANATOMY AND PHYSIOLOGY

22-

16

1.5 points, 0.75 per figure

03
06
01
07

04

02
05

23-

2 points

CODE

A
B
C
D
E
F
G
H
I
J

04

0.2

01

0.2

05

0.2

05

0.2

01

0.2

01

0.2

04

0.2

04

0.2

01

0.2

01

0.2

24-

1 point

GnRH
(Gonadotropin-releasing

Hypothalamus

hormone)

Anterior hypophysis

03

LH

05
02

04

01
Testis

25-

3 points
A

26-

2 points (0.25 x code)

Mechanoreceptor
Chemoreceptor
Photoreceptor

CODE
02, 05, 07, 08

01, 03

0.5

04, 06

0.5

27-

1.5 points

MUSCULAR KIND

28-

CODE
03

01

0.5

0.5

0.5

2 points
A

29-

02

1.5 points

A
B
C
D
E
F

SING
+

0.25

0.25

0.25

0.25

0.25

0.25

30-

1.5 points
Emotions

Other stimuli

hypothalamus

06

02

03

05

Adrenalin

increases
diminishes

increases

Cardiac cost =

04

GENETICS
31-

1 point
A

32-

1 point
A

33.a-

14,1

1 point
A

01

33.b-

1 point
A

34-

1 point
A

35-

2.1 points, 0,3 in the correct


d
Answer: _03, 05, 02, 01, 06, 07, 04

36-

1 point
A

37-

1 point
A

42-

1 point
A

41-

1 point
A

40-

1 point
A

39-

1 point
A

38-

1 point
A

ECOLOGY
43-

1 point
A

44-

2 points
A

45-

12

2 points
A

46-

1 point
A

47-

1 point
A

48-

2 points (1 x line)

49-

3 points

A
B
C
D
E
F

CODE
02

0.5

01

0.5

01

0.5

02

0.5

02

0.5

01

0.5

10

BIOSYSTEMATICS
50-

2 points

F, B and E may be switched

51-

2.5 points
A

52-

1.5 points

A
B
C
D
E

Code
02

0.3

02

0.3

01

0.3

01

0.3

01

0.3

ETOLOGY

11

INTERNATIONAL BIOLOGY OLYMPIAD


PRACTICAL PROBLEMS

2006, Rio Cuarto, Argentina















All IBO examination questions are published under the following Creative Commons license:



CC BY-NC-SA (Attribution-NonCommercial-ShareAlike) https://creativecommons.org/licenses/by-nc-sa/4.0/
The exam papers can be used freely for educational purposes as long as IBO is credited and
new creations are licensed under identical terms. No commercial use is allowed.

17 th INTERNATIONAL BIOLOGY OLYMPIAD


9-16 JULY 2006
Ro Cuarto Repblica Argentina

PRACTICAL TEST

1
Plant Anatomy, Systematics and Physiology

Student code:

17 th INTERNATIONAL BIOLOGY OLYMPIAD


9-16 JULY 2006
Ro Cuarto Repblica Argentina

General remarks about the practical tests


DEAR PRATICIPANTS
The practical tests are organized in four different laboratories.

N 1- Plant Anatomy, Systematics and Physiology


N 2- Animal Anatomy, Systematics and Ecology
N 3- Biochemistry
N 4- Microbiology
You have 1 hour in laboratory N 1
You have 1 hour in laboratory N 2.
You have 1 hour 30 minutes in laboratory N 3,
and 1 hour 30 minutes in laboratory N 4.
You can score maximum 40 points in each laboratory, which means a total of 160
points for the practical test.

Good luck !!!!!!!

Practical test N 1: Plant Anatomy, Systematics and Physiology


In this laboratory task you will have to work on the morphological, taxonomic and
physiological aspects of higher plants in an integrated way.
Aims
A) To identify and compare vegetative organs.
B) To identify different taxa.
C) To relate leaf anatomy to photosynthetic pathways.
Materials:

5 samples (labeled 1-5).


5 slides.
5 coverslides.
1 razor blade.
1 felt-tip marker for glass.
1 tweezers.
2 histological needles.
1 dropper with distilled water and glycerin.
1 Petri dish with Safranin solution (it stains lignin).
1 Petri dish with distilled water.
1 microscope.
Figure 2: microphotographs with details of leaf sectors.

Procedure

Cut cross sections of sample 1.


Place the sections into the Safranin solution.
Transfer the sections to the Petri dish with distilled water to remove the excess of
stain.
Place the sections on a slide with water and glycerin and cover with a coverslide.

Repeat the procedure to obtain histological slides of the remaining samples.


Observe the obtained histological slides with the microscope. Remember to start
observing with the lowest magnification power and then, end up with the 40x objective
lens.
After examining each specimen and your prepared histological slide answer the following
questions:
Q1 : Fill in the organ code number in the appropriate box.
Codes:
01- stem.
02- root.
03- leaf.
04- rhyzome

Sample 1
Code

Q2 : Identify the taxon to which each sample belongs, to write its number in the
appropriate box.
Taxon
Ginkgophyta
Pinophyta
Cycadophyta
Magnoliophyta - Magnoliopsida
Magnoliophyta-Liliopsida

Sample number:

Q3 : The endodermis is a layer of cells that performs an important physiological role.


Indicate with an X the sample/s where this cellular layer is observed.
Sample

Q 4: Plants may differentiate collenchyma and sclerenchyma as supporting tissues. Both


tissues show particular cytological characteristics that allow us to identify them. Circle the
option that contain/s the sample number/s where collenchymatic tissue is observed.
a) 1, 2, 3.
b) 4, 5.
c) 4.
d) 2.
e) 1, 4.

Q 5: Examine carefully the leaf anatomical structure represented in the figure N 1.

Figure N 1: leaf anatomical structure

Could this leaf structure correspond to some of the organs previously cut and be part of
the same plant? Circle the correct option.
YES

NO

If the answer was affirmative, indicate with an X the corresponding sample/s.


Sample

Q 6: Microphotographs with details of leaf sectors are shown (Figure n 2). Select the
codes of those microphotographs corresponding to the sectors pointed out in the leaf
diagram represented in Figure n 1

Figure N 2: microphotographs with details of leaf

01

02

03

04

07

06

08

05

12

13
09

11

10

14

15

16

17
7

18

Q7: Keeping in mind all the anatomical characters observed when completing figure 1 you
could infer that this leaf corresponds to a species that belongs to the Family (Circle the
correct answer):
a) Liliaceae.
b) Fagaceae.
c) Brassicaceae.
d) Poaceae.
e) Araceae.
Q8: Leaf anatomy is related to the environment where the plant grows and it indicates its
photosynthetic pathway. Keeping this in mind, observe again the leaf structure
represented in figure 1 and select the codes that correspond to this structure.
01- It follows the Calvin Cycle photosynthetic pathway or C3 pathway.
02- It has an additional method of fixation of the carbon (not alternative) that
works separately from the Calvin Cycle.
03- It shows a stratified mesophyll.
04- It shows a radiated mesophyll (Kranz).
05- It shows chloroplast dimorphisms and /or sizes.
06- Optimum temperature for photosynthesis is between 15-25.
07- Optimum temperature for photosynthesis is between 30-45.
08- It shows two well-developed sheaths around the vascular bundles.
09- It shows one sheath around the vascular bundles.
10- It does not show sheaths around the vascular bundles.
11- The decarboxylation phase takes place in different structures of the leaf.
Answer: ...... ..........................................................................

Q9: Complete the following comparative table of the three main photosynthetic pathways
of carbon assimilation, keeping in mind the codes for each character.
Enzyme responsible for the initial carboxylation:
01- Ribulose 1,5 - bisphosphate carboxylase-oxygenase (Rubisco).
02- Phosphoenolpyruvate carboxylase (PEPase).
03- Sucrose-phosphate synthase (SPase).
04- Rubisco and PEPase.
05- SPase and PEPase.
Leaf anatomy:
01- stratified.
02- Kranz structure (radiated).
03- succulent.
The CO2 fixation time:
01- Day.

02- Night.
03- Day and night.
First stable product of C02 fixation:
01- Made up of six carbons.
02- Made up of four carbons.
03- Made up of three carbons.
Efficiency in water use:
01- Medium.
02- High.
03- Low.
Photosynthetic rate:
01- Medium.
02- High.
03- Low.

Character
Enzyme responsible for the
initial carboxylation:

C3

C4

CAM

Leaf anatomy:
The CO2 fixation time:
First stable product of CO2
fixation:
Efficiency in water use:
Photosynthetic rate:

Q10: If a plant is placed into a closed chamber and exposed to the light, it is observed that
the CO2 concentration in the air inside the chamber decreases for a while due to
photosynthesis. The decrease is gradual but it never reaches the zero value. A balance is
reached between the CO2 captured by photosynthesis and the one released by respiration
and photorespiration. This balance is known as CO2 compensatory point.
In the following graph the effect of the atmospheric concentration of CO2 on the
photosynthesis rate in plants C3 and C4 is observed. The arrows indicate the
compensatory points of each plant.
Indicate the curve that corresponds to each plant by writing C3 or C4 in the right box.
9

Reference:

: compensatory points of each plant

10

17th INTERNATIONAL BIOLOGY OLYMPIAD


9 - 16 JULY 2006
Ro Cuarto Repblica Argentina

PRACTICAL TEST

2
Animal Anatomy, Systematics and Ecology

Student Code:

17 th INTERNATIONAL BIOLOGY OLYMPIAD


9-16 JULY 2006
Ro Cuarto Repblica Argentina

General remarks about the practical tests


DEAR PARTICIPANTS
The practical tests are organized in four different laboratories.

N 1- Plant Anatomy, Systematics and Physiology


N 2- Animal Anatomy, Systematics and Ecology
N 3- Biocheminstry
N 4- Microbiology
You have 1 hour in laboratories N 1 and N 2.
You have 1 hour 30 minutes in laboratories N 3 and N 4.
You can score maximum 40 points in each laboratory, which means a total of
160 points for the practical test.

Good luck !!!!!!!


2

Practical Test N 2: Animal Anatomy, Systematics and Ecology


Introduction
Bivalves are an important group of molluscs, the second in number of species
after gastropoda. Other names for the class include Pelecypoda, and Lamellibranchia.
Bivalves include all laterally compressed mollusc species; they typically have twopart shells dorsally hinged by strong muscles and ligaments.
The mantle, which secretes the shell, is the dorsal body wall covering the visceral
mass. The mantle cavity is lateral and in most bivalves the gills have a respiratory and
digestive function.
Unlike other molluscs, bivalves lack a radula and feature labial palps which carry
food from the gills to the mouth.
The head is small and it does not feature specific sensory organs.

Task N 1: Bivalve dissection (13 points)


Task N 1 includes part A (10 points) and part B (3 points)
The aim is to compare anatomical structures in three marine bivalves.
Materials:
9 Tray containing three samples of marine bivalves numbered 1, 2 and 3 (stored in
70% alcohol).
9 1 dissection table.
9 1 lancet.
9 1 tweezers.
9 10 color pins (3 green, 3 red, 3 blue and 1 yellow).
9 1 pair of disposable gloves.
9 1 respirator mask.
9 1 magnifying glass.
REMARK: BEFORE STARTING THE PRACTICAL TASK, BE SURE TO HAVE ALL
THE LISTED MATERIALS, OTHERWISE RAISE YOUR HAND TO CALL THE
ASSISTANT.

PART A
Procedure
1- Put on the gloves and respiratory mask.
2- Before starting the dissection, locate the external parts of the bivalve (Figure 1).

Figure 1
3- The valves are hinged by ligaments. In order to identify the internal structure you
have to dissect the bivalve. You must be very careful when separating the valves so as
not to hurt your hands.
Insert the lancet (Figure 2) and cut
bivalve.

,

the adductor muscle/s, according to the

Figure 2
4- In order to separate the valves completely, once the muscle/s is/are cut, you must cut
carefully the ligament in the umbo area.
5- Once the three samples are dissected, identify the structures with different color pins.
Use 3 pins for each bivalve sample (green, red, and blue), and a yellow pin only for
bivalve 2, in the following way:

green pin for the foot.


red pin for labial palps.
blue pin for the gills.
yellow pin for the inhaling siphon. (Only for bivalve 2)

6- After finishing the task you must raise your hand. An assistant will check the
task. The Practical Test Sheet should be signed by both, you and the assistant.

Signatures:
Student.:....................................Assistant:...................................................

PART B
As you have seen during the dissection, the three bivalves show differences in their
muscles.
There exists a muscle classification according to their number and size:
9 Dimyarian isomyarian condition: in which both muscles have similar size.
9 Dimyarian heteromyarian condition: in which both muscles are different in size.
9 Monomyarian condition: Having only one, large adductor muscle to close the
valves.

Complete the table by using the codes below.


Bivalve 1

Bivalve 2

Condition

Codes:
01- Dimyarian isomyarian.
02- Dimyarian heteromyarian.
03- Monomyarian.

Bivalve 3

Task N 2: Bivalve adaptive radiation


Task N 2 includes parts A, B and C.
Most bivalves are filter feeding molluscs, that is to say, they filter marine water to obtain
their food consisting mainly of plankton and suspended organic matter.
The
evolutionary acquisition of feeding by filtering allowed them to colonize many habitats,
thereby giving rise to an important adaptative radiation.
The aims of this part of the test are to determine the habitat of the marine bivalve
samples and to identify the exomorphological and anatomophysiological characteristics
associated to these habitats.
PART A (9 points) Below there are two marine zones, one corresponding to a sandy
beach (Figure 1), and the other to a rocky beach (Figure 2).

Figure 1

High
Tide.

Low
Tide.

Figure 2

High tide

Low
tide

Fill in the corresponding box in each table, indicating the site where the samples given in
this practical task can be found.
Codes:
01- Bivalve 1.

Sandy beach

02- Bivalve 2.

Zone I

03- Bivalve 3.

Zone II

Zone III

Zone IV

Rocky
beach

Zone I

Zone II

Zone III

Zone IV

Zone V

PART B (6 points)- Keeping in mind the zones occupied by bivalves in rocky and sandy
beaches, you must determine the category of the given samples by writing an X in the
corresponding box.
Bivalve 1

Bivalve 2

Bivalve 3

Burrowers in soft
substrate
INFAUNA
Surface dwellers attached
to the substrate
EPIFAUNA
Free swimming

PART C (12 points)- A series of characteristics related to the three given bivalves and
their habitats is given below. Examine your dissected specimens and summarize ALL
their characteristics by writing the appropriate answer codes from the list into the table
below.
Answer code:
01- large, burrowing foot
02- reduced, finger-like foot.
03- highly reduced and barely visible foot.
04- no anterior adductor muscle.
05- no siphons.
9

06- two siphons: incurrent and excurrent (inhaling and exhaling).


07- fringed incurrent siphon.
08- highly developed sensory lobes in the mantle, with tentacles and small ocella.
09- flat lower valve (right)
10- mantle edge with fusion points.
11- byssal threads.
Bivalve 1

Bivalve 2

Bivalve 3

10

Practical Test N 2: Animal Anatomy, Systematics and


Ecology
TASK N 1: Bivalve dissection (13 points)

PART A: One point per pin. Total: 10 points.

PART B: One point per box. Total: 3 points


Bivalve 1
02

Condition

Bivalve 2
01

Bivalve 3
03

Task N 2: Bivalve adaptive radiation (27 points)


Part A: One point per box. Total: 9 points
Sandy beach

Rocky
beach

Zone I
02

Zone I
01

Zone II
02

Zone III
02

Zone IV
03

Zone II

Zone III

Zone IV

Zone V

01

01

01

03

Part B: Two point per each correct mark .Total: 6 points.


Bivalve 1
Burrowers in soft substrate
INFAUNA
Surface dwellers attached to
the substrate EPIFAUNA

Bivalve 2

Bivalve 3

X
X

Free swimming

Part C: One point per correct code. Total: 12 points

Bivalve 1
02- 05- 10- 11

Bivalve 2
01- 06- 07

Bivalve 3
03- 04- 05- 08- 09

17 th INTERNATIONAL BIOLOGY OLYMPIAD


9-16 JULY 2006
Ro Cuarto Repblica Argentina

PRACTICAL TEST

3
Biochemistry

Student code:

17 th INTERNATIONAL BIOLOGY OLYMPIAD


9-16 JULY 2006
Ro Cuarto Repblica Argentina

General remarks about the practical tests


DEAR PARTICIPANTS
The practical tests are organized in four different laboratories.

N 1- Plant Anatomy, Systematics and Physiology


N 2- Animal Anatomy, Systematics and Ecology
N 3- Biochemistry
N 4- Microbiology
You have 1 hour for each laboratory: N 1 and N 2.
You have 1 hour 30 minutes for each laboratory: N 3 and N 4.
You can score maximum of 40 points in each laboratory, which means a total of
160 points for the whole practical test.

Good luck !!!!!!!

Practical test N 3: Biochemistry

Enzymatic determination of glucose

TASK 1:

You have to perform a calibration curve using a standard of glucose,

with known concentration. Then, plot the results as absorbance versus glucose
concentration (15 points)

Important: Raise the red card to call the lab assistant when you are ready to use the
spectrophotometer.

Introduction:
Glucose oxidase (GOD) catalyzes the oxidation of (beta)-D-glucose to D-gluconic acid and
hydrogen peroxide. It is highly specific for (beta)-D-glucose and does not act on (alpha)-Dglucose. The horseradish peroxidase (POD) breaks down hydrogen peroxide into water
and oxygen, using the dye (4-aminophenazone) as an electron donor. At the same time,
the dye is converted to its oxidized form, which is a colored compound. Since the amount
of hydrogen peroxide produced indicates how much reaction has taken place, the
formation of the red color can be used to follow the course of the reaction.

Its major use is in the determination of free glucose in body fluids. Although specific for
(beta)-D-glucose, glucose oxidase can be used to measure total glucose, because as a
result of the consumption of (beta)-glucose, (alpha)-glucose from the equilibrium is
converted to the (beta)-form by mutarotation.

PRINCIPLE
The reaction system is as follows:
GOD
Glucose + O2 + H2O----------------> Gluconic Acid + H2O2

POD
2 H2O2 + 4-AP + 4-Hydroxybenzoate------------ > red quinoneimine

Glucose oxidase reagent: solution containing glucose oxidase, peroxidase, 4aminophenazone (4-AP), and phosphate buffer pH 7.0 containing hydroxybenzoate.

Reagents:
1. glucose oxidase reagent (ready to use).
2. glucose solution (unknown concentration).
3. glucose solution 5 mg. ml-1.
4. distilled water.

Equipment
1. Lab gloves (1pair).

8. Paper towels (3)

2. Marker pen (1).

9. 1000 l tips (30)

3. 1.5 ml microtubes (18).

10. 200 l tips (30)

4. Pipettes (2).
5. Incubator at 37oC .
6. Spectrophotometer (you will use it with lab assistants).
7. Spectrophotometric cuvettes (8).
Instruments:

P1000

P100

Tip ejecting knob


Adjustable Wheel

Pushing Knob
Counter

Adjustment method
You have to pull up the Adjustable Wheel, then you can revolve the adjustable wheel or
knob. Adjust the required volume and push down the Adjustable Wheel.
Remember that minimal and maximal volumes for P100 are 10 l and 100 l
respectively.
For P1000 minimal volume is 100 l and maximal volume is 1000 l.

Usage method:
Please secure the suction tip, after that slightly push down the pushing knob to first stop,
hold and immerse the tip into solution vertically. The immersed depth of the tip is 2-4 mm,
then release the pushing knob slowly and make it return to the original position. Take off
the pipette from the liquid and place the suction tip of the pipette into a special container
receiving the dispensed liquid. The tip must be close to the inner wall of the container.
Depress the pushing knob to the first stop and further more to discharge the solution
completely from the tip. After that, you can take away the pipette and release the button.
Eject the used tip to the trash recipient by pressing the Tip ejecting knob.

EXPERIMENTAL PROCEDURE

1) Label five 1.5-ml microtubes 1/2 through 1/32 with a marker pen. Using the glucose
standard solution (5 mg. ml -1) perform the following serial dilutions (in distilled water) in a
final volume of 100 l: 1/2, 1/4, 1/8, 1/16, and 1/32.
2) Mix well and perform (in a new 1.5-ml microtubes set) the enzymatic determination of
glucose for each dilution according to the following squeme.

1/2

1/4

1/8

1/16

1/32

Blank

Sample volume

10l

10l

10l

10l

10l

Water volume

10 l

Glucose oxidase

1 ml

1 ml

1 ml

1 ml

1 ml

1 ml

reagent volume

3) Mix well and incubate microtubes at 37C for 5 min.


4) Put the content of each microtube in a spectrophotometric cuvette.

5) Read absorbance in a spectrophotometer at 505 nm. You have to use the blank for
calibration. (When you are ready to read in the spectrophotometer, please call the lab
assistant). If you have to wait to use the spectrophotometer, please continue with
the questions. The extra waiting time will not affect the results.
6) Plot the absorbance versus the amount of glucose (in g) on the plotting paper below.

Dilutions
1/2
glucose (g in the reaction
mix)
Absorbance at 505 nm

1/4

1/8

1/16 1/32

TASK 2:

Determination of the glucose concentration in a sample employing the

standard curve obtained before. (10 points)


1) Perform the glucose oxidase reaction to the glucose sample (unknown concentration),
according to the follow scheme.

Sample

Blank

Sample volume

10l

Water volume

10 l

Glucose oxidase reagent volume

1 ml

1 ml

2) Mix well and incubate microtubes at 37C for 5 min.


3) Put the content of each microtube in a spectrophotometric cuvette
4) Read absorbance in the spectrophotometer at 505 nm. You have to use the blank for
calibration. (When you are ready to read in the spectrophotometer please call the lab
assistant)
5) Using the calibration curve, calculate the glucosa concentration of the sample
in g. ml-1.

Absorbance of the sample


Concentration of the sample (in g . ml -1)

QUESTION 1:

DELETED

As many glucose assays measure the peroxide produced

by the glucose oxidase reaction, it is important that the enzyme used for these assays
presents: (1.5 points)
A) a low catalase content.
B) a high catalase content.
C) a low peroxidase content.
D) a high peroxidase content.

WRITE DOWN THE LETTER CORRESPONDING TO CORRECT ANSWER

Answer:................................................................................................

QUESTION 2: Glucose oxidase reagent may contain catalase. If such a condition is not
taken into account the obtained results will give (3 points):
A) underestimation of the glucose in the assay.
B) overestimation of the glucose in the assay.
C) no effect in the assay.

WRITE DOWN THE LETTER CORRESPONDING TO CORRECT ANSWER

Answer:................................................................................................

10

QUESTION 3: The most favorable pH value (the point at which the enzyme is most active)
is known as the optimum pH. Extremely high or low pH values usually result in a complete
loss of enzyme activity due to (1 point):
A) The breakdown of the secondary structure of the protein.
B) The breakdown of the tertiary structure of the protein.
C) The breakdown of the primary structure of the protein.

SELECT ONLY ONE CORRECT ANSWER. MARK IT WITH A CROSS.


A.
A, C

B.

C.
B, C.

A, B
A, B, C.

QUESTION 4: Glucose oxidase from the fungus Aspergillus niger was overexpressed in
yeast. The recombinant glucose oxidase was purified and glycosylation pattern was
analyzed by treatment with endoglycosidase H and -mannosidase. After treatment, an
aliquot was used for SDS-PAGE (electrophoresis in polyacrylamide gels containing
sodium dodecyl sulphate) in reducing conditions. The remaining enzyme was employed for
determination of the KM (Michaelis-Menten constant) with glucose as the substrate.
Michaelis-Menten constant (KM) is the concentration in moles/litre of a substrate at half the
maximum velocity of an enzymatic reaction. (7 points)

11

The values of the KM for each glycoforms are shown below the figure 1.
kDa
200

116
97

66
KM (mM)

20

28

35

60

Figure 1: Analysis of the deglycosylation of Glucose Oxidase by


7.5% acrylamide SDS-PAGE gel electrophoresis. Lane 0 is the
molecular mass standard. Lane 1 untreated enzyme. Lane 2
endoglycosidase H treated enzyme. Lane 3 -mannosidase treated
enzyme. Lane 4 endoglycosidase H and -mannosidase treated
enzyme (fully deglycosylated enzyme).

Which of the following is/are correct conclusion(s) from these results (figure 1)?
A) Glucose oxidase is a homodimer with a molecular mass of 96 kDa.
B) The deglycosylated form has a molecular mass of approximately 68 kDa.
C) Glucose oxidase is glycosylated since the treatment with endoglycosidase H and mannosidase results in a form with lower molecular mass.
D) The polysaccharide moiety of glucose oxidase contains N-acetylglucosamine and
mannose.

MARK THE CORRECT ANSWER/ ANSWERS.

12

Which is/are correct conclusion(s) from the results obtained in the determination of KM for
each glycoform?
A) The affinity of the fully glycosylated enzyme for glucose is higher than the affinity of the
deglycosylated enzyme
B) The glucose oxidase activity is completely abolished in the deglycosylated form
C) The lack of the sugar moiety could cause changes in the structure of the active site of
the enzyme resulting in the observed modifications of the KMs.

MARK THE CORRECT ANSWER/ ANSWERS.

13

QUESTION 5: Another treatment consisted in the purification of the recombinant glucose


oxidase under non-denaturating conditions and in presence of glutaraldehyde. The purified
enzyme was analyzed by SDS-PAGE in reducing (DTT+) as well as in non-reducing (DTT) conditions (without SDS). Prior to loading, the samples were resuspended in loading
buffer with (+) and without (-) DTT (a reducing agent). The obtained results are shown in
figure 2 (4 points)

DTT

DTT

192 kDa
Figure 2: SDS-PAGE analysis of purified
glucose

oxidase.

electrophoresis

in

(non-denaturating

polyacrylamide

gels)

of

recombinant glucose oxidase purified under


non-denaturating conditions.

96 kDa

Taking in account the results obtained from figure 1 and figure 2 the most probable
conformation of the recombinant glucose oxidase is:
A) A monomeric enzyme non-glycosylated.
B) A monomeric enzyme glycosylated.
C) An homodimer consisting of two monomers both glycosylated.
D) An heterodimer consisting of two subunits one of them glycosylated.

MARK THE CORRECT ANSWER.

14

15

17 th INTERNATIONAL BIOLOGY OLYMPIAD


Correct answer sheet
Practical test N 3: Biochemistry
Enzymatic determination of glucose
TASK 1: (15 points )

Dilutions

5 points
1/2

1/4

1/8

1/16

1/32

glucose (g in the reaction


mix) 0.5 pt each

25

12.5

6.25

3.125

1.56

Absorbance at 505 nm
0,5 pt each

0.8

0.4

0.2

0.1

0.05

Total 10 points
Values properly plotted 5 pt
Rect 1 pt
1.0

Units 0,5 pt

OD (505 nm)

0.8

0.6
Spectro1
Spectro2
Spectro3
Spectro4
Spectro5
Spectro6
mean

0.4

0.2

Title 0,5 pt

0.0
0

10

15

20

Glucose (g)

Title 0,5 pt

Units 0,5 pt

25

Full range scale 1 pt


Linneal scale 1 pt

Total 10 points
5 points if interpolation is correct

Absorbance of the sample


Concentration of the sample (in g . ml -1)

X
1100-1400

5 points

Question 2 (3 points)
A

3 points

Question 3 (1 point)
X AB 1 point

XB 0.5 point

Question 4 (9.6 points)


A
0.9

XB
0.9

XC
0.9

XA
2

B
2

XC
2

Question 5 (4 points)
XC

XD
0.9

XA 0.5 point

17th INTERNATIONAL BIOLOGY OLYMPIAD


9 - 16 JULY 2006
Ro Cuarto Repblica Argentina

PRACTICAL TEST

4
MICROBIOLOGY

Student Code:

17 th INTERNATIONAL BIOLOGY OLYMPIAD


9-16 JULY 2006
Ro Cuarto Repblica Argentina

General remarks about the practical tests


DEAR PARTICIPANTS
The practical tests are organized in four different laboratories.

N 1- Plant Anatomy, Systematics and Physiology


N 2- Animal Anatomy, Ecology and Systematics
N 3- Biocheminstry
N 4- Microbiology

You have 1 hour for each laboratory: N 1 and N 2.


You have 1 hour 30 minutes for each laboratory: N 3 and N 4.
You can score maximum of 40 points in each laboratory, which means a total of 160
points for the whole practical test.

Good luck !!!!!!!

Practical test 4
MICROBIOLOGY
There are different systems of bacterium classification, but the most commonly used is
the published in Bergey's Manual of Determinative Bacteriology.
A working outline for the identification of a bacterial strain from the biochemical point of
view is proposed below:
1) Isolate the strain and obtain a pure culture.
2) Carry out a microscopic examination of living cells and also of Gram stained smears.
The morphology and type of Gram staining of the microorganism under study is thus
determined. It is also important to determine the presence of clusters, spores and any
other morphological characteristics that may be of interest.
3) Determine the nutritional characteristics (in general they come off from the methods
used in the previous isolate and culture): photoautotrophs, photoheterotrophs,
chemiautotrophs, chemiheterotrophs.
4) Conduce primary tests: The following group of tests, called primary tests, are used
to determine the genus, group of genera or in some cases, the family to which an
isolate belongs to. The primary tests are, beside Gram staining and morphology
observation, the determination of catalase, oxidase, glucose fermentation, and motility,
among others.

Reagents and Equipment

1. Dropping bottle with Gentian Violet (ready to use)


2. Dropping bottle with Lugol (ready to use)
3. Dropping bottle with Gram decolorizer (ready to use)
4. Dropping bottle with Safranin (ready to use)
5. Dropping bottle with Distilled water
6. 1 tube rack
7. 2 Kahn tubes containing a culture grown in Luria-Bertani medium of organisms
A and B.
3

8. 2 Lab gloves
9. Respiratory mask
10. Marker pen
11. Paper napkin
12. 1 Bunsen burner
13. Microscope
14. Loop
15. 4 Slides
16. Tray with slide holder
17. 1 plastic bottle with water for rinsing
18. 1 disposable glass
19. 1 dropping bottle with immersion oil
20. 1 dropping bottle with 3% H2O2
21. 2 Luria-Bertani agar plates inoculated with organisms A and B.
22. 1 Eppendorf tube with 2 oxidase disks
23. 1 pair of tweezers
24. 2 Kahn tubes
25. 1 Kahn tube with a stopper containing sterile distilled water.
26. 1 plastic Pasteur pipette.
27. 3 plates with eosin methylene blue agar (EMB) medium (one of them inoculated
with organism A, another inoculated with organism B and the last one without
inoculation)
28. 3 tubes with phenylalanine (one of them inoculated with organism A, another
inoculated with organism B and the last one without inoculation).
29. 1 Dropper containing 10% ferric chloride
30. 3 Kahn tubes with SIM (hydrogen Sulfide Indole Motility) medium (one of them
inoculated with organism A, another inoculated with organism B and the last one
without inoculation)
31. 1 Dropping bottle containing Indole reagent
32. 3 Kahn tubes containing UREA broth (one of them inoculated with organism A,
another inoculated with organism B and the last one without inoculation)

33. 3 Kahn tubes with motility indole ornitine medium (MIO)

(one of them

inoculated with organism A, another inoculated with organism B and the last one
without inoculation)
34. 3 Kahn tubes containing Simmons citrate (labeled as SC-A and SC-B and
another one labeled SC without inoculation)
35. Clock located in view of all the students in the laboratory.

Caution:

You must be careful in the manipulation of media and reagents since the
quantities provided allow performing this practical test only once.
If you work carelessly, with abrupt movements, far from the burner, you will
contaminate the medium thereby preventing you from obtaining good results.
You will perform the biochemical tests which basis and interpretation are
detailed below by using the media, reagents, and the given bacteriological information
(charts and diagrams)

Note: Do not discard the tubes with organisms A and B. You will use them in
task 2.

TASK 1: Perform Gram-staining in organisms A and B.

EXPERIMENTAL PROCEDURE

Introduction:

Gram stain differentiates between two major bacterial cell wall types. Some
bacterial species, because of the chemical nature of their cell walls, have the ability to
retain the crystal violet even after the treatment with an organic decolorizer such as a
mixture of acetone and alcohol.

Gram stain technique

1. Make a thin smear of the material to study and allow to air dry.
2. Fix the material to the slide so that it does not wash off during the staining
procedure by passing the slide three or four times through the flame of a
Bunsen burner.
3. Place the smear on a staining rack and overlay the surface with Gentian Violet
solution.
4. After 30 seconds of exposure to the Gentian Violet solution, wash thoroughly
with running water.
5. Next, overlay the smear with Grams iodine solution (lugol) for 30 seconds.
6. Hold the smear between the thumb and forefinger and flood the surface with a
few drops of the acetone alcohol decolorizer until no violet color washes off.
This usually requires 10 seconds or less time.
7. Wash with running water and again place the smear on the staining rack.
Overlay the surface with safranin counterstain for 20 seconds. Wash with
running water.
8. Place the smear in an upright position in a staining rack, allowing the excess
water to drain off and the smear to dry.
9. Examine the stained smear under the 100 x (oil) immersion objective of the
microscope.
10. When you focus the microscope call the assistant.

Results
SELECT THE CORRECT ANSWER, FILLING THE CORRESPONDING BOX

Organism

Gram staining

Positive

Negative

Positive

Negative

Assistant revision

Question
Gram-variability

A) is a term which can be used where two Gram reactions are seen due to an error in
the staining procedure.

B) applies to an organism which changes its cell wall structure from the Gram-positive
type to the Gram-negative type as the culture ages.

C) applies to what is ultimately seen when cells in a culture of gram-positive bacteria


lose the ability to retain the primary stain during the decolorization process.

D) indicates a mixed (i.e., impure) culture.

Write the letter corresponding to the correct answer on the dotted line below:

TASK 2: Biochemical characterization of organisms A and B.

In this part of the practical work you will determine, by means of metabolic tests
(already provided or performed by you), the family and genus of the two organisms
labeled as A and B.

Catalase Reaction:

Some bacteria contain flavoproteins that reduce the oxygen resulting in the production
of hydrogen peroxide (H2O2) or superoxide (O2 -), which are extremely toxic since they
are powerful oxidizers able to destroy the cellular constituents in a short time. Many
bacteria possess enzymes that offer protection against these toxic compounds.

Technique
Perform the catalase test to organisms A and B by adding two drops of H2O2 to a
bacterial suspension (3 loopfuls of the liquid culture labeled as LB-A and LB-B) placed
on the slides.
Note: Write the obtained results in the Biochemical test table using + (for a
positive reaction) or (for a negative reaction)

Question
Which of the following reactions is carried out by the catalase enzyme?
1) H2O2 + NADH + H+ 2 H2O + NAD+
2) H2O2 + H2O2 2H2O + O2
3) O2- + O2- + 2H+ H2O2 + O2
4) 4 O2- + 4H+ 2H2O + 3 O2

Write the number corresponding to the correct answer on the dotted line below:

Oxidase Reaction:

Test used for the detection of the cytochrome-c-oxidase enzyme, which is present in
different genera, e.g. Pseudomonas spp., Neisseria spp., Moraxella spp., Vibrio spp.,
Aeromonas spp.
Oxidase discs contain dimethyl-para-phenylene-diamine, which is the substrate of
cytochrome-c-oxidase enzyme.
Organisms possessing this enzyme, in the presence of atmospheric oxygen and the
substrate contained in the oxidase discs give a red-fuchsia color.

Technique
Perform the oxidase test to organism A and B according to the following instructions:
Oxidase test will be carried out using tubes. From a pure culture, prepare a heavy
suspension in 0.2 ml of sterile distilled water, and add one oxidase disc.
Note: Prepare the bacterial suspension starting from 3 colonies of each one of
the plates labeled as LB-A and LB-B respectively.

Results
Generally, within the first minute and at room temperature, positive results are
detected. A delayed reaction, evidenced after 2 minutes must be considered a negative
result.

Positive: discs show a red-fuchsia color.


Negative: no changes in disc color.
Note: Write the obtained results in the Biochemical test table using + (for a
positive reaction) or (for a negative reaction)

EOSIN METHYLENE BLUE (EMB) AGAR


This medium, is used for the selective isolation of fast growing and with scarcely
nutritional requirements Gram-negative bacteria.
It allows the growth of all Enterobacteriaceae members.
Purpose
This medium combines the Holt-Harris formulation with the Levines one to improve the
selective isolation of Enterobacteriaceae and other Gram-negative bacterium species.
The differentiation between lactose and/or sucrose fermenter organisms from those
organisms which do not ferment them is possible due to the presence of the indicators
eosin and methylene blue. Also, these indicators inhibit the growth of several Grampositive bacteria.
Many strains of Escherichia coli and Citrobacter spp. show colonies with a greenish
metallic sheen in this medium.
Lactose and/or sucrose fermenter organisms show colonies with a dark center
surrounded by a blue or pink color, while lactose and/or sucrose non fermenter
organisms show colorless colonies.
This medium also allows the growth of different organisms in addition to the growth of
the Enterobacteriaceae members, and may be generally differentiated by the
appearance of their colonies.

Instructions

Using the EMB plates provided (labeled as EMB-A and EMB-B for organisms A and B
respectively), determine the sucrose and/or lactose utilization for organisms A and B.

Note: Write the obtained results in the Biochemical test table using + (for a
positive reaction) or (for a negative reaction).

10

Question
Fermentation

A) results in production of acid and possibly gas from the breakdown of sugars.
B) is associated with the type of growth of facultative anaerobes in Thioglycollate
Medium where growth is less dense in the anaerobic region.

C) is generally associated with a positive catalase reaction for an organism.

Write the letter/s corresponding to the correct/s answer/s on the dotted line
below:

Phenylalanine agar (Tubes labeled as Ph)

Phenylalanine agar is recommended for the detection of production of phenylpyruvic


acid from phenylalanine by deamination. A positive reaction results in a green
coloration after the application of 10% ferric chloride.

Instructions

Add 4 or 5 drops of the ferric chloride solution to the phenylalanine slants agar tubes
(labeled as Ph-A and Ph-B for organisms A and B respectively). As the reagent is
added rotate the tubes. An intense green color appearing within 10 minutes indicates
the presence of phenylpyruvic acid.

11

Note: Write the obtained results in the Biochemical test table using + (for a
positive reaction) or (for a negative reaction).

Hydrogen Sulfide indole motility medium (SIM) (Tubes labeled as SIM A and SIM
B)
This medium is used for the detection of hydrogen sulfide, indole production, and
motility in the same tube. Hydrogen sulfide production in this medium is originated from
thiosulphate or sulphate reductases and not by cysteine desulfhidrases. Any
blackening along the line of inoculation is considered as a positive hydrogen sulfide
reaction, and it usually appears between 18-24 hours of inoculation. Motile cultures in
SIM medium show diffuse growth away from the line of inoculation. This is an
appropriated medium for the detection of Listeria's characteristic "umbrella-like"
movement. The high content of tryptophan in this medium makes it very suitable for
detection of indole production.

Instructions

Using the SIM tubes provided (labeled as SIM-A and SIM-B for organisms A and B
respectively), determine the production of hydrogen sulfide and indole, as well as the
motility for organisms A and B.
For indole production detection, add 5 drops of the reagent (labeled as indole) to the
heavy growth obtained in SIM tubes. A pink color promptly developed indicates the
presence of indole.
Note: Write the obtained results in the Biochemical test table using + (for a
positive reaction) or (for a negative reaction).

Question

A negative result for motility


12

A) is indicated if growth occurs only along the line where the medium was stabinoculated.

B) should be confirmed by a wet mount of a young culture of the same organism.


C) may exhibit growth over the surface of the medium.
D) may occur for strictly aerobic, motile organisms.
Write the letter/s corresponding to the correct/s answer/s on the dotted line
below:

UREA BROTH
This medium is suitable for the differentiation of urease producing organism.

Instructions
Using the urea broth tubes provided (labeled as UREA-A and UREA-B for organisms A
and B respectively), determine the production of urease for organisms A and B.

Note: Write the obtained results in the Biochemical test table using + (for a
positive reaction) or (for a negative reaction).

SIMMONS CITRATE AGAR


It is a medium capable to differentiate between bacteria harboring citrate permease
enzymes and those that do not harbor such enzymes.
Instructions
Using the SIMMONS CITRATE tubes provided (labeled as SC-A and SC-B for
organisms A and B respectively), determine the presence of growth for organisms A
and B.
Note: Write the obtained results in the Biochemical test table using + (presence
of growth) or (absence of growth).
13

Motility Indole Ornitine (MIO) Medium


The reactions in this medium are observed as follows:

Ornithine Decarboxylation (ODC). Observe the lower three-quarters (anaerobic


region) of the medium for change in color of the pH indicator; growth must be
present in this part of the tube for correct analysis of result:
o

Gray, blue or purple color: Positive reaction for ornithine decarboxylation


formation of a highly alkaline product, over-neutralizing the acid
produced from glucose fermentation.

Yellow color: Negative reaction. Yellow color is due to the "default" acid
production from glucose fermentation.

Instructions
Using the MIO tubes provided (labeled as MIO-A and MIO-B for organisms A and B
respectively), determine the production of ornithine decarboxylase enzyme for
organisms A and B.

Note: Write the obtained results in the Biochemical test table using + (for a
positive reaction) or (for a negative reaction).

Results:
Write the results of the biochemical tests in the following table (11 points)
Organism

catalase

lactose sucrose motility indole H2S

A
B

Using the tables in the annex indicate (9 points)

14

Phenyl
alanine

ODC

Ureasa citrate

oxidase

Family

Genus

Organism A
Organism B

Questions
1. You have cultures of five organisms as listed below. However, the labels of the
tubes have come off and you need to re-label the tubes correctly! First, you must
consider the various reactions you know for the organisms in question:

Gram

genus

stain

shape

catalase glucose

lactose

phenylalanine citrate

reaction fermentation fermentation deaminase

utilization

Bacillus

rod

+ or

Staphylococcus

coccus

Enterobacter

rod

Morganella

rod

Pseudomonas

rod

a. The results obtained from what specific laboratory procedure will differentiate
Bacillus and Staphylococcus from each other and also from the remaining three
genera?

A) Glucose fermentation
B) Citrate utilization
C) Catalase reaction
D) Gram stain
Write the letter corresponding to the correct answer on the dotted line below:

15

2. Consider the following dilution scheme:

a. Report the total number of CFUs (colony forming units) in the entire 100 ml amount
of the original lake water sample. (TNTC=too numerous to count.)

A) 5.8 10 7 cfu / 100 ml


B) 4.25 10 8 cfu / 100 ml
C) 1.2 10 9 cfu / 100 ml

Write the letter corresponding to the correct answer on the dotted line below:

16

b. Would you expect any change in the answer of the above problem if the first dilution
was made by adding one ml of sample to 9 ml of diluent?

A)

Yes

B)

No

Write the letter corresponding to the correct answer on the dotted line below:

Annex 1
Gram stain (fresh

shape

coccus

coccus

coccus

coccus

rod

rod

rod

rod

rod

rod

rod

rod

coccus

grouping

clusters

clusters

chains

Tetrads

aerobic growing

anaerobic growing

motility

+ or -

catalase

culture)

pairs

+ or + or

+ or + or

oxidase
fermentation of
glucose to acid or

acid+gas

Micrococcus
Staphylococcus
Streptococcus

X
X
X
17

+ (or
)

+ or

Lactococcus

Enterococcus

Clostridium

Bacillus

Alcaligenes

X
X

Pseudomonas

Enterobacterias

Aeromonas

Chromobacterium

Neisseria

18

Gram-Negative cocci and bacilli

Cocci
Bacilli
Group XI
Neisseria
Veillonella
Facultative
anaerobic

Aerobic

Group VIII
Pseudomonas,
Alcaligenes
Halobacterium

motile

Non-motile

Group X
Shigella
Klebsiella

Group IX
Escherichia Proteus
Enterobacter Prividencia
Citrobacter Morganella
Erwinia
Salmonella

Lactose fermentation

Lactose +

Lactose -

Indole +

Indole Glucosa +

citrate +

Citrobacter

citrate -

Escherichia

Urea +

Glucosa -

Urea -

Enterobacter
Klebsiella

19

Salmonella
Shigella
Proteus
Providencia
Morganella

Pseudomonas
Alcaligenes

Motility

motile

Non-motile

citrate +

Lactose +

Citrobacter
Enterobacter

citrate -

citrate +

citrate -

Klebsiella

Shigella

Lactose -

Salmonella
Erwinia

Lactose +
Ureasa -

Lactose Ureasa +

Escherichia

Proteus
Providencia
Morganella

20

Family

Genus

oxidation
catalase

lactose sucrose motility indole SH2

Phenyl

ODC

Ureasa

citrate

oxidase

alanine

Enterobacteriaceae

Pseudomonaceae

Escherichia

Shigella

Salmonella

Citrobacter

+/-

Proteus

+/-

+/-

Morganella

Enterobacter

+/-

+/-

Serratia

+/-

+/-

+/-

Klebsiella

+/-

Pseudomonas

+/-

+/-

+/-

+/-

21

Correct Answer Sheet

Practical test 4
MICROBIOLOGY
TASK 1: Perform Gram-staining in organisms A and B.

Organism

Gram staining

Positive

X Negative

Positive

X Negative

Assistant revision

10 point (5 point organism A and 5 point organism B)

Question

TASK 2: Biochemical characterization of organisms A and B.

Question (1.6 point)


Which of the following reactions is carried out by the catalase enzyme?
2
Question
Fermentation (1.6 point)
A, B
Question
A negative result for motility (1.6 point)

A, B, C, D

Results:
Write the results of the biochemical tests in the following table (11 point )
Organism

catalas
e

Pheny
lactose

motility indole

SH2

lalani

ODC

Ureasa citrate

oxidase

-(0.5p)

- (0.5p)

ne

(0.5p)

(0.5p)

sucrose

+ (0.5p)

(0.5p)

- (0.5p) - (0.5p)

(0.5p)

+
+
(0.5p)

(0.5p) (0.5p) (0.5p) (0.5p)

- / + + / - -(0.5p)

(0.5p) (0.5p) (0.5p) (0.5p)

(0.5p)

+ ( 0.5p)

(0.5p)

Using the tables in the annex indicate (9 point)

Family

Genus

Organism A

Escherichiae (2.25 p)

Escherichia (2.25p)

Organism B

Pseudomonaceae (2.25p)

Pseudomonas (2.25p)

Questions
1. (1.6 point)

D
2. a. (1.6 point)

A
b. (2 point)

INTERNATIONAL BIOLOGY OLYMPIAD


THEORY PROBLEMS

2005, Beijing, China















All IBO examination questions are published under the following Creative Commons license:



CC BY-NC-SA (Attribution-NonCommercial-ShareAlike) https://creativecommons.org/licenses/by-nc-sa/4.0/
The exam papers can be used freely for educational purposes as long as IBO is credited and
new creations are licensed under identical terms. No commercial use is allowed.

16th International Biology Olympiad


Beijing
July, 2005

THEORY EXAMINATION
Part 1

Total time available: 2.5 hours (150 minutes)

Total points available: ~80

GENERAL INSTRUCTIONS

Please check that you have the appropriate examination papers and answer sheets.

It is recommended that you manage your time in proportion to the points allocated for
each question.

IMPORTANT

Use the answer sheets provided to record your answers.

Ensure your name and three digit code numbers are written on the top of each page of the
answer sheets.

Use the 2B pencil provided to fill in the correct answers on the answer sheet.

1.

Various forces are important in the interactions contributing to the tertiary structure
of a protein. The figure below is a diagram showing several possible interactions.
Please match the numbered interactions with their correct names. (1 point)

A. Hydrogen bond
B. Hydrophobic interaction
C. Peptide bond
D. Disulphide bond
E. Ionic bond
Interactions

Answer: A-E

1
2
3
4

2. Which of the following statements about cytokinesis in plant cells is/are NOT correct ?
(1 point)
(1) Plant cells form cell plates
(2) Cytokinesis can start during mitosis
(3) Plant cells have a contracting ring
(4) Membrane fusion connects cell plate and the cytoplasmic membrane of the
mother cell.
(5) Golgi apparatus does not participate in cytokinesis of a plant cell until two
daughter cells are formed.
A. 1, 2, 4,
B. 3
C. 3, 5
D. 4, 5
E. 4
3. DNA ligase is an important enzyme that connects DNA fragments. Which of the
following is/are TRUE about DNA ligase? (1 point)
1) It is important to the DNA replication process
2) It is important in molecular cloning
3) It requires DNA fragments having sticky ends
4) It could cut DNA molecules in the presence of ATP and Mg2+.
5) It requires ATP for its function because the 3-hydroxyl group of a DNA fragment
needs to be phosphorylated before the DNA molecules could be ligated.

A. 1, 2, 3
B. 2, 3, 5.
C. 1, 2
D. 1, 5
E. 1, 2, 4
Questions 4-6: Checkpoints in the cell cycle are very important in regulation of cell
cycle. The following three questions are about the cells cycles and checkpoints.

4. Two animal cells at different phases in the cell cycle can be induced to fuse to form a
single cell with two nuclei. This system provides a very useful tool for studying the
cell cycle.
Which of the following is correct? (1 point)
A. When a cell in M phase is fused with a cell in G1 phase, the nucleus in M phase
stops the mitotic process.
B. When a cell in M phase is fused with a cell in G2 phase, the nucleus in G2 phase
starts the mitotic process.
C. When a cell in G2 phase is fused with a cell in G1 phase, both nuclei start the
mitotic process.
D. When a cell in M phase is fused with a cell in G1 phase, the nucleus in G1
phase starts DNA synthesis.
E. When a cell in M phase is fused with a cell in G1 phase, the nucleus in M phase
stops the mitotic process.

5. Which of the following are true about checkpoints in the cell cycle? (1 point)
(1) If a cell in the G1 phase does not receive a signal at the G1 checkpoint, the cell
usually goes into the G0 phase.
(2) A cell must receive a signal at the G2 checkpoint to go into mitosis.
(3) A cell must receive a signal at the M checkpoint to go into mitosis.
(4) The protein factors that control checkpoints in cell cycle are mostly present in
nuclei.
(5) The cell cycle in unicellular organisms does not have checkpoints.

A. 1, 2
B. 1, 3,
C. 1, 3, 4
D. 2, 3, 4
E. 1, 5
6. In cloning the first mammal, researchers used a mammary cell as the nuclear donor
and fused it with an enucleated egg (cell with nucleus removed). Which of the
following is CORRECT? (1 point)
A. The mammary cell was in G1 phase
B. The mammary cell was in G2 phase
C. The mammary cell was in S phase
D. The mammary cell was in M phase
E. The mammary cell was in G0 phase

7.

Cyanobacteria (blue-green algae) are a group of very important bacteria that perform
photosynthesis. Which of the following is/are TRUE about cyanobacteria. (1 point)

(1)They are gram-negative bacteria


(2) They produce oxygen in photosynthesis
(3) All cyanobacteria can fix nitrogen
(4) Some cyanobacteria can live with fungi symbiotically
(5) The blue-green color of cyanobacteria comes solely from chlorophyll

A. All are correct (1, 2, 3, 4, 5)


B. 1, 2, 3, 4
C. 1, 2, 3
D. 1, 2, 4
E. 1, 2
Questions 8-9 are about biotechnology of transgenic organisms or genetically modified
organisms (GMO).

8.

In creating golden rice that produces betacarotenes in rice kernels, the genes
responsible for betacarotene synthesis are transformed. Which of the following
is/are true? (1 point)

(1) The researcher used normal rice for transformation


(2) The researcher used Ti plasmid for transformation
(3) The researcher used a dicot plant for transformation first followed by
crossing between the dicot plant and the rice plant
(4) Golden rice has a higher nutritional value than normal rice
(5) Beside Agrobacterium, the researcher also used another bacterium,
Escherichia coli in construction of transforming vectors.
A. All are correct
B. 1, 2, 4, 5
C. 1, 2, 3
D. 1, 2
E. 1, 3, 4, 5
9. When a DNA fragment under control of a promoter was transformed into tobacco
plants with Ti plasmid, the transgenic plants showed a lower activity of CO2 fixation.
Biochemical examination found that the transgenic plant had a lower amount of
Rubisco, a key enzyme for Calvin cycle. Which of the following is/are likely to be
the reason(s) for the phenotype? (1 point)
(1) The DNA fragment was transformed into chloroplasts and resulted in
interference with chloroplast transcription.
(2) Genetic exchange between the transformed DNA fragment and host
chromosomal DNA resulted in insertion of Ti plasmid into chromosome,
leading to a lower expression of Rubisco genes

(3) The transformed DNA fragment interfered normal transcription of the


gene encoding large subunit of Rubisco
(4) The transformed DNA fragment interfered normal transcription of the
gene encoding small subunit of Rubisco
A. 1, 2, 5
B. 1, 3, 4
C. 1, 4,
D. 4,
E. 3

10. Which of the following is/are true about endosymbiosis? (1 point)


(1) Both plastid and lysosome are products of endosymbiosis
(2) Eukaryotic cells could engulf other eukaryotic cells to establish a
symbiotic relationship
(3) Cyanobacteria are ancestors of plastids and mitochondria
(4) Cyanobacteria lost their chlorophyll b gene in endosymbiosis.
(5) Flagella of some eukaryotic cells are derived from cyanobacteria

A. 1, 3, 5
B. 1, 2
C. 2, 4
D. 2
E. 4

11. Which of the following graphs correctly displays the relationship of blood flow
velocity in humans as the blood flows from the aorta arteries arterioles
capillaries venules veins venae cavae: (1 point)

12. Lowering the level of a hedge with a hedge trimmer stimulates the hedge to become
bushy because: (1 point)
A. It stimulates the production of ethylene gas.
B. Removing the apical meristems makes more auxin, which stimulates lateral
branch buds to grow.
C. Removing the apical meristems makes less ethylene, which stimulates lateral
branches to grow
D. Removing the apical meristems results in less auxin, which then allows lateral
branches to grow.
E. Removing the lateral buds results in apical dominance under the influence of
cytokinins

10

13. Which of the following is/are true about telomeres? (1 point)

(1) Telomeres are present in all DNA in eukaryotic cells


(2) Telomeres are present in bacterial plasmids
(3) Telomeres are required for replication fork formation
(4) Telomeres are specific sequences present in eukaryotic chromosomes
(5) Telomeres are required for maintaining chromosomal length

A. 1, 3, 5,
B. 3, 4, 5
C. 4, 5
D. 2
E. 3
14. For terrestrial and most aquatic environments, neither animal nor plant life could exist
without the metabolic "services" provided by: (1 point)

A.

chemoheterotrophs

B.

extremophile archaeans

C.

Fungi

D.

Homo sapiens

E.

Fertilizer

11

15. The inner ear of humans, and most other mammals, is sensitive to body position and
balance. What organ(s) is/are responsible for this? (1 point)

A.

cochlea

B.

cochlea and basilar membrane

C.

semicircular canals

D.

semicircular canals and cochlea

E.

semicircular canals, utricle, and saccule

16. Flukes are often parasites in or on another animals. They could cause diseases in
human beings. Blood fluke (Schistosoma mansoni) is a parasitic trematode that
infects men. Which one of the following is NOT true about its life cycles. (1 point)

A. There are two types of larvae in the fluke


B. It reproduce asexually in the human host
C. The larvae need water to swim
D. Its infection of human being is through skin
E. An intermediate host is often required for completion of their life cycle.

12

17. In animal behavior a sign stimulus could trigger a fixed action pattern (FAP). Which
of the following is NOT an example of sign stimulus-FAP? (1 point)

A.

Some moths fold their wings and drop to the ground when they detect an
ultrasonic signal from bats.

B.

A wasp finds its nest according to the surrounding objects.

C.

A newly hatched bird cheeping loudly in begging for food when its parent
returns to nest.

D.

Breeding mayflies lay eggs when they detect water.

18. Some birds (eg Gulls) feed on mollusks. The birds grasp the prey and fly upwards to
a certain height before they drop the prey onto a rock to break the shells. If the shell
was not broken by the first drop, the birds will pick it up and drop it again until it is
broken. In one experiment, researchers found the following relationship between the
drop heights and the number of times it was dropped before the shell broke. (1 point)
Height of drop (m)
1
2
3
4
5
6
12

Number of drops required to break shell


67
46
18
6
5
4
3

According to the optimal foraging theory, which of the following is the most likely height
that the birds would fly to drop the shells?

13

A. 6.5 m
B. 4.5 m
C. 2.5 m
D. 3.5 m
E. 12.5 m
19. The figure below shows cytological and biochemical changes of a human infected by
HIV. There are three curves in the figure labeled as 1 through 3. Which of the
following is CORRECT? (1 point)

A. Curve 1 represents viral numbers


Curve 2 represents concentration of antibodies against HIV
Curve 3 represents Tcell mediated immunity

B. Curve 1 represents Tcell mediated immunity


Curve 2 represents concentration of antibodies against HIV
Curve 3 represents viral numbers

14

C. Curve 1 represents Tcell mediated immunity


Curve 2 represents viral numbers
Curve 3 represents concentration of antibodies against HIV

D. Curve 1 represents concentration of antibodies against HIV


Curve 2 represents Tcell mediated immunity
Curve 3 represents viral numbers

E. Curve 1 represents viral numbers


Curve 2 represents Tcell mediated immunity
Curve 3 represents concentration of antibodies against HIV

20. The figure below shows a generalized life cycle of fungi. Which of the following
is/are TRUE? (1 point)

15

(1) Spores are generally haploid


(2) Cycle I is a sexual life cycle and cycle II is an asexual life cycle
(3) Diploid fungi are formed after plasmogamy
(4) There are two types of mycelia that mate even though they may look
alike.

A. 1, 2,
B. 1, 3
C. 1, 4
D. 1, 2, 4
E. 1, 3, 4

Questions 21-24. The hyperthermophilic archeon, Pyrococcus furiosus, has an unusual


phosphofructokinase. It catalyzes the following reaction:
Fructose-6-phosphate + ADP Fructose-1,6-bisphosphate + AMP
It was found that the addition of glucose, pyruvate, phosphoenolpyruvate, citrate and
fructose-2,6-bisphosphate did not show any effect on the reaction rate. The effects of
ATP and AMP addition were shown as Lineweaver-Burk plots:

16

Answer the following questions:


21. Which of the following statements is TRUE ? (1 point)
A. The reaction is ATP-dependent.
B. The reaction is ADP-dependent.
C. The reaction is AMP-dependent
D. None of the above answers are true.

22. What is the effect of ATP or AMP on the reaction rate? (1 point)
A. Only allosteric Stimulation
B. Only allosteric inhibition
C. Only competitive inhibition
D. Only uncompetitive inhibition
E. Mixed inhibition

17

23. Does this phosphofructokinase play an important role in the regulation of glycolysis
in Pyrococcus furiosus? (1 point)
A. Yes
B. No
C. The conclusion cannot be drawn.

24. Pyrococcus furiosus phosphofructokinase was purified and gave a single band at 52
kDa on SDS-polyacrylamide gel electrophoresis. Its native molecular mass was
determined by gel filtration chromatography to be approximately 190 kDa. The protein
is: (1 point)
A. monomer
B. dimer
C. trimer
D. tetramer
E. hexamer

18

25. Match the following names or descriptions to the right biochemical compounds listed.
(2 points)

Answer
[A-G]

1. Nucleoside found in DNA


2. Phospholipid
3. A yeast fermentation product

1.
2.
3.

4. Monosaccharide

4.

5. Iron-sulfur center

5.

19

26. Antibiotics are antimicrobial substances produced by some organisms to prevent


growth of other organisms. Match the following antibiotics as inhibitors to their
cellular targets: (1 point)

A. Cell wall synthesis


B. Plasma membrane formation

Answer
(A-E)

C. DNA replication

1. Polymyxins
2. Tetracycline

D. RNA transcription

3. Rifampin
4. Penicillin

E. Protein translation

5. Mitomycin

27. Glucose labeled with 14C at C-1 is incubated with the glycolytic enzymes and
necessary cofactors. What is the distribution of 14C in the pyruvate that is formed? (1
point)
A. The label is in the methyl carbon atom of pyruvate.
B. The label is in the carboxyl carbon atom of pyruvate.
C. The label is in both the methyl and carboxyl carbon atoms of pyruvate.
D. The label is in the middle carbonyl carbon atom of pyruvate.

20

28. A common component of NADP, NAD, FMN, FAD, and coenzyme A is: (1 point)
A. A pyrimidine ring
B. A three ring structure
C. An ADP
D. Deoxyribose
E. A triphosphate group
29. Which of the following statements is/are CORRECT? (1 point)
(1) The citric acid cycle (Krebs/TCA cycle) does not exist as such in plants and
bacteria, because its functions are performed by the glyoxylate cycle.
(2) The citric acid cycle oxidizes the acetyl CoA derived from fatty acid
degradation.
(3) The citric acid cycle produces most of the CO2 in anaerobic organisms.
(4) The citric acid cycle provides succinyl CoA for the synthesis of carbohydrates.
(5) The citric acid cycle provides carbon skeletons for amino acid synthesis.
A.

1, 2, 5,

B.

3, 5,

C.

2, 4

D.

2, 3,

E.

2, 5

21

30. Key enzymatic differences between liver, kidney, muscle and brain account for their
differences in the utilization of metabolic fuels. Which of the following does NOT
represent such a biochemical difference? (1 point)

A. The liver contains glucose 6-phosphatase, whereas muscle and the brain do not.
Hence muscle and the brain, in contrast with the liver, do not release glucose
into the blood.
B. The liver has little of the transferase needed to activate acetoacetate to
acetoacetyl CoA. Consequently, acetoacetate and 3-hydroxybutyrate are
exported by the liver and be used by heart muscle, skeletal muscle and the brain.
C. Under conditions of prolonged starvation, the fatty acids stored in the adipose
tissues will be converted into ketone bodies there before being transported to the
brain and muscle for complete oxidation.
D. Lactate dehydrogenase does not appear to exist in the heart muscle. As a result,
the heart depends on aerobic oxidation to obtain the energy for its continuous
pumping.

22

31. An organelle in eukaryotic cell is spherical or ovoid with a diameter of 0.1 to 1.5 m
and consists of a single membrane. It participates in a variety of metabolic processes,
including H2O2-based respiration and lipid metabolism. This organelle is most likely to
be? (1 point)
A. Mitochondrion
B. Peroxisome
C. Endoplasmic reticulum
D. Lysosome
E. Endosome

32. A red alga has two major kinds of photosynthetic pigments: phycobilisomes
(phycobilins) that absorb green light and chlorophylls that absorb red and blue light.
A student performed an experiment and obtained measurement data as shown in table.
Note: light intensity remained constant during the experiment.

Light quality

Photosynthetic oxygen evolution rate

Blue light only

28

Green light only

65

Red light only

47

Blue and green

150

Blue and red

73

Green and red

146

23

Which of the following is/are NOT correct? (2 points)


(1) Blue light absorption was less efficient in photosynthetic electron transfer
because the blue light is mostly absorbed by chlorophyll b.
(2) Red light is more efficiently absorbed by chlorophyll than blue light.
(3) An enhancement effect, caused by the simultaneous excitation of the two
photosystems, is observed in this experiment.
(4) It is predicted that more overlapping is present in the long wavelength region
than in the short wavelength region between phycobilisome absorption
spectrum and chlorophyll absorption spectrum.
A. 1, 2, 4
B. 1, 3, 4
C. 3, 4
D. 1, 2
E. 1

24

33. The figure below shows the nitrogen cycle. Complete the table below according
to the information provided. (1 point)

Bacteria:

Answer:
A through E. Note: there could be more
than one correct answer

(1) Able to form nodules with plants


(2) Able to denitrify
(3) Able to nitrify
(4) Able to use ammonium as energy
source
(5) Able to fix nitrogen from air

25

34. A researcher found that seeds from a plant could inhibit the growth of some fungi.
He isolated some substances from the seeds and performed analyses. The figure
below is the result. He also ran a regular SDS-gel electrophoresis that separated
molecular standard proteins from 14 kDa to 100 kDa.

Treatment 1: no addition of the substance.


Treatment 2: addition of the substance.
Treatment 3: addition of the substance that was treated with -mercaptoethanol
(BME)
Treatment 4: same as treatment 3 except that BME was removed before addition of
the substance to the fungal culture.
Treatment 5: the substance was treated at 80C for 20 min before the addition to the
fungal culture.
Treatment 6: the substance was treated at 80C for 20 min in the presence of BME
before the addition to the fungal culture.
Treatment 7: the substance was treated with trypsin.
Treatment 8: only trypsin was added to the fungal culture.

26

He found no protein could be detected in this molecular mass range with Coomassie stain
(a protein stain) even though the substance(s) showed Coomassie binding in solution.
The substance(s) is colourless, but had a strong absorption in the UV region. Which of
the following is/are correct? (2 point)

(1) The substance(s) contains protein


(2) The substance(s) has a disulphide bond that is important to its function
(3) The substance(s) is stained poorly with Coomassie blue
(4) The substance(s) is a protein with molecular mass smaller than 14 kDa.
(5) The substance(s) is not resistant to trypsin treatment.

A. 1, 2, 3, 4, 5
B. 1, 2, 4
C. 1, 3, 4
D. 1, 4
E. 1, 5

35.

Calculate the pI (isoelectric point) value of aspartic acid. Its pK1 is 2.09, pK2 is
3.86, pK3 is 9.82. (1 point)
A. 5.26
B. 2.98
C. 5.96
D. 6.84

27

36. When we refer to a plant as a short-day plant, the exact meaning is: (1 point)

A. The plant flowers in winter


B. The plant flowers when day is shorter than 12 hours
C. The plant flowers only in the equator area
D. The plant flowers when night is longer than its own critical night length
E. A and D

37. Which of the following is the photoreceptor that responds to day-length? (1 point)
A.

Chlorophyll

B.

Carotenoids

C.

Cytochrome

D.

Phytochrome

E.

Retinal

38. Which of the following statements is correct? (1 point)

A.

A flower is a reproductive organ

B.

A flower lacking any of sepal, petal, stamen or carpel is an imperfect flower

C.

Most grasses have imperfect flowers

D.

Floral parts in all angiosperms are arranged as four whorls

E.

Floral parts are sequentially initiated at the floral meristem

28

39. One of the means to prevent self-fertilization in plants is self-incompatibility. Which


of the following statements is/are TRUE about self-incompatibility? (1 point)
(1)

The plants that show self-incompatibility have a unique stigma structure.

(2)

The flowers of the plants that show self-incompatibility only produce


pollen when stigmas fail to develop.

(3)

Self-incompatibility is analogous to the animal immune response in that


both have the ability to distinguish the cells of self from those
of nonself.

(4)

Pollen from one plant will only develop a pollen tube on its own stigma if
a pollen from another plant is present on the stigma.

(5)

Pollen from one plant will develop a pollen tube on its own stigma, but
will not be able to fertilize the egg.
A. 1, 2
B. 3, 4, 5
C. 4, 5
D. 3
E. 3, 5

29

40. Where do you find cells that undergo meiosis in plants? (1 point)
A. In the shoot apical meristem
B. In the pollen
C. In embryo sacs
D. In the corolla
E. In the ovule
41. Which of the following structures of plants consists of haploid cells? (1 point)
A. Sporophytes
B. Sporocytes
C. Sporangia
D. Tapetum
E. Gametophyte
Questions 42-45. Algae play very important roles in ecosystems. They are also diverse
in pigmentation.
42. Red algae differ from green algae and brown algae in that (1 point)
A. Red algae produce agar
B. Red algae do not produce chlorophyll a
C. Red algae do not have sexual reproduction
D. No unicellular red alga has been found
E. Red algae do not produce flagellated cells in their life cycle.

30

43. Dinoflagellates are a group of algae. Their pigments are similar to brown algae.
Therefore, the pigments of a typical dinoflagellate are similar to: (1 point)

A. Pigments of Chlamydomonas
B. Pigments of Volvox
C. Pigments of a diatom
D. Pigments of a red alga
E. Pigments of blue-green algae

44. According to their pigmentation, which algal group would be most likely to perform
photosynthesis in deepest water? (1 point)

A.

Red algae

B.

Green Algae

C.

Brown algae

D.

Golden algae

31

45.

Seaweeds are large marine algae and they play very important role in marine
ecosystems. Which of the following is/are NOT true about seaweeds? (1 point)

(1) Most seaweeds are brown algae.


(2) Diatoms can sometimes be large enough to be included as seaweeds.
(3) Seaweeds have complicated structures such as leaves.
(4) Seaweeds live in deep water
(5) They use their holdfast to absorb nutrients

A. 1, 2, 3, 4,
B. 2, 3, 4, 5,
C. 1, 3, 4, 5,
D. 1, 2, 4, 5
E. 1, 2, 3, 4, 5
46.

Apoptosis was first described in nematodes and was later found to be present in
many organisms. Which of the following is NOT true about apoptosis? (1 point)
A. It was discovered by cell lineage analysis of nematodes
B. It is a critical process in animal development.
C. It is controlled by a single gene
D. It is found in insects
E. Proteases and nucleases participate in apoptosis.

32

47.

After synthesis, proteins are transported either by non-vesicular transport or by


vesicular transport. Answer A for vesicular or B for Non-vesicular to indicate
how each of the proteins indicated in the table is transported. (0.2x9, 1.8 points)
Proteins:

Answer A or B

1. cytoskeletal proteins
2. Mitchondrial proteins
3. Lysosomal proteins
4. Nuclear proteins
5. Cytoplasmic enzymes
6. integral plasma membrane
(cell surface) proteins
7. secreted proteins
8. Chloroplast proteins
9. Peroxisomal protein

48.

An action potential in neurons is characterized by all of the following except that (1


point)
A.

It is initiated by opening of voltage-gated potassium channels

B.

It is regarded as a regenerative response

C.

It is regarded as an all-or-nothing response

D.

It does not reduce in magnitude with space or time

E.

It is characteristic of transmembrane potential changes that occur in


most axons.

33

49. The resting potential in most neurons is primarily due to the membrane permeability
of (1 point)
A. Calcium
B. Chloride
C. Sodium
D. Potassium
E. Magnesium
50. Which of the following cell cycle phases is usually the shortest in duration? (1 point)
A. G1
B. G0
C. G2
D. S
E. M

34

51. Which of the following is/are often used for protein purification? (1 point)
(1) Gel filtration chromatography
(2) Ion exchange chromatography
(3) Salt precipitation
(4) SDS-electrophoresis
(5) Substrate affinity chromatography
A. all of the above
B. 1, 2, 3, 4,
C. 1, 2, 4, 5
D. 1, 2, 3, 5
E. 2, 3, 4, 5

52. Which of the following is/are important in ATP synthesis? (1 point)


(1) P700
(2) P680
(3) P450
A. 1
B. 2
C. 3
D. 1, 2
E. 1, 2, 3

35

53. Which of the following statements about mRNA is correct? (1 point)


(1) All mRNA has a cap at its 5 end
(2) All mRNA has a poly A tail at its 3 end
(3) Its synthesis is performed by RNA polymerase
(4) The stability of mRNA regulates abundance of the protein it encodes.
(5) The codons on mRNA pair with anti-codons of tRNA through A-T, G-C
hydrogen bonds

A. 1, 2, 3, 4,
B. 3, 4, 5,
C. 1, 2,
D. 3, 4
E. 3
54. Which of the following about tRNA is/are correct? (1 point)
(1) There are stem-loop structures
(2) It consumes ATP in synthesis of aminoacyl tRNA
(3) tRNA is synthesized by RNA polymerase III
(4) tRNA is synthesized as a precursor and was processed before it is
functional.
(5) Although the theoretical number of tRNA molecules is 61, the actually
number of tRNA molecules in most of the cell is smaller, partially because
some anticodons can recognize more than one codon.

36

A. 1, 2, 3
B. 1, 2, 4
C. 1, 2, 5
D. 1, 2, 3, 4, 5,
E. 2, 3, 4, 5

55. Which of the following is/are NOT true about the Freeze-fracture method in electron
microscopy? (1 point)
(1) Low temperature is used to weaken hydrogen bonding.
(2) It is often used to observe structures within membrane.
(3) Particles observed on fractured faces are often liposomes
(4) Both eukaryotic and prokaryotic cells can be observed with this
method
(5) This method actually observes a replica of the specimen.

A. 1, 3
B. 2
C. 3, 4,
D. 4,5
E. 3

37

Questions 56-57. Yeast is one of the ideal organisms for the study of cellular,
developmental and genetic processes. It can grow either on fermentable or nonfermentable carbon sources. With this property, people can isolate and analyze different
yeast mutants associated with certain functions of subcellular organelles.
56. When the yeast mutant cannot grow on oleate (a long chain fatty acid), the mutant has
a defect in which organelle? (1 point)
A. Mitochondria
B. Lysosome
C. Peroxisome
D. Nucleus
E. Endoplasmic reticulum
57. When a yeast mutant cannot grow on glycerol, the mutant has a defect in which
organelle? (1 point)
A. Mitochondria
B. Lysosome
C. Peroxisome
D. Nucleus
E. Endoplasmic reticulum

38

58. Which of the following is NOT a factor influencing membrane fluidity? (1 point)
A. Number of double bonds in the lipids
B. Temperature
C. Flip-flop movement of lipids
D. Cholesterol
Questions 59-61 are about food digestion in the mammalian digestive system.
59. Which of the following is NOT involved directly in protein digestion? (1 point)
A. Trypsin
B. Dipeptidase
C. Aminopeptidase
D. Carboxypeptidase
E. Enteropeptidase
60. Which of the following enzymes is NOT functionally present in small intestine? (1
point)
A. Nucleases
B. Lipase
C. Chymotrypsin
D. Pancreatic amylases
E. Pepsin

39

61. Many hormones are involved in food digestion and absorption. Please match the
functions to the appropriate hormones. (1 point)
A. Regulation of blood sugar
B. Stimulation of bicarbonate release
C. Stimulation of the gallbladder to contract and release bile
D. Stimulation of secretion of gastric juice.
Hormones

Fill your answer

1. Cholecystokinin
(CCK)
2. Gastrin
3. Secretin
4. Insulin

Questions 62-63. Flowering is one of the most sophisticated processes in plants. By


analysis of flowering mutants and through other studies, researchers proposed an ABC
model (hypothesis) to explain gene regulation of flower structures. Three classes of
genes are involved: class A, class B and class C.

40

In this model, a sepal is produced when gene A is active, a petal is produced when genes
A and B are active; Stamen is produced when genes B and C are active and a carpel is
produced when gene C is active. When gene A is missing, gene C takes its place and
when gene C is missing, gene A takes gene Cs place.
62. According to the ABC model, which of the following mutants will produce the
phenotype shown below? (1 point)

41

A. A mutant lacking gene A


B. A mutant lacking gene B
C. A mutant lacking gene C
D. A mutant lacking genes A and B
E. A mutant lacking genes B and C
63. It has been demonstrated that Genes A, B and C encode transcription factors. Which
of the following is NOT a property of transcription factors? (1 point)
A. DNA-binding
B. Interaction with other proteins
C. Degradation by protease
D. RNA binding
E. Participation of other gene regulation

64. PCR (polymerase chain reaction) is one of the most powerful methods in molecular
biology. Which of the following is/are NOT true about PCR? (1 point)
(1) Primers are needed in PCR
(2) A DNA polymerase that can tolerate high temperature is needed in PCR
(3) ATP is needed in PCR
(4) A DNA template is needed in PCR

42

A. 1, 2
B. 2, 3
C. 3
D. 1, 3,
E. 2, 4
65. Nitrogenous wastes of animals are released to their environments in different
forms. Which of the following statements is/are true about animal nitrogenous wastes?
(1 point)
(1) Urea is excreted by many marine fishes.
(2) Ammonia is so toxic that it is rarely excreted as nitrogenous waste by any
animals
(3) The animals in dry environments could excrete uric acid
(4) The form of nitrogenous waste is often an adaptation to animal habitats.
A. 1, 2, 3, 4
B. 1, 4,
C. 1, 2, 4
D. 3, 4
E. 1, 3, 4

43

66. Among the nitrogenous wastes, urea, uric acid and ammonia the order of toxicity
is: (1 point)
A. Ammonia > uric acid > urea
B. Urea > ammonia > uric acid
C. Uric acid > urea > ammonia
D. Ammonia > urea > uric acid
E. Urea > ammonia > uric acid
F. Uric acid > urea > ammonia

Question 67-69. Equilibrium dialysis is a method often used to determine the


dissociation constant KD for a ligand-binding protein. In this method, a protein at a
known concentration is put into several dialysis tubes and each dialysis tube
containing the protein is dialyzed against solutions containing the ligand at various
ligand concentrations. Because the protein cannot move across the dialysis tube
membrane while the ligand can, the ligand is trapped by the protein inside the
dialysis tube and it creates a higher concentration of the ligand in the dialysis tube
than that outside the dialysis tube. The dissociation constant of the ligand can thus be
determined according to the following formula:

44

Where [M] is the concentration of free protein (no bound ligand) in the dialysis
tube, [L] is the concentration of the ligand and [ML] is the concentration of the
protein with bound ligand. Therefore, KD is the ligand concentration when [M]
equals [ML]. [MT] = [M] + [ML].
Where [MT] is the total concentration of the protein
The table below shows the measurement results of a calcium-binding protein.
The protein has a molecular mass of 20 kDa and the concentration of the protein
in equilibrium dialysis is 1 mg.ml-1.

45

Calcium concentration in

Calcium concentration

dialysis solution (M)

in dialysis tube (M)

20

30

50

68

100

129

200

237

400

442

600

647

1000

1050

1500

1548

2000

2049

[M]/[MT]

Please calculate the values of [M]/[MT] at each concentration and plot the data
(Calcium concentration in solution vs [M]/[MT]) on the plotting (graph) paper
shown below.

46

67. How many calcium ions does one protein molecule bind? (1 point)
A. 1
B. 2
C. 3
D. 4
E. It cannot be determined

47

68. What is the KD of the protein? (3 point)


A. 30 M
B. 78 M
C. 95 M
D. 104 M
E. 200 M
69. There are two calcium-binding proteins, Protein X and Protein Y. If Protein X
has a KD of 250 nM and Protein Y has a KD of 400 nM, which of the following
is/are NOT correct? (2 point)
(1) Protein X binds calcium more tightly than Protein Y.
(2) Half of Protein Y will have bound calcium at the concentration of 400 nM.
(3) It is more difficult to release the bound calcium from Protein Y.
(4) When Protein X and Protein Y are mixed at equal molar concentration,
more Protein X will have bound calcium than Protein Y at a calcium
concentration of 250 nM.
(5) When Protein X and Protein Y are mixed at equal molar concentration,
equal amounts of Protein X and Protein Y will have bound calcium at a
calcium concentration of 400 nM.

48

A. 1, 2, 5
B. 2, 4,
C. 3, 4
D. 4, 5
E. 3, 5
70. Which of the following are amniotes? (1 point)
(1) Bony fishes (Osteichthyes)
(2) Reptiles (Reptilia)
(3) Cartilagenous fish (Chondrichthyes)
(4) Jawless fish (Agnatha)
(5) Mammals (Mammalia)
(6) Amphibians (Amphibia)
(7) Birds (Aves)

A. 1, 4, 6, 7
B. 2, 3, 5
C. 2, 5, 7
D. 2, 4, 5, 6
E. 2, 5, 6, 7
F. 4, 5, 6, 7
G. 5, 6, 7

49

71. The figure below shows schematic structures of an amniotic egg. Please name the
structures labelled by numbers 1 through 7. (1 point)
A. amnion B. embryo C. allantois D. chorion. E. yolk sac. F. gut G. allantois cavity

Answer
A-G
1
2
3
4
5
6
7

72. Fill in the appropriate answers based on the functions of the structures shown in the
figure above (question 71). (1 point)

Main Function

Answer:
A-G

(1) It protects the embryo in a fluidfilled cavity that prevents dehydration.


(2) It provides nutrients for the embryo
(3) It functions as a disposal cavity for
metabolic wastes
(4) It is rich in blood vessels and it
forms a sac for collecting waste

50

Questions 73-74. Compare 4 different invertebrates:


(1) Spider
(2) Grasshopper
(3) Millipedes
(4) Shrimp
73. Fill in the appropriate answers according to the following descriptions. (1 point)
A. 1 pair of antennae, 3 pairs of legs
B. 1 pair of antennae, more than 4 pairs of legs
C. 2 pairs of antennae, 4 pairs of legs
D. 2 pairs of antennae, more than 4 pairs of legs
E. No antennae, 3 pairs of legs

Answer: A-F
(1)
(2)
(3)
(4)

F. No antennae, more than 3 pairs of legs

74. Fill in appropriate answers according to organs of excretion and gas exchange.
(1 point)
A. Excretion with Malpighian tubules and gas exchange with tracheal system.
B. Excretion with Malpighian tubules and coxal gland, gas exchange with tracheal
system.
C. Excretion with maxillary gland and gas exchange with gill and tracheal system.
D. Excretion with Malpighian tubules and coxal gland, gas exchange with book lung
E. Excretion with Malpighian tubules and coxal gland, gas exchange with book lung
and tracheal system
F. Excretion with maxillary gland and gas exchange with gill.

Answer
A-F
(1)
(2)
(3)
(4)
51

Questions 75-84. The figure below is a diagram of the ultra-structure of a cell..

75. If you are provided with two electron microscopic pictures, one from pancreas gland
cells and the other from endothelial cells from the proximal tubule of a kidney
nephron. Which of the structure shown in the figure will be more developed in the
pancreas gland cell? (1 point)

Answer: choose one from A through G.

52

76. As the cells grow, the surface area of each cell increases. Which structure is the
location where the lipids are synthesized for plasma membrane (cell surface)
synthesis? (1 point)

Answer: choose one from A through G.

77. If you treat the cells for a short period of time with 3H-Uracil followed by detecting
the labelled cellular structure with autoradiography, which structure will have the
highest concentration of silver grains (strongest labelling)? (1 point)

Answer: choose one from A through G.

78. Which structure is assembled in the nucleus and then transported to cytoplasm?
(1 point)

Answer: choose one from A through G.

79. Erythropoietin (EPO) is hormone that stimulates production of erythrocytes. EPO is


a highly glycosylated protein which can be secreted. Which structure would be
responsible for the initial synthesis of EPO? (1 point)

Answer: choose one from A through G.

53

80. Which structure would be the site for initial glycosylation of EPO? (1 point)
Answer: choose one from A through G.

81. Which structure would be the site for final glycosylation of EPO? (1 point)

Answer: choose one from A through G.

82. Which structure is essential for the transport of EPO inside the cell? (1 point)

Answer: choose one from A through G.

83. The receptor for EPO is a membrane bound protein. Which structure is responsible
for EPOs receptor synthesis? (1 point)

Answer: choose one from A through G.

84. Which structure has the ability to synthesize some proteins that are not encoded by
nucleus. (1 point)

Answer: choose one from A through G.

54

END of PART I

55

16th International Biology Olympiad


Beijing
July, 2005

THEORY EXAMINATION
Part 2

Total time available: 2.5 hours (150 minutes)

Total points available: ~80


There is only one correct option for each question: No negative marking in place

56

Questions 85-92. Sex determination in fruit flies and mammals are both XY type, that is,
XX leads to female and XY leads to male.

85. Some organisms have abnormal sex chromosomes such as XO (only have one X
chromosome) or XXY (extra X chromosome). The most likely cause of the
abnormal sex chromosome is: (1 point)
A. An error occurred in mitosis of the fertilized egg.
B. Gene mutation
C. An error occurred in meiosis during gamete formation.
D. Sex chromosomes in gametes are either lost or doubled in fertilization.
86. In organisms with XXY chromosome type, there is an extra X chromosome. How do
you most conveniently determine if this X chromosome is from sperm or egg? (1 point)

A. Karyotype
B. In situ hybridization
C. RFLP (Restriction fragment length polymorphism)
D. DNA sequencing

57

87. In mammals, XO leads to female and XXY leads to male. In fruit flies, XO leads to
male and XXY leads to female. Which of the following is NOT correct? (1 point)

A.

The Y chromosome in mammals is necessary for formation of a male


organism.

B.

The Y chromosome in mammals is required for the sex organ


development.

C.

The Y chromosome in fruit flies is not functional.

D.

The number of X chromosomes in fruit flies impacts on sex


determination.

88. In mammals with abnormal sex chromosomes, the number of individuals with XO
chromosome type is far fewer than the number of individuals with the XXY
chromosome type. It is therefore predicted that: (1 point)

A. The individuals with the XO chromosome type are less capable of surviving than
those with the XXY chromosome type.
B. The individuals with XO chromosome type are less capable of reproducing than
that with XXY chromosome type.
C. The difference is related to gender of the individuals (XO leads to female and
XXY leads to male).
D. None of the above.

58

89. In both fruit flies and mammals, XX leads to female and XY leads to male. The gene
products encoded by two X chromosomes of female individuals are nearly identical to
those encoded by one X chromosome of male individuals. This is accomplished by
gene dosage compensation. In mammals, it is accomplished by converting one X
chromosome into a Barr body (X inactivation). Which of the following about the Barr
body is/are correct? (1 point)
(1)

Only normal female individuals have Barr bodies.

(2)

Only normal male individuals dont have Barr bodies.

(3)

Barr bodies can always be used to determine gender of human beings.

(4)

The maximum number of Barr bodies is one

(5)

The number of Barr bodies equals the number of X chromosomes


minus one.
A. 1, 3, 5
B. 2, 5
C. 4
D. 5
E. 1, 4, 5

90. No Barr body can be observed in normal female fruit flies because (1 point)

A. The X chromosome of fruit flies is too small


B. There is no mechanism of dosage compensation in fruit flies
C. There is no X inactivation in fruit flies
D. Heterochromatin is difficult to detect in fruit flies.

59

91. The fur color of cats is determined by genes on the X chromosome. XA is the
dominant allele for orange fur, while Xa is the recessive allele for black fur. Which of the
following is true about the fur color of the offsprings from a XAXa female cat and XAY
male cat? (1 point)

A. They are all orange


B. All the female are orange and half the male are orange
C. Regardless of sex, half are orange, the other half have fur that is a mosaic of
orange and black.
D. Those with mosaic fur are all female.

92. One of the genes controlling sweat glands in humans is located on the X chromosome.
Two twin sisters show different phenotypes of the sweat gland. One has no sweat
glands on her left arm while the other has sweat glands on her left arm. Which of the
following statements is/are TRUE? (1 point)

(1) The twins cannot be identical.


(2) They both are heterozygous for the gene.
(3) The reason for the different phenotype is random X inactivation.
(4) X inactivation must occur after the first division of the zygote.

60

A. 1, 2, 3, 4
B. 1
C. 2, 3
D. 3
E. 2, 3, 4
93. Mycorrhizae are symbiotic associations of fungi and plant roots. Which of the
following is/are TRUE about mycorrhizae? (1point)

(1) They are often harmful to plant roots while beneficial to fungi.
(2) They are often beneficial to plants but harmful to fungi
(3) They are helpful for plants to absorb water and minerals.
(4) They could even help the older root region above the root hair area to supply
minerals to plants.
A. 1, 3, 4
B. 2, 3, 4
C. 3, 4
D. 3

61

94. Complete the following sentence: Stomata of a plant open when guard cells (1point)
A. accumulate water by active transport.
B. sense an increase in CO2 in the air spaces of the leaf.
C. become more turgid because of an influx of K+, followed by the osmotic entry of
water.
D. sense that water content of the whole plant is low.

95. Which of the following processes of plants could be regulated by phytochrome?


(1point)
(1) seed germination
(2) flowering
(3) shoot elongation
(4) open and closure of stomata
A. 1, 2, 3, 4
B. 1, 2, 3
C. 1, 2
D. 1

96. If N represents population size, r represents the difference in per capita birth rates and
death rates, K represents the carrying capacity, t represents time, which of the
following equations best describes logarithmic growth of the population? (1point)

62

97. Which of the following is usually the limiting process of phosphorous cycles? (1point)
A. Decomposition
B. Utilization in primary production
C. Release from soil
D. Sedimentation

98. Which of the following ecosystems has the lowest primary production per square
meter? (1point)
A. a salt marsh
B. an open ocean
C. a grassland
D. a tropical rain forest

63

99. Which of the following is/are true about Archaea and Eubacteria? (1point)
(1) They dont have a nuclear envelope
(2) They both have branched chains in membrane lipids
(3) They have one kind of RNA polymerase
(4) They have circular chromosomes.
A. 1, 2, 4
B. 1, 4
C. 2, 3
D. 1, 2, 3

100. Four major groups of fungi are recognized. They are chytrids, zygote fungi, sac
fungi and club fungi. Chytrids differ from other three groups in that (1point)
A. Chytrids dont have sexual reproduction
B. They are all aquatic.
C. They have cell walls made of cellulose
D. They have flagellated cells in their life cycles.

64

101. Chlorophyll a is involved in both light energy absorption and initial electron transfer
of photosynthesis. Which of the following are true about the chlorophyll a? (1point)

(1) The position of chlorophyll a in photosystems has a strong influence on the


function of chlorophyll a.
(2) Chlorophyll a in the photosynthetic reaction center is chemically modified so that
it performs initial electron transfer.
(3) Part of chlorophyll a is structurally related to the heam group found in hemoglobin.
(4) Part of chlorophyll a is structurally related to carotenoids.
A. 1, 2, 3, 4
B. 1, 3
C. 3, 4
D. 1, 2

102. In measurement of photosynthetic electron transfer, intact chloroplasts are isolated


and used to estimate electron transfer rates under different conditions. Which of the
following is CORRECT? (1point)

A. Addition of an uncoupler leads to an increased rate of electron transfer.


B. Cyclic electron transfer starts only when linear electron transfer is inhibited.
C. ATP synthesis could only be observed with continuous light illumination.
D. Oxygen evolution by chloroplast suspension is absolutely dependent upon the
presence of CO2.

65

103. The figure shown below is a diagram of an evolutionary tree. Which of the
following statements about evolution are true and deducible from the figure? (2 points)

(1) All eucaryotic cells contain mitochondria.


(2) Symbiosis of the eucaryotic ancestor with autotrophic cells preceded the symbiosis
with the cell taking advantage of the oxidative metabolism.
(3) There is a common ancestor of eubacteria and eukaryota, archaebacteria are a group
with unique and independent origin.
(4) The ancestral eukaryote was anaerobic.
(5) None of the recent photosynthetic bacteria are related to the chloroplasts.
PTO

66

(6) Mitochondria and chloroplasts have similar genomes.


(7) Mitochondria are present in the cells of the plants, animals and fungi.
(8) Fungi lost chloroplasts during evolution.

(9) Bacteria are a highly homogenous group of organisms which showed rapid
diversification of their genomes and metabolisms during the last billion years.
(10) Chloroplasts and mitochondria are results of independent endosymbiotic events.
A. 1, 2, 5
B. 3, 4, 7
C. 4, 7, 10
D. 6, 8, 10
E. 4, 9, 10

67

104. The figure shown below is an image of a DNA molecule. Structure of the DNA
molecule can undergo dramatic and highly regulated changes during the cell cycle.
Which of the following statements are true about the cross-like structure on the image.
(2 points)

(1)

During the replication all four DNA strands in the double helix are covalently
interconnected.

(2)

In the prophase of the mitosis chromosomes highly condense and get interconnected
via covalent bonds.

(3)

During the prophase of the first meiotic division recombination between sister
chromatids takes place and new covalent bonds are temporarily formed which results
in the formation of the cross-like conformation of the DNA.

(4)

Image is photomontage of the forbidden conformation of the DNA molecule.

68

(5)

During the prophase of the first meiotic division recombination between homologous
chromosomes takes place and new covalent bonds are temporarily formed which
results in the cross-like conformation of the DNA.

(6)

Cross-like structures of the DNA molecules could be observed in the nucleus of the
B-cells and T-lymphocytes during their development.

(7)

Figure shows situation in the cytosol of the bacterial cell where translation and
transcription are not separated spatially.

(8)

Some viruses use formation of the cross-like structures to integrate into the host
chromosomes.

(9)

In the apoptotic cells DNA is cleaved and finally forms unusual cross-like
conformation - useful marker of the final stages of the programmed cell death.

(10) Figure shows unusual type of the replication in the Archaebacteria when three double
helixes are formed from one precursor DNA double helix.
A. 5, 6, 8
B. 1, 3, 8
C. 6, 8, 10
D. 2, 7, 9
E. 4, 6, 10

69

105. The Siamese cat is an example of an animal with melanin synthesized in both sexes
mostly at the body extremities. That makes snout, ears, tail and feet much darker
than the rest of the body. Explanation of this type of the body coloration is that:
(1 point)

A. Only at the body extremities the enzyme tyrosinase (responsible for the synthesis
of the melanin) is synthesized.
B. The only places where one of the X chromosomes that have the dominant gene of
the tyrosinase is NOT inactivated
C. Melanin is synthesized only in the colder parts of the body because the Siamese
cat has a temperature sensitive gene for the enzyme producing melanin.
D. Melanocytes are localized only at the snout, ears, tail and feet the rest of the
body lacks melanocytes.
E. The body extremities are more exposed to the UV-radiation which stimulates
production of the melanin.

70

106. Retinoblastoma (Rb) and p53 genes are examples of anti-oncogenes (tumour
suppressors). Which of the following statements is TRUE? (1 point)
A. Mutation in the p53 gene (when p53 lost its regulatory function) can stop the cell
cycle.
B. Overproduction of the Rb protein in the retina can cause cancer.
C. Cells with a mutated p53 gene are predisposed to malignancy.
D. Cells with a mutated Rb gene are resistant to malignancy.
E. Various viruses incorporated homologs of the p53 and Rb genes into their genomes
and use these proteins for the transformation of the host cell.

107. The extracellular matrix is responsible for the mechanoelastical properties of the
tissues. Which of the following molecules is NOT a component of the extracellular
matrix: (1 point)
A. elastin
B. cytokeratin
C. laminin
D. collagen
E. chondroitin sulphate

71

108. Prions are unique infectious agents formed only from protein called PrP. What are
the true statements about prions? (1 point)

(1)

prion protein has an exceptionally stable conformation

(2)

mutated form of the PrP can predispose to Creutzfedt-Jacob disease in human

(3)

wt form of prion protein is expressed in the brains of the healthy animals

(4)

spongiform encephalopathy is an typical phenotype of the prion caused


disease

(5)

prion disease are restricted only man, cow and sheep - because only these
species express PrP

(6)

prions are small viruses with symmetrical capsid without DNA or RNA

(7)

prion disease is highly infectious and could be transmitted via body fluids

(8)

prion disease could be transmitted via transplantation or cannibalism

(9)

mouse with genetic knock-out for the PrP is resistant to the prion disease

(10)

prion disease could be cured by the bone marrow transplantation

A. 1, 4, 6, 7
B. 2, 3, 4, 5
C. 2, 3, 8, 9
D. 4, 6, 8, 9
E. 1, 3, 9, 10

72

109. Algae were supplied with a radioactive isotope of Carbon, 14C, and allowed to
photosynthesise. After a period of time, the light was switched off and the algae
were left in the dark. The graph shows the relative amount of some radioactive
labelled compounds over the period of the experiment. (1 point)

Which line represents the amount of glycerate 3-phosphate (3GP), ribulose biphosphate
(RuBP) and sucrose formed? (1 point)
Fill out the correct letter of the line in the correct box.

Compound

Line

(1) 3GP
(2) RuBP
(3) Sucrose

73

110. Methylene blue acts as a hydrogen acceptor. It is blue in its oxidised state, but goes
colourless when it is reduced by accepting hydrogen atoms. (1 point)
Methylene blue + hydrogen reduced methylene blue
(blue)
(colorless)
A student wishes to investigate this reaction, he prepares four test tubes as shown below:

Tube A

Tube B

Tube C

Tube D

2 ml

2 ml

2 ml

Glucose solution

2 ml

2 ml

2 ml

Methylene blue solution

1 ml

1 ml

1 ml

Yeast solution

2 ml

2 ml

2 ml

Distilled water

All tubes were incubated at a temperature of 30 C. The colour was recorded at the start
and after intervals of 5 and 15 minutes. The results are shown in the following table.

Colour of content
At start

Tube A

Tube B

Tube C

Tube D

Blue

Blue

Blue

colourless

After 5 minutes

colourless

Blue

Blue

colourless

After 15 minutes

colourless

Blue

Pale blue

colourless

Which test tube can be characterized as a control in this investigation and which test tube
generates an irrelevant (useless) result? (1 point)
Fill out the correct letter below

74

Tube
(1) Control
(2) Useless

111. Morgan crossed Drosophila of two known genotypes, BbVv x bbvv, where B, the
wild-type (grey) body, is dominant over b (black body) and V (wild-type wing) is
dominant over v (vestigial, a very small wing). Morgan expected to see four
phenotypes in a ratio 1:1:1:1. But he observed:
Wild type:

965

Black vestigial:

944

Grey vestigial:

206

Black normal:

185

These results were explained by assuming linkage of alleles together with genetic
recombination (crossing over).
In this particular example the recombinant frequency (defined as the ratio of
recombinants in relation to the total offspring) is: (1 point)
A. 0.205
B. 0.170
C. 0.108
D. 0.900
E. 0.080

75

112. 70% of the population of Beijing is able to taste phenylthiocarbamide. The ability to
taste (T, taster) is dominant over the inability to taste (t, non-taster).
What percentage of the offspring of 'tasters' will be non-tasters? (2 points)
A. 25%
B. 15%
C. 13%
D. 20%
E. 7.5%
Questions 113-116. Wild type individuals of Drosophila have red eyes and strawcoloured bodies. A recessive allele of a single gene in Drosophila causes glass eye and a
recessive allele of a different gene causes ebony body.
A student crosses pure breeding wild type flies with pure breeding flies having glass eye
and ebony body and the resulting F1 flies showed all the wild type phenotype for both
features. On crossing the F1 flies among themselves the student expect a 9:3:3:1 ratio but
the results are not like that. The actual offspring showed:

Eye

Body

Number of flies in F2

Wild

Wild

164

Wild

Ebony

37

Glass

Wild

59

Glass

Ebony

28

76

There are two possibilities:


-

The differences from 9:3:3:1 are coincidental (null hypothesis accepted).

The differences do not occur by coincidence (null hypothesis rejected).

You are required to check this applying the 2 (chi square) test.

For this situation, e.g. degree of freedom, the following diagram with 2 values should be
used:

77

Question 113. The calculated 2 is? (3 points)


A. 10.11
B. 2.84
C. 14.33
D. 11.40
Question 114. Indicate the degree of freedom (df) for this test: (1 point)
A. 2
B. 3
C. 4

Question 115. Determine the probability that the deviation of the observed results from
expected results is due to chance alone. (1 point)
A. About 1%
B. About 2%
C. About 5%
D. About 8%

78

Question 116. To explain the observed deviation of the 9:3:3:1 ratio the student suggested
some possibilities.
(1) linkage of both the alleles
(2) crossing over
(3) incomplete dominance
Which combination of suggestions is the correct explanation? (1 point)
A. 1, 2
B. 1, 3
C. 2, 3
D. 1, 2, 3
117. Which of the following diagram shows the correct representation of the urea content
in the urine of a person on hunger strike, who then died. (1 point)

79

118. Wilhelm von Osten gave performances with his horse called smart Hans. He stated
that he taught his horse to make calculations. But in fact this isnt true at all. He had
taught the horse to respond to his hidden but triggering signals. As a result the
horse made the desired movements: swinging the correct number of times with his
foreleg. After that the horse got some reward.
What kind of learning behaviour is this? (1 point)

A. adaptation
B. conditioning
C. habituation
D. imitation
E. imprinting
F. insight
G. Fixed action pattern

80

119. A snail crawling across a board will withdraw into its shell when you drop a marble
on the board. Repetition of dropping marble will lead to a weaker withdraw action
and in the end the snail will ignore the marble dropping. Which of the following
terms do apply for the disappearance of the withdraw action? (1 point)
(1) adaptation
(2) conditioning
(3) habituation
(4) imprinting
(5) insight
(6) learned behaviour
(7) ritualisation
(8) trial and error
A. 1, 3
B. 2, 4
C. 3, 6
D. 4, 5
E. 5, 6

81

120. Bonsai trees need water with a very low lime content. Which types of water could
be used to water them? (1 point)
(1)

Carbonated mineral water

(2)

Rain water

(3)

Tap water with high water hardness

(4)

Tap water with high water hardness treated by leaving it over night with a
mix of peat and crushed stones and filtrating it before use

(5)

Molten snow

1, 5

2, 5

1, 3

4, 5

2, 4, 5

121. Observe the diagrams 1 to 4 representing cross sections of the ovaries of different
flowers.
Match the numbers in front of the placentation type (A-D) with the corresponding
diagram.

82

A. Axile placentation.
B. free central placentation.
C. Marginal placentation.
D. Parietal placentation.
Match the number with correct placenta type. (1 point)
type

Answer

1
2
3
4

83

122. Which curve shows the correct time course of the production of saliva in a human
after the intake of citric acid? (1 point)

Level of
Saliva
Produced

D
E
Time

Questions 123-125. The behavior of eight Humboldt penguins (Spheniscus humboldti) is


investigated in a larger group of penguins in a zoo enclosure. The animals can be
distinguished by the marks or their individual pattern of black dots on their white thorax.
To document the relationship between penguins, their nearest neighbor (closest animal in
the enclosure) was recorded in short time intervals during day time in a period of several
weeks. The table shows the relatively stable mean values for the frequency of neighbors
for the four male (M1 M4) and four female (F1 F4) penguins.

84

M1
M1
M2
M3
M4
F1
F2
F3
F4

2
5
1
0
3
7
77
95

M2
2
0
9
9
75
1
2
98

M3
5
0
0
0
0
78
6
89

M4
1
9
0
80
8
0
0
98

F1
0
9
0
80
7
0
0
96

F2
3
75
0
8
7
0
0
93

F3
7
1
78
0
0
0
7
93

F4
77
2
6
0
0
0
7

95
98
89
98
96
93
93
92

92

Several months later the same animals were observed again yielding the following values.

M1
M1
M2
M3
M4
F1
F2
F3
F4

4
8
2
1
4
11
60
90

M2
4
0
12
12
65
1
5
99

M3
8
0
0
0
1
62
9
80

M4
2
12
0
70
14
0
1
99

F1
1
12
0
70
10
0
1
94

F2
4
65
1
14
10
0
3
97

F3
11
1
62
0
0
0
10
84

F4
60
5
9
1
1
3
10

90
99
80
99
94
97
84
89

89

During the following years these values tended to remain the same.
123. Analyze the tables and determine the mating system of the Humboldt penguins.
(1 point)
A. promiscuity
B. polyandry
C. polygyny
D. monogamy

85

124. Which is the most common polygamous relationship in these penguins? (1 point)
A. promiscuity
B. polyandry
C. polygyny
D. monogamy

125. Which group of animals do the penguins belong to? (1 point)

A. Ratitae (birds with flat breast and weak breast muscles)


B. Carinatae (birds with strong breast muscles)
C. Neither, they are not birds
126. The substrate(s) of RUBISCO is (are): (1 point)

(1) Phosphoenolpuruvate (PEP)


(2) Ribulose-bis-phosphate (RuBP)
(3) Oxaloacetic acid (OAA)
(4) Phosphoglyceric acid (PGA)
(5) Carbon dioxide (CO2)
(6) Phosphoglyceraldehyde (GAP)
(7) Oxygen (O2)

86

A. 1, 2, 5
B. 1, 5
C. 2, 5
D. 1, 2, 6
E. 2, 5, 7
127. The diagram shows a section through a mammalian ovary. The numbers indicate
different stages of development. (1 point)
Choose the correct sequence of numbers in which the structures develop.

A. 1, 2, 3, 4, 5
B. 5, 4, 3, 2, 1
C. 5, 2, 4, 1, 3
D. 5, 2, 4, 3, 1
E. 2, 4, 1, 3, 5

87

Questions 128-131. PKU and albinism are two autosomal recessive disorders, unlinked
in the human. If a normal couple produced a boy with both disorders and they want to
have the second child:
128. What is the chance of the second child having PKU? (1 point)
A.1/2
B. 1/4
C. 2/3
D. 1/16
129. What is the chance of the second child having both traits? (1 point)
A. 1/2
B. 1/4
C. 1/8
D. 1/16

130. What is the chance of the second child having either PKU or albinism? (1 point)
A. 1/2
B 3/4
C. 3/8
D. 3/16

88

131. What is the chance of their having a normal child? (1 point)


A. 1/16
B. 4/9
C. 9/16
D. 6/16
Questions 132-137. There is a patient who expressed a very rare phenotype. According
to the medical source, this phenotype is seen in 1 in every 100,000 people. The family
history of this patient is given below:

132. How is the trait inherited? (1 point)

A. autosomal recessive
B. autosomal dominant
C. sex-linked recessive
D. sex-linked dominant

89

133. If D= dominant, d = recessive, what is the genotype of II-3? (1 point)

A. DD
B. Dd
C. dd
D. XdY
E. XDY
134. What is the genotype of II-4? (1 point)
A. DD or XDXD
B. Dd or XDXd
C. dd or XdXd

135. If IV-2 married to a man from an unrelated family, what is the chance to get a
normal child? (1 point)
A. 1/2
B. 2/3
C. 100%
D. Cannot be determined
136. For the alleles D and d, which individual should be homozygous? (1 point)
A. III-1
B. III-2
C. III-4

90

D. III-5
E. III-7
137. If this trait is instead quite common in the population, then what is the chance that
IV-4 is heterozygous? (1 point)
A. 1/2
B. 1/4
C. 2/3
D. 100%
138. There are several types of human blood cells such as erythrocytes and monocytes.
They all come from stem cells. Which of the following is/are correct about the stem
cells of blood cells? (1 point)
(1) B cells come from lymphoid stem cells.
(2) T cells come from lymphoid stem cells.
(3) Erythropoietin stimulates production of erythrocytes from myeloid stem cells.
(4) Neutrophils and basophils are derived from the same stem cells.
(5) Lymphoid stem cells come from myeloid stem cells.

A. 1, 2, 3, 4, 5
B. 1, 2, 3, 4
C. 1, 3
D. 1, 2, 4

91

139. Which of the following role(s) do platelets play in the clotting process? (1 point)
(1) They help to form a plug for protection against blood loss.
(2) They release chemical signals for fibrin formation.
(3) They release chemical signals for reducing blood pressure.
A. 1, 2
B. 1, 2, 3
C. 2, 3
D. 1, 3

140. Which of the following is NOT involved in allergic response in human? (1 point)
A. Histamine.
B. Mast cell.
C. Plasma cell
D. Platelets.

141. There are several sensory receptors in human skin. Which of the following is
located deepest in the skin? (1 point)
A. Sensory receptor for pain.
B. Sensory receptor for cold.
C. Sensory receptor for heat.
D. Sensory receptor for strong pressure.

92

142. A mutant zebra fish has a reduced number of hair cells in the neuromast of its lateral
line system. Which of the following will happen? (1 point)
(1) The mutant fish will not be able to detect depth of water.
(2) The mutant fish will swim slowly.
(3) The mutant fish could not detect the sound of its prey.
(4) The mutant fish will show impaired detection of water movement around its body.
A. 1, 2
B. 3, 4
C. 4
D. 2, 4

143. Hemoglobin is responsible for transporting oxygen from the lungs to the tissues.
The Bohr shift is one of the most important properties of hemoglobin. Which of the
following is NOT true about Bohr shift? (1 point)
A. Additional oxygen is bound by hemoglobin in the lungs when pH decreases.
B. Additional oxygen is released from hemoglobin at a lower pH.
C. CO2 is involved in Bohr shift.
D. Bohr shift helps tissues to obtain more oxygen during exercise.

93

144. Which of the following is/are NOT true about the difference in the digestive tracts of
carnivores and herbivores? (1 point)
(1) Carnivores usually have a bigger stomach.
(2) Carnivores usually have a shorter colon.
(3) Herbivores usually have a longer caecum.
A. 1, 2
B. 1
C. 2, 3
D. 3

Questions 145-148. Hemophilia and color blindness are X-linked recessive traits. When
a color-blind woman is married to a hemophiliac man,
145. What is the chance of their having a normal son? (1 point)
A. 50%
B. 0%, all their sons will suffer from color-blind
C. 0%, all their sons will suffer from hemophilia
D. It depends on the recombinant frequency.

94

146. If their son was married to a woman whose mother was colour-blind and their father
was not colour-blind, what is the chance for them to produce a normal daughter?
(1 point)
A. 0%
B. 50%
C. 75%
D. 100%

147. If their daughter was married to a normal man whose father was color-blind, and
produced 1 normal son, 4 normal daughters, 2 color-blind sons, 2 hemophiliac sons
and 1 color-blind, hemophiliac son, the distance between the two genes is: (1 point)
A. 0.5
B. 0.33
C. 0.2
D. 0.1
148. If they have a color-blind daughter, (1 point)
A. There must be a mutation in her fathers germ line.
B. She must have abnormalities other than color-blind.
C. The chance is less than 1/100,000
D. The chance is about 1/1000

95

Questions 149-152. Huntington disease is a rare fatal disease. People with this disease
start to show symptoms in their 40s. Peters father (John) has Huntington disease.
Johns father (Peters grandfather), who also had this disease, had 11 children (5 sons and
6 daughters). Among them, 6 (3 sons and 3 daughters) of them developed the disease and
five died from it.
149. How is the trait inherited? (1 point)
A. autosomal recessive
B. autosomal dominant
C. sex-linked recessive
D. sex-linked dominant
150. What is the possibility that Peter will also develop the disease? (1 point)
A. 50%
B. 25%
C. 75%
D. 67%

96

151. Peter is married to a normal woman. What is the possibility that their first child will
eventually develop the disease? (1 point)
A. 50%
B. 25%
C. 75%
D. 67%
E. 0

152. If Peters mother-in-law died from the same disease, what is the possibility that their
first child will eventually develop the disease? (1 point)
A. 3/16
B. 4/16
C. 7/16
D. 9/16
E. 12/16

153. Trophic levels are indicated below with numbered lines in the flowchart. Write the
appropriate trophic level name in the space provided next to its number. Write
ONLY the letter of the trophic characteristic. (1 point)
NOTE: Left-hand circle in flowchart is Heat; right-hand circle in flowchart is To
detritivores.

97

1. no answer required

5. __________

A. energy used in cellular respiration


B. secondary consumers

2. _________

6. __________

C. tertiary consumers
D. energy in wastes

3. __________

7. __________

E. primary producers
F. primary consumers

4. __________

98

154. Match the biome in the figure below with the appropriate plotted area (a, b, c, d, e, and
f) in the climograph. (1 point)
1. _______ arctic and alpine tundra
2. _______ coniferous forest
3. _______ desert
4. _______ grassland
5. ________ temperate forest
6. ________ tropical forest

155. Referring to the action potential graph below, write the letter (from the graph) that
corresponds to the appropriate action potential action on the right of what is occurring at
that stage of the action potential. (1 point) Note, there could be more than one choice
for each question.

99

1._______ The membrane is unable

to respond to any further stimulation


regardless of intensity

2.__ ______ Sodium gates close, and


potassium gates re-open

3.____ ___ Both sodium and


potassium voltage-gated channels are
closed

4.________ Stimulus opening of


some sodium channels

156. Molting is a process observed in insects. Which of the following statements is/are
true? (1 point)
(1) The exoskeleton of insects is largely made of protein and chitin.
(2) The structure of chitin is similar to that of bacterial cell wall peptidoglycan.
(3) No enzyme has been found to digest chitin.
(4) Molting can be observed in all arthropods.
(5) The only place that is not covered by exoskeleton is the joints between the body
and walking legs.

100

A. 1, 2, 4, 5
B. 1, 4
C. 1, 3, 4, 5
D. 1, 5

157. The mechanism of molting has largely been revealed. The figure below is a diagram
of such a process. Boxes A, B and C represent 3 different growth hormones and molting
hormones. Fill in the answer boxes by choosing the correct letter. (1 point)

101

Answer: A-C

1. brain hormone (BH)


2. juvenile hormone (JH)
3. molting hormone (MH)

158. The figure below shows 4 different circulation systems of vertebrates. From left to
right, these are the circulation systems of (1 point)

A. mammals, reptiles, amphibians, and fish, respectively.


B. fish, amphibians, reptiles, and mammals, respectively.
C. mammals, amphibians, reptiles, and fish, respectively.
D. mammals, amphibians, fish, and reptiles, respectively.

102

159. Match the numbers shown below with correct structures in the figure in question
above (question158). (1 point)

Answer
A-G

1 Sinus venosus
2. Atrium
3 Pulmonary vein
4 Pulmonary artery
5 Conus arteriosus
6 Right Atrium
7 Left ventricle

103

Questions 160-162. The structure of a mammalian kidney is shown below.

104

160. Match the following terms with the correct structures shown in the figure. (1 point)

1.

collecting duct

2.

glomerulus

3.

distal tubule

4.

Bowman`s capsule

5.

proximal tubule

6.

ureter

7.

afferent arteriole

Answer
A-G
1
2
3
4
5
6
7

105

161. The substances that are reabsorbed in the proximal tubule is/are: (1 point)
(1) Na+
(2) Cl(3) Water
(4) Glucose
(5) Amino acids
(6) Urea
A.

1, 2, 3

B.

6,

C.

1, 2, 4, 5,

D.

1, 2, 3, 4, 5

E.

4, 5

162. In the kidney, ultrafiltration occurs in which of the following structures? (1 point)

(1) Nephrons
(2) Bowmans capsule
(3) Proximal tubule
(4) Distal tubule
(5) Collecting duct

106

A. 1, 2
B. 2, 3, 4, 5
C. 3, 4, 5
D. 2, 3, 4
E. 1, 2, 3, 4

Questions 163-166. Sensory transduction by a taste receptor is shown in the figure below.
The sequential events of the transduction is labeled by numbers 1 to 7. A portion of the
cell is magnified. Structures A B and C are different channels

107

163. Structure A which is responsible for event 3 is a (1 point)


A. Potassium channel
B. Calcium channel
C. Sodium channel
D. Neurotransmitter channel
E. Glycine channel

164. Structure C which is responsible for event 5 is a (1 point)


A.

Potassium channel

B.

Calcium channel

C.

Sodium channel

D.

Neurotransmitter channel

E.

Glycine channel

165. Event 4 by structure B (1 point)


A.

depolarizes membrane potential.

B.

increases membrane permeability

C.

transports more sugars molecules into the cell.

D.

transports signal molecules into the cell so that the cell starts to
synthesize neurotransmitters.

E.

transports precursor molecules of neurotransmitters into the cell so


that the cell can synthesize neurotransmitters.

108

166. Which of the following statements is/are true about the action potentials shown as D
and E in the figure? (1 point)
(1) They were both recorded after and before sugar molecules were present,
respectively.
(2) They were recorded before and after sugar molecules were present,
respectively.
(3) The action potential observed after sugar reception is triggered by an
increase of calcium ions which stimulate neurotransmitter release.
(4) The action potential observed after sugar reception is triggered by an
increase of Potassium ions which stimulate neurotransmitter release.
(5) The action potential is recorded from taste sensory receptor cells.

A. 2, 3
B. 1, 3
C. 2, 4
D. 2, 5
E. 2, 4, 5

109

167. Which of the following is NOT a mechanism of animal migration? (1 point)


A. Cruising
B. Piloting
C. Navigation
D. Orientation

168. Both snake and weasel hibernate. Which of the following is correct? (1 point)
A. They will die when temperature decreases below the critical temperature.
B. Weasel will die when temperature decreases below the critical temperature.
C. Snake will die and weasel will wake up when the temperature decreases
below the critical temperature.
D. Weasel keeps low body temperature and slow heart rate during the entire
period of hibernation.

169. It is possible to predict bird diversity based on forest types. Which of the following
is most critical to bird diversity for a forest? (1 point)
A. Forest area
B. Vertical stratification
C. Species composition of plants
D. Coniferous or deciduous forests

110

170. Four quantity pyramids (total number of organisms) are shown below. Which is
representative for plant-aphid-ladybug (ladybird)? (1 point)

171. Which of the following ecosystems has the highest net primary productivity? (1
point)
A. Tropical rain forest
B. Open ocean
C. Northern coniferous forest
D. Farm lands

172. The figure below shows the vertical distribution of some parameters (Chlorophyll,
Phosphate, Primary production and Temperature) in the North Pacific during summer.

111

From left to right, letter a through letter d represent: (1 point)


A. Temperature, phosphate, chlorophyll and primary production
B. Chlorophyll, phosphate, temperature and primary production
C. Primary production, phosphate, temperature, chlorophyll
D. Phosphate, temperature, primary production and chlorophyll.
173. The length of a food chain in a food web is often quite short. Usually, the length is
shorter than 5 links. Which is mostly likely reason for the shortness of the food chain?
(1 point)
A.

The population of final predator is often too large.

B.

The primary producers can sometimes be indigestible.

C.

Only about 10% of energy in one link can be converted to organic matters in
next trophic level.

D.

Wintertime is too long and low temperature limits primary productivity.

112

174. The figure below shows a membrane potential graph detected after a rod cell of
human eyes sees light. Which of the following is the direct trigger for the hyperpolarization? (1 point)

113

A. Retinal switches from cis form to trans form.


B. Cyclic GMP is destroyed.
C. Transducin is activated.
D. Potassium channel is closed
E. Sodium channel is closed.

114















All IBO examination questions are published under the following Creative Commons license:



CC BY-NC-SA (Attribution-NonCommercial-ShareAlike) https://creativecommons.org/licenses/by-nc-sa/4.0/
The exam papers can be used freely for educational purposes as long as IBO is credited and
new creations are licensed under identical terms. No commercial use is allowed.

IBO2005 Theory Test Standard Answers


Question
1-1
1-2
1-3
1-4
2
3
4
5
6
7
8
9
10
11
12
13
14
15
16
17
18
19
20
21
22
23
24
25-1
25-2
25-3
25-4
25-5
26-1
26-2
26-3
26-4
26-5
27
28
29
30
31

Answer
A
B
D
E
C
C
B
A
E
D
B
D
B
D
C
AC
E
B
B
B
A
C
B
E
B
D
G
F
D
A
B

C
E
C
B

Question
32
33-1
33-2
33-3
33-4
33-5
34
35
36
37
38
39
40
41
42
43
44
45
46
47-1
47-2
47-3
47-4
47-5
47-6
47-7
47-8
47-9
48
49
50
51
52
53
54
55
56
57
58
59
60
61-1

Part 1
Answer

A
A
E
CD
CD
AB
CD
B
D
D
AE
DE
E
E
E
C
A
A
C
B
B
A
B
B
A
A
B
B
A
D
E
D
D
D
C
A
AC
A
C
E
E
C

Question
61-2
61-3
61-4
62
63
64
65
66
67
68
69
70
71-1
71-2
71-3
71-4
71-5
71-6
71-7
72-1
72-2
72-3
72-4
73-1
73-2
73-3
73-4
74-1
74-2
74-3
74-4
75
76
77
78
79
80
81
82
83
84

Answer
D
B
A
B
D
BC
E
AD
A
C
E
C
A
F
C
D
B
E
G
A
E
G
C
F
A
B
D

F
B
G
D
F
F
A
C
F
E

IBO2005 Theory Test Standard Answers


Question
85
86
87
88
89
90
91
92
93
94
95
96
97
98
99
100
101
102
103
104
105
106
107
108
109-1
109-2
109-3
110-1
110-2
111
112
113
114
115
116
117
118
119
120
121-1
121-2
121-3

Answer
C
C
C
A
D
C
D
E
C
C
B
D
B
B
B
A
C
C
C
B
A
C
B
B
D
B
C
D
B
A
A
G
B
C
E
B
A
D

Question
121-4
122
123
124
125
126
127
128
129
130
131
132
133
134
135
136
137
138
139
140
141
142
143
144
145
146
147
148
149
150
151
152
153-1
153-2
153-3
153-4
153-5
153-6
153-7
154-1
154-2
154-3

Part 2
Answer
C
B
D
B
B
E
D
B
D
C
C

B
A
D
D
C
A
B
B
B

B
A
B
C
C
A
B
D
F
E
F
E
A

Question
154-4
154-5
154-6
155-1
155-2
155-3
155-4
156
157-1
157-2
157-3
158
159-1
159-2
159-3
159-4
159-5
159-6
159-7
160-1
160-2
160-3
160-4
160-5
160-6
160-7
161
162
163
164
165
166
167
168
169
170
171
172
173
174

Answer
B
C
D
E
DE
AF
B
B
A
B
C
D
A
B
F
E
C
G
D
F
B
C
D
E
A
G
D
A
A
B
A
A
C
C
AB
A
C

INTERNATIONAL BIOLOGY OLYMPIAD


PRACTICAL PROBLEMS

2005, Beijing, China















All IBO examination questions are published under the following Creative Commons license:



CC BY-NC-SA (Attribution-NonCommercial-ShareAlike) https://creativecommons.org/licenses/by-nc-sa/4.0/
The exam papers can be used freely for educational purposes as long as IBO is credited and
new creations are licensed under identical terms. No commercial use is allowed.

16th International Biology Olympiad

Beijing
July 2005

Practical Examination
Part I

Total time available: 90 minutes

The 16th IBO Practical Tests


First name:
Last name
Country:
Code:
Important:
1. Write your name and code on both task paper and answer paper sheets.
2. Make sure that all the results are written on the answer paper unless otherwise
instructed.
3. There are 4 parts in the practical test. Each part lasts 90 min. You should start your
first test according to last digit of your competitor code. For example, if you
have a code of 221, your first practical test will be part I, if you have a code of
223, your first practical test will be part III.
4. Your second practical test is as follows: competitors from part I and part II switch
labs; competitors from part III and part IV switch labs;
5. You go to your third practical test according to the following rules:
If the last digit of your competitor code is 1, you go to practical test part III.
If the last digit of your competitor code is 2, you go to practical test part IV.
If the last digit of your competitor code is 3, you go to practical test part I.
If the last digit of your competitor code is 4, you go to practical test part II.
You should follow the instructions from your guides when switching labs.

Practical tests Part I:


Biochemistry and Molecular Biology

Very important notice: you should start task 1 first and while the
gel electrophoresis is running, start and finish task 2.

Task 1: Separation of plasmid DNA restriction fragments by Agarose Gel


Electrophoresis (24 points

Instruments: Centrifuge, Agarose gel electrophoresis apparatus and Fluorescence gel


imaging systems.

Important:
Raise the blue card on the bench table to ask for help when you want to use the
electrophoresis power supplies.

Introduction
Plasmids are circular double-stranded DNA molecules, which can exist and replicate
independently in bacterial cells. Restriction enzymes can cut the plasmid DNA into
fragments. In the experiment a plasmid and three restriction enzymes BamHI, PstI and
HindIII are provided. You will use the three restriction enzymes to digest the plasmid
DNA and run agarose gel electrophoresis. You need to determine the restriction
3

enzyme sites and calculate the size of restriction fragments between cutting sites
according to migration distance of DNA fragment, which is inversely correlated to the
logarithm of the length of fragment.

Reagents

1. 1TAE buffer Tris-acetate-EDTA


2. DNA dye - GeneFinder containing anthocyanin and sucrose
3. BamHI
4. PstI
5. HindIII
6. Plasmid DNA
7. DNA size standard
8. Distilled water

Equipment
1. Lab gloves
2. Marker pen
3. 0.5 ml centrifuge tubes
4. Centrifuge tube holder
5. Pipettes
6. Centrifuge
7. Incubator
8. Agarose gel electrophoresis apparatus
4

9. Fluorescence gel imaging systems (use it with lab assistants),

Procedure and operation of equipments


1.

Using a Pipette:

A 0-10l pipette is provided for the experiment. The volume is adjusted by turning the
setting ring. The digits of the volume display should be read from top to bottom.
After attaching an appropriate tip, press the control button down to the first stop and
insert the tip into the liquid. Slowly release the button until it reaches a complete stop
to aspirate (suck up) the sample. Then, insert the tip with the liquid to the target
places (tubes or wells) and press the button down slowly to the second stop until all
collected liquid is completely expelled from the tip. Eject the used tip to the trash by
pressing the ejector button.

2.

Using a Centrifuge
Press the stop lever down to open the lid. Load tubes into the rotor. Be sure to balance
the load properly. Close and firmly press down the lid until the lid locks into its

position. The rotor will begin spinning when the lid is completely closed. Let the
centrifuge run for 20 seconds. Push the stop lever, open the lid and remove the tubes
after the rotor has stopped spinning.

3.

Using Restriction enzyme digestions


Type II restriction enzymes recognize certain DNA sequences and digest DNA at the
recognition sites. The plasmid DNA provided to you should be digested by three
different enzymes: BamHI, PstI and HindIII. Add the appropriate amount (1l) of
enzyme(s) to the plasmid DNA in centrifuge tube and close the lid of the tube. Mix it
well by gently tapping the bottom of the centrifuge tube. Incubate the centrifuge tubes
at 37C for 15 min in the incubator.

4.

Using the agarose gel electrophoresis apparatus


The 0.8% Agarose gel with wells is ready for use. Fill the electrophoresis tank with
1TAE buffer and let the buffer cover the gel. The buffer surface should be about 34mm above the agarose gel surface. Load 10 l of the sample, which contain (1)
plasmid DNA cleaved with restriction enzymes and (2) DNA dye, into the wells of
the gel. Please note that the pipette tip should be 1-2 mm above the bottom of the
well so that you can load all of the samples into the wells without puncturing the
bottom of the wells. After loading the samples, close the cover of the electrophoresis
tank. Note that Red wire connects to the anode and Black wire connects to the cathode.
Call the laboratory assistant to turn on the power supply by raising the blue card. Run

the samples at 100 volt for 40 min. After 40 minutes call the assistant to turn off the
power supply by raising the blue card. Every competitor will use one electrophoresis
tank, while every 2 competitors share one power supply.

5. Gel imaging system


This system is operated by lab assistants. Your samples contain a non-toxic dye that
binds DNA fragments for visualization.

Experimental procedure
1. Label eight 0.5-ml centrifuge tubes 1 through 8 with a marker pen, Add solutions
to each tube as follows:

Table 1. Digestion of plasmid DNA with restriction enzymes


No.

Plasmid DNA (l)

BamHI (l)

PstI (l)

HindIII (l)

ddH2O (l)
9

9
1

9
8

7
10

2. Mix well and incubate tubes 1-6 at 37C for 15 minutes. Leave tube 7 in the tube
holder. If you found droplets of the solution on the inside tube wall, you may
used the centrifuge to spin them to the bottom of the tube. The centrifuges are
provided on your table.
3. Put the agarose gel (previously prepared for you) into the electrophoresis tank,
pour 1TAE buffer into the tank and let the buffer cover the gel about 3-4mm.
The gel has 10 wells for sample loading.
4. Add 6 l DNA size standards into the No.8 centrifuge tube.
5. Add 3 l of 5X dye to each tube and mix them well.
6. Load 5 l of DNA size standards (tube No. 8) into the First well of the gel. Load
all of your plasmid samples from the second well through to the eighth well in the
order of Table 1. Please note that tube numbers differ from the lane numbers in
which they are loaded. Use a clean tip for each load. Close the cover of the
8

electrophoresis tank. Call the assistant by raising the blue card to turn on the
power supply. Run the samples at 100 volt for 40 min.
(Note: during your waiting time for completion of electrophoresis, please
undertake task 2 and finish it.)
6. After the electrophoresis has run for 40 min, call the assistant to turn off the power
supply by raising the blue card. Wear gloves and take out the gel holder. 8. Put
your gel into the box with your competitors number. Close the lid and leave the
box on the table. A lab assistant will take the gel image and print a copy for you.

Separation of plasmid DNA restriction fragments with agarose gel


electrophoresis (24 points: 3 points for each lane). The score for this task will be
given by a professor in charge of this test.
Three points for each lane: No DNA, no point; smearing lane with clear bands, minus
1 point; incomplete digestion, minus 1 point; faint bands, minus 1 point.
Your gel image will be posted below once it is printed by the lab assistant.

Task 2: Determination of restriction enzyme sites and DNA fragment size of


restriction fragments. (16 points)

Due to time limitation, you will not be able to run your own gel for size analysis.
However, the figure below is an agarose gel profile of DNA fragments, in which an
identical plasmid was digested with the same three DNA restriction enzymes. The
procedure for digestion and loading positions of each digestion in the gel are identical
to the instruction in task 1. Please answer the following questions according to the
profile below.

10

Question 1. How many sites does this plasmid have for PstI, BamHI and HindIII,
respectively? (3 points)
A. PstI:1, BamHI: 0, HindIII : 2.
B. PstI:2, BamHI : 0, HindIII : 2.
C. PstI:2, BamHI : 1, HindIII : 0.
D. PstI:1, BamHI : 1, HindIII : 1.
Question 2. Linear lambda DNA is often digested with restriction enzymes and used
as a molecular standard in running agarose gels. The figure below is a profile of
lambda viral DNA fragments obtained with HindIII digestion. The numbers on the
right side of the gel are fragment sizes in kb.

11

Which of the following statements isare true? (3 points)


(1) There are 8 sites for HindIII on lambda DNA.
(2) Since lambda DNA can be digested by HindIII, the entire
molecule of lambda DNA must be double stranded.
(3) The profile shown in the figure above is likely to be a
fluorescent image of a dye binding to DNA fragments.
Select which answer A to D is correct
A. 1
B. 1, 2, 3
C. 2, 3
D. 3

Questions 3-5. The gel profile contains eight bands of DNA size standards in lane 1
and the sizes of the DNA fragments in lane 1 are as follows (in bp, base pairs): 200,
500, 800, 1200, 2000, 3000, 4500, 7000. It is known that migration distance of a
DNA fragment is inversely correlated to the logarithm of the fragment length.
Please plot the logarithm of the DNA fragment sizes (kb) versus the migration
distances (cm) on the plotting (graph) paper below, and calculate the sizes (kb) of
the DNA fragments.

12

Question 3. The size (kb) of the smaller restriction fragment between PstI site &
HindIII site is: (3 points)

A. 2.5
B. 0.8
C. 1.1
D. 0.6

13

Question 4. The size (kb) of the smaller restriction fragment between HindIII site &
BamHI site is: (3 points)

A. 0.8
B. 0.4
C. 0.5s
D. 0.6

Question 5. The plasmid length (kb) is: (4 points)


A. 5.2
B. 6.9
C. 4.8
D. 4.3

14

16th International Biology Olympiad

Beijing
July, 2005

Practical Examination
Part II

Total time available: 90 minutes

15

The 16th IBO Practical Tests


First name:
Last name
Country:
Code:
Important:
1. Write your name and code on both the task paper and answer paper sheets.
2. Make sure that all the results are written on the answer paper unless otherwise
instructed.
3. There are 4 parts in the practical test. Each part has 90 min. You should start your
first test according to last digit of your competitor code. For example, if you
have a code of 221, your first practical test will be part I, if you have a code of
223, your first practical test will be part III.
4. Your second practical test is as follows: competitors from part I and part II
exchange labs; competitors from part III and part IV exchange labs;
5. You go to your third practical test according to the following rules:
If the last digit of your competitor code is 1, you go to practical test part III.
If the last digit of your competitor code is 2, you go to practical test part IV.
If the last digit of your competitor code is 3, you go to practical test part I.
If the last digit of your competitor code is 4, you go to practical test part II.
You should follow the instructions from your guides when switching labs.

16

Practical tests Part II:


Cell Biology

This part of examination contains 3 Tasks:

Task 1: Microscopes and cellular structures (15 points)


Task 2: Identification of plants with thin sections (15 points)
Task 3: Karyotype analysis (10 points)

Total Points available: 40


Total Time: 90 minutes

Task 1: Microscopes and Cellular Structures (13.5 points)

Requirement
In this task, you are provided with cell images obtained using different types of
microscopy. You are required to
(1) Distinguish these cell images and choose one name for the microscopic technique
for obtaining each image,
(2) Select one of the techniques for study,
(3) Distinguish between organelles in a given cell image and answer questions.

17

Procedure
You are supplied with two image sheets, Image Sheet 1 and Image Sheet 2.

On Image Sheet 1, seven images (denoted 1-7) of cells or organisms are printed.
These images are obtained with the different microscopic techniques listed below:
A. Light microscopy
B. Fluorescence microscopy
C. Scanning electron microscopy
D. Ultra-thin section transmission electron microscopy
E. Immuno-electron microscopy
F. Negative staining electron microscopy
G. Freeze-fracture electron microscopy

Answer the questions according to the following descriptions.

Descriptions:
1. Image 1 is most likely to be obtained with _____. (0.9 point).
2. Image 2 is most likely to be obtained with _____. (0.9 point).
3. Image 3 is most likely to be obtained with _____. (0.9 point).
4. Image 4 is most likely to be obtained with _____. (0.9 point).
5. Image 5 is most likely to be obtained with _____. (0.9 point).

18

6. Image 6 is most likely to be obtained with _____. (0.9 point).


7. Image 7 is most likely to be obtained with _____. (0.9 point).
Answer the following questions about different microscopic techniques.
8. ____ is appropriate for locating specific molecules in both cells and tissues (0.9
point).
9. ____ is appropriate for visualizing details of cell and tissue surface (0.9 point).
10. ____ is appropriate for analyzing the interior of cell membranes (0.9 point).
11. ____ is appropriate for examining the fine structure of cells (0.9 point).
12. ____ is appropriate for the fine labelling (ultra structural localisation) of
molecular substances in a cell (0. 9 point).

Image Sheet 2 shows the ultrastructure of a cell. Roman numbers (I-III) indicate
different organelles and/or cell components.
A list of organelles and/or cell components is given below (A through F). Answer the
following questions.
A. Lysosome
B. The Golgi apparatus
C. Mitochondrion
D. Microtubule
E. The endoplasmic reticulum
F. Plastid

19

13. The structure indicated by Roman number I is a _____. (0. 9 point).


14. The structure indicated by Roman number II is a _____. (0. 9 point).
15. The structure indicated by Roman number III is a _____. (0.9 point).
16. The structure indicated by Roman number IV is a _____. (0.9 point).

17. The cell shown in Image Sheet 2 is likely to be a cell of _____. (choose one from
below) (0.6 point).
A. Plant
B. Animal
C. Fungus
D. Eubacterium
E. Archeon

20

Task 2: Determination of plant types with thin sections of plant leaves (15 points)

Materials, tools and instrument


(1)

Five (No.1-No.5) Petri dishes, each of which contains some leaf samples.

(2)

A microscope with objective lens of 10x, 20x, 40x.

(3)

Forceps, razor blade, test tube rack, slide, slide cover slip, filter paper.

Background
There are three major types of photosynthesis metabolism in the plants, called C3
metabolism, C4 metabolism and crassulacean acid metabolism. You are now required
to determine which plants are C3 plants and which plants are C4 plants. The
difference between them is that CO2 fixation and sugar synthesis are performed in
different cells in these two types of plants. The different structures of the leaves
between C3 and C4 plants lead to different metabolism.
Task
There are five Petri dishes on the table. Each Petri dish contains pieces of leaves
from a different plant. You are required to determine if the leaves are from C3 plants
or C4 plants.
Procedure
Please follow the procedure below:
(1) Pick up one sample from each dish and make a thin section.
(2) Use several drops of water to wash the section off the blade onto the slide.

21

(3) Remove the excess water with a piece of filter paper, but keep the water around
the sample.
(4) Put the cover slip onto the sample, remove excessive water and observe the
specimen under the microscope.
Answer the following questions.
18. The leaves in Petri dish 1 are (3 points)
A. C3 type.
B. C4 type.
19. The leaves in Petri dish 2 are (3 points)
A. C3 type.
B. C4 type.
20. The leaves in Petri dish 3 are (3 points)
A. C3 type.
B. C4 type.
21. The leaves in Petri dish 4 are (3 points)
A. C3 type.
B. C4 type.
22. The leaves in Petri dish 5 are (3 points)
A. C3 type.
B. C4 type.

22

Task 3. Karyotype analysis (10 points)


Requirement:
In this task, you are asked to perform karyotype analysis. The materials are root tips
from a plant. You will need to use a microscope to observe the cells of the root
meristem tissue and find those cells in mitosis.

Materials, instruments and tools


(1) Root tips (approximately 5-10 mm in length) in a 1.5 ml centrifuge tube.
(2) A microscope with objective lens of 10x, 20x, 40x.
(3) A Carbol Fuchsin (a dye) solution. (It is in a 1.5-ml centrifuge tube, labelled
as CF)
(4) Forceps, razor blade, test tube rack, slide, slide cover, filter paper.
(5) A 1.5 ml centrifuge tube containing approximately 1 ml 1 N (normal) HCl
solution.
Important:
You will use 1 N HCl to treat the root tips. HCl solution is very harmful to your eyes
and skin. Wear gloves and protective goggles when using HCl solution. If HCl
solution comes in contact with any part of your body, please report it immediately to
any instructor in the exam room.

23

Procedure:
You are provided with three root tips of a plant. The following procedure should be
followed so that you can make an appropriate specimen to observe the chromosomes
from cells in mitosis.
(1) Use the forceps to put one or two root tips into the small bottle containing 1 N
HCl.
(2) Put the bottle into the water bath, which has been adjusted to 60C, for 8 min.
Note, your laboratory has several water baths with temperature adjusted to 60C.
The water baths are on the instructors desk.
(3) Very carefully take the root tips out of the HCl solution with a forceps and put
them into the beaker provided containing distilled water. Gently shake it for 1
min.
(4) Take the root tips out of the distilled water. Important: the root tips are now very
fragile. It is recommended that you use the forceps to pick the root tips and dont
touch the tips of the roots.
(5) Put one root tip on a slide. Cut the tissue of the root tip that is rich in dividing
cells. This region is within 1 mm from the root tip. Discard other parts of the root.
(6) Put one drop of Carbol Fuchsin solution onto the root tissue you have just cut off
and leave it to stain for 7 min. Squash the tissue gently with forceps so that the
tissue is dispersed.
(7) Cover the dispersed tissue with a slide cover slip. Push the slide cover slip gently
with a pencil or forceps until the tissue is completely dispersed and separated.

24

(8) Put the slide between two pieces of filter paper and put it on a flat surface. Gently
press the upper filter paper down so that the tissue is further squashed. In the
meantime, extra dye solution is also removed and absorbed by the filter paper.
(9) Observe your slide specimen under the microscope. Note, you might need to use
all objective lenses.
Note: You are provided with three root tips to prepare your specimen. If you fail to
make a good specimen for your observation, please repeat the procedure and make
another preparation. However, the time for your experiment is limited.

Answer the following question:


23. How many pairs of chromosomes are there in the cells (in metaphase) from this
plant? (6 points)
A. 3
B. 4
C. 5
D. 6
E. 7
F. 8
G. 9

25

24. If you found that different metaphase cells had different number of chromosomes,
how do you determine the exact number of chromosomes? (2 points)
A. Count the chromosome numbers from several cells and use the average
number as the chromosome number.
B. Count the chromosome numbers from several cells; the maximum
chromosome number of a cell is the chromosome number of the plant.
C. Count the chromosome numbers of several cells in metaphase; the
chromosome of the plant is the number with highest frequency.

25. The purpose of the treatment of root with 1 HCl at 60C for 8 min is: (2 points)
A. Stimulate cells so that you can observe more cells in metaphase.
B. Dissolve cellulose of cell walls so that the cells are easily separated.
C. Remove ions of the cell wall so that the cells are separated.
D. Dissolve the hemicellulose of cell walls so that the cells are easily separated.
E. Puncture some tiny holes on plasma membrane so that Carbol Fuschin could
penetrate into the cell.

26

16th International Biology Olympiad

Beijing
July, 2005

Practical Examination
Part III

Total time available: 90 minutes

27

The 16th IBO Practical Tests


First name:
Last name
Country:
Code:
Important:
1. Write your name and code on both the task paper and answer paper sheets.
2. Make sure that all the results are written on the answer paper unless otherwise
instructed.
3. There are 4 parts in the practical test. Each part lasts 90 min. You should start your
first test according to last digit of your competitor code. For example, if you
have a code of 221, your first practical test will be part I, if you have a code of
223, your first practical test will be part III.
4. Your second practical test is as follows: competitors from part I and part II
exchange labs; competitors from part III and part IV exchange labs;
5. You go to your third practical test according to the following rules:
If the last digit of your competitor code is 1, you go to practical test part III.
If the last digit of your competitor code is 2, you go to practical test part IV.
If the last digit of your competitor code is 3, you go to practical test part I.
If the last digit of your competitor code is 4, you go to practical test part II.
You should follow the instructions from your guides when switching labs.

28

Practical Exam Part-III


Animal anatomy and ecology
This part contains three tasks:
Task 1. Determination of the distribution pattern and estimation of population size.
(16 Points)
Task 2. Classification of insects. (9.8 points)
Task 3. Shrimp anatomy (14.2 points)

Task 1: Determination of distribution pattern and population size (16 points)


Introduction
Tenebrio molitor is an insect and belongs to Coleoptera. It lives in places used for
food storage such as barns. The majority of the life span of T. molitor is in its larval
stage and its adult stage is quite short. In this experiment, you will study two
ecological aspects of T. molitor: population distribution pattern and population size.
The distribution pattern of a population describes the spatial relationship of
individuals of the population. It is also useful in establishing a reliable sampling
method for the populations. Generally speaking, there are three types of distribution
patterns: random distribution, uniform distribution and aggregated (clumped)
distribution (see the associated figure)

29

If you divide an area into smaller identical squares and count individuals in each
square, you will be able to distinguish the distribution patterns. If the distribution
pattern is uniform, the square deviation (S2) of your sampling will be zero. If the
distribution pattern is random, you will get a typical Poisson (normal) distribution in
your sampling. If the distribution pattern is aggregated, you will not be able to obtain
a Poisson distribution in your sampling. Thus, it is possible to distinguish the three
distribution patterns according to square deviation (S2) and averages of your sampling
(m).

If S2/m=0It is uniform distribution


If S2/m=1it is random distribution
If S2/m >1It is aggregated distribution

30

Here, m = (X1+X2++Xn)/n
S2=[(X1-m)2+(X2-m)2++(Xn-m)2]/(n-1)
X1X2Xn represent the number of individuals in the square 1, 2, . and square
n, respectively, and n represents total number of squares you sampled.

Materials:
A printed photograph of a tray containing some T. molitor is provided. The tray is
divided into 7 x 7 squares.

Task: Determine the distribution pattern of T. molitor.

Procedure
Count the number of the larva in A1, A4, B7, C5, D2, D7, E3, F1, F6, and G3 (total
number of squares is 10), and determine the distribution pattern according to the
formula provided above.
Answer the following questions:

Question 1. The value of S2/m is: (2 points)


A. 0.1
B. 0
C. 1
D. 3.4

31

Question 2. The distribution pattern is (2 points)


A. uniform distribution
B. random distribution
C. aggregated distribution

Question 3. Which of the following could alter the answer of question 2 above:
(2 points) (Note, there might be more than one answer)
A.

Choose the same 10 squares, but reverse the sequential order in your
sampling (i.e. start from G3 and finish with A1).

B.

Choose only the four corner squares (A1, A7, G1 and G7) in sampling and
calculate S2 and m to determine the distribution pattern.

C.

Choose only the central five squares (D3, D4, D5, C4 and E4) in sampling
and calculate S2 and m to determine the distribution pattern.

D.

Redo the sampling by choosing 10 squares randomly and calculate S2 and


m to determine the distribution pattern.

Question 4. Which of the following descriptions about the relationship between


population distribution pattern and individuals of the population is accurate? (2
points)
A. Repulsion among individuals of a population would lead to uniform
distribution.

32

B. Repulsion among individuals of a population would lead to random


distribution.
C. Attraction among individuals of a population would lead to uniform
distribution.
D. When the position of each individual is independent of other
individuals, it would lead to aggregated distribution.
E. When the position of each individual is independent of other
individuals, it would lead to uniform distribution.

The following is to estimate the population size

Population size is one of the most important factors in population ecology. A very
useful tool to estimate population size is Capture-Mark-Recapture method. In this
method, animals are trapped and captured. The captured animals are marked with
tags, collars, etc, and released immediately. After a certain period of time, traps are set
again to capture animals from the same population. A proportion of the marked
(recaptured) animals in the second trapping is assumed equivalent to the proportion of
marked animals in the total population. The population size (N) can be estimated by
the following equation:

N=MR/P
Where M is the number of individuals marked during the first capture, R is the

33

number of individuals in second capture, P is the number of individuals in second


capture that are marked.

In the population of T. molitor, 100 individuals are marked with black dots near their
tails. These marked T. molitor were first released and mixed with other individuals of
the population. A second capture was performed and the result is shown in the printed
photograph provided.

Question 5. The population size of the T. molitor is: (3 points)


A. 550
B. 600
C. 610
D. 627
Question 6. In Capture-Mark-Recapture method, it is assumed that the ratios of M/N
and P/R are identical. Which of the following is/are required to assure an accurate
estimation of population size? (3 points) Note, there might be more than one
correct answer.
A. The marking method should not alter the animals normal activity.
B. Immigration occurs regularly.
C. No birth and no death during the experimental period.
D. The population should have a uniform distribution.
E. The marks on the organism should last longer than the experimental time.

34

Question 7. If after the experiment additional information is obtained that 40


individuals died and 30 individuals moved in between marking and recapture The new
estimated population size would be (2 points) .
A. Equal to what you obtained in question 5.
B. Equal to or smaller than what you obtained in question 5.
C. Equal to or larger than what you obtained in question 5.

35

Task 2. Classification of insects. (9.8 points)


Instruction
There are seven specimens of beetles in the tray on your table. You are required to
name each of them according to the key that follows. You will need to use a
stereoscope, forceps and needle. Note, damage to the specimen will lead to
subtraction of points from your final score of practical test.

A. Opatrum subaratum Faldermann


B. Blaps femoralis femoralis Fischer-Waldheim
C. Coccinella septempunctata Linnaeum
D. Potosia brevitarsis (Lewis)
E. Popillia quadriguttata (Fairmaire)
F. Polyzonus fasciatus (Fairmaire)
G. Chrysochus chinensis Baly

36

Question 8. Fill in the table below according to your classification result and mark
them on your answer sheet: (1.4 x 7 = 9.8 points)

Beetle

Answer A-G

37

Key to 7 species of beetles

1 Tarsus of fore legs, middle legs and hind legs have 5-5-4 segments............................................2
Tarsus of foreleg, middle legs and hind legs have 5-5-5 or 4-4-4 segments....................3
2 Body size small and flat; there is a triangular notch at anterior edge of the labrum; wing
tip at end of wing case invisible................................................ Opatrum subaratum Faldermann
Body size large and elevated; straight at anterior edge of the labrum; wing tip
visible at end of wing case in male individual ........... Blaps femoralis femoralis Fischer-Waldheim
3 Tarsus have 4-4-4; body segments; there are 7 black round dots on the wing
cases..Coccinella septempunctata Linnaeum
Tarsus segments are type 5-5-5; body not semicircular.... 4
4 3rd through 8th antennal segments are branchial (gill-like) ..5
Antennal segments threadlike. 6
5 There is a notch at base of each wing case; there are many white and downy (cottony) dots in shapes
of strips, clouds, or waves on the pronotum and wing cases. .. Potosia brevitarsis (Lewis)
There is no notch at base of wing cases; no downy dots on the pronotum and
wing cases...Popillia quadriguttata (Fairmaire)
6 Body elongate and cylinder-like; compound eyes are reniform (kidney-shaped); antenna at frontal
processes;
there are 2 yellowish transverse strips on each wing casePolyzonus fasciatus (Fairmaire)
Body thickset and oval; round compound eyes; body color deep green, blue,
glaucous (waxy blue grey) or indigo; no transverse strips on wing cases.Chrysochus

38

chinensis Baly

Task 3. Anatomy of a shrimp (14.2 points)


Introduction
Shrimps belong to the Crustacea in the Arthropoda. They have heteronomous
segmentation. The shrimp provided for your exam has a body of 21 segments with an
exoskeleton and jointed appendages.

Materials and instruments

1. One shrimp. Note: you only have one shrimp.


2. Stereoscope
3. Scissors, needle, forceps, insect needles, operational knife.
4. Wax tray
Experiment
Experiment contains two parts: external anatomy of the shrimp and nervous system
anatomy of the shrimp.
(1) External anatomy
Observe the shrimp carefully and answer the following questions.

Question 9. How many pairs of appendages are there in the shrimps head, thorax and
abdomen, respectively? (2 points)

39

A. 2, 4, 10
B. 5, 8, 6
C. 4, 5, 8
D. 3, 6, 7

Question 10. Find the mouthparts of the shrimp and separate the appendages that form
the mouthparts.
How many pairs of appendages is/are the mouthparts composed of? (2 points)
A. 1
B. 2
C. 3
D. 4
E. 5

Questions 11-12. Observe the schematic structures of appendages in the figure below.

40

Question 11. Could you find all of these appendages on the shrimp provided to you?
(2 points)
A. Yes
B. No
Question 12. Sequentially from appendage 1 through appendage 4 shown in the figure,
the main functions of these appendages are: (2 points)
A. 1: Walking, 2: swimming, 3: sensing and holding, 4: sensing and holding
B. 1: Swimming, 2: sensing and holding, 3: swimming, 4: sensing and holding
C. 1: sensing and holding, 2: swimming, 3: walking, 4: sensing and holding
D. 1: sensing and holding, 2: sensing and holding, 3: swimming, 4: walking

Anatomy of the nervous system of the shrimp


Dissect the shrimp and locate the nerve cord. Answer the following questions.
Question 13. The nerve cord of the shrimp is located at: (2 points)

41

A. Dorsal side of the anterior of the body.


B. Ventral side of the posterior of the body.
C. Ventral side of the whole body of the shrimp.
D. Dorsal side of the whole body of the shrimp.

Question 14. There are 4 types of nervous systems schematically shown in the figure
below.

Which nervous system shown above is identical to the nervous system of the shrimp
you observed? (4.2 points)
A. Nervous system A.

Formatted: Numbered +
Level: 1 + Numbering Style:
A, B, C, + Start at: 1 +
Alignment: Left + Aligned
at: 0.63 cm + Tab after:
1.27 cm + Indent at: 1.27
cm

B. Nervous system B.
C. Nervous system C.

Deleted:
Deleted:

D. Nervous system D.

Page Break

Formatted: Left, Line


spacing: Double

42

16th International Biology Olympiad

Beijing
July, 2005

Practical Examination
Part IV

Total time available: 90 minutes

43

The 16th IBO Practical Tests


First name:
Last name
Country:
Code:
Important:
1. Write your name and code on both the task paper and the answer paper sheets.
Deleted: should be

2. Make sure that all the results are written on the answer paper unless otherwise
instructed.
Deleted: has

3. There are 4 parts in the practical test. Each part lasts 90 min. You should start your
first test according to last digit of your competitor code. For example, if you
have a code of 221, your first practical test will be part I, if you have a code of
223, your first practical test will be part III.
4. Your second practical test is as follows: competitors from part I and part II
exchange labs; competitors from part III and part IV exchange labs;
5. You go to your third practical test according to the following rules:
If the last digit of your competitor code is 1, you go to practical test part III.
If the last digit of your competitor code is 2, you go to practical test part IV.
If the last digit of your competitor code is 3, you go to practical test part I.
If the last digit of your competitor code is 4, you go to practical test part II.
You should follow the instructions from your guides when switching labs.

44

Practical Test, Part IV


Plant Biology
Task 1. Plant anatomy and physiology (20 points)
Materials and tools
You are provided with a set of tools and experimental materials. You will need to use
other tools and instruments, including a stereoscope (stereomicroscope), microscope,
Petri dishes, forceps, slides, slide cover slips, and filter paper.
You are provided with a Petri dish containing an aquatic plant.

Finish the following tasks.

Deleted: O

(1) Initially, observe the plant with a stereoscope and answer questions 1, 2 and 3.

Deleted: y
Deleted: first

(2) Take one plant and put it on a slide: cut some roots off and put them on another

Deleted: through

slide and cover the slide with a cover slip. Press the cover slip slightly and

Deleted: ,
Deleted: d

observe the slide under the microscope. Answer questions 4 and 5.

Deleted: ,

(3) Take one plant and put it on a slide. Cut a leaf and put it on another slide. Cover it

Deleted: c

with a slide cover slip and press it gently. Observe the specimen you made and
Deleted: .

answer questions 6 through 8


Questions 1-3 are about external description of the plant.
Deleted:

1. The stem of the plant is: (2 points)

Deleted:
The stem of the pla

A. Vertical
B. Horizontal

45

Deleted: Rosulate

C. Rosette (shortened stem)


Deleted: Acaulescent (

D. Absent

Deleted: no stem)

2. Which one of the following descriptions about its root is correct? (2 points)
A. It contains chlorophyll
B. It is an adventitious root
C. It is a rhizoid
D. It is a spindle-shaped root.

Deleted: n

3. Which of the following descriptions of its leaves is/are correct? (2 points)


Deleted: ul

(1)

The leaves dont have petioles.

(2)

Their leaves are bipinnate.


Deleted: s

(3)

Some leaves dont have chlorophyll

(4)

There are needle-shaped leaves.

A. 1, 2, 3, 4
B. 1, 2
C. 1, 3
D. 2, 4
E. 1, 2, 3

46

Questions 4-5
4.

Which of the following is correct? (2 points)


A. This plant is a vascular plant
Deleted: s

B. This plant contains vessel elements


C. This plant is a bryophyte based on its root structure.
Deleted: is correct

D. None of the above.

5.

A researcher grew the plant for many generations and found that no seeds were
produced. Which of the following could be true based on your observation? (2
points)
(1)

The researcher could have missed the seeds produced.

(2)

This plant is a seedless plant.


Deleted: type of

(3)

This plant does not have sexual reproduction.

Deleted: s
Deleted:

A. 1, 2, 3

Formatted: Indent: Hanging:


9 ch, Numbered + Level: 3
+ Numbering Style: 1, 2, 3,
+ Start at: 1 +
Alignment: Left + Aligned
at: 3.49 cm + Tab after:
4.13 cm + Indent at: 4.13
cm, Tabs: 9.43 ch, List
tab + Not at 11.14 ch

B. 1, 3
C. 1, 2
D. 2,

Deleted:

E. 3
Questions 6-8.
6.

Besides the plant tissues and cells, you should be able to observe some other
immobile cells. Which of the following descriptions is/are correct about these
cells? (2 points)

47

(1)

They are unicellular.

(2)

They are mostly short non-branched filaments.

(3)

Some of them are branched.

(4)

Their nuclei are easily observed.

A.

1,

B.

1, 2, 3, 4

C.

2, 3

D.

2, 3, 4

E.

7. A researcher grew the plants under different conditions and obtained results
shown in the figure below.

48

Condition 1, grown with medium A containing combined nitrogen (nitrate).


The growth rate under this condition was used as 100% growth.
Condition 2, grown with medium A without combined nitrogen.
Condition 3, grown with medium A containing combined nitrogen.
Ampicillin was added to a concentration of 5 M.
Condition 4, grown with medium A without combined nitrogen. Ampicillin
was added to a concentration of 5 M.

Note, medium A is the standard medium for this plant.

Deleted:

Which of the following statements is/are correct based on the results shown above? (4
points)
(1) Ampicillin is inhibitory to plant growth only under nitrogen limiting
condition.
(2) The plant can grow without combined nitrogen.
(3) The root system of this plant could fix nitrogen.
(4) There are at least some microorganisms associated with the plant and
they can fix nitrogen.
(5) Nitrogenase activity is directly inhibited by Ampicillin.

49

A. 1, 3, 5
B. 1, 5
C. 2,
D. 1, 2, 4
E. 4, 5

8. If you would like to obtain a culture of the plant that does not contain any
associated organisms, what is the condition to achieve it? (4 points)
A. Grow it with combined nitrogen plus some ampicillin.
B. Grow it with combined nitrogen.
C. Grow it without combined nitrogen.
D. Grow it without combined nitrogen plus ampicillin.

50

Task 2 Plant pigment characterization (20 points)


Materials and tools
Deleted: through pigment

You are provided with 6 tubes of pigments, labelled pigment I to VI. You are also
provided with a colorless solution in another tube labelled as control. You will need to
use the following instruments:
Deleted: p

Adjustable Spectrophotometer; Cuvette cells; Adjustable pipettes; Filter paper;

Perform the following tasks:

a. Examine the absorption spectra shown in the figure below. The five absorption
spectra are obtained from different organisms and the pigment names are
given in the figure. The major absorptions of these spectra are given. Among
the five pigments in the figure, phycocyanin and phycoerythrin are watersoluble; chlorophyll and carotene are soluble in organic solvents; chlorophyllprotein complexes are soluble in aqueous solution when treated with detergent.
Deleted: p

b. Use the adjustable pipette to transfer 1 ml of each pigment solution to cuvette


cells. Measure the absorptions at the wavelengths in the table below. Record
the results of your measurements in the table.

Deleted:

51

1.Chlorophyll a in water

2.5
2
1.5

678nm

1
0.5
0
400

450

500

550

600

650

700

750

2.Chlorophyll a in ethanol

3
2.5
2

662nm

1.5
1
0.5
0
400

450

500

550

600

650

700

750

3.Phycocyanin
625nm

2
1.5
1
0.5
0
400

450

500

550

600

650

700

750

52

4.Phycoerythrin
2

562nm
1.5
1
0.5
0
400

450

500

600

650

700

750

5.Carotene

450nm

1.5

550

478nm

0.5

0
400

450

Solution

500

550

450nm

600

562nm

650

700

595nm

750

625nm

662nm

678nm

I
II
III
IV
V
VI

53

Answer the following questions:

Deleted: in

Question 9. Which of the pigments would be most efficient at absorbing red light?
(2 points)
A. Phycocyanin
B. Phycoerythrin
C. Carotene
D. Chlorophyll

Question 10. Phycocyanin solution is: (2 points)


A. Solution I.
B. Solution II.
C. Solution III.
D. Solution IV.
E. Solution V.
F. Solution VI.
G. None
Question 11. Phycoerythrin solution is: (2 points)
A.

Solution I.

B.

Solution II.

C.

Solution III.

D.

Solution IV.

54

E.

Solution V.

F.

Solution VI.

G.

None.

Question 12. Chlorophyll solution (in ethanol) is: (2 points)


A. Solution I.
B. Solution II.
C. Solution III.
D. Solution IV.
E. Solution V.
F. Solution VI.
G. None.
Question 13. Carotene solution is: (2 points)
A. Solution I.
B. Solution II.
C. Solution III.
D. Solution IV.
E. Solution V.
F. Solution VI.
G. None.
Question 14. Protein-Chlorophyll complex in detergent-treated solution is: (2 points)
A. Solution I.
B. Solution II.

55

C. Solution III.
D. Solution IV.
E. Solution V.
F. Solution VI.
G. None.
Question 15. Which of the following pigments (1, 2, 3, or 4)is/are present in all algae
and higher plants? (2 points)
(1) Chlorophyll
(2) Carotene
(3) Phycoerythrin
(4) Phycocyanin

A. 1, 2, 3, 4
B. 1, 3, 4
C. 1,
D. 1, 4
E. 1, 2
Question 16. A cyanobacterium contains chlorophyll, carotenoids and phycocyanin as
major pigments. When a culture of the cyanobacterium is extracted with 80% acetone
and centrifuged, what color do you expect to see in the pellet? (3 points)
A. Orange
B. Blue

56

C. Green
D. Purple
Formatted: Font:

E. Colorless

Question 17. In the analysis of proteins with isoelectric focusing (IEF) gel
electrophoresis, one often uses coloured proteins with known pI (isoelectric point)
values as pI standards. Among these proteins are phycocyanin and phycoerythrin. No
chlorophyll proteins are used as IEF gel standard. Which of the following is the
reason why no chlorophyll-proteins are used as IEF gel pI standard? (3 points)
A. Green colour is not visible in the IEF gel.
B. Chlorophyll molecules are too small to be focused.
C. It is often difficult to obtain enough materials of chlorophyll-proteins from
plants.
D. Chlorophyll molecules are not covalently attached to proteins.

57















All IBO examination questions are published under the following Creative Commons license:



CC BY-NC-SA (Attribution-NonCommercial-ShareAlike) https://creativecommons.org/licenses/by-nc-sa/4.0/
The exam papers can be used freely for educational purposes as long as IBO is credited and
new creations are licensed under identical terms. No commercial use is allowed.

IBO2005 Practical Test Standard Answers


Part 1
Question
1
2
3
4
5

Answer
D
D
B
B
D

Part 2
Question
1
2
3
4
5
6
7
8
9
10
11
12
13
14
15
16
17
18
19
20
21
22
23
24
25

Answer
A
F
G
C
B
E
D
B
C
G
D
E
C
B
E
A
A
B
B
A
B
D
B

Part 3
Question
1
2
3
4
5
6
7
8-1
8-2
8-3
8-4
8-5
8-6
8-7
9
10
11
12
13
14

Answer
D
C
BC
A
C
ACE
B
D
C
A
G
B
F
E
B
C
A
C
C
A

Part 4
Question
1
2
3
4
5
6
7
8
9
10
11
12
13
14
15
16
17

Answer
B
B
C
A
D
E
D
A
D
D
B
F
A
G
E
B
D

INTERNATIONAL BIOLOGY OLYMPIAD


THEORY PROBLEMS

2004, Brisbane, Australia















All IBO examination questions are published under the following Creative Commons license:



CC BY-NC-SA (Attribution-NonCommercial-ShareAlike) https://creativecommons.org/licenses/by-nc-sa/4.0/
The exam papers can be used freely for educational purposes as long as IBO is credited and
new creations are licensed under identical terms. No commercial use is allowed.

COMPETITOR #:

15th International Biology Olympiad


Brisbane, 2004

THEORY EXAMINATION # 1

Total time available: 2 hours (150 minutes)


Total points available: ~80

IBO2004-Theory-1-page-1

GENERAL INSTRUCTIONS

Please check that you have the appropriate examination papers and answer sheets.

It is recommended that you manage your time in proportion to the points allotted for each
question.

IMPORTANT

Use the answer sheets provided to record your answers.


Ensure your name and three digit code number is written on the top of each page of the answers.
Using the pencil provided, fill in the appropriate circle on the answer sheet.
Unless otherwise indicated, there is only ONE correct answer for each question.
Part marks are given and no marks are deducted for incorrect answers.

Good luck.

IBO2004-Theory-1-page-2

Questions 1-5. A young family recently bought a Golden Retriever puppy as a pet. They
took delivery of the dog at 8 weeks of age after the pup had been weaned and vaccinated.

Question 1). Following birth, how did the pup find its way to its mothers teat to suckle?
(1 point)
A.

Using tactile cues

B.

Using visual cues

C.

Using auditory cues

D.

Using olfactory cues

E.

Using taste cues

Question 2). Puppies are vaccinated against a range of pathogenic micro-organisms. Which
host immune responses are stimulated to provide protection? (1 point)
A.

Inflammatory immune responses

B.

Adaptive immune responses

C.

Hypersensitivity immune responses

D.

Innate immune responses

E.

Cytokine immune responses

Question 3). Training the dog to obey commands (such as walk, sit, stay) involves modulating
which behavioural pattern? (1 point)
A.

Imprinting

B.

Conditioning

C.

Mimicry

D.

Habituation

E.

Sensitization

IBO2004-Theory-1-page-3

Question 4). The Golden Retriever breed is an inbred line of dogs. The pedigree below is for a
rare, but relatively mild, hereditary disorder of the skin.

1. How is the disorder inherited? (1 point)


A.

Autosomal, recessive

B.

Autosomal, dominant

C.

Sex-linked, recessive

D.

Sex-linked, dominant

2. If D = dominant and d = recessive, what is the genotype of individual III-5 in the pedigree
given above? (1 point)
A.

DD

B.

Dd

C.

dd

IBO2004-Theory-1-page-4

Question 5). Dogs, foxes, jackals, wolves and hyenas were all scored for the presence (+) or
absence (-) of seven phenotypic characters (I-VII). The results are shown in the following Table.

Character

II

III

IV

VI

VII

Dog

Fox

Jackal

Wolf

Hyena

Which phenogram indicates the phylogenetic relationships between these five animal groups
based on the data given? (1 point).

A.

B.
dog
fox
wolf
jackal
hyena

C.

dog
fox
jackal
wolf
hyena
D.

dog
wolf
fox
jackal
hyena

dog
wolf
fox
jackal
hyena

IBO2004-Theory-1-page-5

Questions 6-10.

A 2800 bp plasmid was cut by restriction enzymes in three separate

reactions: one reaction using BamHI and HindIII; one using BamHI and EcoRI; and one
using HindIII and EcoRI. The restriction fragments were separated on an electrophoretic
gel.

origin of
replication

200

2400

2800 bp
plasmid
2000

800

1600

B
Size Markers

BamHI
+
HindIII

BamHI
+
EcoRI

HindIII
+
EcoRI

1600 bp
1400 bp
1200 bp

1000 bp
800 bp
600 bp
400 bp
200 bp

IBO2004-Theory-1-page-6

Question 6). Which of the following statements is FALSE? (1 point)


A.

Plasmids do not possess protein coats

B.

Plasmids are circular double-stranded DNA molecules

C.

Plasmids can be incorporated into the host cells chromosome

D.

Plasmid genes are required for bacterial survival and/or reproduction

E.

Plasmids are generally beneficial to their host cells

Question 7). Which of the restriction sites on the plasmid map (numbered 1-5) correspond to
the restriction enzymes A, B or C. (2 points)
Restriction enzyme

Site

Answer
[A/B/C]

A.

BamHI

B.

EcoRI

C.

HindIII

3
4
5

Question 8). The four sides of the electrophoretic gel are labelled A, B, C and D. Which
represents the cathode ? (1 point)

A.

B.

C.

D.

E.

not possible to determine


IBO2004-Theory-1-page-7

Question 9). The restriction enzyme EcoRI cuts double-stranded DNA as follows:

5 . . G A A T T C . . 3
..C T T A A G..

Which of the following fragments could bind to an EcoRI restriction cut site? (1 point)
(A)

. . CG

(B)

AATTCG . .

. . GCAATT

GC . .

(C)

. . TGAATT
. . AC

(D)

GT . .
TTAACA . .

Question 10). How do single plasmids come to acquire multiple genes for antibiotic resistance?
(1 point)
A.

Transposition

B.

Conjugation

C.

Transcription

D.

Transformation

E.

Transduction

IBO2004-Theory-1-page-8

Question 11-15. A researcher placed ten individuals of each of three different types of
plants at ten different light intensities, ranging from zero to full sunshine, for several days.
The atmosphere was normal air, the temperature was 32C and the plants were well
watered.
The three plant types were:

a C3 plant adapted to growth in full sunshine (a sun plant)

a C3 plant that can only grow in low-light environments (a shade plant)

a C4 plant which, like most C4 plants, is adapted to growth in full sunshine

The researcher then measured the photosynthetic rate of a leaf of each plant, and plotted
the results for plants A, B and C as follows:

photosynthetic rate (relative units)

C
0
0

20

40

60

80

100

light intensity (% of full sunshine)


In the following questions, A, B or C refers to curves A, B or C above or the plants
corresponding to these curves.

IBO2004-Theory-1-page-9

Question 11). Which type of plants gave results A, B and C? (1 point)


Answer
[A/B/C]
1.

C3 sun plant

2.

C3 shade plant

3.

C4 plant

Question 12). Which result (A, B or C) would be obtained for the following? (1 point)
Answer
[A/B/C]
1. wheat, rice, oats, barley, peas and beans
2. plant that usually has the thinnest leaves
3. plant with highest water use efficiency
4. plant that preferentially invests nitrogen (N) into the
manufacture of thylakoid proteins and chlorophyll,
rather than enzymes of CO2 fixation
5. plant with some chloroplasts lacking Rubisco
(Rubisco = ribulose bisphosphate carboxylase/oxygenase)

IBO2004-Theory-1-page-10

Question 13). Curve C shows that photosynthesis by this plant decreases as the light intensity is
increased from 60% to 100% of full sunshine. Why? (1 point)
A. It lacks chlorophyll a
B. It does not close its stomates when water stressed, and therefore becomes dehydrated
under high light
C. It has insufficient Rubisco to make use of high light, and reactive oxygen species
consequently accumulate and damage membranes
D. High light stimulates its mitochondrial (dark) respiration, consequently it respires more
CO2 at night than it photosynthetically fixes by day
E. Its chloroplasts migrate to the sides of the leaf cells, making the leaves transparent and
unable to absorb light for photosynthesis

Question 14). Leaves from the three plants exposed to 60% of full sunshine would undertake
photosynthesis faster if provided with extra light (L) or extra carbon dioxide (D)?
(1 point)
Answer
[L/D]
1. Plant A
2. Plant B
3. Plant C

IBO2004-Theory-1-page-11

Question 15). Photorespiration occurs in a plants chloroplasts when the O2 concentration is


much greater than the CO2 concentration. In this case, O2 instead of CO2 is incorporated by
Rubisco in the Calvin cycle. Rubiscos substrate, which is normally linked to CO2, is which of
the following: (1 point)

A.

3-phosphoglycerate

B.

glycolate 2-phosphate

C.

glycerate 1,3-bisphosphate

D.

3-phosphoglyceraldehyde

E.

ribulose 1,5-bisphosphate

IBO2004-Theory-1-page-12

Questions 16-20.

Karyotypes represent a display of the chromosomes present in

eukaryotic cells. The following diagram shows a normal human male karyotype.

Question 16). Karyotyping is routinely performed on cells that are undergoing which process?
(1 point)
A.

Prophase in meiosis

B.

Anaphase in mitosis

C.

Metaphase in mitosis

D.

Telophase in mitosis

E.

Interphase

IBO2004-Theory-1-page-13

Question 17). How many autosomes are shown in the picture?


A.

22

B.

23

C.

44

D.

46

E.

(1 point)

Question 18). What are the structures circled at position 6 in the diagram called?...
(1 point)
A.

two sister chromatids

B.

an homologous pair of chromosomes

C.

a chromosome

D.

interphase chromosomes

E.

RNA

Question 19). Which answer gives the correct responses to the blanks in the following
statement?
Human chromosomes are typically divided into two arms by their ....... . On human
chromosome maps, the long arm is referred to as ....., while the short arm is called ......
(1 point)
A.

telomere;

p;

B.

centrosome;

q;

C.

centriole;

L;

D.

centrosome;

l;

E.

centromere;

q;

IBO2004-Theory-1-page-14

Question 20). The technology that produced the karyotype shown in the diagram has been
largely superceded and replaced by which of the following?

(1 point)

A. light microscopy together with colour photography


B. fluorescent in situ hybridizing (FISH) DNA probes called chromosome paints
C. electron microscopy and colour lithograph techniques
D. the polymerase chain reaction
E. RNA silencing using short interfering RNAs (siRNA)

Questions 21-25. A new volcanic island forms in the middle of an ocean. A fringing reef
forms as marine currents bring eggs and larvae allowing different species to colonize.
Other factors, such as predation, competition and diseases, are then responsible for
extinctions of some reef species.

Question 21). On average, the reef is colonized each century by ten new species of coral but
10% become extinct each century. How long will it take for the reef fauna to include at least 50
different species of coral? (1 point)
A.

300 years

B.

450 years

C.

500 years

D.

800 years

E.

1200 years

IBO2004-Theory-1-page-15

Question 22). A major change in the direction of marine currents prevents the reef from being
colonized by any new species. Assuming the extinction rate remains the same at 10% per
century, how long will it take for the biodiversity of the reef to be lowered from 50 coral species
to 30 species? (1 point)

A.

300 years

B.

450 years

C.

500 years

D.

800 years

E.

1200 years

Question 23). A marine biologist, wanting to calculate the number of fish (N) that lived on the
reef, captured a sample of individuals (numbering S1), tagged them and released them. One
month later, he collected another sample (numbering S2) and found several marked individuals
amongst them (numbering S3). Which formula can be used to calculate N? (1 point)

A.

N = (S2 x S3) / S1

B.

N = (S1 x S2 x S3)

C.

N = (S1 x S3) / S2

D.

N = (S1 x S2) / S3

E.

N = (S2 + S1) / S3

IBO2004-Theory-1-page-16

Question 24). Three species of reef organisms exhibit the following survivorship curves:

10000
1000
Number of survivors
(log scale)

Species I

100

Species II

10
1

Species III
50
Percentage of maximum life span

100

Which of the following statements is INCORRECT? (1 point)


A.

Juveniles of species III have a higher mortality than those of species I

B.

Death rates in species II are more uniform than those in species I

C.

It is likely that fecundity in species II is lower than that of species III

D.

Adults of species II have longer life expectancy than those of species III

E.

High adult mortality in species I is likely to be offset by high fecundity

IBO2004-Theory-1-page-17

Question 25). Fish caught on the reef were found to be infected with gill parasites. The
intensities of infection were plotted against their frequency, as follows:

Intensity of
infection

Frequency of infected fish

Which statement can be correctly inferred from these results? (1 point)


A.

Parasites accumulate in older hosts

B.

A few hosts contain most parasites

C.

Young hosts acquire most parasites

D.

Parasite dispersion depends on host distribution

E.

Most hosts contain a few parasites

IBO2004-Theory-1-page-18

Questions 26-30. Early in the evolution of life on Earth, certain primitive photosynthetic
cells acquired the ability to split water resulting in the accumulation of oxygen in the
atmosphere. Other cells made use of oxygen by combining it with other molecules and
harnessing the energy released.

Question 26).

The diagrams below depict a mitochondrion (left) showing some of the

biochemistry that occurs in the matrix (O = enzyme) and a magnified view of the inner
membrane (right) showing some of the multi-protein complexes within it.

B
pyruvate

pyruvate

NADH
+H+
NAD+

acetyl CoA

citrate
oxalo-acetate

O2
2H2O

C
D

succinate
H+

IBO2004-Theory-1-page-19

Match the labelled components A to G with the following items: (2 points)


1. Protein complex that manufactures most of the ATP for which respiration is famous
2. Protein that releases CO2
3. Hydrogen ions (H+) moved during electron transport accumulate here, causing the pH
to be at least 1 unit lower than in the matrix
4. Protein that contains copper as a cofactor
5. Protein that synthesises malate
6. Ubiquinone could be found performing its task here
7. Protein that reduces FAD to FADH2

Question 27). Consider the metabolic cycle of eight reactions (Krebs cycle/citric acid cycle)
shown within the diagram of the mitochondrion (cycling from citrate to succinate to oxaloacetate). Which of the following statements is correct? (1 point)
A. The cycle occurs in the intermembrane space of mitochondria
B. The cycle is a central part of lactic-acid fermentation
C. The cycle would cease to operate in the absence of NAD+
D. The cycle directly generates most of a heterotrophic cells ATP
E. The cycle is responsible for the generation of oxygen

IBO2004-Theory-1-page-20

Question 28).

Acetyl-CoA is formed in mitochondria during respiration. It is also made in

other parts of cells, including the cytosol and plastids in plant cells. It is the main molecule used
for the biosynthesis of which combination of the following? (1 point)

1.

The aromatic amino acids tryptophan, tyrosine and phenylalanine

2.

Steroids

3.

The chitin in fungal cell walls and insect exoskeletons

4.

The hydrophobic tail of ubiquinone

5.

The purine bases of DNA and RNA

6.

Fatty acids

7.

Gibberellin plant hormones

A.

1, 5

B.

2, 4, 6, 7

C.

2, 3, 7

D.

1, 4, 7

E.

2, 3, 5, 6, 7

IBO2004-Theory-1-page-21

Question 29).

Which combination of statements is correct for aerobic respiration? (1 point)

1.

H2O is a reducer

2.

CO2 is an oxidant

3.

O2 is an electron acceptor

4.

H2O is an electron donor for organic substances

5.

H2O is one of the end products

6.

Photophosphorylation occurs

7.

Oxidative phosphorylation occurs

8.

Substrate level phosphorylation occurs

A.

1, 2, 8

B.

2, 3, 4, 7

C.

3, 5, 7, 8

D.

1, 4, 7, 8

E.

2, 3, 5, 6, 7

IBO2004-Theory-1-page-22

Question 30). A student obtained a preparation of isolated mitochondria from a liver and
studied respiration by continuously following the O2 uptake of a 1 mL suspension of
mitochondria in a reaction vessel. Additions were made to the suspension as shown. The
following trace was obtained:

malate (50 moles)


oxygen in reaction vessel (!moles)

300

ADP (50 moles) + phosphate (200 moles)


1

2
2,4-dinitrophenol (1 mmole)
3
4

cyanide (1 mmole)
5

0
1

time

The student offers various conclusions for the rates of O2 uptake shown at positions 1-6.
Select either conclusion A or B as correct for each numbered position: (1 point):

1.

A. The mitochondria were damaged and were no longer able to respire


B. Almost no carbohydrate substrate was available for respiration

IBO2004-Theory-1-page-23

2.

A. Malate acted as a carbohydrate substrate for respiration


B. Malate combined directly with O2, removing the O2 from solution

3.

A. Respiration by the mitochondria was coupled to ATP synthesis


B. Inorganic phosphate (Pi) stimulated the activity of Krebs cycle enzymes

4.

A. ADP slowly inactivated Krebs cycle enzymes


B. The supply of ADP was depleted

5.

A. 2,4-dinitrophenol causes H+ ions to leak across the mitochondrial membrane


B. 2,4-dinitrophenol is better than malate as a natural substrate for respiration

6.

A. Cyanide (CN-) inhibits the terminal oxidase of respiration


B. Cyanide can substitute for O2 in respiratory electron transport,
generating HCN instead of H2O

IBO2004-Theory-1-page-24

Questions 31-35. Most higher animals have cardiovascular systems that transport blood
and fluids to body tissues.

Question 31). Vertebrate whole blood consists of plasma and suspended cells or fragments. The
following statements relate to the composition of a normal blood sample.
1) Red cells get their colour from the waste CO2 carried by haemoglobin
2) Erythrocytes are the most abundant cell type in blood
3) Platelets contain a nucleus and DNA
4) Haemoglobin is composed of two polypeptide chains
5) Gamma-globulin is a key protein in plasma
6) All adult blood cells originate in the bone marrow

Which one of the following combinations contains only true statements? (1 point)
A.

3, 4, 5

B.

2, 5, 6

C.

1, 2, 3, 5, 6

D.

4, 5, 6

E.

2, 4, 6

IBO2004-Theory-1-page-25

Question 32). The haematocrit (or packed cell volume, PCV) is the proportion of the total blood
volume occupied by blood cells. The normal haematocrit range for adult males is 40-50%. The
graph below shows the haematocrit results for three patients A, B and C.

Normal Adult Range

Which patient could a doctor correctly diagnose as suffering from the following conditions?
(use D to indicate none) (1 point)
Answer
[A/B/C/D]
1. Dehydration
2. Anaemia
3. Decreased albumin levels
4. No apparent health problems

IBO2004-Theory-1-page-26

Question 33). The amount of oxygen carried in the blood is dependent on the amount of
respiratory pigment (such as haemoglobin) and the partial pressure of oxygen. For a fixed
concentration of haemoglobin, the greater the partial pressure of oxygen the greater the percent
Saturation of Haemoglobin with O2 (%)

saturation of haemoglobin.

100
80

60
40
20

20

40

60

80

100

O2 Partial Pressure (Torr)

Which of the following combinations represents the oxygen equilibrium curves shown above for
conditions A, B and C, in order? [*Exercise blood is blood collected after a period of extreme
exercise.] (1 point)

A.

normal blood,

foetal blood,

exercise blood*

B.

foetal blood,

exercise blood*,

C.

foetal blood,

normal blood,

exercise blood*

D.

exercise blood*, foetal blood,

normal blood

E.

exercise blood*, normal blood,

foetal blood

normal blood

IBO2004-Theory-1-page-27

Question 34). A major role for blood clotting is to help repair damaged blood vessels and tissue
wounds. Which factors are important in this process? (1 point)
A.

erythrocytes, thrombin, fibrin, gamma globulin

B.

monocytes, gamma globulin, thrombin, erythrocytes

C.

lymphocytes, erythrocytes, fibrin, platelets

D.

platelets, thrombin, erythrocytes, fibrin

E.

fibrin, thrombin, platelets, gamma globulin

Question 35). Various abnormal conditions may arise in the cardiovascular system, including:
A) Cholesterol plaque deposits in arteries
B) Overproduction of red blood cells in the bone marrow
C) Single nucleotide mutation in the coding region of the haemoglobin gene
D) High blood pressure
E) Uncontrolled proliferation of progenitor blood cells
F) Reduction in haemoglobin synthesis
Which of these abnormalities cause the following blood disorders? (1 point)
Answer [A/B/C/D/E/F]
1. Sickle cell anaemia
2. Hypertension
3. Atherosclerosis
4. Beta-thalassemia
5. Leukaemia
6. Polycythemia

IBO2004-Theory-1-page-28

Questions 36-38. Reproduction has many features that vary within the plant kingdom.
Question 36). Which of the following statements concerning reproduction is true?
(1 point)
A. Haploid organisms (i.e. organisms whose cells have only one of each chromosome) do
not exist; the only cells that are haploid are gametes (ova and sperm)
B. It is theoretically and practically impossible for an organism to reproduce without
meiosis
C. Sexual reproduction always involves the production of sperm and ova by separate male
and female individuals
D. All cells of all eukaryotic organisms (even fungi) contain a single nucleus (either haploid
or diploid)
E. Sexual reproduction without meiosis is not possible

Question 37). Which of the following is a principal difference between gymnosperms and
angiosperms? (1 point)
A. Gymnosperms produce flagellated sperm that swim in water whereas angiosperms
produce sperm enclosed in pollen
B. Gymnosperms lack seeds whereas angiosperms have seeds
C. In gymnosperms, the ovule develops into a seed whereas in angiosperms the ovary
develops into a seed
D. Gymnosperms lack ovaries that develop into fruits whereas angiosperms have ovaries
that develop into fruits
E. Gymnosperms produce scaly dry fruit whereas angiosperms produce soft juicy fruit

IBO2004-Theory-1-page-29

Question 38). Plant reproduction was studied in Haplopappus gracilis which is diploid and has
only two pairs of chromosomes, one long pair and one short pair (2n = 4). The diagrams below
represent anaphases of individual cells in meiosis or mitosis in a plant that is genetically a
double heterozygote (Aa Bb) for alleles of two genes, one gene on the long chromosome and the
other on the short chromosome. Single lines represent chromatids, and the points of the V's
represent centromeres.

In each case, indicate if the diagram represents a cell in:


meiosis I (= A)
1

meiosis II (= B)
3

(3 points)

mitosis (= C)
6

or an impossible situation (= D)
8

10

11

12

IBO2004-Theory-1-page-30

Questions 39-42). A dichotomous key was constructed for ten groups of animals based on seven
characters.

bilateral symmetry
endoskeleton
hardened
structure with
attached muscles

radial symmetry

no
endoskeleton

A
B

no symmetry

bilateral
symmetry

aquatic

also terrestrial
has
separate
mouth and
anus
no hardened
structure with
attached
muscles

segmented

not segmented

anus and
mouth not
separate

D
E

F
G

has
gastrovascular
system

aquatic

also terrestrial

H
I

no gastrovascular
system

IBO2004-Theory-1-page-31

Question 39). Which groups (A-J) are represented by numbers 1 to 10? (2 points)
Group

Letter

1.

Annelida (annelids)

2.

Arthropoda (crustaceans)

3.

Cnidaria (jelly fish)

4.

Echinodermata (starfish)

5.

Mollusca (bivalves)

6.

Mollusca (gastropods)

7.

Chordata (chordates)

8.

Nematoda (roundworms)

Platyhelminthes (flatworms)

10. Porifera (sponges)

Question 40). Animal body plans vary in the organization of their tissue layers and body cavities. They
may be diploblastic (D) or triploblastic (T). They may be acoelomate (A), pseudocoelomate (P) or
coelomate (C). Indicate the character states for the following animals: (1 point)

1.

Platyhelminthes (flatworms)

2.

Annelida (segmented worms)

3.

Nematoda (roundworms)

tissue layers

body cavities

[D/T]

[A/P/C]

IBO2004-Theory-1-page-32

Question 41). Differences in embryological development allow the differentiation of animals into
protostomes and deuterostomes based on cleavage patterns (radial = R; spiral = S), cell fate
(indeterminate = I, determinate = D) and the development of the mouth of the embryo (arising from
blastopore = B, arising at opposite end to blastopore = O). Which are the correct character states for
deuterostomes? (1 point)
A.

S, D, O

B.

S, D, B

C.

S, I, O

D.

S, I, B

E.

R, D, O

F.

R, D, B

G.

R, I, O

H.

R, I, B

Question 42). Insects may have simple or complex development cycles.


A. They may hatch from eggs looking like small adults.
B. They may undergo gradual changes (incomplete metamorphosis)
C. They may undergo profound changes (complete metamorphosis).

Which developmental cycles do the following insects undergo ? (1 point)

Answer
[A/B/C]
1. Flies
2. Wingless insects
3. Fleas
4. Lice

IBO2004-Theory-1-page-33

Questions 43-47. Bacteria are prokaryotic micro-organisms that grow under a variety of
environmental conditions.

Question 43). Bacterial growth under laboratory conditions at a particular temperature can be
represented as the log number of viable cells plotted against time of incubation.

C
Log (number of cells)
D
B

A
Time

Match the four stages labelled A to D on the graph with the following growth phases. Note: one
phase is not shown and should be entered as E. (1 point)
Growth Phase

Answer
[A/B/C/D/E]

1.

exponential (log) growth phase

2.

death phase

3.

transition phase

4.

stationary phase

5.

lag phase

IBO2004-Theory-1-page-34

Question 44). Most known bacteria grow in the temperature range of 30 - 40C, but thermophiles are
able to survive at temperatures of up to 80C. Indicate which of the following are reasons for the survival
of thermophiles by answering Yes or No. (1 point)

Answer
[Yes/No]
1.

large size

2.

small size

3.

cell membrane composition

4.

accelerated cell repair mechanisms

5.

protein thermostability

Question 45).

Most bacteria (mesophiles) do not tolerate extreme conditions, whereas

extremophiles can survive in environments of high salt, high pressure or extreme temperature
(as shown in the following three graphs).

A) C

D
E

Growth Rate

Salt concentration (M)

IBO2004-Theory-1-page-35

Q) C

Growth Rate

40
Temperature (C)

80

Growth Rate

Hydrostatic Pressure (log atm)

Match the growth characteristics labelled A to F with the following types of bacteria. (1 point).
Type of bacterium

Answer
[A/B/C/D/E/F]

1.

Barophile

2.

Mesophile only

3.

Thermophile only

4.

Halophile only

5.

Psychrophile

6.

Thermophile, halophile

IBO2004-Theory-1-page-36

Question 46).

A Lineweaver-Burk (double reciprocal) plot is shown below depicting the

activity of a bacterial enzyme alone and in the presence of two different substances, A and B.
The y-intercept of each line represents 1/VMAX for the enzyme under the different conditions.

Enzyme plus
substance A
Enzyme plus
substance B
1/V
Enzyme
(without substance
A or B)

1/[S]

Which of the following statements about this enzyme is correct with regard to the
Lineweaver-Burk plot shown above? (1 point)
A. Substance A is a non-competitive inhibitor while substance B is a competitive
inhibitor
B. Substance A is a competitive inhibitor while substance B is a non-competitive inhibitor
C. Substance A is a stimulatory cofactor while substance B is a competitive
inhibitor
D. Substance B is a stimulatory cofactor while substance A is a competitive
inhibitor
E. Both substance A and substance B are stimulatory cofactors

IBO2004-Theory-1-page-37

Question 47).

One way bacterial proteins can maintain stability under extreme conditions is

through disulfide bond bridges between cysteine amino acids. The results below are from an
analysis of a bacterial protein using the reducing agent -mercaptoethanol (BME), which
reduces disulfide bonds. The polypeptide products were separated on the basis of molecular
mass by SDS-polyacrylamide gel electrophoresis. Markers = proteins of known molecular mass
(in kDa).
BME

+BME

Markers
130 kDa
94

67
43
30
20

Based on these results, which of the following diagrams best represents the polypeptide
structure of the native bacterial protein? (Note: represents a disulfide bond) (1 point)

A.

B.

R)

C.

D.

E.

IBO2004-Theory-1-page-38

Questions 48-52. Lipids are vital to cell structure and metabolism; yet fats have a
notorious reputation as being bad for your health.

Questions 48). Which one of the following possible causes of the development of obesity is
INCORRECT? (1 point)
A. Sedentary lifestyle
B. Abnormal hypothalamus function
C. Environmental factors
D. Increased thyroid hormone levels
E. High caloric intake

Question 49). Which statement regarding the hormonal control of fat deposition in adipose
tissue is correct? (1 point)
A. Insulin promotes the formation of triacylglycerol in adipose tissue
B. Insulin promotes the formation of cholesterol in adipose tissue
C. Adrenaline (epinephrine) promotes the formation of triacylglycerol in adipose tissue
D. Adrenaline (epinephrine) promotes the formation of cholesterol in adipose tissue
E. Glucagon promotes the formation of triacylglycerol in adipose tissue
F. Glucagon promotes the formation of cholesterol in adipose tissue

IBO2004-Theory-1-page-39

Question 50). Mitochondria are the primary site in cells for the metabolism of long-chain fatty
acids, and use a process called -oxidation. One cycle of -oxidation of a fatty acid, which has
been prior activated to a coenzyme A (CoA) ester, is shown below:

Based on this diagram, how many cycles of the pathway would be needed for complete
-oxidation of stearic acid (C18:0)? (1 point)
A.

B.

C.

D.

E.

16

F.

18

IBO2004-Theory-1-page-40

Question 51). The melting temperature of a fat-containing substance is dependent on the degree
of unsaturation of the constituent fatty acid chains unsaturated fatty acids reduce the regular
packing together of fatty acids. Examples of two common 18-carbon fatty acids, stearic and
oleic, with different degree of unsaturation, are shown below in diagrammatic form:

Which one of the following substances would you expect to have the highest ratio of
unsaturated/saturated fatty acids? (1 point)
A.

butter

B.

vegetable oil

C.

margarine

D.

peanut paste

E.

cake of soap

IBO2004-Theory-1-page-41

Question 52). In humans, many lipids, including cholesterol, are carried in the blood in large
spherical-shaped lipid-protein complexes called lipoproteins. Their structure reflects the
hydrophilic, hydrophobic or other functional properties of the constituent molecules. Given the
major components of lipoproteins below, predict whether they would be in the core (C) or
surface (S) layer of lipoproteins. (1 point)

Answer
C or S
1. Proteins involved in tissue receptor binding
2. Cholesterol
3. Cholesterol ester (cholesterol esterified to a fatty acid)
4. Phospholipid
5. Triacylglycerol

IBO2004-Theory-1-page-42

Questions 53-57. Photosynthesis occurs in plants when their chloroplasts capture light
energy and convert it to chemical energy stored in sugar and other organic molecules.

Question 53). From several parts of pea plants that had been either kept in light for 12 hours or
in the dark for 12 hours, mRNA was extracted and used in Northern blots with five different
gene probes. The results are shown below.

Leaf

Stem

Root

Tendril

Probe

light

dark

light

dark

light

dark

light

dark

++++

++

+++

+++

+++

++

+++

+++

+++

+++

++

+++

++++

+++

+++

+++

++

++

+++

+++

Which result best indicates that the Rubisco small subunit gene was used as a probe? (1 point)
A.

1.

B.

2.

C.

3.

D.

4.

E.

5.

IBO2004-Theory-1-page-43

Question 54). The main types of plants are C3 plants, C4 plants and CAM plants (CAM stands
for Crassulacean Acid Metabolism). Match the ten items listed in the left-hand column with the
ten corresponding items in the right-hand column. (2 points)

1.

CAM plants at night

A. amylose and amylopectin

2. CAM plants during dry hot days

B. unbranched glucose polymer

3. plastoquinone

C. CO2 fixation followed by malate accumulation


in large vacuoles

4. CO2 compensation point

D. components of thylakoid membranes polar


lipids

5. cellulose

E. UV filters and animal attractants

6. linoleic acid and galactose

F. CO2 evolved when two glycines are converted


to one serine

7. photorespiration

G. stored in organelle with a surrounding


membrane of a monolayer of polar lipids

8. flavonoids

H. electron transporting molecule

9. oils

I. photosynthesis proceeds despite closed stomates

10. starch

J. respiration + photorespiration = photosynthesis

1.

2.

3.

4.

5.

6.

7.

8.

9.

10.

IBO2004-Theory-1-page-44

Question 55). Plants, directly or indirectly, supply all of humanitys food. Match the crops
listed in the left-hand column with the name of the plant organ forming most or all of the
material actually consumed (listed in the right-hand column): (1 point)
1. broccoli

A. axillary bud

2. ginger

B. stem

3. Brussels sprout

C. modified stem (tuber)

4. pumpkin

D. receptacle

5. celery

E. leaves

6. carrot

F. modified stem (rhizome)

7. potato

G. fruit

8. strawberry

H. inflorescence

9. onion.

I. petioles

10. kohl rabi

J. root

1.

2.

3.

4.

5.

6.

7.

8.

9.

10.

Question 56). The accumulation of salt in soil is a major limitation to agriculture. What is the
primary cause of the inability of non-halophyte (i.e. salt-sensitive) plants to grow in soils of
high salt concentration? (1 point)
A. Salt crystals form in the stomata and stop gas exchange
B. Large quantities of Na+ and Cl- enter the plant cells and poison them
C. Salt accumulates in root cells and osmosis follows, causing the cells to burst
D. The oxygen content of the soil is too low
E. The water potential of the soil is too low

IBO2004-Theory-1-page-45

Question 57). Plants obtain from the soil various mineral nutrients with important physiological
roles. Match each of the phrases in the right-hand column with one of the nutrients listed in the
left-hand column. (2 points).

1. calcium

A. cation important in the development of stomatal guard cell turgor

2. nitrogen

B. the form of nitrogen usually available for uptake by plants in


natural ecosystems

3. nitrate

C. required for the biosynthesis of the side chains of the amino acids
cysteine and methionine

4. iodine

D. component of all amino acids, nucleotides and chlorophylls

5. phosphate

E. the "central" atom in a chlorophyll molecule

6. magnesium

F. allows for cell wall cross-linking through pectates

7. potassium

G. important component of DNA and RNA, but not of purine or


pyrimidine bases

8. sulfate

H. most common metal ion in electron-transporting proteins

9. manganese

I. a principal function is in photosynthetic water-splitting

10. iron

J. it is not essential for plant growth

1.

2.

3.

4.

5.

6.

7.

8.

9.

10.

IBO2004-Theory-1-page-46

Questions 58-62. Eukaryotic cells contain many different organelles performing a variety of
functions. The following diagram shows several organelles within a typical animal cell.

Question 58). Identify organelles 1-4 using names selected from the following list. (1 point)
A.

nucleus

B.

Golgi apparatus

C.

plasma membrane

D.

mitochondrion

E.

vesicle

F.

endoplasmic reticulum

G.

centrosome
1.

2.

3.

4.

IBO2004-Theory-1-page-47

Question 59). Which diagram shows the correct organelle interactions for phagocytosis?
(1 point)

IBO2004-Theory-1-page-48

Question 60). In multicellular eukaryotes, adjacent cells are connected to each other to form
tissues and organs. Which of the following is NOT an example of an intercellular connection?
(1 point)
A.

tight junction

B.

plaque junction

C.

plasmodesma

D.

desmosome

E.

gap junction

Question 61). Organelles can be divided into functional groups. Which of the following
functional groupings is correct? (1 point)
A.

nucleus, microfilaments and plasma membrane regulate


cell architecture and movement

B.

endoplasmic reticulum, Golgi apparatus and mitochondria


maintain membrane functionality and secretion

C.

nucleus, ribosomes and smooth endoplasmic reticulum


process genetic information

D.

endoplasmic reticulum, transport vesicles and centrioles


maintain membrane functionality and secretion

E.

microtubules, microfilaments and intermediate filaments


regulate cell architecture and movement

IBO2004-Theory-1-page-49

Question 62). Match the following diagrams to the chemicals listed. (1 point)

Answer
[A/B/C/D/E]
1. Lipid
2. Functional protein
3. Nucleotide
4. Polysaccharide
5. Monosaccharide

IBO2004-Theory-1-page-50

Questions 63-67). The diagram below shows a section through a mammalian ovary.

Question 63). What is the correct sequence in which the labelled structures develop? (1 point)
A.

A-B-D-C-E

B.

D-B-E-A-C

C.

B-D-C-E-A

D.

C-E-B-D-A

E.

E-B-D-C-A

Question 64). Hormonal changes during normal pregnancy are correctly described by which of
the following statements? (1 point)
A.

Estriol (estrogen and progesterone metabolite) excretion is greatest just before


parturition

B.

Human chorionic gonadotropin secretion is greatest in the last 3 months

C.

Human chorionic somatomammotropin secretion is greatest in the first 3 months

D.

Oxytocin secretion is greatest in the middle 3 months

E.

Pregnanediol excretion is greatest in the first 3 months

IBO2004-Theory-1-page-51

Question 65). Which of the listed functions do these hormones play in breast development and
lactation? (1 point)

Hormone

Function

1) Progesterone

A. Plays a background role in breast development

2) Estradiol

B. Stimulates development of alveolar components

3) Prolactin

C. Stimulates growth of ductal system

4) Oxytocin

D. Stimulates milk let-down

5) Insulin

E. No role

Question 66). Where does fertilization of the oocyte by sperm usually occur? (1 point)
A.

Vagina

B.

Cervix

C.

Uterus

D.

Fallopian tubule

E.

Ovary

Question 67). Which of the following act to block polyspermy in humans? (1 point)
A.

Fusion of egg and sperm

B.

Oocyte membrane depolarization

C.

Swelling of fertilized oocyte

D.

Release of cortical granules

E.

All of above

IBO2004-Theory-1-page-52

Questions 68-72. Antibodies are produced by B-lymphocytes when animals encounter


foreign antigenic material. Different antibody isotypes are formed at different times after
exposure; early transient responses are followed by more persistent responses. Antibodies
formed soon after infection can be denatured in the laboratory by treatment with 2mercapto-ethanol while those occurring later are unaffected. You are provided with six
serum samples collected serially from one individual animal.

Serum 1 - collected before immunization with Brucella abortus


Serum 2 - collected 6 days after immunization with Brucella abortus
Serum 3 - collected 42 days after immunization with Brucella abortus
Serum 4 - Serum sample 2 treated with 2-mercapto-ethanol
Serum 5 - Serum sample 3 treated with 2-mercapto-ethanol
Serum 6 - collected 42 days after subsequent immunization with Brucella melitensis

You test the sera against 2 antigens and obtain the following results:

Specimen

Brucella abortus antigen

Brucella melitensis antigen

Serum 1

negative (-)

negative (-)

Serum 2

positive (+)

negative (-)

Serum 3

positive (+)

negative (-)

Serum 4

negative (-)

negative (-)

Serum 5

positive (+)

negative (-)

Serum 6

positive (+)

positive (+)

IBO2004-Theory-1-page-53

Question 68). Which of the following statements is true? (1 point)


A.

The animal has shown an immune response to neither Brucella abortus nor to
Brucella melitensis

B.

No immune response resulted from the immunization with Brucella melitensis

C.

An immune response to the immunization with Brucella abortus has been proven
by these tests

D.

The animal has been previously exposed to both Brucella abortus and to
Brucella melitensis

E.

The animal has been previously exposed to either Brucella abortus or to Brucella
melitensis

Question 69. Which of the following statements is true? (1 point)


A.

Serum 4 and serum 6 can be used to differentiate cultures of Brucella


abortus from those of Brucella melitensis

B.

Brucella abortus and Brucella melitensis have been proven to be the same
organism by these tests

C.

The sera tested have no specificity for either Brucella abortus or for Brucella
melitensis

D.

IgG antibodies to both Brucella abortus and to Brucella melitensis are likely
to be present in Serum 6

E.

No conclusions regarding the specificity of any antibodies can be made from the
data

IBO2004-Theory-1-page-54

Question 70). Subsequent testing of the sera with cultures of Yersinia enterocolitica showed that
sera 3, 5 and 6 gave positive (+) reactions. Which of the following statements is true? (1 point)
A.

These sera can be used as antisera for identifying Yersinia enterocolitica in


culture

B.

Yersinia enterocolitica and Brucella abortus share antigenic determinants

C.

The sera tested have no specificity for either Brucella abortus, for Brucella
melitensis or for Yersinia enterocolitica

D.

Brucella abortus, Brucella melitensis and Yersinia enterocolitica have been


proven to be the same organism by these tests

E.

Antibodies specific to Yersinia enterocolitica have been shown to react in these


tests

Question 71). Which of the following statements is true? (1 point)


A.

IgG antibodies to Brucella abortus have been demonstrated in Serum 2

B.

IgM antibodies to Brucella melitensis have been demonstrated in Serum 6

C.

IgM and IgG antibodies to Brucella abortus have been demonstrated in these
tests

D.

IgM antibodies to both Brucella abortus and to Brucella melitensis have been
demonstrated by these tests

E.

IgM antibodies to both Brucella abortus and to Yersinia enterocolitica have been
demonstrated by these tests

IBO2004-Theory-1-page-55

Question 72). From the above reactions, which of the following could be assumed to contain
IgM antibodies specific for Brucella abortus? (1 point)
A.

Serum 2 and Serum 3

B.

Serum 2 and Serum 4

C.

Serum 2 and Serum 5

D.

Serum 2 and Serum 1

E.

None of the above

Questions 73-77. In the following diagrams, you are given the results of six electrophoretic
gel separations. Bands are shown in numbered lanes in polyacrylamide protein gels,
Western blots, Southern blots and Northern blots and spots are numbered on twodimensional protein gels. Use the information to answer the following questions.

IBO2004-Theory-1-page-56

Question 73). A research scientist had designed a probe that recognizes two distinct fragments
of a gene when digested genomic DNA is examined. Which of the following would confirm
that a section of DNA has been lost from this gene in the cells of a tumour compared to normal
tissue? (1 point)

A. When comparing lane 10 from the tumour and lane 11 from normal tissue
B. When comparing lane 11 from the tumour and lane 10 from normal tissue
C. There would be no change in the observed band pattern
D. When comparing lane 14 from the tumour and lane 13 from normal tissue
E. When comparing lane 14 from the tumour and lane 16 from normal tissue

Question 74). Which of the following would suggest that the cause of a patient suffering a
metabolic disease is caused by changes in the post-translational modification of a protein
involved in the metabolic pathway associated with the disease ? (1 point)

A. Lane 5 from normal tissue with lane 6 from affected tissue


B. Lane 8 from normal tissue with lane 6 from affected tissue
C. Spots 21, 22 and 23 from normal tissue compared to spots 26, 27 and 28 from
affected tissue
D. Spots 31, 32 and 33 from normal tissue compared to spot 25 from affected tissue
E. Spot 20 from normal tissue compared to spot 29 from affected tissue

IBO2004-Theory-1-page-57

Question 75). Which of the following best supports the hypothesis that the developmental
change seen in two different tissues of an embryo is regulated by controlling transcription? (1
point)

A. Comparing lane 2 (brain) with lane 3 (limb bud)


B. Comparing spots 21, 22 and 23 in brain to spot 29 in limb bud.
C. Comparing lane 14 in brain and lane 15 in a limb bud
D. Compare lane 8 (brain) with lane 6 (limb bud)
E. Compare lane 11 (brain) with lane 12 (limb bud)

Question 76). Which lanes on panels A, B, C and D best represent the experiments in which
cells have been transfected with a plasmid encoding a recombinant protein?
[First, the expressed RNA that had been transcribed from the plasmid was analysed by a specific
nucleic acid probe. The expressed protein was then purified and visualized to confirm purity.
Finally, the expressed protein was reacted with a specific antibody probe.]

A.

14, 2 and 6

B.

1, 6 and 9

C.

14, 2 and 8

D.

13, 9 and 5

E.

9, 8 and 2

(1 point)

IBO2004-Theory-1-page-58

Question 77). A gene Z has two promoters. One promoter (PI) is used when the cell is
stimulated by a particular steroid hormone, while the second promoter (PII) is used when the
steroid hormone is absent. The single mRNA from transcription using PII is much shorter than
that of the single mRNA transcript generated when PI is used.

PI

PII

General promoter region

Coding region Z

Assuming no degradation, which two lanes best represents the results of an experiment in which
the RNA transcript from gene Z has been specifically probed with a nucleic acid specific to
gene Z mRNA? [The source of the RNA for the blot comes from cells treated with the steroid
hormone compared to RNA from untreated cells.] (1 point)

A. Comparing lane 15 (steroid-treated cells) and lane 13 (untreated cells)


B. Comparing lane 13 (steroid-treated cells) and lane 14 (untreated cells)
C. Comparing lane 13 (steroid-treated cells) and lane 15 (untreated cells)
D. Comparing lane 14 (steroid-treated cells) and lane 13 (untreated cells)
E. None of the above

END

IBO2004-Theory-1-page-59















All IBO examination questions are published under the following Creative Commons license:



CC BY-NC-SA (Attribution-NonCommercial-ShareAlike) https://creativecommons.org/licenses/by-nc-sa/4.0/
The exam papers can be used freely for educational purposes as long as IBO is credited and
new creations are licensed under identical terms. No commercial use is allowed.

ANSWER SHEET - Theory 1


Q.
1.
2.
3.
4.
5.
6.
7.

8.
9.
10.
11.

12.

13.
14.

15.
16.
17.
18.
19.
20
21.
22.
23.
24.
25.
26.

part

1.
2.

1.
2.
3.
4.
5.

1.
2.
3.
1.
2.
3.
4.
5.
1.
2.
3.

1.
2.
3.
4.

choices

answer

ABCDE
ABCDE
ABCDE
ABCD
deleted
ABCD
ABCDE
ABC
ABC
ABC
ABC
ABC
ABCDE
ABCD
ABCDE
ABC
ABC
ABC
ABC
ABC
ABC
ABC
ABC
ABCDE
LD
LD
LD
ABCDE
ABCDE
ABCDE
ABCDE
ABCDE
ABCDE
ABCDE
ABCDE
ABCDE
ABCDE
ABCDE
ABCDEFG
ABCDEFG
ABCDEFG
ABCDEFG

D
B
B
AorB

worth
(points)
1
1
1
1

C
D
A
C
A
C
B
B
B
A
B
C
A
B
C
A
C
A
C
L
D
D
E
C
C
B
E
B
D
C
D
E
B
G
C
B
F

1
1
2/5
2/5
2/5
2/5
2/5
1
1
1
1/3
1/3
1/3
1/5
1/5
1/5
1/5
1/5
1
1/3
1/3
1/3
1
1
1
1
1
1
1
1
1
1
1
2/7
2/7
2/7
2/7

5.
6.
7.
27.
28.
29.
30.

31.
32.

33.
34.
35.

36.
37.
38.

39.

1.
2.
3.
4.
5.
6.
1.
2.
3.
4.

1.
2.
3.
4.
5.
6.

1.
2.
3.
4.
5.
6.
7.
8.
9.
10.
11.
12.
1.
2.
3.
4.
5.
6.
7.

ABCDEFG
ABCDEFG
ABCDEFG
ABCDE
ABCDE
ABCDE
AB
AB
AB
AB
AB
AB
ABCDE
ABCD
ABCD
ABCD
ABCD
ABCDE
ABCDE
ABCDEF
ABCDEF
ABCDEF
ABCDEF
ABCDEF
ABCDEF
ABCDE
ABCDE
ABCD
ABCD
ABCD
ABCD
ABCD
ABCD
ABCD
ABCD
ABCD
ABCD
ABCD
ABCD
ABCDEFGHIJ
ABCDEFGHIJ
ABCDEFGHIJ
ABCDEFGHIJ
ABCDEFGHIJ
ABCDEFGHIJ
ABCDEFGHIJ

A
E
D
C
B
C
B
A
A
B
A
A
B
C
B
D
A
C
D
C
D
A
F
E
B
E
D
D
B
B
B
C
D
D
D
D
A
D
D
F
E
H
B
D
C
A

2/7
2/7
2/7
1
1
1
1/6
1/6
1/6
1/6
1/6
1/6
1
1/4
1/4
1/4
1/4
1
1
1/6
1/6
1/6
1/6
1/6
1/6
1
1
1/4
1/4
1/4
1/4
1/4
1/4
1/4
1/4
1/4
1/4
1/4
1/4
1/5
1/5
1/5
1/5
1/5
1/5
1/5

40.

41.
42.

43.

44.

45.

46.
47.
48.
49.
50.
51.
52.

53.
54.

8.
9.
10.
1.
2.
3.
1.
2.
3.
4.
1.
2.
3.
4.
5.
1.
2.
3.
4.
5.
1.
2.
3.
4.
5.
6.

1.
2.
3.
4.
5.
1.
2.
3.
4.
5.
6.
7.

ABCDEFGHIJ
ABCDEFGHIJ
ABCDEFGHIJ
DTAPC
DTAPC
DTAPC
ABCDEFGH
ABC
ABC
ABC
ABC
ABCDE
ABCDE
ABCDE
ABCDE
ABCDE
YN
YN
YN
YN
YN
ABCDEF
ABCDEF
ABCDEF
ABCDEF
ABCDEF
ABCDEF
ABCDE
ABCDE
ABCDE
ABCDEF
ABCDEF
ABCDE
CS
CS
CS
CS
CS
ABCDE
ABCDEFGHIJ
ABCDEFGHIJ
ABCDEFGHIJ
ABCDEFGHIJ
ABCDEFGHIJ
ABCDEFGHIJ
ABCDEFGHIJ

G
I
J
TA
TC
TP
G
C
AorB
C
AorB
B
D
E
C
A
N
N
Y
Y
Y
D
E
F
B
C
A
B
B
D
A
C
B
S
S
C
S
C
D
C
I
H
J
B
D
F

1/5
1/5
1/5
1/3
1/3
1/3
1
1/4
1/4
1/4
1/4
1/5
1/5
1/5
1/5
1/5
1/5
1/5
1/5
1/5
1/5
1/6
1/6
1/6
1/6
1/6
1/6
1
1
1
1
1
1
1/5
1/5
1/5
1/5
1/5
1
2/10
2/10
2/10
2/10
2/10
2/10
2/10

55.
56.
57.

58.

59.
60.
61.
62.

63.
64.
65.

66.
67.
68.
69.
70.
71.
72.
73.
74.
75.
76.
77.

8.
ABCDEFGHIJ
9.
ABCDEFGHIJ
10.
ABCDEFGHIJ
deleted
ABCDE
1.
ABCDEFGHIJ
2.
ABCDEFGHIJ
3.
ABCDEFGHIJ
4.
ABCDEFGHIJ
5.
ABCDEFGHIJ
6.
ABCDEFGHIJ
7.
ABCDEFGHIJ
8.
ABCDEFGHIJ
9.
ABCDEFGHIJ
10.
ABCDEFGHIJ
1.
ABCDEFG
2.
ABCDEFG
3.
ABCDEFG
4.
ABCDEFG
ABCDEF
ABCDE
ABCDE
1.
ABCDE
2.
ABCDE
3.
ABCDE
4.
ABCDE
5.
ABCDE
ABCDE
deleted
1.
ABCDE
2.
ABCDE
3.
ABCDE
4.
ABCDE
5.
ABCDE
ABCDE
ABCDE
ABCDE
ABCDE
ABCDE
ABCDE
deleted
ABCDE
ABCDE
ABCDE
ABCDE
ABCDE

E
G
A

2/10
2/10
2/10

E
F
D
B
J
G
E
A
C
I
H
C
E
B
F
B or F
B
E
D
B
E
C
A
E

1
2/10
2/10
2/10
2/10
2/10
2/10
2/10
2/10
2/10
2/10
1/4
1/4
1/4
1/4
1
1
1
1/5
1/5
1/5
1/5
1/5
1

B
C
B
D
A
D
E
C
D
B
C

1/5
1/5
1/5
1/5
1/5
1
1
1
1
1
1

B
D
C
A
E

1
1
1
1
1















All IBO examination questions are published under the following Creative Commons license:



CC BY-NC-SA (Attribution-NonCommercial-ShareAlike) https://creativecommons.org/licenses/by-nc-sa/4.0/
The exam papers can be used freely for educational purposes as long as IBO is credited and
new creations are licensed under identical terms. No commercial use is allowed.

COMPETITOR #:

15th International Biology Olympiad


Brisbane, 2004

THEORY EXAMINATION # 2

Total time available: 2 hours (150 minutes)


Total points available: ~80

IBO2004-Theory-2-page-1

GENERAL INSTRUCTIONS

Please check that you have the appropriate examination papers and answer sheets.

It is recommended that you manage your time in proportion to the points allotted for each question.

IMPORTANT

Use the answer sheets provided to record your answers.

Ensure your name and three digit code number is written on the top of each page of the answers.

Using the pencil provided, fill in the appropriate circle on the answer sheet.

Unless otherwise indicated, there is only ONE correct answer for each question.

Part marks are given and no marks are deducted for incorrect answers.

Good luck.

IBO2004-Theory-2-page-2

Questions 81-85. A 14 year old teenager presents to the family doctor with numerous facial
pimples. The doctor diagnoses a moderate case of acne.

Question 81). Which factor predisposes towards acne? (1 point)


A.

Chocolate

B.

Greasy foods

C.

Puberty

D.

Dry skin

E.

Eczema

Question 82). Acne is characterized by inflamed skin. Which of the following is NOT one of the
principal signs of inflammation? (1 point)
A.

Calor (heat)

B.

Rubor (redness)

C.

Pallor (paleness)

D.

Dolor (pain)

E.

Tumor (swelling)

IBO2004-Theory-2-page-3

Question 83). Acne is caused by infection of the sebaceous glands by Propionibacterium acne. It is
a Gram-positive bacterium which means its cell wall is composed of which of the following?
(1 point)
A.

Thick lipopolysaccharide layer

B.

Thick peptidoglycan layer

C.

Thin lipopolysaccharide layer over a thick peptidoglycan layer

D.

Thick lipopolysaccharide layer over a thin peptidoglycan layer

E.

Thick lipopolysaccharide layer over a thick peptidoglycan layer

Question 84). Which immune cells are delivered to the site of infection to phagocytose invading
bacteria? (1 point)
A.

Lymphocytes

B.

Neutrophils

C.

Eosinophils

D.

Basophils

E.

Plasma cells

Question 85). Treatment with antibiotic drugs helps overcome many bacterial infections. Why is
penicillin toxic to many bacteria? (1 point)
A.

It interferes with DNA replication

B.

It inhibits transcription

C.

It disrupts translation

D.

It blocks protein synthesis

E.

It inhibits cell wall formation

IBO2004-Theory-2-page-4

Questions 86-90. One of the most controversial issues in modern-day biology is the genetic
modification of agricultural crop plants that supply food products for human consumption.
Biologists should be aware of the underlying science of genetic modification.

Question 86). Indicate whether each statement concerning genetically modified (GM) crops in
2004 is True (T) or False (F):

(1 point)
Answer
[T/F]

1. Products from GM crops are now commonly consumed by humans


2. Crops genetically modified to resist attacks by insect larvae are now
produced commercially
3. The consumption of food from GM crops must be dangerous because
transgenic DNA is consumed
4. Many crop plants have been genetically modified to express genes
of Calvin cycle enzymes at higher than normal levels so that these
plants photosynthesise faster
5. Researchers have genetically modified rice to express in developing
grains genes for enzymes that synthesise -carotene (the natural
precursor of vitamin A)

IBO2004-Theory-2-page-5

Question 87). Indicate which of the following major crops have been genetically modified and are
now widely grown commercially by answering Yes (Y) or No (N).

(1 point)

Answer
[Y/N]
1. Maize
2. Wheat
3. Soybean
4. Potato
5. Canola

Question 88). Transgenic plants, like transgenic animals, are genetically-engineered varieties
containing one or more artificially inserted genes. Which of the following methods are used to
introduce genes into crop plants? (1 point)
1.

Agrobacterium tumefaciens infection

2.

Electroporation

3.

Retroviral infection of stem cells

4.

Plant tissue culture

5.

Microinjection

A.

1,3

B.

1,2,5

C.

2,4,5

D.

4,5

E.

all five

IBO2004-Theory-2-page-6

Question 89). Genetic engineers construct recombinant DNA molecules using two enzymes:
restriction endonuclease and DNA ligase. What do these two enzymes do? (1 point)

A. They catalyse different reactions: restriction endonuclease joins fragments into larger
molecules; DNA ligase hydrolyses DNA into smaller fragments
B. They catalyse different reactions: restriction endonuclease hydrolyses DNA into smaller
fragments; DNA ligase joins fragments into larger molecules
C. They both hydrolyse DNA into smaller fragments
D. They both join fragments of DNA into larger molecules
E. They catalyse different hydrolysis reactions: restriction endonuclease hydrolyses bacterial
plasmid DNA; DNA ligase hydrolyses DNA from eukaryotic cells

IBO2004-Theory-2-page-7

Question 90). Scientists coax transformed cells to grow into whole plants by using tissue culture
techniques in which various plant hormones are included in culture media. Match the hormones
listed in the left-hand column with the actions listed in the right-hand column. (1 point)

1. the auxin indoleacetic acid (IAA)

A. promotes growth of
excised apical meristems

2. the auxin 2,4-dichlorophenoxy-

B. promotes organogenesis

acetic acid (2,4-D)

of roots

3. the cytokinin zeatin

C. promotes organogenesis
of shoots

4. gibberellin e.g. GA3

D. maintains valuable tissue samples


in culture for long periods of time

5. abscisic acid (ABA)

E. promotes the growth of a callus


of undifferentiated cells

IBO2004-Theory-2-page-8

Questions 91-95. A transposon has 3 functional requirements: a DNA recognition sequence


where it will integrate into the host genome; DNA sequences called terminal inverted
repeats that occur once the transposon has integrated into the genome; and transposon
DNA that encodes for the protein transposase which regulates movement into and out of
the host genome. A simple transposon is shown in the following diagram.
Terminal inverted repeats

5-CTGACTCTT
3-GACTGAGAA

AAGAGACAG-3
TTCTCTGTC-5
Transposon DNA
Host DNA

Host DNA

5-ACATTAACC
3-TGTAATTGG

ACATTAACC-3
TGTAATTGG-5
Target site duplication

Consider the following ten statements and use them to answer Questions 91 to 95
1.

The genome would be returned to its original state

2.

One transposon would remain in the genome

3.

The two transposons and the host DNA, encoding gene X between the two
integrated transposons could be lost from the host genome

4.

The genome will loose its integrity and will disintegrate

5.

There will be a duplication of gene X on one chromosome

6.

Gene X will be inverted

7.

Gene X will be moved to another chromosome

8.

Gene X will remain in its original position but will also be incorporated into
another chromosome.

9.

The region telemetric to the transposon will be lost

10.

The region centromeric to the transposon will be lost


IBO2004-Theory-2-page-9

Question 91). Movement of transposons is susceptible to errors; for example, if two transposons
integrate into a genome next to each other, it is not possible for the transposase enzyme to
distinguish which terminal repeats belong to which transposon. However, the direction of the
terminal inverted repeats must be conserved in the correct orientation for resolution (removal) of
the transposon. Consider two transposons integrating into a host genome in such a way that they
flank a gene encoding an enzyme for color X. What are the possible outcomes if one or both
transposons were resolved (removed)? (1 point)

A.

B.

1, 2 and 3

C.

D.

3, 5 and 6

E.

IBO2004-Theory-2-page-10

Question 92). Cross-over events during meiosis occur when related sequences align and DNA is
exchanged between chromosomes.

Most occur between sister chromatids and are subtle.

However, gross cross-overs can occur between different chromosomes (interchromosomal) or


between different regions within the same chromosome (intrachromosomal) and are often caused
by transposons. If the two transposons considered in Question 91 were integrated in the same
direction and caused an intrachromosomal cross-over, what would the result be? (1 point)

A.

B.

C.

D.

E.

IBO2004-Theory-2-page-11

Question 93). If the transposons described in Question 92 were in the opposite direction, what
would the result of the crossover be? (1 point)

A.

B.

C.

D.

E.

Question 94). If the two transposons described in Question 92 (both in same direction) caused an
interchromosomal cross-over in which the first transposon in chromosome A crossed-over with the
second transposon contained within sister chromosome B, the resultant cross-over in chromosome
A would lead to which of the following? (1 point)

A.

B.

C.

D.

E.

5+7

IBO2004-Theory-2-page-12

Question 95). Barbara McLintock was awarded the Nobel Prize in 1983 because of her ground
breaking work on transposable elements in maize. Her work focused on the effects that intact
transposons had on chromosomes containing defective transposons.

For this question, consider that the defective transposons contained one terminal inverted repeat
and no transposase. When a strain of maize with intact transposons (strain Ac) was crossed with
maize with a defective transposon incorporated next to gene X (strain Dc), she noticed that a loss
of color could be found in the offspring. This would arise from a change in which of the
following? (1 point)

A.

B.

C.

D.

E.

10

IBO2004-Theory-2-page-13

Questions 96-100.

The cardiovascular system in humans is a closed circulatory system

consisting of a powerful pump, valves and specialized vessels.

Question 96). Name the anatomical features of the heart indicated in the diagram (A-J) using terms
from the list below (1-10). (1 point)

G
F
E

1. Mitral (bicuspid) valve

6. Superior vena cava

2. Right atrium

7.

Right ventricle

3. Pulmonary veins

8.

Left atrium

4. Tricuspid valve

9.

Pulmonary artery

5. Aorta

10. Left ventricle

10

IBO2004-Theory-2-page-14

Question 97). For the structures labelled A, B, C, E, F, H, I and J in the previous Figure, what is
the correct order for the flow of blood entering from the systemic circulation? (1 point)
A.

I, H, F, J, B, C, E, A

B.

I, H, F, A, B, C, E, J

C.

A, F, H, I, B, C, E, J

D.

J, E, C, B, A, F, H, I

E.

A, F, H, J, B, C, E, I

Question 98). Flow through a tube is determined by the equation


where:

v = r4 p/8cl

l = the length of the tube in cm


r = the radius of the tube in cm
p = the pressure difference between the two ends of the tube in dynes per cm2
c = the coefficient of Viscosity in dyne-seconds per cm2
v = volume in cm3 per second

Which of the following will have the greatest effect on the resistance to blood flow? (1 point)
A.

Doubling the length of a blood vessel

B.

Doubling the diameter of a blood vessel

C.

Doubling the blood pressure

D.

Doubling the viscosity of the blood

E.

Halving the viscosity of the blood

IBO2004-Theory-2-page-15

Question 99). The following graph presents the pressure (systolic and diastolic) of a volume of
blood moving through the circulation system via different blood vessels labelled A-E.

Which vessels match the letters A-E: (1 point)


Answer
[A/B/C/D/E/]
1. Venules
2. Capillaries
3. Arterioles
4. Veins
5. Arteries

Question 100). All the following statements are FALSE EXCEPT one. Which? (1 point)
A. A normal blood pressure for an adult is 140/90
B. During exercise blood pressure will tend to increase
C. Sympathetic impulses to the heart and blood vessels tends to decrease blood pressure
D. Decreased cardiac output causes increased blood pressure
E. A heart murmur may be caused by a leaky aortic aneurism
IBO2004-Theory-2-page-16

Question 101-105. Researchers studied the carbon cycle of a lake and summarized the results
in the following scheme (numbers represent flow of carbon in gram/m2 per year).

from the
sea
10

B
151

suspended
detritus

111

bottom
detritus

51
51

20

181
192

136

76

sedentary
bottom fauna
biomass 8.0

54

grazing
bottom fauna
biomass 6.0
48

14

36

zooplankton
biomass 0.3
57

micro-algae
net production 75
biomass 3.0

104

9
29

61

7575

phytoplankton
net production 190
biomass 1.3

macro-algae
net production 50
biomass 15.0

190

50

= dissimilation

IBO2004-Theory-2-page-17

10

B
151

111

51
51

20

181
192

136

54

8.0

76

6.0
48

14

36

0.3
57

104

61

75
3.0

29

75

H
190
1.3

50
15.0

190

50

IBO2004-Theory-2-page-18

Question 101). What is the total biomass of consumers present in the scheme? (1 point)
A.

8.3

B.

9.6

C.

14.0

D.

14.3

E.

29.0

Question 102). What is the total gross primary production of this ecosystem? (1 point)
A.

125

B.

240

C.

265

D.

315

E.

630

Question 103). An indicator for the productive power of an ecosystem is the so-called P/B ratio,
where P = net primary production and B = biomass. Which group has the highest P/B ratio?
(1 point)
A.

D (grazing bottom fauna)

B.

E (sedentary bottom fauna)

C.

F (phytoplankton)

D.

G (micro algae)

E.

H (macro algae)

IBO2004-Theory-2-page-19

Question 104). Macro-algae and micro-algae differ in their net production. Which of the following
statements could explain the difference? (1 point)

A. Per volume of biomass, macro-algae receive more light than micro-algae


B. The proportion of micro-algae involved in photosynthesis is much bigger than
macro-algae
C. Micro-algae contribute more to production but lose it to bottom detritus
D. Dissimilation in macro-algae is relatively much bigger than in micro-algae
E. Macro-algae have a higher biomass, produce more but lose more to dissimilation

Question 105). In the lake, approximately how many days are necessary to renew the entire
biomass of micro-algae? (1 point)

A.

75

B.

61

C.

25

D.

15

E.

IBO2004-Theory-2-page-20

Questions 106-110. All organisms reproduce by at least one of two processes asexual or
sexual. Both processes involve replication of genetic material - producing identical offspring
in the case of asexual reproduction, or variable offspring (combination of gametes from two
different parent organisms) in the case of sexual reproduction. In eukaryotic cells,
replication of DNA is controlled by the cell cycle.

Question 106). Which of the following statements regarding human sex chromosomes is FALSE?
(1 point)
A. X chromosomes contain hundreds of genes
B. Y chromosomes contains the fewest genes of any human chromosome
C. The Y chromosome has very likely evolved from an X chromosome in an ancestral species
D. Most X chromosome genes concern female sexual development
E. Y chromosomes are unique in that they can never undergo crossing over during meiosis

Question 107). Mitosis is a fundamental cellular process that has which of the following
properties? (1 point)
A. The outcome is two daughter cells that are genetically identical because they have each
received an identical set of chromosomes
B. Mitosis is crucial for growth, for the replacement of damaged or aging cells and for asexual
reproduction
C. In many species, haploid cells as well as diploid cells undergo mitosis
D. Two of the above
E. All of the above except D

IBO2004-Theory-2-page-21

Question 108). The amount of DNA in a cell can be determined by measuring the fluorescence of a
dye that binds in direct proportion to the amount of DNA. The histogram below represents the
fluorescence of a eukaryotic germ cell during different stages of cell division (I, II, III, IV and V).

Which of the following sequences represents the correct match of stages I-V with the division
stages numbered 1-5?

(1 point)
1. Anaphase I of meiosis
2. Anaphase II of meiosis
3. Cytokinesis following Telophase II
4. Prophase II of meiosis
5. Prophase I of meiosis

A.

1 = II,

2 = IV,

3 = V,

4 = III,

5=I

B.

1 = I,

2 = II,

3 = III,

4 = IV,

5=V

C.

1 = V,

2 = IV,

3 = III,

4 = II,

5=I

D.

1 = I,

2 = II,

3 = IV,

4 = III,

5=V

E.

1 = IV,

2 = I,

3 = II,

4 = III,

5=V
IBO2004-Theory-2-page-22

Question 109). Cytokinesis is the process that divides the cytoplasm of a parent animal cell into
two daughter cells. From the following list of proteins, which are involved in cytokinesis steps? (1
point)
Tubulin
Fibronectin
Histone
Actin
Myosin
Collagen
Albumin

A.

Tubulin, Fibronectin, Albumin, Actin

B.

Myosin, Collagen, Histone, Tubulin

C.

Actin, Histone, Collagen

D.

Tubulin, Myosin, Actin

E.

Myosin, Fibronectin, Collagen, Albumin

IBO2004-Theory-2-page-23

Question 110). The diagram below represents a eukaryotic cell cycle divided into 5 phases.

E
Non-dividing cells
A

Match phases A-E on the diagram with the cell cycle stages shown in the Table and match the
processes F-J with the appropriate cell cycle stage shown in the Table.
F.

Cytokinesis

G.

Main growth period of the cell

H.

Duplication of DNA

I.

Quiescent cells

J.

Last stage of interphase

Cell cycle stage

1.

G2

2.

3.

G1

4.

5.

Go

(1 point)

Phase

Function

[A,B,C,D,E]

[F,G,H,I,J]

IBO2004-Theory-2-page-24

Questions 111-115.

A dairy cow grazes contentedly on pasture. As evening approaches, she

joins the small procession to the milking shed.

Question 111). Why do the cows go to the milking shed of their own accord? (0.5 point)
A.

Innate behaviour relating to herd instinct (gregarious nature)

B.

Innate behaviour relating to matriarchal dominance (follow-the-leader)

C.

Learnt behaviour in response to food reward (supplementary feed during milking)

D.

Learnt behaviour in response to pain relief (udder pressure relieved by milking)

E.

Learnt behaviour in response to pleasure (suckling stimulus enjoyable)

Question 112) Milk is collected twice daily from the dairy herd. Which of the following statements
is FALSE? (0.5 point)
A.

The hormone oxytocin is responsible for the milk let-down reflex

B.

Milk can only be collected from lactating cows

C.

All cows milked have been pregnant

D.

Milk will only continue to be produced in response to suckling stimuli

E.

Milk is rich in butterfat and colostrum

IBO2004-Theory-2-page-25

Question 113). Cows are ruminants. They have special sacculated digestive tracts to facilitate
fermentative digestion. Name the organs numbered on the diagram using the terms listed below.
(1 point)

A. omasum

B. duodenum

C. reticulum

D. oesophagus

E. rumen

F. ileum

G. jejunum

H. abomasum

I. colon

Answer
[A/B/C/D/E/F/G/H/I]
1.
2.
3.
4.
5.

IBO2004-Theory-2-page-26

Question 114). Digestion of food is facilitated by enzymes produced by various organs. Indicate
the sites of production of the listed enzymes and which substrates they act on. (2 points)
Organs

Food substrates

Enzymes

A. intestines

F. polysaccharides

1. trypsin

B. stomach

G. proteins

2. lipase

C. pancreas

H. fat

3. aminopeptidase

D. salivary gland

4. chymotrypsin

E. liver

5. amylase

Organ

Substrate

(A/B/C/D/E)

(F/G/H)

6. pepsin

Question 115). Cows do not produce cellulase enzymes to digest plant cells. Instead, they rely on
endosymbiotic micro-organisms to do so. Indicate which organisms produce cellulases in the
rumen to digest plant cell walls by answering Yes (Y) or No (N). (1 point)
Answer
[Y/N]
1. viruses
2. bacteria
3. spirochaetes
4. rickettsia
5. protozoa
6. fungi
7. algae
8. helminths

IBO2004-Theory-2-page-27

Questions 116-120. The diagram below shows a single-celled green microalga from the ocean.
Its single chloroplast and several other cellular components are labelled.

A. thylakoid membrane
B. stroma of chloroplast
C. cytosol
D. plasma membrane
E. cell wall
F. flagellum

Question 116). Which components contain the following:

(1 point)
Answer
[A/B/C/D/E/F]

1. orange and yellow carotenoids that harvest light energy for


photosynthesis
2. starch
3. macromolecular polymers that prevent the cell from bursting
if it is placed in fresh water
4. mitochondria

IBO2004-Theory-2-page-28

Question 117). Two mineral nutrients required for the growth of algae are magnesium (Mg) and
manganese (Mn). What are the roles of these minerals?

(1 point)

A. Mg assists osmosis; Mn assists the movement of flagella


B. Mg is in chlorophyll; Mn is in the photosynthetic water-splitting complex
C. Mg is in thylakoid cytochromes; Mn is in the terminal cytochrome oxidase of respiratory
electron transport in mitochondria
D. Mg2+ opens ligand-gated Ca2+ channels; Mn 2+ closes these channels
E. Mg is required for electron transport, Mn is required for membrane transport

Question 118). Indicate whether or not the generation of photosynthetic reducing power by this
alga is directly essential for the indicated cellular processes by answering Yes (Y) or No (N).

(1

point)
Answer
[Y/N]
1. manufacture of sugars from CO2 and H2O
2. conversion of nitrate (NO3-) to the ammonium (NH4+)
required for the biosynthesis of amino acids
3. production of citrate from glucose
4. incorporation of cytosolic phosphate (HPO42-/H2PO4-)
into DNA and RNA
5. assimilation of sulphate (SO42-) into the amino acids
cysteine and methionine

IBO2004-Theory-2-page-29

Question 119). Which statement summarizes the reproductive capabilities of this alga? (1 point)
A. Reproduces by mitosis, in which case genetically variable offspring are produced
B. Reproduces by meiosis, in which case genetically identical offspring are produced
C. Reproduces by mitosis, in which case genetically identical offspring are produced
D. Reproduces by meiosis, in which case genetically variable offspring are produced
E. Reproduces by C and D

IBO2004-Theory-2-page-30

Question 120). Flagella are common among eukaryotic organisms. Some prokaryotic organisms
(bacteria) also possess flagella. Consider the following statements.
1. Bacterial flagella are covered with two membranes
2. Eukaryotic flagella undulate when driving cell movement
3. Both bacterial and eukaryotic flagella use proton gradients as direct energy sources
4. Prokaryotic flagella are formed from actin; eukaryotic flagella are formed from tubulin
5. Prokaryotic flagella are proteinaceous spiral filaments attached to multi-protein rotors
6. All prokaryotic cells have at least one flagellum
7. All eukaryotic flagella are covered with an extension of the plasma membrane
8. All functional eukaryotic flagella contain molecular motor-proteins (dyneins)
9. Prokaryotic flagella can rotate only in the one direction
10. Each eukaryotic flagellum has its own basal body

Which of these statements are true? (2 points)


A.

2, 5, 7, 8, 10

B.

1, 4, 7, 9, 10

C.

3, 5, 6, 8, 10

D.

2, 4, 7, 8, 9

E.

1, 3, 5, 7, 9

IBO2004-Theory-2-page-31

Questions 121-123. Many genotypic and phenotypic traits recognized in animals and plants
have been used in genetic mapping studies and biosystematics.

Question 121). A cladistic classification of part of the Animal Kingdom is shown below.
Salamander
Lung fish
Shark
Sea squirt
(Tunicate)

8
7
Lung

1
Aorta

From the 16 options shown below (A-O), assign the nine most appropriate features to the positions
numbered 1-9. (2 points)
A. Amniotic egg

F. Gill slits

K. Muscular lobed fins

B. Bony fin

G. Hinged jaw

L. Swim bladder

C. Cephalization

H. Internal development of egg

M. Vertebrae

D. Three-chambered heart

I. Limbs

N. Ribs

E. Fibrous protein skeleton

J. Long sticky tongue

O. Tail

IBO2004-Theory-2-page-32

Question 122). Wing structure and function are characteristic for insect orders. Match the wing
characteristics listed in the right-hand column (1-5) with the insect orders listed in the left-hand
column (A-E). (1 point)

A. Odonata

1. One pair of wings, second pair of wings is transformed into halteres

B. Diptera

2. Two pairs of membranous wings

C. Orthoptera

3. Wingless

D. Lepidoptera

4. Two pairs of wings, forewings are leathery, hindwings are membranous

E. Phthiraptera
(Anoplura)

5. Two pairs of wings covered with tiny scales

IBO2004-Theory-2-page-33

Question 123). Three alleles (a, b and c) are linked on a normal chromosome (autosome) of a
plant . An hybrid

ABC
abc
was crossed with a recessive
and the types and numbers of gametes
abc
abc

were recorded as follows:


ABC

414

Abc

70

aBc

28

abC

abc

386

aBC

80

AbC

20

ABc

1. What is the order of these genes on the chromosome? (1 point)


A.

abc

B.

acb

C.

bac

2). What is the correct distance between the three loci (in map units)? (2 points)
distance between a and c

distance between b and c

A.

2.1

2.4

B.

3.4

5.0

C.

5.0

15.2

D.

15.2

3.4

E.

15.2

5.0

IBO2004-Theory-2-page-34

Questions 124-126. A plant biologist studies the sexual reproduction of a moss, a fern, and a
flowering plant (tomato). The biologist makes the following drawings:
X.

moss leaves, seta and capsule

Y.

underside of part of a fern leaf

Z.

cross-section of a tomato flower

Z
4

2
5
3

9
8

Nine structures are numbered 1 to 9.

Question 124). Indicate which structures conform to the following statements: (2 points)
1. Haploid cells that carry out photosynthesis
A.

Only 4, 5

B.

Only 3

C.

Only 1, 2, 6

D.

Only 4, 8

IBO2004-Theory-2-page-35

2. Diploid cells that carry out photosynthesis


A.

Only 1, 2, 3

B.

Only 3

C.

Only 3, 4, 8

D.

Only 4, 8

3. Leaves modified for functions other than photosynthesis


A.

Only 1, 2

B.

Only 1, 2, 6, 7

C.

Only 1, 5, 9

D.

Only 6, 7, 9

4. Structures where meiosis is occurring, or has recently occurred


A.

Only 1, 5, 7, 9

B.

Only 3, 4, 8

C.

Only 1, 2, 5

D.

Only 2, 8

5. Structure(s) that will produce spores destined to germinate and produce a haploid
photosynthetic plant
A.

Only 1

B.

Only 1, 5

C.

Only 1, 5, 7

D.

Only 5, 7

IBO2004-Theory-2-page-36

Question 125). The life cycle of a fern is shown in the diagram below. Five processes (numbered
1, 2, 3, 4, 5) and five stages (labelled I, II, III, IV, V) are indicated.

I
Sporophyte

1
II
Spore

V
Zygote

IV
Gamete (female)

IV
Gamete (male)

III
Gametophyte

Select the processes, or stages, corresponding to the following items: (2 points)

1. Those producing genetic diversity


A.

Only 1, 3

B.

Only 2, 3, 4

C.

Only 1, 4

D.

Only 3

IBO2004-Theory-2-page-37

2. Those undergoing meiosis


A.

Only 1

B.

Only 1, 2, 3

C.

Only 2, 5

D.

Only 3, 4

3. Those with haploid cells


A.

Only I, III, IV

B.

Only II, III, IV

C.

Only I, V

D.

Only IV

4. Those with diploid cells


A.

Only I, II, III

B.

Only I, II, III, V

C.

Only II, V

D.

Only I, V

5. Those stages (or equivalents) not present in the human life cycle
A.

Only I, III

B.

Only II, III

C.

Only III, IV

D.

Only II, III, V

IBO2004-Theory-2-page-38

Question 126). The biologist examines sexual reproduction in a grass. He/she examines a rice
grain, makes a cross-sectional drawing and labels the component parts.

A. pericarp
B. aleurone layer
C. endosperm

D. embryo

For each part, select the ploidy of the tissue (n, 2n, 3n, etc) and if two or more tissues have the
same ploidy, indicate whether they are genetically identical or not, as follows:
2n
2n

genetically identical

2n-i
2n-ii

genetically different

A.

A: 2n

B: n

C: n

D: 2n

B.

A: 2n-i

B: 3n

C: 3n

D: 2n-ii

C.

A: 3n-i

B: 3n-ii

C: 3n-iii

D: 2n

D.

A: 2n-i

B: n

C: 2n-ii

D: 3n

E.

A: 2n

B: 3n-i

C: 3n-ii

D: 2n

IBO2004-Theory-2-page-39

Questions 127-131. Albumin, with a molecular mass of 68,000 Da, is the most abundant
protein in blood plasma, accounting for approximately 60% of all plasma protein.

Question 127). A person was found to have reduced levels of plasma albumin, losses occurring as
the result of kidney damage. Indicate the part of the kidney shown in the following diagram that
you would expect to be the primary site of damage for this patient. (1 point)

A.

B.

C.

D.

E.

F.

G.

7
IBO2004-Theory-2-page-40

Question 128).

A major function of albumin is to maintain blood osmolality (osmotic pressure).

The patient had swelling around the feet. An explanation for this symptom is that loss of albumin
from plasma leads to which of the following? (1 point)

A.

High blood pressure

B.

Loss of tissue fluid

C.

Increased blood supply to feet

D.

Increased blood vessel size

E.

Low blood volume

IBO2004-Theory-2-page-41

Question 129). Albumin is synthesized in liver cells and secreted into the blood plasma. A diagram
showing the ultrastructure of a liver cell is given below.

Which combination of organelles would be involved in the synthesis and transport of albumin to
the plasma membrane for secretion? (1 point)

A.

nucleus, free ribosomes

B.

mitochondria, endosomes

C.

peroxisome, endoplasmic reticulum

D.

endoplasmic reticulum, Golgi apparatus

E.

Golgi apparatus, lysosomes

F.

endosomes, cytosol

IBO2004-Theory-2-page-42

Question 130). Eukaryotic proteins encoded by the nuclear genome are translated from an
initiation AUG codon thus newly synthesized proteins have a methionine residue at the extreme
amino-terminus. However, albumin isolated from human blood plasma does not have an aminoterminal methionine. Which of the following statements accounts for this? (1 point)

A.

Albumin is encoded by the mitochondrial genome, not the nuclear genome

B.

Albumin is modified in the plasma by proteolytic enzymes

C.

Albumin is processed by proteases as it passes through the endomembrane system

D.

The isolation procedure modifies the amino-terminus of the albumin protein

E.

The amino-terminus of albumin is modified by passage across the hepatocyte


cell membrane

IBO2004-Theory-2-page-43

Question 131). The concentration of albumin in the blood plasma of a patient can be determined
using a test based on the specific binding of albumin to a chemical dye. The graph shows the
absorbance readings of the albumin-dye complex at different light wavelengths.

Which absorption spectrum is consistent with the above data for albumin at 2 g/L? (1 point)

E. None of the above

IBO2004-Theory-2-page-44

Questions 132-136. When oxygen levels are low, some cells may resort to the process of
fermentation to obtain energy. Examples include yeast cells used in bakeries, breweries and
wineries.

Question 132). The manufacture of bread, beer and wine all involve alcoholic fermentation of
glucose to ethanol by yeast. Which combination of the following eight statements concerning this
process is correct? (2 points)

1.

Yeast carry out this fermentation because they lack mitochondria

2.

For every molecule of ethanol produced, one molecule of CO2 is evolved

3.

The net generation of ATP is only two molecules per molecule of glucose fermented

4.

More than 80% of the chemical energy of the glucose is released as heat

5.

Glycolysis is an integral part of this fermentation

6.

The conversion of one molecule of citrate to one molecule of malate and two molecules
of CO2 is an integral part of this fermentation

7.

The electron donor for ethanol formation (catalysed by alcohol dehydrogenase) is NADH

8.

CO2 is evolved when beer is produced by alcoholic fermentation, but not when bread or
wine are produced by alcoholic fermentation

A.

1, 2, 3, 4, 5

B.

1, 4, 7, 8

C.

2, 3, 5, 7

D.

2, 3, 6, 7, 8

E.

3, 4, 5, 7, 8

IBO2004-Theory-2-page-45

Question 133). A healthy student decides to enter a 200 metre race. He has not trained to be an
athlete, so he has little chance to prepare except to eat well. On the day of the race, he completes
the run in 27 seconds. He finishes exhausted and suffers from leg-muscle cramps. (1 point)

1. What was the students main source of energy during the race?
A. Muscle glucose and glycogen
B. Free amino acids in his muscle
C. The lipid component of blood low-density lipoproteins
D. Starch being digested from his last meal

2. What was the students leg-muscle metabolism during the race?


A. Predominantly aerobic respiration
B. Both aerobic respiration and fermentation
C. Predominantly fermentation
D. Reactions were fully dependent on the O2 supply delivered by blood haemoglobin

3. Which of the following biochemical pathways were important in his muscle during the race?
A. Fatty acid oxidation
B. Glycolysis
C. Gluconeogenesis
D. NADPH-generating oxidative pentose-phosphate pathway

IBO2004-Theory-2-page-46

4. Why did the student suffer leg cramps?


A Bubbles of CO2 generated by rapid respiration had accumulated in his muscles
B Lactic acid generated by fermentation had accumulated in his muscles
C Extensive exocytosis of Ca2+ from his muscle cells occurred
D Muscle motor proteins (myosin) no longer required ATP to attach to
muscle microfilaments (actin)

Question 134). Glucose in human liver cells can be synthesized from which combination of the
following non-sugar sources? (1 point)
1. adenine
2. alanine
3. lactate
4. palmitate
5. glycerol

A.

Only 1, 2, 3

B.

Only 1, 4

C.

Only 2, 3, 4, 5

D.

Only 4, 5

E.

Only 2, 3, 5

IBO2004-Theory-2-page-47

Question 135). Which statements are FALSE for fermentation? (1 point)


1. Animal cells are capable of fermentation, but plant cells are not
2. In lactic acid fermentation, pyruvate is an electron acceptor instead of O2
3. In ethanol fermentation, pyruvate is an electron acceptor instead of O2
4. Oxidative phosphorylation occurs
5. Substrate level phosphorylation occurs
6. Photophosphorylation occurs

A.

Only 1, 5

B.

Only 1, 2, 6

C.

Only 1, 3, 4, 6

D.

Only 2

E.

Only 2, 5

Question 136). Which of the following statements is correct? (1 point)

A.

Fermentations always involve organic molecules (rather than O2)


acting as the ultimate acceptor of electrons

B.

Fermentations always involve the formation of a single product

C.

Fermentations always involve the production of CO2

D.

Fermentations always involve yeast or bacteria growing


in a large vat of fermentable substrate

E.

Fermentations always involve the formation of ethanol

IBO2004-Theory-2-page-48

Questions 137-141. Mitochondria are double-membrane organelles found in most eukaryotic


cells. They are involved in cellular respiration and metabolism.

Question 137). A widely-held theory is that mitochondria evolved from endosymbiotic bacteria.
Indicate which statements support this theory by answering Yes (Y) or No (N). (1 point)
[Y/N]
1.

Mitochondria have their own DNA

2.

Mitochondria have their own ribosomes

3.

Mitochondria are derived from pre-existing mitochondria by division

4.

Human mitochondrial genes lack introns

5.

Some mitochondrial gene DNA sequences are similar to those of certain


aerobic bacteria

Question 138). Which of the following statements about mitochondria are correct? (Y = correct; N =
incorrect) (1 point)
[Y/N]
1.

Acetyl-CoA conversion to citrate occurs in the Krebs cycle

2.

Glucose oxidation to pyruvate occurs in the mitochondrial matrix

3.

The mitochondrial electron transport chain is located on the inner


mitochondrial membrane

4.

The inner mitochondrial membrane prevents the free movement of


metabolites into and out of the mitochondrion

5.

Mitochondria contain specific membrane transporters


IBO2004-Theory-2-page-49

uestion 139). During oxidation of NADH by the electron transport chain, protons are pumped
across the inner mitochondrial membrane. Which of the following statements about this process is
INCORRECT? (1 point)
A.

Proton pumping is achieved by alternating hydrogen and electron carriers in the


electron transport chain

B.

Ubiquinone (Coenzyme Q) is a hydrogen carrier

C.

Cytochromes are electron carriers but not hydrogen carriers

D.

The ATP synthase complex uses the proton gradient to make ATP

E.

The ATP synthase reaction involves a rotor mechanism

F.

Proton pumping can decrease the pH in the mitochondrial matrix by 1 pH unit

G.

Iron-sulphur (Fe-S) proteins are electron carriers

Question 140). Which of the following statements about mitochondrial DNA and protein synthesis
is correct?

(1 point)

A.

All mitochondrial proteins are coded for by mitochondrial DNA

B.

Mitochondrial DNA accumulates mutations more slowly than


nuclear DNA (per 1,000 base pairs)

C.

Mitochondrial DNA is inherited equally from both parents

D.

Mitochondrial ribosomes are the same size as mammalian ribosomes

E.

Antibiotics are known which inhibit protein synthesis in both bacteria


and mitochondria

F.

Mitochondrial DNA codes for active RNA molecules but not for proteins

G.

The production of ATP by mitochondria is unaffected by age

IBO2004-Theory-2-page-50

Question 141). An electrochemical proton gradient is also responsible for powering bacterial
flagella. During oxidation of glucose, protons are pumped out of the cell to help establish an
electrochemical gradient. The flagellar motor is driven directly by the flux of protons back into the
bacterial cell from the proton gradient. Addition of FCCP, a proton ionophore, to these bacteria in
+

a medium containing a high concentration of K , results in loss of bacterial motility. Which


statement best explains the action of FCCP? (1 point)

A.

FCCP equilibrates the H concentration across the bacterial membrane

B.

FCCP allows K ions across the bacterial membrane

C.

FCCP depletes glucose

D.

FCCP allows proteins across the bacterial membrane

E.

FCCP inhibits glucose oxidation

Questions 142-146. Animals have developed complex patterns of behaviour which scientists
are slowly learning to decipher through rigorous experimentation. Animals live in complex
ecosystems and many varied parameters influence their behaviour.

IBO2004-Theory-2-page-51

Question 142). The following matrix shows elephant head positions relative to their tendency to
fight or fly (with four illustrations labelled 1-4 missing).

The missing illustrations are shown below in random order (labelled I-IV).

Which illustrations (I-IV) correctly match positions (1-4) in the matrix? (1 point)
Position 1

Position 2

Position 3

Position 4

A.

II

III

IV

B.

IV

II

III

C.

II

IV

III

D.

II

IV

III

E.

III

II

IV

IBO2004-Theory-2-page-52

Question 143). Scientists put mallard ducklings (Anas platyrhynchos) into a large outdoor enclosure

and then moved a silhouette shape (pictured below) across the top of the enclosure at intermittent
intervals and in either direction (as shown).

Eight possible behaviours are listed below for different test conditions.
1. The ducklings crouch and run during movement of the shape from left to the right.
2. The ducklings do not respond during movement of the shape from right to the left.
3. The ducklings respond more to the hawk shape than the duck shape.
4. Ducklings reared and tested together with ducks are more likely to crouch
than run in response to the hawk shape.
5. Ducklings will respond more to the duck shape than the hawk shape.
6. Ducklings reared and tested alone are more likely to run than crouch
in response to the hawk shape.
7. Ducklings reared and tested together with ducks are more likely to run
than crouch in response to the hawk shape.
8. Ducklings reared and tested alone are more likely to crouch than run
in response to the hawk shape.

IBO2004-Theory-2-page-53

Which combination of duckling behaviours is correct? (1 point)


A.

1, 5, 8

B.

3, 5, 6

C.

2, 3, 7

D.

3, 4, 6

E.

4, 7, 8

Question 144). Consider three hypothetical communities I, II and III, each containing ten species.
The following graphs show the relative abundance (vertical axis) of each species, when the species
are ranked in decreasing order of abundance on the horizontal axis.

frequency

0.15

0.9

0.3

II

0.1

0.6

0.2

0.05

0.3

0.1

0
1

10

species

species number

III

number

9 10

species

9 10

number

How would you rank the communities in decreasing order of biodiversity? (1 point)
A.

I = II = III

B.

I > III > II

C.

III > II > I

D.

II > III > I

E.

I > II > III

IBO2004-Theory-2-page-54

Question 145). The following graph indicates the variation in vegetation ground cover with
distance from a stock water supply point in a semi-arid pasture during three successive years. The
pasture was grazed in Years 1 and 2 but was ungrazed in Year 3.

% vegetation ground cover

80

60

40
Year 3 ungrazed
Year 1 grazed
Year 2 grazed

20

0
0

10

15

20

Distance from water (km)

The patterns of variation in vegetation cover with distance from water indicate which of the
following? (1 point)
A. Rainfall was approximately equal in Years 1 and 3, and substantially less in Year 2
B. Grazing pressure is much more severe over the entire area in a dry year than in a wet
year
C. Grazing in Years 1 and 2 affects the vegetation cover in Year 3, and grazing pressure is
inversely proportional to distance from the water supply point
D. The animals graze only within 10 km of the water supply point
E. Stocking density was higher in year 1 than year 2
IBO2004-Theory-2-page-55

Question 146). Consider the following schematic diagram of generalized nutrient cycling within
ecosystems. The labelled arrows A to D represent transfer between the pools.

Organic Materials
available as nutrients

Inorganic Materials
available as nutrients

Organic Materials
unavailable as nutrients

Inorganic Materials
unavailable as nutrients

Which of the following statements about the rate of transfer per unit time between these pools is
correct? (1 point)
A. The rate at which transfer A occurs is slower than the rate at which B occurs
B. The rate at which transfer D occurs is faster than the rate at which transfer A occurs
C. All transfers occur at similar rates
D. The rate at which transfer A occurs is faster than the rate at which B occurs
E. These rates are largely unaffected by human actions

IBO2004-Theory-2-page-56

Questions 147-151.

Phospholipids and proteins are the major components of biological

membranes. The structure of 1-palmitoyl-2-linoleoyl-phosphatidylcholine, a common


phospholipid, is shown below.

The structures of the components (labelled A-E) are shown separately below.

IBO2004-Theory-2-page-57

Question 147). Phospholipids are often represented as having a head (H) and tail (T) region.

Indicate where each of the component parts of phosphatidylcholine are found. (1 point)
Answer [H/T]
A. palmitic acid
B. linoleic acid
C. glycerol
D. phosphoric acid
E. choline

Question 148). Which of the components A, B, C, D or E conform to the following descriptions?


(1 point)
Answer
[A/B/C/D/E]
1. This molecule is a polyunsaturated fatty acid component of
1-palmitoyl-2-linoleoyl-phosphatidylcholine
2.

A diet rich in animal fats results in enrichment of this fatty acid


in cell membranes

3.

This molecule is also a component of DNA

IBO2004-Theory-2-page-58

Question 149). Considering the roles of membrane lipids and proteins, and the functions of the
following membranes, match the protein/lipid ratios given below to the following membranes.
(1 point).
A.

Schwann cell membrane (myelin sheath)

B.

Erythrocyte (red blood cell) membrane

C.

Inner mitochondrial membrane


protein/lipid ratio

Answer
[A/B/C]

1.

1:1

2.

4:1

3.

1:4

Question 150). Proteins can span a membrane by means of a stretch of 20 nonpolar amino acids in
an -helical arrangement. Given that the -helix has a pitch of 0.54 nm and has 3.6 amino acid
residues per turn, what is the thickness of the nonpolar central section of the lipid bilayer?
(1 point)
A.

0.18 nm

B.

3.0 nm

C.

5.5 nm

D.

10.2 nm

E.

37.0 nm

IBO2004-Theory-2-page-59

Question 151). Select two of the following molecules which you would expect to be able to
diffuse readily across a biological bilayer membrane. (1 point)

A.

1+2

B.

1+3

C.

1+4

D.

2+3

E.

2+4

F.

3+4

IBO2004-Theory-2-page-60

Questions 152-156. Two species of nectarivorous bird are feeding in the same meadow. Both
species of bird have already migrated some distance from their wintering ground to reach
the meadow. They need to migrate further to reach their breeding grounds. While on the
meadow, each individual bird holds a feeding territory. They are feeding on the nectar in the
flowers of one species of plant.

Question 152). What are the two main resources that the nectar provides? (1 point)
A.

Fat and protein

B.

Pollen and carbohydrate

C.

Carbohydrate and protein

D.

Fat and carbohydrate

E.

Water and protein

Question 153). What will be the birds primary objective during their time on the meadow?
(1 point)
A.

Find a mate

B.

Maximise its net rate of energy gain

C.

Rear young

D.

Minimise its rate of energy consumption

E.

Compete with other species

IBO2004-Theory-2-page-61

Question 154). The types of interaction between the plant and the birds, and between the two
species of bird, are called which of the following, respectively? (1 point)
A.

Predation and mutualism

B.

Predation and competition

C.

Mutualism and predation

D.

Competition and mutualism

E.

Mutualism and competition

Question 155). All else being equal, what type of flowers would the birds prefer to choose?
(1 point)
A.

Flowers with more nectar

B.

Flowers that are purple

C.

Flowers with variable supplies of nectar

D.

Flowers closer to the ground

E.

Flowers that have recently been pollinated

Question 156). A few of the plants have a gene that, when expressed, means they secrete no
nectar. Which of the following statements is true? (1 point)
A.

These plants will have no offspring because the birds will not visit their flowers

B.

Plants that do not secrete nectar cannot set seed

C.

Plants that do not secrete nectar save energy that can be used for growth and other
plant functions

D.

Failure to secrete nectar must be a recessive trait that confers no selective advantage

E.

Plants that secrete more nectar will have more offspring


-------------------------END------------------------------IBO2004-Theory-2-page-62















All IBO examination questions are published under the following Creative Commons license:



CC BY-NC-SA (Attribution-NonCommercial-ShareAlike) https://creativecommons.org/licenses/by-nc-sa/4.0/
The exam papers can be used freely for educational purposes as long as IBO is credited and
new creations are licensed under identical terms. No commercial use is allowed.

ANSWER SHEET
Q.

part

81.
82.
83.
84.
85.
86.
87.
88.
89.
90.
91.
92.
93.
94.
95.
96.

deleted

97.
98.
99.

100
101.
102.
103.
104.
105.
106.
107.
108.
109.

choices

answer

ABCDE
ABCDE
ABCDE
ABCDE

C
B
B
E

1
1
1
1

ABCDE
ABCDE

B
B

1
1

deleted
deleted
deleted
deleted
deleted
deleted
1.
ABCDEFGHIJ
2.
ABCDEFGHIJ
3.
ABCDEFGHIJ
4.
ABCDEFGHIJ
5.
ABCDEFGHIJ
6.
ABCDEFGHIJ
7.
ABCDEFGHIJ
8.
ABCDEFGHIJ
9.
ABCDEFGHIJ
10.
ABCDEFGHIJ
ABCDE
ABCDE
1.
ABCDE
2.
ABCDE
3.
ABCDE
4.
ABCDE
5.
ABCDE
ABCDE
ABCDE
ABCDE
ABCDE
ABCDE
ABCDE
.
ABCDE
ABCDE
ABCDE
ABCDE

D
H
B
G
J
I
F
C
A
E
B
B
D
C
B
E
A
B
D
E
C
B
D
D
E
A
D

1/10
1/10
1/10
1/10
1/10
1/10
1/10
1/10
1/10
1/10
1
1
1/5
1/5
1/5
1/5
1/5
1
1
1
1
1
1
1
1
1
1

deleted
deleted

110.

111.
112.
113.

114.

115.

116.

117.
118.

119.
120.
121.
122.

123.

1.
2.
3.
4.
5.
.
1.
2.
3.
4.
5.
1.
2.
3.
4.
5.
6.
1.
2.
3.
4.
5.
6.
7.
8.
1.
2.
3.
4.
1.
2.
3.
4.
5.

ABCDEFGHIJ
ABCDEFGHIJ
ABCDEFGHIJ
ABCDEFGHIJ
ABCDEFGHIJ
ABCDE
ABCDE
ABCDEFG
ABCDEFG
ABCDEFG
ABCDEFG
ABCDEFG
ABCDEFGH
ABCDEFGH
ABCDEFGH
ABCDEFGH
ABCDEFGH
ABCDEFGH
YN
YN
YN
YN
YN
YN
YN
YN
ABCDEF
ABCDEF
ABCDEF
ABCDEF
ABCDE
YN
YN
YN
YN
YN
ABCDE
ABCDE

deleted
1
ABCDE
2
ABCDE
3
ABCDE
4
ABCDE
5
ABCDE
1.
ABC
2
ABCDE

CJ
DF
AG
BH
EI
C
E
A
D
C
H
E
CG
(CorB)andH
AG
CG
(CorD)andF
BG
N
Y
N
N
Y
Y
N
N
A
B
E
C
B
Y
Y
N
N
Y
E
A

1/5
1/5
1/5
1/5
1/5
1/2
1/2
1/5
1/5
1/5
1/5
1/5
1/3
1/3
1/3
1/3
1/3
1/3
1/8
1/8
1/8
1/8
1/8
1/8
1/8
1/8
1/4
1/4
1/4
1/4
1
1/5
1/5
1/5
1/5
1/5
1
2

B
A
E
C
D
B
E

1/5
1/5
1/5
1/5
1/5
1
2

124.

125.

126.
127.
128.
129.
130.
131.
132.
133.

134.
135.
136.
137.

138.

139.
140.
141.
142.
143.
144.
145.
146.
147.

1.
2.
3.
4.
5.
1.
2.
3.
4.
5.

1.
2.
3.
4.

1.
2.
3.
4.
5.
1.
2.
3.
4.
5.

A.
B.
C.
D.

ABCD
ABCD
ABCD
ABCD
ABCD
ABCD
ABCD
ABCD
ABCD
ABCD
ABCDE
ABCDEFG
ABCDE
ABCDEF
ABCDE
ABCDE
ABCDE
ABCD
ABCD
ABCD
ABCD
ABCDE
ABCDE
ABCDE
YN
YN
YN
YN
YN
YN
YN
YN
YN
YN
ABCDEFG
ABCDEFG
ABCDE
ABCDE
ABCDE
ABCDE
ABCDE
ABCDE
HT
HT
HT
HT

B
D
D
A
B
C
A
B
D
B
B
C
E
D
C
B
C
A
B
B
B
E
C
A
Y
Y
Y
Y
Y
Y
N
Y
Y
Y
F
E
A
B
D
B or A
C
D
T
T
H
H

2/5
2/5
2/5
2/5
2/5
2/5
2/5
2/5
2/5
2/5
1
1
1
1
1
1
1
1/4
1/4
1/4
1/4
1
1
1
1/5
1/5
1/5
1/5
1/5
1/5
1/5
1/5
1/5
1/5
1
1
1
1
1
1
1
1
1/5
1/5
1/5
1/5

148.

149.

150.
151.
152.
153.
154.
155.
156.

E.
1.
2.
3.
1.
2.
3.

HT
ABCDE
ABCDE
ABCDE
ABC
ABC
ABC
ABCDE
ABCDEF
ABCDE
ABCDE
ABCDE
ABCDE
ABCDE

H
B
A
D
B
C
A
B
E
C
B
E
A
C

1/5
1/3
1/3
1/3
1/3
1/3
1/3
1
1
1
1
1
1
1

INTERNATIONAL BIOLOGY OLYMPIAD


PRACTICAL PROBLEMS

2004, Brisbane, Australia















All IBO examination questions are published under the following Creative Commons license:



CC BY-NC-SA (Attribution-NonCommercial-ShareAlike) https://creativecommons.org/licenses/by-nc-sa/4.0/
The exam papers can be used freely for educational purposes as long as IBO is credited and
new creations are licensed under identical terms. No commercial use is allowed.

Country:

PRACTICAL EXAMINATION # 1: MOLECULAR LABORATORY

This practical examination is composed of two 45-minute Tasks:


Task 1: Measurement of enzyme activity (20 points)
Task 2: Separation of proteins by chromatography and electrophoresis (20 points)
After 45 minutes, competitors will swap tasks.
Competitors are requested to follow instructions when swapping from one task to another.
There is to be no discussion or exchange of any materials between competitors when changing
tasks.
Competitors must not commence the new task until instructed.
THESE INSTRUCTIONS ARE FOR TASK 1

EXAMINATION # 1: Molecular Laboratory

TASK 1

GENERAL INSTRUCTIONS

Competitors are advised to read the examination before commencing.

It is recommended that Competitors proportion their time according to the allotted points for each
task and question.

IMPORTANT

All answers must be recorded on the answer sheets provided.

Ensure that your 3 digit code number is written and coded on the top of each page of the answer
sheets.

Using the pencil provided, fill in the appropriate circle on the answer sheet.

EXAMINATION # 1: Molecular Laboratory

TASK 1

TASK 1
MEASUREMENT OF ENZYME ACTIVITY

INTRODUCTION
Alcohol dehydrogenase (ADH) is an enzyme that oxidises ethanol to acetaldehyde (ethanal)
according to the following reaction.

Ethanol + NAD

acetaldehyde + NADH + H

The cofactor nicotinamide adenine dinucleotide (NAD+) is reduced in the reaction. The progress of
the reaction can be monitored by measuring the concentration of NADH at a wavelength of 340 nm.

In this practical task you will use a spectrophotometer to measure the activity of this enzyme at
various concentrations of ethanol.

EXAMINATION # 1: Molecular Laboratory

TASK 1

MATERIALS AND REAGENTS PROVIDED FOR EACH COMPETITOR

Chemicals
Ethanol Solutions:

0.5M, 0.25M, 0.125M, 0.063M

Reaction Buffer:

2mM NAD in 80mM sodium phosphate buffer pH 7.4 containing

40mM KCl
Enzyme:

0.04 mg/mL alcohol dehydrogenase solution

Equipment

Spectrophotometer with wavelength set at 340nm

Adjustable pipettors: 1000L,100L and 20L, plus tips

Yellow sharps bin for tip disposal

1mL plastic cuvettes plus holder

Plastic cuvette stirrers

Laboratory Timer

Marking pens

Safety glasses

Disposable gloves

Graph paper with pre-set axes

Ruler

Black pen

Pink card (to attract attention of demonstrator)

Answer sheet with pencil and eraser

EXAMINATION # 1: Molecular Laboratory

TASK 1

EXPERIMENTAL PROCEDURE

Set up the following duplicate reaction mixtures in 1mL plastic cuvettes, as shown in the Table:

Reaction No.

Reaction Buffer

Ethanol solution

1a

0.9 mL

0.1mL of 0.063M

1b

0.9 mL

0.1mL of 0.063M

2a

0.9 mL

0.1mL of 0.125M

2b

0.9 mL

0.1mL of 0.125M

3a

0.9 mL

0.1mL of 0.250M

3b

0.9 mL

0.1mL of 0.250M

4a

0.9 mL

0.1mL of 0.500M

4b

0.9 mL

0.1mL of 0.500M

EXAMINATION # 1: Molecular Laboratory

TASK 1

Each competitor will be allocated a spectrophotometer. There are two models of


spectrophotometer, but they will both give the same results.

Your spectrophotometer has been set up to measure the absorbance at a wavelength of 340 nm.
Carry out the following steps to prepare for the enzyme activity measurements:
If you are using an Hitachi U-1100 Spectrophotometer

Place the cuvette containing the reaction mixture for reaction 1a in position 1 (front) of the cell
holder.

Close the lid.

To set the absorbance reading to zero, press the 100%T/ 0 ABS button (labelled with red dot).
The absorbance display should read 0.000.

If you are using an Hitachi U-1800 Spectrophotometer

Place the cuvette containing the reaction mixture for reaction 1a in position 1 (front) of the cell
holder.

Close the lid.

To set the absorbance reading to zero, press the AUTO ZERO button (labelled with red dot).
The absorbance display should read 0.000.

EXAMINATION # 1: Molecular Laboratory

TASK 1

Carry out the enzyme activity measurements one at a time, starting with Reaction No. 1a:

(a) Place the cuvette containing the reaction mixture in the spectrophotometer cell.

(b) To start each reaction, add 10 L of Enzyme to the cuvette placed in the spectrophotometer.

(c) Quickly but gently mix the solution in the cuvette with a white plastic stirrer, close the
spectrophotometer lid, set the Absorbance to zero, AND immediately start your timer.

(d) Record the absorbance reading exactly 1 minute later. This is the t = 1 min absorbance reading.
This reading is equal to the rate of enzyme reaction (designated V), expressed as the change
() in absorbance at 340 nm over the 1 minute period (i.e. A340 /min) from t = 0 to t = 1 minute
Enter the values obtained into the Results Table in Column 1.

DATA ANALYSIS AND INTERPRETATION


Complete the Table to determine the average A340 /min for each duplicate pair of reactions. Enter
these values into Column 2.

For the graphing component of the practical test that follows, also determine the values for 1/Vaverage
-1

and enter these in the column of the Table on the right. The values for 1/S (units mM ), where S =
substrate (ethanol) concentration in mM, have been included for you in the last column. For
example, for Reactions 4a and 4b, the actual ethanol concentration in the reactions is 50mM,
-1

therefore 1/S = 0.02 mM .

EXAMINATION # 1: Molecular Laboratory

Reaction
No.

TASK 1

Column

Column

Data for plotting

graph

V
(Average

A340 /min

1/VAverage

A340 /min)

1/S
-1

(mM )

1a
0.160
1b

2a
0.080
2b

3a
0.040
3b

4a
0.020
4b

ENTER THE VALUES FROM COLUMN 2 ONTO THE ANSWER SHEET


(Tasks P1.T1.1 P1.T1.4) (8 points 2 points each)

EXAMINATION # 1: Molecular Laboratory

TASK 1

If your results were plotted as the enzyme rate (V), expressed as A340 /min, versus ethanol
substrate concentration (S), a graph of the shape shown below would be obtained. This shows that
enzyme rate increases with substrate concentration until a maximum rate (Vmax) is reached. From
this graph, Km, a value equal to the substrate concentration at half-maximal velocity, can be
determined.
Vmax
V

Vmax/2

Km
S

A more accurate way to determine the Km is using a plot of 1/V against 1/S. The point where the
line intersects the X-axis is equal to 1/Km. Use the graph with pre-drawn axes provided to plot
a line-of-best-fit for your results.

1/V

- 1/Km

1/S
Km =

From this plot determine Km.

mM

DOUBLE CHECK YOUR Km VALUE (AND UNITS). THEN HOLD UP THE PINK CARD
PROVIDED TO ATTRACT THE ATTENTION OF THE DEMONSTRATOR WHO WILL
RECORD YOUR ANSWER ON THE ANSWER SHEET. (Task P1.T1.5) (6 points)
9

EXAMINATION # 1: Molecular Laboratory

TASK 1

THE DEMONSTRATOR WILL NOW PROVIDE YOU WITH A THEORETICAL GRAPH OF


1/V VERSUS 1/S FOR THE NEXT QUESTION

Alcohol dehydrogenase assays can be used to determine the levels of ethanol added to motor fuels.
Assume that you are a scientist in a testing laboratory and you have been given a 10 mL sample of a
fuel sample to test for the level of added ethanol. Following extraction of the sample to remove all
traces of solvent, you are left with a 100 mL aqueous sample that contains the ethanol. Using
spectrophotometry exactly as before, you test for the ethanol concentration of 0.1 mL of this
aqueous sample. The enzyme rate (V), expressed as A340 /min, that you obtained is 0.175. Assume
that, as before, 0.1 mL of each of the standard ethanol solutions was also tested at the same time,
and these measurments allowed you to plot the graph provided. Use the graph to determine the
molar concentration of ethanol in the original fuel sample.

ENTER THIS VALUE ON THE ANSWER SHEET (Task P1.T1.6) (2 points)

The more usual way of expressing fuel ethanol levels is as a %, i.e. g/100mL. Work out the % fuel
ethanol level for this original sample, given that the molecular formula for ethanol is CH3CH2OH
and using the following atomic masses (g/mol): C = 12, H = 1, O = 16.

ENTER THIS VALUE ON THE ANSWER SHEET (Task P1.T1.7) (2 points)

10

EXAMINATION # 1: Molecular Laboratory

TASK 1

The light absorption spectra for NAD and NADH are shown in the Figure below.

Which of the following statements (1-7) are correct? (Select from answers on next page.)

1. An absorption peak is seen at 340 nm


+

2. Only NAD absorbs light at 340 nm


3. Only NADH absorbs light at 340 nm.
+

4. NAD absorbs light at both 260 nm and 340 nm


5. NADH absorbs light at both 260 nm and 340 nm
6. If the spectrophotometer measurements had been at 330 nm instead of 340 nm the apparent
enzyme rates (A/min) would have been lower
7. If the spectrophotometer measurements had been at 350 nm instead of 340 nm the apparent
enzyme rates (A/min) would have been higher

11

EXAMINATION # 1: Molecular Laboratory

TASK 1

The combination with all of the correct statements is:


A) 1, 2, 4, 6
B) 1, 2, 5, 7
C) 1, 3, 5, 6
D) 1, 2, 5, 6
E) 1, 3, 4, 7
F) 2, 4, 5, 6
G) 1, 2, 4, 7
H) none of the above
I)

all of the above

ENTER YOUR ANSWER ON THE ANSWER SHEET (Task P1.T1.8) (2 points)

END OF TASK 1

PLACE YOUR ANSWER SHEET ON TOP OF THE REST OF YOUR PAPERWORK AT YOUR
WORKSTATION

12

Country:

PRACTICAL EXAMINATION # 1: MOLECULAR LABORATORY

This practical examination is composed of two 45-minute Tasks:


Task 1: Measurement of enzyme activity (20 points)
Task 2: Separation of proteins by chromatography and electrophoresis (20 points)
After 45 minutes, competitors will swap tasks.
Competitors are requested to follow instructions when swapping from one task to another.
There is to be no discussion or exchange of any materials between competitors when changing
tasks.
Competitors must not commence the new task until instructed.
THESE INSTRUCTIONS ARE FOR TASK 2

EXAMINATION #1: Molecular Laboratory

TASK 2

GENERAL INSTRUCTIONS

Competitors are advised to read the examination before commencing.

It is recommended that Competitors proportion their time according to the allotted points for each
task and question.

IMPORTANT

All answers must be recorded on the answer sheets provided.

Ensure that your 3 digit code number is written and coded on the top of each page of the answer
sheets.

Using the pencil provided, fill in the appropriate circle on the answer sheet.

EXAMINATION #1: Molecular Laboratory

TASK 2

TASK 2
SEPARATION OF PROTEINS BY CHROMATOGRAPHY AND ELECTROPHORESIS

IMPORTANT : There are two Parts to this Task.


Please read Part A so that you can plan your time before commencing.

PART A
ION EXCHANGE CHROMATOGRAPHY

INTRODUCTION

Ion exchange chromatography is a technique used to separate proteins on the basis of overall
electric charge. Such a separation depends on the acid-base properties of the relevant proteins and
the charge on the chromatographic matrix used to separate them. Since the net charge on a protein
is dependent on pH, ion exchange chromatography is only effective under conditions of defined pH.
At pH 8.0, cation exchange matrices, such as the Hi-Trap SP to be used in this task, carry a
negative charge and therefore bind positively charged proteins. Ions in the solution with a like
charge to the proteins will compete with those proteins for binding to the matrix. Competing ions in
excess prevent or reverse binding of proteins, thereby causing them to be eluted (washed from the
column) from the matrix.

In this experiment you will be provided with a protein sample containing two proteins, albumin and
cytochrome c. Albumin, a major protein of blood plasma, has a molecular mass of 68,000 Daltons
(Da) and consists of a single chain of amino acids. Cytochrome c functions in electron transport in
mitochondria and consists of a single chain of amino acids that is bound to an iron-containing heme
group that absorbs visible light at a wavelength of 410 nm. Cytochrome c has a total molecular

EXAMINATION #1: Molecular Laboratory

TASK 2

mass of 12,400 Da. So called Ribbon structures of these two proteins are shown below: Albumin
(left), cytochrome c (right).

In this practical session you will use ion exchange chromatography to separate albumin and
cytochrome c.

EXAMINATION #1: Molecular Laboratory

TASK 2

MATERIALS AND REAGENTS PROVIDED PER STUDENT

Chemicals
Protein sample: 4 mg/mL albumin and 4 mg/mL cytochrome c
Buffer 1: 50mM Tris-HCl, pH 8.0 buffer.
Buffer 2: 50mM Tris-HCl, pH 8.0 buffer containing 0.5M NaCl
Protein Assay Reagent

Equipment

Column containing cation exchange matrix (Hi-Trap SP)

Clamp to hold column

2 x 5mL disposable syringe (labelled Buffer 1 and Buffer 2)

1 x 1mL disposable syringe (labelled Protein Sample)

Adjustable pipettes plus tips

Yellow sharps bin for tip disposal

96 well microtitre plate

Plastic beaker (labelled Liquid waste)

Safety glasses

Disposable gloves

Marker pen

Blue card (to attract attention of demonstrator)

Answer sheet with pencil and eraser

EXAMINATION #1: Molecular Laboratory

TASK 2

PROCEDURE

1. Label the microtitre plate (on the attached tape) with your blue Bench Card Number AND
Competitor Number. (For example, if your Bench Card Number is 5 and your Competitor
Number is 14-3, label the plate: 5/14-3).
2. The ion-exchange column is equilibrated with Buffer 1 (50mM Tris-HCl, pH 8.0) and ready
for immediate use.
3. Draw up 5mL of Buffer 1 into the 5-mL Buffer 1 plastic syringe.
4. Remove the black screw-on cap from the outlet plug at the bottom of the column.
5. Attach the syringe to the top of the column by firmly pushing into the black adapter
6. Load 1mL of buffer onto the column by gently and evenly depressing the syringe plunger.
Run the waste into the plastic beaker.
7. Now draw up 0.2mL of the protein sample into a 1mL plastic syringe.
8. Load the protein sample onto the column by gently and evenly depressing the syringe
plunger. While loading, start to collect fractions of 4 drops each into each well of Row A of
the 96 well microtitre plate.
9. Once the sample has been loaded, replace the syringe with the 5mL syringe containing
Buffer 1.
10. Continue to collect 4-drop fractions into each well of Row A.
11. When Row A is complete (Fractions 1-12), replace the screw-on plug to the outlet of the
column and remove the syringe from the column.
12. Fill a new 5mL syringe with Buffer 2 (50mM Tris-HCl, pH 8.0 buffer containing 0.5 M
NaCl).
13. Attach the new syringe to the column, remove the screw-on plug and continue to collect 4drop fractions into Row C of the 96 well microtitre plate.
14. When Row C is complete (Fractions 13-24), replace the screw-on plug to the column.
6

EXAMINATION #1: Molecular Laboratory

TASK 2

15. Using an adjustable pipette, transfer 20L from each well in Row A (Fractions 1-12) into
the corresponding well in Row B.
16. Similarly, transfer 20L from each well in Row C (Fractions 13-24) into the corresponding
well in Row D.
17. Using an adjustable pipette, add 200L of the Protein Assay Reagent to each well of Rows
B and D. This reagent reacts with protein to yield a blue colour that can be measured by
spectrophotometry at 595 nm using a plate reader.
18. Check for and eliminate air bubbles in the wells of your 96-well microtitre plate (do this
carefully, using a clean yellow tip).
19. HOLD UP THE BLUE CARD PROVIDED TO ATTRACT THE ATTENTION OF
THE DEMONSTRATOR to note that your 96-well microtitre plate is ready to be
analyzed. The plate reader will measure the absorbance of the fractions at both 595nm and
410nm. A printout of the results will be returned to you by the demonstrator.

IMPORTANT: MOVE ON TO PART B WHILE YOU ARE WAITING FOR THESE


RESULTS

Questions

ENTER YOUR ANSWERS ON THE ANSWER SHEET

Q1. Which fraction (1-24) contained the first eluted A595 peak

(Task P1.T2.1) (1 point)

Q2. Which fraction (1-24) contained the second eluted A595 peak (Task P1.T2.2) (1 point)
Q3. Which fraction (1-24) contained the A410 peak

(Task P1.T2.3) (1 point)

EXAMINATION #1: Molecular Laboratory

TASK 2

Q4. Subtract the fraction number obtained in answer T2.1 from that obtained in answer T2.2 and
enter the value.

(Task P1.T2.4) (1

point)
Q5. In which fraction (1-24) did the peak of cytochrome c elute?

(Task P1.T2.5) (4 points)

Q6. In your experiment you would have noted that one protein eluted directly from the column but
the second protein needed added salt to be eluted. Consider these statements:

1. The salt neutralised the ionic interaction between the matrix and the second protein eluted
2. The protein that eluted first was more positively charged than the protein eluted with salt
3. The protein that eluted first was more negatively charged than the protein eluted with salt
4. The protein that eluted first did so because it was larger than the protein eluted with salt
5. The protein that eluted first did so because it was smaller than the protein eluted with salt

Which combination of statements is correct?


A. 1, 2
B. 1, 3
C. 2, 3, 4
D. 1, 3, 4
E. 2, 3, 4
F. 1, 3, 5
G. 2, 3, 5
ENTER YOUR ANSWER ON THE ANSWER SHEET

(Task P1.T2.6) (2 points)

EXAMINATION #1: Molecular Laboratory

TASK 2

Q7. In another experiment you added a third protein (Protein X) to the protein sample containing
albumin and cytochrome c and repeated the ion-exchange chromatography separation and detection
exactly as before. The collated results obtained from the plate reader are shown below: The elution
peak corresponding to Protein X is labelled. For convenience, the other two proteins are arbitrarily
labelled A and B.

A595
A410
A

X
12 13

Fraction

Consider the statements below regarding these results:


1. Protein X has a less positive charge overall than protein B
2. Protein X has a more positive charge overall than protein B
3. Protein X contains a non-polypeptide component
4. Protein X is 100% polypeptide
5. Protein X eluted after protein A because it was larger
6. Protein X eluted after protein A because it was smaller

B
24

EXAMINATION #1: Molecular Laboratory

TASK 2

Choose which combination of statements is correct:


A. 1, 3
B. 2, 3
C. 1, 3, 5
D. 2, 3, 6
E. 2, 3, 5
F. 1, 3, 6
ENTER YOUR ANSWER ON THE ANSWER SHEET (Task P1.T2.7) (4 points)

10

EXAMINATION #1: Molecular Laboratory

TASK 2

PART B
TWO-DIMENSIONAL GEL ELECTROPHORESIS

The diagram below shows the results of an experiment where a mixture of albumin, cytochrome c
and other, unknown proteins were separated by two-dimensional gel electrophoresis. In this
technique, the proteins were separated in the first dimension on the basis of their isoelectric point
(pI) followed by separation in the second dimension on the basis of their molecular mass. The
isoelectric point is defined as the pH at which the sum of the positive and negative charges on the
protein is zero. The isoelectric point (pI) for albumin is 4.9 and for cytochrome c is 10.7. The
individual proteins were subsequently detected using a protein stain. Each protein spot has been
given an alphabetical letter identifier.

2
100,000

pH

6
A

F
GH

80,000
60,000

Molecular
Mass (Da)

10

14

12
C

40,000

20,000

M
N
O

QR

S
T
U

5,000

11

EXAMINATION #1: Molecular Laboratory

Answer the following questions:

TASK 2

ENTER YOUR ANSWERS ON THE ANSWER SHEET

Q8. Which spot corresponds to albumin?

(Task P1.T2.8) (2 points)

Q9. Which spot corresponds to cytochrome c?

(Task P1.T2.9) (2 points)

Q10. Phosphorylation is a relatively common modification of proteins that occurs after they have
been synthesised. The proteins affected can have a variable number of attached negatively charged
phosphate groups; this also leads to a slight increase in their molecular mass.

From the data presented in the figure above, choose which group of proteins best represents the
situation where a parent protein has been modified to generate a number of phosphorylated
species that are less abundant. List the proteins in this group in order, starting with the parent
protein through to the most phosphorylated protein.

(Task P1.T2.10) (2 points)

END OF TASK 2
PLACE YOUR ANSWER SHEET ON TOP OF THE REST OF YOUR PAPERWORK AT YOUR
WORKSTATION

12

Country:

PRACTICAL EXAMINATION # 2: CELL BIOLOGY LABORATORY

This practical examination is composed of 3 Tasks:

Task 1: Differential leucocyte count (16 points)


Task 2: Blood group analysis (11 points)
Task 3: Single radial Immunodiffusion antigen analysis (13 points)

Total Points available: 40

Total time available: 90 minutes

GENERAL INSTRUCTIONS

Competitors are advised to read the examination before commencing.

It is recommended that Competitors proportion their time according to the allotted points for
each task and question.

IMPORTANT

All answers must be recorded on the answer sheets provided.

Ensure that your 3 digit code number is written and coded on the top of each page of the
answer sheets.

Using the pencil provided, fill in the appropriate circle on the answer sheet.

EXAMINATION #2: Cell Biology Laboratory

TASK 1

TASK 1: Differential Leucocyte Count.


Requirement
In this task, you are required to perform a differential leucocyte count and answer two
supplementary questions.

Material and equipment


1.

Binocular microscope - with 10X, 40X, 100X (oil immersion) objective lens and
10X eyepiece lens.

2.

Microscope oil immersion lens

3.

Oil for oil immersion microscopy

4.

Cell maturation charts (provided).

5.

Stained blood smear (Wrights stain).

Procedure
You are supplied with a prepared blood smear that has been stained with Wrights stain.
Wrights stain is a standard haematological stain for blood smears. This blood film was
collected from a patient who has a persistent cough and fever. The patient is a male adult and
had a total white cell count (WCC) of 15.0 x 109/L. You are required to complete a
differential leucocyte count and to record your results in the table supplied. The cell
maturation charts will help you identify the leucocytes.

EXAMINATION #2: Cell Biology Laboratory

TASK 1

How to perform a differential leucocyte count


(i)

It is recommended that you use a 100X oil immersion objective lens. Focus on
10X objective. Put a drop of oil on the slide. Turn the turret carefully to bring the
100X objective into the oil. Focus.

(ii)

Identify and count 100 consecutive leucocytes in a longitudinal strip from the tail
end towards the head of the smear as shown in Figure 1a, recording the occurrence
of each cell.

(iii) The lateral edges of the smear must be avoided. It may not be possible to count 100
consecutive leucocytes in a single longitudinal strip due to the thickness of the
smear and the subsequent difficulty in cell identification. If this situation occurs,
adopt the technique of counting from the tail to the head end and back again as
shown in Figure 1b.

FIGURES 1a and 1b

EXAMINATION #2: Cell Biology Laboratory

TASK 1

(iv) Record the results of the differential count on the answer sheet (Table 1), taking care
that the results are expressed as percentages, and that the total recorded adds up to
100%. An example is given in Figure 1c.

FIGURE 1c: Example of a completed differential count

LEUCOCYTES

Absolute count
(109/L)

Neutrophils
(total of band form and segmented)

60

6.0

Lymphocytes

30

3.0

Monocytes

0.8

Eosinophils

0.2

Basophils

Total

100

10

EXAMINATION #2: Cell Biology Laboratory

TASK 1

TABLE 1: Results of differential leucocyte count.

Type of leucocyte

Occurrence

Absolute count

Reference Range

(%)

(109/L)

(109/L)

Neutrophils

2.0 - 7.5

(total of band form and


segmented)

Lymphocytes

1.5 4.0

Monocytes

0.2 0.8

Eosinophils

0.04 - 0.4

Basophils

0.0.- 0.1

Total WCC

Enter results on answer sheet.

100

15.0

4.0 - 10.0

(14 points)

EXAMINATION #2: Cell Biology Laboratory

TASK 1

Questions
P2.T1.1 How could you improve the accuracy of your differential leucocyte count?
A. Count 50 cells.
B. Count 200 cells.
C. Only include cells that are easily identified.
D. Only use the x40 objective.
E. Count all red blood cells in each field.
(1 point)

P2.T1.2 In the differential leucocyte count, calculation of the absolute count from the
percentage of each cell type is an important step because of which of the following factors?
A. Absolute counts provide an indication of anaemia.
B. Percentage counts do not vary with the type of infection.
C. A reference range (normal range) for each cell type can be determined.
D. Leucocyte numbers cannot be validated from a blood smear.
E. All of the above.
(1 point)

EXAMINATION #2: Cell Biology Laboratory

TASK 2

TASK 2: Blood Group Analysis.


Background
Column agglutination blood grouping cards are used to determine the blood group of
individuals in term of the ABO and Rhesus blood groups.

Material and equipment

1.

Images of 12 column agglutination blood grouping cards. Ten are labelled with
patient identification numbers.

2.

Examples of two column agglutination blood group cards (provided).

Procedure and requirement

You are provided with the images of ten (10) blood group cards, each with a unique patient
identification number. You are required to interpret the ABO and Rh D (Rhesus) blood group
for each patient and record the results in the table provided on the answer sheet. Please refer
to Figure 2 for the ABO grouping reactions table and Figure 3 for the Rhesus group reaction
table. Individuals with the D-antigen are described as Rhesus positive (Rh +) and those
without the D-antigen as Rhesus negative (Rh NEG).

EXAMINATION #2: Cell Biology Laboratory

TASK 2

Additional notes on column agglutination cards for blood grouping.


The cards use the principle of column agglutination. If there is a reaction between the
cells and an antibody the cells agglutinate, and become trapped in the column.
A column may contain antisera, e.g. anti-A, anti-B or in the case of the control no
added antisera.
Columns contain micro glass spheres so they trap agglutinated but not single red blood
cells.
A positive reaction is indicated by trapped red blood cells at the top.
A negative reaction is indicated by red blood cells not being trapped and passing
completely through the column.
The columns on the card from left to right are patient cells added to anti-A, patient
cells added to anti-B, patient cells added to anti-D, control column (patient cells only),
A1 cells (strongest form of A cells) added to patient serum, B cells added to patient
serum.
The control has just cells added to allow for the detection of spontaneous
autoagglutination if this occurs, it makes the results invalid.
For a card to be valid the control column must read negative.
For all cards that are invalid, write INVALID in the ABO Blood Group columns.

Be sure to transfer your answers from Table 2 to the answer sheet.

EXAMINATION #2: Cell Biology Laboratory

TASK 2

Figure 2: ABO grouping reactions table


PHENOTYPE

Anti-A

Anti-B

A1 Cells

B Cells

POS

NEG

NEG

POS

NEG

POS

POS

NEG

AB

POS

POS

NEG

NEG

NEG

NEG

POS

POS

Figure 3: Rh D (Rhesus) grouping reactions table


Rh Phenotype

Anti-D

Rh POS

POS

Rh NEG

NEG

TABLE 2: Results of Patient Blood Grouping


RESULTS POS or NEG

Patient
identification
number

(Anti-)

(Anti-)

A1

(Anti-)

ABO blood
B

Control

(cells)

(cells)

group
(A,B,O or AB)

Rh D
(POS
or
NEG)

P 942715
P 945857
P 942675
P 974199
P 926723
P 976348
P 923413
P 981342
P 917300
P 981398
(8 points)
Enter your results on the answer sheet.

10

EXAMINATION #2: Cell Biology Laboratory

TASK 2

Questions
P2.T2.1 If a persons blood group is O Rh POS which of the following ABO antigens are
present on their red blood cells?
A. A antigens only.
B. B antigens only.
C. Both A and B antigens.
D. Neither A nor B antigens.
E. A1 antigens.
(1 point)

P2.T2.2 A person who has the blood group A Rh NEG has which combination of (non red
blood cell stimulated) ABO antibodies?
A. anti-B.
B. anti-A.
C. anti-A,B.
D. anti-H.
E. None.
(1 point)

P2.T2.3 On the basis of the blood grouping reactions you have recorded in Task 2, which of
the patients is the most likely to have been transfused?
A. Patient P 942715

F.

Patient P 976348

B. Patient P 945587

G.

Patient P 923413

C. Patient P 942675

H.

Patient P 981342

D. Patient P 974199

I.

Patient P 917300

E. Patient P 926723

J.

Patient P 981398
(1 point)
11

TASK 3: Single radial immunodiffusion antigen analysis.

Background
Single radial immunodiffusion (SRID) is used to measure the concentration of
immunoglobulins in blood. It is normally carried out by incorporating an antibody in an
agarose gel at a known concentration and placing samples containing an antigen into the
standardised wells in the gel. At completion of immunodiffusion a stable precipitate forms,
with the diameter squared (D2) having a linear relationship with the antigen concentration.

A standard curve of diameter squared (D2) versus antigen concentration plotted on ordinary
graph paper can then be used to estimate the concentration of a number of unknowns. It is
normal for just three standard points to be used to construct the standard curve.

In this task you are required to construct the standard curve for two immunoglobulins (IgG
and IgA), and then determine the immunoglobulin concentration for two patients. There are
also three supplementary questions on the technique.

Requirement
You are required to construct standard curves for two (2) sets of SRID reactions (IgG and
IgA), and then to determine the concentration of the immunoglobulin from each patient.

Material and equipment


1.

2 x images of SRID (IgG and IgA) plates with standards and unknown.

2.

Reading ruler.

3.

Graph paper.

12

Procedure
You are provided with two SRID plates. These plates have been loaded with standards of
varying immunoglobulin concentrations, a control serum and serum from a patient. The
diffusion has been allowed to come to completion.

For each plate, measure the diameter (D) of the precipitation rings (standards, controls and
unknowns) using the reading ruler provided. (Hint: place the gel over the ruler, aligning the
centre of the well with the central line of the ruler). Move the gel until the outer rim of the
precipitin circle just touches the inside of both divergent lines. Read to 0.1 mm accuracy.

Record the measurements in the table provided.

Plot the square of the diameter (D2) of the precipitin rings against the immunoglobulin
concentration of the standards. For the plot use the graph paper provided, with
immunoglobulin concentration on the horizontal (x) axis and ring diameters squared (D2) on
the vertical (y) axis. A line of best fit is to be drawn through the three points. (Hint: The yintercept should be in the range of 10 mm2 to 12 mm2).

Interpolate the IgG and IgA concentrations of the patient serum from the graphs obtained and
record your results on the answer sheet.

There are three supplementary questions

13

TABLE 3: Results from the analysis of the IgG plate.


Well
Number

Description

IgG
Concentration
(g/L)

Standard 1

2.9

Standard 2

9.2

Standard 3

17.6

Control

14.1

10

Patient A

Diameter
(D) (mm)

D2
(mm2)

The IgG concentration of Patient A is _________________g/L


(5 points)
Enter this value on the answer sheet.
TABLE 4: Results from the analysis of the IgA plate.

Well
Number

Description

IgA
Concentration
(g/L)

Standard 1

1.20

Standard 2

3.55

Standard 3

5.55

Control

2.85

10

Patient B

11

Patient B
(1/4 dilution)

Diameter
(D) (mm)

D2
(mm2)

The IgA concentration of Patient B is _________________g/L


(5 points)
Enter this value on the answer sheet.

14

Questions
P2.T3.1 Why is it that the plot of the line does not pass through the origin?
A. The technique is designed for only low concentrations of antibody.
B. The technique is designed for only low concentrations of antigen.
C. The size of the well introduces a zero error.
D. The gel system expands during incubation, introducing an error.
E. Deformation of the gel due to sample application introduces an error.
(1 point)

P2.T3.2 What could cause a poor (non-linear) calibration curve in this technique?
A. Omission of the control sample.
B. Cloudy gel.
C. Patient serum too dilute.
D. Patient serum too concentrated.
E. Incomplete diffusion.
(1 point)

P2.T3.3 How could you improve the accuracy of this technique?


A. Use a thicker agarose gel.
B. Use concentrated antibodies in the wells.
C. Heat the gels in a dry oven at 37 degrees Celsius.
D. Adjust the antibody concentration in the gel.
E. None of the above.
(1 point)

15

Country:

PRACTICAL EXAMINATION # 3: ORGANISMAL LABORATORY

This practical examination is composed of 4 Tasks:

Task 1:

Dissection of the mouthparts of a grasshopper.

Task 2:

Relationship between form, function and ecology in some insect groups.

(10 points)

(10 points)
Task 3:

Identification of insects to species using a dichotomous key.

(14 points)

Task 4:

Vector efficiency of Anopheles mosquitoes in the transmission of malaria.


(6 points)

Total Points available: 40

Total time available: 90 minutes


1

GENERAL INSTRUCTIONS

Competitors are advised to read the examination before commencing.

It is recommended that Competitors proportion their time according to the allotted points for
each task and question.

IMPORTANT

All answers must be recorded on the answer sheets provided.

Ensure that your 3 digit code number is written and coded on the top of each page of the
answer sheets.

Using the pencil provided, fill in the appropriate circle on the answer sheet.

EXAMINATION #3: Organismal Laboratory

Task 1

TASK 1. Dissection of the mouthparts of a grasshopper. (10 points)

Introduction
Grasshoppers are examples of insects with chewing mouthparts. For this task you will be required
to complete three activities.
i.

Identify, dissect out, and display the individual components of the grasshopper
mouthparts and arrange them as shown in Figure 1 below.

ii.

iii.

(5 points)

Label the dissected mouthparts with the numbered pins according to the numbered
names in Table 1.

(3.5 points)

Identify the functions of some of the mouthparts.

(1.5 points)

Materials and Equipment

1. Grasshopper (Valanga irregularis)


2. A set of instruments (2 pairs of forceps, 2 dissecting needles, 1 pair of scissors)
3. Dissecting dish
4. Unnumbered pins for holding the specimen in place in the dissecting dish
5. Pins marked I VII
6. Latex gloves
7. Dissecting microscope
8. A piece of white foam on which to display mouthparts
9. Piece of paper to indicate competitors number

EXAMINATION #3: Organismal Laboratory

Task 1

Table 1. Names of mouthparts


Code

Name of mouthpart

Mandible

II

Labial palp

III

Labrum

IV

Hypopharynx

Maxilla

VI

Maxillary palp

VII

Labium

Figure 1. Labelling of grasshopper mouthparts

EXAMINATION #3: Organismal Laboratory

Task 1

TASK P3.T1.1
1.

Remove the head from the grasshopper body (Please Note, you are allocated only one
grasshopper for this task.)
With a pin secure the head, anterior down, in the wax dissecting dish.
Identify the most posterior component of the mouthparts. Insert your forceps underneath and
remove the part at its base.
Working forwards, remove each component in turn by grabbing it at its base with your
forceps (as close to the head capsule as possible) and pulling it off.

NOTE. Your dissection and display will be photographed, assessed and recorded on a special
control sheet by an attendant. The correctness of the mouthparts preparation and presentation will
be scored. Points will be lost for damage to parts or failure to remove all parts.
If the attendant is busy with another participant, continue with the next task while waiting to have
your dissection assessed.
2.

Arrange the parts on the piece of white foam as shown in Figure 1.

3.

Label the dissected parts with the numbered pins according to the number code in Table 1.

4.

Write your competitor number on the piece of paper pinned to the foam.

5.

Display the tick [] on the green card to indicate to the attendant that you have completed
this task.

6.

Place your dissection to the side of your bench for assessment.


(5 points)

EXAMINATION #3: Organismal Laboratory

Task 1

TASK P3.T1.2 Identify the parts A G of Figure 1 using the number code for the appropriate part
from Table 1.

Labelled Mouthpart

Code for name of mouthpart

A
B
C
D
E
F
G

(3.5 points)
Enter your answer on the answer sheet.
TASK P3.T1.3 By studying each of the mouthparts, determine the primary function. Use the code
number from Table 1 to complete the table below.
Primary function

Code for name of mouthparts

Grinding and crushing of food


Acting as a tongue
Acting as a top lip to form part of
the mouth cavity

(1.5 points)
Enter your answer on the answer sheet.

EXAMINATION #3: Organismal Laboratory

Task 2

TASK 2. Relationship between form, function and ecology in some insect groups. (10 points)

INTRODUCTION
In this task you will investigate the relationship between form, function and ecology in some insect
groups. The task is divided into two parts, Task 2A and Task 2B

Task 2A Determination of the function of insect legs

(5 points)

Introduction
In Task 2A you will study the relationship between the function and structure of the legs of
different insects.
Materials and Equipment
1. A board with 8 insect specimens labelled I VIII
2. Dissecting microscope
3. Slide with plasticine to hold specimens

TASK P3.T2.1 Study the pinned insect specimens provided. To examine each insect, pin the
specimen into the mound of plasticine on the glass slide and place under the dissecting microscope.
Change the position of the pin to view the insect from different angles. To study the underneath of
the specimen, turn the pin upside down and insert the head of the pin into the plasticine. The insect
specimens are labelled I VIII. These insects belong to a number of different orders and have hind
or fore legs modified for special functions. Table 2a below presents a list of these special functions
(Codes A E) and Table 2b provides list of leg modifications necessary to perform the special
functions (Codes a e). Figure 2 explains the terminology used in Table 2b.

EXAMINATION #3: Organismal Laboratory

Task 2

Table 2a. Leg functions

Table 2b. Modifications of leg structure

Code

Function

Code

Modification

Fossorial (digging)

Legs flattened with fringes of hairs/setae

Raptorial (for seizing prey)

Legs with long, narrow coxae, femora with

Saltatorial (jumping)

Gressorial (walking)

Legs short, thickened, spined

Natatorial (swimming)

Legs long with muscular femora

All legs similar in shape and size

strong spines on ventral surface

Figure 2 Terminology of Insect Leg

By closely observing the insect specimens, for each insect leg function (A-E), select one insect
specimen (I-VIII) that has such legs and the type of modification (a-e)
Leg Function

Insect specimen

Leg modification

A. Fossorial (digging)
B. Raptorial (for seizing prey)
C. Saltatorial (jumping)
D. Gressorial (walking)
E. Natatorial (swimming)
(5 points)
Enter your results on the answer sheet.
8

EXAMINATION #3: Organismal Laboratory

Task 2

Task 2B. Relationship between the external morphology and ecology of two ectoparasites
(5 points)

Introduction
Fleas and lice are both external insect parasites of vertebrates. A louse is an example of a parasite
that spends its entire life cycle on its host. A flea is an example of a parasite that does not spend its
entire life cycle on its host. Each possesses morphological adaptations to suit its respective feeding
style and host-associated habitat. This Task examines some of these morphological adaptations and
how they relate to the biology of these insects.
Materials and Equipment
1. 2 slide-mounted specimens
i)

a cat flea (Ctenocephalides felis) and

ii)

a poultry louse (Menopon gallinae)

2. A compound microscope

TASK P3.T2.2 Using the microscope, examine the flea and louse specimens and determine if the
characteristics in the table below are present (+) or absent (-) in each specimen.
Character/Modification

Flea

Louse

body dorsoventrally compressed


tarsal claws
comb-like row of spines on head
body with distinct bristles/setae
elongate mouthparts
obvious eyes
(3 points)
Enter your results on the answer sheet.
9

EXAMINATION #3: Organismal Laboratory

Task 2

QUESTION P3.T2.3 On the basis of your observations, which of the following combinations of
characters would be most important for a parasite that spends its entire life cycle on its host?
(1 point)

A.

Eggs scattered throughout hosts hair/feathers; legs modified for gripping; body dorsoventrally compressed; compound eyes reduced/absent

B.

Eggs cemented onto hosts hair/feathers; legs modified for jumping; body dorso-ventrally
compressed; compound eyes well developed

C.

Eggs scattered throughout hosts hair/feathers; legs modified for gripping; body laterally
compressed; compound eyes reduced/absent

D.

Eggs cemented onto hosts hair/feathers; legs modified for gripping; body dorso-ventrally
compressed, compound eyes reduced/absent

E.

Eggs cemented onto hosts hair/feathers; legs modified for jumping; body laterally
compressed; compound eyes well developed.

10

EXAMINATION #3: Organismal Laboratory

Task 2

QUESTION P3.T2.4. Which combination of characters would most likely be found in an adult
parasite that feeds only on blood?

A.

(1 point)

Piercing and sucking mouthparts; chewing mandibles absent; digestive tract with specialised
area for grinding; muscular pumps to suck blood

B.

Non-piercing mouthparts; chewing mandibles absent; digestive tract not modified for
grinding; muscular pumps to suck blood

C.

Piercing and sucking mouthparts; chewing mandibles absent; digestive tract not modified
for grinding; muscular pumps to suck blood

D.

Non-piercing mouthparts; chewing mandibles present; digestive tract not modified for
grinding; no pumps to suck blood

E.

Piercing and sucking mouthparts; chewing mandibles present; digestive tract not modified
for grinding; muscular pumps to suck blood

11

EXAMINATION #3: Organismal Laboratory

Task 3

TASK P3.T3 Identification of ants to species using a dichotomous key. (14 points)

Introduction
Ants are an important part of most terrestrial ecosystems. They occur in large numbers and are
found in soil, on the surfaces and on vegetation. They can occur around homes where they may be
considered pests but they are gaining increasing significance as bioindicators. For these reasons,
their accurate identification is often required.

Materials and equipment


1. A tray with 10 species of ants in ethanol numbered I - X.
2. A dissecting microscope
3. A dichotomous key
4. A set of instruments (2 pairs of forceps, 2 dissecting needles, a ruler)
5. 3 glass dishes for studying ants under the microscope
6. Plastic pipette

Task P3.T3.1
You are provided with 10 specimens of ants (numbered I to X) and a dichotomous key to ant
species, including the species provided. Figure 3 explains the terminology used in the key.

Identify the ants using the key. You may remove the ants from the vials and place in the glass
dishes for viewing under the microscope. When you have identified each specimen, enter your
answer on the answer sheet by filling in the letter code corresponding to the species identified.

12

EXAMINATION #3: Organismal Laboratory

Task 3

Figure 3: Ant anatomy and terminology.

13

EXAMINATION #3: Organismal Laboratory

Task 3

IDENTIFICATION KEY TO ANT SPECIES


1.

Head and gaster with distinct metallic green or purple lustre; surface of
head, trunk and petiole pitted and rough.................................... Rhytidoponera metallica
Head and gaster not with distinct metallic green or purple lustre;
surface of head, trunk and petiole not pitted and rough................................................. 2.

2.

Colour mainly black or dark brown.............................................................................. 3.


Colour mainly yellow-brown or distinctly black and orange......................................... 6.

3.

Ant length no more than about 3-4 mm ........................................................................ 4.


Ant length more than 5 mm.......................................................................................... 5.

4.

No node on abdominal petiole; tarsi pale yellowish, distinctly


paler than femora .........................................................................Technomyrmex albipes
Abdominal petiole with a single node; tarsi brown ...............................Ochetellus glaber

5.

Propodeum smooth and rounded, without spines .....................Camponotus aeneopilosus


Propodeum with distinct spines............................................................... Polyrhachis sp.

6.

Abdominal petiole 1-segmented; funiculus without a


distinct elongate terminal 3-segmented club ................................................................. 7.
Abdominal petiole 2-segmented; funiculus usually
with a distinct elongate terminal 3-segmented club....................................................... 9.

7.

Gaster pale yellow-green............................................................ Oecophylla smaragdina


Gaster black ................................................................................................................. 8.

Please refer to next page for couplets 8 to 11

14

EXAMINATION #3: Organismal Laboratory

8.

Task 3

Gaster and head black; trunk and petiole orange/brown............ Camponotus consobrinus
Gaster and petiole black; head and trunk orange/brown...............Iridomyrmex purpureus

9.

Propodeum with distinct spines or teeth-like projections ............................................ 10.


Propodeum without distinct spines or teeth-like projections ....................................... 11.

10.

Head and gaster greyish black ...................................................................... Pheidole sp.


Head and gaster pale brown ......................................................... Pheidole megacephala

11.

Funiculus with a distinct terminal 3-segmented club ..................Monomorium pharaonis


Funiculus without a distinct terminal 3-segmented club............. Monomorium destructor
(14 points)

15

EXAMINATION #3: Organismal Laboratory

Task 4

TASK 4. Vector efficiency of Anopheles mosquitoes in the transmission of malaria


(6 points)

Introduction
Malaria is regarded as one of the most prevalent and destructive diseases in the tropics, with over
40% of the worlds population being at risk of infection. The disease is transferred between
humans by mosquitoes, with the main vectors belonging to the genus Anopheles. More than 422
species of Anopheles have been described, of which 68 have been identified as vectors of malaria.
Species differ in their efficiency as vectors, with some being primary or main vectors, and others
acting as secondary or less important vectors. Factors that determine efficiency as vectors include
distribution, feeding and habitat preference, the time of biting and malarial stability.

Table 4a shows the influence of time of biting and habitat preference on vector efficiency.

Table 4a: Influence of time of biting and habitat preference on vector efficiency
Species

Time of Biting Habitat Preference Vector Efficiency

Anopheles 1 10pm - 4am

exophilic

low

Anopheles 2 10pm - 4am

endophilic

high

Anopheles 3 9am 4pm

endophilic

medium

16

EXAMINATION #3: Organismal Laboratory

Task 4

Glossary of terms:
Anthropophilic: Likes to feed on humans
Zoophilic: Likes to feed on animals
Endophilic: Likes to feed and rest indoors
Exophilic: Likes to feed and rest outdoors
Malaria Stability: the chance of the mosquito surviving long enough for the malarial
parasite to become infective. A low value represents unstable malaria, meaning the
mosquito dies before it is capable of spreading infection.

TASK P3.T4.1 Based on the information in Table 4a, the glossary above and the Table below,
rank the six species of Anopheles (Anopheles a Anopheles f) on the answer sheet in rank order
where rank 1 is the most efficient malaria vector and rank 6 is the least efficient malaria vector.
Malaria

Biting

Stability

Species

Distribution

Feeding Preference

Anopheles a

wide

highly anthropophilic

Anopheles b

wide

moderately anthropophilic

exophilic

9am - 4pm

1.8

Anopheles c

wide

zoophilic

exophilic

9am - 4pm

0.5

endophilic

9am - 4pm

1.5

Anopheles d

restricted moderately anthropophilic

Habitat Preference

Time of

endophilic & exophilic 9am - 4am

2.5

Anopheles e

wide

highly anthropophilic

endophilic

10pm - 4am

1.8

Anopheles f

wide

zoophilic

exophilic

9am - 4pm

1.2
(3 points)

17

EXAMINATION #3: Organismal Laboratory

Task 4

TASK P3.T4.2 Figure 4 below is a map of several proposed sites for the construction of a tourist
camp site in an area where Anopheles mosquitoes are found. Table 4b summarises the climatic
conditions of each site. Table 4c lists the five common species of Anopheles (AnophelesI
AnophelesV) found in the vicinity of the five sites. All five species are known vectors of malaria.
Table 4b: Summary of climatic conditions at each proposed campsite
Site

Altitude Daily Max Temp. Daily Min Temp Monthly Rainfall

650m

20C

8C

150mm

200m

25C

13C

100mm

50m

28C

17C

300mm

100m

27C

15C

<50mm

50m

27C

17C

300mm

18

EXAMINATION #3: Organismal Laboratory

Task 4

Table 4c: Species information for Anopheles


Species

Larval Habitat

Anopheles I swamps and stagnant


pools of water

Extra Information
high mortality in temperatures

Anopheles III fast flowing water

0.71

<15C

Anopheles II water collected in tree holes high altitude species (>400m)


and stems of plants

Vector Efficiency

2.49

tolerates temperatures <10C


high larval mortality in

0.22

warm, unshaded waters


Anopheles IV fast flowing water

can tolerate arid conditions

6.54

and high temperatures


Anopheles V swamps and stagnant pools
of water

larvae shelter and feed on

1.36

submerged vegetation

19

EXAMINATION #3: Organismal Laboratory

Task 4

Figure 4: Location and vegetation layout of proposed camp sites.

^^^^^^


^^^ ^^

^ ^ Site 1 ^ ^ lll w lll w
w
lll w lll
w
^^ ^ ^ ^ ^ ^ ^ ^
lll
w Site 3 lll
^ ^ ^
^ ^ ^ ^ w lll w w
lll
w
lll w

^ ^ ^ ^ ^

lll w
w
lll
w
lll

^ ^ ^ ^ ^ ^ ^
lll
III
w

^ ^ ^ ^ ^ ^ ^

ww
Site
2
^ ^
^
^^ w w
w
w


^
^
^
^
^
^
Site 4
ww w w w w

^ ^^^ w w w

w w

w w
w
w
w
w Site 5
w
w
w
w
w
w
w
w
w
w
w
w
w
w w
w
w w

LEGEND

^^

Forested areas

www

Swampy area

lll

Low growing, semi-submerged aquatic vegetation

Fast flowing water e.g. stream/river

Open grassland

10km

20

EXAMINATION #3: Organismal Laboratory

Task 4

Based on the information provided for each species of Anopheles


(Anopheles I Anopheles V), select the site (Site1- Site 5) where it is most likely to occur. There is
only one correct site for each species of Anopheles.

(2 points)

Enter your answer on the answer sheet.

TASK P3.T4.3 Using the information provided select the best site (Sites 1 to 5) to locate a tourist
camp site where the risk of contracting malaria would be the lowest, assuming that each species is
confined to the one camp site and the maximum distance species can infect within is 10km.
(1 point)
Enter your answer on the answer sheet.

21

Country:

PRACTICAL EXAMINATION # 4: ECOLOGY LABORATORY

This practical examination is composed of 2 Tasks:

Task 1: Plant responses to nitrogen nutrition and carbon dioxide levels (20 points)
Task 2: Interactions between two aquatic plants (16 points)

Total Points available: 36

Total time available: 90 minutes

EXAMINATION # 4: Ecology Laboratory

TASK 1

GENERAL INSTRUCTIONS

Competitors are advised to read the examination before commencing.

It is recommended that Competitors proportion their time according to the allotted points
for each task and question.

IMPORTANT

All answers must be recorded on the answer sheets provided.

Ensure that your 3 digit code number is written and coded on the top of each page of the
answer sheets.

Using the pencil provided, fill in the appropriate circle on the answer sheet.

EXAMINATION # 4: Ecology Laboratory

TASK 1

TASK 1: Plant responses to nitrogen nutrition and carbon dioxide levels. (20 points)

INTRODUCTION
Forty plants were grown in a mixture of fine sand and sandy loam topsoil, in controlled
temperature enclosures in a glasshouse with natural illumination. The temperature regime
(25oC day, 20oC night) was near the optimum for the species concerned and the relative
humidity was maintained above 75%. Nutrients were supplied as half-strength Hoagland
solution which provided a sufficient and balanced supply of essential macro- and micronutrients except for nitrogen. Five nitrogen treatments were applied, containing nitrate at
concentrations of 0, 1, 2, 4 or 8 mM. Nutrient solutions were supplied to the soil surface
each day until solution drained from the base of the pot. Carbon dioxide concentrations in
the chambers were regulated to either 350 or 700 parts per million by volume (ppm) by
scrubbing CO2 from the air stream entering the chambers and then adding CO2 at the
required rates.

The plants were grown for 20 weeks and the five youngest fully expanded leaves were
harvested.

TASKS
You have been provided with a picture of the five youngest fully expanded leaves from one
plant at the final harvest of the treatment that supplied nitrate at 8 mM and a CO2
concentration of 350 ppm. Please do not write on or mark the photograph.

EXAMINATION # 4: Ecology Laboratory

TASK 1

Question P4.T1.1 From the images of the leaves you have been given, indicate in your
answer sheet the plant group to which this species belongs:
A. Monocotyledon (Monocot)
B. Gymnosperm (Conifer)
C. Pteridophyte (Fern)
D. Bryophyte (Moss)
E. Dicotyledon (Dicot)
(1 point)

Task P4.T1.2 For each leaf image, use the measuring scale to measure the length of the
leaf from the apex to the point where the leaf is attached to the petiole, and the maximum
width of the leaf measured at right angles to the direction of the measurement of leaf
length. Record your measurement to the nearest 1 mm in your answer sheet.

Leaf number

Maximum Length (mm)

Maximum Width (mm)

1
2
3
4
5
(2 points)

EXAMINATION # 4: Ecology Laboratory

TASK 1

Task P4.T1.3 For another plant from a different treatment, the leaf length and width is
given in Table 1.

Table 1: Length and width of five leaves from one plant from another treatment in
the experiment.
Leaf length

Leaf width

Leaf number

Leaf area (cm )


(mm)

(mm)

112

72

Value a

107

71

Value b

104

68

Value c

99

64

Value d

86

57

Value e

Mean

Value f

Value g

Value h

An estimate of the area of each leaf in Table 1, can be obtained by using the equation:
Area (cm2) = [0.0079 X length (mm) X width (mm)] - 0.252

Calculate the values a to h in Table 1 to the nearest 0.1mm or 0.1 cm2 and record them on
your answer sheet. The value h should be calculated by taking the mean of values a to e.
(2 points)

Task P4.T1.4 Calculate the mean leaf area of the plant in Task P4.T1.3 by using the mean
values for leaf length (value f) and leaf width (value g) in the formula given for estimating
the area of a leaf.

(1 point)

Enter the value (to the nearest 0.1 cm2) on the answer sheet.

EXAMINATION # 4: Ecology Laboratory

TASK 1

TASK P4.T1.5 Compare the values for mean leaf area as calculated by the mean of the
area of each individual leaf with that calculated by using the mean length and mean width
values. Indicate which of the following statements is correct:
A. The values are equal
B. The values are unequal because of an unbalanced data set.
C. The values are unequal because of an error in calculation.
D. The values are unequal because of a normal distribution of leaf area data about
the mean.
E. The values are unequal because of a symmetrical distribution of leaf length data
about the mean.
F. The values are unequal because of an irregular distribution of values within the
data sets.
(2 points)

TASK P4.T1.6 Table 2 shows the mean leaf area for the five youngest fully expanded
leaves of plants grown for 20 weeks under two ambient carbon dioxide concentrations and
five nitrate nutrition regimes.

EXAMINATION # 4: Ecology Laboratory

TASK 1

Table 2: Mean leaf area (cm2) for the five youngest fully expanded leaves on
plants grown under two ambient CO2 concentrations and five nitrate concentrations.
Ambient CO2 concentration (ppm)
Nitrate (mM)
350

700

19.8

17.2

33.0

31.6

44.4

45.6

50.2

51.5

39.5

53.2

On the graph paper provided, prepare a graph comparing mean leaf area against nitrate
concentration in the nutrient solution for each of the two CO2 treatments. From these plots,
which of the following statements is/are most likely to be correct?
I

At both 350 and 700 ppm CO2 and at nitrate concentrations in the nutrient
medium between 0 and 2 mM, leaf area is limited by nitrogen availability.

II

At 700 ppm CO2, photon flux density is likely to limit leaf size at nitrate
concentrations in the nutrient medium higher than 4 mM.

III

This evidence proves that, at 350 ppm CO2, total plant leaf area is reduced when
nitrate supply increases from 4 to 8 mM.

A.

I only

B.

III only

C.

II and III only

D.

I and II only

E.

I, II and III
(2 points)

EXAMINATION # 4: Ecology Laboratory

TASK 1

TASKS P4.T1.7 to P4.T1.10 relate to information in Tables 3a and 3b.


Tables 3a and 3b show the values of leaf mass of the five youngest fully expanded leaves
from one plant grown at 350 ppm CO2 and one plant grown at 700 ppm CO2. You are
asked to determine whether the values for leaf mass at the two CO2 concentrations are
significantly different using unpaired values (Table 3a) and paired values (Table 3b). The
appropriate test is Students t-test. Instructions for the use of this test and a table of values
of t can be found in Appendices A and B. Please also refer to the abbreviations given at the
end of Table 3b. To assist, some of the data has been processed. You may write on the
exam paper but ensure you provide the appropriate answers on the answer sheet.

EXAMINATION # 4: Ecology Laboratory

TASK 1

Table 3a: Leaf mass for plants grown at 350 ppm and 700 ppm CO2. Calculations for unpaired values
Leaf

350 ppm
CO2 (X)

Number
Leaf mass
(mg) (X)

700 ppm
CO2
X-X

(X X)

Leaf mass
(mg) (Y)

Y-Y

(Y Y)

448

619

383161

104

10816

428

593

351649

78

6084

415

484

234256

-31

961

386

479

229441

-36

1296

370

400

160000

-115

13225

Sum ( Xi, (Xi ), etc.)

2575

1358507

32382

Mean (X, Y, D)

515.1

(Xi) /n

6630625
2

0.5

SD = [((Xi ) ((Xi) )/n)/(n-1)]

90.0

Difference between means


2

0.5

S = [((X-X) + (Y-Y) )/(nx+ny-2)]


0.5

t = (X-Y)/(S[2/(nx+ny)] )

EXAMINATION # 4: Ecology Laboratory

TASK 1

Table 3b: Leaf mass for plants grown at 350 ppm and 700 ppm CO2. Calculations for
paired values
Leaf

Leaf mass (mg)

number

350 ppm
CO2 (X)

D = (YX)

D-D

(D D)

700 ppm
CO2 (Y)

448

619

428

593

415

484

386

479

370

400

Sum ( Xi, etc.)

2575

Mean (X, Y, D)

515.1

Difference between means


2

SD = ([((D-D) )/(n-1)]

0.5

0.5

)/(n )

t = (D - ((D-D))/ SD

Abbreviations:
n, number of individuals in sample
S, sum of variates
t, Students t
D, difference between pairs of variates
D, mean of differences between pairs of variates
df, degrees of freedom
P, probability of a significant difference
SD, Standard deviation
10

EXAMINATION # 4: Ecology Laboratory

TASK 1

TASK P4.T1.7 Calculate the standard deviation about the mean leaf mass for the 350 ppm
CO2 treatment in the unpaired value comparison and enter the value (to the nearest 0.1 mg)
in your answer sheet.

(1 point)

TASK P4.T1.8 Use the Students t-test to determine the level of probability that the leaf
mass at 350 ppm CO2 is not significantly different from the leaf mass at 700 ppm CO2 in
the unpaired test (Table 3a) is:
A.

Greater than 0.99

B.

0.95 to 0.99

C.

0.05 to 0.10

D.

0.01 to 0.05

E.

Less than 0.01

(3 points)

TASK P4.T1.9 Use the Students t-test to determine the level of probability that the leaf
mass at 350 ppm CO2 is not significantly different from the leaf mass at 700 ppm CO2 in
the paired test (Table 3b) is:
A.

Greater than 0.99

B.

0.95 to 0.99

C.

0.05 to 0.10

D:

0.01 to 0.05

E.

Less than 0.01

(3 points)

11

EXAMINATION # 4: Ecology Laboratory

TASK 1

TASK P4.T1.10 Indicate which of the following statements is correct::


A.

The two tests produce the same level of significance of the difference between the
means.

B.

The variation in significance of the difference between means in the two tests is due
to an error in the formula.

C.

The variation in significance of the difference between means in the two tests is due
to random variation in one sample.

D.

The variation in significance of the difference between the means in the two tests is
due to non-random variation in one sample.

E.

The variation in significance of the difference between the means in the two tests is
due to non-random but matching variation in both samples
(1 point)

TASKS P4.T1.11 to P4.T1.12 relate to information in Table 4.

Table 4: Leaf mass of plants grown under two ambient CO2 concentrations and five
concentrations of nitrate in the nutrient medium.
Leaf mass (mg)

Nitrate
(mM)

[CO2] 350 ppm

[CO2] 700 ppm

118

103

214

221

310

365

401

514

316

532

12

EXAMINATION # 4: Ecology Laboratory

TASK 1

On the graph paper provided, plot the mean leaf mass for each treatment against the nitrate
concentration in the nutrient solution.

Compare the graphs you have plotted for:


1. leaf area vs. nitrate concentration, and
2. leaf mass vs. nitrate concentration.

TASK P4.T1.11 From the relationships between leaf area, leaf mass and nitrogen
concentration in your graphs, indicate on your answer sheet which of the following
statements is most likely to be correct:
I

At nitrate supply concentrations between 0 and 2 mM, there is a positive


interaction between nitrate concentration in the soil solution and CO2
concentration in the atmosphere in their effects on mean leaf mass.

II

Climate change that results in a doubling of the ambient CO2 concentration is


very likely to increase mean leaf mass significantly in this plant species at all
nitrate concentrations in the root medium between 0 and 8 mM.

III

At 700 ppm CO2, there is no change in leaf mass per unit area at nitrate supply
concentrations between 4 and 8 mM.

A.

I only

B.

III only

C.

I and II only

D.

I and III only

E.

I, II and III
(1 point)

13

EXAMINATION # 4: Ecology Laboratory

TASK 1

TASK P4.T1.12 Indicate which of the following statements are correct:


I

Between 0 and 2 mM NO3-, neither leaf area nor leaf mass is affected by the
concentration of CO2 in the air.

II

The evidence from this study indicates that climate change that results in a
doubling of the ambient CO2 concentration is likely to result in more uniform
plant growth over the surface of the earth

III

Between 4 and 8 mM NO3-, there is a negative interaction between increasing


NO3- and CO2 concentrations in their effects on leaf mass.

A.

I only

B.

II only

C.

III only

D.

I and III only

E.

I, II and III
(1 point)

14

EXAMINATION # 4: Ecology Laboratory

TASK 2

Task 2: Interactions between two aquatic plants

INTRODUCTION
Lemna is a monocotyledon plant and azolla is a fern. Both are floating plants and
reproduce by vegetative means. Azolla has a cyanobacterium associated with its root
system that fixes nitrogen in water with low nitrogen concentrations. The growth of
colonies of these plants can be indicated by counting leaves (lemna) or fronds (azolla), or
by measuring their dry weights at different times. Both of these procedures have
difficulties, and a common ecological expression of the importance of species in a
community is the proportion of land or water surface that they occupy. The following
questions relate to the estimation of the area occupied by two species in a mixed
population. In most ecological studies, it is impossible to measure every individual, and
sampling must be used. The following problem requires you to evaluate a sampling
procedure.

TASKS
To complete TASKS P4.T2.1 to P4.T2.5, you have been provided with a photograph of an
area of fresh water supporting lemna (small bright green leaves) and azolla (the darker
green fern). For reference, we have provided you with a petri dish with samples of lemna
and azolla. You are required to design in part a sampling procedure to estimate the
percentage of area occupied by different components of this plant community. Sampling
may be carried out by (1) random points, (2) randomly placed transect lines, or (3)
randomly placed sampling areas. Please do not write on or mark the photograph.

15

EXAMINATION # 4: Ecology Laboratory

TASK 2

Ten randomly located transect lines (each approximately 50 mm long) have been drawn on
the photograph. Each line has a number that represents the order of its selection and
corresponds to a row in Table 5. Make your measurements in the order indicated by the
line number.
NOTE: You may not need to measure every line. Analyse your data progressively, so that
after you have measured four lines, you calculate the mean and other values as indicated.
With each additional line, your sample size increases, and you should complete another
calculation.

For each transect line that you select, place your measuring scale on the line with the zero
end closest to the line number. Make your measurements on the side of the line closest
to the number. Measure the total length of the line that lies over:
1. lemna
2. azolla
Light coloured areas on the photograph represent open water. Dark coloured areas on the
photograph represent submerged azolla, which should be measured.

Use these values to calculate the percentage of the transect line occupied by each
species. Record the percentage cover values in your work sheet. For each sample size
greater than three (that is four or more sample lines), calculate:
(1) the mean percentage cover of lemna
(2) the standard deviation and confidence limit (probability 0.05) of the mean
percentage cover of lemna. Use the formulae in Appendix A.
(3) the coefficient of variation , which is the ratio (confidence limit)/(mean
percentage cover of lemna).

16

EXAMINATION # 4: Ecology Laboratory

TASK 2

(4) the mean percentage cover of azolla.


(5) the standard deviation and confidence limit (probability 0.05) of the mean
percentage cover of azolla. Use the formulae in Appendix A.
(6) the coefficient of variation , which is the ratio (confidence limit)/(mean
percentage cover of azolla).
Record these values in your work sheet (Table 5).
For each species, plot the coefficient of variation against the sample size.
Repeat the sampling process until the coefficient of variation for the percentage of
area occupied by azolla is less than 0.25. When you have reached this point, enter the
following values in your answer sheet:

TASK P4.T2.1 How many transect lines do you need to measure to obtain a coefficient of
variation for percentage cover of azolla which is less than 0.25?
(2 points)
TASK P4.T2.2 The mean percentage of the area of the photograph occupied by lemna (to
the nearest 1 per cent).

(2 points)

TASK P4.T2.3 The mean percentage of the area of the photograph occupied by azolla (to
the nearest 1 per cent).

(2 points)

TASK P4.T2.4 The confidence limit (P = 0.05) of the mean percentage of the area of the
photograph occupied by lemna (to the nearest 0.1 per cent).
(2 points)
TASK P4.T2.5 The confidence limit (P = 0.05) of the mean percentage of the area of the
photograph occupied by azolla (to the nearest 0.1 per cent).
(2 points)

17

EXAMINATION # 4: Ecology Laboratory

TASK 2

TASK P4.T2.6 Indicate which of the following statements are correct:


I

The confidence limit of the mean area of each species is independent of sample
size.

II

The confidence limit of the mean area for azolla decreases with increasing
sample size

III

For a given sample size, the coefficient of variation for the area of lemna is
greater than the coefficient of variation for the area of azolla because lemna
plants become separated and are redistributed during vegetative growth but
azolla remains in larger colonies.

IV

For a given sample size, the coefficient of variation for the area of lemna is
greater than the coefficient of variation for the area of azolla because the area
occupied by lemna is smaller than the area occupied by azolla.

Sampling by random line transect is completely unreliable.


A. I only
B. V only
C. II and IV
D. II and III
E. II, III and IV
(1 point)

18

EXAMINATION # 4: Ecology Laboratory

TASK 2

Table 5: Percentage Areas (A) of lemna and azolla in a mixture


Lemna
A2

A2

(A)2

Azolla

Sample

Area Mean

SD

CL

CV

Area

number

(%)

(%)

(A)

(A)

A2

Mean

1
2
3
4
5
6
7
8
Abbreviations: SD, standard deviation; CL, confidence limit (P = 0.05); CV, coefficient of variation
Formulae to describe these parameters are included in Appendix A or in the text.

19

A2

(A)2

SD

CL

CV

EXAMINATION # 4: Ecology Laboratory

TASK 2

TASKS P4.T2.7 to P4.T2.10 Ecological studies are often undertaken in order to predict
the future characteristics of plants or plant associations. The data in Table 6 describe the
change in fresh biomass (g) of a colony of lemna growing in a pond with a high nutrient
concentration.

Table 6: Biomass of lemna on selected days after initiation of a culture.


Time

Biomass of plants

(days)

(g)

10

40.2

12

78.5

16

159.7

20

325

24

TASK P4.T2.7 By graphing the data in Table 6 (after transforming the data if necessary),
extrapolate the relationship to estimate the biomass of the colony on Day 24 of the
experiment.
Enter the value (to the nearest gram) on the answer sheet.

(2 points)

TASK P4.T2.8 From the data in Table 6, calculate the Relative Growth Rate (RGR)of the
lemna colony between Day 0 and Day 8, using the formula given below. Enter the value (to
the nearest 0.001 g g-1 day-1) in your answer sheet.
(1 point)

20

EXAMINATION # 4: Ecology Laboratory

TASK 2

RGR = (lnW2 lnW1)


(t2 t1)
where RGR is Relative Growth Rate (g g-1 day-1), W2 is weight (g) at time t2 (days), and
W1 is weight (g) at time t1 (days)

TASK P4.T2.9 From the data in Table 6, calculate the Relative Growth Rate of the lemna
colony between Day 12 and Day 20. Enter the value (to thenearest 0.001 g g-1 day-1)
in your answer sheet.
(1 point)

TASK P4.T2.10 From the graph of lemna biomass vs. time, indicate which of the
following statements are correct:
I

Lemna relative growth rate increased between Day 8 and Day 20

II

The lemna colony grew exponentially between Day 0 and Day 20

III

Lemna growth was not limited by nutrient availability.

IV

Lemna growth was limited by space.

A.

I only

B.

II and III

C.

III only

D.

II and IV

E.

I, III and IV
(1 point)

21

EXAMINATION # 4: Ecology Laboratory

TASK 2

TASKS P4.T2.11 to P4.T2.12 Lemna and azolla have different maximum growth rates,
and when grown in mixture, one species or the other may dominate the site. The data in
Table 7 indicates the initial biomass values for an experiment where different mixtures of
lemna and azolla with a total mass of 10 g were distributed in samples of a medium with a
constant level of nutrition.
Table 7: Distribution of biomass between lemna and azolla in three populations
established as cultures.
Per cent initial
Azolla biomass

Biomass (g)
Azolla initial

Lemna initial

10

25

2.5

7.5

50

75

7.5

2.5

100

10

After three weeks of growth, the plants were separated and weighed, with the results
presented in Table 8.

22

EXAMINATION # 4: Ecology Laboratory

TASK 2

Table 8: Biomass of lemna and azolla grown for three weeks in three cultures as
described in Table 7.
Per cent initial

Final Azolla

Final Lemna

Final total

Azolla biomass

biomass (g)

biomass (g)

biomass (g)

15.6

15.6

25

4.6

13.1

17.7

50

9.6

9.9

19.5

75

14.5

5.4

19.9

100

18.8

18.8

On the same graph, plot the final azolla biomass, final lemna biomass and final total
biomass after three weeks of growth against the percentage of initial azolla biomass.

TASKS P4.T2.11 Indicate which of the following statements most completely describes
the evidence in the graph:
I

The growth of azolla is not affected by the presence of lemna

II

A species that is less vigorous in monoculture should show a relative depression


in growth when grown in mixture with a species that is more vigorous in
monoculture.

III

This experiment proves that azolla releases nitrate ions into the water.

IV

The proportion of lemna in the pond may be expected to increase progressively


with time.

23

EXAMINATION # 4: Ecology Laboratory


A.

I only

B.

II only

C.

II and IV

D.

II, III and IV

E.

I and IV

TASK 2

(2 points)
TASKS P4.T2.12 Indicate which of the following statements is correct:
I

A positive interaction between species is indicated by a curve relating total


biomass to different proportions of the two species (from Table 7) that is convex
in shape.

II

If a species is aggressive in a mixture its final proportion of biomass is greater


than the initial proportion. This characteristic may be independent of the vigour
of the species in a monoculture.

III

This experiment does not demonstrate that the two species interact in their use
of resources during growth.

IV

Lemna derives some benefit from the presence of azolla, but azolla does not
derive any benefit from the presence of lemna.

A.

III only

B.

IV only

C.

I and IV only

D.

II and IV only

E.

I, II and IV only
(2 points)

24

EXAMINATION # 4: Ecology Laboratory

TASK 2

APPENDIX A
A1.1 Determination of standard deviation of a mean
The standard deviation is calculated from the following attributes of a sample:

!X

2
i

, the sum of the squares of each value of the variable X i where i has values from 1

to n.

n is the number of values of the variable X in the sample


2

(! X ) , the square of the sum of all values of the variable X


i

where i has values from 1

to n.

SD =

"X

2
i

(" X )
i

(n ! 1)

A1.2 Confidence limit of a mean


The confidence limit (CL) is derived from the standard deviation by
CL =

t " SD

(n ! 1)

Where t is the value of the distribution of Students t (Appendix B) for the desired level of
probability
(P = 0.05) and the number of degrees of freedom (df) where n is the number of samples in
the analysis. The number of degrees of freedom is (n 1).

25

EXAMINATION # 4: Ecology Laboratory

TASK 2

A1.3 Significance of difference between two sample means


Differences between two samples (variable X and variable Y) can be tested using the
Students t-distribution, which is calculated from the differences between individual values
(Xi and Yi) and the mean values (X and Y) for the two samples where nx and ny are the
numbers of individuals in the two samples. This can be done for either unpaired or paired
values from the two sample groups. If the calculated value of t is greater than the value of t
for a particular level of probability in the table for the degrees of freedom applicable to the
sample, then it can be concluded that the difference between two mean values is real (that
is, significant) at that level of probability.

A1.3.1 Unpaired samples


If the collection of data for the two samples was completely independent, each value from
one sample value must be compared with all values from the other sample. The procedure
can be summarised as

(X ! X )2
(nx +

+ (Y ! Y )
ny ! 2 )

and

(X

"Y)
2
S!
nx + n y

26

EXAMINATION # 4: Ecology Laboratory

TASK 2

The value of t is compared with the value in the Table of distribution of t (Appendix B), for
the level of significance that you want and the number of degrees of freedom, df, where
df = (n-1).

A1.3.2 Paired samples


If the collection of data for the two samples was arranged so that pairs of variables were
measured, the differences between the means can be assessed as follows. The difference
(D) between the two values (X and Y) of each pair enables the mean difference (D) to be
calculated. The algebraic sum of (D D) for all pairs will equal zero. The procedure can
be summarised as

S =

(D-D)2
(n 1)
n

t = D (D D)
S
The value of t is tested in the same way as described above for unpaired samples.

27

EXAMINATION # 4: Ecology Laboratory

TASK 2

APPENDIX B

df

28

INTERNATIONAL BIOLOGY OLYMPIAD


THEORY PROBLEMS

2003, Minsk, Belarus















All IBO examination questions are published under the following Creative Commons license:



CC BY-NC-SA (Attribution-NonCommercial-ShareAlike) https://creativecommons.org/licenses/by-nc-sa/4.0/
The exam papers can be used freely for educational purposes as long as IBO is credited and
new creations are licensed under identical terms. No commercial use is allowed.

14-th International Biology Olympiad


Minsk Belarus,
8th-16th July, 2003

THEORETICAL TEST
Dear competitors!
You will have 4.5 hours for answering all the tasks of parts A and B. Tasks for
part A have only one correct answer. You have to mark it by filling in the circle
opposite the test number on the answer sheet. Answers written in the question paper
will not be taken into account.

Tasks for part B may have several (more than one) correct answers. You must fill
them in the answer sheet part B. The marks for the questions of part B depend on
the number and complexity of the questions.. The marks are shown in the text.

Be attentive while filling in the answer sheet. Make sure the correct circle
corresponding to the appropriate question is filled in. Any corrections in answer
sheet should be avoided!

Note there are some questions which are marked SKIPPED. Do Not answer these.
Please read all possible answers before attempting the question, as many questions
continue over from one page to the next page.

2
PART A
Cell Biology (14 questions, 20 points).

A1. (1 point). List the following proteins in the order of decreasing evolutionary
conservativeness of their primary structure:
1. Somatotropin.
2. Catalytic subunit of a DNA polymerase.
3. Histone H1.
4. Prolamines (storage proteins of cereals).
. 1, 4, 3, 2.
B. 2, 3, 1, 4.
C. 3, 2, 1, 4.
D. 4, 1, 2, 3.
E. 1, 2, 3, 4.
A2. (1 point). What is the common feature of amino acids encoded by codons U, where
is any base, U is uracil?
. Hydrophobicity.
B. Positive charge.
C. Negative charge.
D. Sulfur in the side chain.
E. No common feature.
A3. (1 point). A denatured polypeptide chain containing amino acids of different chemical
properties is shown in the figure.

Amino acid properties:


A and E: Have negatively charged side
groups.

B: With many electropositive atoms.

C and F: Have hydrophobic side


groups.

D: With many electronegative atoms.

3
If renatured, the most stable configuration of the above polypeptide in the cytoplasmic environment
will be:

A.

B.

B D

D B

F
C
C

C.

D.
A
F

A
B D

C
E

A4. (1 point). Nucleoside phosphates can be interphosphorylated enzymatically. Which one of


the following reactions is impossible?
. ADP + ADP = AMP + ATP.
. AMP + GTP = ADP + GDP.
. ATP + GDP = ADP + GTP.
D. ATP + UMP = ADP + UDP.
E. ADP + AMP = ATP + adenosine.
A5. (1 point). Which nucleotides predominate in the genome of extremely thermophilic
bacteria Thermus aquaticus in comparison to E.coli?

A. A-T.
B. C-T.
C. G-A.
D. G-C.
E. T-G.
A6. (2 points). Define from reaction written below:
HOOC

HOOC

COOH

COOH

6.1. (1 point). To which class does the enzyme catalyzing the reaction of formation of
succinic acid (Succinate) from fumaric acid (Fumarate) belong?

A. Isomerase.
B. Dehydrogenase.(Oxidoreductases)
C. Hydrolase.
D. Synthase.
E. Transferase.
6. 2. (1 point). The coenzyme of this reaction is the derivative of which vitamin?
A. B1 - thiamine
B. B2. - riboflavin
C. B6. - pyridoxalphosphate
D. B12. - cyancobalamine
E. Bc. folic acid
A7. (1 point). It is known that cyanides (CN ) and carbon monoxide bind specifically to the
reduced and oxidized form of cytochrome a3 (cyt a3) (part of complex IV of electron
transport chain), respectively, in mitochondria. Which of the following statements are
correct:

1. Cyanides and carbon monoxide are equally toxic to mitochondria.


2. Cyanides are far more toxic for mitochondria than carbon monoxide.
3. Carbon monoxide is more toxic for animals since it is capable of binding other iron-

SKIPPED

containing substances, e.g. hemoglobin.

4. Carbon monoxide is less toxic for animals since it is capable of binding other ironcontaining substances, e.g. hemoglobin.

5. Cyanides are more toxic for animals since they are only capable of binding to
cytochrome a3.
A. 1, 2, 4.
B. 2, 3, 5.
C. 1, 4, 5.
D. Only 4.
. Only 1.

5
A8. (1 point). Lactobacilli lack electron transport chain. However, under special
circumstances, up to 50% of ATP is synthesized by membrane-linked H+ - ATPase.
What are the circumstances to generate a proton gradient to drive ATP formation
mechanism.?
1. If the concentration of lactic acid is higher in the cell than it is in the medium.
2. If the concentration of lactic acid is lower in the cell than it is in the medium.
3. Uniport (unidirectional)of lactic acid.
4. Symport (both in or both out) of lactic acid with H+.
5. Antiport (one in and one out) of lactic acid with H+.
. 1, 3.
B. 1, 4.
C. 1, 5.
D. 2, 5
. 2, 4.
A9. (3 points). The lactose operon of E.coli consists of three genes:
lacZ encodes -galactosidase,
lacY encodes galactosidepermease which carries out lactose transport to the cell,
lacA encodes galactoside-transacetylase.
Lac operon is under the control of LacI (repressor), which is inactive in the presence of
lactose (inductor). There is a wide diversity of the chemical lactose analogs, for example:
Orthonitrophenyl-
-D-galactoside (ONPG) is a substrate for -galactosidase but not an
inductor. The product of this reaction orthonitrophenol is toxic for a cell.
Isopropyl-
-D-thiogalactoside (IPTG) - is an inductor but not a substrate for -galactosidase.
Phenyl-
-D-galactoside (PG) - is a substrate for -galactosidase but not an inducer. The
products of its hydrolysis are nontoxic for a cell.

A9.1. (1 point). Which cells will grow in the medium with PG as the only source of
carbon and energy?

. lacI .
B. lacZ .
C. lacy .
D. lacZ lacy .
E. lacI lacZ .
A9.2. (1 point). Will these cells grow in the medium with ONPG?
A. Yes.
B. No.
A9.3. (1 point). Galactose is a toxic compound for the cells which have galE mutation.
Which cells with this mutation will grow in the IPTG+PG medium (with arabinose
as an additional source of carbon and energy available)?
. lacI .
B. lacZ .
C. lacA .
D. lacI lacA .
A10. ( 2 points). A protein synthesis assay was carried out in vitro. A polyribonucleotide
containing U and C in proportion 1:5 (positions of U and C are random) was used as a
template. Which amino acids and in what proportions will be incorporated into the
synthesized polypeptide molecules?
. 1Phe : 5Pro : 3Leu.
B. 1Leu : 1Pro : 1Ser : 1Phe.
C. 1Phe : 5Ser : 5Pro : 5Leu.
D. 1Phe : 25Pro : 5Ser : 5Leu.
E. 5Leu : 5Pro.

For questions 11 and 12 use the table of genetic code at the beginning of the question paper.
A11. (3 points). The strand of DNA molecule isolated from E. coli bacteria has sequence: 5
GGCCCCCGG 3. Assume that an mRNA is transcribed from the
corresponding double-stranded DNA, the template strand being complementary
to the strand isolated.

A11.1. (1 point). What is the sequence of this mRNA?


. 3 CUCGGUGGGUUCC 5.
B. 5 GUGCCUCCCUGG 3.
C. 5 GGUCCCUCCGUG 3.
D. 5 CCGUCCCGUG 3.
A11.2. (1 point). Which peptide will be synthesized if its translation begins precisely at
5 end of this mRNA? (Assume that start codon is not required).
. - Gly - Tyr - Pro - Ala Asp.
B. - His - Arg - Met - Gly Ile.
C. - Val - Ala - Tyr Pro.
D. - His - Arg - Tyr - Pro Ala.
A11.3. (1 point). When tRNAAla separates from ribosome, which tRNA will bind next?
. tRNATyr.
B. tRNAPro.
C. tRNAVal.
D. tRNAArg.
E. tRNAHis.
A12. (1 point). The transcriptional activity of which kind of RNA polymerase in
eukaryotes can be seen by using a light microscope (without any methods of
colouration)?
A.
B.
C.
D.
E.

RNA-polymerase I.
RNA-polymerase II.
RNA-polymerase III.
Primase.
Impossible to determine.

13. (1 point). Phalloidin, a very toxic compound isolated from the mushroom Amanita
phalloides, has a very high affinity for actin polymers. Phalloidin can be marked by
covalently linking it to a fluorescent molecule, like fluorescein, without affecting its
affinity properties.
If a microscopic slide with methanol-fixed sperm is stained with a reagent containing
fluorescein-marked phalloidin (excess reagent being washed away), which part of the

8
spermatozoids will be glowing under a fluorescence microscope?
. Acrosome.
. Flagellum.
. Head.
D. Mitochondria.
. Whole spermatozoid.
A14. (2 points). On the basis of the following experimental facts, decide which of the four
models (A, B, or D) of Bax and Bcl-2 proteins action in regulation of programmed
cell death (apoptosis) is correct.
Experimental facts:


Mice with inactivated bcl-2 gene had a high rate of apoptosis in various tissues, which
could be corrected by the absence of Bax protein.

Bax gene in a single genome copy was able to promote apoptosis in the absence of
Bcl-2 protein.

However, bcl-2 gene suppressed apoptosis in the absence of Bax protein.

A. Bax protein inhibits the action of Bcl-2 protein, which blocks apoptosis
(look at A in the figure).
B. Bcl-2 protein is an inhibitor of Bax protein, which promotes apoptosis
(look at B in the figure).
C. Bcl-2 and Bax proteins act independently, resulting in either survival or death,
(look at C in the figure).
D. Bcl-2 protein blocks inhibitory action of Bax protein on apoptosis
(look at D in the figure).

9
Plant anatomy and physiology (10 questions, 12 points).
A15. (1 point). If the vascular system of a plant tendril is represented by the only one
closed collateral (xylem & phloem are touching) bundle, the tendril is formed by the
metamorphosis of which organ?

. Shoot.
B. Leaf.
C. Stem.
D. Root.
. Impossible to determine.
A16. (1 point). A transverse microscopic section of a spruce needle leaf is shown in the
diagram below. Which roman numerals indicates the upper surface of the leaf?

II

Phloem
Xylem
III

IV

A. I and II.
B. II and IV.
C. I and III.
D. III and IV.
E. II and III.
17. (1 point). The endosperm in conifers develops from:
. The central nucleus resulting from double fertilization.
B. The ovule after fertilization.
C. The megaspore before fertilization.
D. The megaspore after fertilization.
. The megasporangium cells before fertilization.

10

A18. (1 point). Which compounds are the main substrates for growth of xylophilous fungi
(accomplishing decomposition of wood), which elicit white (1) and brown (2) rot?
1.
A. Cellulose.

2.
Suberin.

SKIPPED
B. Cellulose.

Lignin.

C. Lignin.

Cellulose.

D. Suberin.

Cellulose.

E. Pectin

Hemicellulose.

19. (1 point). Which is the correct rank order of the pH value in cytosol (1), chloroplast
stroma (2) the inside of thylakoids (3) in plant cells exposed to light:
A. 1>2>3.
B. 1>3>2.
C. 2>1>3.
D. 2>3>1.
E. 3>1>2.
A20. (1 point). Spirogyra filaments were placed in a medium, in which strict (obligate)
aerobic bacteria were incubated without access to oxygen for some time. Then part of
the spirogyra filament was illuminated with a narrow beam, which passed through a
prism to obtain a spectrum (see figure below).

400

500

600

700

800 wavelength , nm

In which parts of the filament will the greatest concentration of bacteria be observed?
A. 1,3.
B. 1,4.
C. 2,3.
D. 2,4.
. 3,4.

11
A21. (2 points). Plants of wild type corn whose Rubisco function was normal were
compared with a mutant corn variety whose Rubisco is not able to catalyze an oxygenation
reaction. Which of the following statements regarding the photosynthetic capacity
of this mutant corn and the wild type is correct and why would it be correct?
Assume the same temperature conditions.

Photosynthetic capacity of the


mutant

Reason

It would show much lower capacity


compared to the wild type.

Rubisco in the bundle sheath cell loses its


oxygen fixation capacity.

It would show much lower capacity


compared to the wild type.

Rubisco in the bundle sheath cell loses its


carbon dioxide fixation capacity.

It would show much higher capacity


compared to the wild type.

Since mesophyll cells photorespire,


photosynthetic capacity of the mutant would
not be affected by this mutation.

D.

It would show the same capacity as


the wild type.

Since mesophyll cells photorespire,


photosynthetic capacity of the mutant would
not be affected by this mutation.

E.

It would show the same capacity as


the wild type.

Since CO2 concentration in the bundle sheath


cells is high enough, both wild type and mutant
corn do not photorespire.

12
A22. (2 points). Photosynthesis in plants is dependent on temperature (T) and light intensity
(L). The following graphs show the results of measurements of CO2 consumption for
three plants of the same species under different light intensities. Which combination of
statements concerning limiting factors in the temperature ranges (I) 5 C to 0C and
(II) +20 C to + 30C is correct under the light intensity used?

light intensity
4-fold
CO2-consumption
2-fold

1-fold

-5

10

15

Temperature range from


5 to 0C
(I)

20

25 30
temperature (C)

Temperature range from


+20 to +30C
(II)

A.

T and
L limiting factor.

T and L
not limiting factor.

B.

T limiting,
L not limiting.

T not limiting,
L limiting.

C.

T limiting,
L not limiting.

T limiting,
L not limiting.

D.

T not limiting,
L limiting.

T limiting,
L not limiting.

E.

None of the above combinations is correct

13
A23. (1 point). The result of an experiment which uses guard cell protoplasts of
Vicia faba is given below. Protoplasts were incubated in a suspension medium with
isotonic osmotic pressure. After 30 min under saturating red light they were irradiated
with blue light for 30 sec. During the experiment in which the protoplasts were cultured
the pH of the medium was monitored.

What would be the most plausible conclusion based on the above results?

A. Blue light may help guard cells to take up protons from outside into the cell.
B. Blue light may enhance the ability of guard cells to pump protons out of the cell.
C. Blue light may be a very effective wavelength of light for the respiration of the
guard cells.
D. Blue light may activate all of the protoplasts to give away their energy.
E. Not only blue light but also other wavelengths of light may help guard
cells to transfer protons.

14
24. (1 point). If an oat coleoptile deprived of its epidermis is placed in a physiological
solution with pH = 5.0, relatively fast lengthening of the coleoptile occurs. The action of which
hormone does this experiment imitate?

A. Auxin.
B. Gibberellic Acid
C. Cytokinins.
D. Ethylene.
E. Abscisic Acid

15
Animal Anatomy & Physiology (10 questions, 12 points).

A25. (1 point). In which animals is the volume of the lungs relatively constant during all
the stages of ventilation (breathing)?

A. In insects.
B. In birds.
C. In mammals.
D. In reptiles.

26. (1 point). During the blood flow from the ventricle to atrium in fishes, how does the
pressure change?
A - Atrium.
V - Ventricle.
P - Pressure.

A.

B.

C.

E.

V
D.

16
A27. (1 point). A branched axon is stimulated at the site 1 (see figure below). The excitation
is transferred from site 1 to 2 and then to 3 and 4. The excitation is measured at
these sites. Which statement of impulse frequencies (I) measured at these sites is correct?

3
1

2
4

A. I(1) > I(2) > I(3), I(3) = I(4), I(3) + I(4) = I(2).
B. I(1) > I(2) > I(3), I(3) = I(4), I(3) x I(4) = I(2).
. I(1) < I(2) < I(3), I(3) = I(4).
D. I(1) = I(2) > I(3), I(3) = I(4), I(3) + I(4) = I(2).
E. I(1) = I(2) = I(3) = I(4).
A28. (1 point). Drosophila flies homozygous for the shake mutation are extremely sensitive to
diethyl ether that causes convulsions in homozygous individuals. Convulsions are caused
by abnormalities in nerve impulse conduction. (see graph below). The function of which
structures is impaired in the shake mutations?

mV

shake
wild type

Time

. Na+ -channels.
B. K+ -channels.
C. Ca2+ -channels.
D. K+/Na+ -ATPase.
E. + -pump.

17
A29. (1 point). Daily changes in the concentration of which hormone are represented by
the following graph?

Arrows indicate the time


of food consumption.

A. Thyroxine
B. Glucagon.
C. Insulin.
D. Cortisol.
E. Parathormone.

A30. (1 point). Thyroiditis is an autoimmune disease, which is caused by the hyperactivity of


the thyroid gland. In this disease the TSH (thyroid stimulation hormone) concentration
in the blood is below normal. Antibody binding to hormone receptor sites may activate
or block the receptor.
The cause of this disease is the binding of autoimmune antibodies to:

A. Thyroxin receptors.
B. Thyroxin.
C. TSH receptors.
D.TSH.
E. Thyreoliberin receptors.

18
A31. (3 points). There are two recessive mutations ob and db in mice. These mutations cause
the same phenotype: obesity, adipose tissue hypertrophy and predisposition to obesity
related diseases (hypertension, physiological diabetes insipidus and so on). The
mutations are not linked. Three experiments of parabiosis (surgically joining blood
circulation systems of two mice with different genotypes) were carried out to define the
roles of the products of these genes in weight regulation. Two weeks after the parabiosis,
the weight of each mouse was determined (see table).
ob /ob
Weight

Loss of
weight

wt+

db / db

Without
changes

Without
changes

wt+

Loss of
weight

A31.1. (1 point). What is the consequence of the ob gene:


. Peptide hormone favouring obesity.
B. Peptide hormone favouring loss of weight.
C. Hormone receptor favouring obesity.
D. Hormone receptor favouring loss of weight.
E. Nonpeptide hormone favouring obesity.

A31.2. (1 point). What is the consequence of the db gene:


Peptide hormone favouring obesity.
B. Peptide hormone favouring loss of weight.
C. Hormone receptor favouring obesity.
D. Hormone receptor favouring loss of weight.
E. Nonpeptyde hormone favouring obesity.

ob /ob

+ db / db

Loss of
weight

Without
changes

19

A31.3. (1 point). What segregation by phenotype will be seen in F2 after interbreeding of


individuals with the genotypes ob / ob and db / db ?
. 9:3:3:1.
B. 9 :7.
C. 15:1.
D. 1:2:1.
E. 3:1.
A32. (1 point). If four gold rods are implanted into a tibia-bone of a newborn rat (as
shown in the figure), the distances between which of these rods will be maximally altered
with growth?

2
1
4

3
A. 1 and 2.
B. 2 and 3.
C. 3 and 4.
D. 3 and 1.
A33. (1 point). Quick movement of the individuals of genus Dryocopus (wood-pecker) on tree
trunks is enabled thanks to the fact that:

SKIPPED

A. All its leg fingers are directed forward.

B. Two its leg fingers are directed forward and two leg fingers are directed to the back.
C. Three its leg finger are directed forward and one leg finger is directed to the back .
D. One its leg finger is directed forward and three leg fingers are directed to the back.

20
A34. (1 point). The major difference between humoral immunity and cellular immunity
is that:
A. Humoral immunity is non-specific, whereas cellular immunity is specific for a
particular antigen.
B. Only humoral immunity is a function of lymphocytes
C. Humoral immunity cannot function independently; it is always activated by
cellular immunity.
D. Humoral immunity acts against free-floating antigens, whereas cellular
immunity works predominantly against pathogens that have entered body cells.
E. Only humoral immunity displays immunological memory.

21
Ethology (2 questions, 2 points).

35. (1 point). Which of the following cases result in optimal conditioning (Pavlovian)?
. Unconditional stimulus is delivered before conditional stimulus and unconditional
stimulus is stronger than conditional stimulus.
. Unconditional stimulus delivered before conditional stimulus and unconditional
stimulus is weaker than conditional stimulus.
. Conditional stimulus starts delivered unconditional stimulus and conditional
stimulus is stronger than unconditional stimulus.
D. Conditional stimulus starts delivered unconditional stimulus and conditional
stimulus weaker than unconditional stimulus.

A36. (1 point). The cuckoo (Cuculus canorus) and its hosts is a well studied system of
co-evolution as a long never ending process. A cuckoo lays its eggs in the nest of small
passerines (Passeriformes). The cuckoo and its hosts have adopted different behaviours
that result from the co-evolution between them.
Which combination of the following statements (1 6) are true?

1. The hosts lay their eggs in the afternoon.


2. The cuckoo eats ant eggs.
3. The host is aggressive towards a cuckoo.
4. The cuckoo eggs do not mimic the hosts eggs.
5. The cuckoo is aggressive towards a host.
6. The cuckoo tries to avoid being seen in the host nest.
A. 3 and 6.
B. 4 and 6.
C. 2 and 3.
D. 1 and 5.
E. 4 and 2.

22
Genetics (10 questions, 12 points).

37. (1 point). In birds, for instance chickens, sex is determined by a combination of sex
chromosomes Z and W. At an early age it is difficult to determine their sex. However, it
is commercially very important to distinguish males and females at this age. Using a
genetic marker, it is possible to conduct such crosses so that sex will be determined by
phenotypic expression of the marker gene. On which chromosome must the marker gene
(I) be located and which crossing allows discrimination of the males from females (II)?

Marker gene localization (I)

Crossing (II)

A.

On Z chromosome.

Female with recessive phenotype is crossed with a male


homozygous for dominant allele.

B.

On W chromosome.

Female with recessive phenotype is crossed with a male


homozygous for dominant allele.

C.

On Z chromosome.

Female with dominant phenotype is crossed with a male


homozygous for recessive allele.

D.

On an autosome.

Female with recessive phenotype is crossed with a male


heterozygote.

E.

On Y chromosome.

Female with dominant phenotype is crossed with a male


heterozygote.

A38. (1 point). abcde genes are closely linked on the E. coli chromosome. Short deletions
within this region lead to the loss of some genes. For example:
deletion 1 bde genes
deletion 2 ac genes
deletion 3 abd genes
What is the gene order on the genetic map of the E. coli chromosome?

23
A.
B.
C.
D.
E.

b, c, d, e, a
e, a, c, b, d
a, b, , d, e
c, a, b, d, e
a, b, c, d, e

A39. (2 points). According to the model proposed for floral organization, each whorl is
determined by a unique combination of three genes, namely, A, B and C.
It has been shown that genes A and C mutually repress each other. The expression pattern
of these genes in wild type flowers is shown below.
P

St C

S: sepal formation
P: petal formation
St: stamen formation
C: carpel formation

C B A

gene activity

1 2 3 4
whorls of flower

A39.1. (1 point). The morphology of flower that lacks the functional gene A will be:
.

  St C
1

B.

C P P C
1

D.

C St St C
1

C.

 P St C
1

24
A39.2. (1 point). The whorls of a flower that lacks the functional gene C will be:

A.

B.

St

   C
1

C.

D.

St 

A40. (2 points). Colour of the plant endosperm is determined by a single gene located in the
centromere region. Expression of this gene takes place only in the cells of endosperm.
Experiment 1. Inbred plant line with coloured endosperm (CE) was pollinated by the
pollen of inbred plant line with colourless endosperm (CLE). F1 seeds were with
coloured endosperm.

SKIPPED

Experiment 2. After pollination of F1 plants with pollen of CLE line all F2 seeds were
with coloured endosperm as well.

Experiment 3. After pollination of F2 plants with pollen of CLE line 50% of plant gave
seeds were with coloured and 50% with colourless endosperm.

A40.1. (1 points). According to the results of three experiments, determine which


type of embryo sack is typical for this plant species?

25

. Monosporical.

SKIPPED

B. Bisporical.

C. Tetrasporical.
Meiosis

Mitosis

A40.2. (1 point). What ratio of seeds with coloured and colourless endosperm would be
observed in experiment 2, if the gene of colouration of endosperm were located

SKIPPED

in >50 cM distance from centromere?


. 3:1.
B. 1:3.
C. 7:1.
D. All with coloured endosperm.

E. All with colourless endosperm.

41. (1 point). In humans PKU (phenylketonuria) is a disease caused by an enzyme


dysfunction at step A in the following simplified reaction sequence, and AKU
(alkaptonuria) is due to an enzyme inefficiency in one of the steps summarized as step B
here:

Phenylalanine

tyrosine

2 + 2

26
A person with PKU marries a person with AKU. What are the expected phenotypes for
their children? Note: both diseases (PKU and AKU) are not sex linked. Both parents are not
heterozygous.

A. All children will be ill.


B. All children will be normal
C. Half of their children will have PKU, but the other half will be normal.
D. Half of their children will have AKU, but the other half will be normal.

A42. (1 point). The figure shows the results of electrophoresis of PCR-amplified DNA
fragments obtained from members of a single family: mother (1), father (2) and 9
children. Father and 6 children (3, 5, 7, 8, 10, 11) in this family have symptoms of
Huntingtons disease (HD). Father first showed symptoms of the disease after he was
40 years old; the onset age of the disease in children is shown in the figure near
corresponding DNA fragments. What is the probability of 4th, 6th and 9th child in
this family falling ill with the disease?

9 10 11
2

(CAG)n

11
26
40

27

100

14

30

. Child 4 and child 9 are healthy and will never develop Huntingtons disease,
whereas child 6 has high probability of developing the disease.
. Short PCR fragments correspond to appearance of HD at an early age.
. Child 4, child 6 and child 9 all have chances to develop HD at an older age.

27
D. There is no correlation between the age of children with disease symptoms and the
rate of migration of PCR-amplified fragments.
. Huntington disease is an infectious disease therefore most children of the family
must be ill.

43. (1 point). The long corolla of tobacco is inherited as a recessive monogenic characteristic.
If in a natural population 49% of plants have a long corolla, what is the probability that
the result of test crossing plants with a short corolla from this population in F1 will have
uniformity of progeny?

A. 82,4 %.
B. 51 %.
C. 30 %.
D. 17,7 %.
E. 42 %.
44. (1 point). In a genetically balanced population involving alleles T and t. 51 % of the
individuals show the dominant phenotype. Suddenly the living conditions change
causing death of all recessive individuals before they reach maturity. After this,
conditions return to normality. What will be the frequency of allele t after one
generation?
A. 0,41.
B. 0,3.
C. Impossible to determine.
D. 0,7.
E. 0,58.
45. (1 point). On land the process of evolution proceeds faster than in the sea, because:

A. Life started in the sea.


B. Selection pressure is higher in the sea so surviving is more difficult.
C. More fossils are found in depositions of the sea.
D. Living conditions in the sea are more stable.

28
46. (1 point). The phenomenon of reduction in organism complexity during the process of
evolution is called:

SKIPPED

A. Biological regress.

B. General degeneration.
C. Idioadaptation.
D. Aromorphosis.
E. Disjunction.

29
Ecology (8 questions, 10 points).
A47. (3 points). The shell of the land snail shows variation in both colour and banding
pattern. In order to construct a 5-figure banding formula, bands are numbered
from the top of the largest whorl, as shown in the diagram. 0 is used to represent
the absence of a band and square brackets indicate the fusion of two bands.

A47.1. (1 point). Using the appropriate letter, indicate the banding formula of shell S.
. 030[45].
B. 03045.
C. 02045.
D. 003[45].
A47.2. (1 point). Thrushes (which have good colour vision) smash the shells of land
snails against stones (anvils) in order to feed on the soft inner body. If snail types
P, Q, R and S began in equal numbers in a habitat of grassland, which would be

30
the most popular among birds?

A. P.
B. Q.
C. R.
D. S.
A47.3. (1 point). A survey of broken shells collected from thrush anvils amongst dead
beech leaves in a woodland area was carried out. Predict which of the following
sets of results was obtained.

Options
P
13
11
5
6

A.
B.
C.
D.

Broken shells of each type (%)


Q
R
33
1
1
34
1
14
21
20

S
5
6
32
5

A48. (1 point). Which combination of the following statements, referring to the process of
ecological succession, is correct?

1.

Nutrient availability generally increases.

2.

Species diversity decreases as the process proceeds.

3.

A new group of plant species achieves dominance over time and ousts the previous
species.

4.

The height and biomass of the vegetation usually increases as the process proceeds.

5.

Each group of species modifies the habitat making it more favourable for other species.

A. 1, 2, 3.
B. 2, 3, 4.
C. 3, 4, 5.
D. 1, 3, 4, 5.
E. 1, 2, 4, 5.

31
A49. (1 point). Which matching of factors influencing the growth of a population is correct?

Factors independent of the populations


density.

Factors depending on the


populations density.

A.

Development of territories,
cannibalism.

Wind, parasites, light.

B.

Migration, amount of food.

Temperature, crowding factor.

C.

Development of territories,
temperature.

Humidity, wind, light.

D.

Overcrowding factor, light.

Wind, quality of the soil.

E.

Parasites, predators.

Quality of the soil, humidity.

A50. (1 point). A typical feature of the climax stage of an ecological succession is:

A. The ecosystem is very stable


B. The increase of biomass is at its maximum.
C. The number of plant and animal species continues to increase.
D. The net production of the ecosystem has remarkable but regular differences from
year to year.
A51. (1 point). In ecological pyramids, normally each higher trophic level is smaller. Possible
exceptions leading to inverted pyramids are:
I.

A pyramid of numbers with one big producer.

II.

A pyramid of mass when producers have a very short life cycle.

III.

A pyramid of energy in extremly hot ecosystems.

Which combination is correct?


A. Only I and II.
B. Only II and III.

32
C. Only I and III.
D. I, II and III.
E. None of these.

A52. (1 point). You and your family are stranded on a remote island with one cow and a large
stock of wheat for cow food. To obtain the highest amount of energy and survive for the
longest period of time, you should:
A. Feed the wheat to the cow, then drink the milk.
B. Eat the cow, then eat the wheat.
C. Feed the wheat to the cow, drink the milk, then eat the cow.
D. Drink the milk, eat the cow when milk production ceases, then eat the wheat.

A53. (1 point). If an area has a total energy, K, in the sunlight available, the net energy
productivity of the fourth trophic level in the area is roughly:
A. 10 3 K
B. 10 5 K
C. 10 7 K
D. 10 4 K
E. 10 6 K

33
A54. (1 point). Assume first that the graph below shows the changes in two populations of
herbivores in a grassy field. A possible reason for these changes is that:

No. of
organisms

B
A
Time

A. All of the plant population in this habitat decreased.


B. Population B competed more successfully for food than did population A.
C. Population A produced more offspring than population B did.
D. Population A consumed the members of population B.
E. Over time, both populations will have the same average number.

34
Biosystematics (6 questions, 6 points).

A55. (1 point). To assign ascidia to subphylum Urochordata it is necessary to


know the features of the larval stage of ascidia. Which is the correct combination of
statements I-IV ?

I.

They possess a notochord in the larval stage.

II.

They are highly specialised.

III.

They possess a hollow dorsal neural tube, which in metamorphosis is reduced.

IV.

They possess a propulsive tail, pharynx and branchial slits

A. I.
B. II.
C. I and II.
D. I, III and IV.
E. I and III.
A56. (1 point). Which are the characteristics of Cnidaria ?
A. Oceanic/marine or freshwater, mainly predators.
B. Only oceanic/marine, mainly predators.
C. Oceanic/marine or freshwater, filter feeding.
D. Only oceanic/marine, always filter feeding.
E. Only freshwater, predators or parasites.

57. (1 point). Which of the following statements can be used as evidence to prove the close
evolutionary relationship between Phylum Annelida and Phylum Mollusca?
A. Both of them have bodies with bilateral symmetry.
B. Their digestive systems have similar parts.
C. Their bodies consist of similar tegmata (segments).
D. Both of them have a closed circulatory system.
E. Many molluscs and marine annelids have a trochophore larva in their life cycle.

35
A58. (1 point). Zoologists place chordates and echinoderms on one major branch of the
animal phylogenetic tree, and molluscs, annelids, and arthropods on another major
branch. Which of the following is a basis for this separation?
A. Whether or not the animals have skeletons.
B. What type of symmetry they exhibit.
C. Whether or not the animals have a body cavity.
D. How the body cavity is formed.
E. Whether or not the animals are segmented.
A59. (1 point). Phylogenetic connections between three extant (a, b, c) and two extinct (d, e)
taxonomic groups are shown below in the cladogram. What kind of their association into
a taxon of the highest rank (encircled with dotted line) would be in concord with
principles of natural systematics (monophyletic or paraphyletiv groups)?

SKIPPED
b

d
e

d
e

36
A60. (1 point). There are five species (K, L, M, N, O) in a single family. They belong to the
same genus. The table lists data concerning the presence or absence of six features in these
species:

Species

K.
L.
M.
N.
O.

Features
1
+

3
+

4
+

5
+
+

Based on the assumption that the most probable scheme of phylogenetic development
is that which required the least number of evolutionary changes, indicate the species
that is the most probable ancestor of species O.
A. K
B. L
C. M
D. N

37
PART B
Cell biology (10 questions, 51 points).

B1. (6 points). It is known that ribosomes of cytoplasm, ribosomes of endoplasmic reticulum


(ER) and mitochondrial ribosomes take part in protein biosynthesis. Write the numbers
of the proteins in the list below in the correct box, according to the site of their synthesis.

1. Elastin

5. Glycogen synthase

9. Prothrombin

2. Collagen

6. Receptors for glucagon

10. Keratin

3. Somatotropin

7. Casein

11. Lactate dehydrogenase

4. Actin

8. Phosphofructokinase

12. Tubulin

Answers:
ER-bounded ribosomes

Cytoplasmic ribosomes
Mitochondrial ribosomes

B2. (9 points). The Human condition albinism is inherited in the autosomal recessive manner
(see figure). The cause of this condition is a mutation from wild type allele A to recessive
allele a, which introduces a stop codon into the middle of the gene, resulting in a shortened
polypeptide. The mutation also introduces a new target site for a restriction enzyme, which
makes it possible to detect mutated genes by restriction mapping.

38

Task:
Depict the expected results of Southern-, Northern-, Western-blot hybridization analyses of all
genotypes (, , ). Results of Southern-blot hybridization should be depicted according to the
length of the largest restriction fragment (11 kb) and length markers shown to the left of each
Southern-blot hybridization lane. Markers have to do only with the length of DNA fragments.
Results of Northern- and Western-blot hybridization should be depicted without scale, but taking
into account the respective positions of different restriction fragments for different genotypes.

39
B3. (3 points). Three human-mouse hybrid cell lines have been created (X, Y and Z). The
table below summarizes their characteristics. Each cell line has several human chromosomes
carrying genes coding for particular enzymes.

Human chromosome or enzyme

Line

+
+
+
+
+
+

Chromosome 3
Chromosome 7
Chromosome 9
Chromosome 11
Chromosome 15
Chromosome 18
Chromosome 20
Glutathione reductase
Malate dehydrogenase
Galactokinase

Line Y
+
+

Line Z

+
+

+
+

Identify by giving the number, the human chromosome that carries the gene of each enzyme.
Answers:
Gene of Enzyme

A. Chromosome
number

Glutathione reductase
Malate dehydrogenase
Galactokinase

B4. (3 points). Two independent mutations event of a DNA segment lead to the following
results. Mark the type(s) of mutations observed.
(See Genetic Codes in the front of Part A)

5`

codon

3`

G AG

G AA

T AG

40
A. Point mutation.

E. Neutral mutation.

B. Transition.

F. Missense mutation.

C. Silent mutation.

G. Nonsense mutation.

D. Transversion.

Answer

1:
2:
B5. (3 points). Mark the correct statements by + and the incorrect ones by in the
appropriate box.
A.

In any region of the DNA double helix only one chain of DNA that is usually used as
a template for transcription.

B.

In bacteria the transcription of all classes of RNA is carried out by RNA polymerase
of a single type, whereas in eukaryotic cells three types of RNA polymerase are
used.

C.

Formation of the peptide bond is carried out by enzyme peptidyl transferase, which
binds to large subunit of ribosome after the initiation of translation.

D.

Since the start codon for protein synthesis is AUG, methionine is only found in N
termini of polypeptide chains.

E.

Many antibiotics used in medicine today selectively inhibit protein synthesis only in
prokaryotes because of structural and functional differences between ribosomes of
prokaryotes and eukaryotes.

F.

Modified nucleotides, which are in the composition of tRNA molecule, form as a


result of covalent modification of standard nucleotides after their incorporation into
RNA-transcripts.

41
6. (5 points). Oligoribonucleotide X was treated with phosphatase (for removal of 3 and 5 terminal phosphates), then with RNAase T1, which cleaves all phosphodiester bonds
located in a 3 position of guanosine in a 5-specific manner.
N

N
OH

5'
O
O
P

P
O

OH

P
O

3'

P
O

OH

O
O

N
OH

O
O

G
OH

RNAaseT1
As a result, oligonucleotides L, M and N were generated in equal amounts. Each of them
was further treated with phosphatase and subjected to alkaline hydrolysis. Results are
listed in the table below.

Oligoribonucleotide

Content, mole/mole of oligoribonucleotide

UMP (1), AMP (1), CMP (1), Guanosine (1)

AMP (1), Cytidine (1)

CMP (2) , Guanosine (1)

Then experiment was modified: oligoribonucleotide X after treatment with phosphatase was
hydrolyzed with RNAaseP, which cleaves all phosphodiester bonds in a 3-position of
pyrimidines in a 5 - specific manner.

N
OH

5'
O
O

O
O

P
O

OH

P
O

N
OH

P
O

Py
OH

3'

OH

P
O

RNAaseP
This hydrolysis yielded five products in approximately equimolar concentrations: uridine
monophosphate, cytidine monophosphate and oligonucleotides P, Q and R. After resolution
of

42
the mixture and alkaline hydrolysis of these oligonucleotides data listed in the table below
were obtained.

Oligoribonucleotide

Content, mole/mole of oligoribonucleotide

CMP (1), GMP (1)

GMP (1), AMP (1), Cytidine (1)

AMP (1), CMP (1)

Using the results given above, deduce the nucleotide sequence of oligoribonucleotide X.
Answer: ________________________________________

B7. (5 points). The amino acid cysteine (Cys) has three ionizable groups:
- -amino group
- -carboxyl group
- a side chain that can be negatively charged.
The pK values are 8.18, 1.71 and 10.28, respectively. In the answer table, enter the ionic
charge of cysteine at pH 1, 5, 9 and 12.
Using an appropriate letter for each direction, show migration of cysteine in an electric field
at different pH values.
. To cathode ()
. To anode (+)
. Does not migrate

Also in the table, circle the pH value nearest to the pI (isoelectric point) of this amino
acid.
Answer:
pH
1
5
9

Ionic charge

Migrates toward

43
12
B8. (8 points). Listed in the two tables below are vitamins (A-K) and functions (1-12).

Designatio

Vitamin

n
A.

B1 (thiamine)

B.

B2 (riboflavin)

C.

B6 (pyridoxine)

D.

Folic acid

E.

A (retinol)

F.

D (calciferol)

G.

E (tocoferol)

H.

(menaquinone)

I.

C (ascorbic acid)

J.

B12 (cobalamin)

K.

PP (nicotinic acid / niacin)

Numbe Functions of vitamins or consequences of deficiency


r
1.

Antioxidant

2.

Regulation of calcium and phosphate metabolism

3.

Group transfer to or from amino acids

4.

Precursor of light absorbing group in visual pigments

5.

Blood coagulation

6.

Scurvy

7.

Beri beri

8.

Pellagra

44
9.

Anaemia

10.

------leave this part blank--------

11.

Co-Enzymes of dehydrogenases

12.

Rickets

Match each of the vitamins with its appropriate biological functions and/or lack of deficiency
of this vitamin or its derivatives. There may be more than one answer per question.
Answers:
Vitamin
A.
B.
C.
D.
E.
F.

Function

Vitamin
G.
H.
I.
J.
K.

Function

B9. (4 points). The table below shows haploid or partial diploid lac operon of E.coli, where:
- Gene lacI codes for repressor.
- P and O are promoter and operator, respectively.
- LacZ and lacY represent genes encoding for -galactosidase and -galactoside permease,
respectively.
- Oc is a constitutive mutation in the operator.
- Is represents a mutation in the lacI gene, which causes mutant repressor protein not to
be separated from the operator once it binds to it.
Assume that there is no glucose in the bacterial culture medium. In the following table
write O if - galactosidase is synthesized, and X if it is not.

Strain
1
2
3

Genotype
I Oc Z+ Y
I+ Oc Z / I+ O+ Z+
I P+ Oc Z+ Y+/ I+ P O+ Z+ Y

Lactose absent

Lactose present

45
4

I P O Z Y /I P O Z Y

46
B10. (5 points). Match the number of the organism in the left column with the corresponding
letter for the disease in the right column.
Organism
1. Bacillus anthracis

Disease
A. African sleeping sickness

2. Borrelia burgdorferi

B. Anthrax

3. Escherichia coli

C. Cholera

4. Filarial nematodes

D. Elephantiasis

5. Plasmodium vivax

E. Lyme disease

6. Streptococcus pyogenes

F. Malaria

7. Tryponema pallidum

G. Plague

8. Trypanosoma gambiense

H. Tuberculosis

9. Vibrio cholerae

I. Strep throat

10. Yersinia pestis

J. Syphilis
K. Urinary tract infection

Answers:
1

10

47
Plant anatomy and physiology (6 questions, 29 points).

B11. (5 points). The figure shows a cross section of part of a plant leaf.

Indicate which of the following statements concerning this plant are true (+) and which are false ().

1.

Aquatic (Hydrophytic) habitat.

2.

C4 -photosynthetic pathway.

3.

Kranz anatomy

4.

Mesophyll with isolateral organization.

5.

Terrestrial Dry habitat (Xerophytic) and plants of tropics and subtopics.

6.

C3 photosynthetic pathway.

7.

Pinnate venation.

8.

Asteraceae(Compositae) Family.

9.

Poaceae (Gramineae) Family.

10. Parallel venation.

48
B12. (5 points). Label the plant structures in the following diagram, by inserting the number
in the appropriate circle on the answer sheet.

1. Phloem.
2. Xylem.
3. Endodermis.
4. Fascicular cambium.
5. Epidermis.
6. Parenchyma.
7. Cortex parenchyma.
8. Sclerenchyma.
9. Interfascicular
cambium.

10. Collenchyma.

13. (5 points). The potometer can be used to measure transpiration in a cut shoot such as
rose-bay willow plant, by measuring water uptake.

49
Indicate which of the following statements are true (+) and which are false ().
A.

The potometer is usually assembled under water

B.

The water-filled syringe is used to suck water out of the apparatus when
air bubbles appear.

C.

The shoot must be sealed over the cut point with vaseline immediately
after it is cut from the plant.

D.

The hypodermic needle is used to introduce the air bubble into the
potometer.

E.

Enclosing the shoot in a black plastic bag will reduce the transpiration

F.

The rate of transpiration will be high in still, humid air.

G.

The rate of transpiration will be highest in warm, dry moving air.

H.

The rate of water uptake and the rate of transpiration are not always
equal.

I.

Low cohesive properties between the water molecules create problems for
potometer experiments.

J.

Results from potometer experiments can never be quantitative.

B14. (2,5 points). For a short-day plant, indicate which treatments, as listed below,
would inhibit flowering. All the treatments were conducted at night. Mark correct
statements with +, incorrect statements with .
.

Exposure to red light and far-red light, consecutively.

Exposure to red light, far-red light, and red light, consecutively.

Exposure to red light, far-red light, and white light, consecutively.

50
D.

Exposure to white light and far-red light, consecutively.

Exposure to red light, far-red light, white light, red light, and white light,
consecutively.

B15. (6,5 points). Diffusion and osmosis are important for the passive transport of molecules
in the cell.

01. (2,0 points). The figure shows an experiment with a dialysis (visking)
membrane filled with sugar and starch (colorless) suspended in a beaker with
diluted iodine solution (orange brown). Use + to indicate which colour you
would expect in the beaker and in the tube after several hours of dialysis.
Solution in the
beaker.

Solution in the
dialysis tube.
iodine
solution

Colorless
Orange-brown

sugar
starch

Pink-red
Greenishyellow

Blue-black

02. (2.5 points). In a similar experiment, dialysis membranes are filled with solutions
with different concentrations of molecules and left in beakers with solutions with
different molecule concentrations. The dialysis tubes all have the same mass at the
beginning of the experiment. The size of the molecules is bigger than the pore size of
the membrane. Mark with + the experimental settings in which the beaker
contains a hypotonic solution compared to the dialysis tube, and mark with the
ones which do not.

51
Experiment

Concentration in
the dialysis tube
(M).

0.1

0.8

0.4

0.2

0.4

Concentration in
the beaker (M).

0.8

0.1

0.2

0.4

0.4

Hypotonic
solution.

03. (2 points). The tubes are weighed after several hours of dialysis. Their mass is
compared to that before the dialysis. Write the letters of the experiments in the
order of the final mass of the dialysis tube, beginning with the tube having the
lowest mass.

Order of the tubes with regard to their mass:

Answers:

52
B16. (5 points). Which position of sporangia is characteristic of present day representatives of

the higher plants phyla listed below?

II

III

IV

VI
S

B. S- sporangium

Phylum
Bryophyta (Liverworts
and mosses)
Lycopodiophyta
(Club moss)
Equisetophyta
(Horse-tails)
Pterophyta
(Polypodiophyta)
(Ferns)

Plant number

53
Animal Anatomy & Physiology (6 questions, 26 points).

17. (5 points). The graph indicates the blood levels of three hormones produced in
a pregnant woman.

01. (2 points). Using + (true) and (false), indicate whether each of the following is true
or false.
A.

Hormone A is produced by the ovary

B.

Hormone A is human chorionic gonadotrophin.

C.

Hormone A is prolactin.

D.

Hormone A is made by the chorion.

02. (1 point). Which hormone keeps the smooth muscle of the uterus relaxed during
pregnancy? (mark with +).
A.

Progesterone.

B.

Prolactin.

C.

Oxytocin.

D.

FSH.

E.

LH.

54
03. (2 points). Two other hormones, not shown on the graph, are also produced during
pregnancy. These are prostaglandins and oxytocin. Indicate whether the following
statements are true (+) or false ().

A.

These two hormones are produced by the ovaries.

B.

These two hormones are responsible for milk formation.

C.

These two hormones are responsible for contractions of the uterine wall.

D.

These two hormones are made by the endometrium and pituitary gland,
respectively.

B18. (3 point). Name the germ layers of a metazoan embryo from which the following systems
or organs developed:
.

Brain.

Hair.
1. Ectoderm.

Autonomic ganglia.
2. Endoderm.

D.

Lungs.
3. Mesoderm.

Cardiac muscle.

F.

Cartilage.

B19. (3 points). Match the protein (1 to 6) with its function (A to F):


1. Myoglobin.

A. Blood clotting.

2. Prothrombin.

B. Regulation of water excretion.

3. Ferritin.

C. Light-sensitive pigment of rod cells.

4. Vasopressin.

D. Oxygen-storage in skeletal muscles.

5. Collagen.

E. Iron storage in spleen, liver and bone marrow.

6. Rhodopsin.

F. Major fibrous protein of connective tissue.

55
Answers:
1

B20. (4 points). For the curve below, fill in the circles on the answer sheet using appropriate
numbers from the upper figure. In the table, for every number put a correct letter
corresponding to a term given below.

Respiratory volumes
Number in
the diagram
and curve

1.
2.
3.
4.

A. Expiratory reserve volume.


B. Tidal volume.
C. Inspiratory reserve volume.
D. Vital capacity.

Term
letter

56
21. (7 points). How can the resting potential of a cell change after addition of the biologically

active compounds listed below (compound addition is marked by an arrow )?

01. (5 points). Determine which graph reflects the addition of which compound.

Fill the results in the table.

MV
0

MV
0

T,mc
T,ms

T,mc
T,ms

Nistatin (Na+- ionophore):T,ms

___________

Tetrodotoxin (inhibitor of Na+-channels):

___________

Valinomycin (+- ionophore):

___________

MV
0

T,mc

57

02. (2 points). What is the change of transmembrane potential, in graphs 2


and 3 called?

A. Hyperpolarisation.
B. Depolarisation.
C. Repolarisation.
D. Action potential.
E. Overshoot.

Answers:
2 _______________________________________________
3 _______________________________________________
B22. (4 points). A mutation in the haemoglobin gene (HbS) causes sickle cell disease that
produces a cascade of symptoms such as:

1. Anaemia.
2. Sickle shaped red blood cells.
3. Breakdown of red blood cells.
4. Clumping of cells and clogging of small blood vessels.
5. Heart failure.
6. Kidney failure.
7. Brain damage.
8. Damage to other organ.
9. Paralysis.

In the following diagram, the symptom in the box on top of the arrow causes the
symptom in

58
the box below the arrow. Fill the empty boxes with the number of the appropriate
symptoms.

59
Sickle cell (abnormal) haemoglobin

60
Ethology (2 questions, 12 points).

B23. (3 points). Guppies are often called millionaire fishes because of their abundant progeny.

In 1966, Professor C.M. Breder, then director of the New York aquarium, decided to

perform an experiment, in order to learn more about fish reproduction. He put pair of

Guppies (one adult male and one adult female) into a small aquarium, with 27.5 liters of

SKIPPED

water capacity supplied with enough food and oxygen to maintain up to 300 fish. During

the 6 following months and with an interval of 4 weeks between each breeding (these

fishes are ovoviviparous), the female produced 102, 87, 94, 51 and 89 offspring, it means a

total of 443 guppies. A later recount showed that only 9 were alive: 6 females and 3 males.

The rest had been eaten by their own mothers.

In another aquarium with the same size and conditions, the researcher placed 8 adult
males, 8 adult females and 8 young fishes, a total of 24 guppies. Females got abundant
progeny, too. Data of proliferation during the course of the following 6 months from the
introduction of the original group of 24 guppies in the aquarium, are shown in the
following tables.

FEMALE 1
Week 4
Number of offspring
after each hatching

Week 8

Week 12

Week 16

Week 20

Males

29

24

31

30

33

Females

58

48

64

58

68

61
Total
Number of offspring
Males
counted some hours after
hatching
Females
Total

87

72

95

88

101

Observation: The just hatched guppies were devoured by their own mother

FEMALE 2
Week 4
Number of offspring
after each hatching

Week 8

Week 12

Week 16

Week 20

Males

32

26

33

28

29

Females

65

50

66

56

58

Total

97

76

99

84

87

SKIPPED

Number of offspring
Males
counted some hours after
hatching
Females
Total

Observation: The just hatched guppies were devoured by their own mother

FEMALE 3
Week 4
Number of offspring
after each hatching

Week 8

Week 12

Week 16

Week 20

Males

32

29

25

34

28

Females

64

56

51

69

55

Total

96

85

76

103

83

Number of offspring
Males
counted some hours after
hatching
Females

62
Total

Observation: The just hatched guppies were devoured by their own mother

FEMALE 4
Week 4
Number of offspring after
each hatching

Number of offspring
counted some hours after
hatching

Week 8

Week 12

Week 16

Week 20

Males

28

25

35

30

29

Females

57

49

69

61

60

Total

85

74

104

91

89

Males

Females

Total

SKIPPED

Observation: The just hatched guppies were devoured by their own mother

FEMALE 5
Week 4
Number of offspring after
each hatching

Week 8

Week 12

Week 16

Week 20

Males

33

30

30

23

30

Females

67

59

64

47

60

Total

100

89

94

70

90

63
Number of offspring
counted some hours after
hatching

Males

Females

Total

Observation: The just hatched guppies were devoured by their own mother

FEMALE 6
Week 4
Number of offspring after
each hatching

Number of offspring
counted some hours after
hatching

Week 8

Week 12

Week 16

Week 20

Males

30

29

26

35

25

Females

62

57

53

70

52

Total

92

86

79

105

77

SKIPPED

Males

Females

Total

Observation: The just hatched guppies were devoured by their own mother

FEMALE 7
Week 4
Number of offspring after
each hatching

Number of offspring
counted some hours after

Week 8

Week 12

Week 16

Week 20

Males

29

24

33

28

29

Females

60

50

71

57

62

Total

89

74

104

85

91

Males

64
counted some hours after
hatching

Females

Total

Observation: The just hatched guppies were devoured by their own mother

FEMALE 8

Number of offspring after


each hatching

Number of offspring
counted some hours after
hatching

Males

Week 4 Week 8
26
32

Week 12
33

Week 16
28

Week 20
28

Females

52

65

64

58

57

Total

78

97

97

86

85

SKIPPED

Males

Females

Total

Observation: The just hatched guppies were devoured by their own mother

ORIGINAL NUMBER OF FISH


ADULTS
Males

Females

YOUNGS

65
Initial number of guppies in the
aquarium

N of guppies recounted one


year later

Observations: The young of the original establishment were devoured by the adults.
Some adults of the original establishment died by unknown causes.

Which of the following statements arise from the analysis of the previous data? Mark with X
correct statements.

I.

Guppies eat their own offspring (infanticide behaviour).

II.

Guppies show indiscriminate cannibalism, eating all individuals belonging to its

III.

species.
Guppies show selective cannibalism, eating the individuals belonging to its species

SKIPPED

which are shorter than threshold level.


IV. Guppies show selective cannibalism eating only foreign progeny.

24. (8 points). Two young men (Hans and Henri), behaviour researchers of more or less the
same age and appearance, are going to do some investigations about sexual preferences
of human females. For this purpose they select six nice outdoor cafs popular with
young women and hire two similar bikes of which one is provided with an extra child
saddle
(see diagram).

66

Hans and Henri expect that a man having a bike with a childs saddle is more attractive to
young women. This is checked on a sunny afternoon in July. Hans and Henri make a tour
along the six outdoor cafs, indicated A to F. At every caf they halt for 15 minutes. While
standing in front of the caf with their bikes and pretending they are having a talk
together, they both try individually to make eye contact with as many as possible of the
females sitting outside. The numbers are recorded and after each caf Hans and Henri
change bikes. The results of this experiment are shown in the table.
Number of hits (eye contacts) at caf A to F

Total

Hans

12

10

14

17

12

72

Henri

17

10

10

12

20

78

Total

21

27

24

17

29

32

150

Remark: underlined are the hits obtained by man (Hans or Henri)+bike with child
saddle.

Hans and Henri expect that the man with a bike having an extra child saddle will be more
attractive to females than the man with the bike without a child saddle. Possible arguments
supporting this idea are based on the hypothesis that female organisms often show behaviour
focusing on objects related to survival of species.
01. (1 point). Which of the following statements is a correct Null Hypothesis for
the experiment of Hans and Henri?

67
1. Hans and Henri do have the same attractiveness for females.
2. The attractiveness of a man + bike with childs saddle is the same as man + bike without childs
saddle.

3. The six cafs do not differ in the character of the visiting females.
4. Having eye contact between a male and a female is not an indicator of attraction.
5. The attractiveness of a man+bike with childs saddle is greater than that of a man+bike without
childs saddle.

68
02. (1 point). Hans and Henri do some calculations with their results.
Number of hits per caf
Mean (average)

Standard deviation

Hans

12

3.4

Henri

13

4.5

Hans+Henri

25

5.5

Situation A:

15 (nA)

3.7 (SA)

10 (nB)

1.9 (SB)

Man + bike with childs saddle


Situation B:
Man + bike without childs saddle

You have to check the significance of the differences between situation A and B using
the t-test. The following table should be used.
Level of significance

Critical t-value

10.0 %

2.02

5.0 %

2.57

2.5 %

3.37

1.0 %

4.03

0.5 %

6.86

Calculate the standard deviation of the difference between the means of the two situations A and B
in using the formula:

s = { (s A /n A ) + (s B /n B )}
2

s=

69
03. (1 point). Calculate t, using the formula:

t=

t=d/s

d difference between means (situation A and situation B).

04. (1 point). How sure can we be about rejecting the Null hypothesis (i.e. the difference
between situation A and B is significant)
1.

Less than 75.0 %

2.

In between 75.0 % and 90.0 %

3.

In between 90.0 % and 95.0 %

4.

In between 95.0 % and 97.5 %

5.

In between 97.5 % and 99.0 %

6.

In between 99.0 % and 99.5 %

7.

Over 99.5 %

05. (1 point). Hans and Henri show their results to Paula, their boss. Paula claims that
Hans and Henri made a big mistake looking at the total number of hits per caf
since the six cafs differ too much as a spread of 17 up to 32 is too much. Hans and
Henri do not agree with Paula and want to prove their point of view using the
test. Determine the using the following formula.
( )2
=
2

2 =

06. (1 point). Indicate the degree of freedom (df) for this test:

07. (1 point). Determine the probability (P) for this test,


using the following table. Estimate the answer in %.

70
Probability of random deviation (P)

(df)

1
2
3
4
5
6
7

0.995
0.00
0.01
0.07
0.21
0.41
0.68
0.99

0.975
0.00
0.05
0.22
0.48
0.83
1.24
1.69

0.9
0.02
0.21
0.58
1.06
1.61
2.20
2.83

0.5
0.46
1.39
2.37
3.36
4.35
5.35
6.35

0.3
1.07
2.41
3.67
4.88
6.06
7.23
8.383

0.25
1.32
2.77
4.11
5.39
6.63
7.84
9.04

0.1
2.71
4.61
6.25
7.78
9.24
10.65
12.02

0.05
3.84
5.99
7.82
9.49
11.07
12.59
14.07

0.025
5.02
7.38
9.35
11.14
12.83
14.45
16.0

0.01
6.64
9.214
11.35
13.28
15.09
16.81
18.48

08. (1 point). Which of the following conclusions based upon this test is correct? Look
at the total number of hits per cafe

1. The cafs are different, but the differences are not significant
2. The differences between the cafs are significant
3. The results are dubious or questionable, something must be wrong in the design
of this experiment
4. The cafs are not different, but this is not significant
5. The cafs are not different and this is significant

71
Genetics (7 questions, 26 points).

25. (4 points). For each species listed in the table below, indicate whether it can be
routinely used to study, investigate or manipulate one or more of the numbered items.
1. Obtain gene mutations.
2. Obtain chromosomal mutations in eukaryotes
3. Make gene maps.
4. Investigate meiosis.
5. Investigate mitosis.
6. Investigate X-chromosome.
7. Obtain extranuclear mutations.
8. Use Agrobacterium tumefaciens Ti-plasmid for gene transfer to the cells of given
organisms.
9. Perform the gene transfer by transduction.
10. Investigate the lac-operon regulation.
11. Determine the DNA sequences.
Indicate the correct statements by X in corresponding box of answer table:

Object

Item number(s)
1

Zea mays
Drosophila melangaster
Saccharomyces cerevisiae
Caenorhabditis elegans
Escherichia coli
Bacteriophage
Prions

10

11

72
B26. (5 points). The birth records for 4 children were lost at a hospital. The ABO blood
groups of the four babies are known to be A, B, AB, and O. To determine parentage all of
their parents were tested for blood group. (The father of third child wasnt found). The
results are shown in the following table.
01. (4 points). Match the babies with their parents by marking the right blood types in
the table .

Families

Parents 1

Parents 2

Parents 3

Parents 4

Blood group of each


parent
Father

AB

Mother

Father

Mother

Father

Unknown

Mother

Father

Mother

Blood group of a baby

02. (1 points). What is/are the possible blood group(s) the unknown father could have?

B27. (3 points). Connect the terms widely used in population genetics in the left column with
the correct statement in the right column.
Term

Statement

Inbreeding depression.

Fixes advantageous alleles and removes disadvantageous


alleles.

Gene flow.

Increases genetic diversity within and between subpopulations, but occurs rarely.

73
3

Selection.

Increases variation between sub-populations and decreases


variations within sub-populations.

Outbreeding depression.

Fitness reduces due to increase in homozygosity,


expression of deleterious alleles increases as a
consequence of mating between closely related individuals.

Genetic drift.

Reduction of fitness due to mating of genetically divergent


individuals.

Mutation.

Decreases variation between sub-populations and increases


variation within sub-populations.

Term

An
swers:

B28.. (4 points). In an isolated human population of 8400 persons, the frequency of allele IA is
30% and allele IB is 10%.

What is the number and % of people with each blood group?


Group

People number

O
A
B
AB

B29. (4 points). Suppose that the difference between 10 cm high maize and 26 cm high maize
is due to four pairs of additive genes. The individuals with 10 cm have the aabbccdd
genotype and the 26 cm - AABBCCDD.

01. (1 point). Determine the phenotype of F1 if it is known that the parental plants are
10 cm and 26 cm of high.

74
Answer:
F1:
02. (1 point). How many phenotypes classes would be in F2?
Answer:
F2:

03. (1 point). Determine the phenotypes of F2 if it is known, that the parental plants are
10 cm and 26 cm high.
Answers:

04. (1 point). What fraction of the total number of plants in F2 will be 18 cm high ?

Answer:

B30. (4 points). The following

figure shows the distribution of the concentrations of five

hypothetical proteins in a Drosophila embryo. The anterior end is on the left and the
posterior end is on the right. A and B gene products activate the expression of Q gene,
and C and D gene products repress the expression of Q gene.

75
If one of the A, B, C and D genes is mutated, where would the protein Q be found? Choose the
number of the correct answer.
Expression pattern of Q gene
Mutant A
Mutant B
Mutant C
Mutant D

I.

Would be found in the anterior end of the embryo body.

II.

Would be found in the posterior end of the embryo body.

III.

No significant change

IV.

Expression of Q gene would decrease significantly.

76
31. (2 points). It is known that in some dioecious plants sex can be determined genetically as
in animals. Examine the results of analysis of different types of polyploids and ascertain
the type (mechanism) of sex determination in the given plant species.
Choose the correct statement and put its number in the appropriate box.

Rumex acetosa
Genotype

Sex

2+2

2++Y

2A+X+2Y

2A+X+3Y

2A+2X+Y

2A+2X+2Y

3A+X+2Y

3A+X+3Y

3A+X+4Y

3A+2X

3A+2X+Y

3A+2X+2Y

3A+2X+3Y

3A+3X

3A+3X+Y

3A+3X+2Y

4A+2X+2Y

4A+2X+3Y

4A+2X+4Y

4A+3X

4A+3X+Y

4A+3X+4Y

4A+4X

4A+4X+Y

4A+4X+2Y

5A+5X

6A+4X+4Y

A haploid number of autosomes.

1. Sex determination as in human.


2. Sex determination as in Drosophila.
3. Sex determination as in birds.

4. Sex determination as in bees.

Silene latifolia
Genotype

Sex

2+2
2++Y
2A+X+2Y

2A+2X+Y

3A+2X
3A+2X+Y

3A+3X
3A+3X+Y
4A+X+Y
4A+2X
4A+2X+Y
4A+2X+2Y
4A+3X
4A+3X+Y
4A+3X+2Y
4A+4X
4A+4X+Y
4A+4X+2Y

77
5. In given plants X-chromosome determines maleness and Y-chromosome
determines femaleness.
6. The presence of the Y-chromosome is a necessary and sufficient condition for the formation of
male flowers.
7. Y-chromosome doesnt take part in sex determination.
8. X-chromosome doesnt take part in sex determination.

01. Rumex acetosa

02. Silene latifolia

78
Ecology (5 questions, 19 points).

B32. (3 point). Three pond ecosystems (1, 2 and 3) were used for fish production. When the
total number of fish in each pond was measured, the following pyramids were obtained.
(Age of the fish is divided into six class intervals).

VI
V

VI

IV
III

VI

V
IV
III

V
IV

II

II

III

II
I

1.

2.

3.

Assign to these pyramids the appropriate features from the list below. Using letters indicate
the answer(s) in the table.

A.

Pond with very intensive fish cropping.

B.

Pond with selective cropping of baby fish.

C.

Pond with limited fish cropping.

D.

Eutrophic pond.

E.

Pond cropped regularly.

F.

Pond with excessive turbidity and excessive phytoplankton.

G.

Pond with optimal age structure.


ond
1
2
3

Statement

79
B33. (2.5 points). The following figure shows the food web of a certain ecosystem with five
species (A-E). Arrows indicate the flow of energy. Match the letters to the descriptions of
the species:
(C)
(B)

(E)

(A)
(D)

Reducers
/ decomposers
Destruenten

Producer
Herbivore
Omnivore
Carnivore

34. (8.5 points). Fresh water bodies can be subdivided into still-water systems (lentic
waterbodies = ponds and lakes) and moving water systems (lotic waterbodies =
creeks and rivers). Both groups differ in the abiotic factors and in their flora and fauna.
01. (2,5 points). Indicate with a + which characteristics are typical of the lentic and
lotic systems.
Water system characteristic

Water system type


lotic

Rapid decrease of the light density


with the depth
Normally staggered water
temperature
Occurrence of long-lasting plankton
communities
Streamlined animal bodies
Animals with suction cups (suckers)

lentic

80
02. (3 points). Rivers show a marked profile of various water quality parameters along
their length. Samples taken near the source of the river show different values for
various parameters compared to samples from down stream parts of the river.
Mark the expected tendency of this difference using the symbols + for increase,
- for decrease or = for no change.

From near the rivers source

A.

Water temperature.

B.

Oxygen content.

C.

Turbidity.

D.

Amount of sediments.

E.

Amount of nutrient minerals.

F.

Velocity of the flow.

To lower part of the river.

03. (3 points). The graph shows values measured along a river (river continuum). The
P/R ratio represents the ratio of production to respiration in the given part of the
river. From the graph choose the correct parts of the river for the questions below.

biological
diversity

0,1

0,01

mean particle size


organic material

1,0
0,1
0,01

0,001

5
4
3
2

diversity

P/R-ratio

P/R-ratio

particle size

10

1,0

1 2 3 4 5 6 7 8 9 10 11 12
(source)
(lower parts)
Parts of the river

Answer the three questions. Write the numbers of river parts in the boxes.

81
A.

Which parts of the river are autotrophic?

B.

In which parts is organic material (such as tree leaves) essential for the consumers?

C.

In which parts can predators be found?

35. (1 point). A student wished to estimate the size of a population of an endangered water
beetle species in a small pond. He captured 30 individuals, marked and then released
them back in the pond. After 24 hours, once again he captured 30 individuals. Of the
newly captured individuals, only 14 were marked. Assume that no individuals were
born, died, immigrated to or emigrated from the population during the experiment.
What would be the student`s estimation of the endangered water beetle population in
the pond? Estimated population size of endangered water beetle in the pond is:

36. (4 points). The graph shows the productivity of an aquatic ecosystem measured in terms
of dissolved oxygen produced and consumed by green plants and photosynthetic algae
where PS = photosynthesis and R = respiration.

Dissolved Oxygen (mg/L)

30

20

10

0
PS

PS+R

-10

-20

Study the graph and answer the following questions, writing your answers in the box.

82
01. (1 points). Which bar represents net primary productivity?

02. (3 points). An algal bloom occurs until nutrient levels are exhausted. Then the algae
die off and microbial decomposition begins. How will this affect the graph
parameters PS and R?

02.1. (1 point). What will happen during the algal bloom?

1. PS will be increased, R will be decreased.


2. PS will be decreased, R will be increased.
3. PS and R will not change.
4. PS + R will increase.
5. PS + R will decrease.
6. PS + R will remain unchanged.

02.2. (1 point). What will happen after decomposition has begun?

1. PS will be increased.
2. PS will be decreased.
3. R will be increased.

SKIPPED

4. R will be decreased.

5. PS + R will be increase.

6. PS + R will be decrease.
7. PS + R remain unchanged.

83
02.3. (1 point). How would the graphs (parameters PS, R and PS+R) change if the net
community productivity per dissolved oxygen levels was measured?

1. PS will be increased, R will be decreased.


2. PS will be decreased, R will be increased.

SKIPPED

3. 3. PS and R will not change.


4. PS + R will increase.

5. PS + R will decrease.
6. PS + R will remain unchanged.

84
Biosystematics (4 questions, 16 points).

37. (3 points). Below is a list of extant (living) mammalian genera. Assign them to the
continents and subcontinents where they live and indicate the Order to which they belong.
Insert the number of the animal into the correct boxes of tables 01 and 02.
GENUS
1.

Ursus (Bears)

2.

Cebus (New world monkeys)

3.

Pan (Chimpanzees)

4.

Pongo (Orangutans)

5.

Elephas (Elephants)

6.

Macropus (Kangaroos)

01. (1.8 points). ontinents & subcontinents.


Australia
North America
India
Africa
Europe
Asia
South America

02. (1,2 points). Order


Marsupialia
Proboscidea
Carnivora
Primates

85
38. (3 points). Match the terms in the left column ( 1 to 6) with the names of organisms in the
right column ( A to F).

1. Polyembryony.

A. Fasciola

2. Heterogony.

B. Helix.

3. Metagenesis

C. Apis

SKIPPED

4. Hermaphrodity.

D. Ichnenmon

5. Parthenogenesis.

E. Aurelia.

6. Parthenocarpy.

F. Axolotl (Ambystoma).

Answer:
1

B39. (3 points). The cladogram shows the phylogenetic relationships among seven
hypothetical species.

isEa paraphyletic
group
(A)
01. (2 points).
Species
A Which ofBthe following
C
F
G
H and which is a
polyphyletic group (B)?
1. E + F + G.
2. E + F.
3. E + F + G + H.
4. C + E.
5. B + C.
Answer:

SKIPPED

. ___________________

.____________________
02. (1 point). Which species are most closely related?
1. G and H.
2. G and F.
3. H and F.
4. Evolutionary closeness is equal for all species.

86

40. In the figure is shown a well known organism.


01. (1,2 points). Give its systematic position by choosing suitable numbers from the list
below.

87
1 Animalia;

11 Gastropoda;

21 Drosophila;

2 Arthropoda;

12 Annelida;

22 Aphis;

3 Echinodermata;

13 Protozoa;

23 Leptinotarsa;

4 Mollusca;

14 Viviparus

24 Coleoptera;

5 Fungi;

15 Hymenoptera

25

6 Chilopoda;

16

26 Oligochaeta;

7 Insecta;

17 Arachnida;

27 Lepidoptera;

18 Cnidaria;

28 Anopheles;

9 Plantae;

19 Diptera;

29 Locusta;

20

30.

10 Apis;

Kingdom
Phylum:
Class:
Order:
Genus:

02. (1 point). Choose the number corresponding to the type of the insects leg.

1. Leaping.
2. Burrowing.
3. Swimming.
4. Gathering.
5. Walking.
6. Prehensile.

88
03. (1 point). Using the letters, list the leg structural elements this insect possesses in
sequence (beginning with those closest to the body).
. Femur.
. Tibia.
. Trochanter.
D. Coxa.
E. Tarsus.

04. (1 point). Give the number corresponding to the type of insect mouthpart.

1.Piercing-suctorial.
2. Licking.
3. Biting.
4. Suctorial.

05. (1 point). Select the numbers of organs of other organisms, which are homologous to
the wings of the insect concerned.

1. Sparrow wing.
2. Crayfish gills.
3. Bat wings.
4. Fish dorsal fin.
5. Fish pectoral fin.
6. Potato beetle elytrum.
7. Frog legs.

89

06. (0,8 point). In the answer table assign the developmental stages of this insect
according to the letters in the figure.

1.

Sporocyst.

5. Imago.

2.

Egg.

6. Redia.

3.

Graaf vesicle.

7. Pupa.

4.

Larva.

8. Hydatid cyst.

Answer:

90
07. (1 point). What is the significance of the species for humans?

1. Animal and human parasite.


2. Crop pest.
3. Object of genetic investigation.
4. Entomophagous.
5. Vector of sleeping sickness agent.

INTERNATIONAL BIOLOGY OLYMPIAD


PRACTICAL PROBLEMS

2003, Minsk, Belarus















All IBO examination questions are published under the following Creative Commons license:



CC BY-NC-SA (Attribution-NonCommercial-ShareAlike) https://creativecommons.org/licenses/by-nc-sa/4.0/
The exam papers can be used freely for educational purposes as long as IBO is credited and
new creations are licensed under identical terms. No commercial use is allowed.

14-TH INTERNATIONAL BIOLOGY OLYMPIAD


8-16 JULY / MINSK / BELARUS
PRACTICAL / LABORATORY 1
PLANT PHYSIOLOGY, MORPHOLOGY AND ANATOMY

_____________________________________________________________________________________________

Dear Participants!
In the laboratory PLANT PHYSIOLOGY, MORPHOLOGY AND ANATOMY you
will be given the following three tasks:

Task 1. The study of physical and chemical properties of photosynthetic pigments.


Task 2. The study of angiosperm flowers structure.
Task 3. The study of anatomic structure of a plant organ on a cross section.

Duration of the lab work is 60 minutes.


Maximum number of points 68.
You have to write down your results and answers into the ANSWER SHEET which will
be collected by an assistant when the time elapses. It is not necessary to write anything in the
task sheets.
Result sheets taken away from the laboratory will not be accepted!
Please be careful when performing reactions and do not let the reagents and solutions
come in contact with your skin and clothes! Use gloves when necessary!
Contact the assistant in case of any unforseen situations!

Good luck!

_____________________________________________________________________________

Country________________________
First name______________________ Family name ____________________
Code_____________________

2
Task 1. (35 points) The study of physical and chemical properties of photosynthetic
pigments.

The conversion of the energy of light into chemical energy occurs in plants with the help of
pigment-protein complexes of chloroplast membranes. These complexes include photosynthetic
pigments which determine the activity of the primary photosynthetic processes. An
understanding of photosynthesis is impossible without knowledge of photosynthetic
pigment properties. Chlorophyll and other photosynthetic pigments have several specific
properties: absorption of different wavelengths of light, ability to participate in redox
reactions, solubility in different types of solvents, etc.

You have to study several of these properties of photosynthetic pigments during this task.

Materials and equipment

1.

A stand with tubes.

2.

Pipettes.

3.

Ethanol extract of photosynthetic pigments (Flask A).

4.

20 % solution (Flask B).

5.

Distilled water (Flask C).

6.

Petrolic (petroleum) ether (Flask D).

7.

A sheet of white paper.

8.

A water bath.

9.

A tube holder.

10.

10 % HCl solution (Flask E).

11.

Saturated (CH3COO)2Zn solution (Flask F).

12.

Saturated ascorbic acid solution (Flask H).

3
1.1. (8 points) Transfer 3 ml of pigment solution from flask A into tube 1 and also 3ml
into tube 2.
Add five drops of 20% KOH from flask B and 1 ml of 2 (from flask C) to the tube 1 and
to the tube 2 - only 1 ml of 2.
Fill in the missing components of the chemical reaction you have just observed in scheme
1.1 of the answer sheet. Please use the number corresponding to the appropriate formulae
from the list below.

KOH

H2O

1. C55H72O5N4Mg chlorophyll.
2. C34H30O5N4 MgK2 potassium salt of the chlorophyllic acid.
3. C55H74O5N4 pheophytin (phaeophytin).
4. C20H39OH phytol.
5. CH3OH methanol.
6. C2H5OH ethanol.
7. MgCl2 magnesium chloride.
8. KCl potassium chloride.

1.2. (4 points) Add 1 ml of the petrolic (petroleum) ether (from the flask D) to the tubes
1 and 2, shake well and leave to stand until the fractions separate completely.
Determine the colour of each fraction in the tubes 1 and 2. Write down the results in
the appropriate cells of the table 1.2 of the answer sheet. Please use single letter colour codes
as shown below.
A. violet;
B. blue;
C. green;
D. yellow;

E. red;
F. olive brown;
G. black;
H. colourless;

4
Tube

Reagent

Experiment 1.1.

Experiment 1.2.

ethanol fraction colour

petrolic ether fraction


colour

1.3. (4 points) Which pigments are responsible for the colour of the petrolic fraction
on the tubes 1 and 2? Write down in the answer sheet (1.3) single letter codes for the
compounds from the list below:
1: _______
2: _______
A. anthocyanins;
B. carotenoids;
C. phycobilins;
D. chlorophylls;
1.4. (2 points) Add 3 ml of the pigment extract to the tube 3 (flask ) and add 5 drops
of HCl (flask ). Mix the tube contents thoroughly by shaking. Record the new colour.
Add 1 ml of the saturated (CH3COO)2Zn solution (from the flask F) to the same tube. Heat
the solution on the water bath. Mix by shaking andrecord the new colour of the solution.
Write the results down in the table 1.4 of the answer sheet. Please use single letter colour
codes as shown below.
A. violet;
B. blue;
C. green;
D. yellow;

E. red;
F. olive brown;
G. black;
H. colourless.

Reagent

New colour in the tube

HCl
(CH3COO)2 Zn
1.5. (6 points) In the scheme 1.5 of the answer sheet, please write the possible components

5
of the reaction in the tube 3 after addition of hydrochloric acid to the pigment
extract. Please use the number corresponding to the appropriate formula from the list below.

HCl

1. C55H72O5N4Mg chlorophyll.
2. C34H30O5N4 MgK2 potassium salt of the chlorophyllic acid.
3. C55H74O5N4 pheophytin (phaeophytin).
4. C20H39OH phytol.
5. CH3OH methanol.
6. C2H5OH ethanol.
7. MgCl2 magnesium chloride.
8. KCl potassium chloride.

1.6. (1 point) Add 2 ml of the pigment extract and 2 ml of ascorbic acid (flask H) to the
tube 4. Mix by shaking until the colour changes.
Please note the colour change. Put the results in the table 1.6 in the answer sheet. Please
use the single letter colour codes shown below.
A. violet;
B. blue;
C. green;
D. yellow;

E. red;
F. olive brown;
G. black;
H. colourless.

Extract colour before


reaction

Solution colour after reaction

1.7. (6 points) Complete the scheme of this reaction (1.7 in the answer sheet)
using compound and condition numbers from the two lists below:

Compound 2

Conditions

Compound 1

Compounds:
1. chlorophyll;
2. pheophytin (phaeophytin);
3. ascorbic acid;
Conditions:
4. electrons involved;
5. protons involved;
6. light involved.

1.8. (4 points)
Write the results down in the table 1.8 of the answer sheet. Please use single letter colour
codes shown below.

A. violet;
B. blue;
C. green;
D. yellow;

E. red;
F. olive brown;
G. black;
H. colourless.

Compound

1
2

Colour before reaction

olour after reaction

7
Task 2. (12 points) The study of angiosperm flowers structure.

Materials and equipment


1.

Fixed flower preparations (, , ).

x3

2.

Forceps.

x1

3.

Dissecting needles.

x2

4.

A magnifying glass.

x1

2.1. (6 points) Study the morphology of flowers , , . Using formula numbers (1-14)
from the list below, indicate the correct formula for each flower in the answer sheet.

1. * K5 C5 G
2. * 5 G
3. * K5 C5 5+5 G(3)
4. * K(5) C5 5+5 G(5)
5. * K5 C5 G1
6. * K(5) C5 G(5)
7. K(5) C1,2,2 (5+5) G1
8. K(5) C1,2,2 (9)1 G1
9. * K0 C5 5 G (2)
10. * K2+2 C4 2+4 G(2)
11. K(5) C(2,3) 2,2 G(2)
12. * K(5) C(5) 5 G(2)
13. K0 C(5) (5) G(2)
14. * 3+3 3+3 G(3)

* = polysymmetrical
= monosymmetrical

8
2.2. (3 points) The diagrams show the types of ovaries characteristic of angiosperm
flowers. Using the numbers (1-4) from the table below, record the types of ovaries for the
flowers , and in the answer sheet.

1.

2.

Superior ovary
(Hypogynous flower)

Middle ovary
(Perigynous flower)

4.
Inferior ovary
(Epigynous flower)

3.
Half-inferior ovary
(Half -epigynous
flower)

2.3. (3 points) Please indicate in the answer sheet to which family the plants with flowers
A, B and C belong. Use the numbers (1-10) from the list below.

1. Ranunculaceae (buttercups)
2. Oleaceae
3. Rosaceae.
4. Leguminosae (Fabaceae), Papilionaceae.
5. Fagaceae
6. Cruciferae (Brassicaceae).

7. Labiatae (Lamiaceae).
8. Solanaceae.
9. Compositae (Asteraceae).
10. Liliaceae.

10
Task 3. (21 points) The study of anatomic structure of a plant organ on a cross
section.

Materials and equipment

1. Fixed parts of a plant organ.

2. Microscope Axiostar.

3. Forceps.

4. Dissecting needles.

5. Blade.

6. Glass slides.

7. Cover slips.

8. Dropping bottle with phloroglucin solution.

9. Pipette.

10. 10 % HCl solution (Flask E).

11. Distilled water (Flask C).

Prepare a cross section of the object you are given. Stain this cross section with
phloroglucin and add several drops of HCl. Wash the preparation thoroughly with water
for 2-5 minutes and then cover it with a cover slip. Observe the preparation under the
microscope. Compare the cross section you have just prepared to the schemes
1-6 below and determine which scheme it corresponds to.

11

12

13

14
3.1. (8 points) Please label (using the numbers from the list below) the tissue elements pointed to
by arrows on the scheme corresponding to your cross section in the answer sheet.
1. Endodermis.

11. Periderm.

2. Phloem elements.

12. Sclerenchyma.

3. Phellogen (Cork cambium).

13. Pericycle.

4. Collenchyma.

14. Xylem elements.

5. Phelloderma.

15. Stoma.

6. Chloroplasts.

16. Chlorenchyma.

7. Epidermis.

17. Cambium.

8. Exodermis.

18. Medullary ray (Pith ray).

9. Core (Pith, Medulla).

19. Interfascicular cambium.

10.Aerenchyma.

20. Fibrovascular bundle.

3.2. (9 points) What elements (1-18) are coloured by phloroglucin in the presence
of HCl? Please, mark with + correct answer in the answer sheet.
1. Endoderm cells.

10. Root hair.

2.Elements of phloem.

11. Cells of phellogen (Cork cambium).

3. Cells of phellem (ork).

12. Sclerenchyma fibers.

4. Cells of collenchyma.

13. Pericycle cells.

5. Tracheids.

14. Xylem elements.

6. Vessel cells.

15. Rhizoids.

7. Epidermis.

16. Cells of parenchyma.

8. Trichomes.

17. Cambium cells.

9. Stomata guard cells.

18. Satellite cells.

3.2.
1

10 11 12 13 14 15 16 17 18

15
3.3. (1 point) What compounds are coloured by phloroglucinin the presence of HCl?
Write the corresponding number (1-6) from the list below into the answer sheet.

1. Cellulose.

4. Suberin.

2. Pectin.

5. Cutin.

3. Lignin.

6. Hemicellulose.
3.3.:_____________________________

3.4. (1 point) Determine which organ the cross section was made from. Write the
corresponding number (1-6) from the list below into the answer sheet.
1. Root.

4. Flower stalk.

2. Stem.

5. Runner.

3. Leaf stalk (Petiole).

6. Rhizome.
3.4.:_____________________________

3.5. (1 point) Determine which division of higher plants the plant you study belongs
to. Write the corresponding number (1-4) from the list below into the answer sheet.

1. Lycopodiophyta.

3. Polypodiophyta.

2. Equisetophyta.

4. Pinophyta.
5. Magnoliophyta.
3.5.:_____________________________

3.6. (1 point) Using the cross section you have just prepared, determine which ecological
group (relative to water availability) the plant belongs to. Write the corresponding number (1-4)
from the ecomorph list below into the answer sheet.
1. Hygrophyte.

3. Mesophyte.

2. Hydrophyte.

4. Xerophyte.
3.6.:_____________________________

14-TH INTERNATIONAL BIOLOGY OLYMPIAD


8-16 JULY / MINSK / BELARUS
PRACTICAL / LABORATORY 2:
ANIMAL MORPHOLOGY, ANATOMY AND SYSTEMATICS

CODE:

_____________________________________________________________________________________________

Dear Participants!

In the laboratory "ANIMAL MORPHOLOGY, ANATOMY AND


SYSTEMATICS" you will be given the following three tasks:
Task 1. Detaching pedes (extremities) of crayfish (Astacus) and determination
of their function.

Task 2. Test for knowledge of animal taxa.

Task 3. Determination of species name of freshwater gastropod molluscs.


molluscs.
The duration of the lab work is 60 minutes.
Maximum number of points 66.

You have to write down your results and answers into the ANSWER SHEET
which will be collected by an assistant when the time elapses. It is not necessary to
write anything in the task sheets.

Result lists taken away from the laboratory will not be accepted!

Please note that the results from the task 1 must be shown to the assistant
BEFORE the time limit!
Please do not forget to put zoological objects and instruments in their

original positions when finished, as these will be used by the next group.
Should the mollusc shells become damaged, you can ask for a replacement.

Good luck!

_____________________________________________________________________________

Country________________________
First name______________________ Family name ____________________
Code_____________________

3
Task 1. (36 points) Detaching pedes (extremities) of crayfish (stacus) and determination
of their function.

Material, instruments and equipment

1.

Astacus leptodactylus ().

2.

A set of instruments (2 forceps, scissors, scalpel, dissecting needles).

3.
4.

Dissecting tray
A magnifying glass.

1
1

5.

Cotton sheet.

6.

Latex gloves.

7.

Pins marked 1 to 18.

18

8.

Foam plate for pins.

The narrow-fingered crayfish (Astacus leptodactylus) is quite common in fresh water


bodies in temperate climates which are characterised by a relatively high content of dissolved
oxygen and mineral salts. A magnifying glass is sufficient to study the structure of pedes
(appendages) of crayfish.

You need to observe the details of animal's segmentation, to find its body parts and
sequentially detach the pedes (appendages excluding the first (antennuales or smallest) pair of
antennae) from one side of animal's body, assembling them in order on a plate with the help of
pins. Then it is necessary to determine the function of each ped and write it down in the answer
sheet.
Description of the techniques.
1. Take the animal in your hand, abdominal (ventral) side up. It is recommended to use a
cotton sheet and latex glove. Beware of small spicules on the carapace! Carefully study
the pedes of all body parts (with the help of a magnifying glass if necessary).

4
2. Using forceps sequentially detach all pedes from one side of animal's body. To do this,
hold the ped at its base with the forceps and pull away from the crayfish. You can also use
scissors and/or scalpel if necessary.
3. Assemble the pedes on pins with the corresponding numbers (1, 2, 3, etc.). Start
numbering from the head. Put the pedes on the foam plate in the correct order.

Attention! The practical results of task 1 must be registered by an assistant on a


special control sheet. The correctness of pedes preparation and numbering is scored. If a
ped is damaged in the process of preparation to such an extent that it cannot be recognized,
the points for this ped are not scored.

Please raise your hand when finished with the first task so that your work can be
checked. If the assistant is busy with another participant, you should continue with the next
task, but please note that the results of task 1 are not counted if they were shown to the
assistant after the total time limit (60 minutes).

In the answer list of task 1 each ped has 3 variants of its possible function. Study the table,
determine the function for each ped, then mark the correct function for each ped in the table with
a circle (). Note: a participant gets 1 point for every correct answer and loses
0.5 point for every wrong answer.

Pedes (extremities)

Functions
1.

sensory

respiratory

reproductive

2.

swimming

food grinding

respiratory

3.

transferring food to mouth

respiratory

reproductive

4.

reproductive

transferring food to mouth

sensory

5.

transferring food to mouth

walking

defence/attack

6.

defence/attack

transferring food to mouth

reproductive

7.

reproductive

swimming

respiratory

8.

swimming

capturing and holding food

reproductive

9.

reproductive

respiratory

defence/attack

10.

reproductive

walking

sensory

11.

reproductive

transferring food to mouth

walking

12.

walking

food grinding

sensory

13.

walking

reproductive

defence/attack

14.

walking

respiratory

reproductive

15.

defence/attack

swimming

walking

16.

swimming

food grinding

respiratory

17.

reproductive

sensory

swimming

18.

swimming

transferring food to mouth

respiratory

6
Task 2. (10 points) Animal taxonomy test.

Page 7 has pictures of ten animals numbered with roman numerals. The table below has
the names of animal phyla (AK), subphyla or classes (ak) and genera (110).

Phylum
.
B.
C.
D.
E.
F.
G.

Annelida.
Arthropoda.
Chordata.
Cnidaria.
Echinodermata.
Mollusca.
Nematoda
(Nemathelminthes)

H. Platyhelminthes.
J. Porifera.
K. Protozoa.

Subphylum/Class
.
b.
c.
d.
e.
f.
g.

Anthozoa.
Cephalopoda.
Chelicerata.
Crustacea.
Hydrozoa.
Insecta.
Polychaeta.

h. Scyphozoa.
j. Asteroidea
(Stellaroidea)
k. Trematoda.

Genus
1.
2.
3.
4.
5.
6.
7.

Araneus.
Asterias.
Corallium.
Cyclops.
Fasciola.
Hydra.
Locusta.

8. Musca.
9. Nereis.
10. Sepia.

Please label the taxonomic position of each animal using the information from the table
put the corresponding code for phylum, subphylum/class and genus next to animal picture in the
answer sheet.

II

III

V
IV

VII

VI

IX

VIII

8
Task 3. (20 points) Determination of species name of freshwater gastropod
molluscs.
Materials, instruments and equipment
1.

A tray with 10 shells of gastropod molluscs to be classified.

2.

An accessory tray for used shells.

3.

A ruler.

4.

A set of instruments (forceps, dissecting needles).

5.

A magnifying glass.

Many species of gastropod molluscs live in fresh water. They play an important role in
water ecosystems. Many are specific intermediate hosts of helminthes parasites of humans and
domestic animals. In this connection taxonomic identification of freshwater gastropod molluscs
has not only theoretical, but also applied value.
The tray has 10 numbered shells of gastropod molluscs. The classification key below allows the
identification of species names and includes illustrations explaining the details of shell structure
and measurements. Classify the molluscs you are given and place the numbers written
on their shells next to species names in the table in the answer sheet.

Species name

Viviparus contectus
Bithynia tentaculata
Physa fontinalis
Aplexa hypnorum
Radix ovata
Radix auricularia
Lymnaea stagnalis
Planorbarius corneus
Planorbis planorbis
Segmentina nitida

Shell number

CLASSIFICATION KEY
1a. Shell aperture (opening) has an operculum (lid)(2)
1b. Shell aperture without an operculum (lid)......(3)
2a. Shell is at least 20 mm high, green-brown, sometimes with three dark stripes on the
last turn of the whorl.......Viviparus contectus
2b. Shell is not more than 15 mm high, uniformly brown without
stripes..................................Bithynia tentaculata.
3a. Shell is like a tower or a cone with variable number of turns..(4)
3b. Shell is flat.(8)
4a. Shell is sinistral....(5)
4b. Shell is dextral..(6)
5a. Shell is egg-shaped. Whorl height is less then aperture height. Yellow- brown or light
brown...................................Physa fontinalis.
5b. Shell has spindle-like shape. Whorl height is twice the aperture height. Brown or
dark brown............................Aplexa hypnorum.
6a. Aperture height is significantly more than whorl height....(7)
6b. Whorl height is equal or slightly exceeds aperture height. Shell is up to 60 mm
high.................Lymnaea stagnalis.
7a. Aperture height is approximately twice its width. Shell height is up to 25 mm,
width up to 15 mm..Radix ovata
7b. Aperture height and width are approximately equal. Shell height is up to 40 mm,
width up to 30 mm.....Radix auricularia.
8a. Aperture has bud-like shape, its height exceeds its width .....Planorbarius corneus.
8b. Aperture has another shape, its width exceeds its height. (9)
9a. Diameter of the shell is over 8 mm, walls are thick, 56 turns, curved at top with
flat bottom. Shell walls are opaque, dark-brown .......................Planorbis planorbis.
9b. Diameter of the shell is less than 8 mm, walls are thin, 4-5 turns, curved at top
with plano-concave bottom. Shell walls are semi-transparent,
light-brown. ..Segmentina nitida.

10

dextral shell

sinistral shell

B
E

F
A

Shell measurements of gastropod molluscs:


B shell height,
CD shell width,
E aperture height,
DF aperture width,
BE whorl height.

Should the mollusc shells become damaged, you can ask for a replacement.
Please do not forget to put zoological objects and instruments in their original
positions when finished, as these will be used by the next group.

14-th INTERNATIONAL BIOLOGY OLYMPIAD


8-16 JULY / MINSK / BELARUS
PRACTICAL / LABORATORY 3:
MICROBIOLOGY AND BIOTECHNOLOGY

_____________________________________________________________________________________________

Dear Participants!
In the laboratory MICROBIOLOGY AND BIOTECHNOLOGY you will be given the
following two tasks:

Task 1. Identification of microorganisms.


Task 2. Study of Bacterial cultures expressing different genes.
Duration of the lab work is 60 minutes.
Maximum number of points 64 .
You MUST write down your results and answers on the ANSWER SHEET which will
be collected by an assistant when the time elapses. It is not necessary to write anything in the
task sheets.

Answer sheets taken away from the laboratory will not be accepted!
Please be careful when performing reactions and do not let the reagents and solutions
come into
contact with your skin and clothes!
PLEASE USE HAND DISINFECTION SOLUTIONS AFTER THIS PRACTICAL
EXAMINATION
GOOD LUCK!
_____________________________________________________________________________

Country________________________

First name______________________ Family name ____________________

Code_____________________

2
Task 1. (46 points) Identification of microorganisms.

Materials and equipment

1. Bacterial strains in:


- Three petri dishes with solid media (plate "GCO" 1, plate protease 1, plate
amylase 1);
- tubes with solid medium (for /F-test);
- tubes with broth (for 2S-test and NR-test).
2. Wooden toothpicks for transfer of bacterial biomass from solid medium onto glass slides.
3. Glass slides.
4. Pipettes.
5. solution, 3 %.
6. 22 solution, 3 %.
7. Dimethylparaphenilendiamine (DMPA) solution, 1 % .
8. Lugol's solution (Lugol).
9. Griess solution, 1% (Griess).

Identification of bacteria is based on the study of certain biological properties, mostly


morphological, physiological and biochemical characteristics. You have to identify five bacterial
strains labelled 1-5. For this you will have to perform five biochemical tests (1.1, 1.3, 1.4, 1.6
and 1.8). You will also use the results of the remaining tests given to you (tests 1.2, 1.5, and
1.7). Some tests are followed with additional questions on the corresponding topic that you have
to answer.
Please fill your results in the table "Identification of bacteria" in the answer sheet using the
following symbols: "+" for a positive reaction, "-" for lack of a reaction. A sample table is given
on the next page. Attention! In the column "Gram reaction" you have to put "+" for Grampositive bacteria and "-" for Gram-negative. In the column "O/F-test" put letter "F" for

3
organisms with anaerobic respiration (fermentative metabolism) and letter "O" for organisms
with aerobic respiraction (oxidative metabolism).

Fill all columns of the table except for the last one. Then identify your bacterium using
identification table in the end of the task sheet and put the letter corresponding to the identified
species into the column Result of identification.

nitrate reductase

H2S production

amylase

protease

oxidase

catalase

O\ F-test

Gram reaction

Strain

The presence of:

Result of identification

Identification of Bacteria (30 points)

1
2
3
4
5

PLEASE BE CAREFUL WHEN PERFORMING REACTIONS AND DO NOT LET THE


REAGENTS AND SOLUTIONS CONTACT YOUR SKIN AND CLOTHES!

PLEASE PUT USED PIPETTES, WOODEN TOOTHPICKS, GLASS SLIDES, FILTER


PAPER, ETC. INTO A SPECIAL CONTAINER ON YOUR BENCH!

4
Test 1.1. Gram reaction
To perform this test you need:
1. Biomass of bacterial strains 1-5 (from the GCO plate).
2. solution (3 % ).
3. Five glass slides.
4. Wooden toothpicks.

Attention! You will need the GCO Petri dish later to perform tests 1.3 and 1.4. Please
perform the tests in the suggested order: 1.1, 1.3, 1.4.

The method:
Using a dropping bottle, put a small drop of the 3 % solution onto a glass slide.
Using a toothpick, transfer some biomass (roughly 3-4 mm in diameter) of one strain to the KOH
drop, trying not to transfer the agar. Mix the bacterial mass with the KOH solution thoroughly. If
the mass sticks to the toothpick and moves behind it, the strain is Gram-negative, otherwise
Gram-positive. You may repeat the test if the results are not clear.
Using a new toothpick each time, repeat the test with the remaining strains. Put the results
in the corresponding column of the "Identification of bacteria" table in the answer sheet using
"+" for Gram-positive bacteria and "-" for Gram-negative.

Test 1.2. (O/F- test).


The O/F-test allows the determination of the ability of bacteria to utilise glucose in aerobic
(oxidative metabolism) or anaerobic (fermentative metabolism) conditions.
To determine the ability of your strains to utilise glucose aerobically and anaerobically,
each strain was inoculated in advance into two tubes with agar medium containing the required
mineral salts, glucose and a pH indicator ((water blue and rosolic acid) which is pink at neutral
pH, blue at acidic pH and red at basic pH). To create anaerobic conditions, medium in the tubes

5
labelled 1a 5a was covered with vaseline oil immediately after inoculation, while the tubes 1b
5b had no oil. The tubes were incubated in a thermostat for 24 hours.
Analyse the colour change in the tubes for each strain. Put the results in the column "O/Ftest" in the table "Identification of bacteria" in the answer sheet. Use letter "F" for organisms
with anaerobic respiration (fermentative metabolism) and letter "O" for organisms with aerobic
respiration (oxidative metabolism).

1.3. Catalase test.

To perform this test you need:


1. Biomass of bacterial strains 1-5 (on the GCO plate).
2. Hydrogen peroxide solution (3 % 22).
3. Five glass slides.
4. Wooden toothpicks.
5. Pipettes.
The method:
Using a pipette, put a drop of hydrogen peroxide solution onto a glass slide. Using a
toothpick, transfer some biomass of one strain from the GCO plate to the drop, trying not to
transfer the agar. Mix the bacterial mass with the hydrogen peroxide solution thoroughly. Record
the results while mixing the bacteria with the solution. Repeat the manipulation with the
remaining four strains. Put the results in the corresponding column of the "Identification of
bacteria table in the answer sheet(+ if positive, - if negative).
Question 1.3.1. (2 points) Which reaction(s) is catalysed by catalase?
. 322 +FADH23H2O + O2 +H2 +FAD
. 2H2O22H2O + O2
. H2O22HO.
D. H2O22HO2- + H2
. 2H2O2 + NADH + H+ 2H2O + NAD+
Put your answer code or codes into the line 1.3.1.

1.3.1. ___________________________

Test 1.4. Cytochrome oxidase test.


To perform this test you need:
1. A Petri dish (GCO), with colonies of strains 1-5.
1. 1 % solution of DMPA.

The method:
Using a dropping bottle, put a drop of DMPA onto each colony. 30-60 seconds later the
colonies of oxidase-positive strains turn pink to dark red. Analyse the colony colour of each
strain and fill the results in the corresponding column of the "Identification of bacteria" table in
the answer sheet.

Question 1.4.1. (4 points) Which of the following statements are true for cytochrome
oxidase positive bacteria?
. Capable of using 2 as terminal electron acceptor in the respiratory chain.
. All are unable to undertake anaerobic respiration.
. All are strict aerobes (obligate aerobes).
D. All are strict anaerobes (obligate anaerobes).
. All are facultative anaerobes.
F. Cytochrome oxidase takes part in chemosynthesis in some strains.

Put your answer code or codes into the line 1.4.1.

1.4.1. ___________________________

7
1.5. Proteolytic activity test.

For determination of proteolytic activity you must analyse a Petri dish with media
containing casein, inoculated in advance with strains 1-5. This plate is labelled "protease". Record
the results in the table in the answer sheet.

Test 1.6. Amylase test.

The plate labelled "amylase" contains rich solid medium supplemented with 0.2% of starch
and has been inoculated with strains 1-5 in advance. Cover the surface of this plate with
Lugol's solution (Lugol) and determine which bacteria have the amylolytic activity. Record the
reaction results into the corresponding column of the "Identification of bacteria" table in the
answer sheet.
1.7. Test for hydrogen sulphide generation (2S-test).
Here you must analyse five tubes prepared previously. The tubes contain meat broth that was
inoculated with test strains some time before. The tubes also contain pieces of white indicator paper
saturated with the solution of lead acetate. Record the results in the table in the answer sheet.
Record in the answer sheet the single letter code for the correct answer for each of the two
questions below:
Question 1.7.1. (4 points) When bacteria which are capable of producing 2S grow on
meat broth medium, 2S is generated from:
. RNA.

F. Glycine.

. DNA.

G. Thiamine.

. Arginine.

H. Biotin.

D. Methionine.

I. Taurine.

. Serine.

J. Cysteine.

Put your answer code or codes into the line 1.7.1.

1.7.1.:________________________________________

Question 1.7.2. (2 points) Which reaction(s) is/are responsible for the colour change of
the indicator paper?
. 2CH3COOH + H2S = (CH3CO)2S + 2H2O
. Pb2+ + S2- = PbS
C. (CH3COO)2Pb + H2S = 2CH3COOH + Pb +S
D. 2CH3COOH + H2S = CHSCOOH + 2H2
E. 2CH3COOH + Pb + 2H2S = 2C2H6 + PbSO4 + S
Write your answer code or codes down in the line 1.7.2. of the answer sheet
1.7.2.:________________________________________

1.8. Nitrate reductase test (NR-test).

For this reaction you need:


1.Tubes with suspensions of cells of strains 1-5 marked as NR.
2. Griess reagent, 1 % (Griess).
3.Pipettes.
Add 1 ml of the 1% Griess (Griess) reagent to the suspension of bacteria. The presence of
nitrate reductase activity results in the appearance of red colour within 1 minute. Record the results in
the table in the answer sheet.
Question 1.8.1. (4 points) The presence of nitrate reductase allows:
. The use of nitrate as an electron acceptor in the electron transport chain during
chemosynthesis.
. The use of nitrate as an electron donor in the electron transport chain during respiration.
. The use of nitrate as an electron donor in the electron transport chain during
chemosynthesis.
D. The use of nitrate as an electron acceptor in the electron transport chain during
respiration.
. The use of nitrites as nitrogen source.

9
Write your answer code or codes down in the line 1.8.1. of the answer sheet.
1.8.1.:________________________________________

Use your results and the identification table to identify the species of your strains. Fill the
results in the table in the answer list.
Identification table

A
B
C
D
E
F
G
H
I
J
K
L
M
N

Escherichia coli
Xanthomonas
campestris
Lactobacillus
delbrueckii
Erwinia herbicola
Clavibacter
michiganensis
Staphylococcus
saprophyticus
Pseudomonas
mendocina
Pseudomonas
putida
Sarcina
lutea
Streptobacillus
moniliformes
Agrobacterium
tumefaciens
Pseudomonas
fluorescens
Bacillus subtilis
Streptococcus
lactis

nitrate reductase

H2S production

amylase

protease

oxidase

O\ F-test

catalase

The presence of:


Gram reaction

Genus, species

F
O

+
+

+
+

+
+

F
O

+
+

+
+

+
+

F
F

+
+

+
+

10
Task 2. (18 points) Study of Bacterial cultures expressing different genes.

Materials and equipment


1. Six tubes with cells taken from cultures at different stages of growth.

2. Distilled water (flask ).

3. Dropping bottle with 0.5 catechol solution (flask ).

4. Pipette.

The xylE gene coding the enzyme catechol-2,3-dioxygenase is often used as a


reporter to study the expression of various genes. This enzyme catalyses the conversion of
colourless catechol into a yellow coloured product called - hydroxymuconic semialdehyde.
Fusing the promoterless xylE sequence to the promoter of gene of interest allows the expression
of this gene to be analyzed according to the appearance and intensity of the yellow colour of
reaction products.

Two strains of Escherichia coli have been constructed experimentally in which the xylE
gene was fused to promoters of two different genes, gene C and gene D. Figure 1 shows
growth curves for these bacteria, labelled I and II (I - E. coli with xylE fused to gene promoter,
II - E. coli with xylE fused to gene D promoter). The arrows in Figure 1 show when the cell
samples were taken from the cultures. The number on the tube corresponds to the number of the
arrow in Figure 1.

Determine the phases of culture growth in which genes C and D are expressed.

11
Figure 1

OD600
2

Cell Density at 600nm

1.6

II
1.2

0.8

0.4

4
0
2

10

12

26

Time, hours

To do this you need to perform the following actions:


1) Fill the pipette to the mark using water (from flask A). Pipette this volume to each tube.
2) using the dropping bottle (flask B), add one drop of catechol solution to each tube and mix
the contents of the tube by shaking,
3) leave the tubes at room temperature for 3 to 5 minutes,
4) examine the appearance of yellow colour in each tube.

Determine in which growth phases genes C and D are expressed and fill the table in the
answer sheet, putting the "+" sign in the corresponding column.

The gene is expressed in


Strain

Gene
early log phase

II

late log phase

stationary phase

14-th INTERNATIONAL BIOLOGY OLYMPIAD


8-16 JULY / MINSK / BELARUS
PRACTICAL / LABORATORY 4
GENETICS
____________________________________________________________________________________________________________________

Dear Participants!

In the laboratory "GENETICS" you will be given the following two tasks:

Task 1. Genetic analysis of inheritance of seed coat colour in Phaseolus vulgaris


L.

Task 2. Identification of the trp mutations in the yeast Saccharomyces


cerevisiae.

Duration of the lab work is 60 minutes.


Maximum number of points 61.

You have to write down your results and answers on the ANSWER SHEET
which will be collected by an assistant when the time elapses. It is not necessary to
write anything on the task sheets.

Good luck!

__________________________________________________________________

Country________________________
First name______________________ Family name ____________________
Code_____________________

Task 1. (30.5 points) Genetic analysis of inheritance of seed coat colour in


Phaseolus vulgaris L.

Time for carrying out this task must not exceed 25 minutes
Materials and equipment
1.

Parental sample seeds (1).

sample 1

2.

Parental sample seeds (2).

sample 2

3.

Hybrid seeds (F1).

sample 3

4.

Test cross line seeds (L).

sample 4

5.

Seeds of F generation.

sample 5

6.

Petri dishes for seeds.

7.

Sheet of white paper.

The seed-coat colour of common beans (Phaseolus vulgaris L.) is controlled by a number
of genes, which are responsible for the synthesis of pigments and distribution of the seed coat
colour, as well as modifying genes, that can enhance, attenuate or change colour in another way.
In the preliminary experiments breeding of two types of common beans (1 and 2)
differing in seed-coat colour was conducted. Seeds of F1 plants were cultured. Plants (F1)
gave seeds of F1 phenotype.

On the next stage of the experiment test-crossing of F1 plants with testcross line plants
(L) was conducted. Grown hybrids (Fa) gave seeds of Fa phenotype. For the next analysis, one
seed from each Fa plant was taken.

3
Scheme of the experiment.

CROSSING SCHEME

P1

P2

(sample 1)

(sample 2)

F1

(sample 3)

L
(sample 4)

F
(sample 5)

Stages of the work:


You are given parental sample seeds P1 (sample 1) and P2 (sample 2), hybrid seeds F1
(sample 3), testcross line seeds L (sample 4) and seeds of F generation (sample 5).
Differences between parental samples are determined by different combinations of two
pairs of non-allelic genes A and B (different gene loci). Gene A controls synthesis of pigment
(A = dominant allele -pigment is present, a = recessive gene -pigment is absent). Gene B is a
modifying gene, that influences colour intensity (B = dominant allele modification is present,
and b = recessive allele modification is absent). Different combinations of two pairs of nonallelic genes A and B cause the development of three types of seed-coat colour (Table 1).

4
Table 1
Kind of seeds

Seed-coat colour

Code of the colour

White
w
Yellow-brown
y

Black
b

You should accomplish the next problems:

Determine if parental samples P1 and P2 are pure-breeding lines (homozygous at each gene
locus).

Determine the type of inheritance of seed-coat colour in common beans (presence of


interaction of non-allelic genes A and B ).

Determine the genotypes of the parental forms of P1 and P2, hybrid seeds F1, seeds of F
generation and testcross line seeds L

Determine if the investigated non-allelic genes are linked.


Attention! The differences in viability of zygotes or gametes of different types of
analyzed common bean (Phaseolus vulgaris L.) samples were not detected. Genes A and B
are localized in the nucleus.
Problem 1.1. Determine if the parental samples P1 and P2 are pure-breeding lines
(homozygous by every pair of non-allelic genes) by seed coat colour ? To answer this
question you must analyze F1 seeds.

5
1.1.1. (1.5 points) Look over samples 1 and 2. Specify the seed phenotypes of
parental forms and F1 using the symbols from Table 1 (Page 4). Fill in the table in the answer
sheet:

Plant
seeds

Sample

F1

Seed phenotype

1.1.2. (2 points) Analyse the seed-coat phenotypes of parental samples and F1


hybrids. Select the correct answer. On the answer sheet record in the symbols of correct answers:
. Both parental plants are homozygous.
. Both parental plants are heterozygous.
. Plant 1 is homozygous, plant 2 is heterozygous.
D. Plant 2 is homozygous, plant 1 is heterozygous.
E. Using the data presented it is impossible to determine, if the parental genotypes are
pure-breeding lines.

1.1.2.:__________________________________________
Problem 1.2. Determine the type of inheritance of seed-coat colour in common beans.
You need to analyze the seeds of Fa plants, which were received after breeding of F1 plants
with La plants.
1.2.1. (1 point) Carefully place the seeds from sample 5 (Fa plant seeds) on to the
sheet of white paper. Identify the quantity of the phenotypic classes of Fa by seed-coat colour.
Group the seeds of Fa by phenotypic classes by putting them into Petri dishes for seeds. Using
the codes from Table 1 specify the phenotypes of Fa. Record in the table in the answer sheet.

6
of class

Seed phenotype

Total number of classes

1.2.2. (3 points) Using your findings about the quantity of Fa classes, choose the type
of interaction of non-allelic genes A and B, which control seed-coat colour in
common beans. Record the symbols of correct answers on the answer sheet.
. There is no interaction of non-allelic genes in the experiment conducted.
B. Incomplete dominance.
C. Duplicate genes
D. Epistasis
E. Codominance.
F. Pleiotropic gene action.

1.2.2:________________________________________
Problem 1.3. Determine the genotypes of the parental samples P 1 and P2, hybrid seeds
F1, seeds of F generation and testcross line seeds (L)
1.3.1. (4 points) Specify all of the possible genotypes of P 1, P2, F1, Fa, and La plants
using symbols A and B to mark the dominant alleles, symbols a and b to mark the
recessive alleles of the investigated genes in the boxes of the table below. Fill in
the table in the answer sheet.

7
Seed phenotype
Plants
Black

Yellow-brown

White

1
2
F1
L
F
Problem 4. Determine if the investigated non-allelic genes A and B are linked.

1.4.1. (1 point) Determine frequency of phenotypic classes in Fa by seed colour.


To answer this question calculate the number of seeds in each class. Use the codes from Table 1.
Fill in the table in the answer sheet.

of class

Seed phenotype

Number
of seeds

Total number of seeds

1.4.2. (3 points) Determine the ratio of the different phenotype classes by the colour
of the seeds in Fa. Fill in the answer sheet using the code of the correct answer:

Code

White

Yellow-brown

Black

A.

0.50

0.25

0.25

B.

0.50

0.19

0.31

C.

0.56

0.16

0.28

D.

0.42

0.14

0.44

E.

0.44

0.15

0.41

F.

0.50

0.14

0.36

1.4.2.:__________________________________________

1.4.3. (3 points) To determine whether there is linkage between the genes being
investigated you must specify the expected ratio in Fa in the case of no linkage. You
an receive the points for this task only if your answer for 1.2.2. is correct. Record in
the table in the answer sheet:

Phenotypic class

Ratio (%)

White seeds

Yellow-brown seeds

Black seeds

1.4.4. (3 points) Specify the expected ratio by seed colour in Fa if the


investigated genes A and B are linked completely. You can receive the points for this task only if
your answer for 1.2.2. is correct. Record in the table in the answer sheet:

Phenotypic class

Ratio (%)

White seeds

Yellow-brown seeds

Black seeds

1.4.5. (3 points) Using 2 method, determine whether to reject or not-reject (accept)


your hypothesis
Calculate the 2 value for H0 (null hypothesis)being No linkage using the formula below:
2 = ((i i)2 / i),
where i is the expected frequency of the phenotype class i. i is the
practically observed frequency of the same class. Use two decimal places during your
calculations. Record in the answer sheet by the 2 value (with two decimal places).

1.4.5.________________________________
1.4.6. (3 points) Use the table of 2 distribution to determine what is the maximum
probability (p) of your H0 (null hypothesis)not being rejected (being accepted). Write the codes
of the answers on your answer sheet.

10
Table of 2 distribution
Value (p) of a significance level 2
df
0.99

0.95

0.90

0.75

0.50

0.25

0.10

0.05

0.025

0.01

0.02

0.10

0.45

1.32

2.71

3.84

5.02

6.63

0.02

0.10

0.21

0.58

1.39

2.77

4.61

5.99

7.38

9.21

0.11

0.35

0.58

1.21

2.37

4.11

6.25

7.81

9.35

11.34

0.30

0.71

1.06

1.92

3.36

5.39

7.78

9.49

11.14 13.28

0.55

1.15

1.61

2.67

4.35

6.63

9.24

11.07 12.83 15.09

0.87

1.64

2.20

3.45

5.35

7.84

10.64 12.59 14.45 16.81

1.24

2.17

2.83

4.25

6.35

9.04

12.02 14.07 16.01 18.48

1.65

2.73

3.49

5.07

7.34

10.22 13.36 15.51 17.53 20.09

2.09

3.33

4.17

5.90

8.34

11.39 14.68 16.92 19.02 21.67

10

2.56

3.94

4.87

6.74

9.34

12.55 15.99 18.31 20.48 23.21

A.
B.
C.
D.
E.
F.

< 0.01
> 0.01
< 0.05
> 0.05
0.01
0.05

1.4.6._______________________________
1.4.7. (3 points) Using your value of p, determine if genes A and B are linked
. Calculate the distance between genes A and B (in cM) if they linked. Record in the
answer sheet the code of correct answer.
. There is complete linkage between genes A and B. The distance between the genes
is 6.94 .
. There is complete linkage between genes A and B. The distance between the genes
is 12.36 .
. There is complete linkage between genes A and B. The distance between the genes
is 27.78 .
D. There is incomplete linkage between genes A and B. The distance between the genes
is 6.94 .
. There is incomplete linkage between genes A and B. The distance between the genes
is 12.36 .

11

F. There is incomplete linkage between genes A and B. The distance between the genes
is 27.78 .
G. Genes A and B are not linked. The distance between the genes is 6.94 .
H. Genes A and B are not linked. The distance between the genes is 12.,36 .
I. Genes A and B are not linked. The distance between the genes is 27.78
J. Genes A and B are not linked
1.4.7:__________________________________________

12
Task 2: (30.5 points) Identification of trp mutations in yeast Saccharomyces cerevisiae

Materials and equipment


1. Tubes with culture liquid.

12

2. A plate with 12 wells.

3. A tube with Erlich reagent.

4. A tube with indole solution.

5. A tube with anthranilate solution.

6. A tube with water.

7. 1 ml pipette.

13

8. A sheet of white paper.

9. A container for used pipettes.

10. Paper towels.

You are given the yeast Saccharomyces cerevisiae as an experimental organism. The
scheme of life cycle of this organism is presented below.

n
Meiosis

2n
n

n
n

These yeasts have alternating haploid and diploid phases during their life cycle. The fusion
of haploid cells gives rise to a diploid cell which through meiosis can produce four haploid cells
with different genotypes.

13
The scheme below shows the pathway of tryptophan biosynthesis in the yeast
Saccharomyces cerevisiae. The scheme shows some intermediate products and genes responsible
for the synthesis of enzymes of this pathway.

chorismate

anthranilate
trp2 gene

indole
trp4 gene

tryptophan
trp5 gene

Mutations in the trp genes lead to the accumulation of the intermediates in the culture
liquid. Two intermediates of this biosynthetic pathway, anthranilate and indole, can be detected
in the culture liquid of the corresponding mutants through colour reactions with the Erlich
reagent.

2.1. (1.5 points) Using a special pipette, add 0.5 ml of Erlich reagent to the control
tubes with standard solutions of anthranilate, indole and to the tube with water (with no
anthranilate and indole). Observe the colour change and record it in the table in the answer sheet
using single letter colour code.
Compound

Colour after Erlich reagent addition

Water
Anthranilate
Indole
Colour code:

yellow

R red

N no colour change

14
2.2. (1.5 points) Which compounds will accumulate in the culture liquid if the
mutants are grown in the rich medium? Fill in the table below in the answer sheet using one
letter code.

Mutant

Accumulated intermediate

tr 2
trp 4
trp 5
A anthranilate
Code:
I indole

O neither anthranilate nor indole

2.3. (6 points) Three classes of double mutants have been constructed in haploid S.
cerevisiae named as trpX trpY trpZ +; trpX trpY + trpZ ; trpX + trpY trpZ (sign
denotes mutant genes, sign + denotes wild type genes; all trp genes are located on different
chromosomes).

Three matings between these mutants have been performed as shown in the table below.
Each mating has generated all possible types of haploid progeny.
Please write down in the answer sheet the genotypes of all possible progeny from each
cross.

15

ating

trpX trpY trpZ +

trpX trpY + trpZ

II

trpX trpY trpZ +

+
trpX trpY trpZ

III

trpX trpY + trpZ

trpX + trpY trpZ

Possible progeny genotypes

2.4. (12 points) Clones produced by these matings have then been grown in liquid
medium, cells removed by centrifugation and supernatant collected for analysis. You now
need to identify these clones.
Please test each of the 12 culture liquid samples for the presence of the tryptophan
metabolic intermediates and use these data for the identification of the trpX , trpY and trpZ
mutations. You are given tubes with supernatants from 12 cultures of S. cerevisiae. The tubes are
labelled according to the mating (I, II and III) and clone number (1-4).
To test the accumulation of particular compounds, transfer 1 ml of liquid from each tube to
the wells of the 12-well plate. Use a new pipette for each transfer!
Add 0.5 ml of the Erlich reagent (using a special pipette) to each well containing the 1 ml
of supernatant. Record the colour changes (using a single letter code) in the table in the
answer sheet.
Determine which compound has accumulated in each culture and record this in the same
table in the answer sheet using a single letter code.

16

Tube

Mating

Colour after Erlich


reagent addition

Accumulated
intermediate

I.1
I

trpX trpY trpZ +

trpX trpY + trpZ

I.2
I.3
I.4
II.1

II

trpX trpY trpZ

trpX + trpY trpZ

II.2
II.3
II.4
III.1

III

trpX trpY + trpZ

+
trpX trpY trpZ

III.2
III.3
III.4

Code:

yellow

A anthranilate

R red

I indole

N no colour change

O neither anthranilate
nor indole

2.5. (3 points) Identify the trpX , trpY and trpZ mutations. Write down names of
the genes in which the trpX , trpY and trpZ mutations are located in the table in the answer
sheet.

Gene
trp 2
trp 4
trp 5

Mutation

17
2.6. (3 points) How would the experimental results change if the
trpX and trpY genes were completely linked? Record in the answer sheet
the letter corresponding to the correct answer:
. The number of different progeny genotypes would be reduced.
. The results would not be changed.
. Phenotypically wild type yeast may be produced.
D. The number of single and triple mutants would increase.

2.6.:__________________________________________
2.7. (1.5 points) How many genotype classes would be obtained if the three genes
were located on the same chromosome and were 100 per cent linked? Write the number for each
mating in the answer sheet.

2.7.: I________________________________________
II_________________________________________
III_________________________________________
2.8. (0.5 points) Which mating will give the single mutant accumulating
anthranilate? Write the mating number (I, II or III) in the answer sheet.

2.8.:__________________________________________
2.9. (0.5 point) Write the genotype of this mutant in the answer sheet using the
actual gene names (trp 2, trp 4 or trp 5).

2.9.:__________________________________________
2.10. (1 point) Which of the double mutants has to be mated with this anthranilateaccumulating single mutant to get progeny with wild type genotype? Write the genotype of this
double mutant in the answer sheet using the actual gene names (trp 2, trp 4 or trp 5).
2.10.:__________________________________________

INTERNATIONAL BIOLOGY OLYMPIAD


THEORY PROBLEMS

2002, Jurmala-Riga, Latvia















All IBO examination questions are published under the following Creative Commons license:



CC BY-NC-SA (Attribution-NonCommercial-ShareAlike) https://creativecommons.org/licenses/by-nc-sa/4.0/
The exam papers can be used freely for educational purposes as long as IBO is credited and
new creations are licensed under identical terms. No commercial use is allowed.

13th International Biology Olympiad. Theoretical Test, Part A

Theoretical test

YOU HAVE FOUR HOURS TO ANSWER ALL THE QUESTIONS


Questions in part A have only one correct answer which should be shown by marking
(blacking out) the appropriate field in the answer list.

You ll get one point for every correct answer in part A


For questions in part B youll have to fill in the correct answers in the appropriate fields,
build graphs, etc
The number of points youll get for the part B questions varies, depending on the
complexity of the question

GOOD LUCK !

13th International Biology Olympiad. Theoretical Test, Part A

Part A
Find the one correct answer in each task and mark it in the answer sheet in this way:
The jury will check only the answer sheet!

Cell Biology
A1. In which processes microtubules could be involved?
Beating of cilia

Movements of

Osmoregulation Movement of organelles

and flagella

chromatids

A.

B.

C.

D.

E.

in living cells

A2. Which of the following is an example of microfilament motion in nonmuscular animal


cells?
A. Rigor mortis
B. Flagellar movement
C. Cytokinesis
D. Chromosome movement during meiosis
E.

Beating of cilia

A3. When or where it is possible to observe nucleolus?


A. During meiosis
B. In senescent plant cells
C. During apoptosis
D. In senescent animal cells
2

13th International Biology Olympiad. Theoretical Test, Part A

E. During the elongation of plant cells


A5. Which statement about actin microfilaments is correct?
A. They are found only in animal muscle cells and involved in the contractions of
sacromeres
B.

They are involved in the formation of the cell


cleavage furrow in plant and animal cells

C.

They are found only in plant cells and involved in the movement of chloroplasts

D.

They are found in plant and animal cells and involved in the movement of vesicles

E.

They are found in all eukaryotic cells and involved in the movement of pyruvate
from cytosol to mitochondrial matrix

A6. Which statement about genetic material is incorrect?


A.

There are viruses possessing genomes, built of RNA

B.

There are cell organelles possessing their own RNA genomes

C.

In the cells of bacteria genetic material may persist in extrachromosomal form

D.

Genetic material of eukaryotes is made of DNA

E. Admission of foreign DNA in a cell is not necessarily lethal for the cell, especially in the
case of
eukaryotic cell

13th International Biology Olympiad. Theoretical Test, Part A

A7. Which of the following is not the metabolic role of tricarboxylic acid (TCA) cycle?
A. Completion of carbohydrates oxidation
B. Supply of metabolic precursors for biosynthesis of some amino acids
C. Supply NADH for the respiratory chain
D. Supply NADPH for biosynthetic reactions
E.

Production ATP or GTP

A8. Which statement about the chemiosmotic theory is not correct?


A.

While electrons in the electron transport chain are passing through the transporters
located on the inner membrane of mitochondria, protons are pumped out of the matrix
by the respiratory complexes I, III and IV

B.

This theory explains coupling between oxidation and phosphorylation

C.

The protons return to the mitochondrial matrix through the proton-dependent ATP
synthase

D.

This theory is valid for forming of ATP in photosynthetic electron transport chains

E.

The respiratory proton transport is driven by conformational oscillations of the


energy-coupling membrane bilayer

A9. Which three amino acids can be formed directly in one step from the following metabolic
intermediates: pyruvate, oxaloacetate and -ketoglutarate ?
Pyruvate

Oxaloacetate

-ketoglutarate

A.

Alanine

Aspartate

Glutamate

B.

Lysine

Asparagine

Glutamine

C.

Serine

Arginine

Tyrosine

D.

Threonine

Glycine

Tryptophan

E.

Histidine

Proline

Leucine

13th International Biology Olympiad. Theoretical Test, Part A

A 10. How many different primary structures approximately may represent a 10 residues long
polypeptide, which is a random combination of 20 naturally occurring amino acids?
A. 10
B. 200
C. 40 00
D. 10 000 000 000 000
E. 100 000 000 000 000 000 000

A 11. Apoptotic cell undergoes a series of changes including membrane blebbing,


fragmentation of DNA creating a vacuolar nucleus and following fragmentation of nucleus
forming micronuclei. Researchers used onion cells to study the cell death. Which picture
corresponds to cell with nuclear fragmentation?

13th International Biology Olympiad. Theoretical Test, Part A

A 12. What is the correct sequence of events during immunological responses to viral
infection?
Code
1. Natural killer cell activation
2.

Antibody production

3. Cytotoxic T cell activation


Virus invasion

Time
A.

B.

C.

D.

E.

A 13. The diagram shows a simplified scheme of three


stages (P, Q and R) of aerobic respiration.
What substances are X, Y, Z?
X

A.

Acetyl CoA

NADH2

Lactic acid

B.

Water

CO2

Dehydrogenase

C.

NADH2

CO2

Dehydrogenase

D.

NADH2

Water

Peroxidase

13th International Biology Olympiad. Theoretical Test, Part A

E.

Pyruvate

CO2

NADH2

A 14. Some genes in the genome of bacteria are organised in operons. Which statement about
such operons is correct?
A.

Genes of the operon are arranged in mosaic structures of introns and exons

B.

Translation of all genes of one operon starts at the same initiation codon

C.

All genes of the same operon are not expressed simultaneously

D.

Proteins encoded in the genes of the same operon are translated from one common
mRNA molecule

E.

Translation of all genes from the same operon is terminated at the same common
STOP codon

A 15. Which of the given components is not needed for DNA replication in vivo?
A.

Single stranded DNA template

B.

Deoxy-nucleoside monophosphates (dAMP, dCMP, dGMP, dTMP)

C.

RNA polymerase primase

D.

Single-strand DNA binding proteins

E.

DNA polymerase

A 16. For numerous groups of organisms genes are split in exons and introns. Which
statement about gene expression is correct?
A. The genetic information of only some introns is used for synthesis of proteins
B. A separate promoter induces the transcription of each exon
C. During RNA processing the sequences of introns are removed as a result of splicing
of the pre-mRNA
D. Translation of each exon starts with its own initiation codon (AUG)
E.

During the translation ribosomes are jumping over the intronic part of mRNA
7

13th International Biology Olympiad. Theoretical Test, Part A

13th International Biology Olympiad. Theoretical Test, Part A

A 17. Human hormone insulin is synthesised as pre-protein and modified before secretion in
extracellular space. It contains two polypeptide chains. Which statement about these chains is
correct?
A.

They are synthesised on cytosolic ribosomes and modified in Golgi apparatus

B.

They are synthesised on ER ribosomes and modified in Golgi apparatus

C.

One chain is synthesised on cytosolic ribosomes, another on ER ribosomes and


modified in cytosol and Golgi apparatus

D.

They are synthesised on cytosolic ribosomes and modified in the lumen of lysosomes

E.

They are synthesised on cytosolic ribosomes and modified in cytosol

A 18. Which statement regarding the amount of genomic DNA per cell (M) during the cell
cycle is correct?
A.

MDNA[G1] = MDNA[meiosis prophase II]

B.

MDNA[meiosis prophase II] = 2 x MDNA[meiosis prophase I]

C.

MDNA[G1] = MDNA[G2]

D.

MDNA[G2 after

E.

MDNA[telophase of mitosis] > MDNA[telophase I of

mitosis]

< MDNA[G2 after meiosis]


meiosis]

13th International Biology Olympiad. Theoretical Test, Part A

Plant Anatomy and Physiology


A 19. Which of the following is closer to the centre of a woody stem?
A. Vascular cambium
B. Primary phloem
C. Secondary phloem
D. Secondary xylem
E. Primary xylem

A 20. Carnivorous plants trap insects. What do they obtain from the insects? What do they
primarily use this substance for?
A. They obtain water, because they live in a dry environment
B. They obtain nitrogen to make sugar
C. They obtain phosphorus to make protein
D. They obtain sugars, because they cant produce enough in photosynthesis
E. They obtain nitrogen to make protein

A 21. The diameter of woody stems is continually increasing. What structure ensures that
there is always enough bark to cover the outside of the stem?
A. Vascular cambium
B. Epidermis
C. Phellogen
D. Endodermis
E. Pericycle

10

13th International Biology Olympiad. Theoretical Test, Part A

A 22. The drawing shows s cross - section of a plant root. The lines (1-6) indicate parts and
the arrow (7) indicates a pathway in the root. Which of the statements (A-E) provides a
correct explanation of the drawing?

A. 1 trichome, 2 cortex, 3 phloem, 4 pericycle, 5 endodermis, 6 epidermis,


7 pathway of water and sugars
B. 1 - root hair, 2 cortex, 3 xylem, 4 endodermis, 5 Casparian strip, 6 - epidermis,
7 pathway of water and minerals
C. 1 - root hair, 2 cortex, 3 xylem, 4 Casparian strip, 5 -pericycle, 6 epidermis,
7 pathway of water and minerals
D. 1 root hair, 2 periderm, 3 phloem, 4 endodermis, 5 Casparian strip,
6 epidermis, 7 pathway of phytohormones
E . 1 root hair, 2 endodermis, 3 xylem, 4 epidermis, 5 Casparian strip, 6 - periderm,
7 pathway of water and minerals

A 23. Which cell is incorrectly paired with its tissue?


A. Root hair dermal tissue
B. Palisade parenchyma ground tissue
C. Guard cell dermal tissue
D. Companion cell excretory tissue
E. Tracheid vascular tissue

11

13th International Biology Olympiad. Theoretical Test, Part A

A 25. A plant biochemist received a specimen from a fellow scientist who noticed that the
plants stomates are closed during the day. The biochemist observed that radioactive carbon in
the form of carbon dioxide, fed to the plant at night, was first found in organic acids that
accumulate in the vacuole. During the day it moved to sugars being manufactured in the
chloroplast. What was the conclusion of the biochemist?
A. The plant fixes carbon by crassulacean acid metabolism (CAM)
B. The plant is a C4 plant
C. The plant is a C3 plant
D. The plant is using mitochondria as chloroplasts
E. The carbon fixation reactions occur in different cells
A 26. Red algae grow at depths beyond those to which red and blue light can penetrate in the
ocean. What could account for this?
A. Red algae have accessory pigments that absorb wavelengths of light available at these
depths
B. Red algae use infrared energy to power photosynthesis
C. Red algae have a more efficient light-absorbing system for red and blue light
D. Red algae are heterotroph organisms
E. The red algae must be identified incorrectly

A. 27. Sections are cut from a willow branch and planted in pots of soil in a greenhouse with
the shoot end of the section exposed and the root end in the soil. Roots sprout from the root
end and shoots sprout from the shoot end. Which statement about the sections is true?
A. The sections lacks the property of polarity
B. The concentration of auxin in the sections is the same in all their length
C. The root end will produce shoots
D. Dedifferentiation will be the first step in the process of root and shoot formation
12

13th International Biology Olympiad. Theoretical Test, Part A

E. The root end has special structures forming roots which the shoot ends lack

A 28. Plants have developed many adaptations to maximize the benefits of available water.
Which of the following is one of these adaptations?
A. Reorientation of leaves in order to increase leaf temperature
B. Decreasing the amount of water lost for each gram of fixed carbon
C. Increasing the leaf surface area
D. Decreasing the thickness of the cuticle
E. Growing more leaves during drought
A 29. You need pears for a large party after three days but they are not ripe enough to use.
What is the best way to hasten the ripening process?
A. To place the pears in the dark
B. To place the pears in a refrigerator
C. To place the pears on the windowsill
D. To place the pears in brown paper bags together with ripe apples

13

13th International Biology Olympiad. Theoretical Test, Part A

Animal Anatomy and Physiology


A 30. Which statements concerning human respiratory muscles are true?
1. During inspiration, the external intercostal muscles contract and the diaphragm moves
downwards
2. Internal and external intercostal muscles act in inspiration, and the diaphragm acts only in
expiration
3. During inspiration, only the internal intercostal muscles contract and the diaphragm moves
downwards
4. During expiration external intercostal muscles contract and the diaphragm moves
downwards
5. During gentle expiration, the thorax passively contracts, and then deep expiration can be
finished by contraction of the internal intercostal muscles
6. During inspiration, the internal intercostal muscles contracts and then strong inspiration
can be finished when the diaphragm moves upwards
A. 2 and 4
B. 1 and 5
C. 4 and 6
D. 3 and 5
E. 2 is the only correct answer

A 31. Which of the following are characteristics for animals with an open circulatory system?
A. Haemoglobin, haemocoel, lymph
B. Haemocyanin, haemocoel, haemolymph
C. Haemoglobin, absence of haemocoel, haemolymph
D. Haemocyanin, absence of haemocoel, lymph
E. Haemocyanin, haemocoel, lymph

14

13th International Biology Olympiad. Theoretical Test, Part A

A 32. Which metabolic changes in the cytoplasm of skeletal muscle cells are characteristic of
skeletal muscle fatigue?
1. Increase of creatine phosphate concentration
2. Decrease in the amount of glycogen
3. Increase of H+ - ion concentration
4. Increase of ATP concentration
5. Decrease in lactate concentration
A. 1 and 2
B. 1 and 4
C. 2 and 3
D. 4 and 5
E. 3 and 4

A 33. Which of the following is characteristic for a physically trained person in comparison
with an untrained person?
A. The heart rate can reach a higher level
B. Stroke volume is greater
C. The activity of vagus nerve (nervus vagus) is lower
D. Mechanical resistance of blood vessels is higher
E. Left ventricular and diastolic volume is smaller

15

13th International Biology Olympiad. Theoretical Test, Part A

A 34. Which statements regarding the differences of a compound eye compared with a
vertebrates eye is not correct?
1. Has chromatic aberration
2. The absorption of ultra-violet radiation is lesser
3. Acuity of vision (visus) is lesser
4. The ability to detect movement is lesser
5. The visual field is wider
A. 1 and 5
B. 2 and 3
C. 1 and 4
D. 4 and 5
E. 2 and 5

A 35. Which statement concerning a laboratory animal (white mouse) that lacks a thymus
gland congenitally is true?
A. Cellular immunity does not develop and the antibody synthesis is impaired
B. Only humoral immunity does not develop
C. Only cellular immunity does not develop
D. The immune system is not affected
E. This animal is resistant to viral infections but sensitive to bacterial infections

16

13th International Biology Olympiad. Theoretical Test, Part A

A 36. The numbers in the first column correspond to human, elephant, bat, mouse and carp.
Which number indicates each organism?

Body temperature

Heart rate

Maximal speed of

(C)

(beats/min)

locomotion

Number

(m/s)
1

1-30

30-40

1.5

38

450-550

3.5

31

500-660

14

36.2

22-28

11

36.6

60-90

10

A.

Human

Elephant

Bat

Mouse

Carp

B.

Mouse

Bat

Elephant

Human

Carp

C.

Carp

Mouse

Bat

Elephant

Human

D.

Carp

Mouse

Elephant

Bat

Human

E.

Bat

Mouse

Carp

Human

Elephant

17

13th International Biology Olympiad. Theoretical Test, Part A

18

13th International Biology Olympiad. Theoretical Test, Part A

A 38. Which figure shows the correct blood flow direction in a human?
A.

B.

C.

D.

E.

19

13th International Biology Olympiad. Theoretical Test, Part A

A 39. Which of the figures shows the correct relations between basal metabolic rate per m2
body surface area and age (in years) of human male and females?
A

20

13th International Biology Olympiad. Theoretical Test, Part A

A 40. The Figure shows cutaneous respiration among different vertebrates: excretion of
carbon dioxide (solid bars) and uptake of oxygen (open bars). Which of the version about
cutaneous respiration is true?

Bullfrog (larva)

Bullfrog (adult)

Lungless salamander

Human

Rana catesbeiana

Rana catesbeiana

Ensatina eschscholtzii

Homo sapiens

A.

I
Bullfrog (larva)

II
Bullfrog (adult)

III
Lungless salamander

IV
Human

B.

Human

Bullfrog (adult)

Bullfrog (larva)

Lungless salamander

C.

Lungless

Bullfrog (larva)

Bullfrog (adult)

Human

salamander
D.

Bullfrog (adult)

Bullfrog (larva)

Lungless salamander

Human

E.

Bullfrog (adult)

Human

Bullfrog (larva)

Lungless salamander

21

13th International Biology Olympiad. Theoretical Test, Part A

Ethology
A 41. Fixed action patterns (FAPs) are important components of behaviour. Which statement
about the fixed action patterns is not true?
A. They are highly stereotypical, instinctive behaviours
B. FAPs are triggered by sign stimuli in the environment, and once begun, are continued to
completion
C. A supernormal stimulus often produces a stronger response
D. FAPs diminish the adaptive significance of behaviour
E. FAPs are often released by one or two simple cues associated with the relevant object in
an organism

A 42. Which feature correctly describes the return of salmon to their native stream to spawn?
A. Insight
B. Olfactory imprinting
C. Habituation
D. Classic conditioning
E. Positive taxis

A 43. Why did psychologists fail in teaching chimpanzees to talk like humans?
A. Chimpanzees have a different location and structure of larynx
B. They have weakly developed cerebrum
C. They have thin tongue
D. They have too large teeth
E. They have bad memory

22

13th International Biology Olympiad. Theoretical Test, Part A

A 44. Why do territorial birds,

which are territory owners tend to win when they meet

intruder birds?
A. They are more aggressive and better fighters
B. They have more to gain from a fight and so they are prepared to fight harder. The higher
benefit associated with territory, the harder they fight for it
C. Ownership is simply a conventional settlement
D. Owners always have a larger body size
E.

Both A and D are correct answers

23

13th International Biology Olympiad. Theoretical Test, Part A

Genetics and evolution


A 45. What is the probability for exactly three children to have a dominant phenotype in a
family with four children of heterozygous parents (Aa x Aa)?
A. 42%
B. 56%
C. 36%
D. 44%
E. 60%

A 46. Mouse hair colour is determined by two unlinked loci C and B. Mice with genotype
CC or Cc are agouti, and with genotype cc-albino because pigment production in hair is
blocked. At the second locus, the B allele is dominant to the b, and the B allele determines
black agouti coat colour, but b - brown agouti coat colour.
A mouse with a black agouti coat is mated with an albino mouse of genotype bbcc. Half of
the offspring were albino, one quarter - black agouti, and one quarter were brown agouti.
What was the genotype of the black parent?

A. BBCC
B. BbCc
C. BbCC
D. Bbcc
E. BBcc

24

13th International Biology Olympiad. Theoretical Test, Part A

A 47. After graduation, you and 19 friends (sex ratio close 1:1) build a raft, sail to a deserted
island, and start a new population, totally isolated from the world. Two of your friends carry
(that is, are heterozygous for) the recessive c allele, which in homozygotes causes cystic
fibrosis.
What will be the incidence of cystic fibrosis on your island, if you assume that the frequency
of this allele does not change during the growth of population?
A. 0.05 %
B. 0.0025 %
C. 0.25 %
D. 0.5 %
E. 0.10 %

A 48. Suppose that allele b is sex-linked (located on X chromosome), recessive and lethal. It
kills the zygote or embryo. A man marries a woman who is heterozygous for this gene. What
would be the predicted sex ratio of the children of this couple if they have many children?
Girls

Boys

A.

B.

C.

D.

E.

25

13th International Biology Olympiad. Theoretical Test, Part A

A 49. Two X-linked genetic defects have been studied by genealogical method (family
history): colour blindness and deficiency of certain enzyme in red blood cells. The pedigree
shows the results.
Normal female
1

Normal male

Colour-blind male
3

Enzyme-missing male
Male with both defects

Which individual (-s) show (-s) that crossing over has occurred?
A. 8 and 9
B. 1
C. 7 and 8
D. 7 and 9
E. 5

26

13th International Biology Olympiad. Theoretical Test, Part A

A 50. Which statement about the meiotic behaviour of chromosomes in a translocation


heterozygote is true?
A. Only adjacent chromosomal segregations yield viable gametes
B. Chromosomes form a cross-shaped structure during prophase I
C. All gametes produced by a translocation heterozygote are non-viable
D. All gametes produced by a translocation heterozygote contain either duplications or
deletions
E. The correct answers are A and D

A 51. Dihybrid crosses between tall, spherical seeded plants and short, dented-seeded plants
in the F1 generation produced only tall, spherical seeded plants. A testcross of F1 hybrids with
short, dented-seeded plants produced many more tall, spherical seeded plants and short,
dented-seeded plants than tall, dented-seeded and short spherical seeded plants. Which is the
right conclusion?
A. Genes for tallness and seed shape are located in different chromosomes
B. Genes for tallness and seed shape are located in the same chromosome, and are
completely linked
C. Genes for tallness and seed shape show incomplete linkage
D. Traits for tall, dented-seeded plants are dominant
E. Traits for short, spherical seeded plants are recessive

27

13th International Biology Olympiad. Theoretical Test, Part A

A 52. Which part in the given DNA sequence corresponds to the translating sequence of this
gene?
Promoter
-10
0
5`- TATCTTATGTTCTCAATCTTGAGGAGGAGGTACGCTATGAAGTCTCACGAATGGCTTAATAGTAG-3`

A.
B.
C.
D.
E.

ATGTTCTCAATCTTGAGGAGGAGGTACGCTATGAAGTCTCACGAATGGCTTAATAGTAG
ATGAAGTCTCACGAATGGCTTAATAGTAG
ATGGCTTAATAGTAG
TATCTTATGTTCTCAATCTTGAGGAGGAGGTA
TATCTTATGTTCTCAATCTTGAGGAGGAGGTACGCTATG

A 53. What is the key difference between heterochromatin and euchromatin?


A. Heterochromatin is found only near the centromeres; euchromatin is found near the ends
of chromosomes
B. Euchromatin is true chromatin; heterochromatin is a DNA-protein complex
C. The X chromosome is made up of euchromatin; heterochromatin is found in the Y
chromosome
D. Heterochromatin is found in prokaryotic DNA; euchromatin is found only in eucaryotic
DNA
E. Heterochromatin is transcriptionaly silent, while euchromatin is often transcriptionaly
active

28

13th International Biology Olympiad. Theoretical Test, Part A

A 54. In crossing true-bred yellow and grey fruit flies Drosophila, the following results were
obtained:
Parents

Progeny

Grey female x yellow male

All grey

Yellow female x grey male

All males yellow


All females - grey

Which statement is correct?


A. Alleles for grey and yellow body colour are codominant
B. The allele for grey body colour is X-linked recessive
C. The allele for yellow body colour is X-linked dominant
D. The allele for grey body colour is X-linked dominant
E. The allele for yellow body colour is autosomal recessive

A 55.

In the figure, each column represents a hypothetical haplotype for four RFLP

(restriction fragment length polymorphism) loci, each with two alleles (indicated by 1
or 2), and the disease locus, where n indicates normal allele and m the mutant allele for a Xlinked locus.

Son

Mother

Grandfather

Grandmother

(Continuation see on the next page)


29

13th International Biology Olympiad. Theoretical Test, Part A

Presence or absence of the mutant allele can be detected by some direct molecular assay. The
data that the mutation is present in the mother but absent in both her parents does not tell us
which of the parents was the source of the mutant gamete. The problem is solved by
haplotype analysis using closely linked polymorphic loci. Where did the mutation, received
by the son occur?
A. In mothers germ cells
B. In grandmothers germ cells
C. In grandfathers germ cells
D. In both grandfathers and grandmothers germ cells
E. There is insufficient information to solve this problem

30

13th International Biology Olympiad. Theoretical Test, Part A

Ecology
A 56. Which statement (-s) is (are) correct?
1. Food chains usually have at least 7 levels
2. Food chains are limited in length by energy losses, for example in
respiration
3. Most of the world terrestrial above-ground production is utilized directly
by detritivores
4. Gross energy production is the remaining assimilated energy after
respiration
A. 2, 3 and 4
B. Only 2
C. Only 1
D. 1 and 3
E.

2 and 3

A 57. Which statements are correct?


1.

Some autotrophic bacteria obtain energy oxidizing NH4+ to NO2- or NO2- to NO3-

2.

Some autotrophic bacteria obtain energy reducing NO2- or NO3-

3.

Nitrogen-fixing cyanobacteria can utilize atmospheric nitrogen (N2)


31

13th International Biology Olympiad. Theoretical Test, Part A

4.
5.

The ocean serves as a buffer, stabilizing the atmospheric CO2 concentration


Coral reefs are very productive ecosystems, but they contain a minor portion of

the global amount of assimilated C


A.

3, 4 and 5

B.

2, 3, 4 and 5

C.

1, 4 and 5

D.

1, 3, 4 and 5

E.

Only 4 un 5

32

13th International Biology Olympiad. Theoretical Test, Part A

33

13th International Biology Olympiad. Theoretical Test, Part A

A 60. Which statement (-s) is (are) correct?


1.

Succession after forest logging is an example of secondary succession

2.

Succession after forest fire is an example of secondary succession

3.

Generally, fire is a very important ecological process, as many ecosystems

depend on fire for their renewal


4.

In climax forests, most of the under storey species will have high

competitive ability.
5.

In climax forests, most of the under storey species are stress-tolerant

species
A.

1, 2 and 4

B.

Only 1, 3 and 5

C.

1, 2, 3 and 5

D.

Only 1

E.

Only 3 and 5

34

13th International Biology Olympiad. Theoretical Test, Part A

A 61. Aphids are common prey for ladybird beetles. The figure shows the amount of time

Time spent feeding on a leaf

spent feeding on a leaf by ladybird beetles. What does the figure indicate?

45
40
35
30
25
20
15
10
5
0
0

10

15

20

25

30

35

number of aphid individuals

1. Ladybird beetles become confused when stationary prey is abundant, and have to spend a
longer time capturing an individual
2. The size of the ladybird population is dependent on the number of aphids available
3. Ladybirds do not waste energy searching for aphids when they are in short supply
4. Ladybird beetles have a better chance of spotting from afar a leaf with many aphids as
compared to a leaf with few aphids
5. Ladybird beetles spend more time on leaves where there are more aphids, because their
net energy gain is maximum due to fewer losses from searching
A. Only 1
B. Only 2
C. Only 3
D. 3 and 5
E. All the answers are correct.

35

13th International Biology Olympiad. Theoretical Test, Part A

A 62. Changes that occur in a forest developing on abandoned farmland are represented in the
graph below. What do the curves R, S and T indicate?

Curve R

Curve S

Curve T

A.

Net productivity

Respiration

Succession

B.

Gross productivity

Net productivity

Heterotrophy

C.

Gross productivity

Respiration

Total biomass

D.

Community respiration

Total biomass

Net productivity

E.

Respiration

Total biomass

Gross productivity

36

13th International Biology Olympiad. Theoretical Test, Part A

A 64. In a Latvian pond, a random sample of carp fish consisted of 120 individuals. All
individuals were permanently marked and released without injuring them. On the next day,
150 individuals were captured, of which 50 were marked. Assuming no change in the total
population size between the two days, what is the size of the population in the pond?
A. 3600
B. 6000
C. 170
D. 360
E. 50

37

13th International Biology Olympiad. Theoretical Test, Part A

A 65. Which of the figures below show density-dependent mortality that could play a role in

Number of deaths

number of deaths

population density

population density

mortaility rate

mortality rate

population density

population density

regulation of population size?


A. W, X, Y and Z
B. Y and Z
C. W and X
D. Only Y
E. W, Y and Z

38

13th International Biology Olympiad. Theoretical Test, Part A

A 66. In an experiment to determine the proportion as a percent of cabbage leaf material eaten
by a caterpillar that was converted to biomass, it was observed that the caterpillar ate 2 cm2 of
leaf per day . In order to make an estimate of the conversion several measurements were done.
W. Average dry mass per cm2 of leaf similar to that eaten
X. Total mass of caterpillar faeces per day
Y. Dry mass of caterpillar faeces per day
Z. Mass of carbon dioxide produced per day
Which of the given equations for estimating B, the mass of cabbage leaf converted into
caterpillar biomass per day, is correct?
A.

B = 2W-Y-Z

B.

B = W-Y-Z

C.

B = (2W-Y-Z)/2W

D.

B = 2W-X-Z

E.

B = W-X-Z

39

13th International Biology Olympiad. Theoretical Test, Part A

Biosystematics

A 68. What do all Angiosperms have that all Gymnosperms lack?


A. vascular cambium
B. secondary xylem
C. pericarp
D. cotyledons
E. seeds

40

13th International Biology Olympiad. Theoretical Test, Part A

A 70. Which of the following is false about the life cycle of mosses?
A. Gametophytes arise from a protonema
B. External water is required for fertilization
C. Gametes are produced by meiosis
D. Antheridia and archegonium are produced by the gametophytes
E. Sperms have flagella

A 71. Many benthic marine invertebrates have free-living planktonic larvae. Which of the
following invertebrates all have planktonic larvae?
A. Nematoda, Echinodermata, Polychaeta, Turbellaria
B. Polychaeta, Turbellaria, Echinodermata, Corallium
C. Decapoda, Echinodermata, Corallium
D. Bivalvia, Turbellaria, Porifera, Nematoda
E. Cephalopoda, Gastropoda, Bivalvia, Echinodermata

A 72. Which statement regarding the systematics of following taxa is correct?


A. Phylum Platyhelminthes includes Hirudinea, Turbellaria and Cestoda
B. Phylum Arthropoda includes Chilopoda, Polychaeta, Crustacea
C. Phylum Platyhelminthes includes all the parasitic worms
D. Phylum Arthropoda includes water insects and water mites
E. Phylum Echinodermata and phylum Cnidaria are close relatives, as they possess a radial
symmetric body

41

13th International Biology Olympiad. Theoretical Test, Part A

A 73. A biology student made some comments after examining Turbellaria, tapeworm
(Taenia sp.) and trematoda (Fasciola hepatica). Which of his comments is not true?
A. Reduction in digestive system evolved in conjunction with passing to the parasitic life
style
B. With passing to the parasitic life style, the reproduction capacity has increased
C. The total disappearance of the digestive system in the parasitic species did not cause any
additional change in the body
D. The excretory systems of these animals basically resemble each other
E. The parasitic life style did not cause any change in the basic structural plan of the nervous
system

A 74. Parasites are adapted to the host and its life cycle. Which of the following statements
about parasitic species is correct?
A. Numerous Nematoda, Turbellaria and Cestoda are endoparasites in the intestine of fishes
B. Fleas, lice and most female mosquitoes are ectoparasites of warm-blooded animals
C. Cyclops are the intermediate hosts of Trematoda and Cestoda
D. The parasitic Nematoda and Turbellaria develop directly without an intermediate host
E. Trematodes and Turbellaria are endoparasites of cold-blooded animals

A75. Heterothermy is the ability to reduce body temperature during hibernation until it gets
close to the environmental temperature. Which of the following animal groups include
heterothermal organisms?
A. Rodentia, Chiroptera, Insectivora
B. Only Carnivora
C. Carnivora, Chiroptera
D. Penguins
E. All mammals living in burrows
42

13th International Biology Olympiad. Theoretical test, Part A

Answer key
A1 A2 A3

A5 A6 A7 A8 A9 A10 A11 A12 A13 A14 A15 A16 A17 A18 A19 A20 A21 A22 A23

A
B
C
D
E

A
B
C
D
E
A25 A26 A27 A28 A29 A30 A31 A32 A33 A34 A35 A36

A38 A39 A40 A41 A42 A43 A44 A45 A46

A
B
C
D
E

A
B
C
D
E
A47 A48 A49 A50 A51 A52 A53 A54 A55 A56 A57

A
B
C
D
E

A60 A61 A62

A64 A65 A66

A68
A
B
C
D
E

A70 A71 A72 A73 A74 A75


A
B
C
D
E

43















All IBO examination questions are published under the following Creative Commons license:



CC BY-NC-SA (Attribution-NonCommercial-ShareAlike) https://creativecommons.org/licenses/by-nc-sa/4.0/
The exam papers can be used freely for educational purposes as long as IBO is credited and
new creations are licensed under identical terms. No commercial use is allowed.

13th International Biology Olympiad. Theoretical test, Part B

Part B
Please read carefully all the instructions!
For questions with multiple correct answers, you will be penalised for additional incorrect
responses.
Only the calculators which were provided with the Olympiad materials are permitted.
Mark all the correct answers in the answer sheet! The jury will check only the answer
sheet!

Cell Biology
B 1. The dependence of the initial reaction rate on substrate concentration for 3 different
enzymes (X, Y and Z) is given in the table:
Substrate concentration

Initial rate (arbitrary units)

(arbitrary units)

0.92

0.91

0.032

1.67

1.67

0.176

2. 85

2.86

0.919

3.75

3.75

2.180

4.40

4.44

3.640

10

4.90

5.00

5.000

15

5.80

6.00

7.337

20

6.23

6.67

8.498

30

6.80

7.50

9.397

50

6.00

8.33

9.824

100

4.20

9.09

9.968

1. Plot the initial rates versus substrate concentrations on the answer sheet! (1 point)
2. Which enzyme (X, Y or Z) is a regulatory enzyme with a co-operative behaviour? (1 point)
3. Which of the enzymes (X, Y or Z) is inhibited by its own substrate? (1 point)

13th International Biology Olympiad. Theoretical test, Part B

B 2. For an exponentially growing culture of microorganisms the specific growth rate () is a


parameter, that gives the cell biomass (g) synthesized per gram of existing cell biomass per
unit of time (usually, per hour). This rate () is inversely related to the doubling time of the
culture, td: = ln2/td 0.7/td. Hence, the shorter the doubling time of cells, the higher is the
specific growth rate of the culture.
Two microorganisms, A and B, were inoculated each in a fresh growth medium with an initial
optical density (OD) of 0.1. A lag phase of 1 hr duration was observed for both cultures.
Three hours after inoculation, the OD of culture A was 0.4, while that of the culture B was
1.6.
1. Estimate the specific growth rate for culture A
2. Estimate the specific growth rate for culture B

(2 points)

B 3. Calculate the intracellular millimolar (mM) concentration of potassium in Escherichia


coli, if the measured potassium content is 7.8 micrograms per milligram of dry cell mass.
Assume all potassium ions are free in the cytosol (not bound to macromolecules), and that the
intracellular volume is 2 microlitres per milligram of dry cell mass. The atomic weight of
potassium is 39 Daltons.

(1 point)

B 4. A species of fungus can dissimilate glucose and produce ATP in two ways.
Aerobically: C6H12O6 + 6O2 = 6 CO2 + 6 H2O,
Anaerobically: C6H12O6 = 2 C2H5OH + 2 CO2
This fungus is cultivated in a glucose-containing medium. Half of the total ATP production is
anaerobic.
1. What is the ratio between the rates of aerobic and anaerobic catabolism of glucose?
(Continuation see on the next page)

13th International Biology Olympiad. Theoretical test, Part B

2. What is the expected oxygen consumption (moles per mole of consumed glucose)?
3. What is the expected CO2 evolution (moles per mole of consumed glucose)?
For calculations, assume that glucose is fermented via the usual Embden-Meyerhof-Parnas
glycolytic pathway, and that oxidative phosphorylation proceeds with maximum efficiency.
(3 points)

B 5. For the bacteria Bacillus subtilis, several auxotrophic mutants have been obtained which
need addition of aspartate, threonine or methionine to the growth medium.
Mutant

Amino acid precursors

Amino acid

Metabolite,

that are not synthesized

needed for

accumulating in

by the mutant

growth

the medium

aspA

4. Aspartate

7. Fumarate

5. Methionine

3. Homoserine

metH

5. Methionine

1. Homocystein

thrC

6. Threonine

2. Homoserinephosphate

metA

1. Homocystein

thrB

2. Homoserinephosphate

6. Threonine

3. Homoserine

thrA

3. Homoserine

6. Threonine

4. Aspartate

2. Homoserinephosphate

5. Methionine

1. Homocystein
_________________________________________________________________________

1. What is the biosynthetic pathway for methionine biosynthesis?


2. What is the biosynthetic pathway for aspartate biosynthesis?
3. What is the biosynthetic pathway for threonine biosynthesis?
Indicate the pathway with appropriate numbers from the table (1-7) and arrows in the answer
sheet!

(3 points)

13th International Biology Olympiad. Theoretical test, Part B

B6. Before a lecture, an assistant noticed that comments on an important diagram are lost. He
found many of terms in a textbook, including some which were unrelated to this diagram.
1. Please help the assistant to locate the correct terms for this diagram and to place the
appropriate numeric labels in the table in the answer sheet.

Enzyme

ATP +

+ AMP + PP
UGU

UGU

5
6

7
UGU

10

8
UAU
11

AUG

CGC

GU

AUA ACA

CGU

CCC

UCA

12
13

14
13

(Continuation see on the next page)

13th International Biology Olympiad. Theoretical test, Part B

Term

Number

Term

A-1

Amino acid

A-2

Growing polypeptide

B-1

Pentose

B-2

Growing DNA strand

C-1

Fatty acid

C-2

Growing RNA strand

D-1

Small ribosomal subunit

D-2

Alpha subunit of RNA

Number

polymerase
E-1

tRNA

E-2

Nuclear pore

F-1

IgG

F-2

P site

G-1

Receptor

G-2

Centriole

H-1

Aminoacyl-tRNA

H-2

Large ribosomal

synthetase

subunit

I-1

Protein kinase

I-2

A-site

J-1

Glucokinase

J-2

Z-site

K-1

Aminoacyl-tRNA

K-2

Peptidyl-tRNA

L-1

Inductor

L-2

DNA polymerase

M-1 Operator

M-2

Spliceosome

N-1

N - end

N-2

Adenylate cyclase

O-1

C - end

O-2

Capsomer

P-1

5` - end

P-2

Single stranded DNA

R-1

3` - end

R-2

Codon

S-1

Nucleotide

S-2

Initiation codon

T-1

Lysosome

T-2

Gene

U-1

Sigma subunit of RNA

U-2

Terminal transferase

polymerase
(Continuation see on the next page)

13th International Biology Olympiad. Theoretical test, Part B

2. Which component of this diagram has (give the number) peptidyl transferase activity?
( 5 points)

B 7. The growth of bacteria is studied. For a period of exactly one duplication, the sample is
moved from an environment with a light nitrogen isotope (14N) to an environment with heavy
nitrogen isotope (15N). After this the sample is again transferred to the environment with light
nitrogen for a period of two duplications.
1. What is the composition of double-stranded DNA (in %) of light and heavy nitrogen
isotopes after the experiment?

A. Only light

B. In between

C. Only heavy

From these cells two types of mRNA {mRNA (A) and mRNA (B), respectively, expressed
from two different genes} were isolated. Both mRNAs were found to contain an identical
number of nucleotides. The nucleotide composition of each mRNA was estimated as (see the
table).

mRNA

A%

C%

G%

T%

U%

17

28

32

23

27

13

27

33

(Continuation see on the next page)


2. What is the nucleotide composition of double-stranded genomic DNA in the coding part of
the genes A and B, respectively.
6

13th International Biology Olympiad. Theoretical test, Part B

dsDNA

A%

C%

G%

T%

U%

A
B

3. What curve in the plot below represents the DNA melting profile of the coding part of
genes A and B, respectively?

100
80

60
40
20
0
30

50

70

90

110

% of melting
Temperature oC
Curve

5
(3 points)

13th International Biology Olympiad. Theoretical test, Part B

B 8.

The pie charts show the relative amounts of types of membrane found in two types of cells.
Suggest, why liver cells (answer 1) possess significantly more smooth ER, while pancreatic
cells (answer 2) have more rough ER. Chose the correct statements (A to E) from the left
column and pair them with the appropriate numbers (1 to 5) from the right column.

Process

Structure

Number

In nuclear membrane of pancreatic cells

In glycogen particles of liver cells

C. Higher lipolytic activity

In endoplasmic reticulum of pancreatic cells

D. Higher protein-secretory

In mitochondria of liver cells

In endoplasmic reticulum of liver cells

A. Higher synthesis of lipids

B.

Higher proteolytic activity

activity
E.

Hihger ATP-synthesizing
activity

(2 points)
8

13th International Biology Olympiad. Theoretical test, Part B

B9. The diagram shows an apparatus made by


a student to investigate the effect of
temperature on the activity of ethanol
fermentation of yeast. The conical flask
contains 2.5 g yeast suspended in 2% sucrose
solution. The meniscus moves down the glass
tube (5ml micropippete) during fermentation.

The table shows the amount of suspension (ml) pushed in the glass tube due to CO2
accumulation at regular time intervals.
Time

40 C

100 C

200 C

350 C

550 C

0
0
0.1
0.2
0.3

0.2
1.0
1.9
3.1
4.0

0.4
1.3
2.2
3.3
NO RESULT

0.7
1.2
2.8
4.4
NO RESULT

0
0.1
0.2
0.3
0.4

(min.)
1
2
3
4
5

1. Plot the data on CO2 accumulation at different temperatures.


2. Estimate the average rate of CO2 production (ml CO2/min) for the yeast suspension at 200 C
using the values obtained in the period between 2 and 4 minutes.
3. Estimate the specific rate of CO2 generation (millimoles CO2/(min g)) at 200 C.
4. What would be the specific rate of ethanol accumulation (millimoles ethanol /(min g)), if
the fermentation follows the equation? C6H12O6

2C2H5OH + 2 CO2
(4 points)

13th International Biology Olympiad. Theoretical test, Part B

Plant Anatomy and Physiology


B 10. Write the numbers (each number can be used only once) of the unnamed structures in
the appropriate boxes in the table in the answer sheet.

Phellem

Phellogen
2

BARK
Sieve Tubes
Companion cells
3

Storage Parenchyma
4

Structures
A.

Periderm

B.

Primary phloem

C.

Phloem fibers

D.

Phelloblast

E.

Phelloderm

F.

Secondary phloem

G.

Tracheids

Number

( 2 points)

10

13th International Biology Olympiad. Theoretical test, Part B

B 11. The figure shows a cross section of gymnosperm stem wood. Write in the table in the
answer sheet the appropriate numbers (each number can be used only once) of corresponding
plant structures.

Plant structure

Number

A.

Early wood

B.

Sieve tube

C.

Latewood

D.

Resin duct

E.

Companion cell

F.

Xylem parenchyma

2
(2 points)

11

13th International Biology Olympiad. Theoretical test, Part B

B 12. The following features pertain to specific structures and processes in plants. Write the
number that corresponds to appropriate structure in the table in the answer sheet!

1.

Regulates the inward flow of ions into the roots

2.

A plastid which develops in a plant when it is kept in the dark

3.

A cell type which provides the main support in gymnosperm wood

4.

Provides water movement horizontally across the stem

Plant structure
A.

Tracheids

B.

Epidermis

C.

Endodermis

D.

Resin duct

E.

Rays

F.

Leucoplast

G.

Etioplast

Number

(2 points)

B 14. The diurnal curve indicates the stomatal opening for a typical C3 plant.
12

13th International Biology Olympiad. Theoretical test, Part B

Stomatal conductance is an indication of the capacity for diffusion through stomata and an
indirect measurement of stomatal opening. A stomatal conductance of zero indicates that
stomata are closed (i.e., there is no transpiration).
1. Indicate the times of day in the diagram and mark them on the answer sheet, using the
codes:.
1. Midnight

2. Noon

3. 6:00 a.m.

4. 6:00 p.m

A
B
C
D

Stomatal opening vs. closure is regulated through several internal and external factors.
(Continuation see on the next page)

13

13th International Biology Olympiad. Theoretical test, Part B

2. Indicate and mark in the answer sheet, which of the following corresponds to the factors
depicted in the picture. Use the given code:
1. CO2

2. Light

3. Ca2+

4. Abscisic acid

5. K+

6. H2O

A and B
C
D
(4 points)
B 15. Plants require 16 essential elements - boron (1), calcium (2), carbon (3), chlorine (4),
copper (5), hydrogen (6), iron (7), magnesium (8), manganese (9), molybdenum (10), nitrogen
(11), oxygen (12), phosphorus (13), potassium (14), sulfur (15), zinc (16). The proportional
masses of various elements in plants are shown.

1. Indicate the numbers in the answer sheet, corresponding to each element in the table.
A, B, C
D
E
( 3 points)

14

13th International Biology Olympiad. Theoretical test, Part B

Animal Anatomy and Physiology


B 16. The Figure shows the overall fluid balance in the human gastrointestinal tract.
Calculate three volumes (X, Y and Z) and write them in the answer sheet.

Volume of fluids

Volume of fluids

absorbed in the

entering the gastro-

gastrointestinal tract

intestinal tract

or excreated during

during 24 hours.

24 hours.

...................................

...................................

Ingested with food and

Small intestine

fluids 2 000 ml

absorbs X ml

.................................

...................................
Colon absorbs
1 000 ml
...................................
Water excreation in
the faeces 100 ml

Saliva 1 500 ml
...................................
Gastric secretion
2 000 ml
....................................
Bile 500 ml
..................................
Pancreatic juice Z ml

Sum: Y ml

...................................
Intestinal secretion
1 500 ml
.........................
Sum: 9 000 ml
(1 point)

15

13th International Biology Olympiad. Theoretical test, Part B

B 17. Lesions in various points in the visual pathway produce deficitsat the visual field. The
level of a lesion can be determined by the specific deficit in the visual field. In the diagram of
the cortex the numbers along the visual pathway indicate the sites of lesions. The deficits that
result from lesions are shown as black areas in the visual field maps on the right.
Choose the corresponding deficits that result from lesions at each site and write the numbers
of the lesion sites in table in the answer sheet!

Number of lesion site

A.

D.
E.

(3 points)

16

13th International Biology Olympiad. Theoretical test, Part B

B 18. Please color the arrows in the answer sheet that indicate the direction of the water flow
through the cell membrane of an erythrocyte. Light circles in the figure show the water
molecules, dark circles show the molecules of the dissolved substances.

(1 point)
B 19. Please indicate the direction of the locomotion of protozoan (A) and (B) by arrows in
the boxes in the answer sheet. Arrows in the figure indicate the direction of wave caused by
ciliarys movement.
A.

B.

(1 point)

17

13th International Biology Olympiad. Theoretical test, Part B

B 20. Please mark with crosses in the table in the answer sheet the locations where filtration,
reabsorbtion and secretion take place in the mammal nephron (more than one correct answer
per process possible)!
Process

1. Renal

2. Proximal

3. Henle

4. Distal

5. Late distal tubule

corpuscle

convoluted

loop

convolu-

and collecting duct

tubule

ted tubule

A. Filtration
of fluid that is
isotonic to blood
B. Reabsorption
of water, Na+, K+,
glucose, amino
acids, Cl-, HCO-3,
urea
C. Reabsorption of
water, Na+, K+ and
ClD. Reabsorption
of water, Na+ and
ClE. Reabsorption
of water, Na+,
HCO-3 and urea

(Continuation see on the next page)

18

13th International Biology Olympiad. Theoretical test, Part B

F. Secretion
of H+ and K+
G. Secretion
of H+, NH4+,
urea, and
creatinine
H. Secretion
of urea
(5 points)

19

13th International Biology Olympiad. Theoretical test, Part B

B 22. Arrange in the answer sheet in the correct sequence the arrows (choose from 1-8) that
characterize the sequence of events in stimulation and contraction of muscle. The black
arrows indicate propagation of excitation (action potential) in the membranes; the white
arrows depolarization - induced Ca2+ propagation in the sarcoplasma.

Stimulation

88

(1 point)

20

13th International Biology Olympiad. Theoretical test, Part B

Ethology

21

13th International Biology Olympiad. Theoretical test, Part B

B 25. A blue jay actively hunts on different moths (Catocala ). The hindwings of the moths

are often strikingly coloured yellow, orange, red or other color, but the forewings of the
moths appear cryptic, looking very much like the bark of the trees on which the moths rest.
In other words moths on a cryptic background appears like background. The forewings
cover the hindwings, but when they are distrurbed, the hindwings are suddenly exposed.
On a uniform background the moths are easily seen.
The detection index estimates the ability to spot a moth.
(Continuation see on the next page)

Which statements regarding wing coloration are true? Mark them with crosses in the answer sheet.
A. The forewings are coloured to decrease detection of moths by a predator
22

13th International Biology Olympiad. Theoretical test, Part B

B.

The hindwings are brightly coloured without any significance, it is a random

feature
C.

The hindwings colour may have a startle effect on a bird, causing the bird to stop

momentarily and thus giving the moth time to escape


D.

The forewings cryptic colour does not defend moths completely, because jays learn

to see moths on a cryptic background


E.

Brightly coloured hindwings promote sexual partner recognition

F.

A predator does not discriminate colours


(3 points)

23

13th International Biology Olympiad. Theoretical test, Part B

Genetics and evolution


B 26. In fruit fly Drosophila melanogaster, there is a dominant gene (b+) for grey body
colour and another dominant gene (c+) for normal wings. The recessive alleles (b, c) of these
two genes result in black body colour and curved wings respectively.
Two students Ada and Donald made crosses to determine the distance between these two
genes. Flies with a grey body and normal wings were crossed with flies that had black bodies
and curved wings. The results obtained in Adas and Donalds experiments are shown in the
table.

Ada`s

Grey body,

Black body,

Grey body,

Black body,

normal wings

curved wings

curved wings

normal wings

236

253

50

61

55

56

241

248

experiment
Donalds
experiment
1. What is the distance (in map units) between these two loci? Mark in the answer sheet.

2. What was the genotype of flies with a dominant phenotype in Ada`s (A.) and Donald`s
(B.) experiment? Give the genotypes and show the linkage phase of genes b and c in the
answer sheet!
A. (Adass flies)
B. (Donalds flies)
(3 points)

24

13th International Biology Olympiad. Theoretical test, Part B

B 27. Several mutations (A, B, C, D) have been found in the coding sequence of a gene.
Codon 1
2
3
4
Wt
ATG TGC CCC CGA
A.
--- -C- --- --B.
--- --- --- A-C.
--- --- --- --D.
--- --- --- --* Deletion of 1 nucleotide

5
GTC
----A----

6
GAG
---------

7
GAC
---------

8
CTG
------*--

9
AGC
---------

10
CTG
---------

11
ACG
---------

12
AGC
---------

1. Please translate codons of variants of the given sequence (Wt, A, B, C, D) in one letter
codes of amino acids (use table of genetic codes) and place the answers in the table in the
answer sheet. Note: write ST instead of STOP
Codon
1
2

Wt
A.
B.
C.
D.

UCU
UCC
UCA
UCG
CCU
CCC
CCA
CCG
ACU
ACC
ACA
ACG
GCU
GCC
GCA
GCG

C
S
S
S
S
P
P
P
P
T
T
T
T
A
A
A
A

10

11

12

First base

UUU
UUC
UUA
UUG
CUU
CUC
CUA
CUG
AUU
AUC
AUA
AUG
GUU
GUC
GUA
GUG

U
F
F
L
L
L
L
L
L
I
I
I
M
V
V
V
V

(Phe)
(Phe)
(Leu)
(Leu)
(Leu)
(Leu)
(Leu)
(Leu)
(Ile)
(Ile)
(Ile)
(Met)
(Val)
(Val)
(Val)
(Val)

(Ser)
(Ser)
(Ser)
(Ser)
(Pro)
(Pro)
(Pro)
(Pro)
(Thr)
(Thr)
(Thr)
(Thr)
(Ala)
(Ala)
(Ala)
(Ala)

UAU
UAC
UAA
UAG
CAU
CAC
CAA
CAG
AAU
AAC
AAA
AAG
GAU
GAC
GAA
GAG

A
Y (Tyr)
Y (Tyr)
STOP
STOP
H (His)
H (His)
Q (Gln)
Q (Gln)
N (Asn)
N (Asn)
K (Lys)
K (Lys)
D (asp)
D (asp)
E (Glu)
E (Glu)

UGU
UGC
UGA
UGG
CGU
CGC
CGA
CGG
AGU
AGC
AGA
AGG
GGU
GGC
GGA
GGG

G
C (Cys)
C (Cys)
STOP
W (Trp)
R (Arg)
R (Arg)
R (Arg)
R (Arg)
S (Ser)
S (Ser)
R (Arg)
R (Arg)
G (Gly)
G (Gly)
G (Gly)
G (Gly)

U
C
A
G
U
C
A
G
U
C
A
G
U
C
A
G

Third base

Second base

2. Please arrange in the answer sheet the mutations (A-D) in an order, which shows their
influence on protein functions starting from the most deleterious mutation.
(4 points)

25

13th International Biology Olympiad. Theoretical test, Part B

B 28. A river has two populations of snails; a large population just off the left bank (main
population), and a much smaller one downstream near an island (island population). Consider
a locus that has two alleles, G ang g, in the island population, but is fixed for the G allele in
the main population. Let p be the frequency of the G allele in the island population.

Because of river flow, migration occurs from the large population to the island, but not the
reverse. Assume p= 0.6 before migration. After migration 12% of the islands snails originated
from the main population.
1. Calculate p after the migration!

Following the wave of migration, the island snails reproduce. For some reason, the island
snails, including the new immigrants, have a much higher mutation rate than the main
population. The mutation rate of G g in the island population is 0.003, and there is
essentially no reverse mutation (mutation in the main population is rare, and can also be
ignored).

2. Calculate p in the next generation of island snails?

(2 points)

26

13th International Biology Olympiad. Theoretical test, Part B

B 29. In a specific population, genotype frequencies have been estimated before and after
selection.
a1a1

a1a2

a2a2

0.25

0.50

0.25

0.35

0.48

0.17

Frequency before
selection
(generation F0)
Frequency after
selection
(generation F1)

1. Calculate the selection coefficient of each genotype (a1a1, a1a2, a2a2) and write the answer
in the answer sheet!
2. Against which genotype is selection the strongest? Write the answer in the answer sheet!
(2 points)

B 30. Expression of some autosomal genes depends on whether that gene came from male or
female parent. These are so called imprinted genes (imprinted genes are expressed in a parent
- specific manner). Imprinting of these genes happens during spermatogenesis or oogenesis,
and may silence the allele coming from one parent.

(Continuation see on the next page)

27

13th International Biology Olympiad. Theoretical test, Part B

Problem 1. Imprinted genes can account for many cases of incomplete penetrance. The
pedigree shows the incomplete penetrance of an autosomal dominant gene resulting from
imprinting during oogenesis. A woman II1 is heterozygote for this gene. Analysis of DNA
reveals that III2 and III5 have received the mutant gene from their mother.

I
1

II
1

III
1

IV
1

1. What is the probability of II1 and II2 having an affected child? Mark the answer in the
answer sheet!

2. What is the probability of III1 and III2 having an affected child? Mark the answer in the
answer sheet!

3. What is the probability of III4 and III5 having an affected child? Mark the answer in the
answer sheet!

Problem 2. Parental imprinting gives a deviation from Mendelian patterns of inheritance,


because the same allele may be differently expressed depending on whether it is inherited
from the mother or the father.
(Continuation see on the next page)

28

13th International Biology Olympiad. Theoretical test, Part B

Determine which pedigree show maternal (1) and which paternal (2) imprinting (choose from
pedigrees A,B,C,D).

1. Maternal

2. Paternal

( 5 points)

29

13th International Biology Olympiad. Theoretical test, Part B

B 31. The figures show modes of selection on (Y) a heritable quantitative (continuous) trait
and (Z) a polymorphism inherited as two alleles at one locus. In both cases, the phenotype
was assumed to be inherited additively (i.e. heterozygote intermediate between homozygotes,
there is no interaction among loci that contribute variation to the quantitative trait). The
vertical axis is the proportion of the population with each phenotype. The upper rows of
figures in both Y and Z show the distribution of phenotypes in one generation, before
selection occurs. The shaded portions represent individuals with relative disadvantage (lower
reproductive success). The lower rows of figures in both Y and Z show the distribution of
phenotypes in the following generation, after selection among the parents has occurred. X
marks the mean of the quantitative trait before selection.
(Continuation see on the next page)

30

13th International Biology Olympiad. Theoretical test, Part B

A. Directional selection does not alter the means, but may reduce the variation
B. Disruptive or diversifying selection is unlikely to be exactly symmetrical, and thus
usually shifts the means
C. Directional selection increases the proportion of genotypes with higher values of
the trait
D. Stabilizing selection is unlikely to be exactly symmetrical, and thus usually shifts
the means
E. Stabilizing selection does not alter the means, but may reduce the variation
Match in the answer sheet the statements (A E) with appropriate schemes (1 - 3)
Not all the rows in the table have to be filled.
Statements
A
B
C
D
E

Number of schemes

(3 points)
31

13th International Biology Olympiad. Theoretical test, Part B

B 32. Alkaptonuria is a rare genetic disease. The gene for alkaptonuria (alk) is recessive and
has been located on chromosome 9. Gene alk is linked to the gene I encoding the ABO blood
types. The distance between the alk gene and gene I is 11 map units. A pedigree of a family
with the alkaptonuria is shown below. Affected individuals are indicated by shaded symbols.
In addition, the blood type of family members is given.
AB

3
4 4

AB

1. What are the genotypes of individuals 3 and 4? Give the answer in the answer sheet

2. If individuals 3 and 4 are expecting their fifth child, what is the probability that the child
will have alkaptonuria (a physician has determined that foetus has blood type B)? Give the
answer in the answer sheet!

(2 points)

32

13th International Biology Olympiad. Theoretical test, Part B

Ecology
B 33. The following table shows the commercial fishing catches of smelt and fishing intensity
in the Riga Gulf between 1982 and 1985. The fishing intensity during those years was
estimated by the total time spent by all fishing boats that were concentrated on this species.
The fishing equipment did not change in the investigated time-period.
Year

Catch (tons)

Fishing intensity (relative


units)

1982
1983
1984
1985

100
150
100
150

2
5
5
3

Estimate and write in the answer sheet the relative sizes of the smelt population, starting with
a relative size of 10 units for the year 1982.
1982 = 10
1983= ..
1984= ..
1985= ..
(3 points)
B 34. Figure 1 shows the number of phytoplankton cells, and P and N (inorganic)
concentrations in water samples in a transect along the coast of the Atlantic Ocean around a
fish processing factory.
The nitrogen concentrations in the water were very low and close to the detection limit. The
closest station to the factory was Station 6. Figure 2 shows the results of a nutrient
enrichment study. Water samples taken at the Stations were filtered to remove phytoplankton.
(Continuation see on the next page)

33

13th International Biology Olympiad. Theoretical test, Part B

Then standard amounts of phytoplankton were added to the samples, and then they were
enriched with only N, only P, or were unenriched, and the number of phytoplankton cells in
these enriched or unenriched samples were counted at a later time.
Figure 1
8

7
Phytoplankton
(mill. Cells/ml)

5
P (g/l)

4
3

N (g/l)

2
1
0
0

9 10 11

Station

Algal population (million cells/ml)

30

25
20

N-enriched

15

P-enriched

Unenriched control

10

5
0
1

9 10

Station

Figure 2

1. Which is pollutant or pollutants that factory is emitting? Give the answer in the answer
sheet using the appropriate codes.
Codes: X. Phytoplankton
Z. Nitrogen

Y. Zooplankton

W. Phosphorus

S. Organic substances

2. Which is the main limiting nutrient (A) in this is case?


Which is the second limiting factor (B) in this case?
(3 points)

34

13th International Biology Olympiad. Theoretical test, Part B

B 35.

The diagram shows a 10m x 10m plot located in a forest with two main tree species: Species
X which is shaded grey, and Species Y which is shaded black. The plot is divided into a grid
with step 1m.
1. What are the percentage frequencies of Species X and Species Y using a quadrate size of 2
m x 2m?
2. What are the percentage frequencies of Species X and Species Y using a quadrate size of
5m x 5m. Write the correct answers in the answer sheet.
1. Quadrate of 2. Quadrate of
size 2m x 2m
A.
B.

size 5m x 5m

Species X
Species Y
( 4 points)

35

13th International Biology Olympiad. Theoretical test, Part B

B 36. The Baltic Sea is brackish, receiving salt water from the North Sea, and fresh water
from rivers. Turnover of water in deep layers is much slower than at surface layers.
Stratification of the water column is common in summer. The following figure shows a depth
profile (in July) for oxygen concentration (mg/l), hydrogen sulphide concentration (mg/l),
salinity (PSU) and temperature (C) in the water column.

Match in the table in the answer sheet the labels A, B, C, and D with oxygen concentration,
hydrogen sulphide concentration, salinity and temperature, and the labels E, F and G with
parts of the curves halocline, redoxycline and thermocline.
1

Oxygen concentration

Hydrogen sulphide concentration

Salinity

Temperature

Halocline

Redoxycline

Thermocline
(7 points)

36

13th International Biology Olympiad. Theoretical test, Part B

Biosystematics
B 37. There are 4 flower diagrams shown:

Write the appropriate label (A-D) of a diagram in the table in the answer sheet.
Flower formulas
1
2
3
4
5

Label

Ca(5) Co5 A5+5 G(3)


Ca(5) Co(5) A(5)
Ca5 Co5 A0+5 G(3)
Ca5 Co(5) A5 G(2)
Ca5 Co(5) A5 G(4)
(2 points)

B 38. The pictures below show a seed (1) or a fruit (2). Write the answers (1 or 2) in the table
in the answer sheet.

(2 points)

37

13th International Biology Olympiad. Theoretical test, Part B

(2 points)

B 41. The systematic, morphological, biological and ecological characters of two invertebrate
species are presented. Write the appropriate number of an organism in the table in the answer
sheet!
A. Protist

B. Arthropod

No flagellum, no cilia

Secondarily reduced wings

Complex life cycle

Incomplete metamorphosis

Intracellular parasite

Ectoparasite on mammals

Intermediate host present

Absence of intermediate host

..

...

Code: 1 Plasmodium
4 Trypanosoma

2 Dysentery amoeba
5 Tick

6 Flea

3 Paramaecium
7. Human louse

8 Spider
(2 points)

38

13th International Biology Olympiad. Theoretical test, Part B

B 42. Most birds start to incubate when their clutch is full. There are species which start
incubation after the first egg is laid. Their chicks hatch asynchronously, which is
characteristic to birds of prey and owls (Falconiformes, Strigiformes).
Mark all the correct statements with crosses in the table in the answer sheet.
A.

Food resources for birds of prey, and therefore the number of chicks they
can feed, differ between years significantly

B.

Younger nestlings are fed more often and they catch up with older ones in
the progress of their growth

C.

Birds of prey feed as many chicks of the brood as the food resources
allow in the given year

D.

During years with scarce food resources, food is given mainly to the
oldest nestlings, while the youngest ones starve to death

E.

Older nestlings help to feed younger ones

F.

Room in the nest is not sufficient for several big chicks simultaneously,
therefore they grow up and fly out of the nest one at a time

G.

One fledgling that can reach reproduction age is more important for
species survival then several but not well developed fledglings

H.

The number of fledglings and not their fitness is the most important for
the species survival

(2 points)

39

13th International Biology Olympiad. Theoretical test, Part B

ANSWER KEY
B 1.
Z
1.
Y

2. Z (1 point)
3. X (1 point)

B 2.

1. A = 0.7 g g-1h-1

B 3.

100 mM [1 point]

B 4.

1. 1.16 or 1.18 or 1;19

[1 point]

2. B = 1.4 g g-1h-1

[1 point]

2. 0.30 or 0.32 or 0.353


3. 2.2 or 2.21 or 2.24

B 5.

1. 7 4 3 1 5

2. 7 4

3. 7 4 3 6

[1p x 3 = 3 points]
40

13th International Biology Olympiad. Theoretical test, Part B

B 6. 1.
A-1
B-1
C-1
D-1
E-1
F-1
G-1
H-1
I-1
J-1
K-1
L-1
M-1
N-1
O-1
P-1

Number
1

12
2 or 10

5
11

A-2
B-2
C-2
D-2
E-2
F-2
G-2
H-2
I-2
J-2
K-2
L-2
M-2
N-2
O-2
P-2

R-1

R-2

S-1
T-1
U-1

S-2
T-2
U-2

2. 17

Number
6

3. skipped

8
7
9
10

13 or 14 or
both
13

[2 points per column]


[1 point if there are 1-3 mistakes]
B7.
1.
A.
75 %

B.
25 %

C.
0%

[1 point]
2.
dsDNA
A
B

A%
20
30

C%
30
20

G%
30
20

T%
20
30

U%
0
0

3.

B 8: skipped

41

13th International Biology Olympiad. Theoretical test, Part B

B 9.
1.

35C

20C

10C

55C
4C

[1 point for X axis]


[1 point for Y axis]
2. 1 ml/min

[1 point]

3. 0.017 0.018 mmoles CO2/g .min [1 point]


4. 0.007 0.018 mmoles ethanol/ g .min [1 point]

B 10.

42

13th International Biology Olympiad. Theoretical test, Part B

A.
B.
C.
D.
E.

F.

Number
1
4
2
3

G.

B 11.

A.

Number
3

B.
C.

D.
E.
F.

4
1
2

B 12.

A.
B.
C.
D.
E.
F.
G.

Number
3

2.
A.
B.
C.
D.
E.

3
2
4
1

2.
A and B
C
D

B 15.
1.
A, B, C
D
E

5,6
2
4

3,6,12
2,8,11,13,14,
15
1,5,4,7,9,10,1
6

2.
A.
B.
C.
D.
E.
F.
B 16.

1
X = 7900 ml
4
2

B 13.
1.
A.
B.

A
B
C
D

Y = 9000 ml
Z = 1500 ml
B 17.
A. 3
B. 4
C. 6
D. 2
E. 1
5
F.

B 14.
1.
43

13th International Biology Olympiad. Theoretical test, Part B

or 18.2 or 0.182 or 0.185 units

B 18.
[0.5p
x2=
1
point]
B 19.

2. A. b + c / bc
B. b + c / bc+
[1p x 3 = 3 points]
B 27.
1.

A.
10
L

11
T

12
S

M C P

V E D L S

M C P R

E D L S

W
t

B.
[0.5p x 2 = 1 point]

B 20.
1.

1 2 3 4 5 6 7 8 9
M C P R V E D L S

2.

3.

4. 5.

A.
B.
C.
D.
E.
F.
G.
H.
[5 points]
[1p per each column]
B 21: skipped
B 22.
1,4,6,8
[1 point]
B 23: skipped
B 24: skipped
B 25.
A.
skipped
B.
C.
D.
E.
F.
[1p x 2 = 2 points]
[-1p per incorrect answer]
B. 26.
1. 18.5 units

P R V E D L S

M C P R V E D Stop
D
[S,R,I, Stop = 1 point]
2. DACB [2 points]
B 28.
1. p = 0.648
2. p = 0.646
[1p x 2 = 2 points]
B 29.
1. a1a1= 0
a1a2= 0.3
a2a2= 0.6
2. a2a2
[1p x 2 = 2 points]
[1p for part 1 and 1p for part 2]
B 30.
1. 0 %
2. 50 %
3. 0 %

44

13th International Biology Olympiad. Theoretical test, Part B

1.

[1p x 4 = 4 points]

2.

B 36.

A D
..... .....
[1p x 5 = 5 points]
B 31.
A.
3
B.
1
C.
D.
2
E.
[1p x 3 = 3 points]
[-1p per incorrect answer]

B 32.
1. 3 = JB alk/JC alk
4 = JA alk /JC alk
2. 11 %
[0.5p x 2 = 1 point] +
[1p x 1 = 1 point] = 2 points

C
1
D
2
B
3
A
4
F
5
G
6
E
7
[1p X 7 = 7 points]
B 37.
A
1
2
B
3
D
4
C
5
[0.5p x 4 = 2.0 points]
B 38.
A 2
B 1
C 1
D 2

B 33.

[0.5p x 4 = 2.0 poins]

1983 = 6

B39 & B40 cancelled

1984 = 4

B 41.
A.
B.
2
7
[1p x 2 = 2 points]

1985 = 10
[1p x 3 = 3 points]
B 34.
1. .Z, W or Z, W, S
2. A. Z; B (skipped)
[1p x 2 = 2 points]
B 35.

A.
B.

1.
100
16

2.
100
100

B 42.
A.
B.
C.
D.
E.
F.
G.
H
[0.5p x 4 = 2.0 points]
[ - 0.5p per incorrect answer]

45

INTERNATIONAL BIOLOGY OLYMPIAD


PRACTICAL PROBLEMS

2002, Jurmala-Riga, Latvia















All IBO examination questions are published under the following Creative Commons license:



CC BY-NC-SA (Attribution-NonCommercial-ShareAlike) https://creativecommons.org/licenses/by-nc-sa/4.0/
The exam papers can be used freely for educational purposes as long as IBO is credited and
new creations are licensed under identical terms. No commercial use is allowed.

13th IBO. Practical test, Laboratory I


Animal Systematics and Morphology

Code number:

Laboratory I
Animal Systematics and Morphology
Length of the practical test 60 minutes; 40 points
Tools and Equipments
Microscope, microscopic slides, cover slips, preparation needle, forceps, 10 Petri Dishes,
labelled A to J, permanent marker. There are specimens of one species in each vial.
Introduction
Scientists have investigated the fauna of a freshwater lake. Samples of benthos, plankton and
surface dwelling animals were taken to characterise the fauna of aquatic animals and
relationships among them. The samples were sorted, then preserved in formaldehyde and
finally transferred to a 70% solution of ethyl alcohol, or stored alive.
Tasks
Q. 1. Fill in the answer code of the phylum for the specimens in each vial in the answer sheet.
The jury will check only the answers in the answer sheet!
Answer codes:
01 Arthropoda
02 Annelida
03 Porifera
Vial

(5 points)

04 Platyhelminthes
05 Mollusca

Phylum

13th IBO. Practical test, Laboratory I


Animal Systematics and Morphology

Code number:

Q. 2. Fill in the answer code of the taxonomic units for specimens in each vial in the answer
sheet.
Answer codes:
01 Crustacea
02 Diptera
03 Heteroptera

04 Hirudinea
05 Turbellaria
06 Odonata

07 Coleoptera
08 Lamellibranchiata
09 Euspongia
(5 points)

Vial

Taxon

Q. 3. Mark with crosses in the table in the answer sheet the observed characters for the
species for specimens in each vial.
Characteristic

Vials
A

K. Laterally flattened body


L. Abdomen covered by elytrae
M. Body naked
N. Labium with hooks
O. Piercing-sucking mouth parts
P. Swimming bristles on body
R. Eyes absent
S. Eyes rudimentary
T. Eyes well developed
(10 points)
Q. 4A. Prepare 2 whole-mount microscope slides (I and II) for two specimens with the
characteristics mentioned below. Use the provided materials. Mount the specimens in

13th IBO. Practical test, Laboratory I


Animal Systematics and Morphology

Code number:

glycerine. You will get 1 point for choosing the correct specimen and 1 point for a wellprepared slide.
Please raise your hand when you have prepared both slides!
Slide I
Specimen with clearly visible head capsule, spiracles for breathing of atmospheric air and
swimming bristles on the body.
Slide II
Specimen with antenna and antennula, laterally flattened body and compound eyes,
planktonic.
(4 points)
Q. 4B. Select the common characteristic for both animals and write the answer code in the
space provided .
Answer codes:
01 Free swimming

02 Attached to plants

03 Benthic
(1 point)

Q. 5. Fill in the answer sheet with the appropriate code of gas exchange for the specimens in
each vial.
Answer codes:
01 Spiracles and tracheae

Vial

02 Surface of body

03 Gills or rectal gills

Codes
(5 points)
Q. 6. Based on each pair of characteristics provided, complete the diagrammatic dichotomous
key in the answer list by filling in the letters of specimens (A-J) in the appropriate empty
spaces in the boxes.
3

13th IBO. Practical test, Laboratory I


Animal Systematics and Morphology

Code number:
(10 points)

Answer codes:

11 Definite shaped body

01 Legs present

12 Indefinite shaped body

02 No legs
13 Body segmented
03 Six legs

14 Body without segmentation

04 More than six legs


15 Head capsule developed
05 Planktonic animal

16 No developed head capsule

06 Benthic animal
17 Body naked
07 Adult, no gills

18 Body covered by shell

08 Larva, rectal gills present

09 Swimming legs, live in the water


10 Running legs, live on the water surface

01

04

03

07

02

11

08

12

13

14

09

16

15

10

05

17

18

06

13th IBO. Practical test, Laboratory I


Animal Systematics and Morphology

Code number:

LABORATORY II
PLANT SYSTEMATICS, ANATOMY AND PHYSIOLOGY
Length of the practical test - 60 minutes; 40 points
In the laboratory you must solve 3 tasks
Task 1 Plant Systematics
Task 2 Plant Anatomy
Task 3 Plant Physiology

Task 1 Plant Systematics


Materials and instruments
You will use the instrument set that you received upon registering for the 13 th IBO!
You will also use other instruments and materials: samples No. 1-8, magnifying glass
You have 8 plants (samples 1-8) which can belong to the following plant families (A-J) in the code table:

Code Table
A. Apiaceae

E. Fabaceae

H. Poaceae

B. Asteraceae

F. Geraniaceae

I. Ranunculaceae

C. Brassicaceae

G. Lamiaceae

J. Rosaceae

D. Araceae

By morphological characteristics, determine the respective families (from those given in the
code table), for the samples (1-8)!
(continued on next page)

13th IBO. Practical test, Laboratory I


Animal Systematics and Morphology

Code number:

Q1. Identify each sample in the dichotomous identification key below. Enter the sample number in the provided
windows in the answer sheet.

(8 points)

Write the family codes (A-J) in the windows provided beside the appropriate sample numbers
(8 points)
Identification key
Thesis
Nr.

Sample

Family

Nr.

code
(A-J)

(1-8)
1.

Flowers without perianth. Venation parallel 2.

Flowers with calyx and corolla. Venation netted 3.

2.

Inflorescence spike

Inflorescence panicle as a dense cylinder

3.

Inflorescence head

Flowers single or in an inflorescence, that is not a head 4.

4.

Flowers actinomorphic 5.

Flowers zygomorphic 7.

(continued on next page)

13th IBO. Practical test, Laboratory I


Animal Systematics and Morphology

Tesis
Nr.
5.

Leaves entire or lobed 6.

Leaves separated. Inflorescence of compound umbels. G(2)

6.

Ca5 Co5 A5+5 G(5)

Ca2+2 Co2+2 A2+4 G(2)

7.

Leaves opposite, entire; fruit nutlet

Leaves alternate, compound; fruit legume

Code number:

Sample

Family

Nr.

code

13th IBO. Practical test, Laboratory I


Animal Systematics and Morphology

Code number:

Task 2 Plant Anatomy


Materials and instruments:
You will use the instrument set that you received upon registering for the 13th IBO!
You will also use other instruments and materials: sample No. 9, pith of black elderberry (for fixing in task 2),
microscope, stain mixture, Petri Dish with water, distilled water, microscope, slides and coverslips, razor blade,
filter paper, cloth material.

Cut the pith of black elderberry (Sambucus nigra) lengthwise in half with a razor blade. Holding with fingers,
secure sample No. 9 lengthwise between the halves (Figure 1. A-D). The black elderberry pith is used only for
fixing. Holding the black elderberry pith in one hand and the razor blade in the other, prepare freehand crosssections of sample No. 9 (Figure1E) and place them in the water in the Petri Dish! Choose the three best crosssections (without the black elderberry pith) and place them on the microscope slide. Add a drop of Astra blue
(stains cellulose) and safranin (stains lignin) mixture. After 0.5 minutes, remove the stain with the filterpaper,
and add a drop of distilled water, and remove them with the filter paper. Twice repeat rinsing with water. Add a
drop of water and place the coverslip over the cross-sections.
The quality of the cross-section and the preparation will be assessed!

13th IBO. Practical test, Laboratory I


Animal Systematics and Morphology

Code number:

Figure 1. Preparation of cross-section of sample


No. 9
A Black elderberry pith
B Cut black elderberry pith
C Sample placed between cut black elderberry
pith
D Ready for cutting
E Preparation of cross-section

Q2A. When the preparation is ready, using the low power x10 objective, find the best
cros-section. Raise the card showing 2A, and the assistant of the laboratory task will
assess the quality of the cross-section and preparation, and write the assessment points
in the answer sheet!
(2 points)
Q2B. Using the microscope (objective x10 and x40), study the preparations. Compare the cross-section seen
under the microscope with that in Figure 2. and with its parts [A (Figure 3), B (Figure 4), C (Figure 5), D
(Figure 6)].

Figure 2. Cross-section of sample No. 9.

(continued on next page)

13th IBO. Practical test, Laboratory I


Animal Systematics and Morphology

Code number:

Figure 3. Sample No. 9 cross-section part A.


A.
B.
C.
D.
E.
A.

FIGURE 4. SAMPLE NO. 9 CROSS-SECTION

PART B

F.

G.
H.
B.
Figure 5. Sample No. 9 cross-section part C

K.
L.
M.

C.

Figure 6. Sample No. 9 cross-section part D

N.

O.

D.
(continued on next page)

10

13th IBO. Practical test, Laboratory I


Animal Systematics and Morphology

Code number:

Code Table
No.

Part

No.

Part

1.

Pericycle

13.

Phloem

15.

Palisade parenchyma

4.

Hypodermis

16.

Spongy parenchyma

5.

Casparian strip

17.

Guard cell

6.

Endodermis

18.

Sclerenchyma sheath

7.

Back cavity of stoma

8.

Trichome

20.

Angular collenchyma

9.

Epidermis

21.

Front cavity of stoma

22.

Lobed parenchyma

11.

Resin duct

23.

Xylem

12.

Epithelial cells

24.

Pith

In the answer sheet beside the letters A-O of parts seen on Figures 3-6, write the codes of the correct names of
these parts!

(15 points)

Q2C. Choose the correct plant taxon observed, and enter an x beside the respective code in the answer sheet.

Code table:
A.

Bryophyta

B.

Equisetophyta

C.

Pinophyta

D.

Magnoliophyta
(1 point)

Q2D. Choose the correct ecological group of the plant observed, and enter an x beside the respective code in
the answer sheet.

Code table:

A.

hydrophyte

B.

hygrophyte

11

13th IBO. Practical test, Laboratory I


Animal Systematics and Morphology

C.

mesophyte

D.

xerophyte

Code number:

(1 point)

12

13th IBO. Practical test, Laboratory I


Animal Systematics and Morphology

Code number:

Task 3. Plant Physiology


Materials and instruments:
You will use the instrument set that you received upon registering for the 13 th IBO!
You will also use other instruments and materials: sample No.10 onion fragment, microscope, Ca(NO3)2
solution, distilled water, microscope slides and coverslips, razor blade, filter paper, cloth material.

A characteristic stem modification (bulb) fragment from a representative of the Liliaceae is supplied. Separate
from the bulb fragment one outer fleshy scale leaf, using the instrument set!

Q3A. Determine on which side of the outer fleshy scale leaf can be found the lower
epidermis. Lift the card with sign 3A, the assistant of the laboratory task will arrive,
and you will show him the lower epidermis. His assessment will be entered in the answer
page!
(1 point)

Q3B. Make a preparation: using a razor blade, shave a thin (~5 x 5 mm) piece of the lower epidermis and place
it on the microscope slide. Add one drop of the 1 M Ca(NO 3)2 solution, place a coverslip over the section, and
begin observation of the process occurring immediately under the x10 objective. Raise the card 3B, and the
assistant of the laboratory task will arrive. His assessment of the preparation quality will be entered in the answer
page!
(1 point)

13

13th IBO. Practical test, Laboratory I


Animal Systematics and Morphology

Code number:

Q3C. What is the name of the process seen under the microscope? In the answer sheet, enter an x beside the
code of the correct process.

Code table:

A.

Hemolysis

B.

Dissociation

C.

Association

D.

Plasmolysis

E.

Deplasmolysis

F.

Hemophosphorylation
(1 point)

Q3D. Which of the below concentrations of Ca(NO3)2 solution could also cause the process observed? Enter an
x beside the correct codes of the possible concentrations in the answer sheet.

Code table:
A.

5M

B.

3M

C.

2M

D.

0.2 M

E.

0. 1 M
( 3 points)

14

13th IBO. Practical test, Laboratory I


Animal Systematics and Morphology

LABORATORY III

Code number:
number of the working place

MOLECULAR BIOLOGY
Length of the practical test - 60 minutes; 40 points
Task: Electrophoretic separation of plasmid pX DNA fragments in an agarose gel and construction of a restriction map of the pX plasmid.

The lab assistants will give 5 points for strict following the lab safety regulations and accurate sample loading:
A - wearing the lab gloves during laboratory experiment 1 point,
B - addressing the assistant before usage of the power supply and correct usage of UV transilluminator
1 point,
C - proper use of pipette 1 point,
D - loading the whole amount of the sample in the well - 1 point,
E - not damaging the gel 1 point.

Note: One power supply is used by 3 - 4 students, one UV transilluminator is used by 2 students!

Please wear the gloves during laboratory experiment !

Managing of power supplies is the priority of laboratory assistants !

Technical explanation
Theory
Electrophoresis is a widely used analytical method for separation of molecules by their charge, molecular
weight and size. Frequently electrophoretical separation is performed in gel media where molecules with similar
charges are separated according to their molecular weight and size. The substance, which forms the gel, has to be
dissolved in the buffer solution.

Mapping of plasmid DNA


Plasmids are circular extrachromosomal double-stranded DNA molecules, which are
found in many bacterial species. Restriction enzymes are nucleases, which cleave DNA at the
15

13th IBO. Practical test, Laboratory I


Animal Systematics and Morphology

Code number:

sites where specific 4 6 nucleotide (base) pair (bp) sequences are found; e.g. enzyme called
HaeIII cuts the double stranded DNA at sequence (site) GGCC, but enzyme called EcoRI cuts
the double stranded DNA at sequence (site) GAATTC.
Plasmid DNA mapping is placing of the restriction enzyme cleavage sites relative to each
other on the circular scheme of the plasmid molecule. For this purpose we have to determine
the length of DNA fragments produced by cleavage of the plasmid with different restriction
enzymes. Plasmid molecules can be cut by one or by multiple restriction enzymes
simultaneously. DNA fragments produced at cutting migrate as compact bands, which can be
visualised in the gel by staining with specific dyes. The distance, which DNA fragment
migrates in the gel during electrophoresis (cm from the start point of the migration till the
front edge of the fragment band), is inversely correlated to the logarithm of the length of the
fragment as measured in bp. One of the most common gel substances for electrophoresis is
agarose. Pores of the agarose gel are large enough for separation of molecules with molecular
mass over 100 000 Da.

Equipment
Agarose gel electrophoresis tank
(4, Fig.1). contains two electrodes - cathode (5) and anode (6), respectively. Before
electrophoresis the gel is overlaid with buffer solution (7). Samples, which contain the
mixture of molecules to be analysed, are loaded in the wells (1), which are formed by special
comb during the preparation of the gel (2) on the gel support (3). Before connecting to the
power supply the electrophoresis tank is closed with a cover (8).

Fig 1. Electrophoresis tank with a gel.


Adjustable volume pipettes are used for handling of liquids (Fig.2).

Fig 2. Adjustable volume pipette.

Use of the pipette:


16

13th IBO. Practical test, Laboratory I


Animal Systematics and Morphology

Code number:

1. By turning of adjustment ring (3) and controlling the volume monitor (2) set the
appropriate volume! In this experiment you need to handle two volumes 5 l and 10 l.
Correct setting of these volumes on the monitor is shown in Fig. 3.

0
5
0

5,0 l

1
0
0

10,0 l

Fig. 3 Correct setting of 5 l and 10 l volumes on the monitor of the pipette.


2. Place the yellow tip (1, Fig.2) on the shaft of the pipette.

Do not handle liquid without a tip!

3. Press the button (4,Fig.2) smoothly to the first stop and put the tip in liquid (sample),
(Fig.4,A).
4. Slowly release the button to aspirate the sample (Fig. 4 B).
5. Take the tip with the liquid to the target (other drop of liquid or well in the gel) and press
the button until collected liquid is completely out of the tip (Fig. 4 C).

Take off
the tip with
your fingers

Fig. 4. Steps of liquid handling.

6. Take out pipette from the liquid, release the button (Fig. 4 D) and displace
the used tip in the trash, labelled as:

For each solution or sample use a fresh tip!

You can make some trial pipetting attempts with one tip and buffer solution in the tank before starting to
handle DNA samples.

Reagents and materials


1. Ready for use 0,8% agarose gel, prepared in 0,5x TAE buffer.

17

13th IBO. Practical test, Laboratory I


Animal Systematics and Morphology

Code number:

2. Electrophoresis unit, filled with 0,5x TAE buffer (20 mM Tris-acetate, 0,5 mM EDTA,
pH=8,0).
3. 2x GLB - gel loading buffer, containing 0,05% bromophenol blue in 10% glycerol.
4. St - DNA size standard, premixed with loading buffer (more detailed explanation below)
5. B+C and B+D - 5 l of plasmid pX DNA each, cleaved with restriction enzymes
B + C ; B + D, respectively (detailed explanation see in Q2 below).
To DNA samples fluorescent DNA dye Vistra Green in dilution of 1:10 000 is already added.
For all cleavages DNA of plasmid pX is used. The length of plasmid pX is 4 360 bp.

Experiment (first phase)


Sample loading
1. Load on the gel in each of the wells No.2 and No.5
(Fig. 5) 10 l of DNA size standard St.
2. Add 5 l of 2x GLB to each of cleaved plasmid
DNA samples (B+C and B+D) and load the mix
(10 l) on the gel (B+C in the well No.3 and B+D in
the well No.4, respectively).
Fig. 5. Sample loading.
3. Close the cover of the electrophoresis unit. Call the assistant by raising the hand!

Do not manage power supply; this is the priority of laboratory assistants !

Let the samples run for 20 min. Mind the time, otherwise you lose DNA fragments! Use this time to prepare the
answers to the questions below!

Questions (first set, to be answered while the gel runs)


Q1. It is known that in the electrophoresis buffer with pH 8,0 DNA molecules are migrating
from cathode to anode.
Give the answers by marking the appropriate boxes Q1 in the answer list.
- What is the charge of DNA molecules?
18

13th IBO. Practical test, Laboratory I


Animal Systematics and Morphology

Code number:

A. negative
B. neutral
C. positive
D. Impossible to determine

Which of the mentioned components is the major determinant of the charge of DNA
molecules?
E. purines
F. pyrimidines
G. deoxyriboses
H. phosphate groups
I. hydrogen bonds between the both DNA strands
J. No one of the mentioned
(2 points)

Q2. DNA fragment calculations


Given:
1. The picture of the gel showing electrophoretic separation of DNA size standards and DNA
fragments produced by cutting of the plasmid pX with the restriction enzymes A,B,C,D
(Fig. 6).
2. Size of the plasmid pX is 4 360 bp (base pairs) and each restriction enzyme (A,B,C,D),
cuts the pX DNA at one site (one time) only.
3. The restriction site of enzyme A is taken as the starting point for restriction map of this
plasmid.
4. In a combined cleavage with enzymes A and B DNA is cut in two fragments, shorter of
which is 380 bp long (see Fig. 7).
5. Length of the DNA fragments in the bands of the DNA size standard (Lane 1, Fig. 6.):
3 000; 2 000; 1 500; 1 200; 1 031; 900; 800; 700; 600; 500; 400; 300; 200; 100 (in bp)
Band of 500 bp fragments has elevated width (is darker) in respect to neighbour bands.
Bands of short DNA fragments (under 500 bp) may be weak or lost from the gel.
Estimate
Q2A. What is the size (bp) DNA fragments marked with the Roman numerals (I-VI) in the
DNA size standard Lane 1, Fig.6.? Put the answers in the appropriate cells (I-VI) of the table
Q2A in the answer list.
(3 points)
St

A+C A+D C+D St

19

13th IBO. Practical test, Laboratory I


Animal Systematics and Morphology

Code number:
Fig. 6 Electrophoretic separation of plasmid pX
cleavage fragments.
Lanes are numbered under the gel, the wells are seen
in the upper part.
Lane 1 DNA fragment length standard
Lane 2 cleavage of plasmid pX with enzymes A + C
Lane 3 cleavage of plasmid pX with enzymes A + D
Lane 4 cleavage of plasmid pX with enzymes C + D
Lane 5 DNA fragment length standard

Q2B. Plot the distance migrated by the DNA fragment length standard bands marked with
Roman numerals in Fig.6 (cm) versus the length of DNA fragments (bp) as determined in
your answer Q2A in the coordinates Q2B in your answer list. Make the graph using the
plotted points
On the X-axis - distance from the well to the front (distant) edge of the band (cm); on the Y
axis length of the DNA fragments (bp).
(4 points)
Q2C. Using the graph constructed in paragraph Q2B determine the size (bp) of DNA
fragments presented in lanes 2, 3 and 4, Fig.6. Put the answers in the columns 2, 3 and 4 of
the table Q2C in the answer list, corresponding the gel lanes 2, 3 and 4, respectively. .
(Allowed accuracy 10% of exact value).
(6 points)
Q2D. In the sample A+C (Lane 2, Fig.6) after mixing with gel loading buffer DNA
concentration was 150 ng/l (nanograms per microliter), 10 l were loaded on the gel.
How much DNA (in ng) was loaded on the gel? Put the answers in the column 1 of the
table Q2D in the answer list.
How much DNA (in ng) is contained in each of the bands in the lane 2, Fig.6 (A+C)
(assuming that all the loaded DNA is distributed between the two bands)? Put the answers in
the column 2 (for the band of the largest DNA fragment) and column 3 (for the band of the
smallest DNA fragment) of the table Question 2D in the answer list. (Allowed accuracy
10%).
(6 points)

20

13th IBO. Practical test, Laboratory I


Animal Systematics and Morphology

Code number:

If you still have time until the gel is ready, you


can start considering the possible location of
cutting sites for the restriction enzymes C and D in
the plasmid map (Fig. 7). Use the results from the
answer to Q2C!

Fig. 7
Experiment (second phase)
20 minutes after the beginning of electrophoresis the assistant will disconnect your
electrophoresis tank from the power supply. Do not hesitate to remind the time to the
lab assistant!
Thereafter:
1. Place the gel together with the gel support in a tray and take them to the UV
transilluminator, the number of which is indicated at your working place.
2. Pick up the protecting shield of the transilluminator.
3. Place the gel on the UV table.
4. Close the protecting shield and switch on UV light.

Do not switch on the UV transilluminator, while the protecting shield is


open !
Do not lift the protecting shield while the UV light is on !

!
!

5. Observe the image of DNA bands and draw the pattern of bands in the frame given in the
answer list Q3. DNA bands in your picture must be positioned relative to the DNA size
standard precisely as in your gel
(4 points)
6. Switch off the UV light and leave the gel together with gel support at the transilluminator.
7. Clean the hands with paper towel and continue preparing your answers.
21

13th IBO. Practical test, Laboratory I


Animal Systematics and Morphology

Code number:

Questions (second set, to be answered after the recording the fragment


separation in the gel)
Q4A. Determine the approximate fragment length of cut DNA, comparing the position of the
bands of samples with the bands of DNA size standard. Put the answers in the answer list;
table Q4A, columns 3 and 4, corresponding the lanes 3 and 4, respectively. (Allowed
accuracy 20%).
(4 points)
Q4B. Considering the analysis of the gel depicted in Fig.6 and the data obtained from your
own gel, determine the approximate positions of the cutting sites of the enzymes C and D at
the plasmid map (Q4B, answer list) by writing the letters (C, D) in appropriate boxes.
(6 points)

22

13th IBO. Practical test, Laboratory I


Animal Systematics and Morphology

Code number:

Laboratory IV
Dendroecology: Growth of aspen (Populus tremula) invading a clear cut (previously spruce) by
seed.

Length of the practical test 60 minutes; 40 points

Materials and instruments


You will need the calculator from the instrument set that you received upon registering for the 13 th IBO!

Other materials:
1.

A data sheet. The data sheet is identical to the answer sheet with tables, only with some of the columns
(Table 1) and rows (Table 2) filled in. THIS DATA SHOULD BE COPIED ONTO THE ANSWER
SHEETS. DO NOT WRITE ON THE DATA SHEETS.

2.

10 labelled discs of aspen. DO NOT MAKE ANY MARKS ON THE DISCS

3.

one measuring tape

4.

one ruler

5.

one magnifying glass

23

13th IBO. Practical test, Laboratory I


Animal Systematics and Morphology

Code number:

Introduction
Aspen has invaded an open area created by a clear cut harvest of a spruce forest. Not all of the trees established
in the same year, but over a 8-year period. Now, 18 years after the start of development of the new forest stand,
the vertical and radial growth of aspen will be investigated by retrospective analysis. Tree rings can be used to
measure the tree stem diameter at different tree ages.

Q1. In the following photograph (Figure 1) of a cross-section of an aspen stem, identify the following, by writing
the respective one correct number from the Figure in the answer sheet!

Code Table
A.

bark

B.

pith

C.

latewood

D.

earlywood

E.

meristem

F.

annual ring

G.

branch

H.

vascular cambium

I.

phloem cells

J.

xylem cells

K.

cork cells
(11 points)

24

13th IBO. Practical test, Laboratory I


Animal Systematics and Morphology

Code number:

25

13th IBO. Practical test, Laboratory I


Animal Systematics and Morphology

Code number:

The growth of aspen stems in the area will be used to determine response to possible intraspecific competition. It
is assumed that the effect of competition increases with time, because tree density increases and the trees
increase in size.

Q2. For a tree species, not necessarily aspen, what are the possible effects of intraspecific competition for light
resources:
1. increased mortality rate over time,
2. decreased yearly growth increment in height of stems
3. increased variability among stems of yearly growth increment in height
4. decreased yearly growth increment in diameter of stems
5. increased variability among stems in yearly growth increment in diameter
6. accelerated yearly growth increment in height of stems in response to shading by neighbours

Which of the following is the correct combination of correct answers? Enter your choice in the answer sheet!
A. 1, 2, 4
B. 1, 3, 5
C.

1, 2, 4, 6

D. all of the above


E.

1, 2, 3, 4, 5
(1 point)

The purpose of this practical work is to determine whether radial and/or vertical growth of aspen is related to
time of invasion of aspen individuals. In other words, since the time of invasion can be determined by estimating
the age of stems, does the growth depend on the age of the individual?

Methods and Results


Within a forest plot (20mX20m), all trees growing were cut at tree base, labelled and stem height was measured.
Each student is supplied with 10 segments cut at stem base, which were randomly sampled in the plots. Each

26

13th IBO. Practical test, Laboratory I


Animal Systematics and Morphology

Code number:

stem is labelled, and the height of these stems in metres is provided beside the appropriate tree ring code in
Table 1 of the data sheets. Copy the codes and the heights onto the answer sheets.

T1. Fill in the Table 1!

Count the number of tree rings of each segment, and enter this data in Table 1 of the
answer sheets!

(2 points)

NOTE: Since determination of the first few tree rings is difficult due to rot, the first five tree
rings developed have been determined under a dissecting microscope, and the position
marking the end of five years is marked on each stem. Therefore, you need only to count the
rings developed after this mark, and then add 5 years.

As none of the cut segments are perfectly round (the radial growth differs depending on compass direction),
the mean diameter will be estimated by calculation from the perimeter. Using the cloth tape provided,
measure the perimeter of each stem, and enter this data in Table 1 of the answer sheet!
(2 points)

Using the appropriate formula, use the perimeter measurements to calculate the mean diameter, and enter
this data in Table 1 of the answer sheet!

(1 point)

Using the ruler provided, measure the total width of the first five tree rings (from the centre to the mark on
the segment) and enter this data in Table 1 of the answer sheet!
(1 point)

Using the ruler provided, measure the last five tree rings (at the location of the maximum diameter (marked
by the line), and enter this data in Table 1 of the answer sheet!
(1 point)

G1. Using the graph paper supplied in the answer sheet, produce scatter plot for Graph 1. - Stem age (a) versus
height (h)

(2 points)

27

13th IBO. Practical test, Laboratory I


Animal Systematics and Morphology

Code number:

G2. Using the graph paper supplied in the answer sheet, produce scatter plot for Graph 2. - Stem tree age (a)
versus diameter (d)

(2 points)

G3. Using the graph paper supplied in the answer sheet, produce scatter plot for Graph 3. - Stem age (a) versus
width of first produced five tree rings

(f5)

(2 points)

G4. Using the graph paper supplied in the answer sheet, produce scatter plot for Graph 4. - Stem age (a) versus
width of the last five produced tree rings (l5).

(2 points)

Remember to label the axes with the appropriate codes (a, d, h, f5, l5) and scales.

G5. As the graph of stem age versus height appears to indicate a linear relationship, calculate the linear
regression equation (best-fit line through the points) using the Tables provided (Table 1 and Table 2) and draw
the calculated best-fit line on the appropriate graph
(1 point)

NOTE: In the data sheet supplied, you have been given the correct values of the sum of squared ages
(Xi2) and the value of the sum of age times height (Xi*Yi). Therefore, you do not need to calculate these.
ENTER THESE FROM THE DATA SHEET ONTO THE ANSWER SHEET.

T2. Completely fill in Table 2 on the answer sheets

(8 points)

Check that Table 1 is completely filled in.

28

13th IBO. Practical test, Laboratory I


Animal Systematics and Morphology

Code number:

Discussion
Q3. The Graphs that you have produced suggest that intraspecific competition has resulted in:
1. reduced height of shaded individuals
2. reduced diameter of shaded individuals
3. reduced stem structural support of shaded individuals
4. increased variability in height of individuals of the same age
5. Increased variability in diameter of individuals of the same age

Which of the following is the correct combination of correct answers?


A.

1, 2, 3, 4, and 5

B.

1, 2 and 3

C.

2, 3 and 5

D.

2 and 4

E.

4 and 5
(1 point)

29

13th IBO. Practical test, Laboratory I


Animal Systematics and Morphology

Code number:

Q4. Which of the following comments are suggested by the Graphs that you have produced?

1. The reduced growth of individuals invading the clear cut later suggests that aspen has a stress-tolerant growth
strategy.
2. The Graphs suggest a certain competitive ability, because late-coming stems are able to maintain the same
growth rate in height as the stems which arrived earlier, indicated by the linear relationship between age and
height.
3. In combination, the Graphs suggest that there will be potentially increased mortality of the individuals
invading the stand later.
4. The Graph 4 suggests that the growth strategy of aspen appears to be a ruderal strategy: rapid growth early in
succession, taking advantage of available resources

Which of the following is the correct combination of correct answers?


A. 1 and 3
B. 2 and 4
C.

2, 3 and 4

D.

4
(1 point)

30

13th IBO. Practical test, Laboratory I


Animal Systematics and Morphology

Code number:

Q5. Which of the following comments are suggested by Graphs 3 and 4?

1. The Graphs show that the effect of competition increases with time after clearcut.
2. Only the trees arriving on the site during the first few years (but not necessarily all of these individuals) have
been able to support a high radial growth rate during the last few years .
3. Differences in the amount of shading in the occupied patches may be a reason for the high variability in
Graph 3.
4. Graphs 3 and 4 probably reflect linear relationships that are hidden by high variability.

Which of the following is the correct combination of correct answers?

A. 1, 2 and 3
B. 4
C. 1, 2, 3, and 4
D. 1 and 3
E. 2 and 4
(1 point)

31

13th IBO. Practical test, Laboratory I


Animal Systematics and Morphology

Code number:

Q6. Which of the following problems clearly need to be considered, as they can affect the results shown in the
Graphs?

1. The annual growth increment in tree ring width is related to the current stem diameter. For this reason,
calculation of the relative growth rate (RGR) would have less bias.
2. Differences in the growth of the different stems should also be assessed in relation to differences in
meteorological conditions during the years after the clear cut.
3. Biotic factors such as herbivory (insects, deer, moose which do occur in the area) and disease may have
caused death or damage to a particular age class of stems, biasing the results.
4. The sample size is too low for conclusive results.

Which of the following is the correct combination of correct answers?


A. None of the comments is true
B. All of the comments are true
C. Only 1 and 4 are true
D. Only 2, 3, and 4 are true
E. Only 2 and 3 are true
(1 point)

32

INTERNATIONAL BIOLOGY OLYMPIAD


THEORY PROBLEMS

2001, Brussels, Belgium















All IBO examination questions are published under the following Creative Commons license:



CC BY-NC-SA (Attribution-NonCommercial-ShareAlike) https://creativecommons.org/licenses/by-nc-sa/4.0/
The exam papers can be used freely for educational purposes as long as IBO is credited and
new creations are licensed under identical terms. No commercial use is allowed.

12th International Biology Olympiad


Theoretical Test - Part A
Cell Biology
A 1. The genetic code is:
A.
B.
C.
D.

the set of cellular genes.


the nucleotidic sequence of the gene.
the genetic expression.
the law of correspondence between ribonucleotide sequences
and amino acids.

A 2. During differentiation:
A.
B.
C.
D.

cells loose a substantial part of their genetic information.


each cell of a multicellular organism expresses only one gene at
a time.
different strains of procaryotes can be produced.
only part of the genes of a given cell are active at the same time.

A 3. During meiosis:
A.
B.
C.
D.

tetrads are formed at metaphase II.


crossing over occur at prophase I.
homologous chromosomes are pairing during prophase II.
sister chromatids are separated at anaphase I.

A 4. A given biochemical reaction:


A.
B.
C.
D.

A 5. The study of crossing over frequency between linked genes has been
used:
A.
B.
C.
D.

to estimate the efficiency of chromosome separation at anaphase.


to estimate the physical distance between genes.
to establish the genetic map of the chromosome.
to estimate the frequency of a mutation.

- 77 -

Belgium, July 8 - 15, 2001

always looses energy.


always receives energy.
transmits energy to any other reaction or receives energy from
any other reaction.
transmits energy to another specific reaction or receives energy
from another specific reaction.

12th International Biology Olympiad


A 6. Thinking about cellular respiration, which of the following statements is
correct?
Dinitrophenol (DNP) rapidly kills a cell because:
A.
B.
C.
D.

it destroys and paralyses all the cell proteins.


it forbids the access of the oxygen to the cell.
it forbids the transport of cellular molecules transporting energy.
it increases the oxygen consumption of the cell.

A 7. Under how many forms can a 2000 nucleotides gene exist?


A.
B.
C.
D.

1.
2000.
700.
a practically illimited number.

A 8. ATP is an important molecule in metabolism because:


A.
B.
C.
D.

it has high-energy phosphate bonds.


its phosphate bonds are very easily formed but not easily broken.
it is readily obtained from an organisms environment.
it is extremely stable.

A 9. Which cell type would probably provide the best opportunity to study
lysosomes?
A.
B.
C.
D.

muscle cell.
nerve cell.
phagocytic white blood cell.
leaf cell of a plant.

Belgium, July 8 - 15, 2001

A 10. Large numbers of ribosomes are observed in cells that specialise in


the production of:
A.
B.
C.
D.

lipids.
polysaccharides.
proteins.
glucose.

A 11. A biologist ground up a piece of plant tissue and then centrifuged the
mixture. She obtained some organelles from the sediment in the test tube.
The organelles took up CO2 and gave off O2. The organelles were most
likely:

- 78 -

12th International Biology Olympiad


A.
B.
C.
D.

chloroplasts.
ribosomes.
nuclei.
mitochondria.

A 12. Which one of the following cell structures doesnt contain nucleic
acid:
A.
B.
C.
D.

mitochondria.
nucleus.
rough endoplasmic reticulum.
smooth endoplasmic reticulum.

A 13. Which is the correct series of histones forming the core of a nucleosome?
A.
B.
C.
D.

H1 , H3 and H4.
H2A , H2B ,H3 , H4.
H1 , H2A and H2B.
H1 , H2A , H2B and H4.

A 14. How does cyanide (CN-) cause the death of living things?
A.
B.
C.
D.

by stopping photosynthesis.
by breaking down protein molecules.
by stopping (-oxidation).
by stopping the electron flow through ETS enzymes.

A 15. Which of the following phenomena is responsible for arteriosclerosis?


A.
B.
C.
D.

non modification of LDL receptors.


joining of LDL complex and LDL receptors.
shape modification of the active site of LDL complex.
high concentration of cholesterol.

I- During electron transport, a proton gradient develops across the inner


membrane of mitochondria.
II- A closed membrane or vesicular structure is required for oxidative phosphorylation.
III- ATP synthesis starts when a proton gradient develops in mitochondria.

- 79 -

Belgium, July 8 - 15, 2001

A 16. Which of the following experimental observation(s) support(s) the


chemiosmotic theory about oxidative phosphorylation?

12th International Biology Olympiad


A.
B.
C.
D.

only II.
only III.
only I and II.
I, II and III.

A 17. The oxygen atoms used to form H2O at the end of the oxidative
phosphorylation chain (also named: terminal oxidation), come from:
A.
B.
C.
D.

carbon dioxide.
glucose.
molecular oxygen.
pyruvate.

A 18. Pyruvate is the end product of glycolysis. Consequently, which of the


statements below is true?
A.
B.
C.
D.

there is more energy in 6 molecules of carbon dioxide than in


two molecules of pyruvate.
there is less energy in two molecules of pyruvate than in one
molecule of glucose.
pyruvate is a more oxidized state than carbon dioxide.
there is more energy in 6 molecules of carbon dioxide than in
one molecule of glucose.

A 19. During aerobic respiration, electrons travel downhill from


A.
B.
C.
D.

food Krebs cycle ATP NAD+.


food NADH electron transport chain oxygen.
glucose ATP oxygen.
food glycolysisKrebs cycle NADH ATP.

Belgium, July 8 - 15, 2001

A 20. You eat a nice portion of Belgian fries (fried potatoes) with chicken
meat and a fresh salad. Which of the following molecules of this meal would
normally not be oxidized in aerobic respiration to generate ATP?
A.
B.
C.
D.

polysaccharides.
proteins.
nucleic acids.
lipids.

A 21. In plant cells, ATP is produced in response to light. The electron


transport chain that is involved in the process is located in the:
A.
B.

thylakoid membranes of chloroplasts.


stroma of chloroplasts.

- 80 -

12th International Biology Olympiad


C.
D.

inner membrane of mitochondria.


cytoplasm.

A 22. The substance that is the general biosynthetic precursor of sex hormones and hormones of the adrenal cortex is:
A.
B.
C.
D.

inositol.
lecithin.
phosphatidyl-choline.
cholesterol.

A 23. During replication, the excision of the primers:


A.
B.
C.
D.

is made by a DNAse.
produces OKASAKI fragments.
occurs only in the lagging strands.
occurs in the nucleus.

A 24. The following substances are proteins of the cytoskeleton:


A.
B.
C.
D.

actin, actinomycin, myosin.


desmin, tubulin, dynein.
tubulin, desmin, vinblastin.
cytochalasin, actin, myosin.

A 25. Two bottles of liquid were labelled X and Y. The two bottles contained different 0,2 molar (hypotonic) solutions. Red blood cells in a drop
of blood were placed in each bottle. The cells in X swelled and some of
them burst while those en Y shrunk and became shrivelled. On the basis of
this information which of the following conclusions is most valid?
A.
B.

D.

A 26. The diagrams below refer to variation in the amount of DNA (y axis)
as a function of time (x axis) during cell division. (the units are arbitrary).
Which ones of these diagrams depict what happens during meiosis and
mitosis, respectively?

- 81 -

Belgium, July 8 - 15, 2001

C.

the concentration of the solute in liquid X was higher than in the


red blood cells
the concentration of the solute in liquid Y was higher than in the
red blood cells.
the ionization of the solute was responsible for the shrinking of
the red blood corpuscles in liquid Y.
the diffusion of solute into the red blood corpuscles was respon
sible for the shrinking of the red blood corpuscles in liquid Y.

12th International Biology Olympiad

A.
B.
C.
D.

1 and 2.
4 and 1.
3 and 4.
2 and 3.

A 27. The following short DNA sequence 5' AGGATGCTA 3' can be perfectly hybridized with:
A.
B.
C.
D.

5' AGGATGCTA 3'.


5 UGGUACGAU 3'.
5 ATCGTAGGA 3'.
5' TAGCATCCT 3'.

A 28. The following sequence in registered in a genomic data bank as part


of a coding locus in a genome:
5'.....AGGAGGTAGCACCTTTATGGGGAATGCATTAAACA.......3'.
The ATG underlined represents the initiation codon of the gene located at
that locus. Among the following sequences, which one could be part of the
transcribed mRNA corresponding to that locus?
A.
B.
C.
D.

5' AGGAGGUAGCACCUUUAUGGGGAAUGCAUUAAACA 3'.


5' UCCUCCAUCGUGGAAAUACCCCUUACGUAAUUUGU 3'.
5' ACAAAUUACGUAAGGGGUAUUUCCACGAUGGAGGA 3'.
5' UGUUUAAUGCAUUCCCCAUAAAGGUGCUACCUCCU 3'.

Belgium, July 8 - 15, 2001

A 29. In eucaryotic cells the sequence of organelles involved in the synthesis and secretion of a protein, is:
A.
B.
C.
D.

ribosome, endoplasmic reticulum, Golgi, cell membrane.


ribosome, Golgi, endoplasmic reticulum, cell membrane.
mitochondrion, ribosome, endoplasmic reticulum, cell membrane.
nucleus, mitochonrion, ribosome, Golgi, cell membrane.

A 30. If you extract the DNA of the bacteriophage (X174, you will find that
its composition is 25 % A, 33 % T, 24 % G, and 18 % C. How would you
interpret these results?

- 82 -

12th International Biology Olympiad


A.
B.

C.

D.

the experiments results must be erroneous; something went


wrong.
we could admit that the A percentage approximately equals that
of T, and the same for C and G. Consequently, Chargaffs rules
are not eluted, DNA is double stranded and replicates semi-conservative.
as the A and T, respectively C and G percentages are different,
DNA is single - stranded; it is replicated by special enzymes, following a particular replication pattern, with single - stranded chain
as a template.
because A does not equal T, nor does G equal C, the DNA must
be single - stranded; it replicates by synthetising a complementary strand and uses this double stranded form as a template.

A 31. DELETED
A 32. In the nucleus of a eukaryotic cell
A.
B.
C.
D.

more than 50 % of DNA codes for proteins.


all the proteins are histones.
transcription of DNA takes place only in the heterochromatin.
nucleolar DNA codes for the synthesis of ribosomal RNA.

A 33. During protein synthesis,


A.
B.
C.
D.

aminoacyl tRNA synthetase is involved in the synthesis of the


amino-acids.
the C-C-A end of tRNAs is used to attach the transfer RNAs to
the mRNA.
each new amino acid added to the system locks first in the A site
of the large ribosomal subunit.
peptidyltransferase moves the newly formed peptide from site A
to site P.

A.
B.
C.
D.

targets the HIV envelope proteins which prevents CD4 binding.


inhibits DNA replication of HIV.
binds to metabolic enzymes of the virus.
interferes with the protease activity of HIV.

- 83 -

Belgium, July 8 - 15, 2001

A 34. The drug, AZT, is given to AIDS patients to slow down the progress of
the disease. The structure of the drug is given here. The drug is effective as
it:

12th International Biology Olympiad

A 35. If the steps by which a virion is replicated are designated as follows:


1.
2.
3.
4.
5.
6.

Synthesis of viral protein


Fusion of virion envelope with cell membrane
Assembly of proteins
Removal of capsid
Release of virus from cell
Replication of viral RNA,

Which of the following sequences is the correct one?


4-2-1-6-3-5.
6-4-1-3-5-2.
2-6-4-5-1-3.
2-4-6-1-3-5.

Belgium, July 8 - 15, 2001

A.
B.
C.
D.

- 84 -

12th International Biology Olympiad


Plant anatomy and physiology

A 36. Graph demonstrates the amount of evaporated water during the year
in two equally high trees growing in the same habitat in Belgium. To which
tree species belong the two given curves in the graph?

A.
B.
C.
D.

a) pine (Pinus), b) spruce (Picea).


a) oak (Quercus), b) birch (Betula).
a) pine (Pinus), b) oak (Quercus).
a) oak (Quercus), b) spruce (Picea).

A 37. What is the number of chromosomes of the primary endosperm in the


seed of Pinus?
A.
B.
C.
D.

haploid.
diploid.
triploid.
polyploid.

A 38. DELETED

A 40. The changes (colour, texture and chemical composition) that take
place in the fruits when they ripen are due to:
A.
B.
C.
D.

the CO2 content in the atmosphere.


the temperature variation.
the ethylen synthesis in the plant.
the indolacetic concentration in the fruit.

- 85 -

Belgium, July 8 - 15, 2001

A 39. DELETED

12th International Biology Olympiad


A 41. Pinus pollen is moved from male to female cones by:
A.
B.
C.
D.

wind.
water.
insects.
birds.

A 42. Which of the following statements about fungi is true?


A.
B.
C.
D.

ascospores are formed asexually.


gills of mushrooms are diploid.
mycelia of nearly all fungi are predominantly dikaryotic.
in the dikaryotic condition two unfused nuclei lie within the same
cell.

A 43. How have bryophytes managed to survive on land?


A.
B.
C.
D.

they were the first plants that developed stomata.


they do not require moist environments for their reproductive
cycles.
they grow close to the ground in relatively moist regions.
the sporophyte became independent of the gametophyte.

A 44. The gemmae of Marchantia are homologous to:


A.
B.
C.
D.

seeds.
gametes.
asexual cells.
pollen grains.

Belgium, July 8 - 15, 2001

A 45. Which of the following is not an essential mineral element for plants?
A.
B.
C.
D.

potassium.
magnesium.
calcium.
lead.

A 46. Nitrate reduction:


A.
B.
C.
D.

is performed by plants.
takes place in mitochondria.
is catalyzed by the enzyme nitrogenase.
is known as the nitrogen fixation process.

- 86 -

12th International Biology Olympiad


A 47. In C4 plant photosynthesis
A.
B.
C.
D.

3 PG ( phosphoglyceraldehyde) is the first product of CO2 fixation.


four-carbon acid are formed by PEP (phosphoenolpyruvate) carboxylase in the bundle sheath.
continues at lower CO2 level than in C3 plants.
CO2 released from RuDP (ribulose diphosphate) is transferred to
PEP.

A 48. The diagram below shows a cut across a portion of a green non
woody stem with a vascular bundle. It may be reasonably concluded that:

A.
B.
C.
D.

starch would be found in the cells of area III.


area II differentiated from protoderm.
area IV was stained red of safranin.
area V was interfasicular cambium.

Belgium, July 8 - 15, 2001

- 87 -

12th International Biology Olympiad


A 49. The diagram below is a longitudinal section of a flowering plant.
What is the type of pollination?

A.
B.
C.
D.

self-pollination by pollen from staminate flower to pistillate flower


of the same flower.
transfer of pollen from staminate flower to pistillate flower of other
flowers on the same plant.
wind-pollination.
insect-pollination.

Belgium, July 8 - 15, 2001

A 50. Two similar plants of a species of angiosperm grew under different


amount of sunlight.
Plant I received full sunlight. Plant II received only seven percent sunlight.
Both plants were grown in the same type of soil, and received equal amounts
of water and the necessary mineral ions. At the end of the experiment,
transverse sections were prepared from the leaves of each plant and examined under a microscope. The features seen in a typical section for each
plant are illustrated below.
It is reasonable to suggest that:
A.
B.
C.
D.

the rate of photosynthesis of plant I is less than the rate of respiration for plant II.
the rate of respiration of plant I is less than the rate of respiration
for plant II.
the rate of photosynthesis of plant II is equal to the rate of respiration.
the high concentration of light intensity decreases cellular respiration of plant I.

- 88 -

12th International Biology Olympiad


A 51. DELETED
A 52. The difference between cyanobacteria (blue-green algae) and the
other algae groups (red, brown and green algae) is marked by the fact that:
A.
B.
C.
D.

cyanobacteria are only found in marine habitats.


cyanobacteria always have flagella.
cyanobacteria never are diploid.
cyanobacteria always use H2S for their photosynthesis.

A 53. Diatoms (Bacillariophyta) can reproduce asexually through the process of template formation of silica-plate boxes (epitheca - hypotheca). As
a result, most of the newly formed boxes will decrease in size and ultimately lead to unviable small cell sizes. This problem is avoided:
A.
B.
C.
D.

through the fact that during sexual reproduction the zygote will
increase in size before formation of new silica plates.
through conjugation (fusion of the content of two small cells into
one large cell).
through the fusion of two small silica plates into one larger silica
plate.
through the fusion of four small silica plates into one larger silica
plate.

A 54. Where does the embryo in flowering plants gets its energy from?
A.
B.
C.
D.

the haploid endosperm.


the tapetum layer.
the light that penetrates through the seed coat.
the albumen (triploid endosperm).

Belgium, July 8 - 15, 2001

- 89 -

12th International Biology Olympiad


Animal anatomy and physiology

A 55. In fishes, auditory sensitivity rests on groups of ciliated cells of the


labyrinth wall called neuromasts and attached to a heavy mass (CaCO3
grains or a sort of stone, called otoliths). Neuromasts register movements
of the otoliths relative to the labyrinth wall. Underwater sounds are transmitted in the form of pressure waves that do not result in important water
molecules displacements. Selachian Sandry otoliths are less efficient than
Teleostean ones, which consist in little stones. Choose the group of fishes
provided with the best auditory device:
A.
B.
C.
D.

no particular group.
teleosts which possess a gas bladder.
fishes swimming near the surface of seas and lakes.
skates.

A 56. Here is a diagram of the mammalian heart. Through which of the


following does oxygenated blood enter the heart?

Belgium, July 8 - 15, 2001

A.
B.
C.
D.

1.
2.
3.
10.

A 57. Which of the following statements is correct?


A.
B.
C.
D.

all veins carry blood flowing to the heart.


all veins carry oxygenated blood.
all veins carry deoxygenated blood.
arteries are larger than their corresponding veins.

- 90 -

12th International Biology Olympiad


A 58. The taste quality detected by taste buds located on the posterior
third of the tongue is:
A.
B.
C.
D.

sweet.
acid.
hot.
bitter.

A 59. Erythrocytes of man with blood group A are mixed with blood plasma
of another man. No agglutination is observed. It can be concluded that the
blood group of the plasma donor is:
A.
B.
C.
D.

only B.
A or 0.
A or AB.
A, B or 0.

A 60. The use of stimulant drugs, as cocaine or amphetamine, evokes,


among other effects, a state of intense nervous excitation. Which one of
the following sentences related to these drugs could not explain their effect?
A.
B.
C.
D.

their structure is such that they link to the CNS dopamin recep
tors.
they are less efficiently metabolized by the organism, than the
neurotransmitter they mimic.
they contain an amino group, just as dopamine and adrenaline.
their affinity for dopamine receptors is superior to that of the natur
al neurotransmitter.

A 61. To elevate the level of circulating lactate dehydrogenase (an enzyme) in a rabbit, a scientist injects a solution of sodium lactate. One hour
later he measures the actual lactate dehydrogenase activity in the muscles.
Which physiological mechanism does he challenge in this experiment?
positive feed-back.
negative feed-back.
detoxification.
gene induction.

A 62. Essential oils from such aromatic plants as nutmeg contain large
amounts of aromatic hydrocarbons which, upon simple addition of an amino

- 91 -

Belgium, July 8 - 15, 2001

A.
B.
C.
D.

12th International Biology Olympiad


group, yield amphetamine derivates with hallucinogenic properties. This
reaction has been accomplished in vitro with cellular homogenates converting a fraction of these essential oils to sympathomimetic compounds.
The ingestion of large quantities of powdered nutmeg leads to an intoxication state reminding the amphetamines effects, indicating that some conversion also occurs in vivo.
a) Which of the following enzymatic systems could be responsible for this?
A.
B.
C.
D.

oxydoreductases.
hydrolases.
transaminases.
kinases.

b) Where does this enzymatic activity mainly take place?


A.
B.
C.
D.

in the liver.
in the brain.
in the lung.
in the kidney.

A 63. In fish, inhibition of the ejection of the second polar body can be
achieved by the application of a physical shock (pressure, temperature) at
a precisely determined moment after fertilisation. The resulting embryos
have two sets of DNA from the mother, and one from the father, and thus
are triploid.
a). These individuals are sterile because,
A.
B.
C.

Belgium, July 8 - 15, 2001

D.

they have an odd chromosome number.


their sexual organs grow abnormally.
the spermatozoa are too big, and cant pass through the micropyle.
triploid individuals have an abnormal behaviour, and their aggressiveness doesnt allow the necessary proximity during mating.

b). Which mechanism does explain the effect (non-ejection) of the


physi cal shock on the second polar body?
A.
B.
C.
D.

cytoskeleton desorganisation.
the denaturing of some enzymatic systems.
plasmic membrane rigidification.
disintegration of the polar body inside the oocyte cytoplasm.

- 92 -

12th International Biology Olympiad


A 64. The graph below represents the contraction patterns for three different kinds of muscles. Please match the correct sequence of patterns for
smooth muscle, skeletal muscle and cardiac muscle respectively:
A.

BAC.

B.
C.
D.

CBA.
ABC.
ACB.

A 65. In an experiment designed to find what proportion of cabbage leaf


material eaten by a caterpillar was converted into caterpillar biomass, it
was observed that the caterpillar ate 2 cm2 of leaf per day. In order to make
an estimate of the conversion the following measurements were taken:
W.
X.
Y.
Z.

average dry weight / cm2 of leaf similar to that eaten.


total weight of caterpillar faeces per day.
dry weight of caterpillar faeces per day.
weight of carbon dioxide produced per day.

Which of the following formulae would be the appropriate one to estimate


the weight of cabbage leaf converted into caterpillar biomass?
A.
B.
C.
D.

W X Z.
W Y Z.
2W X carbon content of Z.
W + X Y.
Belgium, July 8 - 15, 2001

- 93 -

12th International Biology Olympiad


A 66. The amount of dissolved oxygen in water changes when its temperature increases. The amount of haemoglobin in body liquids of aquatic
vertebrates depends therefore also on the temperature of water in which
animals live. Which of the curves of the graph describes these changes
best?
A.
B.
C.
D.

curve a.
curve b.
curve c.
curve d.

A 67. Oxygen release from hemoglobin is caused and enhanced by:


A.
B.
C.
D.

low pO2, low pH and low temperature in the tissues.


high pO2, high pH and high temperature in the tissues.
high pO2, low pH and low temperature in the tissues.
low pO2, low pH and high temperature in the tissues.

A 68. When a person suffers severe trauma, such as a broken leg, which
hormone mobilizes aminoacids, sugar and fats to be used in the long-term
stress response?
A.
B.
C.
D.

acetylcholine.
cortisol.
aldosterone.
adrenalin.

Belgium, July 8 - 15, 2001

A 69. DELETED
A 70. Of the following anatomical structures, which one is homologous to
the wing of a bat?
A.
B.
C.
D.

the dorsal fin of a shark.


the arm of a human.
the pelvic fin of a fish.
the wing of a butterfly.

- 94 -

12th International Biology Olympiad


A 71. The plant-louse of oak trees is a small insect, stinging with its mouth
parts into young branches and living on the sucked up liquid. How far do
these lice sting into the branches?
A.
B.
C.
D.

into vessel areas outside from the cambium.


into the cambium.
they sting into the different areas depending on the trees age.
the area depends on the louses age and stage of development.

A 72. Which of these statements about reproduction in invertebrates is


NOT correct?
A.
B.
C.
D.

many invertebrates have separated sexes.


many invertebrates utilise external fertilisation.
some invertebrates have structures that store sperm.
invertebrates do not engage in copulation.

- 95 -

Belgium, July 8 - 15, 2001

Co-ordinators meeting

12th International Biology Olympiad


Ethology
A 73. When a worker bee stings to defend itself, its sting often remains
blocked in the enemys flesh, which in turn causes bees death. Assuming that defence is suicidal for a bee, how could you explain the persistence of such behaviour?
A.
B.
C.
D.

honey bees are naturally aggressive.


sometimes, the sting doesnt remain stuck in the opponents flesh,
and the bee can reuse its weapon.
due to altruism.
because the bees which are more exposed to fighting enemies
outside the hive are also the oldest ones and are thus near the
end of their life expectancy.

A 74. Why are mammals often polygamous, whereas birds are frequently
monogamous?
A.
B.
C.
D.

females available for mating are more frequent in mammals.


mammals, and not birds, display in leks.
external fertilisation in birds requires the presence of the male to
avoid new-laid eggs to be fertilised by competitors.
breast-feeding by the females in mammals, but not in birds, makes
males investment in brood care less necessary.

Belgium, July 8 - 15, 2001

A 75. P. Marler has carried out many experiments to determine the role of
the genes (inheritance) and the environment (learning) in song acquisition
in males of the white-crowned sparrow Zonotrychia leucophrys. His results
are shown in the following illustration:

- 96 -

12th International Biology Olympiad


1.

2.
3.
4.

A.
B.
C.

D.

normal course of development : the bird hears its species song


while young (first 50 days), and initially produces a subsong,
which crystallises into the final adult song characteristic of the
species.
the juvenile bird is isolated from the sound of all songs.
the juvenile bird is allowed to hear normal songs during the first
50 days after birth, but is then deafened.
the juvenile bird is isolated from the sound of all songs during
the first 50 days. It then receive songs of its own species.
What can you conclude from these experiments?
hearing the song of its own species is sufficient to develop a
normal song.
young adults must hear the song of an adult male and need to
train themselves to reproduce it correctly.
chicks must hear the song of an adult male and need to train
themselves before developing a normal song ; learning is only
possible if chicks hear the song during a critical period of 50 days
following birth.
singing does not require any learning and develops according to
a maturation process.

A 76. As shown below, as soon as a domestic hen chick hatches it starts


pecking at grains that look like food, and as it grows older its aim at food
grain improves. Note that if a chick is prevented from pecking at food during its second day, it will still be better at pecking on its third than on its first
day ; however, it will not be as accurate as it would have been if it had been
allowed to practice. What are the necessary mechanisms for the development of an accurate pecking in chicks?

Belgium, July 8 - 15, 2001

- 97 -

12th International Biology Olympiad


A.
B
C.
D.

accurate pecking develops following maturation of the nervous


system.
accurate pecking develops by learning allowing the chicks to dis tinguish between alimentary items.
both processes of maturation and learning are involved in accurate pecking.
there is a critical period - ranging from day 1 to day 7 - during
which the chicks learn how to peck food on the ground.

A 77. Passerine birds may produce different types of sounds and alarm
calls. The songs of males birds differ between species. Basically, they have
two main functions : defending the territory and attracting/stimulating females to mate. Alarm calls are emitted when birds spot a dangerous predator flying over. Alarm calls when a hawk flies over stimulate other nearby
birds to take action to escape. Whereas the songs of male birds are different between species, alarm calls of several species sound similar i.e., they
share certain common acoustic properties. How do you explain this crossspecies similarity of alarm calls?
A.
B.
C.
D.

there is a convergent evolution of alarm calls because their acoustic properties make the call difficult to locate.
there is a convergent evolution of alarm calls because their acoustic properties allow the caller to locate precisely the predator flying over.
there is a convergent evolution of alarm calls with simple acoustic properties, because simple alarm calls are easily and rapidly
learned by young birds.
predators escape when hearing long calls, because such calls
are also emitted by their own predators.

Belgium, July 8 - 15, 2001

A 78. DELETED

- 98 -

12th International Biology Olympiad


Genetics - Evolution
A 79. In cats, black colour is caused by an X-linked allele; the other allele at
this locus causes orange colour. The heterozygote is tortoise-shell. What
kind of offspring would you expect from the cross of a black female and an
orange male?
A.
B.
C.
D.

tortoise-shell female; tortoise-shell male.


black male; orange female.
tortoise-shell female; orange male.
tortoise-shell female; black male.

A 80. A man is brought to court in a paternity case. His blood type is B, Rh


positive. The mother blood type is B, Rh negative. The childs blood type is
A, Rh negative. What can be said about the mans chances of being the
father?
A.
B.
C.
D.

he is not the father.


he might be the father.
he is the father.
he might not be the father.

A 81. The fact that all seven of the garden pea traits studied by Mendel
obeyed the principle of independent assortment means that:
A.
B.
C.
D.

the haploid number of garden peas is 7.


the formation of gametes in plants is obtained by mitosis only.
the seven pairs of alleles determining these traits behave as if
they were located on different chromosomes.
seven pairs of alleles determining these traits are located on the
same pair of homologous chromosomes.

A 82. Given the parents AABBCc x AabbCc, with simple dominance and
independent assortment for each of the three characters. What proportion
of the progeny will be expected to phenotypically resemble the first parent?
1/4.
3/4.
1/8.
3/8.

A 83. Two true-breeding stocks of garden peas are crossed. One parent
had red, axial flowers, and the other had white, terminal flowers. In F1 all
individuals had red, axial flowers. If 1000 F2 offspring resulted from the

- 99 -

Belgium, July 8 - 15, 2001

A.
B.
C.
D.

12th International Biology Olympiad


cross and assuming independent assortment, how many of them would
you expect to have red, terminal flowers?
A.
B.
C.
D.

190.
65.
250.
550.

A 84. Three babies were recently mixed up in a hospital. After consideration of the data below, which of the following represents the correct baby/
parent series of combinations?
______________________________________________
Couple
I
II
III
Blood groups

A and A

A and B

B and O

______________________________________________
Baby
1
2
3
Blood groups

AB

______________________________________________
A.
B.
C.
D.

I-3, II-1, III-2.


I-1, II-3, III-2.
I-2, II-1, III-3.
I-2, II-3, III-1.

A 85. There is good evidence for linkage when:

Belgium, July 8 - 15, 2001

A.
B.
C.
D.

two genes occur together in the same gamete.


a given gene is associated with a specific phenotype.
genes do not segregate during meiosis.
a single gene influences two traits.

A 86. Genes A and B are linked 12 map units apart. A heterozygous individual, whose parents were Aabb and aaBB, would be expected to produce
gametes in the following frequencies:
A.
B.
C.
D.

44 % AB
6 % AB
12 % AB
6 % AB

6 % Ab
44 % Ab
38 % Ab
6 % Ab

- 100 -

6 % aB
44 % aB
38 % aB
44 % aB

44 % ab
6 % ab
12 % ab
44 % ab

12th International Biology Olympiad


A 87. The Darwinian fitness of an individual is measured by:
A.
B.
C.
D.

the number of its offspring that survive to reproduce.


the number of mates it attracts.
its physical strength.
how long it lives.

A 88. Among the various mice dihybrid crosses indicated below, which one
will give the best chance to obtain in one litter a mouse with genotype AABb:
A.
B.
C.
D.

AaBb x AaBb.
AaBb x AABb.
AABB x aaBb.
AaBb x AaBB.

A 89. Two individuals are differing by one character. When crossed together all the offsprings belong to the parental phenotypes. Which of the
following ratios between the two observed phenotypes seem in accordance
with the hypothesis of a single gene difference (monogenic trait):
A.
B.
C.
D.

421/416.
862/281.
762/435.
1201/304.

A 90. Considering an animal strain, which is pure for a particular trait:


A.
B.
C.
D.

crossing of individuals from such a strain will not allow further


selection of variants for that trait.
offspring from such crossing experiments will consist exclusively
in homozygotes at any locus considered, including the locus re
sponsible for the particular trait.
crossing is rendered impossible because sterility invariably arises.
crossing of individuals from such a strain is a way to obtain hy
brids for that trait.

A.
B.
C.
D.

0.
1/2.
2/3.
3/4.

- 101 -

Belgium, July 8 - 15, 2001

A 91. Phenylketonuria (PKU) is a genetic disease caused by a recessive


mutation. If both parents are heterozygous for the gene, what is the probability for a normal child to be heterozygous?

12th International Biology Olympiad


A 92. How many different kinds of gametes can a trihybrid produce, considering that only independent assortment occurs during meiosis (no crossing
over)?
A.
B.
C.
D.

4.
8.
2 x 23.
6.

A 93. Suppose you are able to observe under the microscope the total
number of meiosis occurring in one gonad of a given individual and to outnumber exactly the crossovers between two given loci for which that individual is dihybridic. If the frequency of these particular crossovers is 100%
(that is to say that every meiosis exhibits one crossing over between the
two loci you consider) you anticipate that the total percentage of recombinant gametes would be equal to:
A.
B.
C.
D.

100 %
50 %
25 %
12.5 %

A 94. The sequence of a variant of the human globin protein shows a difference at aminoacid number 40 and another difference at aminoacid number
60 as compared to the normal protein. The number of nucleotides between
the two point mutations in the DNA of the corresponding gene is necessarily:

Belgium, July 8 - 15, 2001

A.
B.
C.
D.

a multiple of 3.
a multiple of 20.
at least 60.
at least 57.

A 95. A tomato plant that is heterozygous for a gene that makes stems
purple or green and also for a gene that makes foliage hairy or hairless is
test crossed to a green hairless plant and the 500 progeny are outnumbered as follows: 42 purple, hairy; 202 purple, hairless; 209 green, hairy;
47 green, hairless. The distance (in cM or m.u.: map units) between these
genes is:
A.
B.
C.
D.

17.8 cM.
35.6 cM.
8.9 cM.
89 cM.

- 102 -

12th International Biology Olympiad


A 96. DELETED
A 97. Which of the following statement is in agreement with the HardyWeinberg law:
A.
B.
C.
D.

in a climax ecosystem, energy flow is constant.


in a panmictic population, the allele frequencies remain constant
from one generation to another.
individuals are taller under high latitudes.
in a population, the mutation frequency compensates the selection pressure.

A 98. As we go from ferns through gymnosperms to angiosperms, the


following evolutionary trend can be observed:
A.
B.
C.
D.

an increasing dominance of the sporophyte and a reduction of


the gametophytic generation.
an increasing dominance of the gametophyte developing vessels and sieve tube elements with companion cells in angiosperms.
an increasing dominance of the sporophyte developing tracheids
and sieve cells with albuminous cells in angiosperms.
an increasing dominance of the sporophyte while the gametophyte remains unaltered.

A 99. During Paleozoic era, tree forms are especially dominant during:
A.
B.
C.
D.

Silurian.
Devonian.
Carboniferous.
Permian.

Belgium, July 8 - 15, 2001

- 103 -

12th International Biology Olympiad


Ecology
A 100. In a big lake under temperate climate, the temperature drop in winter provokes:
A.
B.
C.
D.

a mixing of the mineral nutrient.


the migration of phytoplancton.
a thermal stratification.
a strong mortality of fishes.

A 101. DELETED
A 102. Soil and cave animals share some common features: which of the
following is not correct?
A.
B.
C.
D.

a reduction of pigmentation.
a reduction of visual performances.
a reduction of all sensory organs.
an adaptation to constant abiotic conditions.

A 103. Which ecological unit incorporates abiotic factors?


A.
B.
C.
D.

community.
ecosystem.
population.
species.

A 104. While local conditions such as heavy rainfall or the removal of plants
limits the amount of nitrogen, phosphorus, or calcium available to a particular ecosystem, the amount of carbon available to the system is seldom a
problem. The reason therefore is:

Belgium, July 8 - 15, 2001

A.
B.
C.
D.

that organisms do not need very much carbon.


that plants can make their own carbon using water and sunlight.
that many nutrients come from the soil, carbon comes from the
air.
that plants are better at absorbing carbon from the soil.

A 105. DELETED
A 106. DELETED
A 107. DELETED

- 104 -

12th International Biology Olympiad


A 108. DELETED
A 109.
1.
all autotrophic organisms are also phototrophic.
2.
of the total light energy reaching the atmosphere, only about 1 %
is utilised in photosynthesis.
3.
light intensity and quality changes vertically through a forest
canopy.
4.
light intensity and quality changes vertically through a water column.
Which of the above combination of sentences is correct?
A.
B.
C.
D.

1,3,4.
2,3,4.
3,4.
1,2,3,4.

A 110. Which of the following factors tends to be the most important factor
controlling primary productivity in the oceans?
A.
B.
C.
D.

solar radiation.
temperature.
dissolved oxygen.
nutrients.

A 111. A uniform pattern of dispersion is likely to be found in which type of


habitats and under what condition?
A.
B.
C.
D.

A 112.The cultural eutrophication of lakes often reduces the oxygen content to a risky level. The main reason for this excessive oxygen reduction
is:
A.
B.
C.
D.

the oxygen consumption of plants.


the oxygen consumption of fishes.
the oxygen consumption of decomposers.
the oxidation of nitrates and phosphates.

- 105 -

Belgium, July 8 - 15, 2001

homogeneous distribution of essential resources, high population density.


heterogeneous distribution of essential resources, high population density.
homogeneous distribution of essential resources, low population
density.
heterogeneous distribution of essential resources, low population density.

12th International Biology Olympiad


A 113. Corals are marine cnidarians. Those building coral reefs live in
shallow and translucent water. They contain Zooxanthellae symbiotic algae in their endoderm. The following chemical reactions may help you:
6CO2 + 6 H2O C6H12O6 + 6O2
Ca(HCO3)2 CaCO3 + H2O + CO2
Which of the four following propositions isnt correct?
A.
B.
C.

Belgium, July 8 - 15, 2001

D.

reef corals get exogenous food by several different ways (cnidoblasts, contact digestion, angling with mucous filaments).
reef corals help their symbiotic algae by giving them amino
acids and glucose and receive from them glycerol and growth
vitamins.
reef corals of the euphotic zone have lost cnidoblasts and do not
need any longer because their zooxanthellae cover all their needs.
the carbon dioxide used for precipitating CaCO3 results from zooxanthellae.

- 106 -

12th International Biology Olympiad


Biosystematics
A 114. Which of the following metazoan groups is characterised by a bilateral symmetry?
A.
B.
C.
D.

hydrozoa.
porifera.
cephalochordata.
echinodermata.

A 115. DELETED
A 116. Indicate which of the following cavities has a coelomic origin:
A.
B.
C.
D.

pulmonary cavity.
bladder cavity.
peritoneal cavity.
uterine cavity.

A 117.
I.
II.
III.
IV.
V.

Odonata - Hymenoptera.
Coleoptera - Orthoptera.
Orthoptera - Homoptera.
Hemiptera - Coleoptera.
Lepidoptera - Mecoptera.

In which of the above insect orders do the anterior (front) and posterior
(rear) pairs of wing have different structures and are mouthparts of the
chewing type?
A.
B.
C.
D.

only II.
I and II.
only III.
IV and V.

A.
B.
C.
D.

dry skin with epidermal derivatives and cutaneous respiration.


respiration only with lungs and stable body temperature.
development includes laying of eggs and a caterpillar stage.
a respiratory system which includes trachea and lungs with a
faveolus.

- 107 -

Belgium, July 8 - 15, 2001

A 118. Of the following statements, mark the one, that is a characteristic


feature of reptiles:

12th International Biology Olympiad


A 119. Mark the incorrect statement about sponges:
A.
B.
C.
D.

their skeletons consist of calcium, silicon dioxide or spongin


needles.
they gain food by phagocytosis.
all known sponges live in seas or oceans.
sponges can host many symbiotic species.

A 120. To which family does a plant with the following characteristics belong: 5 basely connate sepals, 5 free petals, many stamens, and a superior
gynoecium consisting of 5 fused carpels?
A.
B.
C.
D.

Solanaceae.
Rosaceae.
Cactaceae.
Ranunculaceae.

Belgium, July 8 - 15, 2001

A 121. DELETED

Sssst...Concentration during the theoretical test

- 108 -















All IBO examination questions are published under the following Creative Commons license:



CC BY-NC-SA (Attribution-NonCommercial-ShareAlike) https://creativecommons.org/licenses/by-nc-sa/4.0/
The exam papers can be used freely for educational purposes as long as IBO is credited and
new creations are licensed under identical terms. No commercial use is allowed.

12th International Biology Olympiad


Theoretical Test - Part B
Cell biology
B 1. [3 points] The diagram pertains to the replication of double-stranded
DNA. DNA shown with arrows indicates the newly synthesized strands and
the direction of replication.
Match each site of action with the appropiate enzyme.

1. Telomerase.
2. DNA topoisomerase.
3. DNA polymerase.
4. DNA helicase.
5. DNA ligase.
this enzyme functions at the square marked I on the diagram.
this enzyme functions at the square marked II on the diagram.
this enzyme functions at the square marked III on the diagram.
B 2. [6 points] The two compartments (A and B) of a tank contain each one
liter of solution and are separated by a membrane whose permeability characters are to be identified.
One has placed in A: 300 g heparin (MW: 6000)
in B: 22.5 g formamide (MW: 45) and 5.8 g NaCl
(MW: 58).
At equilibrium, a small rise in liquid level is observed in compartment A.
Mark your answer by putting an X in the appopriate box.

0.05 molar.
0.5 molar.
2 molar.
5 molar.

- 109 -

Belgium, July 8 - 15, 2001

a) At the start of the experiment, the molar concentration in compartment A


is equal to:

12th International Biology Olympiad


b) At the start of the experiment, the molar concentration in compartment
B is equal to:
0.2 molar.
0.5 molar.
0.6 molar.
0.7 molar.
2.1 molar.
c) According to the results of the experiment, what are the conclusions that
can be made concerning the permeability characteristics of the membrane
that was used?
the membrane is permeable to compounds that have a molecular weight lower than 6000 daltons.
the membrane is permeable only to molecules that are not ionised.
the membrane is permeable only to molecules that are ionised.
the membrane is only permeable to water.
d) At the final equilibrium, the concentration of formamide in compartment
B:
will be unchanged.
will be decreased by half.
will be doubled.

Belgium, July 8 - 15, 2001

will be impossible to calculate.


e) With respect to the molecules that have been placed in the two sides of
the container, this membrane:
has the same permeability characteristics as the plasma membrane of a red blood cell.
is more permeable than the plasma membrane of the red blood
cell.
is less permeable than the plasma membrane of the red blood
cell.

- 110 -

12th International Biology Olympiad


has the same permeability characteristics as as the plasma membrane of a onion cell.
has the same permeability characteristics as the plasma membrane of a red blood cell and a onion cell.
f) What would happen if a few red blood cells were placed in compartment
B at the beginning of the experiment?
they would all explode.
some would swell while others would explode.
they would all shrink.
some would shrink while others be normal.
they would have a normal shape and size.

B 3. [6 points] The bacterium Escherichia coli adapts to lactose present in


his medium by means of the lactose operon. Cytoplasmic components
used for the function of this operon are randomly distributed among boxes
of the following table. Each component is designated by a number.
1.
2.
3.
4.
5.
6.
7.
8.

-galactosidase gene
Repressor
Operator
Lactose
Lactose-permease
Repressor messenger RNA
Regulator gene
Promotors

10. Plasmic membrane


11. RNA polymerase
12. Ribosomal apparatus
13. Transacetylase gene (lacA)
14. Transacetylase
15. -galactosidase
16. Glucose
17. Messenger RNA,
-galactosidase, permease
and transacetylase
18. Galactose

9. Lactose-permease gene

10

11 12 13 14 15 16 17 18

B 4. [5 points] Match each item in column A with one in column B to which

- 111 -

Belgium, July 8 - 15, 2001

What are the elements present in the cytoplasm of the bacteria growing on
a medium without lactose (put an X in the appopriate boxes).

12th International Biology Olympiad


it is most closely associated. (Put the corresponding numbers of column
B in the boxes in front of column A).
Column A

Column B

A. leucoplasts

1. Krebs cycle

B. rough ER

2. bacterial DNA anchorage

C. genome

3. microtubule-organizing center

D. mitochondria

4. protein modification and targeting

E. centriole

5. complete set of genetic instructions

F. mesosome

6. starch storage

G. lysosome

7. immunoglobulin

H. microfilament

8. lipid synthesis

I.

9. digestive enzymes

smooth ER

J. Golgi apparatus 10. cytoskeleton

Belgium, July 8 - 15, 2001

B 5. [2 points] Mannitol / salt agar is a culture medium that contains a high


salt (NaCl) concentration, mannitol (a fermentable sugar) and a chemical
pH indicator that is yellow at acidic conditions and red at alkaline conditions. (Acids are released when microorganisms ferment mannitol). This
medium also contains other carbohydrates that allow growth of
nonfermenting, halophilic organisms (i.e nonfermenting organisms that tolerate high salt concentrations). Nonhalophilic organisms will not grow on
mannitol / salt agar.
For each of the following situations, assume that the organisms described
are the only organisms involved.
Place the number of the term in front of the box of the sentence that best
describes the way the medium is being used.
1.
2.
3.
4.

selective medium
differential medium
both selective and differential medium
neither selective nor differential medium

- 112 -

12th International Biology Olympiad


onto mannitol salt agar you inoculate a halophilic fermenter, a
halophilic mannitol nonfermenter and a nonhalophilic mannitol
fermenter.
onto mannitol salt agar you inoculate a halophilic mannitol
fermenter and a halophilic mannitol nonfermenter.
onto mannitol salt agar you inoculate a halophilic mannitol
nonfermenter that is pigmented yellow, and a halophilic mannitol
nonfermenter that is pigmented red. These two organisms show
the same pigmentation (yellow and red, respectively) on a general
purpose medium such a nutrient agar.
onto mannitol you inoculate a halophilic mannitol nonfermenter
and a nonhalophilic mannitol fermenter.
B 6. [1 point] The diagram below represents a function of the nucleic acid,
DNA. Based on the diagram, what is the most likely nucleotide sequence of
the messenger RNA. Put an X in the appropriate box.

Belgium, July 8 - 15, 2001

- 113 -

12th International Biology Olympiad


Plant anatomy and physiology

Belgium, July 8 - 15, 2001

B 7. [3 points] Observe the diagrams I to VI illustrating different types of


plant cells and identify the cell types. Match the numbers in front of the cell
types (A-F) with the figures.

A.

sieve tube.

B.

collenchyma.

C.

sclereid.

D.

xylem vessels.

E.

epidermis.

F.

palisade cell.

B 8 A drawing of a cross-section of a (dicot) root is presented and 4 questions (a, b, c and d) relate to this figure. In this drawing several tissues and
cell types have been marked (with a letter). A list of tissues and cell types
have been numbered (1-14).
a)[4 points] Match the number with the markings on the drawing.

- 114 -

12th International Biology Olympiad


1. collenchyma
3. cortex
5. companion cells
7. trichome
9. endodermis
11. epidermis
13. root cap

2. xylem
4. phloem
6. mesophyl
8. pericycle
10. cambium
12. xylem parenchyma
14. pollen tube

b) [5 points] Nutrients are taken up from the soil or from nutrient solution
and translocated to the aerial parts of the plant.

in this part of the root some cells contain suberine (cork-like


substances). Which cells do contain suberine? Mark the correct
roman number in the box.
the (dicot) root, depicted in the figure contains dead cells. Which
cells in this section are dead? Mark the correct roman numbers
in the boxes.

- 115 -

Belgium, July 8 - 15, 2001

which tissue (in the figure above) translocates the nutrients from
root to shoot? Mark the correct roman number in the box.

12th International Biology Olympiad


B 9. The relationship between light saturated photosynthetic rate of single
leaves and organic leaf nitrogen in two plant species (represented by open
circles and solid triangles) is presented in 3 graphs (A, B and C) below.
Photosynthesis was measured at three temperatures: 20 , 27 and 34 C.
One of the two plant species used is a C3 plant, the other a C4 plant.

a) [1 point] Which symbol represents the C4 plant, or


Mark the correct answer with a cross in the appropriate box.

b) [3 points] Which temperature corresponds to which graph?


Mark the right combination with crosses in the appropriate boxes.
20 C

27 C

34 C

A
B

Belgium, July 8 - 15, 2001

C
B 10. [1 point] Even when starving, snails still stay green for some time.
Only after longer periods of starving they take up a brown-yellowish color.
Scientists investigated the weight loss (wl) of starving snail. Which results
could they have observed comparing starving green (gr) with starving brownyellow (by) snails? Mark your answer with a X in the box.
wl(gr) = 0 ; wl(by) > 0
wl(gr) > 0 ; wl(by) = 0
wl(gr) > 0 ; wl(by) > 0
wl(by) > wl(by) > 0

- 116 -

12th International Biology Olympiad


B 11. [5 points] Match the terms numbered 1,2,3 with the phrases under
column A
A

contain palisade parenchyma

1. roots

possess a cuticle

2. stems

possess a pronounced periderm

3. leaves

contain an x-shaped central core of xylem


modified to a potato
contain Casparian strips
modified to a carrot
contain an atactostele
contain growth rings
modified to corms (e.g. Crocus)
B 12. DELETED

Belgium, July 8 - 15, 2001

- 117 -

12th International Biology Olympiad


Animal anatomy and physiology
B 13. [3 points] Write down the correct code on the lines of the drawing.

Code

Belgium, July 8 - 15, 2001

1.
2.
3.
4.
5.
6.

Ranvier node.
nucleus.
dendrites
axon initial segment.
Schwann cell.
terminal button.

B 14. [1 point]
1.
most of the invertebrates neurons are not myelinated.
2.
myelin sheats wrap the full length of the axon except in its end
and in the Ranvier nodes.
3.
there are no ribosomes neither in the axons nor in the nerve
endings.
4.
some neurons of the nervous system dont have any axon.
From the list above,which is the correct combination of statements?
1, 2, 3
2, 4
1, 3, 4
all are correct.

- 118 -

12th International Biology Olympiad


B 15. [4 points] Choose a word or phase from the right column which is
related to each word or phrase in the left column by placing a number in
front of the word or phrase in the blank space in front of each word or
phrase on the left (each word can be used only once).
macrophage

1. yellow marrow

specific receptor on a B cell surface

2. contains large
number of granules

cytokines

3. astrocyte

site of differentiation for lymphocytes 4. oligodendrocyte


neutrophil

5. microglia cell

a major protein in plasma

6. complement

target of HIV virus

7. B cell

fat tissue inside a bone

8. antibody

macrophage in brain

9. chemicals
secreted by several types of cell in
immune system
10. T cell
11. peptides and
proteins
12. spleen or thymus
13. bone marrow
14. albumin
Belgium, July 8 - 15, 2001

- 119 -

12th International Biology Olympiad


B 16. DELETED
B 17. [1 point] Two types of muscles participate in the locomotion of an
earthworm. When moving forward, the earthworm first squeezes the front
segments into a long tube (I) and than drags its back part (II). Which muscles
are contracted (C) and relaxed (R) in I and II respectively?
I
Longitudinal muscles/circular muscles
R/C

II
Longitudinal muscles/circular
muscles
R/C

R/C

C/R

C/R

C/R

C/R

R/C

B 18 [1 point] Which substance does not usually appear in the glomerular


filtrate of mammals?
urea
glucose
amino acids
plasma proteins

Belgium, July 8 - 15, 2001

B 19 [1 point] Several parts of the body are involved in the transmission of


a stimulus. Which of the following represents the correct sequence as a
stimulus is carried along the reflex pathway?
sense organ; efferent neuron; spinal cord; afferent neuron;
muscle/gland
muscle/gland; efferent neuron; spinal cord; afferent neuron;
sense organ
sense organ; afferent neuron; spinal cord; efferent neuron;
muscle/gland
sense organ; afferent neuron; efferent neuron; spinal cord;
muscle/gland

- 120 -

12th International Biology Olympiad


B 20. [5 points] Types of nonspecific defense mechanisms.
Indicate whether each of the following is a general (G) physical (P) chemical (C) or biological (B) defense mechanism by putting a X in the appropriate boxes.
General G

Physical P

Chemical C

Biological B

Skin
Interferon
Gastric acid
Mucociliary Blanket
Inflammation
Fever
Lysozyme
Phagocytosis
Bile
Urine

- 121 -

Belgium, July 8 - 15, 2001

Relaxing at the Campus of the Vrije Universiteit Brussel after


the theoretical test

12th International Biology Olympiad


Ethology
B 21. DELETED
B 22. [1point] The following diagrams show the circadian rhytmic activity of
rats. The gray background represents dark phases.
Diagram 1: single record of locomotive activity within 48 hours
Diagram 2: summary activity over a period of 35 days
Diagram 3: activity of an arythmic rat after destruction of the Nucleus
suprachiasmaticus

Belgium, July 8 - 15, 2001

From the given information one can conclude that a normal rat has an
endogenous period of rythmicity of:
Put a X in the appropriate box.
less than 24 hours
exactly 24 hours
more than 24 hours
the period varies strongly
there is not enough information for a conclusion

- 122 -

12th International Biology Olympiad


Genetics - evolution
B 23. [3 points] Consider the following set of eight hybridised human-mouse
cell lines:
Chromosome
Cell line
1

12

13

17

21

Each cell line may carry an intact (numbered) chromosome (+), only its
long arm (q), only its short arm (p), or it may lack the chromosome (-).
The following human enzymes were tested for their presence (+) or absence (-) in cell lines A-H.
Identify the chromosome carrying each enzyme locus. Where possible, identify the chromosome arm.

Belgium, July 8 - 15, 2001

- 123 -

12th International Biology Olympiad


Cell line
Enzyme

Steroid sulphatase

Phosphoglucomutase-3

Esterase D

Phosphofructokinase

Amylase

Galactokinase

Belgium, July 8 - 15, 2001

B 24. [2 points] A certain haploid fungus is normally red due to a carotenoid pigment. Mutants were obtained that were different colours due to
the presence of different pigments: orange (?-), pink (P-), white (W-), yellow
(Y-), and beige (?-). Each phenotype was inherited as if a single gene mutation governed it. To determine what these mutations signified, double mutants were constructed with all possible combinations, and the results were
as follows:
Mutations:
B
PWY-

Ppink
-

Wwhite
white
-

Yyellow
pink .
white
-

B
beige
pink .
white
yellow

a) Determine the sequence of the five mutations in pathway of pigment


biosynthesis. Mark with X correct answer.
P - Y - B - O - W
Y - P - B - W - O

- 124 -

12th International Biology Olympiad


W - P - Y - B - O
O - B - Y - P - W
b) Determine the sequence of the colour intermediates in the biochemical
pathway of pigment biosynthesis. Mark with X correct answer.
white, pink, yellow, beige, orange, red
orange, pink, yellow, white, beige, red
beige, white, orange, pink, yellow, red
white, orange, yellow, pink, beige, red
B25. [4 points] Match each item in column A with one in column B to which
it is most closely associated. Mark the corresponding numbers of column
B in boxes on the left side of column A.
Column A

Column B
1 translation termination

B. alternative form of a gene

2 intercalating agent

C. auxotroph

3 substitution one amino acid


for another

D. ethidium bromide

4 chromosome aberration

E. nonsense mutation

5 substitution one purine for


pyrimidine

F. missense mutation

6 nutritional mutant

G. inversion

7 base analog

H. transversion

8 allele

- 125 -

Belgium, July 8 - 15, 2001

A. 5bromouracil

12th International Biology Olympiad


B 26. [3 points] Study the following pedigree of a trait which has 100%
penetrance.

: unaffected female

: affected female
: unaffected male
: affected male
a) The probable mode of inheritance of this trait is:
X-linked dominant
autosomal recessive
X-linked recessive
autosomal dominant
b) What are the genotypes of the mother and father of V-3?
Aa and a/Y
AA and AA

Belgium, July 8 - 15, 2001

aa and A/Y
Aa and Aa
c) If the individual V-2 marries a homozygous normal person, what is the
probability that their first child will be a carrier?
25 %

100 %

50 %

- 126 -

12th International Biology Olympiad


B 27. [1point]Three alleles are determining the ABO-blood groups: IA, IB
and i. Colour blindness is caused by a recessive X-chromosomal allele.
Inspect the following family trees where colour blindness and blood groups
have been indicated.

Directly after birth a baby of couple I must have been changed with a baby
of couple II. Indicate the numbers of the changed babies.
Answer: .. & ...
B 28. [1 point] A dominant gene , A, causes yellow coat colour in rats. The
dominant allele of another independant gene , R, produces black coat colour.
When the two dominants occur together, they interact to produce gray. When
the two recessives interact, they produce cream colour. A mating between
a gray male and a cream female produced a litter in which 3/8 of the offspring were yellow, 3/8 were gray, 1/8 were black and 1/8 were cream. If
the genotype of the female was aarr, what was the genotype of the male?
Belgium, July 8 - 15, 2001

AARR
AaRr
AaRR
AARr
Aarr

- 127 -

12th International Biology Olympiad


B 29. [1 point] What are the frequencies of recombinant chromatids for the
genes a and b if single crossing over, double crossover between two
chromatids and double crossover between three chromatids take place
as shown in the figure below?

Crossing over Double crossing over


between 2 chromatids

Double crossing over


between 3 chromatids

25 %

50 %

75 %

50 %

50 %

75 %

50 %

0%

50 %

75 %

25 %

0%

Belgium, July 8 - 15, 2001

Ecology
B 30. [1 point] The growth of an animal population depends on the biological characteristics of the individuals within such as: life expectancy, fecundity (number of births given by a female during its life), the duration of the
reproductive period, the distribution of the fecundity along the female age,
the development time (the time between birth and thelaying of the first egg)
and the age at the first egg laying. Moreover, the age structure of the population (i.e. the distribution of the age according to the age pyramids will
influence the way these characteristics will be expressed.
Let us consider two populations A and B. Which one of these two populations will undergo the more rapid growth in relationships with the following
statements, all other elements being constant?
population A is mostly composed of young individuals whereas
population B is characterised by a stable age structure.

- 128 -

12th International Biology Olympiad


inside population A, females reached sexual maturity two times
faster than B however females from B survived 1/3 longer.
in population A female fecundity is uniformly distributed during all
their life, whereas for B it is concentrated at the beginning of adult
life.
in population B female fecundity is two times higher than in A but
life expectancy is two times shorter than in A.
B 31. [1 point] Social Hymenoptera (such as ants, social bees and wasps)
are characterised by the frequent repetition of altruistic behaviours between
individuals. This altruism may even lead to the death of some individuals
such as soldiers. These behaviours are more frequent between close relatives. Reproduction in Hymenoptera is quite special as it relies on haplodiploid parthenogenesis. This means that an unfertilised egg gives birth to
a haploid male while a fertilised egg produces a diploid female. Under these
conditions calculate (in %) the mean degree of relatedness between:
a female and her daughter
a female and her son
two sisters
a sister and her brother
an aunt and her niece

- 129 -

Belgium, July 8 - 15, 2001

B 32. [1 point] In order to measure the biodiversity in tropical forest, the


following experiments was undertaken. A canopy tree was isolated using a
plastic sheet before fumigation with an insecticide. All insects killed were
collected.(this experiment was conducted by Terry Erwin and his staff from
Smithsonian Institution, Panama). Investigators mostly concentrate on
Coleoptera. They collected 1200 species. By studying that group, they observed that 20 % of phytophagous Coleoptera were specific of that trees
species. They concluded that each tree species in tropical forest welcomes
160 species of Coleoptera that are specific of the tree considered. Knowing
that 40 % of the insect species already described on earth are Coleoptera
they infer that, in average, each tree harbours 400 insect species. In tropical forest, only 2/3 of the insect are living in the canopy. Thus if the tree is
taken in its totality (branches, trunk, roots, etc.) it would shelter 600 insect
species.

12th International Biology Olympiad


Finally, as presently 50.000 tree species are identified in tropical
forest, the total number of insect species should be of 30.000.000. Less
than one million of insect species are actually described meaning that we
know a very little part of the total earth biodiversity. Which among the following propositions is not correct?
30 millions of insects is an underestimation as it does not take
into account species living in other countries.
30 millions is an overestimation as most tree species share common species.
this experiment does not give an overall view because it is too
limited in space and time but it just showed that we are lacking
data on biodiversity.
0 millions is an overestimation as it does not take into account
the species already described.
B 33. DELETED
B 34. DELETED
B 35. DELETED
B 36. [6 points] For each figure answer YES or NO to the following question: Does the figure show that population density is affected by birth or
death rates, thereby representing a mechanism able to regulate populations
size.
YES

NO

Belgium, July 8 - 15, 2001

A.
B.
C.
D.
E.
F.

- 130 -

12th International Biology Olympiad


Number
deaths

Mortality
rate

Population density

Birth rate

Population density

Number
deaths

Population density

Net recruitment

Population density

Net recruitment

Population density

Population density

Belgium, July 8 - 15, 2001

- 131 -

12th International Biology Olympiad


Biosystematics
B 37. [4 points] The following table contains a list of some cellular structures and processes. Make a comparison between Eubacteria,
Archaeobacteria and Eucaryotes by making a cross (X) in appropriate
boxes of the table, if these structures/processes are present in those domains.
Cellular structures
and processes
Eubacteria

Archaeobacteria

Eucaryotes

RNA
Introns
Ribosomes
Mitochondria
Diploid stages
Chlorophyll-based
Photosynthesis
Methanogenesis
Nitrogen fixation

Belgium, July 8 - 15, 2001

B 38. [5 points] Among the members of the Animal Kingdom, some have
body cavities, some do not. The following animals (1 - 9) possess different
types of body cavities.
1.
2.
3.
4.
5.
6.
7.
8.
9.

Rotifers
Gastropods
Insects
Sponges
Nematodes
Corals
Oligochaetes
Planaria
Sea anemones

- 132 -

12th International Biology Olympiad


Identify which animals of the list above belong to what group of body cavity
by filling in the blanks with the corresponding numbers. Each group may
have more than one related animal.
Animals with true coeloms (coelomates)
Animals with pseudocoeloms (pseudocoelomates)
Animals with no coeloms (acoelomates)
Animals with gastrovascular cavities
Animals with spongocoel

B 39. [3 points] Taxonomical classification of organisms can be realised in


different ways.
One method is to take in consideration selected characteristics of a number of organisms and construct a grid (data matrix) showing the percentage of similarity between these characteristics.
An example of such a similarity matrix of 10 organisms 1 to 10 is:
100

54

100

80

55

100

63

57

62

100

62

57

64

74

100

81

55

85

63

64

100

50

86

51

56

56

54

100

83

56

86

65

67

87

54

100

50

87

50

56

56

52

85

54

100

10

61

56

62

90

72

65

55

67

55

100

10

- 133 -

Belgium, July 8 - 15, 2001

12th International Biology Olympiad


Based on this matrix it is possible to produce a tree like diagram showing
one group of four related organism (group I), one group of three (two + one)
related organisms (group II) and another group of three organisms (group
III) in the following way:

Indicate which organisms belong to group I, II and III:


Group I:.........................

Belgium, July 8 - 15, 2001

Group II:.........................
Group III:........................

- 134 -

12th International Biology Olympiad


B 40. [5 points] Every insect Order is characterised by a specific structure
and function of wings.
Insects

Order

Number

dragonflies
Odonata
grasshoppers Orthoptera
lice
Anoplura
beetles
Coleoptera
flies
Diptera

1.
2.
3.
4.
5.

Complete the following statements by filling in the spaces, with the number
of the appropriate insect order.
similar in size non-foldable fore wings and hind wings with netlike
venation are characteristic for individuals of the order.
sclerotised fore wings and membranous hind wings with netlike
venation are characteristic for individuals of the order.
secondary reduced wings combined with protruding-sucking
mouth parts are characteristic for ectoparasitic individuals of the
order.
hard fore wings having a protection function and membranous
hind wings are characteristic for individuals of the order.
membranous fore wings and hind wings transformed into halters
having a function of stabilisation during flight are characteristic
for individuals of the order.

- 135 -

Belgium, July 8 - 15, 2001

The test is over, unstress yourself

12 th International Biology Olympiad

Belgium, July 8 - 15, 2001

Answer Key to the Theoretical Part A


A 001
A 002
A 003
A 004
A 005
A 006
A 007
A 008
A 009
A 010
A 011
A 012
A 013
A 014
A 015
A 016
A 017
A 018
A 019
A 020
A 021
A 022
A 023
A 024
A 025
A 026
A 027
A 028
A 029
A 030
A 031
A 032
A 033
A 034
A 035
A 036
A 037
A 038
A 039
A 040

D
D
B
D
C
C
D
A
C
C
A
D
B
D
D
D
C
B
B
C
A
D
D
B
C
B
D
D
A
D
Deleted
D
C
B
D
C
A
Deleted
Deleted
C

A 041
A 042
A 043
A 044
A 045
A 046
A 047
A 048
A 049
A 050
A 051
A 052
A 053
A 054
A 055
A 056
A 057
A 058
A 059
A 060
A 061
A 062
A 063
A 064
A 065
A 066
A 067
A0 68
A 069
A 070
A 071
A 072
A 073
A 074
A 075
A 076
A 077
A 078
A 079
A 080

A
D
C
C
D
A
C
C
D
C
Deleted
C
A
D
B
C
A
D
C
C
D
D/D
A/A
D
B
B
D
B
Deleted
B
A
D
C
D
C
C
A
Deleted
D
A

-1 -

A 081
A 082
A 083
A 084
A 085
A 086
A 087
A 088
A 089
A 090
A 091
A 092
A 093
A 094
A 095
A 096
A 097
A 098
A 099
A 100
A 101
A 102
A 103
A 104
A 105
A 106
A 107
A 108
A 109
A 110
A 111
A 112
A 113
A 114
A 115
A 116
A 117
A 118
A 119
A 120
A 121

C
B
A
D
C
B
A
B
A
A
C
B
B
D
A
Deleted
B
A
C
A
Deleted
C
B
C
Deleted
Deleted
Deleted
Deleted
B
D
C
C
C
C
Deleted
C
A
D
C
B
Deleted

12 th International Biology Olympiad


Answer Key to the Theoretical Part B
B01
B02

a
b
c
d
e
f

B03
B04
B05
B06
B07
B08
B09
B10
B11
B12
B13

B21
B22
B23
B24
B25
B26

1, 2, 3, 6, 7, 8, 9, 10, 11,
12, 13, 16
6, 7, 5, 1, 3, 2, 9, 10, 8,
4
3, 2, 4, 1
UCGAUU
VI, I, IV, V, II, III
4, 12, 2, 10, 8, 9, 3, 11
III, VI, III
#
B, C, A
answer 4
3, 3, 2, 1, 2, 1, 1, 2,
2, 2
Deleted
upper: 2, 4, 1, 5
lower: 3, 6
All are correct
8, 9, 12, 2, 14, 10, 1,5
Deleted
R/C C/R
Plasma proteins
answer 3
G: fever
P: skin, mucociliary
C: interferon, gastric,
Lysozyme, bile, urine
B: inflammation,
phagocytosis

-2-

B27
B28
B29
B30
B31
B32
B33
B34
B35
B36
B37

B38
B39

B40

Deleted
answer 3
Xp, 6q, 13q, 21, 1p, 17q
a
b
a
b
c

answer 3
answer 1
7, 8, 6, 2, 1, 3, 4, 5
answer 3
answer 1
50%
2&5
answer 2
50 %, 0%, 50%
A, A, B, B
50, 50, 75, 25, 12,5
answer 1
Deleted
Deleted
Deleted
Yes: B, D, E
No: A, C, F
Line 1: X, X, X
Line 2: - , X, X
Line 3: X, X, X
Line 4: - , - , X
Line 5: - , - , X
Line 6: X, - , X
Line 7: - , X, Line 8: X, X, 2/3/7, 5, 8, 6/8/9, 4
Group I: 1/3/6/8
Group II: 10/4/5
Group III: 7/9/2
3, 5, 2, 1, 4

Belgium, July 8 - 15, 2001

B14
B15
B16
B17
B18
B19
B20

a
b
a
b

3, 4, 2
0.05 molar
0.6 molar
answer 4
answer 4
answer 5
answer 3

INTERNATIONAL BIOLOGY OLYMPIAD


PRACTICAL PROBLEMS

2001, Brussels, Belgium















All IBO examination questions are published under the following Creative Commons license:



CC BY-NC-SA (Attribution-NonCommercial-ShareAlike) https://creativecommons.org/licenses/by-nc-sa/4.0/
The exam papers can be used freely for educational purposes as long as IBO is credited and
new creations are licensed under identical terms. No commercial use is allowed.

12th International Biology Olympiad

Belgium, July 8 - 15, 2001

Practical test

- 60 -

12th International Biology Olympiad


Practical Test

General remarks about the practical tests


In practical tests you are expected to demonstrate that you know some
basic methods, processes and techniques of biology and that you are able
to use them for experimental purposes.
This will be tested in a series of tests, organised in four different laboratories.
Laboratory 1: Plant anatomy, morphology and taxonomy
Laboratory 2: Animal anatomy, morphology
Laboratory 3: Plant pigment analysis
Laboratory 4: Ethology
You have approximately 60 minutes in each laboratory. You can score 50
points at a maximum in each laboratory, which means a total amount of
200 points for the practical test. If at some task you mark some incorrect
alternative(s) as well as some correct ones, you will get a lower score at
that particular task than if you only mark the correct answers.

Laboratory 1: Plant anatomy,


morphology and taxonomy
Introductory remarks

The dissection case must be used in two laboratories. Its a gift. Take it
with you.

- 61 -

Belgium, July 8 - 15, 2001

In this laboratory you will be given three separated tasks. It is of no importance with which you start. The time to perform all the tasks is 70 minutes.
Please be sure that the task paper given to you fits the task you are performing.

12th International Biology Olympiad


Task 1: making sections of plant organs. Identification of the organs.
Taxonomic classification.
Task 2: identification of 3 representatives of a plant taxonomic group by
selecting distinct parts of the species.
Task 3: identification of flowering plant families on the basis of flower buds
and reference slides of transverse section of the buds.
TASK 1: Making sections of plant organs. Identificarion of the organs.
Taxonomic classification. (Total points 11)
Material
Plant organ samples (2 types A and B)
Instruments and equipment:
Razor blade
Stain and glassware +H2O
Slides and coverslips
Microscope
Reference slides of transverse section of plant organs of 7 different taxonomic groups

Belgium, July 8 - 15, 2001

1. Make a cross section of sample A. Prepare a wet mount and use the
stains if necessary.
You have a set of reference slides (1-7) with cross sections of different
plant organs.
Examine your preparation under the microscope and compare with the reference slides. Identify the corresponding reference slide and fill in the correct number of that slide on the answer form in the blank 1A1.
(1,5 points)
You have a list of organ types. Select the correct number out of the list and
fill in the blank 1A2 on your task 1 answer form. (2 points)
You have a list of taxonomic group. Select the correct number out of the list
and fill in the blank 1 A3 on your task 1answer form. (2 points)
2. Make a cross section of sample B. Proceed as in 1 and fill in blanks on
your task 1answer form .
1B1 (1,5 points)
1B2 (2 points) 1B3 (2 points)
TASK 1. List of organ types
1. Primary root
2. Primary stem
3. Rhizome

- 62 -

12th International Biology Olympiad


4.
5.
6.
7.

Leaf petiole
Leaf rachis
Secondary root
Secondary stem

TASK 1. List of taxonomic groups


1.
2.
3.
4.
5.
6.
7.

Flowering plant monocotyl (Liliaceae)


Flowering plant dicotyl (Magnoliaceae)
Conifer
Fern
Clubmoss
Horsetail
Cycad

TASK 2: Identification of 3 representatives of a plant taxonomic group


by selection of distinct parts of the specimen.
Material
Mixture of plant parts of 3 representative species of different plant taxonomic groups
Binocular microscope
Dissecting equipment (tweezers)
Glassware (petri dishes)

2. You have a list of taxonomic groups: select the appropriate group for the
species in Petri dishes A, B and C and fill in the corresponding number in
the blank code 215, 224, 233 (each 2 points) on the task 2 answer form.

- 63 -

Belgium, July 8 - 15, 2001

1. Transfer the mixture of plant parts into a large petri dish. Select the parts
which belong to the same species (if necessary the binocular microscope
can be used) and transfer the items that belong together.
In petri dish A you have to collect 4 items.
In petri dish B you have to collect 3 items.
In petri dish C you have to collect 2 items.
You have a figure with several plant parts of different plant species belonging to several taxonomic groups. Select those figures, that corresponds
with the items in the petri dishes. Fill in the blanks in 211, 212 , 213 and 214
(each 2 points) for the items belonging to species 1, on the task 2 answer form.
Do the same for species 2 and 3. Fill in for petri dish A and petri dish B
221, 222 and 223 (each 2 points). FilI in for petri dish C 231, 232 (each 2
points) on the task 2 answer form .

12th International Biology Olympiad


TASK 2. List of several taxonomic groups.
1.
2.
3.
4.
5.
6.
7.
8.

Monocotyl
Dicotyl
Cycad
Conifer
Moss
Alga
Horsetail
Clubmoss

Belgium, July 8 - 15, 2001

Task 2. Figure of different plant parts

- 64 -

12th International Biology Olympiad


TASK3: Identification of flowering plant families on the basis of flower
buds and refernce slides of transverse section of buds (Total points
15).
Material
Samples of flower buds of 2 plant species belonging to 2 different families
A and B.
Reference slides of cross sections of flower buds (1, 2, 3)
Binocular microscope
Dissection tools
Petri dishes
3.1. Take the flower bud out of petri dish A. Dissect the flower and observe
its characteristic diagram.
You have a set of selected reference slides (1-3).
Select, on the basis of these characteristics, the corresponding reference
slide and fill in the number in the blank code 311. (1,5 points)
You have a list of families of flowering plants.
Select the correct number corresponding with the family of the sample A
flower bud and fill in the number in the blank code 312 (2 points) on your
answer form task 3.
You have pictures of fruit types.
Select the number of the type of fruit belonging to this family and fill in the
number on the blank code 313 (1,5 points).
3.2. Do the same for the petri dishes B and C.
Fill in for petri dish B the number in the blank code 321 (1,5 points), 322
(2 points), and 323 (1,5 points).
Fill in for petri disch C the number in the blank code 331 (1,5 points), 332
(2 points), and 333 (1,5 points).
TASK 3. List of families of flowering plants
Belgium, July 8 - 15, 2001

1. Ranunculaceae
2. Lamiaceae
3. Oleaceae
4. Fabaceae
5. Apiaceae
6. Asteraceae
7. Poaceae
8. Liliaceae
9. Fagaceae
10. Brassicaceae

- 65 -

12th International Biology Olympiad


Task 3. Pictures of fruit types

Laboratory 2: Animal anatomy, morphology


Introductory remarks

Belgium, July 8 - 15, 2001

The insect to be dissected is a cockroach : Insecta, Dictyoptera, Blattidae,


from the genus Periplaneta. These insects are well known to you. They
have invaded nearly all the buildings in some cities. As they are nocturnal,
the humans may considerably underestimate their number. They are sometimes seen as mark of dirtiness, however, they can be very abundant even
in very clean environment, like hospital. A main factor of their proliferation
is the climate : summer, with both high temperature and humidity are extremely favourable to the species.
Dissection of Periplaneta
Material
Periplaneta
A dissection tray
6 entomological needles
8 coloured needles (one to eight)
Scalpel

- 66 -

12th International Biology Olympiad


H2O
tweezers
fine scissors
slides and cover-glasses
binocular microscope + light
Manipulation
1. add tap water in the dissection tray (1,5 cm high).
2. fix the animal properly. The dissection can be made under binocular
microscope : start cutting it with fine scissors beginning at the anus ; cut
along the two lateral sides as far as the prothorax.
3. lift up the cut off tergite (dorsal plate) and turn them over. (5 points)
4. unroll the digestif tract, lift it away from the body without cutting it and
pin the different sections with labelled needles in the following order
Use needle red 1. for the crop / stomodaeum.
(5 points)
blue 2. for the gizzard / proventiculus
(5 points)
orange 3. for the digestive caeca
(5 points)
black 4. for the mesenteron / midgut
(5 points)
yellow 5. for Malpighi tubules
(5 points)
green 6. for proctodaeum
(5 points)
white 7.for nerve cord
(5 points)
pink 8. fat body
(5 points)
5. Cut a piece of trachea and make a wet mount of it, bring it under the
binocular microscope and ensure you have a thin preparation. Take this
between slide and cover-glass and examine it under the binocular microscope (5 points).
The dissection must be completed within 40 minutes.
The dissection must be checked by an assistant.
When you have finished the dissection, raise your hand to call for an assistant.
The assistant and you must sign the evaluation sheet.
Re-evaluation is supervized by a Professor.

Reversed Phase Thin-Layer Chromatography of Photosynthetic Pigment


Introduction remarks
Thin-Layer Chromatography is an important technique for analysis of molecules. How do biologists and biochemists know so much about molecules

- 67 -

Belgium, July 8 - 15, 2001

Laboratory 3: Plant pigment analysis

12th International Biology Olympiad


such as, for example, proteins, lipids and plant pigments? The answer is of
course that they have been isolating, separating and studying these molecules for more than a century. One important technique for the analysis of
molecules is thin-layer chromatography (TLC). This technique can separate different kinds of molecules based on their relative affinities for a hydrophilic stationary phase and a hydrophobic mobile phase. The stationary
phase is usually a thin layer of silicic acid (silica gel) on a glass or metal
plate, while the mobile phase is a mixture of appropriate solvents. Although
largely replaced by several new and more efficient and accurate techniques,
TLC remains a rather simple but effective technique to make a first qualitative analysis of photosynthetic pigments in samples.
For the analysis of photosynthetic pigments some problems may appear,
such as the formation of pheophytins due to the reaction of pigments with
H2SiO3 molecules, present in the silica gel. The problem can be avoided by
using silica gel from which the polar groups are saturated with C 18alkylchains. This results in a non-polar silica gel and the technique is specified as reversed phase thin layer chromatography (RP-TLC) with an appropriate solvent for the mobile phase (e.g. mixtures of ethylacetate, methanol
and water).
Equipment and material available for each student
1.
2.
3.

Belgium, July 8 - 15, 2001

4.
5.
6.
7.
8.
9.
10.
11.
12.
13.
14.
15.
16.
17.

Simple chromatography tank (high-200/250ml-beaker filled with


small but suitable volume of solvent, covered with petri dish)
sheet of parafilm (spare)
2 TLC-strips cut at 40 x 100 mm , dried for at least 30
min. at 60 - 80 (1 spare strip)
pencil
ruler
piece of aluminium foil
3 test tubes (glass 20 ml)
graduated measuring cylinder (10ml)
chemicals : petroleum ether 40 - 60 (bottle A); acetone 100 %
(bottle B)
distilled water in test tube
fine sand in test tube
mortar + pestle
2 capillary tubes (20 l)
bean plant
task description with experimental protocol
Multiple choice questions
answer leaflets

- 68 -

12th International Biology Olympiad


TASKS:
1. Separation and identification of plant pigments by reversed phase
TLC
2. Solving MC questions related to TLC and Plant Pigments (use
answer leaflet no 1)
3. Completing the answer leaflets no 2 for task evaluation
At the end of the session each student presents two answer leaflets and 1
TLC-plate.
Task 1: Separation and identification of pigments by reversed phase
TLC
Preparation (takes approx. 15 minutes)
1.
2.
3.
4.

Take about 8 cm2 of plant material.


Grind the plant material in a mortar using a bit of sand.
Add 100 % acetone (max. 10 ml) bottle B.
Pour in test tube, decant after 5 minutes (meanwhile you can
prepare the TLC-plate, see step 7).
5. Add approximately 20 drops petroleum ether (bottle A) and ap
proximately 5 ml distilled water.
6. Shake well and allow to separate into two layers (wait 1 min for
the separation).
7. Take a TLC plate and draw carefully and faintly a start line (use pencil)
at 1.5 cm from the bottom (avoid damaging the silica gel layer).
Spotting (takes approx. 5 minutes)
1. Spot the pigment extract on the start line with a small capillary tube in
three different locations (do it gently and avoid damaging the silica
gel layer).
2. Dry the spots by gently blowing.
3. Repeat this 4 to 5 times.

1. Put the TLC plate in the tank and allow to develop in the dark
(cover the jar with aluminium foil). IMPORTANT: Use your free
time to solve the MC questions (task 2).
2. After approx. 20 minutes, take the plate out of the tank. In any case
take the plate out when the front line reaches 1 cm from the top of the
TLC plate.

- 69 -

Belgium, July 8 - 15, 2001

Running (takes approx. 25 minutes)

12th International Biology Olympiad


Interpretation
1. Mark the solvent front line (use pencil).
2. Use the answer leaflet to draw a copy of your TLC plate, including
the front and start line and the pigments observed.
3. Calculate Rf-values for the photosynthetic pigments you have sepa
rated.
Remarks:
Photosynthetic pigments with a Rf-value equal or greater than 0,4 might
be only slightly visible. Examples for this RP-TLC:
neoxanthin Rf =0,6
violaxanthin Rf =0,5
luten
Rf =0,4
Despite the precautions pheophytins may appear. If they show up, you might
see them as a grey-blue haze. We do not consider them as photosynthetic
pigments because they are derived from photosynthetic pigments through
chemical alteration.
4. Mark on your drawing, each of the observed photosynthetic pig
ment spots. Numbering of your spots starts with no 1 for the
spot with the lowest Rf-value. Try to identify four photosynthetic
pigments with the lowest Rf-values.
5. Cover your TLC plate with aluminium foil to prevent deterioration
of the pigment colors.
6. Continue with task 3 by completing n 2 answer leaflet, using all
available information including that from the MC questions.
Task 2: Answer the MC questions related to TLC and Plant Pigments
Choose the one best answer for each question.

Belgium, July 8 - 15, 2001

Scoring for each question:


- Right answer = + 3 points
- Wrong answer = - 1 point ( guess correction)
- No answer = no points
Mark your answers on leaflet no 1
1. Which of the following statement is correct?
A. Plants have chloroplasts and therefore can live without mitochondria.
B. Both chlorophyll and heme contain an extensive system of double
bonds that allow them to absorb visible light.
C. The role of chlorophyll in photosynthesis is equivalent to that of
heme in mitochondrial electron transport.
D. All the statements above are wrong.

- 70 -

12th International Biology Olympiad


2. Study the molecular structure of major plant pigments to predict the
relative solubility in polar or non-polar solvents (see figure on page 17).
To determine polarity of these pigments, one can count the number of
atoms per molecule. Which count is the most suitable?
A.
B.
C.
D.

H-atoms
N-atoms
C-atoms
O-atoms

3. Acetone is very suitable to extract pigments from chloroplasts. Choose a


correct answer that supports this fact.
A.
B.
C.
D.

Non-polar molecules only dissolve in non-polar solvents.


Chloroplast-membranes are polar structure.
Non-polar molecules only dissolve in polar solvents.
The stroma in chloroplasts contains a huge number of non-polar
molecules.

4. Plant material often contains red pigments; their structure reveals that
they have no function in photosynthesis and are soluble in water. A perfect
location for these pigments in plant cells is:
A.
B.
C.
D.

Membranes of chloroplasts.
Membranes of mitochondria.
Lumen of vacuoles.
Cytosol of the cell.

5. Chlorophyll is a complex molecular structure with a magnesium atom


held in the centre of a porphyrin ring, which is structurally similar to the
porphyrin ring that binds iron in heme. Which of the next statements is
correct and completes the functional description of the molecule?

- 71 -

Belgium, July 8 - 15, 2001

A. Electrons within the bond network of the porphyrin ring absorb


light, while the long hydrophilic tail helps to hold the chlorophyll in
the thylakoid membrane.
B. Electrons within the bond chain of the long hydrophilic tail absorb
light, while the network of the porphyrin ring helps to hold the
chlorophyll in the thylakoid membrane.
C. Electrons within the bond network of the porphyrin ring absorb
light, while the long hydrophobic tail helps to hold the chlorophyll
in the thylakoid membrane.
D. Electrons within the bond chain of the long hydrophobic tail ab
sorb light, while the network of the porphyrin ring helps to hold
the chlorophyll in the thylakoid membrane.

12th International Biology Olympiad


6. A researcher has just discovered a new bacteria species that has a
unique photosynthetic pigment. In a TLC, the pigment appears to be reddish yellow. What colours of the visible light spectrum are not absorbed?
A.
B.
C.
D.

Red and yellow.


Blue and violet.
Green and yellow.
Blue, green, and red.

7. If you know the absorption spectrum of a plant pigment, you can


be reasonably certain of:
The wavelength of visible light it absorbs.
The wavelength of visible light it reflects.
Tthe energy of the visible light it absorbs and reflects.
All of the above.

Belgium, July 8 - 15, 2001

A.
B.
C.
D.

- 72 -

12th International Biology Olympiad


ANSWER LEAFLET N 1: MC QUESTIONS (21 points)
Scoring for each question:
right answer
+ 3 points
wrong answer
- 1 point
no answer
no point
Question 1
Question 2
Question 3
Question 4
Question 5
Question 6
Question 7

A
A
A
A
A
A
A

B
B
B
B
B
B
B

C
C
C
C
C
C
C

D
D
D
D
D
D
D

Code number:
ANSWER LEAFLET N 2

(29 points)

Fill in for RP-TLC task evaluation


RP-TLC plate n:...................
Photosynthetic pigment n 1
name:.........................................
Rf -value:...................................
Photosynthetic pigment n 2
name:.........................................
Rf -value:...................................
Photosynthetic pigment n 3
name:.........................................
Rf -value:...................................

(3 points)
(3 points)
(3 points)
(3 points)
(3 points)
(3 points)

Draw a copy of your TLC-plate in the box (5 points).


Mark the most abundant photosynthesis pigment spots.
Present answer leaflet n 1, n 2, and your TLC-plate to the panel of examiners.

- 73 -

Belgium, July 8 - 15, 2001

Photosynthetic pigment n 4
name:.........................................
Rf -value:...................................

(3 points)
(3 points)

12th International Biology Olympiad


Laboratory 4: Ethology
Introductory remarks
In this part, you will see 6 film sequences dealing with the behavior of insects, birds and mammals. Each sequence will be shown twice.
We suggest you proceed as follows:
First, watch the sequence.
Second, read the relevant question.
Third, watch the sequence a second time.
Fourth, answer the question on the answer sheet correctly.
Good Luck
Questions about sequences
1. Butterfly chrysalis : the chrysalis of this species is either green or brown.
Both morphs are cryptic in their natural environment (trees) (8 points).
A.
B.
C.
D.

Chrysalis colour is conditioned by the texture of the substrate on


which the caterpillar moves before pupate.
Chrysalis colour is conditioned by the colour of the twig on which
the caterpillar ran before pupate.
Both A and B are correct.
Neither A nor B is correct

Belgium, July 8 - 15, 2001

2. Path selection by ants: two video sequences are displayed. During the
first sequence, a bridge made of two unequal lengths connects the nest to
a food source. During the second sequence, the ants have access to the
food only by using the long path of the bridge during the first part of the
experiment. The ants, however, can use both the long and the short paths
in the second part of the experiment, which started after a heavy traffic was
first established on the long path during the first part of the experiment
(8 points).
A.
B.
C.
D.

The ants randomly select one or the other path.


The ants are able to evaluate travelled distances and to remem
ber the shorter path.
Path selection depends on the number of ants which had previ
ously travelled along each path.
Ants never travel along a path that they have not marked chemi
cally.

3. Honeybee dance: in the video sequence, a bee loaded with pollen per-

- 74 -

12th International Biology Olympiad


forms a waggled dance for recruiting nest-mates towards a food source.
The bee dance informs nest-mates on the direction they must follow to find
the food source: (8 points)
A.
B.
C.
D.

About 45 right to the sun.


About 135 left to the sun.
South-west from the hive.
North-east from the hive.

4. Social interactions between cows: the video sequences illustrate dominance relationships between feeding cows in five experimental conditions.
Out of the following assertions, which one is correct (8 points)?
A.
B.
C.
D.

Visual stimuli are required for holding dominance position.


A dominant cow always expresses its status whatever the experimental condition.
Dominance is not expressed without head contact between cows.
All the above answers A, B and C are wrong.

5. Maternal behavior in sheep. Three video sequences are shown: the first
illustrates the behavior of a mother and her lamb soon after parturition; the
second, a lamb is washed immediately after parturition before its mother
was able to care for it, after what it is given to its mother who had never
cared for lambs before; the third, the lamb was washed soon after parturition and then given to a foreign ewe who had experienced lamb care before (10 points).
A.
B.
C.
D.

Suckling acceptance requires that the ewe had the opportunity to


care for a lamb after parturition.
An ewe is more prone to accept a lambkin if she marked it by
licking it before.
Washing eliminates all odours, preventing suckling acceptance.
The three answers A, B, C are correct.

A.
B.
C.
D.

Only males take care of chickens.


Only females take care of chickens.
Brood-care by males and females is similar.
Both males and females take care of the chickens, but there is a
division of tasks between them.

- 75 -

Belgium, July 8 - 15, 2001

6. Emperor-pinguin :four videos are shown:1)egg-laying and egg-transfer


to the male ; 2) incubation of eggs by aggregated males ; 3) females join
the males and chickens are tranferred to them; 4) chickens are grouped in
a nursery (8 points).

12th International Biology Olympiad


Answer Key to the practical test
Laboratory 3: Chromatography
Task 2 MC questions
1. b; 2.d; 3.a; 4.c; 5.c; 6.a; 7.d
Task 3
1. carotene; Rf +/- 0.1
2. chlorophyll b; Rf +/- 0.2
3. chlorophyll a; Rf +/- 0.3
4. lutein; Rf +/- 0.4

Belgium, July 8 - 15, 2001

Laboratory 4: Ethology
1.a; 2.c; 3.b; 4.c; 5.a; 6.d

Barbecue at the campus of the Universit Libre de Bruxelles


after the practical test

- 76 -

INTERNATIONAL BIOLOGY OLYMPIAD


THEORY PROBLEMS

2000, Antalya, Turkey















All IBO examination questions are published under the following Creative Commons license:



CC BY-NC-SA (Attribution-NonCommercial-ShareAlike) https://creativecommons.org/licenses/by-nc-sa/4.0/
The exam papers can be used freely for educational purposes as long as IBO is credited and
new creations are licensed under identical terms. No commercial use is allowed.

Answer Key-Theoretical Test-Part A

4. 3. Theoretical Test
General remarks:
This test consists of two parts, A and B. In part A there are 103 multiple
choice questions, each having only one correct answer. In part B there are
46 questions, each of which may have more than one answer.
In order to eliminate the consequence of guessing in the marking of Type
A questions, one point will be deducted for every 5 incorrect answers.
Failing to answer a question will not result in any penalty.
In the marking of Type B questions a percentage of the total mark for that
question will be deducted for each incorrect answer. The minimum mark
for each question with deductions will be zero.
For the multiple choice questions (type A) mark the correct answer with a
cross X in the blank space provided. If you want to change your answer
with a new one in order to cancel it, you may draw a parallel line on your
old mark.

170

correct

delete

Report of the 11th IBO in Antalya

4.3.1. Theoretical Test - Part A


CELL BIOLOGY
1. In which way are the proteins transported from the site of
synthesis to the cell membrane for secretion?
___A) By cytoplasmic movement
___B) By some signal proteins in the cytosol
___C) By protein-carbohydrate complex carrying signals in the cytosol
___D) By cytoskeleton elements
___E) By vesicles
2. What is the major difference between a vacuole and a vesicle?
___A) The membrane is thick in the vacuole but thin in the vesicle
___B) The vesicle is pinched off only from the cell membrane; the
vacuole is pinched off from the Golgi apparatus
___C) The vacuole membrane is carbohydrate rich; the vesicle membrane
is protein rich
___D) The vacuole is near the nucleus; the vesicle is near the Golgi
apparatus
___E) The vacuole has a comparatively slow movement; the vesicle
moves rapidly
3. Which of the following is not a function of the Golgi apparatus?
___A) Addition of sugars to proteins
___B) Storage of lipids
___C) Package of secretion products
___D) Formation of glycolipids
___E) Synthesis of polysaccharides from simple sugars

171

Answer Key-Theoretical Test-Part A

4. Which of the following functions are carried out in the smooth


endoplasmic reticulum?
I)
Addition of carbohydrates to proteins
II)
Synthesis of membrane phospholipids
III) Addition of carbohydrates to lipids
IV) Synthesis of cholesterol
V)
Detoxification of drugs
___A) I, II, IV
___B) II, III, IV
___C) II, IV, V
___D) I, IV, V
___E) I, II, V
5. Deleted
6. Fibroblasts are cells which synthesize proteins of the extracellular
matrix of the connective tissue (collagen fibers), glycoproteins
(fibronectin) and proteoglycans (dermatan sulphate). According to
these features which organelle/organelles has/have the greater
function in these cells?
___A) Rough endoplasmic reticulum and smooth endoplasmic reticulum
___B) Golgi apparatus
___C) Rough endoplasmic reticulum and free ribosomes
___D) Golgi and rough endoplasmic reticulum
___E) Rough endoplasmic reticulum

172

Report of the 11th IBO in Antalya

7. Four structures (I-IV) and some related functional and structural


features (1-7) are given below.
I. Cilia
II.

Basal bodies

III. Centrosome
IV. Flagellum
1- There is a 9+2 arrangement in a ring around a pair of single
microtubules
2- Most of these structures are longer than the cell
3- It is shorter than the cell
4- There are nine groups of three microtubules, fused into triplets
with an empty core
5- These are the main structures for movement
6- They function in the synthesis of spindle fibers
7- They bind cilia and flagella to the cell membrane
Which of the combinations below for structure and function are
correct?
___A) I: 1, 3, 5

II: 3, 4, 5

III: 3, 4, 6

IV: 1, 2, 3

___B) I:1, 4, 5

II: 1, 2, 7

III: 2, 3, 4

IV: 1, 3, 5

___C) I:1, 4, 7

II: 3, 4, 5

III: 2, 3, 6

IV: 2, 3, 4

___D) I:3, 4, 6

II: 2, 4, 7

III: 3, 4, 5

IV: 4, 5, 6

___E) I:2, 4, 6

II: 2, 4, 7

III: 3, 4, 5

IV: 2, 4, 5

8. Which of the following contains a polar head and a non-polar tail


in cell?
___A) Triglycerides

___B) Neutral lipids

___D) Phospholipids

___E) All the above

___C) Wax

173

Answer Key-Theoretical Test-Part A

9. Which of the following are the fibers that attach to the the
cytoplasmic face of spot desmosomes?
___A) Collagen fibers
___B) Cytoskeleton fibers
___C) Elastic fibers
___D) Tubulin protein fibers
___E) Reticular fibers
10. Which of the following is the correct description of a microsome?
___A) It consists of vesicles detached from the Golgi apparatus
___B) It consists of vesicles containing waste products digested by the
lysosomes
___C) It consists of various amounts of ribosomes and fragmented
endoplasmic reticulum
___D) It is a vacuole that contains secretions
___E) It is a ribosome dimer
11. In aerobic respiration glucose is converted to pyruvate in the
___A) Inner mitochondrial membrane

___B) Cytoplasm

___C) Outer mitochondrial membrane

___D) Mitochondrial matrix

___E) Mitochondrial membrane interspace (intermembrane space)


12. Which of the following is the correct description for a porin?
___A) It is a protein in the structure of microtubules
___B) It is a protein located on the outer mitochondrial membrane
___C) It is a protein of the nuclear pores
___D) It is a lipid that functions in the addition of carbohydrates to
proteins
___E) It is a protein that forms the cytoskeleton

174

Report of the 11th IBO in Antalya

13. How are the peroxisomes formed in a cell?


___A) Only by fission
___B) Only by detachment from a big peroxisome
___C) Both by fission and self-replication of a preexisting peroxisome
___D) Only by budding from plasma membrane
___E) Only by self-replication
14. In living cells there are
1. Ribosomes

5. Introns

2. ATP synthesis

6. DNA polymerase

3. Cell membrane

7. Photosynthesis

4. Nuclear envelope

8. Mitochondria

Which of them can exist both in prokaryotic and eukaryotic cells?


___A) 1, 2, 3, 6, 7

___B) 1, 2, 3, 5, 7, 8

___D) 1, 3, 5, 6

___E) 2, 3, 7, 8

___C) 1, 2, 3, 4, 7

15. Specific inhibitor X" of F0F1ATPase is added to a rat liver cell


carrying out the oxidation of glucose under aerobic conditions.
Which of the following would not occur as a result of this
inhibition?
___A) Mitochondrial ATP formation will stop
___B) The citric acid cycle will slow down because of insufficient NAD +
regeneration
___C) The rate of glucose consumption will decrease
___D) Glycolysis will be accelerated
___E) Oxygen consumption will be halted

175

Answer Key-Theoretical Test-Part A

16. Agarose gel electrophoresis was applied to the DNA samples given
below. What will be the order of migration from the well, at the
completion of the electrophoresis?
I- F+ bacterial plasmid
II- F ' bacterial plasmid
III- Hfr E. coli chromosomal DNA
IV- E. coli chromosomal DNA
___A) I, II, III, IV
___B) II, III, I, IV
___D) III, IV, II, I

___C) IV, III, II, I

___E) IV, I, III, II

17. If you observe two DNA samples X and Y (each containing 1200
base pairs) migrating at different rates in an agarose gel, what
would your interpretation be ?
___A) The amount of adenines in sample X is greater
___B) The amount of guanines in sample Y is greater
___C) The percentage of agarose in the gel is greater than 0.8 %
___D) There are intercalating agents in the agarose gel
___E) Samples X and Y have different conformations
18. The enzyme phosphofructokinase;
I- It is the major regulatory enzyme in glycolysis
II- ATP is the substrate for the enzyme
III- ATP is the negative modulator of the enzyme
IV- Citrate activates the enzyme
Which of the following is the correct answer for the above statements
concerning the enzyme phosphofructokinase?
___A) Only IV is correct

___B) Only I and III are correct

___C) Only I, II and III are correct

___D) Only II and IV are correct

___E) I, II, III and IV are correct

176

Report of the 11th IBO in Antalya

19. If oligomycin and 2,4-dinitrophenol are both added;to a


suspension of mitochondria containing substrates, P i (inorganic
phosphate), Mg++ and ADP;
___A) Both O2 consumption and ADP phosphorylation will cease
___B) The rate of O2 consumption will increase but ADP phosphorylation
will cease
___C) Phosphorylation/O2 consumption ratio will remain the same
___D) Phosphorylation/O2 consumption ratio will increase
___E) O2 consumption will decrease but ADP phosphorylation will
continue
20. In anaerobic glycolysis 2 moles of inorganic phosphate (P i) are
used for one mole of glucose consumed. Which of the following
enzymes catalyzes the reaction in which Pi is directly consumed?
___A) Hexokinase
___B) Phosphofructokinase
___C) Pyruvate kinase
___D) Glyceraldehyde-3-phosphate dehydrogenase
___E) Enolase
21. Which of the following cannot use ketone bodies for the
generation of energy?
___A) The brain (in fasting)
___B) The heart muscle
___C) Erythrocytes
___D) The kidney cortex
___E) The skeletal muscle

177

Answer Key-Theoretical Test-Part A

22. Inside the choloplast the potential uses for the G3P
(glyceraldehyde 3-phosphate) produced in the Calvin cycle
include the synthesis of:
___A) Fatty acids
___B) Glycerol
___C) Glucose
___D) Amino acids
___E) All of the above
23. The following statements are about the effect of a competitive
inhibitor in a reaction catalyzed by an enzyme.
I- V max is unchanged
II-

The inhibition can be reversed by increasing the


concentration of the substrate

III- Km increases
IV- The inhibitor binds to the enzyme at a different site than the
active site
Which combination of statement(s) is/are true?
___A) I, II and III

___B) Only I and III

___D) Only IV

___E) I, II, III and IV

___C) Only II and IV

24. Which of the following bonds is not present in the structure of


DNA?
___A) 3-5 phosphodiester bond
___B) N-glycosidic bond
___C) H-bonds
___D) Hydrophobic interactions
___E) Disulphide bonds

178

Report of the 11th IBO in Antalya

25. Consider these two relationships and the four statements about
the aminoacids/proteins and fatty acids/triglycerides
Left

Right

Left

Right

Amino acids:

Proteins

Fatty acids:

Triglycerides

I.

Both molecules on the right consist only of repeated units of


the molecules on the left

II.

In the process of synthesis of both molecules on the right, at


least some electrical charges are neutralized

III. In both relationships, the diversity in the molecules on the


left result in the diversity in the molecules on the right
IV. In the synthesis of both molecules on the right, water is
released
Which of the statement(s) is/are correct?
___A) I, II, III, and IV

___B) II, III and IV

___D) Only III

___E) Only IV

___C) III and IV

26. Five different cell cultures were treated with different


radioactively labelled compounds as follows:
Compound
Cell Culture
Lactose
Cell culture a
Valine
Cell culture b
Thymidine triphosphate
Cell culture c
Glutamic acid
Cell culture d
Alanine
Cell culture e
After an hour the cells were washed, fixed and autoradiographed. In
order to study nuclear activities in vivo which of the cell cultures is
the best?
___A) Cell culture a
___D) Cell culture d

___B) Cell culture b


___E) Cell culture e

___C) Cell culture c

179

Answer Key-Theoretical Test-Part A

27. Which of the molecules is responsible for the autocatalytic


excision of introns and splicing of exons in eukaryotic cells?
___A) RNA polymerase
___B) Ribonuclease
___C) Ribozyme
___D) Reverse transcriptase
___E) Endonuclease
28. The interaction between the anticodon of a tRNA molecule and
the complementary codon of mRNA is achieved by:
___A) The catalysis by peptidyl transferase
___B) ATP energy
___C) The catalysis by amino-acyl-tRNA synthetase
___D) The covalent bonds formed with energy from GTP
___E) H-bonds
29. The lac (lactose) operon is an example of:
___A) Translational control
___B) Posttranslational control
___C) Replication control
___D) Transcriptional control
___E) All of the above

180

Report of the 11th IBO in Antalya

30. The breakdown of glucose in the cell is controlled by the


activation or inactivation of enzymes present in the specific
stages of glycolysis and the citric acid cycle. There are three key
enzymes like this. The conditions that activate or inactivate these
enzymes are given in the table below. Which combination is
correct for the activation of all the three enzymes?
ENZYMES
Phosphofructokinase

___A) High level of ADP+


AMP

Citrate synthase

Isocitrate
dehydrogenase

Low level of
ATP+NADH

Low level of
ATP+NADH

___B)

High level of ATP

High level of
ATP+NADH

Low level of
ATP+NADH

___C)

Low level of ATP

High level of
ATP+NADH

Low level of
ATP+NADH

___D) High level of ADP

High level of
ATP+NADH

High level of
eitherADP or NAD+

Low level of either


ATP or NADH

High level of either


ATP or NAD+

andAMP

___E)

High level of ATP

31. Which of the following statements is false about prokaryotic RNA


polymerase?
___A) The synthesis is in the 5-3 direction
___B) There is only one RNA polymerase enzyme responsible for the
synthesis of rRNA, m-RNA and t-RNA
___C) Its RNA product will hybridize with the DNA template
___D) The transcription starts from the AUG codon in the DNA
___E) The enzyme synthesizes a single transcript that codes for several
polypeptide chains

181

Answer Key-Theoretical Test-Part A

32. Which of the following statement about the regulator gene in


bacterial operon model is true?
___A) It codes for repressor protein
___B) It codes for inducer molecules
___C) It is the binding site of RNA polymerase
___D) It is the binding site of inducer molecules
___E) Provides the transcription or inhibition of transcription of the
structural genes
33. Which of the following statements is false regarding to the
procaryotic mRNA?
___A) It is polycistronic
___B) It does not involve introns
___C) It binds to ribosome from the 5end.
___D) It is synthesized in the nucleus
___E) It can form a single transcript that codes for several polypeptides.
34. Which of the following statements is a false description for a
codon?
___A) It consists of three nucleotides
___B) It is the basic unit of the genetic code
___C) There may be more than one codon for the same amino acid
___D) It is located on the t-RNA
___E) It can never code more than one amino acid

182

Report of the 11th IBO in Antalya

GENETICS AND EVOLUTION


35. A cross was made between two albinos and phenotypically
identical F1 generation was obtained. When F1 was self-crossed,
F2 was observed as 9 normal and 7 albinos. Which of the
following combinations suits this kind of inheritance?
Parents

O f f s p r i n g s(F2)

___A)

AAbb X aaBB 9A-B-

3aaB-

3A-bb

1aabb

___B)

aabb X AAbb

9A-B-

3aaBb

3Aabb

1aabb

___C)

AaBb X AaBb 9A-B-

3aaBb

3Aabb

1aabb

___D)

aaBb X Aabb

9A-B-

3aaB-

3A-bb

1aabb

___E)

AABB X aabb 9A-B-

3aaB-

3Aabb

1aabb

36. In an experimental population, the frequency of the O blood type


is 25%, A is 24%, B is 39% and AB is 12%. Which of the
following is the frequency of the alleles which are responsible for
the blood types A, B and O?
A

___A)

0.3

0.2

0.5

___B)

0.2

0.5

0.3

___C)

0.2

0.3

0.5

___D)

0.5

0.2

0.3

___E)

0.3

0.5

0.2

37. Deleted

183

Answer Key-Theoretical Test-Part A

38. Incomplete penetrance, sex-limited traits, sex-influenced traits,


age-influenced traits and temperature-influenced traits are all
examples of
___A) Linkage

___B) Conditional gene expression

___C) Epistasis

___D) Multiple alleles

___E) Partial dominance

39. Deleted
40. The fruit weights of a squash plant vary between 2 and 4 kg. The
fruit weights are a product of pairs of additive polygenic genes.
Which of the following is true for the F2 generation that resulted
from a 2 kg squash being pollinated with a 4 kg one in terms of
the number of individuals of any weight classes and also
regarding the genotypes of the pollinated 2 kg and 4 kg squashes.
4 kg

3.5 kg

3 kg

2.5 kg

2 kg

2 kg
parent

4 kg
parent

___A)

aabb

AABB

___B)

AaBb

aaBB

___C)

AAbb

aaBB

___D)

AABb

AABB

___E)

aabb

AABB

41. Which of the following is not a reason why recessive alleles are
not observed in the phenotypes of heterozygotes?
___A) The recessive allele codes for a nonfunctioning protein
___B) The recessive allele is linked to the dominant allele
___C) The dominant allele produces so much product as to swamp the
product of the recessive allele
___D) The recessive allele is normal, but the product of the dominant
allele inhibits the function of the recessive allele
___E) The alleles product (e.g.an enzyme) is much less functional and
therefore masked by the dominant alleles product

184

Report of the 11th IBO in Antalya

42.

3
Healthy : Empty

Sick : Dark colored

Carrier : Spotted

On the pedigree (family tree) given above which of the following


would be the genotype of the individuals written as 1, 2 and 3?
1

AA

XX

XY

XY

___B)

Aa

Aa

aa

___C)

aa

XX

XY

XY

___D)

aa

Aa

aa

___E)

Aa

AA

aa

___A)

185

Answer Key-Theoretical Test-Part A

43. An individual of the genotype AaBbCcDd was crossed with the


one aabbccdd and the following results were obtained.
aBCD

42

Abcd

43

ABCd

140

abcD

145

aBcD

AbCd

ABcd

305

abCD

310

Which of the following shows the arrangement of the genes and their
distance (in centimorgans)?

186

Report of the 11th IBO in Antalya

44. Genetic variations are important for populations


___A) So that males and females of parthenogenetic species might be
distinguished
___B) So that evolution is directed
___C) Because they provide the raw material on which selection acts
___D) So that organisms might be classified
___E) To make them more interesting to study
45. When a dominant allele (A) is alone, it causes a brown fur color
but when it is with another allel which has an epistatic effect, the
fur color is white.
According to this, which of the following shows the true genotypes of
the individuals in the family tree given above?
1

Brown

White

Brown

White

White

___A)

Aabb

AaBb

Aabb

AaBb

aabb

___B)

AaBb

aabb

AaBb

aaBb

aabb

___C)

AaBb

aaBb

AaBb

Aabb

Aabb

___D)

AaBb

aabb

AaBb

aaBb

aabb

___E)

aaBb

AaBb

Aabb

aaBb

aabb

187

Answer Key-Theoretical Test-Part A

46. Deleted
47.
Phenotypes of the individuals
Genotypes

Female

Male

AA

Aa

aa

An inheritance scheme of any character is given above. Which of


the following is true for the inheritance type of this character?
___A) This character is sex linked
___B) This character is sex limited
___C) This character is sex influenced
___D) Incomplete penetrance is seen in the inheritance of this character
___E) Codominance is seen in the inheritance of this character

188

Report of the 11th IBO in Antalya

48. In cattle, the polled (hornless) condition is dominant to the horned


condition. Coat colour can be red, white or roan (red with white
patches). Both genes are carried on autosomes and they are not
linked. A cross was carried out between a cow and a bull, both of
which had the roan coat colour and both were heterozygous for
the polled condition. Which of the following statements are true
about the offspring from the cross, assuming that the cross was
carried out several times to produce a lot of offspring?
1. The chance of producing white polled and white horned
offspring is the same.
2. The chance of producing roan polled offspring is three times
that of producing roan horned.
3. There is an equal chance of producing red polled and white
polled offspring.
4. Statistically there should be more roan horned offspring than
any other type.
5. The chance of producing roan polled offspring is twice that of
producing white polled.
___A) 1 & 2
___B) 2 & 3
___C) 3 & 4
___D) 1, 2 & 3
___E) 2, 3 & 5

189

Answer Key-Theoretical Test-Part A

49. In guinea-pigs, there are several alleles involved in determining


the animal's coat color. Cb - black; Cc - creamy; Cs - silver and
Cz - albino. Analyze the results of the following crosses and
determine the most suitable order of alleles referring to
dominance-recessiveness relationships of these alleles.

Crosses

Phenotype of
parents
Black

Silver

Creamy

Albino

black x black

22

black x albino

10

creamy x creamy

30

11

silver x creamy

23

11

12

___A) Cb > Cc > Cs > Cz


___B) Cb > Cs > Cc > Cz
___C) Cc > Cz > Cb > Cs
___D) Cb > Cz > Cs > Cc
___E) Cb > Cc > Cz > Cs
50. Deleted

190

Phenotype of offspring

Report of the 11th IBO in Antalya

51. Which of the following is not a proof that eukaryotic cells evolved
by endosymbiosis?
___A) Similarity between spirochetes and flagellum
___B) Similarity between mitochondrial DNA and procaryotic DNA
___C) Similarity between bacterial and chloroplastic ribosomes
___D) Similarity between chloroplast and cyanobacteria
___E) Similarity of the inhibition of the protein synthesis between the
eucaryotic cells and the mitochondria
52. Which of the following has the best evolutionary adaptation
capacity?
___A) Primitive, heterogenous heredity material, high number of
generation, short life span
___B) Highly specific, homogenous heredity material , high number of
generation, long life span
___C) Highly specific feeding regime, living underground, asexual
reproduction
___D) Living on high mountains, nocturnal, feeding on the most common
plants
___E) Highly tolerant to hereditary changes, low offspring success,
specific feeding regime
53. Which of the following is not a biological characteristic of desert
organisms?
___A) No regular reproductive cycle
___B) Seeds germinate immediately after flowering and fruiting
___C) Fewer stoma (in plant)
___D) Specialized kidney capable of reabsorbing water (in animal)
___E) More succulent plants

191

Answer Key-Theoretical Test-Part A

54. Deleted
55. Biologists assume that the first heredity material to appear was
RNA. Which of the following may be the main reason for that?
___A) RNA was produced in Millers experiment
___B) RNA is structurally more primitive than DNA
___C) The RNA called ribozyme catalyses some chemical reactions
___D) DNA can not stay stable in hydrophobic medium
___E) RNA appears in all animals
56. Which of the following is not evidence that higher plants are
derived from green algae?
___A) Some green algae have multicellular sporophyte and gametophyte
phases
___B) Both plants and algae have cellulose in their cell walls
___C) Both plants and algae have similar photosynthetic and accessory
pigments
___D) Both plants and algae synthesize starch as a main store product
___E) Green algae and higher plants have the same amount of DNA per
cell
57. The statements below are about various pollination strategies in
plants. Which one could be the most disadvantageous for
evolutionof new species?
___A) The stigma can recognize the origin of pollen grains, and does not
accept those from the same flower
___B) The stigma never emerges from the corolla, and only accepts
pollen grains from the same flower
___C) The corolla forms a long tube, only allowing some specialized
pollinators which carry pollen grains from the same species to
enter
___D) The stamens and pistil mature at different times
___E) The stamens and pistil are located in different flowers

192

Report of the 11th IBO in Antalya

58. Which of the following triplets is false for the hearing organ?
Structure

Animal

Function

___A)

Columella

Lizard

Transports the sound from the


membrane to the cochlea

___B)

Weber bones

Fish

Transports the sound created by


the vibration of the swim bladder
to the brain

___C)

Tarsal bones

Salamander Transports the vibrations from


the soil to the inner ear

___D)

Some cranial

Whale

___E)

Utricle (saccule) Mole

Transports the sound from water


to the (otic) bones inner ear
Transports the vibrations from
the soil to the inner ear

59. Deleted

193

Answer Key-Theoretical Test-Part A

PLANT ANATOMY AND PHYSIOLOGY


60. Which of the following can not be stated relating to cyclic
photophosphorylation?
Note: NADPred = NADPH; NADPox = NADP+
___A) It is favored when the cell is more in need of ATP than NADP red
___B) It is favored when NADPox is in short supply
___C) An energized electron is first accepted by ferredoxin
___D) Plastocyanin is the last acceptor of an energized electron before it
reaches the center
___E) In the system, cytochrome f connects ferredoxin to plastoquinone
61. Deleted
62.
I. It is required for activity of some dehydrogenases,
decarboxylases, kinases, oxidases and peroxidases
II.

Under its deficiency, plant tissues become soft and often


flaccid even under low temperature and stress conditions

III. It is required for the photosynthetic reactions involved in the


02 cycle
Which of the following gives the best match of minerals to
thestatements above?
I

II

III

___A) N

Ca

Mg

___B)

Mn

Mg

___C)

Mn

___D) Mn

Ca

Cl

___E)

194

Cl

Report of the 11th IBO in Antalya

63. When the temperature is high and the amount of dissolved oxygen
is higher than that of CO2 in the chloroplasts, in which of the
following plants does growth not slow down?
___A) Wheat
___B) Watermelon
___C) Sunflower
___D) Sugar cane
___E) Rice
64. Deleted
65. Deleted
66. Deleted
67. Deleted
68. Which of the following cannot be referred to as blue-light
responses in higher plants and fungi?
___A) Phototropism in Phycomyces
___B) Carotenoid biosynthesis in Neurospora
___C) Stomatal opening in higher plants
___D) Chloroplast rearrangements
___E) Flavenoid synthesis

195

Answer Key-Theoretical Test-Part A

69. Shootytumors are produced in the stem of an X plant


infected by bacteria in which mutations occur in their T-DNA,
while rooty tumors are formed in the stem of a Y plant.
Which of the following is true for the effects mentioned above.
___A) In the X plant, the genes which are responsible for giberellin
(giberellic acid( synthesis are inactivated while in the Y plant,
genes which are responsible for abscisic acid synthesis are
inactivated.
___B) In the X plant, the genes which are responsible for Indol aceticacid synthesis are inactivated while in the Y plant, genes which
are responsible for zeatin synthesis are inactivated.
___C) In the X plant, the genes which are responsible for zeatin
synthesis are inactivated while in the Y plant, genes which are
responsible for ethylene synthesis are inactivated.
___D) In the X plant, the genes which are responsible for abscisic acid
synthesis are inactivated while in the Y plant, genes which are
responsible ethylene synthesis are inactivated.
___E) In the X plant, the genes which are responsible for cytokinins
synthesis are inactivated while in the Y plant, genes which are
responsible for ethylene synthesis are inactivated.

196

Report of the 11th IBO in Antalya

70. In the figure, changes in water potential in the leaves of a plant


over a period of time are shown. Which of the following is true
according to this situation?
Leaf water
potential (mPa)

0
-0.8

-1.6

4
6
Time (Days)

10

___A) The ABA (Abscissic acid) content decreased, and stomata


resistance increased between the 2nd and 6th days; the process was
reversed between the 6th and 8th days
___B) The ABA content did not change and stomata resistance decreased
between the 2nd and 6th days; the process was reversed between
the 6th and 8th days
___C) The ABA content increased and stomata resistance decreased
between the 2nd and 6th days; the process was reversed between
the 6th and 8th days
___D) The ABA content and stomata resistance increased between the
2nd and 6th days; the process was reversed between the 6 th and 8th
days.
___E) The ABA content decreased and stomata conductance increased
between the 2nd and 6th days; the process was reversed between
the 6th and 8th days.

197

Answer Key-Theoretical Test-Part A

71. Which of the following are limiting or near-limiting nutrients


both in aquatic and terrestrial systems?
___A) Nitrogen-potassium

___B) Potassium-magnesium

___C) Phosphorus-nitrogen

___D) Calcium-magnesium

___E) Iodine-magnesium
72. Which of the following is not true for the auxin transport in
plants?
___A) IAA transport usually does not take place in sieve tubes and xylem
___B) IAA transport usually occurs in parenchymatic cells adjacent to
vascular bundles
___C) Auxin moves rather slowly within the plant
___D) IAA moves mainly from the apex to the base (basipetal direction)
___E) Auxin transport does not require energy
73. Light is perceived by all living organisms in one way or the other.
The pigment which is chosen for this process of photoperception
are carotenoids. Which of the following properties make
carotenoids the right pigment for this function.
___A) Their ability to absorb most of the visible and ultraviolet light
___B) Their high capacity to store and transfer light energy as chemical
energy
___C) As saturated organic compounds, their capability to preserve
themselves against environmental factors such as high energy
currents
___D) Their high affinity for proteins which have a role in perception
___E) The efficiency of long structures of alternating double bonds in
their structure to initiate light sensitive stereoisomerism

198

Report of the 11th IBO in Antalya

74. The changes that take place in climacteric fruits when they ripen
(colour, texture and chemical composition) are mainly due to:
___A) The CO2 content in the atmosphere
___B) The temperature variation
___C) The ethylene synthesis in the fruit
___D) The auxin concentration in the fruit
___E) Giberellin concentration in fruit
75. Deleted
76. The data were obtained relating to the rates of oxygen release and
uptake in plants. The plants were placed in the dark for 12 hours
followed by 12 hours in light. The temperature was constant
throughout the experiment. The results are shown in the graph.

Which of the following is the most accurate estimate of the total


volume of oxygen used by the plants for respiration during 24 hours
of the experimental time?
___A) 50 cm3

___B) 600 cm3

___D) 1200 cm3

___E) 1800 cm3

___C) 1000 cm3

199

Answer Key-Theoretical Test-Part A

77. In which aspect does C4 photosynthesis differ from Crassulacean


acid metabolism (CAM)?
___A) PEP carboxylase is only used in C4 photosynthesis
___B) CO2 fixation in CAM plants occurs at night while it occurs in C4
plants during the day time
___C) Organic acids with four carbons are only produced in C4
photosynthesis
___D) Only plants with crassulacean acid metabolism can carry out
photosynthesis in arid environments
___E) Only plants with C4 photosynthesis can economize water
78. Which of the following cannot be stated with relation to the shoot
apex?
___A) There is only one apical cell in vascular non-flowering plants
___B) There is more than one apical cell in each cell layer in
gymnosperms
___C) There are different apical cells in more than one tissue layer in
angiosperms
___D) An apical cell is pyramidal shaped in non-flowering plants
___E) A shoot apex with a distinct tunica and corpus is found in
gymnosperms
79. Which of the following cannot be stated for the collenchyma?
___A) It is a living tissue found in developing organs
___B) It is formed in the roots only under the effect of light
___C) Its location in petiols is peripheral
___D) It is located at the periphery of woody stems
___E) It is located at the periphery of lamina

200

Report of the 11th IBO in Antalya

80. Which of the following cannot be referred to as a function of the


sporoderm (exine)layer of pollens?
___A) The storage of enzyme proteins for the reactions
___B) Playing a role in the reaction between pollen and stigma
___C) Production of the pollen tube
___D) Protection of pollen against external factors
___E) Realizing pollination
81. Deleted
82. In a flower, flower symmetry is radial, calyx 4 and fused, corolla 4
and is separate, the androecium has 5 stamens and is connected to
the corolla, the gynoecium is compound in 5 parts, superior and
syncarpous. According to the description given above, which of
the formulae in the following is correct?
___A) + K(4) [C4A(5)]G(5)
*

___C) K4 C4A5G5
___E) *K4 [C4A5]G5

___B) + K(4) C4A(5)G(5)


___D) *K(4) [C4A(5)]G(5)

83. Which of the following is true for a C 4 plant in which some leaves
can carry out C3 photosynthesis while others can carry out C4
photosynthesis?
___A) In fact, it is a C3 plant
___B) The leaves which carry out C3 photosynthesis lack Kranz anatomy
___C) PEP (phosphoenolpyruvate) is not synthesized in the leaves which
carry out C4 photosynthesis
___D) It indicates that the C4 pathway was evolved from the C3 pathway
___E) Both C3 and C4 photosynthesis do not occur on the same leaf
84. Deleted

201

Answer Key-Theoretical Test-Part A

BIOSYSTEMATICS
85. Deleted
86.
I.

A small dry single-seeded, indehiscent fruit

II.

A fruit with a single ovary consisting of a single carpel

III. Dehiscent fruit is formed by two carpels with a septum


between the carpels and its length is less than three times of its
width
The above statements describe three different fruit types.

II

III

Which of the following combinations are correct for the fruit types?
___A)
___B)
___C)
___D)
___E)

202

I
Siliqua
Legume
Siliqua
Achene
Achene

II
Legume
Siliqua
Achene
Siliqua
Legume

II
Achene
Achene
Legume
Legume
Siliqua

Report of the 11th IBO in Antalya

87. Deleted
88. Deleted
89. Which one of the following is not a characteristic of a
deuterostomian animal ?
___A) Radial cleavage during the embriyonic development
___B) Regulative development during the embriyonic period
___C) Enterocoelom
___D) Pharyngeal slits on the pharynx
___E) Original (evolutionary origin) bilateral symmetry
90. The main reason for echinoderms living only in the sea is because;
___A) They were adapted to be sessile so they do not have a great
distribution
___B) They appear first in the seas
___C) They live in different habitat types in the sea
___D) They have no excretory system
___E) There are safer places in the deep sea
91. Which one of the following structures of earthworms has similar
functions to the liver of vertebrates?
___A) Typhlosolis
___B) Coelomocytes
___C) Chloragogen cells
___D) Cells that line the inner surface of the small intestine
___E) Calcium gland cells

203

Answer Key-Theoretical Test-Part A

92. The metanephridia of annelids and molluscs are functionally and


structurally similar to the vertebrate kidneys. During the
formation of urine, filtration, reabsorption and secretion
processes occur. Where does filtration occur in the nephridium of
mussels?
___A) On the nephrostom in the nephridium
___B) On the cardiac wall and pericardial glands
___C) On the tubules that are connected to the nephrostom
___D) On the wall of the intestine
___E) On the gill capillaries
93. Which one of the following is not a characteristic of molluscs
(Mollusca)?
___A) Mantle
___B) Radula
___C) Trochophore larva
___D) Spiral cleavage
___E) Regulative development
94. Deleted

204

Report of the 11th IBO in Antalya

ANIMAL ANATOMY AND PHYSIOLOGY


95. Deleted
96. Which of the following is an important feature of primitive
aquatic life forms?
___A) Partially oxygenated blood
___B) An open circulation with no small blood vessels or capillaries
___C) Significantly decreased blood pressure
___D) Highly acidic blood
___E) Carriage of most of the O2 in the the plasma
97.
I-

Partial O2 pressure

II-

pH

III- Amount of 2,3-diphosphoglycerate


IV- Partial CO2 pressure
V-

Body temperature

The factors affecting the dissociation of O 2 from hemoglobin in the


human circulatory system are given above. In which of the following
alternatives does this dissociation occur most easily?
___A) I increases

III decreases

IV increases

___B) II increases

V decreases

III increases

___C) I decreases

IV increases

III increases

___D) V increases

IV decreases

I increases

___E) II decreases

III decreases

V decreases

205

Answer Key-Theoretical Test-Part A

98. A nerve-skeletal muscle isolated preparation is placed in a Ca 2+ free medium appropiate for its survival. If the nerve is electrically
stimulated, which of the following events will occur?
___A) The muscle will not be stimulated
___B) The muscle will be stimulated but will not contract
___C) The muscle will be both stimulated and contract
___D) The muscle will not be stimulated, but even if it is, it will not contract
___E) The muscle may be stimulated, and may contract but it will not relax
99. Which of the following is false about the differences between the
vertebrate skeletal muscles and smooth muscles?
___A) Skeletal muscle is more sensitive to electrical stimulation while
smooth muscle is more sensitive to chemical stimulation
___B) Skeletal muscle has a certain length in the resting state; smooth
muscle has not
___C) Smooth muscle contracts more than skeletal muscle after stretching
___D) Skeletal muscle consumes 10% less of the energy than that of
smooth muscle for the same degree of contraction
___E) Without a nerve connection, skeletal muscle cannot function
normally but smooth muscle can

206

Report of the 11th IBO in Antalya

100. The figure shows a feedback system for the control of the output
of blood from the heart (cardiac output). Which of the following
gives the correct description of the parts played by the
components X, Y and Z?
___A)

X
Monitor

Y
Receptor

Z
Effector

___B)

Monitor

Effector

Receptor

___C)

Receptor

Monitor

Effector

___D)

Receptor

Effector

Monitor

___E)

Effector

Monitor

Receptor

207

Answer Key-Theoretical Test-Part A

101.
I-

The magnitude of the impulse is dependent on the size of the


stimulus
II- The number of fibres which are stimulated increases with the
size of the stimulus
III- The speed at which the impulse travels increases with the size
of the stimulus
IV- The speed at which the impulse travels depends on whether or
not the nerve has a myelin sheath
V- The speed of the impulse conduction is directly proportional to
the diameter of the axon
Which of the following is the correct combination of the statements
given above about the nerve conduction?
___A) I, II and III

___B) II, III and IV

___D) III, IV and V

___E) I, III and V

___C) II, IV, and V

102. Deleted
103. The figure shows some parts of a mammalian eye numbered 1-5.
If light suddenly strikes the eye, which of the following will be
the nervous pathway for the evoked unconditional pupil reflex?
(CNS= Central Nervous System)
___A) From 4 to the CNS and then to 3
___B) From 1 to 4 then to the CNS and
then to 2
___C) From 3 to the CNS and back to 3
___D) From 5 to 1 then to 2
___E) From 4 to the CNS and then to 5

208

Report of the 11th IBO in Antalya

104. When an epinephrine (adrenalin)solution is dropped on the


surface of a frog muscle(M. gastrocnemius) in vitro, the muscle
displays a strong contraction. However,when the epinephrine
solution is injected into the muscle cell, nothing happens. Which
of the following is the reason for this?
___A) Epinephrine induced the antagonistic effect inside the cell
___B) Epinephrine induced the side-effect inside the cell
___C) Epinephrine was not processed by proteolytic enzyme
___D) Epinephrine did not find the receptor inside the cell
___E) Epinephrine was degraded inside the cell
105. Which of the following alternatives constitute the cell groups
that function effectively in the human immune system?
___A) T lymphocyte B lymphocyte Macrophage
___B) T lymphocyte Macrophage Erythrocyte
___C) B lymphocyte Kupffer cell Lipocyte
___D) Dendritic cell Neutrophilic leukocyte Fibroblast
___E) Microglia Histiocyte Megakaryocyte

209

Answer Key-Theoretical Test-Part A

106. The left-hand diagram shows a frog sciatic nerve lying across a
number of electrodes. The electrodes A and B are used for
stimulating and C and D for recording. The right-hand diagram
shows a typical recorded action potential. Based on this
information, which of the following statements is correct?

___A) The duration of the recorded action potential (d) will be


independent of the distance between electrodes C and D
___B) The magnitude of the recorded action potential (m) will be
independent of the distance between electrodes C and D
___C) The first deflection on the recording occurs when electrode C is
negative with respect to D
___D) The duration of the recorded action potential will depend on the
distance between B and C
___E) The recorded action potential can be made monophasic by applying
a local anesthetic at A
107. Deleted
108. Which one of the following is incorrectly matched?
___A) Bird Discoidal cleavage Erythrocyte with nucleus
___B) Frog Mesonephrose kidney Holoblastic inequal cleavage
___C) Reptile Viviparous organisms Telolecithal egg
___D) Fish Deuterostomia Radial cleavage
___E) Mussel Protostomia Mosaic development

210

Report of the 11th IBO in Antalya

109. Diagram shows the rates of filtration (F), reabsorption (R) and
excretion (E) of a substance (X) in relation to its plasma
concentration by the kidneys. Which one of the following
statement is incorrect?

___A) The reabsorption of X is dependent on its plasma concentration


___B) The filtration rate of X is directly proportional to its plasma
concentration
___C) When the plasma concentration of X reaches a certain value, its
excretion rate suddenly increases
___D) The concentration of X in the urine is expected to be higher than
its amount filtered in the glomerulus
___E) The filtration rate of X in the glomerulus is fixed
110. Which of the following hormonal conditions of a woman is
suitable in her late pregnancy?
___A) Estrogen increases, progesterone increases
___B) Estrogen decreases, progesterone decreases
___C) Estrogen increases, progesterone decreases
___D) Estrogen decreases, progesterone increases
___E) Luteinizing Hormone increases, Human Chorionic Gonadotropin
increases
111. Deleted

211

Answer Key-Theoretical Test-Part A

112. Deleted
113. Deleted
114.

Chromosome number
2N
N

Cell number
Which of the following can be said according to the graph given
above?
I

Gamete formation in human

II-

Gamete formation in phanerogamia

III- Gamete formation in queen honey bee and the development of


the male bee
IV- Spore formation and development in ferns
___A) I and II

___B) III and IV

___D) I, II and IV

___E) I, II and III

___C) II and IV

115. Which one of the following is true about determination


(developmental fate of cells)?
___A) Differentiation occurs before determination
___B) In animals, the cells that appear after the first two division are
determined
___C) A determined cell will keep its features wherever it is transported
in the embryo
___D) When a cell is determined its structure (shape) will begin to change
___E) A determined cell has the same transcription model as a
differentiated one

212

Report of the 11th IBO in Antalya

BEHAVIOUR
116. In the picture, a flying dummy (silhouette) of a bird is
illustrated. If the dummy is moved over hatched chicks of a
pheasant
(Phasianus
colchicus) from left to right
(upper arrow) or from right
to left (ie. moving backward)
(lower arrow), the chicks will
react as follows (mark the
correct answer with an X):

I.

In both cases the chicks will react to the silhouette by


crouching
II. In both cases the chicks will not react at all
III. During the movement of the silhouette from left to right
(upper arrow) the chicks will not react
IV. During the movement of the silhouette from right to left (lower
arrow) the chicks will not react
V. During the movement of the silhouette from right to left (lower
arrow) the chicks will react by crouching
VI. During the movement of the silhouette from left to the right
(upper arrow) the chicks react by crouching
___A) Only I
___B) Only II
___C) III and V
___D) IV and VI

___E) III and VI

213

Answer Key-Theoretical Test-Part A

117. Animal aggression comes out in several cases and aggresion is


also motivated by various conditions such as an external
stimulus.
Which of the following is not an agressive behaviour?
___A) The behaviour of the prey that is under the threat of being killed
___B) Behaviour that does not reflect the normal behaviour and
specifications of a group
___C) The behaviour against intruder in order to protect their territory
___D) Behaviour towards other animals that try to steal their own food
___E) Behaviour of a predator against its prey
118.

The location of the food-supply


Tail-wagging dance
When honeybees find a food-supply, they can show the exact place of
the supply to other individuals of the colony by a tail-wagging
dance. An example of this behaviour is illustrated above
According to this example, for a honeybee that makes the illustrated
tail-wagging dance below, which one of the following shows the
location of the food-supply ?

__A)

___B)

___D)

___E)

214

___C)

Report of the 11th IBO in Antalya

119.

It is known that some grasshopper species may make a sound in


order to court the opposite sex and these sounds are species specific.
It is also observed that these sounds seem very different when a close
relative species live in the same area.
The sonograms of the mating songs of the five different species of
male grasshoppers from the genus Chorthippus is given above. Which
of the species (I-V) are living together in the same area?
___A) I and II
___B) III and IV
___C) III and V
___D) IV and V
___E) III, IV and V

215

Answer Key-Theoretical Test-Part A

120. When a goose notices an egg outside her nest, she rises, extends
her neck, touches the egg with her beak, and then rolls it back in
very gently. She completes the same recovery behavior whether
the objects she sees is a beer bottle or golf ball, even when the
object is removed after she has begun to reach for it.
Which of the following statements is correct according to the
situation above?
___A) The reason why the goose rolls back the objects that do not
structurally look like an egg is her lack of recognition
___B) This behaviour is caused by instinct
___C) In order to fill her nest to provide suitable incubation conditions
___D) This behavior is learnt from the parents.
___E) The goose recognises her own egg shape. For this reason, egg
rolling is a fixed action pattern and continues without another
stimulus

216

Report of the 11th IBO in Antalya

ECOLOGY
121. Deleted
122. Which one of the following environmental conditions affects the
dispersal trend of a population positively?
___A) The conditions that cause high mortality sometimes create empty
habitats
___B) Very frequent disturbances in the habitat conditions
___C) Absence of suitable habitats very close to each other
___D) A low level of natality causing the differences between the habitats
___E) A breakdown at any level of the food chain
123. Deleted
124. According to the population growth curves given below, which
population has reached equilibrium by responding properly to
negative feedback mechanisms with time?
___A) limit K

___D)
limit K

___B) limit K

___E)
limit K

___C) limit K

217

Answer Key-Theoretical Test-Part A

125. Deleted
126. A mosquito species which lives in hot and highly humid
environment generally chooses little isolated aquatic habitats to
reproduce and completes its larval development. This species
gives many generations by reproducing in late May and early
October. To increase its population in a given area which in the
following is the most important limiting factor for this species
which is very sensitive to the chemical changes in the habitat
water .
___A) Increase in the saturation deficit in the air during the reproductive
season
___B) Predation
___C) Competition with another species in the microhabitat
___D) Increase in the shadow factor
___E) Increase in relative humidity
127. Which one of the following explanations cannot be given about
the relationship between the carrying capacity and the
environmental response of populations with a high density?
___A) Competition increases
___B) The natality (birth) rate decreases
___C) The negative feedback mechanism works
___D) The environmental response decreases
___E) The mortality rate increases
128. Deleted
129. Deleted.
130. Deleted

218

Report of the 11th IBO in Antalya

131. Which one of the following cannot be said about the distribution
of populations?
132. A park was built in a place that was occupied by a lot of
speciesAtrees. A long time ago they were all cut down no
species A trees remained. Later gardeners planted more
species A trees and also species B trees and species C
trees had species B and C never grew in that region before.
Nobody took care of this garden. After 100 years there were a lot
of new A trees and B trees, but no young C trees . Which
processes refer to the A, B and C trees in that park?
A

___A) Introduced,

Acclimatized,

Reacclimatized

___B)

Acclimatized,

Introduced,

Reacclimatized

___C)

Introduced,

Reacclimatized,

Acclimatized

___D) Reacclimatized,

Acclimatized,

Introduced

___E)

Reacclimatized,

Introduced

Acclimatized,

219

Answer Key-Theoretical Test-Part A

133. The graph represents the changes in the biomass, diversity and
primary production in the ecological succession. Which blocks
on the graph represent the first settler stages and the climax
stage, respectively.
I

II

III

Years
Primary production
Biological diversity
___A) I and II
___B) II and III
___C) I and III
___D) I, II and III
___E) None

220

Biomass

Report of the 11th IBO in Antalya

134. An ecologist wants to investigate if there are any differences in


the vegetation on the north and south facing sides of a valley.
She lays down a rope from the top to the bottom of the slope and
every 2 meters she places a 1 m2 quadrat next to the rope.
Standing above the quadrat she estimates and records the area
occupied by each plant species. This technique involves which of
the following?
1. The use of a point quadrat
2. The recording of % cover
3. The plotting of the results on a kite diagram
4. Random sampling
5. The use of a belt transect
___A) 1 & 2
___B) 2 & 3
___C) 3, 4 & 5
___D) 1, 2 & 3
___E) 2, 3 & 5
135. Deleted

221

Answer Key-Theoretical Test-Part A

4. 4. Answer Key to the Theoretical Test


4.4.1. Part A
1. D
2. E
3. B
4. C
5. Deleted
6. D
7. A
8. D
9. B
10. C
11. B
12. B
13. C
14. A
15. C
16. D
17. E
18. C
19. B
20. D
21. C
22. E
23. A
24. E
25. B
26. C
27. C
28. E
29. D
30. A
31. D
32. A
33. D
34. D

222

35.
36.
37.
38.
39.
40.
41.
42.
43.
44.
45.
46.
47.
48.
49.
50.
51.
52.
53.
54.
55.
56.
57.
58.
59.
60.
61.
62.
63.
64.
65.
66.
67.
68.

A
C
Deleted
B
Deleted
E
B
A
C
C
A
Deleted
B
E
B
Deleted
E
A
A
Deleted
C
E
B
E
Deleted
E
Deleted
D
D
Deleted
Deleted
Deleted
Deleted
E

69. B
70. C
71. D
72. E
73. E
74. C
75. Deleted
76. D
77. B
78. E
79. D
80. E
81. Deleted
82. D
83. D
84. Deleted
85. Deleted
86. E
87. Deleted
88. Deleted
89. D
90. D
91. C
92. B
93. E
94. Deleted
95. Deleted
96. C
97. C
98. A
99. D
100. C
101. C
102. Deleted

103. A
104. D
105. A
106. A
107. Deleted
108. C
109. E
110. A
111. Deleted
112. Deleted
113. Deleted
114. B
115. C
116. C
117. C
118. A
119. A
120. E
121. Deleted
122. A
123. Deleted
124. C
125. Deleted
126. A
127. D
128. Deleted
129. Deleted
130. Deleted
131. A
132. D
133. A
134. E
135. Deleted

Report of the 11th IBO in Antalya

77















All IBO examination questions are published under the following Creative Commons license:



CC BY-NC-SA (Attribution-NonCommercial-ShareAlike) https://creativecommons.org/licenses/by-nc-sa/4.0/
The exam papers can be used freely for educational purposes as long as IBO is credited and
new creations are licensed under identical terms. No commercial use is allowed.

Report of the 11th IBO in Antalya

4.3.2. Theoretical Test - Part B


CELL BIOLOGY
1. Deleted
2. Deleted
3. Deleted
4. In the left column below, you can see some proteins and in the right
one there are some protein functions (1 8). Match the functions
with the proteins by writing their numbers in the blanks. (A
protein may have more than one function) (7 points).
___

Dynein

___

1. Shows channel protein characteristics


2. Possesses ATPase activity

___

Na -K ATPase
Nexin

___

Connexon

4. Transport protein

___

Porin

5. Ion transport protein

___

Keratin

6. Attaches the microtubules

___

Desmin

7. Attaches Z bands to the myofibers in muscle


cells
8. Exists in the cytoskeleton of epithel cells

3. Fasciliates transport through membrane

5.
A) (5') A G C C T A A T G G C C T A (3')
B) (3') T C G G A T T A C C G G A T (5')
The DNA above is replicated in the direction of the arrow. Write the
appropriate letter showing the templates for leading strand and
lagging strand synthesis in the blanks. (2 points)
template for the lagging strand _________
template for the leading strand _________

129

Theoretical Test - Part B

6.
A
C

E
D

F
B

A suitable substrate for DNA polymerase is shown above. Fill in each


blank below with a corresponding letter on the diagram. (3 points).
Primer

_________

Template

_________

3 end of the primer

_________

5 end of the primer

_________

3 end of the template strand

_________

5 end of the template strand

_________

7. Two molecules of DNA (I and II) are the same size (1000 bp) but
differ in base composition. The first one contains 42% and the
second one 66% A+T. (1.5 points).
A) How many G residues are there in DNA I and II? (1 point).
I: _______
II: _______
B) Which molecule (I or II) has a higher Tm. (Tm =dissociation
point) (0.5 point)
_______

130

Report of the 11th IBO in Antalya

8. Match the enzymes involved in the procaryotic replication at the


left with their function at the right by putting the appropiate
numbers in the blanks. (3 points).
DNA Helicase_________
Primase__________

1. Synthesis of RNA primers in the


replication of the lagging strand
2. Unbinds double stranded DNA

DNA polymerase I

3. Removes RNA primers.

3 5 exonuclease
activity_________
DNA Ligase_________

Topoisomerase II_________

4. Seals nicks in the DNA at the


boundaries between Okazaki
fragments.
5. Removes mismatched bases

DNA Polymerase I
53 exonuclease
activity_________

6. Releaves the topological stress


produced by the unwinding of double
stranded DNA.

131

Theoretical Test - Part B

9. Below is a diagram that shows DNA replication. On the diagram,


mark: (5.4 points).
A) 3' ends with the letter a and 5' ends with the letter b, (2
points).

B) The lagging strand with letter A, the leading strand with


letter B , Okazaki Fragments with letter C, and RNA
primers with letter D. (1.4 points).

C) Match the enzymes with the reaction it catalyzes. Put the letter
in front of the enzyme in the appropriate blanks below. (2
points).
E. Primase
F. Ligase
G. DNA Polymerase II
H. DNA Polymerase III
I. DNA Polymerase I
- Enzyme ___________catalyzes the synthesis of fragment I
- Enzyme ___________catalyzes the synthesis of fragment II
- Enzyme ___________catalyzes the synthesis of RNA primer
- Enzyme ___________seals the nick shown as III in the diagram

132

Report of the 11th IBO in Antalya

10. Compare RNA polymerase with DNA polymerase III that


function in the transcription and replication processes in E. coli
on the basis of the parameters (A-H) with their characteristics
given in the table. Put the letters in the appropriate boxes. (5
points).
A) Promotor

B) Origin

C) 35

D) 53

E) dNTP

F) NTP

G) Yes (+)

H) No (-)

RNA
Polymerase

DNA
Polymerase III

The DNA region initially


recognized and bound by the
polymerase
The direction of the
polymerization
The direction of enzyme
movement on the template strand
The type of the nucleotide
substrates added to the growing
chain
35 exonuclease activity
(Proof reading ability)
11. Deleted
12. Deleted

133

Theoretical Test - Part B

13. For each of the following statements, indicate with a P if the


statement applies only to prokaryotes, with an E if the
statement applies only to eukaryotes, and with an E-P if the
statement applies to both eukaryotes and prokaryotes. (2.7
points).
___ A single RNA polymerase transcribes genes that encode mRNA,
tRNA and rRNA.
___ Polimerisation of DNA is in the 5` 3`direction.
___ Sigma () subunit detaches from RNA polymerase shortly after
transcription has initiated
___ The 5 end of the mature mRNA begins with a triphosphate
___ Polymerisation of RNA is in the 5` 3` direction
___ They carry circular DNA
___ There are no introns in mRNA
14. The template strand for mRNA is given below. (5 points).
* (3)
(5) CTT TGA TAA GGA TAG CCC TTC
A) What is the base sequence of the mRNA that can be transcribed
from this strand?
________________________________

B) Using the genetic code table given on the next page, write the
amino acid sequence of the polypeptide coded by this mRNA.
__________________________
C) Suppose the other (complementary) strand is used as a template
for the transcription. What is the amino acid sequence of the
resulting peptide?
__________________________
D) If the labeled base above in the template strand is converted to
A instead of T, what would be the type of the mutation?
Transition (X), transversion (Y), deletion (Z) or insertion (W)
Write the correct letter on the line below.
_________________________________

E) What is the type of this mutation? Neutral (N), silent (S),


missense(M), or nonsense (NS)? Write the correct letter on the
line below.
__________________________
BASE II

134

Report of the 11th IBO in Antalya

BASE
I
U

UUU

phe

UUC

UUA

A
UAU

G
tyr

UGU

IBASEI
II

UCU

ser

cys

phe

UCC

ser

UAC

tyr

UGC

cys

leu

UCA

ser

UAA

stop

UGA

stop

UUG

leu

UCG

ser

UAG

stop

UGG

trp

CUU

leu

CCU

pro

CAU

his

CGU

arg

CUC

leu

CCC

pro

CAC

his

CGC

arg

CUA

leu

CCA

pro

CAA

gin

CGA

arg

CUG

leu

CCG

pro

CAG

gin

CGG

arg

AUU

ile

ACU

thr

AAU

asn

AGU

ser

AUC

ile

ACC

thr

AAC

asn

AGC

ser

AUA

ile

ACA

thr

AAA

lys

AGA

arg

AUG

met

ACG

thr

AAG

lys

AGG

arg

GUU

val

GCU

ala

GAU

asp

GGU

gly

GUC

val

GCC

ala

GAC

asp

GGC

gly

GUA

val

GCA

ala

GAA

glu

GGA

gly

GUG

val

GCG

ala

GAG

glu

GGG

gly

15. Deleted
16. Deleted

135

Theoretical Test - Part B

17.
A) The heart muscle while working aerobically
B) A bacterium culture that cannot grow in the dark
C) A propionic acid bacteria that can grow under anaerobic
conditions in the dark
D) Erythrocytes
Fill in the blanks with the suitable letters above.(2 points)
_____ provides ATP requirement by photosynthesis
_____ provides ATP requirement by converting glucose to lactate
_____ provides ATP requirement by fermentation
_____ provides ATP requirement by oxidative phosphorylation
18. Atmospheric nitrogen (N2) is chemically very stable. Only a few
prokaryotic species can convert atmospheric nitrogen into usable
form by plants. One of the characteristics of these organisms is
that they possess the nitrogenase enzyme complex that can fix the
nitrogen. O2 irreversibly inhibits this enzyme. Considering that
we live in an oxidizing environment, nitrogen fixing organisms
must have a variety of mechanisms for protection from O2.
A) Below are given some characteristics of bacteria. Put an X in
the blank for the ones that can fix nitrogen. (4 points).
_____A free living bacterium under soil like Clostridium
_____Cyanobacteria like Nostoc muscorum possess thick cell walls
_____Bacteria like E.coli that are inhabitants of the intestinal tract
_____Symbiotic bacteria like Rhizobium of leguminous plants that
possess specialized protein leghemoglobin
_____High mutation rate bacteria like Salmonella typhimurium
B) Which is the major product of the nitrogenase enzyme complex?
(Mark with an X)
_____ammonia

136

_____nitrite

_____nitrate

_____nitrogen gas

Report of the 11th IBO in Antalya

GENETICS AND EVOLUTION


19. Consider cases of unknown paternity where the ABO blood group
phenotype of concerned individuals is to be used to help identify
fathers. The frequency of blood group allels in the population is
as follows: p (A) = 0.2, p (B) = 0.3, p (O) = 0.5. Blood group
assessments are made by routine laboratory procedures, which
can assess blood group phenotypes. (4 points)
a. In a case where the mother`s blood group is A and the child`s
blood group is AB, what is the probability that a man chosen
at random from the population will be proven not to have
fathered the child purely on the basis of his blood group. (2
points)
Answer:_______
b. In a case where the mother`s blood group is A and the child`s
blood group is O, what is the probability that a man chosen
randomly from the population will be proven not to have
fathered the child purely on the basis of his blood group. (2
points)
Answer:_______
20. In a certain human population 64% is able to roll their tongue.
This ability is based on a dominant allele.
A roller marries a non-roller.
Calculate the chance of having a roller baby. (4 points)
Answer:__________________%
21. Deleted

137

Theoretical Test - Part B

22. In peas, the allele for green seed color (A) is dominant over the
allele for yellow seed color (a) and the allele for normal leaf (B) is
dominant over the allele for rolled leaf (b). The F 1 generation is
obtained by crossing AABB x aabb. When these F1 plants are
tested again, the following results are found;
117 green seed / normal leaf
115 yellow seed / rolled leaf
76 green seed / rolled leaf
80 yellow seed / normal leaf

2 = (O-E)2 / E
O: Observed value

E: Expected value (6 points)

A) Degree of freedom: ________


B) Calculate the value and check from the table: ________
2

C) According to the result of choice B, decide whether these genes are


linked or not (mark with the letter X)
Genes are linked: ________
Genes are not linked: ________
D) If these genes are linked, calculate the distance between the genes
The distance between the genes is ________ map units

138

Report of the 11th IBO in Antalya

23. (2.5 points).


DIRECTION OF THE LIGHT

According to the structure and the location of the light sensors


illustrated on the scheme above, match the followings
Animal group
________Earthworm
________Squid
________Human being
________Insect
________Planaria

139

Theoretical Test - Part B

PLANT ANATOMY AND PHYSIOLOGY


24. (2 points)

Root water
conductivity

3
4
PHASE

The graph represents the water conductivity of a root. Mark the true
alternative(s) with an X.
___A) The temperature decreases between the phases 1-2 and 4-5
___B) CN- is added to the medium between the phases 0-1 and 2-4
___C) There is enough oxygen between the phases 1-2 and 4-5
___D) The root cannot get enough nutrient between the phases 1-2 and 45

140

Report of the 11th IBO in Antalya

25. (3.6 points)


Critical night
duration

Dark
I
A

Light
II

Critical night length

B
RF
C
RF

FRF

RF

FRF

RF

RF

FRF

RF

D
E
FRF

F
0

12

RF : red or white flash

16

20

24

FRF: far red flash

The figure represents exposure of long-day and short-day plants to a


variety of light regimes. Decide whether long-day (short-night) and
short-day (long-night) plants will flower (+) or not (-) under the
conditions given above.
Long day (short-night) plant

Short-day (long-night) plant

A)
B)
C)
D)
E)
F)

II

26. Deleted

141

Theoretical Test - Part B

27. Deleted
28. Deleted
29. Deleted
30. Label the figure using the numbers (2.1 points)
01 Anther

05 Phyllary

02 Ovarium

06 Nectar

03 Stigma

07 Pappus

04 Corolla

31. A student is studying a tundra plant at different temperatures.


He investigates:

142

Report of the 11th IBO in Antalya

The production of oxygen by means of photosynthesis, and


The net amount of oxygen released into the environment
The diagram shows the results. It is up to you to decide which
curve is production and which curve is release. (2 points)

II

143

Theoretical Test - Part B

Now answer the following two questions:


31.1. At which temperature(s) are both the ATP-production and the
ATP-consumption approximately equal? (1 point)
___A) At 20 oC
___B) At -2,5 oC and at +40 oC
___C) Only in between -10 oC and -2,5 oC
___D) Only in between -10 oC and + 10 oC
___E) In between -10 oC and + 40 oC
31.2. At which temperature(s) are the production of oxygen by
photosynthesis and the consumption of oxygen by dissimilation
equal to each other? (1 point)
___A) At 20 oC
___B) At -2,5 oC and at +40 oC
___C) Only in between -10 oC and -2,5 oC
___D) Only in between -10 oC and + 10 oC
___E) In between -10 oC and + 40 oC

144

Report of the 11th IBO in Antalya

32. Select numbers from the answer key and mark the parts of the
different fruit types (3 points)
Answer key:
1-

Remains of sepals

2-

Receptacle

3-

Seed

4-

Placenta

5-

Pedicel

33. Deleted
34. Deleted

145

Theoretical Test - Part B

35. Which of the following can be expected to happen when glucose is


added to the mineral solution in which plant cells are bathed. Put
a cross (X) in the related boxes.
35. 1. (1 point)

Increases

Decreases

Membrane potential
pH of the medium

35. 2 (1 point)

Taken up
Glucose

146

Not taken up

Report of the 11th IBO in Antalya

BIOSYSTEMATICS
36. Deleted
37. Deleted
38. Which of the following descriptions about vertebrates is/are
correct? Mark the correct ones with an X. (2.5 points)
______The scales of fish are epidermal scales
______The scales of snakes are epidermal scales
______The scales of lizards are epidermal scales
______The scales of pangolins are epidermal scales
______The hairs of humans are derivatives of epidermis
______The horns of deers are epidermal in origin
______The horns of cattles are epidermal in origin
______The horns of rhinoceros are epidermal in origin
39. Deleted
40. Deleted
41.

I. Rhynia
IV. Lycopodium

II. Spirogyra
V. Equisetum

III. Rhizopus/Mucor

Establish the relationships between the given genera names above


with the terms below (You can use any term more than once.) (3.6
points).
______ It appeared in the Carboniferous period
______ The sporangia are generally cluster shaped
______ The zoospores never appear
______ The oldest plant that has no leaves
______ The first real roots are seen in this group
______ The prothallus reaches sexual maturity in 12-15 years time
______ The chloroplasts are helozonic (spiral shaped)
______ Xylem is made of ringed and spiral tracheids
______ In the cross section, the xylem appears star or plate shaped
42. Deleted

147

Theoretical Test - Part B

ANIMAL ANATOMY AND PHYSIOLOGY


43. Some animals and some of their characteristics are shown in the
table . Match these characteristics correctly with the given
animals (mark the appropriate box with an "X") (3.6 points).

Excretes
hipertonic urine
relative to the
body fluids

148

Terrestrial
mammal

Excretes
isotonic urine
relative to the
body fluids

(Mouse)

Excretes
hipotonic urine
relative to the
body fluids

Marine
mammal

Actively
absorbs salt

(whale)

Actively
secretes salt

Lizard

Wastes are
discarded as
uric acid

(Chameleon)

Wastes are
discarded as
urea

Marine fish

Wastes are
discarded as
ammonia

(sea bass )

Does not drink


water

Bird (pigeon)

Fresh water
fish (carp)

Drinks water
regularly

Report of the 11th IBO in Antalya

44.

In the figure, some parts of a mammalian nephron are numbered (IV). Match these numbers with the events or properties given in the
table (a number can be used more than once). (2.5 points).
Cl- is actively pumped out
Blood is filtered
Almost all glucose is reabsorbed
Urine becomes acidic
Na+ is reabsorbed under aldesterone control

149

Theoretical Test - Part B

45. In humans, some mechanisms are activated in the case of a serious


decrease in the red blood cell count. Some sources (organs/tissues),
secreted substances, targets and biological responses are given in
the list (1-13). Examine them and put appropriate numbers in the
appropriate boxes in the table. (2 points).
1. Liver
2. Kidneys
3. Heart
4. Erythopoeitin
5. Lungs
6. Spleen
7. Bone marrow
8. Antidiuretic hormone
9. Renin
10. Androgens
11. Adrenaline
12. Increase in erythropoiesis
13. Increase in the blood glucose level

Stimulus

Decreased red
blood cell count

46. Deleted

150

Stimulated

Secreted

organ/tissue

Substance

Target Biological
Response

Report of the 11th IBO in Antalya

47. In the figure, 4 parameters varying according to the different


parts of the human circulation system (aorta, arteries, arterioles,
capillaries, venules, veins, and vena cava) are plotted (I-IV).
Match the numbers of the curves with the parts of the circulation
system (Put the appropriate number in front of each part.). (2
points).

______ Total cross-sectional area


______ Blood pressure
______ Blood velocity
______ Vessel diameter

151

Theoretical Test - Part B

48. Inspect the following scheme representing the blood circulation of


a human embryo just before birth. The numbers respresent the
blood flow in ml/min per kg body mass of the embryo. (2 points).

48. 1. Indicate the letter of the box which represents the placenta. (1
point).
Answer:________(fill in a letter)
48. 2. Calculate the ratio of blood flow through the lungs just before
and few days after the birth, assuming that the total amount of
blood leaving the heart at both are equal. (1 point).
Answer: (Ratio) Before / After =__________

152

Report of the 11th IBO in Antalya

49. In the human circulation various mechanisms are activated when


blood pressure decreases below or rises above its normal level.
This question is related to a situation where the blood pressure
exceeds its normal level. Indicate the events that take place at
various parts of the circulation system to return the pressure back
to its normal level by marking the appropriate boxes in the table
with an "X" .(3 points).

Stretch receptors

Cardioaccelatory

Cardioinhibitory

center

center

inhibited

stimulated

inhibited

stimulated

inhibited

increases

stimulated

inhibited

stimulated
Cardiac output

Vasomotor
center

Arterioles
decreases

constricted

dilated

50. Deleted

153

Theoretical Test - Part B

51.

Tissue cells

Plasma

Red
blood
cell

The figure shows the reactions occuring during gas and electrolyte
exchange between blood capillaries loaded with O 2 and the tissue
cells. Fill in the empty boxes in the figure and below with the
appropriate numbers out of the 13 substances given below. (3.5
points).
1. Hb (hemoglobin)

6. H2PO4

11. HCO3

2. H2CO3

7. H2SO4

12. HbO2

3. H2O

8. Cl-

13. HHb

4. CO2

9. Na

5. NaHCO3

10. Ca2+

154

II

III

IV

VI

VII

Report of the 11th IBO in Antalya

52. The respiratory quotient RQ of an organism is defined as


RQ = CO2 (produced)/O2(used) .........(in a given time)
The theoretical RQ values of important substrates are approximately:
substrate

RQ

(completely oxidised)
Carbohydrate

1,0

Fat

0,7

Protein

0,9

In practice, the values of RQ will be higher or lower than the


theoretical ones.
What is the effect of the following circumstances on RQ? (2 points).
Put an X in the correct boxes in the following table.
RQ higher

RQ lower

Anaerobic respiration of substrate


Incomplete oxidation of substrate
Fixation of CO2 as CaCO3
Converting carbohydrate into fat

155

Theoretical Test - Part B

53. Figure shows 5 saturation curves of O 2 with hemoglobin each


obtained from a different animal (I-V). The shape of these curves
differ according to the basal metabolism of the animal. Match
these curves with the animals given below (Put the number of the
curve in front of the name of the animal.) (2.5 points).
_____ Elephant

_____ Snake

_____ Man

_____ Mouse

156

_____ Bird (sparrow)

Report of the 11th IBO in Antalya

54. Mark the correct change in the arterial chemoreceptors,


respiratory rate, H+ excretion rate in the kidneys and blood
partial CO2 pressure that takes place in order to correct a drop
in blood pH. (2 points).

Arterial chemoreceptors

Respiratory rate

stimulation

increase

inhibition

decrease

H+ excretion in the kidneys

Blood partial CO2 pressure

increase

increase

decrease

decrease

55. Write the numbers which refer to the extra-embryonic


membranes amnion (1), allantois (2), yolk-sac (3) and chorion (4)
that are seen during the development of organisms given below.
(1.4 points)
Fish

______

Frog

______

Reptile

______

Bird

______

Mammal

______

157

Theoretical Test - Part B

56. The following statements are about calcium and its regulation in
humans. Match correctly the substances given in the answer key
with the statements (put the letter of the substance in front of the
statements). (2 points).
_____It is the vitamin which promotes the accumulation of calcium in the
body
_____It is the gland which secretes calcitonin, the hormone causes
calcium accumulation
_____It is the the place where calcium accumulates in great amounts in
the body
_____It is the gland which secretes hormone that increases the calcium
level in the blood
Answer key:
A. Vitamin D
B. Bones
C. Thyroid
D. Blood
E. Parathyroid gland
F. Vitamin C
G. Adrenal gland

158

Report of the 11th IBO in Antalya

57. Some human endocrine and exocrine glands are numbered in the
figure . In the following statements some functions related to these
glands are given. Match the statements with the glands in the
figure (Put the number of the gland in front of the statements). (3
points)

_____It secretes a hormone which increases the reabsorption of Na + into


the blood
_____Its secretion is increased when blood Ca2+ concentration drops
below its normal level
_____If its secretion is decreased the basal metabolic rate also decreases
_____Its secretion is necessary for the development of cellular immunity
_____Its hormone induces red blood cell production in bone marrow
_____Without its hormone, there will be an excessive water loss from the
body
_____Its secretion is increased after a carbohydrate rich meal
_____Acidic compounds stimulate its hormone secretion
______Its secretion is necessary for the chemical breakdown of proteins

58. Deleted

159

Theoretical Test - Part B

59. Deleted
60. Deleted

ECOLOGY
61. Deleted
62. A food web including 6 different species (A to F) in an ecosystem
is shown in the figure illustrated below. The arrows refer to the
energy flow directions. Match the following according to that
figure. (3.5 points).
B

C
F
D

Producer species
Decomposer species
Consumer species on the first trophic level
Consumer species on the second trophic level
Consumer species on the third trophic level
The species in which biomagnification is seen at the highest level

160

_____
_____
_____
_____
_____
_____

Report of the 11th IBO in Antalya

63. Age distribution in human populations can be shown in three


different types as developing type, stable type and regression
type. Mark the stable type age distribution among the following
figures. (1 point).

161

Theoretical Test - Part B

64. In the tables below, one might see the relationships among the
populations and the results of these influences. Mark the correct
answer(s) by an ellipse in each box. For clarity, an example has
been given for competition. (3.5 points).
Competition
Species
A
B
+
0
0
+
+
+
0
0

Predation
Species
A
B
+
0
0
+
+
+
0
0

Parasitism
Species
A
B
+
0
0
+
+
+
0
0

Neutralism
Species
A
B
+
0
0
+
+
+
0
0

Mutualism
Species
A
B
+
0
0
+
+
+
0
0

Commensalism
Species
A
B
+
0
0
+
+
+
0
0

Amensalism
Species
A
B
+
0
0
+
+
+
0
0

162

Report of the 11th IBO in Antalya

65. In the figure illustrated below the energy flow between the
organisms on different trophic levels in a food web is shown. On
the scheme given below, please fill in the blanks with the numbers
given below for the producers, and 1o, 2o, 3o trophic level
consumers according to their levels. (3.6 points).

Producers
0

_____

2 consumers _____

10 consumers
0

3 consumers

_____
_____

163

Theoretical Test - Part B

66. Deleted
67. The figure given below represents the carbon cycle. Fill in the
blanks with the suitable numbers that corresponds to the
processes concerning the cycle given to you. (2 points).

Processes:
Combustion
Consumption
Death
Photosynthesis
Respiration
Decomposition

164

______________
______________
______________
______________
______________
______________

Report of the 11th IBO in Antalya

68. In the table, the principle components that form an aquatic


ecosystem are given. Put the organisms, and components, in their
places in the table given below. (2 points).
I.

Fungi

II.

Phytoplankton

III.

Inorganic components

IV.

Zooplankton

The part of the ecosystem

The organism or component

Abiotic substance
Producer
Consumer
Decomposer
69. Deleted
70. The density of a population that reaches equilibrium is known as
the carrying capacity in that species for that habitat. When a
population approaches the carrying capacity of any habitat,
which of the following shows a tendency to increase? Mark all
correct answers with an X. (2.5 points).
________ Competition for resources
________ Competition for shelters
________ Competition for mating areas
________ Immigration
________ Accumulation of toxic wastes

165

Theoretical Test - Part B

71. In any habitat that includes a predator species and its prey, it is
known that both of their populations show linked fluctuations.
Show the curve belonging to the predator species by putting an
X in the circle. (1 point).

N1
N

N2

Time

166

Report of the 11th IBO in Antalya

BEHAVIOUR
72. Thorleif Schjelderup- Ebbe reported the results of a study about
the social organization carried out in a poultry yard with Leghorn
hens . He found that there was a kind of order (arrangement) in
the peck that was related with a real hierarchy in the group.
The following matrix shows the peck frequency within a group of
13 females (from A to M). Each datum indicates the times that the
hen identified by the letter in the horizontal line is pecked by one
in the vertical line. (4 points)
M

A 53

45

38

51

35

36

41

29

33

34

41

39

B 42

34

37

28

36

29

40

46

43

53

47

C 36

29

26

44

31

38

24

42

37

32

D 35

27

39

29

36

52

43

31

26

E 48

30

27

43

41

40

36

35

F 43

39

42

40

39

33

31

G 39

38

28

36

41

39

H 35

52

47

37

37

41

42

54

33

29

31

32

K 42

37

L 39

M -

72.1. Which of the following female has the highest hierarchy within
the group? (2 points)
___A) A

___B) B

___C) J

___D) H

___E) M

72.2. Deleted
72.3. Deleted

167

Theoretical Test - Part B

72.4. Which ones of the following may be the advantages of a


hierarchy of dominance? (2 points).
___A) To suppress the aggression.
___B) To diminish the time and the energy invested in fights.
___C) To diminish the mortality of individuals because of the wounds
caused in the combat
___D) A and B.
___E) All the above
73. The coefficient of relatedness ( r ) between various kin pairs
that changes according to the relationships in any diploid animal
((For example a mammalian like Canis lupus (Canidae)) is given
in the table below. (3.5 points).
Parent Offspring

0.50

Identical twins

1.00

Grandparent Grandchild

0.25

First cousins

0.125

Uncle nephew

0.25

73.1. In view of this, in a haplodiploid bumblebee species Bombus


terrestris L, 1758 (Apoidea: Hymenoptera), show the coefficient
of relatedness ( r ) in the situations given below. (2.5 points).
Mother Daughter
________
Father Daughter
________
Mother Son
________
Sisters
________
Brothers
________

168

Report of the 11th IBO in Antalya

73.2. If you take into consideration the knowledge given in the table
above, which of the following statements about the sterilization
of the worker bees given as I, II, III and IV is true? (1 point).
I-

For the transmission of the genetic knowledge to the next


generation, the sterilization of the worker bee, is harmful
for it.

II-

For the transmission of the genetic knowledge to the next


generation, the sterilization of the worker bee, is benefical
for it.

III- For the transmission of the genetic knowledge to the next


generation, there is no difference on the individual level.
IV- This is an example of altruism.
___A) Only I
___B) Only IV
___C) II and III
___D) II and IV
___E) III and IV

169

Theoretical Test - Part B

4.4.2. Part B Answer Key


1. Deleted
2. Deleted
3. Deleted
4.
2, 6
2, 3, 4, 5
6
1, 3, 5
1, 3, 5
8
7

5.
A
B
6.
A
B
C
E
D

170

Report of the 11th IBO in Antalya

F
7.
A) I: 580, II: 340
B) I
8.
DNA Helicase___2___
Primase___1___
DNA polymerase I
3 5 exonuclease activity___5____
DNA Ligase___4____
Topoisomerase II___6___
_
DNA Polymerase I
53 exonuclease activity____3____

9.
A)

171

Theoretical Test - Part B

B)

C)
H
H
E
F
10.
A

172

Report of the 11th IBO in Antalya

11. Deleted
12. Deleted
13.
P
E, P
P
P
E, P
P,E
P
14.
A) 5'GAA GGG CUA UCC UUA UCA AAG
B) Glu-Gly-Leu-Ser-Leu-Ser-LysC) Leu-stop-stop
D)
E)

173

Theoretical Test - Part B

15. Deleted
16. Deleted
17.
B
D
C
A
18.
A)
__X__
__X__
_____
__X__
_____
B)
__X__ammonia
_____ nitrite
_____ nitrate
_____ nitrogen gas
19.
a. 0.49
b. 0.25
20.
5/8 or 62.5%

174

Report of the 11th IBO in Antalya

21. Deleted
22.
A) 3
B) 14.98 or 15.0 > 7.815 - 387.45 or 387 > 7.815
C) Genes are linked
D) Deleted
23.
1
4
5
3
2
24.
A, D
25.
A)

B)

C)

D)

E)

F)

175

Theoretical Test - Part B

26. Deleted
27. Deleted
28. Deleted
29. Deleted
30.

31.
31.1. E
31.2. B

176

Report of the 11th IBO in Antalya

32.
1
2
3

5
5

2
1

33. Deleted
34. Deleted
35.
35.1.
Increases
Membrane

Decreases
X

potential
pH of the medium

35. 2.
Taken up
Glucose

Not taken up

36. Deleted
37. Deleted

177

Theoretical Test - Part B

38.
______
___X__
___X__
___X__
___X__
______
___X__
___X__
39. Deleted
40. Deleted
41.
V
III
II
I
IV
IV
II
I
IV
42. Deleted

178

Report of the 11th IBO in Antalya

43.

X
X

Terrestria
l mammal
(Mouse)

Marine
mammal
(whale)

Marine fish
(sea bass )

Lizard
(Chamele
on)

Bird
(pigeon)
Fresh
water fish
(carp)
Drinks water
regularly
Does not
drink water
Wastes are
discarded as
ammonia
Wastes are
discarded as
urea
Wastes are
discarded as
uric acid
Actively
secretes salt
Actively
absorbs salt
Excretes
hipotonic
urine relative
to the body
fluids
Excretes
isotonic
urine relative
to the body
fluids
Excretes
hipertonic
urine relative
to the body
fluids

X
X

X
X

179

Theoretical Test - Part B

44.

II
I
III
IV
V

45. Stimulus

Stimulated Secreted Target


organ/tissue substance

Decreased red
blood cell count

Biological
Response

12

46. Deleted
47.

IV
I
III
II

48.1. G
48.2. 1:7
49.
Stretch
receptors

Vasomotor
center
inhibited

stimulated

inhibited

Cardioinhibitory
center
stimulated

inhibited

stimulated

180

inhibited

Stimulated
X

Cardioaccelatory
center

Report of the 11th IBO in Antalya

Cardiac output
increases

Arterioles

decreases constricted

dilated

50. Deleted
51.
1

13

11

II

III

IV

VI

VII

52.
RQ
higher

RQ lower

X
X
X
X
53.
II
V
IV
III
I

181

Theoretical Test - Part B

54.
Arterial chemoreceptors

Respiratory rate

stimulation

increase

inhibition

decrease

H+ excretion in the kidneys

Blood partial CO2 pressure

increase

increase

X
55.
3
_______
1, 2, 3, 4
1, 2, 3, 4
1, 2, 3, 4
56.
A
C
B
E
57.
5
3
2

182

decrease

decrease
X

Report of the 11th IBO in Antalya

4
9
1
7
8
6
58. Deleted
59. Deleted
60. Deleted
61. Deleted
62.
A
E
B,D
C
F
F
63.

183

Theoretical Test - Part B

64.
Competition
Species
A
B
+
0
0
+
+
+
0

Neutralism
Species
A
B
+
0
0
+
+
+
0

Predation
Species
A
B
+
0
0
+
+
+
0

Mutualism
Species
A
B
+
0
0
+
+
+
0

Amensalism
Species
A
B
+
0
0
+
+
+
0

184

Parasitism
Species
A
B
+
0
0
+
+
+
0

Commensalism
Species
A
B
+
0
0
+
+

+
-

0
0

Report of the 11th IBO in Antalya

65.

12
7, 8, 9, 10, 11
1, 2, 3, 4, 5, 6
1, 2, 3, 5

66. Deleted
67.

9
3
4,5,7
1
2,8,10
6

68.

III
II
IV
I

69. Deleted

185

Theoretical Test - Part B

70.

__X__
__X__
__X__
_____
__X__

71.

N2

72.1.

72.2.

Deleted

72.3.

Deleted

72.4.

73.1. 0.50
1.00
0.50
0.75
0.50
73.2. D

186

INTERNATIONAL BIOLOGY OLYMPIAD


ALL PROBLEMS

1998, Kiel, Germany















All IBO examination questions are published under the following Creative Commons license:



CC BY-NC-SA (Attribution-NonCommercial-ShareAlike) https://creativecommons.org/licenses/by-nc-sa/4.0/
The exam papers can be used freely for educational purposes as long as IBO is credited and
new creations are licensed under identical terms. No commercial use is allowed.

Eckhard Klieme
Eckhard R. Lucius
Erwin Zabel (Eds.)

Report of the
9th International Biology Olympiad

ipn
ipnmaterialien

Theoretical Test Part A

Horst Binding, Christian Gliesche, Ludger Kappen, Eckhard Klieme,


Ralf Kittler, Sievert Lorenzen, Eckhard R. Lucius, Erhard Lipkow,
Karl Meissner, Hansjrg Rudolph, Jrgen Soll, Helmut Uhlarz, Erwin Zabel

The Tasks of the 9th International Biology Olympiad

5.1

Theoretical Tests

5.2.1 Theoretical Test - Part A


01-10

11-20

21-30

31-40

41-50

51-60

61-70

71-80

10

C(2)

A(2)

A(2)

11

12

13

14

15

16

17

18

19

20

C(2)

B(2)

D(2)

21

22

23

24

25

26

27

28

29

30

B(2)

31

32

33

34

35

36

37

38

39

40

B(2)

C(2)

41

42

43

44

45

46

47

48

49

50

B(2)

C(2)

C(2)

51

52

53

54

55

56

57

58

59

60

B(2)

C(2)

D(2)

61

62

63

64

65

66

67

68

69

70

D(2)

B(2)

71

72

73

74

75

76

77

78

79

80

D(2)

A(2)

E(2)

Tab. 11: Answer codes of the following tasks, part A of the theoretical test
The following part A tasks were skipped: 2, 5, 11, 29, 33, 39, 43, 49, 48.

84

th

Report of the 9 IBO 1998 in Kiel

CELL BIOLOGY
A 1:
How do polypeptides find their way from the place of synthesis
(1 pt)at cytoplasmic ribosomes to the place of their destination in
the mitochondria?
A:
B:

by specific transport along the cytoskeleton


by specific amino-terminal targeting signals

C:
D:

by specific carboxy-terminal targeting signals


it is not necessary because the synthesis takes place on the surface of
the organelles
mitochondria synthesize all proteins inside the organelle

E:
A 2:

What is mRNA editing? (2 pts)

A:

removal of a coding base triplet from the mRNA

B:
C:
D:
E:

alteration of the coding sequence during transcription


change of a single base in the mRNA
addition of a poly-adenylat-tail to the mRNA
reverse transcription from mRNA to DNA

A 3:
How does the
(1 pt)increase in a cell?
A:
B:

only by de novo synthesis


only by division

number

of

mitochondria

and

plastids

C:
D:

by de novo synthesis and division


by inheritance

E:

by fusion of membrane vesicles

A 4:
How is the distribution of light to the photosystem I (PS I) and
(2 pts) the photosystem II (PS II) adjusted to different conditions of light?
A:

by changes in the size of antenna, in the orientation of the chloroplasts

B:

or in the thickness of leaves


by reversible removal of polypeptides of the reactive centre, and un
packing of grana thylakoids

C:
D:
E:

by separation of chlorophyll from PS I and PS II


by inhibiting the electron flow from PS I to PS II
by uncoupling the ATP synthesis

85

Theoretical Test Part A

A 5:
(1 pt)

Which process initiates the proteolytic degradation of proteins?

A:

glycosylation

C:

prenylation

B:

ubiquitination

D:

phosphorylation

A 6:

E: ADP ribosylation

How do you experimentally differentiate between integral and

(2 pts) peripheral membrane proteins?


A: Only peripheral proteins (but not integral) can be removed from the
membrane by 1 M NaCl solution.
B: Only integral membrane proteins can be removed from the membrane
by 0.1 M NaOH.
C: Only integral membrane proteins can be removed from the membrane
by 1 M NaCl.
D: Only peripheral (but not integral) membrane proteins can be solubilized
in detergent.
E: Only peripheral (but not integral) membrane proteins can be affected
by proteases.
A 7:

How are the fibre types forming the cytoskeleton called?

(1 pt)
A:

tubuline, lignin, kinesin

B:

microtubules, myosin, microfilaments

C:

keratin, myosin, kinesin

D:

microfilaments, intermediate filaments, microtubules

E:

actin, myosin, intermediate filaments

A 8:
Animal and plant cells possess channels directly connecting
(1 pt)the cytoplasm of one cell to the cytoplasm of another cell. How
are these structures called?
A:

plasmodesmata, desmosomes

B:

plasmodesmata, Ca2+-ATPase

C:

porin, gap junction

D:

gap junction, plasmodesmata

86

th

Report of the 9 IBO 1998 in Kiel

A 9:

What is the endosymbiosis hypothesis related to?

(1 pt)I. to bacteria living in the epithelium of gizzard


II. to plastids

III. to mitochondriaI

IV. to ribosomes

A:

B:

only II and III

D:

only III and IV

C:

only III

E:

II, III and IV

A 10: The semipermeability of which of the water-permeable


(1 pt)membranes would be sufficient for the process of plasmolysis?
I. plasmalemma
II. tonoplast
III. each biomembrane enclosing a compartment
IV. the middle lamella
A:

only I

C:

I and II

B:

only II

D:

only III

A 11:

E:

III and IV

Which process are microtubules not involved in?

(1 pt)
A:

cytomorphogenesis

C:

mitotic prophase

B:

meiotic anaphase I

D:

transport of Golgi vesicles

E:

cytoplasmic circulation

A 12:

A B-lymphocyte (a cell type of the immune system) produces

(1 pt)and secretes antibodies. Which structures of its protoplast


should therefore be very well developed?
A: only the smooth endoplasmic reticulum
B: only the smooth endoplasmic reticulum and the Golgi apparatus
(dictyosomes)
C: only the rough endoplasmic reticulum and the lysosomes
D: only the rough endoplasmic reticulum and the Golgi apparatus
E: the rough endoplasmic reticulum, the Golgi apparatus and the lysosomes

87

Theoretical Test Part A

A 13: Which of the following reactions can occur in the mitochondria?


(2 pts)
I.
II.

NADP reduction
synthesis of fatty acids

III.
endoxidation
IV. gene expression
A:

I, III, VI, VII

B:

II, III, IV, VII

C:

III, IV, VI, VII

D:

IV, VI, VII, VIII

E:

I, III, V, VIII

A 14:
(1 pt)

V. Calvin
VI. citric acid cycle
VII.oxidative catabolism of fatty acids
VIII. nitrit reduction

Which of the following are found in the cell wall of flowering


plants?

I.

apoplast

IV

intermicellar space

VII

II.

mircofilament

keratin

VIII cutin

III.

microfibre

VI

chitin

IX

A:

I,III,VII,IX,

B:

II,V,VII,VIII,

C:

III,IV,V,VIII

D:

II,VI,IX

E:

I,III,V,VI

A 15:

lignin
suberin

What is a prion?

(1 pt)
A:

a protein

B:

infectious RNA without genes for a protein envelope

C:

the DNA sequence which serves as a template for primer RNA

D:

an early eukaryote free of mitochondria

E:

a multienzyme complex for the biosynthesis of fatty acids

88

th

Report of the 9 IBO 1998 in Kiel

A 16: Which statements concerning the growth factors cytokins and


(1 pt)cytokinins are correct?
I. The plant cytokinins are peptides.
II. The plant cytokins are peptides.
III. The plant cytokinins are purine derivatives.
IV. The plant cytokins are purine derivatives.
V.

The animal cytokinins are peptides.

VI. The animal cytokins are peptides.


VII. The animal cytokinins are purine derivatives.
VIII. The animal cytokins are purine derivatives.
A:

I and VII

B:

II and VIII

C:

II and V

D:

III and VI

E:

IV and VI

A 17: Which classes of lipids have non-polar side chains and polar
(1 pt)head groups?
A:

phospholipids

B:

triglycerides

C:

cholesterol

D:

waxes

E:

glycerol

89

Theoretical Test Part A

ANATOMY AND PHYSIOLOGY OF PLANTS

A 18: The figure below shows the cross-section of a plant or organ.


(2 pts) Distinguish which of the given morphological, anatomical,
systematical and ecological terms are in accordance with the
cross-section. Select a combination of possibilities.
I.
stem axis
II.
root
III. leaf stalk
IV. dicotyledon
V. monocotyledon
VI. swamp or water plant
VII. terrestrial plant
A:

I, IV, VI

C:

II, V, VI

B:

II, V, VII

D:

II, IV, VII

E:

III, IV, VII

A 19: In the following three plant species(I to III) are listed. Among 1 to 7
(2 pts) anatomical and physiological characteristics are mentioned as well.
I.

Opuntia ficus-indica (Cactaceae)

1. water storage tissue of cells with


large vacuoles and chloroplasts

II.

Zea mays (corn, Poaceae)

2. chloroplasts of bundle sheath cells


with reduced grana.

III

Triticum aestivum (wheat, Poaceae) 3. C3-plant


4. C4-plant
5. CAM-plant
6. subepidermal sclerenchyma in the
7. stomata predominantly open at night

Decide which of the following assignments are correct.


A:

I: 1, 3, 6

II: 1, 5, 7

III: 2, 4, 6

B:

I: 2, 4, 6

II: 3, 6

III: 1, 3, 6

C:

I: 2, 4, 6

II: 2, 3, 6

III: 3, 6

90

th

Report of the 9 IBO 1998 in Kiel

D:

I: 1, 5, 7

II: 2, 4, 6

III: 3, 6

E:

I: 1, 5, 7

II: 3, 6

III: 2, 4, 6

A 20:

A C3-plant and a C4-plant are kept together under an airtigthly

(1 pt)sealed glass bell under light. How does the CO2-concentration


change under this glass bell?
A: The CO2-concentration does not change.
B: The CO2-concentration decreases to the CO2-compensation point of
the C3-plant.
C: The CO2-concentration decreases to the CO2-compensation point of
the C4-plant
D: The CO2-concentration increases.
E: The CO2-concentration drops below the value of the CO2compensation point of the C4-plant.
A 21: The protoplast has detached from the cell wall after plasmoly(1 pt)sis. What can mainly be found between the cell wall and the
protoplast?
A:

air

B:

vacuum

C:

water

D:

hypertonic solution

E:

cell sap

A 22: What happens if you put a turgescent cell into its own cell sap
(2 pts) (i.e. a solution with the same water potential as the osmotic
potential of the cell)?
A: There is no change.
B: The cell loses water until its water potential is equal to the water
potential of the surrounding solution.
C: Water is released until incipient plasmolysis occurs.
D: Plasmolysis occurs.
E: The cell bursts.

91

Theoretical Test Part A

A 23:
(1 pt)

By examining the dependence of photosynthesis on photon


supply you get so-called light curves (curves I to V).
Legend: 1: typical sun plant; 2: typical shade-loving plant;

respirat. photosynthesis

respirat. photosynthesis

: respiration in darkness; : light compensation point

light intensity

respirat. photosynthesis

respirat. photosynthesis

light intensity

respirat. photosynthesis

light intensity

light intensity

light intensity

Which of the preceding curves including the inscription is correct?


A:

92

B: II

C:

III

D:

IV

E:

th

Report of the 9 IBO 1998 in Kiel

A 24: The figure shows - be(1 pt)sides evaporation the


daily curves for the
process of transpiration
of a plant underlying
deterioration of water
supply. Which of the
curves expresses the
daily course of the
transpiration in the
cuticule?
A:

curve A

B:

curve B

C:

curve C

D:

curve D

Sunrise

Sunset

A 25: If a respiratory quotient > 1 is measured in an experiment, that


(1 pt)means:
A: In the measured respiration process carbohydrates are used as substrates.
B: In the measured respiration process fats are used as substrates.
C: In the measured respiration process organic acids are used as substrates.
D: In the measured respiration process proteins are used as substrates.

93

Theoretical Test Part A

A 26:
dro(1 pt)

The following figure shows a germ which was grown in hyponics in a glass vessel and which was exposed to light only
from the right-hand side. The growth curvatures you can observe in the stem axis are called:

A:

photonasty

B:

phototropism

C:

phototaxis

D:

photomorphosis

E:

photoperiodism

A 27: Germs were grown in closed chambers (I-IV) with different


(1 pt)nitrogen oxide NOx-concentrations in the air. The following figure
demonstrates the average length of the germs depending on

A: The lower the NOxconcentration the


larger the germs
can grow on average within 8 days.
B: In an atmosphere
free of NOx germs
grow faster from
day 5 to 8 than in
the first four days.
C: For achieving an
average germ
length of 1.5 cm
half of the time that
would be needed
94

Lnge der Keime (cm)

time. The nitrogen oxide concentration (kept constant) is given in


ppm together with the corresponding chamber number.

V 0 ppm NOx
IV 0,5 ppm NOx

2,0
1,8
1,6
1,4
1,2
1,0
0,8
0,6
0,4

III 5 ppm NOx


II 500 ppm NOx
I 5000 ppm NOx

0,2
0,0
1

Tage

th

Report of the 9 IBO 1998 in Kiel

without NOx is sufficient in a concentration of 0.5 ppm NOx.


D: At a concentration of 5000 ppm there is nearly no germ growth, but
1/10 of this concentration promotes growth during the fourth day.
E: After 4 days in chambers with air containing at least 0.5 ppm NOx, no
germ growth can be found any more.

A 28:
(1 pt)

Which function
germination?

does

the

glyoxylate

cycle

A:

reduction of fatty acids

B:

synthesis of ATP

C:

decarboxylation of glyoxylate from photorespiration

D:

linking the -oxidation to gluconeogenesis

E:

synthesis of glycine by transamination

have

during

A 29: Which three sensory systems regulate light-depending develop(1 pt)ment and differentiation of angiosperms?
I.

phytochrome

II.

cryptochrome

III.

protochlorohyllid

IV.

chlorophy

V.

anthocyan

A:

I, III and V

B:

I, II and IV

C:

II, III and V

D:

I, II and III

E:

II, IV and V

95

Theoretical Test Part A

ANATOMY AND PHYSIOLOGY OF ANIMALS AND MEN

A 30:
(1 pt)

The eyes of squids are highly efficient, those of snails are poorly
efficient. Consider whether efficiency of organs may be coupled
with the intensity of their metabolism. If squids were provided
with poorly efficient eyes and snails with highly efficient eyes
whereas the further body organisation remained unchanged in
both cases, this would be:

A:

a disadvantage to squids and an advantage to snails

B:

an advantage to squids and a disadvantage to snails

C:

a disadvantage to both squids and snails

D:

an advantage to both squids and snails

A 31:
(1 pt)

A medusa metabolizes most energy at the rim of its umbrella.


Energy is extracted from small particles which can be phagocy
tizedon their whole way through the tubes of the gastrovascular
system. How do the small particles have to be transported
through the gastrovascular system of the moon jelly (see figure
below) in order to provide the rim of its umbrella with a maximum
amount of food?

gonad

gastric filaments

A:

through tubes marked by A towards the rim of the umbrella

B:

through tubes marked by B towards the rim of the umbrella

C:

through tubes marked by C towards the rim of the umbrella

D:

no preference of particular tubes

96

th

Report of the 9 IBO 1998 in Kiel

A 32:
(1 pt)

In a smooth muscle, the length difference between its relaxed and


its maximally contracted state is

A: larger than in a striated muscle


B: smaller than in a striated muscle
C: as large as in a striated muscle
D: null, because smooth muscles cannot contract themselves but, rather,
prevent tissues from becoming stretched.
A 33:

Let striated skeletal muscles be replaced by smooth ones, and,

(1 pt)

reversely smooth muscles of the intestinal wall by striated ones.


The effect would be:

A: Both locomotion and intestinal peristaltic would be improved.


B: Both locomotion and intestinal peristaltic would be impaired.
C: Both locomotion and intestinal peristaltic would be as before.
D: Locomotion would be impaired, but intestinal peristaltic would be improved.
E: Locomotion would be improved, but intestinal peristaltic would be impaired.
A 34:

Many species of Rotatoria (rotifers; up to 3 mm length) in

(2 pts) freshwater have a reproductive cycle called heterogony: Reproduction occurs temporarily only in females and temporarily
in males and females. If this is an advantage to these species of
Rotatoria, which of the following statements is correct?
A: Several generations of females and males are followed by one with
only females.
B: Several generations of only females are followed by one with females
and males.
C: Every generation of only females is followed by one with females and
males, and vice versa.
D: Several generations of only females are followed by several with females and males, and vice versa.

97

Theoretical Test Part A

A 35:
(1 pt)

Stamina training leads to many effects in man. Which of the


following effects does not apply to stamina training?

A: enlargement of the heart


B: increase of the number of capillaries in muscles
C: increase of the number of mitochondria within muscle cells
D: increased pulse rate during rest
E: increase of the lung volume
A 36:
(1 pt)

Manual work increases the CO2 concentration in the blood. What


is the result of this?
I.

The affinity of hemoglobin to oxygen decreases.

II. Elimination of CO2 through the lungs decreases.


III. Loss of mineral salts through the air breathed out increases.
IV: The brain suffers from oxygen deficiency.
V: The body warms up.
A:

only I

A 37:
(1 pt)

B:

only IV

C:

I, V

D:

II, III

E:

What short-term effects may occur in the human cardiovascular


system at a sudden transition from lying to standing?
I.

The pulse rate is increased.

II. The total peripheral resistance is increased.


III. More blood flows through the kidneys.
IV. More blood flows through the upper and lower limbs.
V. The blood pressure is decreased.
A:

I, III, IV

B:

III, IV, V

C:

I, II, V

D:

only II, III

E:

only IV, V

98

II, IV

th

Report of the 9 IBO 1998 in Kiel

A 38:
(1 pt)

If a fish species from fresh water developed into a marine fish


species, which problems of osmoregulation would have to be
solved?
I

Excess of salts would have to be excreted.

II. Water would have to be reabsorbed from the initial ultrafiltrate of


kidneys.
III. The salt concentration of the blood would have to be adjusted to
that of the sea.
IV. Drinking of sea water would have to be avoided.
V. Permeability of the skin would have to be reduced.
A:

only I, II

C:

all answers

B:

only II, IV

D:

only V

A 39:

E:

only II, IV

In an everse eye (the opposite of inverse), the light-sensitive

(2 pts) region of retinal cells is directed towards the incoming light. The
figure below taken from a text book illustrates how an everse
median eye of a spider could have developed by invagination.
Why should this suggested eye structure be considered incorrect?
Legend:
1:

lens

2:

vitreous body

3:

retinal cell, the sensory area


is opposite to nucleus

4:

optic nerve

A: In the development of eyes, invagination never plays a role.


B: The figure really shows an inverse eye.
C: Generally, sensory cells perceive the stimulus distally and conduct the
impulses proximally.
D: Generally, sensory cells perceive the stimulus proximally and conduct
the impulses distally.
E: The cells of the vitreous body and the cells of the retina are mixed up.
99

Theoretical Test Part A

A 40:
(1 pt)

Bones are dynamic rather than static structures. How may this be
realized?

A: Those who sit crookedly will soon get a crooked back.


B: In case of being bedridden, the internal structure of bones adapts to
the altered stress.
C: Bones do not break as readily as static structures of equal strength.
D: A tooth crown freshly implanted, which initially does not fit well, can
do so after a couple of weeks without intervention of the dentist.
A 41: If a man of 70 kg weight ingests 40 g alcohol, the alcohol level in
(2 pts) his blood will raise one part per thousand. About 1 g alcohol is
eliminated per hour and 10 kg body weight. The man of 70 kg
body weight has been involved in a traffic accident and has run
away. A blood sample has been taken from him after 2 hours. It
contained 0.5 parts per thousand of alcohol. Assuming he did not
ingest any alcohol after the accident - how much alcohol did his
blood contain at the time of the accident?
A:

1.10 parts per thousand

B:

0.95 parts per thousand

C:

0.80 parts per thousand

D:

0.65 parts per thousand

E:

0.55 parts per thousand

100

th

Report of the 9 IBO 1998 in Kiel

ETHOLOGY
A 42:

The mobbing reaction of house swallows (Hirundo rustica).(1 pt)

House swallows mostly breed in colonies. They live on flying insects. Predators spotted during flights for prey or near the nest are noisily attacked in nosedives. This mobbing reaction communicates itself to the other swallows and
supports the defence against enemies. The table shows the ethological status
of the swallows and their share in the mobbing reaction and in the population,
respectively.
birds status

share in the population

share in the mobbing reac-

in per cent

tion in per cent

adult animals, not mated

adult animals, before breeding

11

adult animals, breeding

14

10

adult animals with young animals

51

77

young animals

20

total

100

100

Which of the following hypothesis have to be assumed to be correct according to the results?
I:
II:
III:
IV:
V:
VI
A:

IV, V, VI

The mobbing reaction is an intra-specific signal.


The mobbing reaction is an element of self-defence.
The mobbing reaction is an element of choice of mates.
The mobbing reaction is a kind of maternal care.
The mobbing reaction is a form of commensalism.
The mobbing reaction is an altruistic mode of behaviour.
B:

III, IV, V

C:

II, IV, VI

D:

I, IV, VI

101

Theoretical Test Part A

A 43: The rank of macaques (Macaca sylvanus).


(2 pts)
Macaques are related to rhesus macaques; they live in Morocco and Algeria.
Adult males, females and their young animals form groups in which ranks are
set up by superorder or suborder.
The table below shows the frequencies of aggression within a group of nine
females (A to I):

attacking

being attacked
A

16

16

14

10

18

total

53

33

23

29

10

11 33

14

24

10

10

13

13

16

10

27

52

10 28

36

45 224

Read the matrix and choose which one of the females is the lowest in the rank order.
How do you call the individual loosing conflicts within the group?
I.

female A

II

female I

III.

female E

IV.

female F

V.

female H

VI.

female G

IX.

subdominant

VII. omega animal

VIII. alpha animal

Which of the following combinations is the correct one?


A:

I, III, IV

D:

IV, VI, IX

B:

II, V, VII

E:

V, VII, IX

C:

III, VII, IX

102

th

Report of the 9 IBO 1998 in Kiel

GENETICS/EVOLUTION

A 44:
(1 pt)

Which of the following statements on introns is correct?

A:

They are non-transcribed sequences (spacers) between two genes.

B:

They are transcribed spacers between two genes.

C:

They are located between the coding regions of a gene.

D:

They are located between the coding regions of a mature mRNA.

E:

They are non-coding regions of a polycistronic mRNA.

A 45:

Which statements on sex chromosomes (gonosomes) are cor

(1 pt)rect?
I.

They are always heterochromatic.

II.

They may occur in a euchromatic or heterochromatic form.

III.

They occur in human beings and in all animals.

IV.

They occur in yeast (Saccharomyces cerevisiae).

V.

They occur in some mosses.

VI.

They occur in some flowering plants.

VII. Their homozygosis always leads to female sex.


A:

I, II, III

C:

I, IV, VI

B:

I, III, V

D:

II, V, VI

E:

II, V, VII

A 46: The protoplast fusion in dicotyledons is well suited for the


(1 pt)following objectives A-D. What can also be achieved by cross
hybridization?
A: combination of the genetic material of different plant families
B: studies on the fate of plastids during the development from cells with
mixed plastide populations
C: studies on the fate of mitochondria during the development from cells
with mixed mitrochondria populations
D: transfer of resistance genes from wild species into cultivated species
103

Theoretical Test Part A

A 47: Did Gregor Mendel know some of the phenomena (I- VII )? Under
(2 pts) which capital letter do you only find such phenomena?
I.

decrease of the degree of homozygous-recessives in the gen


eration sequence

II.

extrakaryotic inheritance

III.

break of coupling

IV.

polyploidy

V.

hybrid character of the endosperm (albumens)

VI.

cleistogamy in pea

VII.

homogeneity of the F1 of homozygous parents differing in two


pairs of alleles

A: I, II

C:

V, VI

B: I, III

D:

III, VI

E: IV, VII

A 48: Which of the following statements characterizes a substitution


(1 pt)hybrid?
A: The hybrid of two auxotrophic mutants is prototrophic
B: The deficiency caused by a recessive allele is repaired by another
gene
C: A pair of chromosomes is replaced by a pair of chromosomes of another species.
D: The hybrid competes one of the parents out of the population
E: The plastids of one species are displaced by those of another species
(after back-crossing).
A 49: Which process does not lead to a change in the chromosome
(1 pt)number?
A:

poisoning of the nuclear spindle

B:

non disjunction

C:

endomitosis

D:

centromere fracture

E:

duplication

104

th

Report of the 9 IBO 1998 in Kiel

A 50: The bacteria strains A and B and a certain bacteriophage which is


(1 pt)infectious for both strains grow in one culture. After one day
some bacteria which are equivalent to type A have originated, but
show characteristics based on five alleles in B.
Which processes might have caused this phenomenon?
I.

transformation

IV.

sexduction

II.

transposition

V.

mutation

III.

transduction

VI.

transfection

A:

I, II, III

C:

I, V, VI

B:

I, III, IV

D:

II, IV, VI

E:

, III, IV

A 51: The table sums up characteristics which are original (plesiomor(1 pt)phic) or deduced (apomorphic) within the group of angiosperms.
characteristics
1

vascular bundle collateral closed, herb

vascular bundle collateral open, woody plant

blossom polysymmetric, actinomorphic

blossom monosymmetric, zygomorphic

monospermous indehiscent fruit (e.g. nut)

polyspermous dehiscent fruit (e.g. capsula)

tracheal segments, long

tracheal segments, short

blossoms without nectar

10

blossoms with nectar

11

ovary superior

12

ovary inferior

Which of the following combinations shows deduced (apomorphic) characters


only?
A:

1, 3, 5, 7, 10, 11

C: 2, 4, 6, 8, 9, 12

B:

1, 4, 5, 8, 10, 12

D: 2, 3, 5, 8, 9, 12

E: 1, 4, 6, 7, 10, 12

105

Theoretical Test Part A

A 52: I - V lists names of plants and 1 - 5 lists some characteristics


(1 pt)referring to the reproduction of cormophytes.
I.
II.

Lycopodium (Lycopodiophyta)
Gingko (Pinophyta)

III.
IV.
V.

Pinus (Pinophyta)
Cycas (Pinophyta vel Cycadophyta)
Magnolia (Magnoliophyta)
1. aggregations of sprophylls (strobili) or blossoms unisexual
2. aggregations of sprophylls (strobili) or blossoms bisexual
3. aggregations of sprophylls (strobili) or blossoms sexually
not differentiated
4. fertilization by spermatozoids
5. fertilization by immobile sperms

Which assignments are correct?


A:

I: 1, 4

II: 3, 4

III: 3, 4

IV: 1, 5

V: 3, 4

B:

I: 1, 5

II: 2, 5

III: 1, 4

IV: 2, 5

V: 2, 5

C:

I: 3, 4

II: 1, 4

III: 1, 5

IV: 2, 5

V: 2, 5

D:

I: 3, 4

II: 1, 4

III: 1, 5

IV: 1,4

V: 2, 5

A 53: Which advantage do angiosperms (with double fertilisation and


(1 pt)formation of a secondary endosperm) have in comparison to
gymnosperms (single fertilisation and formation of a primary
endosperm)?
1. The neotenic development of a female prothallium (macrogametophyte)
requires less energy.
2. The triploid endosperm of angiosperms can store more nutrients.
3. Regarding the investment of energy and reserve material, the formation of
a secondary endosperm after double fertilization is more economic.
4. Double fertilization is an additional prevention against hybridization.
5. Double fertilization accelerates the embryonic development.
A:

1, 2, 3

C:

only 1, 3

B:

only 2, 4

D:

2, 4, 5

106

E: 1, 3, 4

th

Report of the 9 IBO 1998 in Kiel

A 54: Lamarck and Darwin each proposed a theory on the origin of


(2 pts) species diversity. According to Lamarcks theory of species
transformation, infusories may originate and have to originate
by abiogenesis up to the present and give rise to the development of higher organisms. A single origin of organisms is sufficient for Darwins theory of natural selection. Which of the
following statements is correct?
A: Lamarck has to be considered as the author of the descent theory
which, however, was first founded by Darwin.
B: Different contemporary species may be related to each other according to Darwins theory, but not according to Lamarcks theory.
C: Long-term evolving of species depends on improvement in performance of organisms according to Darwins theory and on increasing
success of organisms according to Lamarcks theory.
D: Lamarck considered inheritance of acquired characteristics as possible, whereas Darwin did not.
A 55: The well known evolutionary geneticist Maynard Smith stated in
(2 pts) 1989: We need some measure of the survival and reproduction of
the different genotypes. This measure is fitness. Which of the
following statements is correct?
A: While organisms are mortal, genes are potentially immortal. Therefore,
the statement cited is correct.
B: Indeed, fitness is a condition to achieve various kinds of success. Fitness can only be measured by measuring the latter. Therefore, the
statement cited is correct.
C: According to the statement cited, the success of survival and reproduction is a measure of fitness. However, fitness is a condition to
achieve success. Therefore, the statement cited is wrong.
D: Fitness of genotypes can only be measured by the survival and reproductive success of the corresponding organisms. Therefore, the
statement cited is correct.
E: The suggested measure of fitness works well. Therefore, it does not
matter whether the statement cited is right or wrong.
107

Theoretical Test Part A

ECOLOGY
A 56: Why do some columnar cacti in their natural habitat turn their
(1 pt)longitudinal axis to the point where the sun is at its zenith in
summer-time? Evaluate the explanations I - III.
I.
II.
III.

to attain as much light as possible


to heat the apically arranged reproductive organs in the
morning and in the evening
to minimize the increase in temperature of the columnar body

Which of this / these explanation(s) is (are) correct?


A: None of the explanations is correct.
B: Only explanation I is correct.
C: Only explanations I and II are correct.
D: Only explanation III is correct.
E: All three explanations are correct.
A 57:

Which plant life form of the soil flora in a deciduous forest is

(1 pt)developing and fruiting most quickly in spring?


A: therophyts, because they have seeds that already germinate at low
temperatures
B: therophyts, because their subterranean storage organs allow a quick
start of growth
C: geophyts, because their seeds quickly germinate on the soil in spring
D: geophyts, because they can quickly form a shoot from their subterranean storage organs
E: None of the explanation above is correct.
A 58: In which way does a cormophytic plant protect itself against
(1 pt)freezing ( < 10 o C)?
A: Plants develop a dense branching to prevent the loss of heat.
B: Plants protect themselves against freezing by a thick cuticle and thick
hair on the leaves.
C: Plants concentrate salts in their cells (osmotic effect).
D: Plants have to tolerate cellular dehydration as a consequence of iceformation.
E: Freezing is avoided by metabolic heat production.

108

th

Report of the 9 IBO 1998 in Kiel

A 59: Why do you often find cyanobacteria and red algae in places with
(1 pt)low irradiance?
I.

All of them are small and are outcompeted by fast-growing


species.
II. They are strongly photosensitive.
III. They can efficiently utilise light with the help of additional an
tenna pigments
A:
B:
C:

All three explanations are correct.


Only I is correct.
Only II is correct.

D:
E:

Only III is correct.


Only II and III are correct.

A 60:
(1 pt)

Why is a high-moor bog an extreme habitat?


I.

Because it is oligotrophic and permanently cold.

II. Because it allows extreme situations on the surface concerning


climate.
III. Because it is oligotrophic and has a low pH value.
A:

I, II and III

C:

I and II

B:

Only I

D:

II and III

A 61:
(1 pt)

E:

Only III

Which one of the desert plants I and II has to tolerate more heat in
a habitat with the same solar radiation and why?
I. a desert plant with large leaves and water-storing bulbs
II. a desert plant with small but succulent leaves

A: I heats up more intensively than II, because the leaves possess a


large surface.
B: I and II heat up at the same range and have to be equally heat tolerant.
C: II heats up more intensively, because its heat capacity is low.
D: II heats up more intensively than I, because the leaves contain a lot of
water.
E: II heats up more intensively than I, because the leaves of I transpire
more intensively.
109

Theoretical Test Part A

A 62. Rips are an advantage to columnar cacti and globular cacti:


(1 pt)I. because they are in agreement with the concept of xerophytes
by increasing the surface.
II. because predators are irritated.
III. because they decrease the irradiation by partial shading
A:

Only I

C:

I and II

B:

Only II

D:

Only III

A 63:

E:

I, II and III

The ovary needs a lot of heat energy for maturation. In cold

(1 pt)climates this may be a problem. By which means is the perianth


capable of increasing the energy input in a particularly effective
way?
A: by a bell shaped perianth preventing the nightly emission
B: by paraboloid flowers reflecting the solar radiation to the ovary
C: by dark-coloured flowers absorbing a particularly high amount of heat
D: by the symmetry of zygomorphic flowers causing an evenly warming
up of the ovary
E: by an ovary which is bigger than the perianth
A 64:

R-strategists (r-selectionists) are inhabitants of fast originating

(2 pts) and vanishing habitats. They are selected for quick colonisation,
for fast and complete utilisation of these habitats and for fast
search for new favourable places. Which of the following
characteristics apply to r-strategists?
I.

They have a slow development.

II.

They have a relatively large number of offspring.

III.

They have a short life span.

IV.

Their mortality is independent of the population density.

V.

Their population size is relatively constant.

A:

I, II and III

C:

II, III and V

B:

I, II and IV

D:

II, III and IV

110

E:

I, III and V

th

Report of the 9 IBO 1998 in Kiel

A 65: The ecosystems shown in the table differ in the amount of their
(2 pts) net primary production.
selected ecosystems

number

ecosystem

tropical rain forest

savannah

subtropical sand desert

temperate deciduous forest

boreal deciduous forest

tundra

Name the correct order (increasing net primary production ) of the ecosytems shown in the table.
A:

3, 6, 2, 5, 4, 1

B:

3, 6, 5, 2, 4, 1

C:

6, 3, 5, 2, 4, 1

D:

6, 3, 2, 5, 1, 4

A 66:
(1 pt)

Which possible effects does the smoke from the vast forest fires
in tropical rain forest have on the plants in neighbouring areas?
I.

decrease of the photosynthetic rate

II.

increase in the respiration rate

III.

decrease of the plant growth

IV.

increase in the transpiration rate

A:

I and II

B:

II and III

C:

I and III

D:

III and IV

E:

II and IV

111

Theoretical Test Part A

The following figure shows survivorship curves. Inspect the


curves showing the percentage of individuals of populations of
three different species which are still alive at any particular age.

I.

elephant

II.

frog

III.

rabbit

% of still living population

A 67:
(1 pt)

Age (relative time units)

Inspect the survivorship curves. Which of the following combinations is


correct?
A:

I (2),

II (3)

B:

I (1),

II (3)

C:

II (3),

III (1)

D:

I (1),

II (2)

E:

II (2),

III (3)

112

th

Report of the 9 IBO 1998 in Kiel

SYSTEMATICS
A 68: Which characteristics are typical for wind-pollinated (anemophi
(1 pt)lous) plants?
A: smooth stigma, viscous pollen and polyspermous dehiscent fruit
B: smooth stigma, non-viscous pollen and polyspermous indehiscent fruit
C: papillate stigma, non-viscous pollen and monospermous indehiscent
fruit
D: papillate stigma, viscous pollen and monospermous dehiscent fruit
E: papillate stigma, viscous pollen and polyspermous indehiscent
A 69: Where do beans (Fabacae, e.g. soybean Glycine max, syn. Soja
(1 pt)hispida) store reserve material for germination?
A: in the pericarp
B: in the cotyledons of the embryo
C: in the triploid nutritive tissue (endosperm) of the seed
D: in the diploid nutritive tissue (perisperm) of the seed
A 70:
A:

Which storage organ does not exist in monocotyledons? (1 pt)

onion

A 71:
(2 pts)

C:

tuber

B:

turnip

D:

rhizome

I - V lists plant species and 1 - 5 lists morphological and taxonomic characteristics.


I. onion (Allium)
II. peach (Prunus)
III. coconut palm (Cocos)
IV. oak (Quercus)
V. corn (Zea)

1. monocotyledons
2. dicotyledons
3. stone fruit
4. capsula
5. nut or caryopsis

Which assignments are correct?


A:

I: 1, 3

II: 1, 4

III: 1, 4

IV: 2, 4

V: 1, 5

B:

I: 1, 4

II: 2, 5

III: 1, 5

IV: 2, 3

V: 1, 3

C:

I: 1, 4

II: 2, 5

III: 1, 3

IV: 2, 5

V: 1, 3

D:

I: 1, 4

II: 2, 3

III: 1, 3

IV: 2, 5

V: 1, 5

E:

I: 1, 4

II: 2, 3

III: 1, 4

IV: 2, 5

V: 1, 3
113

Theoretical Test Part A

A 72: In which family do symbiotic nitrogen-fixing bacteria in the root


(1 pt)occur?
A:

Rosaceae

C:

Fabaceae (Leguminosae)

B:

Brassicaceae (Cruciferae)

D:

Asteraceae

A 73:

E: Fagaceae

What is correct concerning penguins?

(1 pt)I. Penguins are an intermediate form between birds and mammals.


II. Penguins are densely covered with feathers.
III. Penguins are densely covered with hair.
IV. Penguins are densely covered with chitin fibres.
A:

I, II

C:

only II

B:

I, III

D:

only III

A 74:

E:

only IV

Which of the following animal taxa only occur in the sea?

(2 pts) I.
II.
III.
IV.

starfish and sea urchins (Stellaroidea, Echinoidea)


jellyfish (Cnidaria)
sponges (Porifera)
squids (Cephalopoda)

V. higher developed crustacea (Malacostraca - Crustacea, Astacus)


A:

I, IV

C:

only V

B:

only II, III

D:

I, II, III

A 75:

E:

II, IV, V

In which animals does heterogony occur like in rotifers?

(1 pt)
I.

in water fleas (Cladocera - Daphnia)

II. in plant-lice (Aphidina - Homoptera)


III. in dragon flies (Odonata)
IV. in bivalves (Bivalvia - Lamellibranchiata)
V. in sea stars (Asteroida - Stellaroidea)
A:

only I, II

C:

only V

B:

only III, IV

D:

I, II, III

114

E: III, IV, V

th

Report of the 9 IBO 1998 in Kiel

MICROBIOLOGY / BIOTECHNOLOGY
A 76: A broth culture of Escherichia coli (50 ml inoculated with 0.5 ml
o
(1 pt)of an overnight culture), grown at 37 C for 18 hours, contains:
A: cells of E. coli which all divide at the same rate
B: mostly endospores
C: cells of E. coli which are all in the stationary growth phase
D: cells of E. coli which are in all possible growth phases
A 77: Clear plaques in a bacterial lawn that do not increase in size are
(1 pt)caused by
A:

the parasitic bacterium Bdellovibrio bacteriovorus.

B: bacteriophages.
C: bacteria producing antibiotics.
D: bacteriocin-producing bacteria.
A 78: In a wastewater treatment plant flocks of activated sludge swim
(2 pts) on the surface of the settling tank. Which physiological process
causes this problem?
C: denitrification
D: alcoholic fermentation

A: nitrification
B: sulphate reduction

A 79: It is well known that heterologous DNA which is genetically stable


(2 pts) can be established in a bacterial cell. Which of the following
processes can be used to achieved this?
I. conjugation
II. transformation of plasmid DNA
III. transformation of chromosomal DNA
A:

I, III, V

C:

I, III, IV

B:

I, II, III

D:

II, IV, V

IV. general transducti


V. special transduction

E:

I, II, V

A 80: Which of the following organisms is used to transfer genes into


(1 pt)higher plants?
A:

Escherichia coli

C:

Agrobacterium tumefaciens

B:

Rhizobium trifolii

D:

Salmonella typhimurium
115

Theoretical Test Part A

74

Theoretical Test Part A

74

Theoretical Test Part B

5.2.2

Theoretical Test - Part B

CELL BIOLOGY
B 1:

Stimulating a giant axon of a squid at the time = 0 leads to a

(1 pt)

change in the membrane potential, which is measured with a


microelectrode. The results are shown in the figure below. Label
the figure using the adequate code numbers.

Code numbers:
1.
2.

resting potential
depolarisation

3.
4.
5.
6.

membrane potential (mV)


action potential
osmotic potential
hyperpolarisation

116

th

Report of the 9 IBO 1998 in Kiel

B 2:
Match the molecules or proteins listed below with the various
(2 pts) sub-compartments of mitochondria.

1, 2
6, 9

1. pyruvate ehydrogenase
2. DNA polymerase
3. cytochrome c
4. ATP synthase
5. porin
6. aconitase

4, 10

7. protein import receptor


8. cytochrome b2
9. enzyme of -oxidation of
fatty acids

3, 8

10. succinic acid dehydrogenase

5, 7

117

Theoretical Test Part B

B 3:
S K I P P E D .
(3 pts) Bacterial ribosomes are distinguished from ribosomes coded by
the nucleus because of their sedimentation (Svedberg-constant S)
and their composition of rRNA. Name the ribosomes and their
subunits according to the Svedberg constant and their rRNA
composition. Fill in the correct code number in the corresponding
space in the following table.
Code numbers:
1. 5S rRNA

3. 16S rRNA

5. 23S rRNA

2. 5.8S rRNA

4. 18S rRNA

6. 28S rRNA

bacterial ribosomes ribosomes coded by the nucleus


......70.......S

.......80..............S

large subunit

large subunit

......50.....S

........60.............S

composition of rRNA rRNA composition


....... .1.........

........ .1..............

.........5.........

..........2.............

....................

..........1.............

small subunit

small subunit

.....30...S

..........40..........S

composition of rRNA composition of rRNA

118

..........3.........

...........4.............

.....................

..........................

.....................

..........................

th

Report of the 9 IBO 1998 in Kiel

B 4:
The average time of duplication of mammalian cells in cultures is
(2 pts) 24 hours. Just divided cells growing on a cover-slip were treated
with radioactively labelled (tritium) thymine for 10 minutes and
immediately fixed. Figure I shows a typical microscopic section.
Figure II shows the autoradiography with the dispersion of the
silver grains of the section in figure I using the same magnification. Calculate the average duration of the S-phase in these cells
with a maximum error of 30 minutes.

figure 1

Average

duration

figure 2

of

S-phase:

.....2,5

h..........

119

Theoretical Test Part B

B 5:
The following table shows structures of eukaryotic and prokary(2 pts) otic cells as well as various membranes (types of membrane).
Which cell structures possess which type of membrane (types of
membrane)? Mark the correct answer with a cross.

presence or types of membranes


cell structures

no membrane

monolayer
membrane

1. nucleus

2. lysosomes

3. mitochondria

4. Golgi apparatus

5. peroxisomes

6. rough endoplasmic
reticulum

7. chloroplasts

8. vacuoles

9. ribosomes

10. centrioles

11. nucleolus

12. flagella of

eukaryotes
13. flagella of
prokaryotes

120

double
membrane

th

Report of the 9 IBO 1998 in Kiel

B 6:
Label the marked structures of the chloroplast in the following
(2pts) figure. Use the corresponding code numbers.
Code numbers:
1. thylakoid

5. nucleoid

9. plastidome
(collection of plastids)

13. pyrenoid

2. cristae

6. nucleolus

10. granum

14. grain of starch

3. stroma

7. amyloplast

11. elaiosplast

4. nucleosome

8. lipid droplets 12. prolamella body

10

3 14 1

121

Theoretical Test Part B

B 7:
The following table shows types of plastids and their specific
(2 pts) characteristics. Match the characteristics to the types of plastids
by marking with a cross.

Types of plastids

characteristics
starch formation

Chloroplast

photosynthesis

coloured

contains DNA

Chromoplast

Rhodoplast

Proplastid
Leuko-amyloplast

B 8:
(1 pt)

X
X

A cell produces and secretes a certain protein. Select from the


list only those structures which are involved in these processes,
to our present knowledge - and list the code numbers in the correct order of the processes.
1. peroxisome
2. free ribosome
3. Golgi apparatus
4. secretory vesicle
5. plasma membrane
6. vacuole
7. rough endoplasmic reticulum
8. lysosome
2, 7, 3, 4, 5
correct order of compartments:....................................................

122

th

Report of the 9 IBO 1998 in Kiel

ANATOMY AND PHYSIOLOGY OF PLANTS

B 9:

The following figure shows a simplified schematic drawing of the

(2 pts) most important photochemical complexes and their localisation


in the thylakoid membrane of chloroplasts. Label the spots marked with
with the four integral membrane protein complexes.
a- photosystem I
b- photosystem II
c- cytochrome b6 / f
d- ATP synthase
Mark with arrows (using a yellow pen) the electron flow from water as the
+

e
- donator to NADP as the e terminal acceptor.

stroma

intrathylakoid space

legend: FdR = Ferredoxin-NADP+-Reductase; Fd = Ferredoxin; FeS = Ironsulphur-centre; P700, P680 = chlorophylls of the reaction centres; PC = Plastocyanin; PQ = Plastoquinone; Q = Quinon; Ph = Pheophytine; OEC = water
splitting complex .

123

Theoretical Test Part B

B 10:

Plot with a yellow pen (1) the course of gas exchange of a typical
(3 pts) CAM plant and with a pencil (2) the course of gas exchange of a
C3-plant using the figure.
The malic acid concentrations in the CAM plant vacuoles should
be plotted in blue (3).
light period

CO2 output/h

CO2 input/h

Malic acid concentration

dark period

B 11: S K I P P E D .
(2 pts)
COOAmino acids can be divided in various classes
+
H3N - C - H
according to the polarity of their R-groups.
R

1: serine (Ser)

6: glutamine (Gln)

2: alanine (Ala)

7: arginine (Arg)

3: valine (Val)

8: glutamic acid (Glu)

4: phenylalanine (Phe)

9: aspartic acid (Asp)

5: tyrosine (Tyr)

124

Classify the following amino acids due to the


polarity of their R-groups by arranging the following code numbers in the table.

th

Report of the 9 IBO 1998 in Kiel

classification of amino acids according to the polarity of their R-group (at pH 7)


nonpolar (hydrophobic) R-groups

2, 3, 4

polar, but uncharged R-groups

1, 5, 6

negatively charged (acidic) R-groups

8, 9

positively charged (alkaline) R-groups

B 12:

The figure below is a simplified scheme of the citric acid cycle.

(2 pts) Fill in the missing intermediates in the correct order by writing


down the corresponding code number in the space
provided.
1: oxaloacetate

4: isocitrate 7: 2-oxoglutarate

2: malate

5: fumarate

8: malonate

3: cis-aconitate

6: glyoxylate

9: succinate

Indicate by an arrow - like in the given example - the missing release of


CO2 as well as the three missing dehydrogenations in which the hydrogen is transferred to the NAD+.
Acetyl-CoA

Citrate

CO2

Succinyl-CoA

125

Theoretical Test Part B

B 13: Compare figure A and B. Indicate in the table, which structures in


(2 pts) figure B correspond to those in figure A.

figure A figure B

figure A

figure B

126

th

Report of the 9 IBO 1998 in Kiel

ANATOMY AND PHYSIOLOGY OF ANIMALS AND PLANTS

B 14:

Within the wall of a garden hose, flexible fibres which can hardly

(2 pts) be stretched run in crosswise helices enclosing an angle of


0
= 110 . Only at this angle, the water pressure within the hose
cannot alter he length of the hose, its diameter, or its volume (see
the maximum indicated in the curve presented).

volume of a garden hose

volume of a cylinder, whose


helically fibres within the
wall are of constant length

angle enclosed by the helically running fibres of the wall


In flat- and roundworms, flexible but inelastic fibres run crosswise helically
within the body wall. The cross section of the body is flat in flatworms and circular in roundworms. The body volume is frequently changed by ingestion of
food and defecation.
Therefore, which of the following statements is correct? Mark with a
cross.
= 1100 in flatworms and = 1100 in roundworms
X

= 1100 in flatworms and = 1500 in roundworms


= 1500 in flatworms and = 1100 in roundworms
= 1500 in flatworms and = 1500 in roundworms

127

Theoretical Test Part B

B 15:
(1 pt)

Microscopically small marine annelid worms differ in various aspects from their macroscopically large relatives. Mark the correct
statement with a cross.
In the former, the relation of body surface to body volume is
smaller than in the latter.
In the former, the brain volume occupies a smaller part of the
body volume than in the latter.
A heart is often absent in the former, but always present in the
latter.
External fertilization is widespread among the former and
does not occur among the latter.
Planctic larvae are widespread among the former and do not
occur among the latter.

B 16: The main waste product of protein and purine metabolism is urea
(2 pts) in mammals and uric acid in birds. The solubility of these products
in water is different. If mammals produced uric acid and birds
urea (with an otherwise unchanged body organisation) why
would
this be disadvantageous to both? Mark the correct answer.

In both cases, the embryos would have substantial problems


with excretion.
In both cases, the adults would have substantial problems
with excretion.
The organs of mammals are unsuitable for excreting uric acid.
The organs of birds are unsuitable for excreting urea.
In the case of urea formation, birds would lack important raw
materials for feather formation.
In the case of uric acid formation, mammals would lack important raw materials for hair formation.

128

th

Report of the 9 IBO 1998 in Kiel

B 17: Which one of the following statements related to the neuronal


(2 pts) basis of learning is correct?

Learning immediately induces the formation of specific memory


molecules.
Learning immediately induces the formation of specific memory
synapses.
Learning immediately induces the formation of specific memory
neurons.
Permanently, too many memory molecules are formed.
Learning improves the fitness of a few of them which survive,
while the less fit ones are decomposed.
Too many synapses are formed constantly. Learning improves
the fitness of a few of them which survive, while the less fit
ones are decomposed.
Too many neurons are formed constantly. Learning improves
the fitness of a few of them which survive, while the less fit
ones are broken down.
Selection due to differences in fitness does not occur in the
brain. Neurons are activated by learning and otherwise rest.

129

Theoretical Test Part B

B 18: S K I P P E D .
(3 pts) Inspired by nature, the presented patterns I and II were generated
by a computer. Pattern formation takes place only at the growth
edge (at the bottom in the figures). An activator stimulates pigmentation (responsible for black parts of the patterns), while this
process is inhibited by an inhibitor (responsible for white parts
of the patterns). Patterns form, because the concentrations of
activator and inhibitor vary in time and space. The activator promotes
its own production by autocatalysis and, additionally, promotes
production of the inhibitor by catalysis. Both activator and inhibitor can diffuse and can be broken down. The example shows, the
laws of pattern formation differ only by their rates of diffusion D
and the rate of decomposition R of activator and inhibitor.
In order of increasing size the values are (A stands for activator, I for inhibitor):
DA = 0.005 and 0.1
RA = 0.01 and 0.06

DI = 0.4 in both cases


RI = 0.003 and 0.015

time

start of pattern formation

growth edge

time

start of pattern formation

growth edge

Write down which values correspond to pattern I and which to pattern II.
values of pattern I: 0,005 DA =

0,005

RA =

0,01

RI =

0,015

values of pattern II: 0,1

0,1

RA =

0,06

RI =

0,003

130

DA =

th

Report of the 9 IBO 1998 in Kiel

ETHOLOGY

B 19:

Sexual selection of swallows (Hirundo rustica)

(4 pts) Reproduction and parental care cause costs and yield benefits.
The right mate selection can optimize the cost-benefit-ratio (e.g.
measured by the number of eggs per nest or by the number of the
hatched and fledgling offspring). Male and female swallows differ
in their behaviour and in the length of their outer tail feathers
(see table below).
Characteristics of swallows
swallow Body
length

weight

feathers
tail feather
other than
mil feathers

arrival from
Africa

male

19 cm

18-26 g

identical
to females

106 mm
(100%)

before
(females)

female

19 cm

18-26 g

identical
to males

91 mm (84%
of 106 mm)

after
(males)

Courting male swallows change their flight path in the presence of a female
and show two long feathers in the spread tail fan. These feathers might indicate certain features of males as a secondary sexual characteristic. If that is
correct, experiments with modified ordinary tail feather lengths should yield
various results. In the picture males are shown with ordinary and experimentally modified tail feathers.

131

Theoretical Test Part B

The next table shows the results of three groups of males (A, B, C), which had
different feather lengths.
Results of an ethological experiment
male tail feathers

A: 106 mm (ordinary)

B: 85 mm
(shortened)

C: 127 mm
(lengthened)

time of courting

6.8 days

12.3 days

3.4 days

offspring / season

5.0 young

3.3 young

8.4 young

aggression
(males)

as B, C

as A, C

as A, B

against (males)

Analyse these results and answer the following questions:


(1)

What is the time-saving of a pair (in %) with the duration of incubation of


14 to 16 days, if the female does not mate with male A but with male C?
........21,3.............to .......24,3....... %
(incubation of 14 to 16 days)

(2)

How many offspring on average does a female have with a male C (long
tail feathers) compared to an ordinary male A (result in %)? .....68...... %

(3)

According to the results, what determines the most successful reproduction of breeding pairs? Mark with a cross.
the condition and the behaviour of females in the choosing of
a mate against males (female choice)
the condition and the successful competition of males against
each other (male-male competition)
the condition and the successful competition of females
against each other (female-female competition)
the condition and the behaviour of males in the choosing of a
mate against females (male choice)

132

th

Report of the 9 IBO 1998 in Kiel

B 20:

Preference of male and female body proportions in selective

(4 pts) model experiments.


In a very extensive investigation (n = 10,000 people) children, youths
and adults were each shown two outline pictures of the male and female body (models).

After the comparison each person stated their preferred model. The following
table (next page) shows the results for 4 - 20- year-olds, divided by
I.

age of the experimental subject

II.

sex of the experimental subject

III.

sex of the preferred model

133

Theoretical Test Part B

Selective behaviour of males and females (body outline in double selection)


age (years)

4
5
6
7
8
9
10
11
12
13
14
15
16
17
18
19
20

choice by
(males)
%
45
43
39
35
33
32
28
26
26
28
34
42
50
57
63
67
68

choice by
(males)

%
55
57
61
65
67
68
72
74
74
72
65
57
50
43
37
33
32

choice by
(females)

%
43
44
44
42
39
37
36
36
37
42
50
57
63
66
67
63
58

choice by
(females)

%
57
56
56
58
61
63
64
64
63
58
50
43
37
34
33
34
42

Evaluate the table and mark the correct answer with a cross.
(1)

Which models do younger females prefer?


male models
female models

(2)

Which models do older males prefer?


male models
female models

(3)

Between which ages do females change from non-preference (< 50%) to


preference (> 50%) of male models?
at 14
at 15
at 16

134

p.t.o.

th

Report of the 9 IBO 1998 in Kiel

(4)

Between which ages do males change from no-preference (< 50%) to


preference (> 50%) of female models?
at 14
at 15
at 16

(5)

Which hypotheses correspond to the data?


Mark the correct statements with a cross.
Hypothesis 1: Secretion of growth hormones from the tissue
of the thyroid gland causes the orientation for mates of the
opposite sex. The hormonal effect starts at a different time in
boys and in girls.
Hypothesis 2: Sexual learning optimises the preference of
the opposite sex. The hormonal effect starts at a different time
in boys and in girls.
Hypothesis 3: The neurophysiological maturation of the preference concerning the opposite sex is promoted by the increasing amount of oestrogen ( ) and testosterone ( )
during puberty. It develops at different times during maturation
in boys and girls.

B 21:

Mate selection in grasshoppers (Tettigoniidae, crickets)

(2 pts) In the population of one species A males which are ready to reproduce
sing on fixed locations. The females are orientated towards the songs,
approach the males and get on them (inversion of the sexual role
com-pared to other grasshoppers). The males produce one spermatophore during their lifetime, and it can only be given to one female.
Nevertheless, a female can produce more than one clutch of eggs and
mate with several males in succession. Contrary to the otherwise rare
fema-le reproduction products (compared to the number of male gametes) in this case the females should have a measurably higher success rate in reproduction. An investigation showed the following results:

135

Theoretical Test Part B

average weight of females average number of eggs


before eggs are laid
per female
Rejected by males

3.23 g

30

Accepted by males

3.71 g

48

Answer the following questions according to the results:


(1)

What is the average weight of a female without eggs?


2.43 g
2.24 g
2.32 g

(2)

How many eggs can each of four males fertilize, when it mates with a
female that produces four clutches of eggs in succession and weighs
3.87 g, 3.74 g, 3.52 g and 3.74 g, respectively when copulating?
male 1 with female 1

.....54...... eggs

male 2 with female 1

.....49...... eggs

male 3 with female 1

.....41...... eggs

male 4 with female 1

.....49...... eggs

B 22:

Spectrum of prey and food selection of the oyster catcher

(2 pts)

(Haematopus ostralegus)
Oyster catchers are birds that look for prey in the shallow water of sea
coasts, e.g. bivalve. The shells in a habitat are of different sizes and
these sizes correlate with the amount of energy in the shell meat:
Small shells can be opened fast and easily (costs) but provide less
food energy (benefit).
Big shells provide more energy (benefit), but they are more difficult
and time-consuming to open (costs).

136

th

Report of the 9 IBO 1998 in Kiel

The following table shows


I.

the total amount of energy of big shells of different size

II.

the frequency of distribution of all shells present

III.

the frequency of distribution of the shells selected

Number and energy content (relative units) in shells living in a feeding territory
that were selected by birds
total amount shells present energy pre(n)
sent (E)
of energy
per shell (E)
1
1
1
2
17
34
3
11
33
4
9
36
5
6
30
6
7
42
7
20
140
8
22
176
9
19
171
10
13
130
11
6
66
12
2
24
13
1
13
1 to 13

134

sum 896

shells selected (n)

energy consumed (E)

0
0
1
3
6
7
11
10
14
13
6
2
1

0
0
3
12
30
42
77
80
126
130
66
24
13

74

sum 603

Analyse and evaluate the table and answer the following questions:
(1)

What total amount of energy could be expected from 74 shells that were
randomly selected from 134 shells?
.......495...... energy units E (propotional)

(2)

What is the absolute total amount of energy and the total amount of energy in percentage, that can additionally be consumed from 74 shells
that were selected by birds.
........108........ energy units R (real)

........21,8....... % benefit

137

Theoretical Test Part B

GENETICS/EVOLUTION

B 23:

Which characteristics does an interspecific hybrid from crossing

(2 pts) diploid flowering plants have? From the list below, select and
write down the code numbers of the correct reasons or explanations in the circles below.
properties of the hybrid
3

amphihaploid (haploid form of amphihaploid)


amphidiploid (alloplyploid with double the normal chromosome number)
fertile, but its offspring are sterile
usually fertile, if the parents belong to the same genus
sterile and therefore not suited for plant breeding

It can be made fertile by the toxin colchicin.


It can be made fertile by the phytohormone -indolelacetic
acid (IAA).

Code numbers for reasons and definitions:


1. The interspecific infertility is a significant feature of the species.
2. The embryonic development is impaired.
3. Each type of chromosomes is present only once.
4. Two chromosome sets are present.
5. The formation of fertile gametes is impaired.
6. The treatment facilitates the meiotic chromosome pairing to take place.
7. The treatment facilitates fruit maturation.
8. The treatment does not lead to the mentioned result.

138

th

Report of the 9 IBO 1998 in Kiel

B 24: In DNA replication the 3-end priming problem is known (end rep(2 pts) lication problem). From the list below, write in the circles in the
corresponding genophores (chromosomes DNA) the numbers of
the phenomena by which the problem is solved or avoided.
genophores
1

main genophores of eubacteria

plasmids of eubacteria

main genophores of archaea bacteria

mitochondrial DNA

plastid DNA

chromosomes of nuclei

DNA of lambda-phages

DNA of adenovirus

DNA of retrovirus

code numbers for phenomena:


1. circular nature of DNA
2. pre-replicative circularization of the DNA
3. replication jumps from the 5-end of the leading, to the 3-end of the
lagging strand
4. telomerase activity
5. specific primer-independent DNA replicase
6. transcription / reverse transcription
7. polyadenylization
B 25: Isolated protoplasts of two albino mutants of potatoes were
(2 pts) fused. A variegated plant was obtained. Where are the mutant
genes, which have caused the chlorophyll deficiency, located?
A new mutation, plastid communication and plastid DNA recombination should be excluded.
Mark the correct answer with a cross.
X

in the nucleus of one mutant, in the plastids of the other one


in the nucleus of both mutants
in the plastids of both mutants

139

Theoretical Test Part B

B 26:

Compare the following types of anaphase.

(2 pts) Which of the following statements are correct for anaphase I of


meiosis? Which statements are correct for the anaphase of mitosis? Write the correct code numbers from the list below, in the
spaces beside the anaphase types. Pay attention to the fact that
chromatides are subunits of chromosomes.
1. phase I of meiosis: ........1, 3, 4, 5..........
2. anaphase of mitosis:

..1.............5..........

code numbers:
1.Chromosomes move to the poles.
2.Single chromatids move to the poles.
3.The moving chromosomes consist of two chromatides.
4.The chromosomes can carry two alleles of a gene.
5.One haploid set of chromosomes can move to each pole.
6.There are four spindle poles per cell.
B 27: In the course of the evolution of plants the haploid life phase,
(2 pts) i.e. the gametophyte, has been more and more reduced and the
diploid life phase, i.e. the sporophyte, has become the dominant
generation. Write the number of the following taxa in the correct
order according to the increasing dominance of the diploid life
phase.
I.

Ulothrix (Chlorophyta)

II.

Chlorella (Chlorophyta)

III.

Pinus (Pinophyta)

IV.

Polytrichum (Bryophyta)

V.

Rosa (Magnoliophyta)

VI.

Navicula (Phaeophyta, Diatomeae)

........... II, I, IV, III, V, VI..................


140

th

Report of the 9 IBO 1998 in Kiel

B 28:
(1 pt)

S K I P P E D .
The principle of natural selection is essentially important to Darwins theory of natural selection, and this is essentially important
to our understanding of evolution. According to the principle of
natural selection, exponential growth of populations forces selection to occur again and again.

Which of the following statements is correct? Mark with a cross.

The principle of natural selection is a rule, as it dominates the


evolution of many, but not all species of organisms.
The principle of natural selection is a law which dominates
evolution of all species of organisms only.
The principle of natural selection is a natural law which is only
valid within the biotic world where it dominates the phylogenesis of all species and the ontogenesis of all organisms.
The principle of natural selection is a natural law which in
both the biotic and abiotic world dominates all processes in
which self-replication of basic entries occurs.
The principle of natural selection is only seemingly valid in
evolution. In reality, it dominates nothing else than selforganization of dynamic order in abiotic systems.
The principle of natural selection is not a natural law but,
rather, a tautology: The fittest are the most successful in survival and reproduction and, hence, they are the fittest.

141

Theoretical Test Part B

ECOLOGY

B 29:

The dependence of the net photosynthesis on temperature fol-

(1 pt)

lows a mathematical function. Show the dependence of the net


photosynthesis on temperature ising the axes below, of a C3 plant
at light saturation.

net rate
of photosynthesis

+2
+1
0
-1

temperature

procuction

B 30: The following graph shows the changes in values of various bio(2 pts) logical factors/parameters in relation to increasing density of
fish.

population density

142

th

Report of the 9 IBO 1998 in Kiel

Match the curves 1, 2, 3 and 4 from the graph to the terms in the table.
Match each curve only once.
curve from the graph
total fish production

individual growth

food potential not used

expenditure of energy of each fish in


the search for food

B 31: The following pictures show fish from various locations.


(2 pts)

A. flat fish
at the bottom of shallow sea

B: tuna
in the open water in sea

C: angler-fish
at the bottom of deep sea

D: eel
predominantly at the
bottom of fresh water

Match the fish A, B, C and D from the picture to the gill surfaces in the table.
143

Theoretical Test Part B

Gill surface per gramm body weight (in arbitrary units) of fish of various locations.

B 32:

gill surfaces

fish

2551

902

462

51

The following figure shows a section of a food web in an ecosys-

(3 pts) tem.

chemoautotrophic
producers

bacteriophages

predator (2)
predator

Write down the numbers from the figure at the appropriate place in the
following table.

144

th

Report of the 9 IBO 1998 in Kiel

Parts from a food net in an ecosystem

part from the food web

assigned number from the picture

organic waste

pantophage (omnivore)

saprophage (delritivores)

photoautotrophic organisms (producer)

phytophage (herbivore)

decomposers

mycophages

parasitic animals

zoophage (predator I)

145

Theoretical Test Part B

B 33: Genetically identical seedlings of the sunflower (Helianthus, As(2 pts) teraceae) were planted in two neighbouring gardens A and B.
After three months the plant height was measured with the following results:
garden

number of plants

average height
(in mm)

standard
deviation

A
B

12
16

1.6
1.4

0.3
0.2

How can the differences be evaluated regarding the growth?


(1)

The differences are accidental. The growth conditions in both gardens


are the same (null hypothesis).

(2)

The differences do not occur by coincidence. The growth conditions in


garden A and B are significantly different (null hypothesis is rejected).
You have to check the question with the help of the t-test.
The following t-values should be used.
probability
90 %
95 %
97 %
99 %
99.5 %

critical t-value
1.32
1.71
2.06
2.48
2.78

Exercises:
(1)

Calculate the standard deviation of each difference between the average


values for garden A and B for this t-test. Use the following formula:

S 2 S 2
S = A n + B n
A
B

Result: ....................... S = 0,10 ......................


(2)

Calculate the t-value and determine by which probability the null hypothesis can be rejected (the differences are significant). Mark with a
cross.
probability

146

< 90 %

95 to 97.5 %

99 to 99.5 %

th

Report of the 9 IBO 1998 in Kiel

90 to 95 %

97.5 to 99.0 %

> 99 %

SYSTEMATICS/TAXONOMY

B 34:

The following illustration shows the life cycle of a fungus.

(2 pts)

(1)

Which fungus group does the fungus whose life cycle is shown belong to? Mark the correct answer with a cross.
Ascomycota
Basidiomycota
X

(2)

Zygomycota
Which kind of spores are spread at X?
haploid meiotic spores

haploid mitotic spores


diploid spores

(3)

Which way of life is typical for this fungus?


X

This fungus is a saprophyte.


This fungus is a parasite.

(4)

Where does the meiosis take place?


The meiosis takes place at X.
147

Theoretical Test Part B

The meiosis takes place at Y.

B 35: A synapomorphy is a homologous characteristic of various spe(3 pts) cies (therefore Syn-), that has evolved/is derived (apomorphic)
compared with the primary absence or previous state of that
characteristic in other species. The original characteristic m
evolves into m and m on the one hand and to m* on the other
(see figure).
Note that m is apomorphic compared with m, but not compared
with m. M, N, O and P could be the taxa in which the characteristics occur.

Mark the statements which are compatible with the definition of synapomorphy.
m is a synapomorphy of N compared with m in M
m is a synapomorphy of N
m is a synapomorphy of NO (N plus O) compared with m in M
m is a synapomorphy of O compared with m in M
m is a synapomorphy of O
m is a synapomorphy of O compared with m* in P
m is a synapomorphy of O compared with the primary
absence of m in P
m is a synapomorphy of N compared with the primary
absence of m in M

148

th

Report of the 9 IBO 1998 in Kiel

the secondary lack of m in O is a synapomorphy of O


compared with m in N
B 36:
(1 pt)

A set of species is called monophyletic if it contains exactly all


known species which are decendants from one well character-

ized
but nevertheless hypothetical stem species.
For calling a studied set of species monophyletic the proof of at
least one synapomorphy of this set is
necessary and sufficient
X

necessary, but not sufficient


not necessary, but sufficient
not necessary and not sufficient

B 37: In cladistics (a method of systemactics) the outgroup comparison


(2 pts) plays an important role. It works as follows: The states of the
characteristics a, b and c marked black and white, are present in
the taxa M, N, O as shown in the figure. Suppose MNO (M plus N
plus O) are monophyletic. P is then its outgroup. As the states
marked black occur only within MNO they are apomorphic compared with the states marked white which occur both in MNO and
the outgroup P.

What is correct? Mark with a cross.


1. The states of the characteristics m, n and o marked black are not
only apomorphies but even synapomorphies of MNO, MN and M compared with the states marked white.

149

Theoretical Test Part B

2. Because of the evidence of synapomorphy, the states of m, n and o


marked black are even homologies of MNO, MN and M, respectively.
3. If the states of a, b and c marked black do not occur outside of MNO,
MN and M, respectively, these sets of species are monophyletic.
4. The outgroup comparison method has led to a circular argument.

MICROBIOLOGY/BIOTECHNOLOGY
B 38: The diagram represents a sequence of reactions taking place in a
(2 pts.) bacterium, in which amino acids are produced from one another by
the action of specific enzymes. Numbers 1 to 6 represent different amino acids; letter V to Z represent different enzymes. All the
amino acids are essential for survival. The original strain of the bacterium required only amino acid 1. A mutant strain of the bacterium could only survive when provided with amino acids 1,2 and
5 in its culture medium. Which enzymes were missing in the
mutant strain?
V

4
X

6
Z

In the mutant the following enzymes were missing: ..... V, Z .....


B 39:

Mark the correct answers with a cross. (2 pts)


1. Yeast cells used for wine production tolerate up to 14 % ethanol.
2. Citric acid is produced industrially with the help of Aspergillus niger
in an anaerobic process.
3. By mutagenesis and strain selection bacterial and fungal production
of antibiotics is increased on a higher level than by gene amplification.
4. Cephalosporins are produced by bacteria.
5. The flock of activated sewage sludge is the active microbiological
aggregate in anaerobic waste water treatment.
6. It was found in practice that microorganisms modified by gene tech
nology for in situ remidiation of organic contaminations in the soil or
ground water are inferior to the adapted, natural microbial community
with regard to their cleaning performance.

150

th

Report of the 9 IBO 1998 in Kiel

B 40:
X

What is correct? Mark with a cross. (1 pt)


H2 can be used as an electron donor by chemolithoautotrophic bacteria.
Sulphate reducing bacteria are inferior to methanogenics bacteria when
competing for hydrogen.
H2 is used for the denitrification of drinking water.

151

Theoretical Test Part B

152

th

Report of the 9 IBO 1998 in Kiel

5.2 Practical Tests


5.2.1 Laboratory 1: Systematically-anatomically orientated
Introducing remarks:
In laboratory 1 you have to solve both of the following tasks:
Task 1:

Anatomical examination of a plant leaf

Task 2:

Examination and systematic classification of four different plant


species.

Read the questions for both tasks and then decide in which order you would
like to work.
Please, do not forget to write down your personal data (Surname, First name,
Country, Code Number) in the space provided.

Task 1:

Examination of a plant leaf

(14 pts)
Introducing remarks:
The required equipment and chemicals for the examination are at your bench.
Use the equipment and chemicals suitable for you examinations. Make several
cross sections through the penne of the leaf (figure 1). Stain these cross sections with the aqueous solutions of the dyes Astra-blue (stains cellulose) and
Auramin (stains lignin) and analyse the anatomic structures under the microscope. As supplementation you can make longitudinal and surface sections.

1 (1.1)

Label the given schematic drawing (figure 1). Write down the

(9 pts)

code numbers of the given anatomical structures next to the


correct reference lines.

151

Practical Test Lab 1 (Systematics Anatomy)

Code numbers:
1.

Cuticula

2.

Epidermis

3.

Hypodermis

4.

Spongy parenchyma

5.

Palisade parenchyma

6.

Vascular bundle sheath

7.

Xylem with lignin

8.

Phloem with sieve cells


and parenchyma

9.
10.

Transfusion tissue
Accessory transfusion
tissue

11.

Stomatic cells

1 (1.2) Which of the following systematical groups does the object refer
(2 pts) to? Mark with a cross.
Fern (Polypodiophyta, Pteridophyta)
Gymnosperms
Angiosperms

1 (1.3) Evaluate the object from the ecological point of view.


(3 pts) Mark with a cross.
Hydrophyte (swamp or water plant)
Hygrophyte (plant with adaptions to damp locations)
Mesophyte (plant with adaptions to balanced locations)
Xerophyte (plant with adaptions to dry locations)

152

th

Report of the 9 IBO 1998 in Kiel

Task 2:
(24 pts)

Examination and systematic classification of four different


plant species.

Introducing remarks:
On your bench there are four vessels. In each vessel there is a flowering shoot
of a plant species. The species are labelled A, B, C and D. The required
equipment and a magnifying glass microscope are on your bench as well.
Analyse the given objects A, B, C and D morphologically. Answer the following
questions with the help of the answer code and fill in the corresponding code
numbers in the table below.

1 (2.1) Which floral diagram (D 1 - D 6) belongs to which species? Fill in


(4 pts) your decision (code number) in the table at the end of this task.

use
D1
as code numbers

D2

D3

D4

D5

D6

1 (2.2) Which family name belongs to which species? Fill in the corre(8 pts) sponding code number in the table.
Code numbers:
1. Asteraceae 4.

4. Lamiaceae

2. Brassicaceae

5. Oleaceae

3. Fabaceae

6. Ranunculaceae

7. Rosaceae

153

Practical Test Lab 1 (Systematics Anatomy)

1 (2.3) Which species have which kind of fruit (F 1 - F 7)? Fill in the
(4 pts) corresponding code number in the table.
Code numbers:
F 1. Achene

F 4. Fruit composed of four one-seed nutlets

F 2. Follicular fruit

F 5. Aggregate of nutlets

F 3. Pulse

F 6. Stone fruit, Drupe

F 7.

Berry

1 (2.4) Which species have which inflorescence (B 1 - B 7)? Fill in the


(4 pts) corresponding code number in the table.
B 1. Spike

B 4.

Thyrsus

B 2. Capitulum

B 5.

Bunch, Raseme

B 3. Panicle

B 6.

Solitary flower

B 7.

Volute

1 (2.5) Which species have which leaf arrangement? Fill in the corres(4 pts) ponding code number in the table.
Code numbers:
P 1. alternating
P 2. decussate
P 3. distichous
Table
A
Diagram
Family
Fruit
Inflorescence
Leaf arrangement

154

th

Report of the 9 IBO 1998 in Kiel

5.2.2

Laboratory 2: Physiologically - orientated

Task 1: Model experiment referring to the respiratory chain


(23 pts)

Introducing remarks:
Electron transfer through several redox systems to the terminal acceptor as it happens for instance in the endoxidation of the respiratory chain can be simulated in a
model experiment in a test tube, following the so-called BAUMANN experiment.
Add Fe2+ to a solution of sodium acetate containing the amino acid cysteine, shake it
for a short time and write down the colour immediately and after some minutes.
Shake the mixture again and write down the colour immediately and after some minutes etc.
Information: Cysteine forms a coloured compound with Fe3+, whereas Fe2+ does not.

COO

I: Preparation of the experiment

The following solution is necessary for the experiment:

H3N+C

10 ml of aqueous 0,1 M sodium acetate solution containing

the amino acid cysteine in the concentration of 41 mM

CH2SH

(=solution a).
It should be produced by mixing 0,2 M sodium acetate solution (solution b) with 82 mM cysteine solution (solution c).

CYSTEINE

2 (1.1) How many mg sodium acetate (waterless) have to be weighed for


(3 pts) 00 ml of the 0,2 M sodium acetate solution? For calculation use
the following
atomic masses:

H:1;

Result (no decimals)

O: 16;

C: 12;

Na: 23.

1640 mg

155

Practical Test Lab 2 (Physiology)

2 (1.2) How many g cysteine have to be weighed for 100 ml of the 82 mM


(3 pts) cysteine solution? For calculation use the following atomic
masses:
H: 1;

O: 16;

C: 12;

S: 32;

N: 14.

Round off the result (no decimals):

1g

Obtain solution b (6 ml), solution c (6 ml) and the required Fe SO4 from the
assistant and start the experiment.

II: Procedure
2 (1.3) Mix solution b and solution c in the correct volume ratio to get
(2 pts) 10 ml of the required solution a.
The use of a pipette aid is obligatory.
Present your experiment and the remaining solutions immediately for judgement (score credit) to the laboratory advisor. Leave your pipette aid with the
laboratory advisor.
If you have made a mistake in the procedure you can get solution b and solution c again. In this case you cannot score in (3).

2 (1.4) Add FeSO4 to the solution a, plug the test tube with a stopper and
(1 pt)

shake it strongly. Which colour does the solution have after


shaking? Mark with a cross.
red
white
green
blue-violet

2 (1.5) Leave the test tube open (without stopper) for 1-2 minutes. Which
(1 pt) colour does the solution have after this time? Mark with a cross.

156

th

Report of the 9 IBO 1998 in Kiel

red
white
green
blue-violet

2 (1.6) How often can the colour change be achieved by shaking?


(1 pt) Mark with a cross.
once
up to 5 times
more than 6 times
not repeatable

III: Evaluation of the experiment:


2 (1.7)

Which component(s) is (are) used up in the experiment?

(1 pt)

Mark with a cross


iron and oxygen
cysteine and oxygen
sodium acetate
oxygen
cysteine
none

2 (1.8) The redox potential Eo of the system Fe3+ / Fe2+ is + 0,77 Volt.
(1 pt) Which redox potential does an e- donator of this system have?
Mark with a cross.
greater than + 0,77 V

less than + 0,77 V

equal to + 0,77 V
2 (1.9) Which value does the redox potential Eo of the e- terminal
(1 pt) acceptor have in the performed experiment? Mark with a cross.

157

Practical Test Lab 2 (Physiology)

greater than + 0,77 V


less than + 0,77 V
equal to + 0,77 V

2 (1.10) Which enzyme in the respiratory chain is analogous to the Fe(1 pt) system in the BAUMANN experiment? Mark with a cross.
NADH ubiquinone reductase
cytochrome oxidase
ATP synthase
ubiquinone cytochrome c reductase
2 (1.11) Complete the incomplete schematic drawing of the electron
(8 pt)

transfer in the BAUMANN experiment.


You can score 1 or 2 points per correct item in

H2O

2e

2e

1/202

Cys-s-s-Cys
2 H+

158

th

Report of the 9 IBO 1998 in Kiel

Task 2: Imbibition movements of the awns in Avena sterilis


(17 pts)
Introducing remarks:
The basis for each imbibition movement is the swelling capability of wall substances. If imbibition occurs in all directions equally, it is called isotropic; if one
direction is preferred, the imbibition is called anisotropic. Curvatures can be
the result of a substantially different wall structure as well as a different texture
even in homogeneous material. Texture is the arrangement of fibrils in the
wall layers. The movements to be observed in the proposed experiment are
caused by different textures. Your object is distinguished by two conspicuous
awns. You have to examine their movements. A magnifying glass (direct light
and transmitted light) is available.

I. Procedure:
2 (2.1) Observe the movement of the awns after moistening.
(2 pts) Put the air-dry spikelets together with the awns on a piece of paper and sprinkle the awns with water (use the sprayer) and observe (with the help of the magnifying glass as well) your object.
Which reactions can you observe? Mark with a cross.
No reaction
Both of the awns show a synergic rotation based on decoiling.
The awns do not touch each other.
Both of the awns show a synergic rotation based on spiralisation.
Occasionally they hook together and sometimes start saltatory
movements.
Both of the awns show a synergic rotation based on decoiling.
Occasionally they hook together and sometimes start saltatory
movements.
The movement of one awn is based on decoiling, the movement of
the other one on spiralisation.

159

Practical Test Lab 2 (Physiology)

2 (2.2) Observe the awn movement when drying.


(2 pts) Let the awns dry after the experiment (switch both lamps on full
power and use them for heating). It can take 10 to 15 minutes until you can see a reaction. Observe your object. Which of the following statements is correct? Mark with a cross.
Both awns show a synergic rotation, which is based on spiralisation, occasionally they hook together, and sometimes start
saltatory movements.
Both awns show a synergic rotation, which is based on
decoiling. The awns do not touch each other.
Both awns show a synergic rotation, which is based on
spiralisation.
The awns do not hook together.
Both awns show a synergic rotation, which is based on
decoiling, occasionally they hook together, and sometimes
start saltatory movements.
The movement of one awn is based on decoiling, the
movement of the other awn is based on spiralisation.

II. Evaluation
2 (2.3) Texture and imbibition of cellulose
(3 pts) On your bench there are two pieces of typing paper. According to
the manufacturing process the cellulose fibres are essentially
longitudinally orientated.
2 (2.3a) Give the paper a good spray of water from both sides and decide
(1 pt)

after about 5 minutes whether the paper preferably extents in the


longitude of the fibre (A), cross to the fibre direction (B) or in
both directions equally (C). Mark with a cross.
A
B
C

160

th

Report of the 9 IBO 1998 in Kiel

2 (2.3.b)What is the maximum extention given in percent?


(2 pts) Fill in the result:

2,3 %

2 (2.4) Perform a mental experiment.


(8 pts) In figure 1 the fibre direction of a piece of paper is given in dotted
longitudinal lines. Stripes of paper are labelled a, b and c.
Imagine: The stripes given in figure 1 are cut out, moistened and
clued together identically as shown in column 1 of the table 1.
For drying the stripes are hung up at the upper end (*).
Draw the expected form modifications as a side view in column 2
(table 1).
Column 1

Column 2

2 (2.5) Link the observations regarding the awn movement to the result
(2 pts) of the (4) pilot experiment (mental experiment).
Which texture are the movements of the awns based on?
Mark with a cross.
( aa )
( bb )
( ba )
( ab )
( ac )

161

Practical Test Lab 3 (Genetics Microbiology)

5.2.3
tated

Laboratory 3: Genetically - microbiologically orien-

Task 1: Isolation of bacterial DNA


(12 pts)
Introducing remarks:
DNA should be isolated from cells of a 5-day-old liquid culture of the bacterium
Escherichia coli K 12. Various biochemical steps have to be done in a certain
order under different temperatures and incubation times. The tasks are: (1) to
determine the correct order of these steps, (2) to assign a specific temperature
and incubation time to these steps and (4,5,6) to select the correct substances
or solutions for the relevant steps.

3 (1.1) Determine the correct order of the steps. Write down the code
(4 pts) letters of the following steps in the correct order.
A

winding up the DNA

disruption of the cytoplasmic membrane

precipitation of the uncleaned DNA

disruption of the bacterial cell wall

Order: D, B, C. A...
3 (1.2) Assign one of the temperatures (T) and incubation times (I) given
(3 pts) below to each of the following steps.
Disruption of the cytoplasmic membran

T: .+ 600 C I: 2 min..

Precipitation of the uncleaned DNA:

T: .

Disruption of the bacterial cell wall:

T: . +370 C. I: 10 min.

Incubation time: 30 sec,


Temperature:

0 C,

2 min,

10 min,

15 min,
0

00 C I: 30 sec

30 min

room temperature, +37 C, +600 C,

+1000 C

Realize the following steps in the order you have determined.


162

th

Report of the 9 IBO 1998 in Kiel

3 (1.3) STEP A: Winding up the DNA


(2 pts) Try to wind up the precipitated DNA strands with a wooden rod.
Ask the laboratory advisor to score the result.
(The laboratory advisor is going to sign.)
DNA could be wind up
DNA could not be wind up

3 (1.4) STEP B: Disruption of the cytoplasmic membrane.


(1 pt) To disrupt the cytoplasmic membrane you have to add 0.2 ml of
one of the following solutions.
Mark the correct answer with a cross.
HCl (1 M)
acetone (pure)
glycerine
sodiumdodecylsulphate solution (25 %)
NaOH (1 M)

3 (1.5) STEP C: Precipitation of the uncleaned Dann


(1 pt)

For the precipitation of the uncleaned DNA the lysate has to be


overlayed with a liquid. Select the correct one from the following
liquids and mark it with a cross.
Realize the overlaying in the following way: Let the fluid from a
beaker slowly run down the wall of the test tube. The well-perceptible
whitish strands which separate slowly from the aqueous phase after
overlaying predominately consist of chromosomal DNA.
ethanol (96%, denatured)
double distilled water (aqua bidest.)
HCl (1 M)
NaOH (1 M)
glycerine
163

Practical Test Lab 3 (Genetics Microbiology)

3 (1.6) STEP D: Disruption of the bacterial cell wall


(1 pt) To disrupt the bacterial cell wall a tip of a spatula of one of the
following substances has to be added to a culture of Escherichia
coli K12 (in a test tube).
Mark the correct substance with a cross.
penicillin
lysozyme
urea
CaCl2
trypsine

Task 2: Investigation of the mitosis in the root meristem of a


(25 pts) specimen of the Magnoliopsida (Dicotyledoneae)
Introducing remarks:
Young roots were harvested from plants grown in sand and differently treated:
Treatment (1): Incubation (for 4 hours) in the aqueous solution of a
substance that inhibits chromosome movement
Treatment (2): Incubation (for 24 hours) in ethanol/glacial acetic acid
(3:1)
Treatment (3): Incubation (for 24 hours) in a saturated solution of car
mine in 45 % acetic acid
Treatment (4): Heating in a boiling water bath for 5 minutes
On your bench you can find two different samples.
Sample I:

Root tips only submitted to treatments (2) - (4)

Sample II:

Root tips submitted to treatments (1) - (4)

3 (2.1) Compression preparations


(2 pts) Make compression preparations of the samples I and II in which
as many cells as possible are in the cell cycle, lying in a

164

th

Report of the 9 IBO 1998 in Kiel

monolayer and flattened in such a way that the chromosome


arms are pushed into a focus layer but are not squash-damaged.
Apply a technique you are familiar with or follow the procedure given below.
Separate the apical meristem of a root in a droplet of 45 % acetic
acid on the slide.
Fragmentate the tip by distearing and slight squashing.
Cover it with a cover glass.
Disrupt the tissue clumps by gentle hints with the blunt end of a pair
of forceps held upright down on the cover glass.
Adjust the amount of liquid so that the cells glide but do not float.
In case of insufficient spreading add 45 % acetic acid; slightly lift the cover
glass with a needle and avoid shifting of the cover glass by a pair of forceps.
When spreading is sufficient the preparate is covered with a tissue and a gentle pressure by a finger is applied in order to flatten the cells. Attention! The
danger of squash-damaging is particularly high when the cells are well spread.
Present your preparates (under the microscope) to the laboratory
advisor for quality control and scoring at any time.

3 (2.2) Scetches of different cell stages


(8 pts) Prepare scetches of the stages of the cell cycle using sample I
(a), (b), (c), (e), and (f) and a metaphase-like figure of sample II (d)
on the following blank page. Note the list of the stages of the cell
cycle below. Draw a chromatide and a daughter chromosome
only with a line but otherwise corresponding to your preparate;
unclear regions may be circled with dotted lines.
Stages of the cell cyle:
(a)

interphase (1 pt)

(b)

middle prophase (1 pt)

(c)

metaphase plate in side view (3 pts)

(d)

metaphase-like figure of sample II seen from above (1 pt)

(e)

late anaphase or early telophase (1 pt)

(f)

late telophase (1 pt)

165

Practical Test Lab 3 (Genetics Microbiology)

Blank piece of paper (task 2). Scetches of cell stages.


3 (2.3) Detailed drawings of chromosomes
(10 pts) Draw two metaphase chromosomes from sample I, true to
nature, and label the drawing.
3 (2.3a)
(2 pts)

Draw two significantly different chromosomes.

3 (2.3b) Characterize the drawn chromosomes by using capital letters


(3 pts) according to the following terms.
A:
B:
C:
D:
E:

metacentric
acrocentric
dicentric
acentric
satellite chromosome

3 (2.3c) Label the parts of the larger chromosome using the code
(5 pts) numbers of the following terms.
1. chromosome arm
2. chromatide arm
3. daughter chromosome
4. centromere
Labelled drawings (task 3):

5. centrosome
6. satellite
7. nucleolus organisator region
8. nucleosome

3 (2.4) Chromosome number


(1 pt)

Indicate the chromosome number of sample I. ......12........

3 (2.5) Ploidy level


(1 pt) Indicate the ploidy level of sample I. Consider that a distinctive
chromosome type occurs only once in the haploid chromosome
set. Mark with a cross.
haploid (1 n)
diploid (2 n)
triploid (3 n)

166

th

Report of the 9 IBO 1998 in Kiel

tetraploid (4 n)
3 (2.6) Effects of the treatments (1) - (4) (see introducing remarks p.7)
(1 pt)

Match the treatments (1) - (4) to the effects you achieved. Write
down the numbers of the treatment in the corresponding circles.
inhibition of the centromere fission
inhibition of tubuline association
fixation of protoplasmic structures
shortening of the chromosomes
staining of the chromatine
staining of the nucleus spindle
destruction of the middle lamella
destruction of the primary walls

3 (2.7) Treatment (1)


(1 pt) Which substance was used in the treatment (1)?
Mark with a cross.
colchicine
bromouracile
- amanitin
actinomycine
mitomycine
3 (2.8) Which aim was followed by J.H. Taylor (1958) when he applied
(1 pt)

the same substance? Mark with a cross.


production of polyploid plants
verification of the hypothesis of semiconservative replication
in eukaryotes
inhibition of mRNA synthesis
inhibition of translation
investigation of the cytomorphogenesis in Desmidiacea

167

Practical Test Lab 3 (Genetics Microbiology)

168

th

Report of the 9 IBO 1998 in Kiel

5.2.4

Laboratory 4: Ethologically orientated

Task: Experimental investigation of triad grasp, forceps


grasp
(35 pts) and tool (grasp of man as functions of primate hands).
Introducing remarks:
During the evolution of primates the grasping hand has developed in a way
that it can be used for locomotion as well as for manipulation of objects.
Therefore a differentiated co-operation of hand, eye and brain is necessary.
For grasping or splitting tiny food items the ability to manipulate is particularly
important (fine tuned motor function) to get sufficient food energy per unit of
time and without foreign bodies.
The required equipment and objects are situated on your bench: 2 Petri
dishes, a pair of angle forceps, sea sand (sieved), 20 sunflower seeds and a
stop-watch. Precision scales are available for the weighing procedure.

Task:
4 (1)

Move your thumb and index finger of one hand without any

(2 pts) optical control against each other. In which area do they touch
each other? Mark with a cross.
inner side of the thumb and outer side of the finger
outer side of the thumb and inner side of the finger
outer side of the thumb and outer side of the finger
inner side of the thumb and inner side of the finger
4 (2) How is this co-ordination possible? Mark with a cross.
(2 pts)
by learning
by ontogenetic maturing
by the construction of the skeleton

169

Individual Results and Medals Awarded

by neuronal decisions
by flexor and extensor muscles
4 (3)

Figure 1 shows the print of a mans hand (Dr. Lucius), figure 2

(2 pts) shows the print of a chimpanzees hand. How are the


recognizable structures on the surface of the inner hand of man
and chimpanzee called?
S K I P P E D.
capillary ridges in both
tactile fields in both
prehensile relief of chimpanzees, papillary ridges of man
papillary ridges in both

Print of a chimpanzees hand (figure 1).

Print of a Dr. Eckhard R.Luciuss hand (figure 1).

170

th

Report of the 9 IBO 1998 in Kiel

4 (4)

Investigation of triad grasp, forceps grasp and tool grasp of

(29 pts) man: Arrange 20 sunflower seeds in the sand-filled Petri dish I in
such a way that they lie flat, istributed over the whole dish, and
do not touch each other.
4 (4a) Grasp each seed one after the other with your thumb, index
(5 pts) finger and middle finger at the same time (triad grasp). All 20
seeds are put individually one after the other in the empty Petri
dish II. Measure the required time in minutes and seconds (e.g.
0:49 minutes) with the help of a stop-watch. Smoothen the
sand in dish I, transport all the seeds with the help of a pair of
forceps from dish II to dish I and weigh the sand (on a filter
paper, in mg) that has been transported from dish II when
grasping. Repeat the experiment 10 times, write down all data
and fill in the results in the table on page 11.
4 (4b) Grasp each seed from the smoothened sand with your
(5 pts) thumb and index finger only (forceps grasp). Repeat the
procedure from task (4.1).
Determine the time and the transported amount of sand in 10
experiments and fill in the data in the table on page 11.
4 (4c) Prepare the following experiment as described under (4.1).
(5 pts) However, this time a pair of forceps should be used for picking
up the seeds (tool grasp). Repeat the experiment 10 times,
measure the time and amount of sand and fill in the results in the
table on page 11.
4 (4d) Decide in which of the experiments the smallest amount of sand
(4 pts) (on average) was transported together with the seeds. Mark with
a cross.
smallest amount of sand in (4.1) (triad grasp)
smallest amount of sand in (4.2) (forceps grasp)

171

Individual Results and Medals Awarded

smallest amount of sand in (4.3) (tool grasp)


4 (4e)
Calculate the mean and the standard deviation (x s) of
(10 pts) n = 10 values for the transported amount of sand and the
required time in the experiments ( 4.1) to ( 4.3). Fill in the results
in the table on page 11 and evaluate the significance of the
transported sand with the help of the t-test. Mark below which of
the differences is (are) significant and which is (are)
insignificant.
experiment 1 versus experiment 2

significant

insignificant difference

experiment 1 versus experiment 3

significant

insignificant difference

experiment 2 versus experiment 3

significant

insignificant difference

Table
experiment 1
exp. no.
1
2
3
4
5
6
7
8
9
10
sum
mean

172

time (sec) sand (mg)

experiment 2
time (sec) sand (mg)

experiment 3
time (sec) sand (mg)

th

Report of the 9 IBO 1998 in Kiel

standard
deviation

Evaluation:
(1.) experiment 1:

experiment 2 (sand)

t =_______________;
(2.) experiment 1:

p
experiment 3 (sand)

t = _______________;
(3.) experiment 2:

p
experiment 3 (sand)

t = _______________;

Appendix to the practical test - ethologically orientated


Test for differences in mean between two measuring series.
It has to be tested whether the difference between the results of the three experiments (amount of sand in mg) is statistically significant.
n1 =
n2 =
n3 =

x 1 s1 =

x2 s2 =

x3 s3 =

Degrees of freedom
m1 = n1 + n2 2
m2 = n1 + n3 2
(1.) experiment 1: experiment 2
2

s D = s1 + s2 =

m3 = n2 + n3 2

t=

x1 x2
sD

(2.) experiment 1 : experiment 3


2

s D = s1 + s3 =

t=

x1 x3
sD

(3.) experiment 2 : experiment 3


2

sD = s2 + s3 =

t=

x2 x3
sD

t- table (acc. to STUDENT)


probability p for t

173

Individual Results and Medals Awarded

m
0,05
0,01
0,001
--------------------------------------------------18
2,10
2,88
3,92

174

INTERNATIONAL BIOLOGY OLYMPIAD


THEORY PROBLEMS

1999, Uppsala, Sweden















All IBO examination questions are published under the following Creative Commons license:



CC BY-NC-SA (Attribution-NonCommercial-ShareAlike) https://creativecommons.org/licenses/by-nc-sa/4.0/
The exam papers can be used freely for educational purposes as long as IBO is credited and
new creations are licensed under identical terms. No commercial use is allowed.

INTERNATIONAL BIOLOGY OLYMPIAD


PRACTICAL PROBLEMS

1999, Uppsala, Sweden















All IBO examination questions are published under the following Creative Commons license:



CC BY-NC-SA (Attribution-NonCommercial-ShareAlike) https://creativecommons.org/licenses/by-nc-sa/4.0/
The exam papers can be used freely for educational purposes as long as IBO is credited and
new creations are licensed under identical terms. No commercial use is allowed.

INTERNATIONAL BIOLOGY OLYMPIAD


THEORY PROBLEMS

1997, Ashgabad, Turkmenistan















All IBO examination questions are published under the following Creative Commons license:



CC BY-NC-SA (Attribution-NonCommercial-ShareAlike) https://creativecommons.org/licenses/by-nc-sa/4.0/
The exam papers can be used freely for educational purposes as long as IBO is credited and
new creations are licensed under identical terms. No commercial use is allowed.

IBO 1997, Turkmenistan


THEORETICAL TEST
PART A
1. In plants with flowers (angiosperms), the female gametophyte is very small and consists of
A) 20 to 25 nuclei.
B) 18 to 20 nuclei.
C) 7 to 9 nuclei.
D) 1 to 2 nuclei.
2. In which of the following the gametophyte plant is dominant ?
A) Rhynophyta
B) Bryophyta
C) Lycopodiophyta
D) Pinophyta
E) Equisetophyta
3. A dark -grown oat seedling illuminated from one side with a halogen bulb curves towards the
bulb. A colored filter was inserted between the bulb and the seedling and the effect of illumination
remained unaffected. What was the color of the filter?
A) Red
B) Green
C) Yellow
D) Blue
E) Such an effect is not possible. Only full spectrum white light may have the effect described
above
4. Dichlorophenol indophenol (DCPIP) is a blue dye that is decolorized when reduced. After being
mixed with DCPIP which of the followings would show the greatest change in color?
A) isolated chloroplasts in the dark.
B) isolated chloroplasts in the light.
C) chlorophyll extract in the dark.
D) boiled chloroplasts in the dark.
E) boiled chloroplasts in the light.
5. Phytochrome far red (PFR) inhibits flowering in:
A) Day-neutral plants
B) Vernalized plants
C) Long-day plants
D) Short-day plants
E) Senescing plants
6. In a monocot stem, such as corn, there are no distinct areas of cortex and pith. The tissue that
performs the functions as the cortex and pith is referred to as;
A) Endodermis
B) Ground tissue
C) Meristem
D) Vascular cambium
E) Periderm
7. Which of the following techniques would be most suitable for the study of DNA synthesis in a
cell?
A) Phase contrast microscopy
B) Electron microscopy
C) Autoradiography
D) Serial- sections microscopy
E) Nuclear transplantation

8. In poikilotherm animals, oxidation and phosphorilation is physiologic response to the decreasing


surrounding temperature. In this condition which of the following structures change its function
manner?
A) Nucleus inner membrane
B) Mitochondria inner membrane
C) Mitochondria outer membrane
D) Hyaloplasma
E) Peroxisom
9. Which of the followings occurs in the same direction as evolution?
A) Genetic Drift
B) Gene Flow
C) Natural Selection
D) Mutation
E) Both Genetic Drift and Gene flow
10. In some localities in Africa there are three different distasteful species of butterfly. Different
females of the edible swallow tail butterfly, Papilio dardanus , mimic each of the distasteful models.
So the initial single population has been separated into three different populations that differ in their
color pattern as each has mimicked a different distasteful model. Which of the following explains
this event?
A) Stabilizing selection
B) Directional selection
C) Disruptive selection
D) Bottleneck effect
E) Founders effect
11. Which of the followings is not accepted as prezygotic isolating mechanism in speciation?
A) Gametic Isolation
B) Mechanical Isolation
C) Hybrid Breakdown Isolation
D) Ecological Isolation
E) Temporal Isolation
12. Why are mycoplasmas resistant to penicillin although they are sensitive to tetracycline?
(Tetracycline is a kind of antibiotic that inhibits protein synthesis)
A) Since they lack a cell wall
B) Since they lack nuclear envelope
C) Since they are smaller than viruses
D) Since they are anaerobic
E) Since their cell walls are made of compounds like protein
13. Which of the following livings causes the black bread mold?
A) Ascomycota
B) Basidiomycota
C) Deuteromycota
D) Zygomycota
14. Which of the followings are the fungal partners of nonwoody plants in mycorrhiza symbiotic
relationships?
A) Zygomycota
B) Basidiomycota
C) Deuteromycota
D) Ascomycota
E) Both C and D.
15. Which of the followings controls the amount and kinds of minerals that enter the xylem in the
apoplast route of transportation of water?
A) Semipermeable cell membrane
B) Plasmodesmata

C) Endodermis
D) Cortex parenchyma cells.
E) Pericycle
16. Which of the followings is the origin of multicellular branch roots?
A) Epidermis
B) Cortex
C) Endodermis
D) Tracheid
E) Pericycle
17. In order for a bacterium to produce an eucaryotic protein, which of the followings must first be
isolated from an eucaryotic cell prior to cloning?
A) The protein from the rough endoplasmic reticulum
B) The protein primary DNA transcript from the nucleus
C) The protein of mRNA from the cytoplasm
D) The segments of DNA that control transcription for this protein
E) The introns from the segment of DNA that codes for the protein
18. In humans the majority of ingested calcium is excreted in the:
A) Urine
B) Sweat
C) Plasma
D) Feces
19. The old blind man have not noticed the wasp in his cup of juice. It has string in his throat. What
is the best help for him?
A) To warm his neck
B) To give him some alcohol
C) To give him some cold water
D) To give him some sedative
E) To give him vomitory
20. Which of the following cells recognizes an antigen presented in combination with MHC II?
(Major Histocompatibility Complex II)
A) Suppresser cell
B) Cytotoxic cell
C) Neutrophil cell
D) Helper cell
E) Basophil cell
21. When humans movement become incorrect disorderly trembling and lose equilibrium and there
are pronunciation disorders and speech difficulties the lesion is found in:
A) Spinal cord
B) Thalamus
C) Hypothalamus
D) Medulla oblongata
E) Cerebellum
22. Which of the following structures is formed due to the transformation of the lateral organ?
A) Pressure receptors
B) Semicircular canals
C) Middle ear
D) Cohlea
E) Statocyst
23. Which of the followings depicts the pattern flow of energy and materials through an ecosystem?
A) Food chain
B) Biomass pyramid
C) Numbers pyramid

D) Food web
E) A and D
24. Into a large, tightly stoppered flask filled with nitrogen 100m1 of live yeast
made suspension in minimal medium containing 0.1 mole/l glucose as carbon source was
introduced. After 24 hours somewhat more than 100 ml of carbon dioxide (as measured under
standard conditions) accumulated in the flask (assume no carbon dioxide dissolves in the medium).
What was the approximate concentration of glucose in the medium at that moment?
A) Almost zero
B) 0.01 mole/l
C) 0.05 mole/l
D) 0.075mole/l
E) 0.09 mole/l
25. The principal constitute of myelin is
A) Carbohydrate
B) Lipid
C) Mucopolysaccharide
D) Nucleic acid
E) Protein
26. Various substances can affect the excitability of neuron. Calcium ions make a neuron less
excitable by:
A) Increasing the amount of sodium ions that enter the voltage gated sodium channels
B) Binding to the neurolemma, making it less permeable to potassium ions thus limiting
repolarization and reducing the action potential.
C) Binding to the proteins that make up the sodium channels thus reducing the amount of sodium
ions that can enter.
D) Binding to the active site of the sodium potassium pump which causes repolarization at a
voltage lower than -70 mV
E) Reacting with the negatively charged chlorine ion to form calcium chloride which neutralizes the
membrane potential.
27. In certain terrestrial animals the body surface has evolved as a respiratory structure. This is
characterized by
A)The presence of deep indentations in the body surface to facilitate the diffusion of gas deep into
the body.
B) The exchange of gas directly between all the body cells and the atmosphere
C) Special enzymes that allow for active transport of oxygen into the body and carbon dioxide out
of the body.
D) The counter current exchange of carbon dioxide and oxygen between the air and mucus
covering the animals' body surface.
E) The presence of blood vessels close to the surface to receive and transport oxygen to other
body regions.
28. The function of RUGAE in the stomach is to ______________ .
A) Crush the food
B) Increase the capacity of the stomach.
C) Secrete amylase
D) By muscular contractions, mix the food with gastric juices
E) Increase the surface area for secretion
29. Which statement is false in comparing Metanephridia with protonephridia?
A) Metanephridia have a tuft of cilia to move fluids along while
protonephiridia only have a single flagellum.
B) The metanephridium is open at both ends
C) Metanephridia are surrounded by capillaries which permits useful materials to be reabsorbed
and returned to the circulation
D) Useful materials are reabsorbed in the metanephridia.
E) Water is conserved and urine is concentrated

30. In a colony of honey bees, the dominance of the queen is maintained by the
A) Provision of eggs by the queen
B) Secretion of pheromones by the queen
C) Suppression of sexual development of the workers
D) None of the above
31. Substitution of a nitrogen base in a gene can resulted in:
A) Nonsense mutation
B) Frame shift mutation
C) Missense mutation
D) Silent mutation
E) All kinds of the above mentioned mutations except B
32. The frequency of crossovers between linked genes A and B is 30% between B and C is 40%,
between C and D is 45%, between A and C is 10%, between B and D is 5%. The sequence of
genes on chromosome is :
A) ABCD or DCBA
B) BDCA or ACDB
C) CABD or DBAC
D) DBCA or ACBD
E) DCAB or BACD
33. Some mammals have no collar bone in their shoulder girdle. A typical feature of these
mammals is;
A) Being arboreal
B) Being aquatic
C) Being capable of soaring or active flight
D) Having mobility of forelimbs limited to one plane only
E) Neither of the features given above
34. How old are the earliest known fossils of the genus Homo ?
A) Several hundred thousand years
B) 1-9 million years
C) 10-19 million years
D) 20-100 million years
E) More than 100 million years
35. Which of the followings makes up the inner bark of a woody plant?
A) Primary phloem
B) Secondary phloem
C) Primary xylem
D) Secondary xylem
E) Pith
36. Which of the following statements about the upper and lower epidermis is correct?
A) Lower epidermis cells are living parenchyma cells while upper epidermis cells are not.
B) Upper epidermis cells' wall on the outside of the leaf is thicker than on the inside while it is not
valid for lower epidermis.
C) Upper epidermis cells are relatively transparent while lower epidermis cells contain chloroplasts.
D) The upper epidermis has a thicker cuticle than the lower epidermis.
E) The lower epidermis mostly contains lots of trichomes while there is no trichome in the upper
epidermis.
37. The resemblance between one orchid species and female wasps is so strong that male wasps
mount the flowers and attempt to copulate with them. This is an example of:
A) Parasitism
B) Coevolution
C) Artificial selection
D) Adaptive radiation
E) Selective breeding

38. Fruits that form from a single flower that contains many separate carpels, are referred to as:
A) Accessory fruits
B) Aggregate fruits
C) Dry fruits
D) Multiple fruits
E) Simple fruits
39. How do shade-avoiding plants respond when they absorb more far-red light?
A) They produce more leaves
B) They grow additional parts
C) They branch extremely
D) They grow taller
E) They stop growing
40. Plants respond to wounding by herbivorous insects by producing a plant polypeptide known as
___________ that may trigger the plant to produce proteinase inhibitors that disrupt insect
digestion.
A) Florigen
B) Polyamine
C) Phytoalexin
D) Salicylic acid
E) Systemin
4l. Which of the following models maintains that populations diverge from one another by the slow
accumulation of adaptive characteristics within each population?
A) Punctuated equilibrium
B) Paedomorphosis
C) Hybridization
D) Gradualism
E) Allopoliploidy
42. Proteins synthesized in E-coli have which of the followings at their terminal end?
A) N- formyl- methionine
B) N- acetyl- adenine
C) Adenine triphosphate
D) The AUG codon
E) The UUU codon.
43. Which of the followings are the ancestors of mitochondria and chloroplasts according to
endosymbiosis theory respectively?
A) Purple sulfur bacteria, cyanobacteria
B) Purple non-sulfur bacteria, cyanobacteria
C) Purple sulfur bacteria, green algae
D) Purple non- sulfur bacteria, green algae
E) None of the above
44. Catabolite activator protein __________________ .
A) Increases the affinity of the promoter region for RNA polymerase
B) Decreases RNA polymerase activity
C) Becomes nonfunctional when it binds to cAMP
D) Both A and B are true
E) Both A and C are true
45. The lactose operon regulates the synthesis of ________________ .
A) b - galactosidase
B) Galactoside transacetylase
C) Galactose transferase
D) Both A and B
E) A, B, C are correct

46. TATA box is seen in ________ cells and is the site where ___________ .
A) Procaryotic; RNA polymerase binds
B) Eucaryotic cells; DNA ligase cleaves introns
C) Multicellular eucaryotes, RNA polymerase binds
D) All of the above are true
E) None of the above is true
47. How may recombinant genes be introduced into monocotyledons?
A) By the use of Ti vectors
B) By the use of plasmids as vectors
C) By the use of genetic shotguns
D) By the use of radioactive probes
E) None of the above
48. Which of the followings can be used as DNA vectors?
A) Bacteriophages
B) Plasmids
C) E- coli cells
D) Both A and B
E) Both A and C
49. Immunoglobulins, designated ________ increase antimicrobial activity in mucus.
A) Ig A
B) Ig E
C) Ig G
D) Ig M
E) Ig Z
50. Antibody- mediated responses work best against _______________ .
A) Intracellular pathogens.
B) Extracellular pathogens
C) Toxins
D) B and C
E) All of the above.
51. Which of the followings is wrong?
A) The peroxysomes in plant cells are also called glyoxysomes
B) Peroxysomes and glyoxysomes contain enzymes that degrade fatty acids
C) Peroxysomes are present in the photosynthetic cell of green plants
D) Microbodies are cytochemically defined by their catalase activity
52. Which of the followings is true?
++
A) Dynein is a Ca ATPase and generates the energy for the bending of the ciliurn and flagellum.
B) The initial polymerization of tubulin often occurs at certain regions of the cell termed microtubule
organizing centers (MTOCs).
++
C) If cells are microinjected with Ca microtubule disassembly is blocked and assembly is
promoted.
D) Desmin is a kind of microfilament.
E) Desmin is not found in cardiac or smooth muscle.
53. Which of the following statements is wrong?
A) The nuclear lamina is composed of four different polypeptides termed lamins.
B) The lamins are intermediate filaments.
C) Nuclear envelope contains structural proteins but does not contain any enzymes.
D) Small molecules enter the nucleus more readily than do large ones.
E) All of them is true
54. In eucaryotes there are commonly three different RNA polymerases, which act at different
promoters to transcribe the various kinds of RNA in the cell. Which of the followings about the RNA
polymerase I is true in eucaryotes?

A) It occurs in the nucleoplasm and synthesizes mRNA


B) It occurs in the nucleolus and synthesizes mRNA
C) It occurs in the nucleolus and synthesizes tRNA
D) It occurs in the nucleoplasm and synthesizes mRNA
E) It occurs in the nucleolus and synthesizes rRNA
55. All chordates have _________ in some stage of their life.
A) A spinal chord
B) Gills
C) A notochord
D) Vertebrae
E) Hair
56. A cladogram is based on:
A) Homologous characteristics
B) Analogous characteristics
C) Polyphyletic taxa
D) Fossil evidence only
E) Derived characters only
57. The phenetic system is based primarily on:
A) Known evolutionary relationships
B) Fossil evidence
C) Shared characteristics
D) Ancestral characteristics
E) Both ancestral and shared characteristics
58. A typical R- selected organism might be a_________ : K- selected organism might be a
____________ .
A) Mouse, elephant
B) Elephant, mouse
C) Mouse, rabbit
D) Rabbit, fly
E) Rabbit, mouse
59. ______________ is the least frequent of the dispersal patterns seen in nature and may be
seen in situations in which the environment is particularly homogeneous.
A) Aggregated distribution
B) Clumped distribution
C) Uniform distribution
D) Density - dependent distribution
E) Random dispersion
60. When some animals mature they then migrate long distances to their breeding grounds or to
escape harsh weather. A signal for this type of behavior would be:
A) A conditioned stimulus
B) The length of day
C) Negative reinforcement
D) A sign stimulus
61. For an animal to respond to a pheromone it must:
A) Have a specific receptor
B) Be able to see it coming
C) Be able to smell it
D) Have a thin epidermis or hair so it can penetrate
E) Be of the opposite sex of the individual from which it was produced
62. In polymerase chain reaction technology the two strands of DNA are separated by:
A) Increasing temperature
B) Treating them with restriction enzymes

C) Centrifugation
D) Increasing pH
E) Both A and C are true
63. Which of the following areas of the world produce the most O2 per year?
A) Temperate forests
B) Grasslands
C) Lakes and streams
D) Oceans
64. The concentration of phytoplancton is significantly lower in open seas than in coastal waters. It
is commonly believed that this is caused directly by:
A) Insufficient abundance of mineral nutrients
B) Too low concentration of dissolved carbon dioxide
C) Insufficient mixing of water
D) Massive presence of phytoplanctophagous fish
E) Too high salinity
65. Lianas and epiphytes are especially common in:
A) Tropical rain forests
B) Savannah
C) Temperate grassland
D) Taiga
66. Lichens are most typically formed by the symbiotic association of an alga or cyanobacterium
and a(n) :
A) Ascomycota
B) Deuteromycota
C) Basidiomycota
D) Zygomycota
E) Spirochete
67. The logistic equation:
A) Models an S-shaped growth curve
B) Takes into account the carrying capacity
C) Shows continued exponential growth
D) Both A and B
E) Both B and C
68. At which of the given times should the C02 concentration in the forest be at the highest level
each day during the summer?
A) Just before sunrise
B) Noon
C) Just before sunset
D) Midnight
69. Clumped dispersion is typically influenced by :
A) Social behavior such as family groups
B) Asexual reproduction in plants or invertebrates
C) Limited seed dispersal in plants
D) A patchy distribution of resources
E) All of these
70. The area of greatest seasonal ozone depletion is in the:
A) Stratosphere above the Arctic
B) Stratosphere above the Antarctic
C) Troposphere above the Arctic
D) Troposphere above the Antarctic
E) Lithosphere above the Arctic

71. Which of the followings about the golgi complex is false?


A) The golgi complex under the electron microscope is composed of flattened sacs termed
cisternae
B) Sulfation of proteins doesn't occur in the golgi
C) Glycosylation of proteins occur in the golgi
D) The trans region of golgi faces the plasma membrane and has larger vesicles than cis region of
golgi
72. Studying cells using a phase contrast microscope gives the advantage because:
A) The resolving power is greater
B) The magnification is greater
C) It uses a beam of electrons which can penetrate the tissue of living organisms, thus allowing
one to see the organelles enclosed by the plasma membrane
D) Organelles of cells can be seen without staining which usually results in the death of cells
73. The scanning electron microscope differs from the transmission electron microscope because
the scanning electron microscope:
A) Can view a living specimen
B) Relies on the emission of secondary electrons emitted from a gold-coated object
C) Can view the internal structure of a cell
D) Utilizes a beam of light that can pass through the specimen
E) Gives a two dimensional image of the object being studied
74. Typically, which grows the most slowly?
A) Bread moulds
B) Lichens
C) Yeast
D) Mushrooms
E) Sac fungi (Ascomycota)
75. Which of the followings is characterized by cold temperatures little rain and few or no trees?
A) Taiga
B) Savannah
C) Tundra
D) Desert
76. Which of the following biomes has the greatest species diversity?
A) Tropical Forest
B) Savannah
C) Grassland
D) Temperate deciduous forest
77. The tropical rain forest is characterized by:
A) High density of plants, but nutrient poor soil
B) High density of plants, and nutrient rich soil
C) Low density of plants due to nutrient poor soil
D) Low density of plants, but nutrient rich soil
E) None of the above responses are correct
78. Which the following pairs is mismatched?
A) Behavior; response to environment signals
B) Capacity for behaviour; is learned
C) Behaviour; can be modified
D) Persistent changes in behaviour; learning
E) Behaviour; adaptation for survival
79. In a place where number of individuals establish adjacent territories if one individual gets
further from the center of its territory and approaches the edge of the next, it:
A) Encounters a big fight on entering the next territory
B) Loses control of its mates

C) Passes a neutral line between territories where neither is dominant


D) Becomes more aggressive as it gets closer to the adjacent territory
80. Which of the followings about the plasma membranes is not true?
A) In most plasma membranes the two membrane leaflets (cytoplasmic and exoplasmic leafted)
have same lipid compositions
B) Membrane proteins bind asymmetrically to the two layers of lipid
C) Most membrane proteins and lipids are laterally mobile in the membrane
D) Cell coat(glycocalyx) is found at the exoplasmic face
81. Which of the followings about the cell junctions is false?
A) Many plant cells are interconnected by plasmodesmata which allow metabolites with low
molecular weight and signaling molecules to move between cells
++
B) Ca ion concentration affects the closure of gap junctions
C) Tight junctions consist of connexin molecules; six formed in a hexagon cylinder in one plasma
membrane joined to six arranged in the same array in the adjacent cell membrane
D) Spot desmosomes are found in all epithelial cells
82. Which one of the followings is false for mitochondria?
A) The size and coding capacity of mitochondria DNA varies in different organisms
B) Mitochondria contain their own genetic material
C) Mitochondria ribosomes are different from the cytoplasmic ribosomes
D) Most mitochondrial proteins are synthesized in the mitochondria
E) Plant mitochondria DNAs are larger than the other organisms mitochondria DNA
83. Which of the following plant hormones delay senescence or aging?
A) Auxin
B) Cytokinin
C) Ethylene
D) Gibberellin
E) Abscisic acid
84. Which of the following phylum's members cause sleeping sickness?
A) Rhizopoda
B) Zoomastigina
C) Ciliophora
D) Foraminifera
E) Actinipoda
85. The beginning of cephalization is first seen in the:
A) Scyphozoans
B) Poriferans
C) Platyhelminthes
D) Nematodes
E) Nemerteans
86. In the body, although nerve impulses can travel in both directions along an axon impulses are
usually unidirectional. This is because:
A) Dendrites have a higher threshold than axons
B) The axon terminals have a higher threshold than axons
C) The myelin sheath directs the flow of ions toward the axon terminals
D) The axon terminals contain neurotransmitter receptors and the dendrites do not
E) Dendrites contain no neurotransmitter substance
87. Which of the followings about the archaebacteria is false?
A) They all lack peptidoglycan in their cell walls
B) Their lipids contain an ether linkage between the glycerol and the fatty acid residues
C) Translation is initiated by N-formyl methionine t RNA
D) They live in swamps

88. In chemiosmosis hydrogen ions (protons) release their energy to produce ATP as they pass
through _______________________ .
A) The outer mitochondrial membrane
B) ATP synthase
C) ATP dehydrogenase
D) ATP decarboxylase
E) A group of electron carriers
89. Tetanus toxin inhibits the breakdown of acetyhcholine at the myoneural junction. This effects
muscle contraction by _____________ .
A) Not allowing the muscle to contract
B) Preventing the muscle from relaxing
C) Preventing calcium from entering the muscle
D) Inhibiting potassium from leaving the muscle
E) Stopping the formation of actomyosin
90. Protein leaves the interstitial fluid and enters the circulatory system:
A) Through pores on the arterial side of a capillary bed
B) Through pores on the venous side of a capillary bed
C) By being phagocytized and transported to the blood through eosinophils
D) Via the lymphatics which deposits them into the subclavian vein
E) By active transport mediated by permeases in the capillary epithelium
91. In the citric acid cycle, two acetyl CoA molecules are metabolized to:
A) 2 CO2 + 2 ATP + 2 NADH + 2 FADH
B) 4 C02 + 6 NADH + 2 FADH2 + 2 ATP
C) Fructose 1,6 biphosphate
D) Glucose + 2 C02 + 2 NADH + 2 FADH2 + 2 ATP
E) 2 PGAL
92. Which of the followings is true?
A) Lactose operon in E. coli is only controlled by negative control by the lactose repressor
B) When E.coli is grown in the presence of both glucose and lactose, both glucose and lactose is
utilized
C) Catabalite gene activator protein (CAP) has no influence on transcription until ATP has bound to
it
D) Different proteins that are coded by a same mRNA molecule are always produced in similar
numbers
E) DNA repair genes at many different sites on the chromosome are regulated by only a single
repressor in prokaryots
93. A mRNA "cap'' :
A) Marks the mRNA for degradation
B) Serves as the attachment point for a poly-A tail
C) Prevents translation
D) Decreases the half - life of the mRNA
E) None of the answers is correct
94. Which of the followings can be found on a completed bacterial mRNA?
A) Intron region
B) A leader sequence
C) A primer sequence
D) Poly A tail
E) None of them
95. In an E-coli bacterium, there are three types of DNA polymerase which are called DNA
polymerase I, DNA polymerase III and DNA polymerase II. Which of the followings about the
functions of these enzymes is wrong?
A) All of them have 5' 3' polymerization activity
B) Only DNA polymerase II has not the 5' 3' exonuclease activity

C) Only DNA polymerase I and III have the 3' 5' exonuclease activity
D) DNA polymerase I has the capacity to remove the RNA primers
E) DNA polymerase III makes up the Okazaki fragments
96. Which of the followings is not correct for CAM plants?
A) Crassulacean Acid Metabolism is found in fewer than 5 percent of plants
B) Although the cell could not metabolize normally if the acid accumulated in the cytoplasm,
throughout the night more and more malic acid is produced and stored in the cell's vacuole,
building a reservoir of fixed CO2
C) Malic acid is broken down into C02 and pyruvate during day-time. It is then that CO2 becomes
fixed to RBP
D) CAM synthesis helps to conserve precious water by separating the time of CO 2 fixation; the light
reactions, and the Calvin cycle
97. Which of the following amino acids enter the Crebs cycle via being converted to ketoglutaric
acids?
A) 2- carbon amino acids
B) 3- carbon amino acids
C) 4- carbon amino acids
D) 5- carbon amino acids
98. Intolerance to cold and myxodema are typically seen in ____________ .
A) Hyperthyroidism
B) Hypothyroidism
C) Acromegaly
D) Cretinism
99. What's the effect of insulin in adipose tissue?
A) It stimulates glucose uptake
B) It stimulates lipolysis
C) It stimulates hormone-sensitive lipase and lipoprotein lipase
D) It stimulates urea synthesis
100. The largest white blood cell in humans which is a part of the body's macrophage defense
system is:
A) Neutrophil
B) Lymphocyte
C) Monocyte
D) Eusinophil
E) Basophil

INTERNATIONAL BIOLOGY OLYMPIAD


PRACTICAL PROBLEMS

1997, Ashgabad, Turkmenistan















All IBO examination questions are published under the following Creative Commons license:



CC BY-NC-SA (Attribution-NonCommercial-ShareAlike) https://creativecommons.org/licenses/by-nc-sa/4.0/
The exam papers can be used freely for educational purposes as long as IBO is credited and
new creations are licensed under identical terms. No commercial use is allowed.

8th INTERNATIONAL BIOLOGY OLYMPIAD


PRACTICAL TEST-A
BOTANY AND BOTANIC ECOLOGY
1. MATERIAL NUMBER: 1
The function of this organism is
A) to fix free nitrogen
B) to convert proteins to NH3
C) to make alcoholic fermentation
D) to make lactic acid fermentation
E) to produce amino acid from NH3
2. MATERIAL NUMBER: 1
This organism is a (n).
A) Eucaryot
B) Cyanobacteria
C) Yellow - green algae
D) Diatom
E) Green algae
3. MATERIAL NUMBER: 2
This organism is a(n)
A) Procaryot
B) Metazoan
C) Diatom
D) Cyanobacteria
E) Green algae
4. MATERIAL NUMBER: 2
This organism ..
A) produces NO2 from NH3
B) produces NO3 from NO2
C) produces free nitrogen
D) forms the silica layers of the earth
E) is a zooplankton
5. MATERIAL NUMBER: 2
The species of this organism is
A) Anabaena sp.
B) Escherichia sp.
C) Navicula sp.
D) Chlorella sp.
E) Sargassum sp.
6. MATERIAL NUMBER: 3
It makes..
A) Chemosynthesis
B) Photosynthesis
C) Nitrification
D) Denitrification
E) Fermentation
7. MATERIAL NUMBER: 4
Type of reproduction is..
A) Autogamy

B) Isogamous
C) Anisogamous
D) Conjugation
E) Vegetative
8. MATERIAL NUMBER: 4
The shape of chloroplast is
A) Star
B) Oval
C) Spiral
D) Rod
E) Circle
9. MATERIAL NUMBER: 5
The shape of bacteria is
A) Spirillum
B) Bacillus
C) Coccus
D) Vibrio
E) Staphylococcus
10. MATERIAL NUMBER: 5
The group of bacteria is
A) Gram (+)
B) Gram (-)
C) Gram (+) and Gram (-)
D) One flagellated
E) Poly flagellated
11. MATERIAL NUMBER: 6
The shape of bacteria is
A) Spirillum
B) Bacillus
C) Coccus
D) Vibrio
E) Staphylococcus
12. MATERIAL NUMBER: 6
The group of bacteria is
A) Gram (+)
B) Gram (-)
C) Gram (+) and Gram (-)
D) One flagellated
E) Many flagellated
13. MATERIAL NUMBER: 7
The number of flagella on the bacteria is
A) 0
B) 1
C) 2
D) 4
E) Many
14. MATERIAL NUMBER: 8
The shape of bacteria is

A) Staphylococcus
B) Diplococcus
C) Streptococcus
D) Micrococcus
E) Spirillum
15. MATERIAL NUMBER: 9
The shape of bacteria is
A) Staphylococcus
B) Diplococcus
C) Streptococcus
D) Micrococcus
E) Spirillum
16. MATERIAL NUMBER: 10
Which of the followings includes this organism?
A) Zygomycota
B) Ascomycete
C) Basidiomycete
D) Phaeophceae
E) Rhodophyta
17. MATERIAL NUMBER: 10
The structure includes
A) Only sporangium
B) Only zygote
C) Sporangium and zygote
D) Only egg.
E) Egg and sperms
18. MATERIAL NUMBER: 11
Which of the followings includes this organism?
A) Zygomycota
B) Ascomycete
C) Basidiomycete
D) Phaeophceae
E) Rhodaphyta
19. MATERIAL NUMBER: 11
The organism includes..................in its structure.
A) Zygote
B) Lamellate structure
C) Basidium
D) Gametophyte
E) Ascus
20. MATERIAL NUMBER: 12
The dark structures on the section of this fungus are..
A) Spores
B) Spore vesicles
C) Ascus
D) Zygote
E) Hypha

21. MATERIAL NUMBER: 12


Which of the followings does include this fungus?
A) Zygomycota
B) Ascomycete
C) Basidiomycete
D) Rhodophyta
E) Phaeophyceae
22. MATERIAL NUMBER: 13
This structure of the fern is..
A) Only antheridium
B) Only archegonium
C) Antheridium and archegonium
D) Young sporophyte
E) Strobilus
23. MATERIAL NUMBER: 14
This structure which belongs to lycopsida is...........
A)Only antheridia
B)Only archegonia
C)Antheridia and Archegonia
D)Rhizome
E)Strobila
24. MATERIAL NUMBER: 15
This section of leaf belongs to
A) Fungi
B) A lycopside
C) Horsetail
D) Pine
E) Grass
25. MATERIAL NUMBER: 15
This plant is
A) Hydrophyte
B) Hygrophyte
C) Xerophyte
D) Tropophyte
E) Helophyte
26. MATERIAL NUMBER: 15
The big spaces at the edges are ..
A) Latex
B) Resin channels
C) Secreting pockets
D) Xylem tubes
E) Phloem tubes
27. MATERIAL NUMBER: 15
Stomata occur atrelative to the leaf surface.
A) The same level
B) The upper level
C) The lower level
D) Extremely upper
E)There are no stomata

28. MATERIAL NUMBER: 15


The shape of parenchyma cell is
A) Circle
B) Star
C) Rectangle
D) Isodiametric
E) Hexagonal
29. MATERIAL NUMBER: 16
The pollen belongs toplant.
A) Magnolia
B) Ranunculus
C) Zea
D) Pinus
E) Solanum
30. MATERIAL NUMBER: 16
The distribution of pollen is done by
A) Water
B) Insects
C) Wind
D) Birds
E) Mammalia
31. MATERIAL NUMBER: 17
This section of root belongs to.
A) Cocos
B) Monstrea
C) Orchis
D) Ranunculus
E) Sphagnum
32. MATERIAL NUMBER: 18
This section of root belongs to..
A) Astaraceae
B) Fabaceae
C) Araceae
D) Lamiaceae
E) Cactaceae
33. MATERIAL NUMBER: 19
This section of stem belongs to..
A) Poaceae
B) Solanaceae
C) Oleaceae
D) Phaeophyceae
E) Phagaceae
34. MATERIAL NUMBER: 20
This section of stem belongs to..
A) Fabaceae
B) Pinophyta
C) Lycopodiophyta

D) Liliaceae
E) Muscopsida
35. MATERIAL NUMBER: 21
This section consists of..
A) Only cork cambium
B) Only lenticel
C) Only cork tissue
D) Only cork cambium and lenticel
E) Cork cambium, lenticel, cork
36. MATERIAL NUMBER: 22
This section of leaf belongs to a plant.
A) Polyticum
B) Pinus
C) Marchantia
D) Triticum
E) Quercus
37. MATERIAL NUMBER: 23
This leaf belongs to a.. plant.
A) Hydrophytic
B) Succulent
C) Mesophytic
D) Xerophytic
E) Carnivore
38. MATERIAL NUMBER: 24
This leaf belongs to a.. plant.
A) Hydrophytic
B) Succulent
C) Mesophytic
D) Xerophytic
E) Carnivore
39. MATERIAL NUMBER: 25
This leaf belongs to a.. plant.
A) Hydrophytic
B) Succulent
C) Mesophytic
D) Xerophytic
E) Carnivore
40. MATERIAL NUMBER: 26
This flower belongs to a .. plant.
A) Triticum
B) Lamium
C) Syringa
D) Opuntia
E) Ginkgo
41. MATERIAL NUMBER: 27
This structure which belongs to a lilium is...
A) Ovulary
B) Tetrad

C) Anther
D) Embryo
E) Seed
42. MATERIAL NUMBER: 28
This tissue which is seen at the outermost part of the stem is. ..
A) Epidermis
B) Cork
C) Lenticel
D) Stoma
E) Trichome
43. MATERIAL NUMBER: 29
The supportive tissue in this structure is
A) Sclereid
B) Schlerenchyma
C) Collenchyma
D) Tracheid
E) Trachea
44. MATERIAL NUMBER: 30
The crystal shape in this tissue is a.
A) Raphides
B) Druse
C) Prism
D) Cubic
E) Sand
45. MATERIAL NUMBER: 31
The pink lines between the lumens are.
A) Middle lamella
B) Secondary cell wall
C) Primary cell wall
D) Plasmodesmata
E) Edged passage
46. MATERIAL NUMBER: 32
Which of the followings isnt found in this root structure?
A) Epidermis
B) Cortex
C) Starch
D) Vascular bundle
E) Cork cambium

8th INTERNATIONAL BIOLOGY OLYMPIAD


PRACTICAL TEST-B
ZOOLOGY AND ZOOLOGIC ECOLOGY
1. MATERIAL NUMBER: 1
This organism belongs to
A) Amoebae
B) Radiolarian
C) Foraminifers
D) Paramecium
E) Euspongia
2. MATERIAL NUMBER: 1
This organism moves using
A) Cilia
B) Flagellum
C) Pseudopod
D) Myosin filaments
E) Flow of water
3. MATERIAL NUMBER: 2
This organism belongs to
A) Rhodophyta
B) Phaeophyta
C) Euglenophyta
D) Chlorophyta
E) Poriferan
4. MATERIAL NUMBER: 2
This organism has ..flagella/ flagellum.
A) 1
B) 2
C) 4
D) No
E) Many
5. MATERIAL NUMBER: 3
This organism is
A) Euglena
B) Trypanosome
C) Chlamydomonas
D) Chlorella
E) Vorticella
6. MATERIAL NUMBER: 3
The habitat of this organism is
A) Fresh Water
B) Salty Water
C) Intestine
D) Mouth of Mammals
E) Humans blood
7. MATERIAL NUMBER: 3
This organism moves using

A) Cilia
B) Flagellum
C) Pseudopod
D) Myosin filament
E) Flow of water
8. MATERIAL NUMBER: 4
This organism is
A) Euglena
B) Paramecium
C) Chlorella
D) Plasmodium
E) Vorticella
9. MATERIAL NUMBER: 4
This organism moves using
A) Cilia
B) Flagellum
C) Pseudopod
D) Myosin filament
E) Flow of water
10. MATERIAL NUMBER: 5
The reproductive pattern of this single celled organism is
A) Conjugation
B) Fission
C) Isogamy
D) Anisogamy
E) Autogamy
11. MATERIAL NUMBER: 6
This organism is
A) Euglena
B) Trypanosome
C) Chlamydomonas
D) Chlorella
E) Vorticella
12. MATERIAL NUMBER: 7
This organism belongs to
A) Hydrozoan
B) Scyphozoan
C) Anthozoan
D) Euspongia
E) Oligochaete
13. MATERIAL NUMBER: 7
This reproduction pattern is..
A) Conjugation
B) Isogamy
C) Budding
D) Oogamy
E) Fission

14. MATERIAL NUMBER: 7


The habitat of this organism is
A) Salty Water
B) Human Intestine
C) Human Blood
D) Fresh Water
E) On the mosses
15. MATERIAL NUMBER: 8
This organism belongs to..
A) Hydrozoan
B) Scyphozoan
C) Anthozoan
D) Euspongia
E) Oligochaete
16. MATERIAL NUMBER: 9
This organisms belongs to..
A) Polychaete
B) Oligochaete
C) Hirudinea
D) Crustacean
E) Chelicerata
17. MATERIAL NUMBER: 9
This organism has .. antennae.
A) No
B) 2
C) 2 pairs
D) Brush like
E) 4 pairs
18. MATERIAL NUMBER: 10
This organism belongs to..
A) Chelicerata
B) Thysanuran
C) Isopteran
D) Crustacean
E) Hymenoptera
19. MATERIAL NUMBER: 10
This organism has . antennae.
A) No
B) 2
C) 2 pairs
D) Brush like
E) 4 pairs
20. MATERIAL NUMBER: 10
This organism has .. legs.
A) 4
B) 4 pairs
C) 3
D) 3 pairs
E) No

21. MATERIAL NUMBER: 10


The mouth structure is
A) Chewer
B) Grazer
C) Cheliser
D) Holder
E) Piercer and absorbent
22. MATERIAL NUMBER: 11
.. has this kind of eye structure.
A) Leptinotarsa
B) Hirudo
C) Fasciola
D) Sepia
E) Asteria
23. MATERIAL NUMBER: 12
This metamorphosis occurs.
A) Under Soil
B) On the branches
C) On the mammals skin
D) In fresh water
E) On the death animal body
24. MATERIAL NUMBER: 12
The type of this metamorphosis is.
A) Ametabol
B) Hemimetabol
C) Holometabol
D) Hypermetabol
E) Neoteny
25. MATERIAL NUMBER: 12
The adult of this organism uses.. as food
A) Larva
B) Vegetable
C) Fruit
D) Mammals blood
E) Soil
26. MATERIAL NUMBER: 12
This organism has . antennae.
A) Many
B) No
C) 2
D) 2 pairs
E) 6
27. MATERIAL NUMBER: 13
The type of egg that show this kind of segmentation is.
A) Isolecithal
B) Mesolecithal
C) Telolecithal

D) Centrolecithal
E) Polylecithal
28. MATERIAL NUMBER: 13
The type of segmentation is
A) Superficial
B) Mesoblastic
C) Meroblastic
D) Holoblastic
E) Discoidal
29. MATERIAL NUMBER: 13
. show this kind of division.
A) Frogs
B) Birds
C) Turtles
D) Starfish
E) Salamander
30. MATERIAL NUMBER: 14
This blastula stage is formed by..............type egg.
A) Isolecithal
B) Mesolecithal
C) Telolecithal
D) Centrolecithal
E) Polylecithal
31. MATERIAL NUMBER: 14
The type of segmentation.
A) Superficial
B) Mesoblastic
C) Meroblastic
D) Holoblastic
E) Discoidal
32. MATERIAL NUMBER: 14
This blastula stage is formed by.
A) Frogs
B) Birds
C) Turtles
D) Starfish
E) Salamander
33. MATERIAL NUMBER: 15
This structure is the. tissue of a mammal.
A) Cartilage
B) Bone
C) Pancreas
D) Thyroid
E) Secretory
34. MATERIAL NUMBER: 15
The big gap in the middle is..
A) Secretory Channels
B) Langerhans Islands

C) Secretory Lumen
D) Haversian Channels
E) Chondrocytes sets
35. MATERIAL NUMBER: 16
This blood tissue belongs to..
A) Locusta
B) Myotis
C) Cebus
D) Lumbricus
E) Anuran
36. MATERIAL NUMBER: 17
Circular structure observed in the cross section of skin is..
A) Adipose tissue
B) Vater paccini
C) Crause
D) Ruffini
E) Meissner
37. MATERIAL NUMBER: 17
The duty of this structure is..
A) Storing lipid
B) Sense the pressure
C) Sensing the high temperature
D) Sensing the low temperature
E) Secreting sweat
38. MATERIAL NUMBER: 18
This vessel that cats have is
A) Artery
B) Vena
C) Arteriole
D) Lymphatic
E) Arteriosclerosis Vena
39. MATERIAL NUMBER: 19
The type of epithelium is..
A) Transitional
B) Pseudo Stratified
C) Stratified
D) Cubic
E) Goblet
40. MATERIAL NUMBER: 20
This person, whose bar bodies has been stained, has..
A) XX
B) XY
C) XXX
D) XXY
E) XXXY
41. MATERIAL NUMBER: 21
These jaw bones belongs to
A) Mammalia

B) Piciformes
C) Testudinate
D) Caudata
E) Teleostei
42. MATERIAL NUMBER: 22
This skull belongs to.
A) Anura
B) Myotis
C) Lacerta
D) Clupea
E) Petromyzon
43. MATERIAL NUMBER: 23
This tooth belongs to
A) Strix
B) Ciconia
C) Sycliorhinus
D) Ascidia
E) Dryocopus
44. MATERIAL NUMBER: 24
This human bone is.
A) Atlas
B) Lumbar
C) Thoracic
D) Cervical
E) Axis
45. MATERIAL NUMBER: 25
This human bone is.
A) Atlas
B) Lumbar
C) Thoracic
D) Cervical
E) Axis

8th INTERNATIONAL BIOLOGY OLYMPIAD


PRATICAL TEST-C
TAXONOMY
1- MATERIAL NUMBER: 1
This organism belongs to ...............................
A. Porifera
B. Cnidaria
C. Turbellaria
D. Trematoda
E. Cestoda
2- MATERIAL NUMBER: 1
Stained part is ............ system of that organism.
A. Respiratory
B. Excretory
C. Digestive
D. Nervous
E. Reprodutive
3- MATERIAL NUMBER: 2
This organism belongs to .................................
A. Porifera
B. Cnidaria
C. Turbellaria
D. Trematoda
E. Cestoda
4- MATERIAL NUMBER: 2
....................... is not observed in this slide
A. Intestine
B. Uterus
C. Testis
D. Ovary
E. Protonephridium
5- MATERIAL NUMBER: 3
This organism belongs to ............................
A. Platyhelminthes
B. Nemathelminthes
C. Annelida
D. Oligochaeta
E. Polychaeta
6- MATERIAL NUMBER: 3
The sex of the individuals in the slide is ..........................
A. Only male
B. Only female
C. Male and female
D. Hermaphrodite
7- MATERIAL NUMBER: 4
This organism belongs to ............................
A. Stellaroidea
B. Echinoidea
C. Gastropoda
D. Scyphozoa

E. Ascidia
8- MATERIAL NUMBER: 5
This organism belongs to ............................
A. Turbellaria
B. Trematoda
C. Cestoda
D. Hirudinea
E. Polychaeta
9- MATERIAL NUMBER: 6
This organism belongs to .................................
A. Turbellaria
B. Trematoda
C. Cestoda
D. Hirudinea
E. Polychaeta
10- MATERIAL NUMBER: 6
The name of the species in this slide is ............
A. Nereis
B. Hirudo
C. Asterias
D. Polycellis
E. Scolopendra
11- MATERIAL NUMBER: 7
This organism belongs to .................................
A. Crustacea
B. Chelicerata
C. Tracheata
D. Chilopoda
E. Homoptera
12- MATERIAL NUMBER: 7
This organism has ............................. antennae
A. No
B. One pair
C. Two pairs
D. Four pairs
E. Six
13- MATERIAL NUMBER: 7
The name of the species in this slide is .................................
A. Astacus
B. Daphnia
C. Cyclops
D. Areneus
E. Aphis
14- MATERIAL NUMBER: 8
This organism belongs to ............................
A. Crustacea
B. Chelicerata
C. Tracheata

D. Chilopoda
E. Hymenoptera
15- MATERIAL NUMBER: 8
This organism has ....................... antennae
A. No
B. One pair
C. Two pairs
D. Four pairs
E. Six
16- MATERIAL NUMBER: 8
The name of the species in this slide is .................................
A. Ixodes
B. Scolopendra
C. Pedinculus
D. Leptinotarsa
E. Gerris
17- MATERIAL NUMBER: 8
The the number of legs in this organism is ...........................
A. Two pairs
B. Three pairs
C. Four pairs
D. Six pairs
E. Eight pairs
18- MATERIAL NUMBER: 9
This organism belongs to ................................
A. Diptera
B. Hymenoptera
C. Anoplura
D. Isoptera
E. Odonata
19- MATERIAL NUMBER: 9
Mouth parts are ........................
A. Sucker
B. Chewer
C. Piercing
D. Chelicer
E. Lapping for pollen grains
20- MATERIAL NUMBER: 9
The name of the species in this slide is ...........
A. Ichneumon
B. Leptinotarsa
C. Aphis
D. Anopheles
E. Pedinculus
21- MATERIAL NUMBER: 10
This organism belongs to ...........................
A. Diptera
B. Hymenoptera

C. Anoplura
D. Isoptera
E. Odonata
22- MATERIAL NUMBER: 10
Mouth parts are
A. Piercing
B. Chewer
C. Licking
D. Chelicer
E. Lapping for pollen grains
23- MATERIAL NUMBER: 10
This the number of legs in this organism is .....................
A. Two pairs
B. Three pairs
C. Four pairs
D. Six pairs
E. Eight pairs
24- MATERIAL NUMBER: 10
The name of the species in this slide is ................
A. Drosophila
B. Araneus
C. Formica
D. Gerris
E. Ixodes
25- MATERIAL NUMBER: 10
This is organism has ...................... antennae
A. No
B. One pair
C. Two pair
D. Four pair
E. Six
26- MATERIAL NUMBER: 11
This organism belongs to ..........................
A. Diptera
B. Hymenoptera
C. Anoplura
D. Isoptera
E. Odonata
27- MATERIAL NUMBER: 11
This number of legs in this organism is ............
A. Two pairs
B. Three pairs
C. Four pairs
D. Six pairs
E. Eight pairs
28- MATERIAL NUMBER: 11
The name of the species in this slide is .............
A. Drosophila

B.
C.
D.
E.

Araneus
Pediculus
Gerris
Ixodes

29- MATERIAL NUMBER: 11


This organism has ..................... antennae
A. No
B. One pair
C. Two pair
D. Four pair
E. Six
30- MATERIAL NUMBER: 12
This fruit belongs to ................................
A. Asteraceae
B. Solanaceae
C. Lamiaceae
D. Fabaceae
E. Oleaceae
31- MATERIAL NUMBER: 13
This plant belongs to .............................
A. Solananceae
B. Magloniaceae
C. Rosaceae
D. Ranunculaceae
E. Brassicaceae
32- MATERIAL NUMBER: 13
Ovary position is ....................................
A. Epigin
B. Hypogin
C. Perigin
D. Perigineus
E. Hypogenius
33- MATERIAL NUMBER: 14
This plant belongs to ......................................
A. Araceae
B. Arecaceae
C. Poaceae
D. Orchidaceae
E. Liliaceae
34- MATERIAL NUMBER: 15
Ovary position is ........................................
A. Epigin
B. Hypogin
C. Perigin
D. Perigineus
E. Hypogineus
35- MATERIAL NUMBER: 15
This plant belongs to ......................... (familia)

A.
B.
C.
D.
E.

Solanaceae
Magnoliaceae
Rosaceae
Rununculaceae
Brassicaceae

36- MATERIAL NUMBER: 15


This plant belongs to ...............................
A. Prunus
B. Rosa
C. Pulsatilia
D. Syringa
E. Malus
37- MATERIAL NUMBER: 16
This structure belongs to .............................
A. Rosaceae
B. Fagaceae
C. Brasicaceae
D. Solanaceae
E. Asteraceae
38- MATERIAL NUMBER: 16
This body part of a plant is .........................
A. Stem
B. Bulb root
C. Storage root
D. Leave
E. Bud
39- MATERIAL NUMBER: 17
Leaves of the plant are .................................
A. Tripinnat
B. Paripinnat
C. Trifoliat
D. Imparipinnat
E. Bipinnat
40- MATERIAL NUMBER: 18
This structure belongs to ............................
A. Araceae
B. Arecaceae
C. Poaceae
D. Orchiclaceae
E. Liliaceae
41- MATERIAL NUMBER: 19
This plant belongs to ...................................
A. Asteraceae
B. Arecaceae
C. Aracaea
D. Rosaceae
E. Fabaceae

8th INTERNATIONAL BIOLOGY OLYMPIAD


PRACTICAL PART-D
ECOLOGIC PROBLEMS
1. The amount of food that juneperus bird needs everyday is 10% of its weight.
According to this information how much food does a Juneperus bird with 90 gram
body weight needs everyday.
A) 90 gr. B) 45 gr. C) 9 gr. D) 10 gr. E) 18 gr.

Answer the question 2,3,4,5,6 according to the given information below. In the sitophilus
oryzae population density dependent egg laying number is given below.
Number of mite=128
Number of wheat seeds=200
The number of eggs that a female lay in each day=1,6
The number of eggs found in each seed=2,56
Percentage of adults getting out from eggs=~70
Percentage of seeds that doesnt have any egg=5,0
Female, male ratio=1
2. Number of wheat mite for each seed?
A) 32.8 B) 1.56 C) 3.2 D) 0.64 E) 6.4
3. The total number of eggs that females lay everyday?
A) 204.8
B) 512
C) 327.6
D) 140
E) 102.4
4. The total number of eggs in the seeds is
A) 512
B) 327.6
C) 486.4
D) 163.8
E) 304
5. The number of seeds with egg is
A) 10
B) 6.4
C) 64
D) 100
E) 190
6. The total number of eggs that give rise to adults..
A) 358
B) 229
C) 340
D) 114
E) 212
Answer the question 7,8,9,10,11,12 according to the information given below. The
numbers of female and male individuals in the Panaeus incidus population (for each
season) is given below

..........................Number of.................Number of
............................MALES....................FEMALES
Winter......................130..............................142
Spring.......................60................................67
Summer....................482.............................458
Fall...........................243.............................246
7. Whats the maximum population size in year
A) 915
B) 913
C) 1828
D) 940
E) 518
8. The number of maximum female individuals in a year.
A) 915
B) 913
C) 458
D) 482
E) 1828
9. The season, in which the percentage of males is the highest, is
A) Winter
B) Spring
C) Summer
D) Fall
E) All seasons are same
10. Whats the maximum percentage of male individuals in a year?
A) 54
B) 47.7
C) 51
D) 49
E) 50.05
11. The season in which the mortality number is the highest.
A) Winter
B) Fall
C) Spring
D) Summer
12. The season in which the mortality percent is the highest..
A) Winter
B) Fall
C) Spring
D) Summer
Answer the questions 13,14,15,16,17 according to information given below.
Following figure compares the terrestrial and aquatic areas.
...........................................TERRESTRIAL AREA...................AQUEOUS AREA
Total Area....................................149x106 km2.................................361x106 km2
Total Net Primary Production......120x109 ton/year ............................60x109 ton/year
Total Secondary Production..........0.8x109 ton/year............................3.0x109 ton/year
Total Food
(Both animal and plant made)........1.3x109 ton/year...........................17.0x109 ton/year

13. Total net primary product per km2 on terrestrial area for a year is
A) 85
B) 8.50 x 102
C) 805.369
D) 8.5 x 103
E) 8505 x 102
14. Total secondary product per km2 on terrestrial area for a year is..
A) 53x102
B) 5.369
C) 5.3x103
D) 0.053
E) 0.53
15. Total food per km2 on terrestrial area for a month is..
A) 7.27
B) 0.72x103
C) 2.7x102
D) 0.727
E) 0.00727
16. Total net primary product per km2 on aqueous area for a month
is.
A) 1.3x103
B) 13.850
C) 138
D) 1.3x102
E) 0.0138
17. Total secondary product per km2 on aqueous area for a month
is
A) 6.92
B) 0.0069
C) 609.2
D) 60.29
E) 0.692
18. Total food per km2 on aqueous area for a year is..
A) 47.091
B) 4.7
C) 47.7x103
D) 47.7x102
E) 4.7

INTERNATIONAL BIOLOGY OLYMPIAD


THEORY PROBLEMS

1996, Artek, Ukraine















All IBO examination questions are published under the following Creative Commons license:



CC BY-NC-SA (Attribution-NonCommercial-ShareAlike) https://creativecommons.org/licenses/by-nc-sa/4.0/
The exam papers can be used freely for educational purposes as long as IBO is credited and
new creations are licensed under identical terms. No commercial use is allowed.

IBO 1996 Ukraine Theory questions


1. If you keep a C3 plant and a C4 plant of comparable size in an air-light
glass container and provide them with adequate sun light, you would
expect the C4 plant to.
a)
b)
c)
d)

grow better than the C3 plant after one year in that condition;
grow slower than the C3 plant after one year in that condition;
be able to survive longer than the C3 plant, but ultimately die; +
be able to survive not as long as the C3 plant.

2. When compared to sun plants, shade plants usually:


a) have leaves greener than those of sun plants because they need more
chlorophyll;
b) have thicker cuticles than sun plants because they need to preserve water;
c) have thicker leaves because, this way, they can capture all the incoming
light energy;
d) have thinner leaves because they are already sufficient to capture low light
energy. +
3. The process of photorespiration does not.
a)
b)
c)
d)

occurs during the daytime;


produce phosphoglycolate;
consume oxygen;
generate ATP. +

4. Which of the following genotype frequencies of AA, Aa and aa,


respectively, satisfy the Hardy-Weinberg principle?
a)
b)
c)
d)

0.25, 0.50, 0.25; +


0.36, 0.55, 0.09;
0.64, 0.27, 0.09;
0.29, 0.42, 0.29.

5. When the base composition of DNA from bacterium Mycobacterium


tuberculosis was determined, 18 percent of the bases were found to be
adenine. What is the [G] + [C] content?
a)
b)
c)
d)

18%;
32%;
36%;
64%. +

6. Six tubes containing preparations from animal tissue were set up as


shown in the table. After incubation, in which three tubes would carbon
dioxide be produced?
N
1
2
3
4
5
6
a)
b)
c)
d)

content
glucose + homogenised cells;
glucose + mitochondria;
glucose + cytoplasm lacking organelles;
pyruvic acid + homogenised cells;
pyruvic acid + mitochondria;
pyruvic acid + cytoplasm lacking
organelles.

1,2 and 3;
1,4 and 5; +
3,4 and 6;
3,5 and 6.

7. The diagrams show vertical sections of kidneys of coypu, brown rat


and kangaroo rat, showing the relative size of cortex and medulla.
Coypu are found in fresh water and are never short of water to drink.
Brown rats are able to go some days without drinking. Kangaroo rats
are able to live in deserts without drinking at all. Which kidney belongs
to which animal?

a)
b)
c)
d)

1
brown rat
brown rat
kangaroo rat
kangaroo rat

2
coypu
kangaroo rat
brown rat
coypu

3
kangaroo rat;
coypu;
coypu; +
brown rat.

8. Rockmusicians, boiler-makers, and other workers who are subjected


to loud, high-pitched noises over a period of years frequently become
deaf to high tones because;
a) the cells near the base of the organ of Corti become injured; +
b) the tympanic membrane become injured;
c) the membrane at the opening of the inner ear, called the oval window
becomes injured;

d) the auditory nerve becomes injured.


9. Characteristics of animals with open circulatory system.
a)
b)
c)
d)

haemoglobin, haemocoel, lymph;


haemocyanin, haemocoel, haemolymph;+
haemoglobin, haemocoel is absent, haemolymph;
haemocyanin, haemocoel is absent, lymph.

10. Corneal transplants are highly successful because.


a)
b)
c)
d)

the tissues from the patient and from potential donors are matched; +
the cornea has almost no blood or lymphatic vessels associated with it;
drugs and X-rays have been used to destroy T-lymphocytes;
most patients do not have immune system.

11. The heavy line in the graph above illustrates the relationship
between auxin concentration and cell growth in stem tissues. If the
same range of concentrations were applied to lateral buds, what curve
would probably be produced?

a)
b)
c)
d)
e)

I; +
II;
III;
IV;
either I or III.

12.

The cell represented by number "x" is


a)
b)
c)
d)
e)

B cell;
T cell; +
mast cell;
mucrophage;
plasma cell.

13. When sunlight is on the chloroplast, pH is lowest in the


a)
b)
c)
d)

stroma;
cytosol;
space enclosed by the thylakoid membranes; +
space enclosed by the inner and outer membranes.

14.

What is the type of life cycle of green algae shown in the Fig. above?
a)
b)
c)
d)

haplontic type; +
diplontic type;
isomorphic diplohaplontic type;
heteromorphic diplohaplontic type.

15. Red algae differ from the green algae and brown algae in having:
a)
b)
c)
d)

no chlorophyll a;
no differentiated cells;
no phycocyanin within their cells;
no flagellated stages in their life cycles.+

16. Ribosomes in the cytoplasm of eukaryotic cells are:


a)
b)
c)
d)

the same size and composition as in bacteria;


larger than in bacteria, but of similar composition; +
smaller than in bacteria, and different in composition;
the same size but completely different in composition from the ribosomes
in bacteria.

17. The relationship between oxygen consumption and weight (body


size) of sea anemones was studied. The rate of oxygen consumption
depends on the surface area of the animal. Which of the following
graphs best describes this relationship?

a)
b)
c)
d)
e)

A;
B;
C;
D;
E.

18. Most metabolism of a plant is carried out by the


a)
b)
c)
d)

epidermis;
collenchyma;
sclerenchyma;
parenchyma. +

19. Tissues that form long, tough strands, as in the leaf stalk of celery,
are:
a)
b)
c)
d)

epidermis;
collenchyma;
sclerenchyma; +
parenchyma.

20. The lowest water potentials in the xylem are in the


a) root hairs;

b) vascular cylinder of roots;


c) tracheids of the stem;
d) leaves. +
21. Many fungi look like and live like plants, but they are put in a
separate group because:
1. Fungi are prokaryotic;
2. Many fungi are parasitic;
3. Fungi do not have embryological development;
4. The cell wall of fungi do not contain cellulose;
5. sexual reproduction is lacking in many groups of fungi.
a)
b)
c)
d)

1,3,5;
2,4;
3,4,5;+
4,5.

22. Which of the following features are shared by the Eubacteria?


1. prokaryotic cell;
2. rigid cell wall with peptidoglycan;
3. eukaryotic cell;
4. multiple by binary fission;
5. rigid cell wall with cellulose;
a)
b)
c)
d)
e)

1, 2, 4; +
2, 3, 4;
3, 4, 5;
1, 2;
3, 5.

23. One of the most widely used natural insecticides are the toxins
produced by
a)
b)
c)
d)
e)

Bacillus thuringiensis; +
Salmonella sp.;
Bacillus stereothermophelis;
Bacillus cereus;
Escherichia coli.

24. The diagram represents the respiratory pathway


Which stages involve the action of both a decarboxylase and a
dehydrogenase?

a)
b)
c)
d)

P and Q;
Q and R;
Q and S; +
R and S.

25. For the next ten items refer to the data plotted on the following
graph.
Use statements, which are supported by the data of this graph.

1.
2.
3.
4.
5.
6.
7.
8.

Stems do not respond to auxins.


Plant roots do not respond to auxins.
Roots react differently to auxins than stems do.
Roots respond to different auxins not so as stems do.
Roots excrete auxins and thus protect themselves.
High concentration of auxins increase root growth.
Stem growth is always increased by the addition of auxins.
The growth of a root is promoted by amounts of auxins smaller than those
for stem growth.
9. Root growth is inhibited by stem growth.
10. Root growth is accelerated by stem growth.
a)
b)
c)
d)
e)

3, 8;+
1, 2, 6, 7;
4, 5, 9, 10;
3, 6, 10;
1, 8.

26. The following are statements about the female reproductive system.
Respond by true.
1. Both oestrogen and progesterone are necessary for ovulation to take
place;
2. Oestrogen tends to inhibit the production of FSH by the anterior pituitary
gland;
3. Fertilization of the ovum by the spermatozoon normally takes place in the
uterus;
4. Progesterone production is largely under the control of Lh;
5. Throughout the part of the mensual cycle that follows ovulation, there is
a slight rise in body temperature.
a) 1, 2, 5;
b) 1, 3, 5;
c) 2, 3, 4;
d) 2, 4, 5;+
e) 5.
27. A man of blood group A has 2 children. Plasma from the blood of
one
of them agglutinates his red cells while that from the other does not.
Choose correct statements.
1. Father must be heterozygous group (A).
2. Children must have different mothers.
3. "Agglutinating" child could be group O.
4. Mother of "agglutinating" child must be group O.
5. "Non-agglutinating" child could be group AB.
a)
b)
c)
d)
e)

1, 2, 5;
1, 3, 5;+
2, 3, 4;
2, 4, 5;
5.

28. Ultrafiltration is responsible for the fluid movement that takes


place in the following processes:
a)
b)
c)
d)

concentration of bile;
salivation;
glomerular filtration; +
sweating;

29. If (Rh+) blood is transfused into an (Rh-) woman who has not
previously
been transfused, then:
(Choose correct statements.).
8

a) anti-Rh antibodies will be produced in the organism of women;+


b) the blood is incompatible so red cell agglutination and death may follow;
c) there is no immediate or long term effect as 70% of the Rh population are
heterozygous;
d) provided anti-D antibody is given before the next pregnancy no harm will
be done.
30. The next four items are based on the family tree below. The studied
character concerns the hair.
Choose incorrect statements.

1.
2.
3.
4.

The character wavy hair is dominant


The number one is homozygote
One of the parents of the N2 had probably the same genotype as him
If number 3 and number 4 could have another child he should have lank
hair.

a)
b)
c)
d)
e)

1, 3;
2, 4;
1, 2, 4;+
2, 3, 4;
1, 2, 3, 4.

31.

Mayflies are tasty aquatic insects that are especially vulnerable to


predator during their transition to adult hood, when they are emerging
from the water to mate and lay their eggs. This graph illustrates the
effect of the number of mayflies emerging per day on the total
predation on them.
Choose correct statement.
a) the more mayflies emerging on a June evening, the more likely any one
mayfly is to be taken by a predator. b) the more mayflies emerging on a June evening, the less likely any one
mayfly is to be taken by a predator. +
c) the number of emerging mayflies diminishes steadily from the beginning
to the end of June. 32. Oxygen content reduction makes the glycolyse (glycogenesis)
intensity increased due to:
a)
b)
c)
d)

increase of ADF concentration in cell; +


increase of NAD+ concentration in cell;
increase of ATP concentration in cell;
increase of concentration of peroxides and free radicals.

33. Nucleosomes prevent:


a) the replication and all the stages of DNA-transcription;
b) the beginning of DNA-transcription, but dont prevent its continuation and
replication; +
c) the replication and the beginning of transcription, but dont prevent its
continuation;
d) the continuation of the transcription, but dont prevent its beginning and
replication.

10

34. The main organ which produces glucose from lactic acid is:
a)
b)
c)
d)

kidneys;
liver; +
spleen;
intestines epithelium.

35. Ribosomes consist of:


a)
b)
c)
d)

RNA and proteins; +


RNA, proteins and lipids;
lipids and proteins;
RNA, proteins, lipids and carbohydrates.

36. The movement of flagellum of eukaryotic cells realised by means of


hydrolusis of ATP by:
a)
b)
c)
d)

actin;
myosin;
dynein; +
kinesin;

37. Flagellum of procaryots moves by means of the energy of:


a)
b)
c)
d)

ATP;
electrohimical potential;
phosphorylation;
phosphoendpiruvate;

38. Genetic material transmision from one bacterium to another one


with the help of virus is called:
a)
b)
c)
d)

transposition;
transformation;
transversion;
transduction. +

39. Inner lysosomes medium is:


a)
b)
c)
d)

more acid than that of cytoplasma;+


more alkaline than that of cytoplasma;
has the same pH that is in cytoplasma;
sometimes more acid and sometimes more alkaline.

11

40. HIV agglicts:


a)
b)
c)
d)
e)

T- lymphocytes; +
B-limphocytes;
monocytes;
neutrophilus;
basophilus.

41. A heron standing in a cold water for a long time doesnt get its legs
overchilled because of:
a)
b)
c)
d)

countercirculation in limbs ; +
even thin sat layer under limbsskin;
branched blood stream in limbs:
intensive metabolism in limbs.

42. Decrease of excitability of nerve cells is:


a)
b)
c)
d)

membrane potential decrease after absolute magnitude;


change of rest potential of a cell towards negative; +
membrane potential change from negative figures towards positive ones;
changing of membrane potential sign from positive to negative.

43.It is possible to incubate in agar the culture of agents of:


a)
b)
c)
d)

diabetes;
influenza;
malaria;
dysentery. +

44.Which structure of the mammales is a modified parietal eye?


a)
b)
c)
d)

hypophysis;
cerebellum;
epiphysis; +
diencephalon.

45. The following creatures are natural carriers of the agent plague of
human being in the nature:
a)
b)
c)
d)

wolves, foxes;
birds;
rodents; +
a human being.

46. Which group of modern reptiles is the most ancient:


a) lizards (Sanria);
b) snakes (ophidia);

12

c) turtles (chelonia); +
d) crocodiles (crocodilia);
47. Ectoderm, endoderm and mesoderm develop into tissues and
organs.Which of the following combinations is correct?
Ectoderm
a)
b)
c)
d)

nervous system
nervous system
skin
epidermis

mesoderm
blood
large intestine
bones
liver

endoderm
lungs; +
lungs;
kidneys;
heart.

48. The corn culm diameter is determined by the activity of the following
type of meristem:
a)
b)
c)
d)

primary; +
secondary;
primary and secondary;
first primary then secondary.

49. Secondary thickening of a stem is typical for:


a)
b)
c)
d)

mosses, gymnosperms, angiosperms;


monocotyledoneus, angiosperms, gymnosperms;
monocotyledoneus and dicotyledonous angiosperms;
gymnosperms and dicotyledonous angiosperms. +

50. All gelminthes have such distinctive features as:


a)
b)
c)
d)

absence of digestive system;


no sense organs;
hermaphroditism;
high reproduction potential.

51. Mammals arterial blood flows through veins and venous blood
though arteries:
a)
b)
c)
d)

in the systemic circulation;


in the pulmonary circulation;+
in livers portal system;
during extrasystolic circulation when the blood is being pomped from
hearts ventricle into auricles.

52. Why do mans eyes constantly move while looking at an object?

13

a)
b)
c)
d)
e)

to keep the object in the field of vision;


to provide the direction of light beams to the yellow spot of retina;
to keep the light on the receptors of an eye for a short time;
to provide focus of image on the retina;+
to deadaption of optic neuron.

53. Which of the following fossil plants could be ancestors of the


contemporary highe plants?
a)
b)
c)
d)
e)

Psilophyta;+
Rhuniophyta;
Phaeophita;
Rhodophyta;
Cylorophyta.

54. Which of the following higher plants was brown coal formed from?
a)
b)
c)
d)
e)

jointweeds;
ferns;
lycopodiums;
gymnosperms;
rborescentflowering plants.

55.Which of following functions couldnt be considered as the functiopn


of human digestion system?
a)
b)
c)
d)
e)

physical processing of food;


hydrolysis of food components to monomereses;
elimination with species specifity of food components;
to free the energy during the oxydation of food components;+
antybacterium processing of food.

56. Primary lysosomes are produced in:


a)
b)
c)
d)

cytoplasm;
nucleus;
canalicular apparatus;+
cell center.

57. Proteins that have been produced in the rough endoplasmatic


reticulum are transported to:
a) lisosoms;
b) hyaloplasm;
c) mitohodrium;

14

d) plastids;
e) canalicula apparatus.+
58. Gibberellins (substances of plants growing) are:
a)
b)
c)
d)
e)

phospholipids;
three acyl glycerol;
wax;
steroids;
terpenes.+

59 Gens for ribosomal RNA in chromosomes are situated in the area of:
a)
b)
c)
d)
e)

telomere;
primary stretching;
kinetochore;
satellite;
secondary stretching.+

60 What determines the connection between ribosomes and reticulum?


a)
b)
c)
d)
e)

carbohydrates;
N-terminal protein sequence;+
C-terminal protein sequence;
lipids;
ions.

61. What is the main factor to limit the increasing of general biomass on
the definite territory?
a)
b)
c)
d)
e)

deficiency of O2 and CO2;


deficiency of H2O;
temperature;
flow of sun energy;+
biotyc relations.

62. Spatial structure of actin-myosin system is put into the basis of the
muscular tissues classification. According to this way of approach all
muscular tissues of multicellular are divided into three types:
a)
b)
c)
d)
e)

non-striated, cardial, cross-striated


non-striated, skew-striated, cardial;
non-striated, skew-striated, cross-striated;+
epitelium-muscular, symplastic, cardial;
postural, phasic, paramyosinic.

15

63. Heart systole force is known to quickly change when the


concentration Ca2+ in the extracellular liquid is changed. Removal Ca2+
from perfusive solution leads to stopping of heart activity. Its
connected with:
a) stopping of generation of action potentials in pacemakers;
b) stopping of generation of action potentials in myocard;
c) Action potentials are generated in pacemakers but excitation transmission
is disturbed in the heart;
d) action potentials are generated in the heart but electrochemical
conjugation is corrupted;+
e) action potentials are generated in the heart but excitation transmission is
disturbed via.
64. There are some ways to define the increasing of speed of excitation
transmission via axon. The main of them are:
a)
b)
c)
d)
e)

diameter of axon;
distance between Runves interception and amplitude of action potential;
magnitude of rest potential and size of;
a.&b;+
a.&c.

65. In nature there are only several main mechanisms of moving which
are used by different groups of animals in different ways. This is:
a)
b)
c)
d)
e)

walking, flying, swimming;


ciliary, flagellate, undulating;
muscular, ciliary, amoeboid;+
muscular, reactive, amoeboid;
muscular, undulating, reactive.

66. The high limit of force (in relative magnitudes) is approximately the
same for locomory muscles of different animals. It can be explained in
following way:
a) the force of muscle doesnt depend on area of cross-section;
b) the force of the muscle doesnt depend on it belonging to phasic or
postural groups of muscles;
c) the force of muscles contraction depends on quantaty of myofibrillas that
take part in contraction;+
d) the force of muscles contraction depends on reserve of ATP in muscles
cell;
e) the force of muscles contraction is limited by entrance of O2 .
67. The heart of the homoiotherms has the mechanical structures
(endoskeleton) which formed by fibers of collagen and elastic. Its
function is:

16

a) to provide the length of rest of the cardiac muscle and to prevent its
oversprain;
b) to increase efficiency of the cardiac muscle and to provide its quick
returning to the primary condition;
c) to provide active affluence of blood to the heart;
d) all answers are correct;+
e) there are no correct answers.
68. Which animals have the system of giant axons?
a)
b)
c)
d)
e)

Coelenterata, Plathelminthes, Nemathelminthes;


Plantheminthes, Nemathelminthes, Annelida;
Nemathelminthes, Annelida, Artropoda;
Annelida, Artoropoda, Mollusca;+
Artropoda, Mollusca, Chordata.

69. Which reactions are made with the help of system of giant axons?
a)
b)
c)
d)
e)

quick non differential reactions;+


quick differential reactions;
slow non differential reactions;
slow differential reactions;
a.&b.

70. In human stomach splitting of the great bulk of carbohydrates is


realisad with the help of:
a)
b)
c)
d)
e)

amylase pH 5,8-7,8;+
amylase pH 1,5-1,8;
lactase pH 5,8-7,8;
lactase ph 1,5-1,8;
theres no correct answer.

71. How is increasing of temperature up to an optimum level (for


graving out of pollen to embryo bag of pistic inside flowers of Alpine
plants) can be reached in conditions of rather low temperature of air?
1. Flowers of plants are dark-coloured and are heated up by the sun well;
2. Petals of these flowers are closed a large part of their existence and
consequetly keep warm better.
3. Flowers of these plants have a high level of breath, which is accompanied
by allocation of energy of metabolism as ATF; +
4. Petals of the majority of these flowers, similary to parabolic mirrors,
concentrate solar beams, in the centre of the flower where generative
organs (pistic).+

17

a)
b)
c)
d)
e)

1;
1, 2;
2, 3;
3, 4;
1, 2, 3, 4.

72. Explain, why do many granivorous birds ( sparrow, goldinch) feed


nestlings not by a grain, but by insects.
1. grain and seeds dont ripen to time rearing of nestlings.
2. nestlings digest a grain poorly;
3. animal food is more full value for nestlings.
a)
b)
c)
d)
e)

1;
2;
3;
1, 2;
2, 3.+

73. Competitive inhibitors of ferments have:


a)
b)
c)
d)
e)

similar structure with a molecule of ferment;


similar structure with a molecule of the substratum;+
similar structure with the gene, which codes the ferment;
ability to connect a molecule of the ferment and the substratium;
ability to inhibit ferment.

74.The graph below illustrates action potential of the neuron. Using the
code denote which parts of it are marked with letters.

The codes of the answers:


1. repolarization;
2. rest potential;
3. depolarization;
4. after hyperpolarization;
5. paradoxical phase;
18

6. levelling phase;
7. inhibitory phase;
8. parabiosis;
A
a) 7;
b) 1;
c) 2;
d) 5;

B C
4; 3;
3; 2;
3; 1;
6; 3;

D
1;
5;
4;+
7;

75. Choose the features that confirm the hypothesis of endosymbiosis in


respect of beginning of chloroplastids.
1.
2.
3.
4.
5.
6.
7.
8.

single - membrane,organelles;
double - membrane organelles;
they have the genetic make up of their own that includes circular DNA;
they have no the genetic apparatus of its own;
they have ribosomes that differ from ribosomes of cytoplasme in structure;
they have ribosomes that do not differ from ribosomes of cytoplasm;
they have a lot of common features with bacterium;
they have a lot of common features with blue - green algae.

a)
b)
c)
d)
e)

1, 4, 5, 7;
2, 4, 6, 7;
1, 3, 6, 8;
2, 3, 5, 8;+
there are no correct answers.

76. Which of the following characteristics belong to electrical synapses?


1. They provide quick reactions;
2. They provide slow reactions;
3. They provide synchronous responses;they provide nonsynchronous
responses;
4. They provide the coordination of cells that carry out the same functions;
5. They provide the coordination of cells that carry out different functions;
6. Vertebrates have them;
7. Chordates have them.
a)
b)
c)
d)
e)

1, 2, 6, 8;
1, 3, 5, 7, 8;
2, 4, 6, 7, 8;
1, 3, 5, 7;
2, 4, 6, 8.

77. Diagram I shows an electron microscopic picture. What is depicted?


Please mark the correct answers!

19

1. A cell;
2. A tissue;
3. An organ;
4. A plant organism;
5. An animal organism;
6. A procaryotic cell;
7. An eucaryotic cell;
8. A fatty tissue;
9. A supportive tissue;
10. A cross-section of a root;
11. A cross section of a plant axis;
a)
b)
c)
d)
e)

3, 4, 11;
2, 9, 10;
1, 4, 6;
1, 4, 7;+
3, 5, 8.

78. The plasmatic membrane of all cellss consists of:


a)
b)
c)
d)
e)

lipids, proteins and carbohydrates;+


only lipids;
only proteins;
lipids and carbohydrates;
lipids and proteins

79. As early in the XI and XIIth century knowledgeable healers


recommended applying soil gathered from beneath black alders Alnus
glutinosa to inflammed skin. Please state the reason for their success.
a) black alders form a symbiotic relationships with a certain kind of
fungus.Together they form small root bulbs to assimilate nitrogen which
are called mykorrhiza. This mykorrhiza is also inhabited by streptomycetes
which live near black alder roots.They exude streptomycin. This is
responsible for the healig effect. +

20

b) different kind of penicillium flourish in the soil beneath black alders.


Penicillin, which is dissolved due to the ground humidity, is responsible for
the healing effect.
c) the soil around black alders is relatively sterile due to the substances
contained in the roots. If the soil is applied to a wound, pathogens are
deterred.
80. Microbiologic culture media are produced in a pressure cooker at a
temperature +1210 C. Which of the following organisms survive the
procedure of simply boiling the culture media at +1000 C?
a)
b)
c)
d)
e)

Escherichia coli;
Penicillium notatum;
Bacillus subtilis; +
Salmonella typhimurium;
Convallaria mayalis.

81. Regard the following illustration and decide what kind of structure
is depicted.

a) The depicted structure is the wood of an angiosperm plant, since the


strands of living tissue are made up of several layers;
b) The depicted structure is the wood of a gymnosperm plant;+
c) The depicted structure is the wood of an angiosperm plant, since it
possesses vessel elements;
d) The depicted structure is the wood of an gymnosperm, since the strands of
living tissue are single layered.

21

82. Accessory pigments of photosynthesis transfer the absorbed energy


to photosynthetically active pigments, hereby losing more or less
considerable amount of energy. Which of the below mentioned
substances do not qualify as accessory pigments?
a)
b)
c)
d)
e)

chlorophyll a;+
chlorophyll b;
carotines;
fucoxanthines of phaeophyceae;
phycobiliproteids.

83. Many insects go into diapause at certain times. Which of the


following statements is incorrect?
a) the rate of metabolism is decreased to a minimum during diapause;
b) the growth rate is stagnant during diapause.
c) factors such as a lack of oxygen, vitamin deficiency,lack of nutrition etc.
could effect a stagnation of growth during diapause.
d) diapause is only caused by a shift in temperature and the duration daylight
hours.+
e) even the eggs of insects can go through diapause phases.
84. In situation that conflict between attack flight animals have been
noted to behave in a most peculiar fashion eg. the behaviour
of
fighting cocks interrupting their fight to peck at the ground for food.
What is this kind of behavior called and what this behavior is based on?
I. Feeding
II. Ritualisation
III. Displacement Activity+
IV. Aggressive Behaviour
V. lack of food or hunger
VI. lateral inhibition of two opposing behavior tendencies
VII. irritation of a nerve
VIII. displacement of an initial stimulus from one nerve to another
a) I and V;
b) II and VII;
c) III and VII;
d) III and VI;+
85. The following diagram depicts the human caryogram with a
hereditary disease. What is the syndrome called?
Which of the below mentioned conditions is caused by the which
deficiency?

22

I. Edwards syndrome
II. Down syndrome
III. Turner syndrome
IV. Slanted eyes, skeletal deformity (flattened back of the head) , severe
mental retardation, male or female.
V. Feminine constitution, rudimentary and functionless sexual organs,
infantilism.
VI. Deformed fingers, heart conditions, high voices, lessened beard growth.
a) I and V;
b) II and IV;+
c) II and IV;
d) II and V.
86. The following statements are possible, general connections between
the speed of evolution and sexual or asexual reproduction.
I. The rate of evolution will be higher in a smaller population than it would be
in a larger one.
II. The rate of evolution will behave oppositely proportional to the generative
duration.
III. The rate of evolution will increase in proportion to the variety of the
different ways of life. It will decrease in proportion to the density of competition
which newcomers find in an certain area.
a) only III is right;
b) only I and II are right;
c) all statements are correct;+
d) none of the statements are correct;
87. The rate of hereditariness H can expressed in the following
mathematical manner. Which statement is correct?

23

VG genetic variability;
VP phaenotypic variability;
VE environmentally caused (modificated) variability.
a)
b)
c)
d)

A;+
B;
C;
D.

88. The connections between organism and their environment are


diverse and
numerous. The following illustration depicts a comparison of height
between six different kinds of penguins. Please comment on the picture.
Which factor (I-IV) determines the differences in the penguins' size?
Which rule (V-VIII) describes these circumstances?

I. The nutritional supply;


II. The temperature;
III. The competition to other species;
IV. The different levels of oxygen in their native waters.
V. The pessimum law;
VI. The Allen rule;
VII. The Bergmann rule;
VIII. The ecological potency
a) I and III;
b) I,II and VIII;

24

c) II and VII;+
d) IV and VIII.
89. Brachionus calyciflorus is a frequent inhabitant of eutroph ponds
and lakes. Different kinds are known, especially one with a pair of thorns
on the hind or caudal end of the shell. This appearance, which is quite
frequent with plancton organisms is known as cyclomorphosis:

Brachionus rubens does not possess thorns on its hind end as a


means of defense.
A different kind of single cell organisms, Asplanchna brighwelli, feeds
amongst others on Brachionus calyciflorus and Brachionus rubens by
attacking them from the rear in order to devour the prey. How would
the population density develop if you mixed a culture of Asplanchna
brightwelli with Brachionus calyciflorus ? How would the population
density develop if you mixed a culture of Asplanchna brightwelli with
Brachionus rubens?
I. The predator would survive;
II Predator and prey would survive;
III. The prey would survive.
IV. The predator would survive;
V. The prey would survive;
VI. The population would develop according the Volterra laws.
a) I and IV;
b) II and VI;
c) III and VI;+
d) III and V.
90. The diagrams A to C show population pyramids of a bee populations
in different seasons (spring, summer and winter). Please match the
spring with the corresponding illustration.

25

a) A;+
b) B;
c) C.
91. Reproduction is often connected with a shift from haploid to diploid
or the other way round. Please regard the diagram and decide with
group of organisms show the below mentioned type of shift.

a) eg. for humans;


b) eg. for angiosperm plants;
c) eg. for gymnosperm plants;
d) eg. for and ferms.+
92. The following dichotomous key will be useful in determining the
differences between cereals. At which point in this key would you place
wheat (Triticum)?
1. Cereal ear without stem with a very short stem - organism A
1*. Cereal ears have long stems.
2. On every nodium of the cereal head's axis there is more than on ear organism B.
2*. On every nodium of the cereal head's axis there is only one ear.
3. Ears contain three or more blossoms - organism C.

26

3*. Ears contain two blossoms.


a) A;
b) B;
c) C.+
93. Age of some trees can be determined due to the presence of the
"tree-rings" (annual growth rings) , which represent the yearly growth
of:
a) primary phloem and xylem;
b) secondary phloem and xylem;
c) secondary phloem only;
d) secondary xylem only;+
e) cork.
94. The eye of a squid and the eye of a fish are an example of:
a) secondary adjustment to life in a water habitat;
b) vestigial organs;
c) homologous organs;
d) analogous organs;+
e) two of the above answers are correct.
95.The pedigree represents a women suffering from a certain rare
disease caused by a recessive mutation in one gene. What is the
probability that fourth child in the family ( marked by a"?" ) is also ill?
The mutation in question behaves according to classical Mendelian
genetics.

a)
b)
c)
d)
e)

zero;
0.25;
0.5;
0.125;
0.25 if the gene is autosomal, 0.125 if it is on a sex chromosome.

96. On illumination green plants release oxygen gas. Oxygen atoms


derived
from:
a) reduction of CO2;
b) decomposition of water;
c) decomposition of sugars;
d) photorespirathion;
e) two of the above possibilities may be correct under different
circumstances.
27

97. In root nodules, the nitrogen-fixing bacteria obtain from the host
plant:
a) nitrogen dissolved in vascular sap;
b) oxygen dissolved in vascular sap;
c) nitrates
d) sugars
e) neither of the above;
98. Certain physiological process occurs in a plant only after it has been
illuminated with full-spectrumn white light or its red component; other
monochromatic cannot produce this effect.Based on this, one may
expect that process is regulated via:
a) chlorophyll;
b) a flavonoid pigment;
c) phytochrome;
d) auxins or giberellins;
e) the information given is not sufficient to choose from among above
answers.
99. A medium-strong stimulation of a tactile receptor (touch receptor)
causes in its nerve fibre an action potential, whose oscillograph
representation is shown in fig."X". How will the action potential look like
after a stronger stimulus?

a)
b)
c)
d)
e)

A;
B;
C;
D;
E.

100. After marathonrun oxygen depth has formed in muscles of the


runner;
a rest period after the run, which of the following will take place in the
runners organism with particular intensity?
a) conversion of pyruvate into lactate;
b) conversion of lactate into pyruvate;
c) accumulation of NADH;
d) glycolysis;
e) two of the above;
101. Is it possible that individual cells of a certain species of bacteria
harbor different numbers of a plasmid molecule?

28

a) no, only one plasmid molecule may be present per cell;


b) yes, but only if are different konds of plasmids;
c) yes, but only if the bacteria are cultivated differing conditions;
d) yes, because replication of most plasmids is not very strictly controlled;
e) yes, but only for different strains of the bacterium.
102. Which of the features characteristic for (most) mitochondria and
chloroplasts is NOT indicative of the organelles` endosymbiotic origin:
a) possession of own, usually circular DNA, and own ribosomes;
b) structure and compostion of the organelles` ribosomes;
c) the way these organelles multiplicate;
d) presence of numerous introns in the organelles` genes;
e) all of the above features reflect the similarity to prokaryotes, therefore
support the endosymbiotic theory of the organelles`origin.
103. large doses of ionizing radiation cause in animals so called (post)
radiation disease. Its cause is:
a) disruption of cell membranes;
b) damage to the enzymes catalysing basic metabolic processes;
c) killing of symbiotic microorganisms in the alimentary tract,
and consequently inadequate supply of some vitamins;
d) damage to DNA, interfering with cell multiplication;
e) olways answers is correct.
104. Graph illustrates the speed of enzyme-catalysed reaction (V) as a
function of temperature (temp). Why at temperatures t1 and t2 the
reaction speeds are the same?
a) because all chemical reactions have an optimum temperature, above and
below hey progress at lower speeds;
b) at t1 temperature only a few enzyme molecules are active, but they from
very stable complexes with their substrate molecules, where as at t2 the
opposite is true;
c) at t2 less enzyme molecules are active, than at t1, but each of these
molecules is capable of converting more substrate molecules per time unit
than at t1;
d) at both temps. the number of active enzyme molecules is the same, but at
t1 each of them forms more stable enzyme-substrate complexes than at
t2, and t2 each enzyme molecule works faster, than at t1;
e) all the above explanations are incorrect.
105. Among known extant (contemporary) animal species,
invertebrate species constitute about:
a) several %;
b) a dozen % (between 10 and 20);
c) roughly 50%;
d) roughly 75%;
e) above 90%.

29

106. A group of plants as "algae" is characterized jointly (especially in


older textbooks), because:
a) they are a true monophyletic group;
b) they are polyphyletic, but nevertheless show remarkable similarity
their physiology and chemical constitution (photo - synthetic pigments,
storage material etc);
c) they are polyphyletic, but nevertheless have almost identical life cycles;
d) they are polyphyletic, but neverthelles exhibit similar ecology and degree
of morphological comlexity;
e) more than one the above explanations is correct.
107. Mimicry is the rtzult of evolutionary process when organisms of
one species mimic the features (usually bright coloration) of another
species ( the "master specie"). USUALLY the organisms of the "master
species" are:
a) very closely related to the mimicking species (they form so called sibling
species);
b) much larger, than those of the mimicking species;
c) poisonous or inedible;
d) innocuous, but the organisms of the mimicking species are poisonous;
e) more than one of the above answers is correct.
108. " Fight for survival" is a consequence of:
a) innate tendency to compete;
b) the necessity to fight against natural disasters (floods, volcanic eruptions,
temperature extremes etc);
c) genetic variability;
d) the number of offspring organisms exceeding capacity of their habitat;
e) all of the above.
109. Palinology is the science of fossilized pollen.The best preserved
pollen grains are likely to be found in:
a) crude oil;
b) peat;
c) coal;
d) limestone;
e) sandstone.
110. Some moss (bryophyta) and club-moss (Lycopodiophyta) species
may be
morphologically quit alike, but they differ by numerous characters.
Which of the following is correct:
a) if in the transverse section of the talk) one observes specialized waterconducting cells - the species certainly belongs to club - mosses;
b) if in the cells of the "leaves" one sees an odd number of chromosomes the certainly belongs to mosses;
c) if all spores of the organism are identical - the species certainly belongs to
mosses;
d) all of the above are correct;
e) neither of the above is correct.
30

111. In the blood of an adult man the total content of haemoglobin is,
roughly:
a) several hundred gram;
b) tens of gram (10-100 g);
c) several gram;
d) several hundred milligram;
e) tens of milligram.
112. The action of a chemical inhibiting the enzyme acetylcholinesterase
will cause, as a primary response:
a) complete relaxation of all muscles;
b) relaxation only of skeletal (striated) muscles;
c) contraction of most of the muscles. both striated and smooth;
d) contraction of some of the striated muscles only;
e) contraction of some smooth muscles only.
.113. Penicillin does not harm nondividing bacterial cells, but in its presence bacteria cannot increase in number. From this information it is
reasonable to assume that penicillin inhibits:
a) transcription;
b) protein biosynthesis;
c) biosynthesis of cell-wall component(s);
d) DNA replication;
e) more than one of the above is consistent with the given information.
114. Callus cells were incubated for several hours in a medium
containing all necessary substances, with one of the chemicals being
radioactive( tritium-labelled). Afterwards the cells were fixed for
microscopy; using autoradiography it was observed that the
radioactivity was concentrated exclusively in the nucleus,the
mitochondria and chloroplasts. It is reasonable to assume that the
compound labeled was?
a) an aminoacid;
b) uridine;
c) thymidine;
d) glucose;
e) more than of the above possibilities is very likely.
115. Rough endoplasmic reticulum should be particularly abundant in:
a) reticulocytes ( immature erythrocytes);
b) nerve cells;
c) pancreas cells;
d) smooth muscle cells;
e) it is abundant to a similar degree in all of the above types of cells.
116. Correct ordering of structures, from the least complex one (that is,
containing the lowest number of different kinds of protein molecules) to
the most complex (that is, containing the largest number of different
kinds of protein molecules) is given in answer:
31

a) scrapie prion; HIV virion; eukaryotic ribosome; mitochondrion;


b) eukaryotic ribosome; HIV virion; scrapie prion; mitochondrion;
c) scrapie prion; HIV virion; mitochondrion; eukaryotic ribosome;
d) HIV virion; scrapie prion; eukaryotic ribosome; mitochondrion;
e) scrapie prion; eukaryotic ribosome; HIV virion; mitochondrion.
117.Polyribosomes may be found free in the cytoplasm ( cytosol), or
attached to the endoplasmic reticulum. Which factor determines
whether a molecule of mRNA is translated in "free" or in "bound"
polyribosomes?
a) it depends on the amino acid sequence of the polypeptide coddedfor by
this MrnA;
b) if the mRNA has a poly(A)-tail - it will be translated in bound polysomes, if
it has no tail - it will be translated in free polyribsomes;
c) if the mRNA has a 5' cap - it will be translated in bound polyso- mes, if it
has no cap - it will be translated in free polyribosomes;
d) only mRNAs shorter than can 1000 nucleotides are translated in free
polyribosomes; the longer ones will be translated in bound polyribosomes;
e) in fact it depends on the type of the cell - in certain cels mRNA get
translated mostly in free polyribosomes, in other types of cell - mostly in
bound polyribosomes;
118. The substances given below are all coloured. For which of them
this facthas no direct bearing on its functioning?
a) chlorophyll;
b) phytochrome;
c) vitamin A;
d) haemoglobin;
e) for all the above substances their colour is essential for their functioning.
119. Ecological assemblies K through Q consist of species designated
with numbers l through 8, present a various densities. Individual deities
of these species in any particular assembly are given (as individuals
per square meter) in the table. Which of the above assemblies is the
least susceptible to a massive pest infection (gradation):
assembly
species1
species 2
species 3
species 4
species 5
species 6
species 7
species 8

K
50
30
10
10
0
0
0
0

L
92
4
0
0
1
1
1
1

M
75
5
5
5
5
5
0
0

N
0
25
20
20
20
5
0
0

P
0
2
3
5
40
50
0
0

Q
0
65
20
10
3
2
0
0

a) K;
b) L;
c) M;

32

d) N;
e) P.
120. Tolerance spectra of water temperature and salinity of a
craustacean
species is shown in the graph. This species is likely to be found on New
Year's eve (Christian) in surface waters of:

a) an alpine pond;
b) Aral See;
c) Indian Ocean;
d) The Dead Sea;
e) in more than one of the above.
121. A simplified genealogy tree of seven species is shown. Based
solely
on this tree, which of the following statements is correct:

a) the evolutionary relatioship of tyranosaurus and pigeon is closer than of


tyranosaurus and lizard;
b) turtle is more closely related to lizard than to tyranozaurus;
c) man is more closely related to pigeon than to turtle;
d) two of the above statements are correct;
e) statements A., B. and C. are correct.
122. In the Earth's past there were several episodes of sudden (in the
geological time-scale) extinction of a significant number of species.
During one such period dinosaurs, among other groups, became
extinct. This took place:
33

a) several million years ego (Myr);


b) a dozen 10-20) Myr;
c) somewhat more than 50 Myr;
d) ca. 100 Myr;
e) more than 200 Myr.
123. Animal behaviour patterns, in which an individual endangers its life
to benefit other members of the group, are called altruistic.It is believed
that altruistic behaviour was favoured by kin selection. Which if the
examples given below CANNOT be explained as kin-selection-favoured?
a) suicidal attack by a worker bee guarding its hive;
b) protection of the queen of an ant species by "soldier ants";
c) protection of lion cubs by a lioness NOT being their mother;
d) warning cries of a bird warning other individuals abou approaching
danger;
e) in fact, all of the above behaviour patterns probably arose through kin
selection.
124. School forming by fish, herd forming by herbivores, aggregation
forming by planctonic organisms:
a) reflects the organisms' intrinsic social needs;
b) makes it easier to obtain food;
c) protects individual organisms against a predator attack;
d) helps protect the progeny;
e) is only temporary and solely reflects local abundance food.
125. A retrovirus is causing AIDS, the so called HIV-virus. A retrovirus
contains RNA, which the host cell is transcripted in DHA,with the help of
the enzyme reverse-transcriptase. Which of the following statements is
correct?
I. Viral DNA is synthesized in cells infected with the HIV-virus;
II. Viral RNA is synthesized in cells infected with the HIV-virus;
III. DNA is translated to viral protein.
a)
b)
c)
d)
e)

I;
II;
III;
I, II;
I, II, III.

126 The picture shows a schematic drawing of the cell cycle.

34

Somebody wants to determine the duration of the S-phase. This is


done in adding with tritium labeled compound R to the medium of the
organism with the dividing cells. Which of the following compounds is
the most suitable to be R?
a) adenine;
b) cytosine;
c) guanine;
d) thymine;
e) ATP;
127. Potatoes are stored during one week in pure air, after this during on
week in pure nitrogen and in pure air again. During the experiment the
excretion of CO2 is measured. The diagram shows the results.

The extra amount of CO2 being produced and excreted during the
third week probably originates from:
a) ethanol;
b) ethanal;
c) lactic acid;
d) NAD H2;
128. Four pupils have the following schematic drawings of
chromosomes during Meiosis I.

35

In which drawing(s) the position of the chromosomes allow the


appearance of crossing over?
a) A, B;
b) A, D;
c) A, C;
d) B, D;
e) D, C.
129. A certain plant doesn`t take up radio active substances from the
soil unless fungi are present in and between the root cells. Four
statements derived from this information are stated below. These fungi
do influents:
a) the intake of substances by root hairs;
b) The protection of the root by the rootcap;
c) watertransport by xylem;
d) the growth of the root by meristem;
130. Examine the two drawings of the same stoma different moments.

In which drawing turgor is highest in the guard cell?


In which drawing the concentration of dissolved particles is highest?
a)
b)
c)
d)

Turgor highest
1
1
2
2

concentration highest
1
2
1
2

131. During one hour in light condition the evaporation is measured of


three different but equal sized plants. After this, just above the ground

36

the stem is cut off and during one hour the amount of released fluid is
measured.
Results:
hinese rose
sunflower
tomato

evaporation
6,20 ml
4,80 ml
0,50 ml

released fluid
0,02 ml
0,02 ml
1 0,07 ml

John concludes:
root pressure has a considerable influence on the watertransport
Mary concludes:
capillary forces have a considerable influence on the watertransport.
Who is right?
a) John and Mary;
b) only John;
c) only Mary;
d) neither John,nor Mary;+
132 Examine the figure showing a drawing + magnification of a piece of
a cross section of a root of a dicotyledon plant.

Cell walls are indicated in black.


Four arrows are visible in the drawing.
Indicate which arrow(s) could represent active transport of and ion.
a) 1;
b) 2;
c) 3;

37

d) 4;
133 The influence of pCO2 on the amount of refreshed air in the lungs is
determined of a test person. The experiment is done at sealevel and
high up in mountains at 5800m. The diagram shows the results.

Examine the two following statements:


the sensitivity of the respiratory centre for CO2 increases if pO2
decreases.
the influence of pCO2 on the amount of refreshed air in the lungs
increases if pO2 decreases. What is correct?
a) none of these;
b) only I;
c) only II;
d) both I and II;
134. Inspect the following table with data
total
concentration
mmol/l
I. blood of fish I (sea fish)
1050
II.. blood of fish II (sea fish )
360
III. blood of fish III (fresh water
280
fish)
seawater
1000
fresh water
10

concentration
of Na+ mmol/l)
290
190
130
470
1

Diffusion of water and Na+ may occur in the gulls of these fishes.
Which fish(es) will have for both water and Na+:
excretion / intake <1
a) I;
b) II;

38

c) III.
135 The relative excretion activity is determined of the protozoan
Zoothamnium in different mixtures of seawater and fresh water. The
results are shown in the diagram.

Consider the following conditions:


I. below 15 % seawater
II. 15 seawater
III. 15-40 % seawater
IV. above 40% seawater
Which of these four conditions is most suitable for this protozoan?
a) I;
b) II;
c) III;
d) IV.
136 A bee X has found a location with pollen supplies. Her waggle dance
in a hive is shown in the drawing. Another bee Y also has found a
location with food.

Which of the following drawings represents the waggle dance of bee Y?

39

a) I;
b) II;
c) III;
d) IV;
137 The figure shows possible positions of the black headed gull ( Larus
lanoce phalus).A gull has landed by accident in the territory of another
gull and both gulls have met each other. Which position will probably be
shown by the visitor gull?

a) position 1;
b) position 3;
c) position 5;
d) position 7;
e) position 9;
138 In a certain human population 64% is able to roll the tongue. This
ability is based on a dominant allele. A roller marries a non-roller.
Calculate the chance to have a roller baby.
a) 0,1;
b) 0,225;

40

c) 0,5;
d) 0,625;
e) 0,75.
139. A certain feature of a plant is based on three alleles: E1, E2 and E3.
A pollen grain of this plant is unable to create a pollen tube if one and
the same allele exists in both the pollen grain and the cell of the pistil. A
plant with genotype E1E2 is pollinated with pollen of a plant with
genotype E1E3. Which genotype(s) is (are) to be expected in the
offspring?
a) E1E1 E1E2;
b) E1E3 E2E3;
c) 1E3 E1E2.
140 Three alleles are determining the ABO-bloodgroups: IA, IB and i.
Color blindness is caused by a recessive X-chromosomal allel. Inspect
the following family trees.

As you can see some of the parents and children are colorblind. Also
the bloodgroups are indicated. Directly after birth a baby of couple I
must have been changed with a baby of couple II. Indicate the numbers
of the changed babies.
a) 1 - 3;
b) 2 - 6;
c) 2 - 5;
d) 2 - 4;
e) 1 - 5.
141 The figure shows a karyogram of a sell of a individual with 2n = 4.

41

View the following statements:


I. If chromosome 1 is from the father, then chromosome 2 is from the
mother;
II. Chromosomes 1 and 2 are from the father and 3 and 4 are from the
mother;
III. During meiosis I chromosomes 1 and 2 go to one pole while, 3 and 4 are
going to another pole;
IV. During meiosis I chromosomes 1 and 3 go to one pole while 2 and 4 are
going to another one. Correct are:
a) I and III;
b) I and IV;
c) II and III;
d) II and IV;
142. Someone has investigated the fitness of a population of the coletit
(Parus major) in relation to the number of eggs per nest. He determined:
I. The frequency of number of eggs per nest;
II. The average weight of the young birds at the moment they flew out;
III. The number of birds caught back per nest at the end of the summer;
IV. The average area of a territorium;
The two diagrams A and B are showing the results of two of these
four aspects.

What are the diagrams epresenting?


A
B
a) I
II;
b) II
III;

42

c) I
d) II

IV;
IV.

143. The diagram shows the relationship between the intake of CO2 and
light intensity of leaves of a beech tree in a sunny and in a shadow
position.

Examine the two statements:


I. graph 1 matches with leaves in the shadow position
II. light intensity is a limiting factor for shadow leaves at the situation marked
with lette.
What is correct?
a)
b)
c)
d)

both I and II;


only I;
only II;
none of these;

144. How many kinds of gametes an an individual with n pairs of


hromosomes
produce? (assuming no exchange of part between homologous
hromosomes.).
a) 2n-1;
b) n2;
c) n;
d) 2n;
e) 2.
145. From the of being hatched from the egg, a nestling duck was raised
fully by hand, he or she will think of the raiser as his (her) parents, the
behavioral mechanism of this phenomena is:
a) learning behavior;
b) imprinting;
c) adaptation strategy;
d) recognition.
43

146. When a new male, takes over a lion pride, they sometimes kill or
evict
the cubs already present, this phenomena can be explained from
behavioral
ecology:
a) the male doesn`t like cubs;
b) the male cannot afford too much for caring those cubs;
c) the male breed his own offspring;
d) degeneration of the male`s parental behavior.
147. Honey-bee is a kind of social insects. In its colony, males develop
from
unfertilized eggs and are haploid, while workers develop from normal
fertilized
eggs and are therefore dipliid. Which of the following is the correct
coefficient of realatadness (R) between workers on the colony:
a) 0.5;
b) 0.25;
c) 1;
d) 0.75;
e) 0.375.
148. There is significant correlation between intensity of sexual
selection and mating system. Which of the following is correct?
a) intensity of sexual selection in promiscuity is strong;
b) intensity of sexual selection in monogamy is strong;
c) intensity of sexual selection in polygyny is strong;
d) intensity of sexual selection in polyandry is strong.
149. Which is the determinant factor in distinguishing dominant from
subordinate for animals living in group?
a) aggressive;
b) amicable;
c) territory;
d) recognition.
150. In addition to providing immediate energy, which of the followings
is the most prominent contribution of glycolysis to aerobic respiration?
a) glucose production;
b) NAD + production;
c) ATP production;
d) lactic acid production;
e) pyruvate production.
151. Which of the following is the rate-limiting step in glycolysis?
a) isomerization of glucose-6-phosphate to fructose-6-phosphate;
b) phosphorylation of fructose-6-phosphate;
c) phosphorylation of glucose
d) conversion of pyruvate to lactate.
44

152. Which of the followings is observed in an experimental animal fed


on a diet composed of excessive fat and insufficien carbohydrate?
a) a decrease in acety-CoA synthesis;
b) an extremely high consumption of glucose;
c) a decrease in fat metabolism;
d) ketosis;
e) convertion of fatty acids to glucose.
153. Which of the following substances can pass through the cell
membrane only by means of endocytosis and exocytosis?
a) proteins, amino acids, monosaccharides;
b) oil droplets, gases, fatty acids;
c) proteins, oil droplets, polysacharides;
d) oil droplets, water, ions;
e) pollysaccharides, monosaccharides, water.
154. In a synthetic chain reaction:

if the enzyme E4 has a defect, in this case, at least which of the


followings has to be added to the medium in order the bacteria to grow
normally through autotrophic way?
a) X;
b) A;
c) X and C;
d) C;
e) Y;
155. The following table shows the percentages of nucleic acid bases,
isolated from different species.
Species Adenin Guanin Thymi Cytosin
Uraci
n
1
21
29
21
29
0
2
29
21
29
21
0
3
21
21
29
29
0
4
21
29
0
29
21
5
21
29
0
21
29
According to these information. Which of the following statements is
false?
a) species (1), (2) and (3) carry DNA as the genetic material;
b) species (4) and (5) carry RNA as the genetic material;
c) species (1) and (2) have double-stranded DNA;
d) species (3) has single-stranded DNA;
e) species (4) and (5) have double-stranded and single-stranded RNA
respectively.
45

156. In an intermediary heredity, which of the following crosses gives


rise to offsprings with only two different phenotype?
a) TT x TT;
b) Tt x Tt;
c) Tt x tt;
d) TT x tt;
e) tt x tt;
157. Two organisms with a genotype of TtGg (T=height, G=color) are
mated with each other. Which of the followings the probability for the
offsprings to carry only one of the dominant characters?
a) 9/16;
b) 7/16;
c) 6/16;
d) 3/16;
e) 15/16;
158. Eucaryotic genes may not function properly when cloned into
bacteria.
Which of the followings is not a reason for this?
a) inability to excise introns;
b) destruction by native endonucleases;
c) failure of promoter to be recognised by bacterial RNA polymerase;
d) different ribosome binding sites;
e) using different genetic codes.
159. Suppose that you are trying to clone a specific human gene. After
human DNA fragments have been spliced into plasmid vectors ( shotgun
method) and the plasmids nave been exposed to recipient bacterial
cells, which of the following kind of bacteria you would not find?
a) bacteria that contain free spliced human DNA fragments;
b) bacteria containing plasmid with no human DNA in it;
c) bacteria that contain plasmid DNA with the desired human gene in it;
d) bacteria containing plasmid with irrelevant human gene in it;
e) bacteria that do not contain any plasmid;
160. While the stem of a high plant is branching, if the main stem
develops
better than the branches and becomes dominant to them, which of the
following names is given to such a branching?
a) monocasium;
b) monopodial;
c) dicasium;
d) sympodial;
e) pleiocasium.
161. If a plant is allowed to make photosynthesis in a CO2 containing
medium, to which of the following molecules does the radioactive
carbon
46

incorporate first?
a) PGA;
b) phosphoglyceraldehyde;
c) NADPH;
d) ribulose-1,5 diP;
e) pyruvic acid.
162.Which of the following Cyanophyta member cannot
fix the atmospheric nitrogen?
I. Anabaena;
II. Scenedesmus;
III. Calotrix;
IV. Nostoc.
a) I,IV;
b) III,IV;
c) I,III;
d) I,III,IV;
e) only II;
163. A mollusc sample is given to a biologist. After examining the
sample he says that it belongs to Bivalvia. Which of the followings may
be the key that makes him to reach this conclusion?
a) gills;
b) absence of radula;
e) body symmetry;
c) mantle.
164. Which of the followings occurs if an acetylcholine esterase inhibitor
is added to a synaptic cleft?
a) postsynaptic neuron is continuously stimulated;
b) postsynaptic neuron cannot be stimulated;
c) presynaptic neuron does not release neurotransmitters;
d) presynaptic neuron cannot be stimulated;
e) postsynaptic neuron membrane is hyperpolarised.
165. Which of the following impulse conducting ways is activated if our
head makes rotational movements?
a) utriculuc and sacculuscerebellumbrain cortex;
b) semicircular canals--cerebellum--brain cortex;
c) semicircular canals--brain cortex--cerebellum;
d) utriculus and sacculus--brain cortexcerebellum.
166. An extremely high amount of ammonia is found in a soil sample.
Which of the followings is the reason of this?
a) absence of nitrogen fixing fungi;
b) absence of bacteria which free the bound nitrogen;
c) competition among bacteria;
d) absence of nitrogen fixing bacteria;
e) inability of rocks to make absorbtion.

47

167. With which of the followings, convergent evolution is brought about


between two species?
a) occurence of similar mutations;
b) occurence of hybridization;
c) exposure to similar selection stresses;
d) hybridization of each of the two species with a third one;
e) genetic drift between two species.
168.Choose the correct answer. Some of the cell components are:
1. Cytoplasm;
2. Mitochondrions;
3. Ribosomes;Chromatine;
4. Cell wall;
5. Nucleus;
6. Centriole.
All the procariotic cells contain:
a) 1.,2., 4., 5.
b) 1., 2., 4., 6.
c) 1.,2., 3., 5., 7.
d) 1., 3., 4., 5.
169. The group of animals, developing by the change of the host, is:
a) Hirudinea, Ixodes, Ostrea
b) Astacus, Pediculus, Aphis
c) Ichneumon, Ascidia, Ascaris
d) Plasmodium, Fasciola, Trichinella
170. Hydrogen peroxide is droped out by drops on the slices of boiled
and unboiled potatoes. Foam appears only on slice X, but it does not
appear on slice Y. Which is the correct explanation?
a) slice x is boiled, because hydrogen peroxide affects only carbohydrates
decomposed in monosacharides.
b) slice x is boiled, because ferments in unboiled potatoes make hydrogen
peroxide inactive.
c) slice x is unboiled, because there are ferments in the plants, promoting the
decomposition of hydrogen peroxide, but they denaturate in high
temperatures.
d) slice x is unboiled, because hyrogen peroxide decomposes ferments in it.
171. You can see cross - section diagrams of hydra (1), planaria (2) and
earth- warm (3). Which conclusion is not correct?

a) those animals have no special respiration organs.

48

b) hydra and earth - warm have body cavity, but planaria do not .
c) earth - warm has cyrculatory system, but hidra and planaria do not .
d) all those animals are aerobic organisms.
172. Phenilketonuria is a recessive human genetic disorder, causing the
disturbance of aminoacids methabolism. Phenilketonuria causes :
a) incability to synthesize phenilalanile.
b) incability to decompose phenilalanile.
c) incability to absorbe phenilalanile.
d) incability to include phenilalanile in protein synthesis.
173. In which cases the transpiration in plants intensifies ?
1. The air temperature and the rate of wind increases.
2. The light intensity weekens and the weather becomes calm .
3. The water amount in soil and the air temperature decreases.
4. The air temperature decreases and the night begins.
a)
b)
c)
d)

1 and 3
1 only
4 only
2 only

174. Look at the pictures illustrating the structure of fresh watter fish
nephron (1) and marine fish nephron (2). Which one of the statements is
correct ?

a) marine fish loses more water with urine than fresh water fish.
b) the absence of glomeruli helps marine fish to get rid of spare water.
c) salts reabsorb more intensively in marine fish tubules, than in fresh watter
coiled tubules.
d) glomeruli help fresh water fish to get rid of spare water.
***Following information deals with the questions 175,176
The next items are based on the following diagram illustrating a
coleoptile tip from a plant grown in the dark and placed on blocks
of agar separated by a sheet of mica.
49

175. The tip is exposed to light as shown. The influence effect of the
light on the distribution of auxin will be such as:
a) block A will contain more than block B.
b) block B will contain more than block A.
c) both blocks will contain the same amount.
d) neither block will contain auxin.
176.After exposure to light as shown, agar blocks alone are placed back
on the cut surface on the oat plant so that a half of the (side I) is
covered by block A and an other half ( side II) is covered by block B. It
is likely that
a) neither side will grow.
b) both sides will grow equally fast.
c) side II will grow faster than side I.
d) side I will grow faster than side II.
***Following information deals with the questions 177-180;
The codes of the answers:
1. Body is divided into carapace and abdomen;
2. Head, chest, abdomen carapace - abdomen;
3. Two pairs of antenna;
4. No antennas;
5. One pair of oral members;
6. The stage of larva is presented;
7. No larval forms;
8. Three pairs of ambulatory legs;
9. Four pairs of ambulatory legs;
10. Imagoes breath only through tracheas;
177. Choose the features typical for representatives of the class
Crustacea.
a) 5, 7, 10;
b) 2, 6, 8, 10;
c) 1, 3, 6;
d) 4, 7, 9.
178. Choose the features typical for the representatives of the class
Myriapoda
a) 5, 7, 10;
b) 2, 6, 8, 10;
c) 1, 3, 6;

50

d) 4, 7, 9.
179. Choose the features typical for representatives of the class Insecta.
a) 5, 7, 10;
b) 2, 6, 8, 10;
c) 1, 3, 6;
d) 4, 7, 9.
180. Choose the features typical for representatives of the class
Arachnida.
a) 5, 7, 10;
b) 2, 6, 8, 10;
c) 1, 3, 6;
d) 4, 7, 9.
***Following information deals with the questions 181-183;
The code of the answers:
1. Have highly - developed conductive tissues;
2. Gametophyte prevail;
3. Sporophyte prevail;
4. Male gametes - spermatozoides;
5. Male gametes - spermatozoid
6. Equal - spored;
7. Hetero - spored;
8. Equal - spored or hetero - spored;
9. Have roots;
10. Havent roots.
181. Chose the features which are typical for Pteridophyta.
a) 1, 3, 5, 7, 9;
b) 1, 3, 4, 8, 9;
c) 2, 4, 6, 10.
182. Chose the features which are typical for Bryophyta.
a) 1, 3, 5, 7, 9;
b) 1, 3, 4, 8, 9;
c) 2, 4, 6, 10.
183. Chose the features which are typical for Spermatophyta.
a) 1, 3, 5, 7, 9;
b) 1, 3, 4, 8, 9;
c) 2, 4, 6, 10.
***Following information deals with the questions 184,185;
The code of the answers:
1. Have only pulmonary breathing;
2. Poikilotherma amniotes;
3. Have urinary bladder;
51

4.
5.
6.
7.
8.
9.

Sacrete the products of netrogenous metabolism chiefly in a form of acid;


Sacrete the products of netrogenous metabolism chiefly in a form of urea;
Have mesohepheic kidneys;
Molting is typical for imagoes;
Larva stage is present;
Three - chambered heart;

184. Using the code denote which features are tupical for Amphibia
a) 2, 3, 6, 7, 9;
b) 3, 5, 6, 8, 9;
c) 1, 2, 3, 4, 7, 9;
d) 1, 4, 5, 6, 8, 9;
185. Using the code denote which features are tupical for Reptilia.
a) 2, 3, 6, 7, 9;
b) 3, 5, 6, 8, 9;
c) 1, 2, 3, 4, 7, 9;
d) 1, 4, 5, 6, 8, 9;
***Following information deals with the questions 186-188;
Codes of the answers:
1. Trees bushes, herbaceous plants;
2. Trees,bushes;
3. Trees, herbaceous plants;
4. Haploid endosperm (n);
5. Diploid endosperm (2n);
6. Triploid endosperm (3n);
7. Male gametes - spermatozoons;
8. Male gametes - spermatozoids;
9. Male gametes - spermatozoons or sperma;
10. Insemination isnt connected with water;
11. Insemination is connected with water;
12. Insemination of ones is connected with water, while of others is not;
13. Vegetative organs have vessels;
14. Vegetative organs have not vessels;
15. Have specialized vegetative organs (tubers, rhizomes, bulbs).
186. Chose the features typical for Angiospermatophyta
a) 1, 6, 8, 11, 13, 15;
b) 3, 8, 11, 13, 15;
c) 2, 4, 9, 12, 13;
d) 1, 6, 7, 10, 13, 15;
e) 2, 3, 8, 12, 15.
187. Chose the features typical for Gymnospermatophyta
a) 1, 6, 8, 11, 13, 15;
b) 3, 8, 11, 13, 15;
c) 2, 4, 9, 12, 13;
d) 1, 6, 7, 10, 13, 15;
52

e) 2, 3, 8, 12, 15.
188.Chose the features typical for Pteridophytes
a) 1, 6, 8, 11, 13, 15;
b) 3, 8, 11, 13, 15;
c) 2, 4, 9, 12, 13;
d) 1, 6, 7, 10, 13, 15;
e) 2, 3, 8, 12, 15.
***Following information deals with the questions 189-191;
The code of the answers:
1. Autotroph;
2. Heterotroph;
3. They have chlorophyllin protoplasm;
4. They have chloroplasts;
5. They have mitochondrions;
6. They have a nucleus;
7. Genetic material is situated in cytoplasm;
8. They have cytoplasmic membrane;
9. They have cellulose membrane;
10. They can move;
11. They cant move.
189. Denote, which processes of vital activity and peculiarities of
structure are characteristic for cells of animals.
a) 2, 5, 6 ,8, 10;
b) 1, 4, 5, 6, 8, 9, 11;
c) 2, 5, 6, 8, 9, 11;
d) 1, 2, 7, 8, 9, 10, 11;
e) 1, 3, 7, 8, 9, 10, 11.
190. Denote, which processes of vital activity and peculiarities of
structure are characteristic for cells of fungi
a) 2, 5, 6 ,8, 10;
b) 1, 4, 5, 6, 8, 9, 11;
c) 2, 5, 6, 8, 9, 11;
d) 1, 2, 7, 8, 9, 10, 11;
e) 1, 3, 7, 8, 9, 10, 11.
191. Denote, which processes of vital activity and peculiarities of
structure are characteristic for cells of bacterium.
a) 2, 5, 6 ,8, 10;
b) 1, 4, 5, 6, 8, 9, 11;
c) 2, 5, 6, 8, 9, 11;
d) 1, 2, 7, 8, 9, 10, 11;
e) 1, 3, 7, 8, 9, 10, 11.
***Following information deals with the questions 192,193;

53

The code of the answers:


1. Diameter 25 nm;
2. Diameter 8-10 nm;
3. Diameter 7 nm;
4. Tubular;
5. Consists of various proteins, e.g. karatine;
6. Tubes consist of 4-5 protofilaments;
7. Consists of molecules of tubuline, which is a protein;
8. Consists mainly of actin;
9. Frms horn substance death of cells;
10. Eementary units from which centrioles, nucleous spindles, flagella and
cilia.
192. The eucaryotic cell contains filaments and tubular structures which
are termed cytoskeleton. Please math the microtubuli with the
corresponding statements numbered above
a) 1, 4, 7, 10;
b) 3, 6, 8;
c) 2, 5, 6, 9;
d) 3, 8.
193. Please math the microfilaments with the corresponding
statements as numbered above.
a) 1, 4, 7, 10;
b) 3, 6, 8;
c) 2, 5, 6, 9;
d) 3, 8.
***Following information deals with the questions 194-196;
The codes of answers:
1. Chemical structure derived from trytptophane;
2. Accelerate growth, germination and development of blossoms;
3. Accelerate the maturation of fruit and drop of foliage;
4. Accelerate growth in height;
5. Inhibits (slows down) the rate of metabolism and development;
6. These are derivates of adenine;
7. Induce cell division in young cells;
8. These are grouped under isoprenoid hormones.
194. Phytohormones influence the growth rate and the development of
plants.
Please match cytokinines with the corresponding chemical structure
and functions.
a) 1, 4;
b) 2, 8;
c) 7, 6;
d) 5, 8;
e) 1, 3.

54

195. Please match auxines with the corresponding chemical structure


and functions.
a) 1, 4;
b) 2, 8;
c) 7, 6;
d) 5, 8;
e) 1, 3.
196. Please match gibberellines with the corresponding chemical
structure and functions.
a) 1, 4;
b) 2, 8;
c) 7, 6;
d) 5, 8;
e) 1, 3.
***Following information deals with the questions 197-200;
Figures I-IV show different kinds of oral tools common with insects.

197. Which of these tools is specific for butterflies?


a) I
b) II
c) III
d) IV
198. Which of these tools is specific for honey bee (female worker)?;
a) I
b) II
c) III
d) IV
199. Which of these tools is specific for cockroach?
a) I
b) II
c) III
d) IV
55

200. Which of these tools is specific for mosquito(female)?


a) I
b) II
c) III
d) IV
***Following information deals with the questions 201-204
Scetch diagrams A-D in which you match the following human hereditary
diseases (single gene conditions) with corresponding terms: autosomal, xlinked, dominant and recessive as listed below.

201. Which of the trees correspond to inheritance of the skeletal


deformity eg. short fingers (autosomal dominant gene)
a) A
b) B
c) C
d) D
202. Which of the trees correspond to inheritance of the phenylketonuria
and sickle cell anemia (autosomal recessive gene)
a) A
b) B
c) C
d) D
203. Which of the trees correspond to inheritance of the nystagmism (Xlinked dominant gene)
a) A
b) B
c) C
d) D
56

204. Which of the trees correspond to inheritance of the red-green


colour blindness and haemophilias (X-linked recessive gene)
a) A
b) B
c) C
d) D.

57

INTERNATIONAL BIOLOGY OLYMPIAD


PRACTICAL PROBLEMS

1996, Artek, Ukraine















All IBO examination questions are published under the following Creative Commons license:



CC BY-NC-SA (Attribution-NonCommercial-ShareAlike) https://creativecommons.org/licenses/by-nc-sa/4.0/
The exam papers can be used freely for educational purposes as long as IBO is credited and
new creations are licensed under identical terms. No commercial use is allowed.

INTERNATIONAL BIOLOGY OLYMPIAD


THEORY PROBLEMS
1995, Bangkok, Thailand















All IBO examination questions are published under the following Creative Commons license:



CC BY-NC-SA (Attribution-NonCommercial-ShareAlike) https://creativecommons.org/licenses/by-nc-sa/4.0/
The exam papers can be used freely for educational purposes as long as IBO is credited and
new creations are licensed under identical terms. No commercial use is allowed.

VI INTERNATIONAL
BIOLOGY
OLYMPIAD

1 . Which characteristics are found in club- mo s s


(Eq ui se tum ) !

(Ly co p o d i um )

b ut

no t

in

horsetail

The codes of the answers:


1 . spores have elaters;
2. photosynthesis occurs in microphylls;
3. sporophylls form strobilus;
4. microphylls are arranged in a whorl. Answers:
a) 1, 2; b) 2, 3; c) 2, 4; d) 3, 4.
2. Which is the first group of organisms that could successfully colonize a newly formed volcanic
island?
a) ferns;

b) lichen; c) liverwort; d) algae.

3. A population of mice originally inhabiting the entire area shown


in the figure has become separated into two populations, A and B, by
a new highway. If the environment inhabited by population A
undergoes severe changes and the environment of population does
not. The rate of evolution of population A will probably be:
a) initially slower than population B;
b) initially faster than population B;
c) equal to population B;
d) slower at first then faster than population B.

4. The concentration of ions in the sap of vessels of a tomato


plant is investigated at three places (see the results in the
figure). The difference in concentration of ions between vein
and stem is partly attributed to:
a) evaporation of water from the stomata;
b) capillary force of the xylem vessels;
c) intake of ions by leaf cells;
d) intake of water by leaf cells.
5 . Wha t i s t h e mo st i mpo r t a nt f un ctio n o f g ly co ly si s in a ero bic cel l s?
a) to obtain fat from glucose;
b) to obtain energy from glucose step by step;
c) to allow carbohydrates to enter the Krebs cycle;
d) ability to divide the glucose molecule into two pieces.

6 . T he f re qu en cie s o f re c o mb i na t io n betw ee n g e ne s ( lo ci ) a , b, c, d, e a nd f li n ke d o n t he
sa me c hro mo so me a re: ( a - c) 2 ,5 %, ( f - d) 8 ,5 %, ( b- d) 4 ,5 %, ( d- e) 4 %, ( c- e) 9 ,5 %, ( a - b)
2 0 ,5 %, ( f - a ) 7 ,5 %. T he o rde r o f t he se g e ne s ( lo ci) i s:
a) a, c, d, e, f, b; b) b, c, e, f, d, ; ) a, c, f, e, d, b; d) b, e, f, c, a, d.

7 . Cel l s o f t h e a d re na l co r t ex pro d uce ho r mo ne s w ho se str uc tu re i s si mi l a r t o tha t o f:


a) hemoglobin; b) cholesterol; c) tyrosine; d) adrenalin.

8 . In g ree n pla nt s, w h ic h ev ent ca n co n ti nu e i n a l l fo u r co nd itio n s sho w n be lo w ?


a)increasing net photosynthesis;
b) water absorption;
c) respiration;
d) transpiration;
e) guttation.
9 . Wha t ty pe o f be ha v io r is sho w n w he n pa r en t s h err ing g ul l g iv e a n a la r m ca l l a n d t he
y o ung b ir ds re spo n d by hi di ng ?
a) imprinting;
b) conditioned reflex;
c) reaction to a sign stimulus
d) displacement activity.
1 0 . Wh ic h c ha ra ct er i st ic s bel o ng t o in s ect s?
T he co de s o f t he a n sw e rs:
1 . a do r sa l ro p e la dd er ne rv e co r d;
2 . ma l pig h ia n t u bu le s;
3 . o pe n c irc ula t o ry sy st e m;
4 . g a s e xc ha ng e v ia a t ra c hea sy st e m.
Answers:
a) 1 , 2 ; b ) 2 , 3; ) 1, 2, 4 ; d) 2, 3, 4 .
1 1 . In w hic h a ni ma l ce ll s w o ul d tfe Go lg i a ppa ra t u s b e mo st a b un da nt?
a) voluntary7 muscle cells;

b) red blood cells; c) gland cells; d) ovums.

12. In the diagram, the background squares represent


environmental factors (space, temperature, etc.), and the
irregular polygons enclosing a set of factors represent the
ecological niches of species I, II, III, and IV. Which species is
in danger of elimination, if the resources are limiting?
Note: Each niche is continuous.
a) I; b) II; ) III; d) IV
13. To determine that a green plant releases C0 2 during
respiration, what is necessary in the experiment?
a) using a plant with many leaves;

b) doing the experiment in the dark;

c) submerging the plant in water;

d) using a young plant.

14. Which of the following is an example of active transport?


a) chloride exchange between red blood cells and plasma;
b) sodium reabsorption in the distal tubules of the kidney;
c) movement of oxygen from pulmonary alveoli into blood;
d) oxygen movement within a muscle fiber.

15. The graph shows the relationship between the yield of a crop
and the quantity of positive ions used to fertilize a field. A field is
fertilized with 20 kg/ha of the positive ions and 20 kg/ha of the
negative ions. Are the cations and anions under these conditions
limiting factors for the yield?
a) no, neither of them;
b) only the positive ions;
c) only the negative ions;
d) yes, both of them.
16. Incipient plasmolysis is the moment when:
a) turgor pressure of the cell equals zero;
b) the protoplast completely shrinks away from the cell wall;
c) the cell volume is at a maximum;
d) the cell wall can stretch no further.

17. Based on the figure above, identify plant parts of the same
generation of the life cycle.
a) 111 and VI, I and VI;
) III and V, III and VI;

b) I and V, II and VI;


d) II and VII, IV and VIII.

1 8 . A co nd it io n nec es sa ry f o r s pec ia ti o n is :
a) a high rate of gene mutation;
b) geographical separation of populations;
c) separation of a very small group of individuals from the initial large population;
d) behavioural, geographical, genetic or other barriers preventing gene flow between populations.
1 9 . T he g ro u p o f a na mn io t es is:
a) pigeon, salamander, marsupial;

b) dolphin, seahorse, seal;

c) salmon, toad, skate;

d) raven, woodpecker, newt.

2 0 . In a g e ne po o l w it h e q ua l pro po rt io n s o f a do mi n a nt a n d a r ece s siv e a ll ele s, co mp l ete


se lect io n a g a i ns t t h e re ces s iv e ph eno ty pe i n ea ch g e ne ra tio n w o u ld :
a) make little difference to the proportions of the genotypes;
b) decrease the proportion of the recessive genotype;
c) lead to the extinction of the recessive alleles;
d) increase the proportion of heterozygotes.

2 1 . Wh ic h o f t he f o llo w i ng cha ra ct er s co u ld be fo u nd i n s ea a n e mo ne s a n d so me spo ng e s?


T he co de s o f t he a n sw e rs:
1 . ps eu do co elo m;
2 . int ra ce ll ula r d ig e st io n;
3 . ra d ia l sy mme t ry ;
4 . g a stro v a sc ula r ca v it y .
Answers: a) 1, 2; b) 2, 3; c) 3, 4;

d) 1, 4 .

2 2 . Wh ic h o f t he f o llo w i ng st a t e me nt s i s no t co rre ct?


a) phosphorylation of ADP occurs in the thylakoid membrane;
b) ATP is formed as protons diffuse through ATP synthesis;
c) ATP is consumed during the dark reaction;
d) NADPH and ATP are produced in photosystem II.
2 3 . Wh ic h o f t he f o llo w i n g is o b se rv e d w hen t h e g ro w th ra te o f the p o pu la tio n e q ua l s
zero ?
a) the population is increasing and a strong competition for food and shelter is expected;
b) the population is increasing and high parasitic and predatory activities are expected;
c) the population is decreasing because of the accumulation of toxic waste;
d) the population is near its carrying capacity.
2 4 . A mo u se w a s a l lo w ed t o br ea t h e a i r co nta i n in g a pa rt ic ula r i so to pe o f o xy g en
the mo us e t h e " la bel ed " o xy g e n a t o ms f ir st sh o w ed u p i n:
a) pyruvate;

18

02. In

b) carbon dioxide; c) acetyl-CoA; d) water.

2 5 . Wh ic h a b io t i c f a ct o r s li mit t he di str i but io n o f life in t he o c ea n , b ut do no t us ua l ly


li mit t he d ist ri b ut io n o f lif e o n la n d?
T he co de s o f t he a n sw e rs:
1 . mi n e ra l s;
3 . nit ro g e n ;
2 . lig ht;
4 . o xy g en.
Answers:
a) 1, 3; b) 1, 4;
c) 2, 3;
d) 2, 4.
2 6 . In hig he r p la nt s, t he e v o lut io n o f t he s po ro p hy te ha s c lea r ly do mi n a te d o v e r tha t o f
the g a meto p hy t e ( a c co r di ng t o siz e, a na to mi c co mp l e xity a nd d ura t io n w ith i n th e p la nt ' s
life cy c le). T he pr i ma ry rea so n f o r th i s do mi na nce i s t ha t th e s po ro p h y te:
a) may reproduce vegetatively;
c) has a well-developed conducting tissue;

b) has a well-developed parenchyma;


d) has cells that divide by mitosis.

2 7 . Wha t w o u ld be o b s erv ed w he n a g ra zer is r e mo v e d fro m t he e co sy s te m o f a na t ura l


g ra s sla n d?
T he co de s o f t he a n sw e rs:
1 . a n i nc rea se i n t he int en sit y o f p la nt co mp e tit io n;
2 . a dec rea se in t he i nt e ns it y o f p la nt co mp etit io n ;
3 . a n i nc rea se i n t he v a rie t y o f pla nt s pec ie s;
4 . a dec rea se in t he v a rie t y o f pla nt sp ec ie s.
Answers:
a) 1, 3; b) 15 4; c) 2, 3; d) 2, 4.

2 8 . In w hic h o f t he f o llo w i ng ev e nt s i s c ro ss ing - o v er l i key t o o cc ur?


a) formation of spermatogonia;
b) formation of spores in a fern;
c) formation of egg in a liverwort archegonium;
d) formation of a second plant from a strawberry stolon.
2 9 . Wha t i s t h e i mme d ia t e so u rce o f t he en erg y us ed t o ma ke mo st o f t he A TP i n a n i ma l
cel ls?
a) the transfer of phosphate groups from glucose breakdown products to ADP;
b) the movement of hydrogen ions through a specific membrane;
c) the splitting of glucose into two molecules of pyruvic acid;
d) the movement of electrons along the electron transport chain.
3 0 . In a s ee d, t h e f o o d st o r a g e t i ss ue s fo r th e e mb ry o a re:
a) haploid in gymnosperms, triploid in angiosperms;
b) diploid in gymnosperms, triploid in angiosperms;
c) diploid in gymnosperms, pentaploid in angiosperms;
d) haploid in gymnosperms, diploid in angiosperms.
31. Two cylinders P and Q are cut from a potato. P is placed for 1 hour in distilled water, and Q is placed
for 1 hour in a salt solution with an osmotic value which is identical to the average value of cell sap of the
potato cells. Determine whether the treated cylinders match their original holes in the potato.
a) P does not match, but Q does:

b) P does not match and neither does Q;

) P matches exactly and so does Q;

d) P matches exactly, but Q does not.

32. Select characteristics specific for class Mammalia only.


The codes of the answers:
1. 4-chambered heart;
2. sweat glands;
3. diaphragm;
4. homeothermy;
a) 1, 2, 4, 5; b) 3, 6, 7, 8;

5. pinna;
6. scrotum;
7. hair;
8. viviparity. Answers:
c) 2, 3, 5, 6, 7; d) 1, 4, 6, 7, 8.

33. A cross between two types of white-flowered sweet peas produced all purple-flowered peas in Fr 382
purple-flowered and 269 white-flowered peas were observed in F2. These numbers are consistent with the
9/7 ratio. If the purple F { were crossed to one of the parental types, what proportion of white-flowered peas
would you expect among the progeny?
a)l;

b)0,75;

c) 0,5; d) 0,25;

e) 0.

34. One of the negative consequences of the overuse of antibiotics is:


a) adaptation of the person undergoing treatment to increasing concentrations of the drug;
b) stimulation of the production of antibodies;
c) selection of antibiotic-resistant bacterial strains;
d) increased frequency of mutations, eventually causing cancer.
35. Which characteristics do sunflowers have?

36. U is inserted between the 9-th and 10-th base (counting in 5? - 3f direction) of the following mRNA:
5' GCUAUGCGCUACGAUAGCUAGGAAGC 3f and when it is translated into a peptide, the length of the
peptide chain is*:
a) 4;

b)5;

c) 8;

d) 9. *Use genetic code table.

37. The F t genotypes resulting from a cross between a drone honeybee and a 1 queen honeybee are
males (AB, Ab, aB, ab) and females (AaBb, Aabb, aa Bb, aabb). What are the genotypes of the
parents?
a) aaBb x Ab; b) AaBb x ab; c) Aabb x aB;

d) AaBb x Ab.

38. The following graph represents the probability of capture


of a wood pigeon (Columba palumbus) by a goshawk (Accipiter
gentilis) as a function of the size of the flock. Which of the
above propositions are correct?
The codes of the answers:
1. a solitary wood pigeon has less chance of being captured by
a goshawk than a pigeon in a flock;
2. the goshawks are less successful when they attack larger
flocks of wood pigeons;
3. the goshawks attack only solitary wood pigeons;
4. the per cent attack success is inversely proportional to the
number of pigeons in the flock. Answers:
a) 1 , 3 ; b) 1 , 4 ; c) 2, 3; d) 2, 4.

39. Which is the typical characteristic of an Old World monkey?


a) having a flat nose;

b) lacking a prehensile tail;

c) always having a long tail;

d) exclusively ground dwelling.

40. The hypothesis postulated by A.Oparin and experimentally tested by S.Miller suggests that:
a) the primitive atmosphere contained molecular oxygen;
b) the primitive oceans contained high concentrations of proteins and nucleic acids;
c) bacteria appeared on the earth 3,5 x 109 years ago;
d) organic molecules could have been formed without life.
41. The chief role of ATP in neurotransmission is to:
a) inhibit transport of Na" and K" across the membrane;
b) induce an action potential;
c) increase an action potential when it is already formed;
d) maintain the resting potential.
42. The above nucleotide chain is:
a) DNA; b) mRNA; c) tRNA;

d) rRNA.

43. If frog tadpoles receive insufficient iodide from food and the surrounding water medium, which of the
following may occur?
The codes of the answers:
1. enlargement of thyroid gland;
2. over-secretion of TSH;
3. growth stimulation;
4. exhibition of cretinism;
5. remaining in larval stage;
6. enlargement of the pituitary gland. Answers:
a) 1, 2 , 3 ; b) 3 , 4 , 6 ; c) 2, 4, 6; d) 1, 2, 5.
44. Which of the following is typical for both gymnosperms (Pinophyta)
(Magnoliophyta)?
a) sporophylls differentiating into a carpel and a stigma;
b) haploid endosperm and vascular tissues with tracheids;
c) heterospory and nonflagellated sperm (male gamete);
d) isogamy and wind pollination.

and angiosperms

45. Which combination of the following human gametes will produce a Down syndrome male individual?
The codes of the answers:
1. 23+X;
2. 21+Y;
3. 22+XX;
Answers:
a) 1 and 2; b) 1 and 3; ) 1 and 4;

4. 22+Y.
d) 2 and 3;

e) 3 and 4.

46. If A in the graph represents a population of hawks in a community,


then what would most likely be represented by B?
a) a population of the hawks' predators;
b) a population with which the hawks have a mutualistic relationship;
c) variation in the numbers of producers in that
d) a population which is the hawks' prey.
47.Which of the following cannot reproduce asexually?

48. One locus has 5 alleles: A 1 ,A 2 ,... A 5 . How many different genotypes can exist in a population if
the dominance hierarchy of these alleles is A 1 > A 2 > A 3 > A 4 > A 5 ?
a) 5; b) 10; c) 15; d) 32.
49. An average of 50 yeast cells per unit area was observed under the microscope. After 4 hours
the liquid culture was diluted 10 times. Again a microscopic slide was prepared under the same
conditions as before. An average of 80 cells per unit area was observed this time. What was the
average time between cell divisions?
a) 1/4 hour; b) 1/2 hour; ) 1 hour; d) 2 hours.
50. Consider the pedigree below. If IV-1 is male and IV-2 is female, which of the following
statements is correct?

a)

the probability that IV-1 would have both AD and SLR abnormalities is 1/8;

b) the probability that IV-2 would have both AD and SLR abnormalities is 1/4;
c) the probability that IV-1 would manifest AD abnomiality but not the SLR abnoniiality is 1/8;
d) the probability that IV-2 would manifest AD abnormality but not the SLR abnomiality is 1/8.

51. Which are the possible conditions that could lead to serious hypoglycemia (low blood glucose
level) and unconsciousness?
The codes of the answers:
1. type I diabetic patients (insufficient B-cells) who receive an insulin injection several hours
before a meal;
2. type II diabetic patients (non-functional insulin receptors) who receive an ex cessive insulin
injection;
3. patients with a tumor of the islets of Langerhans who receive an acute injec tion of insulin;
4.injection of insulin to a normal subject after heavy exercise.
Answers:
a) 1 , 3 ;

b)l,4;

) 1, 2, 3; d) 2, 3, 4 .

52. Through how many of membranes would a molecule have to pass from the interior of a chloroplast
thylakoid to the mitochondrial matrix?
a)3;

b)5;

c) 7;

d) 9.

53. Substances can be transported across a membrane against their concentration gradient because:
a) some membrane proteins are ATP-dependent carrier molecules;
b) some membrane proteins act as channels through which specific molecules can enter the cell;
c) the lipid bilayer is permeable to numerous small molecules;
d) the lipid bilayer is hydrophobic.
54. Of the following modes of inheritance, which one could
describe the genetic character appearing in the above pedigree?
The codes of the answers:
1. autosomal dominant;
2. autosomal recessive;
3. sex-linked dominant;
4. sex-linked recessive. Answers:
a) 1; b) 2; ) 1 or 3; d ) 2 o r 3 ;
e) 2 or 4.
55. Which of the following is true for RNA?
a) G + = A + U;

b) G + = + U;

c) G + > A + U;

d) none of the above.

56. Which of the following numbers (lines) correctly matches stimuli specific for receptor cells , and C?

57. If the following DNA is transcribed in the direction shown. The RNA product will be:

a) 5* U G G G A A U G 3'; b ) 5 ' G C A U U C G C C G A 3 ' ;


c) 5' G U A A G G G U 3'; d) 5' A G G U U G 3'.

58. A suitable vector for inserting DNA into the genome of a human cell is:
a) T-plasmid; b) phage;

c) retrovirus;

d) all of the above.

59. Touching the mantle of the siphon of the seahare (Ap ly sia , phylum Mo l lu sca ) , normally
triggers a reflex that protects the mantle by withdrawing it. If the mantle is touched repeatedly,
the withdrawal response becomes progressively weaker. This type of behaviour is called:
a) habitation;

b) a conditioned reflex;

c) trial and error;

d) a chain of reflex.

60. The graph below depicts changes in


the population growth rate of the
Kaibab deer. About how many deer
could this particular environment have
supported in 1930 without some of
them starving to death?

a) 12000;
100000

b) 35000;

) 50000;

d)

61. A given fungus fails to digest starch in a certain culture medium. What are possible causes of
this lack of digestion?
The codes of the answers:
1. this fungus contains no amylase;
2. the amylase in the fungal mycelium is not secreted;
3. there is some substance interfering with starch digestion by the fungus;
4. the only respiratory substrate for this fungus is carbohydrate.
Answers:
a) 1 , 2 ; b ) 3 , 4; ) 1, 2, 3; d) 1, 2, 3, 4 .
62. The figures I-IV illustrate transportation of substances and ions through the cell membranes.
Which of the following statements is correct?
a ) there i s diffusion in all figures;
b) there is active transport in all figures;
c) there is active transport in fig. II and III and
passive transport in fig. I and IV;
d) there is osmosis in fig. I, II and IV;
e) there is active transport in fig. Ill and passive
transport in fig. I, II, and IV.

6 3 . Wh ic h o f t he f o llo w i ng bel o ng t o di co t s (Magnoliopsida)?


a) banana, coconut, cucumber;
b) watermelon, cabbage, eggplant;
c) pineapple, onion, asparagus;
d) poppy, hemp, agave.

6 4 . Wh ic h o f t he f o llo w i ng is no t i mp o rta nt fo r mi g ra ti ng b ir ds i n f in di ng a n d
dete r mi n i ng ro ut e s?
a) auditory stimulation;
b) infrared sensitivity;
c) rotational force of the Earth;
d) using the stars as a compass.

Fo ur ma j o r r ep ro d uc t i v e ho r mo ne s me a s ur ed fro m blo o d s er u m o f a w o ma n d u ri ng a
no r ma l me n st r ua l cy cl e a re s ho w n in t he f ig u r e. If A i s F SH , w ha t a r e , a n d D? W hi ch
is t he mo st s uit a bl e c o nd it io n t o sto re see d s o f mo st tro pi ca l p la nt s so tha t t hey ca n
re ma i n v ia bl e f o r t he lo ng e st t i me ?
65.

a) in an ordinary refrigerator at 5C;


b) in a chamber at 5C with 10% oxygen;
c) in a chamber at 5C with reduced pressure;
d) in a chamber at 30C with humidity maintained at 20%.
6 7 . T he a bo v e da t a s ho w ba ct e ria l g ro w th i n v a rio us me d ia (S. C.M . - si mp l e c ult ur e
me d i u m, a nd U, V, X, Y, Z - re pr es en t d iffe re n t ma te ria ls a d de d t o t he me di u m). Wh ic h
ma ter ia l ca n no t t he ba c t eria sy nt h e siz e?
a)U; b)V; c)X; d)Y; e)Z.

6 8 . Wh ic h o f t he f o llo w i ng a re no t t h e c ha ra ct er s o f xe ro p hy t ic pla nt s?
T he co de s o f t he a n sw e rs:
1 . sho rt ste m;
2 . sto ma ta p re se nt o n bo t h s i de s o f l ea f s ur fa ce s;
3 . enl a rg e me nt o f lea f su r f a ce;
4 . hy po de r ma pr e se nt .
Answers:
a) 1,2; b)2,3; c) 3, 4; d) 1, 4.

6 9 . Wh ic h co do n ca n be mut a t e d by o n e ba se cha ng e t o a "no n se n se" co do n? (Use genetic


code table on the p. 147).
a)GCC; b) GAA;

c) GCA;

d) GGC

7 0 . Wh ic h e nzy me i s no t no r ma lly f o u nd in h u ma n ?
a) DNA-polymerase;

b) hexokinase;

c) chitinase;

d) ATP-synthetase.

7 1 . Wha t i s t h e a n t ico do n c o rre spo n di ng t o t he co do n 5 ! GU A 3 * ?


a) 5' CAU 3'; b) 5' UTC 3;

c) 5' UAC 3';

d) 5' AUG 3.

7 2 . T he me t ho d s o f a g ri c ult ur e u se d by hu ma n ha v e crea te d se rio u s in sec t pro bl e ms


chi efly be ca u s e t h es e p ra ct i ce s:
a) increase the rate of deforestation;
b) provide concentrated areas of food for insects;
c) increase the effectiveness of insecticides over a long period of time;
d) encourage insect resistance to their natural enemies.
7 3 . Wh ic h a re t he d if f e re n ces b et w ee n a h ig h er pla nt cel l a n d a n a ni ma l cel l a nd th ei r
res pe ctiv e me c ha n i s m o f cel l d iv i s io n?
T he co de s o f t he a n sw e rs:
1 . div is io n o f ce nt ro me re;
3 . f u nct io n o f s pi n d le;
2 . div is io n o f cy t o pla s m;
4 . pr es en ce o f c ent rio le s.
Answers:
a) 1 , 2 ; b) 1 , 4 ; c) 2, 4; d) 3, 4.
74. Which of the following is the common characteristic of reptiles, birds and mammals?
a) teeth;

b) diaphragm;

c) oxygenated blood is totally separated from deoxygenated blood;


d) kidneys are in the type of metanephros.
75. Plastides can develop from one another. Which figure is correct?

76. Which DNA is a substrate for DNA-polymerase?

77. Messenger RNA was transcribed in vitro from a double-stranded DNA molecule, which was later
separated into single strands. For each strand of the DNA, the base ratio was analyzed and compared with
that of mRNA. On the basis of the data given in the table, which strand of the double-stranded DNA served
as the template for the mRNA synthesis?

a) DNA-l; b) DNA-2;

c) DNA-3;

d) DNA-4.

78. Which procedure would be used in a choice chamber to decide whether a \f response of
animals to two different light intensities is taxis or kinesis?
a) record the pathway of each animal;
b) record the velocity of the animal movement;
c) count, at intervals, the number of animals in each chamber;
d) count, at intervals, the number of moving and stationary animals.

79. Which substances are found in pancreatic juice?


The codes of the answers:
1. bicarbonate;
2. secretin;
3. bile salts;
4. pepsinogen;
5. lipase.
Answers:
a) 1 , 2 ; b ) l , 5 ;

c) 2, 3, 5;

d) 3, 4, 5 .

80. To determine their need for oxygen, three types of bacteria were grown in three different
tubes. The regions of bacterial growth are shown in the figures above. Which is the correct
classification of the bacterial types.

81. Which part of the nephron is impermeable to water?


a) I:

b)II

c)III:

d) IV:

e) V

8 2 . A do c to r ha s blo o d g r o up 0 , R hf a n d hi s w if e A, P h + . In a n e me rg e ncy sit ua t io n t he


do cto r q u ic kly inv e st ig a t ed t he b lo o d g ro u p o f a pa t ie nt. H i s o w n s er u m a g g l uti na t ed w i th
the pa tie nt ' s blo o d, b ut hi s w if e' s ser u m d i d n o t. Wha t i s the pa t ie nt' s b lo o d g ro u p, a nd is
it po s si bl e t o co nc lu de a bo ut h i s R h- fa cto r to o ?

8 3 . Wh ic h

ho r mo ne s

i nc rea s e

and

dec rea se

b lo o d

g l uco se

l ev e ls,

r es pe ctiv ely ?

8 4 . Wh ic h o f t he f o llo w i ng a ni ma l p hy la po s se ss g i a nt a xo n sy ste ms ?
a)
b)
c)
d)

Coelenterata, Plathelminthes. Nematoda;


Plathelminthes, Nematoda, Annelida;
Annelida, Arthropoda, Mollusca;
Arthropoda, Mollasca, Chordata.

8 5 . Fro m t he f ig ure g iv e n b elo w , c ho o s e th e a nsw e r co d es


t o fil l i n t he bla n ks.
T he co de s o f t he a n sw e rs:
1 . Po lyp o d io p h yta ;
2 . Ma g no lio p hy ta ;
3 . Pino p hy ta ;
4 . pri ma ry g ro w t h;
5 . seco n da ry g ro w t h. An s w ers:
I.
The figure is a plant stem in the division (s)
II.
The stage of growth is (are)
8 6 . Fro m t he f ig u re o f a mus hro o m, cho o se t he co de s o f th e
a nsw e rs t o a n sw er t he f o llo w i ng q ue st io n s.
T he co de s o f t he a n sw e rs:
1 . ba si di u m;
2 . ba s i d io s po r e;
3 . my ce li u m f ro m ba s i dio spo re o r pr i ma ry
my ce li u m;
4 . my ce li u m f ro m st a l k o r sec o n da ry
my ce li u m;
5 . zy g o te.
Answers:
I. In which part of the mushroom does meiosis occur?
II. Where is the diploid zygote formed?
III. Which part of themushroom is dicaryotic?

87. Select plant numbers from the diagram below and write a correct order of the phases of see d
germination anddevelopment of young plants.

88. Put X in the appropriate place to indicate whether each statement concerning photosynthesis
in plant is true or false.
I.

Photolysis occurs in photosystem I

II. Oxygen is released


III. NADH is formed

89. Fill in each blank with one correct answer code.

The codes of the answers:


1. root cap;
2. procambium;
3. ground meristem;
4. primary xylem;
5. photosynthesis;
6. reproduction;

7.
8,
9.
10.
11.
12.
13.

microspore;
parenchyma;
root hairs;
protoderm;
transpiration;
aerenchyma;
respiration.

90. Fill in the blanks with answer codes to indicate the main edible parts of the following fruits.
The codes of the answers:
1. exocarp;
2. mesocarp;
3. endocarp;
4. receptacle;
Answers:
I.Strawberry
II.Banana
III.Peanut
IV.Orange
V.Coconut

5.
6.
7.
8.

perianth;
endosperm;
cotyledon;
aril.

91. List all possible characteristics from the answer codes of the following organisms.
The codes of the answers:
1. autotrophic;
6. having nucleus;
2. heterotrophic;
7. having main genetic materials in cytoplasm;
3. chlorophyll in protoplasm;
8. having cellulose cell wall;
4. having chloroplasts;
9. having mobility;
5. having mitochondria;
10. having no mobility.
Answers:
I.Bacteria ..................
II.Blue-green algae ............. III. Fungi ............
92. The figure shows the response of net carbon
dioxide consumption rates of a single leaf to
ambient C0 2 concentration (ppm) at a light
intensity of 75% full sun.
Indicate with for true and "-" for false
statements.
I.
Plant A is a C4-type because it has a net greater CO.
consumption rate at high CO.
concentration
II.
At a C02 consumption rate equal to zero, there is no
photosynthesis and respiration for both plants A and

III. The CO. consumption rate of plant A will ultimately reach saturation as the C0 2 concentration increases further if
light intensity is held constant...
IV. C4-plants have greater light use efficiency in photosynthetic process than C 3-plants at the C02 concentration of
200 ppm
V.
At a light intensity of 100% full sun, the C02 consumption rate of plant will reach saturation at C02
concentration greater than above in the graph
9 3 . Sa tu ra t io n o f b lo o d he mo g lo bi n w i th o xy g e n is i nfl ue nc ed by th e pa rtia l pre s su re o f
o xy g e n. T h is i s inv es t i g a t ed i n f o ur o rg a n is ms: h u ma n a d ult, h u ma n fet u s, lla ma ( i n t he
An de s) a n d co w . Fo r t w o o f t he m t h e re s ult s a re s ho w n i n t he fig ur e. W ha t co u l d be t he
thre e po s si b le pa ir s o f I a nd I I ?
T he co de s o f t he a n sw e rs:

An s wer s:
I.
P o ss ib i li t y
II.
P o ss ib i li t y
III.
P o ss ib i li t y

........
.......
.....

9 4 . T he fo llo w i ng d ia g ra m sho w s t he pa thw a y o f b lo o d co a g u la tio n. W ha t a re I -V in t he


dia g ra m?
T he co de s o f t he a n sw e rs:
1 . pla t elet s;
2 . thro mb i n;
3 . thro mb o kina se ;
4 . fib ri no g e n;
5 . pro t hro mb i n;
6 . Ca 2 + ;
7. Mg2+;
8 . v ita mi n K .
9 . An s wer s:
I . ..I I I I I . .I V .V
9 5 . Wha t a re I - VI i n t he di a g ra m?
T he co de s o f t he a n sw e rs:
1.
3.
5.
7.
9.

se creto ry v e si cle s;
ri bo so me ;
mi to c ho n dr io n;
ch ro mo so me;
cent rio le;

An s wer s:
I . ... I I . ... I I I . ...

2 . mi cro t u bu le s;
4 . ly so so me;
6 . v a cuo le;
8 . Go lg i bo dy ;
1 0 . n ucl eo l u s.

I V. . .. V. ...

VI . .

9 6 . Id en tify a ll lo w er j a w s o f ma mma l s a n d no n- ma mma li a n v erte bra t es .

Answers:
I.Mammals
II.Non-mammalian vertebrates
9 7 . M a tch a ll po ss ib le a ni ma l g ro u p s i n co lu mn w ith ea c h de sc ri ptio n i n co lu mn A:
I. no c irc ula t o ry sy st e m;
I I. o pe n c irc ula t o ry sy st e m;
I II. clo se d ci rc ula t o ry sy st e m w it ho ut hea rt c ha mb ers ;
IV. hea rt w i t h si ng le - c ir cu i t cir cu la tio n;
V. hea rt w i t h 2 a t ria a nd 2 v ent r ic le s.
T he co de s o f
1 . in se ct;
2 . bir d;
3 . sha r k;
4 . pla na r ia ;

t he a n sw ers ( co l u mn B ) :
5 . cro co di le;
6 . sea ho r se;
7 . a nne li de s (g ene ra l);
8 . hy dra .

Answers:
I .............

II .........

III ..........

IV ......... V .........

9 8 . Id en tify t he f o l lo w ing a ct io n s o f th e sy mp a thet ic a n d pa ra sy mp a thet ic nerv o u s


sy ste ms .
T he co de s o f t he a n sw e rs:
1 . pre pa r e bo dy t o co pe w it h st r e ssf u l co nd itio n;
2 . di la te pu p il s;
3 . inc rea se mo t il it y o f t he int est i ne;
4 . sti mu la t e a d re na l in se c ret io n;
5 . dec rea se hea rt ra t e;
6 . sti mu la t e o xy t o c in rel e a se.
Answers:
I.
Sympathetic nervous system
................
II.
Parasympathetic nervous system
.................
99. The heron (Ardeola ibis) exhibits different types of nesting and breeding' which effect the
success of offspring. The table shows the results:

With these data it is possible to compare the efficiency in energy investment of the parent birds.
Which type of nesting (number 1, 2 or 3 from the table) has the lowest efficiency and which one
has the highest?
Answers:
I. The lowest efficiency
.....
II. The highest efficiency
......................
100. While investigating and comparing the efficiency in
energy investment of the parent birds, some conditions have
to be considered. Choose two possible conditions below.
The codes of the answers:
1. all young birds have to be about the same size during the
investigation;
2. all parent birds have to have about the same mass;
3. the nests have to be close together;
4. the food has to be at about the same distance from all the
nests;
5. the experiment shoud last no longer than one year;
6. all the parent birds shoud feed their young on the same
type of food.
Answers:

.................

101. Choose the correct number (1-5) from the given diagram
of the nitrogen cycle to match each statement (I-V).
Answers:
I.
Fixation of nitrogen by the bacteria in nodules
II.
Absorption of nitrogen compounds by the roots
III. Action of nitrifying bacteria
.....................
IV. Action of denitrifying bacteria ....................
V Action of decomposing bacteria .......................
102. Assume the genotype AABB was crossed with recessive genotype and the x offspring are
mated at random to produce F 2 offspring consisting of 22 A-B-, 5 A- bb, 5 aaB- and 4 aabb.
Estimate the recombination frequency between A
and B?
Answer:
103. Let " X " be the amount of nuclear DNA in a gamete of a diploid organism. Fill in each blank
the amount of nuclear DNA in different stages of the cell cycle of this organism. The DNA amount
per cell in the following stages of the cell cycle (example: 0,5 X , X , 2 X or 4 X )
I.Mid S =
II. G 1 =
. G 2 =
VI. Anaphase =
V. Metaphase =
IV. Prophase =

104. If the initial frequencies of genotypes 1 A 1 , 1 A 2 and A 2 A 2 are 0,04, 0,32 and 0,64,
respectively, what are the genotypic frequencies after one generation of complete selffertilization?
Answers:
I. 1 A 1 ............... II. A 1 A 2 ......... III. A 2 A 2 ...........

105.
The diagram shows genotypic frequencies during
subsequent generations in a population.
I.
What was the frequency of allele A at generation 0,
assuming a Hardy-Weinburg equilibrium at that point?
Answer: frequency of allele A is .....................
II.
What would be the frequency of aa after a very
large number of generations?
Answer: frequency of aa would be

106. In a human population at equilibrium, the frequency of I A =0,2, I B = 0,4 and i = 0,4. What
are the genotypic frequencies of blood group A, B, AB and 0?
Answers:
I.
II.

Group A
Group

III. Group AB
IV. Group 0
107. If the statement is correct for both chloroplasts and mitochondria, mark with "+". If it is
not correct for both, mark with
I. Contain proteins
II. Contain coenzymes for binding hydrogen
+

III. Contain K ions

.....
.....................
.....

IV. Lack DNA


V Can produce ATP
VI. Can produce oxygen

.....

1 0 8 . W ha t w o ul d se qu ent ia l ly ha p pe n if t he sea w a s po ll ute d w it h a la r g e nu mb er o f


o rg a n ic su b st a nce s? W rit e t he co rr ect se qu e nc e o f s uc h c ha ng e s us ing the a n sw er co de s.
T he co de s o f t he a n sw e rs:
1 . the a mo unt o f o xy g en i n w a t er dec rea se s;
2 . the ba ct eria de co mp o si ng t h e d ea d o rg a ni s ms bre ed ra p i dly ;
3 . the a mo unt o f o xy g en i n w a t er i nc rea se s;
4 . ther e i s t he g ro w t h o f b a ct er ia p ro d uc ing H 2 S;
5 . the pla n kt o n ic a lg a e r e pro du ce ra p id ly .
Answer: the correct sequence is

INTERNATIONAL BIOLOGY OLYMPIAD


THEORY PROBLEMS
(Answers)

1995, Bangkok, Thailand















All IBO examination questions are published under the following Creative Commons license:



CC BY-NC-SA (Attribution-NonCommercial-ShareAlike) https://creativecommons.org/licenses/by-nc-sa/4.0/
The exam papers can be used freely for educational purposes as long as IBO is credited and
new creations are licensed under identical terms. No commercial use is allowed.

INTERNATIONAL BIOLOGY OLYMPIAD


PRACTICAL PROBLEMS
1995, Bangkok, Thailand















All IBO examination questions are published under the following Creative Commons license:



CC BY-NC-SA (Attribution-NonCommercial-ShareAlike) https://creativecommons.org/licenses/by-nc-sa/4.0/
The exam papers can be used freely for educational purposes as long as IBO is credited and
new creations are licensed under identical terms. No commercial use is allowed.

INTERNATIONAL BIOLOGY OLYMPIAD


THEORY PROBLEMS
1994, Varna, Bulgaria















All IBO examination questions are published under the following Creative Commons license:



CC BY-NC-SA (Attribution-NonCommercial-ShareAlike) https://creativecommons.org/licenses/by-nc-sa/4.0/
The exam papers can be used freely for educational purposes as long as IBO is credited and
new creations are licensed under identical terms. No commercial use is allowed.

V INTERNATIONAL BIOLOGY OLYMPIAD

1. A scientist made several sections of a


living cell at different levels. After
viewing these sections under an electron
microscope, he drew a complex diagram
as shown on the figure. It would be
reasonable to conclude that this cell is
not:
a) photosynthetic;
b) respiratory;
c) eucaryotic;
d) a plant cell.
2. The two types of cell organelles that
transform energy are:
a) chromoplasts and leucoplasts
b) mitochondria and leucoplasts;
c) mitochondria and chloroplasts
d) mitochondria and chromoplasts.
3. A centriol is:
a) stable structure in the cytoplasm near the nucleus;
b) stable structure within the nucleus;
c) structure, appearing during the mitosis;
d) structure, which is a part of the chromosome.
4. How is it possible that cells of one and the same kind can contain different number of
plasmids:
a) it is because of the circular structure of plasmids;
b) because plasmids are able to replicate independently of chromosomes;
c) it depends on the molecular weight of plasmids;
d) because plasmids are usually smaller than chromosomes.
5. In eucaryotic cells DNA may be found in the:
a) nuclei
b) mitochondria
c) chloroplasts
d) all of these.
6. A poison that interferes with the synthesis of proteins is most likely to interfere with the
function of the:
a) cytoplasm
b) centromeres
c) ribosomes
d) vacuoles.
7. Which one of the following characteristics does not correspond to the pro pe rti es o f w a ter ?
a) bad heat conductor;
b) bad electrical insulator;
c) good solvent for majority of substances in cells;
d) it takes part in majority of chemical reactions in cells.
8. Acco rd ing t o t he f lu i d - mo sa ic mo de l o f t he cel l me mb ra n e :

a) proteins and phospholipids form a regular repeating structure;


b) the membrane is a rigid structure;
c) phospholipids form a double layer, with the polar parts facing each other;
d) proteins are fairly free to move laterally within a double layer of phospholipids.
9. Wh ic h o ne o f t h e f o l l o w ing st a t e me nt s i s no t co rre ct. Act iv e t ra n s p o rt o f a
su b sta nce :
a) can occur against the concentration gradient;
b) can occur against the electrochemical gradient;
c) can be slowed by a factor of at least two by lowering the temperature by 10C;
d) is unaffected by metabolic poisons;
e) shows saturation properties.
10 . A n a b no r ma l s pe ci me n o f so me a ni ma l sp ecie s w a s pro du ce d
by se x ua l re pro du ct io n . T he f ig ure b elo w s ho w s th e c hro mo so me s
o f a so ma t ic c el l o f t hi s o rg a n is m. O n e o f t he g a mete s tha t
co ntr i but ed t o t hi s sp e ci me n ha d a w ro ng n u mb e r o f
chro mo so me s. H o w ma ny c hro mo so me s w ere c o nta i ne d in th is
g a mete ?
a) 4
b)5
c) 6
d) 7.
11 . T hi s qu es t io n r ef er s t o t he f o l lo w ing d ia g r a m .
the S- pha se o f c el l cy c l e t h e ce ll:
a) undergoes cytokinesis;
b) undergoes meiosis;
c) replicates its DNA;
d) undergoes mitosis;
e) enters interphase.

Du ri ng

12 . T he c hro mo so me s n u mb e r i n t o ma to p la nt s is 2 4 . A to ma to ce ll u n derg o es me io si s.
T hre e o f t he re su lt i ng cel ls d eg e nera t e. T he l a st c el l i mme d ia t ely u n derg o es th ree
mi to se s w it ho ut cy t o ki ne si s. H o w ma ny ce ll n ucl ei w i ll y o u fi nd ; ho w ma ny
chro mo so me s do e s ea c h o f t h es e n uc le i co nta in :
a) 4 nuclei with 12 chromosomes each;
b) 4 nuclei with 24 chromosomes each;
c) 8 nuclei with 12 chromosomes each;
d) 8 nuclei with 24 chromosomes each.
l3 . R epa ir reg e ne ra t io n mea n s:
a) general property of the tissues and organs to renew throughout cell life and aging;
b) physiological changes that take place during the period of differentiation and specialization of cells;
c) restoration of tissues and organs after induced injury;
d) replacing of old cells with new ones.
14 . T he un i qu e pro pert ie s o f ea ch a mi no a ci d a re dete r mi n e d by it s p a rtic ula r :
a) R-group
b) amino group
c) type of the peptide bonds;
d) number of the bonds with other amino acids.
15 . A mu ta t io n t ha t c h a ng e s a n a la ni ne re s id u e i n t he int er io r o f a pr o tei n mo le cu le to
v a li ne is fo u n d t o lea d t o a lo s s o f a ctiv ity . T h e rea so n i s :
a) valine is acidic but alanine is basic;
b) alanine is necessary for a-helix formation, whereas valine is not;
c) valine occupies more space than alanine, hence the shape of the protein molecule is changed;
d) the presence of valine changes the isoelectric point of the protein.

16 . Po ly - L- Le uc in e i n a n o rg a ni c so lv e nt i s a - h eli ca l, w her ea s po ly - L- iso le uc in e i s no t.


T he r ea so n f o r t hi s be h a v io u r o f po ly - L- iso le u cin e i s:
a) the methyl group attached to the p-carbon of isoleucine sterically interferes with the a-helix formation;
b) the peptide bonds in poly-L-leucine are different;
c) isoleucine is less soluble in organic solvents than leucine;
d) the isoelectric point of isoleucine is higher.
17 . T he e nzy me s uc ci ni c d ehy dro g ena se no r ma lly ca ta ly z es a rea ctio n inv o lv i ng
su cc in ic a ci d. Ano t he r su b st a nce, ma lo nic a ci d, su ffic ie nt ly re se mb l e s t he s ucc in ic
a ci d to fo r mi ng t e mpo r a ry co mp le xe s w it h t he en zy me , a l tho ug h ma l o ni c a ci d it sel f
ca n no t be ca t a ly z ed by t he s ucc i nic d ehy dro g e na s e. I n t hi s e xa mp l e s ucc i nic a ci d a n d
ma lo ni c a c id a re, re s pe ct iv e ly :
a) a substrate, a competitive inhibitor;
b) a competitive inhibitor, a substrate;
c) a positive modulator, a negative modulator;
d) a negative modulator, a positive modulator.
18 . A p ie ce o f g ut w h ic h co nt a i n s a s ub sta nc e M a n d a n e nzy me t ha t d eg ra d es th i s
su b sta nce i n it i s i mme rse d in a co nta i ne r w it h di sti ll ed w a ter. Wh en the w a te r i s
exa mi n ed o n t he n e xt d a y it i s o b serv e d tha t it co nta in s o n ly a su b sta nce X. B a se d o n
thi s o b serv a t io n w hi ch o ne o f t h e f o l lo w ing sta te me nt s ca n no t be s pec ula t ed :
a) substance X is degraded by the enzyme M;
b) substance M is degraded in the gut;

) substance X is the constituent of substance M;


d) substance X can penetrate the gut wall;
e) molecular weight of substance M is greater than that of substance X.

19. The diagram below represents a student' s view of DNA synthesis occurring in an animal
cell. The arrows represent newly synthesized DNA.
This diagram is:
a) correct as shown;
b) incorrect because DNA synthesis in animal cells is unidirectional;
c) incorrect because DNA synthesis proceeds in 3' - 5' direction;
d) incorrect because DNA synthesis is proceeding in wrong direction on two of the strands.

20. A synthetic mRNA has only two nucleotide types; it contains five times as much U as C:
a) there are 4 possible codons
b) there are 6 possible codons;
c) there are 8 possible codons
d) there are 16 possible codons.

21. The exact replication of DNA is possible due to:


a) the genetic code
b) mitosis
c) the base-pairing rules;
d) the fact that the DNA molecules are enclosed within a nuclear membrane.
22. Which one of the following statements explains why penicillin does work as an antibiotic:
a) penicillin inhibits the nucleic acid synthesis;
b) penicillin inhibits murein synthesis, the material necessary for building of bacterial wall;
c) penicillin inhibits protein synthesis among procaryotes;
d) eucaryotes rapidly decompose penicillin.
23. A cell with eight chromosomes undergoes two mitoses and one meiosis. One of the
daughter cells is fertilized. Which one of the following figures represents the changes in the
chromosome number of the above mentioned cell.
) ;
b) ;
) ;
d) D;
) .

24. Which one the following statements refers to a human cell with 22+X chromosomes?
a) it is a fertilized egg
b) it is a cell which has undergone a meiosis;
c) it is a somatic cell
d) it is a cell which has undergone a mitosis;
e) it is a polyploid cell.
25. Which one of the following substances could be subjected to denaturation?
a) only proteins
b) only nucleic acids:
c) only lipids
d) nucleic acids and proteins.
26. Which one of the following statements is correct? Denaturation is a process of:
a) losing of the secondary and tertiary structure and destroying the primary structure;
b) losing of the secondary and tertiary structure without destroying the primary structure;
c) only losing of quaternary structure;
d) reducing of the chemical reactivity of the polymers.

27. The Krebs cycle takes place in the:


a) mitochondria;
b) cytoplasm;
c) chloroplasts;
d) nucleus.

28. The Krebs cycle is a source of:


a) only ATP
b) nucleoside triphosphates and important intermediate metabolits;
c) only glucose
d) only pyruvic acid.
29. Which of the following substances are products of aerobic respiration:
a) only C02 and H20
b) C02, H20 and ATP
c) oxalic acid
d) citric acid.
30. Which one of the following statements
activity is a way for:
a) regulation of metabolic processes
b) activation of enzyme activity;
c) competitive inhibition of enzyme activity;
d) noncompetitive inhibition of enzyme activity.

is not correct? Allosteric affect on the

enzyme

31. The main functions of the glycocalyx are:


a) structural and protective;
b) mechanosupporting and protective;
c) maintaining the environment immediately around the cell, fulfilling of the selective permeability of
compounds necessary for the cell and extracellular digestion;
d) cell contaction and carrying out of the intercellular exchange.
32. This figure is a cross section at one kind of leaf. What is the type of tissue (cell) of
number 4?
a) palisade cell;
b) parenchyma;
c) sponge cell;
d) sclereid.

33. Cells of xylem vascular


bundles are in the period of their activity:
a) dead
b) alive, only their cell membranes become woody;
c) alive, only their nucleolus disappears;
d) alive, with a big vacuole inside of the cell.

34. Some blue-green algae (Cyanobacteriae) are both autotrophic (photosynthetic) and
heterotrophic (being able to assimilate organic compounds). It has been shown by
experiments that light promotes the assimilation of organic compounds by them. Which one
of the following is the most probable explanation of this phenomenon?
a) the presence of organic compounds inhibits photosynthesis;
b) the uptake and utilization of organic compounds by blue-green algae consume no energy;
c) ATP and NADPH produced in photosynthesis can be used for the uptake and further transformation of
organic compounds;
d) blue-green algae can fix atmospheric nitrogen.
35. Which one of the following processes constitute a part of the light- dependent
photosynthetic reactions?

a) 1, 3 , 6;
b)l , 4 , 8
c ) 2, 3, 6;
d)2 , 4 , 5 ;
e) 5, 7.
36. Which of the following does not affect the number of
gas bubbles per unit of the time in the above figure?
a) volume of the glass container;
b) intensity of the light;
c) number of the leaves on the plant;
d) amount of C02 dissolved in the water;
e) temperature of the water.
37. In C 3 -photosynthesis the C0 2 acceptor is:
a) 3-phosphoglycerine aldehyde
b) pyruvic acid;
c) ribulose-1,5-diphosphate
d) ferredoxin.
38. In photosynthesis, in contrast to mitochondrial respiration:
a) water is decomposed
b) water is formed;
c) electrons are transported with the aid of cytochromes, among others;
d) participation of colorful metallocompounds;
e) more than one of the above are characteristic for photosynthesis only.
39. In the 1930th, C.B. Van Niel correctly hypothesized that oxygen gas released by plants
come from:
a) H20
b) C02
c) C6H]206
d) 03;
40. Auxins function as:
a) stimulators of leaves and fruits drop;
b) stimulators of lateral bud growth;
c) inhibitors of length growth;
d) stimulators of cell elongation and root development.

41. Besides the three factors: photoperiod, temperature and nutrition, another factor, which
also controls the flowering of plants is:
a) the rate of photophosphorylation;
b) the rate of the transport of organic substances within the plant;
c) the age of the plant;
d) the pH of soil solution.
42. Plant cells were incubated for several hours in the presence of radioactive (tritium
labeled) uridine. Afterwards cells were collected, gently homogenized and fractionated to
obtain various organelles. Radioactivity will be found chiefly in:
a) nucleolus, plastids and Golgi apparatus;
b) nucleus, nucleolus and lysosomes;
c) nucleus, ribosomes and vacuoles;
d) nucleus, nucleolus, ribosomes and chloroplasts;
e) all of the mentioned organelles will exhibit roughly the same level of radioactivity.
43. T he n u mb er o f s ee d s i n a po d i s de ter mi ne d by th e nu mb er o f :
a) pistils in the flower
b) carpels in the pistil;
c) ovules in the ovary
d) embryos in the ovular sac;
e) stigmata in the pistil.
44 . Wh ic h o f t he f o l lo w ing r ef e rs t o t he s po ro phy t e i n pla nt s :
a) diploid that produces haploid cells by mitosis;
b) diploid that produces haploid cells by meiosis;
c) haploid that produces gametes by meiosis;
d) haploid that produces gametes by mitosis.
45 . Wh en th e ro o t ca p o f a ma ize s ee dl ing i s e xc i se d, a new ro o t ca p w ill be fo r me d
w ithi n a bo u t 5 da y s . B ut i n a f ly ing s pa ce s hi p , the re i s no reg en era ti o n o f t he ro o t
ca p. W ha t i s t he rea so n ?
a) root cap formation is related to the C02 content of the atmosphere;
b) root cap formation is related to photoperiod;
c) root cap formation is related to temperature difference between day and night;
d) root cap formation is related to gravity.
46 . In h ea lt hy a du lt h u ma n s, re d blo o d c el ls a re fo r me d o nly in:
a) sternum, ribs and vertebrate bodies
b) thigh and shank bones;
c) liver;
d) spleen.
47 . As b lo o d f lo w s t h ro ug h t h e h ea rt t he v a lv e s o pe n a n d clo se a t th e co rre ct
mo me n ts . Th es e v a lv es a re f o r ce d t o o pe n by :
a) the contraction of the powerful muscles of the ventricles;
b) small muscles in the valves themselves;
c) strings attached to them called chordae tendineae;
d) the pressure of the blood itself.
48 . Wh ic h o f t he f o l lo w ing st a t e me nt s e xp la i n s the pe r si sti ng a b no r ma lly hig h lev e l o f
g lu co s e i n t he h u ma n b lo o d :
a) insufficient filtration by the kidneys;
b) excessive amount of thyroid hormones in the blood;
c) muscles in a total resting state (motor hypoactivity, hypodynamics);
d) insufficient hormone secretion by the pancreas;
e) an excessive amount of glucose absorbed by the intestine.

49 . Why th e h u ma n o rg a ni s m ca n no t be fe d o n ly by ca r bo hy d ra te s i s :
a) because the human body is composed mainly of proteins;
b) because man for his life needs many other substances, that cannot produce himself;
c) because the calorific value of the carbohydrates is not very high;
d) because the carbohydrates do not contain nitrogen and the human organism cannot assimilate it from the air.
50 . A s tu de nt a d de d s a l iv a t o st a rc h so lut io n a nd e x pect ed th e s ta rc h to be co nv ert ed
into mo no - a n d d i sa cc h a ri de s. Wh ic h o ne o f t h e fo l lo w ing i s t he be st co n dit io n fo r hi s
ex pe ri me nt ?
a) the mixture was kept at 0C;
b) the mixture was kept at 30C;
c) the mixture was boiled and then kept at 30C;
d) the mixture was kept at 70C.
51 . B a sa l me t a bo li c ra t e i s :
a) the minimal amount of energy needed to maintain life under exactly defined conditions;
b) issue of energy after obtaining nutrition - 100 g of proteins;
c) issue of energy by effort - ten knee-bends;
d) amount of energy needed for converting 1 g of lipids.
52 . M a mma l ia n s pec ie s X a n d Y ha v e si mi la r o v era l l sha pe s a n d bo dy co v er a n d
ex h ib it si mi la r lo co mo t o ry a ct iv it y . H o w ev er, in div i dua ls o f sp eci e s X a re tw ice a s
la rg e a s t ho se o f s pe cie s Y . Th e lo s s o f hea t th ro ug h t hei r su rfa ce , p e r u nit o f bo dy
ma s s, fo r t he se s pe cie s w ill be a bo ut :
a) for X twice as large as for Y
b) for X four times as large as for Y;
c) for Y twice as large as for X
d) neither of the given answers is correct.
53 . A mu s c le ha s bu ilt up a n o xy g en d ebt . W h en t he re is e no ug h o xy g en fo r a ero b i c
res p ira tio n t o re s u me , a ll o f t h e f o l lo w ing w il l o cc ur e x ce pt :
a) lactic acid will be converted into pyruvic acid;
b) 02 will be used up;
c) acetyl-CoA will be converted into CO? and H20;
d) pyruvic acid will be converted into acetyl-CoA;
e) an excess of NADPH will accumulate.
54 . E xc es s iv e co nc en t ra t io n o f t hy ro xi ne ca u s e s un co u pl i ng o f mito ch o nd ria , i. e. i n
sp ite o f t he e le ct ro ns f l o w ing a lo ng t he o x i da ti v e c ha i n no A TP i s pro du ce d. B a se d o n
thi s fa ct o ne sho u ld e x pect p eo p le w it h e xce s s iv e a ct iv ity o f t he t hy r o id to b e :
a) obese in spite of consuming small amounts of food and with body surface colder than in healthy individuals;
b) obese in spite of consuming small amounts of food and with body surface warmer than in healthy
individuals;
c) lean in spite of consuming large amounts of food and with body surface colder than in healthy individuals;
d) lean in spite of consuming large amounts of food and with body surface warmer than in healthy individuals.
55. Blood hydrostatic pressure powers the pro cess of:
a) filtration across the glomerulus;
b) reabsorption of water and dissolved substances at the venule end of capillary;
c) sodium gradient maintenance in the kidney tubules;
d) salt and glucose reabsorption in the kidney tubules;
e) Na+ reabsorption in the loop of Henle.
56 . In th e ma mma l ia n e mb r y o , t he f ir st str uc t ure to d ev e lo p re la tio n sh i p w it h t he
w a ll o f t he ut e ru s is ?
a) the amnion;
b) the chorion
c) the allantois
d) the trophoblast.

57 . Wh ic h o f t he f o l lo w ing i s a co mmo n fu nc ti o n o f t he a dr ena l g la n d s, t he pa ncr ea s


a nd th e l iv er i n hu ma n ?
a) regulation of the metabolic rate
b) construction of the blood vessels;
c) synthesis of urea and uric acid
d) digestion of fats;
e) taking part in the regulation of glucose level in the blood.
58 . T he st r uct ur e o f ho r mo ne s ca n no t be :
a) amino acid derivatives
b) peptides and proteins;
c) steroids
d) carbohydrates.
59 . In a n i nd iv i d ua l w it h sho rt - sig hte d ne ss ( my o pia ) :
a) the eye tends to be longer than average from lens to retina;
b) use of appropriate biconvex lens corrects the defect:
c) close vision is more seriously affected than distant vision;
d) a circular object tends to appear oval.
60 . A d rug t ha t i na ct iv a t es a cet y lc ho l in ee ster a se :
a) inhibits the release of acetylcholine from presynaptic endings;
b) inhibits the attachment of acetylcholine to its receptor protein;
c) increases the duration of acetylcholine stimulating muscle contraction;
d) all of above.
61 . Al l th e f o llo w i ng di sea se s a r e ca u se d by a pro to zo a n e xce pt :
a) malaria
b) sleeping sickness
c) rabies
d) amebic dysentery.
62 . Wh ic h o ne o f t h e f o llo w i ng st a t e me nt s r efe rri ng to t he a xo n i s no t co rre ct ?
a) the axon is the long projection of the neuron;
b) the myelin sheath accelerates the nerve impulse conduction;
c) the axons could be longer than two meters;
d) some axons have not myelin sheath;
e) the speed of nerve impulse conduction does not depend on the axon diameter.
63. Hemopoietic organs are:
The codes of the answers:
1. marrow 2. thymus
nodes; 6. core of the adrenal glands. Answers:
a) 1, 2, 3 and 4;
b) 3, 4 and 6;
) 1, 2, 4 and 6;
d) 1, 2, 5 and 6;
e) no correct answer.

3. islets of Langerhans

4. spleen;

5. lymph

64. To reach the right hand the blood from the stomach and intestine:
The codes of the answers: 1. must pass through the heart (once)
2. must pass through the heart
(twice) 3. do not pass through the heart
4. must pass through the lungs;
5. must pass through
the liver;
6. must pass through the brain. Answers:
a) only 2
b) 1 and 4
c) 2, 4 and 6
d) 2, 4 and 5;e) 3, 4 and 5.

65. A hungry brown rat was placed in a closed metal box with a food slot and a foodreleasing lever.
The rat was allowed to poke around randomly until it accidentally tripped the lever and released a
food pellet. The rat soon learned to trip the lever whenever it wanted food. What type of learning is
this:
a) imprinting;
b) sensitization
c) habituation
d) trial and error.
66. Which process of cell division is essential if Mendel's First Law (the Law of Segregation) is to be
fulfilled:
a) division of the centromere
b) duplication of chromosomes;
c) pairing of homologous chromosomes
d) formation of chiasmata.
67. Man A has a rare X-linked recessive factor which is expressed as trait A. Man has a rare
dominant autosomal factor which is expressed only in males as trait B. Would you distinguish these
two cases just by studying the progeny:
a) no, because the progeny of man A and man can have just the same trait of their respective male
ancestors;
b) yes, because in the progeny of man A and man would not be woman with trait B;
c) no, because man A would not have sons with trait A, and man would not have sons with trait B;
d) yes, because the sons of man A's daughter could be with trait A, whereas the sons of man B's daughter
and the sons of his son could both be with trait B.
68. In numerous bird species males have very colorful and ornate plumage.
Cho o se th e b e st e x pla n a t io n f o r t he ev o l utio n a ry sta b il iza tio n o f s uc h a fea tu re :
a) ornate plumage is a good camouflage among colorful tropical plants;
b) this feature is neither advantageous, nor detrimental, so is neither selected for, nor against;
c) females mate preferentially with ornate males, so the genes "causing" this feature spread among the
population;
d) ornate plumage makes it easier for the birds of the same species to recognize each other, thus potentially
dangerous encounters with individuals of other species are being avoided;
e) bright and ornate plumage serves as a warning against dangerous (often poisonous) species.
69 . Very s i mi la r sp eci e s o f f i sh re lea se t he ir e g g s a nd s pe r m i nto t he sa me w a ter, b ut
the s per m a t o n e s p eci e s ca n no t pe netra t e t he eg g s o f th e o t her s pec ie s. T hi s is a n
exa mp l e o f pre zy g o t i c i so la t io n by :
a) ecological isolation
b) behavioral isolation;
c) mechanical isolation
d) gametic isolation.
70 . In s il k w o r ms , si l k g la n d c el ls a re s pe cia l i zed i n t he sy nth es i s o f la rg e q ua nt it ie s
o f the p ro tei n kn o w n a s si l k- f i bro in. T he s e sa me cel l s do no t ma ke b l o o d sp eci fic
pro te i ns . O ne w o u ld e x pect t ha t s il k - g la n d c el ls ha v e :
a) only silk fibroin genes;
b) the genes for both blood protein and silk fibroin;
c) silk fibroin genes and some other genes, but not blood protein genes;
d) fewer genes than the zygote.
71 . Ple io tro pic g e ne a c t iv it y co u ld b e rev ea l ed ma i nly th ro ug h :
a) studying of genes;
b) studying the interaction of genes of the organism;
c) studying pedigrees (genealogy);
d) studying of phenotypic changes, caused by gene mutations.

72 . Wh ic h o f t he f o l lo w ing co n d it io n s ha v e e na ble d t he a p pea ra n ce o f the liv ing


o rg a n is ms o n dry la n d :
a) decreasing of the vulcanic activity and tectonic processes;
b) decreasing of the sea level and enlarging of the land surface;
c) formation of the ozone layer in the atmosphere;
d) general warming of the climate, increasing of humidity, appearing of the green-house effect.
73 . Co mp a r ing t h e s kel et o n o f ma n a n d o th er pri ma t es th e fo l lo w i ng pec u lia r iti es
co u ld be d et er mi n ed :
The codes of the answers:
1. The thorax of man is laterally flattened and that of the primates is almost cylindrical, with
massive bones to which strong muscles are attached.
2. The bones of the foot of man are shorter.
3. In man the aperture of the lower part of the skull, serving for the connection between the
brain and the spinal cord is situated backwards, while in the primates it is driven more
forward.
4. The molars of man are with U-shaped surface and these of the primates are with X-shaped
surface.
Answers:
a) only 1 and 3
b) only 1 , 2 and 3
c) only 2 and 3 ;
d) only 2 and 4 e) all answers are correct.
74. In a family the father has blood group A and his son has blood group 1 and is a
hemophylic. What is the possible genotype of the mother, the son and the father in that
family?
The codes of the answers:
father
mother
son
A
H
B B H H
1. I IX Y,
I I X X ,
I B IX h Y;
2. I A IX H Y,

I B IX H X h ,

I B IX H Y;

3. I A IX h Y,

I A I B X H X\

I B IX h Y;

4. I A I A X h Y,

I B IX H X\

I B IX H Y

Answers:
a) only 1 and 3
b ) only 2 and 4
c) only 1 and 4 ;
d) only 2 and 3 e) only 2 , 3 and 4 .
7 5. What is the most probable sequence of the genes ABCD if the distance between them is:
A-B = 1,5 map units; B-C = 18 map units and A-D = 18,5 map units.
The codes of the answers:
1. ABCD; 2. BACD;
3. ABDC; 4. ACBD.
Answers:
a) only 1
b ) only 3 ;
c) 2 or 3
d ) only 4 ;
e) all answers are incorrect.

76. After crossing a guinea-pig with black hair and guinea-pig with white hair, all F 1
individuals were with black hair. After analyzing the progeny of a F 1 - female crossed with a
homozygous recessive individual the phenotypes in the backcross were in the following ratio
3 white : 1 black. What are the possible genotypes of the parents and individuals of F 1 ?
The codes of the answers:
1. AA, aa, Aa;
2. AAbb, aaBB, AaBb;
3. AABB, AAbb, AABb;
4. AABB, aabb, AaBb.
Answers:
a) only 1;
b) only 3;
c) only 2;
d) only 4
e) all answers are incorrect.

7 7 . T he g ra p h ic s ho w n in di ca t e s t he a mo un t a nd
co lo r o f l ig ht p en et ra t i o n i n t he s ea .
Co n si de ri ng t hi s da t a , o ne w o u l d e x pect th e b a si c
ma s s o f p hy t o pla n kt o n t o be s i t ua te d i n t he
lev e l :
a) 1-5 m
b) 5-10 m
c) 10-20 m
d) 20-30 m
e) 30-40 m.

7 8 . A pla nt , liv ing i n a ho t , dry env iro n me nt i s


li kely to ha v e :
a) large, dark green leaves with many stomata on both sides;
b) large, light green leaves with stomata on one side only;
c) medium, light-colored hairy leaves with stomata mostly on the underside;
d) small, thick leaves with few stomata.
79 . An ep i phy t e i s a g r o w t h f o r m a da pt ed fo r g a in ing a cc es s to :
a) light
b) pollinators;
c) water
d) nutrients.
80 . T he hig he s t o s mo t i c pr es s ure i s c ha ra cter i s tic fo r th e c el ls o f :
a) hydrophytes
b) mesophytes;
c) xerophytes
d) halophytes.
81 . A c la s s o f st u de nt s ino cu la t e d a f la s k o f le a f bro t h w it h so me po n d mu d a n d kept
in da r kn e s s. Sa mp l e s w ere t a ke n da i ly to a sc e rta i n t he a bu n da n ce o f mi c ro o rg a ni s ms
in t he b ro th o v er a t e n- da y per io d . Th e re s ult s a re sho w n .
Wh ic h o ne o f t h es e st a t e me nt s is no t a
v a li d co nc lu s io n :
a) organism I is probably a producer;
b) organism II probably eats organism I;
c) organism I I succeeds organism I;
d) organism I V is probably a predator.

82. The water-hyacinth was introduced into a tropical country and has
become a major menace to boating, clogging rivers and lakes. A single
plant can produce 1000 offsprings in 50 days.
The reason for its success is:
a) lack of predators;
b) good climate;
c) high reproductive rate;
d) the empty niche it occupied;
e) all of the above answers could be correct.

83. Plants with soft stems and badly developed supportive tissue, few
stem parenchyma cells with large intercellular spaces and with a thin
cuticule, belong to which ecological group?
a) mesophytes
b) xerophytes
c) hydrophytes
d) halophytes.
84. Plants that have mutualistic relations with nitrogen fixing bacteria provide the bacteria with:
a) N2
b) enzymes;
c) sugars;
d) nitrites.

85. The figure represents the changes in the density


of population of species A and species B. What is the
most possible relationship between species A and B?
a) parasite - host;
b) predator - prey;
c) competition;
d) symbiosis.

86. Organisms belonging to two closely related animal species were placed in a container with all
parameters kept constant, and a limiting supply of food. There is no direct interspecies interaction. In
the state of equilibrium:
a) there will be constant numbers of animals of both species (static equilibrium), irrespectively of the initial
numbers;
b) there will be oscillating numbers of animals of both species (dynamic equilibrium);
c) there will be constant numbers of animals of both species (static equilibrium), determined by the initial
numbers;
d) animals of only one species will survive;
e) each of the above situations may take place, depending on the species used for the experiment.

87. If the above ecosystem represented by the food


webs above was sprayed with DDT, the greatest
concentration of this insecticide would be expected in:
a) A;
b)B
c) G
d)F.

88. A potential danger to an isolated population that has been greatly reduced in number is the:
a) loss of genetic variability
b) tendency toward assortative mating;
c) reduced gene flow
d) Hardy-Weinberg disequilibrium.
89. The struggle for existence is a consequence of:
a) each organism leaving more offspring than needed to replace itself;
b) innate competitive tendencies;
c) the inevitable difficulty of coping with climatic conditions;
d) territories and dominance hierarchy.
90. In a stable ecosystem of a pond sudden heavy pollution causes death of all plants. The first visible
change in pond water composition will be decreased in the concentration of:
a) carbon dioxide
b) nitrates
c) oxygen;
d) phosphates e) answers a and are correct.
91. Which of the following promotes diversity of living things:
a) classification of organisms;
b) natural selection in different environments;
c) inheritance from a common ancestor;
d) homeostatic regulation.
92. Which of the following cannot be a result of the ecological succession:
a) increase in the trophic levels in the food chain;
b) increase in the productivity;
c) increase in the stability of the community;
d) gradual decrease in the organic material (humus);
e) increase in the species diversity.

93. Study the pictures and the above key. Which


picture fits key 2a?
Key:
la. single flower;
lb. inflorescence.
2a. pistil flower and perfect flower;
2b. perfect flower only.
3a. one ovary in a flower;
3b. many ovaries in a flower.
Answers:
a) I
b) II ) III
d) IV.

94. Secondary growth of the stem is typical for:


a) Bryophyta, Gymnospermae, Angiospermae;
b) Angiospermae;c) Dicotyledones;
d) Gymnospermae, Dicotyledones.
95. The most diverse and widespread of all contemporary plants are:
a) mosses
b) ferns
c) conifers
d) angiosperms.
96. In what way is the growth of a flowering plant most different from the growth of a
mammal:
a) parts of the organism grow at different rates;
b) the growth at one part might be controlled by another part;

c) the total mass of the organism remains relatively constant once maturity is reached;
d) growth tends to become confined to special groups of cells.
97. Which of the following structures is common in fish, bird and mammalian auditory
(hearing) organ:
a) auriculum
b) semicircular channels;
c) auditory (hearing) channel
d) Eustachian tube;
e) a well developed cochlea.
98. Choose the correct statement:
a) reptiles like fish and amphibians have a body temperature regulating mechanism;
b) the bodies of reptiles are covered by moist, soft scales that help to protect them against dehydratation;
c) due to the development of kidney tubules as water conserving structures, reptiles are able to survive in the
desert;
d) reptiles secrete nitrogenous waste as urea.
99. Of the following groups the first animals to serve as pollinators were:
a) ant
b) beetle
c) bees
d) hummingbird.
100. The sense organ of balance in the crayfish is the:
a) claws
b) ears
c) antennal statocyst;
d) first mouth appendages;e) eye stalks.
101. Heart with two atria and one ventricle can be found in:
a) sparrow
b) toad
c) ray
d) carp.
102. Which animal is reproduced by parthenogenesis as a normal process:
a) hydra;
b) tapeworm
c) earthworm;
d) honey bee.
103. Approximately what percentage of existing animal species are invertebrates:
a) 20 %
b) 50 %
c) 70 %
d) 95 %.
104. Study the following diagrams of cell division. For

"stage of cell division" choose one alternative from the


list below - write the figures in the table.

Fig. 1 (104.1)
Stage of cell division
Number of 2n chromosomes (in
interphase)

Fig. 2 (104.2)

The codes of the answers:


1. mitotic prophase 5. prophase I of meiosis;
2. mitotic metaphase 6. anaphase I of meiosis;
3. mitotic anaphase 7. metaphase II of meiosis;
4. mitotic telophase 8. anaphase II of meiosis.

105. The following biopolymers: nucleic acids, proteins and polysaccharides are characterized by the
following features:
The codes of the answers:
1. linear, almost never branched;
2. linear or branched;
3. homopolymers;
4. heteropolymers.
Put the correct answers into the table using the respective letters.

biopolimers
characteristics
1

nucleic acids

proteins

polysaccharides

106. Which one of the following statements is correct?


The codes of the answers:
protein function 1. antibodies bind to parts of the antigen;
2. antibodies fix oxygen;
3. hemoglobin binds to oxygen;
4. hemoglobin binds to parts of the antigen.
Answers:
a)1 and 4
b) 1 and 3
c) 2 and 4
d) 2 and 3;
107. The following statements refer to the female reproductive system.
Select the correct statements:
a) both estrogen and progesterone are necessary for the ovulation to take place;
b) estrogen tends to inhibit the production of FSH by the anterior pituitary gland;
c) fertilization of the ovum by the sperm normally takes place in the uterus;
d) progesterone production is largely under the control of LH;
e) through out the part of the menstrual cycle that follows ovulation, there is a slight rise of body temperature.
*** Folloyving information deals with the question 108-113.
The bread mould (Neurospora) normally produces its own amino acids from raw materials
through a system of enzymes.
Use the following key for the items 108-113:
a) a logical hypothesis based on the diagram;
b) an illogical hypothesis;
c) a hypothesis unrelated to the diagram;
d) a restatement of information given by the diagram.

10 8 . E nzy me A ca t a ly ze s t he rea ct io n i n w h ic h o rn ith in e i s fo r me d fro m t he pr io r


su b sta nce .
Answer:
a, b, c, d.
10 9 . If g en e w ere no t pr es ent , a rg in in e w o u l d be fo r me d d ire ctly f r o m o r ni th in e .
Answer:
a, b, c, d.
11 0 . If g en e A w ere d es t ro y ed , t he mo u l d w o u l d su rv iv e if o rn it hi ne w ere a d de d to th e
me d i u m.
Answer:
a , b, c, d.
11 1 . If g en e w ere de s t ro y ed , t he mo u l d co ul d su rv iv e if en zy me w ere a d de d to th e
me d i u m.
Answer:
a, b, c , d.
11 2 . G en e d ire ct s t h e f o r ma t io n o f e nzy me Answer:
a, b, c, d.
11 3 . Diff ere nt mo ul d s ha v e dif f er ent a mi no a cid re q uir e me nts .
Answer:
a, b, c, d.
*** Following information deals with the question 114-115.
In a ra ndo m- ma t ing p o pu la t io n o f p la n t A (a cro s s - f ert il izi ng pla nt), the fo l lo w ing
ph eno ty p ic f re qu en cie s a re f o un d:
Ta l l, g ree n st e m - 6 3 % ;
Ta l l, p ur ple st e m - 2 1 % ;
Sho rt, g r ee n st e m - 1 2 %;
Sho rt, pu rp le st e m - 4 %.
T - a do mi na nt a lle le c o nt ro ll ing t a ll s te m ;
t - a re ce s siv e a ll el e co nt ro ll ing s ho rt st e m ;
R - a do mi na n t a l lel e c o nt ro ll ing g ree n ste m ;
r - a r ece s siv e a l le le co nt ro ll ing p ur pl e ste m .
1 1 4 . W ha t i s t he p erc e nt a g e o f t h e p la nt s ha v ing t he TT g e no ty p e ?
a) 84;
b) 63
c) 36;
d) 21;e) 12 f) 9 g)4.
11 5 . W ha t i s t he pe rce n t a g e o f t he pla nt s ha v i ng t he T TR R g e no ty pe ?
a) 84;
b) 63
c) 36;
d) 21;e) 12 f) 9 g) 4.

*** Following information deals with the question 116-120.


a) gene flow;
b) sexually reproducing species;
c) natural selection;
d) mutation;
e) genetic drift.
Use the following key for the phrases.

116. The sole source of new alleles is:


117. Change of increase or decrease in relative allele frequencies of a population is:
118. One or more populations, the members of which interbreed and produce fertile offspring
is:
119. Change in allele frequencies in a population due to immigration, emigration is:
120. Differential survival and reproduction of variant members of a population is:

INTERNATIONAL BIOLOGY OLYMPIAD


PRACTICAL PROBLEMS
1994, Varna, Bulgaria















All IBO examination questions are published under the following Creative Commons license:



CC BY-NC-SA (Attribution-NonCommercial-ShareAlike) https://creativecommons.org/licenses/by-nc-sa/4.0/
The exam papers can be used freely for educational purposes as long as IBO is credited and
new creations are licensed under identical terms. No commercial use is allowed.

INTERNATIONAL BIOLOGY OLYMPIAD


THEORY PROBLEMS
1993, Utrecht, Netherlands















All IBO examination questions are published under the following Creative Commons license:



CC BY-NC-SA (Attribution-NonCommercial-ShareAlike) https://creativecommons.org/licenses/by-nc-sa/4.0/
The exam papers can be used freely for educational purposes as long as IBO is credited and
new creations are licensed under identical terms. No commercial use is allowed.

IVth International Biology Olympiad (Utrecht, the Netherlands, 1993)


Theoretical Test
The theoretical test consists of two parts, A and B.
Part A consists of 111 multiple choice questions.
You have got two answering sheets for this test A:
sheet 1: questions 1 - 100
sheet 2: questions 101 - 111.
Use a pencil to fill out the multiple choice questions on the answering sheets.
Part B consists of 29 open questions.
For this part B you have to fill out your answers in the given space on the test itself.
For each question in part A and B the obtainable scores are indicated in brackets. The total score is:
Totally:
Part A:
136 points
Part B:
70 point
Together:
206 points
The figures and graphs are in an appendix and will be handed out to you separately. You will need these
figures and graphs for both part A and B, so please keep them during the whole contest.
You have three and a half hours to complete the test.
How you divide the available time is up to you, but you are advised not to spend more than two hours on
part A.
Before starting, be sure that your name and participant number are correctly indicated on the two answering
sheets of part A and on test B itself.
GOOD LUCK!

Theoretical test, Part A.


1.(2pt)
As result of a mutation in a cell, a protein is formed with a different type of amino acid at the
place of the amino acid lysine. In answering the question use the information given in the table with mRNA
codons ( fig. 1)

What amino acid is most likely to be built in, aspartic acid (Asp) or methionine (Met) and which nucleotide is
mutated?

A.
B.
C.
D.

amino acid
aspartic acid
aspartic acid
methionine
methionine

nucleotide mutated
adenine
thymine
adenine
thymine

2.(2pt)
The table shows two different mRNA molecules and the two different protein molecules
synthesized from them.
mRNA
protein
.....AGAGAGAGAGAGAGAGAGAGAGAG.....
P
.....AAUGAAUGAAUGAAUGAAUGAAUG.....
Q
How many different types of amino acids can be found in each protein molecule?

A.
B.
C.
D.

P
1
1
2
2

Q
3
4
3
4

3.(1pt)

A gene can be defined as:


A.
C.
D.

.(1pt)

Which one of the following items is not required for protein synthesis?
A.
B.
C.
D.
E.

5.(1pt)

B.
C.
D.

Enzymes loose some or all of their normal activity if their three dimensional structure is
disrupted.
Enzymes provide the activation energy necessary to initiate a reaction.
The activity of enzymes is independent of temperature and pH.
An enzyme acts only once and is then destroyed.

A bacterial mRNA with a length of 360 nucleotides in length codes for a protein of:
A.
B.
C.
D.

7.(1pt)

mRNA
ribosomes
tRNA
endoplasmic reticulum
amino acids

Which one of the following statements is true for enzymes?


A.

6.(1pt)

part of an RNA molecule, characterized by a certain order of nucleotides.


B.
a
particular order of nucleotides in a DNA-molecule.
part of a DNA-molecule, characterized by a certain order of nucleotides, which determines
the synthesis of one protein.
order of amino acids in a protein molecule.

roughly 360 amino acids


roughly 1080 amino acids
exactly 120 amino acids
less than 120 amino acids

What is the most important function of the organella P in


figure 2?
A.
B.
C.
D.

synthesis of proteins
synthesis of ATP with energy from dissimilation
reactions
synthesis of ATP with energy from the sun
transport of proteins

8.(1pt)
When a muscle cell has a shortage of oxygen will the pH
decrease or increase? What substance is responsible for this change in pH?

A.
B.
C.
D.

change in pH
decrease
decrease
increase
increase

caused by
carbon dioxide
lactate (lactic acid)
carbon dioxide
lactate (lactic acid)

9.(1pt)

An enzyme catalyses the reaction P + Q R.


The continuous line in the graph (Fig.3) shows the progress of this reaction in the absence of
the enzyme. At time point t1, the enzyme that catalyses the reaction is added to the reaction
mixture. ([Z] is representing the concentration of a compound Z).

Which line in figure 3 shows how the


reaction will proceed ?
A.
A
B.
B
C.
C
D.
D
E.
E

10.(1pt)

Which of the following processes causes


the highest energy release?
A.
photolysis
B.
glycolysis
C.
Krebs cycle (citric acid cycle)
D.
terminal oxidations

11.(1pt)

What happens during glycolysis and what are the end products?
A.
Oxygen from CO2 is replaced by hydrogen and glucose is formed.
B.
As a result of splitting by fermentation of a glucose molecule, two molecules of tricarbon
acid are formed and 2 molecules of ATP are synthesized.
C.
As a result of the splitting of fat molecules, energy is released, which the cell needs.
D.
Tricarbon acid is split up into carbon dioxide and water, as a result of which 36 molecules
of ATP are synthesized.

12.(1pt)

The concentration of an electrically neutral substance within a certain type of blood cell is much
higher than it is in the surrounding blood plasma, yet the substance continues to move into the
cell. The process by which this substance moves into the cell is called:
A.
osmosis
B.
simple diffusion
C.
facilitated diffusion
D.
active transport

13.(2pt)

An U-shaped tube is filled with two different solutions, R and S, separated by a semi-permeable
membrane (X). R is hypotonic in relation to S, seen by the lighter tone in figure 4.

When an equilibrium is reached in the U-shaped tube of figure 4, are the surfaces equally high or
not, and are the osmotic pressures equal or not (see also figure 5)?
A. The right side is higher, the two solutions are isotonic
B. The right side is higher, the right side is hypertonic
C. The left side is higher, the right side is hypertonic
D. The surfaces are equally high, the two solutions are isotonic
14.(1pt)

Which one of the following pairs is correctly matched?


A.
chloroplast
storage of enzymes
B.
peroxisomes
cellular transportation
C.
nucleolus
site of ribosomal subunit synthesis
D.
lysosomes
power house of cell

15.(1pt)

See figure 6. What letter in figure 6 indicates the end of meiosis I?

A.
B.
C.
D.

A
B
C
D

16.(1pt)

Which organelles take part in the formation of a spindle?


A.
nucleus
B.
mitochondrions
C.
Golgi-complex
D.
endoplasmic reticulum
E.
microtubuli

17.(2pt)

Figure 7 shows the movement of chromosomes during mitosis.


Curve A in figure 7 shows the mean
distance between the centromeres of
the chromosomes and the
corresponding pole of the spindle.
At what time did the anaphase start?
A.
0 minutes
B.
Somewhere between 0 and 10
minutes
C.
10 minutes
D.
After more than 10 minutes

18.(2pt)

In figure 7, curve B represents the distance between


A.
the chromosomes in the metaphase
B.
the centrioles
C.
chromatides of a chromosome
D.
homologuous chromosomes during the anaphase

19.(1pt)

What property/properties make(s) the phospholipids especially well suited to make up cellular
membranes?
A.
they are hydrophobic
B.
they are hydrophilic
C.
they absorb water readily
D.
they are both hydrophobic and hydrophilic

20.(1pt)

If one were observing a cell undergoing mitosis, which of the following would be evidence that it
was a plant rather than an animal cell?
A.
absence of spindle
B.
absence of centrioles
C.
replication of chromosomes
D.
disappearance of nuclear membrane

21.(1pt)

The belonging of a human erythrocyte to serotypes A, B, 0 is determined by chemical markers on


its surface. These markers are:
A.
lipid molecules
B.
oligosaccharides
C.
polypeptides
D.
antibodies
E.
nucleic acids

22.(1pt)

Most plants store energy in the form of:


A.
glycogen
B.
cellulose
C.
fat
D.
glucose
E.
starch

23.(1pt)

Which organelles do not have a membrane?


A.
mitochondrions
B.
plastides
C.
Golgi-apparatus
D.
nucleolus
E.
lysosomes

24.(1pt)

DNA replicates during:


A.
G1 phase
B.
S phase
C.
G2 phase
D.
M phase

25.(1pt)

The molecular formula of glucose is C6H12O6. What would be the molecular formula of a
polymer made by linking ten glucose molecules together by a condensation reaction?
A.
C60H120O60
B.
C6H12O6
C.
C60H102O51
D.
C60H100O50
E.
C60H112O51

26.(1pt)

Cells of the pancreas will incorporate radioactive amino acids into proteins. This labels newly
synthesized proteins and enables to track the location of these proteins in a cell. In this case we
are tracking an enzyme that is eventually secreted by pancreas cells. Which of the following is
the most likely pathway for movement of this protein in the cell?
A.
endoplasmic reticulum - Golgi nucleus
B.
Golgi - endoplasmic reticulum lysosome
C.
nucleus - endoplasmic reticulum Golgi
D.
endoplasmic reticulum - Golgi vesicle that fuses with plasma membrane
E.
endoplasmic reticulum -lysosome vesicles that fuses with plasma membrane

27.(1pt)

A chromosome is in its most extended form during:


A.
interphase
B.
prophase
C.
metaphase
D.
anaphase
E.
telophase

28.(1pt)

Which one of the following elements is needed for the synthesis of chlorophyll, but is not itself a
component of chlorophyll?
A.
carbon
B.
magnesium
C.
nitrogen
D.
iron
E.
oxygen

29.(1pt)

With the help of a light microscope, it can be observed that when cells are dehydrated, plant cells
shrink less than animal cells. This is caused by the difference in:
A.
the flexibility of the plasmalemma
B.
the number of pores in the plasmalemma
C.
the osmotic potential of the cells
D.
the existence of a cell wall in plant cells
E.
the size of the vacuoles

30.(2pt)

From a fresh potato, small lenghts are cut


which are exactly 5 cm. Each one of these
lengths is placed in a mannitol solution. The
concentration of the solution is different for
each length. After 4 hours, the length of each
potato part is measured. In figure 8, the
lengths of the parts are given and compared
with the concentrations of the mannitol
solutions.

Which of the following conclusion(s) is/are correct?


I.
The cells of the potato length in a solution of 40 grams of mannitol per litre still have turgor
after 4 hours.
II.
The cells of the potato length in a solution of 100 grams of mannitol per litre have come
loose from the cell wall after 4 hours.
A.
both
B.
I only
C.
II only
D.
neither
31.(2pt)

If a stem is exposed to light from one side only, it bends towards the light (Fig.9A). This positive
phototropism is caused by substances which are synthesized in the top of the stem. In a stem of
a plant (which had been exposed from all sides) a crosswise incision was made just under the
top. In the incision, a small slide of glass was placed, after which the plant was exposed on the
side of the incision (Fig.9B).

How will the stem now grow, and what is the correct explanation of this phenomenon?
A.
The stem will bend away from the light. The sheet of glass in the incision causes an
increase in the concentration of growth-stimulating substances on the exposed side.
B.
The stem will bend away from the light. The sheet of glass stops the transport of growthinhibiting substances on the exposed side.
C.
The stem keeps growing straight upward. Positive phototropism of the untreated plant is
not caused by a decrease in the growth on the exposed side.
D.
The stem will bend towards the light. Light causes the transport of growth stimulating compounds towards the unexposed side of the stem.
32.(1pt)

Four types of Phytoplankton (I, II, II and


IV) were collected from different depths
of the sea. For each of these types, the
photosynthesis was measured, as
represented in figure 10.
Which type of Phytoplankton was
collected at the greatest depth?
A.
I
B.
II
C.
III
D.
IV

33.(1pt)

Figure 11 shows leaves of the same type of plant


in different conditions. From the edges of the
leaves of plant P, small drops of water emerge,
from plant Q they do not.
From which plants do the leaves have the
strongest osmotic potential?
From which plant do the leaf cells have the
highest turgor pressure?

A.
B.
C.
D.

strongest osmotic potential:


plant P
plant P
plant Q
plant Q

highest turgor pressure:


plant P
plant Q
plant P
plant Q

34.(1pt)

A C3- and a C4-plant are both put into the same glass jar. The jar is kept away from the outside
air and is exposed to light during 12 hours a day for some time. After a week the C3-plant has
died. This happens because:
A.
the C4-plant needs less oxygen for respiration at night
B.
the C4-plant is more resistant to drought
C.
the C4-plant is more efficient in its photosynthesis
D.
the C4-plant exhausts the C3-plant by photosynthesizing its carbon dioxide which is
released during respiration

35.(1pt)

Figure 12 gives information on the diameter of a tree-trunk during a number of subsequent


twenty-four-hour periods.

Friday was a dry, cloudy day.


What do you expect the weather conditions were like on the last day?
A. rainy and calm
B. rainy with a lot of wind
C. sunny day with high humidity
D. sunny day with low humidity
36.(1pt)

Potassium plays an important role in opening and closing of stomata. This role consists of:
A. changing the permeability of the plasmalemma of the guard cells for water
B. changing the osmotic potential of the guard cells
C. regulating the synthesis of the plant hormone called abscisic acid, which is needed for the
conversion of sugars into starch in the guard cells.

D.

regulating the production and degradation of starch and organic acids in the guard cells

37.(1pt)

Under conditions of a high atmospheric humidity hardly any calcium (Ca) is transported to
developing fruits. This is caused by:
A. calcium only being transported through the xylem and this transport not taking place
anymore
B. calcium only being transported through the phloem and this transport not taking place
anymore
C. transpiration stopping and, as a result both xylem and phloem transport stopping
D. the stomata closing and transport to the fruit stopping

38.(1pt)

C4-plants can start photosynthesis with a lower concentration of CO2 in the atmosphere than C3plants. This is because:
A.
respiration of C4-plants is higher
B.
respiration of C4-plants is lower
C.
C4-plants do not have photorespiration
D.
C4-plants do have photorespiration

39.(1pt)

In figure 13, for two plants the


relation is shown between the
day length and the time at
which flowering occurs.

A.
B.
C.
D.
E.
40.(1pt)

plant A is a short-day plant and plant B is a long-day plant


plant A is a long-day plant and plant B is a short-day plant
plant A is a short-day plant and plant B is a day-neutral plant
plant A is a long-day plant and plant B is a day-neutral plant
plant A is a day-neutral plant and plant B is a short-day plant

From two trees, diameter 10 and 40 cm, the bark of the stem is equal in thickness.
The reason for this is:
I.
secondary phloem is produced much less than secondary xylem
II.
dead phloem cells are pressed together
III.
rhytidome is produced and the outer bark peels off
What is correct ?
A.
I and II only
B.
I and III only
C.
II and III only
D.
I, II and III

41.(1pt)

The table in Fig. 14 shows the percentages of identity of alpha-hemoglobin amino acid
sequences among four animal species: L, M, N, P.

The species investigated are: man, chimpanzee, horse and kangaroo, but it is not known which
letter corresponds to which species. Which one of the given keys is most likely to be correct?
man
chimpansee
horse
kangaroo
A.
L
M
N
P
B.
P
N
M
L
C.
L
P
M
N
D.
M
N
L
P
42.(1pt)

In one of the groups listed below more than one heart may be found. In which group?
A. echinoderms
B. bivalves
C. cephalopods
D. non-vertebrate chordates

43.(1pt)

Only one of the following features of the phylum of the Chordata also is present in adult
Tunicata (=Urochordata). Which feature?
A.
possession of a chorda
B.
possession of visceral slits ( = pharyngeal slits)
C.
possession of a tail
D.
possession of a dorsal tubular nervous system

44.(1pt)

One of the following properties can be used to distinguish reptiles from amphibians. Which one?
A.
a closed type of circulatory system
B.
reproductive organs opening into the intestine
C.
a single compartment stomach
D.
a metanephros type of kidney

45.(1pt)

The increase in complexity of the vertebrate circulatory system is represented by one of the
following combinations. Which combination?
A.
toad
- rabbit
- alligator
- shark
B.
shark
- frog
- alligator
- rabbit
C.
shark
- crocodile
- rabbit
- frog
D.
alligator - dog
- shark
- toad

46.(1pt)

Blood loses its oxygen rapidly in one of the following places. Which one?
A.
lung alveoles
B.
blood capillaries in intestines
C.
lung veins
D.
liver portal vein

47.(1pt)

Four possibilities for the transport of carbon dioxide from the body cells to the lungs are listed
below. Which possibility does not exist?
A.
bound to the ferro-ions of hemoglobin in erythrocytes
B.
as a hydrocarbonate ion in the buffering system of the blood
C.
bound to the protein of hemoglobin in erythrocytes
D.
dissolved in blood plasma and in erythrocyte cytoplasm

48.(1pt)

Which process in the kidney nephron is least selective?


A.
secretion
B.
reabsorption
C.
filtration
D.
transport across the epithelium of the collecting duct

49.(1pt)

50.(1pt)

51.(1pt)

The concentration of a substance in Bowman's capsules of a healthy human kidney is 0.1%, while
its concentration in the urine is zero. Which one of the following substances is meant?
A.
calcium phosphate
B.
glucose
C.
sodium chloride
D.
uric acid
Which of the four organs listed below does not belong to the immune system?
A.
lymph nodes
B.
pancreas
C.
thymus
D.
spleen
Which of the following statements about macrophages is
false?
A.
Macrophages look like amoebas.
B.
Macrophages produce antibodies.
C.
Macrophages produce interleukines
D.
Macrophages practise phagocytosis

52.(1pt)
Which organ(s) in Fig. 15 is/are under the control of a
hormone released by the pituitary gland?
A.
B.
C.
D.
E.

1 only
2 only
1 and 2 only
2 and 8 only
1, 2 and 8

53.(1pt)

Which one of the following symptoms can be used as evidence for an insufficiency of the
hormones secreted by the parathyroid glands?
A.
decrease in blood glucose
B.
increase in blood urea
C.
decrease in blood calcium
D.
decrease in blood amino acids

54.(1pt)

The vitelline sac (=yolk sac) is expected to be very small in one of the following groups. In which
one?
A.
in groups that fertilize externally
B.
in groups with embryos that are fed from maternal blood
C.
in groups that fertilize internally
D.
in groups that have an allantoic membrane

55.(1pt)

One of the following traits is common to both avian and mammalian development. Which one?
A.
holoblastic cleavage
B.
primitive streak
C.
trophoblast
D.
yolk plug/mass

56.(1pt)

57.(1pt)

Ecto-, endo- and mesoderm develop into tissues and organs.


One of the following combinations is true. Which one?
ectoderm
mesoderm
A.
brain and spinal cord
blood
B.
brain and spinal cord
colon (=large intestine)
C.
skin
bones
D.
epidermis
liver

entoderm
lungs
lungs
kidneys
heart

One of the alternatives below defines the layers of the retina in the correct sequence. Which one?
N.B! The first layer in each sequence is supposed to be located next to the jellylike vitreous
humor that fills the eyeball.
A.
pigmented cells - bipolar cells
- ganglion cells
- photoreceptors
B.
photoreceptors - pigmented cells
- ganglion cells
- bipolar cells
C.
ganglion cells
- bipolar cells
- photoreceptors
- pigmented cells
D.
photoreceptors - bipolar cells
- ganglion cells
- pigmented cells

58.(1pt)

During starvation or hibernation, the supplies of energy substrates are used up in the order:
A. fats
- proteins
- carbohydrates
B. fats
- carbohydrates
- proteins
C. carbohydrates
- fats
- proteins
D. proteins
- carbohydrates
- fats

59.(1pt)

One of the four statements given below is false. Which one?


A.
Certain innate patterns of behaviour can be modified by experience.
B.
Taxis never occurs together with fixed action patterns.
C.
A super normal stimulus often produces a stronger response.
D.
Fixed action patterns are highly stereotyped instinctive behaviours.

60.(1pt)

61.(1pt)

The red patch of the bill of a herring gull has a special function for its chickens. One of the
statements below is true. Which one? It serves as a:
A. super normal stimulus
B. sign stimulus
C. inhibitor of aggression
D. recognition marker of the parent
Some bees in an observation hive found a white clover field at 500 m. distance. At 10 a.m. they
danced in the hive according to figure 16A.

How would these bees dance at 2 p.m.?


A.
according to figure 16A
B.
according to figure 16B
C.
according to figure 16C
D.
according to figure 16D
62.(1pt)

In the same hive other bees found a group of linden trees; they danced at 10 a.m. according to
figue 16B. How would these bees dance at 2 p.m.?
A.
according to figure 16A
B.
according to figure 16B
C.
according to figure 16C
D.
according to figure 16D

63.(1pt)

A number of moth species use pheromones to achieve the encounter between the sexes.
Generally it is the stationary female that produces this odour, and the male often flies long
distances to find the female.
A. He is activated by the pheromones and flies upwind to the odour source (anemotaxis); the
pheromones induce landing near the female.
B. He is activated by the pheromones and uses the odour gradient (chemotaxis) both for
orientation and landing.
C. He is activated by climatic conditions; after a random flight he spots the female nearby
through the perception of pheromones.
D. He is activated by climatic conditions; celestial cues enable a directed flight; the
pheromones induce landing near the female.

64.(1pt)

During autumn migration the young of some bird species fly with their parents, while in other
species old and young birds migrate separately. This is related to :
A. the number of broods the parents make in a season
B. the average life span of an individual of that species
C. food requirements during migration
D. the direction and distance of the migration being learned or innate

65.(1pt)

Pigeons were trained to collect food from a feeding machine at 10 a.m., by picking at a pigeon
key. Then they were transported 60 degrees eastward to a place P. At this new location they
are expected to pick at the pigeon key (local time in P) at:
A.
10 a.m.
B.
4 a.m.
C.
4 p.m.
D.
2 p.m.

66.(1pt)

The most important factor regulating seasonal migration is:


A.
the change in average air temperature
B.
the change in day length
C.
the reduced availability of food
D.
the increased predator pressure

67.(1pt)

Which of the following alternatives is not an indication of annual photoperiodism?


A.
swarming
B.
moulting
C.
flowering
D.
sleep

68.(1pt)

So-called altruistic behaviour (e.g. the protection of lion pups by `aunts') occurs in natural
populations because (choose the primary reason):
A.
it ensures survival of the species
B.
it ensures survival of kin
C.
it ensures spread of the genes shared by relatives
D.
of group selection mechanisms

69.(1pt)

As basic material for micro evolution serve(s)


A.
modifications
B.
phenotypical plasticity
C.
mutations
D.
phenocopies

70.(2pt)

In Drosophila, the following crossing-over percentages were found:


gene
gene
crossing-over %
bi
ec
1.4
bi
fa
3.9
wi
ec
4.0
wi
fa
1.5
What is the order of these genes?
A. bi - ec - fa - wi
B. bi - ec - wi - fa
C. ec - bi - fa - wi
D. ec - bi - wi - fa

71.(1pt)

Which form of natural selection can lead to differentiation in species and to polymorphism?
A.
directional selection
B.
stabilizing selection
C.
disruptive selection
D.
density dependent selection

72.(1pt)

73.(1pt)

What is the advantage of sexual reproduction over a-sexual reproduction?


Sexual reproduction generally leads to:
A.
a larger amount of offspring
B.
greater genetic variation among the offspring
C.
a greater distrubution area
D.
lower sensivity of the individual to environmental influences
A black female rabbit which had been crossbred with a white male rabbit had a litter of black
youngs. One black young female from this litter was crossbred with a white male. What is to be
expected of the colours of young rabbits from this second crossbreeding when, in this case, fur
colour is determined by an autosomal locus?
A.
3 blacks and 1 white
B.
3 whites and 1 black
C.
blacks only
D.
2 whites and 2 blacks

74.(1pt)

Which new qualities in the organic world occurred in the Precambrium:


A.
the heart with four ventricles and homoiothermia
B.
the flower and seeds
C.
photosynthesis and multicellularism
D.
internal skeleton of bone tissue

75.(1pt)

Which changes in the organic world occurred in the Mesozoicum:


A.
development and spreading of the angiosperms
B.
development and spreading of the reptiles
C.
development and spreading of fish
D.
development and spreading of the green algea

76.(1pt)

Which factor possibly played a selecting role in the evolution of the flower:
A.
the fertilization process becoming independent on the presence of water
B.
development of adaptations to pollination
C.
development of a surplus of nutrients in the seed
D.
development of adaptations to the spreading of fruits

77.(1pt)

What did the evolution of photosynthesis lead to?


A.
the evolution of the biosynthesis of proteins
B.
the evolution of multicellular organisms
C.
the accumulation of carbon dioxide in the atmosphere
D.
the accumulation of oxygen in the atmosphere

78.(1pt)

In which of the following cases the genes perform a multiple function?


A. A pea has a dominant gene which causes the yellow colour of the seeds.
B. The green colour of the seeds of a pea is caused by a recessive gene.
C. Yellow body colour of fruit flies is inherited sex-linked.
D. With the columbine (flower genus Aquilegia), a gene determines the red colour of the flower
and the violet hue in the leaves.

79.(1pt)

How are polyploid organisms obtained:


A. Self-pollination is carried out with plants and homozygous lines are obtained; the
homozygous lines are crossbred among themselves and show heterosis.
B. Individuals are selected which are of interest to man, their sex organs are exposed to
radiation and as a result of changes in genes and chromosomes new hereditary
characteristics occur.
C. For a good productivity, a selection is made in a number of lines, and these are then
crossed.
D. By means of a colchicine treatment, the spindle of a dividing cell collapses and therefore,
the doubled chromosomes do not disperse towards the poles of the meiotic cell.

80.(1pt)

Which two pictures in figure 17 give a summary of the theory of natural selection according to
Darwin:

A.
B.
C.
D.
81.(1pt)

82.(2pt)

I and III
I and IV
II and III
II and IV

In the European population, about 1 in 2500 people suffers from Cystic Fibrosis, a genetically
determined (descared), autosomal disease. Healthy parents have a child suffering from Cystic
Fibrosis. The woman remarries a healthy man. What is the chance of a child from this second
marriage suffering from Cystic Fibrosis:
A.
1 : 25
B.
1 : 50
C.
1 : 100
D.
1 : 625
The pedigree in figure 18 shows an inheritance of a rare form of muscular dystrophy.

The disease is probably caused by a mutation on one locus which is:


A.
recessive, autosomal
B.
dominant, autosomal
C.
recessive, related to the X-chromosome
D.
related to the Y-chromosome
E.
situated in the mitochondrial genome
83.(1pt)

Extinct marine xiphosaurs are found in a large area in 150 million-year-old sediments
(150 Myr BP). From each individual, the length/breadth ratio of the carapax was measured. This
length/breadth ratio is referred to as 's'.
In figure 19, graph P represents the distribution of quality 's' for 150 Myr BP.

In sediments of 100 Myr BP, three different populations of xiphosaurs are found in three different
places. The graphs a, b, c in figure 19 represent the distribution of quality 's' in each of these
three populations. Answer the following two questions:
1
for which of the three populations (a, b, c) is it most probable that species formation occurs?
2
for which of the three areas (a, b, c) is it most probable that the environment remained
constant?
species formation
constant environment
A
b
a
B
c
c
C
c
a
D
b
b
84.(1pt)

85.(1pt)

The order of appearance of the main groups of organisms during evolution can best be described
as follows:
A.
autotrophic, anaerobic heterotrophic, aerobic heterotrophic
B.
aerobic heterotrophic, anaerobic heterotrophic, autotrophic
C.
anaerobic heterotrophic, aerobic heterotrophic, water oxydizing autotrophic
D.
anaerobic heterotrophic, water oxydizing autotrophic, aerobic heterotrophic
The main trend in the evolution of land plant was:
A.
a sharp demarcation of the phases of sporophyte and gametophyte
B.
a shortening of the haploid phase
C.
a shortening of the a-sexual phase
D.
an increase in the complexity of the gametophyte

86.(1pt)

A new characteristic usually appears in evolution as a result of:


A. accumulation of point mutations in a gene which originally encoded for something else
B. duplication of a gene and accumulation of point mutations in one of the copies coming from
that duplication
C. a mutation in a regulator gene
D. genotypical recording of favourable phenotypical adaptations

87.(2pt)

How many different phenotypes can be expected in the F2 of the crossing: AA BB * aa bb when:
I the genes are completely coupled and,
II the genes inherit independently:
I
II
A.
3
4
B.
3
9
C.
4
9
D.
4
16
E.
9
16

88.(3pt)

In the domestic cat, the locus Orange is sex linked and intermediary dominant: heterozygous cats
are 'tortoise'. A tortoise pussy cat (Oo) mates with a red tomcat (oY); the third colour is tabby
(OO, OY). What can be expected for the fur colour of the kittens?
females
males
A.
tortoise
red and tabby
B.
tortoise and red
red and tortoise
C.
tortoise and red
red and tabby
D.
red and tabby
red and tabby

89.(3pt)

In the domestic cat, the autosomal locus White is dominant and epistatic; the locus Orange is sex
linked with allele O tabby and allele o red, while the heterozygous is tortoise. A white female
mates with a tabby tomcat.
The kittens turn out to be:
1 red male
1 tortoise female
1 tabby female
1 white male
1 white female
What is the genotype of the mother:
A.
WW Oo
B.
Ww OO
C.
Ww Oo
D.
Ww oo

90.(1pt)

The following reactions produce the eye pigments B and D in fruit flies:
enzyme X
substrate A ----------------> pigment B

substrate C

enzyme Y
----------------> pigment D

Mixing of both pigments results in a reddish brown eye colour with wild type flies. Absence of
pigment B results in bright red eyes, absence of pigment D results in brown eyes, absence of
both pigments results in colourless (white) eyes.

When a homozygous fly with colourless eyes is crossed with a homozygous wild type fly, which
enzymes can be demonstrated in the F1 descendants:
A.
X only
B.
Y only
C.
X and Y
D.
none
91.(3pt)

92.(1pt)

Crossing of two white dogs with the same genotype results in descendants in the following
relation: 12 white : 3 black : 1 brown. When the brown descendant is crossed with a white which
has the same genotype as the parents, what is the expected relation in this second crossing:
white :
black :
dark brown :
brown
A.
9
3
3
1
B.
1
1
1
1
C.
12
3
1
D.
2
1
1
A simplified representation of the nitrogen cycle in an ecosystem is shown in figure 20.
Which group of bacteria is involved in the conversion of proteins to NH3 ?

A.
B.
C.
D.
E.
93.(1pt)

94.(1pt)

Decomposers
Chemosynthetizers
Nitrogen oxidizers
Free nitrogen fixers
Ammonia oxidizers

Nitrogen is fixed in ecosystems in ways stated below.


One of the statements below is false. Which one?
A.
by cyanobacteria
B.
by electrical discharges in the atmosphere
C.
by industrially synthesized fertilizer
D.
by atmospheric deposition
E.
by denitrification
Water in an eutrophic, fresh-water lake, contains large amounts of blue-green algae, which make
the lake unsuitable for recreation. Which of the following treatments is the most likely to lead,
within the shortest time, to a significant decrease in the abundance of these algae ?
A.
Massive introduction of carnivorous fish (bass, pike etc.).
B.
Massive introduction of zooplancton eating fish.
C.
Complete removal (by selective fishing) of carnivorous fish.
D.
Complete removal (by selective fishing) of zooplancton eating fish.

95.(1pt)

The transfer of energy through a terrestrial ecosystem is often depicted by energy pyramids.
Which of the following statements is true ?
A. Ecological efficiency is highest for top consumers.
B. About 10 % of the energy from one trophic level is incorporated into biomass of the next
level.
C. The energy lost as heat or in cellular respiration is 10 % of the available energy of each
trophic level.
D. Only 25 % of the energy in one trophic level is passed on to the next level.

96.(1pt)

Nitrogen fertilization of rice fields can be low in the presence of one of the following types of
organisms. Which one ?
A.
Water fern (Azolla species)
B.
Green algae
C.
Brown algae
D.
Mosses

97.(1pt)

On the border of two ecosystems (grassland and wood), there is a transitional zone (ecotope).
Characteristic for the fauna of this ecotope is:
A. A smaller species diversity in comparison with the two adjoining ecosystems.
B. A greater species diversity in comparison with the two adjoining ecosystems.
C. A greater number of herbivores in comparison with the two adjoining ecosystems.
D. A greater number of predators and parasites in comparison with the two adjoining
ecosystems.

98.(1pt)

The growth of a population of animals is mainly determined by the combination of two important
parameters. Which?
A.
birth rate and food supply
B.
mortality rate and migration
C.
mortality rate and the size of the area occupied by the population
D.
birth and mortality rate
E.
food sources and the size of the territory

99.(1pt)

Which one of the sequences below is an example of primary succession?


A.
mosses
- lichens
- herbaceous plants
B.
lichens
- herbaceous plants
- mosses
C.
lichens
- mosses
- herbaceous plants
D.
herbaceous plants
- mosses
- lichens
E.
herbaceous plants
- lichens
- mosses

100.(2pt)

In the graph of figure 21, for an area in the temperate


regions, two curves are shown, which are related to
the changes of two quantities during primary
succession.
What do these curves represent?

A.
B.
C.
D.

101.(1pt)

A = the species diversity of herbaceous plants


B = the biomass of the entire vegetation
A = the biomass of the entire population
B = the species diversity of herbaceous plants
A = the total amount of humus on the soil
B = the species diversity of the entire vegetation
A = the species diversity of the carnivores
B = the total biomass of the carnivores
In figure 22, the total amount of humus is
plotted as a function of time, in two
ecosystems.
What are these systems?

A.
B.
C.
D.
102.(1pt)

A
tropical grassland
tropical rain forest
tundra
tropical grassland

B
tropical rain forest
broad-leaved forest from temperate regions
tropical rain forest
grassland from temperate regions

In a laboratory, a competition experiment is executed with two types of bacteria, which compete
for food. During the experiment, it is determined what the relationship is between the frequency
of type 1 in the mixed culture after 1 generation (zt+1) as a function of the frequency of type 1
during the previous generation (zt). In figure 23, the drawn line indicates the observed
relationship between zt+1 and zt, the dotted line is the relationship zt+1 = zt.

What happens to type 1 and type 2 in the long term?


A.
Type 1 co-exists with type 2.
B.
Both type 1 and type 2 grow in number.
C.
Type 1 excludes type 2 from the mixed culture.
D.
Type 2 excludes type 1 from the mixed culture.
103.(2pt)

In a trout hatchery, a density experiment was executed. In figure 24, the eventual density is
plotted as a function of the starting density. The eventual densities were determined before a
new generation of trouts was born in the fish-breeding pond. What is the explanation for the fact
that, for higher starting densities, the eventual density is no longer dependent on the starting
densities?

A.
B.
C.
D.
E.

Accumulation of damaging waste products in the breeding pond.


Exactly compensating density dependent mortality.
Exactly compensating density dependent birth.
Overcompensating density dependent mortality.
Undercompensating density dependent birth.

104.(2pt) In figure 25, the biomass of the macrofauna and the microfauna in various ecosystems is
presented.

Which conclusion can be drawn about the breakdown speed of litter and the accumulation rate of humus.

A.
B.
C.
D.

tundra:
tundra:
tropical rain forest:
tropical rain forest:

breakdown rate of litter


low
low
high
low

accumulation rate of humus


low
high
high
low

105.(2pt) In figure 26, the changes in time of three quantities are shown during the secondary succession
after a forest fire.

These quan

biomass of the vegetation

net primary production

secondary production

net primary production

biomass of the
vegetation

ratio between net primary production


and the biomass of the vegetation

net secondary production

net primary production

net tertiary production

ecosystem respiration

net primary production

Biomass of tertiary producers

106.(1pt)

During the last decades, the concentration of CO2 in the atmosphere has gradually increased.
Which of the following factors does not partly cause this:
A.
cutting down the tropical rain forest.
B.
The combustion of fossil fuels.
C.
The increasing motorized traffic.
D.
The increasing desert forming in the Sahel zone in Africa.
E.
The world-wide increase of the net primary production.

107.(1pt)

Of the following statements, one is not correct. Which one?


A. Interspecific competition between two species always leads to competitive exclusion of one
of the competing species.
B. Niche differentiation occurs with plants as well as with animals.
C. Niche differentiation increases the species diversity in an ecosystem.
D. Self-thinning in plants is the result of intraspecific competition.

108.(2pt)

The decomposition rate of dead material, expressed as the change in dry weight in time, is
often described by means of a negative exponential equation:
mt = m0 . e

-kt

In this equation, mt is the mass at time t, m0 is the mass at time 0 (when the material has just
died off) and k is the decomposition constant. k varies strongly with different types of material.
In which sequences below the value of k is increasing from left to right ?
A.
Wood
- leaves of Prunus (cherry-tree)
- leaves of Fagus (beech)
B.
Wood
- leaves of Fagus
- leaves of Prunus
C.
Leaves of Prunus
- leaves of Fagus
- wood
D.
Leaves of Fagus
- leaves of Prunus
- wood
E.
Leaves of Fagus
- wood
- leaves of Prunus
109.(3pt)

The growth speed of a population can often be described with the logistic growth equation:
dN/dt = rN(K-N)/K

In this equation, N is the number of individuals, r is the intrinsic relative rate, and K is the carrying
capacity.
According to this equation, the equilibrium number of individuals in the population is determined by:
A.
r only
B.
K only
C.
r and K
D.
N and K
E
r and N
110.(2pt) In many experiments it has been observed that the light extinction in a vegetation decreases with
2
2
an increasing Leaf Area Index (LAI : m leaf per m area), in accordance with this equation:
(-k x LAId)

Id = I0 x e

In this equation, Id is the light intensity at depth d in the vegetation (measured from the top!), I0 is
the light intensity above the vegetation, k is the extinction coefficient and LAId the cumulative Leaf
Area Index (measured from the top of the vegetation).
Which statement is not correct ?
k increases with an increasing:
A.
thickness of the leaves
B.
concentration of chlorophyl in the leaves
C.
amount of leaves in the vegetation
D.
reflection of light in the vegetation
111.(2pt)

The curves 1 to 4 in figure 27 are growth curves of bacterial populations under conditions W, X,
Y, and Z.

These conditions are:


Wcontinuous supply of food, water, oxygen, removal of waste products
Xwith time, food is depleted and harmful waste products accumulate
Ywith time, food is depleted and harmless waste products accumulate
Zwith time, food is depleted; as a result of a mutation, a new strain develops that is
capable of metabolizing the waste products secreted by the original strain
Which growth curves and growth conditions do match correctly ?
1
2
3
4
A.
W
X
Y
Z
B.
X
Y
Z
W
C.
Y
Z
W
X
D.
Y
X
Z
W
E.
X
Y
W
Z
End of Part A

Theoretical test (Part B


112.(3pt) Chromosomal DNA, on which gene A is located, is isolated. One of the two
complementary DNA chains is, after separation, paired with gene A mRNA. This
happens under circumstances favouring DNA/RNA hybridisation. After hybridisation a
structure as shown in figure 28 can be observed.

The arrows mark the first and last codon for gene A. Just a small portion of the
flanking DNA is shown. Indicate in the table which chainpart and which number do
match correctly. (ss = single stranded, ds = double stranded).
chainpart:

number:

ss DNA
ss RNA
ds DNA
ds RNA
ds DNA/RNA

113.(4pt) In the model of a gene given below, the organisation of the DNA in and around the
chromosomal "single copy" gene for the polypeptide musculase is shown. Marked are
the initiation and termination sites of transcription, the start and stop codons of
translation and the boundaries of the intron in the gene (). The distances are given in
kilobases (kb), but not drawn on scale.
ini
sta

sto ter

0 1,2 1,7 2,0 5,2 5,8 7,5 8,0


A

Of how many aminoacids does the polypeptide musculase consist? Assume the
musculase does not undergo any posttranslational processing.
Answer:

...........

Of how many nucleotides does the musculase mRNA consist that is used in the
ribosomes? Assume that there is a 100 nucleotides long poly-A-tail connected to the
mRNA at the 3'-terminus before it is operational.
Answer:

.............

114.(4pt) In the wild type of Escherichia coli the polypeptide P consists of 169 aminoacids. The
sequence of the aa 161 to 165 (counted from the N-terminus) is:
161
trp

---

162
his

163
met

164
glu

165
tyr

---

There is a population which has a mutation in the structural gene for polypeptide P, as
a result of which the pp only contains 165 amino acids. The order of the first 161 amino
acids is the same as with the wild type. The order of the others is as follows:
161
trp

---

162
thr

163
tyr

164
gly

165
val

---

Assume in the following questions that the mutant mRNA differs in only one nucleotide
of the wildtpye mRNA.
a.
Give the mRNA codons for the aminoacids 161 to 165 for the wildtype and the
mutant. Use figure 1 for the codons. For each aminoacid only one code will be
the correct one.

B.

amino acid number

161

wild type code


mutant code

UGG
UGG

162

163

164

165

What kind of mutation has happened? Place a tick in the correct column:
substitution mutation
addition mutation
deletion mutation

C.

Wat is the first letter of the aminoacid 166 un the mutant?


Answer:

.............

115.(3pt) Plants can move. We distinguish between nastic movements and tropism.
Give the most common mechanism by which these movements take place, both for
nastic movements and tropism. Choose from the following list:
A.
by means of contractile proteins
B.
by means of elongation of the group of cells involved
C.
by means of restraining cell division of the group of cells involved
D.
by means of stimulating the cell division of the group of cells involved
E.
by means of changing the permeability of the plasmalemma
F.
by means of water absorption of the group of cell involved
Answer:

Tropism:

.......................

Nastic movements :

.......................

116.(3pt) From a one-year-old plant, a growth analysis has been made by measuring a number
of parameters at various points in time. In figure 29, the obtained results are plotted on
the Y-axis, on different scales, against time (X-axis).

Match each of
the parameters with the curves. Place a tick in the correct column.
A

the length growth of the stem


the fresh weight of the whole plant
the dry weight of the whole plant
117.(3pt) Which of the tissues parenchyma, collenchyma and sclerenchyma can have (one of)
the characteristics or functions below? Put a tick in the right column.
Fout! Bladwijzer niet gedefinieerd.

parenchyma

collenchyma

sclerenchyma

secondary lignified cell walls


contain chloroplasts
lacking of protoplasts in matured cells
can become meristematic
cells are always elongated
118.(3pt) In the cases indicated in the table, the water absorption of a plant with maximal
transpiration rate can change. As a result, the plant will absorb less or more water
from the soil, or the water absorption remains the same. Place a tick in the correct
column.

Fout! Bladwijzer niet gedefinieerd.

water absorption
less

same

more

the soil temperature drops from 20 C to 5 C


the soil dehydrates
it starts raining
it gets dark
25% of the leaves are cut off
25% of the roots are cut off
119.(2pt)

Figure 30 shows the pith rays close to the cambium on a tangential cross-section of
the bark of a tree. With this tree, dilatation only takes place in the pith rays.

Quite far from the cambium, the pith rays in the bark, on a tangential cross-section look like
figure:
.........
(Fill in A, B, C or D)
120.(2pt)

The soil of an oat field gets exhausted at the beginning of flowering. Then half of the
field is manured with artificial fertilizer, which contains all elements needed, and in a
form that can easily be taken in by the plants. What will happen with the plants on
that part of the field that was not manured ? Place a tick in the correct column.

Fout! Bladwijzer niet gedefinieerd.

true

false

the leaves sooner turn yellow


the fruits are of less weight
the fruits have lower protein concentration
the fruits have a much lower calcium concentration
121.(2pt)

The following animal phyla are placed in the supposed chronological order in which
they developed during evolution:
1-Protozoa;
2-Porifera;
3-Cnidaria;
4-Platyhelminthes;
5-Annelida;
6-Arthropoda;
7-Chordata.

Fill in which of the phyla mentioned above was the first in evolution in having:
A.
extracellular digestion?
nr:
B.
a nervous system?
nr:
C.
a closed tubular circulatory system?
nr:
D.
a digestive tract with a mouth and an anus separated
nr:
from each other?
122.(4pt)

Fill in which of the phyla from question 121


A.
have a ladderlike nervous system?
B.
belong to the group of the Coelomata?
C.
belong to the group of the Protostomia?
D.
belong to the group of the Bilateria (=Bilateralia)?

nrs:
nrs:
nrs:
nrs:

.......
.......
.......
.......

................
................
................
................

123.(3pt) In the digestive system of the human body, the ingested organic substances are
digested (= broken down) to monomers (= simple substances). Indicate the shortest
way along which one of these monomers, glucose, can reach the kidneys from the
intestine, by putting the numbers of the structures involved in the right sequence.
These numbers are to be chosen from the following list:
1 = jugular veins
7 = aorta
12 = left ventricle
2 = inferior vena cava
8 = renal arteries
13 = carotic arteries
3 = liver
9 = lungs
14 = brain
4 = left atrium
10 = right ventricle
15 = hepatic portal vein
5 = superior vena cava
11 = pulmonary veins
16 = pulmonary arteries.
6 = right atrium
Answer: ............................................................................
124.(2pt)

Indicate in the margin with + or which of the following statements about nutrition
and digestion are true (+) or false ().
A.
In mammals, the protein-hydrolyzing enzyme pepsine is produced in the liver.
B.
In mammals, the production of saliva by the salivary glands is regulated by
nerves originating from the brain.
C.
Gland cells in the wall of the stomach of ruminants produce an enzyme that
digests cellulose.
D.
Which of the amino acids are to be called exogenous/essential amino acids,
as opposed to endogenous/non-essential amino acids, depends on the animal
species under consideration.

125.(2pt) Write down the numbers of the following structures as appearing from Fig.31:
See next page
Fout! Bladwijzer niet gedefinieerd.
A.

distal convoluted tubule:

B.

glomerulus:

C.

collecting tubule (of Bellini):

D.

loop of Henle:

Answer:

126.(2pt) Rh+ blood is transfused into a Rh- woman who has not previously been transfused.
Indicate in the margin with + or which of the following statements are true (+) or false
().
A. Anti-Rh antibodies will be produced by the woman.
B. The transfused blood and the blood of the woman are incompatible, so red cell
agglutination and death may follow.
C. In a subsequent pregnancy, a Rh+ foetus could be threatened by haemolytic
disease.
D. There is no immediate or long term effect as 70% of the Rh+ population are
heterozygous.
127.(2pt) Indicate in the margin with + or which of the following statements about insulin are
true (+) or false ().
A.
Insulin stimulates the conversion of lipids to glucose.
B.
Excess amounts of insulin in the body can cause a state of coma.
C.
Insulin increases the storage of glycogen in the liver.
D.
Insulin increases the blood glucose level.

128.(2pt)

Indicate in the margin with + or which of the following statements about the
female reproductive system are true (+) or false ().
A. Both oestrogen and progesterone are necessary for ovulation to take place.
B. Oestrogen inhibits the production of FSH by the anterior pituitary gland.
C. Fertilization of the ovum by the spermatozoon normally takes place in the
uterus.
D. Progesterone production is largely under the control of LH.

129.(3pt)

Figure 32 shows
changes in the blood
concentration levels of
several hormones
involved in pregnancy,
birth and lactation. The
various curves in the
Figure are labelled with
the letters A to E.

The various hormones involved are indicated in the table below.


Which curve belongs to which hormone? Fill in A, B, C, D or E.

Fout! Bladwijzer niet


gedefinieerd.hormone:

curve

oestrogen
oxytocin
prolactin
progesterone from placenta
progesterone from corpus luteum
130.(2pt)

The following statements are related to the regulation of the processes of moulting
and metamorphosis during the lifecycle of butterflies.
Indicate in the margin with + or which statements are true (+) or false ().
A.
If, early in the second larval stage, the secretion by the corpora allata of
juvenile hormone is inhibited; the next moult will result in a new caterpillar.
B.
If, early in the last larval stage, the secretion by the prothoracic glands of
ecdyson is inhibited; the metamorphosis to a fully developed butterfly will
occur.
C.
If, early in the second larval stage, the secretion by the brain of a hormone
which stimulates the prothoric glands is inhibited; there will be no more
moulting
D.
If, early in the last larval stage, extra juvenile hormone is injected, the next

moult will result in an extra large caterpillar.


131.(2pt) Write down the numbers of the
following structures as
appearing from figure 33.

Fout! Bladwijzer niet


gedefinieerd.A. Eustachian
tube
B.
malleus (hammer)
C
semicircular canals
D.
oval windows

nr:

nr:
nr:
nr:

132(2pt) Write down the numbers of the following structures as appearing from figure 34:

Fout! Bladwijzer niet gedefinieerd.A.


B.
white matter of spinal cord
C.
spinal ganglion
D.
spinal nerve

nr:
nr:
nr:
nr:

133.(2pt) The statements below are partly related to figures 33 and 34.
Indicate in the margin with + or which statements are true (+) or false ().
A.
Bilateral destructions of parts 11, 12, 13 and 14 of figure 33 lead to deafness.
B.
Cutting certain spinal nerves which originate from the neck region of the spinal
cord results in deafness.
C.
Cutting the dorsal roots of spinal nerves which originate from the chest and the

D.

belly regions of the spinal cord results, among others, in the loss of the skin's
sensitivity to tactile contact stimuli in certain regions of the body surface.
Eliminating the left hemisphere of the cerebrum results in the loss of the skin's
sensitivity (=skin senses) mainly in the left part of the body.

134.(2pt) Somebody has a small injury at the retina of the left eye. This injury is situated at the
top of this retina, into the direction of the left ear of this person. He is looking with his
left eye to a paper with a cross and a spot on it. The image of the spot is exactly
projected on the injury of the retina. Choose from figure 35 the correct combination of
the positions of spot and cross on the paper.

Answer (combination):
135.(2pt)

..............

The daily food intake of a 30 year old woman contains 70 g of protein, 70 g of fat and
an unknown amount of carbohydrate.
Assume energy values per gram are 17 kJ for protein, 37 kJ for fat and 16 kJ for
carbohydrate.
A.
How much is the maximum energy yield of the fat in the diet?
Answer: .................. kJ
B.

If the energy needs of the woman are 10800 kJ per day, how much
carbohydrate must she consume to be in energy balance?
Answer: .................. g

136.(2pt) A person eating an entirely vegetarian diet must usually eat a greater mass and
variety of protein per day than a person eating a diet containing animal protein.
Consider the statements A, B, C and D about this. Indicate in the margin with + or
which statements are a correct (+) or incorrect () explanation for the necessary
differences in the two diets.
A.
In general, the diversity of amino acids essential for human beings is not as
great in a plant protein as it is in an animal protein.
B.
In general, the ratio between the amounts of various essential amino acids in
a plant protein differs strongly from that in an animal protein.
C.
In general, the percentage of proteins is lower in vegetable food than in animal
food.
D.
In general, plant proteins are more difficult to be digested and less completely
digestible than animal proteins.
137.(1pt) Figure 36 compares the appearances of two types of cows, A and B.

Which type of cow is better adapted to withstand high temperatures? Fill in A or B.


Answer (Type):...........
138.(2pt) In an experiment in a research station in the tropics both types of cows were moved
between sun and shade conditions. Figure 37 shows the effects of these different
conditions on body temperature and breathing rate.

Indicate for each curve in this Figure which type of cow it represents. Fill in A or B.
Answer

curve 1:

type ........

curve 2:

type ........

curve 3:

type ........

curve 4:

type ........

139.(2pt) Match each of the organisms listed below with its corresponding trophic level
A. earthworm
1. primary producer
B. phytoplankton
2. primary consumer
C. pike
3. secondary consumer
D. grasshopper
4. detritivore
E. frog
5. tertiairy consumer
Answer:

A matches with:

..........(Fill in 1, 2, 3, 4 or 5, etc)

B matches with:

..........

C matches with:

..........

D matches with:

..........

E matches with:

..........

140.(2pt) Match each of the terms with the correct definition.


A mutualism
B commensalism
C cryptic colouration
D aposematic colouration
E mimicry
Answer:

1. camouflage
2. animals with effective defenses are often brightly coloured
3. one partner benefits without significantly affecting the other
4. both partners benefit from the relationship
5. a harmless species resembles an `armed' one

A matches with:
B matches with:
C matches with:
D matches with:
E matches with:
END

.............
.............
.............
.............
.............

(Fill in 1, 2, 3, 4 or 5, etc)

Answer key theoretical test


(Part A)
1
1 - 10

2
2

3
2

C
2

B
2

11 - 20

21 - 30

31 - 40

41 - 50

51 - 60

61 - 70

71 - 80

81 - 90

91 - 100

101 - 110

111

Every question has a score of 1 point with a few exceptions;


these are indicated in superscript.
Questions indicated with '-' were skipped.
Total number of points for part A: 124

2
2

Answer key Theoretical test


112
113A
B
114A

B
C
115
116

117

118

119
120

121A
B
C
D
122A
B
C
D

(Part B)

ssDNA 2,4,6
ds DNA/RNA 1,3,5,7
300
3200
162 163
164 165
CAU AUG GAG UAU
ACA UAU GGA GUA
addition
U
T B
N F
A
B
C
p - s
p c - s
p c c less
less
less
less
less
same
skipped
true
true
false
false
3
3
5
5
4, 5, 6
5, 6, 7
(4), 5, 6
4, 5, 6, 7

Total number of points for part B: 63

123
124A
B
C
D
125A
B
C
D
126A
B
C
D
127A
B
C
D
128A
B
C
D
129

skipped
+
+
9
5
10
7
+
+
+
+
+
+
C
D
E
B
A
130A B
C
+
D
+
131A 15
B
3
C
6, 7
D
17
132A 10
B
13
C
5
D
2

133A
B
C
D
134
135
136A
B
C
D
137
138

139

140

+
+
B
skipped
+
+
+
A
B
A
B
A
4
1
5
2
3
4
3
1
2
5

INTERNATIONAL BIOLOGY OLYMPIAD


PRACTICAL PROBLEMS
1993, Utrecht, Netherlands















All IBO examination questions are published under the following Creative Commons license:



CC BY-NC-SA (Attribution-NonCommercial-ShareAlike) https://creativecommons.org/licenses/by-nc-sa/4.0/
The exam papers can be used freely for educational purposes as long as IBO is credited and
new creations are licensed under identical terms. No commercial use is allowed.

PRACTICAL TEST
4th International Biology Olympiad 1993, Utrecht, the Netherlands
The test consists of five sections: I to V.
There are two groups of participants.
Group A: participants with nos. 1 - 30;
Group B: participants with nos. 31 - 60.
There are five halls, numbered 1 - 5.
You will not do all five sections of the test in the same hall. There are guides to tell
you where you are supposed to go and where your seat is. You are not allowed to
leave before the end of each section.
Group A will do the practical test in the following sequence:
Section I in hall 1
(60 mins.)
Section II in halls 2 and 3 (60 mins.)
Section III in hall 1
(45 mins.)
break
(15 mins.)
Section IV in hall 4
(45 mins.)
Section V in hall 5
(45 mins.)
The sequence for group B is as follows:
Section II in halls 2 and 3 (60 mins.)
Section III in hall 1
(45 mins.)
Section IV in hall 4
(45 mins.)
break
(15 mins.)
Section V in hall 5
(45 mins.)
Section I in hall 1
(60 mins.)
The maximum number of points to be obtained amounts to a total of 201.
The maximum points to be gained from sections I - V separately are as follows:
Section I:
40 points
Section II:
41 points
Section III: 45 points
Section IV: 39 points
Section V:
36 points
The maximum score for each separate question is indicated between brackets in the
margin of the answer sheets.

1
IBO-1993 - Practical Test

PRACTICAL TEST
4th International Biology Olympiad 1993
Section I: The reaction of nematodes to manuring. (60 mins., hall 1)
Some 80 % of all multicellular animals on earth are nematodes. One kilogramme of
soil contains an average of 30,000 individuals, representing some 30 to 60 different
species.
You are going to analyze the effects of manuring on these nematodes.
Preliminary research showed that a certain type of poor sandy soil contained the
following types of nematodes:
plant parasites (pp)
bacteria eaters (ba)
fungus eaters (fu)
carnivores (ca) and
omnivores (om).
Dehydrated nematode-free cattle manure was added to this sandy soil. Six days
later, a second sample was taken for nematode fauna analysis. We now have two
samples, one taken before manuring, the other afterwards. The table below shows
the composition of the nematode fauna in percentages before manuring:
Name
Acrobelus
Aphelenchoides
Diplogaster
Eudorylaimus
Hemicycliophora
Prionchulus
Rhabditis
Rotylenchus
Trichodorus
Tylenchus
Others

Food
ba
fu
ba
om
pp
ca
ba
pp
pp
pp

group %
2
46.5
2
4.5
1
1.5
3
11.0
3
6.0
3
3.5
1
3.0
3
9.5
3
2.0
2
5.5
7.0

Each name is followed by the type of food and a number from 1 to 3. This number
gives information about the life strategy of the species in question. The following
characteristics are distinguished:
Group 1 Enrichment opportunists.
These nematodes can exploit new food sources fast.
They have a life cycle of several days.
They produce numerous small eggs.
They are relatively insensitive to stress.
They are transported by insects.
They are found in soil with high food density and show dormancy periods
(dauerlarvae) when the food supply decreases.
Group 2 General opportunists.
These nematodes have a longer life cycle than the enrichment opportunists.
They are susceptible to stress but are not found in poor soil.
Group 3 Persisters.
These nematodes have a life-span of several months.
The produce few but large eggs.
They are relatively sensitive to stress.

2
IBO-1993 - Practical Test

The relationship between 'enrichment opportunists' (1), 'general opportunists' (2)


and 'persisters' (3) is indicative of changes in the ecosystem of the soil.
The distribution among the groups in percentages:
Group 1 4.5% (3.0% Rhabditis + 1.5% Diplogaster), group 2 = 56.5% and group 3 =
32.0%. This makes a total of 93%. Allowing for the percentage of non-identified
nematodes (7%), we arrive at a composition of 100/93 x 4.5 = 4.8% for group 1,
61.8% for group 2 and 34.4% for group 3.
We can construct a right-angle triangle graph on the basis of the data for group 1
(4.8%) and group 3 (34.4%), producing the point indicated in fig. I.1 (see answer
sheets).
Task
Your task is to analyze the nematode fauna (six days) after manuring. You are then
required to compare your data with the situation before manuring, and interpret any
changes that have occurred.
To this end, you will be given a microscopic specimen amply provided with
nematodes.
-

Find the nematodes with the small magnification, then use the 40x-lens
and identify them on the basis of the pictures supplied.
Return to the small magnification. Begin on the left hand side of the
specimen and work through the specimen systematically and
representatively until you have identified 40 nematodes. Allow at least 45
minutes for this task. Keep a record of your observations.

Process your findings as follows:


1
Write down your answers on the answer sheet in numbers and
percentages.
2
Calculate the percentage ratio of the three groups of nematodes on the
basis of their life strategies, and enter your answer in the answer sheet.
3
Then indicate these percentages in fig. I.1 on the answer sheet.
4
Finally, answer questions I.1 - I.7.

3
IBO-1993 - Practical Test

4
IBO-1993 - Practical Test

Answer sheets section I


(20)

(3)

Results of counting and percentage calculation:


Name

Number

Acrobelus

.........

.........

Aphelenchoides

.........

.........

Diplogaster

.........

.........

Eudorylaimus

.........

.........

Hemicycliophora

.........

.........

Prionchulus

.........

.........

Rhabditis

.........

.........

Rotylenchus

.........

.........

Trichodorus

.........

.........

Tylenchus

.........

.........

Unidentified

.........

.........

Ratio of the three groups of nematodes in %


Group

(2)

.........

.........

.........

Indicate these percentages with a point in fig. I.1.

5
IBO-1993 - Practical Test

Mark the correct answers to the following questions.


There is only one correct answer to each question.

(2)

I.1

In the triangle (fig. I.1) the point has shifted in a certain direction.
What does this indicate?
O Larger presence of predators
O Increased food supply
O Stress
O Increase in number of nematodes
O Shift within food types

(2)

I.2

Which food type has benefited most from the manure?


O Fungus eaters
O Plant parasites
O Carnivores
O Bacteria eaters
O Omnivores

(1)

I.3

The number of which species has increased most, in absolute terms?


O
Prionchulus
O
Acrobeles
O
Rhabditis
O
Tylenchus
O
Rotylenchus

(2)

I.4

Looking at the adult nematodes, it is obvious that they reproduce quickly.


How can you tell?
O
Many females with eggs
O
Comparatively more adult individuals
O
Darker intestines
O
Smaller females
O
Swollen gullet

(2)

I.5

After the experiment, the total number of nematodes has doubled.


Calculate the reproduction factor r (final density/initial density) for Rhabditis.
O
r is smaller than 1
O
r is between 1 and 3
O
r is between 3 and 10
O
r is higher than 10
O
One of the density figures is 0

I.6

In absolute terms, the number of persisters (group 3) declines.


One possible cause of this is that
O
They are affected by fungi which do not distinguish between manure and
nematodes
O
They are eaten by carnivorous nematodes which have begun to develop
in huge numbers
O
They are sensitive to substances like ammonia, produced in the soil.
O
The food of the persisters decreases
O
They lose the competition for space in the soil

(3)

6
IBO-1993 - Practical Test

(3)

I.7

Nematodes are fairly resistant to agitation (ploughing!) of the soil, as long as it


is not too dry. If we take one kilogramme of good agricultural soil, sift it well
(through a 5 mm sieve) and then leave it for a week, what can we expect?
O
No changes: nematodes are quite resistant to agitation
O
Increase of group 1 due to increased microbial activity
O
Decrease of group 1 due to decreased microbial activity
O
Decrease of group 1 as a result of physical disturbance
O
After a week no persisters will be found, since they are left in the sieve.
End of section I

7
IBO-1993 - Practical Test

PRACTICAL TEST
4th International Biology Olympiad 1993
Section II:

The effects of nitrogenous fertilization on the individual plant.


(60 mins., halls 2 and 3)

The plants used for this experiment, the agricultural crop maize (Zea mays), were
grown in a greenhouse under different nitrogen conditions. These nitrogen levels are
indicated with L (low) and H (high). The levels of all other nutriments were the same
for both test groups. In this section, you will study the resulting growth differences.
For this purpose, you will have to
A
weigh
B
determine leaf surface
C
determine root length
You will do this in various places in halls 2 and 3.
There are not enough practical test sets for all of you. The following are available:
5 scales for weighing (takes very little time)
10 sets for determining leaf surface (takes little time)
20 sets for determining root length (takes a bit longer)
It makes no difference which set you use.
Plan your time in such a way that you can do all the tasks.
As you also need time to process your findings, you will hardly have to wait for each
other, if at all.
Task IIA
A number of plants were harvested before this Olympiad. The leaves, stems and
roots of these plants have been collected and dried separately. This is the dried
material that you will find in the plastic pots and cups. Each pot contains leaves,
stems or roots of three plants. The net weight of each pot is indicated on the
outside.
-

Determine the dry weight of the leaves, stems and roots of three plants with
an (electronic) scale.
Do this for both nitrogen levels.
Write the code (LA ... or HA ...) of the pots used and the average dry weight
for each plant on your answer sheet.
Now calculate the ratio between the average dry weight of the shoot
(leaves plus stem) and the average dry weight of the roots as follows:
leaves + stem
root

For maximum growth under fixed nitrogen conditions, the plant could
distribute its biomass across shoot and root in several ways. This would
result in changes in the shoot/root weight ratio. On the answer sheet, a
number of possible explanations are suggested for the differences found.
Indicate in the margin with + or - which of these suggestions can and which
cannot serve as possible explanations.

Task IIB
Apart from the weight of a plant, the leaf surface is also an important factor. One
way of determining it is to measure the lengths of the leaves of a plant and the
widths at half the length of each leaf. The average leaf surface can then be
calculated as follows:
length x width x 0.761.

8
IBO-1993 - Practical Test

The leaves of plants grown at two different nitrogen levels have been placed
under transparent plates. For each nitrogen level, determine the total leaf
surface of one plant.
On the answer sheet, enter both the code of the plants measured (LB ... or HB
...) and the results.

The fresh weight of the leaves is indicated for each individual plant under the plates.
Another experiment showed that the dry weight/fresh weight ratio is equal to:
0.13 for leaves of the high-nitrogen plants
0.15 for leaves of the low-nitrogen plants.
On the basis of these data, the calculation carried out in the first part of task
IIB and the results of task IIA, calculate the average leaf surface of the highnitrogen (H) and the low-nitrogen (L) plants you weighed for task IIA.
Task IIC
Another important factor besides leaf surface is root length. It is determined as
follows. A representative piece of root is placed in a large Petri dish filled with water.
The dish is placed on a grid showing parallel horizontal and vertical lines at fixed
intervals. The larger the number of crossings of root and grid lines, the larger the
root length. The formula for root length calculation is the following:
Root length = the number of crossings x correction factor.
The correction factor for the grids used is 0.987.
20 grids are available. Choose one of them.
Determine the total root length of a piece of root, for both high-nitrogen and
low-nitrogen plants. On the answer sheet, enter the code of the roots (LC ...,
HC ...) and your findings.
Data on the fresh weight of the roots you have weighed are supplied. The dry
weight/fresh weight ratio is equal to:
0.09 for high-nitrogen roots
0.10 for low-nitrogen roots.
On the basis of these data, the calculation carried out in the first part of task
IIC and the results of task IIA, determine the average root length of the highnitrogen (H) and the low-nitrogen (L) plants you weighed in task IIA.
Now calculate, for both high-nitrogen and low-nitrogen levels, the ratio
between average leaf surface and average root length for the plants weighed
in task IIA.
You have now calculated two indexes for the plants from task IIA:
dry weight of the shoot
dry weight of the roots
(task IIA), and
total leaf surface
total root length
-

(task IIC).

Of course, there are physiological reasons that determine the distribution of


the biomass across the various organs of the plant.
Which of the indexes you have determined gives the most information on this
point?
On the answer sheet, you will find a number of arguments to justify the choice
of one of the two indexes above. Indicate which of these suggestions are good
arguments for the particular choice they support, and which are not.

9
IBO-1993 - Practical Test

Answer sheets section II


Task IIA
(4) 1
Average dry weight per plant:
L (code LA ...)
leaves
..........
stem
..........
roots
..........

H (code HA ...)
..........
..........
..........

(2)

Ratio between average dry weight of shoot (stem + leaves) and average
dry weight of roots:
Low nitrogen level: ...............
High nitrogen level: ...............

(7)

Which of the statements below could be adequate explanations for the


differences in weight ratio found?
Indicate your views by putting + (correct explanation) or - (incorrect
explanation) in the margin.
a
Under low nitrogen conditions, the roots take in too little amino
acid to allow fast growth. As a result, the plant will have to invest
more in root development.
b
Low nitrogen conditions slow down the photosynthesis process.
Under such conditions, a plant will have to invest more in leaf
development in order to maintain a sufficient level of
photosynthesis.
c
When there is more nitrogen, the plant contains more proteins, as
a result of which less of its biomass requires other carbon
products. Therefore, under high nitrogen conditions the plant has
to invest less in leaves and more in its roots in order to guarantee
maximum growth.
d
Under high nitrogen conditions the plant will produce more
enzymes for photosynthesis, which, consequently, will increase.
This means that the stomas will have to be opened further and that
more water will evaporate. This water is absorbed by the roots
system. As a result, the plant will have to produce more roots.
e
Under high nitrogen conditions a few roots suffice for adequate
nitrogen intake. As a result, the plant will invest primarily in leaf
development.
f
High nitrogen levels stimulate root development, since the roots
are the first organs to be able to make use of the extra nitrogen.
g
The distribution of biomass across shoot and root is determined
genetically for each individual. For this reason, nitrogen conditions
only influence the growth of the plant as a whole. Consequently,
the differences in shoot/root ratio should be interpreted as genetic
differences, and have no function.

(3)

(5)

Task IIB
1 Total measured surface of the leaves.
Treatment
Leaves surface (cm2)
L (code LB .......)
....................
H (code HB .......)
....................
2 Total calculated average surface of the leaves from task IIA:
Treatment
Average leaf surface (cm2)
L
....................
H
....................
10
IBO-1993 - Practical Test

Task IIC
Total measured length of representative piece of root.
Treatment
Total length of the roots (cm)
L (code LC ......)
....................
H (code HC ......)
....................

(6)

(4)

Total calculated average root length of the plants from task IIA:
Treatment
Average root length (cm)
L
....................
H
....................

(2)

Calculated ratio between average leaf surface and average root length
of the plants from task IIA:
Treatment
average leaf surface
average root length
L
..................
H
..................

(2)

Which index gives the most information? (mark the correct answer)
O
shoot weight/root weight
O
leaf surface/root length

(6)

Which of the arguments below concerning the indexes are correct, and
which are not? Indicate your views by placing + (correct) or - (incorrect) in
the margin.
a

The rate of photosynthesis is directly proportional to the biomass of


the shoot. The rate of respiration is directly proportional to the
biomass of shoot plus root. This is why the shoot weight/root weight
index provides the most information.
The absorption of CO2 by the leaves and of nutriments and water by
the roots are processes that take place at surfaces.
For this reason, the leaf surface/root surface ratio would be the best
index. The leaf surface/root length ratio is an acceptable alternative,
however, and therefore the best of the two indexes given.
Under certain conditions (for instance drought), leaves and roots may
shrink. When they do, leaf surface or root length are no longer very
helpful data. Therefore it is wiser to work with the shoot weight/root
weight index.
The leaf surface/root length ratio gives no information whatsoever
about the investment in the biomass of the stem. As a result, this
index cannot serve as a reliable indicator of growth. This does not
apply to the ratio between shoot weight/root weight, which is therefore
a better parameter.
Leaf surface is a two-dimensional measure, whereas root length is
one-dimensional. As a result, the two cannot be simply combined to
calculate an index. This problem does not arise in the calculation of
the shoot weight/root weight ratio, which is therefore a better
parameter.
Since the shoot weight and the root weight both increase under high
nitrogen conditions, the leaf surface/root length ratio is a better
measure.
End of section II

11
IBO-1993 - Practical Test

PRACTICAL TEST
4th International Biology Olympiad 1993
Section III: The effects of nitrogenous fertilization on the anatomy of the plant
(45 mins., hall 1)
In section II of the practical test you studied maize plants grown at two different
nitrogen levels. For each of these levels, you will now make a cross-section of a
piece of stalk.
The stalks have already been prepared for you for this purpose.
Dye your cross-sections with a solution of iodine in potassium iodide.
Study the anatomical structure of the stalk and the effects of nitrogenous fertilization
on this structure.
-

Study fig. III.1 (supplied on a separate page).


It represents a picture of part of a maize plant stem. The numbers 1 - 9 refer to
the cell types, the letters a - h indicate the tissues.
You are required to name these cell types and tissues.
Use names in list III.A for this purpose.
Enter the number of your choice on the answer sheet.
Also use the cross-sections you cut yourself to complete this task.
Plants grown at high and low nitrogen levels show anatomical differences.
Study these differences on the basis of the cross-sections you have prepared.
Try to find five striking differences between the two cross-sections.
In doing so, pay attention to cell types, tissues or parts of cell types.
Indicate the location of these differences on the answer sheet with the help of
list III.A.
Then indicate the nature of the differences between high and low nitrogen
conditions. For this purpose, use the letters from list III.B.
Attention:
there may be several differences within a single (part of a) cell
type.
Make sure that you also indicate these multiple differences on the answer
sheet.
On the answer sheet, several statements are presented as possible
explanations for the differences found. Indicate which of these are correct and
which are not.

12
IBO-1993 - Practical Test

LIST III.A
1.
2.
3.
4.
5.
6.
7.
8.
9.
10.
11.
12.
13.
14.
15.
16.
17.
18.
19.
20.
21.
22.
23.
24.
25.
26.

Apical Meristem
Lateral Meristem
Cambium
Epidermis
Hypodermis
Exodermis
Cortex
Stele
Pith
Pith Ray
Vascular Bundle
Protophloem
Metaphloem
Secondary phloem
Protozylem
Metaxylem
Secondary Xylem
Parenchyma
Collenchyma
Sclerenchyma
Sclerenchyma Fibre
Scleride
Vessel
Sieve Tube
Sieve Cell
Companion Cell

27.
28.
29.
30.
31.
32.
33.
34.
35.
36.
37.
38.
39.
40.
41.

Subsidiary Cell
Xylem Fibre
Phloem Fibre
Xylem Parenchyma
Phloem Parenchyma
Tracheaid
Bundle Sheath
Pit Pair
Chloroplast
Starch
Diameter Cell
Intercellular Space
Primary Cell Wall
Secondary Cell Wall
Lignin

LIST III.B
A.
B.
C.
D.
E.
F.
G.
H.
I.
J.

present
absent
thicker
thinner
larger
smaller
larger amount
smaller amount
there are more cell layers
there are fewer cell layers

13
IBO-1993 - Practical Test

Answer sheet section III


(12)

Cell types, tissues

Cell type
(in figure III.1

Number
(in list III.A

Tissue
(in figure III.1

(25)

Number
(in list III.A)

Differences
Cell type/Tissue/Cell part
(number from list III.A)

Difference at high level


(letters from list III.B)

Indicate with a + or - in the margin which statements can (+) and which cannot (-)
serve as possible explanations for the differences.
(2)

(2)

(2)

Plants which are cultivated under high nitrate conditions (high nitrate
plants) are able to form many new structures. In doing so, they use up
photosynthesis products, preventing the build up of starch in the plants.
Little energy is wasted on the storage and maintenance of reserve
materials in these plants. Low nitrate plants, on the other hand, need so
much energy for this that less energy is left over for growth.
A low starch content in high nitrate plants suggests that the processing
capacity is fairly equal to the photosynthesis capacity. Furthermore, it
appears that the photosynthesis occurs at a faster rate in these plants
than in low nitrate plants.
The slower rate of growth in low nitrate plants is the result of the smaller
amount of carbohydrates available for redistribution throughout the plant.
The slower rate of photosynthesis in low nitrate plants is a result of the
fact that there are fewer proteins and fewer leaves because more roots
have been formed. Because of this relatively small amount of "green"
biomass, the capacity for photosynthesis is relatively small. Conversely,
the larger root system leads to higher respiration. This causes the lower
16
IBO 1993 Practical Test

(2)

growth rate of low nitrate plants.


Low nitrate plants, while having less capacity for photosynthesis,
nevertheless have a high starch content. Therefore, this product of
photosynthesis is not used for growth. It follows that not photosynthesis
but another process that depends on the nitrate supply is the limiting
factor for growth.
End section III

17
IBO 1993 Practical Test

PRACTICAL TEST
4th International Biology Olympiad 1993
Section IV:

Effects of nitrogen fertilization on species interaction


(45 min., room 4)

In this section we compare the reaction of two plant species to differences in


nitrogen levels (nitrate and/or ammonium):
the grass Lolium perenne and the herb Plantago lanceolata.
Lolium perenne (Gramineae) is a high-growing grass (see fig. IV.1a) and is very
often found in highly-fertilized agricultural grasslands.
Plantago lanceolata (Plantaginaceae) grows particularly in grasslands and
roadsides poor in nutriments. It is a rosette plant; a herb without leaves on its stem
(see fig. IV.1b).

Caution: For task IV.B you must take measurements with costly light meters of
which we only have a limited number. You will have to take turns. Pay
attention to each other and manage your time so that you can take a turn.

18
IBO 1993 Practical Test

Task IV.A
The increase in biomass for both species under the influence of two nitratefertilization levels is given in figure IV.2:

We are concerned with the dry weight in g per pot.


The two species were grown in separate pots.
All pots are identical and rectangular: 9 cm x 10 cm.
-

Calculate what the end yield is in dry weight per m for Lolium as well as for
Plantago based on the data from the graph.
Do this for the unfertilized and the fertilized case and fill in the calculated values
on the answer sheet.

Task IV.B
In the room, there are a number with trays of vegetation from grasslands with a high
(tray 1) and with a low (tray 2) nitrate-fertilization level.
Measure the luminosity (intensity of light) at different heights with the available
light meters. Measure every 10 cm starting at soil level.
Take care that the vegetation in the trays remains intact.
Make a graph on the answer sheet reflecting your findings in percentages of
the original light. Choose your axes yourself.
Rosette plants only grow moderately in the lower 10 cm of vegetation in the
case of nitrogen fertilization. A number of possible explanations for this are
given on the answer sheet. Indicate which suggestions can or cannot serve as
explanations.

19
IBO 1993 Practical Test

Task IV.C
The outcomes of a competition experiment between species 1 and species 2 are
given in figure IV.3.

In this experiment the species were cultivated in monoculture (with 10 plants of one
species per pot) as well as mixture (with 5 plants of species 1 and 5 of species 2 per
pot) during a growing season. The relative yield of the mixture with regard to the
monoculture is shown for species 1 and species 2 in this graph.
-

The line for species 2 bends upwards and the line for species 1 bends
downwards.
A number of suggestions as to why this is so are given on the answer sheet.
Indicate which suggestions can and which cannot serve as possible
explanations.
Finally, indicate on the answer sheet which of the two species is a rosette
plant.
End of section IV

20
IBO 1993 Practical Test

Answer sheets for section IV


Task IV.A
(3)

End yield (dry weight in g per m)


Plant
+ N: high nitrate level

- N: low nitrate level

Lolium Perenne

...................

....................

Plantago lanceolata

...................

....................

(12)

Task IV.B
Graph

21
IBO 1993 Practical Test

Task IV.B continued


(12)

(8)

(4)

Indicate with a + or - in the margin which statements can (+) and which cannot (-)
serve as possible explanations.
a

Rosette plants (for example Plantago) cannot grow better under high nitrate
fertilization conditions because the nitrate inhibits photosynthesis and,
consequently, growth.

Rosette plants are able to grow faster under low nitrate conditions because
more rain water reaches the soil.

The leaves of rosette plants receive little light in the case of high nitrogen
fertilization and therefore are only able to grow moderately.

Rosette plants develop a compact, stocky form in the case of high luminosity.
Under high nitrate fertilization conditions they receive less light and therefore
show higher growth and take on an ascending form.

Rosette plants dry out more quickly in the case of nitrogen fertilization
because all the rainwater reaches the soil by way of grass shoots, therefore
benefiting the grass.

The leaves of rosette plants are low in the vegetation and are overshadowed
by the longer grasses after fertilization, so that they are less able to
photosynthesize.

The leaves of rosette plants are low in the vegetation. Little CO2 is available
there because it is used up by the higher grasses. For this reason less
photosynthesis occurs.

Task IV.C
Indicate with a + or - in the margin which statements can (+) and which cannot (-)
serve as possible explanations.
a

species 1 grows better in mixture than in monoculture because there is less


intraspecific competition than interspecific competition

species 1 grows better in monoculture than in mixture because there is less


intraspecific competition than interspecific competition

species 2 grows better in monoculture than in mixture because there is less


intraspecific competition than interspecific competition

species 2 grows better in mixture than in monoculture because there is more


intraspecific competition than interspecific competition

in both cases, species 1 and species 2, intraspecific competition is the most


prevalent

The rosette plant is species: ............

(fill in: 1 or 2)

End of section IV

22
IBO 1993 Practical Test

PRACTICAL TEST
4th International Biology Olympiad 1993
Section V:

Nitrogen balance of a fertilized and an unfertilized meadow


(45 min., room 5)

Task V.A
The nitrogen cycle for a West European grassland fertilized with nitrogen is given on
the answer sheet.
In figure V.1, a number of processes c.q. phenomena are missing.
Complete the schematic drawing.
Name the processes (phenomena) which are missing from the cycle.
Choose from the provided list of processes (phenomena).
Write the corresponding numbers in the correct places in figure V.1.
Task V.B
There are a number of water samples (samples 1, 2 and 3) in the room.
One sample is collected rainwater. The other two samples are both groundwater
collected at a depth of 50 cm. One of the samples is from an unfertilized natural
grassland and the other from a nearby agricultural grassland fertilized with nitrogen.
From each sample determine:
a) the acidity (pH) using the strip method.
b) the nitrate content using the strip method.
c) the ammonium content using the drip method.
The strip method is simple: dip, compare with the colouring chart and match
with a colour comparison value.
The drip method for ammonium is as follows:
1
Rinse the measuring vessel with the water to be tested and fill to the 5
ml mark.
2
Add 10 drops of reagent 1 and swirl.
3
Add 1 microspoon of reagent 2. Dissolve by swirling and wait or set
aside for 5 minutes.
4
Add 6 drops of reagent 3 and swirl.
5
After 5 minutes, place the measuring vessel on the colouring chart and
match with a colour comparison value.
-

Fill in your measurements on the answer sheet. Pay attention to the units of
measurement.
Fill in on the answer sheet where each sample comes from
For both grasslands, calculate the amount of run-off ammonium + nitrate per
m.
Do this using the fixed concentrations in the water samples and considering
the fact that the rainfall balance (precipitation minus evaporation) is 300 mm.
1 mm precipitation corresponds with 1 litre of rainwater
per m. Fill in your answers on the answer sheet.

23
IBO 1993 Practical Test

Task V.C
Two photographs of grassland A and grassland B are shown in fig. V.2.

Which grassland has the greatest variety of species?


Which grassland has been fertilized with nitrate and ammonium the
most?

24
IBO 1993 Practical Test

In many countries in Europe and America extra nitrate and ammonium is added to
natural areas by "acid rain".
On the answer sheet there are some descriptions of what could happen to the
variety of plant species in originally nutrient-poor nature reserves when extra
nitrate and ammonium are added by way of "acid rain". Indicate which
descriptions are correct or incorrect.

25
IBO 1993 Practical Test

Answer sheet section V


Task V.A

Schematic drawing of the nitrogen cycling in an agricultural grassland in Western


Europe.

Complete the figure. Choose the correct processes/phenomena out of the list and
write the corresponding numbers in the right places in the figure.
List of phenomena (task V.1)
1)
2)
3)
4)
5)
6)
7)
8)
9)
10)
11)

denitrification
mineralization
N-fixation
acid rain
nitrification
sulphate reduction
herbivore
methanogenesis
ammonia volatilization
soil leaching
run off

12)
13)
14)
15)
16)
17)
18)
19)
20)
21)
22)

ammonification
harvest
photosynthesis
assimilation
volcanic emission
soil weathering
soil fixation
N allocation
transpiration
respiration
retranslocation

26
IBO 1993 Practical Test

(8)

Task V.B
The measured values are (pay attention to units of measurement):

sample 1
sample 2
sample 3
(3)

(6)

pH

Ammonium

Nitrate

........
........
........

.............
.............
.............

............
............
............

The numbers of the three water samples are:


sample number
rainwater

........

fertilized grassland

........

unfertilized grassland

........

The amount of run-off ammonium + nitrate is:


in the unfertilized grassland:

...............

in the fertilized grassland:

...............

(1)

Task V.C
The grassland with the greatest variety of species is: .............
(fill in A or B)

(1)

The grassland that has been fertilized with nitrates and ammonium the most is:
...........
(fill in A or B)

(10)

Indicate with a + or - in the margin which descriptions can (+) and which cannot (-)
serve as possible explanations.
a

the variety of plant species does not change as a result of extra nitrogen
compounds in the air, because in almost all cases, phosphor limits growth
and, therefore, no overshadowing by extra growth can occur
the variety of plant species will decrease, because a number of fast-growing
plant species (usually grasses) will increase, and the amount of light close to
the ground is strongly limited causing many low-plant species to disappear
the variety of species will only increase if the initial conditions were very poor
in nutriments, as a result of which hardly any plant species could grow.
Because of the extra nitrogen boost, more plant species can grow in the
beginning and, thus, the variety of species will increase.
the variety of plant species will not change, because all extra nitrogen
compounds conveyed from the air, under dry as well as wet environmental
conditions, are fully denitrificated and are released into the atmosphere as
nitrogen gas
the variety of species will not change because the soil has the effect of a
buffer, so that the concentration of nitrogen compounds available to the plant
is kept practically constant

28
IBO 1993 Practical Test

INTERNATIONAL BIOLOGY OLYMPIAD


PRACTICAL PROBLEMS
1992, Slovakia















All IBO examination questions are published under the following Creative Commons license:



CC BY-NC-SA (Attribution-NonCommercial-ShareAlike) https://creativecommons.org/licenses/by-nc-sa/4.0/
The exam papers can be used freely for educational purposes as long as IBO is credited and
new creations are licensed under identical terms. No commercial use is allowed.

INTERNATIONAL BIOLOGY OLYMPIAD


THEORY PROBLEMS
1991, Russian Federation















All IBO examination questions are published under the following Creative Commons license:



CC BY-NC-SA (Attribution-NonCommercial-ShareAlike) https://creativecommons.org/licenses/by-nc-sa/4.0/
The exam papers can be used freely for educational purposes as long as IBO is credited and
new creations are licensed under identical terms. No commercial use is allowed.

II INTERNATIONAL
BIOLOGY
OLYMPIAD

1. How many triplets of DNA nucleotides code 20 amino acids?


a) 20; b) 61; c) 64; d) 4.
2. The above reaction is catalysed by:
a) dehydrogenase;
b) decarboxylase;
c) oxidase;
d) hydrolase.
3. The function of the so-called "nonsense" codons is:
a) to code each one for several amino acids;
b) to enable the mRNA to attach itself to any ribosome;
c) to code each one for the proper amino acid;
d) to indicate the end of the coded message in the mRNA.
4. A peptide bond is formed between:
a) phosphate and carboxyl;
b) carboxyl and amino;
c) alcohol and aldehyde;
d) aldehyde and amino.
5. Cellulose and glycogen are both:
a) polysaccharides;
b) glucose and glucose-1-phosphate polymers;
c) polynucleotides;
d) disaccharides.
6. The sequence of nucleotides of mRNA is complementary to the sequence of
nucleotides of:
a) the 2 strands of a DNA molecule;
b) one of the strands of a DNA molecule;
c) the molecule of tRNA;
d) all the tRNA molecules.
7. The DNA triplet GCT is complementary to the tRNA anticodon:
a) GCT; b) CGA; c) UGC; d) GCU; e) GCT.
8. Bacteria cultured in an environment containing light nitrogen isotopes (14N) were
transferred to an environment containing heavy nitrogen (15N) for a period corresponding to
one replication and then they were brought back to the initial environment. The analysis of
bacteria DNA composition made after a period corresponding to two successive replication
showed that correlation of 14N/15N in DNA was:
a) 1:1; b) 2:1; c)4:l; d) 8:1.
9. All procaryotic cells have:
a) ribosomes and mitochondria;
b) a plasma membrane and vacuoles;
c) a plasma membrane and nuclear membrane;
d) a plasma membrane and ribosomes.

10. Any kind of cell has a plasma membrane built of:


a) lipids and proteins;
b) lipids only;
c) proteins only;
d) lipids and polysaccharides.
11. At the time when a cell bagging to divide, it has N chromosomes and Q DNA.
What DNA quantity and chromosome number is expected in each daughter cell
after mitosis?
a)NandQ; b) N/2 and Q/2; c) N and Q/2; d) N/2 and Q.
12. Molecules of water get to the leaves by the following way:
a) stomata > mesophyll > xylem; b) xylem > mesophyll > stomata;
c) phloem > xylem > mesophyll; d) phloem > mesophyll > stomata.
13. Meiosis produces:
a) gametes in all cases; b) spores in all cases;
c) spores and gametes in both animals and plants;
d) usually spores in plants and gametes in animals.
14. Antibodies are synthesised:
a) in macrophages only;
b) in lymphocytes only;
c) in macrophages and lymphocytes; d) in macrophages and neutrophils.
15. Secretion of the follicle-stimulation hormone (FSH):
a) affects the ovary only; b) stimulates follicles growth in the ovary;
c) is controlled by the hypothalamus and the ovary;
d) is not known for man.
16. A boy belongs to the 0 blood group, his mother - to the A blood group, and his father - to
the blood group. The probability of his sister belonging to the same type is:
a) 1/16; b) 1/8; c) 1/4; d) 1/2.
17. Autosomes are:
a) nonsex-chromosomes;
b) cells which can reproduce themselves;
c) organelles which can reproduce themselves; d) sex-chromosomes.
18. The certain species of fungus grow on animal facals. How this type of feeding is called?
a) parasitism; b) commensalism; c) saprophytes; d) symbiosis.
19. The human forearm and the bird's wing have the same structure. This suggests that:
a) birds are derived from mammals;
b) birds and mammals have common ancestors;
c) mammals are less adapted to their environment than birds;
d) birds and mammals are well adapted for flying.
20. People with type 0 blood are:
a) universal donors;
b) universal donors and recipients;
c) universal recipients; d) unable to give their blood for any blood transfusion.
21. Insulin injecting into human blood:
a) reduces glycogen synthesis;
b) increases muscles consumption of glucose;
c) reinforces glycogen destruction;
d) increases the blood glucose content.

22. Lung ventilation is necessary to maintain concentration of carbon dioxide and oxygen of the
following relation:
C02
02
a) high high;
b) low low;
c) low high;
d) high low.
23. In a plant cell the organelles separated by two membranes from cytoplasm
are:
a) the nucleus only; b) the mitochondria and the plastides only;
c) the nucleus, the mitochondria and the plastides;
d) the mitochondria, the lysosomes and the plastides.
24. What days of female menstrual cycle fertilization is mostly probable?
a) the 7-th the 9-th;
b) the 8-th the 10-th;
c) the 11-th the 17-th;
d) the 21-th the 28-th.
25. Sexual reproduction in plants and animals:
a) increases variation;
b) reduces variation;
c) is able to promote or reduce variation; d) has no effect on variation.
26. Cells from the skin of a person suffering from Down's syndrome are examined through the
microscope. Which of the following could be observed in each cell?
a) additional chromosome;
b) the shortage of chromosomes;
c) a haploid set of chromosomes;
d) chromosome number differs from that in adjacent cells.
27. Human individual features depend:
a) on genotype entirely; b) on environment impact entirely;
c) on interaction between genotype and environment;
d) on parents genotype entirely.
28. DNA composition analysis showed that one of the listed correlations cannot change:
)/T; b) G/C; c) A+T/G+C; d) A+G/T+C.
29. Bacteria are characterised by the following peculiarity:
a) they have a cell structure; b) they are diploid;
c) they can be seen through the powerful microscope;
d) they are free living, parasitic or symbiotic organisms.
30. Which of the listed below organs produce non-active enzyme forerunner into mammal's
digestive tract?
a) salivary glands; b) pancreas; c) gall bladder; d) liver.
31. Which of the listed below processes is not connected with adrenaline action?
a) stimulation glycogen change to glucose;
b) speeding up the heart contractions;
c) intensification peristalsis of the intestine;
d) pupil's dilating.

32. A simple key for determination four different plants is worked up:
1. The vascular tissues are present...................2;
The vascular tissues are absent...........Plant A.
2. Male gametes have flagellums.........................3;
Male gametes don't have flagellums......Plant B.
3. Spores of one type..................................Plant C;
Spores of two type...................................Plant D.
Which of this plants is an angiosperm one?
a) A; b)B; ) C; d) D.
33. The sporophyte generation of the flower plant is a diploid one. Then its
endosperm is:
a) n; b) 2n; c) 3n; d) 4n.
34. The diagrams illustrate the situation, when:

a) zooplankton reproduces faster, than phytoplankton;


b) one zooplankton generation feeds on one phytoplankton generation;
c) one phytoplankton generation develops food for several zooplankton
generations;
d) one zooplankton generation feeds on several phytoplankton generations.
35. The scheme illustrates some stages and processes of green plant life
cycle. Which of the listed below pairs of processes are able to make the
most valuable contribution to the genetic diversity of species?

a) 1 and 2;
b) 1 and 4;
c) 2 and 5;
d) 3 and 4.
36. Which of the soil processes is useless for vascular plants?
a) nitrogen fixation from atmosphere; b) oxidation ammonium compounds;
c) oxidation nitrites into nitrates;
d) reduction nitrates to nitrogen.
37. Vitamin, which is the most important for growth and crepuscular vision,
is:
a) A; b)B; ) C; d) D.
38. Breathing is more rapid, when physical load is concerned with?
a) high 02 blood concentration;
b) low 02, blood concentration;
c) high C02 blood concentration;
d) low C02 blood concentration.

39. The results of blood testing are represented in the table:


Plasma
Agg
utinant
a
b
+
Agglutination
What blood was taken for analysis?
a) A, Rh+; b) B, Rh+; ) B, Rh"; d) AB, Rh+.
40. While breathing:
a) the diaphragm contracts;
c) abdominal muscles contract;

Antibodies Rh+
-

b) intercostal muscles contract;


d) no muscles contract.

41. The nerve impulse travels in the organism by one of the listed below ways:
a) dendrite > synapse > neuron body > axon;
b) axon > neuron body > dendrite > synapse;
c) dendrite > neuron body > axon > synapse;
d) dendrite > synapse > axon > neuron body.
42. Transfer of Rh is:
a) transfer of genetic information from DA/A to mRNA;
b) transfer of information from mRNA to rRNA;
c) organition of amino acids according to the codons;
d) recognition of anticodons mRNA.
43. Which of J.B. Lamark's statements is turned down nowadays?
a) sometimes it is difficult to distinguish one species from another with the help
morphological features;
b) living beings vary; c) benefit features are heredited;
d) living beings adapt themselves to the environment.
44. Which of the number pyramids correlates with the food chain: plants-aphid (Aphididae) ladybird (Coccinellidae)l

45. Which of the listed birds will get more evolution benefit?
Bird
Number
laid eggs
hatched nestling
reproducing offspring
a)
9
8
2
b)
2
2
2
c)
9
9
3
d)
7
5
4
46. The general cause of verdure correlated variations from the South to the North and from
the plain to the peaks of the mountains, i.e. with increasing latitude as well as altitude is
connected with:
a) while altitude as well as latitude increases temperature declines;
b) sun lights fall angle is smaller on mountain slopes;
c) clouds gather over the mountain peaks;
d) plants have some difficulties about slope setting.

47. In biocenosis nutritious substances cycling isn't concerned with:


a) transference some food elements from an organism to atmosphere;
b) joining the most food elements to food webs through animals;
c) increasing of population density in that regions where food elements storage are more
than in another;
d) number limitation of ecosystem organisms caused by shortage of some food elements.
48. Which of the listed below factors affect is correlated with population number least of all?
a) parasitism;
b) accumulation of metabolism wastes;
c) predatoriness;
d) hard winter.
49. The population is able to increase its number exponentially:
a) when only food is limited;
b) when firstly it occurs in the surrounding proper for this species but not yet inhabit by it;
c) only when predators are absent; d) in laboratory only.
50. The most effective action about improving the environment is:
a) the second use of glass package;
b) active participation in the movement for developing laws protecting the environment;
c) bicycle instead of driving; d) become a vegetarian.
51. In the nucleus of rabbit's cell haploid DNA number weighs 4'10-n g. What weight will
zygote's DNA have in prophase of the first mitosis?
a)4-10-|2g; b)8-10-12g; c) 1,6 10" g; d) 3,2 lO"11 g.
52. In what test does the Escherichia coli not discovering cause troubles?
a) drinking water; b) newborn child fecal; c) adult fecal; d) adult urine.
53. In the desert the limited factor for plants is usually:
a) duration of daylight;
b) salinization of soil;
c) quantity of moisture;
d) temperature.
54. The most ancient paleontologycal illustrations of life on the Earth are dated:
a) 32 million years;
b) 600 million years;
c) 3,5 milliard years;
d) 4,5 milliard years.
55. In what part of human body discovering of bacteria Staphylococcus epidermiclis does not
cause a trouble?
a) skin surface;
b) nasal cavity;
c) abdominal cavity;
d) vagina.
56. 200 amino acids form the protein. The gene specifying its structure:
a) would be longer in a procaryote;
b) would be longer in an eucaryote;
c) would be same length in a procaryote and in an eucaryote.
57. A cilium differs from a flagellum by:
a) its bases doesn't possess 9+2 arrangement of microtubules;
b) flagellum is capable of rotatory motion whereas, cilium is not;
c) cilia are longer; d) there is no difference.

58. How many centrioles are seen in the beginning of prophase in human
cell?
a)l; b)2; c) 4; d) a lot.
59. How many chromosomes one can observe simultaneously when anaphase
finishes in human skin cell?
a) 23; b)46; c) 69; d) 92.
60. Home fly (Musca domestica) is able to adapt to environmental changes
quicker than human because:
a) it is smaller;
b) it is a good flyer;
c) it has a great number of offspring; d) rapid succession of generations occurs.
61. Lipid's binary membranes are not freely permeable to which of the following
molecules?
a) oxygen; b) carbon dioxide; c) amino acids; d) water.
62. The result of Na+-K+-pump function is not:
a) low NaT concentration in the cell;
b) facilitated diffusion of amino acids into the cell;
c) developing the proton's gradient of concentration;
d) high K+ concentration in the cell.
63. If one-cell animal (Amoeba proteus) and the erythrocyte are put into distillate water:
a) both cells would be destroyed;
b) amoeba would be died, and the erythrocyte would go on living;
c) amoeba would go on living and the erythrocyte would be died;
d) both cells would go on living.
64. Which of the following ways of transporting molecules across membranes utilise ATP
directly?
a) diffusion;
b) Na+-K+-pump;
c) facilitated diffusion;
d) transporting across open channels.
65. Of the following crosses, which is a testcross?
a) AA x aa;
b) AA x Aa;
c) Aa x Aa;
d) Aa x aa.
66. When a healthy mouse is infected with Pneumococcus, it is likely to become ill and die if the
Pneumococcus is:
a) heat-killed;
b) is alive, but lacking a polysaccharide coat;
c) a mixture of some heat-killed Pneumococcus having polysaccharide coat and some
Pneumococcus lacking coat;
d) both lacking polysaccharide coat and heat-killed.
67. The eucaryotic cell, with one more chromosome is:
a) diploid; b) haploid; c) aneuploid; d) monoploid.
68. Who proposed a coherent theory in the field of evolution first:
a) Charles Darwin;
b) Jean Baptiste de Lamark;
c) Alfred Wallace;
d) Julian Kuxley.
69. In the rocks of nearly 3,5 milliard years the fossils are found of:
a) viruses; b) dinosaurs; c) bacteria; d) algae.

70. The first fossils of genus Homo were found:


a) in South Australia;
b) in South Africa;
c) in South-East Asia;
d) in Europe.
71. Viruses can contain:
a) only DNA; b) only RNA; c) DNA and RNA; d) DNA or RNA.
72. Charles Darwin's explanation of the way, how evolution occurs is that:
a) God determines, which species should evolve;
b) fitness enable one species to leave more offspring;
c) certain species has "built" in plan of evolution;
d) environmental impact causes adequate species adaptations.
73. The major factor, causing deviations from Hardy-Wainberg equilibrium is:
a) mutations; b) migration; c) selection; d) near-related breeding.
74. First fossils of Australopithecines were found:
a) in Africa; b) in Asia; c) in America; d) in Australia.
75. What feature is not peculiar to Azotobacterl
a) procaryotic; b) chemosynthetic; c) aerobic; d) atmospheric nitrogen fixation.
76. For Trypanosoma the host, in which sexual reproduction occurs, is:
a) human; b) fly; c) antelope; d) none of listed above.
77. Charles Darwin considered that the most doubt about the truth of his theory is:
a) Earth age;
b) lack of blanks in palaeontology history;
c) ways of traits heredity;
d) Charles Darwin had no doubts.
78. All Fungi are characterised by:
a) spore reproduction;
b) heterotrophic food;
c) mycelium;
d) developing basidiocarp.
79. Angiosperms dominate in the modern Earth flora because:
a) they have well-developed tissues securing high metabolism;
b) they have the organ of seed reproduction - the flower;
c) they have C3- and C4-photosynthesis;
d) a + b + c.
80. What plant cells can function after their death?
a) cambium; b) sieve-tubes; c) collenchyma; d) vassels.
81. Malaria is caused by:
a) mosquito bite (Anopheles maculipennis);
c) Plasmodium vivax;

b) moisture air of marshes;


d) Bacillus malaria.

82. Cell walls of Fungi are mainly made of:


a) glucose; b) cellulose; c) pectin; d) chitin.
83. One can consider the bryophytes as the dead-end evolutionary siding because:
a) they have no true roots;
b) their vascular tissues are badly developed;
c) gametophyte dominates in the life cycle;
d) sex reproduction depends on water.

84. You sneeze and have watery eyes every spring. Your physician tests you and says that
you are allergic to pollen of a plant Ambrosia. You guess that Ambrosia is:
a) high plant;
b) insects pollinated;
c) wind pollinated;
d) excreta phytoncids.
85. All helminths are characterised by:
a) the digestive system is absent;
b) high reproduction intensity;
c) the senses are absent;
d) hermaphroditism.
86. There are 24 chromosomes in the cells of pine {Pinus sylvestris) leaves. Then its endosperm
would contains chromosomes:
a) 12 chromosomes; b) 24 chromosomes; c) 36 chromosomes; d) 48 chromosomes.
87. Which of the listed pairs aren't homologous organs?
a) tendrils in grapes (VWs) and tendrils in the garden pea (Pisum);
b) tendrils in grapes (Vitis) and strawberry {Fragaria) runners;
c) tendrils in grapes (Vitis) and hawthorn (Crataegus) prickles;
d) needles (Pimis) and cactus prickles (Mammillaria).
88. The seed germinating on the soil surface has a root growing vertically nevertheless of
seed's posture. It is because of:
a) negative heliotropism;
b) positive geotropism;
c) substratum structure;
d) positive hydrotropism.
89. Organic compounds pass through xylem:
a) after autumn leaf-fall only;
b) at night only;
c) only when buds of leaf-fall trees develop; d) never.
90. Sponges (Porifera) digestion is:
a) out intestine; b) in the cavity; c) in the cell; d) b + c.
91. The common features for Annelida and Arthropoda are:
a) exoskeleton;
b) having extremities;
c) closed circulatory system;
d) segmentation.
92. Lymnaea stagnalis - lung mollusc, inhabiting water permanently, lay his eggs:
a) on above-water plant parts; b) into the bank soil;
c) on underwater plant parts; d) digs into silt.
93. Flatworms (Plathelminthes):
a) don't have any circulatory system and body cells get their feed and oxygen as a result
of diffusion;
b) have closed circulatory system and a heart;
c) have opened circulators' system;
d) don't need any circulatory system because they don't have any organs.
94. Air flows through birds lung:
a) on inhalation from the tail to the head;
b) on exhalation from the head to the tail;
c) on inhalation and on exhalation from the tail to the head;
d) on inhalation from the head to the tail and on exhalation from the tail to the head.

95. Sting of a bee (Apis mellifera) is:


a) modified organ of copulation;
b) modified abdomen segment;
c) modified ovipositor;
d) neither the one nor the others.
96. White bears (Thalarctos maritimus) don't eat penguins (Impennes) in
wildlife because:
a) they don't meet each other in wildlife;
b) penguins swim faster than bears;
c) penguins live within groups and it is dangerous to attack them;
d) penguins meat is toxic.
97. Horsefly {Tabanus) females eat:
a) blood;
b) plant juice;
c) excrements of hoofed animals;
d) no one of the variants.
98. High radiation of human body doesn't cause:
a) destroying of blood cells formation; b) cancer;
c) stomach-intestine bleeding;
d) myocard infarction.
99. There are not more than 6 links in food chains usually because:
a) environmental resources are limited;
b) large amount of energy is lost at every link;
c) predators are unable to eat predators; d) a + b + c.
100. There is age distribution in a population in schemes. Which of the
populations has a greater chance to extinct?

***Following information deals with the questions 101-135.


Decide true or false that or either statement.
101. Glycocalyx (the highest layer of a great number of animal cells) contains
different polysaccharides and proteins.
a) yes; b) no.
102. Cell plasma membrane contains different proteins only.
a) yes; b) no.
103. Water, ions of different salts, mineral and organic molecules pass through the narrow
channels in the cell membrane from the environment into the cell.
a) yes; b) no.
104. Pinocytosis is common for animal, fungi and bacteria cells only.
a) yes; b) no.
105.Polysaccharides prevail in the basis matter of plant cells cytoplasm.
a) yes; b) no.
106. When guard cells turgor is higher than usual, the stomata is opened.
a) yes; b) no.

107. Only cell nucleus contains DNA, where it forms chromosomes.


a) yes; b) no.
108. Ameba, infusoria and great amount of other one-celled animals (Protozoa) feed by
phagocytosis.
a) yes; b) no.
109. Phagocytosis is common for animal cells as well as for plant, bacteria and blue-green
algae (Cyanophyta).
a) yes; b) no.
110. Proteins are synthesized on the membranes of smooth endoplasmic reticulum.
a) yes; b) no.
111. Ribosomes are common for cells of all living beings.
a) yes; b) no.
112. Ribosomes contain proteins, RNA, lipids and polysaccharides.
a) yes; b) no.
113. Mitochondria are found in cytoplasm of the most plant and animal cells.
a) yes; b) no.
114. Lysosomes are formed by the Golgi complex.
a) yes; b) no.
115. In all plant and animal cells there is an organelle located near the nuclear envelope and
named cell center (centrioles).
a) yes; b) no.
116. RNA, DNA and proteins form nucleolus.
a) yes; b) no.
117. Nucleolus are the places where big and small ribosomes parts are
formed.
a) yes; b) no.
118. Human arm and whale's (Balaenoptera) flipper are homologous organs.
a) yes; b) no.
119. Tendrils of the garden pea (Pisum sativum) and tendrils of the cucumber (Cucumis sativus) are
analogous organs.
a) yes; b) no.
120. Barberry (Berbris vulgaris) prickles and dog-rose (Rosa canina) prickles are homologous
organs.
a) yes; b) no.
121. Melanism appears as a result of genetic diversity and is enable to be "caught up" by
natural selection.
a) yes; b) no.
122. Cross-pollination always gives more benefits than self-pollination.
a) yes; b) no.

123. Platelets are formed in the spleen.


a) yes; b) no.
124. Mitochondria were not found in the cells of some anaerobic organisms.
a) yes; b) no.
125. Animals adapt to sweet and salt substances better than to bitter and sour ones.
a) yes; b) no.
126. Some Australian marsupials (Metatheria) species and species of placental (Eutheria)
mammals are convergent ones.
a) yes; b) no.
127. Tympanic membrane is common for every land vertebrate animal ears.
a) yes; b) no.
128. The first full conception about evolution was developed by J.Sant-Iler.
a) yes; b) no.
129. Natural selection is the only force that is capable to maintain the high
level of heterozygotes in wildlife populations.
a) yes; b) no.
130. The term "genetics" was suggested by G.Mendel.
a) yes; b) no.
131. The Eustachian tube preserves the eardrum (tympanic membrane) from destroying when
atmosphere pressure fluctuates.
a) yes; b) no.
132. Tadpole tails disappear because of dead cells digesting by lysosomes.
a) yes; b) no.
133. In cephalopod's eye accommodation achieves by changing the shape of the lens.
a) yes; b) no.
134. Penguins have a keel on the sternum.
a) yes; b) no.
135. Oviparous (Prototheria) occur not only in Australia and neighbour islands. They live in
South America as well.
a) yes; b) no.
***Following information deals with the questions 136-165.
Write the terms proceeding from the listed definitions.
The codes of the answers:
11. Ieucoplast;
21.
1. hair cells of organ of Corti;
12. macroevolution;
22.
2. hemolymph;
13. mantle;
23.
3. divergence;
14. mesoderm;
24.
4. intron;
15. migration;
25.
5. clone;
16. mycorrhiza;
26.
6. commensalism;
17. ontogeny;
27.
7. convergence;
18. vane;
28.

recolonization;
coprophagous;
symbiote;
symplast;
sporophyte;
thermophobic;
tissue;
evolution;

8. rhizome;
9. crossing-over;
10. xerophyte;

19. parthenogenesis;
20. pericardium;

29.
30.

neuston;
etology.

136. The certain or by chance movement of organisms of some species from the
area lines...
137. The permanent or temporary living together organisms of various species
in which one of the partners eats meat residues or excretes of the other
not harming it...
138. Vegetative genetically identical offspring of one organism...
139. An organism which eats excrements of other animals, mainly mammals..
140. A group of living beings inhabiting upper level of water on the border
with air...
141. Artificial return to some territory of the species extinct there before...
142. Origin of living beings from former living forms...
143. Plants of arid habitants stable to overheating and dehydrating because
of a number of adaptive features...
144. Colourless plastides of various form and function in plant cells...
145. Organism inhabiting at permanent low temperature conditions...
146. The exchange of corresponding chromatid segments between homologous
chromosomes responsible for new combinations of various alleles...
147. Reproducing part of hearing system of mammals and humans, which
transfer the energy of sound waves into the nervous impulse...
148. The independent development of similar structures in unrelated organisms
to identical environment...
149. Interaction between members of the population of one species or various
species in order to obtain a mutually required resource available in limited
supply...
150. Variant of bone tissue forming the placoid fish scales and the main
part of mammals teeth...
151. Liquid tissue circulating in insects body...
152. The type of evolution coming into existence of taxons higher than
species...
153. The outer crease of the skin in Mollusca, Brachiopoda, Cirripedia covering
the whole body or its part...
154. Embodiment of plant or animal cells in tissues while out-cells matter
is destroying or dissolving...

155. A symbiotic association between fungi and the roots of a vascular plant...
156. Origin of the group of organisms from common ancestor...
138. Vegetative genetically identical offspring of one organism...
139. An organism which eats excrements of other animals, mainly mammals..
140. A group of living beings inhabiting upper level of water on the border
with air...
141. Artificial return to some territory of the species extinct there before...
142. Origin of living beings from former living forms...
143. Plants of arid habitants stable to overheating and dehydrating because
of a number of adaptive features...
144. Colourless plastides of various form and function in plant cells...
145. Organism inhabiting at permanent low temperature conditions...
146. The exchange of corresponding chromatid segments between homologous
chromosomes responsible for new combinations of various alleles...
147. Reproducing part of hearing system of mammals and humans, which
transfer the energy of sound waves into the nervous impulse...
148. The independent development of similar structures in unrelated organisms
to identical environment...
149. Interaction between members of the population of one species or various
species in order to obtain a mutually required resource available in limited
supply...
150. Variant of bone tissue forming the placoid fish scales and the main
part of mammals teeth...
151. Liquid tissue circulating in insects body...
152. The type of evolution coming into existence of taxons higher than
species...
153. The outer crease of the skin in Mollusca, Brachiopoda, Cirripedia covering
the whole body or its part...
154. Embodiment of plant or animal cells in tissues while out-cells matter
is destroying or dissolving...
155. A symbiotic association between fungi and the roots of a vascular plant...
156. Origin of the group of organisms from common ancestor...
46
157. The course of development of an individual, the whole complex of transformations

from egg to life end (death or the new division of an individual)...


158. Lamellate-like part of a contour feather growing from both sides of
the shaft...
159. One of the forms of sex reproduction when female sex cells develop
without fertilization...
160. Strong sac of connective tissue enclosing the heart of some invertebrate
and all vertebrate animals...
161. Segments of the gene which has not genetical information...
162. Science about behaviour of living beings...
163. Unity of cells of multicellular organism (except sex ones)...
164. The asexual plant generation producing spores...
165. A modified shoot with long thin internodes and scale colourless, rarely green
leaves...
***Following information deals with the questions 166-169.
Compliment the letters of the figures with codes of the answers.
166. What is the chromosome number of the organisms and their generations,
which are marked by letters in the figure?
The codes of the answers: 1. n; 2. n + n; 3. 2n; 4. 3n.

Answers:
. a) b) c) d) e) f) g) h)
i) j) k) 1) m) n) o) ....

167. What terms refer to types of interactions of the living organisms, which are
shown in the figure?
The codes of the answers:
1. neutralism;
3. mutualism;
5. predation;
2. commensalism;
4. parasitism;
6. competition.

Answers: a)

b)

c)

d)

e)

f) ....

168. Find the connection between terms and structures, which are shown in the
figure:
The codes of the answers:
1. uterus cavity;
2. cervix;
3. miometrium;
4. endometrium;
5. vagina;
6. chorion;
7. chorion villus;
8. placenta;
9. amnion;
10. amnion cavity;
11. umbilical cord;
12. allantois;
13. resides of yolk sac.
Answers:
a) b) c)
g)
h) i)

d)
j)

e)
k)

f)
l)

m)....

169. Find the connection between the families of plants and flower diagrams,
which are shown in the figure:
The codes of the answers:
1. Magnoliaceae;
6. Lamiaceae;
2. Ranunculaceae;
7. Solanaceae;
3. Rosaceae;
8. Liliaceae;
4. Fabaceae;
9. Orchidaceae;
5. Brassicaceae;
10. Poaceae.

Answers: a)

b)

c)

d) e)

f) g)

h)

i) j) ....

***Following information deals with the questions 170-172.


Solve following genetic problems.
170. Black and white mice were hybridized. Hybrid individuals of the first generation
were grey (agouti). After their hybridization segregation occured in number of 9
grey, 3 black, 4 white. Find the genotypes of the parents and Fx-cross and F2 -cross.
Write conventional letter signs into the shown below scheme.

black mouse x white mouse


F1
grey mice
F2
9 grey mice; 3 black mice; 4 white mice.
171. When brown-coloured spaniels of thoroughbred lines were crossed with
white-coloured ones the resulting hybrid individuals occured white-coloured. The
hybrids of the second generation showed the segregation in proportion 12 white :
3 black : 1 brown. Find the genotypes of the parents, hybrids Fl and F2. Write
them into the shown below scheme.
p
brown spaniel x white spaniel
F1 white spaniels
F2 12 white spaniels; 3 black spaniels; 1 brown spaniel.
172. In cats the gene of black colour and the gene of red colour are connected with
sex, located in the X-chromosome and produce incomplete dominance. While they
are combined the turtle colour occurs. The turtle-coloured cat gave birth to 5 kittens
in proportion 1 red : 2 turtle : 2 black. That red kitten was female. Find the genotype
and the phenotype of the tom-cat sire, genotypes and sex of kittens. Write these
data into the shown bellow scheme.
p
turtle-coloured cat x tom-cat
The phenotype:
The genotype:
F1
1 red kitten; 2 turtle kittens; 2 black kittens.
Sex:
The genotype:

173. From the listed information collect the data about spider (Arachnida).
The codes of the answers:
1. Members:
a) scorpion (Scorpiones);
b) slater (Oniscoidea);
c) crawfish (Palinuridae); d) cyclop (Cyclopes);
e) mite (Acarina);
f) solifuga (Solifugae).
2. Species number:
a) nearly 1 million; b) nearly 35000; c) nearly 60000.
3. Their body is clearly subdivided into the head, thorax and abdomen.
4. The body is formed by cephalothorax - prosoma and abdomen epistosoma, can consist of different number of segments.
5. Segments are organized into tagmas - the head, the thorax, the abdomen.
In many species the basal of the head is covered by chitin fold - carapace.
6. Antennas are absent.
7. There is one pair of antennas.
8. There are two pairs of antennas.
9. There are mandibular and maxillas.
10. There are cheliceras and pedipalps.
11. There are 3 pairs of walking legs on the thorax.
12. There are 4 pairs of walking and feeding legs.
13. There are 5 pairs of walking legs.
14. The legs are uniramulous.
15. The legs are biramulous.
16. There are simple eyes (ocellus) in different number (from 1-2 pairs
to 8 pairs; 4 pairs usually on the cephalothorax).
17. There are two types of eyes - compound (in the most species) or simple
(ocellus).
18. The eyes are compound (in many species) or there is the unpaired
nauplial eye.
19. The nervous system is formed alike the one in primitive Arthropods
and even Annelids: this is the brain, consisting of overesophageal
double ganglia and ventral nerve cord.
20. The nervous system is concentrated. The dorsal ganglia (brain) is
fused with the subesophageal ganglia. In some species one or more
nervous ganglia of ventral nerve cord are remained.
21. They are the unique invertebrate animals who are capable to distinguish
sounds.
22. The excretory system is formed by Malpighian tubules.
23. Antennas glands or maxillary glands are excretory system.
24. Sexual glands and Malpighian tubules are excretory system.
25. There are two variants of respiratory system: tracheas and/or lungs.
Some small animals do not have neither lungs nor tracheas.
26. They breathe through the gills, but when they are absent - through
the body surface.
27. Breathing occurs by the help of tracheas system or through the body surface.
28. There are respiratory pigments in the blood.
29. The process of development involves incomplete or complete metamorphosis.
30. In the most species the process of development involves metamorphosis. The certain larva nauplius - hatches from the egg.
31. The process of development is a direct one or involves metamorphosis.
32. Moult is common for larva as well as for adults.
33. Adults do not moult.

INTERNATIONAL BIOLOGY OLYMPIAD 1990


THEORY PROBLEMS















All IBO examination questions are published under the following Creative Commons license:



CC BY-NC-SA (Attribution-NonCommercial-ShareAlike) https://creativecommons.org/licenses/by-nc-sa/4.0/
The exam papers can be used freely for educational purposes as long as IBO is credited and
new creations are licensed under identical terms. No commercial use is allowed.

I INTERNATIONAL
BIOLOGY
OLYMPIAD
1. A relict is:
a) a rapidly spreading organism that is not indigenous to the given area;
b) the term for an organism causing dangerous disease;
c) a genetically unstable hybrid;
d) an organism which in the given area is a remainder from earlier times, when it was widespread.
2. As a result of intensive nitrogen use of fertilizers this one has spread to a number of
biotopes which are not typical of it:
a) coltsfoot (Tussilago far far a);
b) viper's bugloss (Echium vulgare);
c) great nettle (Urtica diodica);
d) dandelion (Taraxacum officinale).
3. The highest gross primary production is typical of the ecosystem:
a) beech wood; b) meadow; c) mountain pine forest; d) oak grove.
4. Water organisms characterized by striking active movement are known as:
a) plankton; b) benthos; c) nekton; d) pleuston.
5. Indicate which ecological group is referred to the plants which have a thin, fragile stalk
with reduced mechanical tissue, inside the stem are large intercellular spaces, the leaves are
fleshy with a thin cuticle...
a) mesophyte; b) hygrophyte; c) xerophyte; d) thermophyte.
6. The marking of territory is particularly significant for:
a) finding a partner;
b) genetic adaptation;
c) a successful struggle for survival;
d) competition within a species.
7. Putrefactive bacteria belong to:
a) producers;
c) consumers of the 2-nd order;

b) consumers of the 1-st order;


d) destroyers (reducers).

8. Acid rains have negative influence on nature:


a) influence the quality of solar radiation;
b) increase the movement of Ca2+ and certain metals in the ecosystem;
c) increase the amount of H202 in the atmosphere;
d) cause smog.
9. The eutrofication of water is caused by high content of:
a) potassium and CO., in the water;
b) nitrogen and phosphorous in the water;
c) sodium and calcium in the water;
d) saccharides in the water.
10. The dominant of the herbal layer of an oak and hornbeam wood may be:
a) deadly nightshade (Atropa belladonna):
b) rejuvenating woodland beech (Fagus sylvatica);
c) purple orchis (Orchis rnacidata);
d) starwort (Stellaria holostea).

11. A mountain spruce forest is, in comparison with a field of maize:


a) more stable, does not require additional energy to survive;
b) more stable, but requires considerable input of additional energy;
c) less stable, requires great amount of additional energy to survive;
d) less stable, but due to high capacity for self regulation does not need additional energy.
12. Solar energy is best used per surface unit by:
a) a field of wheat;
b) a meadow;
c) a pine forest;
d) a water meadow.
13. The silver birch (Betula pendula) belong to the trees which are:
a) strongly light loving;
b) shade loving;
c) indifferent to light;
d) light and shade loving.
14. Mushrooms (Fungi) have the following relationship to light:
a) they require it unconditionally for growth;
b) they do not require it for growth, but many types need a certain amount of light for spore
production;
c) they have no need whatever of light for existence;
d) apart from exceptions they require it (light has a positive influence, for instance, on rate of
growth and colouring of gills).
15. Light for the germination of seeds is:
a) necessary only for some plant species;
b) quite essential for all parasitic plants;
c) an essential factor only for light loving plants;
d) is not a limiting factor for germination in any species.
16. Relation of insufficient light and yellowing of leaves:
a) no influence;
b) slows it down, especially at higher temperatures;
c) slows it down, especially at lower temperatures;
d) speeds it up, especially at higher temperatures.
17. Algae and infusoria live together in a solution placed in the light. The infusoria
consumes 0,10 mol of glucose in a week, the algae - 0,12 mol. The weekly production of
glucose amounts to 0,25 mol. What is the net production of oxygen per week in this
medium?
a) 0,03 mol;
b) 0,60 mol; c) 1,32 mol; d) 0,18 mol.
18. "Synanthropic" is the term used for:
a) enriching of local flora and fauna with species from other geographical territories;
b) domestic (domesticised) animal;
c) organisms transferred by man from original territories;
d) animals and plants living in close relationship with man.
19. Phototaxis is:
a) the ecological link of a certain species to a shaded or unshaded site;
b) the amount of light in a certain unit of time required for the development of a given species;
c) orientation (e.g. of insects) evoked by ligh;
d) relation between growth of insect larvae and light conditions.
20. What is an edaphone?
a) instrument for repelling insects (e.g. mosquitoes) with sound waves;
b) community of all organisms living in the soil;
c) part of the sound apparatus of a grasshopper;
d) part of the sound apparatus of a mosquito.

21. A monophagous is:


a) a cell capable of swallowing up foreign particles (e.g. microorganisms);
b) an animal which takes in food only during a single stage of development;
c) an animal with the highest degree of specialization as regards type of food;
d) a virus attacking only narrowly specialized eucaryotic cell.
22. A predator is:
a) a beast of prey living wild which kills and eats other animals;
b) an animal living on or in the body of another animal and feeding on it;
c) an animal which on or in its body carries the germs or development stages of other animals;
d) a species of animal whose existence signals the later existence of another species.
23. A pheromone is:
a) a chemical substance, containing mainly iron compounds, from which the body covering of
insect is made;
b) a hormone enabling relations between individuals of one species of insects;
c) a chemical dye based on iron which gives rise to the metallic sheen of some groups of insects;
d) a hormone enabling relations among individuals of various species of insects.
24. The term "mimicry" means:
a) the microscopic feeler hairs on an insect's body;
b) the special protective similarity of some species of insects to others;
c) the state into which some species of insects fall under the influence of external stimuly
signalling danger;
d) the behaviour of the females of some groups of insects, intended to attract the male.
25. By the concept of "population" we mean:
a) a set of individuals of one species including their developmental stages on a certain territory;
b) a set of individuals of one species on a certain territory without stages of development;
c) set of individuals of all species in a common environment;
d) set of individuals on a certain territory which are not genetically related.
26. The genetic balance of a population in the sequence of generations is expressed by the
Hardy-Weinberg Law, which is expressed mathematically as:
a)H = 2pq;
b) p2+2pq+q2 = 1;
c) (p+q) - (p - q);
d) (p+q) x (p - q) = p2 - q2.
27. The basic unit of an eucaryotic chromosome consisting of DNA and protein is:
a) a nucleotide; b) a nucleoside; c) a nucleosome; d) a nucleoid.
28. All the given factors are mutagenous with the exception of:
a) gamma-radiation; b) UV-radiation;
c) acridine dye; d) acetic acid.
29. A caryotype is:
a) the crossing of chromosomes during meiosis;
b) the marking of sex chromosomes;
c) characteristics of type of cell core (procaryotic or eucaryotic type);
30. In a monohybrid with complete dominance the phenotype splitting proportion
in F2, is:
a) 3:1;
b) 1:2:1; c) 9:3:3:1;
d) 1:1.
31. With regard to sex chromosomes a normal woman is equipped with:
a) XY;
b) XX; c) YY;
d) XO.
32. A new pink flowering plant with genome Cc was planted on an island. It
is an annual, self pollinating plant that has four seeds a year. If in its progeny
there appears a plant with white flowers, then this plant is:
a) polyploid;
b) homozygotically recessive;
c) a hybrid with incomplete dominance;
d) a mutant.

33. In a case where one parent has blood group 0 and the other - AB, the
children may belong to the group:
34. In the interbreeding of individuals with genotypes AABb and aaBb the
following genotype cannot occur in further generations:
a) AABb; b) AaBb; c) AaBB; d) Aabb.
35. The biogenetic law (onthogenesis is an abbreviation of phylogenesis)
was formulated by:
a) E. Haeckel; b) G. Cuvier; c) J.B. Lamarck; d) Ch. Darwin.
36. Among the oil soluble vitamin is:
a) vitamin K; b) pyridoxine; c) thiamin; d) vitamin .
37. Which tree releases allelopathic substances to the environment?
a) beech (Fagus sylvatica);
b) sallow (Salix caprea);
c) bastard acacia (Robinia pseudacacia);
d) small-leaved lime (Tilia cordata).
38. During which process most energy quantity is released?
a) photolysis; b) glycolysis;
c) Krebs cycle (the cycle of citric acid);
d) final oxidation in the respiration chain.
39. Some substances enter a photosynthesising cell and some leave the
a) 0, AB, A, B; b) , ; c) 0, AB; d) AB.
cell:
a) glucose and CO, water and 02;
b) glucose and 02 water and C02;
c) water and 0, glucose and C02;
d) water and C02 - glucose and 02.
40. Which of the given actions are linked with the primary processes of 49 photosynthesis
(with the light phase):
The codes of the answers: 1. photolysis; 2. Calvin cycle;
3. elimination 02;
4. absorption C02.
a) 1, 2; b) 1, 3;
c) 2, 3; d) 2, 4.
41. The energetic effect of anaerobic glycolysis are 2 molecules of:
a) lactic acid;
b) pyruvic acid;
c) ATP;
d) ethanol.
42. In the photosynthesis of C3-plants the acceptor of C02 is:
a) 3-phosphoglycerol aldehyde;
b) ribulose-l,5-diphosphate; 51
c) phosphoenolpyruvic acid;
d) ferredoxin
43. Photorespiration is a process typical for:
a) all plants without exception in roughly the same intensity;
b) mainly C4-plants;
c) mainly C3-plants;
d) this process is not typical for plants. 52
44. The following has a marked capacity for vegetative reproduction:
a) orache (Atriplex);
b) plantain (Plantago);
c) coltsfoot (Tussilago);
d) chicory (Cichorium).
45. Gametophyte is in the life of plants a generation which:
a) creates gametes through the process known as meiosis;
b) arises from gametes;
c) arises as a result of the combining of gametes;
d) is created by cells with a haploid number of chromosomes in the nucleus.

46. Not among the flowering plants is the:


a) hazel (Corylus);
b) poplar (Populas):
c) larch (Larix);
d) walnut (Juglans).
47. An angular stem, symmetrical flowers in tight inflorescences, two longer and two
shorter stamens characterise the family of: 55
a) mustard (Brassicaceae);
b) poppies (Papaveraceae); th
c) dead-nettles (Lamiaceae);
d) figworts (Scrophulariaceae).
48. Willows (Salix) are polinated by... and their seeds are spreaded by..
a) the wind - the wind;
b) the wind - animals;
c) insects - water or animals;
d) insects - the wind.
d) characteristics giving number, shape and construction of all chromosomes of an individual.
49. Mycelium is not formed by:
a) Mucor mucedo;
c) Puccinia graminis;

b) Claviceps purpurea;
d) Saccharomyces cerevisiae.

50. A common trait of Cyanophyta and Rhodophyta is:


a) more or less the same cell construction;
b) more or less the same colouring;
c) production of starch during photosynthesis;
d) in reproduction similar single cell units are formed.
51. Apart from the single cotyledon monocotyledonous plants can be further
characterised by the following traits:
a) cambium, replacement roots, perianth;
b) cambium, main root and side roots;
c) scattered vascular bundles, main root and side roots, perianth;
d) scattered vascular bundles, replacement roots.
52. The flower of buttercup (Ranunculus) is characterised by:
a) arrangement of and in ring, A and G - in spiral, free pistils;
b) arrangement of P in ring, A and G - in spiral, free pistils;
c) arrangement of all parts in ring, free pistils;
d) arrangement of all parts in spiral, pistils grown into the receptacle.
53. In plants with enclosed seeds the seeds originate from:
a) the ovary and sepals;
b) the ovary;
c) the ovule;
d) the fertilized egg cell.
54. Diatoms (Bacillariophyta) are single cell organisms:
a) feeding mainly heterotrophically and living in colonies;
b) moving with the aid of flagella or pseudopods;
c) forming a double cell wall of CaC03;
d) fonning a cell wall of SiO,.
55. With the transition of plants from a watery environment to the dry land
there occurred in phylogenesis mainly the improvement of:
a) mechanical tissues and the methods of reproduction;
b) assimilation tissues and conductive tissues;
c) covering tissues and assimilation tissues;
d) methods of reproduction, conductive and mechanical tissues.
65. The respiratory muscles of a man are:
a) the internal crosswise stomach muscle and diaphragm;
b) the external and internal intercostal muscles and direct stomach muscle;
c) the external and internal intercostal muscles and diaphragm;
d) the large and small chest muscle and diaphragm.

66. When at rest the surface of the membrane of a nerve cell is:
a) electropositive; b) electronegative; c) electroneutral; d) without electric charge.
67. The composition of lymph (sap) is similar to the composition of blood
plasma. Select the correct statement:
a) proteins are lacking in lymph;
b) there are more proteins in honph than in plasma;
c) there are less proteins in lymph than in plasma;
d) the difference is in the quality of Na+ ions.
68. The semilunar valves separate:
a) the chambers of the heart from the arteries; b) the left ventricle from the right atrium;
c) the right ventricle from the left atrium;
d) hollow veins from the right atrium.
69. Progesteron is eliminated by:
a) the Graafian follicle;
b) the corpus luteum (Yellow body);
c) the maturing egg;
d) the ovulating egg.
70. The biosynthesis of proteins takes place in all cells of the human organism
with the exception of:
a) the cells of the pancreatic gland;
b) the cells of the mucous membrane of the intestine;
c) mature red corpuscles (erythrocytes);
d) white blood corpuscles (lymphocytes).
71. The veins (trachea) of the altered root of mistletoe (Viscum) grow through
a pine (Pinus) branch, for instance, into the:
a) pithy part of the vascular bundle (phloem);
b) woody part of the vascular bundle (xylem);
c) pith of the branch;
d) immediate surroundings of individual vascular bundles, but do not penetrate them.
72. Sexual reproduction in Infusoria with gene recombination takes place
by:
a) encystation; b) strobilation; c) conjugation; d) longitudinal splitting.
73. The mesoderm appears first in phylogenesis in the:
a) Scyphozoa; b) Turbellaria;
c) Tracheata;
d) Annelida.
74. The symbiosis between termite (Isoptera) and the microorganisms, living
in their intestines and feeding on cellulose is an example of:
a) competition; b) predation; c) mutualism; d) commensalism.
75. Various tissues and organs are formed from the ecto-, meso- and endoderm.
Which combination is correct?
ectoderm
mesoderm
endoderm
a)

brain and spinal fluid

blood

lungs

b)

brain and spinal fluid

large intestine

lungs

c)

skin

bone

kidneys

d)

epidermis

liver

heart

76. The human organism does not synthesise and must therefore require
from food:
a) purines;
b) pyrimidines;
c) fructose; d) aromatic amino acids.
77. Caterpillars of the cabbage white (Pieris brassicae) form a chrysalis:
a) underground near a nutrition's plant;
b) on various objects above ground (trees, posts, walls);

c) on a food plant;
d) do not make a chrysalis because it is an insect with imperfect metamorphosis.
78. Colorado beetle (Leptinotarsa decemlineata) was brought to Europe from
North America in:
a) 1925;
b) 1938;
c) 1947;
d) 1914.
79. Cricket (Gryllidae) chirp:
a) by rubbing their front wings together;
b) by rubbing the front wing against the back leg;
c) by means of a special organ on the lower side of the thorax;
d) by pushing collected air out through the stigmata.
80. Rotifers (Rotatoria), like all roundworms (Nemathelminthes), do not have a developed.
system.
a) digestive; b) eliminative; c) respiratory; d) nervous.
81. Halters are:
a) moisture loving ribbon like shapes of the spores of horsetails;
b) part of the mandibles of some beetles;
c) the reduced wings, for example, of the mosquitos;
d) the sensitive cilia on the rear part of the body of homoptera.
82. Which type of parasitic worms has a stage of development with life in fresh
water?
a) roundworm (Ascaris lumbricoides);
b) nematode worm (Trichinella spiralis);
c) beef tapeworm (Taeniarhynchus saginatus);
d) liver rot (Fasciola hepatica).
83. Parthenogenesis can occur in all the following organisms with the exception
of:
a) rotifers (Rotatoria);
b) insects (Insecta);
c) crustaceous (Crustacea);
d) spiders (Arachnida).
84. What is the largest number of elements of which an insect's leg may be
made up?
a) 5;
b)6;
c) 9;
d) 15.
85. Planorbis breathe:
a) through gills hidden by part of the shell so that they are actually within the body;
b) through two external gills;
c) through a lung sac formed from the wall of the shell hollow;
d) through a single gill (ctenidium) near the anal opening.
86. Chorda (chorda dorsalis) is preserved in mature age by:
a) sea squirts (Ascidiae);
b) sturgeon (Acipenser);
c) lamprey (Petromyzones);
d) dipnoan fish (Protopterus).
87. The most primitive placental mammals are:
a) insect-eating mammals (Insectivora);
b) marsupials (Marsupialia);
c) rodents (Rodentia);
d) protothere (Monotremata).
88. In herbivores the food returns to the mouth for cuddling from:
a) the crop (ingluvies);
b) the honeycomb (reticidum);
c) the second stomach (rumen);
d) the psalterium (omasum).
89. European species of amphibians live on a diet which is:
a) mainly animal;
b) mainly vegetable;
c) for the young - vegetable, for adult - animal;

d) for young frogs - mixed, for others - animal.


90. The brown frog (Rana temporaria) has teeth:
a) on the upper and lower jaws;
b) on the upper jaw and roof of mouth;
c) on the lower jaw;
d) has no teeth at all.
91. The cloaca is:

a) the sexual organ of fish, amphibians and some reptiles;


b) a spiral surface in the intestines of sharks and some fish;
c) a common opening for the digestive, eliminator)' and sex organs;
d) in water birds a gland serving for the oiling of feathers.
92. European species of lizard can threaten the health of man:
a) by biting with teeth and poison gland;
b) as an intermediary host to some intestinal flatworms (Plathelminthes);
c) by biting and infecting wound with poisonous secretions from the mouth;
d) they are not at all dangerous.
93. Zoochoria is:
a) the broadcasting of seeds or fruits by activity of animals;
b) an infectious disease carried by animal fur;
c) the science of the spreading of animals on Earth;
d) use of animals for "biological warfare" with other animal species.
94. In European amphibians the fertilization of eggs takes place:
a) in the water outside the female's body;
b) in the female's body;
c) in frogs - outside the female's body, in salamanders and newts - inside the body
d) in frogs and newts - in the water, in salamanders - in the female's body.
95. The nest of the great tit (Parus major) is usually situated:
a) in the fork of a branch of a thick bush;
b) mostly under a bush on the ground in grass;
c) is usually woven of grass stalks and hung on a branch 3 m above ground;
d) in a hollow tree.
96. From the following mammals one does not hibernate:
a) great dormouse (Glis glis);
b) european hedgehog (Erinaceus europaeus);
c) common mole (Talpa europaea);
d) long-eared bat (Plecotus auritus).
97. Which bird of prey lives mainly on smaller birds?
a) buzzard (Buteo buteo);
b) marsh harrier (Circus aeruginosus);
c) kestrel (Falco tinnunculus); d) sparrow hawk (Accipiter nisus).
98. Articulates (Arthropoda) are linked in development to:
a) Mollusca; b) Nemathelminthes; c) Annelida; d) Echinodermata.
99. One of the following vertebrate animals does not have ribs:
a) newt (Thturus); b) bat (Myotis); c) sparrow (Passer): d) mole (Talpa).
100. L. Pasteur discovered:
a) vaccination against smallpox; b) blood-coagulation factors;
c) the treatment of rabies;
d) bacteriophages.

***Following Information deals with the questions 101-104.


Body construction of Turbellaria (101), Annelida (102), Crustacea (103), Insecta
(104) is represented on the figures. Numerate on the schemes:
The codes of the answers:
1. nervous system;
3. vascular system;
2. alimentary channel;
4. secretory system.
101.

Answers:
a) ...; b) ...; c) ...; d) ...; e)
102.

Answers:
a) a) ...; b) ...; c) ...; d) ...; e)
103.

Answers:
a) a) ...; b) ...; c) ...; d) ...; e)
104.

Answers:
a) a) ...; b) ...; c) ...; d) ...; e)

***Following information deals with the questions 105-108.


Some types of eyes are represented on the figures: Turbellaria (105), Insecta
(106), Cephalopoda (107) and Mammalia (108). Numerate on the schemes:
The codes of the answers:
1. nervous system;
3. layer of pigmental cells;
2. photosensitive cells;
4. lens.
105.
Answers:

)...; b) .. ).. d) ....


106.
Answers:

)...; b) .. ).. d) ...


107.
Answers:

)...; b) .. ).. d) ....


108.

Answers:
)...; b) .. ).. d) ....
***Following information deals with the questions 109-110.
The are schemes of embryogeny of bird (109) and mammal (110) are represented
on the figures. Numerate on the schemes:
The codes of the answers:
1. yolk sac;
4. chorion;
2. amnion;
5. navel string;
3. allantois;
6. chorionic villus.
109.

Answers:
a ) . . . ; b)

c)

d) . . . .

110. Answers:
a ) . . . ; b)

c)

d) . . . . e)

f)

111. Scheme of construction of sensitive neuron represented on the figure. Numerate on


the scheme:

The codes of the answers:

1. cell body;
2. axon;
3. dendrite;
4. Schwann cell;
Answers:
a)
b)

5. node of Ranvier;
6. presynaptic membrane;
7. postsynaptic membrane.

c)

d)

e)

f)

g) . . . .

112. Stages of the cell's cycle are marked by numbers. Find the according
between the numerals on the figure I and letters, which dente the stages of cell's
cycle, on figure II.

Answers:
a)
b)
113.

c)

d)

e) . . . .

Numerate on the figure infancies

The codes of the answers:


1. blastocoel;
2. blastopore;
3. archenteron;
4. ectoderm;
5. endoderm;
6. mesoderm;
7. building of nervous system;
8. chorda.
Answers:
a ) . . . ; b)
c)
d)
e)

f)

of lancelet:

g)

h)

114. Cycle of the individual development of moss is


represented on the figure. Numerate on the scheme:
The codes of the answers:
1. cells with 2n set chromosomes (sporophyte);
2. cells with n set chromosomes (gametophyte);
3. archegonium;
4. anteridium.
Answers:
a)...; b)
c)
d)
e)
f)
g) ....
115. Cycle of the individual
development of lichen is
represented on the figure.
Numerate on the scheme:
The codes of the answers:
1. cells with 2n set
chromosomes
(sporophyte);
2. cells with n set
chromosomes
(gametophyte);
3. strobilus;
4. microspores;
5. megaspores;
6. archegonium;
7. anteridium.
Answers:
a)...; b)
c)
d)
e)
f)...; g)
h)...; i)
j) ....
116. Cross-cut of pistil is
represented on the figure.
Name its parts with the
help of the codes:
The codes of the answers:
1. egg;
2. sperm;
3. polar nucleus;
4. stigma;
5. ovary;
6. synergids;
7. antipodal cells;
8. style;
9. pollen tube.
Answers:
a)...; b)
c)
d)
e)
g)
h)
i) ....

f)

117. There are schemes of situation of the conductive textures in the cross-cuts of stalk
of extinct rhynia (1), of club-moss (2), of horsetail (3), of fern (4) and pine (5). Numerate on the schemes (ph - phloem, x - xylem).

Answers: a)

b)

c)

d)

e) . . . .

118. On the figures fore-and-aft crosses of flowers of rosace (Rosaceae). Also


compliment the numbers of the
figures.
The codes of the answers:
1. nectary;
2. corolla;
3. cup;
4. pistil;
5. receptacle;
6. strawberry;
7. pear;
8. rose;
9. plum.
Answers: a ) . . . ;

b)

c)

d)

e)

A ) . . . ; B)

C)

D) . . . .

119. Outgoing from the ecological regularity, complement the Figure into squares
with the denominations of such plant's species, which can constitute, as a result, fragment
of defoliated forest with natural species composition. Order of designation of species in a
separate several horizons are not important.
The codes of the answers:
1. durmast oak (Ouercus petraea);
2. small-leaved lime (Tilia cordata);
3. European hazel (Corytus avellana);
4. weymouth pine {Pinus strobus);
5. golden osier {Salix alba);
6. European hornbeam (Carpinus betulus);
7. loranth {Loranthus europaeus);
8. juniper common (Juniperus communis);
9. frail (Phragmites australis);
10. daphne february {Daphne mezereum);
11. annual meadow grass {Poa annua);
12. fescue (Festuca rubra);
13. nightshade black (Solanum nigrum);
14. chickling (Lathyrus vernus);
15. mustard dish (Thlaspi arvense);
16. wood bluegrass {Poa nemoralis);
17. Symphoricarpos alba;
18. Nardus strica.
Answers: a)
b)
c)
d) . . . .

INTERNATIONAL BIOLOGY OLYMPIAD 1990


THEORY PROBLEMS SOLUTIONS















All IBO examination questions are published under the following Creative Commons license:



CC BY-NC-SA (Attribution-NonCommercial-ShareAlike) https://creativecommons.org/licenses/by-nc-sa/4.0/
The exam papers can be used freely for educational purposes as long as IBO is credited and
new creations are licensed under identical terms. No commercial use is allowed.

I IBiO

Você também pode gostar